You are on page 1of 1322

About Pearson

Pearson is the world’s learning company, with presence across 70 countries


worldwide. Our unique insights and world-class expertise comes from a long
history of working closely with renowned teachers, authors and thought leaders, as
a result of which, we have emerged as the preferred choice for millions of teachers
and learners across the world.
We believe learning opens up opportunities, creates fulfilling careers and hence
better lives. We hence collaborate with the best of minds to deliver you class-
leading products, spread across the Higher Education and K12 spectrum.
Superior learning experience and improved outcomes are at the heart of everything
we do. This product is the result of one such effort.
Your feedback plays a critical role in the evolution of our products and you can
contact us - reachus@pearson.com. We look forward to it.

Objective_Maths_JEE Main 2019_Prelims.indd 1 5/18/2016 3:11:34


A Complete Resource Book in

Mathematics
for JEE Main 2017

Dr Dinesh Khattar
Kirori Mal College, University of Delhi

Objective_Maths_JEE Main 2019_Prelims.indd 5 5/18/2018 3:11:35 PM


The aim of this publication is to supply information taken from sources believed to be valid and reliable. This is not an
attempt to render any type of professional advice or analysis, nor is it to be treated as such. While much care has been
taken to ensure the veracity and currency of the information presented within, neither the publisher nor its authors bear
any responsibility for any damage arising from inadvertent omissions, negligence or inaccuracies (typographical or
factual) that may have found their way into this book.

Copyright © 2016  Pearson India Education Services Pvt. Ltd

This book is sold subject to the condition that it shall not, by way of trade or otherwise, be lent, resold, hired out,
or otherwise circulated without the publisher’s prior written consent in any form of binding or cover other than that
in which it is published and without a similar condition including this condition being imposed on the subsequent
purchaser and without limiting the rights under copyright reserved above, no part of this publication may be reproduced,
stored in or introduced into a retrieval system, or transmitted in any form or by any means (electronic, mechanical,
photocopying, recording or otherwise), without the prior written permission of both the copyright owner and the
publisher of this book.

No part of this Book may be used or reproduced in any manner whatsoever without the publisher’s prior written
consent.

ISBN 978-93-325-7030-6

First Impression
Published by Pearson India Education Services Pvt. Ltd, CIN: U72200TN2005PTC057128.

Head Office: 15th Floor, Tower-B, World Trade Tower, Plot No. 1, Block-C, Sector 16, Noida 201 301,
Uttar Pradesh, India.
Registered Office: 4th floor, Software Block, Elnet Software City, TS-140, Block 2 & 9, Rajiv Gandhi Salai,
Taramani, Chennai 600 113, Tamil Nadu, India.
Fax:  080-30461003, Phone: 080-30461060
website: in.pearson.com, Email: companysecretary.india@pearson.com

Compositor: SRS Global, Puducherry


Printed in India by

Objective_Maths_JEE Main 2019_Prelims.indd 6 5/18/2016 5:08:27


Contents

Preface . . . . . . . . . . . . . . . . . . . . . . . . . . . . . . . . . . . . . . . . . . . . . . . . . . . . . . . . . . . i

Chapter 1 Set Theory                                              11–118

Chapter 2 Functions                                               21–260

Chapter 3 Complex Numbers                                          31–368

Chapter 4 Quadratic Equations and Expressions                                41–452

Chapter 5 Matrices                                                51–530

Chapter 6 Determinants                                             61–656

Chapter 7 Permutations and Combinations                                   71–746

Chapter 8 Mathematical Induction                                        81–84

Chapter 9 Binomial Theorem                                          91–946

Chapter 10 Sequence and Series                                       101–1062

Chapter 11 Limits                                               111–1148

Chapter 12 Continuity and Differentiability                                  121–1250

Chapter 13 Differentiation                                           131–1344

Chapter 14 Applications of Derivatives                                    141–1462

Chapter 15 Indefinite Integration                                       151–1552

Chapter 16 Definite Integral and Area                                     161–1686

Chapter 17 Differential Equations                                       171–1748

Chapter 18 Coordinates and Straight Lines                                 181–1854

Chapter 19 Circles                                              191–1956

Chapter 20 Conic Sections (Parabola, Ellipse and Hyperbola)                        201–2062

Chapter 21 Vector Algebra                                          211–2146

Chapter 22 Three Dimensional Geometry                                   221–2236


Chapter 23 Measures of Central Tendency and Dispersion                         231–2324

Objective_Maths_JEE Main 2019_Prelims.indd 7 5/18/2016 3:11:35


viii  Contents

Chapter 24  Probability����������������������������������������������������������������������������������������������������������������������������������� 24.1–24.60

Chapter 25  Trigonometric Ratios and Identities�������������������������������������������������������������������������������������������� 25.1–25.30

Chapter 26  Trigonometric Equations������������������������������������������������������������������������������������������������������������� 26.1–26.22

Chapter 27  Inverse Trigonometric Functions������������������������������������������������������������������������������������������������� 27.1–27.36

Chapter 28  Heights and Distances���������������������������������������������������������������������������������������������������������������� 28.1–28.36

Chapter 29  Mathematical Reasoning���������������������������������������������������������������������������������������������������������������� 29.1–29.6

Mock Test - I �������������������������������������������������������������������������������������������������������������������������������������������������������� M1.1–M1.8

Mock Test - II ������������������������������������������������������������������������������������������������������������������������������������������������������� M2.1–M2.6

Mock Test - III ������������������������������������������������������������������������������������������������������������������������������������������������������� M3.1–M3.6

Mock Test - IV ������������������������������������������������������������������������������������������������������������������������������������������������������� M4.1–M4.8

Mock Test - V ������������������������������������������������������������������������������������������������������������������������������������������������������� M5.1–M5.8

Objective_Maths_JEE Main 2019_Prelims.indd 8 5/18/2018 3:11:36 PM


Preface

About the Series


A Complete Resource Book for JEE Main series is a must-have resource for students preparing for JEE Main examination.
There are three separate books on Physics, Chemistry, and Mathematics; the main objective of this series is to strengthen
the fundamental concepts and prepare students for various engineering entrance examinations. It provides class-tested
course material and numerical applications that will supplement any ready material available as student resource.
To ensure high level of accuracy and practicality, this series has been authored by highly qualified and experienced
faculties for all three titles.
About the Book
This book, A Complete Resource Book in Mathematics for JEE Main 2019, covers both the text and various types of prob-
lems required as per the syllabus of JEE Main examination. It also explains various short-cut methods and techniques to
solve objective questions in lesser time.
Salient Features
• Complete coverage of topics along with ample number of solved examples.
• Large variety of practice problems with complete solutions.
• Chapter-wise Previous 15 years’ AIEEE/JEE Main questions.
• 5 Mock Tests based on JEE Main pattern in the book.
• 5 Free Online Mock Tests as per the recent JEE Main pattern.
It would have been difficult to prepare this book without aid and support from a number of different quarters. I shall be
grateful to the readers for their regular feedback. I am deeply indebted to my parents without whose encouragement this
dream could not have been translated into reality. The cherubic smiles of my daughters, Nikita and Nishita, have inspired
me to treat my work as worship.
Anuj Agarwal from IIT-Delhi, Ankit Katial from National Institute of Technology (Kurukshetra) and Raudrashish
Chakraborty from Kirori Mal College, University of Delhi, with whom I have had fruitful discussions, deserve special
mention.
I earnestly hope that the book will help the students grasp the subject well and respond with a commendable score in
the JEE Main examination. There are a plethora of options available to students for Mathematics, however, ever grateful to
them and to the readers for their candid feedback.
Despite of our best eff orts, some errors may have crept into the book. Constructive comments and suggestions to
further improve the book are welcome and shall be acknowledged gratefully.
Best of luck!

Dinesh Khattar

Objective_Maths_JEE Main 2019_Prelims.indd 9 5/18/2018 3:11:36 PM


Set Theory 1.1

CHaPtER

1 Set Theory

Chapter Highlights
Set, Representation of a set, Types of sets, Operations on sets, Algebra of sets, Cartesian product of two sets,
Relations, Types of relations on a set, Equivalence relation, Congruence modulo m.

seT
i M P o R ta n t P o i n t S
A set is a well defined collection of objects such that given
an object, it is possible to determine whether that object  In roster form, every element of the set is listed only
belongs to the given collection or not. once.
For example, the collection of all students of Delhi  The order in which the elements are listed is immaterial
University is a set, whereas, collection of all good books For example, each of the following sets denotes the same
on mathematics is not a set, since a mathematics book con- set {1, 2, 3}, {3, 2, 1}, {1, 3, 2}.
sidered good by one person might be considered bad or
average by another.
set-builder form
notations In this form, we write a variable (say x) representing any
member of the set followed by a property satisfied by each
The sets are usually denoted by capital letters A, B, C, etc.
member of the set.
and the members or elements of the set are denoted by
For example, the set A of all prime numbers less than
lower-case letters a, b, c etc. If x is a member of the set A,
10 in the set-builder form is written as
we write x ∈ A (read as ‘x belongs to A’) and if x is not a
member of the set A, we write x ∉ A (read as ‘x does not A = {x | x is a prime number less than 10}
belong to A’). If x and y both belong to A, we write x, y ∈ A.
The symbol ‘|’ stands for the words ‘such that’. Sometimes,
we use the symbol ‘:’ in place of the symbol ‘|’.
rePresenTaTion of a seT
Usually, sets are represented in the following two ways: TYPes of seTs
1. Roster form or tabular form empty set or null set
2. Set builder form or rule method
A set which has no element is called the null set or empty
roster form set. It is denoted by the symbol Φ.
For example, each of the following is a null set:
In this form, we list all the members of the set within braces
(curly brackets) and separate these by commas. 1. The set of all real numbers whose square is –1.
For example, the set A of all odd natural numbers less 2. The set of all rational numbers whose square is 2.
than 10 in the roster form is written as: 3. The set of all those integers that are both even and odd.

A = {1, 3, 5, 7, 9} A set consisting of atleast one element is called a non-empty


set.

Objective_Maths_JEE Main 2017_Ch 1.indd 1 01/01/2008 03:14:05


1.2 Chapter 1

singleton set For example, let A = {3, 4}, then the subsets of A are f,
{3}, {4}, {3, 4}. Here, n(A) = 2 and number of subsets of
A set having only one element is called singleton set.
For example, {0} is a singleton set, whose only A = 22 = 4.
member is 0. Also, {3} ⊂ {3, 4} and {2, 3} ⊄ {3, 4}

finite and infinite set Power set


A set which has finite number of elements is called a finite The set of all subsets of a given set A is called the power set
set. Otherwise, it is called an infinite set. of A and is denoted by P(A).
For example, the set of all days in a week is a finite For example, if A = {1, 2, 3}, then
set whereas, the set of all integers, denoted by {…, – 2, – 1, P(A) = [f, {1}, {2}, {3}, {1, 2} {1, 3}, {2, 3}, {1, 2, 3}]
0, 1, 2, …} or {x | x is an integer}, is an infinite set.
An empty set Φ which has no element, is a finite set. Clearly, if A has n elements, then its power set P(A) contains
The number of distinct elements in a finite set A is exactly 2n elements.
called the cardinal number of the set A and it is denoted
by n (A). Trick(s) for Problem solving

equal sets Number of elements in P {P [P(f)]} is 4


or Cardinal Number of P{P[P(f)]} = 4
Two sets A and B are said to be equal, written as A = B, if Since, P (f) = {f}
every element of A is in B and every element of B is in A. Also, P [P (f)] = {f, {f}}
and P {P[P (f)]} = {f, {f}, [{f}], [f, {f}]}
equivalent sets
Hence, n [P {P[P (f)]}] = 4
Two finite sets A and B are said to be equivalent, if n(A) =
n(B).
euler–venn diagrams
caution To express the relationship among sets, we represent them
pictorially by means of diagrams, known as Euler–Venn
Equal sets are equivalent but equivalent sets need not be Diagrams or simply Venn diagrams.
equal.
In Venn diagrams, the universal set U is represented
For example, the sets A = {4, 5, 3, 2} and B = {1, 6, by the rectangular region and its subsets are represented by
8, 9} are equivalent but are not equal.
closed bounded circles inside this rectangular region.

subset oPeraTions on seTs


Let A and B be two sets. If every element of A is an element Union of Two sets
of B, then A is called a subset of B and we write A ⊆ B or
B ⊇ A (read as ‘A is contained in B’ or B contains A’). B is The union of two sets A and B, written as A ∪ B (read as
called superset of A. ‘A union B’), is the set consisting of all the elements which
are either in A or in B or in both. Thus,
A ∪ B = {x: x ∈ A or x ∈ B}
i M P o R ta n t P o i n t S
Clearly, x∈A∪B
 If A ⊆ B and A ≠ B, we write A ⊂ B or B ⊃ A (read as : A ⇒ x∈A
is a proper subset of B or B is a proper superset of A).
or x ∈ B,
Every set is a subset and a superset of itself.
x∉A∪B

and
 If A is not a subset of B, we write A ⊄ B.
⇒ x∉A
 The empty set is the subset of every set.
 If A is a set with n (A) = m, then the number of subsets and x ∉ B.
of A are 2m and the number of proper subsets of A are
For example, if A = {a, b, c d} and B = {c, d, e, f }, then
2m–1.
A ∪ B = {a, b, c, d, e, f }.

Objective_Maths_JEE Main 2017_Ch 1.indd 2 01/01/2008 03:14:05


Set Theory 1.3

Similarly,
U
B – A = {x: x ∈ B and x ∉ A}
A∪B For example,
if A = {1, 2, 3, 4, 5} and B = {1, 3, 5, 7, 9},
A B
then A – B = {2, 4} and B – A = {7, 9}
Fig. 1.1

intersection of Two sets U U

The intersection of two sets A and B, written as A ∩ B (read A–B B–A


as ‘A intersection B’) is the set consisting of all the com-
mon elements of A and B. Thus,
A B A B
A ∩ B = {x: x ∈ A and x ∈ B}
Fig. 1.4(a–b)
Clearly, x∈A∩B
⇒ x ∈ A and x ∈ B,
notE
and x∉A∩B
⇒ x ∉ A or x ∉ B.  A–B≠B–A
 The sets A – B, B – A and A ∩ B are disjoint sets
For example, if A = {a, b, c, d} and B = {c, d, e, f }, then  A – B ⊆ A and B – A ⊆ B
A ∩ B = {c, d}.
 A – f = A and A – A = f

U
symmetric difference of Two sets
A∩B
The symmetric difference of two sets A and B, denoted by
A D B, is defined as
A B A D B = (A – B) ∪ (B – A).
Fig. 1.2 For example, if A = {1, 2, 3, 4, 5} and B = {1, 3, 5, 7, 9}
then A D B = (A – B) ∪ (B – A) = {2, 4} ∪ {7, 9} = {2, 4,
disjoint sets 7, 9}.
Two sets A and B are said to be disjoint, if A ∩ B = f, i.e., A
and B have no element in common.
For example, if A = {1, 2, 5} and B = {2, 4, 6}, then
A ∩ B = f, so A and B are disjoint sets.

U B

Fig. 1.5

A B
complement of a set
Fig. 1.3 If U is a universal set and A is a subset of U, then the com-
plement of A is the set which contains those elements of
difference of Two sets U, which are not contained in A and is denoted by A′ or Ac.
Thus,
If A and B are two sets, then their difference A – B is defined as
A′ = {x: x ∈ U and x ∉ A}
A – B = {x: x ∈ A and x ∉ B}

Objective_Maths_JEE Main 2017_Ch 1.indd 3 01/01/2008 03:14:06


1.4 Chapter 1

For example, (a) (A ∪ B)′ = A′ ∩ B′


if U = {1, 2, 3, 4, …} and A = {2, 4, 6, 8, …}, (b) (A ∩ B)′ = A′ ∪ B′
(c) A – (B ∪ C) = (A – B) ∩ (A – C)
then, A′ = {1, 3, 5, 7, …}
(d) A – (B ∩ C) = (A – B) ∪ (A – C)

U key results on operations on sets


1. A ⊆ A ∪ B, B ⊆ A ∪ B, A ∩ B ⊆ A, A ∩ B ⊆ B
A 2. A – B = A ∩ B′
3. (A – B) ∪ B = A ∪ B
4. (A – B) ∩ B = f
A′
5. A ⊆ B ⇔ B′ ⊆ A′
6. A – B = B′ – A′
Fig. 1.6
7. (A ∪ B) ∩ (A ∪ B′) = A
8. A ∪ B = (A – B) ∪ (B – A) ∪ (A ∩ B)
9. A – (A – B) = A ∩ B
notE 10. A–B=B–A⇔A=B
11. A∪B=A∩B⇔A=B
 U′ = f
12. A ∩ (B D C) = (A ∩ B) D (A ∩ C)
 f′ = U
A ∪ A′ = U

some results about cardinal number
 A ∩ A′ = f
If A, B and C are finite sets and U be the finite universal
set, then
algebra of seTs 1. n (A′) = n (U) – n (A)
2. n (A ∪ B) = n (A) + n (B) – n (A ∩ B)
1. Idempotent Laws: For any set A, we have 3. n (A ∪ B) = n (A) + n(B),
(a) A ∪ A = A where A and B are disjoint non-empty sets
(b) A ∩ A = A 4. n (A ∩ B′) = n (A) – n (A ∩ B)
2. Identity Laws: For any set A, we have: 5. n (A′ ∩ B′) = n (A ∪ B) ‘ = n (U) – n (A ∪ B)
(a) A ∪ f = A 6. n (A′ ∪ B′) = n (A ∩ B) ‘ = n (U) – n (A ∩ B)
(b) A ∩ f = f 7. n (A – B) = n (A) – n (A ∩ B)
(c) A ∪ U = U 8. n (A ∩ B) = n (A ∪ B) – n (A ∩ B′) – n (A′ ∩ B)
(d) A ∩ U = A 9. n (A ∪ B ∪ C) = n (A) + n (B) + n (C) – n (A ∩ B)
3. Commutative Laws: For any two sets A and B, we – n (B ∩ C) – n (C ∩ A) + n (A ∩ B ∩ C)
have 10. If A1, A2, A3, … An are disjoint sets, then
(a) A ∪ B = B ∪ A n (A1 ∪ A2 ∪ A3 ∪ … ∪ An) = n (A1) + n (A2) + n (A3)
(b) A ∩ B = B ∩ A + … + n (An)
4. Associative Laws: For any three sets A, B and C, we 11. n (A D B) = number of elements which belong to
have exactly one of A or B
(a) A ∪ (B ∪ C) = (A ∪ B) ∪ C
(b) A ∩ (B ∩ C) = (A ∩ B) ∩ C
5. Distributive Laws: For any three sets A, B and C, we carTesian ProdUcT of Two seTs
have
If A and B are any two non-empty sets, then cartesian
(a) A ∪ (B ∩ C) = (A ∪ B) ∩ (A ∪ C)
product of A and B is defined as
(b) A ∩ (B ∪ C) = (A ∩ B) ∪ (A ∩ C)
6. For any two sets A and B, we have A × B = [(a, b) : a ∈ A and b ∈ B]
(a) P(A) ∩ P(B) = P(A ∩ B)
(b) P(A) ∪ P(B) ⊆ P(A ∪ B), where P(A) is the power
set of A. caution
7. If A is any set, we have (A′)′ = A.
8. Demorgan’s Laws: For any three sets A, B and C, we A×B≠B×A
have

Objective_Maths_JEE Main 2017_Ch 1.indd 4 01/01/2008 03:14:07


Set Theory  1.5

⇒ 50 = 35 + 24 – n (A ∩ B)
Trick(s) for Problem Solving
⇒ n(A ∩ B) = 59 – 50 = 9.
 If A = f or B = f, then we define A × B = f.
2. Let A = {2, 3, 4} and X = {0, 1, 2, 3, 4}, then which of
 If A has n elements and B has m elements then A × B has

mn elements.
the following statements is correct?
 If A1, A2, …, Ap are p non-empty sets, then their cartesian
(A) {0} ∈ A′ in X
p (B) f ∈ A′ with respect to. X
product, is defined as ∏ Ai = [(a1, a2, a3, …, ap); ai ∈ Ai (C) {0} ⊂ A′ with respect to X
i =1
for all i] (D) 0 ⊂ A′ with respect to X.
Solution: (C)
Key Results on Cartesian Product We have, A′ in X = The set of elements in X which are
not in A = {0, 1}
If A, B, C are three sets, then {0} ∈ A′ in X is false, because {0} is not an e­ lement
1. A × (B ∪ C) = (A × B) ∪ (A × C) of A′ in X.
2. A × (B ∩ C) = (A × B) ∩ (A × C) f ⊂ A′ in X is false, because f is not an element
3. A × (B – C) = (A × B) – (A × C) of A′ in X
4. (A × B) ∩ (S × T) = (A ∩ S) × (B ∩ T), {0} ⊂ A′ in X is correct, because the only element
where S and T are two sets. of {0} namely 0 also belongs to A′ in X.
5. If A ⊆ B, then (A × C) ⊆ (B × C) 0 ⊂ A′ in X is false, because 0 is not a set.
6. If A ⊆ B, then (A × B) ∩ (B × A) = A2 3. If X = {8n – 7 n – 1/n ∈ N} and Y = {49 (n – 1)/n ∈ N},
7. If A ⊆ B and C ⊆ D then A × C ⊆ B × D then
8. If A ⊆ B, then A × A ⊆ (A × B) ∩ (B × A)
(A) X ⊂ Y (B)  Y⊂X
9. If A and B are two non-empty sets having n elements
(C) X = Y (D)  None of these
in common, then A × B and B × A have n2 elements in
common. Solution: (A)
10. A × B = B × A if and only if A = B We have, 8n – 7n – 1
11. A × (B′ ∪ C ′)′ = (A × B) ∩ (A × C)
= (7 + 1)n – 7n – 1 = (nC272 + nC373 + … + nCn7n)
12. A × (B′ ∩ C ′) ‘ = (A × B) ∪ (A × C)
= 49(nC2 + nC37 + … + nCn7n – 2) for n ≥ 2
Solved Examples For n = 1, 8n – 7n – 1 = 0
Thus, 8n – 7 n – 1 is a multiple of 49 for n ≥ 2 and
1. If n (U) = 60, n (A) = 35, n (B) = 24 and n (A ∪ B)′ = 0 for n = 1. Hence, X consists of all positive integral
10 then n (A ∩ B) is multiples of 49 of the form 49 Kn. where Kn = nC2 +
(A) 9 (B) 8
n
C37 + … + nCn7n – 2 together with zero. Also, Y con-
(C)  6 (D)  None of these sists of all positive integral multiples of 49 including
zero. Therefore, X ⊂ Y.
Solution: (A)
4. The set (A ∪ B ∪ C) ∩ (A ∩ B′ ∩ C′)′ ∩ C′ is equal to
We have,
(A) A ∩ B (B)  A ∩ C ′
n(A ∪ B) = n (U) – n(A ∪ B)′ = 60 – 10 = 50 (C) B ∩ C ′ (D)  B′ ∩ C ′
Now, n (A ∪ B) = n (A) + n(B) – n(A ∩ B) Solution: (C)
U (A ∪ B ∪ C) ∩ (A ∩ B′ ∩ C ′)′ ∩ C ′
(A ∪ B)′
= (A ∪ B ∪ C) ∩ (A′ ∪ B ∪ C) ∩ C ′
A B
= [(A ∩ A′) ∪ (B ∪ C)] ∩ C ′
= (f ∪ B ∪ C) ∩ C ′ = (B ∪ C) ∩ C ′
= (B ∩ C ′) ∪ (C ∩ C ′)
A∩B
= (B ∩ C ′) ∪ f = B ∩ C ′

Objective_Maths_JEE Main 2017_Ch 1.indd 5 01/01/2008 03:14:08


1.6  Chapter 1

5. If A, B and C are non-empty subsets of a set, then = {x : (x ∈ R and x ≥ 2) or (x ∈ R and x ≤ 0)}


(A – B) ∪ (B – A) equals
= {x : x ∈ R and x ≥ 2} ∪ {x : x ∈ R and x ≤ 0}
(A) (A ∩ B) ∪ (A ∪ B) (B) (A ∪ B) – (A ∩ B)
(C) A – (A ∩ B) (D) (A ∪ B) – B Similarly,
Solution: (B) B′ = {x : x ∈ R and x ≤ 1} ∪ {x : x ∈ R and x > 3}

(A – B) ∪ (B – A) = (A ∪ B) – (A ∩ B) \ A ∪ B = {x : x ∈ R and 0 < x ≤ 3},


A ∩ B = {x : x ∈ R and 1 < x < 2}
A – B = {x : x ∈ R and 0 < x ≤ 1}
A B 9. If Y ∪ {1, 2} = {1, 2, 3, 5, 9}, then
(A)  The smallest set of Y is {3, 5, 9}
(B)  The smallest set of Y is {2, 3, 5, 9}
A−B A∩B B−A (C)  The largest set of Y is {1, 2, 3, 4, 9}
(D)  The largest set of Y is {2, 3, 4, 9}
6. Let A and B two non-empty subsets of a set X such that
A is not a subset of B then Solution:  (A and C)
(A) A is subset of the complement of B Since the set on the right hand side has 5 elements,
(C) B is a subset of A \ smallest set of Y has three elements and largest
(C) A and B are disjoint set of Y has five elements,
(D) A and the complement of B are non-disjoint \ smallest set of Y is {3, 5, 9}
and largest set of Y is {1, 2, 3, 4, 9}.
Solution: (D)
10. If A has 3 elements and B has 6 elements, then the
Since A ⊄ B, $ x ∈ A such that x ∉ B
minimum number of elements in the set A ∪ B is
Then x ∈ B ′. (A) 6 (B) 3
\ A ∩ B ′ ≠ f (C) f (D)  None of these

7. Two finite sets have m and n elements, then total number Solution: (A)
of subsets of the first set is 56 more that the total number Clearly the number of elements in A ∪ B will be mini-
of subsets of the second. The values of m and n are, mum when A ⊂ B. Hence the minimum number of ele-
(A)  7, 6 (B)  6, 3 (D)  5, 1 (D)  8, 7 ments in A ∪ B is the same as the number of elements
in B, that is, 6.
Solution: (B)
11. Suppose A1, A2, … A30 are thirty sets, each with five
Since the two finite sets have m and n elements, so
elements and B1, B2, …, Bn are n sets each with three
number of subsets of these sets will be 2m and 2n
30 n
respectively. According to the question
m n
elements. Let ∪ Ai = ∪ Bj =S
2 – 2 = 56 i =1 j =1
If each element of S belongs to exactly ten of the Ai′s
putting m = 6, n = 3, we get and exactly nine of the Bj′s then n =
26 – 23 = 56  or  64 – 8 = 56 (A) 45 (B) 35
(C)  40 (D)  None of these
8. Let U = R (the set of all real numbers) If A = {x : x ∈
R, 0 < x < 2}, B = {x : x ∈ R, 1 < x ≤ 3}, then Solution: (A)
(A) A ∪ B = {x : x ∈ R and 0 < x ≤ 3} Given Ai′s are thirty sets with five elements each, so
(B) A ∩ B = {x : x ∈ R and 1 < x < 2} 30
(C) A – B = {x : x ∈ R and 0 < x ≤ 1} ∑ n ( Ai ) = 5 × 30 = 150 (1)
i =1
(D)  All of these
If there are m distinct elements in S and each element
Solution: (D) of S belongs to exactly 10 of the Ai′s, we have
We have 30

A′ = R – A = {x : ∈ R and x ∉ A}
∑ n ( Ai ) = 10 m (2)
i =1

Objective_Maths_JEE Main 2017_Ch 1.indd 6 01/01/2008 03:14:09


Set Theory  1.7

\ From Eq. (1) and (2), we get ∵ n (A ∪ B) = n(A) + n(B) – n(A ∩ B)
10 m = 150 65 = 40 + n (B) – 10
\ m = 15 (3) n (B) = 65 – 40 + 10 = 35
30 30
Similarly ∑ n ( B j ) = 3n and ∑ n ( B j ) = 9 m Number of people who like only tennis
j =1 j =1
= n(B) – n (A ∩ B) = 35 – 10 = 25
\ 3 n = 9 m \ Number of people who like tennis only and not
9m cricket = 25.
⇒ n= = 3 m = 3 × 15 = 45 [from (3)]
3
15. In a group of 1000 people, there are 750 people who
Hence, n = 45.
can speak Hindi and 400 who can speak English. Then
12. If A = {1, 3, 5, 7, 9, 11, 13, 15, 17}, B = {2, 4, …, 18} number of persons who can speak Hindi only is
and N is the universal set, then A′ ∪ ((A ∪ B) ∩ B′) is (A) 300 (B) 400
(A) A (B)  N (C)  600 (D)  None of these
(C) B (D)  None of these
Solution: (C)
Solution: (B) Here
We have,
n(H ∪ E) = 1000, n (H) = 750,
(A ∪ B) ∩ B′ = A
n (E) = 400
[(A ∪ B) ∩ B′] ∪ A′ = A ∪ A′ = N.
Using n(H ∪ E) = n(H) + n(E) – n(H ∩ E)
13. If X and Y are two sets and X′ denotes the complement 1000 = 750 + 400 – n (H ∩ E)
of X, then X ∩ (X ∪ Y)′ equals
⇒ n (H ∩ E) = 1150 – 1000 = 150.
(A) X (B)  Y
(C) f (D)  None of these U
Solution: (C)
H E
X ∩ (X ∪ Y)′ = X ∩ (X ′ ∩ Y ′)
[∵ By De-Morgan’s Law (A ∪ B)′ = (A′ ∩ B′)] 600 150 250

= (X ∩ X ′) ∩ Y ′ = f ∩ Y ′ = f


14. In a group of 65 people, 40 like cricket, 10 like both
cricket and tennis. The number of persons liking tennis
only and not cricket is Number of people who can speak Hindi only
(A) 21 (B) 25 = n(H ∩ E′) = n(H) – n(H ∩ E)
(C)  15 (D)  None of these
= 750 – 150 = 600.
Solution: (B)
Let A be the set of people who like cricket and B the 16. If f : R → R, defined by f (x) = x2 + 1, then the values of
set of people who like tennis. f –1(17) and f –1(–3) respectively are
(A) f, {4, –4} (B)  {3, –3}, f
Then n(A ∪ B) = 65 (C) f, {3, –3} (D)  {4, –4}, f
n (A) = 40, n(A ∩ B) = 10 Solution: (D)
U Let y = x2 + 1. Then for y = 17,
we have x = ± 4 and for y = –3, x becomes ­imaginary
A B
that is, there is no value of x.
30 10 25
Hence, f (17) = {–4, 4}
and f–1(–3) = f

Objective_Maths_JEE Main 2017_Ch 1.indd 7 01/01/2008 03:14:10


1.8  Chapter 1

17. In a statistical investigation of 1,003 families of Using


Kolkata, it was found that 63 families had neither a
n(A ∪ B ∪ C) = n (A) + n(B) + n(C) – n(A ∩ B)
radio nor a TV, 794 families had a radio and 187 had a
TV. The number of families in that group having both – n(B ∩ C) – n(A ∩ C) + n(A ∩ B ∩ C)
a radio and a TV is
Substituting the above values, we have
(A) 36 (B) 41
(C)  32 (D)  None of these 92 = 42 + 51 + 68 – 30 – 28 – 36 + n (A ∩ B ∩ C)
Solution: (B) ⇒ n (A ∩ B ∩ C) = 92 – 161 + 94
Let R be the set of families having a radio and T, the ⇒ n (A ∩ B ∩ C) = 92 – 67 = 25
set of families having a TV, then
n (R ∪ T) = The no. of families having at least one Hence, 25% of the people read all the three papers.
of radio and TV 19. Out of 800 boys in a school, 224 played cricket, 240
= 1003 – 63 = 940 played hockey and 336 played basketball. Of the total,
64 played both basketball and hockey; 80 played
n (R) = 794 and n(T) = 187 cricket and basketball and 40 played cricket and
Let x families had both a radio and a TV i.e., hockey; 24 played all the three games. The number of
boys who did not play any game is

(A) 160 (B) 240 (C) 216 (D) 128
R T
Solution: (A)
794 – x

187 – x

x n (C) = 224, n (H) = 240, n (B) = 336


n (H ∩ B) = 64, n (B ∩ C) = 80
n (H ∩ C) = 0, n (C ∩ H ∩ B) = 24
n (R ∩ T ) = x n (C c ∩ H  c ∩ B c) = n [(C ∪ H ∪ B)c]
The number of families who have only radio = 794 – x = n (U) – n (C ∪ H ∪ B)
and the number of families who have only TV = 187 – x
From Venn diagram, = 800 – [n (C) + n (H) + n (B) – n (H ∩ C)

794 – x + x – 187 – x = 940 – n (H ∩ B) – n (C ∩ B) + n (C ∩ H ∩ B)]

⇒ 981 – x = 940 or x = 981 – 940 = 41 = 800 – [224 + 240 + 336 – 64 – 80 – 40 + 24]

Hence, the required no. of families having both a radio = 800 – [824 – 184] = 984 – 824 = 160.
and a TV = 41. 20. In a certain town 25% families own a phone and 15%
18. In a city, three daily newspapers A, B, C are published. own a car, 65% families own neither a phone nor a car.
42% of the people in that city read A, 51% read B and 2000 families own both a car and a phone. Consider
68% read C. 30% read A and B; 28% read B and C; the following statements in this regard:
36% read A and C; 8% do not read any of the three 1. 10% families own both a car and a phone.
newspapers. The percentage of persons who read all 2. 35% families own either a car or a phone.
the three papers is 3.  40,000 families live in the town.
(A) 25% (B)  18% Which of the above statements are correct?
(C) 20% (D)  None of these (A)  1 and 2 (B)  1 and 3
(C)  2 and 3 (D)  1, 2 and 3
Solution: (A)
Solution: (C)
Let the no. of persons in the city be 100.
Then we have n(P) = 25%, n(C) = 15%,
n (A) = 42, n (B) = 51, n (C) = 68;
n (P ′ ∩ C ′) = 65%, n (P ∩ C) = 2000
n (A ∩ B) = 30, n(B ∩ C) = 28, n(A ∩ C) = 36
n (A ∪ B ∪ C) = 100 – 8 = 92 Since, n (P ′ ∩ C ′) = 65%

Objective_Maths_JEE Main 2017_Ch 1.indd 8 01/01/2008 03:14:10


Set Theory 1.9

\ n(P ∪ C)′ = 65% Now, if x ∈ N, then the solution set is {4}.


Also, if x ∈ I, then the solution set is {0, 4}.
\ n(P ∪ C) = 35%
Further, since there is no root of the form a + ib,
Now, n (P ∪ C) = n (P) + n (C) – n (P ∩ C) where a, b are real and b ≠ 0,
\ if x ∈ S = {a + ib : b ≠ 0, a, b ∈ R} then the
\ 35 = 25 + 15 – n (P ∩ C)
solution set is f.
\ n (P ∩ C) = 40 – 35 = 5
Thus, n(P ∩ C) = 5% relaTions
But, n(P ∩ C) = 2000 Let A, B be any two non-empty sets, then every subset of
A × B defines a relation from A to B and every relation from
\ 5% of the total = 2000 A to B is a subset of A × B.
2000 × 100 If R is a relation from A to B and if (a, b) ∈ R, then
\ total no. of families = = 40000 we write a R b and say that ‘a is related to b’ and if (a, b)
5
∉ R, then we write a R b and say that a is not related to b.
\ n(P ∪ C) = 35%,
Total no. of families = 40,000 and n(P ∩ C) = 5%. key results on relations
21. If P, Q and R are subsets of a set A, then R × (P ′ ∪ Q′)′ 1. Every subset of A × A is said to be a relation on A.
equals 2. If A has m elements and B has n elements, then A × B
(A) (R × P) ∩ (R × Q) (B) (R × Q) ∩ (R × P) has mn elements and total number of different relations
(C) (R × P) ∪ (R × Q) (D) None of these from A to B is 2mn.
3. Let R be a relation from A to B, i.e. R ⊆ A × B, then
Solution: (A)
Domain of R = {a : a ∈ A, (a, b) ∈ R for some b ∈ B}
R × (P′ ∪ Q′)′ = R × [(P ′)′ ∩ (Q ′)′] Range of R = {b : b ∈ B, (a, b) ∈ R for some a ∈ A}
For example, let A = {1, 3, 4, 5, 7}, B = {2, 4, 6, 8} and
= R × (P ∩ Q)
R be the relation ‘is one less than’ from A to B, then
= (R × P) ∩ (R × Q) R = [(1, 2), (3, 4), (5, 6), (7, 8)].
Here, domain of R = {1, 3, 5, 7} and range of R = {2,
22. If sets A and B are defined as
4, 6, 8}.
A = {(x, y) : y = ex, x ∈ R}
B = {(x, y) : y = x, x ∈ R} then
notE
(A) B ⊂ A (B) A ⊂ B
(C) A ∩ B = f (D) A ∪ B = A Domain of a relation from A to B is a subset of A and its
Solution: (C) range is a subset of B
x 2 x3
Since y = ex = 1 + x +
+ +…
2 ! 3! identity relation
\ ex > x ∀ x ∈ R so that the two curves given by y = ex
and y = x do not intersect in any point Hence, there is R is an identity relation if (a, b) ∈ R if a = b, a ∈ A, b ∈ A.
no common point, so that A ∩ B = f. In other words, every element of A is related to only itself.

23. The solution of 3x2 – 12x = 0 when Universal relation


(A) x ∈ N is {4} Let A be any set and R be the set A × A, then R is called the
(B) x ∈ I is {0, 4} universal relation in A.
(C) x ∈ S = {a + ib : b ≠ 0, a, b ∈ R} is f
(D) All of these void relation
Solution: (D) f is called void relation in a set.
We have,
3x2 – 12x = 0
Trick(s) for Problem solving

⇒ 3x (x – 4) = 0 The void and the universal relations on a set A are respec-


tively the smallest and the largest relations on A
⇒ x = 0, 4

Objective_Maths_JEE Main 2017_Ch 1.indd 9 01/01/2008 03:14:11


1.10 Chapter 1

inverse relation Consider any x, y, z ∈ I.


Let R ⊆ A × B be a relation from A to B. Then R–1 ⊆ B × A 1. Since x – x = 0 = 0 . m ⇒ x – x is divisible by m
is defined by ⇒ (x, x) ∈ R ⇒ R is reflexive.
2. Let (x, y) ∈ R ⇒ x – y is divisible by m
R–1 = [(b, a): (a, b) ∈ R] ⇒ x – y = mq, for some q ∈ I
Thus, ⇒ y – x = m (–q)
⇒ y – x is divisible by m
(a, b) ∈ R ⇔ (b, a) ∈ R– 1, ∀ a ∈ A, b ∈ B ⇒ (y, x) ∈ R
Thus, (x, y) ∈ R ⇒ (y, x) ∈ R ⇒ R is symmetric.
i M P o R ta n t P o i n t S 3. Let (x, y) ∈ R and (y, z) ∈ R
⇒ x – y is divisible by m and y – z is divisible by m
 dom (R–1) = range (R) and range (R–1) = dom (R) ⇒ x – y = mq and y – z = mq′ for some q, q′ ∈ I
 (R–1)–1 = R. For example, if R = [(1, 2), (3, 4), (5, 6)] ⇒ (x – y) + (y – z) = m (q + q′)
then R–1 = [(2, 1), (4, 3), (6, 5)] ⇒ x – z = m (q + q′), q + q′ ∈ I
and (R–1)–1 = [(1, 2), (3, 4), (5, 6)] = R. ⇒ (x, z) ∈ R
dom (R) = (1, 3, 5), range (R) = (2, 4, 6) Thus, (x, y) ∈ R and (y, z) ∈ R ⇒ (x, z) ∈ R, so R is
and dom (R–1) = (2, 4, 6), range (R–1) = (1, 3, 5) transitive.
So, dom (R–1) = range (R) and range (R–1) Hence the relation R is reflexive, symmetric and transitive
= dom (R) and therefore it is an equivalence relation.

caution
TYPes of relaTions on a seT
Every identity relation is reflexive but every reflexive relation
Let A be a non-empty set, then a relation R on A is said to need not be an identity relation. Also, identity relation is
be: reflexive, symmetric and transitive.

1. Reflexive: If a R a, ∀ a ∈ A, i.e., if
(a, a) ∈ R, ∀ a ∈ A congrUence modUlo m
2. Symmetric: If a R b ⇒ b R a, ∀ a, b ∈ A, i.e., if
(a, b) ∈ R ⇒ (b, a) ∈ R, ∀ a, b ∈ A Let m be a positive integer and x, y ∈ I, then x is said to be
3. Anti-symmetric: If a R b and b R a ⇒ a = b, ∀ a, b ∈ A congruent to y modulo m, written as x ≡ y (mod m), if x – y
4. Transitive: If a R b and b R c ⇒ a R c, ∀ a, b, c ∈ A is divisible by m.
i.e., (a, b) ∈ R and (b, c) ∈ R ⇒ (a, c) ∈ R, ∀ a, b, c ∈ A For example, 155 ≡ 7 (mod 4) as
155 - 7 148
= = 37 (integer)
eQUivalence relaTion 4 4
A relation R on a non-empty set A is called an equivalence 27 - 5 22
But 27 ≡/ 5 (mod 4) as = (Not an integer)
relation if and only if it is 4 4
1. reflexive i M P o R ta n t P o i n t S
2. symmetric and
3. transitive  The universal relation on a non-void set is reflexive
 The identity and the universal relations on a non-void set
That is, R satisfies following properties:
are symmetric and transitive
1. a R a, ∀ a ∈ A  The identity relation on a set is an anti-symmetric relation
2. a R b ⇒ b R a, ∀ a, b ∈ A  The relation R on a set A is symmetric if and only if R = R–1
3. a R b, b R c ⇒ a R c, ∀ a, b, c ∈ A  If R and S are two equalvalence relations on a set A, then
R ∩ S is also an equivalence relation on A.
For example, let I be the set of all integers, m be a positive  The union of two equivalence equivalence relations on a
integer. Then the relation, R on I is defined by, set is not necessarily an equivalence realtion on the set.
R = [(x, y): x, y ∈ I, x – y is divisible by m]  The inverse of an equivalence relation is an equivalence
relation.

Objective_Maths_JEE Main 2017_Ch 1.indd 10 01/01/2008 03:14:12


Set Theory  1.11

R is symmetric since if | a – b | > 0, then


caution
| b – a | = | a – b | > 0.
A reflexive relation on a set is not necessarily symmetric. Thus a R b ⇒ b R a
R is not transitive. For example,
consider the numbers 3, 7, 3. Then we have 3 R 7
Equivalence Classes of an Equivalence since | 3 – 7 | = 4 > 0 and 7 R 3 since | 7 – 3 | = 4 > 0.
Relation But 3 R 3 since | 3 – 3 | = 0 so that | 3 – 3 | >/ 0.
Let R be an equivalence relation in A (≠ f), Let a ∈ A. Then 26. Let R be a relation defined as a R b if 1 + ab > 0. Then,
the equivalence class of a, denoted by [a] or { a }is defined the relation R is
as the set of all those points of A which are related to a (A) Reflexive (B) Symmetric
under the relation R. Thus, [a] = {x ∈ A : x Ra} (C)  Transitive (D)  None of these
It is easy to see that
Solution: (A) and (B)
1. b ∈ [a] ⇒ a ∈[b]
Here relation R is reflexive since 1 + a × a > 0 ∀ real
2. Two equivalence classes are either disjoint or identical
numbers a. It is symmetric since 1 + ab > 0 ⇒ 1 +
ba > 0. However R is not transitive: consider three real
Solved Examples 1
numbers 2, – and –2. We have
6
24. Let n be a fixed positive integer. Let a relation R be ⎛ 1⎞ 2
defined on I (the set of all integers) as follows: a R b 1 + 2 × ⎜ - ⎟ = > 0
⎝ 6⎠ 3
if n/(a – b), that is, if a – b is divisible by n, then, the
relation R is ⎛ 1⎞ 4
and 1 + ⎜ - ⎟ (– 2) = > 0
(A)  reflexive only ⎝ 6⎠ 3
(B)  symmetric only ⎛ 1⎞ ⎛ 1⎞
(C)  transitive only Hence, 2 R ⎜ - ⎟ and ⎜ - ⎟ R (– 2)
⎝ 6⎠ ⎝ 6⎠
(D)  an equivalence relation
But 2 R – 2 since 1 + 2 (– 2) = – 3 >/ 0
Solution: (D)
R is reflexive since for any integer a we have a – a = 0 27. Let R be a relation defined as a R b if | a | ≤ b. Then,
and 0 is divisible by n. Hence, a R a ∀ a ∈ I. relation R is
R is symmetric, let a R b. Then by definition of R, (A) Reflexive (B) Symmetric
a – b = nk where k ∈ I. Hence, b – a = (–k) n where (C)  Transitive (D)  None of these
–k ∈ I and so b R a. Thus we have shown that Solution: (C)
a R b  ⇒  b R a R is not reflexive, if – a is any negative real number,
R is transitive, let a R b and b R c. Then by definition then | – a | > – a so that – a R – a. R is not symmetric
of R, we have a – b = k1n and b – c = k2n, where k1, k2 consider the real numbers a = – 2 and b = 3. Then a R
∈ I. It then follows that b since |– 2 | < 3. But b R a since | 3 | > – 2.
R is transitive: let a, b, c be three real numbers such
a – c = (a – b) + (b – c) = k1n + k2n = (k1 + k2)n that
where k1 + k2 ∈ I | a | ≤ b and | b | ≤ c.
25. Let R be a relation defined as a R b if | a – b | > 0. Then, But | a | ≤ b ⇒ b ≥ 0, and so | b | ≤ c ⇒ b ≤ c.
the relation R is It follows | a | ≤ c.
(A) Reflexive (B) Symmetric Thus a R b and b R c ⇒ a R c.
(C)  Transitive (D)  None of these
28. N is the set of natural numbers. The relation R is
Solution: (B) defined on N × N as follows (a, b) R (c, d) ⇔ a + d = b
R is not reflexive since | a – a | = 0 and so | a – a | 0. + c. Then, R is
Thus a R a for any real number a. (A)  Reflexive only (B)  Symmetric only
(C)  Transitive only (D)  an equivalence relation

Objective_Maths_JEE Main 2017_Ch 1.indd 11 01/01/2008 03:14:13


1.12  Chapter 1

Solution: (D) Solution: (B)
We have, (a, b) R (a, b) for all (a, b) ∈ N × N since a + The relation ‘less than’ is only transitive because
b = b + a.
x < y, y < z ⇒ x < z, x, y, z ∈ N
Hence, R is reflexive.
R is symmetric: we have \ x R y, y R z ⇒ x R z
(a, b) R (c, d) ⇒ a + d = b + c ⇒ d + a = c + b 31. If R and R′ are symmetric relations (not disjoint) on a
set A, then the relation R ∩ R′ is
⇒ c + b = d + a ⇒ (c, d) R (a, b)
(A) reflexive (B) symmetric
R is transitive: let (C)  transitive (D)  None of these
(a, b) R (c, d) and (c, d) R (e, f ). Solution: (B)
Then by definition of R, we have Since R ∩ R′ are not disjoint, there is at least one
ordered pair, say, (a, b) in R ∩ R′.
a + d = b + c and c + f = d + e,
But (a, b) ∈ R ∩ R′ ⇒ (a, b) ∈ R and (a, b) ∈ R′
⇒ a + d + c + f = b + c + d + e since R and R′ are symmetric relations, we get
or a + f = b + e. (b, a) ∈ R and (b, a) ∈ R′
Hence, (a, b) R (e, f ) and consequently (b, a) ∈ R ∩ R′
similarly if any other ordered pair (c, d) ∈ R ∩ R′,
Thus, (a, b) R (c, d) and (c, d) R (e, f ) then we must also have, (d, c) ∈ R ∩ R′
⇒ (a, b) R (e, f ) Hence, R ∩ R′ is symmetric
32. With reference to a universal set, the inclusion of a
29. Let S = {1, 2, 3, 4, 5} and let A = S × S. Define the subset in another, is relation which is
relation R on A as follows (a, b) R (c, d) if and only if
(A) symmetric only (B) equivalence
ad = cb. Then, R is
(C)  reflexive only (D)  None of these
(A)  reflexive only
(B)  symmetric only Solution: (C)
(C)  transitive only Let the universal set be
(D)  equivalence relation
U = {x1, x2, x3 …xn}
Solution: (D) We know every set is a subset of itself. Therefore,
Given that S = {1, 2, 3, 4, 5} and A = S × S inclusion of a subset is reflexive Now the elements of
A relation R on A is defined as follows: “(a, b) R the set {x1} are included in the set {x1, x2} but con-
(c, d)” if and only if ad = cb verse is not true i.e.,
(A) R is reflexive, since ab = ba {x1} ⊂ {x1, x2} but {x1, x2} ⊄ {x1}
⇒  ba = ab, therefore,
Hence, the inclusion of a subset is not symmetric.
(a, b) R (b, a) ∀ a, b ∈ S
Thus, the inclusion of a subset is not an equiva-
(B) R is symmetric,
lence relation.
since (a, b) R (c, d)
⇒  ad = cb ⇒ cd = da 33. Let R be the relation on the set R of all real numbers
⇒ (c, d) R (a, b) ∀ a, b ∈ S defined by a R b if | a – b | ≤ 1. Then R is
(C) R is transitive (a, b) R (c, d) and (c, d) R (e, f ) (A) reflexive (B) symmetric
⇒  ad = cb and cf = ed ⇒ adcf = cb ed (C) transitive (D) anti-symmetric
⇒  cd (af ) = cd (be) ⇒ af = eb
⇒ (a, b) R (e, f ) ∀ a, b, c, d, e, f ∈ S Solution:  (A and B)

30. The relation ‘less than’ in the set of natural numbers is | a – a | = 0 < 1 so a R a ∀ a ∈ R.
(A)  only symmetric \ R is reflexive
(B)  only transitive a R b ⇒ | a – b | ≤ 1 ⇒ | b – a | ≤ 1 ⇒ b R a.
(C)  only reflexive
\ R is symmetric
(D)  equivalence relation

Objective_Maths_JEE Main 2017_Ch 1.indd 12 01/01/2008 03:14:14


Set Theory  1.13

2 R 3/2, 3/2 R 2 but 2 ≠ 3/2 so R is not anti-symmetric. z1 - z2


Now, z1 R z2  ⇒  is real
Finally, 1 R2, 2 R 3 but 1 R 3 as z1 + z2
| 1 – 3 | = 2 > 1. ( a1 - a2 ) + i (b1 - b2 )
⇒ is real
34. If R is a relation ‘<’ from A = {1, 2, 3, 4} to B = {1, 3, ( a1 + a2 ) + i (b1 + b2 )
5} i.e., (a, b) ∈ R if a < b, then R o R–1 is ( a - a2 ) + i (b1 - b2 ) ( a1 + a2 ) - i (b1 + b2 )
⇒ 1 ×
(A)  [(1, 2), (1, 5), (2, 3) (2, 5) (3, 5), (4, 5)] ( a1 + a2 ) + i (b1 + b2 ) ( a1 + a2 ) - i (b1 + b2 )
(B)  [(3, 1), (5, 1), (5, 2), (5, 3), (5, 4)] is real
(C)  [(3, 3), (3, 5), (5, 3), (5, 5)]
(D)  [(3, 3), (3, 4), (4, 5)] ( a1 - a2 ) ( a1 + a2 ) + (b1 - b2 ) (b1 + b2 )
+ i [(b - b2 ) ( a1 + a2 ) - (b1 + b2 ) ( a1 - a2 )]
Solution: (C) ⇒  1
( a1 + a2 ) 2 + (b1 + b2 ) 2
Here is real
R = [(1, 3), (1, 5), (2, 3), (2, 5), (3, 5), (4, 5)]
⇒ (a1+ a2) (b1 – b2) – (a1 – a2) (b1 + b2) = 0
\ R–1 = [(3, 1), (5, 1), (3, 2), (5, 2), (5, 3), (5, 4)]
⇒ 2a2b1 – 2b2a1 = 0
Hence, a1 b1 a a
R 0 R–1 = [(3, 3), (3, 5), (5, 3), (5, 5)] ⇒ =  or  1 = 2
a2 b2 b1 b2

35. A relation R on the set of complex numbers is defined a2 a3
z - z2 and =
by, z1 R z2 ⇔ 1 is real, then the relation R is b2 b3

z1 + z2 Similarly,
(A) equivalence (B) only reflexive a2 a3
(C)  only transitive (D)  None of these z2 R z3  ⇒  =
b2 b3

Solution: (A) Therefore,
z - z1
Since 1 = 0, which is real ∀ z1 ∈ C, therefore z1 R z2 and z2 R z3
z1 + z1
R is reflexive. a1 a2
⇒ =
z - z2 b1 b2
For z1, z2 ∈ C, z1 R z2  ⇒  1 is real 
z1 + z2 a2 a3
and =
⎛ z - z2 ⎞ ⎛ z - z1 ⎞ b2 b3
⇒ – ⎜ 1 is real  ⇒  ⎜ 2 is real 

⎝ z1 + z2 ⎠ ⎝ z2 + z1 ⎟⎠ a1 a3
⇒ =
⇒ z2 R z1. b1 b3

For ⇒ z1 R z3
z1, z2, z3 ∈ C,
\ R is transitive.
let z1 = a1 + ib1, z2 = a2 + ib2 and z3 = a3 + ib3. Hence R is an equivalence relation.

EXERCISES

Single Option Correct Type

1. Let F1 be the set of all parallelograms, F2 the set of 2. (i) Let R be the relation on the set R of all real numbers
rectangles, F3 the set of rhombuses, F4 the set of 1
defined by setting a R b if | a – b | ≤ . Then R is
squares and F5 the set of trapeziums in a plane then 2
F1is equal to (A)  Reflexive and symmetric but not transitive
(A) F2 ∩ F3 (B)  F2 ∪ F3 ∪ F4 ∪ F1 (B)  Symmetric and transitive but not reflexive
(C) F3 ∩ F4 (D)  None of these (C)  Transitive but neither reflexive nor symmetric
(D)  None of these

Objective_Maths_JEE Main 2017_Ch 1.indd 13 01/01/2008 03:14:17


1.14  Chapter 1

3. n/m means that n is a factor of m, then the relation ‘/’ is 12. If A = { x : x2 = 1} and B { x : x4 = 1}, then A D B is
(A)  reflexive and symmetric. equal to:
(B)  transitive and reflexive. (A) {i, – i} (B)  {–1, 1}
(C)  reflexive, transitive and symmetric. (C)  {–1, 1, i,–i} (D)  None of these
(D)  reflexive, transitive and not symmetric.
13. Which of the following is a singleton set?
4. Set A and B have 3 and 6 elements respectively. What (A) {x : | x | < 1, x ∈ Z}
can be the minimum number of elements in A ∪ B? (B) {x : | x | = 5, x ∈Z}
(A) 18 (B) 9 (C) 6 (D) 3 (C) {x : x2 = 1, x ∈Z}
5. Let R be a relation defined on the set of natural num- (D) {x : x2 + x + 1 = 0, x ∈R}
bers N as 14. Consider the following relations:
R = [(x, y): x ∈ N, y ∈ N, 2x + y = 41]. Then (1) A– B = A – (A ∩ B)
(A)  Domain of R = {1, 2, 3, …, 19, 20} (2) A = (A ∩ B) ∪ (A– B)
(B)  Range of R = {39, 37, 35, 9, 7, 5, 3, 1} (3) A – (B ∪ C) = (A – B) ∪ (A – C)
(C) R is reflexive Which of these is/are correct?
(D) R is symmetric (A) 1 and 3 (B)  2 only
6. Let A = {x: x ∈ R, | x | < 1} (C)  2 and 3 (D)  1 and 2
B = {x: x ∈ R, | x – 1 | ≥ 1} 15. Let R be a reflexive relation on a finite set A having n
and  A ∪ B = R – D, then the set D is elements, and let there be m ordered pairs in R. Then
(A) {x: 1 < x ≤ 2} (B)  {x: 1 ≤ x < 2} (A) m ≥ n (B)  m≤n
(C) {x: 1 ≤ x ≤ 2} (D)  None of these (C) m = n (D)  None of these
7. Consider the set A of all determinants of order 3 with 16. If two sets A and B are having 99 elements in common
entries 0 or 1 only. Let B be subset of A consisting of then the number of elements common to each of the
all determinants with value 1. Let C be the subset A of sets A × B and B × A are
consisting of all determinants with value –1. Then (A) 299 (B) 992 (C) 100
(A) C is empty. (D) 18 (E) 9
(B) B has as many elements as C.
(C) A = B ∪ C. 17. The relation R defined on the set A = [1, 2, 3, 4, 5] by
(D) B has twice as many elements as C. R = [(x, y): |x2 – y2| < 16] is given by,
(A)  [(1, 1), (2, 1) (3, 1), (4, 1), (2, 3)]
8. Let A and B be two sets then (A ∪ B)′ ∪ (A′ ∩ B) is (B)  [(2, 2), (3, 2) (4, 2), (2, 4)]
equal to (C)  [(3, 3), (3, 4) (5, 4), (4, 3), (3, 1)]
(A) B′ (B)  B (C)  A (D)  A′ (D)  None of these
9. If A is the set of even natural numbers less than 8 and 18. Let L denotes the set of all straight lines in a plane. Let
B is the set of prime numbers less then 7, then the a relation R be defined by a R b ⇔ a ^ b, a, b ∈ L.
number of relations from A to B is Then R is
(A) 29 (b) 92 (C) 32 (D) 29 – 1 (A) reflexive
10. If P, Q and R are subsets of a set A, then R × (P ′ ∪ Q ′)′ (B) symmetric
equals (C) transitive
(D)  None of these
(A) (R × P) ∩ (R × Q) (B) (R × Q) ∩ (R × P)
(C) (R × P) ∪ (R × Q) (D)  None of these 19. Let R = [(2,3), (3,4)] be a relation defined on the set
of natural numbers. The minimum number of ordered
11. For real numbers x and y, define a relation R, x R y if
pairs required to be added in R so that enlarged rela-
and only if x – y + 2 is an irrational number. Then the
tion be comes an equivalence relation is
relation R is
(A) 3 (B) 5 (C) 7 (D) 9
(A) reflexive
(B) symmetric 20. The solution of 8x ≡ 6 (mod 14) is
(C) transitive (A)  [8], [6] (B)  [6], [14]
(D)  an equivalence relagtion (C)  [6], [13] (D)  [8], [14], [16]

Objective_Maths_JEE Main 2017_Ch 1.indd 14 01/01/2008 03:14:17


Set Theory  1.15

21. Let R be the set of real numbers. 22. Let X = {1, 2, 3, 4, 5}. The number of different ordered
Statement 1: A = {(x, y) ∈ R × R : y – x is an integer} pairs (Y, Z) that can formed such that Y ⊆ X, Z ⊆ X and
is an equivalence relation of R. Y ∩ Z is empty, is
Statement 2: B = {(x, y) ∈ R × R : x = a y for some (A) 52 (B) 35 (C) 25 (D) 53
rational number a} is an equivalence relation of R.
23. Let R be the real line. Consider the following subsets
(A) Statement 1 is false, Statement 2 is true
of the plane R × R.
(B) Statement 1 is true, Statement 2 is true; Statement 2
is a correct explanation for Statement 1 S = [(x, y) : y = x + 1 and 0 < x < 2], T = [(x, y): x – y is
(C) Statement 1 is true, Statement 2 is true; Statement 2 an integer]. Which one of the following is true?
is not a correct explanation for Statement 1 (A) Neither S nor T is an equivalence relation on R
(D)  Statement 1 is true, Statement 2 is false (B) Both S and T are equivalence relations on R
(C) S is an equivalence relation on R but T is not
(D) T is an equivalence relation on R but S is not

Previous Year’s Questions

24. Let R = {(1, 3), (4, 2), (2, 4), (2, 3), (3, 1)} be a relation S = {(x, y) : y = x + 1 and 0 < x < 2}, T = {(x, y) : x - y
on the set A = {1, 2, 3, 4}. The relation R is [2004] is an integer}. Which one of the following is true?
(A) a function (B) reflexive  [2008]
(C) not symmetric (D) transitive (A) neither S nor T is an equivalence relation on R
(B) both S and T are equivalence relations on R
25. Let R = {(3, 3), (6, 6), (9, 9), (12, 12), (6, 12), (3, 9), (C) S is an equivalence relation on R but T is not
(3, 12), (3, 6)} be a relation on the set A = {3, 6, 9, 12} (D) T is an equivalence relation on R but S is not
be a relation the set A = {3, 6, 9, 12}. The relation is
 [2005] 29. If A, B and C are three sets such that A ∩ B = A ∩ C
(A)  reflexive and transitive only and A ∪ B = A ∪ C, then [2009]
(B)  reflexive only (A) A = B (B)  A=C
(C)  an equivalence relation (C) B = C (D)  A∩B=f
(D)  reflexive and symmetric only 30. Let S be a non-empty subset of R. Consider the follow-
26. Let W denote the words in the English dictionary. ing statement: [2010]
Define the relation R by: [2006] P: There is a rational number x ∈ S such that x > 0.
R = {(x, y) ∈ W × W | the words x and y have at least Which of the following statements is the negation of
one letter in common}. Then R is the statement P ?
(A)  not reflexive, symmetric and transitive (A) There is no rational number x ∈ S such that x ≤ 0
(B)  reflexive, symmetric and not transitive (B)  Every rational number x ∈ S satisfies x ≤ 0
(C)  reflexive, symmetric and transitive (C) x ∈ S and x ≤ 0 ⇒ x is not rational
(D)  reflexive, not symmetric and transitive (D)  There is a rational number x ∈ S such that x ≤ 0

27. The set S = {1, 2, 3, …, 12) is to be partitioned into 31. Let R be the set of all real numbers. [2011]
three sets A, B, C of equal size. Thus, A ∪ B ∪ C = S, Statement 1: A = {(x, y) ∈R × R : y − x is an integer}
A ∩ B = B ∩ C = A ∩ C = f. The number of ways to is an equivalence relation on R.
partition S is [2007] Statement 2: B = {(x, y) ∈ R × R : x = α y for some
12 ! 12 ! rational number α} is an equivalence relation on R.
(A)  (B)  (A) Statement 1 is true, Statement 2 is true; Statement 2
3!( 4 !)3 3!(3!) 4
is not a correct explanation for Statement 1
12 ! 12 ! (B)  Statement 1 is true, Statement 2 is false.
(C)  3
(D) 
( 4 !) (3!) 4 (C)  Statement 1 is false, Statement 2 is true.
28. Let R be the real line. Consider the following subsets (D) Statement 1 is true, Statement 2 is true; Statement 2
of the plane R × R. is a correct explanation for Statement 1

Objective_Maths_JEE Main 2017_Ch 1.indd 15 01/01/2008 03:14:18


1.16  Chapter 1

32. Let A and B be two sets containing 2 elements and 4 33. Let A and B be two sets containing four and two
elements respectively. The number of subsets of A × B ­elements respectively. Then the number of subsets of
having 3 or more elements is [2013] the set A × B, each having at least three elements is:
(A) 220 (B) 219  [2015]
(C) 211 (D) 256 (A) 256 (B) 275 (C) 510 (D) 219

Answer keys

Single Option Correct Type


1. (B) 2. (A) 3. (B) 4. (C) 5.  (A and B) 6. (B) 7. (B) 8. (D) 9. (A)
10. (A) 11. (A) 12. (A) 13. (A) 14. (D) 15. (A) 16. (B) 17. (D) 18. (B) 19. (C)
20. (C) 21. (D) 22. (B) 23. (D)

Previous Year’s Questions


24. (C) 25. (A) 26. (B) 27. (C) 28. (D) 29. (C) 30. (B) 31. (B) 32. (B) 33. (D)

Hints and Solutions

Single Option Correct Type


1. Since every rectangle, rhombus and square is a parallelo- 5. The relation R can be written as
gram so, F1 = F2 ∪ F3 ∪ F4∪ F1 R = [(1, 39), (2, 37), (3, 35), …(10, 21), (11, 19), … (19, 3),
The correct option is (B) (20, 1)]
1
2. R is reflexive since | a – a | = 0 < for all a ∈ R. \ Domain of R = {1, 2, 3, …, 19, 20}
2
1 1 Range of R = {39, 37, 35, 9, 7, 5, 3, 1}
R is symmetric since | a – b | < ⇒|b–a|< R is not reflexive since (x, x) ∉ R ∀ x ∈ N.
2 2
R is not transitive: we take three numbers For example, (1, 1) ∉ R
3 1 1 R is not symmetric since (1, 39) ∈ R but (39, 1) ∉ R.
, , , then
4 3 8 The correct option is (A and B)
3 1 5 1 1 1 5 1 6. We have
- = < and - = < A = {x : x ∈ R, – 1 < x < 1}
4 3 12 2 3 8 24 2
and  B = {x : x ∈ R, x – 1 ≤ – 1 or x – 1 ≥ 1}
3 1 5 1

But - = > = {x : x ∈ R, x ≤ 0 or x ≥ 2}
4 8 8 2
\ A ∪ B = R – D,
3 1 1 1 3 1 where D = {x : x ∈ R, 1 ≤ x < 2}

Thus R and R but R
4 3 3 8 4 8 The correct option is (B)
The correct option is (A) 7. We know that the interchange of two adjacent rows (or
3. ‘/’ is reflexive since every natural number is a factor of itself, ­columns) changes the value of a determinant only in sign and
that is n/n for n ∈ N. ‘/’ is transitive: if n is a factor of m not in magnitude. Hence corresponding to every element D
and m is a factor of p, then n is surely a factor of p. Thus of B there is an element D′ in C obtained by interchanging
‘n/m’ and ‘m/p’ ⇒ ‘n/p’. However ‘/’ is not symmetric: for two adjacent rows (or columns) in D. It follows that
­example, 2 is factor of 4 but 4 is not a factor of 2. n (B) ≤ n (C)
The correct option is (B) that is, the number of elements in B is less than or equal to
4. n (A ∪ B) = n (A) + n (B) – n (A ∩ B) the number of elements in C.
= 3 + 6 – n (A ∩ B) Similarly,  n (C) ≤ n (B)
Since maximum number of elements in A ∩ B = 3, Hence,  n (B) = n (C)
\ minimum no. of elements in A ∪ B = 9 – 3 = 6. That is, B has as many elements as C.
The correct option is (C) The correct option is (B)

Objective_Maths_JEE Main 2017_Ch 1.indd 16 01/01/2008 03:14:19


Set Theory  1.17

8. (A ∪ B)′ ∪ (A′ ∩ B) = (A′ ∩ B′) ∪ (A′ ∩ B) 17. Here, R = {(x, y) : |x2 – y2| < 16} and A = { 1, 2, 3, 4, 5}
= (A′ ∪ A′) ∩ (A′ ∪ B) ∩ (B′ ∪ A′) ∩ (B′ ∪ B)
\ R = [(1,2), (1,3), (1,4); (2,1), (2,2), (2,3), (2,4); (3,1),
= A′ ∩ [A′ ∪ (B ∩ B′)] ∩ U
(3,2) (3,3), (3,4); (4,1), (4,2), (4,3); (4,4), (4,5); (5,4), (5,5)]
The correct option is (D)
= A′ ∩ (A′ ∪ f) ∩ U

18. Here a ^ b ⇒ b ^ a. Hence, R is symmetric.
= A′ ∩ A′ ∩ U = A′ ∩ U = A′
The correct option is (B)
The correct option is (C)
19. To make it reflexive, we need to add (2, 2), (3, 3), (4,4).
9. A = {2,4,6}, B = {2,3,5}
To make symmetric, it requires (3, 2), (4, 3) to be added.
Number of relations from A to B = 23×3 = 29 To make transitive, (2, 4) and (4, 2) must be added, so, the
The correct option is (A) relation.
10. R × (P′ ∪ Q′)′ = R × [(P′)′ ∩ (Q′)′] R = [(2,2), (3,3), (4,4) (2,3),(3, 2), (3, 4), (4,3), (2,4),(4,2)]
= R × (P ∩ Q) = (R × P) ∩ (R × Q) The correct option is (C)
The correct option is (A) 20. Since 8x ≡ 6 (mod 14) i.e, 8x –6 = 14 k for k ∈ I.
11. Clearly x R x as x – x + 2 = 2 is an irrational number. The values 6 and 13 satisfy this equation, while 8, 14, and 16
Thus, R is reflexive. Also ( 2 , 1) ∈ R as 2 –1 + 2 = 2 do not.
2 – 1 is an irrational number but (1, 2 ) ∉ R as 1– 2 + The correct option is (C)
2 = 1 is a rational number. So, R is not symmetric, 21. Statement 1 is true

1R 2 and 2 2 R 2 We observe that
Since, 1 R 2 and 2 R 2 but 1 is not related to 2 . So R is Reflexivity: xRx as x – x = 0 is an integer, ∀ x ∈ A
not transitive. Symmetric
The correct option is (A) Let (x, y) ∈ A
12. A = {–1, 1}, B = {–1, 1, –i, i} ⇒ y – x is an integer
A – B = f, B–A= {–i, i} ⇒ x – y is also an integer
(A – B) ∪ (B – A) ={–i, i} Transitivity: Let (x, y) ∈ A and (y, z) ∈ A
The correct option is (A) ⇒ y – x is an integer and z – y is an integer
13. | x | < 1 ⇒ –1 < x < 1  (x = 0 integer satisfies it) ⇒ y – x + z – y is also an integer
The correct option is (A) ⇒ z – x is an integer
14. A – B = A – (A ∩ B) is correct ⇒ (x, z) ∈ A
A = (A ∩ B) ∪ (A - B) is correct Because of the above properties A is an equivalence relation
over R
A B
Statement 2 is false as ‘B’ is not symmetric on 
We observe that
0Bx as 0 = 0 ⋅ x∀ x ∈  but (x, 0) ∉ B
The correct option is (D)
22. Every element has 3 options. Either set Y or set Z or none, so
number of ordered pairs = 35.
A–B A–(A ∩ B) The correct option is (B)
(3) is false. 23. T = {(x, y) : x – y ∈ I}
\ (1) and (2) are true. as 0 ∈ I T is a reflexive relation.
The correct option is (D) If x – y ∈ I ⇒ y – x ∈ I
15. The set consists of n elements and for relation to be reflexive \ T is symmetrical also
it must have at least n ordered pairs. It has m ordered pairs If x – y = I1 and y - z = I2
therefore m ≥ n. Then x – z = (x – y) + (y – z) = I1 + I2 ∈ I
The correct option is (A) \ T is also transitive.
16. n [(A × B) ∩ (B × A)] Hence, T is an equivalence relation.
= n [(A ∩ B) × (B ∩ A)] = n (A ∩ B) ⋅ n (B ∩ A) Clearly  x ≠ x + 1  ⇒ (x, x) ∉ S
= n (A ∩ B) . n (A ∩ B) = (99)(99) = 992 \ S is not reflexive.
The correct option is (B) The correct option is (D)

Objective_Maths_JEE Main 2017_Ch 1.indd 17 01/01/2008 03:14:21


1.18  Chapter 1

Previous Year’s Questions


24. (2, 3) belongs to R but (3, 2) is not. 29. The correct option is (C)
Hence R is not symmetric. 30. P: there is a rational number x ∈ S such that x > 0
The correct option is (C) ~P: Every rational number x ∈ S satisfies x ≤ 0
25. R is Reflexive and transitive only so not an equivalence The correct option is (B)
relation. 31. x − y is an integer
e.g. (3, 3), (6, 6), (9, 9), (12, 12) [Reflexive] x – x = 0 is an integer ⇒ A is Reflexive
(3, 6), (6, 12), (3, 12) [Transitive]. x − y is an integer ⇒ y − x is an integer ⇒ A is symmetric
The correct option is (A) x − y, y − z are integers
26. Clearly (x, x) ∈ R ∀ x ∈ W. So, R is reflexive. As sum of two integers is an integer.
Let (x, y) ∈ R, then (y, x) ∈ R as x and y have at least one ⇒ (x − y) + (y − z) = x − z is an integer
letter in common. So, R is symmetric. ⇒ A is transitive. Hence statement 1 is true.
But R is not transitive for example x
Also = 1 is a rational number ⇒ B is reflexive
Particularly if x = DELHI, y = DWARKA and z = PARK, x
x y
then = a is rational ⇒ need not be rational
x x
(x, y) ∈ R and (y, z) ∈ R but (x, z) ∉ R. 0 1
The correct option is (B) i.e., is rational ⇒ is not rational
12! 1 0
27. The required number of ways is 12C4 × 8C4 × 4C4 = Hence B is not symmetric
( 4!)3
The correct option is (C) ⇒ B is not an equivalence relation.
28. Given T = {(x, y) : x - y ∈ I} The correct option is (B)
As 0 ∈ I, T is a reflexive relation. 32. A × B will have 8 elements.
If x - y ∈ I ⇒ y - x ∈ I \ The number of subsets of A × B having 3 or more
\ T is symmetrical also ­elements is 28 -8 C0 -8 C1 -8 C2 = 256 - 1 - 8 - 28 = 219
If x - y = I1 and y - z = I2 The correct option is (B)
Then x - z = (x - y) + (y - z) = I1 + I2 ∈ I 33. n(A) = 4, n(B) = 2
\ T is also transitive. n(A × B) = 8
Hence T is an equivalence relation. \ Number of subsets having atleast 3 elements


Clearly x ≠ x + 1 ⇒ (x, x) ∉ S
\ S is not reflexive.

( )
= 28 - 1 + 8 C1 + 8 C2 = 219

The correct option is (D)
The correct option is (D)

Objective_Maths_JEE Main 2017_Ch 1.indd 18 01/01/2008 03:14:22


CHaPtER

2 Functions

Chapter Highlights
Function or mapping, Features of a mapping f : X → Y, Value of a function, Domain and range of a function,
Intervals in R, Method to find the domain of a function, Method to find the range of a function, Types of
functions, Method to check whether the function f : X → Y is one–one or many-one, Onto or surjective function,
Method to check whether the function f : X → Y is onto or Into, Bijective function, Some important functions,
Identity function, Modulus function or absolute value function, Greatest integer function/step function/floor
function, Least integer function/ceiling function, Fractional-part function, Signum function, Reciprocal function,
Exponential function, Logarithmic function, Polynomial function, Rational function, Trigonometric functions,
Inverse trigonometric functions, Two ways of defining a function, Explicit and implicit functions, Operations on
functions, Composition of functions, Properties of composite functions, Inverse functions, Method to find the
inverse of a function, Properties of inverse functions, Odd and even functions, Properties of odd and even
functions, Periodic function, Short-cut method to check the periodicity of a function.

Function oR Mapping FeatuReS oF a Mapping F : X → Y


Let X and Y be any two non-empty sets and there be 1. For each element x ∈ X, there exist a unique element
correspondence or association between the elements of y ∈ Y.
X and Y such that for every element x ∈ X, there exists a 2. The element y ∈ Y is called the image of x under the
unique element y ∈ Y, written as y = f (x). Then we say that mapping f.
f is a mapping or function from X to Y, and is written as 3. If there is an element in X which has more than one
image in Y, then f : X ∈ Y is not a function. But distinct
f : X → Y such that y = f (x), x ∈ X, y ∈ Y
elements of X may be associated to the same element
X Y of Y.
4. If there is an element in X which does not have an
image in Y, then f : X → Y is not a function.
f

x y = f (x)
caution
 If f : X → Y is a function, then there may be some ele-
ments in Y, which are not images of elements of X.
i M P o R ta n t P o i n t S  But there should not be any x left (element of X) for
which there is no elements in set Y.
 If f : X → Y be a function from a non-empty set X to
another non-empty set Y, where X, Y ⊆ R (set of all real
numbers), then we say that f is a real valued function
or in short a real function. value oF a Function
Throughout this chapter a ‘function’ will mean a ‘real

The value of a function y = f (x) at x = a is denoted by f (a).
function’.
It is obtained by putting x = a in f (x).

Objective_Maths_JEE Main 2017_Ch 2.indd 1 01/01/2008 03:18:01


2.2 Chapter 2

notE
Range
0
 If for some value of x say x = a, f(a) takes the form ,
we say that f(a) is indeterminate. 0 Domain Co-domain
 If for some value of x say x = a, the denominator vanishes,
we say that f (a) is undefined (or does not exist) A B
f
a p Domain = {a, b, c, d} = A
b q
Co-domain = {p, q, r, s} = B
Solved exaMple c r
Range = {p, q, r}
d s
x −1 2
1. If f (x) = , for every real number x, then the Fig. 2.2
x2 + 1
minimum value of f
(A) does not exist because f is unbounded notE
(B) is not attained even though f is bounded
(C) is equal to 1  Range is the subset of co-domain
(D) is equal to –1  Range can never exceed co-domain for a given function.
Solution: (D)  The projection of the graph of y = f(x) on the x-axis is
equal to the domain of f whereas the projection on the
We have, y-axis is equal to the range of f.
x2 −1 2
f (x) ==1– 2 .
x +1
2
x +1
2
f (x) will attain its minimum value when is tRick(S) FoR pRobleM Solving
x2 + 1
maximum, i.e. when x + 1 is minimum i.e. at x = 0.
2

\ minimum value of f (x) is f (0) = –1. The total number of functions from set A to set B containing
m and n elements respectively is nm.

doMain and Range oF a Function


inteRvalS in R
If f : X → Y be a function, then the set X is said to be the
domain of f and range of f The set of all numbers lying between two given real
= set of all image points in Y under the map f. numbers is called an interval in R.
Let a and b be any two real numbers such that a < b,
= f (X) = { f (x) : f (x) ∈ Y; x ∈ X}
then we define the following types of intervals.
The set Y is also called the co-domain of f. Clearly f (X) ⊆ Y
1. Closed interval [a, b]
as shown in Fig. 2.1.
= closed interval from a to b
Y
= {x : x ∈ R; a ≤ x ≤ b}
X f (x)
= set of all real numbers lying between a and b includ-
f Range ing the end points a and b.

x y = f (x)
a b
Domain
2. Open interval (a, b) or ]a, b[
Co-domain = open interval from a to b
Fig. 2.1
= {x : x ∈ R; a < x < b}
= set of all real numbers lying between a and b, exclud-
In other words, we can say ing the end points a and b.
Domain = All possible values of x for which f(x)
exists.
Range = For all values of x, all possible values of f(x) a b
as shown in Fig. 2.2.

Objective_Maths_JEE Main 2017_Ch 2.indd 2 01/01/2008 03:18:03


Functions 2.3

3. Closed-open interval [a, b) and open–closed interval logarithmic Functions


(a, b]
1. logb a is defined when a > 0, b > 0 and b ≠ 1.

exponential Functions
a b a b
Fig. 2.3
1. ax is defined for all real values of x, where a > 0.

[a, b) = {x : x ∈ R; a ≤ x < b} notE


(a, b] = {x : x ∈ R; a < x ≤ b}
4. Real number set R as an open interval  (x – a) (x – b) > 0 ⇒ x < a or x > b, for a < b
We introduce two special numbers –∞ and +∞, where  (x – a) (x – b) < 0 ⇒ a < x < b, for a < b
–∞ = a number less than any real number,  |x| < a ⇒ – a < x < a
+∞ = a number greater than any real number.
 |x| > a ⇒ x < – a or x > a
–∞ < x for all x ∈ R, and x < ∞ for all x ∈ R.
⎧a > b k , if b > 1
Hence, the set R can be thought of as the open interval  logb a > k ⇒ ⎪⎨
(–∞, ∞), so that ⎩⎪a < b , if b < 1
k

If loga x > loga y, then if a > 1 then x > y and if 0 < a < 1
R = (–∞, ∞) = {x : x ∈ R; –∞ < x < ∞} 

then x < y
Also, the infinite intervals in R can be given by,  x 2 = |x|
(–∞, a), (a, + ∞), (–∞, a], [a, + ∞) 
n
x n = |x|, if n is even and n
x n = x, if n is odd.

notE
Solved exaMpleS
The numbers +∞ and –∞ do not follow the ordinary rules of
0 ∞ 2. The domain of the function f (x) = x − 1 − x 2 is
arithmetic, i.e., ∞ – ∞ ≠ 0, 0 × ∞ ≠ 0, ≠ 1, ≠ 1, ∞ + ∞
0 ∞
≠ 2 ∞, 1∞ ≠ 1, 00 ≠ 1 etc. ⎡ 1 ⎤ ⎡ 1 ⎤
(A) ⎢ −1, − ⎥∪ ⎢ , 1⎥
⎣ 2⎦ ⎣ 2 ⎦
(B) [– 1, 1]
Method to Find the doMain oF a
⎛ 1⎤ ⎡ 1 ⎞
Function (C) ⎜ −∞, − ⎥ ∪ ⎢ , + ∞⎟
⎝ 2⎦ ⎣ 2 ⎠
algebraic Functions ⎡ 1 ⎤
(D) ⎢ , 1⎥
1. Denominator should be non-zero. ⎣ 2 ⎦
2. Expression under the even root should be non-negative.
Solution: (D)
trigonometric Functions For f (x) to be defined, we must have

1. sin x and cos x are defined for all real values of x. x– 1 − x 2 ≥ 0 or x ≥ 1 − x2 > 0
2. tan x and sec x are defined for all real values of x except 1
p \ x2 ≥ 1 – x2 or x2 ≥
x = (2n + 1) , where n ∈ Z. 2
2 Also, 1 – x2 ≥ 0 or x2 ≤ 1
3. cot x and cosec x are defined for all real values of x 1 ⎛ 1 ⎞ ⎛ 1 ⎞
except x = np, where n ∈ Z. Now, x2 ≥ ⇒ ⎜ x − ⎟ ⎜ x+ ⎟ ≥0
2 ⎝ 2⎠ ⎝ 2⎠
1 1
inverse trigonometric Functions ⇒ x≤– or x ≥
2 2
1. sin x and cos x are defined for – 1 ≤ x ≤ 1.
–1 –1
Also, x ≤ 1 ⇒ (x – 1) (x + 1) ≤ 0 ⇒ – 1 ≤ x ≤ 1
2
2. tan–1x and cot–1x are defined for all real values of x.
3. sec–1x and cosec–1x are defined for x ≤ – 1 or x ≥ 1. 1 ⎡ 1 ⎤
Thus, x > 0, x2 ≥ and x2 ≤ 1 ⇒ x ∈ ⎢ , 1⎥
2 ⎣ 2 ⎦

Objective_Maths_JEE Main 2017_Ch 2.indd 3 01/01/2008 03:18:07


2.4  Chapter 2

3. The domain of the function 1 25


⇒ ≤ x ≤ (1)
⎧ ⎛ 1 ⎞⎫ 3 4
(x) = sin–1 ⎨log 2 ⎜ x 2 ⎟ ⎬ is
f  40–6x
C8x–10 is defined if
⎩ ⎝ 2 ⎠⎭
(A)  [– 2, – 1) ∪ [1, 2] (B)  (– 2, – 1] ∪ [1, 2] 40 – 6x > 0, 8x – 10 ≥ 0 and 40 – 6x ≥ 8x – 10
(C)  [– 2, – 1] ∪ [1, 2] (D)  (– 2, – 1) ∪ (1, 2) 20 5 25
⇒ x< ,x≥ and x ≤ 
Solution: (C) 3 4 7
For f (x) to be defined, we must have 5 25
⇒ ≤x≤ (2)
4 7
⎛1 ⎞ 1
– 1 ≤ log2 ⎜ x 2 ⎟ ≤ 1 ⇒ 2–1 ≤ x2 ≤ 21 From Eq. (1) and (2), we get
⎝2 ⎠ 2
5 25
[Q the base = 2 > 1] ≤x≤
⇒ 1 ≤ x2 ≤ 4 (1) 4 7
But, 24 – x ∈ N,
Now, 1 ≤ x2 ⇒ x2 – 1 ≥ 0 i.e. (x – 1) (x + 1) ≥ 0 \ x must be an integer,
⇒ x ≤ – 1 or x ≥ 1 (2)
\ x = 2, 3.
Also, x2 ≤ 4 ⇒ x2 – 4 ≤ 0 i.e. (x – 2) (x + 2) ≤ 0 Hence, domain ( f ) = {2, 3}.
⇒ – 2 ≤ x ≤ 2 (3) ⎛ 1 − | x |⎞
6. The domain of the function f (x) = cos −1 ⎜ is
From Eq. (2) and (3), we get the domain of f ⎝ 2 ⎟⎠
= ((– ∞, – 1] ∪ [1, ∞)) ∩ [– 2, 2] (A) (– ∞, – 3) ∪ (3, ∞)
(B)  [– 3, 3]
= [– 2, – 1] ∪ [1, 2].
(C) (– ∞, – 3] ∪ [3, ∞)
4. The domain of the function (D) f
1
f (x) = is Solution: (B)
x − x + x4 − x + 1
12 9
⎛ 1 − | x |⎞ 1 −| x |
cos–1 ⎜ is defined if – 1 ≤ ≤1
(A) (– ∞, – 1) (B)  (1, ∞) ⎝ 2 ⎟⎠ 2
(C)  (– 1, 1) (D)  (– ∞, ∞)
⇒ – 2 ≤ 1 – | x | ≤ 2 ⇒ – 3 ≤ – | x | ≤ 1
Solution: (D)
⇒ – 1 ≤ | x | ≤ 3
f (x) is defined for
x12 – x9 + x4 – x + 1 > 0 |x| ≥ – 1 is true for all real values of x.
⇒ x4 (x8 + 1) – x (x8 + 1) + 1 > 0 |x| ≤ 3 ⇒ – 3 ≤ x ≤ 3
⇒ (x8 + 1) x (x3 – 1) + 1 > 0 ⎛ 1 − | x |⎞ ⎛ 1 − | x |⎞
Also cos −1 ⎜ is defined if cos–1 ⎜ ≥0
If x ≥ 1 or x ≤ – 1, then the above expression is positive. ⎝ 2 ⎟⎠ ⎝ 2 ⎟⎠
If – 1 < x ≤ 0, the above inequality still holds. 1− | x |
If 0 < x < 1, then x12 – x (x8 + 1) + (x4 + 1) > 0 ⇒ ≥ cos 0 = 1
2
[Q x4 + 1 > x8 + 1 and so x4 + 1 > x (x8 + 1)]
The domain of f = (– ∞, ∞). ⇒ 1 – |x| ≥ 2

5. The domain of the function ⇒ |x| ≤ – 1  (Absurd)


(x) = 24 – xC3x – 1 + 40 – 6xC8x – 10 is,
f  \ Domain ( f ) = [– 3, 3]
(A)  {2, 3} (B)  {1, 2, 3} 7. The domain of the function
(C)  {1, 2, 3, 4} (D)  None of these
⎛ 1⎞
f  (x) = log1/ 2 ⎜ x − ⎟ + log 2 4 x 2 − 4 x + 5 is
Solution: (A) ⎝ 2⎠
C3x–1 is defined if,
24–x
⎡1 ⎞ ⎛1 ⎞
(A)  ⎢ , ∞⎟ (B)  ⎜⎝ , ∞⎟⎠
24 – x > 0, 3x – 1 ≥ 0 and 24 – x ≥ 3x – 1 ⎣2 ⎠ 2
1 25 (C) (– ∞, ∞) (D)  None of these
⇒ x < 24, x ≥ and x ≤
3 4 

Objective_Maths_JEE Main 2017_Ch 2.indd 4 01/01/2008 03:18:10


Functions  2.5

Solution: (B) 1
is defined if log (3 – x) ≠ 0 and 3 – x > 0
⎛ 1⎞ 1 log (3 − x )
log1/ 2 ⎜ x − ⎟ is defined if x – > 0
⎝ 2⎠ 2 ⇒ 3 – x ≠ e0 = 1 and x < 3 ⇒ x ≠ 2 and x < 3 (2)
1 From Eq. (1) and (2), we get domain of f
i.e., x >
2
= [– 6, 6] ∩ ((– ∞, 3) \ {2}) = [– 6, 3)\{2}.
log 2 4 x 2 − 4 x + 5 is defined if 4x2 – 4x + 5 > 0
11. The domain of the function
⎡⎛ 1⎞
2

⇒ 4 ⎢⎜ x − ⎟ + 1⎥ > 0 which is true for all real x. ⎛ 3 ⎞
f (x) = cos–1 ⎜ is
⎢⎣⎝ 2⎠ ⎥⎦ ⎝ 4 + 2 sin x ⎟⎠
⎛1 ⎞ ⎡ p p ⎤
\ Domain of f = ⎜ , ∞⎟ . (A)  ⎢ − + 2np , + 2np ⎥
⎝2 ⎠
⎣ 6 6 ⎦
8. The domain of the function ⎛ p p ⎞
(B)  ⎜ − + 2np , + 2np ⎟
⎡ ⎛ 16 − x 2 ⎞ ⎤ ⎝ 6 6 ⎠
(x) = cos ⎢log ⎜
f  ⎟ ⎥ is
⎢⎣ ⎝ 3 − x ⎠ ⎥⎦ ⎛ p p ⎞
(C)  ⎜ − + 2np , + 2np ⎟
⎝ 6 6 ⎠
(A)  (– 4, 4) (B)  (– 4, 3)
(C) (– ∞, – 4) ∪ (3, ∞) (D)  None of these ⎡ p p ⎞
(D)  ⎢ − + 2np , + 2np ⎟
Solution: (B) ⎣ 6 6 ⎠
16 − x 2 Solution: (A)
f (x) is defined if >0
3− x 3
f (x) is defined if – 1 ≤ ≤1
⇒ 16 – x2 > 0 and 3 – x > 0 4 + 2 sin x
⇒ (x – 4) (x + 4) < 0 and x < 3 Since, 4 + 2 sin x > 0 for all real x, therefore
3 1
⇒ – 4 < x < 4 and x < 3 or – 4 < x < 3 ≤ 1 ⇒ 3 ≤ 4 + 2 sin x ⇒ sin x ≥ –
4 + 2 sin x 2
\ Domain of f = (– 4, 3).
p p
9. The domain of the function f (x) = log2 log3 log4 x is ⇒ – + 2np ≤ x ≤ + 2np, n ∈ Z
6 6
(A) [4, ∞) (B)  (4, ∞) ⎡ p p ⎤
(C) (– ∞, 4) (D)  None of these Domain of  f = ⎢ − + 2np , + 2np ⎥ .
⎣ 6 6 ⎦
Solution: (B)
f (x) is defined if log3 log4 x > 0, log4 x > 0 and x > 0 1−| x|
12. The domain of definition of f (x) = is
⇒ log4 x > 3° = 1, x > 4° and x > 0 2 −| x|
(A) (–∞, ∞)\[–1, 1]
⇒ x > 41, x > 1 and x > 0 ⇒ x > 4 (B) (–∞, ∞)\[–2, 2]
Domain of f = (4, ∞). (C)  [–1, 1] ∪ (–∞, –2) ∪ (2, ∞)
(D)  None of these
10. The domain of the function
Solution: (C)
⎛ 2 − | x |⎞
−1
1−| x|
f (x) = cos ⎜ + [log (3 − x )]−1 is
⎝ 4 ⎟⎠ f (x) is defined if ≥ 0 and 2 – |x| ≠ 0
2 −| x|
(A)  [– 6, 3)\{2} (B)  [– 6, 2) ∪ (2, 3]
(1 − | x |) ( 2 − | x |)
(C)  [– 6, 3] (D)  [– 6, 3) ⇒ ≥ 0 and x ≠ –2, 2
( 2 − | x |) 2
Solution: (A)
⇒ (|x| – 1) (|x| – 2) ≥ 0 and x ≠ –2, 2
⎛ 2 − | x |⎞ 2 −| x |
cos ⎟⎠ is defined for – 1 ≤ ≤1 ⇒ |x| ≤ 1 or |x| > 2
–1
⎜⎝
4 4
⇒ –1 ≤ x ≤ 1 or (x < –2 or x > 2)
⇒ – 4 ≤ 2 – | x | ≤ 4 ⇒ – 6 ≤ – | x | ≤ 2
Domain of f = [– 1, 1] ∪ (– ∞, – 2) ∪ (2, ∞)
⇒ – 2 ≤ | x | ≤ 6 ⇒ | x | ≤ 6 ⇒ – 6 ≤ x ≤ 6 (1)

Objective_Maths_JEE Main 2017_Ch 2.indd 5 01/01/2008 03:18:13


2.6  Chapter 2

Solution: (A)
13. The domain of the function f (x) = 1 − 1 − 1 − x 2 is
f (x) is defined if
(A) (– ∞, 1) (B)  (–1, ∞)
⎡ 1⎤ ⎡ 1⎤
(C)  [0, 1] (D)  [–1, 1] x2 – 5x + 6 ≠ 0, ⎢ x + ⎥ > 0, ⎢ x + ⎥ ≠ 1
⎣ 2⎦ ⎣ 2⎦
Solution: (D)
x – 5x + 6 ≠ 0 ⇒ (x – 2) (x – 3) ≠ 0 ⇒ x ≠ 2, 3
2
(1)
f (x) is defined if
⎡ 1⎤ 1
⎢ x + 2 ⎥ > 0 ⇒ x ≥ 2 (2)
1 – 1 − 1 − x 2 ≥ 0, 1 – 1 − x 2 ≥ 0 and 1 – x2 ≥ 0 ⎣ ⎦
1 – x2 ≥ 0 ⇒ (x + 1) (x – 1) ≤ 0 ⇒ – 1 ≤ x ≤ 1
Clearly for these values, the other two inequalities ⎡ 1⎤ ⎡ 1 3⎞
⎢ x + 2 ⎥ ≠ 1 ⇒ x ∉ ⎢ 2 , 2 ⎟⎠ (3)
hold. ⎣ ⎦ ⎣
Thus domain of f = [– 1, 1]. From Eq. (1), (2) and (3), we get domain of f
14. The domain of the function ⎡3 ⎞
=⎢ , 2⎟ ∪ (2, 3) ∪ (3, ∞).
(x) = log10 [1 – log10 (x2 – 5x + 16)] is
f  ⎣2 ⎠
(A)  (2, 3) (B)  [2, 3] −1
(log16 x 2 )
(C)  (2, 3] (D)  [2, 3) 17. The domain of the function f (x) = e sin is
⎡1 ⎤ ⎡ 1⎤ ⎡1 ⎤
Solution: (A) (A)  ⎢ , 4 ⎥ (B)  ⎢ −4, − ⎥ ∪ ⎢ , 4⎥
f (x) is defined if ⎣4 ⎦ ⎣ 4⎦ ⎣4 ⎦
1 – log10 (x2 – 5x + 16) > 0 and x2 – 5x + 16 > 0 ⎡ 1⎤
(C)  ⎢ −4, − ⎥ (D)  None of these
⎛ 5⎞ 39
2
⎣ 4⎦
⇒ log10 (x2 – 5x + 16) < 1 and ⎜ x − ⎟ + >0
⎝ 2⎠ 4 Solution: (B)
⇒ x – 5x + 16 < 10 = 10
2 1
f (x) is defined if
⎡ ⎛ 5⎞
2
39 ⎤ – 1 ≤ log16 x2 ≤ 1 ⇒ 16–1 ≤ x2 ≤ 161
 Q
⎢ ⎜ x − ⎟ + > 0 for all real x ⎥
⎢⎣ ⎝ 2⎠ 4 ⎥⎦ ⇒
1
≤ x2 ≤ 16
⇒ x – 5x + 6 < 0
2 16
1 ⎛ 1⎞ ⎛ 1⎞
⇒ (x – 3) (x – 2) < 0 ⇒ 2 < x < 3 x2 ≥ ⇒ ⎜ x − ⎟ ⎜ x + ⎟ ≥ 0
16 ⎝ 4 ⎠ ⎝ 4⎠
\ Domain of f = (2, 3)
1 1
1 ⇒ x≤– or x ≥ (1)
15. The domain of the function f (x) = is 4 4
| sin x | + sin x x2 ≤ 16 ⇒ (x – 4) (x + 4) ≤ 0 ⇒ – 4 ≤ x ≤ 4 (2)
(A)  (– 2np, 2np) (B) (2np, (2n + 1) p) From Eq. (1) and (2), we get domain of f
⎛ p p⎞
(C)  ⎜ ( 4 n − 1) , ( 4 n + 1) ⎟ (D)  None of these ⎡ 1⎤ ⎡1 ⎤
⎝ 2 2⎠ = ⎢ − 4, − ⎥ ∪ ⎢ , 4 ⎥ .
⎣ 4⎦ ⎣4 ⎦
Solution: (B) 18. The domain of the function
f (x) is defined if | sin x | + sin x > 0 (x) = log 2 log 2 log 2 ...log 2 x is
f 

⇒ sin x > 0 ⇒ 2np < x < 2np + p n times

\ Domain of f = (2np, (2n + 1) p). (A) (2n – 1, ∞) (B) [2n, ∞)


(C) (2n – 2, ∞) (D)  None of these
16. The domain of the function
Solution: (D)
(x) = log ⎡
f  1⎤
x 2 − 5 x + 6 is
⎢x + 2⎥
⎣ ⎦
f (x) is defined if
(1) x > 0
⎡3 ⎞ ⎡3 ⎞ (2) log2 x > 0 ⇒ x > 20 = 1
(A)  ⎢ , 2⎟ ∪ (2, 3) ∪ (3, ∞) (B) 
⎢ , ∞⎟
⎣2 ⎠ ⎣2 ⎠ (3) log2 log2 x > 0 ⇒ log2 x > 20 = 1 ⇒ x > 21 = 2
⎡1 ⎞ (4) log2 log2 log2 x > 0 ⇒ log2 log2 x > 20 = 1
(C)  ⎢ , ∞⎟ (D)  None of these
⎣2 ⎠ ⇒ log2 x > 21 = 2
⇒ x > 22

Objective_Maths_JEE Main 2017_Ch 2.indd 6 01/01/2008 03:18:17


Functions  2.7

(5) log2 log2 log2 log2 x > 0 ⇒ log2 log2 log2 x > 20 = 1 Solution: (C)
⇒ log2 log2 x > 21 f (x) is defined if
–(log3 x)2 + 5 log3 x – 6 > 0 and x > 0
2
⇒ log2 x > 22 ⇒ x > 22
M  M  M ⇒ (log3 x – 3) (2 – log3 x) > 0 and x > 0
Continuing like this, we get x > 22
2...( n−1) times
⇒ (log3 x – 2) (log3 x – 3) < 0 and x > 0
2...( n−1) times ⇒ 2 < log3 x < 3 and x > 0
\ Domain of f = ( 22 , ∞).
⇒ 32 < x < 33 ⇒ 9 < x < 27
1
19. The domain of the function f (x) = is \ Domain of f = (9, 27).
[ x] − [ x] − 6
2

22. The domain of definition of the function y (x) given by


(A) (– ∞, – 2) ∪ [4, ∞) (B) (– ∞, – 2] ∪ [4, ∞)
the equation 2x + 2y = 2 is
(C) (– ∞, – 2) ∪ (4, ∞) (D)  None of these
(A) 0 < x ≤ 1 (B)  0 ≤ x ≤ 1
Solution: (A) (C) – ∞ < x ≤ 0 (D)  – ∞ < x < 1
f (x) is defined for
Solution: (D)
[x]2 – [x] – 6 > 0 ⇒ ([x] – 3) ([x] + 2) > 0 We have, 2x + 2y = 2 ⇒ 2y = 2 – 2x
⇒ [x] < –2 or [x] > 3 log ( 2 − 2 x )
⇒ y=
But [x] < – 2 ⇒ [x] = –3, –4, –5, ... log 2 
\ x < –2 For y to be defined, 2 – 2x > 0 ⇒ 2x < 2
Since, 2x is an increasing function, therefore we get x < 1.
Also, [x] > 3 ⇒ [x] = 4, 5, 6, ...
\ x ≥ 4 23. The domain of the function
\ Domain of f = (–∞, –2) ∪ [4, ∞). ⎛ x ⎞
(x) = cot–1 ⎜ 2
f  ⎟ , x ∈ R is
20. The domain of the function ⎝ x − [x2 ] ⎠
f (x) = log3 ⎡ − log ⎛1 + 1 ⎞ − 1⎤ is (A) R – {± n , n ∈ N} (B)  R – { n, n ≥ 0, n ∈ I}
⎢ 1 ⎜ ⎟
⎣ 2 ⎝ x1 5 ⎠ ⎥⎦ (C) R (D)  R – {0}
(A) (– ∞, 1) (B)  (0, 1) Solution: (B)
(C) (1, ∞) (D)  None of these x
Domain of cot–1x is R and is defined if
Solution: (B) x2 ≠ [x2] x − [x2 ]
2

(∵ x2 ≥ [x2])
f (x) is defined if
⇒ x2 ≠ 0 or +ve integer.
⎛ 1 ⎞ 1
– log 1 ⎜1 + 1 5 ⎟ – 1 > 0, 1 + 1 5 > 0,  x ≠ 0 Hence, domain = R – { n : n ≥ 0, n ∈ Z}.
2 ⎝ x ⎠ x
⎛ 1 ⎞
⇒ log 1 ⎜1 + 1 5 ⎟ < –1, x1/5 + 1 > 0,  x ≠ 0 Method to Find the Range of
2 ⎝ x ⎠
−1 a Function
1 ⎛ 1⎞
⇒ 1 + > ⎜ ⎟ , x > (–1)5,  x ≠ 0
x15
⎝ 2⎠ 1. Find the domain of the function y = f (x).
1 2. If the domain is an infinite interval, solve the equation
⇒ > 1, x > –1 and x ≠ 0 y = f (x) and find x in terms of y to get x = g (y). Find the
x1 5
real values of y for which x is real. The set of values of y
⇒ 0 < x < 1 and x > –1 ⇒ 0 < x < 1.
so obtained constitutes the range of f. Note that if finite
\ Domain ( f ) = (0, 1). number of values of x are excluded from the domain,
21. The domain of the function find the values of y for these values of x and exclude
(x) = log3 [–(log3 x)2 + 5 log3 x – 6] is
f  these values of y from the range of f found earlier.
3. If the domain is a finite interval, find the least and
(A)  (0, 9) ∪ (27, ∞) greatest value of y for values of x in the domain. If a is
(B)  [9, 27] the least value and b the greatest value of y, then range
(C)  (9, 27) ( f ) = [a, b].
(D)  None of these

Objective_Maths_JEE Main 2017_Ch 2.indd 7 01/01/2008 03:18:19


2.8  Chapter 2

Solved Examples ⎧1 ⎫
(A) (–∞, ∞)\ ⎨ , 1⎬ (B) (–∞, ∞)
⎩5 ⎭
24. The range of the function f (x) = 3 x 2 − 4 x + 5 is (C) (–∞, ∞)\{1} (D)  None of these
⎡ 11 ⎤ ⎛ 11 ⎞ Solution: (B)
(A)  ⎢ −∞, ⎥ ⎜ −∞, 3 ⎟
(B) 
⎣ 3⎦ ⎝ ⎠ f (x) is defined if x2 + x – 6 ≠ 0
i.e., (x + 3) (x – 2) ≠ 0 i.e. x ≠ – 3, 2
⎡ 11 ⎞ ⎛ 11 ⎞
(C)  ⎢ , ∞⎟ (D) 
⎜ 3 , ∞⎟ \ Domain ( f ) = (– ∞, ∞)\{– 3, 2}
⎢⎣ 3 ⎠ ⎝ ⎠
x 2 − 3x + 2
Let y= 2
Solution: (C) x + x −6
f (x) is defined if 3x2 – 4x + 5 ≥ 0 ⇒ x y + xy – 6y = x2 – 3x + 2
2

⎡ 4 5⎤ ⎡⎛ 2 ⎞ 11⎤
2 ⇒ x2 (y – 1) + x (y + 3) – (6y + 2) = 0
⇒ 3 ⎢ x2 − x + ⎥ ≥ 0 ⇒ 3 ⎢⎜ x − ⎟ + ⎥ ≥ 0 For x to be real, (y + 3)2 + 4 (y – 1) (6y + 2) ≥ 0
⎣ 3 3⎦ ⎢⎣⎝ 3⎠ 9 ⎥⎦
which is true for all real x ⇒ 25y2 – 10y + 1 ≥ 0 i.e. (5y – 1)2 ≥ 0
\ Domain ( f ) = (– ∞, ∞) which is true for all real y.
\ Range of  f = (–∞, ∞).
Let y = 3x 2 − 4 x + 5 ⎛ x2 ⎞
27. The range of the function y = sin–1 ⎜ is
⇒ y2 = 3x2 – 4x + 5 i.e. 3x2 – 4x + (5 – y2) = 0 ⎝ 1+ x 2 ⎟⎠
11 ⎛ p⎞ ⎡ p⎞
For x to be real, 16 – 12 (5 – y2) ≥ 0 ⇒ y ≥ (A)  ⎜ 0, ⎟ (B) 
3 ⎝ 2⎠ ⎢0, 2 ⎟⎠
⎡ 11 ⎞ ⎣
\ Range of y = ⎢ , ∞⎟ . ⎡ p⎤
⎢⎣ 3 ⎠ (C)  ⎢0, ⎥ (D)  None of these
⎣ 2⎦
25. The range of the function f (x) = loge (3x2 – 4x + 5) is
Solution: (B)
⎛ 11⎤ ⎡ 11 ⎞ x2
(A)  ⎜ −∞, log e ⎥ (B) ⎢log e , ∞⎟ Clearly, for y to be defined, ≤ 1 which is true
⎝ 3⎦ ⎣ 3 ⎠ 1+ x 2
for all x ∈ R. So, the domain = (– ∞, ∞).
⎡ 11 11⎤
(C)  ⎢ − log e , log e ⎥ (D)  None of these ⎛ x2 ⎞
⎣ 3 3⎦ x2
Now, y = sin–1 ⎜ 2⎟
⇒ = sin y
⎝ 1+ x ⎠ 1+ x 2
Solution: (B)
x2
f (x) is defined if 3x2 – 4x + 5 > 0 ⇒ x=
1+ x 2 
⎡ 4 5⎤ ⎡⎛ 2 ⎞ 11⎤
2

⇒ 3 ⎢ x 2 − x + ⎥ > 0 ⇒ 3 ⎢⎜ x − ⎟ + ⎥ > 0, For x to be real, sin y ≥ 0 and 1 – sin y > 0


⎣ 3 3⎦ ⎢⎣⎝ 3⎠ 9 ⎥⎦
⎡ p⎞
which is true for all real x. ⇒ 0 ≤ sin y < 1 ⇒ y ∈ ⎢0, ⎟
\ Domain ( f ) = (– ∞, ∞) ⎣ 2⎠
⎡ p⎞
Let y = loge (3x2 – 4x + 5) ⇒ ey = 3x2 – 4x + 5. \ Range = ⎢0, ⎟
⎣ 2⎠
⇒ 3x2 – 4x + (5 – ey ) = 0
For x to be real, p2
28. The range of the function y = 3 sin − x 2 is
11 16
16 – 12 (5 – ey) ≥ 0 ⇒ 12 ey ≥ 44 ⇒ ey ≥
3 ⎡ 3 ⎤ ⎡ 3 3 ⎤
11
⇒ y ≥ loge (A)  ⎢0, ⎢−
⎥ (B)  , ⎥
3 ⎣ 2⎦ ⎣ 2 2⎦
⎡ 11 ⎞ ⎡ 3 ⎤
\ Range of f = ⎢log e , ∞⎟ . (C)  ⎢ − , 0 ⎥ (D)  None of these
⎣ 3 ⎠ ⎣ 2 ⎦
x 2 − 3x + 2 Solution: (A)
26. The value of the function f (x) = 2 lies in the
x + x −6 p2
interval For y to be defined, – x2 ≥ 0
16

Objective_Maths_JEE Main 2017_Ch 2.indd 8 01/01/2008 03:18:23


Functions  2.9

⎛p ⎞ ⎛p ⎞ ⎛ p⎞ ⎛ p⎞ ⎛ 1⎞
⇒ ⎜ − x ⎟ ⎜ + x ⎟ ≥ 0 ⇒ ⎜ x − ⎟ ⎜ x + ⎟ ≤ 0
(A)  ⎜⎝ − 1, − ⎟⎠
[–2, –1] (b) 
⎝4 ⎠ ⎝4 ⎠ ⎝ 4⎠ ⎝ 4⎠ 2
p p (C)  (–2, –1) (D)  None of these
⇒ – ≤ x ≤
4 4
Solution: (C)
⎡ p p⎤
\ Domain of y = ⎢ − , ⎥ We have, [2 sin x] + [cos x] = –3
⎣ 4 4⎦
only if [2 sin x] = –2 and [cos x] = –1
⎡ p p⎤ p2 ⎡ p⎤ ⇒ –2 ≤ 2 sin x < –1 and –1 ≤ cos x < 0
Clearly, for x ∈ ⎢ − , ⎥ , − x 2 ∈ ⎢0, ⎥ . 1
⎣ 4 4⎦ 16 ⎣ 4⎦ ⇒ –1 ≤ sin x < − and –1 ≤ cos x < 0
2
⎡ p⎤ 7p 11p p 3p
Since sin x is an increasing function on ⎢0, ⎥ . ⇒ <x< and <x<
⎣ 4⎦ 6 6 2 2
p2 p 7p 3p
Therefore, sin 0 ≤ sin − x 2 ≤ sin ⇒ <x<
16 4 6 2
p2 3 3 For the above values of x, sin x + 3cos x = 2 sin
⇒ 0 ≤ 3 sin − x2 ≤ ⇒0≤y≤ ⎛p ⎞
16 2 2 ⎜⎝ + x ⎟⎠ lies between –2 and –1.
3
⎡ 3 ⎤
\ Range of y = ⎢0, ⎥ \ Rage of f (x) is (–2, –1).
⎣ 2⎦
e− x
29. The range of the function 31. The range of the function y = is
1 + [ x]
⎡ ⎛ 4 − x2 ⎞ ⎤ (A) (–∞, ∞) (B)  R – {0}
f (x) = sin ⎢log ⎜ ⎟ ⎥ is
⎢⎣ ⎝ 1 − x ⎠ ⎥⎦ (C) R (D)  None of these

(A)  [0, 1] (B)  (–1, 0) Solution: (B)


(C)  [–1, 1] (D)  (–1, 1) We have,
e− x
Solution: (C) y=
1 + [ x] 
For f (x) to be defined,
⇒ (1 + [x])y = e–x > 0
4 − x2
> 0, 4 – x2 > 0 and 1 – x ≠ 0 ⇒ (1 + [x])y > 0
1− x
⇒ y > 0 if 1 + [x] > 0 and y < 0 if 1 + [x] < 0
Since 4 − x 2 </ 0,
\ y ∈ R – {0}
\ We have 1 – x > 0 and 4 – x2 > 0
⇒ x < 1 and (x – 2) (x + 2) < 0
⇒ x < 1 and – 2 < x < 2 Types of Functions
⇒ –2 < x < 1
\ Domain of  f = (– 2, 1).
One–One or Injective Function
A function f : X → Y is said to be one-one or injective if
⎛ 4 − x2 ⎞
Since –∞ < log ⎜ ⎟ <∞ distinct elements of X have distinct images in Y, as shown
⎝ 1− x ⎠ in Fig. 2.4.
⎡ ⎛ 4 − x2 ⎞ ⎤ f
⇒ –1 ≤ sin ⎢log ⎜ ⎟⎥ ≤ 1
⎢⎣ ⎝ 1 − x ⎠ ⎥⎦
\ Range of  f = [–1, 1].
30. If [2 sin x] + [cos x] = –3, then the range of the function
f (x) = sin x + 3 cos x in [0, 2p] (where [⋅] denotes the X Y
greatest integer function) is Fig. 2.4

Objective_Maths_JEE Main 2017_Ch 2.indd 9 01/01/2008 03:18:28


2.10 Chapter 2

Many–one Function
notE
A function f : X → Y is said to be many-one if there exists
atleast two distinct elements in X whose images are same,  Any continuous function f(x) which has atleast one local
as shown in the Fig. 2.5 maxima or local minima is many-one.
f  All even functions are many-one.
 All polynomials of even degree defined on R have atleast
one local maxima or minima and hence are many one on
the domain R. Polynomials of odd degree can be one-one
or many-one.

. X Y
Fig. 2.5 tRick(S) FoR pRobleM Solving

Method to check whetheR the Number of One–One Functions (Injections)


Function f : X → Y iS one–one oR If X and Y are any two finite sets having m and n elements
respectively, then the number of one-one functions from X to
Many-one Y would be
⎧nP , if n ≥ m ⎫
1. Consider any two points x, y ∈ X. =⎨ m ⎬
⎩ 0, if n < m⎭
2. Put f (x) = f (y) and solve the equation.
3. If we get x = y only, then f is one-one, otherwise it is
many-one.
onto oR SuRJective Function
tRick(S) FoR pRobleM Solving
A function f : X → Y is said to be onto or surjective if every
Derivative Test to Check the Injectivity element of Y is the image of atleast one clement in X under
If a function is either strictly increasing or strictly decreasing
the map f , as shown in the Fig. 2.6
X Y
in the whole domain (or equivalently, f ′(x) > 0 or f ′(x) < 0, ONTO
∀ x ∈ X), then it is one-one, otherwise it is many-one.
Graphical Test
If any straight line parallel to x-axis intersects the graph of
the function atmost at one point, then the function is one-one,
otherwise it is many-one (i.e., it intersects the graph of the
function in atleast two points).
Fig. 2.6
Y Y

A function f : X → Y is an into function if it is not an onto


function, as shown in the Fig. 2.7
(0, 1)
X Y
X′ X X′ X INTO
O O
f (x) = ax + b f (x) = ax (0 < a < 1)
Y′ Y′
(i) One–One Function (ii) One–One Function
Y Y

Fig. 2.7

In other words, if the function f : X → Y is such that there is


X′ X X′ X′ atleast one element in Y which is not image of any element
O f (x) = O
x2 f (x) = x
in X, then we say that f is a function of A into B, i.e., f is
Y′ Y′ called into function if, for some y ∈ Y, there does not exist
(i) Many–One Function (ii) Many–One Function any x ∈ X such that y = f (x).

Objective_Maths_JEE Main 2017_Ch 2.indd 10 01/01/2008 03:18:29


Functions 2.11

Method to check whetheR the (A) one-one and into (B) many-one and into
Function f : X → Y iS onto oR into (C) one-one and onto (D) many-one and onto
Solution: (A)
1. Find the range of the function f.
2. If range of f = Y (the co-domain), then f is onto, other- Since, for different x, 4x and 4| x | are different positive
wise it is into. numbers,
\ f is one–one. Also, f is not onto as its range (0, ∞)
is a proper subset of its co-domain R.
notE
33. Let f : R → R be a function defined by,
For an ‘onto’ function, the range overlaps or equals
co-domain, whereas for an into function, the range does not f (x) = x + x 2 , then f is
overlap but fits inside the co-domain. (A) injective (B) surjective
(C) bijective (D) None of these
Solution: (D)
tRick(S) FoR pRobleM Solving
We have,
Number of Onto Functions (Surjections) f (x) = x + x 2 = x + | x |
If X and Y are any two finite sets having m and n elements Clearly, f is not one-one as f (– 1) = f (– 2) = 0 but – 1
respectively, where 1 ≤ n ≤ m, then the number of onto ≠ –2.
functions from X to Y is given by Also, f is not onto as f (x) ≥ 0 ∀ x ∈ R,
n

∑ ( −1) n−r n
Cr r m \ range of f = (0, ∞) ⊂ R.
r −1

Any polynomial function f is onto if degree is odd and into if


degree of f is even. SoMe iMpoRtant FunctionS
constant Function
biJective Function A function f : R → R defined as f (x) = c, ∀ x ∈ R, where c
A function f : X → Y is said to be bijective, if f is both one- is a constant, is called a constant function. Its domain is R
one and onto, as shown in the Fig. 2.8 and range is singleton set {c}.
The graph of a constant function is a straight line par-
X Y allel to x-axis as shown in the Fig. 2.9. It is above or below
f the x-axis according as c is positive or negative. If c = 0,
then the straight line coincides with x-axis.
Y

Fig. 2.8 (0, c) f (x) = c

tRick(S) FoR pRobleM Solving X


O
Number of One–One Onto Functions (Dijections)
If X and Y are any two finite sets having the same number of Fig. 2.9
elements, say n, then the number of bijective functions from
X to Y is n!.
identity Function
The function f : R → R defined as f (x) = x, ∀ x ∈ R, is called
Solved exaMpleS the identity function. Its do-main is R and range is also R.
The graph of the identity function is a straight line
32. The function f : R → R defined by, passing through origin and inclined at an angle of 45º with
x-axis, as shown in the Fig. 2.10.
f (x) = 4x + 4|x| is

Objective_Maths_JEE Main 2017_Ch 2.indd 11 01/01/2008 03:18:30


2.12  Chapter 2

Y where [x] = integral part of x or greatest integer not


greater than x or greatest integer less than or equal to x.
(x) = n,
i.e. f 
f (x) = x
X′ X where n ≤ x < n + 1, n ∈ Z (the set of integers).
O Its domain is R and range is Z. The graph of the
­greatest integer function is as shown in Fig. 2.12:
Y′ Y

Fig. 2.10
3

2
Modulus function or absolute
1
value function
X
The function f : R → R, defined as –3 –2 –1 0 1 2 3
–1
⎧ x, if x > 0 ⎫
⎪ ⎪ –2
f (x) = | x | = ⎨0, if x = 0 ⎬ 
⎪ − x, if x < 0⎪ –3
⎩ ⎭
is called the absolute value function or modulus function. Fig. 2.12
Its domain is R and its range is [0, ∞). The graph of the
modulus function is as shown in the Fig. 2.11.
Trick(s) for Problem Solving
Y
 [x] ≤ x < [x] + 1
⎪⎧[ x ] + [ y ] if { x} + { y } < 1⎪⎫
[x + y] = ⎨
f(


⎩⎪[ x ] + [ y ] + 1, if { x} + { y } ≥ 1⎭⎪
=
x)


x)
=

f(
–x

X where {x} denotes the fractional part of x.


O
 n ≤ x < n + 1 ⇔ [x] = n

 n ≤ [x] ≤ n ⇒ n ≤ x < n + 1
1 2 1 2
Fig. 2.11
 x – 1 < [x] ≤ x

 [[x]] = [x]

Trick(s) for Problem Solving  [n + x] = n + [x], where n is any integer

⎧0 if x ∈ Z
 For every x ∈ R, |x| = max {x, –x}  [x] + [– x] = ⎨
⎩ −1 if x ∉ Z
 For every x ∈ R, |x| = x 2
 If [f(x)] ≥ n, then f(x) ≥ n
 |x + y| ≤ |x| + |y|
If [f(x)] ≤ n, then f(x) < n + 1
 |x + y| = |x| + |y| if x, y have the same sign and

|x + y| < |x| + |y| if x, y have opposite signs where n is any integer.


 |x – y| ≥ ||x| + |y||  ⎡ x + ⎡ y + ⎡⎣ z + ⎡⎣w + [u ]⎤⎦ ⎤⎦ ⎤ ⎤ = [ x ] + [ y ] + [ z ] + [w ] + [u ]
⎣ ⎣ ⎦⎦
 |x – y| = ||x| – |y|| if x, y have same sign and |x| ≥ |y|

 |x – y| > ||x| + |y|| if x, y have opposite signs

 |xy| = |x| |y|


Least integer function/ceiling
function
Greatest integer function/Step The function f : R → R defined as f (x) = (x) is called least
function/Floor Function integer function,
where (x) = least integer greater than or equal to x.
The function f : R → R defined as f (x) = [x] is called the i.e., f (x) = n,
greatest integer function,
where n – 1 < x ≤ n, n ∈ Z (the set of integers)

Objective_Maths_JEE Main 2017_Ch 2.indd 12 01/01/2008 03:18:31


Functions  2.13

Its domain is R and range is Z. The graph of the least 1 x!


(A)  (B) 
integer function is as shown in Fig. 2.13: log [1 − | x |] {x}
Y log ( x − 1)
(C) x! {x} (D) 
1 − x2
Solution: (C)
Since only x! {x} is defined so x! {x} represents a
function.
–3 –2 –1 O
X ⎡ 1 ⎤
1 2 3 35. Range of the function f defined by f (x) = ⎢ ⎥
⎣ sin{x} ⎦
(where [.] and {.} respectively denote the greatest inte-
ger and the fractional part functions) is
(A) Z, the set of integers
(B) N, the set of natural numbers
Fig. 2.13
(C) W, the set of whole numbers
(D)  {2, 3, 4, ...}
Fractional-part function Solution: (B)
The function f : R → R defined as f (x) = x – [x] or f (x) = Q {x} ∈ [0, 1)
{x}, where {x} denotes the fractional part of x, is called \ sin {x} ∈ [0, sin 1) but f (x) is defined if sin {x} ≠ 0
the ­fractional-part function. Its domain is R and range is ⎛ 1 ⎞
1
[0, 1). The graph of the fractional part function is as shown \ ∈ ,∞ .
in Fig. 2.14: sin{x} ⎜⎝ sin 1 ⎟⎠
Y ⎡ 1 ⎤
\ ⎢ ⎥ ∈{1, 2, 3.....} .
⎣ sin{x} ⎦

–3 –2 –1 0 1 2 3
X Signum function
The function f : R → R defined as
⎧x ⎫
Fig. 2.14 ⎪ for x ≠ 0 ⎪
(x) = ⎨ x
f  ⎬
⎪⎩0 for x = 0⎪⎭
Trick(s) for Problem Solving
is called the signum function.
 If 0 ≤ x < 1, then {x} = x Its domain is R and range is the set {– 1, 0, 1}. The
 If x is an integer, then x = [x] ⇒ {x} = 0 graph of the signum function is as shown in Fig. 2.15:
⇒ {[x]} = 0 Y
 [{x}] = 0
(0, 1)
 0 ≤ {x} < 1

⎧0 if x ∈ integer X
 {x} + {– x} = ⎨ 0
⎩ −1, if x ∉ integer (0, –1)
 If x ∉ Z and x > 0, then {–x} = 1 – {x}
Fig. 2.15

Solved Examples Reciprocal function


1
34. Which of the following is a function ([.] denotes the The function f : R\{0} → R defined by f (x) = , is called
x
greatest integer function, {.} denotes the fractional
the reciprocal function. Its domain as well as range is R\{0}.
part function)?
The graph of the reciprocal function is as shown in Fig. 2.16:

Objective_Maths_JEE Main 2017_Ch 2.indd 13 01/01/2008 03:18:33


2.14 Chapter 2

Y Y Y

f(x) = loga x
a>1

(1, 0)
X X
X 0 (1, 0) 0
O
f (x) = loga x
a<1
Fig. 2.18(a) Fig. 2.18(b)

Fig. 2.16 notE


 loga a = 1, loga 1 = 0, provided a > 0, a ≠ 1
exponential Function
⎧ −∞, if a >1
 loga 0 = ⎨
Let a (≠ 1) be a positive real number. Then the function ⎩+ ∞, if 0 < a <1
f : R → R, defined by f (x) = ax, is called the exponential  loge x is also denoted as: ln x.
function. Its domain is R and range is (0, ∞). The graph of  If loga x1 > loga x2 then x1 > x2 if a > 1 and x1 < x2 if
the exponential function is as shown in Fig. 2.17(a) and (b). 0 < a < 1.
Y Y
f (x) = a x f (x) = a x
a>1 a<1 polynoMial Function
(0, 1) (0, 1) A function f : R → R, defined by f (x) = a0 + a1 x + a2 x2 +
... + anxn, where n ∈ N and a0, a1, a2, ..., an ∈ R, is called a
X X polynomial function.
0 0
If an ≠ 0, then n is called the degree of the polynomial.
The domain of a polynomial function is R.
Fig. 2.17(a) Fig. 2.17(b)
Rational Function
p (x)
notE A function of the form f (x) = , where p (x) and q (x)
q (x)
are polynomials over the set of real numbers and q (x) ≠ 0,
 ax = e x loge a, (a > 0).
is called a rational function. Its domain is R\{x | q (x) = 0}.
 aloga x = x, (a > 0, a ≠ 1).
logc b
 log ab = , a, b, c > 0 and a, c ≠ 1 tRigonoMetRic FunctionS
logc a
1 table 2.1
 log ba = , provided a ≠ 1, b ≠ 1 and a, b > 0
loga b Function Domain Range
 If a x1 > a x2 then x1 > x2 if a > 1 and x1 < x2 if 0 < a < 1
y = sin x R [– 1, 1]
y = cos x R [– 1, 1]
 π 
logaRithMic Function y = tan x R \ (2n + 1) n ∈ Z 
2
R
 
Let a (≠ 1) be a positive real number. Then the function y = cot x R \ {np | n ∈ Z} R
f : (0, ∞) → R, defined by f (x) = loga x, is called the  π 
logarithmic function. Its domain is (0, ∞) and range is y = sec x R \ (2n + 1) n ∈ Z  (– ∞, – 1] ∪ [1, ∞)
 2 
R. The graph of the logarithmic function is as shown in
y = cosec x R \ {np | n ∈ Z} (– ∞, – 1] ∪ [1, ∞)
Fig. 2.18(a) and (b):

Objective_Maths_JEE Main 2017_Ch 2.indd 14 01/01/2008 03:18:35


Functions  2.15

Inverse Trigonometric Functions Illustration


1. y = sin2x, x ∈ R.
Table 2.2
2. y = log x + x – 2ex, x > 0.
Function Domain Range

 π π
Implicit Function
y = sin–1x –1≤x≤1  − 2 , 2  When the variables x and y occur together in an equation
y = cos x–1
–1≤x≤1 [0, p] f (x, y) = 0, in which y cannot be expressed explicitly in
terms of x, then y is said to be an implicit function of x.
 π π
y = tan–1x –∞<x<∞  − , 
2 2 Illustration
y = cot–1x –∞<x<∞ (0, p)
1. xy = tan(x + y)
 π   π 2. exy + xy – 2 = 0.
y = cosec–1x (–∞, – 1] ∪ [1, ∞)  − 2 , 0 ∪  0, 2 
 
 π π  Operations on Functions
y = sec–1x (– ∞, – 1] ∪ [1, ∞) 0, 2  ∪  2 , π 
 
Let f and g be two real functions with domain D1 and D2
respectively. Then,
Two Ways of Defining a Function
1. The sum function ( f + g) is defined by,
1. Uniform Definition: If a function is defined as y =
( f + g) (x) = f (x) + g (x), ∀ x ∈ D1 ∩ D2
f (x), x ∈ [a, b], we say that it is uniformly defined.
Illustration The domain of f + g is D1 ∩ D2
 (i)  y = f (x) = sin x, x ∈ R, 2. The difference function ( f – g) is defined by,
(ii)  y = f (x) = x2 + 1, x ∈ [–1, 1]. ( f – g) (x) = f (x) – g (x), ∀ x ∈ D1 ∩ D2
2. Piecewise Definition: If a function y = f (x), x ∈ The domain of f – g is D1 ∩ D2
[a, b] assumes different forms in different subsets of 3. The product function fg is defined by,
[a, b], we say that it is piecewise defined, as shown in
the Fig. 2.19 ( fg) (x) = f (x) ⋅ g (x), ∀ x ∈ D1 ∩ D2
The domain of fg is D1 ∩ D2

Y
⎛ f⎞
4. The quotient function ⎜ ⎟ is defined by,
⎝ g⎠
y=1 ⎛ f⎞ f ( x)
1 ⎜⎝ g ⎟⎠ (x) = g ( x ) , ∀ x ∈ D1 ∩ D2\[x : g (x) = 0]
y

(0, 1) –
=

x
1

=
y

f
x

The domain of is D1 ∩ D2\[x : g (x) = 0]


X g
–1 0 (1, 0)
5. The scalar multiple function cf is defined by,
Fig. 2.19 (cf ) (x) = c ⋅ f (x), ∀ x ∈ D1
Illustration
The domain of cf is D1
⎧ 1, −1 ≤ x < 0

y = f (x) = ⎨1 − x, 0 ≤ x < 1 Trick(s) for Problem Solving
⎪ x − 1, x ≥ 1

In a function f(x) is such that
 f(xy) = f(x) ⋅ f(y) ⇒ f(x) = x , n ∈ R
n

Explicit and Implicit Functions  f(x + y) = f(x) ⋅ f(y) ⇒ f(x) = a


kx

 f(xy) = f(x) + f(y) ⇒ f(x) = k log x or f(x) = 0


Explicit Function
 f(x + y) = f(x) = f(y) ⇒ f(x) = k, where k is constant
A function y is said to be an explicit function of x, if the ⎛ 1⎞ ⎛ 1⎞
 f(x) × f ⎜ ⎟ = f(x) + f ⎜ ⎟ ⇒ f(x) = ± x + 1
n
dependent variable y can be expressed totally in terms of ⎝ x⎠ ⎝ x⎠
the independent variable x.

Objective_Maths_JEE Main 2017_Ch 2.indd 15 01/01/2008 03:18:38


2.16  Chapter 2

Composition of Functions ⎧ x, x ∈ Q, x ∈Q
⎪ 1 − x, x ∉ Q, x ∈Q
Let f and g be two real functions with domain D1 and D2 ⎪
= ⎨
respectively. ⎪1 − x, 1 − x ∈ Q, x ∉Q
If range of f ⊆ domain of g, then composite function ⎪⎩1 − (1 − x ), 1 − x ∉ Q, x ∉Q
(gof ) is defined by,
⎧ x, x ∈ Q
(gof ) (x) = g [ f (x)], ∀ x ∈ D1 = ⎨
⎩ x, x ∉ Q
Also, if range of g ⊆ domain of f, then composite function \ fof (x) = x, x ∈ [0, 1].
( fog) is defined by,
37. If g [ f (x)] = | sin x | and f [g (x)] = (sin x )2, then
( fog) (x) = f [g (x)], ∀ x ∈ D2
(A) f (x) = sin2x, g (x) = x
(B) f (x) = sin x, g (x) = |x|
caution (C) f (x) = x2, g (x) = sin x
(D) f and g cannot be determined.
gof exists only if range f ⊆ domain and fog exists only if
range g ⊆ domain if Solution: (A)

When f (x) = sin2x and g (x) = x ,


Properties of Composite Functions ( fog) (x) = f [g (x)] = f ( x ) = (sin x )2

Let f : X → Y and g : Y → Z be two functions. and (gof ) (x) = g [ f (x)] = g (sin2x) = |sin x|

1. If both f and g are one-one, then so is gof. When f (x) = sin x and g (x) = |x|
2. If both f and g are onto, then so is gof. ( fog) (x) = f (g (x)) = f (|x|) = sin |x| ≠ (sin x )2
3. If gof is one-one, then f is one-one but g may not be
one-one. When f  (x) = x2 and g (x) = sin x 
4. If gof is onto, then g is onto but f may not be onto. ( fog) (x) = f [g (x)] = f (sin x ) = (sin x )2
5. If f and g are bijective, then so is gof.
6. It may happen that gof may exist and fog may not exist. and (gof ) (x) = g [ f (x)] = g (x2) = sin x 2 
Moreover, even if both gof and fog exist, they may not = sin | x | ≠ | sin x |
be equal.
38. Let f  be a function with domain [–3, 5] and let
Solved Examples g (x) = |3x + 4|. Then the domain of ( fog) (x) is
⎛ 1⎞ ⎡ 1⎤
(A)  ⎜ −3, ⎟ (B) 
⎢⎣ −3, 3 ⎥⎦
36. Let f  be a function defined on [0, 1] such that ⎝ 3⎠
⎧x x ∈Q ⎡ 1⎞
f (x) = ⎨ (C)  ⎢ −3, ⎟ (D)  None of these
⎩1 − x, x ∉Q ⎣ 3⎠
Then for all x ∈ [0, 1],  fof (x) is Solution: (B)
(A) a constant (B) 1 + x
( fog) (x) = f [g (x)] = f (|3x + 4|)
(C) x (D)  None of these
Since the domain of f is [–3, 5],
Solution: (C)
We have, \ –3 ≤ |3x + 4| ≤ 5 ⇒ |3x + 4|≤ 5

⎧x x ∈Q ⇒ –5 ≤ 3x + 4 ≤ 5
f (x) = ⎨ ⇒ –9 ≤ 3x ≤ 1
⎩1 − x, x ∉Q
1
⎧ f ( x ), f ( x) ∈ Q ⇒ –3 ≤ x ≤
\ fof (x) = ⎨ 3
⎩1 − f ( x ), f ( x) ∉ Q
⎡ 1⎤
\ Domain of fog is ⎢ −3,
⎣ 3 ⎥⎦

Objective_Maths_JEE Main 2017_Ch 2.indd 16 01/01/2008 03:18:40


Functions 2.17

inveRSe FunctionS Solved exaMpleS


If f : X → Y be a one-one onto (bijection) function, then the
mapping f –1 : Y → X which associates each element y ∈ Y 39. Let f : (4, 6) → (6, 8) be a function defined by f (x)
with element x ∈ X such that f (x) = y, is called the inverse ⎡x⎤
fucntion of the function f : X → Y. = x + ⎢ ⎥ (where [ ⋅ ] denotes the greatest integer
⎣2⎦
We define inverse function f –1 : Y → X by the rule
function), then f – 1 (x) is equal to
y = f (x) ⇔ f –1 (y) = x, ∀ x ∈ X, ∀ y ∈ Y
⎡x⎤
(A) x − ⎢ ⎥ (B) – x – 2
as shown in the Fig. 2.20: ⎣2⎦
1
X Y (C) x – 2 (D)
⎡x⎤
x+⎢ ⎥
f ⎣2⎦
f –1(y) = x y = f (x)
Solution: (C)
Since f : (4, 6) → (6, 8),
f –1
\ f (x) = x + 2
\ f (x) = x – 2
–1

Fig. 2.20
40. Let f : [4, ∞) → [4, ∞) be a function defined by,
f (x) = 5x (x – 4), then f –1 (x) is
notE
(A) 2 – 4 + log5 x (B) 2 + 4 + log5 x
For the existence of inverse function, it should be one-one x ( x − 4)
and onto. ⎛ 1⎞
(C) ⎜ ⎟ (D) None of these
⎝ 5⎠
Solution: (B)
Method to Find the inveRSe oF a Let y = 5x (x–4) ⇒ x (x – 4) = log5 y
Function ⇒ x2 – 4x – log5 y = 0
Let f : X → Y be a bijective function. 4 ± 16 + 4 log5 y
⇒ x=
1. Put f (x) = y. 2
2. Solve the equation y = f (x) to obtain x in terms of y. = (2 ± 4 + log5 y )
Interchange x and y to obtain the inverse of f.
But x ≥ 4, so x = (2 + 4 + log5 y )
Aliter: let g be the inverse of f. Simplify the equation
f (g(x)) = x to find g(x). \ f –1 (y) = 2 + 4 + log5 y

a x − a− x
i M P o R ta n t P o i n t S 41. The inverse of the function f (x) = is
a x + a− x
If (x, y) is a point on the graph of an invertible function f, 1 ⎛1− x⎞ 1 ⎛1+ x ⎞
(A) log a ⎜ (B) log a ⎜
then the corresponding point on the graph of f –1 is (y, x). 2 ⎝ 1 + x ⎟⎠ 2 ⎝ 1 − x ⎟⎠
Thus, the graphs of f and f –1 are mirror image of each other
in the line y = x. ⎛1+ x⎞
(C) log a ⎜ (D) None of these
⎝ 1 − x ⎟⎠
Solution: (B)
pRopeRtieS oF inveRSe FunctionS a x − a− x a2 x − 1
Let y= =
1. Inverse of a bijection is also a bijection function. a x + a− x a2 x + 1
2. Inverse of a bijection is unique. y −1 ( a 2 x − 1) − ( a 2 x + 1)
3. ( f – 1)– 1 = f ⇒ = 2x
y +1 ( a − 1) + ( a 2 x + 1)
4. If f and g are two bijections such that (gof ) exists then
(gof )– 1 = f – 1og– 1. (Using componendo and dividendo)

Objective_Maths_JEE Main 2017_Ch 2.indd 17 01/01/2008 03:18:43


2.18 Chapter 2

y −1 −2 1+ y pRopeRtieS oF odd and even


⇒ = ⇒ a2x =
y +1 2a 2 x 1− y FunctionS
⎛1+ y⎞ 1. The graph of even function is always symmetric with
⇒ 2x loga a = loga ⎜
⎝ 1 − y ⎟⎠ respect to y-axis. The graph of odd function is always
symmetric with respect to origin.
1 ⎛1+ y ⎞ 2. The product of two even functions is an even function.
⇒ x= log a ⎜
2 ⎝ 1 − y ⎟⎠ 3. The sum and difference of two even functions is an
even function.
odd and even FunctionS 4. The sum and difference of two odd functions is an odd
function.
odd Function 5. The product of two odd functions is an even function.
6. The product of an even and an odd function is an odd
A function f (x) is said to be odd if f (–x) = –f (x) for every
function.
real number x in the domain of f.
7. It is not essential that every function is even or odd. It
Illustration is possible to have some functions which are neither
even nor odd, e.g. f (x) = x2 + x3, f (x) = loge x, f (x) = ex.
y = f (x) = sin x is odd.
8. The sum of even and odd function is neither even nor
odd function.
even Function 9. Zero function f (x) = 0 is the only function which is
A function f (x) is said to be even if f (– x) = f (x) for every even and odd both.
real number x in the domain of f.
tRick(S) FoR pRobleM Solving
Illustration
The graph of an odd function is symmetric about origin
y = f (x) = x2 is even. 

and it is placed either in the first and third quadrant or in


the second and fourth quadrant.
 The graph of an even function is symmetric about the
notE y-axis.
 To express a given function f (x) as the sum of an even and
 (x, y) is a point on the graph of an even function if an
odd function, we write
only if (–x, y) is a point on the graph.
1 1
f(x) = [f(x) + f(–x)] + [f(x) – f(–x)]
Y 2 2
1 1
where [f(x) + f(–x)] is an even function and [f(x) –
2 2
(–x, y) (x, y) f(–x)] is an odd function.
 f(x) = 0 is the only function which is both even and odd.
X′ X
O  If f(x) is an odd function, then f ′(x) is an even function
Y′ provided f(x) is differentiable on R.
 If f(x) is an even function, then f ′(x) is an odd function
 (x, y) is a point on the graph of an odd function if and provided f(x) is differentiable on R.
only if (–x, –y) is a point on the graph.  If f and g are even functions, then fog is also an even
Y function, provided fog is defined.
(x, y)  If f and g are odd functions, then fog is also an odd function,
provided fog is defined.
 If f is an even function and g is an odd function, then fog is
X′ X an even function.
O
 If f is an odd function and g is an even function, then fog is
an even function.
(–x, –y)  For a real domain, even functions are not one-one.
Y′
 Even functions are many-one functions, because they are
symmetric about y-axis.

Objective_Maths_JEE Main 2017_Ch 2.indd 18 01/01/2008 03:18:45


Functions  2.19

45. A function whose graph is symmetrical about the


Solved Examples ­origin is given by
(A) f (x) = (3x + 3– x)
42. Let f (x) = (–1)[x] (where [ . ] denotes the greatest ­integer
function), then (B) f (x) = cos [log (x + 1 + x 2 )]
(A)  Range of f is {–1, 1} (C) f (x + y) = f (x) + f (y) ∀ x, y ∈ R
(B) f is an even function (D)  None of these
(C) f is an odd function Solution: (C)
(D)  lim f (x) exists, for every integer n A function whose graph is symmetrical about the ori-
x→n
gin must be odd.
Solution: (A)
f (x) = (–1)[x] = {–1, 1}, since [x] ∈ Z. (3x + 3–x) is an even function.
43. If f is an even function defined on the interval [– 5, 5], Since cos x is an even function and log (x + 1+ x2 )
then the real values of x satisfying the equation is an odd function,
⎛ x +1⎞ \ cos (log (x + 1 + x 2 )) is an even function.
(x) = f ⎜
f  are
⎝ x + 2 ⎟⎠
If f (x + y) = f (x) + f (y) ∀ x, y ∈ R, then f (x) must be
−1 ± 5 −3 ± 5 odd.
(A)  (B) 
2 2 46. Let the function f (x) = 3 sin x – 4 cos x + log (|x| +
−2 ± 5
(C)  (D)  None of these 1 + x 2 ) be defined on the interval [0, 1]. The odd
2 extension of f (x) to the interval [– 1, 1] is
Solution:  (A, B)
(A)  3 sin x – 4 cos x + log (|x|+ 1+ x2 )
⎛ x +1⎞ x +1
Since f (x) = f ⎜ ⎟ ⇒x=
⎝ x + 2⎠ x+2  (B)  3 sin x + 4 cos x – log (|x|+ 1+ x2 )
⇒ x2 + x – 1 = 0 (C)  3 sin x + 4 cos x + log (|x|+ 1+ x2 )
−1 ± 5 (D)  None of these
⇒ x= (1)
2
Solution: (B)
Since f (x) is an even function defined on [–5, 5],
To make f (x) an odd function in the interval [–1, 1], we
\ f (– x) = f (x), ∀ x ∈ [– 5, 5] re-define f (x) as follows :
⎛ x +1⎞ ⎧ f ( x ), 0 ≤ x ≤ 1⎫
⇒ x = –⎜ ⇒ x2 + 3x + 1 = 0 (x) = ⎨
f  ⎬
⎝ x + 2 ⎟⎠ ⎩ − f ( − x ), − 1 ≤ x < 0⎭

−3 ± 5 ⎧⎪3 sin x − 4 cos x + log (| x | + 1 + x 2 ), 0 ≤ x ≤ 1 ⎫⎪
⇒ x= (2) =⎨
2 ⎬
⎪⎩ −[ −3 sin x − 4 cos x + log (| x | + 1 + x 2 )], − 1 ≤ x < 0⎪⎭
From Eq. (1) and (2), the values of x are
−1 ± 5 −3 ± 5 ⎧⎪3 sin x − 4 cos x + log (| x | + 1 + x 2 ), 0 ≤ x ≤ 1⎫⎪
and =⎨ ⎬
2 2  ⎪⎩3 sin x + 4 cos x − log (| x | + 1 + x 2 ), − 1 ≤ x < 0⎪⎭
44. The function f (x) = sec [log (x + 1 + x 2 )] is Thus, the odd extension of f (x) to the interval [–1, 1] is
(A) even (B) odd
3 sin x + 4 cos x – log (|x| + 1+ x2 )
(C)  constant (D)  None of these
Solution: (A)
Periodic Function
Since the function sec x is an even function and log (x +
1 + x 2 ) is an odd function, therefore the function sec A function f (x) is said to be a periodic function of x,
p­ rovided there exists a real number T > 0 such that
[log (x + 1 + x 2 )] is an even function.
(x + T) = f (x), ∨ x ∈ R.
f 

Objective_Maths_JEE Main 2017_Ch 2.indd 19 01/01/2008 03:18:48


2.20  Chapter 2

The smallest positive real number T, satisfying the above a


where cos q = 
condition is known as the period or the fundamental period a + b2
2

of f (x).
= a 2 + b 2 sin (kx + q ),
Solved Examples 2p
which is a periodic function of period .
| k|
2 4
47. The period of the function 3(sin p x + x −[ x ]+ sin p x ) , where [⋅] 49. Let f  be a real valued function with domain R satisfying
denotes the greatest integer function, is
f (x + k) = 1 + [2 – 5 f (x) + 10 { f (x)}2 – 10 { f (x)}3 + 5
1
(A)  (B)  1 { f (x)}4 – { f (x)}5]1/5
2
for all real x and some positive constant k, then the
(C) 2 (D) None periodic
period of the function f (x) is
Solution: (B) (A) k (B)  2k
1 − cos 2p x (C)  Non periodic (D)  None of these
sin2 p x = .
2
2p Solution: (B)
Since cos 2p x is a periodic function with period = 1, We have,
2p
therefore sin2 p x is periodic with period 1. (1) f  (x + k) = 1 + [1 + {1 – f (x)}5]1/5
x – [x] is a periodic function with period 1. (2)
1 ⇒ f (x + k) – 1 = [1 – ( f (x) – 1)5]1/5
sin4 p x = (sin2p x)2 = (1 – cos 2p x)2
4 ⇒ f (x + k) = [1 – {f (x)}5]1/5,
1
= (1 + cos2 2p x – 2 cos 2p x) where f (x) = f (x) – 1
4
1 ⇒ f (x + 2k) = [1 – {f (x + k)}5]1/5
= (3 + cos 4p x – 4 cos 2p x)
8 ⇒ f (x + 2k) = [1 – {1 – (f (x))5}]1/5 = f (x), ∀ x ∈ R
2p
Since, cos 4p x is a periodic function with period
4p ⇒ f 
(x + 2k) – 1 = f (x) – 1
1
= and cos 2p x is a periodic function with period ⇒ f (x + 2k) = f (x), ∀ x ∈ R
2
2p \ f 
(x) is periodic with period 2k.
= 1, therefore, period of sin4 p x is equal to
2p
⎛ 1⎞ L.C.M. (1, 1) 1 50. The period of the function f (x) = sin x is
L.C.M. ⎜1, ⎟ = = = 1 (3)
⎝ 2⎠ H.C.F. (1, 2) 1 (A) p (B)  2p
p
From Eq. (1), (2) and (3), we get (C)  (D)  None of these
2
p x + x − [ x ] + sin 4 p x )
2
Period of 3(sin =1
Solution: (D)
48. The period of the function f (x) = a sin kx + b cos kx is
Let f (T + x) = f (x) ⇒ sin T + x = sin x 
2p 2p
(A)  (B) 
k | k| ⇒ T + x = np + (–1)n x 

p Clearly, from here, no positive value of T independent


(C)  (D)  None of these
| k| of x is possible because x on RHS can be cancelled
Solution: (B) only when T = 0.
\ f (x) is a non-periodic function.
We have,
f (x) = a sin kx + b cos kx 51. The period of the function f (x) = tan x is
(A) p (B)  2p
⎛ a b ⎞
= a2 + b2 ⎜ sin kx + cos kx ⎟ p
⎝ a +b
2 2
a +b
2 2
⎠ (C)  (D)  None of these
 2
= a 2 + b 2 (cos q sin kx + sin q cos kx),

Objective_Maths_JEE Main 2017_Ch 2.indd 20 01/01/2008 03:18:52


Functions  2.21

Solution: (A) 55. The function f (x) = k |cos x| + k2 |sin x| + f (k) has


p
Let f (T + x) = f (x) period if k is equal to
2
⇒ tan (T + x ) = tan x ⇒ tan (T + x) = tan x (A) 1 (B) 2
(C)  3 (D)  None of these
⇒ T + x = np + x, n ∈ Z
Solution: (A)
Clearly, from here, the least positive value of T inde-
pendent of x is p. Therefore, f (x) is a periodic function Since |sin x| + |cos x| is a periodic function with period
p p
of period p. , therefore period of f (x) will be  when k = 1.
2 2 
52. The period of the function f (x) = cos x2 is
|sin x | − |cos x |
(A) 2p (B)  p 56. The period of the function f (x) = is
|sin x + cos x |
p p
(C)  (D)  None of these (A)  (B)  2p
2 2
Solution: (D) (C) p (D)  None of these
Let f (T + x) = f (x) ⇒ cos (T + x)2 = cos x2 Solution: (C)
⇒ (T + x)2 = 2np ± x2 We have,
Clearly, from here, no positive value of T independent |sin(p + x )| − |cos (p + x )|
(p + x) =
f 
of x is possible because x2 on R.H.S. can be cancelled |sin(p + x ) + cos(p + x )| 
only when T = 0. |sin x | − |cos x |
\ f (x) is a non periodic function. = = f (x) for all x
|sin x + cos x |
53. The period of the function \ f (x) is periodic with period p.
⎛ p x⎞ ⎛ px ⎞
(x) = cos ⎜
f  − sin ⎜ is
⎝ n ! ⎟⎠
57. The period of the function
⎝ ( n + 1)!⎟⎠
⎧1, when x is a rational⎫
(A)  2 (n + 1)! (B)  2 (n!) f (x) = ⎨ ⎬ is
(C) (n + 1) (D)  Not periodic ⎩0, when x is irrational ⎭
Solution: (A) (A) 1 (B) 2
(C)  non-periodic (D)  None of these
Since sin x and cos x are periodic functions with period
2p. Solution: (D)
⎛ p x⎞ 2p
\ Period of cos ⎜ = = 2 (n!)
⎝ n! ⎟⎠ p / n !
For every rational number T, we have

⎛ px ⎞ 2p ⎧1, when x is a rational⎫


and period of sin ⎜ = = 2 (n + 1)! f (T + x) = ⎨ ⎬ = f (x),
⎝ ( n + 1)!⎟⎠ p / ( n + 1)! ⎩0, when x is irrational ⎭
\ Period of f (x) = L.C.M. of {2 (n!), 2 (n + 1)!} = 2 but there is no least positive value of T for which
(n + 1)! f (T + x) = f (x) because there are infinite number of
rational numbers between any two rational numbers.
54. If the period of the function f (x) = sin ( [n] x), where Therefore, f (x) is a periodic function having no funda-
[n] denotes the greatest integer less than or equal to n, mental period.
is 2p, then
(A) 1 ≤ n < 2 (B)  1 < n < 2 58. Which of the following functions has period p ?
(C) 1 ≤ n ≤ 2 (D)  None of these ⎛ 2p x ⎞ ⎛ p x⎞
(A) 2 cos ⎜ ⎟ + 3 sin ⎜
Solution: (A) ⎝ 3 ⎠ ⎝ 3 ⎟⎠
Sin x is a periodic function with period 2p, therefore (B) |tan x| + cos 2x
2p p⎞ p⎞
sin ( [n] x) is a periodic function with period . ⎛ ⎛
[n] (C) 4 cos ⎜ 2p x + ⎟ + 2 sin ⎜ p x + ⎟
But the period of f (x) is 2p (given). ⎝ 2 ⎠ ⎝ 4⎠
2p (D)  None of these
\ = 2p ⇒ [n] = 1 ⇒ [n] = 1 ⇒ 1 ≤ n < 2.
[n]

Objective_Maths_JEE Main 2017_Ch 2.indd 21 01/01/2008 03:18:55


2.22  Chapter 2

Solution: (B) x 2p
The period of cos x is 2p and that of cos is
⎛ 2p x ⎞ ⎛ p x⎞ = 4p. 2 1 /2
Period of 2 cos ⎜ ⎟ + 3 sin ⎜
⎝ 3 ⎠ ⎝ 3 ⎟⎠ Hence, the period of f (x) is 4p.
⎛ 2p 2p ⎞
= L.C.M. ⎜ , = 6. 62. p is the period of the function
⎝ 2p / 3 p / 3 ⎟⎠ (A) |sin x| + |cos x|
⎛ p⎞ ⎛ p⎞ (B) sin4x + cos4x
Period of 4 cos ⎜ 2p x + ⎟ + 2 sin ⎜⎝ p x + ⎟⎠
⎝ 2⎠ 4 (C)  sin (sin x) + sin (cos x)
⎛ 2p 2p ⎞ 1 + 2 cos x
= L.C.M. ⎜ , = 2
⎝ 2p p ⎟⎠
(D) 
sin x ( 2 + sec x )
⎛ 2p ⎞
Period of |tan x| + cos 2x = L.C.M. ⎜ p ,
⎝ ⎟ = p. Solution: (D)
p
2⎠ The period of |sin x| + |cos x| and sin4x + cos4x is ⋅
2 
59. The period of the function f (x) = x [x] is sin (sin x) + sin (cos x) has period 2p. The function
(A) 1 (B) 2 1 + 2 cos x
(C)  non periodic (D)  None of these can be written in a simplified form as
sin x ( 2 + sec x )
Solution: (C) cos x
= cot x, so it has period p.
Let n ≤ x < n + 1 sin x
Then, f (x) = x ⋅ n, where n changes with x. x2 + 1
63. If f (x) = , ([⋅] denotes the greatest integer
[ x]
Clearly no constant k > 0 is possible for which f (x) =
f (x + k) corresponding to all x. ­function), 1 ≤ x < 4, then
\ f (x) is a non periodic function. ⎡ 17 ⎞
(A)  range of f is ⎢ 2, ⎟
60. The period of the function ⎣ 3⎠
px (B) f is monotonically increasing in [1, 4]
(x) = 3x + 3 – [3x + 3] + sin
f  ,
2
where [x] denotes the greatest integer ≤ x, is 17
(C)  the maximum value of f (x) is
(A) 4 (B) 1 3
(C) 2 (D) Non-periodic 17
(D)  the maximum value of f (x) is
Solution: (A) 4
px Solution: (A)
3x + 3 – [3x + 3] has the period 1 and sin has x2 + 1
2 We have, f (x) = .
2p [ x]
the period i.e., 4. Therefore, the period of f (x) is
p /2
When x ∈ [1, 2) then f (x) = x2 + 1 ⇒ Rf = [2, 5).
L.C.M. (1, 4) = 4.
x2 + 1 ⎡5 ⎞
61. The value of n ∈ I for which the function When x ∈ [2, 3) then f (x) = ⇒ Rf = ⎢ , 5⎟ .
sin nx
2 ⎣2 ⎠
(x) =
f  has 4p as its period is
⎛ x⎞ x2 + 1 ⎡10 17 ⎞
sin ⎜ ⎟ When x ∈ [3, 4) then f (x) = ⇒ Rf = ⎢ , ⎟ .
⎝ n⎠ 3 ⎣3 3⎠
\ Rf = [2, 17/3).
(A) 2 (B) 3 (C) 4 (D) 5
64. The number of solutions of the equation a f (x) + g (x) = 0,
Solution: (A)
1
For n = 2, we have a > 0, g (x) ≠ 0 and has minimum value is
2
sin 2 x 2 sin x cos x (A)  One (B)  Two
(x) =
f  =
⎛ x⎞ ⎛ x⎞ (C) Zero (D) Infinitely many
sin ⎜ ⎟ sin ⎜ ⎟
⎝ 2⎠ ⎝ 2⎠
 Solution: (C)
4 sin x / 2 cos x / 2 cos x x We have
= = 4 cos cos x.
⎛ ⎞
x 2 a f (x) + g (x) = 0, a > 0
sin ⎜ ⎟
⎝ 2⎠

Objective_Maths_JEE Main 2017_Ch 2.indd 22 01/01/2008 03:18:59


Functions  2.23

Since minimum value of g (x) is 1/2 68. If f (x + y, x – y) = xy, then the arithmetic mean of
\ g (x) > 0 and a f (x) > 0 f (x, y) and f ( y, x) is
\ a f (x) + g (x) > 0, ∀ x (A) y (B) x
(C)  0 (D)  None of these
Hence number of solutions is zero.
Solution: (C)
65. If f : R → R, g : R → R be two given functions then
f (x) = 2 min { f (x) – g (x), 0} equals Let x + y = a and x – y = b
(A) f (x) + g (x) – |g (x) – f (x)| a+b a−b
⇒ x= and y =
(B) f (x) + g (x) + |g (x) – f (x)| 2 2 
(C) f (x) – g (x) + |g (x) – f (x)| \ f (x + y, x – y) = xy
(D) f (x) – g (x) – |g (x) – f (x)| a+b a−b a2 − b2
⇒ f (a, b) = ⋅ =
Solution: (D) 2 2 4 
We have, x −y
2 2
y2 − x2
\ f (x, y) = and f (y, x) =
f (x) = 2 min { f (x) – g (x), 0} 4 4 
⎧0 f ( x) > g( x) \ Arithmetic mean of f (x, y) and f (y, x) is zero.
=⎨
⎩2[ f ( x ) − g ( x )], f ( x ) ≤ g ( x )  ⎛ 1⎞ 1
69. If 3 f (x) + 5 f ⎜ ⎟ = – 3, ∀ x (≠ 0) ∈ R, then f (x) =
⎝ x⎠ x
⎧ f ( x ) − g ( x ) − | f ( x ) − g ( x ) |, f ( x) > g( x)
= ⎨ 1 ⎛3
⎩ f ( x ) − g ( x ) − | f ( x ) − g ( x ) |, f ( x) ≤ g( x) ⎞ 1 ⎛ 3 ⎞
 (A)  ⎜⎝ + 5 x − 6⎟⎠ (B)  ⎜⎝ − + 5 x − 6⎟⎠
14 x 14 x
\ f (x) = f (x) – g (x) –|g (x) – f (x)|
1 ⎛ 3 ⎞
66. If f : R → R is a function satisfying the property
(C)  ⎜ − + 5 x − 6⎟⎠ (D)  None of these
14 ⎝ x
f (2x + 3) + f (2x + 7) = 2, ∀ x ∈ R, then the period of
f (x) is Solution: (B)
(A) 2 (B) 4 (C) 8 (D) 12 We have,
⎛ 1⎞ 1
3 f (x) + 5 f ⎜ ⎟ = – 3, ∀ x (≠ 0) ∈ R(1)
Solution: (B) ⎝ x⎠ x
We have, ⎛ 1⎞
⇒ 3 f ⎜ ⎟ + 5 f (x) = x – 3 (2)
f  (2x + 3) + f (2x + 7) = 2 (1) ⎝ x⎠
Replace x by x + 1, ⎡ 1⎤
⎢ Replacing x by x ⎥
f (2x + 5) + f (2x + 9) = 2 (2)  ⎣ ⎦
Now replace x by, Multiplying Eq. (1) by 3 and (2) by 5 and subtracting,
we get
x + 2, f (2x + 7) + f (2x + 11) = 2 (3)
⎛3 ⎞
From (1) – (3), we get 9 f (x) – 25 f (x) = ⎜ − 9⎟ – (5x – 15)
⎝x ⎠
f (2x + 3) – f (2x + 11) = 0 3
⇒ – 14 f (x) = – 5x + 6
(2x + 3) = f (2x + 11) ⇒ T = 4
i.e., f  x
67. If T1 is the period of the function y = e3(x–[x]) and T2 is 1 ⎛ 3 ⎞
⇒ f (x) = ⎜ − + 5 x − 6⎟⎠ , ∀ x (≠ 0) ∈ R
the period of the function y = e3x–[3x] ([⋅] denotes the 14 ⎝ x
greatest integer function), then
T
(A) T1 = T2 (B)  T1 = 2 caution
3
(C) T1 = 3T2 (D)  None of these There are certain function which are periodic but don’t have
a fundamental period. For example the constant function
Solution: (C)
f(x) = c is periodic as
(x) = e3{x} ⇒ T1 = 1
Let g  f(x + T) = f(x)
and f (x) = e{3x} ⇒ T2 = 1/3 is true for every real number T, but it does not have a fun-
damental period since the least positive value of T cannot
\ T1 = 3T2. be obtained.

Objective_Maths_JEE Main 2017_Ch 2.indd 23 01/01/2008 03:19:02


2.24  Chapter 2

Short-Cut Method to Check the


Periodicity of a Function ⎧L.C.M. of {T1, T2 , T3 } if h( x ) is an odd function

= ⎨1
1. Put f (T + x) = f (x) and solve this equation to find the ⎪⎩ 2 L.C.M. of {T1, T2 , T3 } if h( x ) is an even function
positive values of T independent of x.
 If f(x) is periodic with period T, then kf  (ax + b) is also
2. If no positive value of T independent of x is obtained, T
then f (x) is a non-periodic function. periodic with period , where a, b, k ∈ R and a, k ≠ 0.
| a|
3. If positive values of T independent of x are obtained,
 sin x, cos x, sec x and cosec x are periodic functions with
then f (x) is a periodic function and the least positive period 2p.
value of T is the period of the function f (x).
 tan x and cot x are periodic functions with period p.

 |sin x|, |cos x|, |tan x|, |cot x|, |sec x| and |cosec x|
Trick(s) for Problem Solving are periodic functions with period p.
 sin x, cos x, sec x and cosec x are periodic functions
n n n n
 Constant function is periodic with no fundamental period. with period 2p when n is odd or p when n is even.
1
 If f(x) is periodic with period T, then and f( x ) are  tan x and cot x are periodic functions with period p.
n n

also periodic with same period T. f ( x)


 If f(x) is a periodic function with period T and g(x) is any
 If f1(x), f2(x) and f3(x) are periodic functions with periods function such that range of f ⊂ domain of g, then gof is
T1, T2 and T3 respectively. Also, if h(x) = af1(x) ± bf2(x) ± also periodic with period T.
cf3(x), then, period of [h(x)]

EXERCISES

Single Option Correct Type

1. Let f (x) = x3 + x2 + 100 x + 7sin x, then the equation 5. The graph of the function cos x cos (x + 2) –cos2 (x + 1)
1 2 3 is
+ + = 0 has
y − f (1) y − f ( 2) y − f (3) (A) a straight line passing through (0, –sin2 1) with
(A)  one real root (B)  two real roots slope 2
(C)  more than two real roots (D)  no real root (B)  a straight line passing through (0, 0)
(C)  a parabola with vertex (1, –sin2 1)
2. If b2 – 4ac = 0 and a > 0, then the domain of the func-
(D) a straight line parallel to x-axis passing through
tion f (x) = log (ax3 + (2a + b) x2 + (2b + c) x + 2c) is
⎛p ⎞
⎧ b⎫ ⎧ b⎫ the point ⎜ , − sin 2 1⎟
(A)  (– 2, ∞)\ ⎨ − ⎬ (B)  [– 2, ∞)\ ⎨ − ⎬ ⎝2 ⎠
⎩ 2a ⎭ ⎩ 2a ⎭ x2 − 8
6. Let f : R → R be a function defined by, f (x) = 2 ,
⎧ b⎫ then f is x +2
(C) (– ∞, – 2)\ ⎨ − ⎬ (D)  None of these
⎩ 2a ⎭ (A)  one-one but not onto
3. If ex + ef (x) = e, then range of the function f is (B)  one-one and onto
(A) (–∞, 1] (B)  (–∞, 1) (C)  onto but not one-one
(C) (1, ∞) (D)  [1, ∞) (D)  neither one-one nor onto
1 1
4. Which of the following functions is are injective 7. If f (x) = 64x3 + 3 and a, b are the roots of 4x + = 3,
map(s)? then x x
(A) f (x) = x2 + 2, x ∈ (– ∞, ∞) (A) f (a) = 12 (B)  f (b) = 11
(B) f (x) = |x + 2|, x ∈ [– 2, ∞) (C) f (a) = f (b) (D)  None of these
(C) f (x) = (x – 4) (x – 5), x ∈ (– ∞, ∞)
8. If the functions f, g, h are defined from the set of real
4 x 2 + 3x − 5 numbers R to R such that
(D) f (x) = , x ∈ (– ∞, ∞)
4 + 3x − 5 x 2

Objective_Maths_JEE Main 2017_Ch 2.indd 24 01/01/2008 03:19:04


Functions  2.25

⎧0, if x ≤ 0 14. The image of the interval [1, 3] under the mapping
f (x) = x2 – 1, g (x) =x 2 + 1 , h (x) = ⎨ f : R → R, given by f (x) = 2x3 – 24x + 107 is
⎩ x, if x ≥ 0
(A)  [0, 89] (B)  [75, 89]
then the composite function (hofog) (x) =
(C)  [0, 75] (D)  None of these
⎧0, x=0
⎪ 2 ⎧0, x = 0 ⎛ 1⎞
(A)  ⎨ x , x > 0 (B)  15. If 2 f (x) – 3 f ⎜ ⎟ = x2, x is not equal to zero, then f (2)
⎨ 2 ⎝ x⎠
⎪− x 2 , x < 0 ⎩x , x ≠ 0 is equal to

7 5
(A) – (B) 
⎧0, x ≤ 0 4 2
(C)  ⎨ 2 (D)  None of these
⎩x , x > 0 (C)  – 1 (D)  None of these
2x − 1
9. If S is the set of all real x and such that 16. Let f (x) = (1 + b2) x2 + 2bx + 1 and m(b) the minimum
is positive, then S contains 2 x + 3x 2 + x
3
value of f (x) for a given b. As b varies, the range of
⎛ 3⎞ ⎛ 3 1⎞ m(b) is
(A)  ⎜ −∞, − ⎟ (B)  ⎜ − ,− ⎟ ⎛ 1⎤
⎝ 2⎠ ⎝ 2 4⎠ (A)  [0, 1] (B)  ⎜⎝ 0, ⎥
2⎦
⎛ 1 1⎞ ⎛1 ⎞ ⎡1 ⎤
(C)  ⎜ − , ⎟ (D) 
⎜⎝ , 3⎟⎠ (C)  ⎢ , 1⎥ (D)  (0, 1]
⎝ 4 2⎠ 2 ⎣2 ⎦
10. The number of values of x, where the function f (x) = 17. Let f : R → R, g : R → R be two functions given by
cos x + cos ( 2 x) attains its maximum, is f (x) = 2x – 3, g (x) = x3 + 5. Then ( fog)–1 (x) is equal to
(A) 0 (B) 1 ⎛ x − 7⎞
1/ 3
⎛ x + 7⎞
1/ 3

(A)  ⎜ (B) 
(C) 2 (D) infinite ⎝ 2 ⎟⎠ ⎜⎝
2 ⎠

11. The distinct linear function (s) which map (s) [–1, 1] ⎛ 7⎞
1/ 3
⎛ x − 2⎞
1/ 3

onto [0, 2] is (are) (C)  ⎜ x − ⎟ (D) 


⎜⎝ ⎟
⎝ 2⎠ 7 ⎠
(A) x + 1, – x + 1 (B)  x – 1, x + 1
(C) – x + 1 (D)  None of these 18. The functions f (x) = log (x – 1) – log (x – 2) and g (x) =
⎛ x − 1⎞
12. Let f (x) = max. {(1 – x), (1 + x), 2}, ∀ x ∈ R. Then log ⎜ are identical when x lies in the interval
⎝ x − 2 ⎟⎠
⎧1 + x, x ≤ −1 (A) [1, 2] (B) [2, ∞]

(A) f (x) = ⎨2, − 1 < x < 1 (C) (2, ∞) (D)  (– ∞, ∞)
⎪1 − x, x ≥ 1
⎩ ⎧ − 1, x<0

⎧1 − x, x ≤ −1 19. Let g (x) = 1 + x – [x] and f (x) = ⎨ 0, x = 0 . Then,
⎪ ⎪ 1,
(B) f (x) = ⎨1, −1 < x < 1 ⎩ x>0
⎪1 + x, x ≥ 1 for all x, f [g (x)] is equal to
⎩ (A) x (B) 1 (C) f (x) (D) 
g (x)
⎧1 − x, x ≤ −1
⎪ 1
(C) f (x) = ⎨2, − 1 < x < 1 20. The domain of the function y = log is
⎪1 + x, x ≥ 1 | sin x |

(A) R\{np : n ∈ Z} (B) R\(– p, p)
(D)  None of these (C) R\{2np : n ∈ Z} (D) (– ∞, ∞)
13. If f (x) = sin [p 2] x + sin [–p 2] x, where [⋅] denotes the x 2 + 34 x − 71
greatest integer function, then 21. If x is real, then the expression
x2 + 2x − 7
⎛p⎞ (A)  cannot lie between 5 and 9
(A) f ⎜ ⎟ = 1 (B)  f (p) = 2 (B)  always lies between 5 and 9
⎝ 2⎠
(C)  is not real
⎛p⎞ (D)  None of these
(C) f ⎜ ⎟ = –1 (D)  None of these
⎝ 4⎠

Objective_Maths_JEE Main 2017_Ch 2.indd 25 01/01/2008 03:19:07


2.26  Chapter 2

22. If f (x) is an odd periodic function with period 2, then 31. The range of the function
f (4) equals sin (p [ x 2 + 1])
f (x) = where, [ ] is greatest integer
(A) – 4 (B) 4 (C) 2 (D) 0 x4+ 1
function, is
23. The function (A)  [0, 1] (B)  [– 1, 1]
f (x) = cot–1 [ ( x + 3) x ] + cos–1 ( x 2 + 3 x + 1 ) is (C)  {0} (D)  None of these
defined on the set S, where S is equal to
⎛ 1⎞ 5
(A)  {– 3, 0} (B)  [– 3, 0] 32. If a f (x) + bf ⎜ ⎟ = x + , (a ≠ b), then f (x) is equal to
(C)  [0, 3] (D)  (– 3, 0) ⎝ x⎠ x

24. If f (x) = acos x and g (x) = (sin x)a, a ∈ N, then


1 ⎛ 1⎞
(A)  ⎜⎝ x + ⎟
a − b2
2
x⎠
(A) f (x) > g (x), ∀ x
(B) f (x) < g (x), ∀ x 1 ⎡ 1 ⎤
(C) f (x) = g (x), for infinitely many values of x (B) 
a − b2
2 ⎢ x(5a − b) + x (5b − a) ⎥
⎣ ⎦
(D) f (x) ≠ g (x), for any x
1 ⎡ 1 ⎤
25. Let f be a function satisfying f (x + y) = f (x) f (y) for all (C)  ⎢ x( a − 5b) + x (5a − b) ⎥
a − b2
2
n ⎣ ⎦
x, y ∈ . If f (1) = 3, then ∑ f ( r ) is equal to (D)  None of the above
r =1
3 3
(A)  (3n − 1) (B)  n ( n + 1) 33. Let f : R → R defined by,
2 2
(x) = x3 + x2 + 100x + 5 sin x, then f is
f 
(C) 3n + 1 – 3 (D)  None of these
(A)  many–one onto (B)  many–one into
26. Let f (x) be defined for all x > 0 and be continuous. Let (C)  one–one onto (D)  one–one into
⎛ x⎞
f (x) satisfy f ⎜ ⎟ = f (x) – f ( y) for all x, y and f (e) = 1. 34. Let f  be a real valued function with domain R ­satisfying
⎝ y⎠ 1
Then 0 ≤ f (x) ≤ and for some fixed a > 0
⎛ 1⎞ 2
(A) f (x) is bounded (B)  f ⎜ ⎟ → 0 as x → 0
⎝ x⎠ 1
(x + a) = − f ( x ) − ( f ( x )) 2 ∀ x ∈ R,
f 
(C) x f (x) → 1 as x → 0 (D)  f (x) = log x 2
then the period of the function f(x) is
27. If g (x) = 1 + x and f [g (x)] = 3 + 2 x + x, then f (x) = (A) a (B) 2a
(A) 1 + 2x2 (B)  2 + x2 (C)  non-periodic (D)  None of these
(C) 1 + x (D)  2+x
35. Let f (x) = sin x + cos x, g(x) = x2 – 1. Then g( f (x)) is
28. Range of values of f (x) = 1 + sin x + sin3x + sin5x …, invertible for x ∈
⎛ −p p⎞ ⎡ p ⎤ ⎡ p ⎤
x∈ ⎜ , is
⎝ 2 2 ⎟⎠ ⎢− 2 , p ⎥
(A)  ⎢ − , 0 ⎥ (B) 
⎣ 2 ⎦ ⎣ ⎦
(A) (0, 1) (B) (– ∞, ∞)
(C)  (– 2, 2) (D)  None of these ⎡ p p⎤ ⎡ p⎤
(C)  ⎢ − , ⎥ (D) 
⎢0, 2 ⎥
⎣ 4 4⎦ ⎣ ⎦
29. The function f : (– ∞, – 1] → (0, e5] defined by,
⎛ y y⎞
36. If f ⎜ 2 x + , 2 x − ⎟ = xy, then f (m, n) + f (n, m) = 0
3
−3x + 2
f (x) = e x is ⎝ 8 8⎠
(A)  Many one and onto
(A)  only when m = n
(B)  Many one and into
(B)  only when m ≠ n
(C)  One-one and onto
(C)  only when m = – n
(D)  One-one and into
(D)  for all m and n.
30. The domain of the function
37. If f (x) is defined on (0, 1), then the domain of d­ efinition
⎛ ⎛ 1 ⎞ ⎞ of f (ex) + f (ln|x|) is
(x) = log 2 ⎜ − log1/ 2 ⎜1 + 4 ⎟ − 1⎟ is
f 
⎝ ⎝ x⎠ ⎠ (A) (– e, – 1)
(A) 0 < x < 1 (B)  0 < x ≤ 1 (B) (– e, – 1) ∪ (1, e)
(C) x ≥ 1 (D)  x>1 (C) (– ∞, – 1) ∪ (1, ∞)
(D) (– e, e)

Objective_Maths_JEE Main 2017_Ch 2.indd 26 01/01/2008 03:19:11


Functions  2.27

⎡1⎤ ⎡1 1 ⎤ ⎡1 2 ⎤ 44. If the graph of y = ax3 + bx2 + cx + d is symmetric


38. The value of ⎢ 2 ⎥ + ⎢ 2 + 100 ⎥ + ⎢ 2 + 100 ⎥ +  + about the line x = k, then the value of a + k is
⎣ ⎦ ⎣ ⎦ ⎣ ⎦
⎡ 1 99 ⎤ c
⎢ 2 + 100 ⎥ is (A) −
2b
(B)  c
⎣ ⎦
(A) 49 (B) 50 (C) 51 (D) 98 (C) c – bd (D)  None of these
1 ⎛ p x⎞
39. The domain of definition of 45. If function f (x) = − tan ⎜ ⎟ ; (–1 < x < 1) and
2 ⎝ 2⎠
log 0.3 | x − 2 |
(x) =
f  is g(x) = 3 + 4 x − 4 x 2 , then the domain of gof is
|x|
(A)  [1, 2) ∪ (2, 3] ⎡ 1 1⎤
(B)  [1, 3] (A)  (– 1, 1) (B)  ⎢ − ,
(C)  – (1, 3] (D)  None of these ⎣ 2 2 ⎥⎦

40. Let f : R → R be a function defined by, f (x) = ⎡ 1⎤ ⎡ 1 ⎤


(C)  ⎢ − 1, ⎥ (d) 
⎢ − , − 1⎥
| x |3 + | x | ⎣ 2⎦ ⎣ 2 ⎦
− , then the graph of f (x) lies in which
1 + x2 46. If f : R → R is a function such that f (x) = x3 + x2f  ′(1) +
quadrant(s)? xf  ″(2) + f  ″′(3) for all x ∈ , then f (2) – f (1) =
(A)  I and II (B)  I and III (A) f (0) (B) –f (0) (C)  f  ′(0) (D) –f  ′(0)
(C)  II and III (D)  III and IV
⎧ p⎫
41. The domain of definition of the function 47. Let f :  → A = ⎨ y : 0 ≤ y < ⎬ be a function such
⎩ 2⎭
sin − 1 x + x2 + 1 + x − [ x ] + log x that f (x) = tan (x + x + k), where k is a constant. The
–1 2

(x) =
f  is minimum value of k for which f  is an onto function, is
⎛ ⎛ 1 ⎞⎞
sin x + cos x
e + log ⎜ sin ⎜ ⎟⎟ (A) 1 (B) 0
⎜⎝ ⎝ − x 2 ⎠ ⎟⎠ 1
(C)  (D)  None of these
(A)  (– 1, 1) (B)  (0, 1) 4
(C)  (1, 0) (D)  None of these 48. Suppose f : [2, 2] → R is defined by,
42. If f : R → R and g : R → R are given by f (x) = |x| and ⎧ −1 for − 2 ≤ x ≤ 0
(x) = ⎨
f  ,
g(x) = [x] for each x ∈ R, then {x ∈ R : g[ f (x)] ≤ f ⎩ x − 1 for 0 ≤ x ≤ 2
[g(x)]} = then {x ∈ (–2, 2) : x ≤ 0 and f (|x|) = x} =
(A) Z ∪ (– ∞, 0) (B)  (– ∞, 0) (A) {– 1} (B) {0}
(C) Z (D)  R (C) {– 1/2} (D)  f
43. The function 49. Let f (x) = [x]2 + [x + 1] – 3, where [x] is greatest i­ nteger
1 1 less than or equal to x, then
(x) = sin −1 ( x − x 2 ) + 1 −
f  + 2
| x | [ x − 1] (A) f (x) is a many one and into function
is defined in the interval (where [⋅] is the greatest (B) f (x) = 0 for infinite number of values of x
integer) (C) f (x) = 0 for only two real values
⎛ (D)  None of these
1+ 5⎞
(A) x ∈ ⎜ 2 ,
⎝ 2 ⎟⎠ 50. If q2 – 4pr = 0, p > 0, then the domain of the function
f (x) = log [ px3 + (p + q)x2 + (q + r)x + r] is
⎛ 1+ 5⎞
(B) x ∈ ⎜1, ⎧ q ⎫
⎝ 2 ⎟⎠ (A) R – ⎨ − ⎬
⎩ 2 p⎭
⎡1 − 5 1 + 5 ⎤
(C) x ∈ ⎢ , ⎥ ⎡ ⎧ q ⎫⎤
(B) R – ⎢( − ∞, − 1] ∪ ⎨ − ⎬ ⎥
⎣ 2 2 ⎦
⎣ ⎩ 2 p ⎭⎦
⎛ 1+ 5⎞
(D) x ∈ ⎜ − 2 , ⎡ ⎧ q ⎫⎤
⎝ 2 ⎟⎠ (C) R – ⎢( − ∞, − 1) ∩ ⎨ − ⎬ ⎥
⎣ ⎩ 2 p ⎭⎦
(D)  None of these

Objective_Maths_JEE Main 2017_Ch 2.indd 27 01/01/2008 03:19:13


2.28  Chapter 2

51. If f : (3, 6) → (1, 3) is a function defined by f (x) =


59. If f : R → S, defined by f (x) = sin x − 3 cos x + 1 , is
⎡x⎤ onto then the interval of S is
x − ⎢ ⎥ (where [ ] dentoes the greatest integer func-
⎣3⎦ (A)  [– 1, 3] (B)  [– 1, 1]
tion), then f –1(x) = (C)  [0, 1] (D)  [0, 3]
(A) x – 1 (B)  x + 1 60. If a and b are natural numbers and
(C) x (D)  None of these
(x) = sin ( a 2 − 3 ) x + cos ( b 2 + 7 ) x
f 
52. If [x] denotes the integral part of x, then the domain of the
function f (x) = sin– 1 [2x2 – 3] + log2 [log1/2(x2 – 5x + 5)] is is periodic with finite fundamental period, then period
of f (x) is
⎛ 5 ⎤ ⎡ 5⎞ (A)  p (B)  2p
(a) ⎜ − , − 1⎥ (B) ⎢1,
⎝ 2 ⎦ ⎣ 2 ⎟⎠
(C) 2p ( a 2 − 3 + b 2 + 7 )
⎛ 5 ⎤ ⎡ 5⎞
(C)  ⎜ − , − 1⎥ ∪ ⎢1, (D)  None of these (D)  p ( a 2 − 3 + b 2 + 7 )
⎝ 2 ⎦ ⎣ 2 ⎟⎠
⎛ 1⎞
61. If 2f (x) + 3 f ⎜ ⎟ = x2 – 1, then f (x) is
2 f ( n) + 1 ⎝ x⎠
53. If f (n + 1) = , n = 1, 2, … and f (1) = 2, then
f (101) equals 2 (A)  a periodic function (B)  an even function
(C)  an odd function (D)  None of these
(A) 52 (B) 49 (C) 48 (D) 51
⎛ x1 − x2 ⎞
62. If f (x1) – f (x2) = f ⎜ for x1, x2 ∈ [– 1, 1] then
54. If f (x) =
cos 2 x + sin 4 x
for x ∈ R, then f (2002) = f (x) is ⎝ 1 − x1 x2 ⎟⎠
sin 2 x + cos 4 x ⎛1 − x⎞ ⎛1 − x⎞
(A) log ⎜ tan −1 ⎜
(B) 
(A) 1 (B) 2 (C) 3 (D) 4 ⎝ 1 + x ⎟⎠ ⎝ 1 + x ⎟⎠
55. Let f (x) = x + 1 and f (x) = x – 2, then the values of x ⎛1 + x⎞ ⎛1 + x⎞
satisfying| f (x) + f (x)| = |f (x)| + |f (x)| are (C) log ⎜ tan −1 ⎜
(D) 
⎝ 1 − x ⎟⎠ ⎝ 1 − x ⎟⎠
(A) (–∞, 1) (B)  (2, ∞)
(C) (–∞, –2) (D)  (1, ∞) 63. Let f : R → R be a periodic function such that
(T + x) = 1 + {1 – 3f (x) + 3[ f (x)]2 – [ f (x)]3}1/3,
f 
56. A function f  from the set of natural numbers to ­integers
defined by, where T is a fixed positive number, then period of f (x) is
⎧n − 1 (A) T (B)  2T
⎪⎪ 2 , when n is odd (C) 3T (D)  None of these
(n) = ⎨
f  is
⎪− n , when n is even 64. The domain of the function
⎪⎩ 2
(A)  neighter one–one nor onto − log 0.3 ( x − 1)
(x) =
f  is
(B)  one–one but not onto − x 2 + 3 x + 18
(C)  onto but not one–one (A)  [2, 6] (B)  (2, 6)
(D)  one–one and onto both (C)  [2, 6) (D)  None of these
⎛ p⎞ tan 2 a 65. Suppose f (x) = (x + 1)2 for x ≥ – 1. If g(x) is the func-
57. If a ∈ ⎜ 0, ⎟ then x 2 + x + is always
⎝ 2⎠ x2 + x tion whose graph is the reflection of the graph of f (x)
greater than or equal to with respect to the line y = x, then g(x) equals
(A)  2 tan a (B)  1 1
(C) 2 (D) sec2 a (A)  − x − 1 , x ≥ 0 (B)  ,x>–1
( x + 1) 2
58. The function f (x) is defined in [0, 1], then the domain
of definition of the function f [log (1 – x2)], is (C)  x + 1 , x ≥ – 1 (D)  x −1, x ≥ 0
(A) x ∈ {0}
sin101 x
(B) x ∈ [ − 1 + e , − 1] ∪ [1, + 1 + e ] 66. The function f (x) = , where [x] denotes the
⎡x⎤ 1
(C) x ∈ (– ∞, ∞) ⎢⎣ p ⎥⎦ + 2
(D)  None of these integral part of x is

Objective_Maths_JEE Main 2017_Ch 2.indd 28 01/01/2008 03:19:17


Functions  2.29

(A)  an odd function ⎡ 1 ⎤


(B)  an even function 74. Range of the function f defined by f (x) = ⎢ ⎥
(C)  neither odd nor even function ⎣ sin{x} ⎦
(D)  both odd and even function (where [ ⋅ ] and { ⋅ } respectively denote the greatest
integer and the fractional part functions is:)
1
67. The domain of the function f (x) = is (A)  Z, the set of integers
|x|− x (B)  N, the set of natural numbers
(A) (–∞, ∞) – {0} (B)  (–∞, ∞) (C)  W, the set of whole numbers
(C) (0, ∞) (D)  (–∞, 0) (D)  {2, 3, 4, …}
68. The total number of injective mappings from a set with 75. If f : R → R, g : R → R be two given functions then
n element to a set with n elements, for m > n, is f (x) = 2 min {|f (x) – g(x)|, 0} equals
m! m! (A) f (x) + g(x) – |g(x) – f (x)|
(A)  (B)  (B) f (x) + g(x) + |g(x) – f (x)|
n !( m − n)! ( m − n)!
(C) f (x) – g(x) + |g(x) – f (x)|
(C) nm (D)  zero
(D) f (x) – g(x) – |g(x) – f (x)|
69. Let f : N → Y be a function defined as f (x) = 4x + 3,
76. Let f (x) be a function defined on [0, 1] such that
where Y = {y ∈ N : y = 4x + 3 for some x ∈ N}. Show
that f is invertible and its inverse is ⎧x x ∈Q
f (x) = ⎨
3y + 4 y+3 ⎩1 − x x ∉ Q
(A) g(y) = g(y) = 4 +
(B)  Then, for all x ∈ [0, 1],  fof (x) is
3 4
(A) a constant (B) 1 + x
y+3 y−3 (C) x (D)  None of these
(C) g(y) = g(y) =
(D) 
4 4
77. If a function f : R → R be such that f (x – f (y)) = f ( f (y))
70. For real x, let f (x) = x3 + 5x + 1, then + xf (y) + f (x) – 1, ∀ x, y ∈ R, then f (x) =
(A) f  is one–one but not onto R x2 x2
(B) f  is onto R but not one–one (A)  − 1 (B)  +1
2 2
(C) f  is one–one and onto R x 2

(D) f  is neither one–one nor onto R (C) 1 − (D)  None of these
2
71. Let f (x) = (x + 1)2 – 1, x ≥ –1 78. A function whose graph is symmetrical about the
Statement 1: The set {x : f (x) = f –1 (x)} = {0, –1} ­origin is given by
Statement 2: f is a bijection. (A) f (x) = (3x + 3–x)
(A) Statement 1 is true, Statement 2 is true; Statement 2 (B) f (x) = cos [log( x + 1 + x 2 )]
is a correct explanation for Statement 1 (C) f (x + y) = f (x) + f (y) ∀ x, y ∈ R
(B) Statement 1 is true, Statement 2 is true; Statement 2 (D)  None of these
is not a correct explanation for Statement 1 79. Which of the following functions is (are) injective
(C)  Statement 1 is true, Statement 2 is false map(s)?
(D)  Statement 1 is false, Statement 2 is true
(A) f (x) = x2 + 2, x ∈ (– ∞, ∞)
72. The period of the function (B) f (x) =|x + 2|, x ∈ [ − 2, ∞ )
⎧1, when x is a rational (C) f (x) = (x – 4) (x – 5), x ∈(– ∞, ∞)
(x) = ⎨
f  is
⎩0, when x is irrational 4 x 2 + 3x − 5
(D) f (x) = , x ∈ (– ∞, ∞)
(A) 1 (B) 2 4 + 3x − 5 x 2
(C)  non-periodic (D)  None of these 80. Let f be a function with domain [–3, 5] and let g(x) =
1 1 1 |3x + 4|. Then, the domain of (fog)(x) is
73. Let f1(n) =1 +
+ + ... + , then f1(1) + f1(2) + f1(3) ⎛ 1⎞ ⎡ 1⎤
2 3 n (A)  ⎜ − 3, ⎟ (B)  − 3, ⎥
+ … + f1(n) is equal to ⎝ 3⎠ ⎢
⎣ 3⎦
(A) nf1(n) – 1 (B)  (n + 1)f1(n) + n ⎡ 1⎞
(C) (n + 1)f1(n) – n (D)  nf1(n) + n (C)  ⎢ − 3, ⎟ (D)  None of these
⎣ 3⎠

Objective_Maths_JEE Main 2017_Ch 2.indd 29 01/01/2008 03:19:20


2.30  Chapter 2

81. If for a real number x, [x] denotes the greatest integer 88. If f : R → R is a function such that f (x) = x3 + x2 f ′(1)
less than or equal to x, then for any n ∈ N + x f ′′(2) + f ′′′(3) for all x ∈ R, then f (2) – f (1) =
⎡ n + 1⎤ ⎡ n + 2 ⎤ ⎡ n + 4 ⎤ ⎡ n + 8 ⎤ (A) f  (0) (B)  – f (0)
⎢ 2 ⎥ + ⎢ 4 ⎥ + ⎢ 8 ⎥ + ⎢ 16 ⎥ + ... = (C) f ′(0) (D)  – f ′(0)
⎣ ⎦ ⎣ ⎦ ⎣ ⎦ ⎣ ⎦
(A) n (B)  n–1 89. If q2 – 4pr = 0, p > 0, then the domain of the function
(C) n + 1 (D)  n+2 f (x) = log {px3 + (p + q)x2 + (q + r)x + r} is
⎧ − 1, x < 0 ⎧ q ⎫
⎪ (A)  R − ⎨ − ⎬
82. Let g(x) = 1 + x – [x] and f (x) = ⎨0, x = 0 . Then, for ⎩ 2 p⎭
⎪1, x > 0
all x, f [g(x)] is equal to ⎩ ⎡ ⎧ q ⎫⎤
(B)  R − ⎢( − ∞, − 1] ∪ ⎨ − ⎬⎥
(A) x (B) 1 (C) f (x) (D)  g(x). ⎣ ⎩ 2 p ⎭⎦
83. If g(x) = 1 + x  and f [g(x)] = 3 + 2 x + x , then f (x) = ⎡ ⎧ q ⎫⎤
(A) 1 + 2x2 (B) 2 + x2 (C) 1 + x (D) 2 + x (C)  R − ⎢( − ∞, − 1) ∩ ⎨ − ⎬⎥
⎣ ⎩ 2 p ⎭⎦
84. The domain of the function f (x) =
(D)  None of these
⎛ ⎛ 1 ⎞ ⎞
log2 ⎜ − log1/ 2 ⎜1 + 4 ⎟ − 1⎟ is 90. If a and b are natural numbers and
⎝ ⎝ x⎠ ⎠
(A) 0 < x < 1 (B)  0 < x ≤ 1
f (x) = sin ( )
a 2 − 3 x + cos ( )
b2 + 7 x
(C) x ≥ 1 (D)  x>1 is periodic with finite fundamental period, then period
85. Let f be a real valued function with domain R satisfy- of f (x) is:
1 (A) p (B)  2p
ing 0 ≤ f (x) ≤ and for some fixed a > 0
2
1
(C) 2p ( a2 − 3 + b2 + 7 )
(x + a) = − f ( x ) − ( f ( x )) 2 ∀ x ∈ R,
( )
f 
2 (D)  p a2 − 3 + b2 + 7
then the period of the function f (x) is
(A) a (B)  2a sin101 x
(C)  non-periodic (D)  None of these 91. The function f (x) = , where [x] denotes the
⎡x⎤ 1
+
⎢⎣ p ⎥⎦ 2
86. If f (x) is defined on (0, 1), then the domain of defini-
tion of f (ex) + f (log |x|) is integral part of x, is
(A) (– e, – 1) (B)  (– e, – 1) ∪ (1, e) (A)  an odd function
(C) (– ∞, – 1) (D)  (– e, e) (B)  an even function
(C)  neither odd nor even
1 1 (D)  both odd and even functon
87. The function f (x) = sin–1(x – x2) + 1 − + 2
| x | [ x − 1]
92. If the graph of y = ax3 + bx2 + cx + d is symmetric
is defined in the interval (where [ ⋅ ] is the greatest
about the line x = k, then the value of a + k is
integer):
⎛ c
1+ 5⎞ (A) − (B)  c
(A)  x ∈ ⎜ 2 , 2b
⎝ 2 ⎟⎠
(C) c – bd (D)  None of these
⎛ 1+ 5⎞
(B)  x ∈ ⎜1, 93. The domain of definition of the function f (x) =
⎝ 2 ⎟⎠
ln {x} + x − 2{x} , where { } denotes the fractional
⎡1 − 5 1 + 5 ⎤ part, is
(C)  x ∈ ⎢ , ⎥ (A) {0} ∪ [1, ∞) (B) (1, ∞)
⎣ 2 2 ⎦
(C) (1, ∞) – I+ (D)  None of these
⎛ 1+ 5⎞
(D)  x ∈ ⎜ − 2 , 94. The domain of the function f (x) = ln (1 – 2 |cos x|)
⎝ 2 ⎟⎠ + ecos–1(2x/p ) is

Objective_Maths_JEE Main 2017_Ch 2.indd 30 01/01/2008 03:19:23


Functions  2.31

(A) (2np, (2n + 1)p), n ∈ I


⎛ p −p⎞ ⎛p p⎞
(A)  ⎜ − , ∪ , (B) [2np, (2n + 1)p], n ∈ I
⎝ 2 3 ⎟⎠ ⎜⎝ 3 2 ⎟⎠
(C) R
⎡ p − p ⎞ ⎡p p ⎤ (D)  None of these
(B)  ⎢ − , ⎟ ∪⎢ , ⎥
⎣ 2 3 ⎠ ⎣3 2⎦ 103. The domain of the function
⎡ p p⎞ ⎛p p⎤ ⎛ p ⎞
(C)  ⎢ − , − ⎟ ∪ ⎜ , ⎥ f (x) = [sin x] cos ⎜ is
⎣ 2 3⎠ ⎝ 3 2⎦ ⎝ [ x − 1] ⎟⎠
(D)  None of these (A) (1, 2) (B)  R – [1, 2)
(C) R – (1, 2) (D)  None of these
95. The domain of the function
x − [ x]
{ }
f (x) = ln sgn(9 − x 2 ) + [ x ]3 − 4[ x ], where [⋅] 104. The range of the function y =
1 − [ x] + x
is
denotes integral part, is
(A)  (– 2, 1) ∪ (2, 3) (B)  [– 2, 1) ∪ [2, 3) ⎛ 1⎞ ⎡ 1⎤
(A) ⎜ 0, ⎟ (B) 
⎢0, 2 ⎥
(C)  [– 2, 1] ∪ [2, 3) (D)  [– 2, 1) ∪ [2, 3] ⎝ 2⎠ ⎣ ⎦
96. The domain of the function ⎡ 1⎞ ⎛ 1⎤
(C) ⎢0, ⎟ (D)  ⎜⎝ 0, ⎥
f (x) =
(log0.2 x ) + (log0.2 x 3 ) (log0.2 0.0016 x ) + 36 is
3
⎣ 2 ⎠ 2⎦
105. If the domain for y = f (x) is [–3, 2], then the domain
(A)  (0 , 125) (B)  [0, 125] of g(x) = f {|[x]|} is
(C)  (0, 125] (D)  None of these (A)  (– 2, 3) (B)  [– 2, 3]
97. The range of the function y = [x2] – [x]2, x ∈ [0, 2] (C)  [– 2, 3) (D)  (– 2, 3]
where [⋅] denotes the integral part, is 106. If {x} and [x] represent fractional and integral part of
(A) {0} (B) {0, 1} 2000
{x + r}
(C)  {1, 2} (D)  {0, 1, 2} x, then the value of [x] + ∑ is
r = 1 2000
98. The range of the function
(A) x (B)  2000 x
⎡ 1⎤ ⎡ 1⎤
y = sin −1 ⎢ x 2 + ⎥ + cos −1 ⎢ x 2 − ⎥ , where [·] (C)  0 (D)  None of these
⎣ 2 ⎦ ⎣ 2⎦
n
denotes the integral part, is 107. If f : (0, p) → R be defined by f (x) = ∑ ([1 + sin k x ]),
(A) (0, p) (B)  [0, p] k =1
(C) {p} (D)  {0, p} where [x] denotes the integral part of x, then the range
ax 2 + 2 x + 1 of f (x) is
99. Let f (x) = . If f : R → [– 1, 2] is onto,
2x2 − 2x + 1 (A) {n – 1, n + 1} (B)  {n – 1, n, n + 1}
then the values of a are (C) {n, n + 1} (D)  None of these
(A) (– ∞, 2) (B)  [2, ∞) 108. Consider a function f (n) defined for all n ∈ N. The
(C) (– ∞, – 7] ∪ [– 2, ∞) (D)  None of these function satisfies the following two conditions
100. Let f : N → N, where f (x) = x + (– 1)x – 1. Then,  (i)  f (1) + f (2) + f (3) + … to ∞ = 1
(A)  f –1(x) = x + (– 1)x –1 (B)  f –1(x) = x (ii) f (n) = {(1 – p)p–1} { f (n + 1) + f (n + 2) + … to ∞}
(C)  f (x) = x – (– 1)
–1 x–1
(D)  None of these where 0 < p < 1. Then, f (2) is equal to
(A) p (1 – p) (B)  1 – p
a x2 + 6x − 8
101. Let f : R → R, where f (x) = . The values (C) 1 + p (D)  None of these
a + 6 x − 8x 2
of a, for which f to be onto, are ⎛ x x
109. If f (x) = lim ⎜ +
(A)  (2, 14) (B)  [2, 14) n→∞ ⎝ x + 1 ( x + 1)( 2 x + 1)
(C)  (2, 14] (D)  [2, 14]
x ⎞
+ +  to n terms⎟ then range of f (x) is
102. Domain of definition of the function ( 2 x + 1)(3 x + 1) ⎠
⎛ 2| x| ⎞ (A)  {0, 1} (B)  {– 1, 0}
f (x) = sin x + sin −1 ⎜ is
⎝ 1 + x 2 ⎟⎠ (C)  {– 1, 1} (D)  [0, 1]

Objective_Maths_JEE Main 2017_Ch 2.indd 31 01/01/2008 03:19:26


2.32  Chapter 2

110. If p and q are positive integers, f is a function defined (A) (4n – 1) + 4n x (B) (x + 1)4–n – 1
for positive numbers and attains only positive values (C) (x + 1)4n –1 (D)  (4–n – 1)x + 4n
such that f (x f (y)) = xpyq, then
114. Let f (x + p) = 1 + [2 – 3 f (x) + 3( f (x))2 – ( f (x))3]1/3,
(A) p = q (B)  p = q2 ∀ x ∈ R, where p > 0. Then, f (x) is periodic with
(C) p = q (D) 
2
p| = |q|
|  period.
111. If the function f satisfies the relation f (x + y) + (A) p (B)  2p
f (x – y) = 2 f (x) f (y) ∀ x, y ∈ R and f (0) ≠ 0, then f (x) (C) 4p (D)  None of these
is
n
(A)  an even function
(B)  an odd function
115. If ∑ f ( x + ka)
k =0
= 0, where a > 0, then the period of

(C)  odd if f (x) > 0 f (x) is


(D)  neither even nor odd
(n + 1)a
(A) a (B) 
112. Let f : R – {2} → R be a function satisfying 2f (x) +
a
(C)  (D)  None of these
⎛ 2 x + 29 ⎞ n +1
3f ⎜ = 100x + 80 ∀ x ∈ R – {2}, then f (x) =
⎝ x − 2 ⎟⎠
116. If y = log3x and S = (3, 27), the set onto which the set
60( 2 x + 29) S is mapped is
(A)  16 – 40x –
x−2 (A)  (0, 3) (B)  (1, 4)
3( 2 x + 29) (C)  (1, 3) (D)  (0, 2)
(B) 100x + 80 –
x−2
117. The values of x for which the functions f (x) = x – 3
30( 2 x + 29)
(C)  40 – 16x + and f (x) = 4 – x satisfy the inequality |  f (x) + f (x)| <
x−2 |  f (x)| + |f (x)| are
(D)  None of these
(A) [3, 4] (B) (– ∞, ∞)
113. Let g: R → R be given by g(x) = 3 + 4x. If gn(x) = gogo (C) (– ∞, ∞) – [3, 4] (D)  None of these
… og(x), then g–n(x) (where g–n(x) denotes inverse of
gn(x)) is equal to

More Than One Option Correct Type

118. If f is an even function defined on the interval [– 5, 5], 120. Let f (x) be defined for all x > 0 and be continuous. Let
then the real values of x satisfying the equation
⎛ x⎞
f (x) satisfy f ⎜ ⎟ = f (x) – f (y) for all x, y and f (e) = 1.
⎛ x + 1⎞ ⎝ y⎠
f (x) = f ⎜ are Then
⎝ x + 2 ⎟⎠
(A) f (x) is bounded
−1 ± 5 −3 ± 5
(A)  (B)  ⎛ 1⎞
2 2 (B) f ⎜ ⎟ → 0 as x → 0
⎝ x⎠
−2 ± 5 (C) x f (x) → 0 as x → 0
(C)  (D)  None of these
2 (D) f (x) = log x
119. The distinct linear function which maps [– 1, 1] onto | x |3 + | x |
121. Let f : R → R be a function defined by f (x) = – ,
[0, 2] is 1 + x2
(A) x – 1 (B)  x + 1 then the graph of f (x) lies in which quadrant
(C) – x + 1 (D)  – x ­– 1 (A) I (B) II
(C) III (D) IV

Objective_Maths_JEE Main 2017_Ch 2.indd 32 01/01/2008 03:19:28


Functions  2.33

⎛ x − x2 ⎞ (A)  domain of f is [1, 2)


122. If f (x1) – f (x2) = f⎜ 1 for x1, x2 ∈ [–1, 1], then (B)  domain of f is [1, 3)
f (x) is ⎝ 1 − x1 x2 ⎟⎠
⎧ p⎫
⎛1 − x⎞ ⎛1 − x⎞
−1
(C)  range of f is ⎨log ⎬
(A) log ⎜ (B) 
tan ⎜ ⎩ 2⎭
⎝ 1 + x ⎟⎠ ⎝ 1 + x ⎟⎠ (D)  range of f is {0}
⎛1 + x⎞ ⎛1 + x⎞
(C) log ⎜ tan −1 ⎜
(D)  128. If the function f: [1, ∞) → [1, ∞) is defined by f (x) =
⎝ 1 − x ⎟⎠ ⎝ 1 − x ⎟⎠ 2x(x – 1), then
123. Which of the following functions have period 2? (A) f is one-one (B)  f is onto
⎛p ⎞ 1 + 1 + 4 log 2 x
(A) {x} + cos p x (B)  tan ⎜ [ x ]⎟ (C) f –1(x) =
⎝2 ⎠ 2
(C) sin x + {x} (D)  sin (cos x)
1 − 1 + 4 log 2 x
124. The values of x for which the domain of definition of (D) f –1(x) =
2
1
the function, f (x) = , where [.] 9x
[| x − 1|] + | 7 − x | − 6 129. Let f (x) = x . Then,
9 +3
denotes the greatest integer part, is not defined are
(A) f (x) + f (1 – x) = 1
(A)  (0, 1] (B)  [7, 8)
(B) f (x) + f (1 – x) = –1
(C)  {2, 3, 4, 5, 6} (D)  [0, 1] ∪ [7, 8]
⎛ 1 ⎞ ⎛ 2 ⎞
x(sin x + tan x ) (C) f ⎜ +f⎜
125. If f (x) = , where [ ] denotes greatest ⎝ 1996 ⎟⎠ ⎝ 1996 ⎟⎠
⎡x + p⎤ 1
⎢⎣ p ⎥⎦ 2 − ⎛ 3 ⎞ ⎛ 1995 ⎞
+f ⎜ ⎟ +… + f ⎜ = 998
⎝ 1996 ⎠ ⎝ 1996 ⎟⎠
integer function, then
(A) f (x) is an odd function if x = np ⎛ 1 ⎞ ⎛ 2 ⎞
(D) f ⎜ +f⎜
(B) f (x) is an even function if x ≠ np ⎝ 1996 ⎟⎠ ⎝ 1996 ⎟⎠
(C) f (x) is an odd function if x ≠ np
⎛ 3 ⎞ ⎛ 1995 ⎞ 1
(D) f (x) is an even function if x = np +f ⎜ ⎟ +… + f ⎜ ⎟ = 997
⎝ 1996 ⎠ ⎝ 1996 ⎠ 2
ex − e− x
1 26. If f : R → R be defined by f (x) = , then 130. Let n be a positive integer with f (n) = 1! + 2! + 3! +
2
(A) f  is one-one … + n! and P(x) and Q(x) be polynomials in x such
(B)  f  is onto that f (n + 2) = P (n) f (n + 1) + Q(n) f (n) for all n ≥ 1,
(C)  f –1(x) = log( x − x 2 + 1) then
(A) P(x) = x + 3
(D) f –1(x) = log( x + x 2 + 1) (B) Q(x) = –x – 2
127. Let f (x) = sin–1(log[x]) + log(sin–1[x]), where [ ] (C) P(x) = –x – 2
denotes the greatest integer function. Then, (D) Q(x) = x + 3

Passage Based Questions


Passage 1 (A) 1 ≤ n < 2 (B)  1 < n < 2
A function f (x) is called periodic if there exists a positive (C) 1 ≤ n ≤ 2 (D)  None of these
real number T independent of x such that f (x + T) = f (x) 132. The period of the function f (x) = cos(sin x) + cos
for all x in the domain of x. The smallest such value of T is (cos x) is
called the fundamental period of f. p p
If f (x) is periodic having period T, then k f (ax + b) is (A)  (B) 
2 4
also periodic having period T/|a|
p
131. If the period of the function f (x) = sin ( )
[n] x , where
(C) 
6
(D)  None of these

[n] denotes the greatest integer less than or equal to n,


is 2p, then

Objective_Maths_JEE Main 2017_Ch 2.indd 33 01/01/2008 03:19:31


2.34  Chapter 2

Passage 2 If the domain consists of finite number of points, then


For composite functions, if T1, T2, … be the fundamental the range consists of set of corresponding f (x) values.
periods of the various functions involved, then the period If the domain consists of whole real line or real
of the composite function is the L.C.M. of (T1, T2, …). But line minus some finite points, then express x in terms of
in the case of functions where modulus is involved, the y as x = g(y). The values of y for which g is defined is the
L.C.M. rule gives the period of the function but it may not required range.
be the fundamental period. If the domain is a finite interval, find the intervals in
For example, according to the L.C.M. rule, which f (x) increases/decreases and then find the extreme
Period of |sin x| + |cos x| = L.C.M. of (p, p) = p, but it values of the function in those intervals. The union of those
is not the fundamental period since intervals is the required range.

⎛ p⎞ ⎛ p⎞ 1 37. If ex + ef (x) = e, then range of the function f  is


sin ⎜ x + ⎟ + cos ⎜ x + ⎟ = |cos x| + |sin x| (A) (–∞, 1] (B)  (–∞, 1)
⎝ 2⎠ ⎝ 2⎠
p (C) (1, ∞) (D)  [1, ∞)
which shows that the fundamental period is .
Thus, the period of |sin px| + |cos qx| 2 sin(p [ x 2 + 1])
138. The range of the function f (x) = ,
x4 + 1
⎛p p⎞ where [⋅] denotes the greatest integer function, is
= L.C.M. of ⎜ , ⎟ if p ≠ q
⎝ p q⎠ (A)  [0, 1] (B)  [–1, 1]
1 ⎛p p⎞ (C)  {0} (D)  None of these
= L.C.M. of ⎜ , ⎟ if p = q
2 ⎝ p q⎠ 139. Range of values of f (x) = 1 + sinx + sin3x + sin5x, x ∈
⎛ p p⎞
133. The function f (x) = k |cos x| + k2 |sin x| + f (k) has ⎜⎝ − , ⎟⎠ is
p 2 2
period if k is equal to
2 (A)  (0, 1)
(A) 1 (B) 2 (B) (– ∞, ∞)
(C)  3 (D)  None of these (C)  (– 2, 2)
(D)  None of these
134. The period of the function f (x) = 3x + 3 – [3x + 3] +
px Passage 4
sin ,
2 The domain of a function y = f (x) is the set of input values
where [x] denotes the greatest integer ≤ x, is x for which the operation f is being defined (real).
(A) 4 (B) 1 For finding the domain of a function, the denomina-
(C)  2 (D)  None of these tor should not be equal to 0. Also, expression under the
p is the period of the function
135. even root should be greater than or equal to 0.
(A) |sin x| + |cos x|
If domain of the functions, y = f (x) and y = g(x) are
(B) sin4x + cos4x
D1 and D2, respectively, then the domain of f (x) ± g(x) or
(C)  sin (sin x) + sin (cos x) f ( x)
f (x) ⋅ g(x) is D1 ∩ D2, whereas the domain of is D ∩
1 + 2 cos x g( x) 1
(D)  D2 – {g(x) = 0}.
sin x( 2 + sec x ) loga x is defined if x, a > 0 and a ≠ 1.
Sin–1x and cos–1x are defined for – 1 ≤ x ≤ 1
136. The period of the function f (x) = sin 5x + cos 3 x is
(A)  3 p (B)  p 140. If [x] denotes the integral part of x, then the domain of
(C)  non-periodic (D)  None of these the function f (x) = sin–1[2x2 – 3] + log2[log1/2(x2 – 5x
+ 5)] is
Passage 3
⎛ 5 ⎤ ⎡ 5⎞
The range of a function y = f (x) is the set of all possible (A) ⎜ − , − 1⎥ (B) 
⎢1,
output values f (x) corresponding to every input x in the ⎝ 2 ⎥⎦ ⎢⎣ 2 ⎟⎠
domain of f and is denoted as f (A) if A is the domain. For
⎛ 5 ⎤ ⎡ 5⎞
finding the range of a function y = f (x), first of all, find the (C) ⎜ − , − 1⎥ ∪ ⎢1, (D)  None of these
domain of f. ⎝ 2 ⎥⎦ ⎢⎣ 2 ⎟⎠

Objective_Maths_JEE Main 2017_Ch 2.indd 34 01/01/2008 03:19:33


Functions  2.35

Passage 5 1 41. If f (x) satisfies x + |  f (x)| = 2f (x), then f –1(x) satisfies


Consider a one-one onto function y = f (x) having domain A (A) 3x + |  f –1(x)| = 2f –1(x)
and range B, i.e. f : A → B. Then, there exists a function g (B) x + f –1(x) = 2f –1(x)
such that x = g(y) which therefore has domain B and range (C) f –1(x) – |x| = 2x
A. i.e. g : B → A such that f (x) = y ⇔ g(y) = x, ∀ x ∈ A and
(D) 3x – |  f –1(x)| = 2f –1(x)
y ∈ B. g is said to be the inverse of f.

Match the Column Type

142.   II. Range of the function (B) (–∞, ∞)


f (x) = loge (3x2 – 4x + 5)
Column-I Column-II
⎡ 11 ⎞
   I. Domain of the function R–{ n,
(A)  III. The value of the function (C)  ⎢ , ∞⎟
f (x) =
1 n ≥ 0, n ∈ I} x 2 − 3x + 2 ⎣ 3 ⎠
f (x) = 2 lies in
|sin x | + sin x x + x−6
the interval
  II. Domain of the function (B) (2np,
  IV. The range of the (D)  [–1, 1]
f (x) = (2n + 1)p)
function f (x) =
⎡ ⎛ 1 ⎞ ⎤ ⎡ ⎛ 4 − x2 ⎞ ⎤
log3 ⎢ − log1/ 2 ⎜1 + 1/ 5 ⎟ − 1⎥
⎣ ⎝ x ⎠ ⎦ sin ⎢log ⎜ ⎟⎥
⎢ ⎝ 1− x ⎠⎥
⎣ ⎦
III. Domain of the function (C)  (0, 1)
f (x) = log3 ⎡⎣ − (log3 x ) 2
144. The range of the function
+ 5 log3 x − 6 ]
Column-I Column-II
  IV. Domain of the function (D)  (9, 27)
⎛ ⎞    I. y = log (A)  [0, 1)
x 5
f (x) = cot −1 ⎜ ⎟, { 2 (sin x – cos x) + 3}
⎝ x − [x ] ⎠
2 2

x∈R   II. y = log2 {2 – log 5


(B) (–∞, 1]
(16 sin2x + 1)}
143. e x − e −| x |
III. y = (C)  [0, 2]
Column-I Column-II e x + e| x |
⎡ 11 ⎞ ex − e− x ⎡ 1⎞
   I. Range of the function (A)  ⎢log e , ∞⎟   IV. y = x ,x≥0 (D)  ⎢0, ⎟
⎣ 3 ⎠ e + e− x ⎣ 2⎠
f (x) = 3 x 2 − 4 x + 5

Assertion-Reason Type
Instructions: In the following questions an Assertion (A) 145. Assertion: If 3 f (x) – f (1/x) = ln x4, x > 0, then the
is given followed by a Reason (R). Mark your responses area of the region bounded by f (ex), x-axis, the lines
from the following options. x = 1 and x = 1 is 0.
(A)  Assertion(A) is True and Reason(R) is Reason: f (x) = ln x, x > 0.
True; Reason(R) is a correct explanation for ax
Assertion(A) 146. Assertion: If f (x) = x ( a > 0) , then
(B)  Assertion(A) is True, Reason(R) is True; 2 n −1
a + a
⎛ r ⎞
Reason(R) is not a correct explanation for ∑ 2 f ⎜ ⎟ = 2n – 1
r =1 ⎝ 2n ⎠
Assertion(A)
(C) Assertion(A) is True, Reason(R) is False Reason: f (x) + f (1 – x) = 1 ∀ x
(D) Assertion(A) is False, Reason(R) is True

Objective_Maths_JEE Main 2017_Ch 2.indd 35 01/01/2008 03:19:36


2.36  Chapter 2

147. Assertion: The range of the function f (x) = g(x)  + 3− x


where g(x) = and h(x) =
h(x) where g(x) = − x + 4 x − 3 and h(x) =
2
( x − 1) ( x − 2) ( x − 3)
⎡ 3x − 2 ⎤
⎛p ⎛ p ⎞⎞ sin −1 ⎢ ⎥ is [0, 2) – {1}
sin ⎜ ⎜ sin ( x − 1)⎟ ⎟ is [0, 2] ⎣ 2 ⎦
⎝2 ⎝ 2 ⎠⎠
Reason: Domain of h(x) is [0, 2)
Reason: Maximum and minimum values of both g
149. Assertion: Suppose, f (x) = (x + 1)2 for x ≥ –1. If g(x)
and h are attained at 2 and 1, respectively.
is the function whose graph is the reflection of the
148. Assertion: If [x] denotes the integral part of x, then graph of f (x) with respect to the line y = x, then g(x) =
domain of the function f (x) = g(x) + h(x), x – 1, x ≥ 0.
Reason: g(x) is the inverse of f (x)

Previous Year’s Questions

150. The period of sin2θ is : [2002]


1 55. The range of the function f ( x ) = 7 − x Px − 3 is [2004]
(A)  p 2 (B)  p
(A)  {1, 2, 3} (B)  {1, 2, 3, 4, 5}
(C) 2p (D)  p/2
(C)  {1, 2, 3, 4} (D)  {1, 2, 3, 4, 5, 6}
151. The domain of sin−1 [log3 (x/3)] is : [2002]
156. If f: R → S, defined by f (x) = sinx − 3 cosx + 1, is
(A) [1, 9] (B) [−1, 9] onto, then the interval of S is [2004]
(C) [−9, 1] (D)  [− 9, −1]
(A) [0, 3] (B) [−1, 1]
152. The period of the function f  (x) = sin4 x + cos4 x is : (C) [0, 1] (D) [−1, 3]
 [2002]
p 157. The graph of the function y = f (x) is symmetrical
(A) p (B)  about the line x = 2, then [2004]
2
(C) 2p (D)  None of these (A) f (x + 2) = f (x − 2)
(B) f (2 + x) = f (2 − x)
153. A function f from the set of natural numbers to inte- (C) f (x) = f (−x)
gers defined by (D) f (x) = − f (−x)
⎧n −1 sin −1 ( x − 3)
158. The domain of the function f ( x ) = is
⎪ 2 , when is odd 9 − x2

f ( n) = ⎨ is  [2003]  [2004]
⎪ n (A)  [2, 3] (B)  [2, 3)
⎪ − , when n is even (C)  [1, 2] (D)  [1, 2)
⎩ 2
159. Let f : (−1, 1) → B, be a function defined by
(A)  one-one but not onto
2x
(B)  onto but not one-one f ( x ) = tan −1 , then f is both one-one and onto
(C)  one-one and onto both 1 − x2
(D)  neither one-one nor onto when B is the interval [2005]

154. Domain of definition of the function ⎛ p⎞ ⎡ p⎞


(A) ⎜ 0, ⎟ ⎢0, 2 ⎟⎠
(B) 
⎝ 2⎠ ⎣
3
f ( x) = + log10 ( x 3 − x ) , is [2003]
4−x 2 ⎡ p p⎤ ⎛ p p⎞
(C) ⎢ − , ⎥ (D)  ⎜ − , ⎟
(A)  (1, 2) ⎣ 2 2 ⎦ ⎝ 2 2⎠
(B) (−1, 0) ∪ (1, 2) 160. A real valued function f (x) satisfies the functional
(C)  (1, 2) ∪ (2, ∞) equation f (x−y) = f (x) f (y) −f (a − x) f (a + y) where
(D) (−1, 0) ∪ (1, 2) ∪ (2, ∞) a is a given constant and f (0) = 1, f (2a−x) is equal to
 [2005]

Objective_Maths_JEE Main 2017_Ch 2.indd 36 01/01/2008 03:19:38


Functions  2.37

(A) −f (x) (B)  f (x) 163. For real x, let f (x) = x3 + 5x + 1, then [2009]
(C) f (A) + f (a−x) (D) 
f (x) (A) f is one-one but not onto R
(B) f is onto R but not one-one
⎛ p p⎞ (C) f is one-one and onto R
161. The largest interval lying in ⎜ − , ⎟ for which the
⎝ 2 2⎠ (D) f is neither one-one nor onto R
⎛x ⎞ 1
function f ( x ) = 4 − x + cos −1 ⎜ − 1⎟ + log(cos x ) is
2

⎝2 ⎠ 164. The domain of the function f ( x) = is


| x | −x
defined, is [2007]  [2011]
⎛ p p⎞ (A) (0, ∞) (B)  (− ∞, 0)
(A) [0, π] (B) 
⎜⎝ − , ⎟⎠
2 2 (C) (− ∞,∞) − {0} (D)  (− ∞, ∞)
⎡ p p⎞ ⎡ p⎞
(C) ⎢ − , ⎟ (D)  ⎢0, ⎟ ⎛ 1⎞
⎣ 4 2 ⎠ ⎣ 2⎠ 165. If f ( x ) + 2 f ⎜ ⎟ = 3 x, x ≠ 0, and
⎝ x⎠
S = {x ∈ R : f ( x ) = f ( − x )}; then S:
162. Let f : N → Y be a function defined as f (x) = 4x + 3,
where Y = {y ∈ N: y = 4x + 3 for some x ∈ N}. Show  [2016]
that f is invertible and its inverse is [2008]
(A)  contains more than two elements.
3y + 4 y+3 (B)  is an empty set.
(A) g(y) = (B)  g (y) = 4+ (C)  contains exactly one element
3 4
y+3 y −3 (D)  contains exactly two elements
(C) g (y) = (D)  g (y) =
4 4

Answer keys

Single Option Correct Type


1. (B) 2. (A) 3. (B) 4. (B) 5. (B) 6. (D) 7. (C) 8. (B) 9.  (A, D) 10. (B)
11. (A) 12. (C) 13. (A) 14. (B) 15. (A) 16. (D) 17. (A) 18. (C) 19. (B) 20. (A)
21. (A) 22. (D) 23. (A) 24. (C) 25. (A) 26. (D) 27. (B) 28. (B) 29. (D) 30. (A)
31. (C) 32. (C) 33. (C) 34. (B) 35. (C) 36. (D) 37. (A) 38. (B) 39. (A) 40. (D)
41. (D) 42. (D) 43. (A) 44. (A) 45. (A) 46. (B) 47. (C) 48. (C) 49.  (A, B) 50. (B)
51. (B) 52. (D) 53. (A) 54. (A) 55. (B) 56. (D) 57. (A) 58. (A) 59. (A) 60. (B)
61. (B) 62.  (A, B, C) 63. (B) 64. (C) 65. (D) 66. (B) 67. (D) 68. (D) 69. (D)
70. (C) 71. (C) 72. (D) 73. (C) 74. (B) 75. (D) 76. (C) 77. (C) 78. (C) 79. (B)
80. (B) 81. (A) 82. (B) 83. (B) 84. (A) 85. (B) 86. (A) 87. (A) 88. (B) 89. (B)
90. (B) 91. (B) 92. (A) 93. (C) 94. (C) 95. (B) 96. (C) 97. (D) 98. (C) 99. (C)
100. (A) 101. (D) 102. (B) 103. (B) 104. (C) 105. (C) 106. (A) 107. (C) 108. (A) 109. (A)
110. (C) 111. (A) 112. (A) 113. (B) 114. (B) 115. (B) 116. (C) 117. (C)

More than One Option Correct Type


1 18.  (A) and (B) 119.  (B) and (C) 120.  (C) and (D) 121.  (C) and (D) 122.  (A), (B) and (C)
123.  (A) and (B) 124.  (A), (B) and (C) 125.  (C) and (D) 126.  (A), (B) and (D) 127.  (A) and (C)
128.  (A), (B) and (C) 129.  (A) and (D) 130.  (A) and (B)

Passage Based Questions


131. (A) 132. (A) 133. (A) 134. (A) 135. (D) 136. (C) 137. (B) 138. (C) 139. (B) 140. (D)
141. (D)

Objective_Maths_JEE Main 2017_Ch 2.indd 37 01/01/2008 03:19:39


2.38  Chapter 2

Match the Column Type


142. I. → (B); II. → (C); III. → (D); IV. → (A)
143. I. → (C); II. → (A); III. → (B); IV. → (D)
144. I. → (C); II. → (B); III. → (D); IV. → (A)

Assertion-Reason Type
145. (A) 146. (A) 147. (A) 148. (A) 149. (A)

Previous Year’s Questions


150. (B) 151. (A) 152. (B) 153. (C) 154. (D) 155. (A) 156. (D) 157. (B) 158. (B) 159. (D)
160. (A) 161. (D) 162. (D) 163. (C) 164. (B) 165. (D)

Hints and Solutions

Single Option Correct Type


2
1. We have, f (x) = x3 + x2 + 100x + 7 sin x ⎛ b⎞
⇒ a ⎜ x + ⎟ ≠ 0  [Q b2 – 4ac = 0]
⇒ f ′(x) = 3x2 + 2x + 100 + 7 cos x > 0, ∀ x ∈ R ⎝ 2a ⎠
\ f (x) is an increasing function b
⇒ x ≠ –
⇒ f (1) < f (2) < f (3) 2a
b
Let a = f (1), b = f (2) and c = f (3), then a < b < c(1) \ f (x) is defined for x ≠ – and x + 2 > 0
2a
Then given equation is
⎧ b⎫
1 2 3 \ Domain of f = (– 2, ∞)\ ⎨ − ⎬ .
+ + =0 ⎩ 2a ⎭
y−a y−b y−c The correct option is (A)
⇒ (y – b)(y – c) + 2(y – a)( y – c) + 3( y – a)( y – b) = 0 (2) 3. We have,
Let f(y) = ( y – b)( y – c) + 2( y – a)( y – c) + 3( y – a)( y – b), ex + ef (x) = e ⇒ ef (x) = e – ex ⇒ f (x) = log (e – ex).
Then, f (a) = (a – b)(a – c) > 0 For f (x) to be defined, e – ex > 0 ⇒ e1 > ex ⇒ x < 1
and f (b) = 2(b) = 2(b – a)(b – c) < 0 \ Domain of f = (–∞, 1)
and f (c) = 3(c – a)(c – b) > 0 Let y = log (e – ex) ⇒ ey = e – ex
So the equation (2), i.e. f ( y) = 0 has one real root between a ⇒ ex = e – ey
and b and other between b and c.
⇒ x = log (e – ey)
The correct option is (B)
For x to be real, e – e > 0 ⇒ e1 > ey ⇒ y < 1
y

2. f (x) = log (ax3 + (2a + b) x2 + (2b + c) x + 2c)


\ Range of f = (–∞, 1).
= log ((ax2 + bx + c) (x + 2))
The correct option is (B)
= log (ax2 + bx + c) + log (x + 2)
4. The function f (x) = x2 + 2, x ∈ (– ∞, ∞) is not injective as
Since a > 0 and D = 0, f (1) = f (–1) but 1 ≠ –1.
\ ax2 + bx + c ≥ 0 ∀ x ∈ R The function f (x) = (x – 4) (x – 5), x ∈ (–∞, ∞) is not one-one
\ log (ax2 + bx + c) is defined if ax2 + bx + c ≠ 0 as f (4) = f (5) but 4 ≠ 5.
⎛ b c⎞ 4 x 2 + 3x − 5
⇒ a ⎜ x 2 + x + ⎟ ≠ 0
The function, f (x) = , x ∈ (–∞, ∞) is also not
⎝ a a⎠ 4 + 3x − 5 x 2
injective as f (1) = f (–1) but 1 ≠ –1.
⎡⎛ b⎞
2
⎛ b 2 − 4 ac ⎞ ⎤
⇒ a ⎢⎜ x + ⎟ − ⎜
⎥ ≠0 For the function, f (x) = |x + 2|, x ∈ [– 2, ∞).
⎢⎣⎝ 2a ⎠ ⎝ 4 a 2 ⎟⎠ ⎥⎦
Let f (x) = f (y), x, y ∈ [– 2, ∞) ⇒ |x + 2| = |y + 2|

Objective_Maths_JEE Main 2017_Ch 2.indd 38 01/01/2008 03:19:41


Functions  2.39

⇒ x + 2 = y + 2
For y to be positive,
⇒ x = y. (x + 1) (2x + 1) x (2x – 1) > 0

So, f is an injection. ⎛ 1 ⎞ ⎛1 ⎞
⇒ x ∈ (– ∞, – 1) ∪ ⎜ − , 0⎟ ∪ ⎜ , ∞⎟ .
The correct option is (B) ⎝ 2 ⎠ ⎝2 ⎠
1 Thus, (a) and (d) are the correct answers.
5. y = {cos( 2 x + 2) + cos 2 −[1 + cos( 2 x + 2)]}
2 The correct option is (A) and (D)
1 10. The maximum value of f (x) = cos x + cos ( 2 x) is 2 which
or y = − (1 − cos 2) = –sin2 1 i.e., constant
2 occurs at x = 0. Also, there is no value of x for which this
p
\ graph is a line parallel to x-axis. Also, when x = , value will be attained again.
2
2⎛p ⎞ The correct option is (B)
y = − cos ⎜ + 1⎟ = – sin 1 and hence it passes through the
2
⎝2 ⎠ 11. Let the required function be f (x) = ax + b.
⎛p ⎞ If a > 0, then f (–1) = 0 and f (1) = 2.
point ⎜ , − sin 2 1⎟ .
⎝2 ⎠ ⇒ –a + b = 0 and a + b = 2
The correct option is (B) ⇒ a = 1 and b = 1.
6. Since f (x) = f (–x) If a < 0, then f (–1) = 2 and f (1) = 0
\ f is not one–one. ⇒ – a + b = 2 and a + b = 0
x2 − 8
Let y ∈ R. Then f (x) = y ⇒ y = 2 ⇒ a = –1 and b = 1.
x +2
8 + 2y Hence, f (x) = x + 1  or  f (x) = –x + 1.
⇒ x2 =
1− y The correct option is (A)
For x to be real, (8 + 2y) (1 – y) ≥ 0 12. For x ≤ – 1, 1 – x ≥ 2 and 1 – x ≥ 1 + x
and 1 – y ≠ 0 ⇒ (y + 4) (y – 1) ≤ 0 and y ≠ 1 \ max [(1 – x), 2, (1 + x)] = 1 – x
⇒ – 4 ≤ y < 1 For –1 < x < 1, 0 < 1 – x < 2 and 0 < 1 + x < 2.
\ Range of f = [– 4, 1) ⊂ R \ max [(1 – x), 2, (1 + x)] = 2.
\ f is not onto. For x ≥ 1, 1 + x ≥ 2, 1 + x > 1 – x
The correct option is (D) \ max [(1 – x), 2, (1 + x)] = 1 + x
7. We have, ⎧1 − x, x ≤ −1
1 1 ⎪
f (a) = 64 a3 + 3 = (4a)3 + 3 Hence, f (x) = ⎨2,
−1 < x < 1.
a a
⎪1 + x, x ≥ 1
⎛ 1⎞
3
1⎛ 1⎞ ⎩

= ⎜ 4a + ⎟ − 3 ⋅ 4a ⋅ ⎜ 4a + ⎟ The correct option is (C)
⎝ a⎠ a⎝ a⎠
13. We have,

= (3) – 12 ⋅ 3 = 27 – 36 = – 9.
3

f  (x) = sin [p 2] x + sin [– p 2] x


⎡ 1 ⎤
⎢Since a, b are roots of 4 x + x = 3⎥ = sin 9x + sin (– 10) x
⎢ ⎥ = sin 9x – sin 10x
⎢\ 4 a + 1 = 3 ⎥
⎢⎣ ⎥⎦ ⎛p⎞ 9p
a \ f  ⎜ ⎟ = sin – sin 5p = 1 – 0 = 1
⎝ 2⎠ 2
Similarly, f (b) = – 9 f (p) = sin 9p – sin 10p = 0
\ f (a) = f (b) = –9
⎛p⎞ 9p 10p 1
The correct option is (C) f ⎜ ⎟ = sin – sin = – 1.
⎝ 4⎠ 4 4 2
8. We have,
( )
The correct option is (A)
( fog) (x) = f [g (x)] = f x2 + 1 14. Since the given function has minimum value 75 which is

( ) attained at x = 2 and maximum value 89 which is attained at


2

= x +1
2
– 1 = x2 x = 3. Hence, the range of f is [75, 89].
⎧0 if x = 0⎫ The correct option is (B)
\ (hofog) (x) = h [( fog) (x)] = h (x2) = ⎨ 2
⎬.
⎩ x if x ≠ 0 ⎭

The correct option is (B) ⎛ 1⎞
15. We have, 2 f (x) – 3 f ⎜ ⎟ = x2, x ≠ 0.
⎝ x⎠
2x − 1 2x − 1
9. Let y = = Putting x = 2, we get
2 x 3 + 3x 2 + x x ( 2 x + 1)( x + 1)
⎛ 1⎞
( 2 x − 1) 2 2  f (2) – 3 f ⎜ ⎟ = 4 (1)

= ⎝ 2⎠
( 2 x + 1)( x + 1) x ( 2 x − 1)

Objective_Maths_JEE Main 2017_Ch 2.indd 39 01/01/2008 03:19:45


2.40  Chapter 2

1 x 2 + 34 x − 71
Now, putting x = , we get
21. Let y =
2 x2 + 2x − 7
⎛ 1⎞ 1 ⇒ (y – 1) x2 + (2y – 34) x – 7y + 71 = 0

2  f  ⎜ ⎟ – 3 f (2) = (2)
⎝ 2⎠ 4 For x to be real,
7 (2y – 34)2 ≥ 4 (y – 1) (71 – 7y)  (QDiscriminent ≥ 0)
Solving (1) and (2), we get f (2) = –
.
4 ⇒ y2 + 289 – 34y ≥ – 7y2 – 71 + 78y
The correct option is (A)
⇒ 8y2 – 112y + 360 ≥ 0
16. f (x) = (1 + b2)
⇒ y2 – 14y + 45 ≥ 0
⎧ 2 2b b2 ⎫ b2 ⇒ (y – 9) (y – 5) ≥ 0
⎨ +
x x+ 2 2⎬
− +1
⎩ 1+ b 2
(1 + b ) ⎭ 1 + b 2 ⇒ y ≤ 5 or y ≥ 9
2 \ y cannot lie between 5 and 9.
⎛ b ⎞ 1 1
= (1 + b ) ⎜ x +
2
+ ≥
1 + b 2 ⎟⎠
The correct option is (A)
⎝ 1 + b2 1 + b2
22. Since f (x) is an odd periodic function with period 2
1
\ m (b) =
. So, range of m (b) = (0, 1]. \ f (– x) = – f (x) and f (x + 2) = f (x)
1 + b2
\ f (2) = f (0 + 2) = f (0)

The correct option is (D)
and  f (– 2) = f (– 2 + 2) = f (0)

17. Let y = ( fog) (x) = f [g (x)] = f (x3 + 5)
Now,  f (0) = f (– 2) = – f (2) = – f (0)


= 2 (x3 + 5) – 3 = 2x3 + 7.
1/ 3
⇒ 2 f (0) = 0 i.e. f (0) = 0

y−7 ⎛ y − 7⎞
\ x3 =
⇒x= ⎜ \ f (4) = f (2 + 2) = f (2) = f (0) = 0

2 ⎝ 2 ⎟⎠
Thus, f (4) = 0
1/ 3
⎛ y − 7⎞ The correct option is (D)
⇒ ( fog) (y) = ⎜
–1
⎝ 2 ⎟⎠ 23. For the two components to be meaningful, we must have
1/ 3 x (x + 3) ≥ 0 and 0 ≤ x2 + 3x + 1 ≤ 1.
⎛ x − 7⎞
⇒ ( fog)–1 (x) = ⎜
Hence, (x + 3) x = 0 i.e., x = 0, – 3.
⎝ 2 ⎟⎠
\ S = {– 3, 0}
The correct option is (A)
The correct option is (A)
18. Since f (x) = log (x – 1) – log (x – 2). p
Domain of f (x) is x > 2 or x ∈ (2, ∞)(1) 24. Since f (x) = g (x), ∀ x ∈ (4n + 1) , n ∈ Z
2
⎛ x − 1⎞ x −1 \ f (x) = g (x) for infinitely many values of x.
g (x) = log ⎜
⎟ is defined if >0
⎝ x − 2⎠ x−2 The correct option is (C)
⇒ x ∈ (– ∞, 1) ∪ (2, ∞)(2) 25. Since, f (x + y) = f (x) f (y)
From Eq. (1) and (2), x ∈ (2, ∞). ⇒ f (x) = ax
The correct option is (C) Also, given that f (1) = 3
19. We have, ⇒ a1 = 3,
⎧1 + n − n = 1, x = n ∈ Z \ a = 3
g (x) = ⎨

⎩1 + n + k − n = 1 + k , x = n + k Hence, f (x) = 3x
n
where n ∈ Z, 0 < k < 1.
Now, S = ∑ f ( r ) ⇒ S = 31 + 32 + … + 3n

⎧ − 1, g ( x) < 0 r =1

⎪ ⎛ 1 − 3n ⎞
Now,  f [g (x)] = ⎨
0, g ( x) = 0
= 3⎜

⎩ 1, g ( x) > 0 ⎝ 1 − 3 ⎟⎠
3 n
Clearly, g (x) > 0 for all x. So, f [g (x)] = 1, for all x. \ S =
(3 − 1) .
2
The correct option is (B)
The correct option is (A)
20. Clearly, y is defined for all x ∈ R except
26. Taking f (x) = log x, we see that
when sin x = 0 i.e., x = np : n ∈ Z
⎛ x⎞
\ Domain of f = R \ {np : n ∈ Z} f  ⎜ ⎟ = f (x) – f ( y)

⎝ y⎠
The correct option is (A)

Objective_Maths_JEE Main 2017_Ch 2.indd 40 01/01/2008 03:19:48


Functions  2.41


Clearly f (x) is not bounded sin (p [ x 2 + 1])
⇒ f (x) =
=0
⎛ 1⎞ x4 + 1
and  f  ⎜ ⎟ = – log x → ∞ as x → 0.

⎝ x⎠ Hence, Range of f = Rf = {0}

Also, x f (x) = x log x → 0 as x → 0.

The correct option is (C)

The correct option is (D)
⎛ 1⎞ 5
27. We have, g (x) = 1 + x 32. af ( x ) + bf ⎜ ⎟ = x + (1)
⎝ x⎠ x
and  f [g (x)] = 3 + 2 x + x(1)
⎛ 1⎞ 1
af ⎜ ⎟ + bf ( x ) = + 5 x (2)
Also, f [g (x)] = f (1 + x )(2)
⎝ x⎠ x

From Eq. (1) and (2), we get
Multiply Eq. (1) by a and equation (2) by b, then subtract
f (1 +
x )=3+2 x + x. 5a b
(a2 – b2) f (x) = ax +
− – 5bx
Let 1 + x = y or x = ( y – 1)2.
x x
\ f ( y) = 3 + 2 ( y – 1) + ( y – 1)2
1 ⎛ 1 ⎞
\ f (x) = 2
⎜ x( a − 5b) + (5a − b)⎟⎠

= 3 + 2y – 2 + y2 – 2y + 1 = 2 + y 2 (a − b2 ) ⎝ x
\ f (x) = 2 + x2
The correct option is (C)

The correct option is (B) 33. f (x) = x3 + x2 + 100x + 5 sin x
sin x \ f ′(x) = 3x2 + 2x + 100 + 5 cos x = 3x2 + 2x + 94 + (6 +
28. We have, f (x) = 1 + 5 cos x) > 0
cos 2 x
\ f is an increasig function and consequently a one-one
cos 2 x (cos x ) + sin x ( 2 cos x sin x )
⇒ f ′ (x) = function.
cos 4 x
Clearly,  f (–∞) = –∞, f (∞) = ∞ and f (x) is continuous,
cos x (cos x + 2 sin 2 x )
2
1 + sin 2 x ­therefore range f = R = codomain f. Hence, f  is onto.
= 4
=
cos x cos3 x
⇒ f ′ (x) > 0 The correct option is (C)
\ f (x) is increasing function. 34. We have, f (x + 2a) = f ((x + a) + a)
1
⎛ sin x ⎞ = − f ( x + a) − ( f ( x + a)) 2
lim ⎜1 + 2 ⎟
= –∞ 2
−p ⎝ cos x⎠
x→ 2
2
1 1 ⎛1 ⎞
⎛ sin x ⎞ = −
− f ( x ) − ( f ( x )) 2 − ⎜ − f ( x ) − ( f ( x )) 2 ⎟
and lim ⎜1 +
⎟ =∞ 2 2 ⎝2 ⎠
x→ ⎝ cos 2 x ⎠
p
2
1 1
\ Range = (–∞, ∞)
= −
− f ( x ) + ( f ( x )) 2
2 4

The correct option is (B)
1 ⎛1 ⎞
3
29. We have,  f (x) = e x − 3 x + 2 = − ⎜ − f ( x )⎟ = f (x)

2 ⎝2 ⎠
Let g (x) = x3 – 3x + 2
Hence, f (x) is periodic with period 2a.
⇒ g′ (x) = 3x2 – 3 = 3 (x2 – 1)
The correct option is (B)
⇒ g′ (x) ≥ 0, for x ∈ (–∞, –1]
35. g[ f (x)] = (sin x + cos x)2 – 1
\ g (x) is increasing function
⇒ g[ f (x)] = sin 2x
\ f (x) is one-one. ⎡ p p⎤
We know that sin x is bijective only when x ∈ ⎢ − , ⎥ .
Now, Range of f (x) is (0, e4], but co-domain is (0, e5]. p p ⎣ 2 2⎦
Thus, f (x) is bijective if − ≤ 2x ≤
\ f (x) is into function. 2 2
p p
The correct option is (D) ⇒ − ≤ x ≤ .
4 4
30. For f  to be defined, we must have The correct option is (C)
⎛ 1 ⎞ 1 y y
log1/ 2 ⎜1 + 4 ⎟ < – 1 ⇒ 1 + 4 > (2– 1)– 1 = 2 which is 36. Let 2 x + = a and 2 x − = b,
⎝ x ⎠ x 8 8
1 a+b
­possible only if 4 > 1 i.e. 0 < x < 1. then x = and y = 4(a – b)
x 4
Hence, the domain of the given function is {x : 0 < x < 1}. ⎛ y y⎞
Given, f ⎜ 2 x + , 2 x − ⎟ = xy
The correct option is (A) ⎝ 8 8⎠
31. Since [x2 + 1] is an integer ⇒ f (a, b ) = a2 – b 2

\ sin (p[x2 + 1]) = 0 ⇒ f (m, n) + f (n, m) = m2 – n2 + n2 – m2 = 0 for all m, n.


The correct option is (D)

Objective_Maths_JEE Main 2017_Ch 2.indd 41 01/01/2008 03:19:52


2.42  Chapter 2

37. Since the domain of f is (0, 1), 1 1


\ 0 < ex < 1 and 0 < ln |x| < 1 1 − is defined when 1 − ≥0
|x| |x|
⇒ log 0 < x < log 1 and e0 < |x| < e1 ⇒ x ≤ – 1, x ≥ 1
(2)
⇒ – ∞ < x < 0 and 1 < |x| < e 1

and 2 is defined when x2 – 1 < 0, x2 – 1 ≥ 1
⇒ x ∈ (– ∞, 0) and x ∈ ((– ∞, – 1) ∪ (1, ∞)) ∩ (– e, e) [ x − 1]
⇒ x ∈ (– ∞, 0) and x ∈ (– e, – 1) ∪ (1, e) ⇒ x ∈ (– e, – 1) i.e., x ∈ (–∞, − 2 ) ∪ (– 1, 1) ∪ ( − 2 , ∞)(3)

The correct option is (A)
From Eq. (1), (2) and (3), we get
1 n ⎛
38. We have + < 1 if n ≤ 49 1+ 5⎞
2 100 x ∈ ⎜ 2,
⎝ 2 ⎟⎠
⎡1 n ⎤
\ ⎢ + ⎥ = 0 if n ≤ 49 The correct option is (A)
⎣ 2 100 ⎦
44. Graph is symmetric about x = k
1 n
Again + ≥ 1 if n ≥ 50 if f(k – x) = f (k + x)
2 100
⇒ a(k – x)3 + b(k – x)2 + c(k – x) + d
⎡1 n ⎤
\ ⎢ + ⎥ = 1 if 50 ≤ n ≤ 99 = a(k + x)3 + b(k + x) + c(k + x) + d
⎣ 2 100 ⎦
⇒ 2ax3 – (6ak2 + 4bk + 2c)x = 0
\ The desired sum = 50.

It is true for all x if a = 0 and 6ak2 + 4bk + 2c = 0

The correct option is (D) c c
i.e. a = 0 and k = – ⇒a+k=− .
log 0.3 | x − x | 2b 2b
39. We have, f (x) = The correct option is (A)
|x|
45. Domain of f and domain of composite function gof are same.

Now, by definition, the domain of f  is defined as
The correct option is (A)
log | x − 2 |
0.3 ≥ 0, |x| ≠ 0 46. Since,
|x|
f  (x) = x3 + x2f ′(1) + xf ′′(2) + f ′(3)(1)

Since denominator i.e. |x| is always positive
⇒ f ′(x) = 3x2 + 2xf ′(1) + f ′′(2)(2)
\ log0.3 |x – 2| ≥ 0 ⇒ log0.3 |x – 2| ≥ log0.3(1)

⇒ f ′′(x) = 6x + 2f ′(1)(3)
⇒ |x – 2| ≤ 1
⇒ f ′′′(x) = 6 i.e., a constant function
⇒ –1 ≤ x – 2 ≤ 1
Hence, f ′(3) = 6
\ 1 ≤ x ≤ 3 (1)
Using Eq. (3), we have
and |x – 2| > 0, which is always true, but x ≠ 2 (2)
f ′′(2) = 12 + 2f ′(1)(5)
Hence, domain of f = Df = [1, 2) ∪ (2, 3]
Substituting x = 1, in equation (2), we have
The correct option is (A)
f ′(1) = 3 + 2f ′(1) + f ′′(2)
40. Since, f (– x) = f (x)
⇒ f ′(1) = 3 + 2f ′(1) + 12 + 2f ′(1)  [using (5)]
⇒ f(x) is an even function, its graph will be symmetrical
⇒ 3f ′(1) = – 15
about y-axis.
⎛ | x |3 + | x | ⎞ \ f ′(1) = – 5 (6)
Also, f (x) = − ⎜ ⇒ f (x) = – (+ ive) = – ive
⎝ 1 + x 2 ⎟⎠ Substituting respective value in Eq. (5), we have
i.e., the graph of f (x) completely lies below the x-axis, and is
f ′′(2) = 2
also symmetric about y-axis (as discussed above). Hence, the polynomial f(x) can be written as
\ The graph of f (x) lies in III and IV quadrants.
f (x) = x3 – 5x2 + 2x + 6

The correct option is (D) Therefore, f (2) – f (1)
= (8 – 20 + 4 + 6) – (1 – 5 + 2 + 6)
41. Since, − x 2 is not defined for any real x,
= –2 – 4 = –6
\ domain of the given real fucntion is null set.
\ f (2) – f (1) = – 6 = – f (0).
The correct option is (D)
The correct option is (B)
42. g[ f (x)] ≤ f [g(x)] ⇒ g(|x|) ≤ f ([x]) ⇒ [|x|] ≤ |[x]|
This is true for each x ∈ R. ⎧ p⎫
47. f :  → A, where A = ⎨ y : 0 ≤ y < ⎬
The correct option is (D) ⎩ 2⎭
43. sin– 1 (x – x2) is defined when – 1 ≤ x – x2 ≤ 1 Also, the function f is defined as
f (x) = tan–1(x2 + x + k)
1− 5 1+ 5

≤x≤ (1)
2 2

Objective_Maths_JEE Main 2017_Ch 2.indd 42 01/01/2008 03:19:55


Functions  2.43

For the function f to be onto, the range must overlap


Now, inside the given domain, we will always have
completely on the codomain or range must be equal to
­ ⎡x⎤
codomain such that ⎢3⎥ = 1
⎣ ⎦
Rf = A
\ f (x) = x – 1 throughout its domain
⎡ p⎞
⇒ Rf = ⎢0, ⎟ i.e. 1st Quadrant which is possible if and ⇒ y = x – 1
⎣ 2⎠
only if ⇒ x = y + 1 = f–1 (y)
x2 + x + k ≥ 0 ⇒ D ≤ 0 \ f –1 (x) = x + 1
⇒ 12 – 4 ⋅ 1 ⋅ (k) ≤ 0 The correct option is (B)
1 52. For f (x) to be defined,
\ k ≥
4   (i) [2x2 – 3] = –1, 0, 1
The correct option is (C)
⇒  – 1 ≤ 2x2 – 3 < 2 ⇒ 2 ≤ 2x2 < 5
⎛ 1⎞ ⎛ 1⎞ 1 1 5
48. By verification, f ⎜ − ⎟ = f ⎜ ⎟ = –1=− ⇒ 1 ≤ x2 <
⎝ 2⎠ ⎝ 2⎠ 2 2 2
Hence, f (|x|) = x ⎧ 1 ≤ x 2 ⇒ x ≤ −1 or x ≥ 1

The correct option is (C)
⇒  ⎨ 2 5 5 5
49. Given that ⎪x < ⇒ − <x<
⎩ 2 2 2
f  (x) = [x]2 + [x + 1] – 3
5 5
⇒ f (x) = [x]2 + [x] + 1 – 3
⇒  − < x ≤ −1 or 1 ≤ x < (1)
2 2
⇒ f (x) = [x]2 + [x] – 2
 (ii)  x2 – 5x + 5 > 0
⇒ f (x) = ([x] + 2)([x] – 1)
5− 5 5+ 5
Now, f (x) = 0
⇒  x < or x > (2)
2 2
⇒ ([x] + 2)([x] – 1) = 0
(iii) log1/2 (x – 5x + 5) > 0
2

⇒ either [x] + 2 = 0 or [x] – 1 = 0 0


⎛ 1⎞
⇒ either [x] = – 2 or [x] = 1 ⇒  x2 – 5x + 5 < ⎜ ⎟
⎝ 2⎠
⇒ either – 2 ≤ x < – 1 or 1 ≤ x < 2 ⇒  x2 – 5x + 5 < 1
i.e., f (x) = 0 for infinite number of values of x. ⇒  x2 – 5x + 4 < 0
Also, f (x) will not be one-to-one ⇒ 1 < x < 4 (3)
5− 5
Since, f (x) = (| x | + 2)(| x | − 1)
From Eq. (1), (2) and (3), 1 ≤ x <
      2
Integer
  Integer
 The correct option is (D)
Integer

Hence,  f (x) is into on the set of real numbers (R) as 2 f ( n) + 2


53. We have, f (n + 1) = and f (1) = 2
co-domain. 2
The correct option is (A) and (B) \ f (101) = f (1) + 100 × 1/2 = 2 + 50

50. Given, q2 – 4pr = 0 and p > 0 \ f (101) = 52.

For f (x) to be defined,
The correct option is (A)
px3 + (p + q)x2 + (q + r)x + r > 0 cos 2 x + sin 4 x
54. f (x) =
⇒ px2(x + 1) + qx(x + 1) + r(x + 1) > 0 sin 2 x + cos 4 x
⇒ (x + 1)(px2 + qx + r) > 0 cos 2 x + sin 2 x(1 − cos 2 x )
⇒ f (x) =

q sin 2 x + cos 2 x (1 − sin 2 x )
⇒ x > – 1 and x ≠ −
2p sin 2 x + cos 2 x − sin 2 x cos 2 x
[Since q2 – 4pr = 0 ⇒ f (x) =

q q sin 2 x + cos 2 x − sin 2 x cos 2 x
\ at x = − , px2 + qx + r = 0 and at x ≠ − , px2 + qx
2p 2p ⇒ f (x) = 1 ⇒ f(2002) = 1
+ r > 0]
The correct option is (A)
⎡ ⎧ q ⎫⎤
\ Domain = R − ⎢( − ∞, −1] ∪ ⎨ − ⎬ ⎥ 55. We know that
⎣ ⎩ 2 p ⎭⎦
|x + y| = |x| + |y|

The correct option is (B) if x and y are of same sign i.e.,
⎡x⎤ either x and y are > 0 or x and y are < 0
51. Given that f (x) = x − ⎢ ⎥
⎣3⎦ Since, | f (x) + f (x)| = | f (x)| + |f (x)|
where f : (3, 6) → (1, 3)

Objective_Maths_JEE Main 2017_Ch 2.indd 43 01/01/2008 03:19:58


2.44  Chapter 2

i.e., |x + 1 + x – 2| = |x + 1| + |x – 2|
⎛ 1⎞
therefore, either x + 1 > 0 and x – 2 > 0 61. We have, 2f (x) + 3 f ⎜ ⎟ = x2 – 1 (1)
⎝ x⎠
⇒ x > – 1 and x > 2 1
Putting in place of x, we get
\ x > 2 x
or x + 1 < 0 and x – 2 < 0 ⎛ 1⎞ 1
3 f ( x ) + 2 f ⎜ ⎟ = 2 − 1 (2)
⇒ x < – 1 and x < 2 ⎝ x⎠ x
\ x < – 1 ( 2 x 2 + 3)(1 − x 2 )
Solving Eq. (1) and (2), we get f (x) = ,
Hence, for the given equality to be valid, we have 5x 2
which is a non-periodic even function.
x ∈ (– ∞, – 1) ∪ (2, ∞) The correct option is (B)
Since, option (B) is the subset of the solution set, Hence, 62. When x1 = –1 and x2 = 1,
option (B) is true. ⎛ −1 − 1 ⎞
then  f (– 1) – f (1) = f ⎜
= f (–1)
⎝ 1 + 1(1) ⎟⎠
The correct option is (B)
56. f : N → Z ⇒ f (1) = 0,

f  (1) = 0, f (2) = –1, f (3) = 1, f (4) = –2, ⎛1 − x⎞
which is satisfied when f (x) = tan − 1 ⎜

f  (5) = 2, and f (6) = –3 so on. ⎝ 1 + x ⎟⎠
When x1 = x2 = 0, then

⎛ 0 − 0⎞
f (0) – f (0) = f ⎜
= f (0) ⇒ f (0) = 0
1 0 ⎝ 1 − 0 ⎟⎠
2 –1
3 1
When x1 = –1 and x2 = 0, then

4 –2 ⎛ −1 − 0 ⎞
f (–1) – f (0) = f ⎜
= f (–1) ⇒ f (0) = 0,
5 2 ⎝ 1 − 0 ⎟⎠
6 3 ⎛1 − x⎞
which is satisfied when f (x) = log ⎜

⎝ 1 + x ⎟⎠
In this type of function every element of set A has unique ⎛1 + x⎞
and f (x) = log ⎜
.
image in set B and there is no element left in set B. ⎝ 1 − x ⎟⎠
Hence, f  is one-one and onto function. The correct option is (A), (B) and (C)
The correct option is (D) 63. Given: f (T + x) = 1 + [(1 – f (x))3]1/3
57. Since A.M. ≥ G.M. = 1 + (1 – f (x))
tan 2 a ⇒ f (T + x) + f (x) = 2 (1)
\
x2 + x + ≥ 2 tan 2 a
x +x
2 ⇒ f (2T + x) + f (T + x) = 2 (2)
(2) – (1) ⇒ f (2T + x) – f (x) = 0
⎛ p⎞
= 2 tan a   ⎜ tan a > 0 as 0 < a < ⎟
⇒ f (2T + x) = f (x)
⎝ 2⎠
Also, T is positive and least therefore period of f (x) = 2T.

The correct option is (A)
The correct option is (B)
58. 0 ≤ log (1 – x2) ≤ 1 ⇒ 1 ≤ 1 – x2 ≤ e
Now, 1 – x2 ≤ e ∀ x ∈ R − log 0.3 ( x − 1)
64. Since, f (x) =
But 1 – x2 ≥ 1 is possible only when x = 0. − x 2 +3 x + 18
The correct option is (A) log 0.3 ( x − 1)
⇒ f (x) =

59. f (x) is onto, x 2 − 3 x − 18
\ S = range of f (x)
By definition, f (x) is defined, if
⎛ p⎞
Now f (x) = sin x − 3 cos x + 1 = 2 sin ⎜ x − ⎟ + 1 log ( x − 1)
⎝ 3⎠ 2 0.3 ≥0
x − 3 x − 18
⎛ p⎞ ⎛ p⎞
Q –1 ≤ sin ⎜ x − ⎟ ≤ 1, – 1 ≤ 2 sin ⎜ x − ⎟ + 1 ≤ 3
\ either log0.3 (x – 1) ≥ 0 and x2 – 3x – 18 > 0

⎝ 3⎠ ⎝ 3⎠
⇒ 1 < x ≤ 2  and  x < –3  or  x > 6

\ f (x) ∈ [– 1, 3] = S
No solution
The correct option is (A) or log0.3 (x – 1) ≤ 0 and x2 – 3x – 18 < 0

60. Since the function is periodic, a2 – 3 and b2 + 7 should be ⇒ x ≥ 2 and – 3 < x < 6 ⇒ x ∈ [2, 6)

perfect squares, which is possible only if a = 2, b = 3 in
Hence, domain of f (x) = [2, 6).

which case f (x) = sin x + cos 4x, whose period is 2p.

The correct option is (C)
The correct option is (B)

Objective_Maths_JEE Main 2017_Ch 2.indd 44 01/01/2008 03:20:02


Functions  2.45

65. Just interchange x and y and in first case y = 0, now x ≥ 0 71. There is no information about co-domain therefore f (x) is
x = (y + 1)2 x ≥ 0, y ≥ – 1 not necessarily onto.
or y = x − 1 x ≥ 0 The correct option is (C)
The correct option is (D) 72. For every rational number T, we have,
⎧1, when x is a rational⎫
sin101 x f (T + x) = ⎨
⎬,
66. We have, f (x) = ⎩0, when x is irrational ⎭
⎡x⎤ 1
⎢p ⎥ + 2 but there is no least positive value of T for which f (T + x) =
⎣ ⎦
f (x) because there are infinite number of rational numbers
sin101 ( − x ) between any two rational numbers. Therefore, f (x) is a peri-
⇒ f (–x) =

⎡ x⎤ 1 odic function having no fundamental period.
⎢− p ⎥ + 2
⎣ ⎦ The correct option is (D)
Case I: when x = np (n ∈ Z) 73. In the sum, f1(1) + f1(2) + f1(3) + … + f1(n), 1 occurs n times,
sin101 ( − np ) 1 1
f (– x) =
=0 occurs (n – 1) times, occurs (n – 2) times and so on.
⎡ − np ⎤ 1 2 3
⎢ p ⎥+ 2 \ f1(1) + f1(2) + f1(3) + ... + f1(n)
⎣ ⎦
1 1 1
Case II: When x ≠ np, n ∈ Z = n ⋅ 1 + (n – 1) ⋅
+ (n – 2) ⋅ + … + 1 ⋅
2 3 n
− sin101 ( x ) ⎛ 1 1 1⎞ ⎛ 1 2 3 n −1⎞
(–x) =
f  = n ⎜1 + + + ... + ⎟ − ⎜ + + + ... +

⎡ x⎤ 1 ⎝ 2 3 n⎠ ⎝ 2 3 4 n ⎠
⎢− p ⎥ + 2
⎣ ⎦ ⎡⎛ 1 ⎞ ⎛ 1 ⎞ ⎛ 1⎞ ⎛ 1⎞ ⎤
− sin101 ( x ) ⎡ ⎡x⎤ ⎡ x⎤ ⎤ = nf1 ( n) − ⎢⎜1 − ⎟ + ⎜1 − ⎟ + ⎜1 −
⎟⎠ + ... + ⎜⎝1 − ⎟⎠ ⎥
⇒ f (–x) =
⎢Q ⎢ p ⎥ + ⎢ − p ⎥ = − 1⎥ ⎣⎝ 2 ⎠ ⎝ 3 ⎠ ⎝ 4 n ⎦
⎡x⎤ 1 ⎣ ⎣ ⎦ ⎣ ⎦ ⎦
−1 − ⎢ ⎥ + = nf1(n) – [n – f1(n)] = (n + 1) f1(n) – n
⎣p ⎦ 2  
The correct option is (C)
sin101 ( x ) 74. Q {x} ∈ [0, 1)
⇒ f (–x) =
= f (x)
⎡x⎤ 1 \ sin {x} ∈ [0, sin 1) but f (x) is defined if sin {x} ≠ 0
⎢p ⎥ 2 +
⎣ ⎦
1 ⎛ 1 ⎞
\ f (x) is an even function. \
∈ ,∞
sin{x} ⎜⎝ sin1 ⎟⎠
The correct option is (B)
67. The given function f is well defined only when | x | – x > 0 ⎡ 1 ⎤
\ ⎢ ⎥ ∈{1, 2, 3.....}
⇒ x < 0 ⎣ sin{x} ⎦
Required domain is (–∞, 0) The correct option is (B)
The correct option is (D) 75. We have,
⎧ n P if n ≥ m f  (x) = 2 min {| f (x) – g (x)|, 0}
68. Number of one–one functions = ⎨ m
⎩ 0 if n < m ⎧0 f ( x) > g( x)
The correct option is (D)
= ⎨
⎩2( f ( x ) − g ( x )), f ( x ) ≤ g ( x )
69. Function is increasing
y−3 ⎧ f ( x ) − g ( x ) − | f ( x ) − g ( x ) |, f ( x) > g( x)
x= = g(y)
= ⎨
4 ⎩ f ( x ) − g ( x ) − | f ( x ) − g ( x ) |, f ( x) ≤ g( x)
The correct option is (D) \ f (x) = f (x) – g (x) – |g (x) – f (x)|
70. Given f (x) = x3 + 5x + 1 The correct option is (D)
Now f ′(x) = 3x2 + 5 > 0, ∀ x ∈ R 76. We have,
\ f (x) is strictly increasing function ⎧x x ∈Q
\ It is one–one (x) = ⎨
f 
⎩1 − x, x ∉Q
Clearly, f (x) is a continuous function and also increasing
on R, ⎧ f ( x) , f ( x) ∈Q
\ fof (x) = ⎨

Lt f ( x ) = − ∞ and Lt f ( x ) = ∞ ⎩1 − f ( x ) , f ( x) ∉ Q
x → −∞ x→∞
⎧ x, x ∈ Q, x ∈Q
\ f (x) takes every value between –∞ and ∞.

⎪ 1 − x, x ∉ Q, x ∈Q

Thus, f (x) is onto function. ⎪

= ⎨

The correct option is (C) ⎪1 − x, 1 − x ∈ Q, x ∉Q
⎪⎩1 − (1 − x ), 1 − x ∉ Q, x ∉Q

Objective_Maths_JEE Main 2017_Ch 2.indd 45 01/01/2008 03:20:05


2.46  Chapter 2

⎧ x, x ∈ Q ⎡ n + 1⎤ ⎡ n ⎤

=⎨ ⇒ [n] = ⎢
⎥+⎢ ⎥
⎩ x, x ∉ Q ⎣ 2 ⎦ ⎣2⎦
\ fof (x) = x, x ∈ [0, 1]. ⎡ n + 1⎤ ⎡ n ⎤
⇒ n = ⎢
⎥+⎢ ⎥
The correct option is (C) ⎣ 2 ⎦ ⎣2⎦
77. We have, ⎡n ⎤
+ 1⎥
f (x – f (y)) = f (f (y)) + xf (y) + f (x) – 1 (1) ⎡ n + 1⎤ ⎡ n ⎤ ⎢ 2

= ⎢ ⎥+⎢ ⎥+⎢ ⎥   [Using (1)]
Put  x = f (y) = 0 ⎣ 2 ⎦ ⎣4⎦ ⎣ 2 ⎦
then  f (0) = f (0) + 0 + f (0) – 1
⎡ n + 1⎤ ⎡ n + 2 ⎤ ⎡ n ⎤
\ f (0) = 1 (2)
= ⎢ ⎥+⎢ ⎥+⎢ ⎥
⎣ 2 ⎦ ⎣ 4 ⎦ ⎣4⎦
Again, put x = f (y) = l in (1) ⎡n ⎤
Then,  f (o) = f (l) + l2 + f (l) – 1 + 1⎥
⎡ n + 1⎤ ⎡ n + 2 ⎤ ⎡ n ⎤ ⎢ 4
⇒ 1 = 2f (l) + l2 – 1

= ⎢ ⎥+⎢ ⎥+⎢ ⎥+⎢ ⎥
⎣ 2 ⎦ ⎣ 4 ⎦ ⎣8⎦ ⎣ 2 ⎦
2 − l2 l2
[Using (1)]
\ f (l) = =1−
2 2
x2 ⎡ n + 1⎤ ⎡ n + 2 ⎤ ⎡ n + 4 ⎤ ⎡ n ⎤
Hence, f (x) = 1 − = ⎢
⎥+⎢ ⎥+⎢ ⎥+⎢ ⎥
2 ⎣ 2 ⎦ ⎣ 4 ⎦ ⎣ 8 ⎦ ⎣8⎦
The correct option is (C)
Continuing in this manner, we have,
78. A function whose graph is symmetrical about the origin ⎡ n + 1⎤ ⎡ n + 2 ⎤ ⎡ n + 4 ⎤
must be odd.
⎢ 2 ⎥ + ⎢ 4 ⎥ + ⎢⎣ 8 ⎥⎦ + ... = n
⎣ ⎦ ⎣ ⎦
(3x + 3–x) is an even function. The correct option is (A)
Since cos x is an even function and log (x + 1 + x 2 ) is an 82. We have,
odd function,
⎧1 + n − n = 1, x = n ∈ I
\ cos (log (x + 1 + x 2 )) is an even function.
g (x) = ⎨
If f (x + y) = f (x) + f (y) ∀ x, y ∈ R, then f (x) must be odd. ⎩1 + n + k − n = 1 + k , x = n + k ,
The correct option is (C) where n ∈ I, 0 < k < 1

79. The function f (x) = x2 + 2, x ∈ (–∞, ∞) is not injective as ⎧ −1, g ( x ) < 0

f (1) = f (–1) but 1 ≠ –1 Now, f [g (x)] = ⎨ 0, g ( x ) = 0

The function f (x) = (x – 4) (x – 5), x ∈ (– ∞, ∞) is not one-one ⎪ 1, g ( x) > 0

as f (4) = f (5) but 4 ≠ 5
Clearly, g (x) > 0 for all x. So, f [g (x)] = 1, for all x.

4 x 2 + 3x − 5
The function, f (x) = , x ∈ (–∞, ∞) is also not
The correct option is (B)
4 + 3x − 5 x 2
injective as f (1) = f (–1) but 1 ≠ –1 83. We have, g (x) = 1 + x
For the function, f (x) = |x + 2|, x ∈ [– 2, ∞) and f [g (x)] = 3 + 2 x + x(1)

Let f (x) = f (y), x, y ∈ [–2, ∞) ⇒ |x + 2| = |y + 2|
Also, f [g (x)] = f (1 + x )(2)

⇒ x + 2 = y + 2

From (1) and (2), we get
⇒ x = y
f (1 + x ) = 3 + 2 x + x.
So,  f is an injection.
Let 1 + x = y or x = ( y – 1)2
The correct option is (B)
\ f ( y) = 3 + 2 ( y – 1) + ( y – 1)2
80. ( fog) (x) = f [g (x)] = f (|3x + 4|)
= 3 + 2y – 2 + y2 – 2y + 1 = 2 + y2
Since the domain of f is [–3, 5],
\ f (x) = 2 + x2
\ –3 ≤ |3x + 4| ≤ 5 ⇒ |3x + 4| ≤ 5
The correct option is (B)
⇒ –5 ≤ 3x + 4 ≤ 5
1 84. For f to be defined, we must have
⇒ –9 ≤ 3x ≤ 1 ⇒ –3 ≤ x ≤
3 ⎛ 1 ⎞ 1
log1/ 2 ⎜1 + 4 ⎟ < – 1 ⇒ 1 + 4 > (2– 1)– 1 = 2 which is pos-
⎡ 1⎤ ⎝ x ⎠ x
\ Domain of fog is ⎢ −3, ⎥
⎣ 3⎦ 1
sible only if 4 > 1, i.e., 0 < x < 1
The correct option is (B) x
81. For any x ∈ R, we have Hence, the domain of the given function is {x: 0 < x < 1}

⎡ x ⎤ ⎡ x + 1⎤
The correct option is (A)
[x] = ⎢ ⎥ + ⎢ ⎥ (1)
⎣2⎦ ⎣ 2 ⎦

Objective_Maths_JEE Main 2017_Ch 2.indd 46 01/01/2008 03:20:09


Functions  2.47

85. We have, f (x + 2a) = f ((x + a) + a) Substituting respective value in (5), we have
1 f ′′(2) = 2
= −
f ( x + a) − ( f ( x + a)) 2
2 Hence, the polynomial f (x) can be written as
1 1 ⎛1 ⎞
2
f (x) = x3 – 5x2 + 2x + 6
=
− − f ( x ) − ( f ( x )) 2 − ⎜ − f ( x ) − ( f ( x )) 2 ⎟
2 2 ⎝2 ⎠ Therefore, f (2) – f (1)
= (8 – 20 + 4 + 6) – (1 – 5 + 2 + 6)
1 1 = –2 – 4 = – 6
=
− − f ( x ) + ( f ( x )) 2
2 4
\ f (2) – f (1) = –6 = – f (0)
1 ⎛1 ⎞ The correct option is (B)
=
− ⎜ − f ( x )⎟ = f (x)
2 ⎝2 ⎠ 89. Given, q2 – 4pr = 0 and p > 0
Hence, f (x) is periodic with period 2a. For f (x) to be defined
The correct option is (B) px3 + (p + q)x2 + (q + r)x + r > 0
86. Since the domain of f is (0, 1), ⇒ px2(x + 1) + qx(x + 1) + r(x + 1) > 0
\ 0 < ex < 1 and 0 < ln |x| < 1 ⇒ (x + 1)(px2 + qx + r) > 0
⇒ log 0 < x < log 1 and e0 < |x| < e1 q
⇒ x > – 1 and x ≠ −   [Since q2 – 4pr = 0
⇒ –∞ < x < 0 and 1 < |x| < e 2p
q q
⇒ x ∈ (–∞, 0) and x ∈ ((–∞, –1) ∪ (1, ∞)) ∩ (–e, e) \ at x = − , px2 + qx + r = 0 and at x ≠ − , px2 + qx +
r > 0] 2 p 2 p
⇒ x ∈ (–∞, 0) and x ∈ (–e, –1) ∪ (1, e)
⎡ ⎧ q ⎫⎤
⇒ x ∈ (–e, –1) \ Domain = R − ⎢( − ∞, −1] ∪ ⎨ − ⎬ ⎥
The correct option is (A) ⎣ ⎩ 2 p ⎭⎦
The correct option is (B)
87. sin– 1 (x – x2) is defined when – 1 ≤ x – x2 ≤ 1
90. Since the function is periodic, a2 – 3 and b2 + 7 should be
1− 5 1+ 5 perfect squares, which is possible only if a = 2, b = 3 in

≤x≤ (1)
2 2 which case f (x) = sin x + cos 4x, whose period is 2p.
1 1 The correct option is (B)

1− is defined when 1 − ≥0
|x| |x| sin101 x
⇒ x ≤ – 1 or x ≥ 1
(2) 91. We have, f (x) =
⎡x⎤ 1
1 ⎢p ⎥ + 2

and 2 is defined when x – 1 < 0 or x – 1 ≥ 1
2 2 ⎣ ⎦
[ x − 1]
i.e., x ∈ (–∞, − 2 ) ∪ (–1, 1) ∪ ( − 2 , ∞)(3)
sin101 ( − x )
⇒ f (–x) =

⎡ x⎤ 1

From (1), (2) and (3), we get ⎢− p ⎥ + 2
⎣ ⎦
⎛ 1+ 5⎞
x ∈ ⎜ 2,
Case I: when x = np (n ∈ Z )
⎝ 2 ⎟⎠
sin101 ( − np )
The correct option is (A) (–x) =
f  =0
⎡ − np ⎤ 1
88. Since, ⎢ p ⎥ 2 +
⎣ ⎦
f (x) = x3 + x2f ′(1) + xf ′′(2) + f ′(3)(1) Case II: When x ≠ np, n ∈ Z
⇒ f ′(x) = 3x2 + 2xf ′(1) + f ′′(2)(2)

− sin101 ( x )
⇒ f ′′(x) = 6x + 2f ′(1)(3)
f (–x) =

⎡ x⎤ 1
⇒ f ′′′(x) = 6 i.e., a constant function
⎢− p ⎥ + 2
⎣ ⎦
Hence, f  ′(3) = 6
⎡ ⎡x⎤ ⎡ x⎤
− sin101 ( x ) ⎤

Using equation (3), we have ⇒ f (–x) =

⎡x⎤ 1 ⎢Q ⎢ p ⎥ + ⎢ − p ⎥ = − 1⎥
−1 − ⎢ ⎥ + ⎣ ⎣ ⎦ ⎣ ⎦ ⎦
f ′′(2) = 12 + 2f ′(1)(5) ⎣ p ⎦ 2   
Substituting x = 1, in equation (2), we have

sin101 ( x )
f ′(1) = 3 + 2f ′(1) + f ′′(2) ⇒ f (–x) =
= f (x)
⎡x⎤ 1
⇒ f ′(1) = 3 + 2f ′(1) + 12 + 2f ′(1)  {using (5)}
⎢p ⎥ 2+
⎣ ⎦
⇒ 3f ′(1) = –15
\ f (x) is an even function.

\ f ′(1) = –5
(6)
The correct option is (B)

Objective_Maths_JEE Main 2017_Ch 2.indd 47 01/01/2008 03:20:12


2.48  Chapter 2

92. Graph is symmetric about x = k if f (k – x) = f (k + x)


The domain of definition is the darkened portion on the
⇒ a(k – x)3 + b(k – x)2 + c(k – x) + d X-axis
⎡ p p⎞ ⎛p p⎤
= a(k + x)3 + b(k + x) + c(k + x) + d i.e. x ∈ ⎢ − , − ⎟ ∪ ⎜ , ⎥

⎣ 2 3⎠ ⎝ 3 2⎦
⇒ 2ax3 – (6ak2 + 4bk + 2c)x = 0
It is true for all x if a = 0 and 6ak2 + 4bk + 2c = 0 The correct option is (C)
c c 95. For the sgn operation to be defined, we have
i.e., a = 0 and k = – ⇒a+k= − .
2b 2b 9 – x2 > 0 i.e., –3 < x < 3 (1)
The correct option is (A) For the square root operation to be defined, we have
93. For the ln operation to be defined, we have {x} > 0 [x]3 – 4[x] ≥ 0
Plotting the curves y = {x} and y = 0, we can see that the ⇒ [x] ([x] + 2) ([x] – 2) ≥ 0
values of x satisfying the above inequality, are
⇒ –2 ≤ [x] ≤ 0 or [x] ≥ 2
x ∈ R – I(1)
i.e., –2 ≤ x < 1 or x ≥ 2 (2)
For the square root operation to be defined, we have
The union of the intervals (1) and (2) gives the domain as
{x} ≤ x/2 [–2, 1) ∪ [2, 3)
The correct option is (B)
96. we have

f (x) =
(log0.2 x )3 + (log0.2 x 3 ) (log0.2 0.0016 x ) + 36

= (log0.2 x )3 + (3 log0.2 x ) {log0.2 x + log0.2 (0.2)4 } + 36

= l 3 + 3l ( l + 4) + 36   [putting log0.2 x = l]


= l 3 + 3l 2 + 12l + 36 = ( l + 3) ( l 2 + 12)
Plotting the curves y = {x} and y = x/2 as shown above, we For the square root operation to be defined, we have
can see that the values of x satisfying the above inequality, (l + 3) (l2 + 12) ≥ 0
are
i.e., l + 3 ≥ 0  (Q l2 + 12 is a positive quantity)
x ∈ {0} ∪ [1, ∞)(2)
i.e., log0.2 x ≥ – 3 i.e., x ≤ (0.2)–3 = 53 = 125
The domain of definition is the intersection of (1) and (2)
Also, x > 0 for log to be defined
which gives x ∈ (1, ∞) – I +
Hence, the required domain of definition is (0, 125]
The correct option is (C)
The correct option is (C)
94. For the ln operation to be defined, we have
97. We have,
1 – 2 |cos x| > 0
1 y = [x2] – [x]2, x ∈ [0, 2]
i.e., |cos x| < (1) i.e., y = [x ],
2
0≤x<1
2
For the inverse cos operation to be defined, we have y = [x2] – 1, 1≤x<2
2x = [x2] – 1, x=2
–1≤ ≤1
p = 0, x=2
p p i.e., y = 0, 0≤x<1
i.e., − ≤ x ≤ (2)
2 2 = 1 – 1 = 0, 1≤ x < 2
To find the values of x satisfying both (1) and (2), let us plot
= 2 – 1 = 1, 2≤x< 3
1
the curves y = |cos x| and y = in the interval [–p/2, p/2] as = 3 – 1 = 2, 3≤x<2
shown below 2
= 0, x=2
Hence, the range is {0, 1, 2}
The correct option is (D)
98. For the inverse sin operation to be defined, we have
⎡ 1⎤
−1 ≤ ⎢ x 2 + ⎥ ≤ 1
⎣ 2⎦
1 3 3
i.e., −1 ≤ x 2 + < 2 i.e., − ≤ x 2 < (1)
2 2 2

Objective_Maths_JEE Main 2017_Ch 2.indd 48 01/01/2008 03:20:15


Functions  2.49

For the inverse cos operation to be defined, we have a x2 + 6x − 8


101. Let y =
⎡ 1⎤ a + 6 x − 8x 2
−1 ≤ ⎢ x 2 − ⎥ ≤ 1
⎣ 2⎦ i.e., (a + 8y)x2 + 6(1 – y)x – (ay + 8) = 0
1 According to the given condition, y takes all real values,
i.e., −1 ≤ x 2 − < 2
2 for real x. In other words, the above quadratic equation in x
−1 5 should have real roots for every real y
i.e., ≤ x 2 < (2)
2 2 i.e., D ≥ 0, ∀ y ∈ R
Intersection of inequalities (1) and (2) gives i.e., 36(1 – y)2 + 4(ay + 8) (a + 8y) ≥ 0, ∀ y ∈ R
−1 3 3 i.e., (9 + 8a) y2 + (a2 + 46)y + (9 + 8a) ≥ 0, ∀ y ∈ R
≤ x 2 < i.e., 0 ≤ x 2 <   [Q x2 cannot be negative]
2 2 2 i.e., D ≤ 0 and coefficient of y2 > 0
Now, we have i.e., (a2 + 46)2 ≤ 4(9 + 8a)2 and 9 + 8a > 0
1
y = sin–1 (0) + cos–1(– 1) = p, 0 ≤ x2 < i.e., a2 + 46 ≤ 2(9 + 8a) and a > – 9/8
2
i.e., a2 – 16a + 28 ≤ 0 and a > – 9/8
1 3
y = sin–1 (1) + cos–1(0) = p, ≤ x2 < i.e., 2 ≤ a ≤ 14 and a > – 9/8
2 2
Hence, the range is {p} i.e., 2 ≤ a ≤ 14
The correct option is (C) Hence, f : R → R is onto for 2 ≤ a ≤ 14
99. We have, The correct option is (D)
ax 2 + 2 x + 1 ⎛ 2| x|⎞
f (x) = 102. The function f (x) = sin −1 ⎜ is defined for
2x2 − 2x + 1 ⎝ 1 + x 2 ⎟⎠
which is defined ∀ x ∈ R, since 2| x| 2| x|
2 ≤ 1 i.e., ≤1
⎛ 1⎞ 1 1 + x2 1 + x2
2x2 – 2x + 1 = 2 ⎜ x − ⎟⎠ + ≠ 0 for any real x. ⎡ 2| x| ⎤
⎝ 2 2  ⎢Q is a positive quantity⎥
Now, if f: R → [–1, 2] is onto, then ⎣ 1 + x 2

ax 2 + 2 x + 1 i.e., x2 – 2 |x| + 1 ≥ 0
−1 ≤ ≤ 2, ∀ x ∈ R
2x2 − 2x + 1 i.e., (|x| – 1)2 ≥ 0, which is true ∀ x ∈ R(1)
Solving the left-hand inequality, we have The function sin x is defined for, sin x ≥ 0
(a + 2)x2 + 2 ≥ 0 i.e., 2np ≤ x ≤ (2n + 1)p, n ∈ I(2)
which is true ∀ x ∈ R if a ≥ –2 (1) Intersection of inequalities (1) and (2) gives the domain as
Solving the right-hand inequality, we have [2np, (2n + 1)p], n ∈ I
(a – 2)x2 + 6x – 1 ≤ 0 The correct option is (B)
which is true for all x ∈ R if coefficient of x2 < 0 and D ≤ 0 103. [sin x] is always defined.
i.e., a < 2 and 36 + 4 (a – 2) ≤ 0 ⎛ p ⎞
cos ⎜ is also defined everywhere except when
i.e., a < 2 and a ≤ –7 ⎝ [ x − 1] ⎟⎠
i.e., a ≤ – 7 (2) [x – 1] = 0
Hence, from (1) and (2) the permissible values of a are ⇒ 0 ≤ x – 1 < 1
given by
⇒ 1 ≤ x < 2
(–∞, –7] ∪ [–2, ∞)
Hence, domain ∈ R – [1, 2)
The correct option is (C)
The correct option is (B)
100. We have, f (x) = x + (–1)x – 1, x ∈ N
x − [ x] {x}
Thus, we have 104. Here, y = =
1 + x − [ x ] 1 + {x}
f (1) = 1 + 1 = 2, f (2) = 2 – 1 = 1
Thus, domain = (–∞, ∞)
f (3) = 3 + 1 = 4, f (4) = 4 – 1 = 3
{x}
f (5) = 5 + 1 = 6, f (6) = 6 – 1 = 5 and so on. So, from y = we have,
1 + {x}
The graph of f (x) consists of the points (1, 2), (2, 1), (3, 4),
(4, 3), (5, 6), (6, 5)… Thus, if (a, b) is a point on the graph, y + y {x} = {x}
then (b, a) is also a point on the graph. Hence the inverse of y
⇒ {x} =
f  is f  itself 1− y
y
i.e., f –1(x) = x + (– 1)x – 1, x ∈ N Here, 0 ≤ {x} < 1 so, 0 ≤ <1
1− y
The correct option is (A)

Objective_Maths_JEE Main 2017_Ch 2.indd 49 01/01/2008 03:20:18


2.50  Chapter 2

1 ⇒ f (2) = {(1 – p)p–1} {1 – 1 + p – f (2)}


⇒ 0 ≤ y <
2 = [(1 – p)p–1] [p – f (2)]
⎡ 1⎞ = (1 – p) – (1 – p)p–1f (2)
Hence, range = ⎢0, ⎟
⎣ 2⎠ ⇒ f (2) {1 + (1 – p)p–1} = 1 – p
The correct option is (C) ⇒ f (2) = p(1 – p)
105. For g(x) = f |[x]| to be defined, we must have The correct option is (A)
–3 ≤ |[x]| ≤ 2 x x x
109. Let Sn = + + +
⇒ 0 ≤ |[x]| ≤ 2 [as |x| ≥ 0 ∀ x] x + 1 ( x + 1) ( 2 x + 1) ( 2 x + 1) (3 x + 1)
⇒ –2 ≤ [x] ≤ 2 [as |x| ≤ a ⇒ –a ≤ x ≤ a] x
... +
⇒ –2 ≤ x < 3  [by definition of greatest integer function] (( n − 1) x + 1) ( nx + 1)
Hence, domain of g(x) is [– 2, 3). ⎛ 1 ⎞ ⎛ 1 1 ⎞ ⎛ 1 1 ⎞
= ⎜1 − + − + −
The correct option is (C) ⎝ 1 + x ⎟⎠ ⎜⎝ 1 + x 1 + 2 x ⎟⎠ ⎜⎝ 1 + 2 x 1 + 3 x ⎟⎠
106. We know that {x + r} ={x} as r ∈ Integer
⎛ 1 1 ⎞
+ ... + ⎜ −
{x + r} ⎝ 1 + ( n − 1) x 1 + nx ⎟⎠
2000 2000
{x}
\ [x] + ∑
r =1 2000
= [x] + ∑
r =1 2000
1
= 1 −
⎡ {x} {x} ⎤ 1 + nx
= [x] + ⎢ + + ... + up to 2000 times ⎥
⎣ 2000 2000 ⎦ ⎧1 when x ≠ 0
\ f (x) = lim Sn = ⎨
2000 {x}
n →∞
⎩0 when x = 0
= [x] + = [x] + {x} = x
2000 \ Range f = {0, 1}
The correct option is (A) The correct option is (A)
n
1
107. f (x) = ∑ (1 + [sin k x])
k =1
110. For x =
f ( y)
, we have

= n + [sin x] + [sin 2x] + ... + [sin n x](1) ⎛ 1⎞ 1 yq


p f ⎜ x ⋅ ⎟ = . y q ⇒ f (1) =
Case I: When kx ≠ for k = 1, 2, 3, …, n ⎝ x⎠ f ( y) p
{ f ( y )} p
2
p yq/ p
Since 0 < kx < p and kx ≠ ⇒ f (y) =
2 { f (1)}1/ p
\ 0 < sin kx < 1, for k = 1, 2, ..., n For y = 1, we have f (1) = 1
\ [sin kx] = 0, for k = 1, 2, 3, ..., n \ f (y) = yq/p or f (x) = xq/p(1)
\ From (1), f (x) = n Hence, f (x . yq/p) = xp . yq
p
Case II: When exactly one of x, 2x, 3x, ..., nx is Let yq/p = z ⇒ y = zp/q
2
p ⇒ f (x ⋅ z) = xp . zp or f (x) = xp(2)
Here, not more than one of x, 2x, 3x, ...nx can be
2 From (1) and (2), we have
In this case, one of sin x, sin 2x, …, sin nx is 1 and others lie xq/p = xp
between 0 and 1 q
⇒ = p  or  q = p2
\ From (1), f (x) = n + 1 p
Hence, range of f = {n, n + 1} The correct option is (C)
The correct option is (C) 111. Given, f (x + y) + f (x – y) = 2 f (x) f (y)(1)
108. We have, Interchange x and y in (1), we get
f (n) = {(1 – p)p–1} {f (n + 1) + f (n + 2) + … to ∞} f (y + x) + f (y – x) = 2 f (y) f (x)(2)
Put n = 1 From (1) and (2), f (x – y) = f (y – x)
f (1) = {(1 – p)p–1} {f (2) + f (3) + … to ∞} Putting y = 2x, we get f (x) = f (– x)
= {(1 – p)p–1} {1 – f (1)} The correct option is (A)
= (1 – p)p–1 – (1 – p)p–1 f (1) 112. We have,
⇒ f (1) {1 + (1 – p)p–1} = (1 – p)p–1 3 ⎛ 2 x + 29 ⎞
f (x) = − f ⎜ + 50x + 40 (1)
⇒ f (1) × p–1 = (1– p)p–1 2 ⎝ x − 2 ⎟⎠
⇒ f (1) = 1 – p 2 x + 29
Replacing x by in the given functional equation
Put n = 2, x−2
we get
f (2) = {(1 – p)p–1} {f (3) + f (4) + …}

Objective_Maths_JEE Main 2017_Ch 2.indd 50 01/01/2008 03:20:22


Functions  2.51

⎡ ⎛ 2 x + 29 ⎞ ⎤ ⇒ g(x + 2p) = [1 – {{1 – {g(x)}3}1/3}3]1/3


⎢ 2⎜ ⎟ + 29 ⎥ from (1) and (2), we have
⎛ 2 x + 29 ⎞ −3 ⎢ ⎝ x − 2 ⎠ ⎥ + 50 ⎛ 2 x + 29 ⎞ + 40
f⎜ f
⎝ x − 2 ⎟⎠ 2 ⎢ ⎛ 2 x + 29 ⎞ ⎥ ⎜⎝ x − 2 ⎟⎠ ⇒ g(x + 2p) = [1 – {1 – {g(x)}3}]1/3
= ⎢ ⎜ ⎟ −2 ⎥ ⇒ g(x + 2p) = [1 – 1 + {g(x)}3]1/3
⎣ ⎝ x−2 ⎠ ⎦
⇒ g(x + 2p) = [{g(x)}3]1/3
⎛ 2 x + 29 ⎞ 3 ⎛ 2 x + 29 ⎞
⇒ f⎜ ⎟ = − f ( x ) − 50 ⎜ + 40 (2) ⇒ g(x + 2p) = g(x)
⎝ x−2 ⎠ 2 ⎝ x − 2 ⎟⎠
which shows f (x + 2p) – 1 = f (x) – 1
Using (2) in (1), we get or, f (x + 2p) = f (x)
9 ⎛ 2 x + 29 ⎞ Hence, f (x) is periodic with period 2p.
(x) =
f  f ( x ) + 75 ⎜ − 60 + 50 x + 40
4 ⎝ x − 2 ⎟⎠ The correct option is (B)
9 ⎛ 2 x + 29 ⎞
⇒ f ( x ) − f ( x ) = 20 − 50 x − 75 ⎜ 115. Given f (x) + f (x + a) + … + f (x + an) = 0 (1)
4 ⎝ x − 2 ⎟⎠
Replace x by x + a, we get
5 ⎛ 2 x + 29 ⎞ f  (x + a) + f (x + 2a) + ... + f {x + a(n + 1)} = 0 (2)
⇒ f ( x ) = 20 − 50 x − 75 ⎜
4 ⎝ x − 2 ⎟⎠ Subtracting (2) from (1), we get
⎛ 2 x + 29 ⎞ f  (x) – f {x + a (n + 1)} = 0
⇒ f (x) = 16 − 40 x − 60 ⎜
⎝ x − 2 ⎟⎠ ⇒ f (x) is periodic with period a(n + 1)
The correct option is (A) The correct option is (B)
113. Here, 116. Since y = log3 x is an increasing function, so S is mapped
g2(x) = (gog) (x) = g{g (x)} = g(3 + 4x) onto the set (log33, log327) = (1, 3)
⇒ g2(x) = 3 + 4 (3 + 4x) The correct option is (C)
⇒ g2(x) = 15 + 42x 117. We have f (x) + f (x) = x – 3 + 4 – x = 1, so that
⇒ g2(x) = (42 – 1) + (42)x | f (x)| + |f (x)| = 1
On generalizing, we have Furthermore,
gn(x) = (4n – 1) + (4n)x ⎧x − 3 if x ≥ 3
| f (x)| = ⎨ ;
Then, for finding inverse, gn(x) = y ⎩3 − x if x < 3
⇒ y = (4n – 1) + (4n)x ⎧4 − x if x ≤ 4
and |f (x)| = ⎨
⇒ x = (y + 1 – 4n)4–n ⎩x − 4 if x > 4
⇒ g–n(y) = (y + 1 – 4n)4–n
⎧7 − 2 x if x < 3
⇒ g–n(x) = (x + 1 – 4n) 4–n ⎪
⇒ | f (x)| + |f (x)| = ⎨ 1 if 3 ≤ x ≤ 4
The correct option is (B) ⎪2 x − 7 if x > 4
114. We have, ⎩
f (x + p) = 1 + [2 – 3 f (x) + 3 { f (x)}2 – { f (x)}3]1/3 We need those points for which the L.H.S. is greater than 1.
Clearly, we can exclude values of x between 3 and 4. Now,
⇒ f (x + p) = 1 + [1 + {1 – f (x)}3]1/3
for values of x less than 3, 7 – 2x is greater than 1, and for
⇒ f (x + p) – 1 = [1 – { f (x) – 1}3]1/3 values of x greater than 4, 2x – 7 is greater than 1.
⇒ g(x + p) = [1 – {g(x)}3]1/3(1) Therefore, the given inequality is true for values of x given
where g(x) = f (x) – 1 and g(x + p) = f (x + p) – 1 by (–∞, ∞) – [3, 4].
⇒ g(x + 2p) = [1 – {g(x + p)}3]1/3(2) The correct option is (C)

More Than One Option Correct Type


⎛ x + 1⎞ x +1
118. Since f (x) = f ⎜ ⇒x= \ f (– x) = f (x), ∀ x ∈ [–5, 5]
⎝ x + 2 ⎠⎟ x+2
⎛ x + 1⎞
⇒ x2 + x – 1 = 0 ⇒ x = – ⎜ ⇒ x2 + 3x + 1 = 0
⎝ x + 2 ⎟⎠
−1 ± 5
⇒ x = (1) −3 ± 5
2 ⇒ x = (2)
Since f (x) is an even function defined on [– 5, 5], 2

Objective_Maths_JEE Main 2017_Ch 2.indd 51 01/01/2008 03:20:24


2.52  Chapter 2

From (1) and (2), the values of x are 2p


123. (A) The period of cos p x is
= 2, and period of {x} is 1
−1 ± 5 −3 ± 5 p
and Hence, period of the given function is L.C.M. of (1, 2) = 2
2 2
⎛p ⎞ ⎛p ⎞
The correct option is (A) and (B) (B) Solving tan ⎜ [ x + T ]⎟ = tan ⎜ [ x ]⎟
⎝2 ⎠ ⎝2 ⎠
119. Let the required function be f (x) = ax + b
If a > 0, then f (–1) = 0 and f (1) = 2 i.e., [x + T] – [x] = 2n
⇒ –a + b = 0 and a + b = 2 gives a value of T independent of x only if T is an
­integer. In that case, the above equation reduces to
⇒ a = 1 and b = 1
[x] + T – [x] = 2n
If a < 0, then f (–1) = 2 and f (1) = 0
i.e., T = 2n
⇒ –a + b = 2 and a + b = 0
Hence, period of f (x), is the smallest positive value of
⇒ a = –1 and b = 1
T, i.e., 2.
Hence, f (x) = x + 1  or  f (x) = –x + 1
(C) We have period of sin x = 2p and period of {x} = 1
The correct option is (B) and (C)
Hence, period of the given functions is L.C.M. of (2p, 1)
120. Taking f (x) = log x, we see that
which does not exist since 2p is an irrational number.
⎛ x⎞ Hence, the function is not periodic
f  ⎜ ⎟ = f (x) – f ( y)
⎝ y⎠ (D) Let us solve
Clearly, f (x) is not bounded sin{cos (x + T)} = sin{cos x}
⎛ 1⎞ i.e., cos (x + T) = np + (– 1) n cos x, n ∈ I
and f ⎜ ⎟ = – log x → ∞ as x → 0 Putting n = 0, gives cos (x + T) = cos x,
⎝ x⎠
Also, x f (x) = x log x → 0 as x → 0 which gives T = 2p as the smallest positive value. For
The correct option is (C) and (D) no other value of n can a value of T be found indepen-
dent of x.
121. Since, f (– x) = f (x)
Hence, the required fundamental period is 2p.
⇒ f (x) is an even function, its graph will be symmetrical
about y-axis. The correct option is (A) and (B)
⎛ | x |3 + | x | ⎞ 124. The function is defined for all real values of x except those
Also, f (x) = − ⎜
⎝ 1 + x 2 ⎟⎠ which satisfy the equation
⇒ f (x) = –(positive) = negative [|x – 1|] + [|7 – x|] – 6 = 0 (1)
i.e., the graph of f (x) completely lies below the x-axis, and Case I: (1 < x < 7)
is also symmetric about y-axis (as discussed above). Equation (1) reduces to
\ The graph of f (x) lies in III and IV quadrants. [x – 1] + [7 – x] – 6 = 0
The correct option is (C) and (D) i.e., [x] – 1 + [– x] + 7 – 6 = 0 or [x] + [– x] = 0
122. When x1 = –1 and x2 = 1, which is true ∀ x ∈ I
⎛ −1 − 1 ⎞ Thus, every integer in (1, 7) satisfies equation (1).
then f (– 1) – f (1) = f ⎜ = f (– 1) Case II: (x ≤ 1)
⎝ 1 + 1(1) ⎟⎠
⇒ f (1) = 0, Equation (1) reduces to
⎛1 − x⎞ [1 – x] + [7 – x] – 6 = 0
which is satisfied when f (x) = tan − 1 ⎜
⎝ 1 + x ⎟⎠ i.e., 1 + [– x] + 7 + [– x] – 6 = 0
When x1 = x2 = 0, then
i.e., [– x] = – 1 i.e., – 1 ≤ – x < 0 or 0 < x ≤ 1
⎛ 0 − 0⎞
f (0) – f (0) = f ⎜ = f (0) ⇒ f (0) = 0 Thus, equation (1) is satisfied ∀ 0 < x ≤ 1
⎝ 1 − 0 ⎟⎠
Case III: (x ≥ 7)
When x1 = –1 and x2 = 0, then Equation (1) reduces to
⎛ −1 − 0 ⎞ [x – 1] + [x – 7] – 6 = 0
f (–1) – f (0) = f ⎜ = f (– 1) ⇒ f (0) = 0,
⎝ 1 − 0 ⎟⎠ i.e., [x] – 1 + [x] – 7 – 6 = 0 or [x] = 7
⎛1 − x⎞ \ 7 ≤ x < 8
which is satisfied when f (x) = log ⎜
⎝ 1 + x ⎟⎠ Thus, equation (1) is satisfied ∀ 7 ≤ x < 8. The union of the
⎛1 + x⎞ intervals obtained in the above three cases gives the domain
and, f (x) = log ⎜
⎝ 1 − x ⎟⎠ of definition as
The correct option is (A), (B) and (C) R – (0, 1] – [7, 8) – {2, 3, 4, 5, 6}
The correct option is (A), (B) and (C)

Objective_Maths_JEE Main 2017_Ch 2.indd 52 01/01/2008 03:20:26


Functions  2.53

x(sin x + tan x ) x(sin x + tan x ) 127. We have,


125. f (x) = =
⎡x + p⎤ 1 ⎡x⎤ 1 f (x) = sin–1(log [x]) + log(sin–1[x])(1)
⎢ p ⎥−2 ⎢p ⎥ + 1 − 2 Let g(x) = sin–1(log [x])(2)
⎣ ⎦ ⎣ ⎦
x(sin x + tan x ) and, h(x) = log(sin–1[x])(3)
=
⎡x⎤ Now for g(x);
⎢ p ⎥ + 0.5 – 1 ≤ log [x] ≤ 1 {as sin–1 q exists when – 1 ≤ q ≤ 1}
⎣ ⎦
− x(sin( − x ) + tan( − x )) and, [x] > 0 {as log [x] exists when [x] > 0}
⇒ f (–x) =
⎡− x⎤ 1
⇒ ≤ [x] ≤ e and [x] > 0
⎢ p ⎥ + 0.5 e
⎣ ⎦
⇒ [x] = 1, 2
⎧ x(sin x + tan x )
⎪⎪ , x ≠ np ⇒ x ∈ [1, 3) (4)
⎡x⎤
⇒ f (–x) = ⎨ −1 − ⎢ ⎥ + 0.5 Again, from (3), we have
⎪ ⎣p ⎦
h(x) = log (sin–1[x]) exists when;
⎪⎩ 0 , x = np
sin–1 [x] > 0 and – 1 ≤ [x] ≤ 1
⎛ ⎞ ⇒ [x] > 0 and –1 ≤ [x] ≤ 1
⎜ x(sin x + tan x ) ⎟
⇒ f (–x) = – ⎜ ⎟ , when x ≠ np ⇒ 0 < [x] ≤ 1 ⇒ [x] = 1
⎜ ⎡ x ⎤ + 0.5 ⎟ ⇒ x ∈ [1, 2) (5)
⎜⎝ ⎢ p ⎥ ⎟⎠
⎣ ⎦ ⇒ Domain of f (x) is [1, 2)
and f (–x) = 0, when x = np Now, for range,
Hence, f (x) is an odd function (if x ≠ np) and f (x) is an even we know, f (x) = sin–1(log[x]) + log(sin–1[x])
function (if x = np)
where x ∈ [1, 2) ⇒ [x] = 1
The correct option is (C) and (D)
\ Range of f (x) = sin–1(log 1) + log(sin–1 1)
126. Let us check for invertibility of f (x)
⎛p⎞
e x + e− x = sin–1(0) + log ⎜ ⎟
(A) one-one: we have, f (x) = ⎝ 2⎠
2 = log(p/2)
e2 x + 1
⇒  f ′(x) =
 , which is strictly increasing as e2x ⎧ p⎫
2e x ⇒ Range of f (x) = ⎨log ⎬
> 0 for all x. ⎩ 2⎭
Thus, f is one-one The correct option is (A) and (C)
(B) Onto; Let y = f (x) 128. (A) one-one:
2

e x + e− x f  (x) = 2 x − x
⇒  y = , where y is strictly monotonic 2

2 ⇒  f ′(x) = 2 x − x (2x – 1) · log2


Hence, range of f (x) = ( f (–∞), f (∞)) For f (x) to be one-one, it should be strictly increasing
⇒  range of f (x) = (–∞, ∞) or strictly decreasing.
So, range of f (x) = co-domain So, f ′(x) > 0
2

Hence, f (x) is one-one and onto ⇒  2 x − x (2x – 1) > 0, where 2x2 – x > 0 for all x
1
e2 x − 1 ⇒ 2x – 1 > 0 or x >
(C) To find f –1 : y = 2
2e x
 Thus, for given domain [1, ∞),   f (x) is always
⇒  e – 2e y – 1 = 0
2x x
­increasing. Hence, f is one-one
2 y ± 4 y2 + 4 (B) onto: As f (x) is strictly increasing
⇒  ex =
2 ⇒ Range f (x) ∈ [ f (1), f (∞))
⇒  x = log(y ± y2 + 1 ) ⇒ Range f (x) ∈ [1, ∞)
⇒  Range of f (x) = Co-domain of f (x), thus, f is onto.
⇒  f –1(y) = log ( y ± y 2 + 1) (C) Inverse:
Since, e f
−1
( x)
is always positive, so, neglecting ­negative As  f is one-one and onto, f –1 can be obtained.
sign. Let y = f (x)
Hence,  f –1(x) = log(x + x 2 + 1 ) ⇒  y = 2x2 – x
⇒  x2 – x = log2 y
The correct option is (A), (B) and (D)
⇒  x2 – x – log2 y = 0

Objective_Maths_JEE Main 2017_Ch 2.indd 53 01/01/2008 03:20:29


2.54  Chapter 2

1 ± 1 + 4 log 2 y ⎛ 2 ⎞ ⎛ 1994 ⎞
⇒ x = ⇒ f⎜ + f⎜ =1
2 ⎝ 1996 ⎟⎠ ⎝ 1996 ⎟⎠

1 + 1 + 4 log 2 y ⎛ 3 ⎞ ⎛ 1993 ⎞
⇒ f –1(y) = [as y > 0, ∀ x ∈ D] ⇒ f⎜ + f⎜ =1
2 ⎝ 1996 ⎟⎠ ⎝ 1996 ⎟⎠
The correct option is (A), (B) and (C) … … …
9x … … …
129. f (x) = x (1) ⎛ 997 ⎞ ⎛ 999 ⎞
9 +3 ⇒ f ⎜ + f⎜ =1
91 − x ⎝ 1996 ⎟⎠ ⎝ 1996 ⎟⎠
and, f (1 – x) = 1 − x
9 +3 ⎛ 998 ⎞ ⎛ 998 ⎞ ⎛ 998 ⎞ 1
⇒ f⎜ + f⎜ = 1 or f ⎜ =
9 ⎝ 1996 ⎟⎠ ⎝ 1996 ⎟⎠ ⎝ 1996 ⎟⎠ 2
9 x 9
⇒ f (1 – x) = = Adding all the above expressions, we get
9 9 + 3.9 x
x
+3 ⎛ 1 ⎞ ⎛ 2 ⎞ ⎛ 1995 ⎞
9 f ⎜ + f⎜ + ... + f ⎜
⎝ 1996 ⎟⎠ ⎝ 1996 ⎟⎠ ⎝ 1996 ⎟⎠
9
f  (1 – x) = (2) 1
3(3 + 9 x ) = (1 + 1 + 1 + ... + 997) +
2
1 1
Adding (1) and (2), we get = 997 + = 997
2 2
9x 9
(x) + f (1 – x) =
f  + The correct option is (A) and (D)
9 + 3 3(3 + 9 x )
x
130. We have f (n + 2) – f (n + 1)
3. 9 x + 9 3(9 x + 3) = (n + 2) ! = (n + 2) (n + 1) !
= =
3(9 + 3)
x
3(9 x + 3) = (n + 2) [ f (n + 1) – f (n)]
\ f (x) + f (1 – x) = 1 (3) ⇒ f (n + 2) = (n + 3) f (n + 1) – (n + 2) f (n)
1 2 3 998 \ P (x) = x + 3 and Q(x) = –x – 2
Now, putting x = , , , ... , in (3), we get
1996 1996 1996 1996 The correct option is (A) and (B)
⎛ 1 ⎞ ⎛ 1995 ⎞
f ⎜ + f⎜ =1
⎝ 1996 ⎟⎠ ⎝ 1996 ⎟⎠

Passage Based Questions


p
131. sin x is a periodic function with period 2p, therefore 133. Since |sin x| + |cos x| is a periodic function with period ,
2p p 2
sin ( [n] x) is a periodic function with period . therefore period of f (x) will be when k = 1.
[n] 2
But the period of f (x) is 2p (given). The correct option is (A)
px
2p 134. 3x + 3 – [3x + 3] has the period 1 and sin has the
\ = 2p ⇒ [n] = 1 ⇒ [n] = 1 ⇒ 1 ≤ n < 2 2
[n] 2p
period i.e., 4. Therefore, the period of f (x) is L.C.M.
The correct option is (A) p /2
132. f (x + T) = f (x) (1, 4) = 4.
⇒ cos (sin (x + T) + cos (cos (x + T) The correct option is (A)
= cos (sin x) + cos (cos x) p
135. The period of |sin x| + |cos x| and sin4x + cos4x is sin (sin x)
If x = 0, then cos (sin T) + cos (cos T) 2
1 + 2 cos x
⎛ p⎞ ⎛ p⎞ + sin (cos x) has period 2p. The function
= cos(0) + cos (1) = cos ⎜ cos ⎟ + cos ⎜ sin ⎟ sin x ( 2 + sec x )
⎝ 2 ⎠ ⎝ 2⎠ cos x
p can be written in a simplified form as = cot x, so it
On comparing, we get T = . has period p. sin x
2
The correct option is (A) The correct option is (D)

Objective_Maths_JEE Main 2017_Ch 2.indd 54 01/01/2008 03:20:34


Functions  2.55

2p 2p ⎛ sin x ⎞
136. The period of sin 5x is and that of cos 3x is . and, lim ⎜1 + =∞
p ⎝ 2 ⎟
2p 2p
5 3 x→ cos x⎠
2
As and do not have a common multiple, f (x) is
5 3 \ Range = (–∞, ∞)
non-periodic.
The correct option is (B)
The correct option is (C)
140. For f (x) to be defined,
137. We have,
(1) [2x2 – 3] = –1, 0, 1
ex + ef (x) = e ⇒ ef (x) = e – ex
⇒ –1 ≤ 2x2 – 3 < 2 ⇒ 2 ≤ 2x2 < 5
⇒ f (x) = log (e – ex)
5
For f (x) to be defined, e – ex > 0 ⇒ 1 ≤ x2 <
2
⇒ e1 > ex ⇒ x < 1
⎧ 1 ≤ x 2 ⇒ x ≤ −1 or x ≥ 1
\ Domain of f = (– ∞, 1) ⎪
⇒ ⎨ 2 5 5 5
Let y = log (e – ex) ⇒ ey = e – ex ⎪x < ⇒ − <x<
⇒ e = e – e
x y ⎩ 2 2 2
⇒ x = log (e – ey) 5 5
⇒ − < x ≤ −1 or 1 ≤ x < (1)
For x to be real, e – ey > 0 2 2
⇒ e1 > ey ⇒ y < 1 (2) x2 – 5x + 5 > 0
\ Range of f = (– ∞, 1) 5− 5 5+ 5
⇒ x < or x > (2)
The correct option is (B) 2 2
138. Since [x2 + 1] is an integer, (3) log1/2 (x2 – 5x + 5) > 0
\ sin (p[x2 + 1]) = 0 ⎛ 1⎞
0

sin (p [ x 2 + 1]) ⇒ x2 – 5x + 5 < ⎜ ⎟ ⇒ x2 – 5x + 5 < 1


⇒ f (x) = =0 ⎝ 2⎠
x4 + 1 ⇒ x2 – 5x + 4 < 0
Hence, Range of f = Rf  = {0} ⇒ 1 < x < 4 (3)
The correct option is (C) 5− 5
From Eqs. (1), (2) and (3), 1 ≤ x <
2
sin x The correct option is (D)
139. We have, f (x) = 1 +
cos 2 x
141. If f (x) ≥ 0, then x + f (x) = 2f (x)
cos 2 x (cos x ) + sin x ( 2 cos x sin x )
⇒ f ′(x) = or, f (x) = x
cos 4 x
\ f –1(x) = x, when f–1(x) ≥ 0 (1)
cos x (cos x + 2 sin x )
2 2
1 + sin 2 x
= = Also, when f (x) ≤ 0, x – f (x) = 2f (x)
cos 4 x cos3 x x
⇒ f ′(x) > 0. or, f (x) =
3
\ f (x) is increasing function. \ f –1(x) = 3x, when f –1(x) ≤ 0 (2)
⎛ sin x ⎞ Clearly, option (d) satisfies both (1) and (2)
lim ⎜1 + 2 ⎟
=–∞
−p ⎝ cos x⎠ The correct option is (D)
x→
2

Match the Column Type


142.     I.  f (x) is defined if | sin x | + sin x > 0 1 ⎛ 1⎞
−1

⇒ sin x > 0 ⇒ 2np < x < 2np + p ⇒ 1 + > ⎜ ⎟ , x > (– 1)5, x ≠ 0
x1 5 ⎝ 2 ⎠
\  Domain of f = (2np, (2n + 1) p) 1
The correct option is (B) ⇒  > 1, x > – 1 and x ≠ 0
x1 5
 II.  f (x) is defined if
⇒ 0 < x < 1 and x > – 1 ⇒ 0 < x < 1
⎛ 1 ⎞ 1
– log1/2 ⎜1 + 1 5 ⎟ – 1 > 0, 1 + 1 5 > 0, x ≠ 0 \  Domain ( f ) = (0, 1)
⎝ x ⎠ x
The correct option is (C)
⎛ 1 ⎞
⇒  log1/2 ⎜1 + 1 5 ⎟ < – 1, x1/5 + 1 > 0, x ≠ 0
⎝ x ⎠

Objective_Maths_JEE Main 2017_Ch 2.indd 55 01/01/2008 03:20:37


2.56  Chapter 2

III. f (x) is defined if For x to be real, (y + 3)2 + 4 (y – 1) (6y + 2) ≥ 0


–(log3 x)2 + 5 log3 x – 6 > 0 and x > 0 ⇒ 25y2 – 10y + 1 ≥ 0 i.e., (5y – 1)2 ≥ 0
⇒ (log3 x – 3) (2 – log3 x) > 0 and x > 0 which is true for all real y.
⇒ (log3 x – 2) (log3 x – 3) < 0 and x > 0 \  Range of f = (– ∞, ∞).
⇒ 2 < log3 x < 3 and x > 0 The correct option is (B)
⇒ 32 < x < 33 ⇒ 9 < x < 27   IV. For f (x) to be defined,
Domain of f = (9, 27).
4 − x2
The correct option is (D) > 0, 4 – x2 > 0 and 1 – x ≠ 0
1− x
x
IV. 
Domain of cot–1x is R and is defined if Since 4 − x 2 </ 0,
x − [x2 ]
2
x2 ≠ [x2]  (Qx2 ≥ [x2]) \  we have 1 – x > 0 and 4 – x2 > 0
⇒  x2 ≠ 0 or positive integer. ⇒  x < 1 and (x – 2) (x + 2) < 0
Hence, domain = R – { n : n ≥ 0, n ∈ I}. ⇒  x < 1 and – 2 < x < 2
The correct option is (A) ⇒  – 2 < x < 1
143. I.  f (x) is defined if 3x2 – 4x + 5 ≥ 0 \  Domain of f = (– 2, 1)
⎡ 4 5⎤ ⎡⎛ 2 ⎞ 11⎤
2

⇒ 3 ⎢ x 2 − x + ⎥ ≥ 0 ⇒ 3 ⎢⎜ x − ⎟ + ⎥ ≥ 0 ⎛ 4 − x2 ⎞
⎣ 3 3⎦ ⎢⎣⎝ 3⎠ 9 ⎥⎦ Since – ∞ < log ⎜ ⎟ <∞
⎝ 1− x ⎠
which is true for all real x
\  Domain ( f ) = (–∞, ∞) ⎡ ⎛ 4 − x2 ⎞ ⎤
⇒  – 1 ≤ sin ⎢log ⎜ ⎟⎥ ≤ 1
Let y = 3x 2 − 4 x + 5 ⎢⎣ ⎝ 1 − x ⎠ ⎥⎦
⇒  y2 = 3x2 – 4x + 5 i.e., 3x2 – 4x + (5 – y2) = 0 \  Range of f = [– 1, 1].
11 The correct option is (D)
For x to be real, 16 – 12 (5 – y2) ≥ 0 ⇒ y ≥
3 144.     I.  We have,

\  Range of y = ⎢
⎡ 11 ⎞
, ∞⎟
y = log 5 { 2 (sin x − cos x ) + 3}
⎢⎣ 3 ⎠
⎧ ⎛ p⎞ ⎫
The correct option is (C) = log 5 ⎨2 sin ⎜ x − ⎟ + 3⎬
⎩ ⎝ 4⎠ ⎭
 II.  f (x) is defined if 3x2 – 4x + 5 > 0
which is defined for values of x such that
⎡ 4 5⎤ ⎡⎛ 2 ⎞ 11⎤
2
p⎞
⇒ 3 ⎢ x 2 − x + ⎥ > 0 ⇒ 3 ⎢⎜ x − ⎟ + ⎥ > 0, ⎛
2 sin ⎜ p − ⎟ + 3 > 0
⎣ 3 3⎦ ⎢⎣⎝ 3⎠ 9 ⎥⎦ ⎝ 4⎠
which is true for all real x. which is true " x ∈ R
\  Domain ( f ) = (– ∞, ∞) Now, we have
Let, y = loge (3x2 – 4x + 5) ⇒ ey = 3x2 – 4x + 5 ⎛ p⎞
–2 ≤ 2 sin ⎜ x − ⎟ ≤ 2
⇒ 3x2 – 4x + (5 – ey ) = 0 ⎝ 4⎠
For x to be real, ⎛ p⎞
11 i.e., 1 ≤ 2 sin ⎜ x − ⎟ + 3 ≤ 5
16 – 12 (5 – ey) ≥ 0 ⇒ 12 ey ≥ 44 ⇒ ey ≥ ⎝ 4⎠
3
⇒  y ≥ loge 11 ⎧ ⎛ p⎞ ⎫
3 i.e., 0 ≤ log 5 ⎨2 sin ⎜ x − ⎟ + 3⎬ ≤ log 5 5
⎩ ⎝ 4⎠ ⎭
⎡ 11 ⎞ i.e., 0 ≤ y ≤ 2
Range of f = ⎢log e , ∞⎟
⎣ 3 ⎠ Hence, the range is y ∈ [0, 2]
The correct option is (A) The correct option is (C)
f (x) is defined if x2 + x – 6 ≠ 0
III.   II.  The function is defined for values of x such that
i.e., (x + 3) (x – 2) ≠ 0 i.e., x ≠ – 3, 2 2 – log (16 sin2x + 1) > 0
5
\  Domain ( f ) = (– ∞, ∞)\{– 3, 2}
Also, we have
x 2 − 3x + 2 2 – log 5 (16 sin2x + 1) ≤ 2
Let y = 2
x + x −6 [Q log (16 sin2x + 1) ≥ 0]
5
⇒  x2y + xy – 6y = x2 – 3x + 2 Together, we have
⇒  x2 (y – 1) + x (y + 3) – (6y + 2) = 0 0 < 2 – log (16 sin2 x + 1) ≤ 2
5

Objective_Maths_JEE Main 2017_Ch 2.indd 56 01/01/2008 03:20:41


Functions  2.57

i.e., –∞ < log2{2 – log 5 (16 sin2 x + 1)} ≤ log22   IV. We have,
i.e., –∞ < y ≤ 1 e x − e− x e2 x + 1 − 2 2
y= x −x
= = 1−
Hence, the range is y ∈(–∞, 1] e +e e2 x + 1 1 + e2 x
The correct option is (B) Now, we have " x ≥ 0, 2 ≤ 1 + e2x < ∞
III. 
We have, 1 1
i.e., ≥ >0
ex − ex 2 1 + e2 x
y = x = 0, x < 0
e + e− x −2
i.e.,  −1 ≤ <0
e x − e− x 1 − e− 2x 1 + e2 x
= x = ,x≥0 2
e + ex 2 i.e.,  −1 + 1 ≤ 1 − < 0 +1
Now, we have " x ≥ 0, 0 < e–2x ≤ 1 1 + e2 x
Hence, the range is y ∈ [0, 1)
i.e.,  – 1 ≤ – e – 2x < 0 i.e., 0 ≤ 1 – e–2x < 1
The correct option is (A)
1 − e− 2x 1 1
i.e., 0 ≤ < i.e., 0 ≤ y <
2 2 2
⎡ 1⎞
Hence, the range is y ∈ ⎢0, ⎟
⎣ 2⎠
The correct option is (D)

Assertion-Reasoning Type
145. We have, 147. We have,
⎛ 1⎞
3 f (x) – f ⎜ ⎟ = 4 ln x g(x) = − x 2 + 4 x − 3 and
⎝ x⎠
Putting 1/x in place of x, we have ⎛p ⎛p ⎞⎞
3 f (1/x) – f (x) = – 4 ln x h(x) = sin ⎜ sin ⎜ ( x −1)⎟ ⎟
⎝2 ⎝ 2 ⎠⎠
Solving the above equations, we have
Since – x2 + 4x – 3 = 1 – (x – 2)2, maximum value of g =
f (x) = ln x ⇒ f (ex) = x
g(2) = 1.
Hence, required area is
Also, g(1) = 0
1
1
⎡ x2 ⎤ Therefore, minimum value of g = g(1) = 0
∫ x dx = ⎢ ⎥ = 0
−1 ⎣ 2 ⎦ −1 Now, h(2) = 1 and h(1) = 0
The correct option is (A) Hence, maximum and minimum values of both g and h are
attained at 2 and 1, respectively. Further, g and h are both
ax
146. Given, f (x) = (1) continous in [1, 2] Hence, Range of f = [ f (1), f (2)] = [0, 2]
ax + a
The correct option is (A)
a1 − x a 148. Domain of g(x): g(x) is defined if
Now, f (1 – x) = 1− x
= (2)
a + a a + ax 3 – x ≥ 0 and (x – 1) (x – 2) (x – 3) ≠ 0
From (1) and (2), we have f (x) + f (1 – x) = 1 (3) ⇒ x ≤ 3 and x ≠ 1, 2, 3
⎛ r ⎞ ⎛ 2n − r ⎞ \ Domain of g (x) = (–∞, 3) – {1, 2, 3}
⇒ f⎜ ⎟+ f⎜ =1
⎝ 2n ⎠ ⎝ 2n ⎠⎟ Domain of h(x):
2n −1
⎛ r ⎞
2n −1
⎛ 2n − r ⎞ ⎡ 3x − 2 ⎤ ⎡ 3x − 2 ⎤
h(x) = sin − 1 ⎢ ⎥ ⇒ –1 ≤ ⎢ 2 ⎥ ≤ 1
⇒ ∑ f ⎜⎝ 2n ⎟⎠ + ∑ f ⎜⎝
r =1 r =1 2n ⎠
⎟ = 2n – 1

⎣ 2 ⎦ ⎣ ⎦
Case I:
2n −1 2n −1
⎛ r ⎞ ⎛ t ⎞
⇒ ∑ f ⎜⎝ 2n ⎟⎠ + ∑ f ⎜⎝ 2n ⎟⎠ = 2n – 1 ⎡ 3x − 2 ⎤
If ⎢ = – 1 ⇒ –1 ≤
3x − 2
<0
r =1 r =1 ⎥
⎣ 2 ⎦ 2
 (Putting 2n – r = t)
⇒ –2 ≤ 3x – 2 < 0
2n −1
⎛ r ⎞ 2
Hence, 2 ∑ f ⎜ ⎟ = 2n – 1 \ 0 ≤ x < (1)
r =1 ⎝ 2n ⎠ 3
The correct option is (A)

Objective_Maths_JEE Main 2017_Ch 2.indd 57 01/01/2008 03:20:45


2.58  Chapter 2

Case II: \ Domain of f = [0, 2) – {1}


⎡ 3x − 2 ⎤ The correct option is (A)
If ⎢ ⎥ =0 149. Since, g(x) is a function whose graph is the reflection of the
⎣ 2 ⎦
3x − 2 graph of f (x) in the line y = x
⇒ 0 ≤ < 1 ⇒ 0 ≤ 3x – 2 < 2 ⇒ g(x) is the inverse of f (x) by definition
2
⇒ 2 ≤ 3x < 4 i.e., g(x) = f –1(x)
2 4 Now, let y = f (x) = (x + 1)2, " x ≥ – 1  {given}
\ ≤ x < (2)
3 3 ⇒ (x + 1) = ± y,y≥0
Case III:
⇒ x = –1 ± y
⎡ 3x − 2 ⎤ 3x − 2
⇒ either x = –1 + y  or x = –1 –
If ⎢ ⎥=1⇒1≤ < 2 ⇒ 2 ≤ 3x – 2 < 4 y
⎣ 2 ⎦ 2
⇒ f (y) = –1
y – 1   Not possible, Q x ≥ –1  (given)
4
\ ≤ x < 2 (3)
3 \ f –1(x) = x – 1 = g(x), x ≥ 0
Thus, from (1), (2) and (3), we have
The correct option is (A)
Domain of h(x) = [0, 2)

Previous Year’s Questions


150. Key Idea : Period of the functions sin θ and cos θ is 2π. 153. Clearly the function is both one-to-one and onto
1 − cos 2q 1 1 Because if n is odd, values are set of all non-negative
Since, sin2θ = = − cos 2q
2 2 2 integers and if n is an even, values are set of all negative
2 p integers.
∴  Period of sin2θ = =p
2 The correct option is (C)
The correct option is (B) 154. 4−x2 ≠ 0
151. Key Idea : Domain of inverse function sin−1 x = [−1, l] and ⇒ x ≠ ±2
⎡ p p⎤ And, x3−x > 0
range of sin −1 x = ⎢ − , ⎥ .
⎣ 2 2⎦ ⇒ x( x + 1)( x − 1) > 0 .
Since, domain of sin−1 x = [− l, 1] The correct option is (D)
⎛ x⎞ 7− x
\ −1 ≤ log3 ⎜ ⎟ ≤ 1 155. f (x) = Px − 3
⎝ 3⎠

x Now, 7 − x ≥ 0 ⇒ x ≤ 7
⇒ 3−1 ≤ ≤ 3 And, x − 3 ≥ 0 ⇒ x ≥ 3,
3
Again, 7 − x ≥ x − 3 ⇒ x ≤ 5
⇒ 1 ≤ x ≤ 9
⎡ ⎛ x⎞ ⎤ ⇒ 3 ≤ x ≤ 5 ⇒ x = 3, 4, 5
∴  Domain of sin −1 ⎢log3 ⎜ ⎟ ⎥ is [l, 9]. ⇒ Range is {1, 2, 3}.
⎣ ⎝ 3⎠ ⎦
The correct option is (A)
The correct option is (A)

152. Q f (x) = sin4 x + cos4x 156. −2 ≤ sin x − 3 cos x ≤ 2 ⇒ −1 ≤ sin x − 3 cos x + 1 ≤ 3


= (sin2 x + cos2x)2− 2 sin2 x cos2x ⇒ range of f (x) is [−1, 3].
1 Hence the range set S is [−1, 3].
= 1 − ( 2 sin x cos x ) 2
2 The correct option is (D)
1 3 1 157. If the curve y = f (x) is symmetric about the line x = 2 then
= 1 − (sin 2 x ) 2 = + cos 4 x
2 4 4 f  (2 + x) = f (2 − x).
Q cos x is periodic with period 2π. The correct option is (B)
2p p
∴  The period of function f (x) = = . 1 58. Since 9 − x2 > 0 and −1 ≤ x − 3 ≤ 1 ⇒ x ∈[2, 3)
4 2
The correct option is (B)
The correct option is (B)

Objective_Maths_JEE Main 2017_Ch 2.indd 58 01/01/2008 03:20:49


Functions  2.59

∴ It is one-one
⎛ 2x ⎞
159. Given f ( x ) = tan −1 ⎜ for x ∈ (−1, 1)
⎝ 1 − x 2 ⎟⎠ Clearly, f (x) is a continuous function and also increasing on
R,
⎛ p p⎞
Clearly range of f ( x ) = ⎜ − , ⎟ Lt f (x) = −∞ and Lt f (x) = ∞
⎝ 2 2⎠
x→−∞ x→∞

⎛ p p⎞ ∴ f (x) takes every value between −∞ and ∞.


∴  co-domain of function = B = ⎜ − , ⎟
⎝ 2 2⎠ Thus, f (x) is onto function.
The correct option is (D) The correct option is (C)
160. f (2a – x) = f (a− (x−a)) = f (A)  f (x−a) −f (0) f (x) 1
164. ⇒| x | − x > 0 ⇒| x |> x ⇒ x is negative
= −f (x) [∵ x = 0, y = 0 in the given functional equation f (0) | x | −x
= f  2 (0) − f  2(a) x ∈ (− ∞, 0)
⇒ f   (A)  = 0 ⇒ f  (A)  = 0]. The correct option is (B)
2

The correct option is (A) 165. f ( x ) + 2 f (1 / x ) = 3 x  (1)


x
161. f (x) is defined if −1 ≤ − 1 ≤ 1 and cos x > 0 1
2 x→ ⇒ f (1 / x ) + 2 f ( x ) = 3 / x (2)
x
p p
i.e. if 0 ≤ x ≤ 4 and − < x < ⎛3 ⎞
2 2 f ( x ) + 2 ⎜ − 2 f ( x )⎟ = 3 x
⎝x ⎠
⎡ p⎞
\ x ∈ ⎢ 0, ⎟ 6
⎣ 2⎠ ⇒ 3 f ( x ) = − 3x
x
The correct option is (D)
2
162. Function is increasing ⇒ f ( x ) = −x
x
y −3 2
So, x = = g( y) For S, f ( x ) = f ( − x ) ⇒ −x=0
4
x
The correct option is (D)
⇒ x = ± 2
1 63. Given f (x) = x3 + 5x + 1. The correct option is (D)
The differential f  ′ (x) = 3x2 + 5 > 0, ∀x ∈ R
∴ f  (x) is strictly increasing function

Objective_Maths_JEE Main 2017_Ch 2.indd 59 01/01/2008 03:20:52


Complex Numbers 3.1

CHaPtER

3 Complex Numbers

Chapter Highlights
Imaginary numbers, Integral powers of i, Complex numbers, Conjugate of a complex number, Modulus of a
complex number, Square roots of a complex number, Argand plane and geometrical representation of complex
numbers, Polar form of a complex number, Particular cases of polar form, Eulerian representation of a complex
number, Logarithm of a complex number, Vectorial representation of a complex number, Roots of a complex
number, Geometry of complex numbers.

IMAgINARY NUMBERS TRICk(S) FOR PROBLEM SOLvINg


Square root of a negative number is called an imaginary
 For any n ∈ N
number.
⎡ 1, when n is even⎤
1. i2n = (i2)n = (–1)n = ⎢ ⎥
Illustration 1 ⎣ − 1, when n is odd ⎦
−1, −4 , −7 , −18 , and so on are all imaginary ⎡ i, when n is even⎤
2. i2n+1 = (i2n) i = (–1)ni = ⎢ ⎥.
numbers. ⎣ − i, when n is odd ⎦
−1 is denoted by the Greek letter i (pronounced as  The sum of four consecutive powers of i is zero. For
example,
iota), where i is a number such that i2 = –1. Thus,
i10 + i11 + i13 + i14 = 0
−2 = 2i , −3 = 3i , −4 = 2i.
 Also, for any n ∈ N, the value of i–n is found out by writing
1
this as n and solving i n.
notE i
 Thus, any integral power of i can be expressed in terms of
±1 or ± i.
If a < 0, then a = |a| i.
The symbol ‘i’ was first introduced by the famous
mathematician, Leonhard Euler (1707–1783) in 1748, For any two real numbers a and b, a × b = ab is true
possibly because ‘i’ is the first letter of the Latin word only when at least one of a and b is either zero or positive. If
‘imaginarius’. both a and b are positive real numbers, then the calculation
− a × − b = ( − a) ( − b ) = ab is wrong.
The correct calculation is
INTEgRAL POwERS OF I
− a × − b = ( − 1 a )( − 1 b )
We have
i= −1 , i2 = –1. = (i a ) (i b )
Therefore,
= i2( a × b ) = (–1) ( ab )
i3 = i2 × i = (–1) × i = –i,
i4 = i2 × i2 = (–1) × (–1) = 1 = – ab

Objective_Maths_JEE Main 2017_Ch 3.indd 1 01/01/2008 03:25:42


3.2 Chapter 3

Thus, the calculation −2 × −3 = ( −2) × ( −3) ⎡ i (1 − i ) ⎤


= (1 + i) ⎢ ⎥
= 6 is wrong. ⎣ 1− i ⎦
= (1 + i) i = –1 + i
The correct result is −2 × −3 = (i 2 ) (i 3 )
4. The least positive integer n for which
= i2( 2 × 3) n
⎛1+ i⎞ 2 −1 1 + x 2
=– 6 ⎜⎝ 1 − i ⎟⎠ = sin , where x > 0, is
p 2x
(A) 1 (B) 2
SOLvEd EXAMPLES (C) 4 (D) None of these

⎛1+ i ⎞ ⎛1− i ⎞
8 8 Solution: (C)
1. The value of ⎜ + is equal to
⎝ 2 ⎟⎠ ⎜⎝ 2 ⎟⎠ 1 + x2
–1
For sin 2 x to be defined,
(A) 4 (B) 6 (C) 8 (D) 2
1+ x 2
Solution: (D) –1 ≤ ≤1
2x
8 8
⎛1+ i ⎞ ⎛1− i ⎞ 1 + x2
We have, ⎜ + or ≤1
⎝ 2 ⎟⎠ ⎜⎝ 2 ⎟⎠ 2x
p p⎤ ⎡ p
8
p⎤
8 or 1 + x2 ≤ 2x

= ⎢cos + i sin ⎥ + ⎢cos − i sin ⎥ or (1 – x)2 ≤ 0 or x = 1
⎣ 4 4⎦ ⎣ 4 4⎦
= cos 2p + i sin 2p + cos 2p – i sin 2p Now,
n n
⎛1+ i ⎞ ⎛ (1 + i ) 2 ⎞ n
= 2 cos 2p = 2 (1) = 2 [By De-Moivre’s theorem] ⎜⎝ 1 − i ⎟⎠ = 1 ⇒ ⎜ 2 ⎟ =1⇒i =1
⎝ ⎠
2. i − −i is equal to
1 COMPLEX NUMBERS
(A) i 2 (B) (C) 0 (D) – i 2
i 2 An expression of the form x + iy, where x and y are real
Solution: (A, D)
numbers and i = −1 , is called a complex number. It is
1
We have, i=0+i⋅1= (0 + 2i) usually denoted by z, i.e.,
2
1 1 z = x + iy
=(1 + i2 + 2 ⋅ 1 ⋅ i) = (1 + i)2
2 2 x is called the real part and y the imaginary part of z and
1 may be denoted by Re (z) and Im (z) respectively.
\ i =± (1 + i) If y = 0, z is called purely real and if x = 0, z is called
2
purely imaginary.
1
\ −i = ± (1 – i) The set of complex numbers is denoted by C.
2 If x = 0 and y = 0, the complex number reduces to
Hence, 0 + i ⋅ 0 = 0, which is called the zero complex number.
1
i − −i = ± [(1 + i) – (1 – i)] = ± 2i
2 i M P o R ta n t P o i n t S
13
3. The value of the sum ∑ (i n + i n +1 ), where i = −1,  We observe that the system of complex numbers includes
equals n =1 the system of real numbers, i.e., R ⊂ C.
(A) i (B) i – 1 (C) –i (D) 0  Every real number is a complex number.
 0 is both purely real and purely imaginary number.
Solution: (B)  A complex number is an imaginary number if and only if its
13 13
imaginary part is non-zero. Here, real part may or may not be
∑ (i n + i n +1 ) = ∑ i n (1 + i) zero. 4 + 3i is an imaginary number but not purely imaginary.
n =1 n =1  All purely imaginary numbers except zero are imaginary
⎡ i (1 − i13 ) ⎤ numbers but an imaginary number may or may not be
= (1 + i) ⎢ ⎥
⎣ 1− i ⎦ purely imaginary.

Objective_Maths_JEE Main 2017_Ch 3.indd 2 01/01/2008 03:25:47


Complex Numbers  3.3

EQUALITY OF COMPLEX NUMBERS x 1x 2 + y 1y 2 + i ( x 2 y 1 − x 1y 2 )


= 
Two complex numbers are said to be equal if and only if x 22 + y 22
their real parts and imaginary parts are separately equal. x1 x2 + y1 y2 i ( x2 y1 − x1 y2 )
= +
i.e., a + ib = c + id x22 + y22 x22 + y22

⇔ a = c and b = d. Multiplicative Inverse of a Non-zero
i.e., z1 = z2 Complex Number
⇔ Re (z1) = Re (z2) and Im (z1) = Im (z2) Multiplicative inverse of a non-zero complex number z = a
+ ib is defined as
1 1 1 a − ib a − ib
caution z–1 = = = × = 2
z a + ib a + ib a − ib a + b2 
Inequality relation does not hold good in case of complex a b
numbers having non-zero imaginary parts. For example, the = 2 2
−i 2
statement 8 + 5i > 4 + 2i makes no sense. a +b a + b2 
Re (z ) [ − Im (z )]
i.e., z– 1 = +i
| z |2 | z |2
Algebra of Complex Numbers 
Addition Solved Examples
For two complex numbers z1 = a1 + ib1 and z2 = a2 + ib2,
their sum is defined as 5. The number of integral solutions of the equation
z = z1 + z2 = (a1 + a2) + i (b1 + b2) (1 – i)x = 2x are
(A) 1 (B) 2
Subtraction (C)  0 (D)  None of these
For two complex numbers z1 = a1 + ib1 and z2 = a2 + ib2, the Solution: (C)
subtraction of z2 from z1 is defined as
Let k be an integral solution of the given equation.
z1 – z2 = z1 + (– z2) = (a1 – a2) + i (b1 – b2)
( 2)
k
Then, (1 – i)k = 2k ⇒ | (1 – i)k | = 2k ⇒ = 2k,
Multiplication which is possible only if k = 0.
Multiplication of two complex numbers z1 = a + ib and
6. Let z1 and z2 be two non real complex cube roots of
z2 = c + id is defined as
unity and |z – z1|2 + |z – z2|2 = l be the equation of a
z1z2 = (ac – bd) + i(ad + bc) circle with z1, z2 as ends of a diameter, then the value
of l is
Trick(s) for Problem Solving (A) 4 (B) 3 (C) 2 (D)  2
Solution: (B)
The product of complex numbers can be easily computed if
we actually carry out the multiplication as given below:
We have,
(a + ib) (c + id) = ac + iad + ibc + i2bd |z – w |2 + |z – w2|2 = l
= ac + i (ad + bc) – bd  (∵ i2 = − 1) ⇒ l = |w – w2|2 = |w2 + w4 – 2w3|
= (ac – bd) + i (ad + bc)
= |w2 + w – 2| = |– 1 – 2| = 3

Division
Conjugate of a Complex Number
Division of two complex numbers
z1 = x1 + iy1 and z2 = x2 + iy2, Conjugate of a complex number z = a + ib is defined as
where x2 + iy2 ≠ 0, is defined as z = a – ib.
z1 x + iy 1 ( x + iy1 ) ( x2 − iy2 )
= 1 = 1 For example, 4 + 5i = 4 – 5i and 4 − 5i = 4 + 5i.
z2 x 2 + iy 2 ( x2 + iy2 ) ( x2 − iy2 )


Objective_Maths_JEE Main 2017_Ch 3.indd 3 01/01/2008 03:25:50


3.4 Chapter 3

where
i M P o R ta n t P o i n t S ac + bd
A=
c2 + d 2
Geometrically, the conjugate of z is the reflection or point and
image of z in the real axis.
bc − ad
B= .
Y c2 + d 2
Imaginary axis

TRICk(S) FOR PROBLEM SOLvINg


P(z)
a + ib
To put the complex number in the form A + iB we
c + id
θ should multiply the numerator and the denominator by the
O X
–θ conjugate of the denominator.

Q(z ) MOdULUS OF A COMPLEX NUMBER


Modulus of a complex number z = a + ib, denoted as mod
(z) or |z|, is defined as
Properties of Conjugate
1. (z ) = z |z| = a2 + b 2 , where a = Re (z), b = Im (z).
2. z = z if and only if z is purely real
Sometimes, |z| is called absolute value of z. Note that |z| ≥ 0.
3. z = – z if and only if z is purely imaginary
4. z + z = 2 Re (z) and z – z = 2i Im (z) For example, if z = 3 + 2i, then |z| = 32 + 2 2 = 13.
5. z1 + z2 = z1 + z2
Properties of Modulus
6. z1 − z2 = z1 − z2
1. |z| ≥ 0 and |z| = 0 if and only if z = 0, i.e., x = 0, y = 0
7. z1 z2 = z1 ⋅ z2
2. |z| = | z | = |– z| = | − z | .
⎛z ⎞ z 3. z z = |z|2
8. ⎜ 1 ⎟ = 1 , z2 ≠ 0
⎝ z2 ⎠ z2 4. –|z| ≤ Re (z) ≤ |z| and – |z| ≤ Im (z) ≤ |z|
9. If z = f (z1), then z = f (z1 ) 5. |zn| = |z|n

( )
10. z n = ( z ) n
6. |z1z2| = |z1| |z2|

11. z1 z2 + z1 z2 = 2 Re ( z1 z2) = 2 Re (z1 z2 ) z1 |z |


7. = 1
z2 | z2 |
a + ib
Method of Writing the Complex Number 8. |z1 ± z2| ≤ |z1| + |z2|
in the form A + iB c + id
9. |z1 – z2| ≥ |z1| – |z2|
We have, 10. |z1 + z2|2 + |z1 – z2|2 = 2 (|z1|2 + |z2|2)
a + ib (a + ib ) ( c − id )
=
c + id (c + id ) (c − id ) 11. |z1 + z2|2 = |z1|2 + |z2|2 + 2 Re (z1 z )
2
2 2 2
[Multiplying the Nu. and the Dn. by 12. |z1 – z2| = |z1| + |z2| – 2 Re (z1 z )
2
the conjugate of the Dn.] 2 2 2
13. |z1 + z2| = |z1| + |z2|
( ac + bd ) + i (bc − ad )
= z1 ⎛z ⎞
c2 + d 2 ⇒ is purely imaginary or Re ⎜ 1 ⎟ = 0
ac + bd bc − ad z2 ⎝ z2 ⎠
= 2 +i 2
c + d2 c + d2
= A + iB,

Objective_Maths_JEE Main 2017_Ch 3.indd 4 01/01/2008 03:25:55


Complex Numbers 3.5

z +1 1
i M P o R ta n t P o i n t S ⇒ =
z −1 iy
Geometrically |z| represents the distance of point P from 2 z 1 + iy
the origin. i.e., |z| = OP
⇒ = (by componendo and dividendo)
2 1 − iy
Y
1 + iy 1 + y2
⇒ z= ⇒ |z| = =1
P(z) 1 − iy 1 + y2

9. If |z – i| < 1, then |z + 12 – 6i|


(A) <14 (B) <16
(C) >14 (D) =14
X Solution: (A)
O M
Given, |z – i| < 1
Now, |z + 12 – 6i| = | (z – i) + (12 – 5i)|
TRICk(S) FOR PROBLEM SOLvINg ≤ |z – i| + |12 – 5i|
Most of the complex equations are solved using the property
zz = |z|2. (∵ |z1 + z2| ≤ |z1| + |z2|)
< 1 + 13 = 14
Hence |z + 12 – 6i| < 14.
SOLvEd EXAMPLES
10. The maximum value of |z| when z satisfies the condi-
7. The solution of the equation |z| – z = 1 + 2i is 2
tion z + = 2 is
3 3 z
(A) – 2i (B) + 2i
2 2 (A) 3–1 (B) 3 +1
3
(C) 2 – i (D) None of these (C) 3 (D) 2+ 3
2
Solution: (A) Solution: (B)
We have, |z| – z = 1 + 2i 2 2 2 2
We have, |z| = z + − ≤ z+ + .
z z z | z|
⇒ x 2 + y 2 – (x + iy) = 1 + 2i, 2
⇒ |z| ≤ 2 + ⇒ |z|2 ≤ 2|z| + 2
where z = x + iy |z |
⇒ |z|2 – 2|z| + 1 ≤ 1 + 2 ⇒ (|z| – 1)2 ≤ 3
⇒ x 2 + y 2 – x = 1 and y = – 2
⇒ – 3 ≤ |z| – 1 ≤
3 ⇒ 1 – 3 ≤ |z| ≤ 1 + 3
[Comparing real and imaginary parts]
That is, the maximum value of |z| is 1 + 3.
3
⇒ x= and y = – 2.
2 11. If |z| = Max. {|z – 1|, |z + 1|}, then
3 1
\ The solution of the given equation is – 2i. (A) |z + z | = (B) z + z = 1
2 2
z −1
8. If is purely imaginary, then (C) |z + z | = 1 (D) None of these
z +1
(A) |z| > 1 (B) |z| < 1 Solution: (C)
(C) |z| = 1 (D) None of these We have, |z| = |z – 1|
Solution: (C) ⇒ |z|2 = |z – 1|2 ⇒ z z = (z – 1) ( z – 1)
z −1
Let = iy, where y is real ⇒ zz = zz – z –z + 1 ⇒ z +z = 1
z +1

Objective_Maths_JEE Main 2017_Ch 3.indd 5 01/01/2008 03:26:00


3.6  Chapter 3

Also, |z| = |z + 1| ⇒ |z|2 = |z + 1|2 From Eq. (2), we can determine the sign of xy. If xy > 0,
then x and y will have same sign. Thus,
⇒ z z = (z + 1) ( z + 1) = z z + z + z + 1
⇒ z + z = –1, ⎡ ⎛ 2
a + b2 + a⎞ ⎛ a2 + b 2 − a ⎞ ⎤
a + ib = ± ⎢ ⎜ ⎟ +i ⎜ ⎟ ⎥⎥
\ |z + z | = 1 ⎢ ⎜ 2 ⎟⎠ ⎜⎝ 2 ⎟⎠
⎢⎣ ⎝ ⎥⎦
z −2 If xy < 0, then
12. If (z ≠ –2) is purely imaginary then |z| is equal to
z +2
(A) 1 (B) 2 (C) 3 (D) 4
⎡ ⎛ 2
a + b2 + a ⎞ ⎛ a2 + b2 − a ⎞ ⎤
a + ib = ± ⎢ ⎜ ⎟ −i ⎜ ⎟ ⎥⎥
⎢ ⎜ 2 ⎟⎠ ⎜⎝ 2 ⎟⎠
Solution: (B) ⎢⎣ ⎝ ⎥⎦
Let z = x + iy
z−2 x + iy − 2 ( x − 2) + iy Trick(s) for Problem Solving
Then, = =
z+2 x + iy + 2 ( x + 2) + iy  Square roots of z = a + ib are:
[( x − 2) + iy ][( x + 2) − iy ] ⎡ |z| + a |z| − a ⎤
= ± ⎢ +i
( x + 2) 2 + y 2 ⎥ for b > 0 and
 ⎣ 2 2 ⎦
( x 2 + y 2 − 4) + i ( 4 y )
= ⎡ |z| + a |z| − a ⎤
( x + 2) 2 + y 2 ± ⎢ −i ⎥ for b < 0
 ⎣ 2 2 ⎦
z −2
Since is purely imaginary, ⎧ ⎫
z +2 a + tb + a − tb = ± 2⎨ a2 + b2 + a ⎬
⎩ ⎭
\ x2 + y2 – 4 = 0 where b > 0

⇒ x2 + y2 = 4 ⇒ |z|2 = 4 ⇒ |z| = 2.


⇒ z+ z =± 2 { z +a }
where lm(z) > 0
⎧ ⎫
Also, a + tb − a − tb = ± 2⎨ a2 + b2 − a ⎬ t
Square Roots of a Complex Number ⎩ ⎭
where b > 0
Let z = a + ib and let the square root of z be the complex
number x + iy. Then
⇒ z− z =± 2 { }
z − a t
where b > 0
a + ib = x + iy
or (a + ib) = (x + iy)2 = (x2 – y2) + (2xy) i
Solved examples
Equating real and imaginary part, we get
a = x2 – y2(1) 13. If 3 a − ib = x – iy, then 3 a + ib =
and b = 2xy (2) (A) x + iy (B)  x – iy
(C) y + ix (D)  y – ix
Now, x2 + y2 = ( x 2 − y 2 )2 + 4 x 2 y 2 
Solution: (A)
2 2
= a + b (3) We have, 3
a − ib = x – iy
Solving the equations (1) and (3), we get
⇒ a – ib = (x – iy)3 = x3 – 3x2 ⋅ iy + 3x (iy)2 – (iy)3
⎛ a2 + b 2 + a ⎞ = (x3 – 3xy2) – i (3x2y – y3)
x = ± ⎜ ⎟
⎜⎝ 2 ⎟⎠ \ a + ib = (x3 – 3xy2) + i (3x2y – y3)

= x3 + 3x2 ⋅ (iy) + 3x (iy)2 + (iy)3
⎛ a2 + b2 − a ⎞
and y=± ⎜ ⎟ = (x + iy)3
⎜⎝ 2 ⎟⎠
 \ 3 a + ib = x + iy.

Objective_Maths_JEE Main 2017_Ch 3.indd 6 01/01/2008 03:26:04


Complex Numbers  3.7

14. The complex number z satisfying the equations |z – i| = 1


or z2 = = – i
|z + 1| = 1 is i
(A) 0 (B) 1 + i Now, z = i ⇒ |z| = |i| = 1
(C) –1 + i (D)  1–i
and z2 = – i ⇒ |z2| = | –i|
Solution:  (A, C)
⇒ |z|2 = 1 ⇒ |z| = 1
Let z = x + iy. Then,
Thus, in both cases |z| = 1.
|(x + iy) – i| = |(x + iy) + 1| = 1
17. The greatest value of |z + 1| if |z + 4| ≤ 3 is
or x 2 + ( y − 1)2 = ( x + 1) + y 2 = 1 (A) 4 (B) 5
(C)  6 (D)  None of these
\ x2 + y2 – 2y + 1 = x2 + y2 + 2x + 1
i.e., x = – y (1) Solution: (C)
and x2 + y2 – 2y + 1 = 1 (2) We have,
|z + 1| = |z + 4 – 3| = |(z + 4) + (– 3)|
From Eq. (1) and (2), x2 + x2 + 2x = 0; or x (x + 1) = 0
≤ |z + 4| + |– 3| = |z + 4| + 3
\ x = 0, –1;
≤ 3 + 3 = 6  (∵ |z + 4| ≤ 3)
\ y = 0, 1
Hence, the greatest value of |z + 1| is 6.
\ z = x + iy = 0, –1 + i.
15. The complex number z satisfying the equations Argand Plane and Geometrical
|z| – 4 = |z – i| – |z + 5i| = 0, is Representation of Complex Numbers
(A)  3 – i (B) 2 3 – 2i
Let O be the origin and OX and OY be the x-axis and y-axis
(C)  – 2 3 – 2i (D) 0 respectively. Then, any complex number z = x + iy = (x, y)
Solution:  (B, C) may be represented by a unique point P whose coordinates
We have two equations are (x, y).
The representation of complex numbers as points in a
|z| – 4 = 0 and |z – i| – |z + 5i| = 0 plane forms an Argand diagram.
Putting z = x + iy, these equations become The plane on which complex numbers are repre-
sented is known as the complex plane or Argand’s plane
|x + iy| = 4 i.e., x2 + y2 = 16 (1) or Gaussian plane. The x-axis is called the real axis and
and |x + iy – i| = |x + iy + 5i| y-axis the imaginary axis.
The complex number z = x + iy is known as the affix
or x2 + (y – 1)2 = x2 + (y + 5)2 of the point (x, y) which it represents.
i.e. y = –2 (2)
2
Putting y = –2 in (1), x + 4 = 16  or  x = ±2. Polar Form of a Complex Number
Hence, the complex numbers z satisfying the given
Let O be the origin and OX and OY be the x-axis and y-axis
equations are
respectively. Let z = x + iy be a complex number repre-
z1 = 2 – 2i, and z2 = – 2 – 2i. sented by the point P(x, y).
Draw PM ^ OX. Then,
16. If i z3 + z2 – z + i = 0, then
OM = x and PM = y. Join OP
(A) |z| < 1 (B)  |z| > 1
(C) |z| = 1 (D)  |z| = 0 Let OP = r and ∠XOP = q.
Solution: (C) Then

Given, iz3 + z2 – z + i = 0 z = x + iy = r (cos q + i sin q)

⇒ i z2 (z – i) – (z – i) = 0 This form of z is called polar or trigonometric form.

⇒ (z – i) (i z2 – 1) = 0 ⇒ z = i

Objective_Maths_JEE Main 2017_Ch 3.indd 7 01/01/2008 03:26:05


3.8 Chapter 3

Comparing real and imaginary parts, we get


 If x > 0, y < 0 (i.e., z is in fourth quadrant), then
x = r cos q (1) ⎛||
y⎞
arg z = q = – tan–1 ⎜ ⎟ .
and y = r sin q (2) ⎝ x⎠
Squaring Eq. (1) and (2) and adding, we get  Argument of the complex number 0 is not defined.
⎧0, if x > 0
r2 = x2 + y2 or r = x 2 + y 2 = |z|  arg (x + i0) = ⎨
⎩p , if x < 0
Thus, r is known and is equal to the modulus of the complex ⎧p /2, if y > 0
number z.  arg (0 + iy) = ⎨ .
⎩3p /2, if y < 0
Substituting the value of r in Eq. (1) and (2), we get
x y
cos q = and sin q = (3)
x2 + y 2 x2 + y 2 Properties of Argument
y 1. arg (z1z2) = arg (z1) + arg (z2)
Dividing Eq. (2) by (1), we get tan q = .
x ⎛z ⎞
The form z = r (cos q + i sinq ) = reiq of the complex 2. arg ⎜ 1 ⎟ = arg z1 – arg z2
⎝ z2 ⎠
number z is called exponential form.
Any value of q satisfying (3) is known as amplitude ⎛z⎞
3. arg ⎜ ⎟ = 2 arg z
or argument of z and written as q = arg (z) or q = amp z. ⎝z ⎠
4. arg (zn) = n arg z
i M P o R ta n t P o i n t S ⎛z ⎞ ⎛z ⎞
5. If arg ⎜ 2 ⎟ = q, then arg ⎜ 1 ⎟ = 2kp – q where k ∈ I
⎝ z1 ⎠ ⎝ z2 ⎠
The unique value of q such that – p < q ≤ p for which x =
r cos q and y = r sin q, is known as the principal value of 6. arg z = – arg z
the argument.
The general value of the argument is (2np + q),
7. arg ( z z ) = arg z ( ) = arg (positive real number) = 0
2

where n is an integer and q is the principal value of arg (z).


While reducing a complex number to polar form, we SOLvEd EXAMPLES
always take the principal value.
The complex number z = r (cos q + i sin q) can also 18. The inequality |z – 4 | < |z – 2| represents the region
be written as rcisq. given by,
(A) Re (z) > 0
r ( c os θ + i s in θ )
(B) Re (z) < 0
(C) Re (z) > 3
(D) None of these
rcis θ
Solution: (C)

Given |z – 4 |2 < |z – 2|2


TRICk(S) FOR PROBLEM SOLvINg ⇒ |(x – 4) + iy |2 < | (x – 2) + iy |2

 If x > 0, y > 0 (i.e., z is in first quadrant), then ⇒ (x – 4)2 + y2 < (x – 2)2 + y2


⎛ y⎞ ⇒ – 4x < – 12 ⇒ 4x > 12; x > 3
arg z = q = tan– 1 ⎜ ⎟ .
⎝ x⎠
 If x < 0, y > 0 (i.e., z is in second quadrant), then ⇒ Re (z) > 3.
⎛ y⎞ ⎛p⎞ ⎛p⎞
arg z = q = p – tan–1 ⎜ ⎟ . 19. If zr = cos ⎜ r ⎟ + i sin
⎝||
x⎠ ⎝3 ⎠ ⎜⎝ r ⎟⎠ , r = 1, 2, 3, …, then
3
 If x < 0, y < 0 (i.e., z is in third quadrant), then
z1 z2 z3 … ∞ =
⎛ y⎞
arg z = q = – p + tan–1 ⎜ ⎟ . (A) i (B) –i
⎝ x⎠
(C) 1 (D) –1

Objective_Maths_JEE Main 2017_Ch 3.indd 8 01/01/2008 03:26:08


Complex Numbers  3.9

Solution: (A) ⇒ |x + iy – 4|2 < |x + iy – 2|2


⎛p⎞ ⎛p⎞ (Putting z = x + iy)
zr = cos ⎜ r ⎟ + i sin ⎜ r ⎟ ,
Since
⎝3 ⎠ ⎝3 ⎠ ⇒ (x – 4)2 + y2 < (x – 2)2 + y2
r = 1, 2, 3, … ⇒ x2 – 8x + 16 + y2 < x2 – 4x + 4 + y2
we have, z1 · z2 · z3 … ∞ ⇒ –4x < – 12 ⇒ x > 3 ⇒ Re (z) > 3
⎛ p p⎞ ⎛ p p⎞
= ⎜ cos + i sin ⎟ ⎜ cos 2 + i sin 2 ⎟ 1− i 3
⎝ 3 3 ⎠ ⎝ 3 3 ⎠ 22. The argument of
is
1+ i 3
⎛ p p⎞ p 2p 4p 2p
⎜⎝ cos 3 + i sin 3 ⎟⎠ ...∞  (A)  (B)  (C)  (D) –
3 3 3 3 3 3
⎛p p p ⎞ ⎛p p p ⎞ Solution: (D)
= cos ⎜ + 2 + 3 + ...⎟ + i sin ⎜ + 2 + 3 + ...⎟
⎝3 3 3 ⎠ ⎝ 3 3 3 ⎠ 1− i 3 (1 − i 3 )2 −2 − 2 3 i
= =
⎛ p ⎞ ⎛ p ⎞ 1+ i 3 4 4 
⎜ 3 ⎟ ⎜ ⎟
= cos ⎜ + i sin ⎜ 3 ⎟ 1 3
1⎟ 1 = − −
2 2
i .
⎜1− ⎟ ⎜1− ⎟
⎝ 3⎠ ⎝ 3⎠
 ⎛ 1 3 ⎞ 2p
p p \ arg ⎜ − − i⎟ = – (p – tan–1 3) = –
= cos + i sin = 0 + i ⋅ 1 = i ⎝ 2 2 ⎠ 3 
2 2
10 23. arg bi (b > 0) is
⎛ 2p k 2p k ⎞ p p
20. The value of ∑ ⎜ sin − i cos is (A) p (B) 
11 ⎟⎠
⎝ (C) – (D) 0
k =1 11 2 2
(A) 1 (B) – 1 (C) i (D) –i Solution: (B)
Since b > 0, bi represents a point on the positive side
Solution: (C)
of the imaginary axis on which the argument of every
We have, p
10 point is .
⎛ 2p k 2p k ⎞ 2
∑ ⎜⎝ sin 11
− i cos
11 ⎟⎠ 24. Let zk (k = 0, 1, 2, …, 6) be the roots of the equation
k =1 
10 6
⎛ 2 2p k 2p k ⎞
= ∑ ⎜ −i sin
⎝ 11
− i cos
11 ⎟⎠
(z + 1)7 + z7 = 0, then ∑ Re (z k ) is equal to
k =1  k =0
10 10 2p k 3 7 7
⎛ 2p k 2p k ⎞ i (A) 0 (B)  (C) – (D) 
= –i ∑ ⎜ cos + i sin =–i ∑e 11
11 ⎟⎠
⎝ 2 2 2
k =1 11 k =1 Solution: (C)

⎡ 10 i 2p k ⎤
= – i ⎢ ∑ e 11 − 1⎥ Let zk = xk + iyk,
⎢ k =0 ⎥
⎣ ⎦ we have (zk + 1)7 + z k7 = 0
= – i (sum of 11th roots of unity – 1)
⇒ (zk + 1)7 = – zk7 ⇒ |zk + 1|7 = |zk|7
= – i (0 – 1) = i.
21. The inequality |z – 4| < |z – 2| represents the region ⇒ |zk + 1| = |zk| ⇒ |xk + iyk + 1|2 = |xk + iyk|2
given by
⇒ (xk + 1)2 + y k2 = xk2 + yk2 
(A)  Re (z) > 0 (B)  Re (z) < 0
(C)  Re (z) > 2 (D)  None of these 1
⇒ 2xk + 1 = 0  or  xk = –
Solution: (D) 2
6 6
We have, 7
Thus, ∑ Re ( z k ) = ∑ x k = – 2 .
|z – 4| < |z – 2| ⇒ |z – 4|2 < |z – 2|2 k =0 k =0

Objective_Maths_JEE Main 2017_Ch 3.indd 9 01/01/2008 03:26:13


3.10 Chapter 3

25. If arg (z) < 0, then arg (–z) – arg (z) =


(A) p (B) –p
notE
p p e iq + e − iq e iq − e − iq
(C) – (D) cos q = and sin q =
2 2 2 2i
Solution: (A)
As –q = arg (z) < 0,
SOLvEd EXAMPLE
we take z = r [cos (– q ) + i sin (– q )]
= r (cos q – i sinq ) ( 3 + i ) 4 n +1
26. For any integer n, the argument of z = is
(1 − i 3 ) 4 n
(–z) p p
(A) (B)
r 6 3
π –θ
p 2p
(C) (D)
O –θ 2 3
r (E) All of the above
(Z) Solution: (A)
We have,
⇒ – z = r (– cos q + i sinq )
( 3 + i ) 4 n +1
= r [cos (p – q ) + i sin (p – q )] z=
(1 − i 3 ) 4 n
\ arg (–z) = p – q 4 n +1
Thus, ⎛ ip ⎞
arg (–z) – arg (z) = p – q + (q ) = p ⎜ 2e 6 ⎟ i ( 4 n +1)
p
⎜⎝ ⎟⎠ 2 4 n +1e 6
= 4n
= p
PARTICULAR CASES OF POLAR FORM ⎛ −i p ⎞ − i 4n
⎜ 2e 3 ⎟ 24n e 3

⎜⎝ ⎟⎠
1. 1 = 1 + i0 = cos 0 + i sin 0
2. –1 = – 1 + i0 = cos p + i sin p i (12 n +1)
p pi
p p = 2⋅e 6 = 2 ⋅ e 2 np i ⋅ e 6
3. i = 0 + i1 = cos + i sin
2 2 = 2 ⋅ epi/6 (∵ e2npi = 1)
⎛ p⎞ ⎛ p⎞
4. –i = 0 + i (– 1) = cos ⎜ − ⎟ + i sin ⎜ − ⎟ p
⎝ 2⎠ ⎝ 2⎠ \ arg z = .
6
⎡ ⎛ p⎞ ⎛ p⎞⎤
5. 1 – i = 2 ⎢cos ⎜ − ⎟ + i sin ⎜ − ⎟ ⎥ LOgARITHM OF A COMPLEX NUMBER
⎣ ⎝ 4⎠ ⎝ 4⎠⎦
⎡ ⎛ 3p ⎞ ⎛ 3p ⎞ ⎤ log (x + iy) = loge (reiq) = loge eiq = loge r + iq
6. –1 – i = 2 ⎢cos ⎜ − ⎟ + i sin ⎜ − ⎟ ⎥
⎣ ⎝ 4 ⎠ ⎝ 4 ⎠⎦ ⎛ y⎞
= loge ( x 2 + y 2 ) + i tan −1 ⎜ ⎟
⎝ x⎠
EULERIAN REPRESENTATION OF A
loge (z) = loge |z| + i arg (z)
COMPLEX NUMBER
Since eiq = cos q + i sin q, thus any non zero complex num- caution
ber z = x + iy = r (cos q + i sin q ) can be represented in
Eulerian form as Since the argument of a complex number is not unique, the
log of a complex number cannot be unique. In general,
z = reiq = r (cos q + i sin q ), loge (z) = loge |z| + i [2kp + arg (z)], k ∈ I
where |z| = r and q = arg (z).

Objective_Maths_JEE Main 2017_Ch 3.indd 10 01/01/2008 03:26:16


Complex Numbers 3.11

notE SOLvEd EXAMPLES


ip
ip ⎛ ip ⎞ 27. If z = cos q + i sin q, then
log i = log e 2 = , log (log i) = log ⎜ ⎟ 1
2 ⎝ 2⎠
(A) z n + n = 2 cos nq
⎛p⎞ ip z
= log i + log ⎜ ⎟ = + log (p/2).
⎝ 2⎠ 2 1
(B) z n + n = 2n cos nq
z
1
(C) z n − n = 2i sin nq
vECTORIAL REPRESENTATION OF A z
COMPLEX NUMBER
1
(D) z n − n = (2i)n sin nq
z
If P is the point (a, b) on the argand plane corresponding to
the complex number z = a + ib. Solution: (A, C)
Then We have,
 1 1
OP i=ˆ aiˆ + bjˆ , = = cos q – i sin q.
z cos q + i sin q

\ OP = a 2 + b 2 = |z| \ zn = (cos q + i sinq)n = cos nq + i sin nq,
1
and and = (cos q – i sinq)n = cos nq – i sin nq
 ⎛ b⎞ zn
arg(z) = direction of the vector OP = tan–1 ⎜ ⎟ . 1
⎝ a⎠ Hence, z n + n = 2 cos nq
z
de’Moivre’s Theorem n 1
and z − n = 2i sin nq.
If n is any integer, then z
z 2n − 1
28. If z = cos q + i sin q, then 2 n =
(cos q + i sin q)n = cos nq + i sin nq z +1
(A) i cot nq (B) i tan nq
TRICk(S) FOR PROBLEM SOLvINg (C) tan nq (D) cot nq
(n is an integer)
 If n is any rational number, then cos nq + i sin nq is one of
the values of (cos q + i sinq)n. Solution: (B)
 (cos q + i sinq )– n = cos (–n)q + i sin (–n)q We have,
= cos nq – i sin n q z 2n − 1 (cos q + i sin q )2 n − 1
=
 (cos q – isinq )n = [cos (–q ) + i sin (–q )]n (cos q + i sin q )2 n + 1
z 2n + 1
= cos (–nq ) + isin (–nq )
cos 2nq + i sin 2nq − 1
= cos nq – isin nq =
cos 2nq + i sin 2nq + 1
1
 = (cos q + isin q )–1 = cos q – i sin q (Using De Moivre’s Theorem)
cos q + i sinq
 The theorem cannot be applied to (cos q + isinf)n i.e., q (1 − 2 sin 2 nq ) + 2i sin nq cos nq − 1
must be same with cos and sin both.
=
(2 cos2 nq − 1) + 2i sin nq cos nq + 1
 The theorem is not directly applicable to (sin q + i cosq )n,
i sin nq cos nq + i 2 sin 2 nq
rather n = (∵ i2 = –1)
⎡ ⎛p ⎞ ⎛p ⎞⎤ cos 2 nq + i sin nq cos nq
(sin q + icosq )n = ⎢cos ⎜ − q ⎟ + i sin ⎜ − q ⎟ ⎥
⎣ ⎝2 ⎠ ⎝2 ⎠⎦
i sin nq (cos nq + i sin nq )
= = i tan nq.
⎛p ⎞ ⎛p ⎞ cos nq (cos nq + i sin nq )
= cos n ⎜ − q ⎟ + i sin ⎜ − q ⎟
⎝2 ⎠ ⎝2 ⎠
 (cosq1 + i sinq1) (cos q2 + isin q2) … (cosqn + isin qn)
29. If a = cos a + i sin a, b = cos b + i sin b,
a b c
= cos (q1 + q2 + … + qn) + isin (q1 + q2 + … + qn) c = cos g + i sin g and + + = – 1, then
b c a
cos (b – g ) + cos (g – a) + cos (a – b ) =

Objective_Maths_JEE Main 2017_Ch 3.indd 11 01/01/2008 03:26:21


3.12  Chapter 3

(A) 0 (B) 1 31. If (sin q1 + i cos q1) (sin q2 + i cos q2) … (sin qn + i cos
(C)  –1 (D)  None of these qn) = a + ib, then a2 + b2 =
Solution: (C) (A) 4 (B) 2
(C)  1 (D)  None of these
We have,
1 1 Solution: (C)
= cos a – i sin a, = cos b – i sin b
a b Given expression
a n
⎛ ⎛p ⎞ ⎛p ⎞⎞
= (cos a + i sina) (cos b – i sinb )
Now
b = ∏ ⎜⎝ cos ⎜⎝ 2 − q r ⎟⎠ + i sin ⎜⎝ 2 − q r ⎟⎠ ⎟⎠
r =1
a 
or = cos (a – b ) + i sin (a – b ) ⎛p
n

n
⎛p ⎞
b = cos ∑ ⎜ − q r ⎟ + i sin ∑ ⎜ − q r ⎟

r =1 2
⎠ ⎝
r =1 2

b 
Similarly, = cos (b – g ) + i sin (b – g ) = cos a + i sin a,
c
n
c ⎛p ⎞
and
a
= cos (g – a) + i sin (g – a) where a= ∑ ⎜⎝ 2 − q r ⎟⎠
r =1 
a b c
Putting these values in + + = –1, = a + ib
b c a
we get \ a2 + b2 = cos2 a + sin2 a = 1.
[cos (a – b ) + cos (b – g ) + cos (g – a)]
32. If z2 – 2zcosθ + 1 = 0, then z2 + z–2 is equal to
+ i [sin (a – b ) + sin (b – g ) + sin (g – a)]
(A) 2cos2θ (B) 2sin2θ (C)  2 cosθ (D)  2 sinθ
= –1 = – 1 + 0 i
Solution: (A)
Comparing real part on both sides, we get
We have,
cos (a – b ) + cos (b – g ) + cos (g – a) = –1 z2 – 2zcosθ + 1 = 0
30. If n is a positive integer, then ( 3 + i)n + ( 3 – i)n is 2 cos q ± 4 cos 2 q − 4
equal to ⇒ z= = cos q ± cos 2 q − 1
np np 2 
(A) 2n cos (B) 2n + 1 cos
6 6 = cos q ± − sin 2 q = cos q ± i 2 sin 2 q 
n–1 np
(C) 2 cos (D)  None of these = cosθ ± isinθ.
6
Solution: (B) When z = cosθ + isinθ

Let 3 = r cos q and 1 = r sin q z2 + z–2= cos2θ + isin2θ + (cos2θ – isin2θ)


so that = 2cos2θ
1 p and when z = cosθ – isinθ,
r2 = 4 and tan q = ⇒ r = 2, q =
3 6  2 –2
z + z = cos2θ – isin2θ + cos2θ + isin2θ
n
⎛ p p ⎞ = 2cos2θ
\ ( 3 + i)n = 2n ⎜ cos + i sin ⎟
⎝ 6 6⎠ 
⎧ ⎛ np ⎞ ⎛ np ⎞ ⎫ Roots of a Complex Number
or ( 3 + i)n = 2n ⎨cos ⎜ ⎟ + i sin ⎜ ⎟ ⎬ (1)
⎩ ⎝ 6⎠ ⎝ 6 ⎠⎭
If z = r (cos q + i sinq ) and n is a positive integer, then
Similarly,
1 1
⎧ ⎛ np ⎞ ⎛ np ⎞ ⎫ ⎡ ⎛ 2k p + q ⎞ ⎛ 2k p + q ⎞ ⎤
n n
( 3 – i) = 2 ⎨cos ⎜ ⎟ − i sin ⎜ ⎟ ⎬ (2) z n = r n ⎢cos ⎜ ⎟⎠ + i sin ⎜⎝ ,
⎩ ⎝ 6⎠ ⎝ 6 ⎠⎭ ⎣ ⎝ n n ⎟⎠ ⎥⎦
Adding Eq. (1) and (2), we obtain where k = 0, 1, 2, 3, … (n – 1).
⎛ np ⎞
( 3 + i)n + ( 3 – i)n = 2 ⋅ 2n cos ⎜ ⎟ Cube Roots of Unity
⎝ 6 ⎠

⎛ np ⎞ Let z = 11/3 or z3 – 1 = 0
= 2n + 1 cos ⎜ ⎟
⎝ 6⎠ ⇒ (z – 1) (z2 + z + 1) = 0


Objective_Maths_JEE Main 2017_Ch 3.indd 12 01/01/2008 03:26:24


Complex Numbers  3.13

−1 + i 3 −1 − i 3 5. x2 – xy + y2 = (x + yw) (x + yw2), in particular, x2 – x +


i.e., z = 1, ,  1 = (x + w) (x + w2)
2 2
6. x2 + y2 + z2 – xy – xz – yz = (x + yw + zw2) (x + yw2 + zw)
−1 + i 3 7. x3 + y3 + z3 – 3xyz = (x + y + z) (x + w y + w2z) (x + w2y
Put w= ,
2 + w z)
−1 − i 3
then w2 = .
2 Solved Examples
2
Thus cube roots of unity are 1, w, w .
33. If 1, w, w2 be the three cube roots of unity, then (1 + w)
Properties of Cube Roots of Unity (1 + w2) (1 + w4) (1 + w8) … to 2n factors =
1. 1 + w + w2 = 0 (A) 1 (B) –1
2. w3 = 1 (C)  0 (D)  None of these
3. w3n = 1, w3n + 1 = w, w3n + 2 = w2 2p i Solution: (A)
2 2 3
4. w = w and ( w ) = w, w w = w , w = e 3 , We have,
2p i

w2 = e 3 (1 + w) (1 + w2) (1 + w4) (1 + w8) … to 2n factors
5. If a + bw + cw2 = 0, then a = b = c provided a, b, c are = (1 + w) (1 + w2) (1 + w3 ⋅ w) (1 + w6 ⋅ w2) ... to 2n
real. factors
6. If these roots are marked on the argand plane, then
these are vertices of an equilateral triangle with = (1 + w) (1 + w2) (1 + w) (1 + w2) … to 2n factors
­circumcentre at origin, as shown in the Fig. 3.1. (∵ w3 = w6 = … = 1)
Imaginary = [(1 + w) (1 + w) … to n factors]
axis
–1 , 3
[(1 + w2) (1 + w2) … to n factors]
2 2 = (1 + w)n (1 + w2)n = [(1 + w) (1 + w2)]n
= (1 + w + w2 + w3)n = (0 + 1)n = 1

3 (∵ 1 + w + w2 = 0, w3 = 1).
Real axis
O 2π (1, 0)
3 34. If 1, w, w2 are the three cube roots of unity, then (1 – w
+ w2) (1 – w2 + w4) (1 – w4 + w8) … to 2n factors =
–1 , 3 (A) 2n (B)  22n
4n
2 2 (C) 2 (D)  None of these
Fig. 3.1 Solution: (B)
We have,
Fourth Roots of Unity
(1 – w + w2) (1 – w2 + w4) (1 – w4 + w8)
The four, fourth roots of unity are given by the solution set
of the equation x4 – 1 = 0 (1 – w8 + w16) … to 2n factors
⇒ (x2 – 1) (x2 + 1) = 0 ⇒ x = ± 1, ± i = (1 – w + w2) (1 – w2 + w) (1 – w + w2)
Fourth roots of unity are vertices of a square which lies on (1 – w2 + w) … to 2n factors.
coordinate axes.
[∵ w4 = w, w8 = w2, w16 = w and so on]
Some Useful Relations
1. x2 + y2 = (x + iy) (x – iy) = (– 2w) (– 2w2) (– 2w) (– 2w2) … to 2n factors
2. x3 + y3 = (x + y) (x + yw) (x + yw 2) = (22 w3) (22 w3) … to n factors
3. x3 – y3 = (x – y) (x – yw) (x – yw 2)
4. x2 + xy + y2 = (x – yw) (x – yw 2), in particular, x2 + x + [∵ (– 2w) (– 2w2) = 22 w3 = 22]
1 = (x – w) (x – w 2) = (22)n = 22n.

Objective_Maths_JEE Main 2017_Ch 3.indd 13 01/01/2008 03:26:26


3.14  Chapter 3

Solution: (B)
35. − 1 − − 1 − − 1 − ... to ∞ =
We have, (x – 1)3 + 8 = 0
2
(A) 1 (B) –1 (C) w (D) 
w ⇒ (x – 1)3 = – 8
Solution:  (C, D) \ x – 1 = (– 8)1/3 = – 2, – 2w, – 2w2
Let x= − 1 − − 1 − − 1 − ... to ∞ Hence, x = –1, 1 – 2w, 1 – 2w2

2
Then x= − 1 − x  or x = – 1 – x 39. (i + 3 )100 + (i – 3 )100 + 2100 =
or x2 + x + 1 = 0 (A) 1 (B) – 1
(C)  0 (D)  None of these
− 1 ± 1 − 4 ⋅1⋅1 −1 ± − 3
\ x= = Solution: (C)
2 ⋅1 2 
−1 + 3 i 2 2w
− 1 ± 3i We have, i+⋅ = 3 =
= = w or w2. 2 i i 
2
−1 − 3 i 2 2w 2
6 6 and i– 3 = ⋅ =
⎛ 3 + i⎞ ⎛ i − 3⎞ 2 i i 
36. ⎜ ⎟ +⎜ ⎟ =
⎝ 2 ⎠ ⎝ 2 ⎠ 100 100
\ (i + 3 ) + (i – 3 ) + 2 100

(A) –2 (B) 2 (C) –1 (D) 1 100


⎛ 2w 2 ⎞
100
⎛ 2w ⎞
= ⎜ +⎜ + 2100
Solution: (A) ⎝ i ⎟⎠ ⎝ i ⎠


We have,
2100
⎛ −1 + 3 i ⎞ = (w100 + w200) + 2100
3+i i 3 + i2 i100
= =–i ⎜ ⎟ = –iw
2 2i ⎝ 2 ⎠ = 2100 (w + w2) + 2100
i− 3 i2 − i 3 ⎛ −1 − 3 i ⎞
and = =–i ⎜ ⎟ = –iw
2 = –2100 + 2100 = 0.
2 2i ⎝ 2 ⎠
⎛ 3 + i⎞
6
⎛ i − 3⎞
6 nth Roots of Unity
6 2 6
Hence, ⎜ 2 ⎟ + ⎜ 2 ⎟ = (–iw) + (–iw ) Since 1 = cos 0 + i sin 0, therefore,
⎝ ⎠ ⎝ ⎠
(1)1/n = (cos 0 + i sin 0)1/n
= i6(w6 + w12)
2p r + 0 2p r + 0
= –1 (1 + 1) = –2. = cos + i sin ; r = 0, 1, 2, …, (n – 1)
n n
37. The common roots of the equations z3 + 2 z2 + 2 z + 1 = 0 2p r 2p r
and z1985 + z100 + 1 = 0 are = cos + i sin ; r = 0, 1, 2, …, (n – 1)
n n
(A) –1, w (B)  –1, w2 2 rp
2 i
(C) w, w (D)  None of these =e n ; r = 0, 1, 2, …, (n – 1)
(i2p/n)
Solution: (C) = 1, e , e(i4p/n), …, e[i2(n – 1)p/n]
3 2
We have, z + 2 z + 2 z + 1 = 0
= 1, a, a2, a3, …, a n – 1,
⇒ (z + 1) (z2 + z + 1) = 0
where a = e(i2p/n)
2
Its roots are – 1, w and w . The root z = – 1 does not
satisfy the equation z1985 + z100 + 1 = 0 but z = w and Properties of nth Roots of Unity
z  =  w2 satisfy it. Hence, w and w2 are the common 1. 1 + a + a2 + … + a n – 1 = 0
roots. 2. 1 ⋅ a ⋅ a2 ⋅ … a n–1 = (–1)n–1
38. If the cube roots of unity are 1, w, w2, then the roots of 3. The n, nth roots of unity lie on the unit circle |z| = 1 and
the equation (x – 1)3 + 8 = 0 are form the vertices of a regular polygon of n sides.
(A)  –1, 1 + 2w, 1 + 2w2 (B)  –1, 1 – 2w, 1 – 2w2 4. nth roots of unity form a G.P. with common ratio
(C)  –1, –1, –1 (D)  None of these e(i2p/n).

Objective_Maths_JEE Main 2017_Ch 3.indd 14 01/01/2008 03:26:30


Complex Numbers  3.15

2. Section Formula: If R(z) divides the line segment


Solved examples joining P(z1) and Q(z2) in the ratio m1 : m2(m1, m2 > 0)
then
40. If r is non-real and r = 5 1, then the value of m z + m2z1
 (i)  For internal division, z = 1 2
(
1 + r + r 2 + r −2 − r −1 is equal to ) m1 + m 2
(A) 2 (B) 4 m1z 2 − m 2 z 1
(ii)  For external division, z =
(C)  8 (D)  None of these m1 − m 2
3. Equation of the Perpendicular Bisector: If P(z1) and
Solution: (B)
Q(z2) are two fixed points and R(z) is moving point
|1 + r + r2 + r–2 – r–1| = |1 + r + r2 + r3 – r4| (see Fig. 3.3) such that it is always at equal distance
from P(z1) and Q(z2) then locus of R(z) is perpendicu-
[Q r5 = 1 ⇒ r3 ⋅ r2 = 1 or r–2 = r3 lar bisector of PQ
and r4 ⋅ r = 1  or  r–1 = r4] i.e., PR = QR or |z – z1| = |z – z2|
= |1 + r + r2 + r3 + r4 – 2r4| ⇒ |z – z1|2 = |z – z2|2

1 − r5 P(z1)
− 2r 4 = 0 − 2r 4 (Q r5 = 1)
1− r
= 2|r|4 = 2(1) = 2 (Q |r| = 1 as r5 = 1)
R(z)
2|1+ r + r + r −2 − r −1 |
= 22 = 4
2

\
41. The values of (16)1/4 are
(A) ±2, ±2 i (B)  ±4, ±4 i Q(z2)
(C) ±1, ±i (D)  None of these
Fig. 3.3
Solution: (A)
After solving,
We have
z ( z1 − z2 ) + z ( z1 − z2 ) = |z1|2 – |z2|2
1/4 4 1/4 1/4
(16) = (2 ) = 2 (1)
4. Equation of a Straight Line
= 2 (cos 0 + i sin 0)1/4  (i) Parametric form: Equation of a straight line join-
⎧ 1 1 ⎫ ing the points having affixes z1 and z2 is z = t z1 +
= 2 ⎨cos (2 k p + 0) + i sin (2 k p + 0) ⎬ ,
⎩ 4 4 ⎭ (1 – t)z2, where t ∈ R
 k = 0, 1, 2, 3 (ii) Non-parametric form: Equation of a straight
line joining the points having affixes z1 and z2 is
= 2 × 1, 2 × i, 2 × –1, 2 × –i = ±2, ±2i
z z 1
z1 z1 1 = 0
Geometry of Complex Numbers
z2 z2 1
1. Distance Formula: The distance between two points

⇒  z ( z 1 − z 2 ) − z ( z 1 − z 2 ) + z 1z 2 − z 2 z 1 = 0
P(z1) and Q(z2) is given by PQ = |z2 – z1| = |affix of
Q – affix of P| (see Fig. 3.2)
Trick(s) for Problem Solving
Q(z2)
 Three points z1, z2 and z3 are collinear if,
z1 z1 1
z2 z2 1 = 0
z3 z3 1
 If three points A(z1), B(z2), C(z3) are collinear then slope
P(z1) of AB = slope of BC = slope of AC
Fig. 3.2 z1 − z2 z2 − z3 z1 − z3
⇒ = =
z1 − z2 z2 − z3 z1 − z3

Objective_Maths_JEE Main 2017_Ch 3.indd 15 01/01/2008 03:26:32


3.16  Chapter 3

(iii) General equation of a straight line: The general


Trick(s) for Problem Solving
equation of a straight line is of the form az + az
+ b, where a is complex number and b is real If z is a variable point and z1, z2 are two fixed points in the
number. argand plane, then
 (iv) 
Slope of a line: The complex slope of the line 1. |z – z1| = |z – z2| ⇒ Locus of z is the perpendicular
a coeff. of z bisector of the line segment joining z1 and z2.
az + az + b = 0 is – =– and real
a coeff. of z 2. |z – z1| + |z – z2| = constant (≠ |z1 – z2|)
Re(a) ⇒ Locus of z is an ellipse
slope of the line az + az + b is – = –i
Im(a) 3. |z – z1| + |z – z2| = |z1 – z2|
(a + a )
. ⇒ Locus of z is the line segment joining z1 and z2
(a − a ) 4. |z – z1| – |z – z2| = |z – z2|
 (v) Length of perpendicular: The length of perpen- ⇒ Locus of z is a straight line joining z1 and z2 but z
dicular from a point z1 to the line az + az + b = 0 does not lie between z1 and z2.
az 1 + az 1 + b az 1 + az 1 + b 5. |z – z1| – |z – z2| = constant (≠ |z1 – z2|)
is given by or . ⇒ Locus of z is a hyperbola.
a+a 2a
6. |z – z1|2 + |z – z2|2 = |z1 – z2|2 ⇒ Locus of z is a
5. Equation of a circle circle with z1 and z2 as the extremities of diameter.
  (i) The equation of a circle whose centre is at point 7. |z – z1| = k |z – z2|, (k ≠ 1) ⇒ Locus of z is a circle.
having affix z0 and radius r is |z – z0 | = r.
⎛ z − z1 ⎞
(ii) If the centre of the circle is at origin and radius r, 8. arg ⎜ = a (fixed) ⇒ Locus of z is a segment of
then its equation is |z| = r (see Fig. 3.4). ⎝ z − z2 ⎟⎠
circle.
P(z) ⎛ z − z1 ⎞ p
9. arg ⎜ = ± ⇒ Locus of z is a circle with z1 and
r ⎝ z − z2 ⎟⎠ 2
z2 as the vertices of diameter.
C(z0) ⎛ z − z1 ⎞
10. arg ⎜ = 0 or p ⇒ Locus of z is a straight line
⎝ z − z2 ⎟⎠
­passing through z1 and z2.
Fig. 3.4

(iii) |z – z0 | < r represents interior of a circle |z – z0| = r


whereas |z – z0| > r represents exterior of the circle Time Saving Tips
|z – z0| = r.
 ||z1| – |z2|| ≤ |z1 + z2| ≤ |z1| + |z2|
 
A C = AB eiq  or (z3 – z1) = (z2 – z1)eiq Thus |z1| + |z2| is the greatest possible value of |z1 + z2|
z3 − z1 and ||z1| – |z2|| is the least possible value of |z1 + z2|.
or = eiq
z2 − z1 1
 If z + = a, the greatest and least values of |z| are
z
 (iv) If A, B and C are three points in argand plane such
that AC = AB and ∠CAB = q then use the rota- a + a2 + 4 − a + a2 + 4
respectively and .
tion about A to find eiq, but if AC ≠ AB use coni 2 2
 The area of the triangle whose vertices are z, iz and z + iz
method.
1
  (v)  If four points z1, z2, z3 and z4 are con-cyclic then is |z|2.
2
(z 4 − z 1 ) (z 2 − z 3 )  The area of the triangle with vertices z, wz and z + wz is
= real
(z 4 − z 2 ) (z 1 − z 3 ) 3
|z|2.
4
⎛ (z − z 3 ) (z 4 − z 1 ) ⎞  If z1, z2, z3 be the vertices of an equilateral triangle and z0
  or arg ⎜ 2
= ±p, 0
⎝ ( z 1 − z 3 ) ( z 4 − z 2 ) ⎟⎠ be the circumcentre, then z12 + z22 + z32 = 3z02 .
 If z1, z2, z3 be the vertices of a triangle, then the triangle is

equilateral iff (z1 – z2)2 + (z2 – z3)2 + (z3 – z1)2 = 0

Objective_Maths_JEE Main 2017_Ch 3.indd 16 01/01/2008 03:26:36


Complex Numbers  3.17

z1 z
or z12 + z22 + z32 = z1z2 + z2z3 + z3z1 = (1 + 0 + i – 1 + 0) = 1 i
3 3
or
1
+
1
+
1
=0 z1 ⎛ p p⎞
= cos + i sin ⎟
z1 − z2 z2 − z3 z3 − z1 3 ⎜⎝ 2 2⎠ 
 The equation |z – z|2 + |z – z2|2 = k (where k is a real
43. If z1 and z2 (≠ 0) are two complex numbers such that
1
number) will represent a circle with centre at (z1 + z2) z1 − z2
2 = 1, then
1 1 z1 + z2
and radius 2k − | z1 − z2 |2 provided k ≥ |z – z |2.
2 2 1 2 (A) z2 = ikz1, k ∈ R (B) z2 = kz1, k ∈ R
 The one and only one case in which |z1| + |z2| + … +
(C) z2 = z1 (D)  None of these
|zn| = |z1 + z2 + … + zn| is that the numbers z1, z2, … zn
have the same amplitude. Solution: (A)
 If three points z1, z2, z3 are connected by relation az1 + We have,
bz2 + cz3 = 0 where a + b + c = 0, then the three points z1 − z 2 z /z − 1
are collinear. =1⇒ 1 2 = 1
z z1 + z 2 z 1/z 2 + 1
 If z is a complex number, then e is periodic.

 If three complex numbers are in A.P., then they lie on a z1 z


straight line in the complex plane. ⇒ −1 = 1 +1
z2 z2

z1
⇒ lies on the perpendicular bisector of the
z2
Solved Examples ­segment joining A (–1 + 0i) and B (1 + 0i).
z1
42. The centre of a square ABCD is at z = 0. If A is z1, then \ = ai for some a ∈ R
z2
the centroid of triangle ABC is
z2 1 −i
z ⎛ p p⎞ ⇒ = =
(A)  1 ⎜ cos + i sin ⎟ z1 ai a 
3 ⎝ 2 2⎠
\ z2 = i kz1 for some k ∈ R
z
(B)  1 (cos p + i sin p) 44. If z = x + iy and ‘a’ is a real number such that |z – ai| =
3
⎛ p p⎞ |z + ai|, then locus of z is
(C) z1 ⎜ cos + i sin ⎟
⎝ 2 2⎠ (A) x-axis (B)  y-axis
(C) x = y (B)  x2 + y2 = 1
(D)  None of these
Solution: (A)
Solution: (A)
p
Since A is z1 and ∠AOB = We have, |z – ai| = |z + ai|
2
⎛ p p ⎞ ⇒ |x + i (y – a)|2 = |x + i (y + a)|2
\ B is z1 cos + i sin
⎜⎝ 2 2 ⎟⎠ ⇒ x2 + (y – a)2 = x2 + (y + a)2
⇒ 4ay = 0; y = 0, which is x-axis.
B B(z1)
45. The locus represented by |z – 1| = |z + i| is
(A)  a circle of radius 1
O (0, 0) (B)  an ellipse with foci at 1 and –i
(C)  a line through the origin
(D)  a circle on the join of 1 and –i as diameter
C D
Solution: (C)
Also, c is z1(cos p + i sin p) We have, |z – 1| = |z + i|
\ Centroid of DABC is
⇒ |(x – 1) + iy| = |x + i (y + 1)|
z1 ⎛ p p ⎞
1 + cos + i sin + cos p + i sin p ⎟ ⇒ (x – 1)2 + y2 = x2 + (y + 1)2
3 ⎜⎝ 2 2 ⎠

⇒ x + y = 0, which is a line through the origin.

Objective_Maths_JEE Main 2017_Ch 3.indd 17 01/01/2008 03:26:40


3.18  Chapter 3

46. The centre of a regular polygon of n sides is located at 48. The equation |z – 1|2 + |z + 1|2 = 4 represents on the
the point z = 0, and one of its vertex z1 is known. If z2 Argand plane
be the vertex adjacent to z1, then z2 is equal to (A)  a straight line
⎛ 2p 2p ⎞ (B)  an ellipse
(A) z1 ⎜ cos ± i sin ⎟ (C)  a circle with centre origin and radius 2
⎝ n n⎠
(D)  a circle with centre origin and radius unity
⎛ p p⎞
(B)  z1 ⎜ cos ± i sin ⎟ Solution: (D)
⎝ n n⎠

We have, |z – 1|2 + |z + 1|2 = 4 (1)
⎛ p p⎞
(C) z1 ⎜ cos ± i sin ⎟ 2 2 2 2
⎝ 2n 2n ⎠ ⇒ (x – 1) + y + (x + 1) + y = 4
(D)  None of these (Putting z = x + iy)
⇒ 2 (x2 + y2 + 1) = 4
Solution: (A)
Let A be the vertex with affix z1. There are two possi- \ x2 + y2 = 1 or |z|2 = 1
2p
bilities and can be obtained by rotating z1 through ⇒ |z| = 1 (since |z| cannot be –ve)
either in clockwise or in anti-clockwise direction. n
Thus, the Eq. (1) represents all points z on the circle
12p
± with centre origin and radius unity.
z2 = z1e n   (Q |z2| = |z1|)
49. The locus of the point z satisfying the condition
O
z −1 p
arg = is
z +1 3
(A)  a straight line (B)  circle
(C)  a parabola (D)  None of these

C(z2) B(z2) Solution: (B)


A(z1)
z −1 p
We have, arg =
47. The locus of the complex number z in the Argand z +1 3
1− iz x + iy − 1 p
plane if = 1, is ⇒ arg = (Putting z = x + iy)
z −i x + iy + 1 3
(A) a circle (B)  x-axis y y p
⇒ tan −1 − tan −1 =
(C) y-axis (D)  None of these x −1 x +1 3
Solution: (B) ⎛ z1 ⎞
Let z = x + iy ⎜⎝∵ Arg z = Arg z1 − Arg z2 ⎟⎠
 2
1− iz y y
Given, = 1 −
p
z −i ⇒ tan −1 x − 1 x2 + 1 =
y 3
1− i ( x + iy ) 1+ 2
⇒ = 1
x + iy − i x −1
2y p
1 + y − ix ⇒ = tan = 3
⇒ = 1 2
x + y −12 3
x + i ( y − 1)
2
(1 + y )2 + x 2 ⇒ x2 + y2 – y – 1 = 0, which is a circle.
⇒ = 1 3
x 2 + ( y − 1)2 n
⎛ z −i ⎞
50. If w = ⎜ , n integral, then w lies on the unit
⇒ (1 + y)2 + x2 = x2 + (y – 1)2 ⎝ 1 + iz ⎟⎠
⇒ 1 + y2 + 2 y + x2 = x2 + y2 – 2 y + 1 circle for
⇒ 4 y = 0 (A)  only even n (B)  only odd n
(C)  only positive n (D) all n
⇒ y = 0, which is the equation of x-axis.

Objective_Maths_JEE Main 2017_Ch 3.indd 18 01/01/2008 03:26:43


Complex Numbers  3.19

Solution: (D) 54. Let z1 and z2 be two non real complex cube roots of
⎛ z −i ⎞
n n unity and |z – z1|2 + |z – z2|2 = λ be the equation of a
⎛ z −i ⎞
We have, w= ⎜ = ⎜ circle with z1, z2 as ends of a diameter, then the value
⎝ 1 + iz ⎟⎠ ⎝ i ( z − i ) ⎟⎠
 of λ is
n
⎛ 1⎞ n (A) 4 (B) 3 (C) 2 (D)  2
= ⎜ ⎟ = (–i)
⎝ i⎠
Solution: (B)
\ |w | = |(–i)n| = |–i|n = 1 for all n. We have,
\ w lies on unit circle for all n. |z – ω |2 + |z – ω2|2 = λ
51. The equation z z + a z + a z + b = 0, b ∈ R ­represents ⇒ λ = |ω – ω2|2 = |ω2 + ω4 – 2ω3|
a circle (not point circle) if = |ω2 + ω – 2| = |– 1 – 2| = 3
(A) |a|2 > b (B)  |a|2 < b
(C) |a| > b (D)  |a| < b 55. The region in the Argand diagram defined by |z – 3| +
|z + 3| < 6 is the interior of the ellipse with major axis
Solution: (A) along
We have, z z + a z + a z + b = 0 (A)  real axis (B)  imaginary axis
⇒ z z + a z + a z + a a = a a – b (C) y = x (D)  y=–x
⇒ (z + a) ( z + a ) = a a – b Solution: (A)
⇒ |z + a| = |a| – b 2 2
The equation |z – (3 + 0i)| + |z – (–3 + 0i)| < 6 rep-
This represents a circle (not point circle) if |a|2 > b. resents the interior of ellipse with foci at (3, 0) and
(–3, 0). So, major axis is along real axis.
52. If z4 = (z – 1)4, then the roots are represented in the
argand plane by the points that are 56. If the area of the triangle on the argand plane formed
by the complex numbers –z, iz, z – iz is 600 square
(A) collinear
units, then |z| is equal to
(B) concyclic
(C)  vertices of a parallelogram (A) 10 (B) 20
(D)  None of these (C)  30 (D)  None of these

Solution: (A) Solution: (B)
We have, z4 = (z – 1)4 Area of the triangle on the argand plane formed by the
2 np i
3 2
complex numbers – z, iz, z – iz is |z| .
⎛ z − 1⎞ 1/4 2
⇒ ⎜⎝ z ⎟⎠ = 1 = e
4 ,  n = 0, 1, 2, 3 3 2
\ |z| = 600 ⇒ |z| = 20
Since for all these values of z, 2
z −1 57. If |z + z | + |z – z | = 8, then z lies on
= 1 so they lie on the line bisecting perpendic-
z (A)  a circle
ularly the join of z = 1 and z = 0. (B)  a straight line
(C)  a square
53. The equation z2 + z 2 – 2|z|2 + z + z = 0 represents a (D)  None of these
(A) straight line (B) circle Solution: (C)
(C) hyperbola (D) parabola
We have, |z + z | + |z – z | = 8
Solution: (D)
⇒ 2|x| + 2|y| = 8 or |x| + |y| = 4
We have, z2 + z 2
– 2|z|2 + z + z = 0
⎛ z + 2i ⎞
⇒ (x + iy)2 + (x – iy)2 – 2(x2 + y2) + x + iy + x – iy = 0 58. If Im ⎜ = 0, then z lies on the curve
⎝ z + 2 ⎟⎠
(Putting z = x + iy)
⇒ 2x2 + 2 (iy)2 – 2x2 – 2y2 + 2x = 0 (A) x2 + y2 + 2x + 2y = 0
(B) x2 + y2 – 2x = 0
1
⇒ – 4 y2 + 2x = 0  or  y2 = x, (C) x + y + 2 = 0
2 (D)  None of these
which is a parabola.

Objective_Maths_JEE Main 2017_Ch 3.indd 19 01/01/2008 03:26:47


3.20  Chapter 3

Solution: (C) ⎛ 1 i 3⎞
Let z = x + iy and |1 – w2| = 1 − ⎜ − − 
⎝ 2 2 ⎟⎠
z + 2i x + iy + 2i x + ( y + 2) i
Then, = =
z+2 x + iy + 2 ( x + 2) + iy  3 i 3
= + = 3 .
[ x + ( y + 2) i ] [( x + 2) − iy ] 2 2
=
( x + 2) 2 + y 2  Therefore, 1, w, w2 form an equilateral triangle.
( x 2 + y 2 + 2 x + 2 y ) + i ( 2 x + 2 y + 4) 60. If |z – 1| + |z + 3| ≤ 8, then the range of values of |z – 4|
=
( x + 2) 2 + y 2 is

⎛ z + 2i ⎞ (A)  (0, 8) (B)  [0, 8]
Since Im ⎜ = 0 ⇒ x + y + 2 = 0
⎝ z + 2 ⎟⎠ (C)  [1, 9] (D)  [5, 9]
which represents a straight line. Solution: (C)
59. The cube roots of unity Given |z – 1| + |z + 3| ≤ 8
(A)  lie on the circle |z| = 1 \ z lies inside or on the ellipse whose foci are (1, 0)
(B)  are collinear and (– 3, 0) and vertices are (– 5, 0) and (3, 0).
(C)  form an equilateral triangle Y
(D)  None of these
Solution:  (A, C)
(4, 0)
Clearly, cube roots of unity 1, w, w2 satisfy |z| = 1. X
(–5, 0) (–3, 0) O (1, 0) (3, 0)
2 2
⎛ 3⎞ ⎛ 3⎞
Also, |1 – w |2 = ⎜ ⎟ + ⎜ ⎟ = 3
⎝ 2⎠ ⎝ 2 ⎠
⇒ |1 – w | = 3 Now, |z – 4| is distance of z from (4, 0). Minimum
­distance is 1 and maximum is 9.
|w – w2| = | 3 i| = 3

EXERCISES

Single Option Correct Type

1. If a, b, c, p, q, r are three complex numbers such that 4. The number of solutions of the equation z2 + |z|2 = 0,
p q r a b c where z ∈ C is
+ + = 1 + i and + + = 0, then the value
a b c p q r (A) one (B) two
p2 q2 r 2 (C) three (D) infinitely many
of 2 + 2 + 2 is
a b c 5. If w is the nth root of unity, then
(A) 2i (B)  i (1 + w + w2 + … + wn –1) is
(C) –2i (D)  None of these
(A) 2 (B) 0 (C) 1 (D) –1
2. The complex numbers sin x + i cos 2x and cos x –
6. The complex number which satisfies the equation
i sin 2x are conjugate to each other, for
(A) x = np (B)  x=0 z+ 2 |z + 1| + i = 0 is
⎛ 1⎞ (A)  2 – i (B) 
–2 – i
(C) x = ⎜ n + ⎟ p (D)  no value of x
⎝ 2⎠ (C) 2 + i (D) –2 + i
3. If z1 and z2 are two non-zero complex numbers such 7. z1, z2 are two non-real complex numbers such that
that |z1 + z2| = |z1| + |z2|, then arg z1 – arg z2 is equal to z1 z 2
p p + = 1. Then z1, z2 and the origin
(A) – p (B) – (C)  p (D)  z 2 z1
2 2

Objective_Maths_JEE Main 2017_Ch 3.indd 20 01/01/2008 03:26:49


Complex Numbers  3.21

(A)  are collinear 16. |z1 + z2| = |z1| + |z2| is possible if


(B)  form right angled triangle 1
(A) z2 = z1 (B) z2 =
(C)  form right angle isosceles triangle z1
(D)  form an equilateral triangle (C) arg z1 = arg z2 (D) |z1| = |z2|
⎡ a − ib ⎤ 17. If z = x + iy, x, y real, then |x| + |y| ≤ k |z|, where k is
8. tan ⎢i log is equal to
⎣ a + ib ⎥⎦ equal to
2ab a2 − b 2 (A)  1 (B)  2
(A)  2 (B) 
a + b2 2ab (C)  3 (D)  None of these
2ab 18. If (1 + i) (1 + 2i) (1 + 3i) … (1 + ni) = a + ib then 2 × 5
(C)  (D) 
ab
2
a −b 2 × 10 … (1 + n2) =
(A) a – ib (B)  a2 – b2
9. If ( 3 + i)100 = 299 (a + ib), then b = 2
(C) a + b 2
(D)  None of these
(A)  3 (B)  2
19. Let z1 = a + ib, z2 = p + iq be two unimodular complex
(C)  1 (D)  None of these
numbers such that Im ( z 1z 2 ) = 1.
10. The real value of a for which the expression If w1 = a + ip, w2 = b + iq, then
1 − i sin a
is purely real is (A)  Re (w1 w2) = 1 (B)  Im (w1 w2) = 1
1 + 2i sin a
p p (C)  Re (w1 w2) = 0 (D)  Im (w1 w 2 ) = 1
(A) (2n + 1) (B) (n + 1)
2 2 3 a b
20. If + =
a + ib = x + iy, then
(C) np (D)  None of these x y
11. The locus of z which satisfies the inequality (A)  4 (x2 + y2) (B)  4 (x2 – y2)
2 2
(C)  2 (x – y ) (D)  None of these
log0.3 |z – 1| > log0.3 |z – i| is given by,
(A) x + y > 0 (B)  x – y < 0 21. If z = a + ib where a > 0, b > 0, then
(C) x + y < 0 (D)  x – y > 0 1 1
(A) |z| ≥ (a – b) (B) |z| ≥ (a + b)
2 2
12. If centre of a regular hexagon is at origin and one
of the vertices on argand diagram is 1 + 2i then its 1
(C) |z| < (a + b) (D)  None of these
­perimeter is 2
(A) 2 5 (B) 6 2 (C) 4 5 (D) 6 5 z −z
22. The complex numbers z1, z2 and z3 satisfying 1 3 =
z2 − z3
13. If z1, z2, z3 are three complex numbers, then z1 Im 1 − 3i
( z2 z3 ) + z2 Im ( z 3 z 1 ) + z3 Im ( z 1z 2 ) is equal to are the vertices of a triangle which is
2
(A) 1 (B) –1 (A)  of area zero (B)  right angled isosceles
(C)  0 (D)  None of these (C)  equilateral (D)  obtuse angled isosceles
4
2z 1 z − z2 23. If (1 + x + x2)n = a0 + a1 x + a2 x2 + … + a2n x2n, then
14. If is purely imaginary number, then 1
3z 2 z1 + z 2 a0 + a3 + a6 + … =
is equal to (A) 3n (B)  3n – 1
n–2
3 2 4 (C) 3 (D)  None of these
(A)  (B) 1 (C)  (D) 
2 3 9 24. If 1, a1, a2, …, an – 1 are the n nth roots of unity, then
6 5
15. If x = (4 – 3i) , then the product of all of its roots is (1 – a1) (1 – a2) (1 – a3) … (1 – an – 1) =
(where q = – tan–1 (3/4)) (A) n + 1 (B)  n
(A) 55 (cos 5q + i sin 5q) (C) n – 1 (D)  None of these
(B) –55 (cos 5q + i sin 5q) 25. The closest distance of the origin from a curve given
(C) 55 (cos 5q – i sin 5q) as = 0 (a is a complex number) is
(D) –55 (cos 5q – i sin 5q) |a| Re a Im a
(A) 1 (B)  (C)  (D) 
2 |a| |a|

Objective_Maths_JEE Main 2017_Ch 3.indd 21 01/01/2008 03:26:54


3.22  Chapter 3

26. |z – 1| + |z + 3 | ≤ 8, then the range of values of |z – 4| is 35. The equation z3 + iz – 1 = 0 has


(A)  (0, 8) (B)  [0, 8] (C)  [1, 9] (D)  [5, 9] (A)  three real roots
(B)  one real root
27. The roots of the equation z4 + 1 = 0 are
(C)  no real roots
(A) (± 1 ± i) (B)  (± 2 ± 2i) (D)  no real or complex roots
1
(C)  (± 1 ± i) (D)  None of these 36. If all the roots of z3 + az2 + bz + c = 0 are of unit
2
­modulus, then
28. The integral solution of the equation (1 – i)n = 2n is (A) |a| ≤ 3 (B)  |b| > 3
(A) n = 0 (B)  n=1 (C) |c| ≤ 3 (D)  None of these
(C) n = – 1 (D)  None of these
37. Let z1 and z2 be two complex numbers such that
29. The greatest value of the moduli of complex numbres z1 z2
+ = 1, then
4 z2 z1
z satisfying the equation z− = 2 is
z (A) z1, z2 are collinear
(B) z1, z2 and the origin from a right angled triangle
(A)  5 (B)  5 –1 (C) z1, z2 and the origin form an equilateral triangle
(C)  5 + 1 (D)  None of these (D)  None of these
30. The locus of the complex number z in an argand plane 38. If S (n) = i n + i –n, where i = −1 and n is a positive
satisfying the equation integer, then the total number of distinct values of S (n)
p
Arg (z + i) – Arg (z – i) = is is
2 (A) 1 (B) 2 (C) 3 (D) 4
(A)  boundary of a circle (B)  interior of a circle
(C)  exterior of a circle (D)  None of these 1 1
39. If z1 ≠ –z2 and |z1 + z2| = + , then
z 2 z1 z 2
31. is always real, then
z −1 (A)  at least one of z1, z2 is unimodular
(A) z lies only on a circle (B) z1 × z2 is unimodular
(B) z lies only on the real axis (C) both z1, z2 are unimodular
(C) z lies either on the real axis or on a circle (D)  None of these
(D)  None of these
z − 2z 2 40. If z = x + iy satisfies amp (z – 1) = amp (z + 3i) then the
32. z1 and z2 are two complex numbers such that 1 value of (x – 1) : y is equal to
2 − z 1z 2
(A)  2 : 1 (B)  – 1 : 3
is unimodular whereas z2 is not unimodular. Then |z1| =
(C)  1 : 3 (D)  None of these
(A) 1 (B) 2 (C) 3 (D) 4
41. If z1, z2, z3, z4 are the four complex numbers repre-
33. If for the complex numbers z1 and z2, |z1 + z2| = |z1 – z2|, sented by the vertices of a quadrilateral taken in order
then amp z1 ~ amp z2 = z −z p
p such that z1 – z4 = z2 – z3 and amp 4 1 = then
(A) p (B)  z2 − z1 2
2 the quadrilateral is a
p
(C)  (D)  None of these (A) square
4
(B) rhombus
34. The locus of the complex number z in an argand plane (C) rectangle
satisfying the inequality (D)  a cyclic quadrilateral
⎛ | z − 1| + 4 ⎞ ⎛ 2⎞ 42. Let z be a complex number with modulus 2 and argu-
log1/ 2 ⎜ >1 ⎜⎝ where | z − 1| ≠ 3 ⎟⎠ is
⎝ 3 | z − 1| − 2 ⎟⎠ 2p
ment , then z is equal to
(A)  a circle 3
(B)  interior of a circle (A) –1 + i 3 (B)  1 – i 3
(C)  exterior of a circle
(D)  None of these 1 i 3
(C) − + (D)  None of these
2 2

Objective_Maths_JEE Main 2017_Ch 3.indd 22 01/01/2008 03:26:56


Complex Numbers  3.23

⎛ | z |2 − | z | +1⎞ ⎛ 2k ⎞ ⎛ 2k ⎞
⎟ < 2, then the locus of z is (A) tan–1 ⎜ 2 ⎟ (B) tan–1 ⎜
43. If log
3⎜
⎝ 2 + |z | ⎠ ⎝ k + 1⎠ ⎝ 1 − k 2 ⎟⎠
(A) |z| < 5 (B)  |z| = 5 (C)  –2 tan–1(k) (D)  2 tan–1(k)
(C) |z| > 5 (D)  None of these 24 2
⎛ 1⎞
⎛z
44. If |z| = 1, then the value of ⎜
− 1⎞
is
51. 1 + x2 = ∑ ⎜⎝ x n − x n ⎟⎠ is equal to
3x , then
+ 1⎟⎠
n =1
⎝z
(A) 48 (B)  – 48
(A) 0 (B) purely real (C) ±48(w – w2) (D) 1 ± 48w
(C)  purely imaginary (D)  complex number
52. For all complex numbers z1, z2 satisfying |z1| = 12 and
45. If z1 and z2 are complex numbers, such that z1 + z2 is a |z2 – 3 – 4i| = 5, the minimum value of |z1 – z2| is
real number, then (A) 0 (B) 2 (C) 7 (D) 17
(A) z1 = – z2
(B) z2 = z 1 53. For any two complex numbers z1 and z2 with |z1| ≠ |z2|
2 2
(C) z1 and z2 are any two complex numbers 2 z 1 + i 3z 2 + 3z 1 + i 2 z 2 is
(D) z1 = z1 , z2 = z 2
(A)  less than 5 |z1|2 + |z2|2
46. The locus of the points representing the complex num-
(B)  greater than 10 |z1z2|
bers which satisfy |z| – 2 = 0, |z – i| – |z + 5i| = 0 is:
(C)  equal to 2|z1|2 + 3 |z2|2
(A)  a circle with centre at origin
(D) zero
(B)  a straight line passing through origin
(C)  the single point (0, –2) 54. If the complex numbers z1, z2, z3 are in AP, then they
(D)  None of these lie on a
47. Let the affix of 2 – 4i be P. Then OP is rotated about (A) circle (B) parabola
O through an angle of 180° and is stretched 5/2 times. (C) line (D) ellipse
The complex number corresponding to the new posi- 55. If the roots of (z – 1)25 = 2w2(z + 1)25 where w is a
tion of P is complex cube root of unity are plotted in the argand
(A)  5 – 10i (B) 5 + 10i plane, they lie on
(C) –5 + 10i (D)  None of these (A)  a straight line (B)  a circle
48. If P, P′ represent the complex number z1 and its addi- (C)  an ellipse (D)  None of these
tive inverse respectively then the complex equation of 56. Let A0A1A2A3A4A5 be a regular hexagon inscribed in a
the circle with PP′ as a diameter is circle of unit radius. Then the product of the lengths of
z ⎛z ⎞ the line segments A0A1, A0A2 and A0A4 is
(A)  = ⎜ 1 ⎟ (B)  zz + z 1z 1 = 0 3 3 3
z1 ⎝ z ⎠ (A)  (B)  3 3 (C) 3 (D) 
4 2
(C)  zz1 + zz1 (D)  None of these
57. If z1 and z2 are the two complex roots of equal magni-
49. If a, b, c, p, q, r are three non-zero complex numbers p
tude and their arguments differ by , of the quadratic
p q r a b c 2
such that + + = 1 + i and + + = 0, then
a b c p q r equation ax2 + bx + c = 0 (a ≠ 0) then a (in terms of b
p2 q2 r 2 and c) is
value of 2 + 2 + 2 is b 2 b2
a b c (A)  (B) 
(A) 0 (B) –1 2c c
(C) 2i (D) –2i b
(C)  (D)  None of these
2c
z −z
50. If z1, z2 are two complex numbers such that 1 2 = 1 58. Common roots of the equations z3 + 2z2 + 2z + 1 = 0
z1 + z2
and z1985 + z100 + 1 = 0 are
and tz1 = kz2 where k ∈  , then the angle between
(A) w, w2 (B) 1, w, w2
(z1 – z2) and (z1 + z2) is 2
(C) –1, w, w (D) –w, – w2

Objective_Maths_JEE Main 2017_Ch 3.indd 23 01/01/2008 03:27:00


3.24  Chapter 3

⎡1 ⎤ 68. Two circles in complex plane are


59. sin–1 ⎢ ( z − 1) ⎥ , where z is non-real, can be the angle C1: |z – i| = 2
⎣i ⎦
of a triangle if C2: |z – 1 – 2i| = 4. Then
(A)  Re (z) = 1, Im (z) = 2 (A) C1 and C2 touch each other.
(B)  Re (z) = 1, –1 ≤ Im (z) ≤ 1 (B) C1 and C2 intersect at two distinct points.
(C)  Re (z) + Im (z) = 0 (C) C1 lies within C2.
(D)  Re (z) = Im (z) (D) C2 lies within C1.
5 2
⎛ 1⎞ 1
60. If x2 – x + 1 = 0 then the value of ∑ ⎜ x n + n ⎟ is 69. The conjugate of a complex number is . Then the
n =1
⎝ x ⎠ complex number is i − 1
(A) 8 (B) 10 −1 1 −1 1
(A)  (B)  (C)  (D) 
(C)  12 (D)  None of these i −1 i +1 i +1 i −1
1+ i 4
61. The triangle formed by the points 1, and i as 70. If z − = 2, then the maximum value of |z| is equal to
­vertices in the Argand diagram is 2 z

(A) scalene (B) equilateral (A)  3 + 1 (B) 
5 +1
(C) isosceles (D) right-angled

(C)  2+ 2
2 (D) 
62. If the quadratic equation z2 + (a + ib)z + c + id = 0,
where a, b, c, d are non-zero real numbers, has a real 71. If z1z2 ∈ C, z21 + z22 ∈ R, z1(z21 – 3z22) = 2 and z2(3z21
root, then – z22) = 11, then the value of z21 + z22 is
(A) d2 – abd – c2 = 0 (B)  d2 – abd + b2c = 0 (A) 2 (B) 3 (C) 4 (D) 5
2 2
(C) d + abd + c = 0 (D)  None of these
72. If 1 − C 2 = nc – 1 and z = eiq,
63. If |z – i| ≤ 2 and z0 = 5 + 3i, the maximum value of |iz c ⎛ n⎞
+ z0| is then (1 + nz ) ⎜1 + ⎟ =
2n ⎝ z⎠
(A) 7 (B) 9 (A) 1 + c cosq (B)  1 – c cosq
(C)  13 (D)  None of these (C) 1 + 2c cosq (D)  1 – 2c cosq
( )
64. The solutions of the equation z z − 2i = 2 (2 + i) are 73. Let ‘a’ be a complex number such that | a | < 1 and z1,
z2,..., zn be the vetices of a polygon such that zk = 1 +
(A) 3 + i, 3 – i (B) 1 + 3i, 1 – 3i
(C) 1 + 3i, 1 – i (D)  1 – 3i, 1 + i a + a2 + … + ak, then the vertices of the polygon lie
within the circle
65. Let a, b be real and z be a complex number. If z2 + a z 1 1
+ b = 0 has two distinct roots on the line Re z = 1, then (A) | z | = (B)  |z – a | =
|1 − a | |1 − a |
it is necessary that
(A) b ∈ (1, ∞) (B)  b ∈ (0, 1) 1 1
(C)  z − = (D)  None of these
(C) b ∈ (–1, 0) (D)  | b | = 1 1− a |1 − a |
66. If w (≠1) is a cube root of unity, and (1 + w)7 = A + Bw. 74. All the roots of the equation a1z3 + a2z2 + a3z + a4 = 3,
Then (A, B) equals where |ai| ≤ 1, i = 1, 2, 3, 4 lie outside the circle with
(A)  (–1, 1) (B)  (0, 1) (C)  (1, 1) (D)  (1, 0) centre origin and radius
1 2
z2 (A)  (B) 
67. If z ≠ 1 and is real, then the point represented by 3 3
z −1 (C)  1 (D)  None of these
the complex number z lies
(A)  either on the real axis or on a circle passing 75. If z4 = (z – 1)4, then the roots are represented in the
through the origin. argand plane by the points that are
(B)  on a circle with centre at the origin. (A) collinear
(C) either on the real axis or on a circle not passing (B) concyclic
through the origin. (C)  vertices of a parallelogram
(D)  on the imaginary axis. (D)  None of these

Objective_Maths_JEE Main 2017_Ch 3.indd 24 01/01/2008 03:27:03


Complex Numbers  3.25

76. The maximum value of |z| when z satisfies the ­condition 86. The locus of the complex number z in an argand plane
2 satisfying the equation
z+ = 2 is p
z Arg (z + i) – Arg (z – i) = is
2
(A)  3 – 1 (B)  3+1 (A)  boundary of a circle (B)  interior of a circle
2+ 3
(C)  3 (D)  (C)  exterior of a circle (D)  None of these

77. If |z + z | + |z – z | = 8, then z lies on 87. If for the complex numbers z1 and z2, |z1 + z2| = |z1 – z2|,
then amp z1 ~ amp z2 =
(A)  a circle (B)  a straight line p
(C)  a square (D)  None of these (A) p (B) 
2
p
78. The complex number which satisfies the equation (C)  (D)  None of these
4
z + 2 |z + 1| + i = 0 is 88. The locus of the complex number z in an argand plane
(A)  2 – i (B)  –2 – i satisfying the inequality
(C) 2 + i (D)  –2 + i
⎛ | z − 1| + 4 ⎞ ⎛ 2⎞
log1/ 2 ⎜ ⎟ > 1 ⎜ where | z − 1| ≠ ⎟ is
⎡ a − ib ⎤ ⎝ 3 | z − 1| − 2 ⎠ ⎝ 3⎠
79. tan ⎢i log is equal to
⎣ a + ib ⎥⎦ (A)  a circle (B)  interior of a circle
2ab a2 − b 2 (C)  exterior of a circle (D)  None of these
(A)  2 (B) 
a + b2 2ab
89. Let z1 and z2 be two complex numbers such that
2ab z1 z 2
(C)  (D)  ab + = 1, then
a2 − b 2 z 2 z1
80. |z1 + z2| = |z1| + |z2| is possible if (A) z1, z2 are collinear
1 (B) z1, z2 and the origin from a right angled triangle
(A) z2 = z 1 (B)  z2 =
z1 (C) z1, z2 and the origin form an equilateral triangle
(C) arg z1 = arg z2 (D) |z1| = |z2| (D)  None of these

81. If z = x + iy, x, y real, then |x| + |y| ≤ k |z|, where k is 90. If P, P′ represent the complex number z1 and its addi-
equal to tive inverse respectively, then the complex equation of
the circle with PP′ as a diameter is
(A)  1 (B)  2
(C)  3 (D)  None of these z ⎛z ⎞
(A)  = ⎜ 1⎟ (B)  zz + z 1z 1 = 0
z1 ⎝z ⎠
82. If z = a + ib where a > 0, b > 0, then
1 1 (C) zz 1 + zz 1 (D)  None of these
(A) |z| ≥ (a – b) (B) |z| ≥ (a + b)
2 2 91. If a, b, c, p, q, r are three non-zero complex numbers
1 p q r a b c
(C) |z| < (a + b) (D)  None of these such that + + = 1 + i and + + = 0, then
2 a b c p q r
p2 q2 r2
83. If (1 + x + x2)n = a0 + a1 x + a2 x2 + … + a2n x2n, then value of 2 + 2 + 2 is
a0 + a3 + a6 + … = a b c
(A) 0 (B) –1 (C) 2i (D) –2i
(A) 3n (B)  3n – 1
(C) 3 n–2
(D)  None of these z1 − z2
92. If z1, z2 are two complex numbers such that =1
84. The closest distance of the origin from a curve given z1 + z2
as a z + a z + aa = 0 (a is a complex number) is and tz1 = kz2 where k ∈ R, then the angle between
|a| Re a Im a (z1 – z2) and (z1 + z2) is
(A) 1 (B)  (B)  (D) 
2 |a| |a| ⎛ 2k ⎞ ⎛ 2k ⎞
n n (A) tan–1 ⎜⎝ k 2 + 1⎟⎠ (B) tan–1 ⎜⎝ 1 − k 2 ⎟⎠
85. The integral solution of the equation (1 – i) = 2 is
(A) n = 0 (B)  n=1 (C)  –2 tan–1(k) (D)  2 tan–1(k)
(C) n = – 1 (D)  None of these

Objective_Maths_JEE Main 2017_Ch 3.indd 25 01/01/2008 03:27:08


3.26  Chapter 3

24 2 100. If a, b, c are real, a2 + b2 + c2 = 1 and b + ic = (1 + a)z,


⎛ n 1⎞
93. 1 + x = 2 3x , then

⎜⎝ x − n ⎟⎠ is equal to
x then
1 + iz
=
n =1 1 − iz
(A) 48 (B) –48 a − ib a + ib
(C) ±48(w – w2) (D) 1 ± 48w (A)  (B) 
1+ c 1+ c
94. For any two complex numbers z1 and z2 with |z1| ≠ |z2| a + ib a − ib
2 2
(C)  (D) 
2 z 1 + i 3z 2 + 3z 1 + i 2 z 2 is 1 − c 1− c
101. If z1, z2 are two complex numbers and wk, k = 0, 1, …,
(A)  less than 5 (|z1|2 + |z2|2) n −1
(B)  greater than 10 |z1z2| n – 1 are the nth roots of unity, then ∑ | z1 + z2 w k |2
k =0
(C)  equal to 2|z1|2 + 3 |z2|2
(A) < n (|z1|2 + |z2|2) (B)  = n (|z1|2 + |z2|2)
(D) zero
(C) > n (|z1|2 + |z2|2) (D)  can’t say
95. If the roots of (z – 1)25 = 2w2(z + 1)25 (where w is a
complex cube root of unity) are plotted in the argand 102. The equation |z – z1|2 + |z – z2|2 = k, k ∈ R represents
plane, they lie on a circle if
1 1
(A)  a straight line (B)  a circle (A) k ≥ | z 1 − z 2 |2 (B)  k ≤ | z1 − z2 |2
(C)  an ellipse (D)  None of these 2 2
1 1
96. If z1 and z2 are the two complex roots of equal magni- (C) k ≥ | z 1 + z 2 |2 (D)  k ≤ | z 1 + z 2 |2
p 2 2
tude and their arguments differ by , of the quadratic
2 103. f (z) when divided by z – i gives remainder i; when
equation ax2 + bx + c = 0 (a ≠ 0) then a (in terms of b divided by z + i gives remainder i + 1. When f (z) is
and c) is divided by z2 + 1, the remainder is
b 2 b2 i ⎛ 1⎞ i ⎛ 1⎞
(A)  (B)  (A)  z + ⎜ i − ⎟ (B)  z − ⎜ i + ⎟
2c c 2 ⎝ 2⎠ 2 ⎝ 2⎠
b i ⎛ 1⎞ −i ⎛ 1⎞
(C)  (D)  None of these (C)  z + ⎜ i + ⎟ (D)  z + ⎜ i + ⎟
2c 2 ⎝ 2⎠ 2 ⎝ 2⎠
z −z
97. The complex numbers z1, z2 and z3 satisfying 1 3
z2 − z3 104. The value of the expression (w – 1) (w – w2) (w – w3)
1 − 3i … (w – wn–1), where w is the nth root of unity, is
= are the vertices of a triangle which is
2 (A) nwn–1 (B)  nwn
(A)  of area zero (C) (n – 1) w n
(D) (n – 1) wn–1
(B)  right angled isosceles
(C) equilateral ⎛ p⎞
105. If |z – i| = 1 and arg (z) = q, q ∈ ⎜ 0, ⎟ , then the value
(D)  obtuse angled isosceles ⎝ 2⎠
2
of cot q − is equal to
98. z1, z2 are two non-real complex numbers such that z
z1 z 2 (A) 0 (B) i (C) – i (D) 1
+ = 1. Then, z1, z2 and the origin
z 2 z1 4 + 3i
106. The reflection of the complex number in the
(A)  are collinear straight line iz = is 1 + 2i
(B)  form right angled triangle
(A) 2 + i (B)  2–i
(C)  form right angle isosceles triangle
(C) 1 + 2i (D)  1 – 2i
(D)  form an equilateral triangle
107. If i = −1 , then
99. If z1 and z2 (±0) are two complex numbers such that
334 365
z1 − z2 ⎛ 1 i 3⎞ ⎛ 1 i 3⎞
= 1, then 4 + 5 ⎜ − + +3 ⎜− + is equal to
z1 + z2 ⎝ 2 2 ⎟⎠ ⎝ 2 2 ⎟⎠
(A) z2 = ikz1, k ∈ R (B)  z2 = kz1, k ∈ R (A)  1 – i 3 (B)  – 1 + i 3
(C) z2 = z1 (D)  None of these (C) i 3 (D)  – 3i

Objective_Maths_JEE Main 2017_Ch 3.indd 26 01/01/2008 03:27:13


Complex Numbers  3.27

108. Let b z + b z = c, b ≠ 0, be a line in the complex (A)  0 (B)  real and positive
plane, where b is the complex conjugate of b. If a (C)  real and negative (D)  purely imaginary
point z1 is the reflection of a point z2 through the line, 116. If the complex numbers z1, z2, z3 are the vertices A,
then z 1 b + z2 b = B, C respectively of an isosceles right angled triangle
(A) 4c (B)  2c with right angle at C, then
(C) c (D)  None of these (z1 – z2)2 = k (z1 – z3) (z3 – z2), where k =
109. Let z1 and z2 be roots of the equation z2 + pz + q = 0, (A) 1 (B) 2
where the coefficients p and q may be complex (C)  4 (D)  None of these
­numbers. Let A and B represent z1 and z2 in the com-
117. If the origin and the two points represented by com-
plex plane. If ∠AOB = a ≠ 0 and OA = OB, where O
a plex numbers A and B form vertices of an equilateral
is the origin, then p2 = k cos2 , where k = A B
2 triangle, then + =
(A)  q (B)  2q B A
(C) 4q (D)  None of these (A) 1 (B) – 1
(C)  2 (D)  None of these
110. If z1, z2, z3 are complex numbers such that |z1| = |z2| =
118. If 2 2 x 4 = ( 3 – 1) + i ( 3 + 1), then
1 1 1
|z3| = + + = 1, then |z1 + z2 + z3| is 1 1
z1 z2 z3 x = cos (2np + k) + i sin (2np + k);
4 4
(A)  equal to 1 (B)  less than 1 n = 0, 1, 2, 3, where k =
(C)  greater than 3 (D)  equal to 3
p 5p
(A)  (B) 
111. If |z| ≤ 1, |w | ≤ 1, then |z – w |2 12 12
(A) ≤ (|z| – |w |)2 – (Arg z – Arg w)2 7p
(B) ≤ (|z| – |w |)2 + (Arg z – Arg w)2 (C)  (D)  None of these
12
(C) ≤ (|z| – |w |)2 + 2 (Arg z – Arg w)2 p
(D)  None of these 32 ⎡ 10 ⎛ 2q p 2q p ⎞ ⎤
112. Suppose, z1, z2, z3 are the vertices of an equilateral
119. ∑ ∑
(3 p + 2) ⎢ ⎜ sin
⎢⎣ q =1 ⎝ 11
− i cos ⎥ =
11 ⎟⎠ ⎥
p =1 ⎦
triangle inscribed in the circle |z| = 2. If z1 = 1 + i 3 (A)  8 (1 – i) (B)  16 (1 – i)
then z2 and z3 are equal to (C)  48 (1 – i) (D)  None of these
(A)  – 2, 1 – i 3 (B)  2, 1 – i 3
120. The three vertices of a triangle are represented by
(C)  – 2, 1 + i 3 (D)  None of these the complex numbers 0, z1 and z2. If the triangle is
3n ­equilateral, then
113. If k = , where n is an even positive integer, then
2 (A) z 12 + z 22 + z 1 z 2 = 0 (B)  z 12 + z 22 = z1 z2
k
(C) z 22 − z 12 = z1 z2 (D) z 12 − z 22 = z1 z2
∑ (−3)r −1 ⋅ 3n C2r −1 =
r =1 121. If |z – 25i| ≤ 15, then |maximum amp (z) – minimum
(A) 0 (B) 1 amp (z)| is equal to
(C)  – 1 (D)  None of these
⎛ 3⎞ ⎛ 3⎞
(A) sin–1 ⎜ ⎟ – cos–1 ⎜ ⎟
114. If a and b are real numbers between 0 and 1 such that ⎝ 5⎠ ⎝ 5⎠
the points z1 = a + i, z2 = 1 + bi and z3 = 0 form an p ⎛ 3⎞
equilateral triangle, then a and b are (B)  + cos −1 ⎜ ⎟
2 ⎝ 5⎠

(A) 2 + 3 , 2 – 3 (B)  2 – 3,2– 3 ⎛ 3⎞
(C) p – 2 cos–1 ⎜ ⎟
(C)  2 – 3 , 2 + 3 (D)  None of these ⎝ 5⎠
115. Let z1 and z2 be complex numbers such that z1 ≠ z2 ⎛ 3⎞
(D) cos–1 ⎜ ⎟
and |z1| = |z2|. If z1 has positive real part and z2 has ⎝ 5⎠
z + z2
negative imaginary part, then 1 may be 122. If z2 + (p + iq) z + r + is = 0 where p, q, r, s are
z1 − z 2 ­non-zero, has real roots, then

Objective_Maths_JEE Main 2017_Ch 3.indd 27 01/01/2008 03:27:17


3.28  Chapter 3

(A) pqs = s2 + q2r pqr = r2 + p2s


(B)  1 1
(A) |z – a| = (B) |z – 1| =
(C) prs = q2 + r2p qrs = p2 + s2q
(D)  |1 − a | |1 − a |
123. If z1 and z2 are any two complex numbers, then 1 1
(C) z − = (D)  None of these
1− a |1 − a |
z 1 + z 12 − z 22 + z1 − z 12 − z 22 is equal to
131. If A, B, C are the angles of a triangle and eiA, eiB, eiC
(A) |z1 + z2| (B)  |z1| are in A.P., then the triangle must be
(C) |z2| (D)  None of these (A)  right angle (B)  isosceles triangle
z (C)  equilateral (D)  None of these
124. If z = x + iy lies in IIIrd quadrant, then also lies in m
the IIIrd quadrant if z −1 ⎛ pi + 1⎞
132. e 2 mi cot p
⋅⎜ =
(A) y > x > 0 (B)  y < x < 0 ⎝ pi − 1⎟⎠
(C) x < y < 0 (D)  x > y > 0 (A) 0 (B) 1
125. If in an argand plane points z1, z2, z3 are the vertices (C)  – 1 (D)  None of these
of an isosceles triangle right angled at z2, then 133. If z1 and z1 represent adjacent vertices of a regular
(A) z12 + 2 z22 + z32 = 2 z2 (z1 + z3) Im ( z 1 )
polygon of n sides and if = 2 – 1, then n
(B) z 12 + z 22 + z 32 = 2 z2 (z1 + z3) is equal to Re ( z 1 )
(C) z 12 + z 22 + 2 z 32 = 2 z2 (z1 + z3) (A) 4 (B) 8
(D) 2 z12 + z22 + z32 = 2 z2 (z1 + z3) (C)  16 (D)  None of these

126. In the Argand diagram, if O, P and Q represent 134. If z1, z2, z3 are non-zero, non-collinear complex num-
respectively the origin and the complex numbers z 2 1 1
bers such that = + , then the points z1, z2, z3
and z + iz, then the ∠OPQ is z1 z 2 z 3
p p p 2p lie
(A)  (B)  (C)  (D)  (A)  in the interior of a circle
4 3 2 3
(B)  on a circle passing through origin
1 27. If z satisfies |z + 1| < |z – 2|, and w = 3z + 2 + i, then (C)  in the exterior of a circle
(A) |w + 1| < |w – 8| (B)  |w + 1| < |w – 7| (D)  None of these
(C) w + w > 7 (D)  |w + 5| < |w – 4| 135. If |z – 25 i| ≤ 15, then the least positive value of arg
128. If P (x) and Q (x) are two polynomials such that f (x) = z is 4
4
P (x3) + x Q (x3) is divisible by x2 + x + 1, then (A)  p – tan– 1 (B) tan– 1 3
3
(A)  P (x) is divisible by (x – 1) but Q (x) is not divis-
4
ible by x – 1 (C) –p + tan– 1 (D)  None of these
(B)  Q (x) is divisible by (x – 1) but P (x) is not divis- 3
ible by x – 1 136. If |z – 4 + 3i| ≤ 2, then the least and the greatest ­values
(C) Both P (x) and Q (x) are divisible by x – 1 of |z| are
(D) f (x) is divisible by x – 1 (A)  3, 7 (B)  4, 7
⎛ 8p ⎞ ⎛ 8p ⎞ (C)  3, 9 (D)  None of these
129. If a = cos ⎜ ⎟ + i sin ⎜ ⎟ , then
⎝ 11 ⎠ ⎝ 11 ⎠ 137. If z1, z2 are two complex numbers and c > 0 such that
Re (a + a2 + a3 + a4 + a5) is equal to |z1 + z2|2 ≤ (1 + c) |z1|2 + k |z2|2, then k =
1 1
(A)  (B)  – (A)  1 – c (B) 
c – 1 (C)  1 + c–1 (D)  1 – c–1
2 2
(C)  0 (D)  None of these 138. If |z – 4 + 3i| ≤ 1 and m and n are the least and greatest
x4 + x2 + 4
130. Let p be a complex number such that |a| < 1 and z1, values of |z| and k is the least value of on
z2, …, zn be the vertices of a polygon such that zk = 1 the interval (0, ∞), then k is equal to x
+ a + a2 + … a k, then the vertices of the polygon lie (A)  m (B)  n
within the circle (C)  m + n (D)  None of these

Objective_Maths_JEE Main 2017_Ch 3.indd 28 01/01/2008 03:27:21


Complex Numbers  3.29

1 39. If n > 1, then the roots of zn = (z + 1)n lie on a 1


1 46. If a, b are the roots of z + = 2(cos q + sin q ) Then,
(A) circle z
(A) |a – i| > |b – i| (B) |a – i| < |b – i|
(B)  straight line
(C) |a – i| = |i – b | (D) |a – i| = |b – i|
(C) parabola
(D)  None of these 147. If at least one value of the complex number z = x + iy
2
140. Let z be a complex number satisfying z + z + 1 = 0. satisfies the condition z + 2 = a2 – 3a + 2 and the
If n is not a multiple of 3, then the value of zn + z2n = inequality z + i 2 < a2 , then
(A) 2 (B) –2
(C) 0 (D) –1 (A) a > 2 (B)  a=2
(C) a < 2 (D)  None of these
141. If 1, a1, a2, a3 and a4 be the roots of x5 – 1 = 0, then
148. If a is the nth root of unity, then 1 + 2a + 3a2 + … to
w −a w −a w −a w −a n terms is equal to
2 1 ⋅ 2 2 ⋅ 2 3 ⋅ 2 4 =
w − a1 w − a 2 w − a 3 w − a 4 n n
(A) − (B)  −
(A) 1 (B)  w (1 − a ) 2 1 − a
w2
(C)  (D)  None of these 2n 2n
(C) − (D)  −
142. If z1 and z2 both satisfy the relation z + z = 2|z – 1| 1−a (1 − a ) 2
p
and arg (z1 – z2) = , then the imaginary part of 149. Let O, A, B be three collinear points such that
4 OA  ·  OB  = 1. If O and B represent the complex
(z1 + z2) is
(A) 0 (B) 1 ­numbers o and z, then A represents
(C)  2 (D)  None of these 1 1
(A)  (B) 
z z
143. All the roots of the equation a1 z3 + a2 z2 + a3 z +
a4  =  3, where |ai| ≤ 1, i = 1, 2, 3, 4, lie outside the (C) z (D)  z2
circle with centre origin and radius
150. ABCD is a rhombus. Its diagonals AC and BD inter-
1 sect at the point M and satisfy BD = 2AC. If the points
(A)  1 (B) 
3 D and M represent the complex numbers 1 + i and
2
(C)  (D)  None of these 2  –  i, respectively, then A represents the complex
3 number
1 1 1 1 i i i 3
144. If + + + =, where a, b, c are (A) 3 − or 3 + (B)  3 + or 1 + i
a+w b +w c +w d +w 2 2 2 2
real and w is a non-real cube root of unity, then (C)  3 – i or 1 – 3i (D)  None of these
(A) a + b + c + d = – 2abcd
151. The locus represented by the complex equation
1 1 1 1
(B)  + + + =2 ⎛p ⎞
1+ a 1+ b 1+ c 1+ d |z – 2 – i| = | z | sin ⎜ − arg z ⎟ is the part of
⎝4 ⎠
1 1 1 1 2
(C)  2
+ 2
+ 2
+ 2
=– 2 (A)  a pair of straight lines
a+w b +w c +w d +w w
(B)  a circle
(D) abc + bcd + abd + acd = 4
(C)  a parabola
145. If z1 + z2 + z3 = A, z1 + z2 w + z3 w2 = B and z1 + z2 (D)  a rectangular hyperbola
w2 + z3 w = C, where 1, w, w2 are the three cube roots 152. If z1, z2, z3 are three points lying on the circle |z| = 2,
of unity, then |A|2 + |B|2 + |C|2 =
then the minimum value of |z1 + z2|2 + |z2 + z3|2 +
(A)  3 (|z1|2 + |z2|2 + |z3|2) |z3 + z1|2 is equal to
(B)  2 (|z1|2 + |z2|2 + |z3|2) (A) 6 (B) 12
(C) (|z1|2 + |z2|2 + |z3|2) (C) 15 (D) 24
(D)  None of these

Objective_Maths_JEE Main 2017_Ch 3.indd 29 01/01/2008 03:27:25


3.30  Chapter 3

More than One Option Correct Type

153. The centre of a regular polygon of n sides is located 160. If |z1 – z2| = |z1| + |z2|, then
at the point z = 0, and one of its vertex z1 is known. If
⎛z ⎞ p
z2 be the vertex adjacent to z1, then z2 is equal to (A) arg ⎜ 1 ⎟ =
⎝ z2 ⎠ 2
⎛ 2p 2p ⎞
(A) z 1 ⎜ cos + i sin
⎝ n n ⎟⎠ ⎛z ⎞
(B) arg ⎜ 1 ⎟ = (2n + 1)p, n ∈ I
⎛ p p⎞ ⎝ z2 ⎠
(B) z1 ⎜ cos + i sin ⎟ (C) z 1z 2 + z 1z 2 ≤ 0
⎝ n n⎠
(D) z1 = l z2, l ∈ R
⎛ 2p 2p ⎞
(C) z 1 ⎜ cos − i sin
⎝ n n ⎟⎠ 161. If z1 = a + ib and z2 = c + id are two complex numbers
such that |z1| = |z2| = 1 and Re (z1 · z 2 ) = 0 then for the
⎛ p p⎞
(D) z 1 ⎜ cos − i sin ⎟ pair of complex numbers w1 = a + ic and w2 = b + id
⎝ n n⎠
(A)  Re (w1 w ) = 0 (B)  Re (w1 w ) = 1
2 2
154. i − −i is equal to (C) |w1| = 1 (D)  None of these
1
(A) i 2 (B)  (C) 0 (D) – i 2 162. If z1, z2, z3 are the vertices of an equilateral triangle in
i 2
the complex plane and z0 is the centroid, then
155. If z1, z2, z3, z4 are the four complex numbers repre-
1 1 1
sented by the vertices of a quadrilateral taken in order (A)  + + =0
z −z p z1 − z 2 z 2 − z 3 z 3 − z1
such that z1 – z4 = z2 – z3 and amp 4 1 = then
z 2 − z1 2 (B) (z1 – z2)2 + (z2 – z3)2 + (z3 – z1)2 = 0
the quadrilateral is a
(A) square (C) z12 + z22 + z32 = 3 z 02
(B) rhombus (D) z 12 + z 22 + z 32 = z1 z2 + z2 z3 + z3 z1
(C) rectangle
(D)  a cyclic quadrilateral 163. If a, b, c, …, k are the roots of the equation
xn + p1 xn – 1 + p2 xn – 2 + … + pn – 1 x + pn = 0
156. The sum
m (p1, p2, …, pn are real) and (1 + a2) (1 + b2) … (1 + k2)
4 n +1 ⎡ m + 1
⎧ ⎛ 2p k ⎞ ⎛ 2p k ⎞ ⎫⎤
∑ ⎢∑ ⎨sin ⎜ ⎟ − i cos ⎜
⎝ m ⎟⎠ ⎭⎥
⎬⎥ is = x2 + y2, then
m =1 ⎢
⎣ k =1m ⎩ ⎝ m ⎠ ⎦ (A) x = 1 – p2 + p4 …

(A)  independent of n (B) y = p1 – p3 + p5 – …
(B)  purely imaginary (C) x = 1 + p2 + p4 + …
(C)  purely real (D) y = p1 + p3 + p5 + ….
(D)  a root of x4n + 1 + 1 = 0 164. If z1, z2, z3 and z4 are the vertices of a square PQRS in
157. z1 = a + ib and z2 = c + id are complex numbers such order, then
that |z1| = |z2| = 1 and Re ( z 1z 2 ) = 0. If w1 = a + ic and (A) z4 + z2 = z3 + z1
w2 = b + id (a, b, c, d ∈ R), then (B) |z1 – z2| = |z2 – z3| = |z3 – z4| = |z4 – z1|
(A) |w1| = 1 (B)  |w2| = 1 (C) |z3 – z1| = |z4 – z2|
(C) Re ( w1w2 ) = 0 (D)  Re ( w1w2 ) = 1 z − z3
(D)  The real part of 1 is zero
z2 − z4
1
arg ( z 2 + z z 1/ 2 ), then
158. If arg (z3/8) = 165. If z1, z2, z3 are the vertices of an isosceles triangle and
2 right angled at z2, then
(A) |z| = 1 (B)  z=
(C) Re (z) = 0 (D)  Im (z) = 0 (A) z12 + z32 + 2z22 = 2 (z1 + z3) z2
159. If z12 + 2z22 + z32 = 2z2(z1 + z3), where z1, z2, z3 are the (B) z12 + z32 = 2 z2(z1 + z3 – z2)
vertices of a triangle, then the triangle is (C) (z1 – z2)2 + (z2 – z3)2 = 0
(A) isosceles (B) right angled z − z2
(C) equilateral (D) obtuse angled (D)  1 is imaginary
z2 − z3

Objective_Maths_JEE Main 2017_Ch 3.indd 30 01/01/2008 03:27:29


Complex Numbers  3.31
⎛z ⎞ p
166. A, B, C are the points representing the complex (C) arg ⎜ 1 ⎟ = ±
⎝ z2 ⎠ 2
numbers z1, z2, z3, respectively on the complex
plane and the circumcentre of the triangle ABC lies (D) DOAB is isosceles
at the o­ rigin. If the altitude AD of the triangle ABC 168. If f (x) and g(x) are two polynomials such that the
meets ­circumcircle again at P, then P represents the polynomial h (x) = x f (x3) + x2g(x6) is divisible by
­complex number x2 + x + 1, then
zz
(A) − z 1z 2 z 3 (B)  − 1 2 (A) f (1) = g (1) (B)  f (1) = – g(1)
z3
(C) h(1) = 0 (D)  h(–1) = 0
z 1z 3 z2 z3
(C) − (D)  −
z2 z1 1 69. If a is the fifth root of unity, then
167. If the points A and B are represented by the non-zero (A) |1 + a + a2 + a3 + a4| = 0
complex numbers z1 and z2 on the argand plane such (B) |1 + a + a2 + a3| = 1
that |z1 + z2| = |z1 – z2| and O is the origin, then p
(C) |1 + a + a2| = 2 cos
(A)  orthocentre of DOAB lies at O 5
p
z1 + z 2 (D) |1 + a| = 2 cos
(B)  circumcentre of DOAB is 10
2

Passage Based Questions


Passage 1 ⎛m b⎞
(C) (a2 + b2)m/2n cos ⎜ tan −1 ⎟
If n be a rational number, then ⎝n a⎠
cos nq + i sin nq (D)  None of these
is the value or one of the values of (cos q + i sin q)n. 1 72. The values of (16)1/4 are
If p and q be the integers prime to each other (q ≠ 0), (A)  ±2, ±2 i (B)  ±4, ±4 i
then all the values of (cos q + i sin q) p/q are given by (C) ±1, ±i (D)  None of these
⎡ p⎤ ⎡ p⎤ 173. The roots of the equation z4 + 1 = 0 are
cos ⎢( 2k p + q ) ⎥ + i sin ⎢( 2k p + q ) ⎥ ,
⎣ q⎦ ⎣ q⎦ (A) (±1 ± i) (B) (±2 ± 2i)
1
where k = 0, 1, 2, … q – 1. (C)  (±1 ± i) (D)  None of these
There are apparently two different ways of calculat- 2
ing the values of (cos q + i sin q) p/q, (where p and q are Passage 2
integers prime to each other and q ≠ 0), namely,
If z = r (cos q + i sin q) and n is a positive integer, then
1. by writing (cos q + i sin q)p/q as
⎡ ⎛ 2k p + q ⎞ ⎛ 2k p + q ⎞ ⎤
{(cos q + i sin q)p}1/q, z1/n = r1/ n ⎢cos ⎜ ⎟ + i sin ⎜ ,
2. by writing (cos q + i sin q)p/q as ⎣ ⎝ n ⎠ ⎝ n ⎟⎠ ⎥⎦
{(cos q + i sin q)1/q}p. where k = 0, 1, 2, 3, … (n – 1), gives n, nth roots of the
complex number z.
It can be easily seen that if p and q are prime to each other,
In particular, since 1 = cos 0 + i sin 0, therefore
the q values in each case are the same, so that each of the
two ways will yield the same result. (1)1/n = (cos 0 + i sin 0)1/n
16 2p r + 0 2p r + 0
⎛ 2 rp 2 rp ⎞ = cos + i sin ; r = 0, 1, 2, …, (n – 1)
170. The value of ∑ ⎜⎝ sin 17
+ i cos
17 ⎟⎠
is n n
r =1 2p r 2p r
= cos + i sin ; r = 0, 1, 2, …, (n – 1)
(A) 1 (B) –1 (C) i (D) –i n n
m/n 2 rp
171. One of the values of (a + ib) + (a – ib)m/n is i
=e n ; r = 0, 1, 2, …, (n – 1)
⎛m b⎞
(A)  2 (a + b ) 2 2 m/n
cos ⎜ tan −1 ⎟ = 1, e p , e(i 4p/n), …, e(i 2(n–1)p/n)
(i 2 /n)
⎝n a⎠
⎛m b⎞ = 1, a, a2, a3, … an–1, where a = e(i 2p/n)
(B)  2 (a2 + b2)m/2n cos ⎜ tan −1 ⎟
⎝n a⎠ These are the n, nth roots of unity.

Objective_Maths_JEE Main 2017_Ch 3.indd 31 01/01/2008 03:27:32


3.32  Chapter 3

Clearly, z7 + 1 = 0 ⇒ z7 = – 1 = cis (2p + 1)p, where p is an


1. 1 + a + a2 + … + a n–1 = 0 integer.
2. 1 · a · a … an–1 = (– 1)n–1 Therefore, z = cis [(2p + 1)p/7], p = 0, 1, …, 6
3. nth roots of unity form a G.P. with common ratio On putting p = 0, 1, 2, 3, 4, 5, 6, the roots are seen
e(i2p/n). to be cos (p/7) ± i sin (p/7), cos (3p/7) ± i sin (3p/7), cos
(5p/7) ± i sin (5p/7), –1.
174. If 1, w, w2, … wn – 1 are the n, nth roots of unity and Equations of the type pz2n + qzn + r = 0, where p, q
z1 and z2 are any two complex numbers, then and r are complex numbers and p ≠ 0.
n −1
∑ | z 1 + w k z 2 |2 =
zn =
−q± q 2 − 4 pr
k =0 2p
2 2
n [|z1| + |z2| ]
(A)  Denoting these values of zn by a and b, we have two equa-
(B) (n – 1) [|z1| + |z2| ] 2 2 tions zn = a and zn = b, each of which can be solved by the
(C) (n + 1) [|z1|2 + |z2|2] method given in the above example.
(D)  None of these Equations of the type a (pz + q)n + b (rz + s)n = 0:
175. If 1, a1, a2, …, an – 1 are the n, nth roots of unity, then pz + q
The substitution = w reduces the given equation
(1 – a1) (1 – a2) (1 – a3) … (1 – an – 1) = to the form rz + s
(A) n + 1
awn + b = 0 (i)
(B)  n
(C)  n–1 which can be solved by the method given above. If wk be
(D)  None of these. a root of the equation (i), the corresponding root zk of the
given equation is obtained by solving the equation
176. If 1, a, a2, …,an – 1 are the n nth roots of unity then
n −1 pz k + q
1 = wk.
∑ 2 − ai
is equal to rz k + s
i =1
178. The roots of the equation x6 + x3 + 1 = 0 are
( n − 2 )2 n − 1 + 1 n
(A)  (B) (n – 2) × 2 ⎛ pp ⎞ ⎛ pp ⎞
2n − 1 cos ⎜ ⎟ ± i sin ⎜ ⎟ , where p =
⎝ 9 ⎠ ⎝ 9 ⎠
( n − 2) ⋅ 2 n − 1
(C)  (D)  None of these (A) 2 (B) 8 (C) 14 (D) 20
2n − 1
179. The roots of the equation z4 + 4z2 + 16 = 0 are
177. If 1, w, w2, …, wn – 1 are the n, nth roots of unity, then
(1 – w) (1 – w)2 … (1 – wn – 1) is equal to (A) ± 1 + i 3 ±1− i
(B)  3
(A) 0 (B) 1 (C) ± 2 + 2 3 i (D) ±2−2 3i
(C)  n (D)  n2
1 80. The roots of the equation (2 + z)6 + (2 – z)6 = 0 are
Passage 3 (A) ±2i tan p/12
Solution of Equations (B) ±2i tan 5p/12
Certain types of algebraic equations can be solved with the (C) ±2i
help of De’Moivre’s theorem (D) ±2
Equations of the type pzn + q = 0: 181. The roots of the equation z4 – z3 + z2 – z + 1 = 0 are
If pzn + q = 0, where p and q are complex numbers, and ⎛ pp ⎞ ⎛ pp ⎞
cos ⎜ + i sin ⎜ where p =
p ≠ 0, then ⎝ 5 ⎟⎠ ⎝ 5 ⎟⎠
zn = – q/p (A)  1, 3, 5, 7, 9
(B)  1, 3, 7, 9
The roots of the given equation are, therefore, the n values
(C)  3, 5, 7, 9
of (– q/p)1/n. For example, consider the equation z7 + 1 = 0.
(D)  None of these

Objective_Maths_JEE Main 2017_Ch 3.indd 32 01/01/2008 03:27:34


Complex Numbers  3.33

Match the Column Type

( )
182. 100
  (III) If 3+i = 299 (C) 22n
Column-I Column-II (a + ib), then b =
2 2
⎛p⎞ ⎛p⎞ (A)  i – 1 (IV) 1(2 – w) (2 – w2) + 2 (D)  n ( n + 1) − n
  (I) If zr = cos ⎜ r ⎟ + i sin ⎜ r ⎟ r 4
⎝3 ⎠ ⎝3 ⎠ (3 – w) (3 – w2) + … +
= 1, 2, 3, …, then z1z2z3 … ∞ = (n – 1) (n – w) (n – w2) =
  (II)  If iz3 + z2 – z + i = 0, then |z| = (B) 1 184.
z −2 (C) 2 Column-I Column-II
  (III) If ( z ≠ − 2) is purely
z +2 6 6
imaginary, then |z| = ⎛ 3 + i⎞ ⎛ i − 3⎞ 7
  (I)  ⎜ ⎟ +⎜ ⎟ (A)  −
(IV) The value of the sum (D)  i – 1 ⎝ 2 ⎠ ⎝ 2 ⎠ 2
13
∑ (i n + i n +1 ) ,   (II) If
z −1
is purely imaginary, (B) 0
n =1 z +1
where i = − 1 , equals then |z| =

( ) ( )
100 100
183. If 1, w, w2 be the three cube roots of unity, then   (III)  i + 3 + i− 3 + 2100 = (C) –2

Column-I Column-II (IV) Let zk = (k = 0, 1, 2, … 6) be the (D) 1


roots of the equation (z + 1)7
  (I) (1 + w) (1 + w2) (1 + w4) (A)  3 6
(1 + w8) … to 2n factors + z7 = 0, then ∑ Re ( z k ) is
k =0
  (II) (1 – w + w2) (1 – w2 + w4) (B) 1 equal to
(1 – w4 + w8) … to 2n
factors =

Assertion-Reason Type
Instructions: In the following questions an Assertion (A) is 186. Assertion: The locus of the point z satisfying the
given followed by a Reason (R). Mark your responses from z −1 p
condition arg = is a parabola
the following options: z +1 3
(A)  Assertion(A) is True and Reason(R) is z
True; Reason(R) is a correct explanation for Reason: Arg 1 = Arg z1 – Arg z2
z2
Assertion(A)
(B)  Assertion(A) is True, Reason(R) is True; 187. Assertion: If the area of the triangle on the argand
Reason(R) is NOT a correct explanation for plane formed by the complex numbers – z, iz, z – iz is
Assertion(A) 600 square units, then |z| = 20
(C)  Assertion(A) is True, Reason(R) is False Reason: Area of the triangle on the argand plane
(D)  Assertion(A) is False, Reason(R) is True formed by the complex numbers –z, iz, z – iz is
3 2
185. Assertion: If a = cos a + i sin a, b = cos b + i sin b, |z |.
a b c 2
c = cos g + i sin g and + + = –1, then cos (b – g )
b c a 188. Assertion: If |z – 1| + |z + 3| ≤ 8, then the range of
+ cos (g – a) + cos (a – b) = –1 values of |z – 4| is [1, 9]
Reason: (cos a1 + i sin a1) (cos a2 + i sin a2) = Reason: |z – 1| + |z + 3| ≤ 8 ⇒ z lies inside or on the
cos(a1 + a2) + i sin (a1 + a2) ellipse whose foci are (1, 0) and (–3, 0) and vertices
are (–5, 0) and (3, 0).

Objective_Maths_JEE Main 2017_Ch 3.indd 33 01/01/2008 03:27:37


3.34  Chapter 3

189. Assertion: The greatest value of the moduli of ­complex 1


191. Assertion: If z0 =
(1 + i ) , then
4 2
numbers z satisfying the equation z − = 2 is
z Pn(z) = (1 + z0) (1 + z02) (1 + z022) … (1 + z02n) =
5 +1
⎛ 1 ⎞
Reason: For any two complex numbers z1 and z2, (1 + i ) ⎜1 − 2 ⎟ , where n > 1 is a positive integer.
⎝ 2 ⎠
n

|z1 – z2| ≥ |z1| – |z2| n +1


1 − z 02
190. Assertion: The locus of the points representing Reason: Pn(z) =
1− z0
the complex numbers satisfying |z| – 2 = 0, |z – i| –
|z + 5i| = 0 is the single point (0, –2) 192. Assertion: If amp . [z1 (z3 – z2)] = amp . [z3(z2 – z1)],
Reason: If z is a variable point and z1, z2 are two fixed then 0, z1, z2, z3 are concyclic.
points in the argand plane, then |z – z1| = |z – z2| ⇒ Reason: For four concyclic points z1, z2, z3, z4,
locus of z is the perpendicular bisector of the line ( z1 − z3 ) ( z2 − z4 )
is purely real.
­segment joining z1 and z2. ( z1 − z4 ) ( z2 − z3 )

Previous Year’s Questions

193. If ω is an imaginary cube root of unity, then (1 + ω ⎛ x y⎞


1 ⎜⎝ p + q ⎟⎠
– ω 2)7 equals: [2002]
198. If z = x − iy and z3 = p + iq , then is equal to
(A) 128 ω (B)  –128 ω ( p2 + q2 )
(C) 128 ω2 (D) –128 ω2  [2004]
(A) 1 (B) –2
194. Let z1 and z2 be two roots of the equation z2 + az + b = 0, (C) 2 (D) –1
z being complex. Further, assume that the origin, z1
and z2 form an equilateral triangle, then 1 99. If | z2 –1| = | z |2+ 1, then z lies on [2004]
 [2003] (A)  the real axis
(A) a2 = b (B)  a2 = 2b (B)  an ellipse
(C) a2 = 3b (D)  a2 = 4b (C)  a circle
(D)  the imaginary axis.
195. If z and ω are two non-zero complex numbers such
p 200. If the cubes roots of unity are 1, ω, ω2 then the roots
that | zω | = 1, and Arg (z) − Arg (ω) = , then Z w
2 of the equation [2005]
is equal to [2003] 3
(x − 1) + 8 = 0, are
(A) 1 (B) –1 (A) −1, −1 + 2ω, −1 −ω2
(C) i (D)  −i (B) −1, −1, −1
x (C) −1, 1 − 2ω, 1 − 2ω2
⎛1+ i⎞ (D) −1, 1 + 2ω, 1 + 2ω2
196. If ⎜ = 1 , then [2003]
⎝ 1 − i ⎟⎠
201. If z1 and z2 are two non-zero complex numbers such
(A) x = 4n, where n is any positive integer that | z1 + z2 | = | z1 | + | z2 | then argz1− argz2 is equal to
(B) x = 2n, where n is any positive integer  [2005]
(C) x = 4n + 1, where n is any positive integer p
(D) x = 2n + 1, where n is any positive integer (A)  (B)  −π
2
p
197. Let z, w be complex numbers such that z + iw = 0 (C) 0 (D)  −
and arg zw = π. Then arg z equals [2004] 2

p 5p z
202. If w = and | w | = 1, then z lies on [2005]
(A) (B)  1
4 4 z− i
3
3p p (A)  an ellipse (B)  a circle
(C) (D) 
4 2 (C)  a straight line (D)  a parabola

Objective_Maths_JEE Main 2017_Ch 3.indd 34 01/01/2008 03:27:39


Complex Numbers  3.35

10
⎛ 2k p 2k p ⎞ z2
203. The value of ∑ ⎜ sin + i cos ⎟ is 211. If z ≠ 1 and is real, then the point which is rep-
⎝ 11 11 ⎠ z −1
 k =1 [2006] resented by the complex number z lies [2012]
(A) i (B) 1 (A) either on the real axis or on a circle passing
(C)  –1 (D)  −i through the origin
204. If z2 + z + 1 = 0, where z is a complex number, then (B)  on a circle with centre at the origin
the value of (C) either on the real axis or on a circle not passing
2 2 2 2 through the origin
⎛ 1⎞ ⎛ 1⎞ ⎛ 1⎞ ⎛ 1⎞ (D)  on the imaginary axis
⎜ z + ⎟ + ⎜ z 2 + 2 ⎟ + ⎜ z 3 + 3 ⎟ + ... + ⎜ z 6 + 6 ⎟
⎝ z⎠ ⎝ z ⎠ ⎝ z ⎠ ⎝ z ⎠
212. If z is a complex number of unit modulus and argu-
is [2006]
⎛ 1+ z ⎞
(A) 18 (B) 54 ment q , then ⎜ equals [2013]
(C) 6 (D) 12 ⎝ 1 + z ⎟⎠
p
205. If | z + 4 | ≤ 3, then the maximum value of |z + 1| is (A)  − q (B)  q
2
 [2007]
(C)  p − q (D)  −q
(A) 4 (B) 10
(C) 6 (D) 0
213. If z is a complex number such that z ≥ 2 , then the
1
206. The conjugate of a complex number is Then the 1
complex number is i − 1 [2008] minimum value of z +  [2014]
2
−1 1 5
(A)  (B)  (A)  is equal to
i −1 i +1 2
−1 1 (B)  lies in the interval (1, 2)
(C)  (D) 
i +1 i +1 5
(C)  is strictly greater than
2
4 3 5
*207. If Z − = 2 , then the maximum value of | Z | is (D)  is strictly greater than but less than
Z 2 2
equal to [2009]
214. A complex number z is said to be unimodular if
(A)  3 + 1 (B)  5+1
| z | = 1. Suppose z1 and z2 are complex numbers such
(C) 2 (D) 2 + 2 z − 2 z2
that 1 is unimodular and z2 is not unimodular.
208. The number of complex numbers z such that | z− 1| 2 − z1 z2
= |z + 1| = |z−i| equals [2010] Then the point z1 lies on a [2015]
(A) 1 (B) 2 (A)  straight line parallel to y-axis.
(C)  ∞ (D) 0 (B)  circle of radius 2.
(C)  circle of radius 2.
209. Let α, β be real numbers and z a complex number. If
(D)  straight line parallel to x-axis.
z2 + α z+ β = 0 has two distinct roots on the line
Re(z) = 1, then it is necessary that [2011] 2 + 3i sin q
215. A value of θ for which is purely imagi-
(A) β ∈(−1, 0) (B)  | β | = 1 1 − 2i sin q
(C) β ∈(1,∞) (D)  β ∈(0, 1) nary, is [2016]
⎛ 1 ⎞ p
210. If ω (≠ 1) is a cube root of unity, and (1 + ω)7 = A + (a) sin −1 ⎜ ⎟ (B) 
⎝ 3⎠ 3
Bω. Then (A, B) equals [2011]
(A)  (1, 1) (B)  (1, 0) p ⎛ 3⎞
(c) sin −1 ⎜ ⎟
(D) 
(C)  (–1, 1) (D)  (0, 1) 6 ⎝ 4 ⎠

Objective_Maths_JEE Main 2017_Ch 3.indd 35 01/01/2008 03:27:43


3.36  Chapter 3

Answer keys

Single Option Correct Type


  1. (A) 2. (D) 3.  (A, C) 4. (D) 5.  (B) 6. (B) 7.  (D) 8. (C) 9. (A) 10. (C)
  11.  (D) 12. (D) 13.  (C) 14. (B) 15.  (B) 16. (C) 17.  (B) 18. (C) 19.  (D) 20. (B)
  21.  (B) 22. (C) 23.  (B) 24. (B) 25. (B) 26. (C) 27.  (C) 28. (A) 29.  (C) 30. (A)
  31.  (C) 32. (B) 33. (B) 34. (C) 35.  (C) 36. (A) 37.  (C) 38. (C) 39.  (B) 40. (C)
  41.  (C, D) 42. (A) 43.  (A) 44. (C) 45.  (B) 46. (C) 47.  (C) 48. (A) 49. (C)
  50. (C) 51.  (B) 52. (B) 53.  (B) 54.  (A, C) 55.  (B) 56. (C) 57. (A) 58. (A) 59.  (B)
  60. (A) 61.  (C) 62. (B) 63.  (A) 64. (C) 65. (A) 66. (C) 67. (A) 68. (C) 69. (C)
  70. (B) 71.  (D) 72. (A) 73.  (C) 74.  (B) 75. (A) 76.  (B) 77. (C) 78.  (B) 79. (C)
  80. (C) 81. (B) 82.  (B) 83. (B) 84.  (B) 85. (A) 86.  (A) 87. (B) 88.  (C) 89. (C)
  90.  (A) 91. (C) 92.  (C) 93. (B) 94.  (B) 95. (B) 96.  (A) 97. (C) 98.  (D) 99. (A)
100.  (B) 101. (B) 102.  (A) 103. (C) 104.  (A) 105. (B) 106. (D) 107. (C) 108.  (C) 109. (C)
110.  (A) 111. (B) 112.  (A) 113. (A) 114.  (B) 115. (D) 116.  (B) 117. (A) 118.  (B) 119. (C)
120.  (B) 121. (C) 122. (A) 123.  (D) 124. (C) 125.  (B) 126. (C) 127.  (A) 128. (D) 129. (B)
130. (C) 131.  (C) 132. (B) 133.  (B) 134. (B) 135.  (B) 136. (A) 137.  (C) 138. (B) 139.  (B)
140. (D) 141.  (B) 142. (C) 143.  (C) 144. (B) 145. (A) 146. (D) 147.  (A) 148. (B) 149.  (A)
150. (A) 151.  (C) 152. (B)

More than One Option Correct Type


153.  (A) and (C) 154.  (A) and (D) 155.  (C) and (D) 156.  (A) and (B) 157.  (A), (B) and (C)
158.  (A), (B) and (D) 159.  (A) and (B) 160.  (B) and (C) 161.  (A) and (C)
162.  (A), (B), (C) and (D) 163.  (A) and (B) 164.  (A), (B), (C) and (D)
165.  (A), (B), (C) and (D) 166.  (B), (C) and (D) 167.  (A), (B) and (C) 168.  (A), (B) and (C)
169.  (A), (B) and (C)

Passage Based Questions


170.  (D) 171. (B) 172.  (A) 173. (C) 174.  (A) 175. (B) 176.  (A) 177. (C) 178.  (A), (B) and (C)
1 79.  (A) and (B) 180.  (A), (B) and (C) 181. (B)

Match the Column Type


1 82. (I) → (D); (II) → (B); (III) → (C); (IV) → (A)
183.  (I) → (B); (II) → (C); (III) → (A); (IV) → (D)
184. (I) → (C); (II) → (D); (III) → (B); (IV) → (A)

Assertion-Reason Type
185. (A) 186. (D) 187. (A) 188. 
(A) 189. (A) 190. 
(A) 191. (A) 192. 
(A)

Previous Year’s Questions


1 93. (D) 194. (C) 195.  (D) or (C) 196. (A) 197. (C) 198. (B) 199. (D) 200. (C) 201. (C)
202. (C) 203. (D) 204. (D) 205. (C) 206. (C) 207. (B) 208. (A) 209. (C) 210. (A)
211. (A) 212. (B) 213. (B) 214. (B) 215. (A)

Objective_Maths_JEE Main 2017_Ch 3.indd 36 01/01/2008 03:27:43


Complex Numbers  3.37

Hints and Solutions

Single Option Correct Type


p q r
1. We have, + + =1+i ⇒ x2 = 2 [(x + 1)2 + 1] = 2 (x2 + 2x + 2)
a b c
2
\ x2 + 4x + 4 = 0
⎛ p q r⎞ ⇒ (x + 2)2 = 0
⇒ ⎜ + + ⎟ = 1 – 1 + 2i = 2i

⎝ a b c⎠
or x = –2
p2 q2 r2 ⎛ qr rp pq ⎞ \ z = –2 – i.

+ + + 2⎜ + + = 2i
a 2
b 2
c 2⎝ bc ca ab ⎟⎠ The correct option is (B)
p 2
q2 r2 2 pqr ⎛ a b c ⎞ z

+ + + + + = 2i 7. If 1 = z, the given equation becomes
a 2
b 2
c 2
abc ⎜⎝ p q r ⎟⎠ z2
2 2
z2 – z + 1 = 0 i.e., z = – w and – w2
p q r2 z

2
+ = 2i
2
+ or, 1 = – w ⇒ z1 = – z2w
a b c2 z2
The correct option is (A) OB = |z2 – 0| = |z2|
2. sin x + i cos 2x and cos x – i sin 2x are conjugate of each OA = |z1 – 0| = |– z2w | = |z2| |–w | = |z2|
other and AB = |z2 – z1| = |z2 + z2w |
if sin x + i cos 2x = cos x − i sin 2 x = |z2| |1 + w | = |z2| |–w2| = |z2|
⇒ sin x + i cos 2x = cos x + i sin 2x Thus z1, z2 and origin form an equilateral triangle.
⇒ sin x = cos x and cos 2x = sin 2x The correct option is (D)
⇒ tan x = 1 and tan 2x = 1, 8. Let a = r cos q and b = r sin q
which is not possible for any value of x. b
\ tan q =
The correct option is (D) a
3. Since |z1 + z2| = |z1| + |z2|, a − ib r (cos q − i sin q )
Now = = (cos q – i sinq)2
\ P, O, Q where P is the affix of z1, O is the affix of origin a + ib r (cos q + i sin q )
and Q the affix of z2 must lie in the same straight line. Thus,
= cos 2q – i sin 2q = e– 2iq
arg z1 – arg z2 = ± p. a − ib
\ i log
= i log e– 2iq = i (– 2 iq) = 2q
The correct option is (A) and (C) a + ib b
4. Let z = x + iy, then 2
⎡ a − ib ⎤ 2 tan q a
\ tan ⎢i log
= tan 2q = =
z2 + |z|2 = 0 ⇒ (x + iy)2 + |x + iy|2 = 0 ⎣ a + ib ⎥⎦ 1 − tan 2 q b2
1− 2
⇒ x2 – y2 + 2ixy + x2 + y2 = 0 2ab a
⇒ 2x2 + 2ixy = 0
= 2 2
a −b
⇒ 2x2 = 0 and 2xy = 0
The correct option is (C)
⇒ x = 0 and xy = 0
9. Since ( 3 + i)100 = 299 (a + ib)
Clearly y can be any real number. Hence, we will get
infinitely many solutions. \ ( 3 – i)100 = 299 (a – ib)

The correct option is (D) \ ( 3 + i)100 – ( 3 – i)100 = 299 (2ib) = 2100 (ib)

5. As w is the nth root of unity so, wn – 1 = 0 ⇒ i100 [1 – 3 i]100 – i100 [–1 +
3 i]100 = 2100 (ib)
⇒ (w – 1) (1 + w + w2 +, …, + wn – 1) = 0 or (– 2w)100 – (2w2)100 = 2100 (ib)

Hence, 1 + w + w2 +, …, + wn – 1 = 0
or w – w2 = (ib) or
3 i = ib
or w – 1 = 0 i.e., w = 1
\ b = 3
The correct option is (B)
The correct option is (A)
6. Since z + 2 |z + 1| + i = 0 10. We have,
\ x + i (y + 1) + 2  |x + iy + 1| = 0 1 − i sin a (1 − i sin a ) (1 − 2i sin a )
=
\ y + 1 = 0 (∵ |x + iy + 1| is real) 1 + 2i sin a (1 + 2i sin a ) (1 − 2i sin a )
\ y = – 1 1 − 3i sin a − 2 sin 2 a
\ x + 2 |x – i + 1| = 0
=
1 − 4i 2 sin 2 a

Objective_Maths_JEE Main 2017_Ch 3.indd 37 01/01/2008 03:27:47


3.38  Chapter 3

(1 − 2 sin 2 a ) − 3i sin a x1x2 … x6 = 55 [cos (5p + 5q) + i sin (5p + 5q)]


=
,
1 + 4 2 sin a = 55 (– cos 5q – i sin 5q)
which is purely real if sin a = 0, i.e., if a = np, where n is an = – 55(cos 5q + i sin 5q)
integer. The correct option is (B)
The correct option is (C) 16. Given |z1 + z2| = |z1| + |z2|
11. By the given condition, |z – 1| < |z – i|
Squaring, |z1 + z2|2 = (|z1| + |z2|)2

⇒ |x – 1 + iy| < |x + i (y – 1)|
⇒ |z1|2 + |z2|2 + 2Re |z1| |z2| = |z1|2 + |z2|2 + 2 |z1| |z2|

⇒ (x – 1)2 + y2 < x2 + (y – 1)2
⇒ 2Re |z1| | z2 | = 2 |z1| |z2|

⇒ –2x < – 2y ⇒ x > y ⇒ x – y > 0
⇒ |z1| | z2 | cos (q1 – q2) = |z1| |z2|

The correct option is (B)
⇒ cos (q1 – q2) = 1

12. Perimeter = 6 × distance of vertex 1 + 2i
⇒ q1 – q2 = 0  or  q1 = q2

 from the centre (0, 0).
or arg z1 = arg z2

= 6 [(1 – 0) + i (2 – 0)] = 6 5

The correct option is (C)
The correct option is (D)
13. Let z1 = x1 + iy1 17. For every a ∈ R, |a| = a 2
z2 = x2 + iy2 \ |a|2 = a2
z3 = x3 + iy3 Now (|x| – |y|)2 ≥ 0
z2 z3 = (x2 – iy2) (x3 + iy3) ⇒ |x|2 + |y|2 – 2 |x| |y| ≥ 0
= (x2x3 + y2y3) + i (x2y3 – x3y2) ⇒ 2 |x| |y| ≤ |x|2 + |y|2

⇒ Im ( z2 z3 ) = x2y3 – x3y2 ⇒ |x|2 + |y|2 + 2 |x| |y| ≤ 2 |x|2 + 2 |y|2

\ z1 Im ( z2 z3 ) = (x1 + iy1) (x2y3 – x3y2) ⇒ (|x| + |y|)2 ≤ 2 (x2 + y2)

Similarly
⇒ (|x| + |y|)2 ≤ 2 |z|2

z2 Im ( z3 z1 ) = (x2 + iy2) (x3y1 – x1y3)
\ |x| + |y| ≤ 2 |z|.
z3 Im ( z1z2 ) = (x3 + iy3) (x1y2 – x2y1)
The correct option is (B)
Adding we get the result = 0
18. We have, |1 + i|2 × |1 + 2i|2 × |1 + 3i|2 … |1 + ni|2
The correct option is (C)
2 z1 = |a + ib |2
14. ∵ is purely an imaginary number, ⇒ (1 + 1) × (1 + 4) × (1 + 9) … (1 + n2) = a2 + b2
3 z2
2z z 3 ⇒ 2 × 5 × 10 … (1 + n2) = a 2 + b 2.
\ let 1 = ib  or  1 = ib (1) The correct option is (C)
3 z2 z2 2
4
19. |z1| = 1, |z2| = 1
z1 4
−1 3
4 ib − 1 ⇒ a2 + b2 = 1
(1)
z1 − z2 z2
\
= = 2 p2 + q2 = 1
(2)
z1 + z2 z1 3
+1 ib + 1
z2 2 Im ( z1 z2 ) = Im (a + ib) (p – iq) = bp – aq

4 \ bp – aq = 1
(3)
4 ⎛ 9b 2 ⎞
3 ⎜ + 1⎟ w1w 2 = (a + ip) (b – iq) = (ab + pq) + i (bp – aq)
ib − 1 ⎜⎝ 4 ⎟⎠
2
= (ab + pq) + i (1)

= = =1
( )
4 4
3 ⎛ 9b 2 ⎞ \ Im w1w 2 = 1

ib + 1 + 1⎟
2 ⎜
⎜⎝ 4 ⎟⎠
The correct option is (D)

The correct option is (B) 3
20. We have, a + ib = x + iy
15. x6 = (4 – 3i)5
a + ib = (x + iy)3 = (x3 – 3xy2) + i (3x2y – y3)

⎛ 4 3i ⎞
x6 = 56 ⎜ − ⎟ = 55 (cos q + i sin q)5 \ a = x3 – 3xy2 and b = 3x2y – y3

⎝5 5⎠
a b
⎛ 3⎞ \
= x2 – 3y2 and = 3x2 – y2
where  q = –tan–1 ⎜ ⎟ = 55 (cos 5q + i sin 5q) x y
⎝ 4⎠
a b
x = 55/6 (cos 5q + i sin 5q)1/6
\
+ = x2 – 3y2 + 3x2 – y2 = 4 (x2 – y2)
x y
⎡ ⎛ 2kp + 5q ⎞ ⎛ 2kp + 5q ⎞ ⎤
The correct option is (B)
= 55/6 ⎢cos ⎝⎜
⎟ + i sin ⎝⎜
⎠ ⎠⎟ ⎥⎦
⎣ 6 6

Objective_Maths_JEE Main 2017_Ch 3.indd 38 01/01/2008 03:27:51


Complex Numbers  3.39

21. As (a – b)2 ≥ 0, a2 + b2 ≥ 2ab(1) 27. We have, z4 + 1 = 0 ⇒ z4 = –1


⇒ z = (cos p + i sin p)1/4
But |z| =
a 2 + b 2 ; so from Eq. (1), |z|2 ≥ 2ab
1 1
\ |z| + a2 + b2 ≥ a2 + b2 + 2ab
2 ⇒ z = cos (2kp + p) + i sin (2kp + p), k = 0, 1, 2, 3.
4 4
or |z|2 + |z|2 ≥ (a + b)2;
p p 3p 3p
⇒ z = cos + i sin , cos + i sin ,
\ 2 |z|2 ≥ (a + b)2
4 4 4 4
5p 5p 7p 7p
\
2 |z| ≥ a + b as |z| is positive. cos + i sin , cos + i sin
1 4 4 4 4
\ |z| ≥
(a + b) 1 1 1 1
2 = (1 + i), (– 1 + i), (– 1 – i), (1 – i).
2 2 2 2

The correct option is (B) 1
Hence, the four roots of z4 + 1 = 0 are (±1 ± i).
z1 − z3 1 − i 3 z3 2
22. = The correct option is (C)
z2 − z3 2 π 28. Put 1 = r cos q and – 1 = r sin q
⎛ −p ⎞ ⎛ −p ⎞ 3 p

= cos ⎜ + sin ⎜ = e– i p/3 ⇒ r = 2 and q = –
⎝ 3 ⎟⎠ ⎝ 3 ⎟⎠ 4
Then given equation takes the form
z1 − z3
\
= e − i p /3 = 1 z1 z2 ⎡ ⎛ p⎞ ⎛ p⎞⎤
n
z2 − z3 ( 2 )n ⎢cos ⎜ − ⎟ + i sin ⎜ − ⎟ ⎥ = 2n

p ⎣ ⎝ 4⎠ ⎝ 4⎠⎦
and angle between z1 – z3 and z2 – z3 is .
3 ⎡ np np ⎤
\ triangle is equilateral. or 2n/2 ⎢cos
− i sin = 2n
The correct option is (C) ⎣ 4 4 ⎥⎦
23. Putting x = 1, w, w2 respectively,
Equating real and imaginary parts, we get
np np
3n = a0 + a1 + a2 + … + a2n
cos = 2n/2 and – sin =0
4 4
(1 + w + w2)n = a0 + a1 w + a2 w2 + … + a2n w2n
These are satisfied only for n = 0.

(1 + w2 + w4)n = a0 + a1 w2 + a2 w4 + … + a2n w4n
Hence, n = 0 is the only solution.

Adding these, 3n + (1 + w + w2)n + (1 + w2 + w4)n

The correct option is (A)
= 3a0 + a1 (1 + w + w2) + a2 (1 + w2 + w4)
4 4
+ a3 (1 + w3 + w6) + … 29. We have, 2 = z − ≥ |z| –
z z
\ 3n + 0n + 0n = 3a0 + 3a3 + 3a6 + …
4
\ 3n – 1 = a0 + a3 + a6 + … ⇒ |z| –
≤2
z
The correct option is (B)
⇒ |z|2 – 2 |z| – 4 ≤ 0 or (|z| – 1)2 – 5 ≤ 0

24. Let n 1 = x; \ xn = 1; \ xn – 1 = 0
⇒ (|z| – 1)2 ≤ 5 or |z| – 1 ≤ ⇒ |z| ≤ + 1

\ xn – 1 = (x – 1) (x – a1) (x – a2) … (x – an – 1)
\ (x – a1) (x – a2) (x – a3) … (x – an – 1)
Hence the greatest value of |z| is 5 + 1.
n
x −1 1− x n
The correct option is (C)
=
= = 1 + x + x2 +, …, + xn – 1. p
x −1 1− x 30. We have, Arg (z + i) – Arg (z – i) =
2
Putting x = 1, we get
⎛ z + i⎞ p
(1 – a1) (1 – a2) (1 – a3) … (1 – an – 1) = n. ⇒ Arg ⎜
=
⎝ z − i ⎟⎠ 2
The correct option is (B)
25. The closest distance = length of the perpendicular from the ⎛ z + i⎞
\ Re ⎜
=0
origin on the line a z + a z + a a = 0 ⎝ z − i ⎟⎠

a( 0 ) + a | 0 | + a a | a |2 | a| ⎛ z + i⎞ ⎛ z + i⎞
=
= = ⎜⎝ ⎟ +⎜ ⎟
2| a | 2| a | 2 z − i⎠ ⎝ z − i⎠ ⎛ z + i⎞ ⎛ z + i ⎞

= 0 ⇒⎜ + =0
The correct option is (B) 2 ⎝ z − i ⎟⎠ ⎜⎝ z − i ⎟⎠
26. Given |z – 1| + |z + 3| ≤ 8 ⇒ (z + i) ( z + i) + (z – i) ( z – i) = 0

\ z lies inside or on the ellipse whose foci are (1, 0) and ⇒ z z + i (z + z ) – 1 + z z – i (z + z ) – 1 = 0

(–3, 0). Vertices are (–5, 0) and (3, 0). Now, |z – 4| is ­distance ⇒ 2 (z z ) = 2

of z from (4, 0). ⇒ z z = 1 or |z|2 = 1

Minimum distance is 1 and maximum is 9. ⇒ |z| = 1

The correct option is (C)

Objective_Maths_JEE Main 2017_Ch 3.indd 39 01/01/2008 03:27:57


3.40  Chapter 3

The equation represents a circle centred at origin and radius | z − 1| + 4 1



< <1
1 unit. 3 | z − 1| − 2 2
The correct option is (A) (∵ loga x is a decreasing function if a < 1)
31. Let z = x + iy ⇒ |z – 1| + 4 < 3 |z – 1| – 2
z2 ⇒ 2 |z – 1| > 6 ⇒ |z – 1| > 3

Since is real = k (say), where k ∈ R
z −1 which is an exterior of a circle.
( x + iy ) 2 The correct option is (C)
\
=k
x + iy − 1 35. Suppose x is a real root
⇒ x2 + 2i xy – y2 = k (x – 1) + i ky
Then x3 + ix – 1 = 0 ⇒ x3 – 1 = 0 and x = 0
⇒ x2 – y2 = k (x – 1)
(1) There is no real number satisfying these two equations.
2xy = ky
(2) The correct option is (C)
Eq. (2) gives either y = 0 or k = 2x
36. Let a, b, g  be the roots
If k = 2x, then x2 – y2 = 2x2 – 2x
\ a + b + g  = –a
⇒ x2 + y2 – 2x = 0, which is a circle.
\ |–a| = |a + b + g | ≤ |a| + |b | + |g | = 1 + 1 + 1
Thus, lies either on the real axis y = 0 or on a circle.
\ |a| ≤ 3

The correct option is (C) The correct option is (A)
z1 − 2 z2 z z
32. Clearly =1 37. We have, 1 + 2 = 1
2 − z1z2 z2 z1
⎛ z − 2 z2 ⎞ ⎛ z1 − 2 z2 ⎞ ⇒ z12 + z22 = z1 z2

⇒ ⎜ 1
⎜⎝ 2 − z z ⎟⎠ = 1
⎝ 2 − z1z2 ⎟⎠ 1 2 ⇒ z12 + z22 + z32 = z1 z2 + z2 z3 + z3 z1,

⇒ z1z1 − 2 z1z2 − 2 z1z2 + 4 z2 z2
where z3 = 0.

= 4 − 2 z1z2 − 2 z1z2 + z1z2 z1z2
⇒ z1, z2 and the origin form an equilateral triangle.

⇒ |z1|2 + 4 |z2|2 = 4 + |z1|2 |z2|2

The correct option is (C)
⇒ |z1|2 [1 – |z2|2] = 4 [1 – |z2|2] 1 i 2n + 1
38. We have, S (n) = in + i–n = in + n
=
⇒ |z1|2 = 4 (Q |z2| ≠ 1) i in
( −1) n + 1
⇒ |z1| = 2
= , n = 1, 2, 3, 4, …
in
The correct option is (B)
\ values of S (n) are 0, – 2, 2, 0, – 2, 2, …

33. Let z1 = x1 + iy1 and z2 = x2 + iy2
\ Total number of distinct values of S (n) is 3.

Now z1 + z2 = (x1 + x2) + i (y1 + y2)

The correct option is (C)
and z1 – z2 = (x1 – x2) + i (y1 – y2)
As |z1 + z2| = |z1 – z2|, we get 1 1 z1 + z2
39. We have, |z1 + z2| = + =
(x1 + x2)2 + (y1 + y2)2 = (x1 – x2)2 + (y1 – y2)2 z1 z2 z1 z2
or, x1 x2 + y1 y2 = 0 (1) ⎛ 1 ⎞
y1 ⇒ |z1 + z2| ⎜1 −
=0
Now amp z1 – amp z2 = tan– 1 x – tan– 1 2
y ⎝ | z1 z2 | ⎟⎠
1 x2
y1 y2 ⇒ |z1 z2| = 1.(∵ z1 ≠ –z2)

x x2 The correct option is (B)
= tan– 1 1
y1 y2 40. We have, amp (z – 1) = amp (z + 3i)
1+ ⋅ y y+3
x1 x2 ⇒ tan–1 = tan– 1
x y − y2 x1 x −1 x
= tan– 1 2 1 ⇒ xy = (x – 1) (y + 3)
x1 x2 + y1 y2

= tan–1 ∞, by (1) ⇒ 3 (x – 1) = y
p \ (x – 1) : y = 1 : 3.
\ |amp z1 – amp z2| = .
2 The correct option is (C)
The correct option is (B) z1 + z3 z + z4
41. We have, z1 – z4 = z2 – z3 or  = 2
34. We have, 2 2
⎛ | z − 1| + 4 ⎞ ⎛ 1⎞
i.e., the diagonals bisect each other.
log1/ 2 ⎜ > 1 = log1/ 2 ⎜ ⎟
⎝ 3 | z − 1| − 2 ⎟⎠ ⎝ 2⎠ \ It is a parallelogram.

Objective_Maths_JEE Main 2017_Ch 3.indd 40 01/01/2008 03:28:02


Complex Numbers  3.41

z4 − z1 p 48. Midpoint of P and P′ is centre of circle C such that



Also, amp =
z2 − z1 2 z + ( − z1 )
1 =0
⇒ angle at z1 is a right angle.
2
\ It is a rectangle and hence a cyclic quadrilateral.
\ Centre of circle lies at origin.

The correct option is (C) and (D) Now, the equation of circle with centre at origin and radius
2p |z1| or |–z1| is
42. Given: |z| = 2 and arg (z) =
.
3 |z – 0| = |z1|
2p
\ If z = r (cos q + i sinq), then r = 2 and q =
⇒ |z|2 = |z1|2 P(–z1)
3
⇒ z · z = z1 · z1
⎛ 2p 2p ⎞ ⎛ 1 3 ⎞
\ z = 2 ⎜ cos
+ i sin ⎟ = 2⎜− + i ⎟
⎝ 3 3⎠ ⎝ 2 2 ⎠ z z ⎛z ⎞
\
= 1 = ⎜ 1⎟ P(z1)

= (–1 + i 3) z1 z ⎝ z⎠

The correct option is (A) The correct option is (A)
⎛ | z | − | z | +1⎞
2 49. We have
2
43. log 3 ⎜ 2 + | z| ⎟ < 2
⎛ p q r⎞
(1 + i)2 = ⎜ + + ⎟
⎝ ⎠ ⎝ a b c⎠
| z |2 − | z | +1

< ( 3 )2 p2 q2 r2 ⎛ qr rp pq ⎞
2 + | z| ⇒ 1 – 1 + 2i =
+ + + 2⎜ + +
a 2
b 2
c2 ⎝ bc ca ab ⎟⎠
⇒ |z|2 – |z| + 1 < 6 + 3 |z|

p2 q2 r2 2abc ⎛ a b c ⎞
⇒ |z|2 – 4 |z| – 5 < 0 ⇒ (|z| – 5) (|z| + 1) < 0
⇒ 2i =
+ + + + +
a 2
b 2
c 2
pqr ⎜⎝ p q r ⎟⎠
⇒ |z| – 5 < 0, since |z| + 1 > 0 ⇒ |z| < 5

2 2

The correct option is (A) p q r2 2abc

= 2
+ 2
+ + (0)
a b c2 pqr
44. We have, |z| = 1, or x2 + y2 = 1
p2 q2 r2
or x2 + y2 – 1 = 0.

2
+ 2
+
= 2i
a b c2
⎛ z − 1⎞ ⎡ ( x − 1) + iy ⎤ ⎡ ( x + 1) − iy ⎤ The correct option is (C)

Now ⎜ ⎟ = ⎢ ⎥×⎢ ⎥
⎝ z + 1⎠ ⎣ ( x + 1) + iy ⎦ ⎣ ( x + 1) − iy ⎦ 50. Interpretating according to Coni’s Theorem. Let the angle
( x 2 + y 2 − 1) + 2iy 2iy between the lines joining
=
=
( x + 1) 2 + y 2 ( x + 1) 2 + y 2 z1, z2 and z1, –z2 be a
z −z
(x2 + y2 – 1 = 0) \ 1 2 = cos a + t sin a
z1 + z2
The correct option is (C)
Using Componendo and Dividendo, we have
45. Let z1 = x1 + iy1
2z 1 + cos a + i sin a
\ z1 = x1 – iy1 1 =
−2 z2 cos a − 1 + i sin a
Now z2 = z1 ⇒ z1 + z2 = z1 + z1
= x1 + iy1 + x1 – iy1 ⎛a⎞ a a
2 cos 2 ⎜ ⎟ + i 2 sin cos
z ⎝ 2⎠ 2 2
= 2x1, which is real. ⇒ 1 =

z2 2⎛a⎞ a a
Hence, result holds if z2 = z1 . −2 sin ⎜ ⎟ + i sin cos
⎝ 2⎠ 2 2
The correct option is (B)
z1 a
46. |z – i| = |z + 5i| represents equation of perpendicular bisector ⇒
= i cot
z2 2
of points (0, 1) and (0, –5) i.e., y = –2, now |z| = 2 is x2 + a
y2 = 4 ⇒ iz1 = –cot z2

2
⇒ x2 + 4 = 4 ⇒ x = 0 But iz1 = kz2 
⎡ a ⎤
⎢⎣ where, k = − cot 2 (say)⎥⎦
\ z represents a single point (0, –2). a
Now, k = –cot

The correct option is (C) 2
a 2k
47. If a complex number z is rotated through an angle 180º, then ⇒ cot = –k ⇒ tan a = 2

it’s new position is –z. So, 2 – 4i is –2 + 4i and stretching it 5/2 2 k −1
2k
times means modulus 5/2 times of previous complex number ⇒ tan a = –

5 1− k2
i.e., (–2 + 4i) = –5 + 10i
2 ⎛ 2k ⎞
The correct option is (C) ⇒ a = tan–1 ⎜⎝ − 1 − k 2 ⎟⎠ = –2 tan–1 k

Objective_Maths_JEE Main 2017_Ch 3.indd 41 01/01/2008 03:28:07


3.42  Chapter 3

z1 − z2 z1 + z3
Now,
= cos a + i sin a ⇒ z2 =

z1 + z2 2
where a is the angle between (z1 – z2) and (z1 + z2).
⇒ B is the mid point of the line AC

⇒ a = –2tan–1k is the angle between (z1 – z2) and (z1 + z2).
⇒ A, B, C are collinear


The correct option is (C) \ z1, z2, z3 lie on a line

51. x2 – 3x + 1 = 0
The correct option is (A) and (C)

3 ± 3−4 3 i p p 55. If z is a root of (z – 1)25 = 2w2(z + 1)25, then


⇒ x =
= ± = cos + i sin 25 25
2 2 2 6 6 ⎛ z − 1⎞ z −1
⎜⎝ ⎟ = 2w2 ⇒ = 2 |w2| = 2
⎛ n 1⎞
2
⎛ 1 ⎞ np z + 1⎠ z +1
x − n⎟ = ⎜ x 2n + 2n − 2⎟ = –2 + 2 cos
⎝⎜ x ⎠ ⎝ x ⎠ 3 z −1

= 21/25
24
⎛ 1⎞
2 24
⎛ np ⎞ z +1

∑ ⎜⎝ x n − x n ⎟⎠ = ∑ ⎜⎝ −2 + 2 cos 3⎠

As 21/25 ≠ 1, we get z lies on a circle.
n =1 n =1

⎡ p 2p 24p ⎤ The correct option is (B)


= –48 + 2 ⎢cos + cos + ... + cos
⎣ 3 3 3 ⎥⎦ 1 3
56. We have (A0  A1)2 = + = 1
4 4
⎡ ⎛ 2p 23p ⎞ ⎛ 24p ⎞ ⎤ ⇒ A0A1 = 1
+2 ⎢cos ⎜ + ⎟ .sin ⎜⎝ ⎟
⎣ ⎝ 3 6 ⎠ 6 ⎠ ⎥⎦ ⎛ 3⎞ 2 2
= – 48 ⎛ 3⎞ 9+3
p A0A1 = ⎜ ⎟ + ⎜ ⎟ =
=3
sin ⎝ 2⎠ ⎝ 2 ⎠ 4
6
= 0 – 48 = – 48 Imaginary
The correct option is (B) axis
52. C1(0, 0) is centre of bigger circle and C2 (3, 4) is centre of
smaller circle –1 , 3 1, 3
A2 A1
2 2 2 2
C1B = r1 = 12 (radius of bigger circle)
C2A = r2 = 5 (radius of smaller circle)
C1C2 = 5, minimum value of |z1 – z2| = AB A3 A0 Real
(–1, 0) (1, 0) axis

C1 –1 3 –1 3
A4 A5
2 2 2 2
C2
A B ⇒ A0A2 = 3
Similarly, A0A4 = 3
Now, C1C2 + C2A + AB = C1B
Thus, (A0A1) (A0A2) (A0A4) = 3
⇒ AB = 12 – 5 – 5 = 2
The correct option is (C)

The correct option is (B) b
57. z1 + z2 = – (1)
2 2 a
53. 2 z1 + i 3 z2 + 3 z1 + i 2 z2 c
z1z2 = (2)
a
= ( 2z + i 1 3 z2)( 2 z1 − i 3 z2 ) z2 = iz1(3)
+ ( 3z +i
1 2 z )(
2 3 z1 − i 2z ) 2
From Eq. (1) and (2)
−b
2 2 z1(1 + i) =
= 5 (|z1|2 + |z2|2) > 5 · 2 z1 z2 = 10 |z1 z2| a
−b b2 −b 2
⇒ z1 = (1 – i) ⇒ z12 = (–2i) = i
Since A · M > G · M for |z1| ≠ |z2| 2a 4a2 2a 2
The correct option is (B)
From Eq. (2) and (3)
54. Let z1, z2, z3 be affixes of points A, B, C respectively. Since, c −c −b 2 c b2
z1, z2, z3 are in A.P., therefore z12 = = i⇒ 2 i= i⇒a=
ai a 2a a 2c
2z2 = z1 + z3

The correct option is (A)

Objective_Maths_JEE Main 2017_Ch 3.indd 42 01/01/2008 03:28:13


Complex Numbers  3.43

58. z3 + 2z2 + 2z + 1 = 0 d2 ad

− +c=0
⇒ (z3 + 1) + 2z (z + 1) = 0 b 2
b
⇒ (z + 1) (z2 – z + 1 + 2z) = 0 or d2 – abd + b2c = 0
⇒ (z + 1) (z2 + z + 1) = 0 The correct option is (B)
⇒ z = –1, w, w2 63. |iz + z0| = |iz – i2 + z0 – 1|

Now equation z1985 + z100 + 1 = 0 gets satisfied by w and w2 = |i (z – i) + 5 + 3i – 1|
but not by –1. So, common roots are w and w2. = |i (z – i) + (4 + 3i)|
The correct option is (A) \ |iz + z0| ≤ |i (z – i)| + |4 + 3i|
z −1 ≤ 1.2 + 5 = 7
59. should be real
i \ Maximum value of |iz + z0| is 7
x + iy − 1
i.e., = y – i (x – 1) is real The correct option is (A)
i
⇒ x – 1 = 0 i.e., x = 1 ( )
64. z z − 2i = zz + 2iz = 2 (2 + i) gives
z −1 2 2
x + y – 2y = 4 and 2x = 2,
\ sin–1 = sin–1 y
i on equating the real and imaginary parts.
\ –1 ≤ y ≤ 1
\ y2 – 2y – 3 = 0 giving y = 3, –1
The correct option is (B)
The solutions are 1 + 3i and 1 – i.
60. Since, x2 – x + 1 = 0 (given)
The correct option is (C)
\ Solving for x, we have x = –w and x = –w2
65. Let the roots of the given equation be 1 + ip and 1 – ip, where
Case I:  x = w p∈
5 2
⎛ 1 ⎞ ⇒ b = product of roots
\ S =
∑ ⎜⎝ w n + w n ⎟⎠ = (1 + ip)(1 – ip) = 1 + p2 > 1, ∀p ∈ 
n =1
⇒ b ∈ (1, ∞)
( )
5 2
⇒ S = ∑ w n + w 2 n

The correct option is (A)
n =1
66. (1 + w)7 = A + Bw
⇒ S = (–1)2 + (–1)2 + 22 + (–1)2 + (–1)2(\ S = 8)
⇒ (–w2)7 = A + Bw
Case II:  x = w2
2 ⇒ –w2 = A + Bw
5 ⎡
⎛ 1 ⎞ ⎤
n
\ S = ∑ ⎢w 2 n + ⎜ 2 ⎟ ⎥
⇒ 1 + w = A + Bw
n =1 ⎢
⎝w ⎠ ⎥
⎣ ⎦ ⇒ A = 1, B = 1
⇒ (A, B) = (1, 1)
( )
5 2
⇒ S = ∑ w 2 n + w n
=8
The correct option is (C)
n =1

The correct option is (A) z2 z2


67. =
61. Points A, B and C are at equal distance from origin O and as z −1 z −1
∠ BOA = 45º clearly BC = AB. So, triangle is isoceles. ⇒ zzz − z 2 = zz z − z 2
⇒ | z |2 ( z − z ) − ( z − z )( z + z ) = 0
Imaginary
axis ⇒ ( z − z )[| z |2 − ( z + z )] = 0
Either z = z ⇒ real axis
C (0, 1) 1, 1 or |z|2 = z + z ⇒ zz − z − z = 0
B
2 2 represents a circle passing through origin.
The correct option is (A)
45° 68. C1: |z – i| = 2 ⇒ x2 + (y – 1)2 = 22
Real axis
O A (1, 0)
C2: |z – 1 – 2i| = 4 ⇒ (x – 1)2 +(y – 2)2 = 42

C 1C2 = 2
The correct option is (C)
r1 = 2, r2 = 4
62. Let a be a real root of the given equation.
⇒ C1C2 < |r1 – r2|
Then, a2 + (a + ib) a + c + id = 0
⇒ one circle lies inside the other. Now point (2, 2) lies
⇒ a2 + aa + c = 0 and ba + d = 0 outside circle C1 and inside circle C2.
2
⎛ d⎞ ⎛ b⎞ ⇒ C1 lies inside C2
⇒ ⎜ − ⎟ + a ⎜ − ⎟ + c = 0

⎝ b⎠ ⎝ d⎠ The correct option is (C)

Objective_Maths_JEE Main 2017_Ch 3.indd 43 01/01/2008 03:28:16


3.44  Chapter 3

69. Put –i in place of i 1 1


z − =
−1 1− a | 1 − a|
Hence .
i +1 The correct option is (C)
The correct option is (C)
74. We have,
⎛ 4⎞ 4 4 4 |3| = |a1z3 + a2z2 + a3z + a4|
70. |z| = ⎜ z − ⎟ + ⇒ | z | = z − +
⎝ z⎠ z z z
⇒ 3 ≤ |a1z3| + |a2z2| + |a3z| + |a4|

4 4 4 ⇒ 3 ≤ |a1| |z3| + |a2| |z2| + |a3| |z| + |a4|

⇒ |z| ≤ z −
+ ⇒| z|≤ 2+
z |z| |z|
⇒ 3 ≤ |z|3 + |z|2 + |z| + 1
(Q |ai| ≤ 1)
⇒ |z|2 – 2|z| – 4 ≤ 0
2 3 2
⇒ 3 < 1 + |z| + |z| + |z| < 1 + |z| + |z| + … ∞

[| z | − ( 5 + 1)][| z | − (1 − 5 )] ≤ 0 1 1
⇒ 3 <
⇒ 1 – |z| <
⇒ 1 − 5 ≤ | z | ≤ 5 + 1
1−| z| 3

The correct option is (B) 2
\ |z| >

71. We have, z1(z21 – 3z22) = 2 (1) 3
2 2
The correct option is (B)
and, z2(3z 1 – z 2) = 11
(2)

Multiplying (2) by i and adding it to (1), we get 75. We have, z4 = (z – 1)4
2 np i
z31 – 3z22z1 + i(3z21z2 – z32) = 2 + 11i
⎛ z − 1⎞
⇒ ⎜
= 11/4 = e 4 , n = 0, 1, 2, 3.
⇒ (z1 + iz2)3 = 2 + 11i(3)
⎝ z ⎟⎠

Again, multiplying (2) by i and subtracting it from (1), we
Since for all these values of z,
get z −1
= 1 so they lie on the line bisecting perpendicularly
z31 – 3z22z1 – i(3z21z2 – z32) = 2 – 11i
z
⇒ (z1 – iz2)3 = 2 – 11i(4)
the join of z = 1 and z = 0.

Multiplying (3) and (4), we get
The correct option is (A)
(z21 + z22)3 = 4 + 121 ⇒ z21 + z22 = 5.
2 2 2 2
76. We have, |z| = z + − ≤ z+ + .

The correct option is (D) z z z | z|
2
72. We have, 1 − c 2 = nc – 1 ⇒ |z| ≤ 2 +
⇒ |z|2 ≤ 2 |z| + 2
| z|
⇒ 1 – c2 = n2c2 + 1 – 2nc
⇒ |z|2 – 2 |z| + 1 ≤ 1 + 2 ⇒ (|z| – 1)2 ≤ 3

c 1

= (1) ⇒ – 3 ≤ |z| – 1 ≤ 3 ⇒ 1 – 3 ≤ |z| ≤ 1 + 3
2n 1 + n2 That is, the maximum value of |z| is 1 + 3
c ⎛ n⎞

Now, (1 + nz ) ⎜1 + ⎟ The correct option is (B)
2n ⎝ z⎠
77. We have, |z + z | + |z – z | = 8
1 ⎧ 2 ⎛ 1⎞ ⎫ ⇒ 2 |x| + 2 |y| = 8 or |x| + |y| = 4
= ⎨1 + n + n⎜ z + ⎟⎬
1+ n ⎩ 2 ⎝ z⎠⎭ The correct option is (C)

=
1
1 + n2
(1 + n 2
+ n( 2 cos q ) ) 78. Since z + 2 |z + 1| + i = 0
\ x + i (y + 1) + 2 |x + iy + 1| = 0

⎛ 2n ⎞ \ y + 1 = 0  [Q |x + iy + 1| is real]

= 1+ ⎜ ⎟ cos q = 1 + ccosq (Using (1))
\ y = –1
⎝ 1 + n2 ⎠

The correct option is (A) \ x + 2 |x – i + 1| = 0
1 − ak + 1
2 k ⇒ x2 = 2 [(x + 1)2 + 1] = 2 (x2 + 2x + 2)
73. We have, zk = 1 + a + a + … a =
1− a \ x2 + 4x + 4 = 0
1 −ak + 1 \ (x + 2)2 = 0
⇒ zk – =
1− a 1− a \ x = –2
1 | a |k + 1 1 \ z = –2 – i

zk − = < (Q |a| < 1)
1− a |1 − a | |1 − a | The correct option is (B)

Therefore, vertices z1, z2, …, zn of the polygon lie within the 79. Let a = r cos q and b = r sin q
circle. b
\ tan q =
a

Objective_Maths_JEE Main 2017_Ch 3.indd 44 01/01/2008 03:28:21


Complex Numbers  3.45

a − ib r (cos q − i sin q ) = 3a0 + a1 (1 + w + w2) + a2 (1 + w2 + w4)



Now, = = (cos q – i sinq)2
a + ib r (cos q + i sin q ) + a3 (1 + w3 + w6) + …
= cos 2q – i sin 2q = e– 2iq
\ 3n + 0n + 0n = 3a0 + 3a3 + 3a6 + …
a − ib \ 3n – 1 = a0 + a3 + a6 + …
\ i log
= i log e– 2iq = i (– 2 iq) = 2q
a + ib The correct option is (B)
⎡ a − ib ⎤ 84. The closest distance = length of the perpendicular from the
\ tan ⎢i log
= tan 2q origin on the line az + az + aa = 0
⎣ a + ib ⎥⎦
b a( 0 ) + a | 0 | + a a | a |2 | a |
2 = = =
2 tan q a 2| a | 2| a | 2
= =
1 − tan 2 q b2 The correct option is (B)
1− 2
a 85. Put 1 = r cos q and – 1 = r sin q
2ab p
= 2 ⇒ r = 2 and q = –
a − b2 4
The correct option is (C) Then, given equation takes the form
80. Given |z1 + z2| = |z1| + |z2| ⎡ ⎛ p⎞ ⎛ p⎞⎤
n

2 2 ( 2 )n ⎢cos ⎜ − ⎟ + i sin ⎜ − ⎟ ⎥ = 2n

Squaring, |z1 + z2| = (|z1| + |z2|)
⎣ ⎝ 4 ⎠ ⎝ 4⎠⎦
⇒ |z1|2 + |z2|2 + 2Re |z1| |z2|
⎡ np np ⎤
or, 2n/2 ⎢cos − i sin = 2n

= |z1|2 + |z2|2 + 2 |z1| |z2| ⎣ 4 4 ⎥⎦
⇒ 2Re |z1| | z2 | = 2 |z1| |z2|
Equating real and imaginary parts, we get
⇒ |z1| | z2 | cos (q1 – q2) = |z1| |z2|
np np
cos = 2n/2 and – sin =0
⇒ cos (q1 – q2) = 1 ⇒ q1 – q2 = 0 or q1 = q2
4 4
or, arg z1 = arg z2
These are satisfied only for n = 0

The correct option is (C) Hence, n = 0 is the only solution.
The correct option is (A)
81. For every a ∈ R, |a| = a 2 p
\ |a|2 = a2
86. We have, Arg (z + i) – Arg (z – i) =
2
Now, (|x| – |y|)2 ≥ 0
⎛ z + i⎞ p
⇒ Arg ⎜ =
⇒ |x|2 + |y|2 – 2 |x| |y| ≥ 0
⎝ z − i ⎟⎠ 2
⇒ 2 |x| |y| ≤ |x|2 + |y|2
⎛ z + i⎞
\ Re ⎜
=0
⇒ |x|2 + |y|2 + 2 |x| |y| ≤ 2 |x|2 + 2 |y|2
⎝ z − i ⎟⎠
⇒ (|x| + |y|)2 ≤ 2 (x2 + y2) ⇒ (|x| + |y|)2 ≤ 2 |z|2
⎛ z + i⎞ ⎛ z + i⎞
\ |x| + |y| ≤ 2 |z| ⎜⎝ ⎟ +⎜ ⎟
z − i⎠ ⎝ z − i⎠

=0

The correct option is (B) 2
82. As (a – b)2 ≥ 0, a2 + b2 ≥ 2ab(i) ⎛ z + i⎞ ⎛ z + i ⎞
⇒ ⎜
+ =0
⎝ z − i ⎟⎠ ⎜⎝ z − i ⎟⎠
But |z| = a 2 + b 2 ; so from (i), |z|2 ≥ 2ab

\ |z|2 + a2 + b2 ≥ a2 + b2 + 2ab
⇒ (z + i) ( z + i) + (z – i) ( z – i) = 0
or, |z|2 + |z|2 ≥ (a + b)2;
⇒ z z + i (z + z ) – 1 + z z – i (z + z ) – 1 = 0
\ 2 |z|2 ≥ (a + b)2
⇒ 2 (z z ) = 2 ⇒ z = 1 or |z|2 = 1
⇒ |z| = 1
\
2 |z| ≥ a + b as |z| is positive.
The equation represents a circle centered at origin and radius
1
\ |z| ≥ (a + b) 1 unit
2 The correct option is (A)
The correct option is (B)
87. Let z1 = x1 + iy1 and z2 = x2 + iy2
83. Putting x = 1, w, w2, respectively,
Now z1 + z2 = (x1 + x2) + i (y1 + y2)
3n = a0 + a1 + a2 + … + a2n
and, z1 – z2 = (x1 – x2) + i (y1 – y2)
(1 + w + w2)n = a0 + a1 w + a2 w2 + … + a2n w2n
As |z1 + z2| = |z1 – z2|, we get
(1 + w2 + w4)n = a0 + a1 w2 + a2 w4 + … + a2n w4n
(x1 + x2)2 + (y1 + y2)2 = (x1 – x2)2 + (y1 – y2)2

n 2 n 2 4 n
Adding these, 3 + (1 + w + w ) + (1 + w + w )
or, x1 x2 + y1 y2 = 0
(1)

Objective_Maths_JEE Main 2017_Ch 3.indd 45 01/01/2008 03:28:26


3.46  Chapter 3

y1 y p2 q2 r2
Now, amp z1 – amp z2 = tan– 1
– tan– 1 2 ⇒
+ +
= 2i
x1 x2
y1 y2 a2 b2 c2
− The correct option is (C)
x1 x2 x y − y2 x1
= tan– 1
= tan– 1 2 1 92. Interpretating according to Coni’s theorem. Let the angle
y1 y2 x1 x2 + y1 y2
1+ ⋅ between the lines joining
x1 x2
z1, z2 and z1, –z2 be a
= tan–1 ∞, by (1) z −z
p \ 1 2 = cos a + t sin a
\ |amp z1 – amp z2| = z1 + z2
2
The correct option is (B) Using Componendo and Dividendo, we have
88. We have, 2z 1 + cos a + t sin a
⎛ 1⎞ 1 =
− 3 > 1 = log1/ 2 ⎜ ⎟ −2 z2 cos a − 1 + t sin a
⎝ 2⎠
⎛ a⎞ a a
| z − 1| + 4 1 2 cos 2 ⎜ ⎟ + t 2 sin cos
⇒ < <1 z ⎝ 2⎠ 2 2
3 | z − 1| − 2 2 ⇒ 1 =

z2 2⎛a⎞ a a
[∵ loga x is a decreasing function if a < 1] −2 sin ⎜ ⎟ + t sin cos
⎝ 2⎠ 2 2
⇒ |z – 1| + 4 < 3 |z – 1| – 2 z1 a a

= t cot ⇒ tz1 = – cot z2
⇒ 2 |z – 1| > 6 ⇒ |z – 1| > 3 z2 2 2
which is an exterior of a circle.
But  tz1 = kz2 ⎧⎨where, k = − cot a (say)⎫⎬

The correct option is (C) ⎩ 2 ⎭
z z a a
89. We have, 1 + 2 = 1
Now, k = –cot ⇒ cot = –k
z2 z1 2 2
⇒ z12 + z22 = z1 z2
2k 2k
⇒ tan a = 2
⇒ tan a = –
⇒ z12 + z22 + z32 = z1 z2 + z2 z3 + z3 z1,
k −1 1− k2
where z3 = 0.
⎛ 2k ⎞
⇒ a = tan–1 ⎜⎝ − 1 − k 2 ⎟⎠ = –2 tan–1 k

⇒ z1, z2 and the origin form an equilateral triangle.


The correct option is (C) z1 − z2
Now,
= cos a + i sin a
z1 + z2
90. Mid-point of P and P′ is centre of circle C such that
where a is the angle between (z1 – z2) and (z1 + z2)

z1 + ( − z1 )
=0 ⇒ a = –2tan–1k is the angle between (z1 – z2) and (z1 + z2).

2

The correct option is (C)
\ Centre of circle lies at origin.
Now, the equation of circle with centre at origin and radius 93. x2 – 3x + 1 = 0
|z1| or |–z1| is 3 ± 3−4 3 i p p
|z – 0| = |z1| ⇒ x =
= ± = cos + i sin
2 2 2 6 6
⇒ |z|2 = |z1|2
P(–z1) 2
⎛ n 1⎞
⎛ 1 ⎞
= ⎜ x 2 n + 2 n − 2⎟
⎜⎝ − n ⎟⎠
x
⇒ z · z = z1 · z1
⎝x x ⎠
z z ⎛z ⎞ np
\
= 1 = ⎜ 1⎟ P(z1) = –2 + 2 cos

z1 z ⎝ z⎠ 3
24 2 24
⎛ 1⎞ ⎛ np ⎞
The correct option is (A) ⇒
∑ ⎜⎝ x n − x n ⎟⎠ = ∑ ⎜⎝ −2 + 2 cos 3⎠

91. We have, n =1 n =1
2
⎛ p q r⎞ ⎡ p 2p 24p ⎤
(1 + i)2 = ⎜ + + ⎟ = –48 + 2 ⎢cos + cos + ... + cos
⎝ a b c⎠ ⎣ 3 3 3 ⎥⎦
p2 q2 r2 ⎛ qr rp pq ⎞ ⎡ ⎛ 2p 23p ⎞ ⎛ 24p ⎞ ⎤
⇒ 1 – 1 + 2i =
+ + + 2⎜ + + +2 ⎢cos ⎜ + ⎟⎠ .sin ⎜⎝
a 2
b 2
c2 ⎝ bc ca ab ⎟⎠ ⎣ ⎝ 3 6

6 ⎠ ⎥⎦
= – 48
p 2
q 2
r 2
2abc ⎛ a q r ⎞ p
⇒ 2i =
+ + + + + sin
a 2
b 2
c 2
pqr ⎜⎝ p b c ⎟⎠ = 0 – 48 = –48
6
2 2 2
p q r 2abc
The correct option is (B)
=
+ + + (0)
a2 b2 c2 pqr

Objective_Maths_JEE Main 2017_Ch 3.indd 46 01/01/2008 03:28:32


Complex Numbers  3.47

2 2 OB = |z2 – 0| = |z2|
94. 2 z1 + i 3 z2 + 3 z1 + i 2 z2
OA = |z1 – 0| = |–z2w| = |z2| |–w |
= ( 2z + i 1 3 z2)( 2 z1 − i 3 z2 ) = |z2|

+ ( 3z +i 2 z )( 2z )
and,  AB = |z2 – z1| = |z2 + z2w|
1 2 3 z1 − i 2
= |z2| |1 + w | = |z2| |–w2| = |z2|
2 2 2 2
= 5 (|z1| + |z2| ) > 5 · 2 z1 z2 = 10 |z1 z2| Thus, z1, z2 and origin form an equilateral triangle.
The correct option is (D)
Since A · M > G · M for |z1| ≠ |z2|


The correct option is (B) z1 − z2 z /z − 1
99. We have, =1⇒ 1 2 =1
25 2 25 z1 + z2 z1/ z2 + 1
95. If z is a root of (z – 1) = 2w (z + 1) , then
⎛ z − 1⎞
25
z −1
25 z1 z
= 2w2 ⇒ = 2 |w2| = 2 ⇒ −1 = 1 +1
⎜⎝ ⎟ z2 z2
z + 1⎠ z +1
z1
z −1 ⇒ lies on the perpendicular bisector of the segment

= 21/25 z2
z +1 joining A (–1 + 0i) and B (1 + 0i).
As 21/25 ≠ 1, we get z lies on a circle.
z1
\ = ai for some a ∈ R

The correct option is (B) z2
z 1 −i
b ⇒ 2 = =
96. z1 + z2 = – (1) z1 ai a
a
c \ z2 = i kz1 for some k ∈ R
z1z2 = (2)
a The correct option is (A)
z2 = iz1(3)
100. We have,
From equations (1) and (3), we have b + ic
−b −b 1+ i
z1(1 + i) = ⇒ z1 = (1 – i) 1 + iz 1+ a 1 + a − c + ib
a 2a = =
1 − iz 1 − i b + ic 1 + a + c − ib
b2 −b 2 1+ a
⇒ z12 =
2
i (–2i) =
4a 2a 2 [1 + a − c + ib] [1 + a + c + ib]
=
From equations (2) and (3), we have
[1 + a + c − ib] [1 + a + c + ib]
c −c z0
z12 = = i = (1)
ai a (1 + a + c) 2 + b 2
2 2
−b −c Now,  R (z0) = (1 + a – c) (1 + a + c) – b2

2
i= 2
i
2a a
= (1 + a)2 – c2 – b2 = 1 + a2 + 2a – (1 – a2)
b2 = 2a2 + 2a = 2a (1 + a)
\ a =

2c and, Im(z0) = 2b (1 + a).

The correct option is (A)
z3
Thus, z0 = 2 (1 + a) (a + ib)
z1 − z3 1 − i 3 Also, denominator of (1) = 1 + a2 + c2 + 2a + 2c + 2ac + b2
97. =
z2 − z3 2 π = 2 + 2a + 2c + 2ac
3
⎛ −p ⎞ ⎛ −p ⎞ = 2 (1 + a) (1 + c)
= cos ⎜ + sin ⎜ = e–ip/3
⎝ 3 ⎟⎠ ⎝ 3 ⎟⎠ 1 + iz a + ib
Therefore, =
z1 − z3 z1 z2 1 − iz 1+ c
\
= e − i p /3 = 1
z2 − z3 The correct option is (B)
p 101. We have,
and angle between z1 – z3 and z2 – z3 is .
\ triangle is equilateral.
3
(
|z1 + z2 wk|2 = ( z1 + z2 w k ) z1 + z2 w k )
The correct option is (C)
z = ( z1 + z2 w k ) ( z1 + z2 w − k )
98. If 1 = z, the given equation becomes
z2
⎡⎣ w k = ei ( 2p k / n) ⇒ w k = e − i ( 2p k / n) = w − k ⎤
2
z – z + 1 = 0 i.e., z = – w and – w
2 ⎦⎥
z
or, 1 = – w ⇒ z1 = – z2w
= | z1 |2 + | z2 |2 + z1 z2 w k + z1z2 w − k
z2

Objective_Maths_JEE Main 2017_Ch 3.indd 47 01/01/2008 03:28:39


3.48  Chapter 3

Therefore, we have, Putting x = w on both sides, we have


n −1 n −1 xn − 1 ⎛ 0 ⎞
∑ | z1 + z2 w k |2 = n (| z1 |2 + | z2 |2 ) + z1z2 ∑ wk (w – 1) (w – w2) … (w – wn–1) = lim ⎜ ⎟
x→w x − w ⎝ 0⎠
k =0 k =0
n −1 nx n −1
= lim = nwn–1
+ z1z2 ∑ w −k
x→w 1
k =0
The correct option is (A)
⎡ n −1 n −1 ⎤
= n (| z1 |2 + | z2 |2 ) ⎢ ∑ w k = 0 = ∑ w − k ⎥ . 105. We have,
⎣k =0 k =0 ⎦ z – i = eia ⇒ z = i + eia
The correct option is (B) = cos a + i (1 + sin a)
102. We have, |z – z1|2 + |z – z2|2 = k ⎛ 1 + sin a ⎞
\ q = arg (z) = tan −1 ⎜
2 2 2
⇒ 2 | z | + | z1 | + | z2 | − 2 Re( z z1 ) − 2 Re( z z2 ) = k ⎝ cos a ⎟⎠
1 + sin a
⇒ 2 | z |2 − 2 Re { z ( z1 + z2 )} = k − (| z1 |2 + | z2 |2 ) ⇒ tan q =
cos a
1
⇒ | z |2 − Re { z ( z1 + z2 )} = ( k − | z1 |2 − | z2 |2 ) 2
\ cot q − =
cos a

2
2 z 1 + sin a cos a + i (1 + sin a )
2
z1 + z2 1 1
⇒ z− − | z1 + z2 |2 = ( k − | z1 |2 − | z2 |2 ) cos a 2 [cos a − i (1 + sin a )]
2 4 2 = −
2
1 + sin a cos 2 a + (1 + sin a ) 2
⇒ z−
z1 + z2
2
=
1
2
k−
1
4
{
| z1 |2 + | z2 |2 − 2 Re( z1 z2 )}
=
cos a

2 [cos a − i (1 + sin a )]
1 + sin a 2 (1 + sin a )
1 1 = i.
= k − | z1 − z2 |2
2 4 The correct option is (B)
2
z1 + z2 1 106. We have,
⇒ z− = ( 2k − | z1 − z2 |2 )
2 4 4 + 3i ( 4 + 3i ) (1 − 2i )
z1 + z2 z1 = =
which will represent a real circle having centre at 1 + 2i (1 + 2i ) (1 − 2i )
2
10 − 5i
1 1 = =2–i
and radius = 2k − | z1 − z2 |2 , provided k ≥ | z1 − z2 |2 5
2 2 which represents the point whose coordinates are (2, –1)
The correct option is (A) Also, we have,
103. Let a z + b be the remainder when f (z) is divided by z2 + 1. iz = z
Then, we have
⇒ i(x + iy) – (x – iy) = 0 [Putting z = x + iy]
f (z) = (z2 + 1) g(z) + Az + B
⇒ i(x + y) – (x + y) = 0
Given: f (z) when divided by z – i gives remainder i
⇒ (i – 1) (x + y) = 0
⇒ f (i) = i
which represents the line y = –x
⇒ (i2 + 1) g(i) + Ai + B = i
Hence, reflection of the point (2, –1) in the line y = –x gives
⇒ Ai + B = i (1) the point (1, –2) which is equivalent to 1 – 2i in the argand
Also, f (z) when divided by z + i gives remainder i + 1 plane.
⇒ f (– i) = i + 1 The correct option is (D)
⇒ (i2 + 1) g(– i) – B = i + 1 1 3
⇒ –Ai + B = i + 1 (2) 107. We know that w = − + i
2 2
Solving equations (1) and (2), we get 334 365
⎛ 1 i 3⎞ ⎛ 1 i 3⎞
A = i/2 and B = i + 1/2 Thus, 4 + 5 ⎜ − + + 3 ⎜− +
⎛ i⎞ ⎛ 1⎞ ⎝ 2 2 ⎟⎠ ⎝ 2 2 ⎟⎠
\ remainder is ⎜ ⎟ z + ⎜ i + ⎟
⎝ 2⎠ ⎝ 2⎠ = 4 + 5w334 + 3w365
The correct option is (C) = 4 + 5 (w3)111 w + 3 (w3)123 w2
104. We have, = 4 + 5 (1)111 w + 3 (1)123 w2
xn – 1 = (x – 1) (x – w) (x – w2) … (x – wn–1) = 4 + 5w + 3w2 = 1 + 2w + 3 (1 + w + w2)
xn − 1 = 1 + 2w + 3 (0) = 1 + 2w = 1 – 1 + 3 i = 3 i.
⇒ = (x – 1) (x – w2) … (x – wn–1)
x−w The correct option is (C)

Objective_Maths_JEE Main 2017_Ch 3.indd 48 01/01/2008 03:28:45


Complex Numbers  3.49

108. The given line is ⎛ a⎞ a


⇒ p2 ⎜1 + cot 2 ⎟ = 4q cot2
b z + b z = c(1) ⎝ 2⎠ 2
Let A (z1) be a reflection of a
B (z2) in the line (1). ⇒ p2 cosec2 = 4q cot2
2
Let P (z) be any point on the line a
⇒ p2 = 4q cos2
(1). We have, 2
\ k = 4q
AP = BP
The correct option is (C)
⇒ |AP|2 = |BP|2
110. Since |z1| = |z2| = |z3| = 1,
⇒ |z – z1|2 = |z – z2|2 we get, z1 z1 = z2 z2 = z3 z3 = 1.
⇒ (z – z1) ( z – z1 ) = (z – z2) ( z – z2 ) 1 1 1
⇒ ( z2 – z1 ) z + (z2 – z1) z + z1 z – z2 z2 = 0 (2) Now, 1 = + + = | z1 + z2 + z3 |
z1 z2 z3
Since (1) and (2) represent the same line, we get
b b c = z1 + z2 + z3 = |z1 + z2 + z3|.
= = = k (say)
z2 − z1 z2 − z1 z1 z1 − z2 z2 The correct option is (A)
⇒ k ( z2 – z1 ) = b , k (z2 – z1) = b, k (z1 z1 – z2 z2 ) = c 111. Let z = r1 (cos a + i sin a)
Now, z1 b + z2 b and, w = r2 (cos b + i sin b) ⇒ Arg z = a, Arg w = b
|z| = r1, |w| = r2.
= z1 {k (z2 – z1)} + z2 {k ( z2 – z1 )}
Given: |z| ≤ 1, |w | ≤ 1 ⇒ r1 ≤ 1, r2 ≤ 1
= k { z1 z2 – z1 z1 + z2 z2 – z2 z1 }
Now, consider |z – w |2 = |(r1 cos a – r2 cos b)
= k (z2 z2 – z1 z1 ) = c.
+ i (r1 sin a – r2 sin b )2|
The correct option is (C)
= (r1 cos a – r2 cos b)2 + (r1 sin a – r2 sin b)2
109. We have, z1 + z2 = – p and z1 z2 = q
We know that = r12 (cos2a + sin2a) + r22 (cos2b + sin2b)
–2 r1 r2 (cos a cos b + sin a sin b)
z1 | z1 |
= (cos a + i sin a) = r12 + r22 – 2 r1 r2 cos (a – b)
z2 | z2 |
Since |z1| = |z2|  (Q OA = OB) = (r1 – r2)2 + 2 r1 r2 [1 – cos (a – b)]
z1 cos a + i sin a ⎛ a − b⎞
we get = = (r1 – r2)2 + 4 r1 r2 sin2 ⎜⎝ 2 ⎟⎠
z2 1
2
Applying Componendo and Dividendo, we get ⎛ a − b⎞
≤ (r1 – r2)2 + 4 × 1 × 1 ⎜
z1 + z2 cos a + i sin a + 1 ⎝ 2 ⎟⎠
=
z1 − z2 cos a + i sin a − 1 or, |z – w|2 ≤ (|z| – |w|)2 + (Arg z – Arg w)2.
a a a The correct option is (B)
2 cos 2 + 2i sin cos
= 2 2 2 112. A(z1), B(z2), C(z3) lie on |z| = 2 whose centre is at O(0, 0)
a a a
−2 sin 2 + 2i sin cos and radius 2.
2 2 2 p
z1 = 1 + 3 i hence |z| = 2 and Arg (z1) =
a ⎡ a a⎤ 3
2 cos
cos + i sin
2 ⎢⎣ 2 2 ⎥⎦ a
= = –i cot
a ⎡ a a⎤ 2
2i sin ⎢cos + i sin
2 ⎣ 2 2 ⎥⎦
p a
⇒ = i cot
z1 − z2 2
Squaring we obtain
p2 a
= –cot2
( z1 + z2 ) 2 − 4 z1z2 2
p2 a
⇒ 2
= –cot2 In turn |z2| = |z3| = 2 and Arg (z2) = Arg (z1) + 120º = 180º
p − 4q 2
\ z2 = –2
2 2 2 a a
⇒ p = – p cot + 4q cot2 Further, Arg (z3) = Arg (z2) + 120º = 300º
2 2

Objective_Maths_JEE Main 2017_Ch 3.indd 49 01/01/2008 03:28:54


3.50  Chapter 3

which gives imaginary values of a and b.


⎡ ⎛ p⎞ ⎛ p⎞⎤
Hence,  z3 = 2 ⎢cos ⎜ 2p − ⎟ + i sin ⎜ 2p − ⎟ ⎥ Hence, a = b = 2 – 3
⎣ ⎝ 3 ⎠ ⎝ 3⎠⎦
The correct option is (B)
⎡ p p⎤ ⎛ 1 i 3⎞
= 2 ⎢cos − i sin ⎥ = 2 ⎜ − 115. Let z1 = x1 + iy1 and z2 = x2 + iy2
⎣ 3 3⎦ ⎝2 2 ⎟⎠
where, x1 ≠ x2, y1 ≠ y2 and x12 + y12 = x22 + y22
= 1 – 3 i
z1 + z2 ( x + x2 ) + i ( y1 + y2 )
Thus, z2 = –2 and z3 = 1 – i 3 Now, = 1
z1 − z2 ( x1 − x2 ) + i ( y1 − y2 )
The correct option is (A)
113. We know that, =
[( x1 + x2 ) + i ( y1 + y2 )] [( x1 − x2 ) − i ( y1 − y2 )]
(1 + x)3n = 1 + 3nC1 x + 3nC2 x2 + … + 3nC3n x3n(1) ( x1 − x2 ) 2 + ( y1 − y2 ) 2
(1 – x)3n = 1 – 3nC1 x + 3nC2 x2 + … + (– 1)3n 3nC3n x3n(2) [( x12 − x22 ) + ( y12 − y22 )] + i [ x1 y1 − y1x2 + y2 x1
Subtracting (2) from (1) gives − y2 x2 − x1 y1 + x1 y2 − x2 y1 + x2 y2 ]
3n 3n 3n 3n 3 3n 5
=
(1 + x) – (1 – x) = 2 [ C1 x + C3 x + C5 x + …] ( x1 − x2 ) 2 + ( y1 − y2 ) 2
3n 3n 2 3n 4
= 2x [ C1 + C3 x + C5 x + …] 2 i ( x1 y2 − y1x2 )
=
Putting x = i 3 , we get ( x1 − x2 ) 2 + ( y1 − y2 ) 2
x1 y
(1 + i 3 )3n – (1 – i 3 )3n = a purely imaginary or 0 if = 1.
x2 y2
= 2i 3 [3nC1 – 3 × 3nC3 + 32 × 3nC5 …] x1 y
If = 1 then x1 + iy1 = k (x2 + iy2)
Therefore, 3nC1 – 3 3nC3 + 32 × 3nC5 … x2 y2
1 If k = 1, z1 = z2, which is not true and if k ≠ 1, |z1| ≠ |z2|.
= [(1 + i 3 )3n – (1 – i 3 )3n]
2i 3
z1 + z2
⎡⎛
1 i 3 ⎞ ⎛ 1 i 3 ⎞ ⎤⎥
3n
1 \ is purely imaginary.
= × 23n ⎢⎜ + − − z1 − z2
2i 3 ⎢⎝ 2 2 ⎟⎠ ⎜⎝ 2 2 ⎟⎠ ⎥
⎣ ⎦ The correct option is (D)
23n −1 116. Since |CA| = |CB| and ∠ACB = 90º
= [(cos np + i sin np) – (cos np – i sin np)]
i 3 \ (z2 – z3) = ± i (z1 – z3)
23n −1 ⇒ (z2 – z3)2 = – (z1 – z3)2
= 2i sin np = 0 as n is an integer.
i 3 ⇒ z22 + z32 − 2 z2 z3 = − z12 − z32 − 2 z1 z3
The correct option is (A) z12 + z22 − 2 z1 z2 = 2 ( z1 z3 − z1 z2 − z2 z3 + z22 )
114. We know that the triangle with vertices z1, z2, z3 is an
⇒ (z1 – z2)2 = 2 [z1 – z3] [z3 – z2]
­equilateral if
The correct option is (B)
z12 + z22 + z32 = z1 z2 + z2 z3 + z3 z1
117. Let A = z1, B = z2 and C = z3,
\ The triangle with vertices z1 = a + i, z2 = 1 + bi and
z3 = 0 will be equilateral if where A, B, C are vertices of equilateral triangle.
(a + i)2 + (1 + bi)2 + 0 = (a + i) (1 + bi) + 0 + 0 Given that third point C is origin, so z3 = 0.
⇒ a2 – 1 + 2ai + 1 – b2 + 2bi = (a – b) + i (1 + ab) Let z2 – z3 = a, z3 – z1 = b, z1 – z2 = g
⇒ a2 – b2 = a – b (1) or, z2 = a, – z1 = b, z1 – z2 = g
and 2 (a + b) = 1 + ab (2) \ a + b + g = z2 – z1 + z1 – z2 = 0
[Equating real and imaginary parts] or, a + b + g = 0 (1)
(1) ⇒ (a – b) [a + b – 1] = 0 Since the triangle is equilateral triangle,
⇒ a = b or a = 1 – b. \ BC = CA = AB
Subsitituting the value of a – b in (2), we get or, |(z2 – 0)| = |0 – z1| = |z1 – z2|
2 (a + a) = 1 + a2 ⇒ a2 – 4a + 1 = 0 or, |a| = |b | = |g |
4 ± 16 − 4 or, |a|2 = |b |2 = |g |2
⇒ a = =2± 3
2 or, a a =b b =g g = k (say)
Since 0 < a < 1 and 0 < b < 1,
k k k
\ a = b = 2 – 3 . \ a = , b = , g =
a b g
Substituting a + b = 1 in (2), we get
2 = 1 + a (1 – a) ⇒ a2 – a + 1 = 0

Objective_Maths_JEE Main 2017_Ch 3.indd 50 01/01/2008 03:28:59


Complex Numbers  3.51

Substituting values of a , b and g in (1), we get ⎛ 2p 9p ⎞ 10p ⎛ 2p 9p ⎞ 10p


sin ⎜ + sin cos ⎜ + sin
k k k ⎝ 11 11 ⎟⎠ 11 ⎝ 11 11 ⎟⎠ 11
+ + =0 = −i
a b g p p
sin sin
11 11
1 1 1
or, + + =0 = 0 – i (– 1) = i (1)
a b g p
32 ⎡ 10 ⎛ 2qp 2qp ⎞ ⎤
1 1 1 \ S = ∑ (3 p + 2) ⎢ ∑ ⎜ sin − i cos ⎟⎥
or, + + =0 ⎢⎣ q =1 ⎝ 11 11 ⎠ ⎥⎦
z2 − z1 z1 − z2 p =1
32 32 32
z1 ( z1 − z2 ) − z2 ( z1 − z2 ) + z1 z2
or, =0 = ∑ (3 p + 2) i p =3 ∑ pi p + 2 ∑ i p
z1 z2 ( z1 − z2 ) p =1 p =1 p =1

or, z12 – z1 z2 – z1 z2 + z22 + z1 z2 = 0 = 3A + 2B(2)


Now,  A = i + 2i2 + 3i3 + … + 32i32
or, z12 + z22 = z1 z2
⇒ Ai = i2 + 2i3 + … + 31i32 + 32i33
z12 z22 ⇒ A (1 – i) = i + i2 + … + i32 – 32i33
or, + =1
z1 z2 z1 z2
i 32 − 1
z z =– 32i = – 32i [∵ i32 = 1]
or, 1 + 2 = 1 i −1
z2 z1 −32i 32i (1 + i )
\ A = = = 16 (1 – i)(3)
A B 1− i 2
or, + = 1.
B A and,  B = i + i2 + … + i32 = 0 (4)
The correct option is (A) Hence,  S = 3 × 16 (1 – i) = 48 (1 – i).
118. Given 2 2 x4 = ( 3 – 1) + i ( 3 + 1) The correct option is (C)
120. Since the triangle is equilateral
⎛ 3 − 1⎞ ⎛ 3 + 1⎞
⇒ x4 = ⎜ ⎟ + i⎜ ⎟ \ |z1 – 0| = |z2 – z1| = |0 – z2|
⎝ 2 2 ⎠ ⎝ 2 2 ⎠
⇒ |z1|2 = |z2 – z1|2 = |z2|2
2 2
⎛ 3 − 1⎞ ⎛ 3 + 1⎞ ⇒ z1 z1 = (z2 – z1) × ( z2 – z1 ) = z2 z2
⇒ |x4|2 = ⎜ ⎟ +⎜ ⎟ =1
⎝ 2 2 ⎠ ⎝ 2 2 ⎠ z z z −z
Now,  z1 z = z2 z2 ⇒ 1 = 2 = 1 2 .
\ |x4| = 1 1 z2 z1 z2 − z1
⎡⎛ 3 + 1⎞ ⎛ 3 − 1⎞ ⎤ ( z1 − z2 ) z2
Also,  z2 z2 = (z2 – z1) ( z2 – z1 ) = (z2 – z1)
and,  arg (x4) = tan– 1 ⎢⎜ ⎟ ⎜ ⎟⎥ z1
⎢⎣⎝ 2 2 ⎠ ⎝ 2 2 ⎠ ⎥⎦
\ z1 z2 = (z2 – z1) (z1 – z2) = z2 z1 – z12 − z22 + z1 z2
⎛ 3 + 1⎞ 5p ⇒ z12 + z22 = z1 z2.
= tan– 1 ⎜ ⎟ = 75º = 12
⎝ 3 − 1⎠ The correct option is (B)
⎡ ⎛ 5p ⎞ ⎛ 5p ⎞ ⎤ 121. We have, max. amp (z) = amp (z2),
\ x4 = 1 ⎢cos ⎜ 2np + ⎟ + i sin ⎜ 2np + ⎟ ⎥ min. amp (z) = amp (z1).
⎣ ⎝ 12 ⎠ ⎝ 12 ⎠ ⎦
n
Using (cos q + i sin q) = cos nq + i sin nq, we have
1⎛ 5p ⎞ 1⎛ 5p ⎞
x = cos ⎜ 2np + ⎟⎠ + i sin ⎜⎝ 2np + ⎟⎠ ;
4⎝ 12 4 12
n = 0, 1, 2, 3
The correct option is (B)
10
⎛ 2qp 2qp ⎞
119. ∑ ⎜⎝ sin 11
− i cos ⎟
11 ⎠ ⎛ 15 ⎞ ⎛ 3⎞
q =1 Now, amp (z1) = q1 = cos–1 ⎜ ⎟ = cos–1 ⎜ ⎟
⎝ 25 ⎠ ⎝ 5⎠
⎧ 2p 4p ⎫ p
= ⎨sin + sin + ... + 10 terms⎬ and,  amp (z2) = + q2
⎩ 11 11 ⎭ 2
p ⎛ 15 ⎞
⎧ 2p 4p ⎫ = + sin–1 ⎜ ⎟
i ⎨cos + cos + ... + 10 terms⎬ 2 ⎝ 25 ⎠
⎩ 11 11 ⎭ p ⎛ 3⎞
= + sin–1 ⎜ ⎟
2 ⎝ 5⎠

Objective_Maths_JEE Main 2017_Ch 3.indd 51 01/01/2008 03:29:07


3.52  Chapter 3

p 3 3 z
\ |max. amp (z) – min. amp (z)| = + sin −1 − cos −1 Now, lies in the IIIrd quadrant if A < 0
2 5 5 z
p p 3 3 i.e., if x2 – y2 < 0 or x2 < y2 i.e., if x < y < 0.
= + − − cos −1 − cos −1
2 2 5 5 The correct option is (C)
⎛ 3⎞ 125. Let BA = BC
= p – 2 cos–1 ⎜ ⎟
⎝ 5⎠ ⇒ |z1 – z2|2 = |z3 – z2|2
The correct option is (C) ⇒ (z1 – z2) ( z1 – z2 ) = (z3 – z2) ( z3 – z2 )(1)
122. Putting z = x + iy, we get Again,
(x + iy)2 + (p + iq) (x + iy) + r + is = 0 \ ∠ABC = 90º
⇒ (x2 – y2 + px – q y + r) + i (2xy + py + qx + s) = 0 BA
\ arg = 90º
⇒ x2 – y2 + px – q y + r = 0 (1) BC
z −z z −z
and, 2xy + py + qx + s = 0 (2) ⇒ arg 1 2 = 90º ⇒ real part of 1 2 = 0
z3 − z2 z3 − z2
If the roots are real, then y = 0
\ (1) gives x2 + px + r = 0 1 ⎡ z1 − z2 z1 − z2 ⎤
(3) ⇒ ⎢ + ⎥ =0
and (2) gives pqx + s = 0 (4) 2 ⎣ z3 − z2 z3 − z2 ⎦
s
From (4), x = –
q
s 2 ps
Putting in (3), we get 2 − +r =0
q q
or, r2 – pqs + rq2 = 0 ⇒ pqs = s2 + rq2.
The correct option is (A)
1 23. We know that |z1 + z2|2 + |z1 – z2|2

= 2 [|z1|2 + |z2|2](1)
z1 − z2 z −z z −z z −z
2 ⇒ = – 1 2 ⇒ 1 2 = 2 3 (2)
Now, ⎡ z1 + z12 − z22 + z1 − z12 − z22 ⎤ z3 − z2 z3 − z2 z1 − z2 z3 − z2
⎣⎢ ⎦⎥
2 2 (1) × (2) ⇒ (z1 – z2)2 = – (z2 – z3)2
= z1 + z12 − z22 + z1 − z12 − z22 + 2 | z12 − ( z12 − z22 )|
⇒ z12 + z22 + z32 = 2 z2 (z1 + z3)
= 2 |z1|2 + 2 | z12 − z22 | + 2 | z22 | [By (1)] The correct option is (B)
= 2 |z1|2 + 2 |z2|2 + 2 | z12 − z22 | 126. Let z = r (cos q + i sin q), then
= |z1 + z2|2 + |z1 – z2|2 + 2 |z1 + z2| |z1 – z2| z + iz = r (cos q + i sin q) + ir (cos q + i sin q)
= (|z1 + z2| + |z1 – z2|)2 = r [(cos q – sin q) + i (sin q + cos q)]
Taking square root of both sides, we get ⎡ ⎛ p⎞ ⎛ p⎞⎤
= 2 r ⎢cos ⎜ q + ⎟ + i sin ⎜ q + ⎟ ⎥
⎣ ⎝ 4 ⎠ ⎝ 4⎠⎦
z1 + z12 − z22 + z1 − z12 − z22 = |z1 + z2| + |z1 – z2|.

The correct option is (D)


124. Since z = x + iy lies in the third quadrant
\ x < 0, y < 0. Again z = x – iy
z x − iy ( x − iy )( x − iy ) x 2 − y 2 − 2ixy
\ = = =
z x + iy x2 + y2 x2 + y2
x2 − y2 2 xy
= 2 2
− i = A + iB
x +y x + y2
2

2
x −y 2
2 xy In DOPQ,
where,  A = and B = – p
PQ2 = r2 + ( 2r ) 2 − 2r ( 2r ) cos
x2 + y2 x2 + y2 4
Since x < 0, y < 0 = r2 + 2r2 – 2r2 = r2
− 2 xy \ PQ = r,
\ <0 p
x2 + y2 \ ∠OPQ =
2
\ B < 0 The correct option is (C)

Objective_Maths_JEE Main 2017_Ch 3.indd 52 01/01/2008 03:29:12


Complex Numbers  3.53

127. Given |z + 1| < |z – 2| 1 31. We have, eiA, eiB, eiC are in A. P.


and,  w = 3z + 2 + i ⇒ 2eiB = eiA + eiC
\ w = 3 z + 2 – i ⇒ 2 (cos B + i sin B)
\ w + w = 3 (z + z ) + 4 (i) = (cos A + cos C) + i (sin A + sin C)
Now, |z + 1|2 < |z – 2|2 ⇒ 2 cos B = cos A + cos C
⇒ (z + 1) ( z + 1) < (z – 2) ( z − 2) and,  2 sin B = sin A + sin C
⇒ (z + 1) ( z + 1) < (z – 2) ( z – 2) A+C A−C
⇒ 2 cos B = 2 cos cos (1)
⇒ z z + z + z + 1 < z z – 2z – 2 z + 4 2 2
A+C A−C
⇒ 3z + 3 z < 3 ⇒ z + z < 1 (ii) and, 2 sin B = 2 sin cos (2)
2 2
From (i) and (ii), we get
Dividing (1) by (2), we get
w + w < 3 × 1 + 4 = 7 ⇒ w + w < 7
⎛ A+C⎞ B 3B
Clearly, |w + 1| < |w – 8| gives cot B = cot ⎜ = tan ⇒ cos =0
⎝ 2 ⎟⎠ 2 2
|w + 1|2 < |w – 8|2 3B p p 2p

⇒ (w + 1) ( w + 1) < (w – 8) ( w – 8) ⇒ =  or B = ⇒A+C= .
2 2 3 3
⇒ w w + w + w + 1 < w w – 8 w – 8w + 64 Putting this value in (1), we get
⇒ 9 (w + w ) < 63 ⇒ w + w < 7 p p ⎛ A−C⎞ ⎛ A−C⎞
2cos = 2 cos cos ⎜ ⇒ cos ⎜ =1
The correct option is (A) 3 3 ⎝ 2 ⎟⎠ ⎝ 2 ⎟⎠
128. We have, x2 + x + 1 = (x – w) (x – w2) ⎛ A−C⎞ p
⇒ ⎜ = 0 or A = C ⇒ A = B = C = .
Since f (x) is divisible by x2 + x + 1, f (w) = 0, f (w2) = 0 ⎝ 2 ⎟⎠ 3
\ P(w3) + w Q (w3) = 0 or P(1) + w Q(1) = 0 (1) The correct option is (C)
6 2 6 2
and,  P(w ) + w Q (w ) = 0 or P(1) + w Q(1) = 0 (2) 132. Let cot–1 p = q, then p = cot q
m m
From (1) and (2) we obtain ⎛ pi + 1⎞ ⎛ i cot q + 1⎞
= e 2 miq ⋅ ⎜
−1

\ e 2 mi cot p
⋅⎜
P(1) = 0 and Q(1) = 0. ⎝ pi − 1⎟⎠ ⎝ i cot q − 1⎟⎠
\ Both P(x) and Q(x) are divisible by x – 1 ⎛ i (cot q − i ) ⎞
m
⎛ cot q − i ⎞
m
= e 2 miq ⋅ ⎜ = e 2 miq ⋅ ⎜
Since f (x) = P(x) + x Q(x), we get f (x) is divisible by x – 1. ⎝ i (cot q + i ) ⎟⎠ ⎝ cot q + i ⎟⎠
The correct option is (D) m m
⎛ cos q − i sin q ⎞ ⎛ e −i q ⎞
⎛ 8p ⎞ ⎛ 8p ⎞
8p = e 2 miq ⋅ ⎜ = e 2 miq ⋅ ⎜ i q ⎟
⎝ cos q + i sin q ⎟⎠
i
129. We have, a = cos ⎜ ⎟ + isin ⎜ ⎟ = e 11 ⎝ e ⎠
⎝ 11 ⎠ ⎝ 11 ⎠
= e2miq (e–2iq)m = e2miq × e–2miq = e0 = 1
Re (a + a2 + a3 + a4 + a5)
The correct option is (B)
a + a2 + a3 + a4 + a5 + a + a2 + a3 + a4 + a5 133. Since z1 and z1 are the adjacent vertices of a regular
=
2 2p
­polygon of n sides, we have, ∠ z1 0 z1 =
−1 + (1 + a + a 2 + a 3 + a 4 + a 5 + a + a 2 + a 3 + a 4 + a 5 ) n
= and, |z1| = | z1 |
2 Thus, z1 = z1 e2pi/n
−1 + 0
= (sum of 11, 11th roots of unity) Let z1 = r (cos q + i sin q) = reiq
2
1 ⇒ z1 = re– iq
= –
2 Since z1 = z1 e2pi/n
The correct option is (B) ⇒ reiq = re– iq e2pi/n = re2pi/n – iq
1 − ak + 1
130. We have, zk = 1 + a + a2 + … + ak = 2p p
1− a ⇒ q = – q or q =
1 − a k +1 n n
⇒ zk – = ⎡ p p⎤
1− a 1− a Therefore, z1 = r (cos q + i sin q) = r ⎢cos + i sin ⎥
⎣ n n⎦
1 | a |k +1 1 ⎛p⎞
⇒ zk − = <   (∵ |a| < 1) r sin ⎜ ⎟
1− a |1 − a | |1 − a | Im ( z1 ) ⎝ n⎠
Now, = 2 –1⇒ = 2 –1
\ Vertices of the polygon z1, z2, …, zn lie within the circle Re ( z1 ) ⎛p⎞
r cos ⎜ ⎟
1 1 ⎝ n⎠
z − = p p
1− a |1 − a | ⇒ tan = 2 – 1 = tan ⇒n=8
n 8
The correct option is (C) The correct option is (B)

Objective_Maths_JEE Main 2017_Ch 3.indd 53 01/01/2008 03:29:21


3.54  Chapter 3

134. We have,
2 1 1 z +z
= + = 3 2
z1 z2 z3 z2 z3
2 z2 z3
⇒ z1 = .
z2 + z3
⎛ z − z4 ⎞ ⎛ z1 − z3 ⎞
Now, ⎜ 2
⎝ z1 − z4 ⎟⎠ ⎜⎝ z2 − z3 ⎟⎠
⎛ ⎞ ⎛ 2 z2 z3 ⎞
− z3
⎜ z −z ⎟ ⎜z +z ⎟
2 3
= ⎜ 2 4
⎟ ⎜ ⎟ Greatest value of |z| = OB = OC + CB = 5 + 2 = 7.
⎜ 2 z2 z3 − z ⎟ ⎜ z2 − z3 ⎟ Thus, 3 ≤ |z| ≤ 7.
⎜⎝ z + z 4⎟
⎠ ⎜⎝ ⎟⎠
2 3
The correct option is (A)
z2 z3 ( z2 − z3 )
=  [taking z4 = 0] 137. Since Re (z1 z ) ≤ | z1z2 |
2 z2 z3 ( z2 + z3 )( z2 − z3 ) 2

z2 + z3 \ |z1|2 + |z2|2 + 2 Re (z1 z ) ≤ |z1|2 + |z2|2 + 2 |z1 z |


2 2
1 ⇒ |z1 + z2|2 ≤ |z1|2 + |z2|2 + 2 |z1| |z2|(1)
= (a real number).
2 Also, Since A.M. ≥ G.M.
Hence, points z1, z2, z3 and origin are concyclic and 2
⎛ 1 ⎞
­therefore, z1, z2, z3 lie on a circle passing through the origin. ( c | z1 |) 2 + ⎜ |z | 1
⎝ c 2 ⎟⎠ ⎧ 1 ⎫2
The correct option is (B) \ ≥ ⎨c ⋅| z1 |2 ⋅ | z2 |2 ⎬
2 ⎩ c ⎭
135. Since |z – 25i| ≤ 15, therefore, distance between z and 25i is
less than or equal to 15. (∵ c > 0)
1
⇒ c |z1|2 + |z2|2 ≥ 2 |z1| |z2|
c
1
\ |z1|2 + |z1|2 + 2 |z1| |z2| ≤ |z1|2 + |z2|2 + c |z1|2 + |z2|2
c
⇒ |z1|2 + |z2|2 + 2 |z1| |z2| ≤ (1 + c) |z1|2
+ (1 + c–1) (|z2|2)(2)
From (1) and (2), we get
|z1 + z2|2 ≤ (1 + c) |z1|2 + (1 + c–1) |z2|2
\ k = 1 + c–1
The correct option is (C)
Thus, point z will lie in the interior and boundary of the 138. We have, 1 ≥ |z – (4 – 3i)|
circle whose centre is (0, 25) and radius is 15.
⎧| z | − | 4 − 3i | ⎧| z | − 5
Let OP be tangent to the circle at point P. ≥ ⎨ ⇒1≥ ⎨
Let ∠POX = q. Then, ∠OCP = q ⎩| 4 − 3i | − | z | ⎩5 − | z |
Now, OC = 25, CP = 15 ⇒ |z| ≤ 6 and |z| ≥ 4
\ OP = 20. ⇒ 4 ≤ |z| ≤ 6 ⇒ m = 4, n = 6
OP 20 4 x4 + x2 + 4 4
Now, tan q = = = Let y = = x3 + x +
CP 15 3 x x
⎛ 4⎞
\ Least positive value of arg z = q = tan–1 ⎜ ⎟ 1 1 1 1
⎝ 3⎠ = x3 + x + + + +
The correct option is (B) x x x x
136. Let z = x + iy 1 1 1 1
Since x ∈ (0, ∞), \ x3, x, , , , are all positive
Given, |z – 4 + 3i| ≤ 2. x x x x
numbers whose product is 1.
\ |x + iy – 4 + 3i| ≤ 2
\ their sum y will be least when
or, ( x − 4) 2 + ( y + 3) 2 ≤ 2 1
x3 = x = ⇒x=1
or, (x – 4)2 + (y + 3)2 ≤ 22 x
\ least value of y = 6
Thus, z lies in the interior or on the boundary of the circle
whose centre is (4, –3) and radius is 2. \ k = 6
Least value of |z| = OA = OC – AC = 5 – 2 = 3. Hence, k = n
The correct option is (B)

Objective_Maths_JEE Main 2017_Ch 3.indd 54 01/01/2008 03:29:26


Complex Numbers  3.55

139. We have, zn = (z + 1)n w −1


n
⇒ = (w2 – a1) (w2 – a2) (w2 – a3) (w2 – a4)(3)
⎛ z + 1⎞ w2 −1
⇒ ⎜ = 1 = cos 0 + i sin 0
⎝ z ⎟⎠ Dividing (2) by (3), we get
z +1 w −a w −a w −a w −a (w 2 − 1) 2
⇒ = (cos 2p r + i sin 2p r)1/n 2 1 ⋅ 2 2 ⋅ 2 3 ⋅ 2 4 =
z w − a1 w − a 2 w − a 3 w − a 4 (w − 1) 2
2p r 2p r
= cos + i sin
n n w 4 + 1 − 2w 2 w + 1 − 2w 2
= =
where, r = 0, 1, 2, …, n – 1. w 2 + 1 − 2w w 2 + 1 − 2w
1 2p r 2p r − w 2 − 2w 2 − 3w 2
⇒ 1 + = cos + i sin = = =w
z n n − w − 2w − 3w
1 2 rp pr pr The correct option is (B)
⇒ 1 + = 1 – 2 sin + i × 2 sin × cos
z n n n 142. Let z = x + iy
1 2 rp rp rp
⇒ = – 2 sin + 2i × sin cos We have, z + z = 2 |z – 1|
z n n n z+z
1 ⇒ = |z – 1|
⇒ z = 2
⎛ rp ⎞ ⎡ rp rp ⎤ ⇒ x = |x + iy – 1| ⇒ x = |(x – 1) + iy|
i ⎜ 2 sin ⎟ cos + i sin
⎝ n ⎠ ⎢⎣ n n ⎥⎦ ⇒ x2 = (x – 1)2 + y2 ⇒ 2x = 1 + y2.
1 ⎡ rp rp ⎤ If z1 = x1 + iy1 and z2 = x2 + iy2
= ⎢ cos − i sin
⎛ r p ⎞ ⎣ n n ⎥⎦ then, 2x1 = 1 + y12 (1)
i ⎜ 2 sin ⎟⎠
⎝ n
and, 2x2 = 1 + y22 (2)
i rp 1 1
⇒ x + iy = − cot − ⇒x=– Subtracting (2) from (1), we get
2 n 2 2
1 2 (x1 – x2) = y12 – y22
Hence, all the points lie on the line x = –
2 ⇒ 2 (x1 – x2) = (y1 + y2) (y1 – y2)(3)
The correct option is (B)
p
140. We have, z2 + z + 1 = 0 (1) But given arg (z1 – z2) =
4
⇒ (z – 1) (z2 + z + 1) = 0,
⎛ y − y ⎞ p y1 − y2
\ z3 = 1. i.e., tan– 1 ⎜ 1 2
= ⇒ =1
If n is not a multiple of 3, then we can write ⎝ x1 − x2 ⎟⎠ 4 x1 − x2
n = 3m + r, where m ∈ I and r = 1 or 2, \ y1 – y2 = x1 – x2,(4)
then 2n = 6m + 2r \ From (3) and (4) we get
If r = 1, then 2r = 2 y1 + y2 = 2
\ zn + z2n = (z3)m × zr + (z3)2n × z2r = zr + z2r \ Im (z1 + z2) = 2.
= z + z2 = –1 [Using (1)] The correct option is (C)
If r = 2, then 2r = 4 143. We have,
\ 2n = 3 (m + 1) + 1 a1 z3 + a2 z2 + a3 z + a4 = 3
\ zn + z2n = (z3)m × zr + (z3)m + 1 × z1 = z2 + z = –1 ⇒ |3| = |a1 z3 + a2 z2 + a3 z + a4|
Hence, zn + z2n = – 1 ⇒ 3 ≤ |a1 z3| + |a2 z2| + |a3 z| + |a4|
The correct option is (D) ⇒ 3 ≤ |a1| |z3| + |a2| |z2| + |a3| |z| + |a4|
141. Since 1, a1, a2, a3, a4 are the roots of the equation x5 – 1 = 0. ⇒ 3 ≤ |z|3 + |z|2 + |z| + 1  (Q |ai| ≤ 1)
\ (x5 – 1) = (x – 1) (x – a1) (x – a2) (x – a3) (x – a4) ⇒ 3 ≤ 1 + |z| + |z|2 + |z|3 < 1 + |z| + |z|2 + |z|3 + … ∞
x5 − 1
⇒ = (x – a1) (x – a2) (x – a3) (x – a4)(1) ⇒ 3 < 1 + |z| + |z|2 + |z|3 + … ∞
x −1
1 1
Putting x = w in (1), we get ⇒ 3 < ⇒ 1 – |z| <
1 −| z | 3
w5 − 1 2 2
= (w – a1) (w – a2) (w – a3) (w – a4) ⇒ – |z| < 0 ⇒ |z| >
w −1 3 3
w2 −1 The correct option is (C)
⇒ = (w – a1) (w – a2) (w – a3) (w – a4)(2)
w −1 1 1 1 1 2
and putting x = w2 in (1), we get 144. Since, + + + =
a+w b+w c+w d +w w
w10 − 1
2 = (w2 – a1) (w2 – a2) (w2 – a3) (w2 – a4) \ w is the root of the equation
w −1

Objective_Maths_JEE Main 2017_Ch 3.indd 55 01/01/2008 03:29:32


3.56  Chapter 3

1 1 1 1 2 \ From (1) and (2), we conclude


+ + + =
a+ x b+ x c+ x d + x x |A|2 + |B|2 + |C|2 = 3 [|z1|2 + |z2|2 + |z3|2].
4 3 2
⇒ 2x + (S a) x + 0 × x – (S abc) x – 2 abcd = 0 The correct option is (A)
Let a, b, g  be the other roots, then 1
146. z + = 2 (cos q + i sin q ) = 2eiq
Sa z
w + a + b + g = – (1)
2 \ z2 – 2eiq z + 1 = 0
Sab = 0
ab + aw + bw + g w + bg + g a = 0 (2) ⇒ z = eiq ± e 2iq − 1
S abc e2iq – 1 = cos 2q + i sin 2q – 1
Sabg = (3)
2 = 2 sinq (– sin q + i cos q)
abg w = –abcd(4) ⎡ ⎛p ⎞ ⎛p ⎞⎤
= 2 sin q ⎢cos ⎜ + q ⎟ + i sin ⎜ + q ⎟ ⎥
Since complex roots occurs in conjugate pairs ⎣ ⎝ 2 ⎠ ⎝ 2 ⎠⎦
\ g = w = w2.
\ Let a = eiq + e 2iq − 1
\ From (2),
ab + w (a + b) + w × w2 + w2 (a + b) = 0 ⎛p q⎞
= cos q + sin q cos ⎜ + ⎟
⎝ 4 2⎠
⇒ ab + (w + w2) (a + b) + w3 = 0
⇒ ab + (– 1) (a + b) + 1 = 0 ⎡ ⎛p q⎞⎤
+ i ⎢sin q + 2 sin q sin ⎜ + ⎟ ⎥
⇒ ab – a – b + 1 = 0 ⎣ ⎝ 4 2⎠ ⎦
⇒ a (b – 1) – (b – 1) = 0 ⇒ (a – 1) (b – 1) = 0 2
⎧ ⎛p q⎞⎫
\ either a = 1 or b = 1 \ |a – i|2 = ⎨cos q + 2 sin q cos ⎜ + ⎟ ⎬
⎩ ⎝ 4 2⎠ ⎭
Hence one root is unity 2
1 1 1 1 ⎧ ⎛p q⎞ ⎫
\ + + + =2 + ⎨sin q + 2 sin q sin ⎜ + ⎟ − 1⎬
a +1 b +1 c +1 d +1 ⎩ ⎝ 4 2⎠ ⎭
The correct option is (B) ⎛p q ⎞
= 2 + 2 2 sin q cos ⎜ + − q ⎟
⎝4 2 ⎠
145. We have, |A|2 + |B|2 + |C|2 = A A + B B + C C (1)
But  A A = (z1 + z2 + z3) ( z1 + z2 + z3 ) ⎛p q⎞
− 2 2 sin q sin ⎜ + ⎟ = 2
= z1z1 + z2 z2 + z3 z3 + z1 ( z2 + z3 ) ⎝ 4 2⎠
+ z2 ( z3 + z1 ) + z3 ( z1 + z2 ) Similarly, |b – i|2 = 2
= | z1 |2 + | z2 |2 + | z3 |2 + z1 ( z2 + z3 ) \ |a – i| = |b – i|
+ z2 ( z3 + z1 ) + z3 ( z1 + z2 ) The correct option is (D)
B B = (z1 + z2 w + z3 w2) ( z1 + z2w + z3w 2 ) 147. If z = x + iy is a complex number satisfying the given con-
ditions, then
= (z1 + z2 w + z3 w2) ( z1 + z2w 2 + z3w )
a2 – 3a + 2 = z + 2
[∵ w = w2 and (w 2 ) = w]
= z1z1 + z2 z2w 3 + z3 z3w 3 + z1 (z2 w + z3 w2) = z+i 2+ 2−i 2
4 2 2
+ z2 (z3 w + z1 w ) + z3 (z1 w + z2 w )
≤ z+i 2 + 2 |1 − i |
= | z1 |2 + | z2 |2 + | z3 |2 + z1 ( z2w + z3w 2 )
< a2 + 2
+ z2 ( z3w + z1w 2 ) + z3 ( z1w + z2w 2 ) (2)
⇒ –3a < 0 ⇒ a > 0
Similarly, C C = | z1 |2 + | z2 |2 + | z3 |2 + z1 ( z2w 2 + z3w )
+ z2 ( z3w 2 + z1w ) + z3 ( z1w 2 + z2w ) (3) Since z + 2 = a2 – 3a + 2 represents a circle with centre
Adding (1), (2) and (3), we get
2 2 2
at A − ( )
2 , 0 and radius a 2 − 3a + 2 and z + 2i
A A + B B + C C = 3 [|z1| + |z2| + |z3| ] 2
< a represents the interior of the circle with centre at
+ z1 [z2 (1 + w + w2) + z3 (1 + w2 + w)] (
B 0, 2 ) and radius a. Therefore, there will be a com-
+ z2 [z3 (1 + w + w2) + z1 (1 + w + w2)] plex number satisfying the given condition and the given
+ z3 [z1 (1 + w + w2) + z2 (1 + w2 + w)] inequality if the distance AB is less than the sum or differ-
ence of the radii of the two circles, i.e., if
= 3 [|z1|2 + |z2|2 + |z3|2][∵ 1 + w + w2 = 0]

Objective_Maths_JEE Main 2017_Ch 3.indd 56 01/01/2008 03:29:39


Complex Numbers  3.57

As given,
(− ) + (0 + 2 )
2 2
2 −0 < a 2 − 3a + 2 ± a 1 1 5
AC = BD ⇒ AM = DM ⇒ AM =
⇒ 2 ± a < a 2 − 3a + 2 2 2 2

⇒ 4 + a ± 4a < a2 – 3a + 2
2
⇒ AD = |z3 – z1| = DM 2 + AM 2
⇒ –a < – 2  or  7a < – 2
2
−2 ⎛ 5⎞
( ) 5
2
⇒ a > 2  or  a < = 5 +⎜ ⎟ =
7 ⎝ 2 ⎠ 2
But,  a > 0,
\ a > 2 Therefore, in D AMD,
The correct option is (A) 5 2 5/2 1
cos q = = and sin q = =
148. Let Sn = 1 + 2a + 3a2 + … + nan–1 5/2 5 5/2 5
\ a Sn = a + 2a2 + … + (n – 1)an–1 + nan z −z
Now, by rotation of complex numbers we know that 3 1
Subtracting, we get, z2 − z1
|z −z |
Sn (1 – a) = 1 + a + a2 + … + an–1 – nan = 3 1 eiq (anticlockwise rotation)
| z2 − z1 |
1 (1 − a n )
= − na n z3 − (1 + i ) 5/2
1−a ⇒ = (cos q + i sin q )
1−a nan
−n n 1 − 2i 5
\ Sn = − = .
(1 − a ) 2 1 − a 1−a z3 − (1 + i ) i
⇒ = 1 + (using values of cosq and sinq)
The correct option is (B) 1 − 2i 2
149. Let A represent z1 2+i i
⇒ z3 = (1 − 2i ) + (1 + i ) ⇒ z3 = 3 −
2 2
z1 z2
Similarly, taking clockwise rotation we get another possible
0 A B position of A as

OA · OB = 1, \ |z1 – 0| × |z – 0| = 1
Since z3 − z1 |z −z |
1 = 3 1 e − iq
⇒ |z1| = z2 − z1 | z2 − z1 |
|z|
⎛1 − i⎞ 3
⇒ z3 = ⎜ (1 − 2i ) + (1 + i ) ⇒ z3 = 1 − i
⎛ z − 0⎞
Also, arg ⎜ 1
⎛z ⎞
= 0 ⇒ arg ⎜ 1 ⎟ = 0 ⎝ 2 ⎟⎠ 2
⎝ z − 0 ⎟⎠ ⎝ z⎠ i 3
So, A represents the complex numbers 3 − or 1 − i
⇒ arg z1 = arg z 2 2
The correct option is (A)
If q is the argument of z, then
151. Let z = x + iy = r (cosq + i sin q), then the equation is
z = |z| eiq
1 iq 1 z 1 ⎛ 1 1 ⎞
\ z1 = e = 2 | z | eiq = = |(x – 2) + i (y – 1)| = r ⎜ cos q − sin q ⎟
|z| |z| zz z ⎝ 2 2 ⎠
1 1
\ A is = ( r cos q − r sin q )
z 2
The correct option is (A) 1
150. Let ABCD be the rhombus and M be the point of intersec- or, ( x − 2) 2 + ( y − 1) 2 = ( x − y)
2
tion of the diagonals AC and BD
which is the part of a parabola with focus (2, 1) and ­directrix
x – y = 0.
The correct option is (C)
152. |z1 + z2|2 + |z2 + z3|2 + |z3 + z1|2
= 2 (| z1 |2 + | z2 |2 + | z3 |2 ) + ( z1z2 + z1z2
+ z2 z3 + z2 z3 + z3 z1 + z1z3 )
= 24 + ( z1z2 + z1z2 + z2 z3 + z2 z3 + z3 z1 + z3 z1 ) (1)
Also,
| z1 + z2 + z3 |2 ≥ 0
Let point D be z1 = 1 + i and point M be z2 = 2 – i ⇒ z1z2 + z1z2 + z2 z3 + z2 z3 + z3 z1 + z3 z1 ≥ – 12
Also, let point A be z3 \ |z1 + z2|2 + |z2 + z3|2 + |z3 + z1|2 ≥ 12
Then, z2 – z1 = 1 – 2i and |z2 – z1| = 5 = MD The correct option is (B)

Objective_Maths_JEE Main 2017_Ch 3.indd 57 01/01/2008 03:29:47


3.58  Chapter 3

More than One Option Correct Type


m +1
153. Let A be the vertex with affix z1. There are two possibili-
2p ⇒ S1 = –i ∑ a k ⇒ S1 = –i(a + a2 + … + am+1)
ties and can be obtained by rotating z1 through either in k =1
n
clockwise or in anticlockwise direction.
±
2p
⇒ S1 =
(
−ia a m +1 − 1 ) = –i ⎧⎪⎨ a
− a ⎫⎪

m

z2 = z1e n   {Q |z2| = |z1|} a −1 ⎪⎩ a − 1 ⎭⎪


Since, a m = (cos 2p + i sin 2p) = 1 {using (1)}
O
a −1
⇒ S1 = i =i
a −1
4 n +1
Now,  S = ∑ S1
m =1
4 n +1
(
i i n+1 − 1 ) = i (i − 1) = i

C(z2)
A(z1)
B(z2) Thus,  S = ∑ im =
i −1 i −1
m =1
The correct option is (A) and (C) which is purely imaginary and independent of n. Also, i is
1 not a root of x4n + 1 + 1 = 0.
154. We have, i=0+i×1= (0 + 2i)
2 The correct option is (A) and (B)
1 1 157. We have,
= (1 + i2 + 2 × 1 × i) = (1 + i)2
2 2 |a + ib| = 1 ⇒ a + ib = cos q + i sin q
1
\ i =± (1 + i) |c + id| = 1 ⇒ c + id = cos f + i sin f
2
⇒ a = cos q, b = sin q, c = cos f, d = sin f
1
\ −i = ± (1 – i) Now, z1z2 = (cos q + i sin q) (cos f – i sin f)
2
1 ⇒ Re( z1z2 ) = cos q cos f + sin q sin f
Hence, i − −i = ± [(1 + i) – (1 – i)] = ± 2i
2 Thus, = 0 ⇒ cos (q – f) = 0 (1)
The correct option is (A) and (D) p p
⇒ q – f = or −
2 2
z1 + z3 z + z4
155. We have, z1 – z4 = z2 – z3 or = 2 Now, |w1|2 = a2 + c2 = cos2q + cos2f
2 2
i.e., the diagonals bisect each other. ⎛ p⎞
= cos 2 ⎜ f ± ⎟ + cos 2 f
⎝ 2⎠
\ It is a parallelogram.
z −z p = sin2f + cos2f = 1
Also, amp 4 1 =
z2 − z1 2 Thus, |w1| = 1. Similarly, |w2| = 1
⇒ angle at z1 is a right angle. Now, w1 w2 = (cos q + i sin q) ⋅ (cos f – i sin f)
\ It is a rectangle and hence a cyclic quadrilateral. ⇒ Re( w1 w2 ) = (cos q sin q + cos f sin f)
The correct option is (C) and (D) 1
= (sin 2q + sin 2f )
156. Let 2
⎡ m + 1 ⎧ ⎛ 2p k ⎞
m = sin (q + f) cos (q – f)
4 n +1
⎛ 2p k ⎞ ⎫ ⎤
S = ∑ ⎢ ∑ ⎨sin ⎜ ⎟⎠ − i cos ⎜⎝ ⎟ ⎬⎥ = 0 [From (1)]
m =1 ⎢
⎝ m m ⎠ ⎭ ⎥⎦
⎣ k =1 ⎩ m

The correct option is (A), (B) and (C)


⎛ 2p k ⎞ ⎛ 2p k ⎞ 158. We have,
Now, let tk = sin ⎜ − i cos ⎜
⎝ m ⎟⎠ ⎝ m ⎟⎠ 1
arg (z3/8) = arg ( z 2 + zz1/ 2 )
⎧ ⎛ 2p k ⎞ ⎛ 2p k ⎞ ⎫ 2
⇒ tk = –i ⎨cos ⎜ ⎟ + i sin ⎜⎝ ⎟⎬
⎩ ⎝ m ⎠ m ⎠⎭ ⇒ 2 arg (z3/8) = arg ( z 2 + z z1/ 2 )
⎛ 2p ⎞ ⎛ 2p ⎞ ⇒ arg ( z 3/ 4 ) − arg ( z 2 + z z1/ 2 ) = 0
Assuming, a = cos ⎜ ⎟ + i sin ⎜ ⎟ (1)
⎝ m⎠ ⎝ m⎠ [Q 2 arg (z) = arg (z2)]
⇒ tk = –ia k ⎛ z 2 + z z1/ 2 ⎞
m +1 ⇒ arg ⎜ ⎟ =0
z 3/ 4
Now, S1 = ∑ (t k ) ⎝ ⎠
k =1
⎛ z ⎞
⇒ Im ⎜ z 5 / 4 + 1/ 4 ⎟ = 0
⎝ z ⎠

Objective_Maths_JEE Main 2017_Ch 3.indd 58 01/01/2008 03:29:53


Complex Numbers  3.59

z ⎛ 5/ 4 z ⎞ \ a2 + b2 = 1 ⇒ b2 + b2 l2 = 1
⇒ z 5 / 4 + = ⎜ z + 1/ 4 ⎟ or, b2 (1 + l2) = 1 (3)
z1/ 4 ⎝ z ⎠
⎛ 1⎞
z z and,  c2 + d 2 = 1 ⇒ d 2 ⎜1 + 2 ⎟ = 1
⇒ z 5 / 4 + = ( z )5 / 4 + ⎝ l ⎠
z1/ 4 ( z )1/ 4
or, d2 (1 + l2) = l2(4)
z ( z )1/ 4 zz1/ 4
⇒ z 5 / 4 + = ( z )5 / 4 + b2 1
|z| 1/ 2
| z |1/ 2 \ 2
or d2 = b2 l2
=
d l2
d2
z 5 / 4 − ( z )5 / 4 Now, |w1| = a2 + c2 = b2 l2 + 2
⇒ z 5 / 4 − ( z )5 / 4 = l
| z |1/ 2 b2 l 2
= b2 l2 + = b 2
(1 + l2) = 1
⎛ 1 ⎞ l2
⇒ {z 5/ 4
− ( z )5 / 4 } ⎜1 − 1/ 2 ⎟ = 0
⎝ |z| ⎠ Also,  Re (w1 w 2 ) = Re {(a + ic) (b – id)}
\ z = z or |z| = 1. ⎛ d⎞
= (ab + cd) = b2 l + d ⎜ − ⎟
The correct option is (A), (B) and (D) ⎝ l⎠
159. We have, z12 + 2z22 + z32 = 2z2 (z1 + z3) b2 l 2
= b2 l – = 0.
⇒ (z12 – 2z1z2 + z22) + (z22 + z32 – 2z2z3) = 0 l
⇒ (z1 – z2)2 + (z3 – z2)2 = 0 Hence, |w1| = 1 and Re (w1 w 2 ) = 0
⇒ (z1 – z2) = ± i (z3 – z2) The correct option is (A) and (C)
⇒ (z1 – z2) = (z3 – z2) ei(p/2), (z3 – z2)e–i(p/2) 162. Let A, B, C be the vertices of the equilateral triangle
­represented by the complex numbers z1, z2, z3, respectively.
p
Then, AB = BC = AC and ∠A = ∠B = ∠C = .
p 3
z3 − z1 i z −z z −z
i.e.,  or  \ = e 3 = 1 2 = 2 3 (1)
z2 − z1 z3 − z2 z1 − z3
⇒ (z3 – z1) (z3 – z2) = – (z1 – z2)2
⇒ z12 + z22 + z32 = z1 z2 + z2 z3 + z3 z1 (2)
Thus, in either case the triangle is an isosceles right angled
⇒ [(z1 – z2)2 + (z2 – z3)2 + (z3 – z1)2] = 0 (3)
triangle, right angled at z2.
Now, 3 z0 = z1 + z2 + z3, [z0 is centroid]
The correct option is (A) and (B)
160. We have, \ z12 + z22 + z32 + 2 (z1 z2 + z2 z3 + z3 z1) = 9 z02
|z1 – z2|2 = (|z1| + |z2|)2 ⇒ z12 + z22 + z32 = 3 z02 (4)
⇒ |z1|2 + |z2|2 – 2 |z1| |z2| cos q From (1), we also have
(z3 – z1) (z3 – z2) + (z1 – z2) (z1 – z2) (z1 – z3 + z3 – z2) = 0
= |z1|2 + |z2|2 + 2 |z1| |z2|
⇒ (z2 – z1) (z3 – z1) + (z1 – z2) (z3 – z2)
⇒ cos q = –1 where q = arg (z1/z2)
+ (z1 – z2) (z1 – z3) = 0
⇒ arg (z1/z2) = (2n + 1) p, n ∈ I
Dividing by (z1 – z2) (z2 – z3) (z3 – z1), we get
⇒ z1/z2 lies on the negative real axis
1 1 1
Hence, we can write z1 = l z2, l ∈ R– + + =0
z2 − z1 z1 − z3 z3 − z2
Also, we have,
Re (z1/z2) ≤ 0 The correct option is (A), (B), (C) and (D)
z z 163. Since a, b, c, … k are the roots of the given equation, we
⇒ 1 + 1 ≤ 0 or z1z2 + z2 z1 ≤ 0 have the identity
z2 z2
The correct option is (B) and (C) xn + p1 xn – 1 + p2 xn – 2 + … + pn – 1 x + pn
161. Let z1 = a + ib, z2 = c + id. ≡ (x – a) (x – b) (x – c) … (x – k)(1)
Then, |z1| = |z2| = 1 ⇒ a2 + b2 = 1 and c2 + d2 = 1 (1) In the identity (1), put x = i
Also, Re (z1 z2 ) = 0 ⇒ Re {(a + ib) (c – id)} = 0 Then, in + p1 in – 1 + p2 in – 2 + … + pn – 1 i + pn
or, ac – bd = 0 (2) ≡ (i – a) (i – b) (i – c) … (i – k)
or, ac = – bd or, in [1 + p1 i– 1 + p2 i– 2 + p3 i– 3 + p4 i– 4 + …
a a + pn – 1 i– (n – 1) + pn i– n]
or = = l (say)
b −c ≡ (i – a) (i – b) (i – c) … (i – k).

Objective_Maths_JEE Main 2017_Ch 3.indd 59 01/01/2008 03:29:59


3.60  Chapter 3

1 1 z2 − z3 BC i p / 4 1 ip /4
But,  i – 1 = = – i, i– 2 = 2 = –1 = e = e (2)
i i z1 − z3 AC 2
1 1
i –3 = 3 = i, i –4 = 4 = 1 etc.
i i
\ The above identity may be written as
in [(1 – p2 + p4 – …) – i (p1 – p3 + p5 – …)]
≡ (– 1)n (a – i) (b – i) (c – i) … (k – i)(2)
Similarly, putting x = –i in (1), we shall obtain

(–i)n [(1 – p2 + p4 + …) + i (p1 – p3 + p5 …)]
≡ (–1)n (a + i) (b + i) (c + i) … (k + i)(3) From equations (1) and (2), we get
Multiplying (2) and (3), we get ⎛ z3 − z1 ⎞
⎜⎝ z − z ⎟⎠ 2 ei p / 4
(–1)n × i2n [(1 – p2 + p4 …)2 – i2 (p1 – p3 + p5 …)2] 2 1 = =2
⎛ z2 − z3 ⎞ 1 ip /4
= (–1)2n (a2 – i2) (b2 – i2) (c2 – i2) … (k2 – i2) e
⎜⎝ z − z ⎟⎠ 2
∵ (–1)n i2n = (–1)n (–1)n = (–1)2n = 1, this gives 1 3

(1 – p2 + p4 …)2 + (p1 – p3 + p5 …)2 z −z z −z


⇒ 3 1 · 1 3 = 2
z2 − z1 z2 − z3
= (a2 + 1) (b2 + 1) (c2 + 1) … (k2 + 1)
⇒ –(z3 – z1)2 = 2 (z22 – z2z3 – z1z2 + z1 z3)
The correct option is (A) and (B)
164. Since the diagonals of the square bisect each other, ⇒ z12 + z32 – 2 z1z3 = – 2 z22 + 2 z2z3 + 2 z1z2 – 2 z1z3
z +z ⇒ z12 + z32 + 2 z22 = 2 z2(z1 + z3)
\ mid-point of PR = 1 3 (1)
2
z2 + z4 or, z12 + z32 = 2z2 (z1 + z3 – z2)
mid-point of QS = (2)
2 or, z12 + z22 + z22 + z32 – 2z1z2 – 2z2z3 = 0
From (1) and (2), we get z1 + z3 = z2 + z4 or, (z1 – z2)2 + (z2 – z3)2 = 0
⇒ (a) is correct 2
⎛ z − z2 ⎞
or, ⎜ 1 = – 1 = cos p + i sin p
⎝ z2 − z3 ⎟⎠
z1 − z2 p p
or, = cos + i sin = i (imaginary)
z2 − z3 2 2
The correct option is (A), (B), (C) and (D)
p
166. We have, ∠ DAC = − C and OC = OD
Since all the sides of a square are equal 2
z
\ = cos (p – 2C) + i sin (p – 2C)
⇒ PQ = QR = RS = SP z3
⇒ |z1 – z2| = |z2 – z3| = |z3 – z4| = |z4 – z1| z
or, = – cos 2C + i sin 2C(1)
⇒ (b) is correct z3
We know that the diagonals of a square are equal Again, ∠ AOB = 2C and OA = OB
\ PR = QS z1
\ = cos 2C + i sin 2C(2)
⇒ |z1 – z3| = |z2 – z4| ⇒ (c) is correct z2
Since the diagonals are perpendicular to each other Multiply (1) and (2), we get
zz
z1 − z3 z −z 1 = – 1
\ = 1 3 (cos a + i sin a) where a is 90º z2 z3
z2 − z4 z2 − z4
z − z3
This is purely imaginary. Therefore, real part of 1 is
zero. z 2 − z4

⇒ (d) is correct
The correct option is (A), (B), (C) and (D)
165. Given AB = BC (isosceles triangle), ∠B = 90º
p
\ ∠C = ∠A =
4
z3 − z1 AC i p / 4
\ = e = 2 ei p / 4 (1)
z2 − z1 AB

Objective_Maths_JEE Main 2017_Ch 3.indd 60 01/01/2008 03:30:04


Complex Numbers  3.61

− z2 z3 − z1 z2 z2 z3 − z1 z3 168. The roots of x2 + x + 1 = 0 are w and w2


⇒ z = = =
z1 z1 z1 z2 z2 So, h(w) = 0 and h(w)2 = 0
 (∵ z1 z1 = z2 z2 ) ⇒ w f (1) + w2g(1) = 0 and w2 f (1) + wg(1) = 0
− z1 z2 ⇒ f (1) = g(1) = 0
= . \ h(1) = f (1) + g(1) = 0.
z3
The correct option is (B), (C) and (D) The correct option is (A), (B) and (C)
167. We have, 2p 2p
169. We have, a = cos
+ i sin
|z1 + z2| = |z1 – z2| 5 5
⇒ ( z1 + z2 ) ( z1 + z2 ) and, 1 + a + a2 + a3 + a4 = 0
= ( z1 − z2 ) ( z1 − z2 ) \ |1 + a + a2 + a3| = |–a4| = |a |4 = 1
⇒ z1z2 + z2 z1 = 0 (1) Also, |1 + a + a2| = |–a3 (1 + a)| = |1 + a|(1)

z1 ⎛z ⎞ 2p 2p
⇒ = − ⎜ 1⎟ = 1 + cos + i sin
5 5
z2 ⎝ z2 ⎠
z1 p⎛ p p⎞
⇒ is purely imaginary = 2 cos ⎜ cos + i sin ⎟⎠
z2 5⎝ 5 5
Also, from (1), p
= 2 cos
2 2 2 5
|z1 – z2| = |z1| + |z2|
p
⇒ DOAB is a right angled triangle, right angled at O. Again, from (i), |1 + a| = |1 + a + a2| = 2 cos .
5
z + z2 The correct option is (A), (B) and (C)
So, orthocentre lies at O and circumcentre = 1 .
2
The correct option is (A), (B) and (C)

Passage Based Questions


170. It is obvious that we must consider the 17th root of unity. Now, by De Moivre’s theorem, one of the values of
Consider the equation z17 – 1 = 0. The roots of this equation are ⎛ mq mq ⎞
(a + ib)m/n is rm/n ⎜ cos + i sin ⎟
2 rp 2 rp ⎝ n n ⎠
z = cos + i sin , r = 0, 1, …, 16 (i)
17 17 and one of the values of
Since the sum of the roots of (i) is zero, therefore,
⎛ mq mq ⎞
16 (a – ib)m/n is rm/n ⎜ cos − i sin ⎟
⎛ 2rp 2rp ⎞ ⎝ n n ⎠
∑ ⎜⎝ cos + i sin ⎟ =0
r=0 17 17 ⎠ Hence, one of the values of (a + ib)m/n + (a – ib)m/n is
Equating real and imaginary parts on both sides, we have mq
= 2rm/n cos
16
2r p 16
2r p n
∑ cos = 0, ∑ sin =0
17 17 ⎛m b⎞
r=0 r=0 = 2 (a2 + b2)m/2n cos ⎜ tan −1 ⎟ , using (ii).
16
⎝n a⎠
2r p
\ ∑ cos 17
= – cos 0 = – 1, The correct option is (B)
r =1
172. We have, (16)1/4 = (24)1/4 = 2 (1)1/4
16
2r p = 2 (cos 0 + i sin 0)1/4
∑ sin = – sin 0 = 0
r =1 17 ⎧ 1 1 ⎫
= 2 ⎨cos ( 2kp + 0) + i sin ( 2kp + 0)⎬ , k = 0, 1, 2, 3
Hence, the desired sum = 0 + i (– 1) = –i ⎩ 4 4 ⎭
The correct option is (D) = 2 × 1, 2 × i, 2 × –1, 2 × –i = ±2, ±2i.
171. Let a + ib = r (cos q + i sin q)(i) The correct option is (A)
Then, r cos q = a and r sin q = b 173. We have, z4 + 1 = 0 ⇒ z4 = – 1
b b ⇒ z = (cos p + i sin p)1/4
⇒ r2 = a2 + b2, tan q = or q = tan– 1 .(ii)
a a 1 1
Taking conjugates of both sides of (i), we get ⇒ z = cos (2kp + p) + i sin (2kp + p),
4 4
a – ib = r (cos q – i sin q).  k = 0, 1, 2, 3.

Objective_Maths_JEE Main 2017_Ch 3.indd 61 01/01/2008 03:30:09


3.62  Chapter 3

p p 3p 3p 1 77. Let an = 1
⇒ z = cos + i sin , cos + i sin ,
4 4 4 4 (an – 1) = (a – 1) (a – w) (a – w2) … (a – wn – 1)
5p 5p 7p 7p an − 1
cos + i sin , cos + i sin ⇒ (a – w) (a – w2) … (a – wn – 1) =
4 4 4 4 a −1
1 1 1 1 an − 1
= (1 + i), (– 1 + i), (– 1 – i), (1 – i). 2 n–1
⇒ lim (a – w) (a – w ) … (a – w ) = lim
2 2 2 2 a→1 a→1 a − 1
Hence, the four roots of z4 + 1 = 0 are (±1 ± i). ⇒ (1 – w) (1 – w2) … (1 – wn – 1) = n
The correct option is (C) The correct option is (C)

174. Since 1, w, w2, … wn – 1 are the n, nth roots of unity 178. Solving x6 + x3 + 1 = 0 as a quadratic in x3, we get
n −1 n −1
−1± 1− 4 −1 i 3
\ ∑ wk = 0 and ∑ (w )k =0 x3 = = ± = r (cos q ± i sin q)
k =0 k =0 2 2 2
n −1 n −1
−1 3
Now, ∑ | z1 + w k z2 |2 = ∑ ( z1 + w k z2 )( z1 + (w )k z2 ) \ r cos q =
2
, r sin q =
2
k =0 k =0
n −1 ⇒ r2 = 1 and tan q = − 3
= ∑ z1z1 + z1z2 (w )k + z1z2 w k + z2 z2 (w k )(w )k 2p 2p 2p
k =0 ⇒ r = 1 and q = ⇒ x3 = cos ± i sin
n −1 n −1 n −1 n −1
3 3 3
= ∑ | z1 |2 + ∑ z1z2 (w )k + ∑ z1z2w k + ∑ | z2 |2 ⎡ ⎛ 2p ⎞ ⎛ 2p ⎞ ⎤
1/3
\ x = ⎢cos ⎜ 2kp + ⎟⎠ ± i sin ⎜⎝ 2kp + ⎟
3 ⎠ ⎥⎦
k =0 k =0 k =0 k =0
⎣ ⎝ 3
= n |z1|2 + 0 + 0 + n |z2|2 = n (|z1|2 + |z2|2)
p p
The correct option is (A) = cos(6 k + 2) ± i sin (6 k + 2) , k = 0, 1, 2
9 9
n
175. Let = 1 = x; ⎛ pp ⎞ ⎛ pp ⎞
Hence, cos ⎜ ± i sin ⎜ , p = 2, 8, 14 are the required
\ x = 1; n
⎝ 9 ⎟⎠ ⎝ 9 ⎟⎠
roots.
\ xn – 1 = 0
The correct option is (A), (B) and (C)
\ xn – 1 = (x – 1) (x – a1) (x – a2) … (x – an – 1)
\ (x – a1) (x – a2) (x – a3) … (x – an – 1) 179. Solving z4 + 4z2 + 16 = 0 as a quadratic in z2, z2 =
xn − 1 1 − xn −4± 16 − 64
= = = 1 + x + x2 + … + xn – 1 = − 2 ± 2 3i
x −1 1− x 2
Putting x = 1, we get Let z2 = − 2 ± 2 3 i = r (cos q ± i sin q)(1)
(1 – a1) (1 – a2) (1 – a3) … (1 – an – 1) = n \ r cos q = – 2 and r sin q = 2 3
The correct option is (B) 2p
2 n–1
⇒ r2 = 16 and tan q = − 3 ⇒ r = 4, q = (2)
176. Since 1, a, a , …, a are the n, nth roots of unity, 3
1/ 2
\ xn – 1 = (x – 1) (x – a) (x – a 2) … (x – a n – 1) ⎛ 2p 2p ⎞
z = 2 ⎜ cos ± i sin ⎟ , by (1) and (2)
⇒ log (xn – 1) = log (x – 1) + log (x – a) ⎝ 3 3⎠
+ log (x – a 2) + … + log (x – a n – 1) ⎡ ⎛ 2p ⎞ 1 ⎛ 2p ⎞ 1 ⎤
= 2 ⎢cos ⎜ 2kp + ⎟⎠ ⋅ ± i sin ⎜⎝ 2kp + ⎟⋅
Differentiating both sides with respect to ‘x’, we get ⎣ ⎝ 3 2 3 ⎠ 2 ⎥⎦
x n −1 1 1 1 1  k = 0, 1
= + + + ... +
n
x −1 x −1 x − a x − a2 x − a n −1 ⎛ p p⎞ ⎛ 4p 4p ⎞
= 2 ⎜ cos ± i sin ⎟ , 2 ⎜ cos ± i sin ⎟
⎝ 3 3⎠ ⎝ 3 3⎠
Putting x = 2, we get
⎛ 1 i 3⎞ ⎛ 1 i 3⎞
n 2n −1 1 1 1 1 = 2 ⎜ ±
n = + + + ... + ⎟,−2⎜ ± ⎟
2 −1 1 2 − a 2−a 2
2 − a n −1 ⎝2 2 ⎠ ⎝2 2 ⎠

n ⋅ 2n −1 n −1
1 Hence, ± 1 ± i 3 are the required roots.
\
2 −1n
–1= ∑ 2−a i The correct option is (A) and (B)
i =1
2+ z
n −1
1 n ⋅ 2n −1 − 2n + 1 ( n − 2) 2n −1 + 1 180. Let = w(i)
Hence, 
2 − ai
= ∑ 2n − 1
=
2n − 1
2−z
i =1 Then,  w6 = – 1 = cis (2p + 1)p, where p is an integer.
The correct option is (A)
\ w = cis [(2p + 1) p/6], where p = 0, 1, … 5

Objective_Maths_JEE Main 2017_Ch 3.indd 62 01/01/2008 03:30:18


Complex Numbers  3.63

From (i), we have, tan p/12, ±2i tan 5p/12, ±2i.


2 ( w − 1) The correct option is (A), (B) and (C)
2 + z = w (2 – z) or z =
w +1 181. Multiplying the given equation by (z + 1) and simplifying,
2 [cos{( 2 p + 1) p / 6} + i sin{( 2 p + 1) p / 6} − 1] we obtain, z5 + 1 = 0, whose roots are z = cos (pp /5) + i sin
=
cos{( 2 p + 1) p / 6} + i sin{( 2 p + 1) p / 6} + 1 (pp /5), p = 1, 3, 5, 7, 9.
2 [2i sin{( 2 p + 1) p / 12} cos{( 2 p + 1) p / 12} For p = 5, the root z = –1 corresponds to z + 1 = 0
Hence, the required roots are
− 2 sin 2 {( 2 p + 1) p / 12}]
= cos (pp /5) + i sin (pp /5); p = 1, 3, 7, 9.
2i sin {( 2 p + 1) p / 12} cos{( 2 p + 1) p / 12}
The correct option is (B)
+ 2 cos 2 {( 2 p + 1) p / 12}
= 2i tan [(2p + 1) p/12], p = 0, 1, …, 5
On giving the values 0, 1, …, 5 the roots are seen to be ± 2i

Match the Column Type


⎛p⎞ ⎛p⎞
182.   I. Since zr = cos ⎜ r ⎟ + i sin ⎜ r ⎟ , \  x2 + y2 – 4 = 0
⎝3 ⎠ ⎝3 ⎠
r = 1, 2, 3, … ⇒  x2 + y2 = 4 ⇒ |z|2 = 4 ⇒ |z| = 2
we have, z1 · z2 · z3 …∞ The correct option is (C)
⎛ p p⎞⎛ p p⎞ 13 13 ⎧⎪ i (1 − i13 ) ⎫⎪
= ⎜ cos + i sin ⎟ ⎜ cos 2 + i sin 2 ⎟
⎝ 3 3 ⎠ ⎝ 3 3 ⎠ IV. ∑ (i n + i n +1 ) = ∑ i n (1 + i) = (1 + i) ⎨ ⎬
n =1 n =1 ⎪⎩ 1 − i ⎪⎭
⎛ p p⎞
⎜⎝ cos 3 + i sin 3 ⎠⎟ … ∞ ⎧ i (1 − i ) ⎫
3 3 = (1 + i) ⎨ ⎬
⎩ 1− i ⎭
⎛p p p ⎞ ⎛p p p ⎞ = (1 + i) i = – 1 + i
= cos ⎜ + 2 + 3 + ...⎟ + i sin ⎜ + 2 + 3 + ...⎟
⎝3 3 3 ⎠ ⎝3 3 3 ⎠ The correct option is (A)
⎛ p ⎞ ⎛ p ⎞ 183.    I. We have, (1 + w) (1 + w2) (1 + w4) (1 + w8) … to 2n
⎜ 3 ⎟ ⎜ ⎟ factors
= cos ⎜ + i sin ⎜ 3 ⎟
1⎟ 1 = (1 + w) (1 + w2) (1 + w3 × w) (1 + w6 × w2)
⎜1− ⎟ ⎜1− ⎟
⎝ 3⎠ ⎝ 3⎠  … to 2n factors
p p = (1 + w) (1 + w2) (1 + w) (1 + w2) … to 2n factors
= cos + i sin =0+i×1=i
2 2  [∵ w3 = w6 = … = 1]
The correct option is (D)
= [(1 + w) (1 + w) … to n factors]
 II. Given, i z3 + z2 – z + i = 0  [(1 + w2) (1 + w2) … to n factors]
⇒  i z2 (z – i) – (z – i) = 0 = (1 + w) (1 + w ) = [(1 + w) (1 + w2)]n
n 2 n
1
⇒ (z – i) (i z2 – 1) = 0 ⇒ z = i or z2 = = – i. = (1 + w + w2 + w3)n = (0 + 1)n = 1
i
Now, z = – i ⇒ |z| = |i| = 1  [∵ 1 + w + w2 = 0, w3 = 1]
and, z2 = – i ⇒ |z2| = |–i| ⇒ |z|2 = 1 ⇒ |z| = 1 The correct option is (B)
Thus, in both cases |z| = 1    II. We have,
The correct option is (B) (1 – w + w2) (1 – w2 + w4) (1 – w4 + w8)
III. Let z = x + iy (1 – w8 + w16)… to 2n factors
z−2 x + iy − 2 ( x − 2) + iy = (1 – w + w2) (1 – w2 + w) (1 – w + w2)
Then,  = =
z+2 x + iy + 2 ( x + 2) + iy (1 – w2 + w) … to 2n factors.

=
[( x − 2) + iy ][( x + 2) − iy ]  [∵ w4 = w, w8 = w2, w16 = w and so on]
2 2
( x + 2) + y = (– 2w) (– 2w2) (– 2w) (– 2w2) … to 2n factors
( x 2 + y 2 − 4) + i ( 4 y ) = (22 w3) (22 w3) … to n factors
= 2 2
( x + 2) + y [∵ (– 2w) (– 2w2) = 22 w3 = 22]
z−2 = (22)n = 22n
Since is purely imaginary,
z+2 The correct option is (C)

Objective_Maths_JEE Main 2017_Ch 3.indd 63 01/01/2008 03:30:22


3.64  Chapter 3

III. Since ( 3 + i)100 = 299 (a + ib) z −1


 II. Let = iy, where y is real
\ ( 3 – i) 100 99
= 2 (a – ib) z +1
z +1 1
\ ( 3 + i) 100
– ( 3 – i)100 = 299 (2ib) = 2100 (ib) ⇒  =
z −1 iy
100 100 100 100 100
⇒  i [1 – 3 i] –i [– 1 + 3 i] =2 (ib)
2z 1 + iy
or,  (– 2w) 100
– (2w ) 2 100
=2 100
(ib) ⇒  =   [by componendo and dividendo]
2 1 − iy
or, w – w2 = (ib) or 3 i = ib
1 + iy 1 + y2
\  b = 3 ⇒  z = ⇒ |z| = =1
1 − iy 1 + y2
The correct option is (A)
The correct option is (D)
IV. The given expression
n −1 + 3 i 2 2w
III. We have, i + 3 = ⋅ =
= ∑ ( n − 1) ( n − w ) ( n − w 2 ) 2 i i
2
−1 − 3 i 2 2w 2
n and, i– 3 = ⋅ =
= ∑ ( n − 1) ( n − w ) ( n − w 2
) 2 i i
1
[Q (1 – 1) (1 – w) (1 – w2) = 0] \ (i + 3 )100 + (i – 3 )100 + 2100
100
⎛ 2w 2 ⎞ 100
n
n2 ( n + 1) 2 ⎛ 2w ⎞
= ⎜ +⎜ + 2100
= ∑ ( n3 − 1) =
4
−n
⎝ i ⎟⎠ ⎝ i ⎠

1
The correct option is (D) 2100
= (w100 + w200) + 2100
184.   I. We have, i100
3+i i 3 + i2 ⎛ −1 + 3 i ⎞ = 2100 (w + w2) + 2100
= =–i ⎜ ⎟ = – iw = –2100 + 2100 = 0
2 2i ⎝ 2 ⎠
The correct option is (B)
i− 3 i2 − i 3 ⎛ −1 − 3 i ⎞ 2
and,  = =–i ⎜ ⎟ = – iw IV. Let zk = xk + iyk, we have (zk + 1)7 + zk7 = 0
2 2i ⎝ 2 ⎠
⇒ (zk + 1)7 = – zk7 ⇒ |zk + 1|7 = |zk|7
⎡ 1 ⎤
 ⎢∵ i = − i ⎥ ⇒ |zk + 1| = |zk| ⇒ |xk + iyk + 1|2 = |xk + iyk|2
⎣ ⎦
⎛ 3 + i⎞ ⎛ i − 3⎞
6 6
⇒ (xk + 1)2 + yk2 = xk2 + yk2
6 2 6
Hence, ⎜ 2 ⎟ + ⎜ 2 ⎟ = (– iw) + (– iw ) 1
⎝ ⎠ ⎝ ⎠ ⇒ 2xk + 1 = 0 or xk = –
6 6 12 2
= i (w + w ) 6 6
7
= –1 (1 + 1) = –2. Thus, ∑ Re ( zk ) = ∑ xk =–
2
The correct option is (C) k =0 k =0
The correct option is (A)

Assertion-Reasoning Type
185. We have, Comparing real and imaginary parts, we get
1 1 cos (a – b ) + cos (b – g ) + cos (g – a) = –1
= cos a – i sin a, = cos b – i sin b
a b The correct option is (A)
a
Now,  = (cos a + i sina) (cos b – i sinb ) z −1 p
b 186. We have, arg =
a z +1 3
or, = cos (a – b ) + i sin (a – b ).
b x + iy − 1 p
b ⇒ arg = [Putting z = x + iy]
Similarly, = cos (b – g ) + i sin (b – g ) x + iy + 1 3
c
c y y p
and,  = cos (g – a) + i sin (g – a). ⇒ tan −1 − tan −1 =
a x −1 x +1 3
a b c
Putting these values in + + = – 1, we get
b c a ⎛ z1 ⎞
[cos (a – b ) + cos (b – g ) + cos (g – a)]
 ⎜⎝∵ Arg z = Arg z1 − Arg z2 ⎟⎠
2
+ i [sin (a – b ) + sin (b – g ) + sin (g – a)]
= –1 = –1 + 0 i.

Objective_Maths_JEE Main 2017_Ch 3.indd 64 01/01/2008 03:30:30


Complex Numbers  3.65

y y Hence, the greatest value of |z| is 5 + 1.



⇒ tan x − 1 x +1 = p
−1 The correct option is (A)
y2 3
190. |z – i| = |z + 5i| represents equation of perpendicular bisector
1+ 2
x −1 of points (0, 1) and (0, –5), i.e., y = –2, now |z| = 2 is x2 +
2y p y2 = 4
⇒ 2 = tan = 3
x + y2 − 1 3 ⇒ x2 + 4 = 4 ⇒ x = 0
2 z represents a single point (0, –2).
⇒ x2 + y2 – y – 1 = 0, which is a circle.
3 The correct option is (A)
The correct option is (D) 191. We have,
(1 – z0) Pn(z) = (1 – z02) (1 + z02) (1 + z022) … (1 + z02n)
187. Area of the triangle on the argand plane formed by the com-
= (1 – z022) (1 + z022) (1 + z023) … (1 + z02n)
3 2
plex numbers –z, iz, z – iz is |z| .
2 = (1 – z023) (1 + z023) … (1 + z02n)
3 2
\ |z| = 600 ⇒ |z| = 20. = (1 – z02n+1)(22m · 22m = 22m+1)
2
2n
The correct option is (A)
n

i 2 ⎛ i⎞ i2
Now, z0 = ⇒ z02n+1 = (z02)2n = ⎜ ⎟ =
188. Given, |z – 1| + |z + 3| ≤ 8 2 ⎝ 2⎠ 22
n

\ z lies inside or on the ellipse whose foci are (1, 0) and n 2n


Now, since 2 is divisible by 4, if n > 1 ⇒ i = 1
(–3, 0) and vertices are (–5, 0) and (3, 0).
1 1 ⎛ 1 ⎞
Y Thus, 1 – z02n+1 = 1 − ⇒ Pn(z) = ⎜⎝1 − 2 ⎟⎠
1 − z0
n n

22 2
⎛ 1 ⎞
= (1 + i ) ⎜1 − 2 ⎟ .
⎝ 2 ⎠
n

(4, 0)
X The correct option is (A)
(–5, 0) (–3, 0) O (1, 0) (3, 0)
192. Since amp ⋅ [z1 (z3 – z2)] = amp ⋅ [z3 (z2 – z1)]
⎛ z ( z − z2 ) ⎞
\ amp. ⎜ 1 3 =0
⎝ z3 ( z2 − z1 ) ⎟⎠
Now, |z – 4| is distance of z from (4, 0). Minimum distance
is 1 and maximum is 9. z1( z3 − z2 )
⇒ is purely real.
The correct option is (A) z3 ( z2 − z1 )
4 4 Hence, 0, z1, z2, z3 are concyclic
189. We have, 2 = z − ≥ |z| –
z z ⎛ ( z1 − z3 ) ( z2 − z4 )
4 ⎜⎝∵for four concyclic points ( z − z )( z − z )
1 4 2 3
⇒ |z| – ≤2
z ⎞
is purely real⎟
⇒ |z|2 – 2 |z| – 4 ≤ 0 or (|z| – 1)2 – 5 ≤ 0 ⎠
⇒ (|z| – 1)2 ≤ 5 or |z| – 1 ≤ 5 ⇒ |z| ≤ 5 +1 The correct option is (A)

Previous Year’s Questions


193. Key Idea : If w is a cube root of unity, then 1 + ω + ω2 = 0 1 94. Given z12 + z22 − z1z2 = 0
and ω3 = 1 ⇒ (z1 + z2)2 – 3z1z2 = 0
(l + ω + w2) = 0 and ω3 = 1 ⇒ a2 = 3b.
(1 + ω – ω2)7 = (−ω2 − ω2)7 (∵1 + ω + ω2 = 0) The correct option is (C)
= (− 2ω2)7
⎛ z⎞ p
= –27⋅ω14 195. arg( z ) − arg(w ) = arg ⎜ ⎟ =
⎝w⎠ 2
= –128(ω3)4ω2
⇒ | zw | = 1
= − 128 ω2(∵ ω3= 1)
⇒ z w = −i or + i .
The correct option is (D)
The correct option is (D) or (C)

Objective_Maths_JEE Main 2017_Ch 3.indd 65 01/01/2008 03:30:34


10
3.66  Chapter 3 ⎛ 2kp 2kp ⎞
∑ ⎜⎝ sin 11
+ i cos ⎟
11 ⎠
k =1
10 10
2kp 2kp
= ∑ sin + i ∑ cos
2
1 + i (1 + i )
196. = =i 11 11
1− i 2 k =1 k =1

⎛1+ i⎞
x
= 0 + i (−1) = −i
⇒⎜ = ix
⎝ 1 − i ⎟⎠ The correct option is (D)
204. The given equation z2 + z + 1 = 0
⇒ x = 4n .
The correct option is (A) ⇒ z = ω or ω2.
1 1
Z ⎛ z⎞ So, z+ = ω + ω2 = −1, z2 + 2
197. Here w = ⇒ arg ⎜ z ⎟ = p z z
i ⎝ i⎠ 1
= ω + ω = −1, z + 3 = ω3 + ω3 = 2,
2 3

⇒ 2arg( z ) − arg(i ) = p z
1 1 1
p z4 + 4 = –1, z5 + 5 = –1 and z6 + 6 = 2.
⇒ 2 arg( z ) − =p z z z
2
∴ The given sum = 1 + 1 + 4 + 1 + 1 + 4 = 12
3p
⇒ arg( z ) = . The correct option is (D)
4
205. From the Argand diagram, maximum value of | z + 1| is 6.
The correct option is (C)
Alternative:
198. z = (p + iq)3 = p(p2− 3q2)− iq(q2− 3p2)
x y | z + 1| = | z + 4 − 3| ≤ | z + 4| + |−3| = 6.
+
x y p q
⇒ = p 2 − 3q 2 and = q 2 − 3 p 2 ⇒ 2 = −2 .
p q ( p + q2 )
The correct option is (B)

( )
2
199. Since | z 2 − 1 |2 = | z |2 +1 , we have


( z − 1)( z
2 2
)
− 1 = | z |4 + 2 | z |2 +1
(–7, 0) (–4, 0) (–1, 0)

⇒ z 2 + z 2 + 2 zz = 0 ⇒ z + z = 0
⇒ R (z) = 0 ⇒ z lies on the imaginary axis.
The correct option is (D)
200. Given equation (x − 1)3 + 8 = 0 implies that (x − 1) The correct option is (C)
= (−2) (1)1/3 206. Put −i in place of i
−1
⇒ x − 1 = −2 or −2ω or −2ω2 Hence, the complex conjugate is
i +1
Or n = −1 or 1 − 2ω or 1 − 2ω2. The correct option is (C)
The correct option is (C)
⎛ 4⎞ 4 4 4
201. | z1 + z2| = | z1 | + | z2| ⇒ z1 and z2 are collinear and are to the 207. One can write Z = ⎜ Z − ⎟ + ⇒ Z = Z − +
⎝ Z⎠ Z Z Z
same side of origin; hence argz1− argz2 = 0.
The correct option is (C) 4 4 4
⇒ Z ≤ Z− + ⇒ Z ≤ 2+
Z Z Z
z z
202. As given w = ⇒| w |= = 1 ⇒ distance of z 2
1 1 ⇒ Z −2 Z −4≤0
z− i z− i

⎛ 1⎞
3 3
( (
∴ Z − 5 +1 Z − 1− 5 ≤ 0 ))( ( ))
from origin and point ⎜ 0, ⎟ is same.
⎝ 3⎠ ⇒1 − 5 ≤ Z ≤ 5 + 1
Hence z lies on the bisector of the line joining points (0, 0) The correct option is (B)
and (0, 1/3). 208. Let z = x + iy
Hence z lies on a straight line. |z− 1| = |z + 1| ⇒ Re z = 0 ⇒ x = 0
The correct option is (C) |z−1| = |z−i| ⇒ x = y
203. Given sum |z + 1| = |z−i| ⇒ y = −x
10
2kp 2kp ⎞ Only (0, 0) will satisfy all conditions.

∑ ⎜⎝ sin 11
+ i cos ⎟
11 ⎠
⇒ Number of complex number z = 1
k =1
The correct option is (A)
10 10
2kp 2kp
= ∑ sin + i ∑ cos
k =1 11 k =1 11

Objective_Maths_JEE Main 2017_Ch 3.indd 66 01/01/2008 03:30:39


Complex Numbers  3.67

209. Suppose roots are 1 + pi, 1 + qi


z1 − 2 z2
Sum of roots 1 + pi + 1 + qi = −α which is real 214. Given that =1
2 − z1z2
⇒ roots of 1 + pi,1 − pi
Product of roots = β = 1 + p2 ∈(1, ∞) ⇒ ( z1 − 2 z2 ) ( z1 − 2 z2 )
p ≠ 0 since roots are distinct.
The correct option is (C) = ( 2 − z1z2 ) ( 2 − z1z2 )

210. 1 + ω = −ω2 2 2
⇒ z1 − 2 z2 − z1 − 2 z2 z1 + 4 z2
(1 + w )7 = ( −w 2 ) = −w14 = −w 2

= 1 + w = A + Bw ⇒ ( A, B ) = (1,1) 2 2
= 4 − 2 z1z2 − 2 z1z2 + z1 z2

The correct option is (A)
2 2 2 2
211. Let z = x + iy (\ x ≠ 1 as z ≠ 1 ) ⇒ z1 + 4 z2 − z1 z2 − 4 = 0

z 2 = ( x 2 − y 2 ) + i( 2 xy )

z2 ⇒ z1
2
(1 − z ) − 4 (1 − z ) = 0
2
2
2
2

is real ⇒ its imaginary part = 0


z −1 ⇒ z1 = 2( as z2 ≠ 1)

⇒ 2 xy( x − 1) − y( x 2 − y 2 ) = 0
The correct option is (B)
⇒ y( x 2 + y 2 − 2 x ) = 0 215. We have,
⇒ y = 0; x 2 + y 2 − 2 x = 0 2 + 3i sin q
z =
1 − 2i sin q
\z lies either on real axis or on a circle through origin.
The correct option is (A) ( 2 + 3i sin q )(1 + 2i sin q )
⇒ z =
212. Given z = 1 ⇒ zz = 1 1 + 4 sin 2 q

1+ z 1+ z ( 2 − 6 sin 2 q ) + 7i sin q
\ = =z. =
1+ z 1+ 1 1 + 4 sin 2 q
z For z to be purely imaginary, we have Re (z) = 0
The correct option is (B)
1
⇒ 2 − 6 sin 2 q = 0 ⇒ sin q = ±
213. z ≥2 3
1 1 1 3
z+ ≥ z − ≥2− ≥ . ⎛ 1 ⎞
2 2 2 2 ⇒ q = ± sin −1 ⎜ ⎟
⎛ 1 ⎞ 3 ⎝ 3⎠
Hence, minimum distance between z and ⎜ − , 0⎟ is
⎝ 2 ⎠ 2 The correct option is (A)
The correct option is (B)

Objective_Maths_JEE Main 2017_Ch 3.indd 67 01/01/2008 03:30:44


Quadratic Equations and Expressions 4.1

CHaPtER
Quadratic Equations and
4 Expressions

Chapter Highlights
Quadratic equation, Common roots, Symmetric function of the roots, Graph of a quadratic expression, Greatest
and least values of a quadratic expression, Nature of roots of a quadratic equation with respect to one or two
real numbers, Relation between roots and coefficients of a polynomial equation, Formation of a polynomial
equation from given roots, Sign of a polynomial expression, Rational algebraic expression

QUadRaTiC eQUaTioN TRiCk(S) FoR pRoBlem SolviNG


2
An algebraic expression of the form: ax + bx + c, where
For the quadratic equation ax2 + bx + c = 0
a (≠0), b, c ∈ R is called a real quadratic expression.
 One root will be reciprocal of the other if a = c.
An equation of the form: ax2 + bx + c = 0, where
 One root is zero if c = 0.
a (≠0), b, c ∈ R is called a real quadratic equation.
The numbers a, b, c are called the coefficients of the  Roots are equal in magnitude but opposite in sign if b = 0.

quadratic equation and the expression b 2 − 4 ac is called  Both roots are zero if b = c = 0.

its discriminant. Discriminant of a quadratic equation is  Roots are positive if D > 0, a and c are of same sign and b

usually denoted by D or Δ. is of opposite sign.


 Roots are of opposite sign if a and c are of opposite

Roots of the Quadratic equation sign.


 Roots are negative if D > 0 and a, b, c are of the same
A root of the quadratic equation
sign.
ax2 + bx + c = 0 (1)  Roots are rational ⇔ D is a perfect square

2  Roots are irrational ⇔ D is positive but not a perfect


is a number a (real or complex) such that aa + ba + c = 0.
square.
The roots of the quadratic Eq. (1) are given by, c
 If a + b + c = 0, then 1, are the roots of the equation
a
−b ± b 2 − 4 ac ax2 + bx + c = 0
x=
2a c
and if a – b + c = 0, then the roots are –1 and – .
a
Nature of Roots of the 2
 If ax + bx + c = 0 is satisfied by more than two values, it

Quadratic equation is an identity and a = b = c = 0 and vice-versa.


2
 If ax + bx + c = 0, where a, b, c ∈ R, has one root p + iq,
1. If D < 0, then roots a, b are imaginary
then the other root will be p – iq. Hence, the imaginary
2. If D > 0, then roots a, b are real and distinct roots occur in conjugate pair.
3. If D = 0, then roots a, b are real and equal

Objective_Maths_JEE Main 2017_Ch 4.indd 1 01/01/2008 03:36:04


4.2  Chapter 4

1 1
 If ax2 + bx + c = 0, where a, b, c are rational, has one root Solving Eq. (1) and (2), we get a = − and b = .
5 5
p + q then the other root will be p − q . Hence, irra-
But these values do not satisfy Eq. (3). Hence the
tional roots occur in conjugate pair if the coefficients are
rational.
given equation cannot be an identify for any values
of a and b.
 The quadratic equation whose roots are reciprocals of the

roots of ax2 + bx + c = 0 is cx2 + bx + a = 0 (i.e., the coef- 3. If x2 + lx + m is an integer for every integer x, then
ficients are written in reverse order). (A) l is always an integer but m need not be an integer
 If a = 1, b, c ∈ Z and the roots are rational numbers, then
(B) m is always in integer but l need not be an integer
these roots must be integers. (C) l and m, both are always integer
2
 The condition that the roots of the equation ax + bx + c =
(D)  None of these
0 may be in the ration m : n is mnb2 = ac(m + n).
2 3
 If sum of roots of ax + bx + c = 0 is equal to the sum of
Solution: (C)
c b a
their reciprocals, then ab2, bc2, ca2 are in A.P. or , ,
Let g(x) = x2 + lx + m, then g(x) is an integer for every
are in H.P. b a c integer x.
Therefore, g(0) = m ⇒ m is an integer
Also,
Solved examples g(1) = 1 + l + m ⇒ l is an integer (Q m is integer)

1. If a + b + c = 0 and a, b, c are rational, then the roots of 4. The equation 125x + 45x = 2.27x has
the equation (b + c – a) x2 + (c + a – b) x + (a + b– c) = 0 (A)  no solution
are (B)  one solution
(A) rational (B) irrational (C)  two solutions
(C) imaginary (D) equal (C)  more than two solutions
Solution: (A) Solution: (B)
We have, The given equation can be written as
D = (c + a – b)2 – 4 (b + c – a) (a + b – c) (5/3)3x + (5/3)x = 2
= (a + b + c – 2b)2 – 4 (a + b + c – 2a) Putting (5/3)x = t, the equation becomes
(a + b + c – 2c)
t3 + t – 2 = 0
2 2
= (– 2b) – 4 (­­– 2a) (– 2c) = 4 (b – 4ac) ⇒ t3 – 1 + (t – 1) = 0
2 2
= 4 [(– a – c) – 4ac] = 4 (a – c) ⇒ (t – 1) (t2 + t + 1) + (t – 1) = 0
2
= [2 (a – c)] = perfect square ⇒ (t – 1) (t2 + t + 2) = 0
\ Roots are rational ⇒ t = 1
2. The number of values of the pair (a, b ) for which the or 2
t + t + 2 = 0
equation a (x + 1)2 + b (x2 – 3x – 2) + x + 1 = 0, ∀ x ∈ R is 2
But t + t + 2 = 0 does not have real solutions.
(A) 1 (B) 0
(C)  infinite (D)  None of these Therefore, t = 1 ⇒ (5/3)x = 1 ⇒ x = 0.
Solution: (B) 5. For a > 0, the roots of the equation
The equation logax a + logx a2 + log a x a3 = 0, are given by:
2

–4/3 –3/4
2
(a + b)x + (2a – 3b + 1)x + (a + 2b + 1) = 0 (A) a (B)  a1/2 (D) 
a (C)  a–1
is an identify in x if Solution: (A)
a + b = 0 (1) We have,
2a – 3b + 1 = 0 (2) log a a 2 log a a 3 log a a
+ + =0
log a a + log a x log a x 2 log a a + log a x
and a – 2b + 1 = 0 (3)

Objective_Maths_JEE Main 2017_Ch 4.indd 2 01/01/2008 03:36:05


Quadratic Equations and Expressions  4.3

1 2 3 So, D = 4 (a2 + 1)2 – 12 (a2 – 3a + 2) > 0


⇒ + + = 0  (let loga x = t)
1+ t t 2 + t a 2 − 3a + 2
and product of roots = <0
2t + t 2 + 2t 2 + 6t + 4 + 3t 2 + 3t 3
⇒ =0
t (1 + t ) ( 2 + t ) ⇒ a2 – 3a + 2 = (a – 1) (a – 2) < 0
⇒ 6t2 + 11t + 4 = 0 \ 1 < a < 2
2
⇒ 6t + 8t + 3t + 4 = 0 Clearly for these values of a, D > 0.
⇒ (2t + 1) (3t + 4) = 0 Hence, 1 < a < 2.
1 4 9. The number of real solutions of the equation 271/x +
⇒ t=– ,–
2 3 121/x = 2 × 81/x is
1 4 (A) one (B) two (C) infinite (D) zero
⇒ loga x = – , –
2 3 Solution: (D)
–1/2 –4/3
\ x=a ,a The given equation can be written as
6. The number of solutions of the equation sin (ex) = 5x + ⎛ 3⎞
3/ x
⎛ 3⎞
1/ x

5– x is ⎜⎝ ⎟⎠ +⎜ ⎟ = 2.
2 ⎝ 2⎠
(A) 0 (B) 1 (C) 2 (D) infinite 1/ x
⎛ 3⎞
Put ⎜ ⎟ = t, then the equation becomes
Solution: (A) ⎝ 2⎠
Put 5x = y. Then the given equation becomes
t3 + t – 2 = 0  ⇒ (t – 1) (t2 + t + 2) = 0.
2
1 ⎛ 1 ⎞
sin (ex) = y + = ⎜ y− x
⎟ + 2  (Q 5 > 0) But t2 + t + 2 = 0 has no real roots,
y ⎝ y ⎠ \ t = 1
1/ x
⇒ sin (ex) ≥ 2. ⎛ 3⎞ 1
⇒ ⎜⎝ ⎟⎠ = 1  ⇒  = 0
Which is not possible for any real value of x. 2 x
Hence, the given equation has no real solution. which is not possible for any value of x.
2/3 1/3 3 2
7. If x = 2 + 2 + 2 then the value of x – 6x + 6x is
12 x
(A) 3 (B) 2 10. For all real values of x,
(C)  1 (D)  None of these 4x2 + 9

Solution: (B) ≤2 (C) 
(A) ≤1 (B)  >1  (D) 
>2
We have, Solution: (A)
x – 2 = 22/3 + 21/3 12 x
Let = y,
Cube both sides, we get 4x2 + 9
(x – 2)3 = 22 + 2 + 3 ⋅ 22/3 ⋅ 21/3 (x – 2) Now, 4yx2 – 12x + 9y = 0
= 6 + 6 (x – 2) As x is real,
or 3 2
x – 6x + 12x – 8 = –6 + 6x.  D = 144 – 4 ⋅ 4y ⋅ 9y ≥ 0  ⇒  1 – y2 ≥ 0
\ x3 – 6x2 + 6x = 2. ⇒ y2 ≤ 1;
8. The values of a, for which the quadratic equation \ |y| ≤ 1.
3x2 + 2 (a2 + 1) x + (a2 – 3a + 2) = 0 possesses roots of
opposite sign, are 12 x
Hence, ≤ 1.
(A) 1 < a < 2 (B)  a ∈ (2, ∞) 4x2 + 9
(C) 1 < a < 3 (D)  None of these 11. If x2 – 3x + 2 be one of the factors of the expression
Solution: (A) x4 – px2 + q, then
Roots are of opposite sign if (a) roots are real and (A) p = 4, q = 5 (B)  p = 5, q = 4
­distinct, (b) product is negative. (C) p = –5, q = –4 (D)  None of these

Objective_Maths_JEE Main 2017_Ch 4.indd 3 01/01/2008 03:36:07


4.4  Chapter 4

Solution: (B) ⇒ rp – rq = pq – pr
Since x2 – 3x + 2 is one of the factors of the expression ⇒ 2rp = q (p + r)
x4 – px2 + q, therefore, on dividing the expression by
factor, remainder = 0 i.e., on dividing x4– px2 + q by 2 p+r 1 1
\ = = + .
x2 – 3x + 2, the remainder q pr p r 
(15 – 3p) x + (2p + q – 14) = 0 p a b
14. If c ≠ 0 and the equation = + has two
On comparing both sides, we get 2x x+c x−c
equal roots, then p can be
15 – 3p = 0  or  p = 5
(A) ( a − b ) 2 (B) 
( a + b )2
and 2p + q – 14 = 0  or  q = 4. (C) a + b (D)  a–b
12. If the roots of x2 – bx + c = 0 are two consecutive Solution:  (A, B)
­integers, then b2 – 4c is We can write the given equation as
(A) 1 (B) 0 p ( a + b) x + c ( b − a)
(C)  2 (D)  None of these =
2x x 2 − c2 
Solution: (A)
or p (x2 – c2) = 2 (a + b) x2 – 2c (a – b) x
The roots of the equation are given by,
or (2a + 2b – p) x2 – 2c (a – b) x + pc2 = 0
2
b ± b − 4c For this equation to have equal roots
x =
2 
c2(a – b)2 – pc2 (2a + 2b – p) = 0
b + b 2 − 4c ⇒ (a – b)2 – 2p (a + b) + p2 = 0 (Q c2 ≠ 0)
If a=
2 
p – (a + b)]2 = (a + b)2 – (a – b)2 = 4ab
⇒ [ 
2
b − b − 4c
and b= ⇒ p – (a + b) = ± 2 ab 
2 
Then, a – b = 1 ⇒ p = a + b ± 2 ab = ( a ± b ) 2 

15. If (7 − 4 3 ) x − 4 x + 3 + (7 + 4 3 ) x2 − 4 x +3
2

⇒ b 2 − 4c = 1 = 14, then
2 the value of x is given by
⇒ b – 4c = 1.
(A)  2, 2 ± 2 (B) 2 ± 3,3
13. If p (q – r) x2 + q (r – p) x + r (p – q) = 0 has equal (C) 3 ± 2 , 2 (D)  None of these
2
roots, then = Solution: (A)
q
Since (7 + 4 3 ) (7 − 4 3 ) = 1,
1 1
(A) p + (B)  +r \ the given equation becomes
r p
1
1 1 y+ = 14
(C) p + r (D)  + y
p r 2
− 4x + 3
where y = (7 − 4 3 ) x
Solution: (D) 
Since p (q – r) + q (r – p) + r (p – q) = 0 ⇒ y2 – 14y + 1 = 0 ⇒ y = 7 ± 4 3 
\ one root is 1
Now y= 7±4 3
r ( p − q)
\ other root = . ⇒ x2 – 4x + 3 = –1
p (q − r) 
⇒ x = 2, 2
Since roots are equal
rp − rq Also, y= 7−4 3
\ = 1
pq − pr ⇒ x2 – 4x + 3 = 1 ⇒ x = 2 ± 2 

Objective_Maths_JEE Main 2017_Ch 4.indd 4 01/01/2008 03:36:12


Quadratic Equations and Expressions  4.5

Sum and Product of the Roots 17. If the ratio of the roots of lx2 + nx + n = 0 is p : q, then
If a and b are roots of ax2 + bx + c = 0, then q p l
(A)  + + =0
p q n
−b Coefficient of x
Sum of roots = a + b = =–
a Coefficient of x 2  p q n
(B)  + + =0
c Constant term q p l
Product of roots = ab = =
a Coefficient of x 2  q p l
(C)  + + =1
Formation of Equation with Given Roots p q n

If a and b are roots of p q n


(D)  + + =1
f (x) = ax2 + bx + c = 0, q p l

then f (x) = (x – a) (x – b ) = 0 Solution: (B)


Let the roots be a and b.
= x2 – (a + b )x + ab = 0
−n n
i.e., x2 – (sum of the roots) x + (product of the roots) = 0. Then a+b= ; ab = ;
l l
a p
caution and =
b q
A quadratic equation with all odd integer coefficients cannot p q n a b
have rational roots. Now, + + = + + ab
q p l b a
n n
− +
a + b + ab l l = 0
Solved examples = =
ab n
16. If r be the ratio of the roots of the equation l
( r + 1) 2 18. In a quadratic equation with leading coefficient 1, a
ax2 + bx + c = 0, then = student reads the coefficient 16 of x wrongly as 19 and
r
a2 b2 obtain the roots as – 15 and – 4. The correct roots are
(A)  (B)  (A)  6, 10 (B)  –6, –10
bc ca
2 (C)  –7, –9 (D)  None of these
c
(C)  (D)  None of these
ab Solution: (B)
Since coefficient of x = 16,
Solution: (B)
\ sum of roots = –16
Given equation is ax2 + bx + c = 0 (1) Since constant term = (–15) (–4) = 60,
Let the root of equation (1) be a and ra, then \ correct answer is –6, –10.
−b 1 1 1
a + ra = (2) 19. If the roots of the equation + = are
a x+a x+b c
c equal in magnitude but opposite in sign, then their
and ra2 = (3)
a product is
From Eq. (2), 1 −1 2
b (A)  (a2 + b2) (B)  (a + b2)
a=– (4) 2 2
a ( r + 1)
1 −1
Putting the value of a in Eq. (3), we get (C)  ab (D)  ab
2 2
rb 2 c
2 2
= Solution: (B)
a ( r + 1) a
We have, ((x + b) + (x + a)c = (x + a) (x + b)
b2 ( r + 1) 2
or, = ⇒ 2
x + bx + ax – 2cx + ab – bc – ca = 0
ac r 

Objective_Maths_JEE Main 2017_Ch 4.indd 5 01/01/2008 03:36:21


4.6  Chapter 4

Now, let roots be a and b, then 22. If a, b are the roots of x2 – 2px + q = 0 and g, d are
roots of x2 – 2rx + s = 0 and a, b, g, d are in A.P., then
a + b = 0, ab = ab – bc – ac
(A) p – q = r2 – s2 (B)  s – q = r2 – p2
a + b = 0 ⇒ b + a = 2c (C) r – s = p2 – q2 (D)  None of these
and ab = ab – (b + a) c Solution: (B)
( a + b) 2 We have, a + b = 2p;
⇒ ab = ab –
2  ab = q, g + d = 2r and g d = s
1 2 2
⇒ ab = (– a – b )  Q  a, b, g, d are in A.P.
2
1 \ b – a = d – g ⇒ (b – a)2 = (d – g )2
\ ab = – (a2 + b2)
2 ⇒ (b + a)2 – 4ba = (d + g )2 – 4dg
20. If sin q and cos q are the roots of the equation ax2 + ⇒ 4p2 – 4q = 4r2 – 4s;
bx + c = 0, then
or s – q = r2 – p2
(A) (a – c)2 = b2 – c2 (B) (a – c)2 = b2 + c2
2 2 2
(C) (a + c) = b – c (D)  (a + c)2 = b2 + c2 23. The rational values of a and b in ax2 + bx + 1 = 0 if
Solution: (D) 1
is a root, are
Since sin q and cos q are the roots of the equation 4+ 3
ax2 + bx + c = 0 (A) a = 13, b = – 8 (B)  a = – 13, b = 8
b c (C) a = 13, b = 8 (D)  a = – 13, b = ­– 8
\ sin q + cos q = − and sin q cos q =
a a Solution: (A)
2
Now (sin q + cos q ) = 1 + 2 sin q cos q 1 4− 3 4− 3
One root = × =
b2 2c a + 2c 4+ 3 4− 3 13 
\ =1 + =
a2 a a  4+ 3
\ other root =
⇒ 2 2
b = a (a + 2c) = a + 2ac 13 
\ The quadratic equation is
⇒ b + c2 = a2 + 2ac + c2 = (a + c)2
2

⎛ 4 + 3 4 − 3⎞ 4+ 3 4− 3
Hence, (a + c)2 = b2 + c2 x 2 − ⎜ + ⎟ x+ ⋅ = 0
⎝ 13 13 ⎠ 13 13
21. In copying a quadratic equation of the form x2 + px +
q = 0, a student wrote the coefficient of x incorrectly or 13x2 – 8x + 1 = 0
and the roots were found to be 3 and 10; another stu- This equation must be identical with ax2 + bx + 1 = 0;
dent wrote the same equation but he wrote the ­constant \ a = 13  and  b = –8.
term incorrectly and thus he found the roots to be
4 and 7. The roots of the correct equation are 24. If a and b are rational and a, b be the roots of x2 +
(A)  5, 6 (B)  4, 6 2ax + b = 0, then the equation with rational coefficients
(C)  4, 5 (D)  None of these one of whose roots is a + b + a 2 + b 2 is
Solution: (A) (A) x2 + 4ax – 2b = 0 (B)  x2 + 4ax + 2b = 0
In case of the first student, product of the roots = 3 × (C) x2 – 4ax + 2b = 0 (D)  x2 – 4ax – 2b = 0
10 = q. So the correct value of q is 30. Solution: (B)
In case of the second student, sum of the roots
Since a, b are roots of x2 + 2ax + b = 0
= 4 + 7 = –p.
So the correct value of p is –11. a + b = –2a and ab = b
2
\ The correct equation is x – 11x + 30 = 0 Let y = a + b + a 2 + b2 
or (x – 5) (x – 6) = 0; ⇒ (y + 2a)2 =a2 + b 2 = (a + b)2 – 2ab = 4a2 – 2b
\ x = 5, 6. ⇒ y2 + 4ay + 2b = 0
\ Roots of the correct equation are 5, 6. So, the required equation is x2 + 4ax + 2b = 0.

Objective_Maths_JEE Main 2017_Ch 4.indd 6 01/01/2008 03:36:23


Quadratic Equations and Expressions  4.7

25. If c, d are the roots of the equation (x – a) (x – b) – k = 0, − b − i 4 ac − b 2


then the roots of the equation (x – c) (x – d) + k = 0 are and b= 
2a
(A) c, d (B)  a, c (C)  b, d (D)  a, b
\ a = b .
Solution: (D)
We have, (x – a) (x – b) – k = 0
COMMON ROOTS
2
⇒ x – (a + b) x + ab – k = 0 (1)
One Root Common
Since the roots of Eq. (1) are c and d
If a is a common root of the equations
\ c + d = a + b,(2)
a1x2 + b1x + c1 = 0(1)
and cd = ab – k(3)
Now (x – c) (x – d) + k = 0 and a2x2 + b2x + c2 = 0(2)

⇒ x2 – (c + d) x + cd + k = 0 then we have
a1a 2 + b1a + c1 = 0
⇒ x2 – (a + b) x + ab = 0
 [Putting the values of a + b and and a2a 2 + b2a + c2 = 0
 ab from Eqs (2) and (3)]
a2 a
⇒ (x – a) (x – b) = 0 ⇒ x = a, b. These give =
b1c2 − b2 c1 c1a2 − c2 a1

26. If the roots of the equations x2 – bx + c = 0 and x2 – cx+ 1
b = 0 differ by the same quantity then b + c is equal to = ( a1b2 − a2 b1 ≠ 0).
a1b2 − a2 b1
(A) 4 (B) 1 (C) 0 (D) –4
Thus, the required condition for one common root is
Solution: (D)
( a1b2 − a2 b1 ) (b1c2 − b2 c1 ) = (c1a2 − c2 a1 ) 2 and the value
We know that if a, b are roots of the equation c a − c2 a1 b c − b2 c1
of the common root is a = 1 2 or 1 2 .
Ax2 + Bx + C = 0, a1b2 − a2 b1 c1a2 − c2 a1
B 2 − 4 AC Both Roots Common
then a–b=
A  If the Eq. (1) and (2) have both roots common, then these
Equating the value of a – b from both the given equations will be identical. Thus the required condition for
­equations, we get both roots common is
a1 b c
b 2 − 4c = c 2 − 4b  = 1 = 1
a2 b2 c2
⇒ b2 – 4c = c2 – 4b
Trick(s) for Problem Solving
⇒ b2 – c2 = –4 (b – c)
⇒ (b – c) (b + c + 4) = 0  To find the common root of two equations, make the coef-
ficient of second degree terms in two equations equal and
⇒ b + c = –4  (Q b ≠ c) subtract. The value of x so obtained is the required com-
27. If a, b are non-real roots of ax2 + bx + c = 0, (a, b, mon root.
 If two quadratic equations with real coefficients have an
c ∈ R), then
imaginary root common, then both roots will be common and
(A) ab = 1 (B)  a=b the two equations will be identical. The required condition is
a= b
(C)  ab = 1 (D)  a1 b c
= 1 = 1
Solution: (D) a2 b2 c2
If two quadratic equations have an irrational root common,
b2 – 4ac < 0

Here then both roots will be common and the two equations will
be identical. The required condition is
− b + i 4 ac − b 2
\ a=
a1 b
= 1 = 1
c
2a  a2 b2 c2

Objective_Maths_JEE Main 2017_Ch 4.indd 7 01/01/2008 03:36:27


4.8  Chapter 4

 If a is a repeated root of the quadratic equation


30. If a, b, c ∈ R and the equations ax2 + bx + c = 0 and
x3 + 3x2 + 3x + 2 = 0 have two roots in common, then
f(x) = ax2 + bx + c = 0,
then a is also a root of the equation f  ′(x) = 0.
(A) a = b ≠ c (B)  a=b=–c
(C) a = b = c (D)  None of these
 If a is repeated common root of two quadratic equations

f(x) = 0 and f(x) = 0, then a is also a common root of the Solution: (C)


equations f  ′(x) = 0 and f  ′(x) = 0. We have, x3 + 3x2 + 3x + 2 = 0
⇒ (x + 1)3 + 1 = 0
Solved examples ⇒ (x + 1 + 1) [(x + 1)2 – (x + 1) + 1] = 0

2
⇒ (x + 2) (x2 + x + 1) = 0
28. The value of k so that the equations x – x – 12 = 0 and
kx2 + 10x + 3 = 0 may have one root in common, is − 1 ± 3i
⇒ x = –2,
43 − 43 2 
(A)  (B) 3 (C) – 3 (D) 
16 16 ⇒ x = –2, w, w 2
Solution:  (B, D) Since a, b, c ∈ R, ax2 + bx + c = 0 cannot have one real
Let a be the common root and one imaginary root. Therefore, two common roots
of ax2 + bx + c = 0 and x3 + 3x2 + 3x + 2 = 0 are w, w2.
Then, a2 – a – 12 = 0  and  ka2 + 10a + 3 = 0
b
Solving the two equations, we get Thus, − = w + w2 = –1
a
a2 a 1 ⇒ a = b
= =
117 − 12k − 3 10 + k  c
and = w ⋅ w2 = 1 ⇒ c = a
a
⇒ (– 12k – 3)2 = 117 (10 + k)
⇒ a = b = c
⇒ 9 (4k + 1)2 = 117 (10 + k)
31. If the equations k (6x2 + 3) + rx + 2x2 – 1 = 0 and
2
⇒ (4k + 1) = 13 (10 + k) 6k  (2x2 + 1) + px + 4x2 – 2 = 0 have both the roots
common, then the value of 2r – p is
⇒ 16k2 + 8k + 1 = 130 + 13k
(A) 0 (B) 1
⇒ 16k2 – 5k – 129 = 0 (C)  –1 (D)  None of these
⇒ 16k2 – 48k + 43k – 129 = 0 Solution: (A)
− 43 The two equations can be written as
\ k = 3  or  k =
16 
x2 (6k + 2) + rx + (3k – 1) = 0 (1)
2 2
29. If the equations ax + bx + c = 0 and x + 2x + 3 = 0
and x2 (12k + 4) + px + (6k – 2) = 0 (2)
have a common root, then a : b : c =
(A)  2 : 4 : 5 (B)  1 : 3 : 4 Divide by 2, we get
(C)  1 : 2 : 3 (D)  None of these p
x2 (6k + 2) + x + (3k – 1) = 0 (3)
2
Solution: (C)
Comparing Eq. (1) and (3), we get
For the equation x2 + 2x + 3 = 0,
p
Discriminant = (2)2 – 4 ⋅ 1 ⋅ 3 < 0. r =
2
\ roots of x2 + 2x + 3 = 0 are imaginary. Since the
\ 2r – p = 0
equations x2 + 2x + 3 = 0 and ax2 + bx + c = 0 are given
to have a common root, therefore both roots will be 32. If the equations x2 – ax + b = 0 and x2 + bx – a = 0 have
common. Hence both the equations are identical. a common root, then
a b c (A) a + b = 1
\ = =
1 2 3 (B) a = b
(C) a – b = 2
i.e. a : b : c = 1 : 2 : 3.
(D) a + b = 0 or a – b = 1

Objective_Maths_JEE Main 2017_Ch 4.indd 8 01/01/2008 03:36:28


Quadratic Equations and Expressions  4.9

Solution: (D) 2
= ⎛ p2 − ⎞
1
Let a be a common root of the given equations. ⎜⎝ 2⎟
+2+ 2 ≥ 2+ 2 
2p ⎠
Then a2 – aa + b = 0  and  a2 + ba – a = 0 Therefore, minimum value of a4 + b4 is 2 + 2
⇒ (a + b) a – (a + b) = 0
⇒ (a + b) (a – 1) = 0 GRAPH OF A QUADRATIC EXPRESSION
⇒ a + b = 0 or a = 1 We have, y or f (x) = ax2 + bx + c where a, b, c ∈ R, a ≠ 0.
If a = 1, 1. The shape of the curve y = f (x) is a parabola
2. The axis of the parabola is y-axis (incase b = 0) or
then 1 – a + b = 0 ⇒ a – b = 1. parallel to y-axis.
3. If a > 0, then the parabola opens upwards.
SYMMETRIC FUNCTION of the roots 4. If a < 0, then the parabola opens downwards
5. For D > 0, parabola cuts x-axis in two distinct points
A function of a and b is said to be a symmetric function if it
remains unchanged when a and b are interchanged. a > 0, D > 0
For example, a 2 + b 2 + 2ab is a symmetric ­function x-axis
of a and b whereas a 2 – b 2 + 3ab is not a symmetric
­function of a and b.
x-axis
a > 0, D > 0
Trick(s) for Problem Solving Fig. 4.1(a) Fig. 4.1(b)

In order to find the value of a symmetric function of a and 6. For D = 0, parabola touches x-axis in one point.
b, express the given function in terms of a + b and ab. The
a > 0, D = 0
following results may be useful. x-axis
2 2 2
 a + b = (a + b ) – 2ab
3 3 3
 a + b = (a + b ) – 3ab (a + b )
4 4 3 3 2 2 x-axis
 a + b = (a + b ) (a + b ) – ab (a + b ) a > 0, D = 0
5 5 3 3 2 2 2 2
 a + b = (a + b ) (a + b ) – a b (a + b )
Fig. 4.2(a) Fig. 4.2(b)
 |a – b | = (a + b )2 − 4ab 7. For D < 0, parabola does not cut x-axis.
 a2 – b2 = (a + b ) (a – b )
a < 0, D < 0
3 3 2
 a – b = (a – b ) [(a + b ) – ab] x-axis
4 4 2 2
 a – b = (a + b ) (a – b ) (a + b )

x-axis
a > 0, D < 0
Solved Example Fig. 4.3(a) Fig. 4.3(b)

1
33. If a and b be the roots of the equation x2 + px – = 0,
2 p2 GREATEST AND LEAST VALUES OF A
where p ∈ R, then the minimum value of a4 + b 4 is QUADRATIC EXPRESSION
(A)  2 (B)  2+ 2 1. If a > 0, then the quadratic expression y = ax2 + bx + c
(C) 2 − 2 (D) 2 has no greatest value but it has least value
Solution: (B) 4 ac − b 2 b
 at x = –
4a 2a
a 4 + b 4 = (a 2 + b 2) – 2a2b 2
2 2 2
2. If a < 0, then the quadratic expression y = ax2 + bx + c
= [(a + b) – 2ab] – 2(ab ) has no least value but it has greatest value
2
⎛ 1 ⎞ 1 1 4 ac − b 2 b
= ⎜ p2 + 2 ⎟ − 4
= p4 + +2  at x = −
⎝ p ⎠ 2p 2 p4  4a 2a

Objective_Maths_JEE Main 2017_Ch 4.indd 9 01/01/2008 03:36:31


4.10  Chapter 4

Sign of Quadratic Expression 2. If both roots of f (x) = 0 lie between k1 and k2, then
a+b
We have, y or f (x) = ax2 + bx + c where a, b, c ∈ R, a ≠ 0. D ≥ 0, a f (k1) > 0, a f (k2) > 0 and k1 < < k2
2
1. If a > 0 and D < 0, then f (x) > 0 for all x ∈ R i.e., f (x)
is positive for all real values of x. k1 α β k2
2. If a < 0 and D < 0, then f (x) < 0 for all x ∈ R i.e., f (x) 3. If k1 and k2 lie between the roots of f (x) = 0, then D ≥ 0,
is negative for all real values of x. a f (k1) < 0 and a f (k2) < 0.
3. If a > 0 and D = 0, then f (x) ≥ 0 for all x ∈ R i.e., f (x) is
positive for all real values of x except at vertex, where α β
k1 k2
f (x) = 0.
4. If a < 0 and D = 0, then f (x) ≤ 0 for all x ∈ R i.e. f (x) is
negative for all real values of x except at vertex, where Trick(s) for Problem Solving
f (x) = 0.
5. If a > 0 and D > 0, let f (x) = 0 have two real roots a and 1. Let f(x) = 0 be a polynomial equation. Let p and q be two
b(a < b ), then f (x) > 0 for all x ∈ (–∞, a) ∪ (b, ∞) and real numbers, p < q.
f (x) < 0 for all x ∈ (a, b ). (a) If f(p) · f(q) < 0, then the equation f(x) = 0 has odd
6. If a < 0 and D > 0, let f (x) = 0 have two real roots a and number of real roots between p and q.
b(a < b ). Then f (x) < 0 for all x ∈ (–∞, a) ∪ (b, ∞) and (b) If f(p) · f(q) > 0, then the equation f(x) = 0 has
f (x) > 0 for all x ∈ (a, b ). either no real root or even number of real roots
between p and q.
(c) If f(p) = f(q), then the equation f  ′(x) = 0 has at
Nature OF ROOTS OF A QUADRATIC least one real root between p and q (This is due to
EQUATION WITH RESPECT TO ONE OR TWO Rolle’s Theorem)
REAL NUMBERS 2. (a) If the coefficients of the polynomial equation f(x) =
0 have p changes of signs, then the equation f(x) =
Let f (x) = ax2 + bx + c, where a, b, c ∈ R, a ≠ 0. Let a, 0 will have atmost p, positive roots.
b(a  < b ) be the roots of the corresponding quadratic (b) If the coefficients of the polynomial equation f(–x) =
­equation. Let k, k1, k2 ∈ R and k1 < k2. 0 have q changes of signs, then the equation f(x) =
0 will have atmost q, negative roots.
Nature of Roots with Respect to (c) The polynomial equation f(x) = 0 will have atmost
One Real Number p + q real roots where p and q are the changes of
signs of coefficients in f(x) and f(–x). (This is due to
1. If both the roots of f (x) = 0 are greater than k, then Descarte’s Rule of signs)
b
D ≥ 0, a f (k) > 0 and k < – For example, consider
2a f(x) = 2x5 – 6x4 + 7x3 – 8x2 + 5x + 3
k α β + – + – + +
Then, f(–x) = –2x5 – 6x4 – 7x3 – 8x2 – 5x + 3
2. If both the roots of f (x) = 0 are less than k, then D ≥ 0, – – – – – +
b
a f (k) > 0 and k > – Clearly, f(x) has 4 changes of signs and f(–x) has
2a only one change of sign, Therefore, the equation
f(x) = 2x5 – 6x4 + 7x3 – 8x2 + 5x + 3 = 0 has
α β k
atmost four positive roots and one negative root.
3. If one root is less than k and other is greater than k, Also, the equation has atmost (4 + 1) = 5 real
then D > 0 and a f (k) < 0 roots.
3. (a) A polynomial equation f(x) = 0 has exactly one root
α k β equal to a if f(a) = 0 and f  ′(a) ≠ 0.
(b) A polynomial equation f(x) = 0 has exactly two roots
Roots with Respect to Two Real Numbers equal to a if f(a) = 0, f  ′(a) = 0 and f ′′ (a) ≠ 0.
1. If exactly one root of f (x) = 0 lies in the interval (k1, k2), (c)  In general, a polynomial equation f(x) = 0 has
exactly n roots equal to a if
then D > 0 and f (k1) · f (k2) < 0
f(a) = f  ′(a) = f ′′ (a) = … = f n–1(a) = 0
k1 α k2 β and     f n(a) ≠ 0

Objective_Maths_JEE Main 2017_Ch 4.indd 10 01/01/2008 03:36:33


Quadratic Equations and Expressions  4.11

RELATION BETWEEN ROOTS AND FORMATION OF A POLYNOMIAL


COEFFICIENTS OF A POLYNOMIAL EQUATION FROM GIVEN ROOTS
EQUATION If a1, a2, a3, … an are the roots of a polynomial equation
Let f (x) = a0xn + a1xn – 1 + a2xn – 2 + … + an – 1x + an = 0, a0, of degree n, then the equation is
a1, a2, …, an ∈ R, a0 ≠ 0 be a polynomial equation of degree xn – s1xn – 1 + s2xn – 2 – s3xn – 3 + … + (–1)nsn = 0
n, having n roots a1, a2, … an. Then,
where sr = Sa1a2 … ar.
1. Sum of all roots
s1 = a1 + a2 + … + an Particular Cases
a a
= Sa1 = – 1 = (–1)1 1 Quadratic Equation
a0 a0
If a, b are the roots of a quadratic equation, then the
2. Sum of the product of two roots
­equation is x2 – (a + b)x + ab = 0.
s2 = a1a2 + a1a3 + …
a a Cubic Equation
= Sa1a2 = 2 = (–1)2 2
a0 a0 If a, b, g are the roots of a cubic equation, then the equation is
3. Sum of the product of three roots
x3 – s1x2 + s2x – s3 = 0
s3 = a1a2a3 + a2a3a4 + …
a a or x3 – (a + b + g )x2 + (ab + ag + bg )x – abg = 0
= Sa1a2a3 = – 3 = (–1)3 3 and so on.
a0 a0 Biquadratic Equation
a If a, b, g, d are the roots of a biquadratic equation, then the
In general, sr = Sa1a2 … ar = (–1)r r .
a0 equation is
x4 – s1x3 + s2x2 – s3x + s4 = 0
Particular Cases
or
Quadratic Equation
x4 – (a + b + g + d )x3 + (ab + ag + ad + bg + bd + g d )x2
If a, b are roots of the quadratic equation ax2 + bx + c = 0,
then – (abg + abd + agd + bgd)x + abgd = 0
b c
a + b = – and ab =
a a SIGN OF A POLYNOMIAL EXPRESSION
Cubic Equation Step 1: Factorize the given polynomial expression as
If a, b, g are roots of the cubic equation ax3 + bx2 + cx + d
= 0, then f (x) = ( x − a1 ) k ( x − a 2 ) k ( x − a 3 ) k …
1 2 3

b ( x − a n −1 ) k ( x − a n ) k
n −1 n

s1 = a + b + g  = –
a where k1, k2, … kn ∈ N and
c
s2 = ab + ag + bg =  a1, a2, a3, … an ∈ R (a1 < a2 < a3 … < an)
a
d Step 2: Plot the points a1, a2, a3, …, an on the real line.
s3 = abg = –
a Step 3: Mark plus sign in the interval of the right of the
Biquadratic Equation largest of these numbers i.e., on the right of an.
If a, b, g, d are roots of the biquadratic equation ax4 + bx3 + –∞ α1 α2 α 3 .... αn ∞
cx2 + dx + e = 0, then Step 4: If kn is even, put ‘+’ sign of the left of an and if kn
b
s1 = a + b + g  + d = – is odd, put ‘–’ sign on the left of an.
a Step 5: Consider the next interval and put a sign in it using
c
s2 = ab + ag + ad + bg + bd + gd = the above rule. Thus, consider all the intervals.
a Step 6: The solution of f (x) > 0 is the union of all the inter-
d vals in which there is a ‘+’ sign and the solution of
s3 = abg + abd + agd + bgd = –
a f (x) < 0 is the union of all the intervals in which
e
s4 = abgd = there is a ‘–’ sign.
a

Objective_Maths_JEE Main 2017_Ch 4.indd 11 01/01/2008 03:36:35


4.12  Chapter 4

For example, consider the polynomial expression 36. The equation


58
⎛ 3⎞ x + 3 − 4 x − 1 + x + 8 − 6 x − 1 = 1 has
f (x) = (x + 2)40 (x + 1)31 ⎜⎝ x − 2 ⎟⎠ (x – 4)37
(A)  no solution
(B)  one solution
+ – + – +
(C)  two solutions
–2 –1 3/2 4 (D)  more than two solutions
⎛ 3⎞ Solution: (D)
Thus, f (x) > 0 if x ∈ (–∞, –2) ∪ ⎜ − 1, ⎟ ∪ (4, ∞)
⎝ 2⎠ Put x − 1 = t ⇒ x – 1 = t2 or x = t2 + 1,
⎛3 ⎞ The given equation reduces to
and f (x) < 0 if x ∈ (–2, –1) ∪ ⎜ , 4⎟ .
⎝2 ⎠
t 2 + 1 + 3 − 4 t + t 2 + 1 + 8 − 6t = 1
where t ≥ 0.
Solved Examples ⇒ |t – 2| + |t – 3| = 1,
⎛ 1⎞ where t ≥ 0. This equation will be satisfied if 2 ≤ t ≤ 3.
34. If the expression ⎜ ax − 1 + ⎟ is non-negative for all
⎝ x⎠ Therefore, 2 ≤ x − 1 ≤ 3 or 5 ≤ x ≤ 10.
positive real x, then the minimum value of a must be
1 \ The given equation is satisfied for all values of x
(A) 0 (B)  lying in [5, 10].
2
1 37. If the roots of the equation
(C)  (D)  None of these
4 x2 – 2ax + a2 + a – 3 = 0
Solution: (C) are real and less than 3, then
We have, (A) a < 2 (B)  2≤a≤3
1 ax 2 − x + 1 (C) 3 < a ≤ 4 (D)  a > 4
ax – 1 + ≥0⇒ ≥ 0
x x Solution: (A)
⇒ 2
ax – x + 1 ≤ 0 as x > 0 We can write the given equation as

It will hold if a > 0 and D ≤ 0 (x – a)2 = 3 – a


1 This shows that a ≤ 3 and x = a ± 3 − a
⇒ a > 0 and 1 – 4a ≤ 0 ⇒ a ≥
4 Both the roots of the given equation will be less than
1 3 if the larger of the two roots is less than 3, that is, if
Therefore, the minimum value of a is .
4
a + 3 − a < 3
35. If a, b are the roots of the equation x2 – 3x + a = 0, a
⇒ 3 − a – (3 – a) < 0
∈ R and a < 1 < b, then a belong to
⎛ 9⎞ ⇒ 3 − a (1 − 3 − a ) < 0
(A)  ⎜ − ∞, ⎟ (B) (–∞, 2)
⎝ 4⎠ ⇒ a < 3 and 1 − 3 − a < 0
⎛9 ⎞
(C) (2, ∞) (D)  ⎜⎝ , ∞⎟⎠ But 3 − a > 1 ⇒ 3 – a > 1 or a < 2
4
Solution:  (B, C) Thus, a < 3 and a < 2 ⇒ a < 2
Since 1 lies between the roots of the given equation,
therefore, D > 0 and f (1) < 0 ⇒ 9 – 4a > 0 and 1 – 3 + 38. If f (x) = x2 + 2bx + 2c2 and g(x) = – x2 – 2cx + b2 such
a<0 that min f (x) > max g(x), then the relation between
9 b and c is:
⇒ a< and a < 2
4 (A) |c| < |b| 2 (B) 0 < c < b 2
\ a < 2 (C) |c| < |b| 2 (D)  |c| > |b| 2

Objective_Maths_JEE Main 2017_Ch 4.indd 12 01/01/2008 03:36:38


Quadratic Equations and Expressions  4.13

Solution: (D) ⇒ x2 – 2ax – a2 = 0
2 2
D 4b − 8c ⇒ x = (1 + 2 ) a, (1 − 2 ) a
min f (x) = – =–
4a 4   Q  x ≥ a and a ≤ 0
2 2
= –(b – 2c ) (upward parabola)
\ x = (1 − 2 ) a.
D 4c 2 + 4b 2
max g(x) = – = 41. If f (x) = x – [x], x (≠0) ∈ R, where [x] is the greatest
4a 4 
2 2
integer less than or equal to x, then the number of solu-
=b +c  (downward parabola)
⎛ 1⎞
tions of f (x) + f ⎜ ⎟ = 1 are
Now 2c2 – b2 > b2 + c2 ⎝ x⎠
⇒ c2 > 2b2 ⇒ |c| > 2 |b| (A) 0 (B) 1 (C) infinite   (D) 2
39. For the equation |x2| + |x| – 6 = 0, the roots are Solution: (C)
(A)  real and equal ⎛ 1⎞
We have, f (x) + f ⎜ ⎟ = 1
(B)  real with sum 0 ⎝ x⎠
(C)  real with sum 1 1 ⎡1⎤
(D)  real with product 0 ⇒ x – [x] + − = 1
x ⎢⎣ x ⎥⎦
Solution: (B) 1 ⎡1⎤
⇒ x + − 1 = [x] + ⎢ ⎥
For, x < 0, |x| = – x x ⎣x⎦ 
\ equation is 2
x +1− x
⇒ = (integer) k (say)
x2 – x – 6 = 0 ⇒ x = – 2, 3 x
 Q x < 0 ⇒ x2 – (k + 1) x + 1 = 0
\ x = –2 is the solution Since x is real, so (k + 1)2 – 4 ≥ 0
For, x ≥ 0, |x| = x, ⇒ k2 + 2k – 3 ≥ 0  ⇒ (k + 3) (k – 1) ≥ 0
\ equation is ⇒ k ≤ – 3  or  k ≥ 1
x2 + x – 6 = 0 ⇒ x = 2, –3 Therefore, number of solutions is infinite.
 Q x ≥ 0
42. If (log5x)2 + log5x < 2, then x belongs to the interval
\ x = 2 is the solution.
⎛ 1 ⎞ ⎛1 1 ⎞
Hence, x = 2, –2 are the solutions and their sum is zero. (A)  ⎜ , 5⎟ (B) 
⎜⎝ 5 , ⎟
⎝ 25 ⎠ 5⎠
40. If a ≤ 0, then the root of the equation (C) (1, ∞) (D)  None of these
x2 – 2a | x – a | – 3a2 = 0 is
Solution: (A)
(A) (1 − 2 )a (B)  ( − 1 + 6 )a We have, (log5 x)2 + log5 x < 2
(C) (1 + 2 )a (D) 
− (1 + 6 )a Put log5 x = a then a2 + a < 2
Solution:  (A, B) ⇒ a2 + a – 2 < 0
If x – a < 0, |x – a| = –(x – a) ⇒ (a + 2) (a – 1) < 0
\ equation becomes x2 + 2a (x – a) – 3a2 = 0
⇒ –2 < a < 1 or –2 < log5 x < 1
⇒ x2 + 2ax – 5a2 = 0
\ 5–2 < x < 5
⇒ x = – (1 + 6 ) a, ( − 1 + 6 ) a 1
 Q x < a ≤ 0 i.e., < x < 5
25
\ x = ( − 1 + 6 ) a 43. The greatest negative integer satisfying x2 – 4x –
77 < 0 and x2 > 4 is
If x – a ≥ 0, |x – a| = x – a
(A) –4 (B) –7
\ the equation becomes x2 – 2a (x – a) – 3a2 = 0 (C)  –6 (D)  None of these

Objective_Maths_JEE Main 2017_Ch 4.indd 13 01/01/2008 03:36:41


4.14  Chapter 4

Solution: (C) 46. The solution set of the inequality log ⎛ π ⎞ (x2 – 3x


sin ⎜ ⎟
+ 2) ≥ 2 is ⎝ 3⎠
We have, x2 – 4x – 77 < 0 and x2 – 4 > 0
⎛1 ⎞ ⎛ 5⎞
⇒ (x + 7) (x – 11) < 0 and (x – 2) (x + 2) > 0 (A)  ⎜ , 2⎟ (B) 
⎜⎝1, ⎟⎠
⎝2 ⎠ 2
⇒ –7 < x < 11 and x < – 2 or x > 2
⎡1 ⎞ ⎛ 5⎤
\ – 7 < x < –2 (C)  ⎢ , 1⎟ ∪ ⎜ 2, ⎥ (D)  None of these
⎣2 ⎠ ⎝ 2⎦
44. The solution of the inequation 4–x + 0.5 – 7.2–x < 4,
Solution: (C)
x ∈ R, is
(A) (–2, ∞) (B)  (2, ∞) We have, log ⎛π⎞
(x2 – 3x + 2) ≥ 2
sin ⎜ ⎟
⎝ 3⎠
⎛ 7⎞ 3
(C)  ⎜ 2, ⎟ (D)  None of these ⇒ x2 – 3x + 2 ≤
⎝ 2⎠ 4
Solution: (A) 5
⇒ ≤ 0 x2 – 3x +
The given inequation is 4
⇒ 4x2 – 12x + 5 ≤ 0
4–x + 0.5 – 7.2–x < 4, x ∈ R
Let 2–x = t ⇒ 4x2 – 10x – 2x + 5 ≤ 0
\ 2t2 – 7t < 4 ⇒ (2x – 5) (2x – 1) ≤ 0
⇒ 2t – 7t – 4 < 02 1 5
⇒ ≤ x ≤ (1)
2 2
⇒ (2t + 1) (t – 4) < 0
Also, x2 – 3x + 2 > 0
1
⇒ – < t < 4 ⇒ (x – 1) (x – 2) > 0
2
but 2–x > 0 ⇒ x < 1  or  x > 2 (2)
so 0 < t < 4
From Eqs (1) and (2), we get
⇒ 0 < 2–x < 4
⎡1 ⎞ ⎛ 5⎤
⇒ –2 < x < ∞ or x ∈ (–2, ∞) x ∈ ⎢ , 1⎟ ∪ ⎜ 2,
⎣2 ⎠ ⎝ 2 ⎥⎦
x x
⎛ 1⎞ ⎛ 1⎞ 47. The values of a which make the expression x2 – ax +
45. The real values of x for which 372 ⎜ ⎟ ⎜ ⎟ > 1,
are
⎝ 3⎠ ⎝ 3⎠ 1 – 2a2 always positive for real values of x, are
(A) x ∈ [0, 64] (B)  x ∈ (0, 64) 2 2 2 2
(A) − < a < ≤a≤
(B) –
(C) x ∈ [0, 64) (D)  None of these 3 3 3 3
2
Solution: (C) (C) a < 1 (D)  0<a<
3
The given inequation is valid only when Solution: (A)
x ≥ 0 (1) Since the coefficient of x2 is 1 which is positive, there-
The given inequation can be written in the form fore the given expression is positive for all real values
of x if D < 0.
372 − x − x
> 1
⇒ (–a)2 – 4 (1 – 2a2) < 0
⇒ 72 – x – x > 0 (Q 3 > 1)
⇒ 9a2 – 4 < 0
⇒ x+ x – 72 < 0
⇒ (3a + 2) (3a – 2) < 0
⇒ ( x + 9) ( x – 8) < 0 2 2
⇒ – < a < .
But x + 9 > 0 for all x ≥ 0 3 3

\ x –8<0⇒ x < 8 48. If the roots of the equation x2 – 2ax + a2 + a – 3 = 0 are


real and less than 3, then
\ 0 ≤ x < 64  [from (1)].

Objective_Maths_JEE Main 2017_Ch 4.indd 14 01/01/2008 03:36:44


Quadratic Equations and Expressions  4.15

(A) a < 2 (B)  2≤a≤3 1


⇒ t+ ≥ 2 2 (where t = 2x)
(B) 3 ≤ a ≤ 4 (D)  a > 4 t
Solution: (A) ⇒ t2 – 2 2 t + 1 ≥ 0
Since the roots of the given equation are real ⇒ [t − ( 2 − 1)] [t − ( 2 + 1)] ≥ 0
\ B2 – 4AC ≥ 0
⇒ t≤ 2 − 1  or t ≥ 2 +1 
⇒ 4a2 – 4 (a2 + a – 3) ≥ 0 but t > 0
⇒ –a + 3 ≥ 0  or  a ≤ 3 (1) ⇒ 0 < 2x ≤ 2 − 1  or 2x ≥ 2 +1 
Since the root is less than 3, so f (3) > 0 ⇒ –∞ < x ≤ log2 ( 2 − 1) 
2 2
⇒ 3 – 2a (3) + a + a – 3 > 0 or x ≥ log2 ( 2 + 1) 
⇒ 2
a – 5a + 6 > 0 or (a – 2) (a – 3) > 0  (but not acceptable as x < 0)

⇒ a < 2  or  a > 3 (2) ⎡1 ⎞


\ x ∈ (–∞, log2 ( 2 − 1) ] ∪ ⎢ , ∞⎟
From Eq. (1) and (2), we have a < 2. ⎣2 ⎠ 
2
51. If a < b, then the solution of x + (a + b) x + ab < 0, is
49. The value of k for which the number 3 lies between the given by
roots of the equation x2 + (1 – 2k) x + (k2 – k – 2) = 0
(A) x < –b or x < –a (B)  a<x<c
is given by,
(B) x < a or x > b (D) –b < x < –a
(A) 2 < k < 5 (B)  k < 2
(C) 2 < k < 3 (D)  k > 5 Solution: (D)

Solution: (A) x2 + (a + b) x + ab < 0
Let f (x) = x2 + (1 – 2k) x + k2 – k – 2 ⇒ (x + a) (x + b) < 0
The number 3 lies between the roots of the given ⇒ x + a < 0, x + b > 0
­equation, if f (3) < 0. or x + a > 0, x + b < 0
2
Now, f (3) = 9 + (1 – 2k) 3 + k – k – 2 ⇒ x < –a, x > –b
= 10 – 7k + k2 = k2 – 7k + 10 or x > ­–a, x < –b
⇒ –b < x < –a
Hence f (3) < 0
or –a < x < –b
⇒ k2 – 7k + 10 < 0
Since a < b
⇒ (k – 2) (k –­5) < 0 ⇒ 2 < k < 5.
\ –a > –b
x |x|
50. Solution of 2 + 2 ≥ 2 2 is Hence –b < x < –a.
(A) (–∞, log2 ( 2 + 1) 52. The conditions that the equation ax2 + bx + c = 0 has
(B)  ⎣⎡log 2 ( )
2 + 1 , ∞) both the roots positive is that
(A) a and b are of the same sign
⎛1 ⎞ (B) a, b and c are of the same sign
(C)  ⎜ , log 2 ( 2 − 1)⎟
⎝2 ⎠ (C) a and c are of the same sign opposite to that of b
⎡1 ⎞ (D) b and c are of the same sign opposite to that of a
(D) (–∞, log2 ( 2 − 1) ] ∪ ⎢ , ∞⎟
⎣2 ⎠ Solution: (C)
Solution: (D) Since both the roots are positive
We have, 2x + 2x ≥ 2 2 (x ≥ 0) −b c
\ >0, >0
1 a a 
⇒ 2x ≥ 2 ⇒x≥
2 b c
⇒ <0, >0
and 2x + 2–x ≥ 2 2 (x < 0) a a 
\ a and c have same sign opposite to that of b.

Objective_Maths_JEE Main 2017_Ch 4.indd 15 01/01/2008 03:36:46


4.16  Chapter 4

53. The smallest value of x2 – 3x + 3 in the interval To Find the Values of a Rational Expression in x,
⎛ 3⎞ Where x is Real
⎜⎝ − 3, ⎟⎠ is
2
Trick(s) for Problem Solving
(A) –20 (B) –15
3  Put the given rational expression equal to y and form the

(C) 5 (D) 
4 quadratic equation in x.
 Find the discriminant D of the quadratic equation obtained
Solution: (D)
2 in step 1.
⎛ 3⎞ 9
We have, x2 – 3x + 3 = ⎜ x − ⎟ + 3 −  Since x is real, therefore, put D ≥ 0. We get an inequation
⎝ 2⎠ 4 in y.
2
 Solve the above inequation for y. The values of y so
⎛ 3⎞ 3
= ⎜x − ⎟⎠ + obtained determine the set of values attained by the given
⎝ 2 4
rational expression.
3
\ smallest value = , which lies in the interval
4
⎛ 3⎞
⎜⎝ − 3, ⎟⎠ .
2 Trick(s) for Problem Solving

RATIONAL ALGEBRAIC EXPRESSION The general quadratic expression ax2 + 2hxy + by2 + 2gx +
2fy + c in x and y may be resolved into two linear rational
P (x ) factors if
An expression of the form where P(x) and Q(x) are
Q (x ) abc + 2fgh – af 2 – bg2 – ch2 = 0
polynomials and Q(x) ≠ 0, is known as a rational algebraic
a h g
expression.
or h b f =0
Sign Scheme for a Rational Algebraic g f c
Expression in x  If sum of coefficients of a polynomial equation a0 + a1x +
a2x2 + … + an xn = 0 is zero, then x = 1 is always atleast
Step 1: Factorise the numerator and denominator of the one root of equation e.g., if a(b – c) x2 + b(c – a) x + c (a
given rational expression into linear factors. Make – b) = 0, then as Sa (b – c) = 0, x = 1 is atleast one root
the coefficient of x positive in all factors. of this equation.
Step 2: Find the real values of x by equating all the factors 2 2
 Least value of the expression (x – y) + (y – z) + (z – x)
2

to zero. is 0.
Step 3: If n distinct real values of x are obtained then the n
 Sum of real roots of the equation an |x| + an – 1 |x|
n–1
+
entire line will be divided into (n + 1) parts. … + a0 = 0 is 0, e.g. if |x| = 2 satisfies the equation, then
Step 4: Plot all these points on the number line in order. x = 2 and x = –2 are real roots, their sum is 0.
1
Step 5: Start with ‘+’ sign from extreme right and change 2
 Length of latus rectum of parabola y = ax + bx + c is .
the sign alternatively in other parts. a

– + – + – +
–∞ ∞
Solved Examples
caution 54. The sum of the real roots of the equation
If the rational expression in x occurs under modulus sign, |x – 2|2 + |x – 2| – 2 = 0 is
then first of all remove the modulus sign and then proceed.
(A) 2 (B) 6 (C) 4 (D) 8
In order to remove the modulus sign, the following
results may be useful: Solution: (C)
 |x| = k ⇔ x = ±k
Put |x – 2| = t.
 |x| < k ⇔ = – k < x < k

 |x| > k ⇔ = x < – k or x > k. The given equation becomes


t2 + t – 2 = 0 or (t + 2) (t – 1) = 0

Objective_Maths_JEE Main 2017_Ch 4.indd 16 01/01/2008 03:36:48


Quadratic Equations and Expressions  4.17

Since t + 2 = |x – 2| + 2 > 0 57. If x is real, then the maximum value of 3 – 6x – 8x2 is


\ we get t – 1 = 0 17 33
(A)  (B) 
⇒ |x – 2| = 1 ⇒ x – 2 = ±1 8 8
21
⇒ x = 3, 1. (C)  (D)  None of these
8
Thus, the sum of roots is 4.
Solution: (B)
x 2 + 2 x − 11 Let y = 3 – 6x – 8x2
55. If x is real, the expression takes all real
x −3
values except those which lie between a and b, then then 8x2 + 6x + y – 3 = 0.
a and b are Since x is real,
(A)  –12, –4 (B)  –12, 2 (C)  4, 12 (D)  –4, 4
\ 62 – 4 ⋅ 8 (y – 3) ≥ 0,
Solution: (C)
or 36 – 32y + 96 ≥ 0
x 2 + 2 x − 11
Let y = or 32y ≤ 132
x −3 
⇒ xy – 3y = x2 + 2x – 11 132
\ y≤
32 
⇒ x2 + (2 – y) x + (3y – 11) = 0 (D ≥ 0)
33
or y≤
⇒ (2 – y)2 – 4 (3y – 11) ≥ 0 8 
⇒ 4 + y2 – 4y – 12y + 44 ≥ 0 33
Hence, maximum value of y = .
⇒ y2 – 16y + 48 ≥ 0 8
1 − x + x2
⇒ y2 – 12y – 4y + 48 ≥ 0 58. For all real x, the minimum value of is
1 + x + x2
⇒ (y – 4) (y – 12) ≥ 0 1
(A) 0 (B)  (C) 1 (D) 3
⇒ y ≤ 4  or  y ≥ 12 3
Solution: (B)
x 2 − 3x + 4 1 − x + x2
56. For real values of x, the expression 2 lies Let z=
between x + 3x + 4 1 + x + x2 
1 1 ⇒ z + zx + zx2 = 1 – x + x2
(A) − and 7 (B)  and 7
7 7 ⇒ zx2 – x2 + zx + x + z – 1 = 0
1
(C)  and 3 (D)  None of these ⇒ x2 (z – 1) + x (z + 1) + (z – 1) = 0
3
Solution: (B) For real x,
x 2 − 3x + 4 B2 – 4AC ≥ 0
Let y= 2
x + 3x + 4  ⇒ (z + 1)2 – 4 (z – 1) (z – 1) ≥ 0
⇒ yx + 3xy + 4y = x2 – 3x + 4
2
⇒ z2 + 2z + 1 – 4z2 + 8z – 4 ≥ 0
⇒ (y – 1) x2 + 3 (y + 1) x + 4 (y – 1) = 0
⇒ – 3z2 + 10z – 3 ≥ 0
Since x is real,
⇒ – 3z2 + 9z + z – 3 ≥ 0
\ discriminant ≥ 0
⇒ – 3z (z – 3) + 1 (z – 3) ≥ 0
⇒ 9 (y + 1)2 – 16 (y – 1)2 ≥ 0
⇒ (z – 3) (– 3z + 1) ≥ 0
⇒ 9 (y2 + 2y + 1) – 16 (y2 – 2y + 1) ≥ 0 1
⇒ ≤ z ≤ 3
⇒ –7y2 + 50y – 7 ≥ 0 ⇒ 7y2 – 50y + 7 ≤ 0 3
1
⎛ 1⎞ 1 \ minimum value of z = .
⇒ (y – 7) ⎜ y − ⎟ ≤ 0 ⇒ ≤ y ≤ 7. 3
⎝ 7⎠ 7

Objective_Maths_JEE Main 2017_Ch 4.indd 17 01/01/2008 03:36:50


4.18  Chapter 4

x2 − 2x + 4 Let 3x + 1 = y
59. Given that, for all real x, the expression 2
x + 2x + 4 Then y ∈ R for all x ∈ R.
1
lies between and 3. The values between which the
3 9 ⋅ 32 x + 6 ⋅ 3x + 4 32 x + 2 + 2 ⋅ 3x + 1 + 4
9 ⋅ 32 x + 6 ⋅ 3x + 4 \ =
expression lies are 9 ⋅ 32 x − 6 ⋅ 3x + 4 32 x + 2 − 2 ⋅ 3x + 1 + 4 
9 ⋅ 32 x − 6 ⋅ 3x + 4
y2 + 2 y + 4
(A)  0 and 2 (B)  –1 and 1 =
y2 − 2 y + 4 
1
(C)  –2 and 0 (D)  and 3. From Eq. (1),
3
1 y2 + 2 y + 4
Solution: (D) < < 3
3 y2 − 2 y + 4
1 x2 − 2x + 4
Given < < 3 for all x ∈ R.
3 x2 + 2x + 4 1 9 ⋅ 32 x + 6 ⋅ 3x + 4
\ < < 3.
3 9 ⋅ 32 x − 6 ⋅ 3x + 4
1 x2 + 2x + 4
⇒ < < 3 for all x ∈ R.(1)
3 x2 − 2x + 4

EXERCISES

Single Option Correct Type


3x

1. The roots of the equation 2 x + 2 ⋅ 3 x − 1 = 9 are given by (A)  two real roots
⎛ 2⎞ (B) ­two positive roots
(A) log2, ⎜ ⎟ – 2 (B)  3, – 3 (C)  two negative roots
⎝ 3⎠
(D) ­one positive and one negative root
log 3
(C)  – 2, 1 – (D)  1 – log23, 2 7. If a, b, c, d and p are distinct real numbers such that
log 2
(a2 + b2 + c2) p2 – 2 (ab + bc + cd) p + (b2 + c2 + d 2) ≤ 0
2. If a, b, c are positive real numbers, then the number of then a, b, c and d
real roots of the equation ax2 + b |x| + c = 0 is (A)  are in A.P. (B)  are in G.P.
(A) 0 (B) 2 (C)  are in H.P. (D) ­satisfy ab = cd
(C)  4 (D)  None of these
8. Let S denotes the set of all values of x for which the
3. If x2 – x + 1 = 0, then value of x3n is equation 2x2 – 2 (2a + 1)x + a (a + 1) = 0 has one root
(A) 0 (B) –1 (C) 1 (D) –1, 1 less than a and other root greater than a, then S equals
(A)  (0, 1) (B)  (–1, 0)
4. The number of negative integral solutions of
(C)  (0, 1/2) (D)  None of these
x2 · 2x + 1 + 2|x – 3| + 2 = x2 · 2(|x – 3| + 4) + 2x – 1 is
9. Let a, b, c be positive real numbers, such that bx2 +
(A) 4 (B) 2 (C) 1 (D) 0
2
( ( a + c) 2 + 4b 2 ) x + (a + c) ≥ 0, ∀ x ∈ R, then a, b, c
5. If a and b (a < b), are the roots of the equation x + are in:
bx + c = 0, where c < 0 < b, then
(A)  G.P. (B)  A.P.
(A) 0 < a < b (C)  H.P. (D)  None of these
(B) a < 0 < b < |a |
(C) a < b < 0 10. If the ratio of the roots of x2 + bx + c = 0 and x2 + qx +
(D) a < 0 < |a | < b r = 0 be the same, then
(A) r2c = b2q (B)  r2b = c2q
6. If a and b are the roots of x2 + px + q = 0 and a4 and
b 4 are the roots of x2 – rx + s = 0, then the equation (C) rb2 = cq2 (D)  rc2 = bq2
x2 – 4qx + 2q2 – r = 0 has always

Objective_Maths_JEE Main 2017_Ch 4.indd 18 01/01/2008 03:36:52


Quadratic Equations and Expressions  4.19

π 20. If ax2 + bx + c = 0, a ≠ 0, a, b, c ∈ R has distinct real


11. If 0 ≤ x < , then the solution of the equation
2 roots in (1, 2) then a and 5a + 2b + c have
2 2

16sin x + 16cos x = 10 is given by x equal to (A)  same sign (B)  opposite sign
π π π π (C)  not determined (D)  None of these
(A)  , (B)  ,
6 3 3 2 21. If a < 0 the positive root of the equation x2 – 2a |x – a|
π π – 3a2 = 0 is
(C)  , (D)  None of these
6 2 (A) a ( −1 − 6 ) (B)  a ( −1 + 6 )

12. If one of the roots of the equation x2 – (p + 1) x + p2 + (C) a (1 − 2 ) (D)  None of these
p – 8 = 0 is greater than 2 and the other root is smaller 22. If px2 + qx + r = 0 has no real roots and p, q, r are real
than 2, then p is such that such that p + r > 0, then
11 (A) p – q + r ≤ 0 (B)  p + r ≥ q
(A)  − < p < 3 (B)  –2 < p < 3
3 (C) p + r = q (D)  None of these
(C) 2 < p < 3 (D)  None of these
23. Given lx2 – mx + 5 = 0 does not have two distinct real
3
13. The common roots of the equations x + 2x + 2x + 2
roots, the minimum value of 5l + m is
1 = 0 and 1 + x130 + x1988 = 0 are (where w is a none (A) 5 (B) –5 (C) 1 (D) –1
real cube root of unity)
24. The set of possible values of l for which l2 – (l2 –
(A) w (B)  w2
5l + 5)x + (2l2 – 3l – 4) = 0 has roots whose sum and
(C)  –1 (D)  None of these
product are both less than 1 is
14. If ‘x’ satisfies |x2 – 3x + 2| + |x – 1| = x – 3, then ⎛ 5⎞
(A) x ∈ f (B)  x ∈ [1,2] (A)  ⎜ −1, ⎟ (B)  (1, 4)
⎝ 2⎠
(C) x ∈ [3, ∞) (D)  x ∈ (–∞, ∞)
⎡ 5⎤ ⎛ 5⎞
15. The number of solutions (s) of the equation (C)  ⎢1, (D) 
⎜⎝1, ⎟⎠
⎣ 2 ⎥⎦ 2
3 x 2 + 6 x + 7 + 5 x 2 + 10 x + 14 ≤ 4 − 2 x − x 2 is 25. If 1 lies between the roots of 3x2 – 3sin q – 2cos2q = 0
(A) one (B) two (C) four (D) infinite then
−1 1 −1
16. If (a2 – 1) x2 + (a – 1) x + a2 – 4a + 3 = 0 is an identity (A)  < sin q < (B)  < sin q < 0
in x, then the value of a is 2 2 2
1
(A) 1 (B) 3 (C) –1 (D) –3 (C)  < sin q < 1 (D)  None of these
2
17. Both the roots of the equation (x – b) (x – c) + (x – a)
(x – c) + (x – a) (x – b) = 0 are always 26. If a, b are the roots of the equation 375x2 – 25x – 2 = 0
n
(A) positive
(C)  real
(B) negative
(D)  None of these
and Sn = a n + b n, then Lt
n →∞
∑ Sr is
r =1
2 7 1
18. If a, b are the roots of the equation x + px + q = 0 then (A)  (B) 
a 12 12
is a root of the equation 35
b (C)  (D)  None of these
(A) px2 + (2q – p2) x + p = 0 12
(B) qx2 + (p2 – 2q) x + q = 0 27. The solution set of (x)2 + (x + 1)2 = 25, where (x) is the
(C) qx2 + (2q – p2) x + q = 0 least integer greater than or equal to x, is
(D)  None of these (A)  (2, 4) (B)  (–5, 4] ∪ (2, 3]
(C)  [–4, –3) ∪ [3, 4) (D)  None of these
19. If a, b, c ∈ R and quadratic equation x2 + (a + b) x +
( 3− x )
c = 0 has no real roots then 28. Number of solutions of log2 (9 – 2x) = 10log 10
is
(A) c (a + b + c) > 0 (A) 1
(B) c + c (a + b + c) > 0 (B) 2
(C) c + c (a + b – c) > 0 (C) 3
(D) c (a + b – c) > 0 (D)  None of these

Objective_Maths_JEE Main 2017_Ch 4.indd 19 01/01/2008 03:36:55


4.20  Chapter 4

29. If ax2 + bx + 6 = 0 does not have two distinct real roots a +b b


(A) g  = g  = a +
(B) 
a ∈ R, b ∈ R, then the least value of 3a + b is 2 2
(A) 4 (B) –1 (C) 1 (D) –2 (C) g  = a (D)  a<g<b
30. If a, b be the roots of x2 + px – q = 0 and g, d be the 39. Number of solutions of the equation x2 – 2 – 2[x] = 0
(a − g )(a − d ) ([⋅] denotes greatest integer function) is
roots of x2 + px + r = 0, q + r ≠ 0, then = (A) 1 (B) 2
(b − g )(b − d )
(C)  3 (D)  None of these
(A) 1 (B) q (C)  r (D)  q+r 2 2

x+2
1 40. The number of real roots of the equation 2sin x − 2cos x =
31. Number of integral solutions of 2 > is 1 is
x +1 2
(A) 2 (B) 1
(A) 0 (B) 1 (C) 2 (D) 3 (C)  infinite (D)  None of these
32. If the ratio of the roots of lx2 + mx + v = 0 is equal to the 41. If the absolute value of the difference of roots of the
ratio of the roots of x2 + x + 1 = 0, then l, m, v are in
equation x2 + px + 1 = 0 exceeds, 3 p , then
(A)  A.P. (B)  G.P.
(A) p < –1 or p > 4 (B)  p > 4
(C)  H.P. (D)  None of these
(C) –1 < p < 4 (D)  0 ≤ p < 4
33. If c < a < b < d, then roots of the equation bx2 + (1 – b
42. If the roots of x2 + ax + b = 0 are c and d, then roots of
(c + d)) x + bcd – a = 0
x2 + (2c + a) x + c2 + ac + b = 0 are
(A)  are real and one lies between c and a
(A) 1, d – c (B)  0, d – c
(B)  real and distinct in which one lies between a and b
(C) 1, c – d (D)  None of these
(C)  real and distinct in which one lies between c and d
(D)  roots are not real 43. If the equation x2 + 2 (k + 1)x + 9k – 5 = 0 has only
negative roots, then
34. If the roots of the equation ax2 + bx + c = 0 are of the
a a +1 (A) k ≤ 0 (B)  k ≥ 0 (C)  k ≥ 6 (D)  k ≤ 6
form and , then the value of (a + b + c)2 is
a −1 a 44. If the product of the roots of the equation x2 – 3kx +
(A) b – 2ac 2
(B)  b – 4ac 2 2e2 ln k – 1 = 0 is 7, then for real roots the value of k is
(C) 4b2 – ac (D) 2b2 – ac equal to
(A) 1 (B) 2 (C) 3 (D) 4
35. If a, b be roots of x2 + px + 1 = 0 and g, d be the roots
of x2 + qx + 1 = 0, then (a – g ) (b – g ) (a + d ) (b + d ) = 45. The solution set of
x
(A) p2 + q2 (B)  p2 – q2 ⎛ 3⎞ 2
⎜⎝ ⎟⎠ = x – x – 9 is
2
(C) q – p 2
(D)  None of these 5
(A) {0} (B) {1}
36. If a and b are odd integers then [x]2 + a [x] + b = 0 (C) f (D)  None of these
(where [·] denotes greatest integer function) has
(A)  finite number of roots 46. The equation esin x – e–sin x – 4 = 0 has
(B)  infinite number of roots (A)  infinite number of real roots
(C)  no roots (B)  no real roots
(D)  None of these (C)  exactly one real root
(D)  exactly four real roots
37. If log9 (x2 – 5x + 6) > log3 (x – 4), x belongs to
(A) (–∞, 4) 47. Suppose the cube x3 – px + q has three distinct real
(B) (4, ∞) roots where p > 0 and q > 0. Then which one of the
(C) (–∞, –4) ∪ (4, ∞) following holds?
p
(D)  no real value of x (A) The cubic has minima at and maxima at
3
38. Let a, b, c be real numbers, a ≠ 0. If a is a root of p

a2x2 + bx + c = 0, b is a root of a2x2 – bx – c = 0 and 3
p
0 < a < b, then the equation a2x2 + 2bx + 2c = 0 has a (B) The cubic has minima at – and maxima at
3
root g  that always satisfies p
3

Objective_Maths_JEE Main 2017_Ch 4.indd 20 01/01/2008 03:36:56


Quadratic Equations and Expressions  4.21

p p (A) nx1n − 1 + a (B) 
n (x1)n – 1
(C) The cubic has minima at both and –
3 3 nx1n − 1 + b
(C) nx1 + b (D) 
p p
(D)  The cubic has maxima at both and – 56. If the roots of the equation x2 – 2ax + a2 + a – 3 = 0 are
3 3 real and less than 3, then
48. The quadratic equations x2 – 6x + a = 0 and x2 – cx (A) a < 2 (B)  2≤a≤3
+ 6 = 0 have one root in common. The other roots of (B) 3 ≤ a ≤ 4 (D)  a > 4
the first and second equations are integers in the ratio 1 − x + x2
4 : 3. Then the common root is 57. For all real x, the minimum value of is
1 + x + x2
(A) 1 (B) 4 (C) 3 (D) 2 1
(A) 0 (B)  (C) 1 (D) 3
3
49. If the roots of the equation bx2 + cx + a = 0 be x2 − 2x + 4
­imaginary, then for all real values of x, the expression 58. Given that, for all real x, the expression 2
x + 2x + 4
3b2x2 + 6bcx + 2c2 is 1
lies between and 3. The values between which the
(A)  greater than 4ab (B)  less than 4ab 3
(C)  greater than –4ab (D)  less than –4ab 9 ⋅ 32 x + 6 ⋅ 3x + 4
expression lies are
50. The equation 9 ⋅ 32 x − 6 ⋅ 3x + 4
(A)  0 and 2 (B)  –1 and 1
x + 3 − 4 x − 1 + x + 8 − 6 x − 1 = 1 has 1
(C)  –2 and 0 (D)  and 3.
(A)  no solution 3
(B)  one solution 59. The value of k for which the number 3 lies between
(C)  two solutions the roots of the equation x2 + (1 – 2k) x + (k2 –
(D)  more than two solutions k – 2) = 0 is given by
51. If x, y ∈ [0, 10], then the number of solutions (x, y) of (A) 2 < k < 5 (B)  k<2
(C) 2 < k < 3 (D)  k>5
the inequation 3sec x − 1 9 y 2 − 6 y + 2 ≤ 1 is
2

(A) 2 (B) 4 (C) 6 (D) infinite 60. The number of negative integral solutions of x2 · 2x + 1
+ 2|x – 3| + 2 = x2 · 2(|x – 3| + 4) + 2x – 1 is
52. The equation (x – n)m + (x – n2)m + (x – n3)m + … +
(x – nm)m = 0 (m is odd positive integer), has (A) 4 (B) 2 (C) 1 (D) 0
(A)  all real roots 61. If a and b (a < b ), are the roots of the equation x2 +
(B)  one real and (n – 1) imaginary roots bx + c = 0, where c < 0 < b, then
(C)  one real and (m – 1) imaginary roots (A) 0 < a < b (B)  a < 0 < b < |a |
(D)  no real root (C) a < b < 0 (D)  a < 0 < |a | < b
53. If f (x) = x – [x], x (≠0) ∈ R, where [x] is the greatest 62. If the ratio of the roots of x2 + bx + c = 0 and x2 + qx +
integer less than or equal to x, then the number of solu- r = 0 be the same, then
⎛ 1⎞ (A) r2c = b2q (B)  r2b = c2q
tions of f (x) + f ⎜ ⎟ = 1 are
⎝ x⎠ (C) rb2 = cq2 (D)  rc2 = bq2
(A) 0 (B) 1 (C) infinite (D) 2 63. The number of solutions of |[x] – 2x| = 4, where [x] is
2
54. If x – (a + b + c)x + (ab + bc + ca) = 0 has imaginary the greatest integer ≤ x, is
roots, where a, b, c ∈ R+, then a , b , c (A) 2 (B) 4 (C) 1 (D) infinite
(A)  can be the sides of a triangle 64. If a, b are the roots of the equation x2 + px + q = 0 then
(B)  cannot be the sides of a triangle a
is a root of the equation
(C)  nothing can be said b
(D)  None of these (A) px2 + (2q – p2) x + p = 0
55. If x1, x2, x3, …, xn are the roots of the equation xn + ax + (B) qx2 + (p2 – 2q) x + q = 0
b = 0, then the value of (x1 – x2)(x1 – x3)(x1 – x4) … (C) qx2 + (2q – p2) x + q = 0
(x1 – xn) is equal to (D)  None of these

Objective_Maths_JEE Main 2017_Ch 4.indd 21 01/01/2008 03:36:58


4.22  Chapter 4

65. If ax2 + bx + c = 0, a ≠ 0, a, b, c ∈ R has distinct real 75. The solution set of (x)2 + (x + 1)2 = 25, where (x) is the
roots in (1, 2) then a and 5a + 2b + c have least integer greater than or equal to x, is
(A)  same sign (B)  opposite sign (A)  (2, 4)
(C)  not determined (D)  None of these (B)  (–5, 4] ∪ (2, 3]
(C)  [–4, –3) ∪ [3, 4)
66. If a < 0, the positive root of the equation x2 – 2a
(D)  None of these
|x – a| – 3a2 = 0 is
(A) a ( −1 − 6 ) (B)  a ( −1 + 6 ) 76. Let S denote the set of all values of S for which the
(C) a (1 − 2 ) (D)  None of these equation 2x2 – 2 (2a + 1)x + a (a + 1) = 0 has one root
less than a and other root greater than a, then S equals
67. If px2 + qx + r = 0 has no real roots and p, q, r are real
(A)  (0, 1) (B)  (–1, 0)
such that p + r > 0, then
(A) p – q + r ≤ 0 (B)  p + r ≥ q ⎛ 1⎞
(C)  ⎜ 0, ⎟ (D)  None of these
(C) p + r = q (D)  None of these ⎝ 2⎠

68. Given lx2 – mx + 5 = 0 does not have two distinct real 77. Solution of 2x + 2| x | ≥ 2 2 is
roots, the minimum value of 5l + m is (A) (–∞, log2 ( 2 + 1)
(A) 5 (B) –5 (C) 1 (D) –1 (B)  (0, 8)
69. If 1 lies between the roots of 3x2 – 3sin q – 2cos2 q = 0 ⎛1 ⎞
(C)  ⎜ , log 2 ( 2 − 1)⎟
then ⎝2 ⎠
−1 1 −1 ⎡1 ⎞
(A)  < sin q < (B)  < sin q < 0 (D) (–∞, log2 ( 2 − 1) ] ∪ ⎢ , ∞⎟
2 2 2 ⎣2 ⎠
1 78. If f (x) = x2 + 2bx + 2c2 and g(x) = – x2 – 2cx + b2 such
(C)  < sin q < 1 (D)  None of these
2 that min. f (x) > max. g(x), then the relation between
b and c is
70. If a, b are the roots of the equation 375x2 – 25x – 2 = 0
n (A) |c| < |b| 2 (B) 0 < c < b 2
and Sn = a n + b n, then Lt
n →∞
∑ Sr is (C) |c| < |b| 2 (D) |c| > |b| 2
r =1
7 1
(A)  (B)  79. If the roots of the equation x2 – 2ax + a2 + a – 3 = 0 are
12 12 real less than 3, then:
35 (A) a < 2 (B)  2≤a≤3
(C)  (D)  None of these
12 (C) 3 < a ≤ 4 (D)  a > 4
71. If ax2 + bx + 6 = 0 does not have two distinct real roots x +1 ( x + 1) 2
80. The solution set of + |x + 1| = is
a ∈ R, b ∈ R, then the least value of 3a + b is x x
(A) 4 (B) –1 (C) 1 (D) –2
(A) {x | x ≥ 0}
72. If the ratio of the roots of lx2 + mx + v = 0 is equal to the (B) {x | x > 0} ∪ {–1}
ratio of the roots of x2 + x + 1 = 0, then l, m, v are in (C)  {– 1, 1}
(A)  A.P. (B)  G.P. (D) {x | x ≥ 1 or x ≤ –1}
(C)  H.P. (D)  None of these
81. If a, b are the roots of the equation ax2 + bx + c = 0,
73. If c < a < b < d, then roots of the equation bx + (1 – b2
(a ≠ 0) and a + d, b + d are the roots of Ax2 + Bx +
(c + d)) x + bcd – a = 0 C = 0, (A ≠ 0) for some constant d, then
(A)  are real and one lies between c and a b 2 − 4 ac B 2 − 4 AC
(B)  real and distinct in which one lies between a and b (A)  =
a2 A2
(C)  real and distinct in which one lies between c and d 2 2
(D)  roots are not real b − 2ac B − 2 AC
(B)  =
2 a2 A2
74. If the roots of x + ax + b = 0 are c and d then roots of
x2 + (2c + a) x + c2 + ac + b = 0 are b 2 − 8ac B 2 − 8 AC
(C)  =
(A) 1, d – c (B)  0, d – c a2 A2
(C) 1, c – d (D)  None of these (D)  None of these

Objective_Maths_JEE Main 2017_Ch 4.indd 22 01/01/2008 03:37:02


Quadratic Equations and Expressions  4.23

82. Let a, b, c be real, if ax2 + bx + c = 0 has two real roots 90. If ax2 + 2bx + c = 0 and a1x2 + 2b1x + c1 = 0 have a
c b a b c
a and b, where a < – 1 and b > 1 then 1 + + is common root and , , are in A.P., then a1, b1,
a a c are in a1 b1 c1
1
(A) < 0 (B)  >0
(C) ≤ 0 (D)  None of these. (A)  A.P. (B)  G.P.
(C)  H.P. (D)  None of these
83. If a, b, c are in G.P., then the equations ax2 + 2bx + c =
d e 91. If x is real, then the minimum value of
0 and dx2 + 2ex + f = 0 have a common root if , , ( a + x ) (b + x )
f a b (x > – c), for a > c, b > c is
are in (c + x )
c
(A) H.P. (B) G.P. (A) ( a − b + c − b ) 2
(C)  A.P. (D)  None of these (B) ( a − c + b − c ) 2
84. If the equations x2 + abx + c = 0 and x2 + acx + b = 0 (C) ( a − c − b − c ) 2
have a common root, then their other roots satisfy the (D)  None of these
equation
(A) x2 + a (b + c) x + a2bc = 0 92. If the ratio of the roots of a1x2 + b1x + c1 = 0 be equal
a
(B) x2 – a (b + c) x + a2bc = 0 to the ratio of the roots of a2x2 + b2x + c2 = 0, then 1 ,
b1 c1 a2
(C) x2 – a (b + c) x – a2bc = 0 , are in
(D)  None of these b2 c2
(A)  A.P. (B)  G.P.
85. If (ax2 + bx + c) y + a'x2 + b'x + c' = 0, then the condi-
(C)  H.P. (D)  None of these
tion that x may be a rational function of y is
(A) (ac' – a' c)2 = (ab' – a' b) (bc' – b'c) 93. If a, b be the roots of the equation x2 – px + q = 0 and
(B) (ab' – a ' b)2 = (ac' – a ' c) (bc' – b'c) a > 0, b > 0, then the value of a1/4 + b1/4 is

( ) , where k is equal to
(C) (bc' – b ' c)2 = (ab' – a ' b) (ac' – a'c) k
(D)  None of these p + 6 q + 4 q1/ 4 p+2 q

86. If n and r are positive integers such that 0 < r < n, then 1 1 1
(A) 1 (B)  (C)  (D) 
the roots of the quadratic equation nCr–1 x2 + 2 ⋅ nCr x + 2 3 4
n
Cr+1 = 0 are 2
94. If a, b are the roots of the equation x + px + 1 = 0 and
(A)  real and distinct c, d are the roots of the equation x2 + qx + 1 = 0, then
(B) rational (a – c) (b – c) (a + d) (b + d) =
(C)  rational but not integer (A) p2 – q2 (B)  q2 – p2
(D) imaginary (C) p2 + q2 (D)  2 (p2 – q2)
87. If the equations x2 – px + q = 0 and x2 – ax + b = 0 95. If q ≠ 0 and the equation x3 + px2 + q = 0 has a root of
have a common root and the other root of the second multiplicity 2, then p and q are connected by
equation is the reciprocal of the other root of the first,
(A) p2 + 2q = 0
then (q – b)2 =
(B) p2 – 2q = 0
(A) aq (p – b)2 (B)  bq (p – a)2 (C) 4p3 + 27q + 1 = 0
2
(C) bq (p – b) (D)  None of these (D) 4p3 + 27q = 0
88. If the two equations ax2 + bx + c = 0 and 2x2 – 3x + 96. If the roots of the equation ax2 + bx + c = 0, are of the
4 = 0 have a common root, then a a +1
(A) 6a = 4b = –3c (B) 3a = –4b = 3c form and , then the value of (a + b + c)2 is
a −1 a
(C) 6a = –4b = 3c (D)  None of these (A) b2 – 2ac (B) 2b2 – ac
2
89. If a, b, c are rational and ax2 + bx + c = 0 and 3x2 + (C) b – 4ac (D) 4b2 – 2ac
x – 5 = 0 have a common root, then 3a + b + 2c = 97. If the sum of the roots of the quadratic equation ax2 +
(A) 0 (B) 1 bx + c = 0 is equal to the sum of the squares of their
(C)  2 (D)  None of these a b c
reciprocals, then , and are in
c a b

Objective_Maths_JEE Main 2017_Ch 4.indd 23 01/01/2008 03:37:05


4.24  Chapter 4

(A)  arithmetic progression 105. If b > a, then the equation (x – a) (x – b) –1 = 0 has


(B)  geometric progression (A)  both roots in (–∞, a)
(C)  harmonic progression (B)  one root in (–∞, a) and other in (b, ∞)
(D)  arithmetico-geometric progression (C)  both roots in (b, ∞)
98. If both the roots of the quadratic equation x2 – 2kx + (D)  both roots in [a, b]
k2 + k – 5 = 0 are less than 5, then k lies in the interval 106. The quadratic equation
(A) (–∞, 4) (B)  [4, 5] ( x + b)( x + c) ( x + c)( x + a) ( x + a)( x + b)
+ + =1
(C) (5, 6] (D) (6, ∞) (b − a)(c − a) (c − b)( a − b) ( a − c)(b − c)
99. If for real number a, the equation (a– 2) (x– [x])2 + has
2  (x  – [x]) + a2 = 0 (where [x] denotes the greatest (A)  two real and distinct roots
integer ≤ x) has no integral solution and has exactly (B)  imaginary roots
one solution in (2, 3), then a lies in the interval (C)  equal roots
(A)  (–1, 2) (B)  (0, 1) (D)  infinite roots
(C)  (–1, 0) (D)  (2, 3) 107. The equation ax4 – 2x2 – (a – 1) = 0 will have real and
100. Let a, b, c be distinct positive numbers such that unequal roots if
each of the quadratics ax2 + bx + c, bx2 + cx + a (A) a < 0, a ≠ 1 (B)  a > 0, a ≠1
and cx2 + ax  + b is non-negative for all x ∈ R. If (C) 0 < a < 1 (D)  None of these
a2 + b2 + c2
R= , then 108. If the equation x2 + [a2 – 5a + b + 4] x + b = 0 has
ab + bc + ca
roots –5 and 1, where [a] denotes the greatest integer
(A) 1 ≤ R < 4 (B)  1 < R ≤ 4
less than or equal to a, then the set of values of a is
(C) 1 ≤ R ≤ 4 (D)  1 < R < 4
⎛ 5−3 5 5 + 3 5⎞
101. The set of values of a for which the equation (x2 + x (A) ⎜ ,
⎝ 2 2 ⎟⎠
+ 2)2 – (a – 3) (x2 +x + 2) (x2 + x + 1) + (a – 4) (x2 +
x + 1)2 = 0 has at least one real root is ⎛ 5 + 3 5⎞
(B) ⎜ 0,
⎛ 19 ⎞ ⎡ 19 ⎤ ⎝ 2 ⎟⎠
(A) ⎜ 5, ⎟ (B) 
⎢5, 3 ⎥
⎝ 3⎠ ⎣ ⎦
⎛ 5 − 3 5 ⎤ ⎡5 + 3 5 ⎞
⎡ 19 ⎞ ⎛ 19 ⎤ (C) ⎜ −1, ⎥∪⎢ , 6⎟
(C) 5
⎢ , 3 ⎟⎠ (D) 
⎜⎝ 5, ⎥ ⎝ 2 ⎦ ⎣ 2 ⎠
⎣ 3⎦
(D)  None of these
102. If all real values of x obtained from the equation
109. Let a1, b be the roots of the equation x2 – ax + p = 0
4x – (a – 3)2x + a – 4 = 0 are non-positive, then a
and g, d be the roots of the equation x2 – bx + q = 0.
belongs to
If a, b, √, d are in increasing G.P., then the value of
(A)  [4, 5] (B)  (4, 5] q+ p
(C)  [ 4, 5) (D)  (4, 5) is equal to
q− p
103. Let f (x) = x2 + ax + b be a quadratic polynomial, b2 − a2 b2 + a2
where a and b are integers. If for a given integer n, (A)  2 (B) 
f (n) f (n + 1) = f (m) for some integer m, then the value b + a2 b2 − a2
of m is b+a b−a
(C)  (D) 
(A) n (n + 1) + an + b (B)  n (n + 1) + a + bn b−a b+a
(C) n (n + 1) + a + b (D)  None of these 1
2
110. If tn denotes the nth term of an A.P. and tp = and tq
x + nx − 2 q
104. If for any real x, we have –1 ≤ 2 ≤ 2 , then n 1
x − 3x + 4 = , then which of the following is necessarily a root
belongs to p
(A) [− 40 + 6, −1] of the equation (p + 2q – 3r) x2 + (q + 2r – 3p) x + (r
+ 2p – 3q) = 0
[ − 40 + 6, 40 − 6]
(B)  (A) tp (B)  tq
(C) [−1, 40 − 6]
(C) tpq (D)  tp + q
(D)  None of these

Objective_Maths_JEE Main 2017_Ch 4.indd 24 01/01/2008 03:37:08


Quadratic Equations and Expressions  4.25

111. If the roots of the equation 4x2 + 4ax + b = 0 are real (A) |b| ≤ 4a (B)  |b| ≥ 4a
and differ at most by a, then b lies in (C) |b| = 2 a (D)  None of these
⎛ a2 ⎞ ⎛ a2 2 ⎞ 113. If a, b, c, d are real numbers, then the number of real
(A) ⎜ 0, ⎟ (B)  ⎜ 2 ,a ⎟ roots of the equation (x2 + ax – 3b) (x2 – cx + b) (x2 –
⎝ 2⎠ ⎝ ⎠
dx + 2b) = 0 are
(C) [0, a2] (D) 
(0, a2)
(A) 3 (B) 4
112. The roots of the equation ax2 + bx + c = 0, where (C)  6 (D)  at least 2
a ∈ R+, are two consecutive odd positive integers, then

More than One Option Correct Type

114. If a ≤ 0, then the root of the equation x2 – 2a |x – a| – 121. If a, b are the roots of ax2 + bx + c = 0 and a4, b 4
3a2 = 0 is are the roots of lx2 + mx + n = 0, then the roots of the
(A) (1 − 2 )a (B)  ( − 1 + 6 )a equation a2 l x2 – 4 aclx + 2c2l + a2 m = 0 are
(C) (1 + 2 )a − (1 + 6 )a
(D)  (A) real (B) imaginary
(C)  opposite in sign (D)  equal
2
115. If x – 3x + 2, be one of the factors of the expression
x4 – px2 + q, then 122. If a, b, c are positive rational numbers such that
a > b > c and the quadratic equation (a + b – 2c)x2 +
(A) p = 5 (B)  q=4
(b + c –2a) x + (c + a – 2b) = 0 has a root in the inter-
(C) p = 4 (D)  q=5
val (–1, 0,) then
p a b (A) c + a < 2b
116. If c ≠ 0 and the equation
= + has two
2x x+c x−c (B)  both roots of the given equation are rational
equal roots, then p can be (C) the equation ax2 + 2bx + c = 0 has both negative
(A) ( a − b ) 2 (B)  ( a + b )2 real roots
(C) a + b (D)  a–b (D) the equation cx2 + 2ax + b = 0 has both negative
real roots
117. For a > 0, the roots of the equation logax a + logx a2 +
log a x a3 = 0, are given by
2 123. If the equation x2 + a2x + b2 = 0 has two roots each of
which exceeds a number c, then
(A) a1/2 (B) 
a–1/2 (C) 
a4/3 (D) 
a–4/3
(A) a4 > 4b2
1 18. Solution of |x2 + 4x + 3| + 2x + 5 = 0 is
(B) c2 + a2c + b2 > 0
(A) 4 (B) –4
(C)  –1 –  3 (D) 1 + 3 a2
(C) − >c
2
119. If the roots of 10x3 – cx2 – 54x – 27 = 0 are in har-
monic progression, then the roots are a2
(D) – <c
−3 −3 1 2
(A)  (B)  (C) 3 (D)  124. If b2 ≥ 4ac for the equation ax4 + bx2 + c = 0, then all
5 2 3
the roots of the equation will be real if
120. If the equation x2 + 9y2 – 4x + 3 = 0 is satisfied for real
(A) b > 0, a < 0, c > 0
values of x and y, then
(B) b < 0, a > 0, c > 0
(A) 1 ≤ x ≤ 3 (C) b < 0, a > 0, c > 0
(B) 2 ≤ x ≤ 3 (D) b > 0, a < 0, c < 0
1 1
(C) − ≤ y ≤ 125. If the equation x2 + (a – b) x – a – b + 1 = 0, where a,
3 3
b ∈R, has unequal real roots for all b ∈ R, then
1
(D)  ≤ x ≤ 1. (A) a < 0 (B)  a>0
3
(C) a >1 (D)  a<1

Objective_Maths_JEE Main 2017_Ch 4.indd 25 01/01/2008 03:37:10


4.26  Chapter 4

Passage Based Questions


Passage 1 129. The value of a for which the equation (1– a2) x2 +
Let a1x2 + b1x + c1 = 0 and a2x2 + b2x + c2 = 0 be two 2ax – 1 = 0 has roots belonging to (0,1) is
­quadratic equations such that a1a2 ≠ 0 and a1b2 ≠ a2b1. 1+ 5
(A) a > a>2
(B) 
If the two equations have a common root a, then 2
a1a2 + b1a + c1 = 0 and a2a2 + b2a + c2 = 0. 1+ 5
(C)  < a < 2 (D) a > 2
Eliminating a using cross-multiplication method gives the 2
condition for a common root. Solving the two equations 130. The values of a for which each one of the roots of
simultaneously, the common root can be obtained. x2 –4ax + 2a2 – 3a + 5 = 0 is greater than 2, are
If the two equations have both roots common, then (A) a ∈ (1, ∞) (B)  a=1
a1 b1 c1 (C) a ∈ (– ∞, 1) (D)  a ∈ (9/2, ∞)
= = .
a2 b2 c2
Passage 3
126. If a, b, c ∈ R and the equations ax2 + bx + c = 0 and
The maximum number of positive real roots of a polyno-
x3 + 3x2 + 3x + 2 = 0 have two roots common, then
mial equation f (x) = 0 is the number of changes of signs
(A) a = b ≠ c (B)  a=b=–c from positive to negative and negative to positive in f (x).
(C) a = b = c (D)  None of these For example, consider the equation f (x) = x3 + 6x2 + 11x –
6 = 0. The signs of the various terms are:
Passage 2
Let k be any point such that k ∈ R and a, b are the roots of +++–
the quadratic equation f (x) = ax2 + bx + c = 0. If k lies out- Clearly, there is only one change of sign in the given
side and is less than both the roots then the equation must expression. So, the given equation has at most one positive
have real and distinct roots and the sign of f (k) is same as real root.
the sign of ‘a’. Also, k is less than the x-coordinate of the The maximum number of negative real roots of a pol-
vertex of the parabola y = ax2 + bx + c. ynomial equation f (x) = 0 is the number of changes of signs
If k lies between both the roots, then the sign of f (k) from positive to negative and negative to positive in f (–x).
is opposite to the sign of ‘a’. For example, for the equation f (x) = x4 + x3 + x2 – x – 1 =
If k lies outside and is greater than both the roots, 0, there are three changes of signs in f (–x). So, the given
then the sign of f (k) is same as the sign of ‘a’. Also, k is equation has atmost three negative real roots.
greater than the x-coordinate of the vertex of the parabola If f (x) and f (–x) do not have any changes of signs, the
y = ax2 + bx + c. equation f (x) = 0 has no real roots.
If both the roots of the equation lie between two real Now, consider the polynomial
numbers k1 and k2, then equation must have real and dis-
tinct roots and the sign of f (k1) and f (k2) is same as the sign Pn (x) = 1 + 2x + 3x2 + … + (n + 1) xn.
of a. Also, the x-coordinate of the vertex of the parabola 131. If n is even, the number of real roots of Pn(x) is
y = ax2 +bx + c lies between k1 and k2.
(A)  0 (B)  n
127. The values of ‘a’ for which the roots of the equation (C)  1 (D)  None of these
(a + 1) x2 – 3ax + 4a = 0 (a ≠ – 1) to be greater than
132. If n is odd, the number of real roots of Pn (x) is
unity are
(A)  0 (B)  n
−16 (C)  1 (D)  None of these
(A)  ≤ a < −1 (B) –2 < a < – 1
7
(C) 0 < a < 1 (D)  None of these 133. If a1 < a2 < a3 < a4 < a5 < a6, then the equation (x – a1)
(x – a3) (x – a5) + 3 (x – a2) (x – a4) (x – a6) = 0 has
128. The values of ‘a’ so that 6 lies between the roots of (A)  three real roots
the equation x2 + 2 (a – 3) x + 9 = 0, are (B)  a root in (–∞, a1)
3 3 (C)  no real root in (a1, a2)
(A) a > − a<−
(B) 
4 4 (D)  no real root in (a5, a6)
3 3
(C) a > (D)  a<
4 4

Objective_Maths_JEE Main 2017_Ch 4.indd 26 01/01/2008 03:37:12


Quadratic Equations and Expressions  4.27

Match the Column Type


134. 135.
Column-I Column-II Column-I Column-II
2 2
  (I) If the roots of the equation (a (A) A.P.   (I) If a, b be the roots of x + px – (A) 0
+ b2)x2 + 2(bc + ad)x + (c2 + q = 0 and g, d be the roots of
d2) = 0 are real, then a2, bd, x2 + px + r = 0, q + r ≠ 0, then
c2 are in (a − g ) (a − d )
=
  (II) If a(b – c)x2 + b(c – a)x + c(a (B) H.P. (b − g ) (b − d )
– b) = 0 has equal roots, then
a, b, c are in   (II) The number of solutions of the (A) 2
equation sin(ex) = 5x + 5–x is
  (III) If the sum of the roots of the (C) G.P.
equation ax2 + bx + c = 0   (III) If x = 2 + 22/3 + 21/3, then the (A) 6
is equal to the sum of the value of x3 – 6x2 + 6x is
reciprocals of their squares, (IV) The minimum value of (A) 1
then bc2, ca2 and ab2 are in 1 5
| x | + | x + | + | x − 3 | + | x − | is
(IV) If a, b, c, d and p are distinct (D) A.G.P. 2 2
real numbers such that (a2 +
b2 + c2)p2 – 2(ab + bc + cd)p
+ (b2 + c2 + d2) ≤ 0 then a, b,
c and d are in

Assertion-Reason Type
Instructions: In the following questions an Assertion (A) is 137. Assertion: If the equation x2 + 2 (k + 1)x + 9k – 5 = 0
given followed by a Reason (R). Mark your responses from has only negative roots, then k ≤ 6
the following options: Reason: The equation f (x) = 0 will have both roots
(A)  Assertion(A) is True and Reason(R) is negative if and only if
True; Reason(R) is a correct explanation for  (i) Discriminant ≥ 0,
Assertion(A) (ii)  Sum of roots < 0,
(B)  Assertion(A) is True, Reason(R) is True; (iii)  Product of roots > 0
Reason(R) is not a correct explanation for
Assertion(A) 138. Assertion: If the equations x2 + bx + ca = 0 and x2 +
(C)  Assertion(A) is True, Reason(R) is False cx + ab = 0 have a common root, then their other roots
(D)  Assertion(A) is False, Reason(R) is True will satisfy the equation x2 + ax + bc = 0
Reason: If the equations x2 + bx + ca = 0 and x2 +
136. Assertion: If the roots of the equations x2 – bx + c = 0 cx + ab = 0 have a common root, then a + b + c = 0
and x2 – cx + b = 0 differ by the same quantity, then
b + c is equal to –4.
Reason: If a, b are the roots of the equation Ax2 +
B 2 − 4 AC
Bx + C = 0, then a − b =
A

Previous Year’s Questions

l39. If α ≠ β with a2 = 5α − 3 and β2 = 5β − 3, then the (A) 3x2 + 19x + 3 = 0 (B)  3x2− 19x + 3 = 0
equation having α/β and β/α as its roots, is [2002] (C) 3x2− 19x− 3 = 0 (D)  x2− 16x + 1 = 0

Objective_Maths_JEE Main 2017_Ch 4.indd 27 01/01/2008 03:37:13


4.28  Chapter 4

139. The number of real roots of 32 x − 7 x + 7 = 9 is


2
148. If both the roots of the quadratic equation x2− 2kx +
 [2002] k2 + k− 5 = 0 are less than 5, then k lies in the interval
(A) Zero (B) 2  [2005]
(C) 1 (D) 4 (A) (5, 6] (B) (6, ∞)
(C) (−∞, 4) (D)  [4, 5]
140. If the sum of the roots of the quadratic equation
ax2 + bx + c = 0 is equal to the sum of the squares of 149. All the values of m for which both roots of the equa-
a b c tions x2− 2mx + m2− 1 = 0 are greater than –2 but less
their reciprocals, then , and , are in [2003] than 4, lie in the interval [2006]
c a b
(A)  arithmetic progression. (A) –2 < m < 0 (B)  m > 3
(B)  geometric progression. (C) –1 < m < 3 (D)  1 < m < 4
(C)  harmonic progression. 3 x 2 + 9 x + 17
(D) arithmetic-geometric-progression. 150. If x is real, the maximum value of
is
3x 2 + 9 x + 7
141. The number of real solutions of the equation x2− 3 [2006]
| x | + 2 = 0 is [2003] (A) 1/4 (B) 41
(A) 2 (B) 4 (C) 1 (D) 3 (C) 1 (D) 17/7
142. The value of ‘a’ for which one root of the quadratic 151. If the difference between the roots of the equation
equation (a2−5a + 3) x2 + (3a – 1) x + 2 = 0 is twice x2 + ax + 1 = 0 is less than 5 , then the set of possi-
as large as the other, is [2003] ble values of a is [2007]
2 2 (A) (−3, 3) (B)  (−3, ∞)
(A)  (B)  −
3 3 (C) (3, ∞) (D)  (−∞, −3)
1 1 152. The quadratic equations x2− 6x + a = 0 and x2− cx +
(C)  (D)  −
3 3 6 = 0 have one root in common. The other roots of
143. If (1 − p) is a root of quadratic equation x2 + px + the first and second equations are integers in the ratio
(1 − p) = 0, then its roots are [2004] 4 : 3. Then the common root is [2008]
(A)  0, 1 (B)  –1, 2 (A) 1 (B) 4
(C) 0, –1 (D)  –1, 1 (C) 3 (D) 2

144. If one root of the equation x2 + px + 12 = 0 is 4, while 153. If the roots of the equation bx2 + cx + a = 0 be imagi-
the equation x2 + px + q = 0 has equal roots, then the nary, then for all real values of x, the expression 3b2x2
value of ‘q’ is [2004] + 6bcx + 2c2 is [2009]
49 (A)  greater than 4ab
(A)  (B)  4 (B)  less than 4ab
4
(C) 3 (D) 12 (C)  greater than − 4ab
(D)  less than − 4ab
145. If 2a + 3b + 6c = 0, then at least one root of the equa-
tion ax2 + bx + c = 0 lies in the interval [2004] 154. If a and β are the roots of the equation x2−x + 1 = 0,
then the value of α2009 + β2009 = [2010]
(A)  (0, 1) (B)  (1, 2)
(C)  (2, 3) (D)  (1, 3) (A)  −1 (B)  1
(C)  2 (D)  −2
146. The values of α for which the sum of the squares of
the roots of the equation x2− (a − 2)x − a − 1 = 0 155. The equation esin x –e–sin x – 4 = 0, for x real, has
assume the least value is [2005]  [2012]
(A) 1 (B) 0 (A)  infinite number of roots
(C) 3 (D) 2 (B)  no roots
(C)  exactly one root
147. If roots of the equation x2− bx + c = 0 be two (D)  exactly four roots
­consectutive integers, then b2− 4c equals [2005]
156. The real number k for which the equation,
(A)  −2 (B)  3
2 x 3 + 3 x + k = 0 has two distinct real roots in [0, 1]
(C) 2 (D) 1
 [2013]

Objective_Maths_JEE Main 2017_Ch 4.indd 28 01/01/2008 03:37:14


Quadratic Equations and Expressions  4.29

(A)  lies between 2 and 3 ( 1, 0) ∪ (0,1)


(A) − ( , 2)
(B) 1
(B)  lies between −1 and 0
(C)  does not exist ( 2, −1) (D) 
(C) − ( −∞, −2) ∪ ( 2, ∞ )
(D)  lies between 1 and 2
159. Let a and b be the roots of equation x 2 − 6 x − 2 = 0.
157. If the equations x2 + 2x + 3 = 0 and
2 a − 2a8
ax + bx + c = 0, a, b, c ∈ R have a common root, then If an = a n − b n, for n ≥ 1,then the value of 10
a : b : c is [2013] 2a9
is equal to [2015]
(A)  3 : 2 : 1 (B)  1 : 3 : 2
(C)  3 : 1 : 2 (D)  1 : 2 : 3 (A) − 6 (B)  3
(C) − 3 (D)  6
158. If a ∈ R and the equation −3 ( x − [ x ]) + 2 ( x − [ x ]) + a 2 = 0
2

3 ( x − [ x ]) + 2 ( x − [ x ]) + a 2 = 0 (where [ x ] denotes the greatest integer ≤ x )


2

has no integral solution, then all possible values of a


lie in the interval [2014]

Answer keys

Single Option Correct Type


1. (C) 2. (A) 3. (C) 4. (D) 5. (B) 6. (A) 7. (B) 8. (D) 9. (B) 10. (C)
11. (A) 12. (B) 13.  (A, B, C) 14. (A) 15. (A) 16. (A) 17. (A) 18. (C) 19. (B)
20. (A) 21. (C) 22. (B) 23. (D) 24. (D) 25. (C) 26. (B) 27. (B) 28. (A) 29. (C)
30. (A) 31. (D) 32. (B) 33. (C) 34. (B) 35. (C) 36. (C) 37. (B) 38. (D) 39. (A)
40. (C) 41. (B) 42. (B) 43. (C) 44. (B) 45. (C) 46. (B) 47. (A) 48. (D) 49. (C)
50. (D) 51. (B) 52. (C) 53. (C) 54. (A) 55. (A) 56. (A) 57. (B) 58. (D) 59. (A)
60. (D) 61. (B) 62. (C) 63. (B) 64. (C) 65. (A) 66. (C) 67. (B) 68. (D) 69. (C)
70. (B) 71. (D) 72. (B) 73. (C) 74. (B) 75. (B) 76. (D) 77. (D) 78. (D) 79. (A)
80. (B) 81. (A) 82. (A) 83. (C) 84. (B) 85. (A) 86. (A) 87. (B) 88. (C) 89. (A)
90. (B) 91. (B) 92. (B) 93. (D) 94. (B) 95. (D) 96. (C) 97. (C) 98. (A) 99. (C)
100. (D) 101. (D) 102. (B) 103. (A) 104. (C) 105. (B) 106. (D) 107. (C) 108. (C) 109. (B)
110. (C) 111. (C) 112. (B) 113. (D)

More than One Option Correct Type


1 14.  (A) and (B) 115.  (A) and (B) 116.  (A) and (B) 117.  (B) and (D) 118.  (B) and (C)
119.  (A), (B) and (C) 120.  (A) and (C) 121.  (A) and (C) 122.  (A), (B), (C) and (D)
123.  (A), (B) and (C) 124.  (C) and (D) 125.  (B) and (C)

Passage Based Questions


126. (C) 127. (A) 128. (B) 129. (B) 130. (D) 131. (A) 132. (C) 133. (A)

Match the Column Type


1 34. (I) → (C); (II) → (B); (III) → (A); (IV) → (C)
135. (I) → (D); (II) → (A); (III) → (B); (IV) → (C)

Objective_Maths_JEE Main 2017_Ch 4.indd 29 01/01/2008 03:37:16


4.30  Chapter 4

Assertion-Reason Type
136. (A) 137. (D) 138. (A)

Previous Year’s Questions


139. (A) 140. (B) 141. (C) 142. (B) 143. (A) 144. (C) 145. (A) 146. (A) 147. (D) 148. (C)
149. (C) 150. (C) 151. (C) 152. (A) 153. (D) 154. (C) 155. (B) 156. (B) 157. (C) 158. (D)
159. (A) 160. (B)

Objective_Maths_JEE Main 2017_Ch 4.indd 30 01/01/2008 03:37:16


Quadratic Equations and Expressions  4.31

Hints and Solutions

Single Option Correct Type


1. We have, 2x + 2 ⋅ 33x /(x – 1) = 9 = 32 6. The discriminant = 16q2 – 4 (2q2 – r)
3x

⇒ (x + 2) log 2 + log 3 = 2 log 3 = 8q2 + 4r = 8q2 + (α4 + β4) > 0
x −1
∴ roots are real.
⎛ 3x ⎞
⇒ (x + 2) log 2 + ⎜ − 2⎟ log 3 = 0 The correct option is (A)
⎝ x −1 ⎠
7. We have, (a2 + b2 + c2) p2 – 2 (ab + bc + cd) p
⎛ 1 ⎞
⇒ (x + 2) ⎜ log 2 +
log 3⎟ = 0 + (b2 + c2 + d2) ≤ 0
⎝ x −1 ⎠
⇒ (ap – b)2 + (bp – c)2 + (cp – d)2 ≤ 0
log 3
⇒ x = –2 or x = 1 –
. ⇒ (ap – b)2 + (bp – c)2 + (cp – d)2 = 0
log 2
(a, b, c, d, p ∈ R)
The correct option is (C)
⇒ ap – b = 0, bp – c = 0, cp – d = 0
2. Since a, b, c are all +ve
b c d
∴ ax2 + b | x | + c > 0 for all real x ⇒
= = =p
a b c
∴ ax2 + b | x | + c ≠ 0 for any real x
⇒ a, b, c, d are in G.P.
∴ no real solution is possible.
The correct option is (B)
The correct option is (A)
8. The required a satisfies the inequality
3. Since x2 – x + 1 = 0
2a2 – 2(2a + 1)a + a(a + 1) < 0
∴ (x – 1) (x2 – x + 1) = 0
⇒ a(a + 1) > 0 ⇒ a ∈ (–∞, –1) ∪ (0, ∞)
⇒ x3 – 1 = 0
The correct option is (D)
⇒ x3 = 1,
9. (a + c)2 + 4b2 – 4b (a + c) ≤ 0
∴ x3n = 1
⇒ (a – 2b + c)2 ≤ 0
The correct option is (C)
⇒ a – 2b + c = 0
4. The given equation can be written as
⇒ 2b = a + c
⎡ 1⎤
2 x +1 ⎢ x 2 − ⎥ = 2| x − 3 | ⋅ 4 [4 x 2 − 1] ⇒ a, b, c are in A.P.
⎣ 4⎦
The correct option is (B)
⎡ 2 1⎤ 10. Let α, β be the roots of equation x2 + bx + c = 0 and α ′, β ′

= 16 · 2|x – 3| ⎢⎣ x − 4 ⎥⎦ be the roots of the x2 + qx + r = 0. Then,

⇒ 2x – 3 = 2|x – 3| α + β = – b; aβ = c, α ′ + β ′ = – q, α ′ β ′ = r.
1


[∵ x2 = does not give negative integral value] a a′ a+b a ′ + b′
4 It is given that = ⇒ =
b b′ a −b a ′ − b′
⇒ x – 3 = ± (x – 3)
(a + b ) 2 (a ′ + b ′ ) 2
⇒ either x – 3 = x – 3 or x – 3 = – x + 3 ⇒ =
⇒ 2x = 6 or x = 3 (a − b ) 2 (a ′ − b ′ ) 2
∴ Given equation does not give any negative integral b2 q2

= ⇒ b2r = q2c.
solution. b 2 − 4c q2 − 4r
The correct option is (D)
The correct option is (C)
5. We have α + β = – b, aβ = c 16
11 Let 16sin x = y, then 16cos x = 161−sin x =
2 2 2

As c < 0, b > 0, we get y


The given equation becomes
α<0<β 16
y+ = 10 ⇒ y2 – 10y + 16 = 0 or y = 2, 8
Also, β = – b – α < – α = |α | y
Thus, α < 0 < β < |α |. 16sin x = 2 ⇒ 24 sin x = 2(1)
2 2

Now,
The correct option is (B) 2
⇒ 4 sin x = 1

Objective_Maths_JEE Main 2017_Ch 4.indd 31 01/01/2008 03:37:19


4.32  Chapter 4

1 π ⎛ π⎞ 18. Sinceα, β are roots of the equation x2 + px + q = 0


∴ sin x
=± ⇒x=  ⎜⎝Q0 ≤ x < ⎟⎠
2 6 2 ∴ α + β = – p and aβ = q
2

and 16sin x = 8
2
⎛a⎞ a
4 sin 2 x 3 Now q ⎜ ⎟ + ( 2q − p 2 ) + q
⇒ 2
=2 ⎝ b⎠ b
3 π ⎛ π⎞ 1 1

⇒ sin x = ± ⇒x=  ⎜⎝Q 0 ≤ x < ⎟⎠ = 2 [q (α + β)2 – p2ab] = 2 (qp2 – p2q) = 0
2 3 2 b b
The correct option is (A) a
Thus, is a root of the equation
12. The given condition is fulfilled if and only if b
qx2 + (2q – p2) x + q = 0
f (2) = 4 – 2 (p + 1) + p2 + p – 8 < 0
The correct option is (C)
⇒ (p – 3) (p + 2) < 0 ⇒ –2 < p < 3
19. Since f (x) has no real roots, f (x) has same sign for every x
The correct option is (B)
∴ f (0) · f (1) > 0
13. (x3 + 1) + 2x (x + 1) = 0
The correct option is (B)
or (x + 1) [x2 + x + 1] = 0 ⇒ x = – 1, ω, ω 2.
20. Let x1 and x2 be two roots of ax2 + bx + c = 0
Of these x = ω, ω 2 satisfy the equation
1 < x1 < 2 and 1 < x2 < 2
1 + x130 + x1988 = 0
⎛ b c⎞
The correct option is (A), (B) and (C) Now a (5a + 2b + c) = a2 ⎜⎝ 5 + 2 a + a ⎟⎠
14. We have, |x2 – 3x + 2| + |x – 1| = x – 3

= a2(5 + 2(–1) (x1 + x2) + x1x2)
⇒ x ≥ 3 ⇒ x2 – 3x + 2 + x – 1 = x – 3
= a2[(x1 – 2) (x2 – 2) + 1] > 0
⇒ x2 – 3x + 4 = 0
2
Hence a and 5a + 2b + c are of same sign
⎛ 3⎞ 9 The correct option is (A)
⇒ ⎜ x − ⎟ + 4 − = 0
⎝ 2⎠ 4 21. Since a < 0, in case of positive root of the equation x > a
⇒ No solution. ∴ The equation is x2 – 2a (x – a) – 3a2 = 0
The correct option is (A) ⇒ x2 – 2ax – a2 = 0
15. We have, 2a ± 4 a 2 + 4 a 2 2a ± 2 2a
Thus, the roots are =
3 x 2 + 6 x + 7 = 3 ( x + 1) 2 + 4 ≥ 2 2 2
= a(1 + 2 ) or a(1 − 2 )

and 5 x 2 + 10 x + 14 = 5( x + 1) 2 + 9 ≥ 3 ∴ the only positive root possible is a(1 − 2 ) .
∴ L.H.S. ≥ 5 The correct option is (C)
R.H.S. = 4 – 2x – x2 = 5 – (x + 1)2 ≤ 5 22. Let α + iβ, α – iβ be the roots
∴ the equation holds only when r
Then, a 2 + b 2 = >0
L.H.S. = R.H.S. = 5 p
∴ x = – 1. ∴ p, r must be of the same sign.
The correct option is (A) Since p + r > 0
16. Equating the coefficients of similar powers of x, we get ∴ p, r are both positive.
a 2 − 1 = 0 ⇒ a = ±1 ⎫ If q < r, p – q + r > 0
⎪ If q > 0, (p + r)2 – (p – r)2 = 4pr ≥ q2
a − 1 = 0 ⇒ a = 1 ⎬ 
2 ⎪ (∵ roots are non-real)
a − 4 a + 3 = 0 ⇒ a = 1, 3⎭
∴ (p + r)2 ≤ q2 + (p – r)2 ≥ q2
∴ common value of a = 1.
∴ p + r ≥ q
The correct option is (A)
The correct option is (B)
17. The given equation can be written as
23. Let f (x) = lx2 – mx + 5
3x2 – 2x (a + b + c) + bc + ca + ab = 0
Since lx2 – mx + 5 = 0 does not have two distinct real roots,
Discriminant = 4 (a + b + c)2 – 12 (bc + ca + ab) therefore either f (x) ≥ 0 ∀ x ∈ R, or f (x) ≤ 0 ∀ x ∈ R
= 4 (a2 + b2 + c2 – bc – ca – ab) But f (0) = 5 > 0
= 2 [(b – c)2 + (c – a)2 + (a – b)2] ≥ 0 ∴ f (x) ≥ 0 ∀ x ∈ R
Hence, the roots are real. ∴ f (–5) ≥ 0 ⇒ 25l + 5m + 5 ≥ 0 ⇒ 5l + m ≥ –1
The correct option is (C) Hence, minimum value of 5l + m is –1.

Objective_Maths_JEE Main 2017_Ch 4.indd 32 01/01/2008 03:37:23


Quadratic Equations and Expressions  4.33

The correct option is (D) If x = n + k, n ∈ Z, 0 < k < 1, then


24. Let α, β be the roots of the given equation, (x)2 + (x + 1)2 = 25
then α + β = l2 – 5λ + 5 < 1 ⇒ (n + 1)2 + (n + 2)2 = 25
⇒ l2 – 5λ + 4 < 0 ⇒ 2n2 + 6n – 20 = 0
aβ = 2l2 – 3λ – 4 < 1 ⇒ n2 + 3n – 10 = 0
⇒ 2l2 – 3λ – 5 < 0 ⇒ n = 2, –5
∴ (λ – 4) (λ – 1) < 0 or 1 < λ < 4 ∴ x = 2 + k, –5 + k, where 0 < k < 1
5 ∴ x > 2, x > –5
and (2λ – 5) (λ + 1) < 0 or –1 < λ <
2 ∴ Solution set is (–5, –4] ∪ (2, 3]
⎛ 5⎞ ⎛ 5⎞ The correct option is (B)
∴ Required set = ⎜ −1, ⎟ ∩ (1, 4) = ⎜1, ⎟ .
⎝ 2⎠ ⎝ 2⎠
28. log2 (9 – 2x) = 3 – x
The correct option is (D)
⇒ 9 – 2x = 23–x
25. Since coefficient of x2 > 0 and 1 lies between the roots of 8
3x2 – 3sin θ – 2cos2 θ = 0 ⇒ 9 – 2x = x
2
∴ f (1) < 0
⇒ 9 · 2x – 22x = 8
⇒ 3 – 3sin θ – 2cos2θ < 0
⇒ 22x – 9 · 2x + 8 = 0
⇒ 1 + 2(1 – cos2 θ) – 3sin θ < 0
⇒ (2x – 8) (2x – 1) = 0
⇒ 2sin2 θ – 3sin θ + 1 < 0
⇒ x = 3  or  x = 0
⇒ (2sin θ – 1) (sin θ – 1) < 0
But x = 3 is not the solution of original equation,
1
⇒ < sin θ < 1 ∴ x = 0.
2
The correct option is (A)
29. Since ax2 + bx + 6 = 0 does not have two distinct real roots.
Trick(s) for Problem Solving
∴ b2 – 24a ≤ 0
If one root is less than k and other is greatter than k, then Let 3a + b = y
D > 0 and af (k) < 0, where ∴ 3a = y – b
f(x) = ax2 + bx + c, a, b, c ∈ R, a ≠ 0 ∴ b2 – 8(y – b) ≤ 0
i.e., b2 + 8b – 8y ≤ 0

The correct option is (C) Since b is real
n ∴ 64 + 32y ≥ 0 ⇒ y ≥ –2
26. ∑ Sr = (α + β ) + (a 2 + b 2) + … + (a n + b n)
∴ Min. value of y i.e., 3a + b = –2.
r =1

= (α + a 2 + … + a n) + (β + β 2 + … + β n) The correct option is (D)
n
30. Here α + β = –p ⇒ α + β = γ + δ
Lt
n→∞
∑ Sr = (α + a 2 + … + ∞) + (β + β 2 + … + ∞)
r =1 γ + δ = –p
a b

= + Now (α – γ) (α – δ) = α 2 – α(γ + δ) + g δ = α 2 – α (α + β ) + r
1− a 1− b
= –αβ + r = –(–q) + r = q + r
a − ab + b − ab

= By symmetry (β – γ ) (β – δ ) = q + r
1 − (a + b ) + ab
∴ Ratio is 1.
a + b − 2ab

= The correct option is (A)
1 − (a + b ) + ab
x+2 1
25 4 31. − >0
+ x2 + 1 2
375 375 29 1
= = =
25 2 348 12 − x2 − 1 + 2x + 4
1− − ⇒
>0
375 375 2( x 2 + 1)
The correct option is (B) 3 + 2x − x2
27. If x = n ∈ Z, (x)2 + (x + 1)2 = 25 ⇒
>0
2( x 2 + 1)
⇒ n2 + (n + 1)2 = 25
Since denominator is positive
⇒ 2n2 + 2n – 24 = 0 ∴ 3 + 2x – x2 > 0

⇒ n2 + n – 12 = 0 ⇒ –1 < x < 3

⇒ n = 3, – 4 ⇒ x = 0, 1, 2

∴ x = 3, – 4
The correct option is (D)

Objective_Maths_JEE Main 2017_Ch 4.indd 33 01/01/2008 03:37:25


4.34  Chapter 4

32. Let α, β be the roots of lx2 + mx + v = 0 ∴ Discriminant can’t be perfect square.



m v
The correct option is (C)
∴ α + β = – , aβ =
l l loge ( x 2 − 5 x + 6) loge ( x − 4)
2 37. >
m loge 9 loge 3
(a + b ) 2 l 2 m2 ⇒ x2 – 5x + 6 > x2 – 8x + 16

= =
ab v lv 10
l ⇒ 3x – 10 > 0 ⇒ x >  (1)
3
a b m2 2
Also, x – 5x + 6 > 0 ⇒ x > 3 or x < 2  (2)

+ +2=  (1)
b a lv and  x – 4 > 0 ⇒ x > 4  (3)
Let γ, δ be the roots of x2 + x + 1 = 0
Common solution from Eqs (1), (2) and (3) is x > 4
∴ γ + δ = –1, g δ = 1
The correct option is (B)
(g + d ) 2 g d 38. Let f (x) = a2x2 + 2bx + 2c = 0

= 1 ⇒ + + 2 = 1 (2)
gd d g Given: a2a 2 + bα + c = 0
a g and a2β 2 – bβ – c = 0

Since = ,
b d Now,  f (α) = a2a 2 + 2bα + 2c
∴ from Eq. (1) and (2)
= (a2a 2 + bα + c) + (bα + c)
m2 = bα + c = – a2a 2
= 1 ⇒ m2 = lv
lv ⎫
∴ λ, µ, v are in G.P. f (β ) = a2β 2 + 2bβ + 2c ⎬

The correct option is (B) = 3bβ + 3c = 3 (bβ + c)
33. Given equation can be written as = 3a2β 2
bx2 + x – bcx – bdx + bcd – a = 0
Since 0 < α < β
⇒ bx (x – c) – bd (x – c) + x – a = 0 ∴ α, β are real
⇒ b (x – c) (x – d) + (x – a) = 0 ∴ f (α) < 0, f (β) > 0
Let f (x) = b (x – c) (x – d) + (x – a) ∴ f (γ ) = 0 where α < γ < β
f (c) = c – a < 0; f (d) = d – a > 0
Hence, one root of the given equation lies between c and d. Trick(s) for Problem Solving
The correct option is (C)
a a +1 b a a +1 c Let f(x) = 0 be a polynomial equation. Let p and q be two real
34. We have, + = − and ⋅ = numbers, p < q. If f(p) ⋅ f(q) < 0, then the equation f(x) = 0
a −1 a a a −1 a a
has atleast one real roots between p and q.
2a 2 − 1 b c+a
⇒ 2 = − and α =
a −a a c−a
2 The correct option is (D)
⇒ (c + a) + 4ac = –2b(c + a)
39. Let us see the graph of y = x2 – 2 and y = [x]
⇒ (c + a)2 + 2b(c + a) + b2 = b2 – 4ac
⇒ (a + b + c)2 = b2 – 4ac Y
The correct option is (B)
35. Here α + β = p; aβ = 1 ⇒ aβ = g δ
γ + δ = q; g δ = 1
Now, (α – γ ) (β – γ) (α + δ ) (β + δ )
X
= [aβ – γ (α + β ) + g 2] [aβ + δ(α + β ) + d 2] O
= [1 + g p + g 2] [1 – pδ + d 2]
= [(g 2 + 1) + g p] [(d 2 + 1) – pd]
= (–qγ + g p) (–qδ – pδ )
= g δ (q2 – p2) = 1 (q2 – p2)
The correct option is (C) If [x] = –1

36. Let y = [x] We have x2 – 2 + 2 = 0 ⇒ x = 0 not possible

∴ The given equation y2 + ay + b = 0 must have integral [x] = 0 ⇒ x = ± 2 not possible

roots which is not possible as a and b are odd integers. [x] = 1 ⇒ x = ± 4 = ± 2 not possible

Objective_Maths_JEE Main 2017_Ch 4.indd 34 01/01/2008 03:37:28


Quadratic Equations and Expressions  4.35

[x] = 2 ⇒ x = ± 6 44. Product of roots = 2 e2 ln k – 1 = 7 (given)


2

⇒ x = 6 is the only solution. ⇒ 2e ln k − 1 = 7


The correct option is (A) ⇒ 2k2 – 1 = 7
2 2 2
⇒ k = ±2
40. Let 2sin x = Z ⇒ 2cos x =
Z ⇒ k = 2 (Since ln k is defined for k > 0)
Therefore, the given equation becomes The correct option is (B)
2 45. x – x2 – 9 = –(x2 – x + 9)
Z– – 1 = 0 ⇒ Z = 2 or Z = –1
Z
2 2 ⎡⎛ 1⎞ 35 ⎤
2
⇒ 2sin x = 2 or 2sin x = –1 (not possible)
= – ⎢⎜ x − ⎟ + ⎥ < 0 for all x ∈ R
⎢⎣ ⎝ 2 ⎠ 4 ⎥⎦
π
⇒ sin2 x = 1 ⇒ x = (2n + 1) , n ∈ 1
2 ∴ no. solution i.e., solution set = f

The correct option is (C)
⎡ ⎛ 3⎞ x ⎤
41. Given: |α – β | > 3p
 ⎢Q ⎜ ⎟ > 0 for all x ∈ R ⎥
⎢⎣ ⎝ 5 ⎠ ⎥⎦
If α, β are the roots of x2 + px + 1 = 0, then
The correct option is (C)
α + β = –p, aβ = 1
46. Let esin x = t
∴ (α – β)2 > 3p
⇒ t2 – 4t – 1 = 0
⇒ (α + β)2 – 4aβ > 3p
⇒ (–p)2 – 4 · 1 > 3p 4 ± 16 + 4
⇒ t =

⇒ p2 – 3p – 4 > 0 2
⇒ (p – 4) (p + 1) > 0 ⇒ t = esin x = 2 ± 5

⇒ p > 4, p > –1 or p < 4, p < –1 ⇒ esin x = 2 –
5 , esin x = 2 + 5
⇒ p > 4 or p < –1
e sin x
=2– 5 < 0,
But p is not –ve (∵ If p is –ve, then 3 p is not real) ⇒ sin x = ln(2 + 5 ) > 1
⇒ p > 4 So both rejected.
The correct option is (B) Hence no solution
42. If f (x) = x2 + ax + b The correct option is (B)
f  (x + c) = x2 + (2c + a) x + c2 + ac + b 47. Let f (x) = x3 – px + q
∴ Roots of the given equation are 0 and d – c. Now for maxima/minima
 (since roots of x2 + ax + b = 0 are c and d.) f ′(x) = 0
The correct option is (B) ⇒ 3x2 – p = 0
43. Let f (x) = x2 + 2 (k + 1) x + 9k – 5. Let α, β be the roots of
f (x) = 0. The equation f (x) = 0 will have both negative roots
if and only if –(p/3)
  (i) Disc. ≥ 0
–(p/3)
 (ii)  α + β < 0 and
(iii)  f (0) > 0
Now, discriminant ≥ 0
p
⇒ 4 (k + 1)2 – 36k + 20 ≥ 0  ⇒  k2 – 7k + 6 ≥ 0 ⇒ x2 =

3
⇒ (k – 1) (k – 6) ≥ 0
p
⇒ k ≤ 1 or k ≥ 6  (1) ∴ x = ±

3
(α + β) < 0
The correct option is (A)
⇒ –2 (k + 1) < 0
48. Let α and 4β be roots of x2 – 6x + a = 0 and α, 3β be the roots
⇒ k + 1 > 0 ⇒ k > –1 (2) of x2 – cx + 6 = 0, then
and aβ > 0 α + 4β = 6 and 4aβ = a
⇒ 9k – 5 > 0 α + 3β = c and 3aβ = 6.
5
⇒ k >  (3) We get aβ = 2 ⇒ a = 8
9
From Eqs (1), (2) and (3) we get k ≥ 6. So the first equation is x2 – 6x + 8 = 0 ⇒ x = 2, 4
The correct option is (C) If α = 2 and 4β = 4 then 3β = 3

Objective_Maths_JEE Main 2017_Ch 4.indd 35 01/01/2008 03:37:31


4.36  Chapter 4

If α = 4 and 4β = 2, then 3β = 3/2 (non-integer) 1 ⎡1⎤


∴ common root is x = 2. ⇒ x +
− 1 = [x] + ⎢ ⎥
x ⎣x⎦
The correct option is (D) 2
x +1− x
49. bx2 + cx + a = 0 ⇒ = (integer) k (say)
x
Roots are imaginary ⇒ c2 – 4ab < 0 ⇒ x2 – (k + 1) x + 1 = 0
⇒ c2 < 4ab ⇒ –c2 > –4ab Since x is real, so (k + 1)2 – 4 ≥ 0
Given expression has minimum value ⇒ k2 + 2k – 3 ≥ 0 ⇒ (k + 3) (k – 1) ≥ 0
4(3b 2 )( 2c 2 ) − 36b 2c ⇒ k ≤ –3  or  k ≥ 1.
=

4(3b 2 ) Therefore, number of solutions is infinite.
12b 2c 2 The correct option is (C)
=–
= –c2 > –4ab. 54. Since the given equation has imaginary roots
12b 2

The correct option is (C) ⇒ D < 0 or (a + b + c)2 – 4(ab + bc + ca) < 0
⇒ (a2 + b2 + c2 – 2ab – 2bc + 2ac) < 4ac
50. Put x − 1 = t ⇒ x – 1 = t2
⇒ (a – b + c)2 < 4ac
or x = t2 + 1, the given equation reduces to
⇒ − 2 ac < a – b + c

t 2 + 1 + 3 − 4t + t 2 + 1 + 8 − 6t
= 1 where, t ≥ 0.


( a + c + 2 ac ) > b
( a + c ) > b or
2
⇒ |t – 2| + |t – 3| = 1, where t ≥ 0. This equation will be

a + c > b.
satisfied if 2 ≤ t ≤ 3.
Similarly, b + c > a and a + b > c.
Therefore, 2 ≤
x − 1 ≤ 3 or 5 ≤ x ≤ 10.
Therefore, a , b , c can be the sides of a triangle.
∴ The given equation is satisfied for all values of x lying

The correct option is (A)
in [5, 10].
55. We have,

The correct option is (D)
xn + ax + b = (x – x1) (x – x2) … (x – xn)
2
x −1
51. We have, 3sec 9 y2 − 6 y + 2 ≤ 1 x n + ax + b
⇒ (x – x2) (x – x3) … (x – xn) =
x − x1
2 2 2
⇒ 3sec
x
y2 − y+ ≤1 x n + ax + b
3 9 ⇒ (x1 – x2) (x1 – x3) … (x1 – xn) = lim
x→ x
1 x − x1
sec 2 x ⎛ 1⎞ 1
2
= nx1n –1 + a.
⇒ 3
⎜⎝ y − ⎟⎠ + ≤ 1
3 9 The correct option is (A)
2
⎛ 1⎞ 1 1 56. Since the roots of the given equation are real
Now, sec2x ≥ 1 ⇒ 3sec2x ≥ 3 and
⎜⎝ y − ⎟⎠ + ≥ , so we
3 9 3 ∴ B2 – 4AC ≥ 0 ⇒ 4a2 – 4 (a2 + a – 3) ≥ 0
1 ⇒ –a + 3 ≥ 0 or a ≤ 3.  (1)
must have sec2x = 1 and y – = 0.
3 Since the root is less than 3, so f (3) > 0
1 ⇒ 32 – 2a (3) + a2 + a – 3 > 0
⇒ x = 0, π, 2π, 3π and y = .
3 ⇒ a2 – 5a + 6 > 0 or (a – 2) (a – 3) > 0
∴ There are 4 solutions.
The correct option is (B) ⇒ a < 2 or a > 3 (2)
52. Let From (1) and (2), we have a < 2.
f (x) = (x – n)m + (x – n2)m + (x – n3)m + … + (x – nm)m. The correct option is (A)
⇒ f ′(x) = m(x – n)m–1 + m(x – n2)m–1 + … + m(x – nm)m–1 1 − x + x2
57. Let z =
Since m is odd, (m – 1) is even. Therefore, f ′(x) = 0 has no 1 + x + x2
real root. ⇒ z + zx + zx2 = 1 – x + x2

⇒ f (x) = 0 has one real and (m – 1) imaginary roots.
⇒ zx2 – x2 + zx + z + z – 1 = 0
The correct option is (C) ⇒ x2 (z – 1) + x (z + 1) + (z – 1) = 0

⎛ 1⎞ For real x, B2 – 4AC ≥ 0

53. We have, f (x) + f ⎜ ⎟ = 1
⎝ x⎠ ⇒ (z + 1)2 – 4 (z – 1) (z – 1) ≥ 0

1 ⎡1⎤ ⇒ z2 + 2z + 1 – 4z2 + 8z – 4 ≥ 0

⇒ x – [x] +
− =1
x ⎢⎣ x ⎥⎦ ⇒ – 3z2 + 10z – 3 ≥ 0 ⇒ – 3z2 + 9z + z – 3–≥ 0

⇒ – 3z (z – 3) + 1 (z – 3) ≥ 0

Objective_Maths_JEE Main 2017_Ch 4.indd 36 01/01/2008 03:37:35


Quadratic Equations and Expressions  4.37

1 62. Let α, β be the roots of equation x2 + bx + c = 0 and α ′, β ′


⇒ (z – 3) (– 3z + 1) ≥ 0 ⇒
≤ z ≤ 3.
3 be the roots of the x2 + qx + r = 0. Then,
1 α + β = – b; aβ = c, α ′ + β ′ = – q, α ′ β ′ = r.
∴ minimum value of z =
.
3
a a′ a+b a ′ + b′

The correct option is (B) It is given that = ⇒ =
b b′ a −b a ′ − b′
1 x2 − 2x + 4 (a + b ) 2 (a ′ + b ′ ) 2
58. Given, < < 3 for all x ∈ R. ⇒
=
3 x2 + 2x + 4 (a − b ) 2 (a ′ − b ′ ) 2
1 x2 + 2x + 4 b 2
q2

< < 3 for all x ∈ R.  (1) ⇒
= ⇒ b2r = q2c.
3 x2 − 2x + 4 b 2 − 4c q2 − 4r
Let 3x + 1 = y
The correct option is (C)
Then, y ∈ R for all x ∈ R.
63. If x = n ∈ I, |n – 2n| = 4
9 ⋅ 32 x + 6 ⋅ 3x + 4 32 x + 2 + 2 ⋅ 3x + 1 + 4 ∴ n = ± 4,

=
9 ⋅ 32 x − 6 ⋅ 3x + 4 32 x + 2 − 2 ⋅ 3x + 1 + 4 If x = n + k, n ∈ I, 0 < k < 1 then
2
y + 2y + 4 |n – 2 (n + k)| = 4

=
y2 − 2 y + 4 ∴ |–n – 2k| = 4.
1
1 y2 + 2 y + 4 It is possible if k = .

From (1), < <3 2
3 y2 − 2 y + 4 The correct option is (B)
1 9 ⋅ 32 x + 6 ⋅ 3x + 4 64. Since α, β are roots of the equation x2 + px + q = 0

< < 3.
3 9 ⋅ 32 x − 6 ⋅ 3x + 4 ∴ α + β = – p and aβ = q.
2
The correct option is (D) ⎛a⎞ a
Now, q ⎜ ⎟ + ( 2q − p 2 ) + q
59. Let f (x) = x2 + (1 – 2k) x + k2 – k – 2 ⎝ b⎠ b
The number 3 lies between the roots of the given equation, if 1 1
f (3) < 0 = 2 [q (α + β) – p ab] = 2 [qp2 – p2q] = 0.
2 2
b b
Now, f (3) = 9 + (1 – 2k) 3 + k2 – k – 2 a
Thus, is a root of the equation
= 10 – 7k + k2 = k2 – 7k + 10 b
Hence, f (3) < 0 ⇒ k2 – 7k + 10 < 0 qx2 + (2q – p2) x + q = 0.
⇒ (k – 2) (k – 5) < 0 ⇒ 2 < k < 5. The correct option is (C)
The correct option is (A) 65. Let x1 and x2 be two roots of ax2 + bx + c = 0
60. The given equation can be written as 1 < x1 < 2 and 1 < x2 < 2
⎛ b c⎞

⎡ 1⎤
2 x + 1 ⎢ x 2 − ⎥ = 2| x − 3 | [4 x 2 − 1] Now, a (5a + 2b + c) = a2 ⎜⎝ 5 + 2 a + a ⎟⎠
⎣ 4⎦


⎡ 2 1⎤ = a2(5 + 2(–1) (x1 + x2) + x1x2)

= 4 · 2|x – 3| ⎢⎣ x − 4 ⎥⎦ = a2{(x1 – 2) (x2 – 2) + 1} > 0
Hence, a and 5a + 2b + c are of same sign.
⇒ 2x – 1 = 2|x – 3|

 1 The correct option is (A)
[∵ x2 = does not give negative integral value]
4 66. Since a < 0, in case of positive root of the equation x > a
⇒ x – 1 = ± (x – 3) ∴ The equation is x2 – 2a (x – a) – 3a2 = 0
⇒ either x – 1 = x – 3 or x – 1 = – x + 3 ⇒ x2 – 2ax – a2 = 0
⇒ 1 = 3 (not possible) or 2x = 4 2a ± 4 a 2 + 4 a 2 2a ± 2 2a
i.e., x = 2.
Thus, the roots are =
2 2
∴ Given equation does not give any negative integral = a(1 + 2 ) or a(1 − 2 )
solution.
∴ the only positive root possible is a(1 − 2 ) .
The correct option is (D)
The correct option is (C)
61. We have α + β = –b, aβ = c
67. Let α + iβ, α – iβ be the roots
As c < 0, b > 0, we get α < 0 < β.
r
Also, β = – b – α < – α = |α | Then, a2 + b2 = >0
p
Thus, α < 0 < β < |α |.
∴ p, r must be of the same sign.
The correct option is (B)

Objective_Maths_JEE Main 2017_Ch 4.indd 37 01/01/2008 03:37:39


4.38  Chapter 4

Since p + r > 0 ∴ p, r are both positive. 72. Let α, β be the roots of lx2 + mx + v = 0
If q < 0, p – q + r > 0 m v
∴ α + β = – , aβ =
If q > 0, (p + r)2 – (p – r)2 = 4pr ≥ q2 l l

[∵ roots are non-real] m2
∴ (p + r)2 ≥ q2 + (p – r)2 ≥ q2 (a + b ) 2 2 m2
= l =




∴ p + r ≥ q ab v lv
The correct option is (B) l
68. Let f (x) = lx2 – mx + 5 a b m2

+ +2=  (1)
Since lx2 – mx + 5 = 0 does not have two distinct real roots, b a lv
therefore, Let γ, δ be the roots of x2 + x + 1 = 0

either f (x) ≥ 0 ∀ x ∈ R, or f (x) ≤ 0 ∀ x ∈ R ∴ γ + δ = –1, g δ = 1

But f (0) = 5 > 0 (g + d ) 2 g d
∴ f (x) ≥ 0 ∀ x ∈ R ∴
= 1 ⇒ + + 2 = 1  (2)
gd d g
∴ f (–5) ≥ 0 ⇒ 25l + 5m + 5 ≥ 0 ⇒ 5l + m ≥ –1 a g
Hence, minimum of 5l + m is –1.
Since = ,
b d
The correct option is (D) ∴ from (1) and (2)

69. Since coefficient of x2 > 0 and 1 lies between the roots of
m2
3x2 – 3sin θ – 2cos2 θ = 0 = 1 ⇒ m2 = lv
lv
∴ f (1) < 0
∴ λ, µ, v are in G.P.
⇒ 3 – 3sin θ – 2cos2θ < 0
The correct option is (B)
⇒ 1 + 2(1 – cos2 θ) – 3sin θ < 0
73. Given equation can be written as
⇒ 2sin2 θ – 3sin θ + 1 < 0
bx2 + x – bcx – bdx + bcd – a = 0;
⇒ (2sin θ – 1) (sin θ – 1) < 0
⇒ bx (x – c) – bd (x – c) + x – a = 0
1
⇒ < sin θ < 1 ⇒ b (x – c) (x – d) + (x – a) = 0
2
Let f (x) = b (x – c) (x – d) + (x – a)
The correct option is (C)
n f (c) = c – a < 0; f (d) = d – a > 0
70. ∑ Sr = (α + β) + (α 2 + β 2) + … + (α n + β n) The correct option is (C)
r =1
74. If f (x) = x2 + ax + b

= (α + α 2 + … + α n) + (β + β 2 + … + β n)
n f  (x + c) = x2 + (2c + a) x + c2 + ac + b
Lt
n→∞
∑ Sr = (α + α 2 + … + ∞) + (β + β 2 + … + ∞) ∴ roots of the given equation are 0 and d – c.
r =1
a b  (since roots of x2 + ax + b = 0 are c and d.)

= +
1− a 1− b The correct option is (B)
a − ab + b − ab 75. If x = n ∈ Z, (x)2 + (x + 1)2 = 25

= ⇒ n2 + (n + 1)2 = 25
1 − (a + b ) + ab
⇒ 2n2 + 2n – 24 = 0
a + b − 2ab

= ⇒ n2 + n – 12 = 0
1 − (a + b ) + ab
⇒ n = 3, – 4
25 4
+ ∴ x = 3, –4
375 375 29 1
= = = If x = n + k, n ∈ Z, 0 < k < 1, then
25 2 348 12
1− −
375 375 (x)2 + (x + 1)2 = 25
The correct option is (B) ⇒ (n + 1)2 + (n + 2)2 = 25
71. Since ax2 + bx + 6 = 0 does not have two distinct real roots ⇒ 2n2 + 6n – 20 = 0
∴ b2 – 24a ≤ 0 ⇒ n2 + 3n – 10 = 0
Let 3a + b = y ∴ 3a = y – b ⇒ n = 2, –5
∴ b2 – 8(y – b) ≤ 0 i.e., b2 + 8b – 8y ≤ 0 ∴ x = 2 + k, –5 + k, where 0 < k < 1
Since b is real ∴ 64 + 32y ≥ 0 ⇒ y ≥ –2 ∴ x > 2, x > –5
∴ Min. value of y, i.e., 3a + b = –2. ∴ Solution set is (–5, –4] ∪ (2, 3]
The correct option is (D) The correct option is (B)

Objective_Maths_JEE Main 2017_Ch 4.indd 38 01/01/2008 03:37:42


Quadratic Equations and Expressions  4.39

76. The required a satisfies the inequality If x < – 1, 1 + |x| – |x + 1| = 0


2a2 – 2(2a + 1)a + a(a + 1) < 0 ⇒ 1 – x + x + 1 = 0 ⇒ 2 = 0 (absurd)
⇒ a(a + 1) > 0 ⇒ a ∈ (–∞, –1) ∪ (0, ∞) If –1 ≤ x < 0, 1 + |x| – |x + 1| = 0
The correct option is (D) ⇒ 1 – x – (x + 1) = 0 ⇒ x = 0 (not possible)
77. We have, 2x + 2x ≥ 2 2 (x ≥ 0) If x ≥ 0, 1 + x – (x + 1) = 0 ⇒ 0 = 0
1 ⇒ x can have any value in the interval
⇒ 2x ≥ 2 ⇒ x ≥ ∴ x = –1, x > 0.(Q x ≠ 0)
2
and, 2x + 2–x ≥ 2 2 (x < 0) The correct option is (B)
1 81. As α, β are roots of ax2 + bx + c = 0, we have
⇒ t + ≥ 2 2 (where t = 2x)
t α + β = –b/a, aβ = c/a
⇒ t2 – t + 1 ≥ 0 Now, (α – β )2 = (α + β )2 – 4aβ
⇒ (t − ( 2 − 1)) (t − ( 2 + 1)) ≥ 0 ⎛ b⎞ 4c
2
b 2 − 4 ac

= ⎜− ⎟ − = (1)
⇒ t ≤
2 − 1 or t ≥ 2 + 1 but t > 0 ⎝ a⎠ a a2
x
⇒ 0 < 2 ≤
2 − 1 or 2x ≥ 2 +1 Now, as α + δ, β + δ are the roots of Ax2 + Bx + C = 0,

⇒ –∞ < x ≤ log2 ( 2 − 1)
we have  α + δ + β + δ = –B/A, (α + δ ) (β + δ ) = C/A.

or, x ≥ log2 ( 2 + 1) 
(but not acceptable as x < 0) Now, (α – β )2 = [(α + δ ) – (β + δ )]2



= (α + δ + β + δ )2 – 4 (α + δ ) (β + δ )
⎡1 ⎞
∴ x ∈ (–∞, log2 ( 2 − 1) ] ∪ ⎢ , ∞⎟
B2 4C B 2 − 4 AC
⎣2 ⎠
= − = (2)
A2 A A2

The correct option is (D)

From (1) and (2), we get
D 4b 2 − 8c 2
78. min. f (x) = – =– b 2 − 4 ac B 2 − 4 AC
4a 4 2 = .
a A2

= –(b2 – 2c2) (upward parabola)
2 2
The correct option is (A)
D 4c + 4b
max. g(x) = –
= 82. Given equation is ax2 + bx + c = 0
4a 4
2 2
Since α, β are the roots of the given equation

=b +c  (downward parabola) −b c
Now, 2c2 – b2 > b2 + c2
∴ α + β = , aβ = .
a a

⇒ c2 > 2b2 ⇒ |c| > 2 |b| Also, since α < – 1, β > 1
The correct option is (D) ∴ aβ < – 1
79. We can write the given equation as c c
⇒ < – 1 or +1<0
(x – a)2 = 3 – a a a
Let f (x) = ax2 + bx + c
This shows that a ≤ 3 and x = a ± 3 − a
As f (1) f (–1) > 0,
Both the roots of the given equation will be less than 3 if the
larger of the two roots is less than 3, that is, if ∴ (a + b + c) (a – b + c) > 0
a + 3−a < 3 or, (a + c)2 – b2 > 0 or (a + c)2 > b2
2 2

3 − a – (3 – a) < 0 ⎛ c⎞ ⎛ b⎞
or, ⎜1 + ⎟⎠ > ⎜⎝ ⎟⎠
⎝ a a

3 − a (1 − 3 − a ) < 0
⎛ c⎞ b ⎡ ⎛c ⎞ ⎤
⇒ a < 3 and 1 − 3 − a < 0
⇒ ⎜1 + ⎟ < −
 ⎢Q ⎜⎝ a + 1⎟⎠ < 0 ⎥
⎝ a⎠ a ⎣ ⎦
But 3 − a > 1 ⇒ 3 – a > 1 or a < 2

Thus, a < 3 and a < 2 ⇒ a < 2
c b
or, 1 +
+ <0.

The correct option is (A) a a
The correct option is (A)
| x + 1| | x + 1 |2
80. + | x + 1| = 83. Roots of ax2 + 2bx + c = 0 are given by
|x| |x|
⎧ 1 | x + 1 |⎫ − 2b ± 4b 2 − 4 ac −b
⇒ | x + 1 | ⎨ +1− x=
=
⎬ =0 2a a
⎩| x | |x| ⎭
∴ |x + 1| = 0 or 1 + |x| – |x + 1| = 0.
(Since b2 = ac as a, b, c, are in G.P.)

|x + 1| = 0 ⇒ x = –1.
This is root of dx2 + 2ex + f = 0

Objective_Maths_JEE Main 2017_Ch 4.indd 39 01/01/2008 03:37:48


4.40  Chapter 4

⎛ − b⎞ ⎛ − b⎞
2 86. The discriminant of the given equation is
∴ d ⎜
+ 2e ⎜ + f =0
⎝ a ⎟⎠ ⎝ a ⎟⎠ D = 4 [(nCr)2 – nCr – 1 nCr + 1]
⇒ db2 – 2eba + a f = 0
= 4 (a – b),
⇒ dac – 2eba + a f = 0
(Q b2 = ac) where a = (nCr)2, b = nCr – 1 · nCr + 1
n
⇒ 2eb = dc + a f
a Cr ⋅ n Cr
Now, 
= n
2e dc + af dc + af b Cr − 1 ⋅ n Cr + 1

= = (Q b2 = ac)
b b2 ac n! n! ( r − 1)! ( n − r + 1)!
d f
= ⋅
= + r ! ( n − r )! r ! ( n − r )! n!
a c
( r + 1)! ⋅ ( n − r − 1)!
d e f

⇒ , , are in A.P. n!
a b c
r +1 n − r +1 ⎛ 1⎞ ⎛ 1 ⎞
The correct option is (C)
= ⋅ = ⎜1 + ⎟ ⎜1 + >1
r n−r ⎝ r⎠ ⎝ n − r ⎟⎠
84. Let α, β be the roots of x2 + abx + c = 0 and α, γ  be the roots
of x2 + acx + b = 0, a being the common root. ∴ a > b ⇒ D > 0

∴ α + β = – ab(1) ⇒ roots of given equation are real and distinct.

aβ = c(2)
The correct option is (A)
1
α + γ  = – ac(3) 87. Let α and β be the roots of x2 – px + q = 0 and α and be
2
the roots of x – ax + b = 0. b
aγ  = b (4)
From (1) – (3), Then,  α + β = p and aβ = q.
β – γ = a (c – b) 1 a
Also,  a + = a and = b.
From (2)– (4), b b
α (β – γ) = c – b ⎛ a⎞ ⎛ 1⎞
2 2
Now, (q – b)2 = ⎜ ab − ⎟ = a 2 ⎜ b − ⎟

a (b − g ) c−b ⎝ b⎠ ⎝ b⎠

= ;
b −g a ( c − b) 2
a ⎡ ⎛ 1⎞⎤
1
=⋅ ba ⎢(a + b ) − ⎜ a + ⎟ ⎥
or α = .
b ⎣ ⎝ b⎠⎦
a
∴ From (2) and (4),
= bq (p – a)2.
b The correct option is (B)
= c, i.e., β = ac 88. The roots of 2x2 – 3x + 4 = 0 are imaginary, because disc. =
a
g (–3)2 – 4 · 2 · 4 < 0. Hence, the common root must be
and, 
= b, ­imaginary. But imaginary roots occur in pair. Hence both the
a
i.e., γ = ab.
roots will be common, i.e., two equations will be identical.
So their coefficients will be proportional
∴ The quadratic equation whose roots are β, γ is a b c
i.e., = = ,
x2 – (β + γ ) x + bγ = 0 2 −3 4
or, x2 – (ac + ab) x + ac · ab = 0; ∴ 6a = –4b = 3c.
or, x2 – a (b + c) x + a2bc = 0. The correct option is (C)
The correct option is (B) 89. We have, 3x2 + x – 5 = 0.
85. The given equation can be written as Its discriminant = 1 – 4 · 3 (– 5) = 61, which is positive but
(ay + a′) x2 + (by + b′) x + (cy + c′) = 0. not a perfect square. Hence, both the roots of 3x2 + x – 5 = 0
The condition that x may be a rational function of y is, must be irrational as the irrational roots occur in conjugate
pair. But one root of ax2 + bx + c = 0 and 3x2 + x – 5 = 0 is
(by + b′)2 – 4 (ay + a′) (cy + c′) is a perfect square;
common. Hence, both the roots of ax2 + bx + c = 0 must also
that is, (b2 – 4ac) y2 + (2bb′ – 4ac′ – 4a′c) y + b′2 – 4a′c′ is a be irrational, that is, both the roots of the given equations are
perfect square. common. Thus, both the equations are the same.
The corresponding quadratic equation has discriminant = 0 a b c
∴ = = = k (say)
that is, 4 (bb′ – 2ac′ – 2a′c)2 – 4 (b2 – 4ac) (b′2 – 4a’c’) = 0; 3 1 −5
or, (ac′ + a′c)2 – 4aa′cc′ = abb′c + a′bb′c – a′c′b2 – acb′ 2 ⇒ a = 3k; b = k, c = –5k.

or, (ac′ – a′c)2 = (ab′ – a′b) (bc′ – b′c). ∴ 3a + b + 2c = 9k + k – 10k = 10k – 10k = 0.
The correct option is (A) The correct option is (A)

Objective_Maths_JEE Main 2017_Ch 4.indd 40 01/01/2008 03:37:52


Quadratic Equations and Expressions  4.41

90. Let α be the common root.


Dividing (1) by (3), we get
Then,  aa2 + 2bα + c = 0 a (1 + k ) ba a ba
= 1 2 , or = 1 2 .(5)
and,  a1a2 + 2b1α + c1 = 0 b (1 + k ) a1b2 b a1b2
By cross-multiplication, we get
Dividing (2) by (4), we get
a2 a 1 ka 2 ca
= = 2 = 1 2 ;
2(bc1 − b1c) ca1 − ac1 2 ( ab1 − ba1 ) kb a1c2
⇒ (ca1 – ac1)2 = 4 (bc1 – b1c) (ab1 – ba1) 
(1) 2
⎛a⎞ c1a2
a b c
or ⎜ ⎟ =
Q
, , are in A.P., ⎝ b⎠ a1c2
a1 b1 c1
2
b a c b ⎛ba ⎞ ca

− = − = k (say) or, ⎜ 1 2 ⎟ = 1 2 
(Using (5))
b1 a1 c1 b1 ⎝ a1b2 ⎠ a1c2
⇒ ab1 – a1b = – ka1b1 and bc1 – b1c = – kb1c1.
⎛b ⎞
2
c1a2 a12 ca a c
⇒ ⎜ 1 ⎟
= × 2= 11 = 1⋅ 1
b a c b ca − ac1 ⎝b ⎠ a c a c a a2 c2
Also, 2k = k + k =
− + − = 1 2 1 2 2 2 2
b1 a1 c1 b1 a1c1 a1 b1 c1
∴ , , are in G.P.
or, ca1 – ac1 = 2ka1c1.
a2 b2 c2
∴ From (1), 4 k 2 a12c12 = 4 (–ka1b1) (–kb1c1)
The correct option is (B)
or, a1c1 = b12. Hence a1, b1, c1 are in G.P.
93. Since α, β are the roots of the equation

The correct option is (B) x2 – px + q = 0
∴ α + β = p and aβ = q.
( a + x ) (b + x )
91. Let y =
(c + x ) Now, (a1/4 + b1/4)4 = [(a1/4 + b1/4)2]2

2
⇒ x + (a + b) x + ab = cy + xy

= [a1/2 + b1/2 + 2 (aβ)1/4]2
⇒ x2 + (a + b – y) x + ab – cy = 0.
2


= ⎡ a + b + 2 ab + 2(ab )1/ 4 ⎤
For real x, B2 – 4AC ≥ 0
⎢⎣ ⎥⎦
⇒ (a + b – y)2 – 4ab + 4cy ≥ 0
2

= ⎡ p + 2 q + 2 ( q)1/ 4 ⎤
⇒ (a + b)2 + y2 – 2 (a + b) y – 4ab + 4cy ≥ 0
⎣⎢ ⎦⎥
⇒ (a – b)2 + y2 – 2 (a + b – 2c) y ≥ 0


=p+6 q + 4 q1/ 4 p+2 q
⇒ y2 – 2 (a + b – 2c) y + (a – b)2 ≥ 0

1/ 4
⇒ [ y − ( ( a − c) − (b − c)) 2 ]
∴ a1/4 + b1/4 = ⎡ p + 6 q + 4 q1/ 4
p+ 2 q⎤
⎣⎢ ⎦⎥
× [ y − ( ( a − c) + (b − c)) 2 ] ≥ 0
The correct option is (D)
∴ y ≤ ( ( a − c) − (b − c)) 2 94. Since a and b are the roots of the equation
or, y ≥ ( ( a − c) + (b − c)) 2 .
x2 + px + 1 = 0
∴ a + b = –p(1)

Hence, the minimum value of y is
and, ab = 1 (2)
( ( a − c) + (b − c)) 2 Also, since c and d are the roots of the equation
The correct option is (B) x2 + qx + 1 = 0
92. Let the ratio of the roots be k. Then, the roots of ∴ c + d = –q(3)
a1x2 + b1x + c1 = 0 are α, kα and, cd = 1 (4)
and the roots of a2x2 + b2x + c2 = 0 are β, kβ. Now, (a – c) (b – c) (a + d) (b + d)
−b1 = (ab – bc – ac + c2) (ab + db + ad + d2)
∴ α + kα = (1)
a1 = [ab – c (b + a) + c2] ⋅ [ab + d (a + b) + d2]
c
α ⋅ kα = 1 (2) = (1 + cp + c2) (1 – pd + d2)
a1
[Putting the values of a + b and ab]
−b2
β + kβ =
(3) = 1 + cp + c2 – pd – cdp2 – c2pd + d2 + cpd2 + c2d2
a2
= 1 + (c2 + d2) + c2d2 – cdp2 + p (c – d) + cpd (d – c)
c
β ⋅ kβ = 2 .(4)
= 1 + [(c + d)2 – 2cd] + c2d2 – cdp2 + p (c – d) + cpd (d – c)
a2
= 1 + (q2 – 2) + 1 – p2 + p (c – d) + p (d – c)

Objective_Maths_JEE Main 2017_Ch 4.indd 41 01/01/2008 03:37:58


4.42  Chapter 4

[Putting the values of c + d and cd] ⎛ a + b⎞ 2


2

= 2 – 2 + q2 – p2 = q2 – p2. ⇒ α + β = ⎜ –
⎝ ab ⎟⎠ ab
The correct option is (B) 2 2
95. Let f (x) = x3 + px2 + q = 0 ⎛ b⎞ ⎛ − b /a ⎞ 2 b ⎛ b⎞ 2a
(1) ⇒ ⎜ − ⎟ = ⎜ – ⇒– = ⎜ ⎟ –
⎝ a⎠ ⎝ c /a ⎟⎠ c /a a ⎝ c⎠ c
Since f (x) = 0 has a root of multiplicity 2
2
∴ f (x) = 0 and f ′ (x) = 3x2 + 2px = 0 have a common root. 2a ⎛ b⎞ b 2a b ⎡b c ⎤
⇒ = ⎜ ⎟ + ⇒ = +
The roots of 3x2 + 2px = 0 are x = 0 and x = –2p/3. c ⎝ c⎠ a c c ⎢⎣ c a ⎥⎦
But x = 0 is not a root of f (x) = 0 (Q q ≠ 0) 2a b c c a b
⇒ = + ⇒ , , are in A.P.
∴ common root of f (x) = 0 and f ′ (x) = 0 is x = –2p/3. b c a a b c
∴ (–2p/3)3 + p (–2p/3)2 + q = 0 a b c
⇒ , , are in H.P.
⇒ 4p3 + 27q = 0. c a b
The correct option is (D) The correct option is (C)
a a +1 98. Discreminent equals –4(k – 5) ≥ 0 ⇒ k ≤ 5. The quadratic
96. Since and are roots of the equation equation at x = 5 must be positive and sum of the roots must
a −1 a
be less than 10. These conditions imply k2 – 9k + 20 > 0. So,
ax2 + bx + c = 0
k < 4.
a a + 1 −b

+ = The correct option is (A)
a −1 a a
99. Let y = x – [x]
a a +1 c

and  · = ∴ the given equation becomes
a −1 a a
f (y) = (a – 2) y2 + 2y + a2 = 0 (1)
2a 2 − 1
−b Since x is not an integer,

=  (1)
a2 − a a ∴ y = x – [x] ≠ 0
a +1 c Then,  a ≠ 0 [Q of (1)]

and,  =
a −1 a When 2 < x < 3, [x] = 2
⇒ aα + a = cα – c

c+a ⇒ 0 < x – [x] <1 i.e. 0 < y < 1
⇒ α (c – a) = a + c or α =
.
c−a Since given equation has exactly one solution in the interval
2 (2, 3)
⎛ c + a⎞
2⎜ −1 ∴ (1) has exactly one solution in the interval (0,1)
⎝ c − a ⎟⎠ b
∴ From (1),
2
=– This is possible if f (0) f (1) < 0
⎛ c + a⎞ c+a a
[∵ otherwise, the equation (1) has either no or two
⎜⎝ c − a ⎟⎠ − c − a
 solutions in (0,1)]
2 2
2 ( c + a) 2 − ( c − a) 2 b ⇒ a (a – 2 + 2 + a ) < 0 ⇒ a (a + 1) < 0

2 2 2
=– ⇒ –1 < a < 0 i.e., a ∈ (–1, 0)
( c + a) − ( c − a ) a
The correct option is (C)
(c + a) 2 + 4 ac b

=– 100. Given, b2 ≤ 4 ac, c2 ≤ 4ab and a2 ≤ 4ac
2a 2 + 2 ac a
Equality cannot hold simultaneously
⇒ (c + a)2 + 4ac = – 2b (a + c)
[∵ a, b, c are different]
⇒ (c + a)2 + 2b (a + c) + 4ac = 0
∴ a2 + b2 + c2 < 4 (ab + bc + ca) ⇒ R < 4
⇒ (c + a)2 + 2b (a + c) + b2 = b2 – 4ac
Also, a2 + b2 + c2 – ab – bc – ca
⇒ (a + b + c)2 = b2 – 4ac. 1
The correct option is (C) ⎡⎣(b − c) 2 + (c − a) 2 + ( a − b) 2 ⎤⎦ > 0 = ⇒ R > 1
2
97. Given equation is The correct option is (D)
ax2 + bx + c = 0 101. The given equation can be written as
Let α, β be the roots of this equation. (z + 1)2 – (a – 3) z (z + 1) + (a – 4) z2 = 0
b c [Putting x2 + x + 1 = z]
then, α + β = –
and aβ =
a a ⇒ (1 + 3 – a + a – 4) z2 + (2 + 3 – a) z + 1 = 0
1 1 a2 + b 2 1
Also,  α + β = 2 + 2 =
⇒ (5 – a) z + 1 = 0  or  z =
a b a 2b 2 a−5
1
(a + b ) 2 − 2ab ⇒ x2 + x + 1 – =0

= a−5
(ab ) 2

Objective_Maths_JEE Main 2017_Ch 4.indd 42 01/01/2008 03:38:04


Quadratic Equations and Expressions  4.43

a−6 Also, the coefficient of x2 = 1 > 0.


⇒ x2 + x + =0
a−5 Hence, a and b both lie between the roots of the equation
whose roots will be real if discriminant ≥ 0 f (x) = 0
4( a − 6 ) 3a − 19 ∴ The equation (x – a) (x – b) – 1 = 0 has one root in
⇒ 1 – ≥0⇒ ≤0
a−5 a−5 (–∞, a) and other in (b, ∞)[∵ b > a]
19 The correct option is (B)
∴ 5 < a ≤
3 106. The given quadratic equation is satisfied by x = –a, x = –b
The correct option is (D) and x = –c, Hence, the quadratic equation has three roots,
102. Given equation can be written as which is only possible if it is an identity hence it has infinite
(2x)2 – ( a– 4)2x – 2x + a– 4 = 0 roots.
⇒ (2x – 1) (2x – a + 4) = 0 The correct option is (D)
⇒ 2x = 1, 2x = a – 4 107. Putting x2 = y, the given equation in x reduces to
Since x ≤ 0 and 2x = a– 4 (∵ x is non-positive) ay2 – 2y – (a – 1) = 0  (1)
∴ 0 < a – 4 ≤ 1 i.e., 4 < a ≤ 5 i.e., a ∈ (4, 5] The given biquadratic equation will have four real and dis-
tinct roots, if the quadratic equation (1) has two distinct and
The correct option is (B)
positive roots. For that, we must have
103. Let f (x) = (x – α) (x – β)(1)
D > 0 ⇒ a2 – a + 1 > 0, which is true ∀ a ∈ R
Now, f (n) f (n + 1) = (n – α) ( n– β) ( n + 1– α) ( n + 1 – β)
Product of roots > 0 ⇒ 0 < a < 1
= (n – α) ( n + 1– β) (n– β) (n + 1 – α)
sum of roots > 0 ⇒ a > 0
= {n (n + 1) – n (α + β) – α + ab} {n (n + 1)
Hence, the acceptable values of a are 0 < a < 1.
– n (α + β) –β + ab}
The correct option is (C)
= {n (n + 1) + na + b–α } {n (n + 1) + na + b – b}
108. Since –5 and 1 are the roots.
= (m – α) (m – β), where m = n (n + 1) + an + b
Product of roots = –5 × 1 = b ⇒ b = –5
The correct option is (A)
and, sum of roots = –5 + 1= –[ a2 – 5a + b + 4]
104. We have,
⇒ [a2– 5a – 1] = 4 ⇒ 4 ≤ a2 – 5a – 1< 5
x 2 + nx − 2 ⇒ a2 – 5a – 5 ≥ 0 and a2 – 5a – 6 < 0
−2 ≤0
x 2 − 3x + 4
5 − 45 5 + 45
⇒ a ≤  or a ≥ and – 1 < a < 6
x 2 − ( n + 6) x + 10 2 2
⇒ ≥0
x 2 − 3x + 4 5−3 5 5+3 5
⇒ – 1 < a ≤  or  ≤a<6
⇒ x2 – (n + 6) x + 10 ≥ 0 2 2
[∵ x2 – 3x + 4 >0 ∀ x ∈ R, as its D < 0 and a > 0] ⎛ 5 − 3 5 ⎤ ⎡5 + 3 5 ⎞
The above inequality will be true for all real x if its discrim- ⇒ a ∈ ⎜ −1, ⎥∪⎢ , 6⎟
⎝ 2 ⎥⎦ ⎢⎣ 2 ⎠
inant ≤ 0
The correct option is (C)
i.e., (n + 6)2 – 40 ≤ 0
109. We have,
⇒ –( 40 – 6) ≤ n ≤ ( 40 – 6) (1)
α + β = α, aβ = p
Also, we have,
and,  γ + δ = b, gδ = q
x 2 + nx − 2 2 x 2 + ( n − 3) x + 2
2
+ 1≥ 0 ⇒ ≥0 If r (r > 1) be the common ratio of the increasing G.P. α, β,
x − 3x + 4 x 2 − 3x + 4 γ, δ then
⇒ 2x2 + (n – 3) x + 2 ≥ 0 β = ar, γ = ar2 and δ = ar3
The above inequality will be true for all real x if its discrim-
The above equations then reduce to
inant ≤ 0
α (1 + r) = a, a2r = p
⇒ (n – 3) 2 – 16 ≤ 0
and,  ar2 (1 + r) = b, a2 r5 = q
⇒ 1 ≤ n ≤ 7  (2)
b q
Drawing the number line for inequalities (1), (2) and taking ⇒ r2 = and r4 =
a p
their intersection we get 2
q ⎛ b⎞
n ∈ [ −1, 40 − 6] ⇒ = ⎜ ⎟
p ⎝ a⎠
The correct option is (C)
q+ p b2 + a2
105. Let f (x) = (x – a) ( x – b) –1 Hence, we have = 2 .
q− p b − a2
⇒ f (a) = –1 and f (b) = –1.
The correct option is (B)

Objective_Maths_JEE Main 2017_Ch 4.indd 43 01/01/2008 03:38:07


4.44  Chapter 4

110. The sum of the coefficients of the equation = 0 Also, we have for real roots
∴ x = 1 is a root of the equation. Let a be the first term (4a)2 – 16 b ≥ 0 i.e., b ≤ a2

and d be the common difference of given A.P. Therefore, 0 ≤ b ≤ a2
1 The correct option is (C)
tp = a + (p – 1)d = (1)
q 112. Let the roots be α and α + 2, where α is an odd positive
1
and, tq = a + (q –1) d = (2) integer. Then, aa2 + bα + c = 0  (1)
p 2 2
and a (α + 2) + b (α + 2) + c = 0 ⇒ aa + bα + c + (4 aα
1
Solving (1) and (2), a = d = + 4a + 2b) = 0
pq
⇒ 2a (1 + α) + b = 0 [using (1)]
∴ tpq = a + (pq – 1)d = 1
⇒ b = – 2a (1 + α)
∴ tpq is the root of the given equation.
⇒ b2 = 4a2 (1 + α)2 ⇒ b2 ≥ 4a2 (1 + 1)2
The correct option is (C) 
[∵ α ≥ 1as α is odd positive integer]
111. Let α, β be the roots of the equation 4x2 + 4ax + b = 0, then 2 2
⇒ b ≥ 16 a or |b| ≥ 4a
we have,
b The correct option is (B)
α + β = –a and aβ =
4 113. The discriminants of the given quadratic equations are,
According to the given condition,
D1 = a2 + 12b, D2 = c2– 4b and D3 = d2 – 8b
|α – β | ≤ a
∴ D1+ D2 + D3 = a2 + c2 + d2 ≥ 0
⇒ (α + β)2 – 4 α β ≤ a2
⇒ At least one of D1, D2, D3 is non-negative. Hence, the
⇒ a2 – b2 ≤ a2 equation has at least two real roots.
⇒ b ≥ 0 The correct option is (D)

More than One Option Correct Type


114. If x – a < 0, |x – a| = –(x – a) For this equation to have equal roots
∴ equation becomes x2 + 2a (x – a) – 3a2 = 0 c2 (a – b)2 – pc2 (2a + 2b – p) = 0
⇒ x2 + 2ax – 5a2 = 0 ⇒ (a – b)2 – 2p (a + b) + p2 = 0 [∵ c2 ≠ 0]
⇒ x = – (1 + 6 ) a, ( −1 + 6 ) a ⇒ [p – (a + b)]2 = (a + b)2 – (a – b)2 = 4ab
 ∵ x < a ≤ 0 ⇒ p – (a + b) = ± 2 ab
∴ x = ( −1 + 6 ) a ⇒ p = a + b ± 2 ab = ( a ± b ) 2
If x – a ≥ 0, |x – a| = x – a The correct option is (A) and (B)
∴ the equation becomes x2 – 2a (x – a) – 3a2 = 0 117. We have,
⇒ x2 – 2ax – a2 = 0 log a a 2 log a a 3 log a a
⇒ x = (1 + 2 ) a, (1 − 2 ) a + + =0
log a a + log a x log a x 2 log a a + log a x
 ∵  x ≥ a and a ≤ 0 1 2 3
∴ x = (1 − 2 ) a. ⇒ + + = 0 (let loga x = t)
1+ t t 2 + t
The correct option is (A) and (B)
2t + t 2 + 2t 2 + 6t + 4 + 3t 2 + 3t
115. Since x2 – 3x + 2 = 0 is one of the factors of the expression ⇒ =0
x4 – px2 + q, therefore, on dividing the expression by factor, t (1 + t ) ( 2 + t )
remainder = 0, i.e., on dividing x4– px2 + q by x2 – 3x + 2, ⇒ 6t2 + 11t + 4 = 0
the remainder ⇒ 6t2 + 8t + 3t + 4 = 0
(15 – 3p) x + (2p + q – 14) = 0. ⇒ (2t + 1) (3t + 4) = 0
On comparing both sides, we get 1 4
⇒ t = – , –
15 – 3p = 0 or p = 5 and 2p + q – 14 = 0 or q = 4. 2 3
The correct option is (A) and (B) 1 4
⇒ loga x = – , –
116. We can write given equation as 2 3

p ( a + b ) x + c( b − a ) ∴ x = a–1/2, a–4/3
= The correct option is (B) and (D)
2x x 2 − c2
118. When x2 + 4x + 3 ≥ 0 i.e. x ≥ – 1 or x ≤ – 3
or, p (x – c ) = 2 (a + b) x2 – 2c (a – b) x
2 2

Then, |x2 + 4x + 3| + 2x + 5 = 0
or, (2a + 2b – p) x2 – 2c (a – b) x + pc2 = 0

Objective_Maths_JEE Main 2017_Ch 4.indd 44 01/01/2008 03:38:10


Quadratic Equations and Expressions  4.45

⇒ x2 + 4x + 3 + 2x + 5 = 0 ⇒ x2 – 4x + 3 ≤ 0
⇒ x2 + 6x + 8 = 0 ⇒ (x – 1) (x – 3) ≤ 0
⇒ (x + 2) (x + 4) = 0 ⇒ 1 ≤ x ≤ 3.
⇒ x = – 2 or x = –4. The correct option is (A) and (C)
Thus, x = –4 as x ∈ {x : x ≥ – 1} ∪ {x : x ≤ – 3} 121. We have,
When x2 + 4x + 3 < 0 i.e. – 3 < x < – 1 b c
α + β = − , ab =
Then, |x2 + 4x + 3| + 2x + 5 = 0 a a
⇒ –(x2 + 4x + 3) + 2x + 5 = 0 m n
and,  α4 + β4 = − , a 4b 4 =
⇒ x2 + 4x + 3 – 2x – 5 = 0 ⇒ x2 + 2x – 2 = 0 l l
The given equation
−1 ± 4 + 8
⇒ x = = −1 ± 3 a2 lx2 –4aclx + 2c2l + a2m = 0
2
has discriminant
⇒ x = −1 − 3 because −1 + 3 does not lie between D = 16 a2 c2 l2 – 4a2 l (2c2 l + a2 m)
–3 and –1. = 8a2c2 l 2 –4a4 lm
Hence, we have either x = –4 or x = −1 − 3 .
⎛ 2c 2 m ⎞ ⎡ −m ⎤
The correct option is (B) and (C) = 4a4l2 ⎜ 2 − ⎟ > 0 ⎢ = a 4 + b 4 > 0⎥
⎝ a l⎠ ⎣ l ⎦
119. The roots of the equation
10x3 – cx2 – 54x – 27 = 0 are in H.P. Hence, the roots are real.
Putting x = 1/y, we get Also, we have,
2c 2l + a 2 m
27y3 + 54y2 + cy – 10 = 0 has roots in A.P. product of the roots =
a 2l
Let the roots of the equation in y be
2c 2 m
α – β, α, α + β. Then, = + = 2a 2 b 2 – (α 4 + β 4)
−54 a2 l
sum of roots = α – β + α + α + β = 3α = = –2
27 = –(a 2 – b 2)2 < 0
∴ α = –2/3
−2 ∴ The roots are of opposite signs.
Q α = satisfies the equation The correct option is (A) and (C)
3
8 4 2c 122. Given, a > b > c  (1)
∴ −27 + 54 ⋅ − − 10 = 0 The given equation is
27 9 3
⇒ c = 9. (a + b – 2c)x2 + (b + c–2a) x + (c + a –2b) = 0  (2)
10
Product of the roots = (α – β) (α) (α + β) = Since (2) has a root in the interval (–1, 0),
27
2 ⎛4 ⎞ 10 ∴ f (–1) f (0) < 0
or, − ⎜ − b 2 ⎟ =
3 ⎝9 ⎠ 27 ⇒ (2a – b–c) (c + a–2b) < 0  (3)
From (1),
or b2 = 1 ⇒ β = ± 1.
5 2 1 a > b ⇒ a – b > 0 and a > c ⇒ a – c > 0
∴ Roots of y-equation are − , − ,
3 3 3 ∴ 2 a – b – c > 0  (4)
3 3 From (3) and (4), c + a – 2b < 0
or, roots of x-equation are − , − , 3.
5 2 ⇒ c + a < 2b
The correct option is (A), (B) and (C) The correct option is (A)
120. Given equation is x2 + 9y2 – 4x + 3 = 0 (1) Again, sum of the coefficients of the equation = a + b – 2c +
b + c– 2a + c + a – 2b = 0, ∴ one root is 1 and the other root
or, x2 – 4x + 9y2 + 3 = 0.
c + a − 2b
Since x is real, ∴ (–4)2 – 4 (9y2 + 3) ≥ 0 is , which is a rational no.
a + b − 2c
⇒ 16 – 4 (9y2 + 3) ≥ 0 ⇒ 4 – 9y2 – 3 ≥ 0 (∵ a,b, c are rational)
⇒ 9y2 – 1 ≤ 0 ⇒ (3y – 1) (3y + 1) ≤ 0 ∴ both the roots of the equation are rational
−1 1 The correct option is (B)
⇒ ≤ y≤ .
3 3 Since c + a < 2b
Equation (1) can also be written as ⇒ 4b2 > (c + a)2 = c2 + a2 + 2ac
9y2 + 0y + x2 – 4x + 3 = 0 ⇒ 4b2 – 4ac > c2 + a2 – 2ac = (c – a)2 > 0
Since y is real ∴ Discriminant of ax2 + 2bx + c > 0
∴ 02 – 4.9 (x2 – 4x + 3) ≥ 0

Objective_Maths_JEE Main 2017_Ch 4.indd 45 01/01/2008 03:38:13


4.46  Chapter 4

Also, each of a, b, c is positive, The given equation will have four real roots if (1) has two
∴ the equation ax2 + 2bx + c = 0 has real and negative roots non-negative roots. This can happen if
The correct option is (C) −b
≥ 0, af (0) ≥ 0, b2 – 4ac ≥ 0
Similarly, (D) is also correct. a
⇒ – ab ≥ 0, ac ≥ 0 [∵ b2 – 4ac ≥ 0 is given]
The correct option is (A), (B), (C) and (D)
Thus, a and b must have opposite signs whereas a, – b and
123. ∵ Roots are real
c must have the same sign.
∴ B2 – 4AC > 0 ⇒ a4 > 4b2
⇒ a > 0, b < 0, c > 0 or  a < 0, b > 0, c < 0.
⇒ (A) is correct.
The correct option is (C) and (D)
If f (x) = x2 + a2x + b2

125. The quadratic equation
∵ c lies outside the roots.
x2 + (a – b) x – a – b + 1 = 0
∴ f (c) > 0, ⇒ c2 + a2c + b2 > 0
will have unequal real roots if
⇒ (B) is correct.
D = (a – b)2 + 4 ( a + b – 1) > 0
Further, if α, β are the roots, then α > c and β > c ⇒ α + β > 2c
⇒ b2 + (4 – 2 a) b + a2 + 4a – 4 > 0  (1)
−a2 This inequality will hold for all b ∈ R if and only if discrim-
⇒ – a2 > 2c ⇒ >c
2 inant of the quardratic expression on l.h.s. of (1) < 0
⇒ (C) is correct. ⇒ ( 4 – 2a)2 – 4 (a2 + 4a – 4) < 0
The correct option is (A), (B) and (C) ⇒ 16 – 16 a + 4a2 – 4a2 – 16 a + 16 < 0
124. Put x2 = y. The given equation becomes ⇒ 32 – 32a < 0 ⇒ a > 1
f (y) = ay2 + by + c = 0  (1) The correct option is (B) and (C)

Passage Based Questions


126. We have, x3 + 3x2 + 3x + 2 = 0
16
⇒ (x + 1)3 + 1 = 0 ⇒ – − ≤ a ≤ −0 (1)
7
⇒ (x + 1 + 1) {(x + 1)2 – (x + 1) + 1} = 0
⇒ (x + 2) (x2 + x + 1) = 0 3a 3a
α + β > 2 ⇒ >0 ⇒ −2>0
a +1 a +1
−1 ± 3i
⇒ x = – 2, 3a − 2a − 2 a−2
2 ⇒ >0 ⇒ >0
a +1 a +1
⇒ x = – 2, ω, w 2.
⇒ a < – 1 or a > 2 (2)
Since a, b, c ∈ R, ax2 + bx + c = 0 cannot have one real and and, (a + 1) f (1) > 0.
one imaginary root, therefore, two common roots of ax2 + ⇒ (a + 1) (a + 1–3a + 4a) > 0
bx + c = 0 and x3 + 3x2 + 3x + 2 = 0 are ω, w 2. ⇒ (a + 1) (2a + 1) > 0
b
Thus, − = ω + w 2 = –1 ⇒ a <–1 or a > –1/2  (3)
a
c From (1), (2) and (3), we get:
⇒ a = b and = ω ⋅ w 2 = 1
a –16
⇒ c = a ≤ a < –1 i.e., a ∈ [–16/7, –1)
7
⇒ a = b = c.
The correct option is (A)
The correct option is (C)
128. Let f (x) = x2 + 2 (a – 3) x + 9. If 6 lies between the roots of
127. Let f (x) = (a + 1)x2 – 3ax + 4a and let α,β be the roots of
f (x) = 0, then we must have the following:
the equation f (x) = 0. The equation will have roots greater
than 1 iff (i)  Disc > 0, and
  (i) Disc. ≥ 0 (ii)  f (6) < 0 (∵ coeff. of x2 is positive).
 (ii)  α + β > 2 Now,
(iii) (a + 1) f (1) > 0 Disc > 0 ⇒ 4 (a – 3)2 – 36 > 0 ⇒ (a – 3)2 – 9 > 0
Now, Disc. ≥ 0 ⇒ 9a2 –16a (a + 1) ≥ 0 ⇒ a2 – 6a > 0
⇒ –7 a2 – 16a ≥ 0 ⇒ a (a – 6) > 0 ⇒ a < 0 or a > 6 (1)
⇒ a (7a + 16) ≤ 0 and,  f (6) < 0 ⇒ 36 + 12 (a – 3) + 9 < 0

Objective_Maths_JEE Main 2017_Ch 4.indd 46 01/01/2008 03:38:15


Quadratic Equations and Expressions  4.47

3 131. When x > 0, Pn (x) > 0 and so Pn (x) = 0 can have no positive
⇒ 12 a + 9 < 0 ⇒ a < – (2)
4 real roots.
From (1) and (2), we get; Now, Pn (x) = 1 + 2x + 3x2 + … + (n +1) xn
a < –3/4 i.e., a ∈ (–∞, –3/4).
⇒ xPn (x) = x + 2x2 + 3x3 + … + n xn + (n + 1) xn + 1
The correct option is (B)
⇒ (1 – x)Pn (x) = 1+ x + x2 + … + xn – (n + 1) xn + 1
129. Let f (x) = (1– a2) x2 + 2ax – 1.
1 − ( n + 2) x n +1 + ( n + 1) x n + 2
Then, f (x) = 0 has roots between 0 and 1 if ⇒ Pn (x) =
(1 − x ) 2
(i)  Disc ≥ 0 and
For negative values of x, Pn (x) will vanish whenever
(ii)  (1 – a2) f (0) > 0 and (1–a2) f (1) > 0
f (x) = 1 – (n + 2) x n + 1 + (n + 1) xn+ 2 = 0
Now, Disc ≥ 0 ⇒ 4a2 + 4(1–a2) > 0, which is always true.
(1 – a2) f (0) > 0 Now,  f (–x) = 1 – (n + 2) (–1)n + 1 xn+1 + (n + 1) (–1)n + 2 x n + 2

⇒ –(1 – a2) > 0 If n is even, there is no change of sign in this expression and
⇒ a2 – 1 > 0 so there is no negative real root of f (x).
⇒ a < –1 or a > 1 (1) The correct option is (A)
and,  (1 – a2) f (1) > 0 132. As discussed in the above problem, if n is odd, there is one
change of sign therefore f (x) can have at most one negative
⇒ ( 1 –a2) (2a – a2) > 0
real root. In this case
⇒ a (a – 1) (a +1) (a – 2) > 0
f (–1) = – 2n – 2 < 0, f (0) = 1 > 0
⇒ a < – 1 or a > 2 or 0 < a < 1  (2)
The correct option is (C)
From (1) and (2), we get:
So, the negative real root lies between –1 and 0.
a < –1 or a > 2.
133. Let f (x) = (x – a1) (x – a3) (x – a5) + 3 (x– a2) (x – a4) (x – a6)
The correct option is (B)
As x → ∞, f (x) → ∞
130. Let f (x) = x2 – 4ax + 2a2 – 3a + 5. The conditions for both
f (a1) = 3 (a1 – a2) (a1 – a4) (a1 – a6) < 0
the roots to exceed 2 are
Similarly,
  (i) Disc. ≥ 0
f (a2) > 0, f (a3) > 0, f (a4) < 0, f (a5) < 0, f (a6) > 0.
 (ii)  f (2) > 0 and
Thus, f (x) changes sign in each of two intervals (a1, a2),
(iii)  sum of the roots > 4
9 (a3, a4) and (a5, a6). Since f (x) = 0 is a cubic root in x,
Solving these three conditions, we get a > .
2 ∴ It will have I root in each of the above sub-intervals.
⎛9 ⎞
Hence, a ∈ ⎜ , ∞⎟ . The correct option is (A)
⎝2 ⎠
The correct option is (D)

Match the Column Type


134.
  (I) Since the roots of the given equation are real, therefore ⇒ [b (c + a)]2 + (2ac)2 – 2 · 2ac · b (c + a) = 0
the discriminant ≥ 0 ⇒ [b (c + a) – 2ac]2 = 0
⇒  4 (bc + ad)2 – 4 (a2 + b2) (c2 + d2) ≥ 0
⇒  b (c + a) = 2ac
⇒  b2c2 + a2d2 + 2abcd – a2c2 – a2d2 – b2c2 – b2d2 ≥ 0
2ac
⇒ (ac – bd)2 ≤ 0. ⇒  b =
a+c
But (ac – bd)2 cannot be negative as it is a square of ∴  b is H.M. of a and c, i.e., a, b, c are in H.P.
real number
The correct option is (B)
∴  ac – bd = 0; or b2d2 = a2c2.
Let α and β be the roots of the given equation;
(III) 
Hence, a2, bd, c2 are in G.P.
b c
The correct option is (C) then,  α + β = − and aβ =  .
a a
(II) Since the roots are equal,
1 1
∴  B2 – 4AC = 0 Given,  α+β= 2 + 2
a b
⇒  b2 (c – a)2 – 4ac (b – c) (a – b) = 0
⇒  b2 (c2 + a2 – 2ac) – 4ac [ab – ac – b2 + bc] = 0 a 2 + b2 (a + b ) 2 − 2ab
= 2
= ,
⇒  b2 (c2 + a2 – 2ac + 4ac) + 4a2c2 – 4abc (c + a) = 0 (ab ) (ab ) 2

Objective_Maths_JEE Main 2017_Ch 4.indd 47 01/01/2008 03:38:16


4.48  Chapter 4

b 2 2c Now, (α – γ ) (α – δ ) = a2 – α (γ + δ ) + gδ



b 2 b 2 − 2ca = a2 – α(α + β) + r
⇒  − = a 2 a =
a c c2 = –aβ + r = –(–q) + r = q + r
a2 Similarly, (β – γ ) (β – δ ) = q + r
⇒ 2ca = bc + ab2
2 2
∴  Ratio is 1.

Hence, bc2, ca2 and ab2 are in A.P. The correct option is (D)
The correct option is (A)  (II)  Put 5x = y. Then, the given equation becomes
2
We have, (a2 + b2 + c2) p2 – 2 (ab + bc + cd) p + (b2 +
(IV)  1 ⎛ 1 ⎞
c2 + d2) ≤ 0 sin (ex) = y + = ⎜ y− x
⎟ + 2 [Q 5 > 0]
y ⎝ y⎠
⇒ (ap – b)2 + (bp – c)2 + (cp – d)2 ≤ 0
 ⇒  sin (ex ) ≥ 2. which is not possible for any real
⇒ (ap – b)2 + (bp – c)2 + (cp – d)2 = 0
value of x.
(Q a, b, c, d, p ∈ R)
Hence, the given equation has no real solution.
⇒  ap – b = 0, bp – c = 0, cp – d = 0
The correct option is (A)
b c d (III)  We have,
⇒  = = =p
a b c x – 2 = 22/3 + 21/3. Cube both sides, we get
⇒  a, b, c, d are in G.P. (x – 2)3 = 22 + 2 + 3 ⋅ 22/3 ⋅ 21/3 (x – 2)
The correct option is (C) = 6 + 6 (x – 2)
135.   (I) We have, α + β = –p or,  x – 6x2 + 12x – 8 = – 6 + 6x.
3

γ + δ = –p ∴  x3 – 6x2 + 6x = 2.
⇒  α + β = γ + δ The correct option is (B)

Assertion-Reasoning Type
1 36. We know that if α, β are roots of the equation (α + β) < 0 ⇒ –2 (k + 1) < 0
Ax2 + Bx + C = 0, ⇒ k + 1 > 0 ⇒ k > –1  (2)
5
B 2 − 4 AC and,  aβ > 0 ⇒ 9k – 5 > 0 ⇒ k >  (3)
then, α–β= . 9
A The correct option is (D)
Equating the value of α – β from both the given equations, 2
138. Let α, β be the roots of x + bx + ca = 0 and α, γ  be the roots
we get of x2 + cx + ab = 0, then we have,
b 2 − 4c = c 2 − 4b ⇒ b2 – 4c = c2 – 4b a2 + bα + ca = 0 and a2 + cα + ab = 0
2 2
⇒ b – c = – 4 (b – c) ⇒ (b – c) (b + c + 4) = 0 Subtracting, we have,
⇒ b + c = – 4(Q b ≠ c) (b – c) α + a( c – b) = 0 ⇒ α = a
The correct option is (A) Putting α = a in equation x2 + bx + ca = 0, we have
137. Let f (x) = x2 + 2 (k + 1) x + 9k – 5. Let α, β be the roots of a2 + ab + ca = 0
f (x) = 0. The equation f (x) = 0 will have both negative roots i.e.,  a + b + c = 0 (1)
if and only if Also, we have
  (i) Disc. ≥ 0 aβ = ca and aγ = ab
 (ii)  α + β < 0 and ⇒ β = c and γ = b
(iii)  f (0) > 0 Now, β + γ  = b + c and bγ = bc. Hence β, γ  will be the roots
Now, discriminant ≥ 0 of the equation
⇒ 4 (k + 1)2 – 36k + 20 ≥ 0 x2 – (b + c) x + bc = 0
⇒ k2 – 7k + 6 ≥ 0 i.e., x2 + ax + bc = 0. [Using (1)]
⇒ (k – 1) (k – 6) ≥ 0 The correct option is (A)
⇒ k ≤ 1 or k ≥ 6  (1)

Objective_Maths_JEE Main 2017_Ch 4.indd 48 01/01/2008 03:38:18


Quadratic Equations and Expressions  4.49

Previous Year’s Questions


139. Key Idea : The equation having α and β as its roots, is 1 42. x2 – 3 | x | + 2 = 0
x2− (α + β )x + aβ = 0. ⇒ (| x | –1) (| x | – 2) = 0, as | x |2 = x2
Since a2 = 5α − 3 ⇒ α2− 5α + 3 = 0
⇒ x = ±1, ± 2 .
and b 2 = 5β − 3 ⇒ b 2− 5β + 3 = 0 The correct option is (B)
The above two equations imply that α and β are the roots of 143. β = 2α
the equation 1 − 3a
x2−5x + 3 = 0. 3a = 2
a − 5a + 3
α + β = 5 and αβ = 3 2
2a 2 = 2
a b a 2 + b 2 (a + b ) 2 − 2ab a − 5 a+3
Now + = =
b a ab ab (3a − 1) 2 1
So, =
25 − 6 19 9( a 2 − 5a + 3) 2 a 2 − 5a + 3
= =
3 3 2
⇒a= .
a b 3
and . = 1
b a The correct option is (A)
a b 144. (1 − p)2 + p (1 − p) + (1 − p) = 0(since (1 − p) is a root of
Thus, the equation having and as its roots is given by
b a the equation x2 + px + (1 − p) = 0)
⎛a b⎞ a b ⇒ (1− p)(1− p + p + 1) = 0
x2 − ⎜ + ⎟ x + ⋅ = 0
⎝ b a⎠ b a ⇒ 2(1− p) = 0 ⇒ (1 − p) = 0 ⇒ p = 1

19 Now, the sum of roots is α + β = -p and the product αβ =
⇒ x2 − x +1= 0 1 − p = 0 (where β = 1 − p = 0)
3
⇒ α + 0 = –1 ⇒ α = –1 ⇒ Roots are 0, –1
⇒ 3 x 2 − 19 x + 1 = 0
The correct option is (C)
The correct option is (A)
145. Since 4 is one of the root of x2 + px + 12 = 0, we have
140. Key Idea : If the discriminant of ax2 + bx + c = 0 is positive,
then this equation has two real roots. 16 + 4p + 12 = 0 ⇒ p = −7
We have 32 x − 7 x + 7 = 32
2
And the equation x2 + px + q = 0 has equal roots
49
⇒ 2x2 − 7x + 7 = 2 ⇒ D = 49 − 4q = 0 ⇒ q = .
4
⇒ 2x2 − 7x + 5 = 0 The correct option is (A)

D = b 2 − 4 ac ax 3 bx 2
146. Let f  ′(x) = ax2 + bx + c, then f ( x ) = + + cx + d
3 2
= ( −7) 2 − 4 × 2 × 5
\ 1
= 49 − 40 ⇒ f ( x ) = ( 2ax 3 + 3bx 2 + 6cx + 6 d ), Now f (1) = f (0) = d,
6
=9
then according to Rolle’s theorem
Since the discriminant is positive, the equation has two real 2
roots. ⇒ f  ′(x) = ax + bx + c = 0 has at least one root in (0, 1)
The correct option is (B) The correct option is (A)
141. Let α, b be the roots 147. x2− (a − 2)x − a − 1 = 0
1 1 Sum of roots, α + β = a − 2
a+b = 2 + 2 Product, αβ = − (a + 1)
a b
α2 + β2 = (α + β)2− 2αβ
a+b =
(a + b )2 − 2ab = a2− 2a + 6 = (a − 1)2 + 5
(ab ) 2 ⇒ a = 1
2
⎛ b ⎞ b − 2ac The correct option is (A)
⎜⎝ − ⎟⎠ =
a c2 148. Let α, α + 1 be the roots of the equation, then
2 2 α + (α + 1) = b
⇒ 2a c = b(c + ba)
a b c α(α + 1) = c
⇒ , , are in HP ∴  b2− 4c = (2α + 1)2 − 4α (α + 1) = 1
c a b
The correct option is (C) The correct option is (D)

Objective_Maths_JEE Main 2017_Ch 4.indd 49 01/01/2008 03:38:22


4.50  Chapter 4

Minimum value
−b
149. = 2k < 10 ⇒ k < 5 4(3b 2 )( 2c 2 ) − 36b 2c 2 12b 2c 2
a = 2
=− = − c 2 > −4 ab.
c 4(3b ) 12b 2
or, = k 2 + k − 5 < 25 ⇒ ( k + 6)( k − 5) < 0 The correct option is (C)
a
⇒k <5 1± 1− 4
155. x2 − x + 1 = 0 ⇒x=
⇒ k ∈ ( −∞, 4) 2
The correct option is (C) 1 ± 3i
x=
150. Equation x2− 2mx + m2− 1 = 0 2
⇒ (x − m)2− 1 = 0
1 3 π π
⇒ (x − m + 1) (x − m − 1) = 0 a= +i , b = cos − i sin
2 2 3 3
⇒ x = m − 1, m + 1
⎛π⎞
According to question a 2009 + b 2009 = 2 cos 2009 ⎜ ⎟
⎝ 3⎠
− 2 < m − 1, m + 1 < 4
⇒ m > − 1 and m < 3 ⎡ 2π ⎤ ⎛ 2π ⎞
= 2 cos ⎢668π + π + = 2 cos ⎜ π + ⎟
⇒ − 1 < m < 3. ⎣ 3 ⎥⎦ ⎝ 3⎠
The correct option is (C) 2π ⎛ 1⎞
= −2 cos = −2 ⎜ − ⎟ = 1
2
3 x + 9 x + 17 3 ⎝ 2⎠
151. Let y = , then
3x 2 + 9 x + 7 The correct option is (B)
3x2 (y − 1) + 9x (y − 1) + 7y − 17 = 0
156. esin x − e − sin x = 4 ⇒ esin x = t
Now, D ≥ 0 (Q x is real) implies that
1
81(y − 1)2 − 4x3 (y − 1) (7y − 17) ≥ 0 t− −4
t
⇒ (y − 1) (y − 41) ≤ 0 ⇒ 1 ≤ y ≤ 41 4 ± 16 + 4
The correct option is (C) t 2 − 4t − 1 − 0 ⇒ t =
2
152. Given equation x2 + ax + 1 = 0
4±2 5
The sum and product of the roots ⇒t = ⇒t = 2± 5
2
α + β = −a 1
αβ = 1 esin x = 2 ± 5 −1 ≤ sin x ≤ 1 ≤ esin x ≤ e
e
Now, | α − β | = (a + b ) 2 − 4ab sin x
= 2 + 5 not possible
e
sin x
⇒ |α − β | = a2 − 4 e = 2 − 5 not possible
∴ Hence no solution
⇒ a2 − 4 < 5
The correct option is (B)
⇒ a − 4 < 5 ⇒ a2− 9 < 0
2
157. If 2 x 3 + 3 x + k = 0 has 2 distinct real roots in [0, 1], then
⇒ a ∈ (−3, 3)
f  ′ (x) will change sign.
The correct option is (A)
But f  ′(x) = 6x2 + 3 > 0
153. Let α and 4β be roots of x2− 6x + a = 0 and α, 3β be the
So, no value of k exists.
roots of x2− cx + 6 = 0, then
The correct option is (C)
α + 4β = 6 and 4αβ = a
158. Both the roots of the equation x 2 + 2 x + 3 = 0 are
α + 3β = c and 3αβ = 6 imaginary.
We get αβ = 2 ⇒ a = 8 Since, a, b, c ∈ R.
So the first equation is x2− 6x + 8 = 0 ⇒ x = 2, 4 So, if one root is common then both roots are common
Now, if α = 2 and 4β = 4 then 3β = 3
a b c
If α = 4 and 4β = 2, then 3β = 3/2 (non-integer) Hence, = =
1 2 3
∴ common root is x = 2.
The correct option is (D) a : b : c = 1 : 2 : 3.
The correct option is (D)
154. Given equation bx2 + cx + a = 0 has imaginary roots
159. a 2 = 3t 2 − 2t
⇒ c2− 4ab < 0 ⇒ c2< 4ab ⇒ –c2 > −4ab
For non-integral solution
Since 3b2 > 0, the expression 3b2x2 + 6bcx + 2c2 has 2
­minimum value. 0 < a < 1

Objective_Maths_JEE Main 2017_Ch 4.indd 50 01/01/2008 03:38:27


Quadratic Equations and Expressions  4.51

Subtract (2) from (1)


a ∈ ( −1, 0) ∪ (0,1).
[Note: It is assumed that a real solution of given equation ⇒ a10 = 6 a9 + 2a8
exists.]
a10 − 2a8
The correct option is (A) ⇒ = 3.
2a9
160. x 2 = 6 x + 2 ⇒ a 2 = 6a + 2
The correct option is (B)
⇒ a 10 9
= 6a + 2a  8 (1)
and b10 = 6b 9 + 2b 8  (2)

Objective_Maths_JEE Main 2017_Ch 4.indd 51 01/01/2008 03:38:28


Matrices 5.1

CHaPtER

5 Matrices

Chapter Highlights
Matrix, Types of matrices, Algebra of matrices, Symmetric matrix, Orthogonal matrix, Idempotent matrix,
Involutory matrix, Nilpotent matrix, Singular matrix

MATrix TYpES of MATricES


A rectangular array of mn numbers in the form of m row Matrix
horizontal lines (called rows) and n vertical lines (called
columns), is called a matrix of order m by n, written as A matrix having only one row is called a row matrix or a
m × n matrix. row vector. For example, A = [2 3 –4 1] is a row matrix of
Such an array is enclosed by [ ] or ( ) or || ||. An m × n order 1 × 4.
matrix is usually written as
column Matrix
⎡ a11 a12 a13 … a1n ⎤
⎢a ⎥ A matrix having only one column is called a column matrix
a22 a23 … a2 n
A = ⎢ 21 ⎥ or a column vector.
⎢ M M M M ⎥ ⎡ 2 ⎤
⎢ ⎥
⎣ am1 am 2 am3  amn ⎦ For example, A = ⎢⎢ 7 ⎥⎥ is a column matrix of order
In compact form the above matrix is represented by A = ⎢⎣ -3 ⎥⎦
[aij] × n. The numbers a11, a12, … etc. are known as elements 3 × 1.
of the matrix A, aij belongs to the ith row and jth column
and is called the (i, j)th element of the matrix A = [aij]. Zero Matrix or null Matrix
⎡3 7 2⎤ A matrix each of whose elements is zero, is called a zero
For example, A = ⎢ ⎥ is a matrix hav- matrix or a null matrix.
⎣ 0 -1 9 ⎦
⎡0 0⎤ ⎡0 0 0⎤
ing 2  rows and 3 columns. Its order is 2 × 3 and it has For example, the matrices ⎢ ⎥ ⎢ ⎥ , are
6 elements: ⎣0 0⎦ ⎣0 0 0⎦
null matrices of order 2 × 2 and 2 × 3 respectively.
a11 = 3, a12 = 7,
a13 = 2, a21 = 0, Square Matrix
a22 = –1, a23 = 9. A matrix in which number of rows is equal to the number of
columns, say n, is called a square matrix of order n.
⎡ 1 -3 4 ⎤
caution ⎡1 2⎤ ⎢ ⎥
For example, the matrices ⎢ ⎥, ⎢3 4 2⎥
⎣ 3 4 ⎦ ⎢
m × n does not indicate multiplication ⎣ 5 3 6 ⎥⎦
are square matrices of order 2 and 3 respectively.

Objective_Maths_JEE Main 2017_Ch 5.indd 1 01/01/2008 04:07:57


5.2  Chapter 5

Diagonal Matrix ⎡2 0 0 0⎤
⎢3 4 0 0 ⎥⎥
A square matrix A = [aij]n × n is called a diagonal matrix if
A= ⎢ is a lower triangular matrix.
all the elements except those in the leading diagonal are ⎢1 3 5 0⎥
zero, i.e., aij = 0 for i ≠ j. In other words ⎢ ⎥
⎣2 4 6 7⎦
A = diag. [a11 a22 a33 … ann]
Trace of a Matrix
⎡6 0 0 ⎤
The sum of the diagonal elements of a square matrix A is
For example, the matrix A = ⎢⎢ 0 4 0 ⎥⎥ . is a diagonal called the trace of A and is denoted by tr (A). For example,
⎢⎣ 0 0 -2 ⎥⎦ if
matrix, and is denoted by A = diag. [6 4 –2]. ⎡ 1 3 4⎤
A = ⎢⎢ 2 -1 6 ⎥⎥ , then tr (A) = 1 – 1 + 4 = 4.
Scalar Matrix ⎢⎣ 3 1 4 ⎥⎦
A square matrix in which every non-diagonal element is
zero and all diagonal elements are equal, is known as scalar Sub Matrix
matrix. A matrix which is obtained from a given matrix by delet-
⎡5 0⎤ ing any number of rows or columns or both is called a
For example, the matrices A = ⎢ ⎥ and B  =
⎣0 5⎦ ­sub-matrix of the given matrix. For example,
⎡2 0 0⎤ ⎡5 3 1 ⎤
⎢ 0 2 0 ⎥ are scalar matrices of order 2 and 3 respectively. ⎡ 2 -1 ⎤ ⎢ ⎥
⎢ ⎥ ⎢ 3 5 ⎥ is a sub-matrix of the matrix ⎢ 4 2 -1 ⎥ .
⎢⎣ 0 0 2 ⎥⎦ ⎣ ⎦ ⎢⎣ 6 3 5 ⎥⎦

Unit Matrix Equality of Matrices


A square matrix in which every non-diagonal element is Two matrices A and B are said to be equal if they are of
zero and every diagonal element is 1, is called a unit matrix same order and all the corresponding elements are equal. It
or an identity matrix. Thus, a square matrix A = [aij]n × n is is written as A = B. For example,
a unit matrix if
⎡2 3 4⎤ ⎡ 2 1+ 2 1+ 3 ⎤
⎧0 when i ≠ j A= ⎢ ⎥ and B = ⎢
aij = ⎨ ⎣5 1 0⎦ ⎣ 2+3 1 0 ⎥⎦
⎩1 when i = j
are equal matrices, whereas
A unit matrix of order n is denoted by In or I. For example,
⎡1 0 0⎤ ⎡3 4⎤ ⎡ 2 5 1⎤
⎡1 0⎤ C= ⎢ ⎥ and ⎢ ⎥
I2 = ⎢ ⎢ ⎥ ⎣2 1⎦ ⎣ 2 3 1⎦
⎥ and I3 = ⎢ 0 1 0 ⎥ are unit matrices
⎣0 1⎦ ⎢⎣ 0 0 1 ⎥⎦ are not equal, because their orders are not same.
of order 2 and 3 respectively.
Algebra of Matrices
Upper Triangular Matrix
A square matrix A = [aij] is called upper triangular matrix if Addition of Matrices
aij = 0 for all i > j. For example, the matrix Let A and B be two matrices each of order m × n. Then the
sum matrix A + B is defined only if matrices A and B are of
⎡1 2 3 4⎤
⎢0 same order. The new matrix, say C = A + B is of order m × n
5 2 3 ⎥⎥
A= ⎢ is an upper triangular matrix. and is obtained by adding the corresponding elements of A
⎢0 0 1 4⎥ and B. Thus, if A = [aij]m × n and B = [bij]m × n are two matri-
⎢ ⎥ ces of same order then the sum A + B is defined to be the
⎣0 0 0 5⎦
matrix of order m × n such that A + B = C = [aij + bij] for
Lower Triangular Matrix all i and j.
⎡ 2 3 4⎤ ⎡ 4 -2 3 ⎤
A square matrix A = [aij] is called a lower triangular matrix For example, if A = ⎢ ⎥ and B = ⎢ ⎥
if aij = 0 for all i < j. For example, the matrix ⎣ 1 0 5⎦ ⎣ 1 1 4⎦

Objective_Maths_JEE Main 2017_Ch 5.indd 2 01/01/2008 04:07:59


Matrices 5.3

⎡ 2 + 4 3 - 2 4 + 3⎤ ⎡6 1 7⎤ For example, if
Then C = A + B = ⎢ ⎥ = ⎢ ⎥,
⎣ 1 + 1 0 + 1 5 + 4⎦ ⎣2 1 9⎦ ⎡2 3 ⎤ ⎡ -3 4 ⎤
whereas the addition of A = ⎢⎢ 6 1 ⎥⎥ and B = ⎢ 2 5⎥
⎢ ⎥
⎢⎣ 7 -2 ⎥⎦ ⎢⎣ 6 3 ⎥⎦
⎡2 3 4⎤ ⎡ 4 -2 ⎤ 
A= ⎢ ⎥ and B = ⎢ 1 1 ⎥
⎣ 1 0 5 ⎦ ⎣ ⎦ ⎡ 2 + 3 3 - 4 ⎤ ⎡ 5 -1 ⎤
is not defined since the two matrices are not of same order. then A – B = ⎢⎢ 6 - 2 1 - 5 ⎥⎥ = ⎢⎢ 4 -4 ⎥⎥ .
If A is any matrix, the negative of A, denoted by – A, is the ⎢⎣ 7 - 6 -2 - 3 ⎥⎦ ⎢⎣ 1 -5 ⎥⎦
matrix obtained by replacing each entry in A by its nega-
tive. For example, if Multiplication of a Matrix by a Scalar
Let A = [aij] be an m × n matrix and k be any scalar. Then
⎡ 2 -1 ⎤ ⎡ -2 1 ⎤
the matrix obtained by multiplying each element of A by k
A = ⎢⎢ 5 4 ⎥⎥ , then – A = ⎢ -5 -4 ⎥
⎢ ⎥ is called the scalar multiple of A by k and is denoted by kA.
⎢⎣ -6 0 ⎥⎦ ⎢⎣ 6 0 ⎥⎦ Thus, if A = [aij]m × n, then kA = [kaij]m × n. For example, if

⎡1 2 3⎤
caution A = ⎢⎢ 4 -6 8 ⎥⎥
⎢⎣ 0 2 5 ⎥⎦
Two matrices cannot be added if they are of different order 
then ⎡1 3⎤
⎢2 1
⎡ 3 6 9 ⎤ 2⎥
1 ⎢ ⎥
Properties of Addition of Matrices 3A = ⎢⎢ 12 -18 24 ⎥⎥ and A = ⎢2 -3 4 ⎥
2 ⎢
1. Matrix addition is commutative. If A and B are two ⎢⎣ 0 6 15 ⎥⎦ 5⎥
matrices of the same order, then A + B = B + A. ⎢0 1 ⎥
⎣ 2⎦
2. Matrix addition is associative. If A, B and C are three
matrices of the same order, then (A + B) + C = A + (B Properties of Scalar Multiplication
+ C). 1. If A and B are two matrices of the same order and k be
3. Existence of additive identity. If O is the zero matrix of a scalar, then
the same order as that of the matrix A, then A + O = A
k (A + B) = kA + kB
= O + A.
4. Existence of additive inverse. If A is any matrix, then A 2. If k1 and k2 are two scalars and A is a matrix, then
+ (– A) = O = (– A) + A. (k1 + k2) A = k1A + k2A
5. Cancellation laws hold good in case of addition of 3. If k1 and k2 are two scalars and A is a matrix, then
matrices. If A, B, C are matrices of the same order,
then (k1k2)A = k1 (k2A) = k2 (k1A)
A+B=A+C⇒B=C 4. If A is any matrix, then 1A = A.
(left cancellation law)
and B + A = C + A ⇒ B = C Multiplication of Matrices
(right cancellation law)
Two matrices A and B can be multiplied only if the number of
columns in A (pre-multiplier) is same as the number of rows
notE in B (post multiplier). For example, if A = [aij]m × n and B =
The zero matrix plays the same role in matrix addition as the [bjk]n × p are two matrices of order m × n and n × p respec-
number zero does in addition of numbers. tively, then their product AB is of order m × p and is defined as
n
(AB)i, k = ∑ aij b jk = ai1 b1k + ai2 b2k + … + ain bnk
j =1
Subtraction of Matrices ⎡ b1k ⎤
Let A and B be two matrices of the same order. Then by ⎢b ⎥
A – B, we mean A + (–B). In other words, to find A – B we = [ai1 ai2 … ain] ⎢ 2 k ⎥
⎢ M ⎥
subtract each element of B from the corresponding element ⎢ ⎥
of A. ⎣ bnk ⎦

Objective_Maths_JEE Main 2017_Ch 5.indd 3 01/01/2008 04:08:01


5.4  Chapter 5

= (ith row of A) (kth column of B). c14 = (1) (6) + (–4) (8) = –26
(AB)ik = Sum of the product of elements of ith row of A with c22 = (5) (4) + (3) (7) = 41
the corresponding elements of kth column of B.
c23 = (5) (1) + (3) (3) = 14

caution c24 = (5) (6) + (3) (8) = 54


c31 = (0) (–2) + (2) (2) = 4
If the number of columns in A is not equal to the number
of rows in B, we cannot find the product AB of the two c32 = (0) (4) + (2) (7) = 14
matrices A and B.
c33 = (0) (1) + (2) (3) = 6
c34 = (0) (6) + (2) (8) = 16.
remarks ⎡ -10 -24 -11 -26 ⎤
Thus, ⎢
AB = ⎢ -4 41 14 54 ⎥⎥
 If A and B are square matrices of the same order, say n,
then both the products AB and BA are defined and each is ⎢⎣ 4 14 6 16 ⎥⎦

a sqaure matrix of order n.
The product BA is not defined since the number of ­columns
 In the matrix product AB, the matrix A is called ­pre-multiplier

(prefactor) and B is called post-multiplier (post-factor). of B is not equal to the number of rows of A. This shows that
matrix multiplication is not commutative. That is, for any
 The rule of multiplication of matrices is row column wise

(or  → ↓ wise), viz.., the first row of AB is obtained by two matrices A and B, it is usually the case that AB ≠ BA
multiplying the first row of A with first, second, third, … (even if both products are defined).
columns of B respectively. Similarly second row of A with
first, second, third, … columns of B respectively and so on. Properties of Matrix Multiplication
1. Multiplication is distributive over matrix addition. If
⎡ 1 -4⎤ A, B, C are m × n, n × p and n × p matrices respectively,
⎡ -2 4 1 6 ⎤
Let A = ⎢⎢5 3 ⎥⎥ and B = ⎢ ⎥ then
⎢⎣0 2 ⎥⎦ ⎣ 2 7 3 8⎦
A (B + C) = AB + AC
be two matrices. 2. Multiplication is associative. If A, B, C are matrices of
order m × n, n × p and p × r respectively, then
Since the number of columns in A are equal to the number (AB)C = A (BC)
of rows in B, the product AB is defined. As order of matrix 3. Multiplicative identity. If A is an m × n matrix and In
A is 3 × 2 and B is 2 × 4, the product AB will be of order the identity matrix of order n × n and Im the identity
3 × 4. matrix of order m × m, then
⎡ c11 c12 c13 c14 ⎤ Im A = A and AIn = A
⎢ ⎥
AB = ⎢ c21 c22 c23 c24 ⎥

In particular if A is a square matrix of order n, then
⎢⎣ c31 c32 c33 c34 ⎥⎦
AIn = In A = A
The entry c11 is obtained by summing the products of each 4. AB = 0 (null matrix) does not necessarily imply
entry in row 1 of A by the corresponding entry in column that A = 0 or B = 0 or both = 0. For example, if A =
1 of B, i.e., ⎡0 -1 ⎤
⎢0 ≠ 0 and
c11 = (1) (–2) + (–4) (2) = –10 ⎣ 0 ⎥⎦
Similarly, for c21, we use the entries in row 2 of A and those ⎡1 1⎤ ⎡0 0⎤
in column 1 of B: B= ⎢
⎥ ≠ 0, then AB = ⎢ ⎥
⎣0 0⎦ ⎣0 0⎦
c21 = (5) (–2) + (3) (2) = –4. 5. If A is a square matrix of order n, then A2 is defined as
Also, c12 = (1) (4) + (–4) (7) = –24 AA. In general Am = AA … A (m times), where m is any
positive integer.
c13 = (1) (1) + (–4) (3) = –11 6. If I be a unit matrix, then I = I 2 = I 3 = … = I n.

Objective_Maths_JEE Main 2017_Ch 5.indd 4 01/01/2008 04:08:02


Matrices  5.5

4. If A and B are two matrices such that AB = B and BA =


Solved examples A, then A2 + B2 =
⎡ i 0⎤ (A) 2AB (B)  2BA
4n
1. If A = ⎢ ⎥ , n ∈ N, then A equals (C) A + B (D)  AB
⎣ 0 i ⎦
Solution: (C)
⎡0 i ⎤ ⎡0 0⎤ We have,
(A)  ⎢ ⎥ (B) 
⎢0 0⎥
⎣ i 0 ⎦ ⎣ ⎦ A2 + B2 = AA + BB = A (BA) + B (AB)
⎡1 0⎤ ⎡0 i ⎤ (Q AB = B and BA = A)
(C)  ⎢ ⎥ (D) 
⎢ i 0⎥
⎣0 1⎦ ⎣ ⎦ = (AB) A + (BA) B
Solution: (C) = BA + AB = A + B
We have,  (Q AB = B and BA = A)
⎡ i 0⎤ ⎡ i 0⎤ ⎡i 0 ⎤ ⎡ -1 0 ⎤
2
5. If A and B are square matrices of same order such that
A2 = ⎢ ⎥⎢ ⎥ = ⎢ ⎥=⎢ ⎥.
⎣ 0 i ⎦ ⎣ 0 i ⎦ ⎣⎢ 0 i 2 ⎥⎦ ⎣ 0 -1 ⎦ (A + B)2 = A2 + B2 + 2AB, then
(A) AB = BA (B)  A=B
⎡ -1 0 ⎤ ⎡ -1 0 ⎤ ⎡1 0⎤ (C) A = B′ (D)  A = –B
A4 = ⎢ ⎥ ⎢ ⎥ = ⎢ ⎥
⎣ 0 -1 ⎦ ⎣ 0 -1 ⎦ ⎣0 1⎦ Solution: (A)
⎡1 0⎤ ⎡1 0⎤ ⎡1 0⎤ We have,
⎢ 0 1 ⎥ ⎢ 0 1 ⎥ ... ⎢ 0 1 ⎥
\ A = ⎣
4n ⎦⎣ ⎦ ⎣ ⎦ = ⎡ 1 0 ⎤ . (A + B)2 = A2 + B2 + 2AB
n times ⎢0 1⎥
⎣ ⎦
⇒ (A + B) (A + B) = A2 + B2 + 2AB
2
2. If AB = A and BA = B, then B is equal to
⇒ A2 + AB + BA + B2 = A2 + B2 + 2AB
(A) B (B)  A (C) 1 (D) 0
⇒ BA = AB
Solution: (A)
⎡ i -i ⎤ ⎡ 1 -1⎤ 8
Since BA = B, 6. If A = ⎢ ⎥ and B = ⎢ -1 1 ⎥ the A equals
⎣ -i i ⎦ ⎣ ⎦
\ (BA) B = BB = B2
(A) 64B (B)  –64B
⇒ B (AB) = B2  ⇒  BA = B2(Q AB = A) (C) –128B (D) 
128B
⇒ B = B2(Q BA = B) Solution: (D)
⎡ 3 -4 ⎤ We have,
n
3. If X = ⎢ ⎥ , the value of X  is
⎣ 1 -1 ⎦ ⎡ i - i⎤ ⎡ 1 - 1⎤
A=⎢ ⎥ =i ⎢ ⎥ = i B
⎣- i i ⎦ ⎣ -1 1 ⎦
⎡ 3n -4 n ⎤ ⎡ 2+n 5-n⎤
(A)  ⎢ ⎥ (B) 
⎢ n - n ⎥⎦ ⎡ 1 - 1⎤ ⎡ 1 - 1⎤
⎣ n -n ⎦ ⎣ ⇒ A2 = (iB)2 = i2 B2 = –B2 = – ⎢ ⎥⎢ ⎥
⎡ 3n ⎣ -1 1 ⎦ ⎣ -1 1 ⎦ 
( -4) n ⎤
(C)  ⎢ ⎥ (D)  None of these ⎡- 2 2 ⎤
⎢⎣ 1n ( -1) n ⎥⎦ = ⎢ ⎥ = – 2B
⎣ 2 - 2⎦
Solution: (D)
We have, ⇒ A4 = (–2B)2 = 4B2 = 4(2B) = 8B
⎡ 3 -4 ⎤ ⎡ 3 -4 ⎤ ⎡ 5 -8 ⎤ ⇒ A8 = (A4)2 = (8B)2 = 64B2 = 128B
X2 = X × X = ⎢ ⎥ ⎢ ⎥ = ⎢ ⎥
⎣ 1 -1 ⎦ ⎣ 1 -1 ⎦ ⎣ 2 -3 ⎦ 7. If A and B are two matrices such that A + B and AB are
For n = 2, matrices in (A), (B) and (C) do not match with both defined, then
(A) A and B are two matrices not necessarily of same
⎡ 5 -8 ⎤
⎢ 2 -3 ⎥ order
⎣ ⎦ (B) A and B are square matrices of same order

Objective_Maths_JEE Main 2017_Ch 5.indd 5 01/01/2008 04:08:05


5.6  Chapter 5

(C)  number of columns of A = number of rows of B Transpose of a Matrix


(D)  None of the above.
Let A be an m × n matrix. Then, the n × m matrix obtained
Solution: (B) by interchanging the rows and columns of A is called the
Both matrices are of same order for A + B to be defined. transpose of A, and is denoted by A′ or At. Thus,
Now let both are of order m × n. But AB is defined only
1. if order of A is m × n, then, the order of A′ is n × m.
when number of columns in A is same as number of
2. (i, j)th element of A = ( j, i)th element of A′.
rows in B. So both are square matrices of same order.
For example,
⎡ 1⎤
1 ⎡ 2 -3 -1 ⎤
8. If A = ⎢ 2 ⎥ then A is
64
if A= ⎢
⎢ ⎥ ⎥ ,
⎣0 1⎦ ⎣4 2 3 ⎦

⎡ 1 32⎤ ⎡ 1 0⎤ ⎡ 2 4⎤
(A)  ⎢ ⎥
⎣32 1 ⎦
(B) 
⎢32 1 ⎥
⎣ ⎦ then A′ = ⎢⎢ -3 2 ⎥⎥
⎢⎣ -1 3 ⎥⎦ 3× 2
⎡1 32⎤ 
(C)  ⎢ ⎥ (D)  None of these
⎣0 1 ⎦ Properties of the Transpose of a Matrix
Solution: (C) 1. Let A and B be two matrices of order m × n, then
(A ± B)′ = A′ ± B′.
⎡ 1⎤ ⎡ 1⎤
1 ⎢1 ⎡1 1⎤ 2. Let A be a matrix of order m × n and k be a scalar, then
A2 = ⎢ 2⎥ 2⎥ = ⎢ ⎥ (kA)′ = kA′.
⎢ ⎥ ⎢ ⎥ ⎣0 1⎦ 
⎣0 1⎦ ⎣0 1⎦ 3. Let A and B be two matrices of order m × n and n × p
respectively. Then, (AB)′ = B′A′.
⎡1 1⎤ ⎡1 1⎤ ⎡1 2⎤
A4 = ⎢ ⎥ ⎢ ⎥ = ⎢ ⎥ 4. The double transpose of any matrix is the original
⎣0 1⎦ ⎣0 1⎦ ⎣0 1 ⎦  matrix. For example, if A is any matrix, then (A′)′ = A.
⎡1 2⎤ ⎡1 2⎤ ⎡1 4 ⎤
A8 = ⎢ ⎥ ⎢ ⎥ = ⎢ ⎥
⎣0 1⎦ ⎣0 1⎦ ⎣0 1 ⎦ Solved Examples

⎡1 32⎤ 10. If A is 3 × 4 matrix and B is a matrix such that A′B and
Similarly, A64 = ⎢ ⎥
⎣0 1 ⎦  BA′ are both defined. Then B is of the type
(A) 3 × 4 (B)  3 × 3 (C)  4 × 4 (D)  4 × 3
⎡ 1 0⎤ 100
9. If A = ⎢ ⎥ , then A is equal to Solution: (A)
⎣1/2 1 ⎦
Let the order of B be m × n.
⎡ 1 0⎤ ⎡ 1 0⎤ Since A is 3 × 4 matrix,
(A)  ⎢ ⎥ (B) 
⎢50 1 ⎥
⎣110 1 ⎦ ⎣ ⎦ \  A′ is 4 × 3 matrix.
Since A′ B is defined,
⎡ 1 0⎤ \  number of columns of A′ must be equal to
(C)  ⎢ ⎥ (D)  None of these
⎣ 25 1 ⎦ number of rows of B,
\  m = 3.
Solution: (B) Also, since BA′ is defined,
\  number of columns of B must be equal to
⎡ 1 0 ⎤ ⎡ 1 0 ⎤ ⎡1 0 ⎤ ⎡ 1 0⎤
A2 = ⎢ ⎥ ⎢ ⎥ = ⎢ ⎥ = ⎢ ⎥ number of rows of A′,
⎣1/2 1 ⎦ ⎣1/2 1 ⎦ ⎣1 1 ⎦ ⎣ 2(1/2) 1 ⎦  \  n = 4.
\  B is 3 × 4 matrix.
⎡ 1 0⎤ ⎡ 1 0⎤ ⎡ 1 0⎤
A3 = A2 × A = ⎢ ⎥ ⎢ ⎥ = ⎢ ⎥ ⎡ 3
⎣ 2(1/2) 1 ⎦ ⎣1/2 1 ⎦ ⎣3(1/2) 1 ⎦  1 ⎤
⎢ ⎥ ⎡1 1⎤
2 2 ⎥
Continuing in this way, we get 11. If P = ⎢ ,A= ⎢ T
⎥ and Q = PAP , then
⎢ –1 3 ⎥ ⎣ 0 1⎦
⎢ ⎥
⎡ 1 0⎤ ⎡ 1 0⎤ ⎣ 2 2 ⎦
A100 = ⎢ ⎥ = ⎢ ⎥
⎣100(1/2) 1 ⎦ ⎣50 1 ⎦  PT(Q2005)P is equal to

Objective_Maths_JEE Main 2017_Ch 5.indd 6 01/01/2008 04:08:09


Matrices 5.7

⎡1 2005⎤ ⎡ 3/ 2 2005⎤
(A) ⎢ (B) ⎢ ⎥ i M P o R ta n t P o i n t S
⎣0 1 ⎥⎦ ⎣ 1 0 ⎦
Elements of main diagonal of a skew-symmetric matrix are
⎡ 1 2005⎤ ⎡1 3/ 2 ⎤ all zero, because by definition,
(C) ⎢ ⎥ (D) ⎢ ⎥
⎣ 3/ 2 1 ⎦ ⎣0 2005 ⎦ aii = –aii ⇒ 2aii = 0
or aii = 0 for all values of i.
Solution: (A)
We have,
Q = P A PT Properties of Symmetric and
Skew-symmetric Matrices
⇒ PT Q = APT (Q as PTP = I = PPT) 1. If A is a square matrix, then
(a) A + A′ is symmetric
\ PT Q2005 P = A PT Q2004 P
(b) A – A′ is skew-symmetric.
= A2 PT Q2003 P = A3 PT Q2002 P 2. If A and B are two symmetric (or skew-symmetric)
matrices of the same order, then so is A + B.
= A2004 PT (QP) 3. If A is symmetric (or skew-symmetric) matrix and k is a
scalar, then kA is also symmetric (or skew-symmetric).
= A2004 PT (PA) 4. If A and B are symmetric matrices of the same order,
then the product AB is symmetric if and only if
(Q = P A PT ⇒ Q P = P A) = A2005 AB = BA.
5. Every square matrix can be expressed uniquely as the
⎡1 2005⎤
= ⎢ sum of a symmetric and a skew-symmetric matrix.
⎣0 1 ⎥⎦ 6. The matrix B′AB is symmetric or skew-symmetric

according as A is symmetric or skew-symmetric.
SYMMETric MATrix 7. All positive integral powers of a symmetric matrix are
symmetric.
A square matrix A is said to be symmetric if A′ = A. That 8. All positive odd integral powers of a skew-symmetric
is, the matrix A = [aij]n × n is said to be symmetric provided matrix are skew-symmetric and positive even integral
aij = aji for all i and j. powers of a skew-symmetric matrix are symmetric.
For example, 9. If A and B are symmetric matrices of the same order,
then
⎡2 1 5 ⎤ (a) AB – BA is a skew-symmetric matrix.
A = ⎢⎢ 1 0 -3 ⎥⎥ is symmetric, (b) AB + BA is a symmetric matrix.
⎢⎣ 5 -3 6 ⎥⎦ 10. If A is any square matrix, then AA′ and A′A are both
symmetric matrices.
Since
⎡2 1 5 ⎤ SoLvED ExAMpLES
A′ = ⎢⎢ 1 0 -3 ⎥⎥ = A
⎢⎣ 5 -3 6 ⎥⎦ 12. Which of the following is correct?
(A) B′AB is symmetric if A is symmetric
Skew Symmetric Matrix (B) B′AB is skew-symmetric if A is symmetric
(C) B′AB is symmetric if A is skew-symmetric
A square matrix A is said to be skew symmetric, if A′ = –A. (D) B′AB is skew-symmetric if A is skew-symmetric
That is, the matrix A = [aij]n × n is skew-symmetric if aij =
–aji for all i and j. Solution: (A, D)
For example, Let A be a symmetric matrix.
⎡ 0 5 7⎤ Then A′ = A.
A = ⎢ -5 0 3 ⎥⎥ = –A

Now, (B′AB)′ = B′A′ (B′)′ [Q (AB)′ = B′ A′ ]
⎢⎣ -7 -3 0 ⎥⎦
= B′A′B [Q (B′ )′ = B]

Objective_Maths_JEE Main 2017_Ch 5.indd 7 01/01/2008 04:08:10


5.8  Chapter 5

= B′AB[Q A′ = A] 1 ⎡ 2 -2 ⎤ ⎡ 2 2 ⎤ 1 ⎡ 8 0 ⎤
Also, AA′ = = 
⇒ B′AB is a symmetric matrix. 8 ⎣⎢ 2 2 ⎥⎦ ⎢⎣ -2 2 ⎥⎦ 8 ⎢⎣ 0 8 ⎥⎦
Now, let A be a skew-symmetric matrix. ⎡1 0⎤
= ⎢ ⎥ = I
Then A′ = –A. ⎣0 1⎦
\ (B′AB)′ = B′A′(B′)′[Q (AB)′ = B′A′] Similarly, A′A = I.
= B′A′B[Q (B′)′ = B] Hence A is orthogonal.
= B′(–A)B[Q A′ = –A]
Solved Examples
= –B′AB
\ B′AB is a skew-symmetric matrix. 15. If A is an orthogonal matrix, then |A| is
13. If A is symmetric as well as skew symmetric matrix, (A) 1 (B) –1
then A is (C)  0 (D)  None of these
(A) diagonal (B) null Solution: (A, B)
(C)  triangular (D)  None of these Since the matrix A is orthogonal
Solution: (B) A′A = AA′ = In
Let A = [aij] ⇒ |A′A| = |AA′| = |In|
Since A is skew-symmetric,
⇒ |A′| |A| = 1
\ aii = 0 and aij = –aji (i ≠ j)
⇒ |A| |A| = 1
A is symmetric as well, so aij = aji for all i and j.
or |A|2 = 1
\ aij = 0 for all i ≠ j
\ |A| = ±1
Hence, aij = 0 for all i and j i.e. A is a null matrix.
⎛ 0 2b c ⎞
14. If A is a 3 × 3 skew-symmetric matrix, then trace of
16. Let A = ⎜ a b – c⎟ be an orthogonal matrix then
A is ⎜ ⎟
(A) |A| (B)  1 ⎝ a –b c ⎠
(C)  –1 (D)  None of these the values of a, b, c are
1 1
Soluiton: (A) (A) b = ± ,c=±
As A is a skew symmetric matrix 6 3
1 1
⇒ aii = 0 ∀ i  ⇒ trace(A) = 0 (B)  a = ± ,c=±
2 6
Also, |A| = |A′| = |–A| = (–1)3|A| 1 1
(C) a = ± ,b=±
⇒ 2|A| = 0  ⇒ |A| = 0 2 6
(D)  All of these
\ trace(A) = |A|.

Solution: (D)
Orthogonal Matrix ⎛ 0 2b c ⎞ ⎛0 a a⎞
A = ⎜ a b – c⎟ and A′ = ⎜ 2b b – b ⎟
A square matrix of order n × n is said to be orthogonal if ⎜ ⎟ ⎜ ⎟
AA′ = In = A′ A. For example, if ⎝ a –b c ⎠ ⎝ c –c c ⎠

As A is orthogonal
1 ⎡ 2 -2 ⎤
A = ⎢ ⎥ , \ AA′ = I
2 2 ⎣2 2 ⎦
⎛ 0 2b c ⎞ ⎛ 0 a a⎞ ⎛ 1 0 0⎞
⎡ 2 2⎤
1 ⇒ ⎜ a b – c ⎟ ⎜ 2b b – b ⎟ = ⎜ 0 1 0 ⎟
then A′ = ⎢ -2 2 ⎥ ⎜ ⎟ ⎜ ⎟ ⎜ ⎟
2 2 ⎣ ⎦ ⎝ a –b c ⎠ ⎝ c –c c ⎠ ⎝ 0 0 1⎠


Objective_Maths_JEE Main 2017_Ch 5.indd 8 01/01/2008 04:08:13


Matrices 5.9

⎛ 4b 2 + c 2 2b 2 – c 2 – 2b 2 + c 2 ⎞ niLpoTEnT MATrix
⎜ 2 2 ⎟
⇒ ⎜ 2b - c a2 + b2 + c2 a2 – b2 – c2 ⎟ A square matrix A is said to be nilpotent matrix if there
⎜ 2 2 2 2 2
a2 + b2 + c2 ⎠
⎟ exists a positive integer m such that Am = 0. If m is the least
⎝ –2b + c a – b –c
positive integer such that Am = 0, then m is called the index
⎛ 1 0 0⎞ of the nilpotent matrix A.
 = ⎜ 0 1 0⎟ For example, if
⎜ ⎟
⎝ 0 0 1⎠ ⎡ ab b2 ⎤
 A= ⎢ ⎥,
\ Using definition of equality of two matrices ⎢⎣ - a 2 - ab ⎥⎦
4b2 + c2 = 1, 2b2 – c2 = 0, a2 + b2 + c2 = 1
⎡ ab b2 ⎤ ⎡ ab b2 ⎤
On solving them, then A2 = AA = ⎢ ⎥ ⎢ 2 ⎥
⎢⎣ - a 2 - ab ⎥⎦ ⎢⎣ - a - ab ⎥⎦

1 1 1
a=± ,b=± ,c=±
2 6 3 ⎡0 0⎤
= ⎢ ⎥ = 0.
⎣0 0⎦
iDEMpoTEnT MATrix Hence, the matrix A is nilpotent of the index 2.
2
A square matrix A is said to be idempotent if A = A.
For example, if SoLvED ExAMpLE
⎡ 2 -2 -4 ⎤ ⎛ 2 –2 –4 ⎞
A = ⎢⎢ -1 3 4 ⎥⎥ , 17. The matrix A = ⎜ –1 3 4⎟ is
⎢⎣ 1 -2 -3 ⎥⎦ ⎜ ⎟
⎝ 1 –2 –3⎠
⎡ 2 -2 -4 ⎤ ⎡ 2 -2 -4 ⎤ (A) Nilpotent (B) Idempotent
then A = AA = ⎢⎢ -1 3
2
4 ⎥⎥ ⎢⎢ -1 3 4 ⎥⎥ (C) Orthogonal (D) Involutary
⎢⎣ 1 -2 -3 ⎥⎦ ⎢⎣ 1 -2 -3 ⎥⎦ Solution: (B)

In order to get the result, we go to the definition of
⎡ 2 -2 -4 ⎤ these matrices and check
= ⎢⎢ -1 3 4 ⎥⎥ = A (A) AA′ = 1 for othogonal matrix.
⎢⎣ 1 -2 -3 ⎥⎦ (B) If A2, A3 … or Ak is zero i.e. A2 = 0 or A3 = 0 or Ak
Hence, A is idempotent. = 0 then A is said to be nilpotent.
(C) If A2 = I then A is involutary matrix.
(D) If A2 = A then A is said to be idempotent and by
invoLUTorY MATrix
checking, we get A2 = A.
A square matrix A is said to be involutory matrix if A2 = I.    \ A is idempotent matrix.
For example, if
⎡ -5 -8 0 ⎤ notE
A = ⎢⎢ 3 5 0 ⎥⎥ ,
 Minimum number of zeros in a triangular matrix is given
⎢⎣ 1 2 -1 ⎥⎦ n(n - 1)
by, , where n is order of matrix.
2
⎡ -5 -8 0 ⎤ ⎡ -5 -8 0 ⎤  A triangular matrix A = [aij]n × n is called strictly triangular
then A = ⎢⎢ 3 5 0 ⎥⎥
2 ⎢ 3 5 0 ⎥
⎢ ⎥ if aij = 0 for 1 ≤ i ≤ n.
⎢⎣ 1 2 -1 ⎥⎦ ⎢⎣ 1 2 -1 ⎥⎦  The multiplication of two triangular matrices is a triangu-
 lar matrix.
⎡1 0 0⎤ 1 1
 If A is involutary matrix, then (I + A) and (I - A) are
= ⎢⎢ 0 1 0 ⎥⎥ = I
2 2
idempotent and (I + A) (I – A) = 0.
⎢⎣ 0 0 1 ⎥⎦  Trace of a skew symmetric matrix is always 0.
Hence, A is involutory.

Objective_Maths_JEE Main 2017_Ch 5.indd 9 01/01/2008 04:08:15


5.10  Chapter 5

Singular Matrix Adjoint of a Square Matrix


A square matrix A is said to be singular matrix if deter- Let A = [aij] be a square matrix of order n and let Cij be the
minant of A denoted by det A or |A| is zero, i.e., |A| = 0, cofactor of aij in the determinant |A|. Then the adjoint of A,
­otherwise, it is a non-singular matrix. denoted by adj A, is defined as the transpose of the cofactor
⎡ 0 1 -1 ⎤ matrix.
For example, the matrix A = ⎢⎢ 4 -3 4 ⎥⎥
The adjoint of a square matrix A is obtained on
replacing each (i, j)th element of A by the cofactor of the
⎢⎣ 4 -3 4 ⎥⎦ ( j, i)th element in |A|.
0 1 -1 For example, if
⎡ 1 2 3⎤
is singular as 4 -3 4 = 0
A = ⎢⎢ -1 0 1 ⎥⎥ ,
4 -3 4
⎢⎣ 4 3 2 ⎥⎦
then we have
caution C11 = –3 C12 = 6 C13 = –3
See next chapter for expansion of the determinant. C21 = 5 C22 = –10 C23 = 5
C31 = 2 C32 = –4 C33 = 2
⎡ -3 5 2 ⎤
Solved Examples Thus, adj A = ⎢ 6 -10 -4 ⎥⎥

⎢⎣ -3 5 2 ⎥⎦
18. Let A and B be two non-null square matrices. If the 
product AB is a null matrix, then Properties of the Adjoint of a Matrix
(A) A is singular (B)  B is singular 1. If A is a square matrix of order n, then
(C) A is non-singular (D)  B is non-singular
A (adj A) = |A| In = (adj A) A,
Solution: (A, B)
where In is a square matrix of order n.
Let B be non-singular, then B–1 exists. 2. If A is a square matrix of order n, then adj (A′) = (adj A)′.
Now, AB = O(given) 3. If A and B are two square matrices of the same order,
then adj (AB) = (adj B) (adj A).
⇒ (AB) B–1 = OB–1
4. adj (adj A) = |A|n–2 A, where A is a non-singular matrix.
(post-multiplying both sides by B–1) 2

5. |adj A| = |A|n–1 and |adj (adj A)| = | A |( n-1) , where A is a


⇒ A (BB–1) = O (by associativity) non-singular matrix.
6. Adjoint of a diagonal matrix is a diagonal matrix.
⇒ AIn = O(Q BB–1 = In)
7. adj(Am) = (adj A)m, m ∈ N
⇒ A = O 8. adj(kA) = kn–1(adj A), k ∈ R
But A is a non-null matrix. Hence, B is a singular matrix. 9. adj(In) = In
Similarly it can be shown that A is a singular matrix. 10. adj(O) = O
19. Which of the following is correct? 11. A is symmetric ⇒ adj A is also symmetric.
(A) Skew symmetric matrix of even order is always 12. A is diagonal ⇒ adj A is also diagonal.
singular. 13. A is triangular ⇒ adj A is also triangular.
(B)  Skew symmetric matrix of odd order is 14. A is singular ⇒ |adj A| = 0
non-singular.
(C)  Skew symmetric matrix of odd order is singular. Solved Examples
(D)  None of the above.
⎡ 1 2 -1 ⎤
Solution: (C)
20. If A = ⎢⎢ -1 1 2 ⎥⎥ , then det. [adj (adj A)] is
Since the determinant of a skew symmetric matrix of
⎢⎣ 2 -1 1 ⎥⎦
odd order is zero,
\  The matrix is singular. (A) (14)4 (B) (14)3 (C) (14)2 (D) (14)1

Objective_Maths_JEE Main 2017_Ch 5.indd 10 01/01/2008 04:08:16


Matrices  5.11

Solution: (A) 5. Let A, B, C be square matrices of the same order n. If A


We know that adj (adj A) = |A|n–2 A if |A| ≠ 0, provided is a non-singular matrix, then
order of A is n. (a) AB = AC ⇒ B = C (Left cancellation law)
(b) BA = CA ⇒ B = C (Right cancellation law)
\ adj (adj A) = |A| A(as n = 3) Note that these cancellation laws hold only if the
\ det [adj (adj A)] = |A|3 det A = |A|4. matrix A is non-singular.
6. If A is a non-singular matrix such that A is symmetric
1 2 -1 then A–1 is also symmetric.
But |A| = -1 1 2 = 14 7. If A is a non-singular matrix, then |A–1| = |A|–1.
2 -1 1 8. If A, B, C are three invertible matrices of the same
order then ABC is invertible and more over
\ det [adj (adj A)] = (14)4.
(ABC)–1 = C –1B–1A–1
21. If A is a singular matrix, then adj A is 9. A = diag (a1a2 … an) ⇒ A–1 = diag (a1–1 a2–1 … an–1)
(A) non-singular (B) singular 10. A is a scalar matrix ⇒ A–1 is also a scalar matrix.
(C) symmetric (D) not defined 11. A is triangular, |A| ≠ 0 ⇒ A–1 is also triangular.
Solution: (B) 12. Every invertible matrix possesses a unique inverse.
Let A be of order n × n.
Since A × adj A = |A| I Solved Examples
\ |A × adj A| = |A|n 22. Let A be an invertible matrix, which of the following is
⇒ |A| |adj A| = |A|n not true?
(A) (A′)–1 = (A–1)′
\ |adj A| = |A|n–1
(B) A–1 = |A|–1
Since A is singular,
(C) (A2)–1 = (A–1)2
\ |A| = 0.
(D)  None of these
\ |adj A| = 0.
Solution: (B)
Hence, adj A is singular.
A–1 = |A|–1 is not true, as L.H.S. is a matrix and R.H.S.
is a number.
Inverse of a Square Matrix
23. If B is a non-singular matrix and A is a square matrix,
Let A be any n-rowed square matrix. The inverse of A
then det (B–1AB) is equal to
denoted by A–1 is determined by the formula:
(A)  det (A–1) (B)  det (B–1)
1
A–1 = (adj A) (C)  det (A) (D)  det (B)
| A|
Solution: (C)
It may be noted that
det (B–1AB) = det (B–1) det A det B
AA–1 = A–1A = I
= det (B–1) × det B × det A = det (B–1B) × det A
Properties of the Inverse of a Matrix = det (I) × det A = 1 × det A = det A.
1. A square matrix is invertible if and only if it is
non-singular. ⎡1 0 0 ⎤ ⎡1 0 0⎤
2. The inverse of the inverse is the original matrix itself, 24. If A = ⎢⎢0 1 1 ⎥⎥ , I = ⎢0 1 0⎥
⎢ ⎥
i.e., (A–1)–1 = A. ⎢⎣0 –2 4 ⎥⎦ ⎢⎣0 0 1 ⎥⎦
3. The inverse of the transpose of a matrix is the trans-
1 2
pose of its inverse, i.e. (A′)–1 = (A–1)′. A–1 = ⎡ A + cA + dI ⎤
4. If A and B are two invertieble matrices of the same 6⎣ ⎦
order, then AB is also invertible and moreover where c, d ∈ R, the pair of values (c, d ) are
(A)  (6, 11) (B)  (6, –11)
(AB)–1 = B–1A–1 (C)  (–6, 11) (D)  (–6, –11)

Objective_Maths_JEE Main 2017_Ch 5.indd 11 01/01/2008 04:08:17


5.12  Chapter 5

Solution: (C) or IX = A–1B


Given or X = A–1B
⎡1 0 0 ⎤ ⎡6 0 0 ⎤
1⎢ Hence X = A–1B is the unique solution of AX = B, | A | ≠ 0.
⎢ ⎥
A = ⎢0 1 1 ⎥ , A = ⎢0 4 –1⎥⎥
–1
6 Criterion of Consistency
⎢⎣0 –2 4 ⎥⎦ ⎢⎣0 2 1 ⎥⎦

Let AX = B be a system of n linear equations in n variables.
⎡1 0 0 ⎤ ⎡1 0 0 ⎤ ⎡1 0 0⎤
1. If |A| ≠ 0, then the system of equations is consistent
A = ⎢0 1 1 ⎥ ⎢0 1 1 ⎥ = ⎢0 –1 5 ⎥⎥
2⎢ ⎥ ⎢ ⎥ ⎢
and has a unique solution given by X = A–1B.
⎢⎣0 –2 4 ⎥⎦ ⎢⎣0 –2 4 ⎥⎦ ⎢⎣0 –10 14 ⎥⎦ 2. If |A| = 0 and (adj A) B = 0, then the system of equa-

tions is consistent and has infinitely many solutions.
⎡c 0 0⎤ ⎡d 0 0 ⎤ 3. If |A| = 0 and (adj A) B ≠ 0, then the system of equa-
cA = ⎢⎢0 c c ⎥⎥ , dI = ⎢0 d 0⎥
⎢ ⎥
tions is inconsistent, i.e., it has no solution.
⎢⎣0 –2c 4c ⎥⎦ ⎢⎣ 0 0 d ⎥⎦ Homogeneous Equations

The system of equations AX = B is said to be homogene-
1 2
\ A–1 = ⎡ A + cA + dI ⎤ ous if the constants b1, b2, … bn are all zero. That is, if the
6⎣ ⎦
 matrix B is a zero matrix and the system is of the form
⇒ 6 = 1 + c + d,  (By equality of matrices) AX = O
\ (–6, 11) satisfy the relation where O is a null matrix of order n × 1.
1. If |A| ≠ 0, then its only solution X = 0, is called the
Solution of a System of Linear trivial solution.
Equations by Matrix Method 2. If |A| = 0, then AX = O has a non-trivial solution. It will
Consider a system of linear equations have infinitely many solutions.

a11x1 + a12x2 + … + a1nxn = b1


Solved Example
a21x1 + a22x2 + … + a2nxn = b2
25. The value of a for which the system of equations ax +
M M M
y + z = 0, x + ay + z = 0, x + y + z = 0, possess non-zero
an1x1 + an2x2 + … + annxn = bn. solutions are given by,
(A)  1, 2
We can express these equations as a single matrix equation
(B)  1, –1
(C) 1
⎡ a11 a12  a1n ⎤ ⎡ x1 ⎤ ⎡ b1 ⎤
⎢a ⎥ ⎢x ⎥ ⎢b ⎥ (D)  None of these
⎢ 21 a22  a2 n ⎥ ⎢ 2 ⎥ = ⎢ 2 ⎥
⎢ M M M ⎥ ⎢ M ⎥ ⎢ M ⎥ Solution: (C)
⎢ ⎥ ⎢ ⎥ ⎢ ⎥ The set of homogeneous equations will have a non-
⎣ an1 an 2  ann ⎦ ⎣ xn ⎦ ⎣ bn ⎦ zero solution if D = 0
A X B
a 1 1
Let |A| ≠ 0, so that A–1 exists uniquely. Pre-multiplying both i.e., 1 a 1 = 0
sides of AX = B by A–1, we get 1 1 1

A–1 (AX) = A–1B ⇒ a = 1

or (A–1A)X = A–1B

Objective_Maths_JEE Main 2017_Ch 5.indd 12 01/01/2008 04:08:19


Matrices  5.13

EXERCISES

Single Option Correct Type

1. If A and B are symmetric matrices and AB = BA, then ⎛ 3 –4 ⎞


8. If A = ⎜ then An equals
A–1B is a ⎝ 1 –1⎟⎠
(A)  symmetric matrix
⎛ 3n –4 n⎞ ⎛ 3n 4 n ( –1) n⎞
(B)  skew-symmetric matrix (A)  ⎜ ⎜
(B)  ⎟
(C)  identity matrix ⎝ n – n ⎟⎠ ⎜⎝ 1n ( –1)n ⎟⎠
(D)  None of these ⎛ 3 + n – ( 4 + n)⎞
(C)  ⎜ (D)  None of these
⎡2 6 4⎤ ⎝ n – n ⎟⎠
2. If the product of the matrix B = ⎢⎢ 1 0 1 ⎥⎥ with a
9. If A = [aij] is a scalar matrix of order n × n such that
⎢⎣ –1 1 –1⎥⎦ aij = k for all i, then trace of A is equal to
⎡ –1 0 1 ⎤ n
(A) kn (B) 
matrix A has inverse C = ⎢ 1 1 3⎥⎥ , then A–1 equals
⎢ k
(C) nk (D)  None of these
⎢⎣ 2 0 2⎥⎦
⎡ 1/ 2 1/ 2 ⎤
⎡ –3 –5 5 ⎤ ⎡ –3 5 5⎤ 10. The matrix A = ⎢ ⎥ is
⎢⎣ -1/ 2 -1/ 2 ⎥⎦
(A)  ⎢⎢ 0 9 14 ⎥⎥ (B)  ⎢0 0
⎢ 9 ⎥⎥
⎢⎣ 2 2 6 ⎥⎦ ⎢⎣ 2 14 16 ⎥⎦ (A) unitary (B) orthogonal
(C) nilpotent (D) involutary
⎡ –3 –5 –5⎤ ⎡ –3 –3 –5⎤
⎢ ⎥ 11. The value of q in [0, 2 p] such that the matrix
(C)  ⎢ 0 9 2 ⎥ (D) ⎢⎢ 0 9 2 ⎥⎥
⎢⎣ 2 14 6 ⎥⎦ ⎢⎣ 2 14 6 ⎥⎦ ⎡ 2 sin q - 1 sin q cos q ⎤
⎢ ⎥
⎢ sin(q + p ) 2 cos q - 3 tan q ⎥
⎡a 2 ⎤ 3 ⎢cos(q - p ) tan(p � q ) 0 ⎥⎦
3. If A = ⎢
2 a ⎥ and |A | = 125 then the value of a is ⎣
⎣ ⎦ is skew-symmetric, is
±2 (C) 
(A) ±1 (B)  ±3 (D) 
±5 (A) p/2 (B)  p/3 (C)  p/4 (D) 
p/6
4. If A is an involutory matrix and I is unit matrix of the
⎡ cos 2q cos q sin q ⎤
same order then, (I – A)(I + A) = 12. If E(q) = ⎢ ⎥ and q and f differ
(A) 0 (B) A (C)  I (D) 2A ⎢⎣cos q sin q sin 2 q ⎥⎦
p
5. Matrix A is such that A2 = 2A – I, where I is unit matrix by an odd multiple of , then E(q) E(f) is a
then for n ≥ 2, An = 2
(A)  null matrix (B)  unit matrix
(A) nA – (n – 1)I (B)  nA – I (C)  diagonal matrix (D)  None of these
(C) 2n–1 A – (n – 1)I (D) 2n–1A – I
13. If A and B are two square matrices such that B =
6. If AB = A and BA = B, where A and B are square –A–1BA, then (A + B)2 =
­matrices, then (A) 0 (B)  A2 + B2
(A) B2 = B and A2 = A (C) A2 + 2AB + B2 (D)  A+B
(B) B2 = A and A2 = B
(C) AB = BA ⎡ 1 tan q / 2⎤
14. If A = ⎢ and AB = I, then B =
(D)  None of these ⎣ - tan q / 2 1 ⎥⎦
7. If A is a square matrix, B is a singular matrix of same q q
order, then for a positive integer n, (A–1BA)n equals (A) cos2 2 A (B)  cos2 2 AT
(A) A–n BnAn (B)  AnBn A–n q
–1 n
(C) A B A (D)  n (A–1 B A) (C) cos2 2 I (D)  None of these

Objective_Maths_JEE Main 2017_Ch 5.indd 13 01/01/2008 04:08:21


5.14  Chapter 5

15. For each real number x such that –1 < x < 1, let A(x) be ⎡a b ⎤
21. Let A = ⎢ ⎥ , be a 2 × 2 matrix where a, b, c, d take
⎡1 – x⎤
x+ y ⎣c d ⎦
the matrix (1 – x)–1 ⎢ – x 1 ⎥⎦ and z = 1 + xy . Then the values 0 or 1 only. The number of such matrices

which have inverses is:
(A) A(z) = A(x) + A(y)
(A) 8 (B) 7 (C) 6 (D) 5
(B) A(z) = A(x)[A(y)]–1
(C) A(z) = A(x) A(y) 22. Let A be a 2 × 2 matrix with real entries. Let I be the
(D) A(z) = A(x) – A(y) 2  × 2 identity matrix. Denote by tr (A), the sum of
diagonal entries of A. Assume that A2 = I.
16. The inverse of a skew symmetric matrix of odd order is
Statement 1: If A ≠ I and A ≠ –I, then det A = –1.
(A)  a symmetric matrix Statement 2: If A ≠ I and A ≠ –I, then tr (A) ≠ 0.
(B)  a skew symmetric matrix (A) Statement 1 is false, Statement 2 is true
(C)  diagonal matrix (B) Statement 1 is true, Statement 2 is true, Statement 2
(D)  does not exist is a correct explanation for Statement 1
17. The number of solutions of equations x2 – x3 = 1, –x1 + (C) Statement 1 is true, Statement 2 is true; Statement 2
2x3 = 2, x1 – 2x2 = 3 is is not a correct explanation for Statement 1
(A) zero (B) one (D) Statement 1 is true, Statement 2 is false
(C) two (D) infinite 23. Let A be a square matrix all of whose entries are inte-
gers. Then which one of the following is true?
⎡a b ⎤
18. If ⎢ ⎥ is to be the square root of two-rowed unit (A) If det A = ±1, then A–1 exists but all its entries are
⎣ g –a ⎦ not necessarily integers
matrix, then a, b and g should satisfy the relation (B) If det A ≠ ±1, then A–1 exists and all its entries are
(A) 1 + a2 + bg = 0 (B)  1 – a2 – bg = 0 non-integers
2
(C)  1 – a + bg = 0 (D)  a2 + bg – 1 = 0 (C) If det A = ±1, then A–1 exists and all its entries are
integers
19. Let A and B be two symmetric matrices of order 3.
(D)  If det A = ±1, then A–1 need not exist
Statement 1: A(BA) and (AB)A are symmetric matrices.
Statement 2: AB is symmetric matrix if matrix multi- 24. If B, C are square matrices of order n and if A = B + C,
plication of A with B is commutative. BC = CB, C2 = 0, then for any positive integer p, Ap+1 =
(A) Statement 1 is false, Statement 2 is true Bk[B + (p + 1)C], where k =
(B) Statement 1 is true, Statement 2 is true; Statement 2 (A) p (B)  p + 1 (C)  p + 2 (D)  p – 1
is a correct explanation for Statement 1
(C) Statement 1 is true, Statement 2 is true; Statement 2 ⎡ 1 2 -1 ⎤
is not a correct explanation for Statement 1 25. If A = ⎢⎢ -1 1 2 ⎥⎥ , then det. (adj (adj A)) is
(D) Statement 1 is true, Statement 2 if false
⎢⎣ 2 -1 1 ⎥⎦
⎛ 1 0 0⎞
(A) (14)4 (B) (14)3 (C) (14)2 (D) (14)1
20. Let A = ⎜ 2 1 0⎟ . If u1 and u2 are column matrices
⎜ ⎟
⎝ 3 2 1⎠ ⎡a b c ⎤
26. If abc = p and A = ⎢⎢ c a b ⎥⎥ such that AA′ = I, then
⎛1 ⎞ ⎛ 0⎞
⎢⎣ b c a ⎥⎦
such that Au1 = 0 and Au2 = ⎜1 ⎟ , then u1 + u2 is
⎜ ⎟
⎜ ⎟ ⎜ ⎟ a, b, c are the roots of the equation
⎝ 0 ⎠ ⎝ 0⎠
equal to:
(A) x3 + p = 0 (B)  x3 ± x2 + p = 0
⎛ -1⎞ ⎛ -1⎞
(C) x3 ± 3x2 + p = 0 (D)  x3 ± 2x2 + p = 0
(A)  ⎜ 1⎟ (B) 
⎜ 1⎟
⎜ ⎟ ⎜ ⎟ 27. If A is a singular matrix, then adj A is
⎝ 0⎠ ⎝ -1⎠
(A) non-singular (B) singular
⎛ -1⎞ ⎛ 1⎞ (C) symmetric (D) not defined
⎜ ⎟ ⎜ -1⎟
(C)  -1 (D) 
⎜ ⎟ ⎜ ⎟ 28. Matrix A is such that A2 = 2A – I, where I is unit matrix
⎝ 0⎠ ⎝ -1⎠ then for n ≥ 2, An =

Objective_Maths_JEE Main 2017_Ch 5.indd 14 01/01/2008 04:08:23


Matrices  5.15

(A) nA – (n – 1)I (B)  nA – I 38. A and B are two non-singular matrices of the same
(C) 2n – 1 A – (n – 1)I (D)  2n – 1A – I order such that An = I for some positive integer n > 1.
29. For each real number x such that –1 < x < 1, let A(x) be Then, BAn–1 B–1 – BA–1 B–1
(A)  is a null matrix
– x⎤ ⎡1 x+ y (B)  is an identity matrix
the matrix (1 – x)–1 ⎢ – x
1 ⎥⎦ and z = 1 + xy . Then,
⎣ (C)  a singular matrix
(A) A(z) = A(x) + A(y) (B)  A(z) = A(x)[A(y)]–1 (D)  None of these
(C) A(z) = A(x) A(y) (D)  A(z) = A(x) – A(y) 39. The number of different matrices which can be formed
30. The inverse of a skew-symmetric matrix of odd order is using 12 different real numbers is
(A)  a symmetric matrix (A)  6 (12)! (B)  12 (12)!
(B)  a skew-symmetric matrix (C)  4 (12)! (D)  None of these
(C)  diagonal matrix 40. A skew-symmetric matrix A satisfies the relation A2 +
(D)  does not exist I = 0, where I is a unit matrix. Then, A is
31. If A is a square matrix, B is a singular matrix of same (A)  Idempotent matrix (B)  Orthogonal matrix
order, then for a positive integer n, (A–1BA)n equals (C)  Nilpotent matrix (D)  None of these
(A) A–n BnAn (B)  AnBn A–n 41. Let A be an n × n matrix such that An = aA, where a is
–1 n
(C) A B A (D)  n (A–1 B A) a real number different from 1 and –1. Then, the matrix
A + In is
32. If A is an invertible matrix, then
(A) singular
(A) adj A′ = (adj A)′ (B) adj A′ = adj A
(B)  non-singular, i.e., invertible
(C) adj A′ = A′ (D)  None of these
(C)  scalar matrix
33. If A is a non-singular square matrix of order n, then adj (D)  None of these
(adj A) is equal to
42. If adj B = A and P, Q are two unimodular matrices, i.e.,
(A) |A|n A (B)  |A|n – 1A
n–2 |P| = 1= |Q|, then (Q–1 B P–1)–1 is equal to
(C) |A| A (D)  None of these
(A) PAQ (B)  PBQ (C)  QAP (D)  QBP
34. If x, y, z are in A.P. with common differences d and the
4 5 x ⎡ a⎤
⎢ 1 n⎥
rank of the matrix 5 6 y is 2 then the values of 43. If A = ⎢ ⎥ , then
6 k z ⎢-a 1⎥
d and k are ⎢⎣ n ⎥⎦
x 1 n
(A)  ; arbitrary number (B)  arbitrary number, 7 (A)  lim An = 0 (B)  lim A =0
4 n→∞ n→∞ n
x
(C) x, 5 (D)  , 6. 1
2 (C)  lim An = 0 (D)  None of these
n→∞ n2
35. If D = diag (a1 a2 a3 … an), where ai ≠ 0 for all i = 1,
44. If A k = 0 for some value of k and (I – A) p = I + A + A2 +
2, …, n, then D–1 is equal to
… + A k – 1, then p is
(A) In
(B) D (A) –1 (B) –2

(C) diag ( a1-1 a2-1 a3-1 ... an-1 ) (C)  –3 (D)  None of these
(D)  None of these 45. If A satisfies the equation x3 – 5x2 + 4x + kI = 0, then
A–1 exists if
36. If A is a non-singular matrix such that AA′ = A′A and
B = A–1A′, then BB′ is (A) k ≠ –1 (B)  k≠0
(C) k ≠ 1 (D)  None of these
(A) I (B)  B–1
–1
(C) (B )′ (D)  None of these 46. If M is a 3 × 3 matrix, where M ′M = I and det M = 1,
3 n –1 then det (M – I) =
37. If A = 0 and A ≠ I for n = 1, 2 then (I – A) is
(A) 0 (B) 1
(A) I + A (B)  I + A + A2
2 (C)  – 1 (D)  None of these
(C) I – A + A (D)  None of these

Objective_Maths_JEE Main 2017_Ch 5.indd 15 01/01/2008 04:08:24


5.16  Chapter 5

More than One Option Correct Type

47. Let A and B be two non-null square matrices. If the 54. Let A, B, C be 2 × 2 matrices with entries from the set
product AB is a null matrix, then of real numbers. Define operation ‘*’ as follows
(A) A is singular 1
A * B = ( AB + BA) , then
(B) B is singular 2
(C) A is non-singular
(A) A * I = A
(D) B is non-singular
(B) A * A = A2
⎡ –1 2 5 ⎤ (C) A * B = B * A
48. The rank of the matrix ⎢ 2 –4 a – 4 ⎥⎥ is
⎢ (D) A * (B + C) = A * B + A * C
⎢⎣ 1 –2 a + 1 ⎥⎦ 55. If A and B are two matrices such that AB = BA, then
"n∈N
(A)  1 if a = 6 (B)  2 if a = 1
(A) AnB = BAn
(C)  3 if a = 2 (D)  1 if a = –6
(B) (AB)n = AnBn
49. The system of equations 2x – 3y + 6z – 5t = 3, y – 4z + (C) (A + B)n = nC0 An + nC1 An–1 B + nC2 An–2 B2 + … +
t = 1, 4x – 5y + 8z – 9t = k has n
Cn Bn
(A)  no solution if k ≠ 7
(D) A2n – B2n = (An – Bn) (An + Bn)
(B)  no solution if k = 7
(C)  infinite solutions if k ≠ 7 ⎡ 1 0 - 2⎤
(D)  infinite solutions if k = 7 56. If A = ⎢⎢ - 2 1
–1
0 ⎥⎥ , then
50. Which of the following is correct? ⎢⎣ - 1 1 0 ⎥⎦
(A) If A is a symmetric matrix, then An is symmetric,
(A) |A| = 2
n∈N
(B) If A is a skew-symmetric matrix then An is sym- ⎡ 1 ⎤
⎢ 2 0 -1⎥
metric if n is even, n ∈ N ⎢ ⎥
(C) If A is a skew-symmetric matrix then An is 1
(B) adj. A = ⎢ -1 0⎥
skew-symmetric if n is odd, n ∈ N ⎢ 2 ⎥
(D)  All of these ⎢ ⎥
⎢- 1 1
0⎥
51. If A is a non-singular matrix, then ⎢⎣ 2 2 ⎥⎦
(A) A–1 is symmetric if A is symmetric (C) |adj. A| = 4
(B) A–1 is skew-symmetric if A is symmetric 1
(D) |A′| =
(C) |A–1| = |A| 2
(D) |A–1| = |A|–1
⎡1 - 1 1 ⎤
52. Which of the following is true? 57. If A = ⎢ 2 - 1 0 ⎥⎥ , then

(A)  Transpose of an orthogonal matrix is also ⎢⎣1 0 0 ⎥⎦
orthogonal
(B) Every orthogonal matrix is non-singular (A) A3 = I (B)  A–1 = A2
n
(C) Product of the two orthogonal matrices is also (C) A = A, " n ≠ 4 (D)  None of these
orthogonal
⎡ 0 1⎤
(D) Inverse of an orthogonal matrix is also orthogonal 58. If A = ⎢ ⎥ , the values of a, b such that (a I +
⎣ -1 0 ⎦
53. Suppose, a, b, c are real numbers such that abc = 1. If
bA)2 = A2 arc
⎡a b c ⎤
the matrix A = ⎢⎢ b c a ⎥⎥ is such that A′A = I, then
1 1 1 1
(A) ± ,± ±
(B)  ,∓
⎢⎣ c a b ⎥⎦ 2 2 2 2

i i i i
the value of a3 + b3 + c3 is (C) ± ,± (D) ± ,∓
(A) 1 (B) 2 (C) 3 (D) 4 2 2 2 2

Objective_Maths_JEE Main 2017_Ch 5.indd 16 01/01/2008 04:08:26


Matrices  5.17

Passage Based Questions


Passage 1 Passage 2
A matrix is said to be of rank r when it contains at least one A non-zero matrix A is said to be in Echelon from if it
non-zero minor of order r and no such minor of order r + 1. ­satisfies the following conditions
The rank of a matrix is denoted by r(A).
1. All non-zero rows of A, if any, precede the zero rows
By means of elementary transformations every non-
2. The number of zeroes preceding the first non-zero ele-
zero matrix of rank r can be reduced to one of the following
ment in a row, is less than the number of such zeros in
forms
the succeeding row.
⎡ Ir M 0 ⎤ ⎡Ir ⎤ 3. The first non-zero element in a row, is unity.
⎢ ⎥
(A)  ⎢⎥ (B)  ⎢⎥
⎢ ⎥ For example, is in Echelon form.
⎢⎣ 0 0 ⎥⎦ ⎢⎣ 0 ⎥⎦ The number of non-zero rows of a matrix in the Echelon
(C) [ I r M 0 ] (D) [Ir] form, is its rank. For example, the rank of the above
matrix is 3. By elementary transformations, we reduce
Where Ir is a r-rowed unit matrix. These are called the normal the matrix to Echelon form and then find its rank as
forms of the given matrix and the value of r is the rank of the the rank of a matrix remains unaltered by elementary
matrix. In order to find the rank of a given matrix, reduce the transformations.
matrix to its normal form. This process of reducing a matrix of
⎡ 2 3 4⎤
rank r to its normal form is known as ‘The sweep-out process’.
61. The rank of the matrix A = ⎢⎢ 3 1 2⎥⎥ is
⎡0 1 2 1⎤
⎢⎣ - 1 2 2⎥⎦
59. The rank of the matrix A = ⎢⎢ 1 2 3 2⎥⎥ is
(A) 1 (B) 2
⎢⎣ 3 1 1 3⎥⎦
(C) 3 (D) can’t determine
(A) 1 (B) 2 (C) 3 (D) 4
⎡ 1 3 4 3⎤
⎡ 1 - 1 2 - 3⎤ ⎢
62. The rank of the matrix A = ⎢ 3 9 12 9⎥⎥ is
⎢4 1 0 2⎥⎥ ⎢⎣ - 1 - 3 - 4 - 3⎥⎦
60. Rank of the matrix A = ⎢ is
⎢0 3 1 4⎥
⎢ ⎥ (A) 1 (B) 2 (C) 3 (D) 0
⎣0 1 0 2⎦
(A) 1 (B) 2 (C) 3 (D) 4

Assertion-Reason Type
Instructions: In the following questions an Assertion (A) is (C)  Assertion(A) is True, Reason(R) is False
given followed by a Reason (R). Mark your responses from (D)  Assertion(A) is False, Reason(R) is True
the following options:
63. Assertion: A is the n × n matrix whose elements are all
(A)  Assertion(A) is True and Reason(R) is True;
‘1’ and B is the n × n matrix whose diagonal elements
Reason(R) is a correct explanation for Assertion(A)
are all ‘n’ and other elements are ‘n – r’. Then, (B – rI )
(B) Assertion(A) is True, Reason(R) is True; Reason(R)
[B – (n2 – nr + r)I] = 0 because
is not a correct explanation for Assertion(A)
Reason: A2 is a scalar multiple of A.

Previous Year’s Questions


⎡a b⎤ ⎡a b ⎤
64. If A = ⎢ ⎥ and A2 = ⎢ ⎥ , then [2003] (C) a = a2 + b2, b = a2 - b2
⎣b a ⎦ ⎣b a ⎦ (D) a = 2ab, b = a2 + b2
(A) a = a2 + b2, b = ab
(B) a = a2 + b2, b = 2ab

Objective_Maths_JEE Main 2017_Ch 5.indd 17 01/01/2008 04:08:27


5.18  Chapter 5

⎛ 0 0 -1⎞ 72. The number of 3 × 3 non-singular matrices, with four


65. Let A = ⎜ 0 -1 0 ⎟ . The only correct statement entries as 1 and all other entries as 0, is [2010]
⎜ ⎟ (A) 5 (B) 6
⎝ -1 0 0 ⎠
(C)  at least 7 (D)  less than 4
about the matrix A is [2004]
(A) A is a zero matrix 73. Let A and B be two symmetric matrices of order 3.
(B) A2 = I  [2011]
(C) A–1 does not exist Statement 1: A(BA) and (AB)A are symmetric matrices.
(D) A = (-1)I, where I is a unit matrix Statement 2: AB is symmetric matrix if matrix multi-
plication of A and B is commutative.
⎛ 1 -1 1 ⎞ ⎛ 4 2 2⎞ (A) Statement 1 is true, Statement 2 is true; Statement 2
⎜ ⎟ ⎜
66. Let A = 2 1 -3 (10) B = -5 0 a ⎟ . If B is is not a correct explanation for Statement 1
⎜ ⎟ ⎜ ⎟ (B)  Statement 1 is true, Statement 2 is false.
⎝1 1 1 ⎠ ⎝ 1 -2 3 ⎠
(C)  Statement 1 is false, Statement 2 is true.
the inverse of matrix A, then α is [2004]
(D) Statement 1 is true, Statement 2 is true; Statement 2
(A) -2 (B) 5 (C) 2 (D) -1 is a correct explanation for Statement 1
67. If A2 - A + I = 0, then the inverse of A is [2005] ⎛100 ⎞
(A) A + I (B)  A 74. Let A = ⎜ 210⎟ . If u1 and u2 are column matrices such
(C) A - I (D)  I-A ⎜ ⎟
⎝ 321 ⎠
⎡1 0 ⎤ ⎡1 0 ⎤ ⎛1 ⎞ ⎛ 0⎞
68. If A = ⎢ ⎥ and I = ⎢0 1 ⎥ , then which one of the
⎣1 1 ⎦ ⎣ ⎦ ⎜
that Au1 = 0 and Au2 = ⎜1 ⎟ , then u1 + u2 is equal to

⎜ ⎟ ⎜ ⎟
following holds for all n ≥ 1, by the principle of math-  ⎝ 0⎠ ⎝ 0⎠ [2012]
ematical induction [2005]
n
(A) A = nA - (n - 1)I ⎛ -1⎞ ⎛ -1⎞ ⎛ -1⎞ ⎛1 ⎞
(B) An = 2n-1A - (n - 1)I (A)  1   (B)  1   (C)  -1   (D)  ⎜ -1⎟
⎜ ⎟ ⎜ ⎟ ⎜ ⎟
(C) An = nA + (n - 1)I ⎜ ⎟ ⎜ ⎟ ⎜ ⎟ ⎜ ⎟
(D) An = 2n-1A + (n - 1)I ⎝0 ⎠ ⎝ -0⎠ ⎝0 ⎠ ⎝ -1⎠
75. The number of values of k, for which the system of
69. If A and B are square matrices of order n × n such that equations [2013]
A2 - B2 = (A - B) (A + B), then which of the following
(k + 1)x + 8y = 4k
will always be true? [2006]
kx + (k + 3)y = 3k − 1
(A) A = B has no solution, is
(B) AB = BA (A) 1 (B) 2 (C) 3 (D) infinite
(C)  either of A or B is a zero matrix
(D)  either of A or B is an identity matrix 76. If A is a 3 × 3 non-singular matrix such that AA′ =
A′ A and B = A-1 A′ , then BB′ equals [2014]
⎛ 1 2⎞ ⎛ a 0⎞
70. Let A = ⎜ ⎟ and B = ⎜ , a, b ∈ N. Then (A) I + B (B)  I -1
(C)  B (D)  ( B -1 )′
⎝ 3 4⎠ ⎝ 0 b⎟⎠
 [2006] ⎡1 2 2 ⎤
(A)  there cannot exist any B such that AB = BA 77. If A = ⎢⎢ 2 1 -2⎥⎥ is a matrix satisfying the ­equation
(B) there exist more than one but finite number of B’s
such that AB = BA ⎢⎣ a 2 b ⎥⎦
(C)  there exists exactly one B such that AB = BA AAT = 9I, where I is 3 × 3 identity matrix, then ordered
(D)  there exist infinitely many B’s such that AB = BA pair (a, b) is equal to [2015]
⎡ 5 5a a ⎤ (A) (-2, 1) (B)  (2, 1)
(C) (-2, -1) (D)  (2, -1)
71. Let A = ⎢⎢0 a 5a ⎥⎥ , If |A2| = 25 then |a | equals
 ⎢⎣0 0 5 ⎥⎦ [2007] ⎡5a -b ⎤ T
78. If A = ⎢ ⎥ and A adj A = A A , then 5a + b is
(A) 5 (B) 1 ⎣3 2 ⎦
(C) 1/5 (D) 5 equal to [2016]
(A) 13   (B) –1   (C) 5   (D) 4

Objective_Maths_JEE Main 2017_Ch 5.indd 18 01/01/2008 04:08:30


Matrices  5.19

Answer keys

Single Option Correct Type


1. (A) 2. (C) 3.  (C) 4. (A) 5.  (A) 6. (A) 7.  (C) 8. (D) 9.  (C) 10. (C)
11.  (D) 12. (A) 13.  (B) 14. (B) 15.  (C) 16. (D) 17. (A) 18. (B) 19. (C) 20. (D)
21. (C) 22. (D) 23. (C) 24.  (A) 25.  (A) 26. (B) 27. (B) 28. (A) 29. (C) 30.  (D)
31.  (C) 32. (A) 33. (C) 34. (B) 35. (C) 36.  (A) 37. (B) 38. (A) 39.  (C) 40.  (B)
41. (B) 42.  (A) 43. (B) 44.  (A) 45. (B) 46. (A)

More than One Option Correct Type


47.  (A) and (B) 48.  (B) and (D) 49.  (A) and (D) 50.  (A), (B), (C) and (D)
51.  (A) and (D) 52.  (A), (B), (C) and (D) 53.  (B) and (D) 54.  (A), (B), (C) and (D)
55.  (A), (B), (C) and (D) 56.  (B) and (D) 57.  (A) and (B) 58.  (A) and (D)

Passage Based Questions


59.  (C) 60. 
(D) 61. (B) 62.  (A)

Assertion-Reason Type
63.  (A)

Previous Year’s Questions


6 4. (B) 65. (B) 66. (B) 67. (D) 68. (A) 69. (B) 70. (D) 71. (C) 72. (C) 73. (A)
74. (D) 75. (A) 76. (B) 77. (C) 78. (C)

Hints and Solutions

Single Option Correct Type


1. We have, AB = BA = B′A′ = (AB)′ ⎡a 2 ⎤ 3 3
3. Given: A = ⎢ ⎥ and |A | = |A| = 125.
⇒ AB is symmetric. ⎣2 a⎦
Also,  ABA–1 = BAA–1 = B(Q AB = BA)
Now |A| = a2 – 4

⇒ A–1ABA–1 = A–1B ⇒ BA–1 = A–1B
⇒ (a 2 – 4)3 = 125 = 53

Therefore, (A–1B)′ = (BA–1) = (A–1)′B′ = A′B
⇒ a 2 – 4 = 5
(Q A–1 and B are symmetric)
⇒ a = ±3
Thus, the matrix A–1B is symmetric.
The correct option is (C)
The correct option is (A)
4. Since A is an involutory matrix
2. We have (BA)–1 = C ⇒ A–1B–1 = C ⇒ A–1 = CB
\ A2 = I
⎡ –1 0 1 ⎤ ⎡ 2 6 4 ⎤
⇒ I – A2 = 0
\ A = ⎢⎢ 1 1 3⎥⎥ ⎢⎢ 1 0 1 ⎥⎥
–1
\ (I – A) (I + A) = 0
⎢⎣ 2 0 2⎥⎦ ⎢⎣ –1 1 –1⎥⎦
The correct option is (A)
⎡ –3 –5 –5⎤ 5. A2 = 2A – 1 (1)

= ⎢⎢ 0 9 2 ⎥⎥ Multiplying by A, we have
⎢⎣ 2 14 6 ⎥⎦ A3 = 2A2 – A = 2(2A – I) – A
[Using (1)]

The correct option is (C) ⇒ A3 = 3A – 2I

Objective_Maths_JEE Main 2017_Ch 5.indd 19 01/01/2008 04:08:31


5.20  Chapter 5

p
Again, multiplying by A, we get The simultaneous equations hold in [0, 2p] if q =
6
A4 = 3A2 – 2AI ⇒ A4 = 3(2A – I) – 2A [Using (1)] The correct option is (D)
⇒ A4 = 6A – 3I – 2A 12. Let S = E(q) E(f)
⇒ A4 = 4A – 3I ⎡ cos 2 q cos q sin q ⎤ ⎡ cos 2f cos f sin f ⎤
Hence, by induction, we have ⇒ S = ⎢
⎥ ⎢ ⎥
⎢⎣cos q sin q sin 2 q ⎥⎦ ⎢⎣cos f sin f sin 2f ⎥⎦
An = nA – (n – 1)I
The correct option is (A) ⎡cos q cos f cos(q � f ) cos q sin f cos(q � f )⎤
⇒ S = ⎢

6. We have AB = A ⎣cos f sin q cos(q � f ) sin q sin f cos(q � f ) ⎦
⇒ A(BA) = A(Q BA = B)
⎡ p p⎤
⇒ (AB) A = A ⎢cos q cos f cos( 2n + 1) 2 cos q sin f cos( 2n + 1) 2 ⎥
⇒ AA = A(\ AB = A) ⇒ S = ⎢

⎢cos f sin q cos( 2n + 1) p sin q sin f cos( 2n + 1) p ⎥
⇒ A2 = A ⎢⎣ 2 2 ⎥⎦
Again BA = B
⎡0 0 ⎤ p
⇒ B (AB) = B(Q AB = A) \ S = ⎢ ⎥ [Q q – f = (2n + 1) (given)]
⇒ (BA)B = B ⎣0 0 ⎦ 2
The correct option is (A)
⇒ BB = B ⇒ B2 = B
13. Given, B = –A–1BA
The correct option is (A)
\ AB = –AA–1BA = –IBA = –BA(\ AB = –BA)
7. Consider n = 2
\ (A–1BA)2 = (A–1BA)(A–1BA) Now (A + B)2 = (A + B)(A + B) = A2 + AB + BA + B2

= (A–1B)(A–1)(BA) = A–1B2A
= A2 + B2(\ BA = – AB)
Again for n = 3, Thus, (A + B) = A2 + B2
2


we have The correct option is (B)
(A–1BA)3 = (A–1B2A)(A–1BA) = A–1B3A q q
\ Generalizing the case (A–1BA)n = A–1BnA 14. |A| = 1 + tan2 2 = sec2 2
The correct option is (C) AB = I  ⇒  B = IA–1

8. Consider ⎡ q⎤
1 – tan ⎥
⎛ 3 –4⎞ ⎛ 3 –4⎞ ⎛ 5 –8⎞ ⎡1 0 ⎤ ⎢ 2
⎢0 1 ⎥ ⎢ q ⎥
2
A = ⎜
= ⎜
⎝ 1 –1⎟⎠ ⎜⎝ 1 –1⎟⎠ ⎝ 2 –3⎟⎠ ⎣ ⎦ ⎢ tan 1 ⎥
⎢⎣ 2 ⎥⎦ q
But when n = 2 no choice among (A), (B), (C) are match =
= cos2. AT
with A2 q 2
sec 2
2
The correct option is (D)
The correct option is (B)
9. Trace of A = a11 + a22 + … ann = k + k + … + k = nk.
The correct option is (C) ⎛ x + y⎞ ⎡ 1 + xy ⎤
15. A(z) = A ⎜ ⎟ = ⎢ ⎥
⎝ 1 + xy ⎠ ⎣ (1 - x )(1 - y ) ⎦
⎡ 1 1 ⎤ ⎡ 1 1 ⎤
⎢ 2 2 ⎥⎥ ⎢ 2 2 ⎥⎥ ⎡ ⎛ x + y⎞⎤
10. A2 = ⎢ ⎢ ⎢ 1 –⎜ ⎥
⎢ 1 1 ⎥ ⎢ 1 1 ⎥ ⎢ ⎝ 1 + xy ⎟⎠ ⎥
⎢– – ⎥ ⎢– – ⎥ ⎢ ⎛ x + y⎞ ⎥
⎣ 2 2⎦ ⎣ 2 2⎦
⎢– 1 ⎥
⎡0 0 ⎤ ⎢⎣ ⎜⎝ 1 + xy ⎟⎠ ⎥⎦
A2 = ⎢ ⎥ =O
⎣0 0 ⎦ \ A(x) ⋅ A(y) = A(z)
\ A is nilpotent The correct option is (C)
The correct option is (C) 16. Let A be a skew symmetric matrix of order n. By definition
11. The matrix can be written as A′ = –A
⎡ 2 sin q - 1 sin q cos q ⎤ ⇒ |A′| = |–A|  ⇒ |A| = (–1)n |A|
⎢ ⎥
⎢ - sin q 2 cos q - 3 tan q ⎥ ⇒ |A| = –|A|(Q n is odd)
⎢ - cos q - tan q 0 ⎥⎦ ⇒ 2|A| = 0  ⇒ |A| = 0

\ A–1 does not exist.


The above matrix is skew symmetric if
2 sin q – 1 = 0 and 2 cos q – 3 = 0

The correct option is (D)

Objective_Maths_JEE Main 2017_Ch 5.indd 20 01/01/2008 04:08:34


Matrices  5.21

17. The system is 0x1 + x2 – x3 = 1 ⇒ (a, d, b, c) = (0, 0, 1, 1), (0, 1, 1, 1), (1, 0, 1, 1),

–x1 + 0x2 + 2x3 = 2 (1, 1, 0, 0),
x1 – 2x2 + 0x3 = 3
(1, 1, 0, 1), (1, 1, 1, 0)
⇒ 6 matrices

⎡ 0 1 –1⎤ ⎡ x1 ⎤ ⎡1 ⎤
⎢ ⎥
⇒ ⎢⎢ –1 0 2 ⎥⎥ ⎢ x2 ⎥ = ⎢⎢ 2⎥⎥ or AX = B

The correct option is (C)

⎣⎢ 1 –2 0 ⎥⎦ ⎢⎣ x3 ⎥⎦ ⎢⎣ 3⎥⎦ ⎡a b ⎤ 2
⎡ a 2 + bc ab + bd ⎤
22. Let A = ⎢ ⎥ so that A = ⎢ ⎥
Clearly, |A| = 0
⎣ c d ⎦ ⎢⎣ ac + dc bc + d 2 ⎥⎦
⎡ 4 2 2⎤ ⎡1 0 ⎤

= ⎢ ⎥
Now Adj A = ⎢⎢ 2 1 1 ⎥⎥
⎣0 1 ⎦
⎢⎣ 2 1 1 ⎥⎦ ⇒ a2 + bc = 1 = bc + d2 and (a + d)c = 0 = (a + d)b

\ (Adj A) B ≠ 0 ⇒ system is inconsistent
Since A ≠ 1, A ≠ 1, a = –d and hence


The correct option is (A) 1 - bc b
det A =

⎡a b ⎤ c - 1 - bc
18. Since ⎢ is a square root of I2 i.e., two rowed unit
⎣g –a ⎥⎦ = –1 + bc – bc = –1
matrix Statement 1 is true.
2
⎡a b ⎤ ⎡1 0 ⎤ But tr. A = 0, and hence statement-2 is false.
\ ⎢
=⎢
⎣g –a ⎥⎦ ⎥
⎣0 1 ⎦ The correct option is (D)
23. Each entry of A is integer, so the cofactor of every entry is
⎡1 0 ⎤ ⎡a b ⎤ ⎡a b ⎤
⇒ ⎢
⎥=⎢ an integer and hence each entry in the adjoint of matrix A is
⎣0 1 ⎦ ⎣ g –a ⎥⎦ ⎢⎣ g –a ⎥⎦ integer.
1
⎡1 0 ⎤ ⎡a + bg ab – ab ⎤ Now det A = ±1 and A–1 =
2
(adj A)
⇒ ⎢
⎥ = ⎢ ⎥ det( A)
⎣0 1 ⎦ ⎢⎣ ga – ga gb + a 2 ⎥⎦ ⇒ all entries in A–1 are integers.
⎡1 0 ⎤ ⎡a + bg
2 ⎤0 The correct option is (C)
⇒ ⎢
⎥ =⎢ ⎥
⎣0 1 ⎦ ⎢⎣ 0 2
a + bg ⎥⎦ 24. Ap+1 = (B + C)p+1
= p+1C0 Bp+1 + p+1C1 B pC + p+1C2 B p–1 C2
\ a2 + bg = 1 ⇒ 1 – a2 – bg = 0
 + … + p+1Cr B p+1–rCr + …
The correct option is (B)
[using BC = CB]
19. Clearly both statements are true but statement  2 is not a
­correct explanation of statement 1. But C 2 = 0
The correct option is (C) ⇒ C 3 = C 4 = … = C r = 0.
Thus, Ap+1 = p+1C0 Bp+1 + p+1C1 BpC

⎛ 1⎞
20. A(u1 + u2) = ⎜ 1⎟ , |A| = 1 = B p+1 + (p + 1)B pC = B p[B + (p + 1)C].
⎜ ⎟ Therefore, k = p.
⎝ 0⎠
1 The correct option is (A)
–1
A = adj A 25. We know that adj (adj A) = |A|n–2A if |A| ≠ 0, provided order
| A|
of A is n.
⎡1 ⎤
\ adj (adj A) = |A| A(as n = 3)
u1 + u2 = A-1 ⎢⎢1 ⎥⎥


\ det (adj (adj A)) = |A|3 det A = |A|4.
⎣⎢0 ⎦⎥
⎡ 1 2 -1 ⎤
⎡ 1 0 0⎤ ⎡ 1⎤
⎢ But |A| = ⎢ -1 1 2 ⎥⎥ = 14.

–1
A = ⎢ -2 1 0 ⎥ = ⎢⎢ -1⎥⎥

⎢⎣ 2 -1 1 ⎥⎦
⎢⎣ 1 -2 1⎥⎦ ⎢⎣ -1⎥⎦
\ det (adj (adj A)) = (14)4.
The correct option is (D)
The correct option is (A)
21. |A| = ad – bc
26. We have,
If A is invertible then |A| ≠ 0
⇒ ad ≠ bc ⎡a b c ⎤ ⎡a c b⎤
⇒ (i) ad = 0 and bc = 1 AA′ = ⎢⎢ c a b ⎥⎥ ⎢⎢ b a c ⎥⎥

or (ii) ad = 1 and bc = 0 ⎢⎣ b c a ⎥⎦ ⎢⎣ c b a ⎥⎦

Objective_Maths_JEE Main 2017_Ch 5.indd 21 01/01/2008 04:08:37


5.22  Chapter 5

⎡ a 2 + b 2 + c 2 ac + ab + bc ab + bc + ca ⎤ Again, for n = 3, we have



⎢ ⎥ (A–1BA)3 = (A–1B2A)(A–1BA) = A–1B3A

= ⎢ ca + ab + bc a 2 + b 2 + c 2 cb + ba + ac ⎥ = I.
⎢ 2 2 2 ⎥ \ Generalizing the case (A–1BA)n = A–1BnA
⎢⎣ ab + cb + ac bc + ca + ab a + b + c ⎥⎦
The correct option is (C)
\ a2 + b2 + c2 = 1 and ab + bc + ca = 0. 32. Since A is invertible matrix, therefore |A| ≠ 0.
⇒ a + b + c = ±1 Since |A| = |A′|
Also, abc = p. Therefore, a, b, c are the roots of the equation \ |A′| ≠ 0 i.e., A′ is invertible.
x3 ± x2 + p = 0.
Now, A adj A = |A| In
The correct option is (B)
⇒ (A adj A)′ = (|A| In)′ ⇒ (adj A)′ A′ = |A| In (1)
27. Let A be of order n × n.
Also, (adj A′) A′ = |A′| In
Since A × adj A = |A| I
⇒ (adj A′) A′ = |A| In  (Q |A| = |A′|)(2)
\ |A × adj A| = |A|n
From (1) and (2),
⇒ |A| |adj A| = |A|n
we have (adj A′) A′ = (adj A)′ A′
\ |adj A| = |A|n–1
⇒ adj A′ = (adj A)′ (by cancellation law)
Since A is singular,
The correct option is (A)
\ |A| = 0.
33. We know that B (adj B) = |B| In for every square matrix B of
\ |adj A| = 0. order n.
Hence, adj A is singular. Replacing B by adj A,
The correct option is (B) we get (adj A) (adj (adj A)) = |adj A| In
28. A2 = 2A – I(1) ⇒ adj A (adj (adj A)) = |A|n–1 In
Multiplying by A, we have (Q |adj A| = |A|n–1)
A3 = 2A2 – A = 2(2A – I) – A {Using (1)} ⇒ A {adj A (adj (adj A))} = A {|A|n–1 In}
3
⇒ A = 3A – 2I  (Pre-multiplying both sides by A)
Again, multiplying by A, we get ⇒ (A adj A) (adj (adj A)) = |A|n–1 (AIn)
A4 = 3A2 – 2AI ⇒ A4 = 3(2A – I) – 2A
{Using (1)}  (by associativity)
⇒ A4 = 6A – 3I – 2A
⇒ |A| In (adj adj A) = |A|n–1 A
⇒ A4 = 4A – 3I
 (Q A adj A = |A| In)
Hence, by induction, we have An = nA – (n – 1)I
⇒ |A| (adj adj A) = |A|n–1 A
The correct option is (A)
⇒ adj adj A = |A|n–2 A
⎛ x + y ⎞ ⎡ 1 + xy ⎤  (Q |A| ≠ 0 \ dividing both sides by |A|)
29. A(z) = A ⎜ = ⎢ ⎥
⎝ 1 + xy ⎟⎠ ⎣ (1 - x )(1 - y ) ⎦ The correct option is (C)
⎡ ⎛ x + y ⎞⎤ x⎤ ⎡4 5 ⎡4 5x ⎤
⎢ 1 –⎜ ⎥
⎢ ⎝ 1 + xy ⎟⎠ ⎥ 34. Let A = ⎢⎢ 5 6
y ⎥⎥ ~ ⎢⎢ 5 6 x + d ⎥⎥
⎢ ⎛ x + y⎞ ⎥ ⎣⎢ 6 k
z ⎥⎦ ⎢⎣ 6 k x + 2d ⎥⎦
⎢– 1 ⎥
⎢⎣ ⎜⎝ 1 + xy ⎟⎠ ⎥⎦
[Q x, y, z are in A.P.]
\ A(x) ⋅ A(y) = A(z). ⎡4 5 x ⎤

~ ⎢5 6 x + d ⎥⎥
The correct option is (C)
30. Let A be a skew-symmetric matrix of order n. By definition ⎢⎣ 0 k - 7 0 ⎥⎦
A′ = –A [Applying R3 → R3 + R1 – 2R2]

⇒ |A′| = |–A| ⇒ |A| = (–1)n |A| Rank of A is 2 if k – 7 = 0 i.e. k = 7 and d is any arbitrary

⇒ |A| = –|A|[Q n is odd] number.
⇒ 2 |A| = 0 ⇒ |A| = 0.
The correct option is (B)
\ A–1 does not exist.
⎡ a1 0 0  0 ⎤
The correct option is (D) ⎢0 a 0  0 ⎥⎥
31. Consider n = 2 35. We have, D = ⎢ 2
⎢M M M M M ⎥
\ (A–1 BA)2 = (A–1BA)(A–1BA) ⎢ ⎥
⎣0 0 0  an ⎦
= (A–1B)(AA–1)(BA) = A–1B2A

Objective_Maths_JEE Main 2017_Ch 5.indd 22 01/01/2008 04:08:38


Matrices  5.23

a1 0 0  0 ⇒ A–1 ⋅ A.A.A′ = A–1A
0 a2 0  0 ⇒ IAA′ = I ⇒ AA′ = I
|D| =
= a1 a2 a3 … an \ A is orthogonal matrix.
M M M M M
The correct option is (B)
0 0 0  an
41. Let B = A + In. Since A = B – In, the condition An = a A can
⎡ a2 a3 ...an 0 . . 0 ⎤ be written in the form
⎢ 0 a1a3 ...an . . 0 ⎥ (B – In)n = a (B – In)
adj D = ⎢

⎢ M M M M M ⎥ ⇒ Bn–nC1 Bn–1 + nC2 Bn–2 + … + (–1)nIn
⎢ ⎥ = a B – a In
⎣ 0 0 . . a1a2 ...an -1 ⎦
⇒ Bn–nC1 Bn–1 + nC2 Bn–2 + … + (–1)n–1 B – a B
1 1
\ D–1 =
adj D = = –a In – (–1)n In
| D| a1a2 a3 ...an
⇒ B (Bn–1–nC1 Bn–2 + nC2 Bn–3 + … + (–1)n–1 In – a In)
⎡ a2 a3 ... an 0 ⋅ 0 ⎤ = [(–1)n+1 – a] In
⎢ a1 a3 ... an ⋅ ⎥
0 0 Since (–1)n+1 – a ≠ 0, \ B is invertible.

= ⎢ ⎥
⎢ M M M M ⎥ \ A + In is invertible i.e., non-singular.
⎢ ⎥
⎢⎣ 0 0 ⋅ a1 a2 ...an - 1 ⎥⎦ The correct option is (B)
42. Writing Q–1BP–1 as (Q–1B)P–1, we have
⎡ 1 ⎤
⎢a 0 . 0 ⎥ adj [(Q–1B)P–1] = (adj. P–1) (adj Q–1 B)

⎢ 1 ⎥
⎢ ⎥ [Q adj (AB) = adj B. adj. A]

1
⎢ 0 . 0 ⎥ -1 -1 -1 -1 = (adj. P–1) (adj B) (adj Q–1)

⎢ a2 ⎥ = diag. ( a1 a2 a3 ... an ) ⎡ -1 adj A ⎤
⎢ M M M M ⎥ = (|P–1| (P–1)–1 A (|Q–1| (Q–1)–1)
⎢Q A = ⎥
⎢ ⎥ ⎣ | A| ⎦
= |P–1| PA |Q–1| Q

⎢ 1 ⎥
⎢ 0 0 .
⎣ an ⎥⎦ =

1
PA | Q -1 | Q [Q (P–1)–1 = P, (Q–1)–1 = Q]
|P|

The correct option is (C)
= PAQ(Q |P| = |Q| = 1)

36. We have, B = A–1A′

The correct option is (A)
Then,  BB′ = (A–1A′) (A–1A′)′ = (A–1A′) A(A–1)′
= A–1A′ A(A′)–1 [by property of inverse] ⎛ a⎞
1
⎜ n⎟ 1 ⎛ n a⎞ 1
= A–1 (AA′) (A′)–1 = I.[A′A = AA′ given] 43. A = ⎜ ⎟ = ⎜ ⎟⎠ = n B
The correct option is (A) ⎜- a 1⎟
n ⎝ - a n
⎝ n ⎠
37. We have,
(I – A) (I + A + A2) = I – A3 = 1. ⎛ n a⎞
where, B = ⎜

This proves that I – A and I + A + A2 are inverse of each other. ⎝ - a n ⎟⎠
The correct option is (B) Put n = r cos q, a = r sin q,

a
38. We have, \ n2 + a2 = r2, q = tan -1

n
An = I  ⇒  A(An–1) = I  ⇒  An–1 = A–1
1 ⎛ r cos q r sin q ⎞
Thus, BAn–1 B–1 – BA–1 B–1 = BA–1 B–1 – BA–1 B–1 = 0. \ A = ⎜

n ⎝ - r sin q r cos q ⎟⎠
The correct option is (A)
39. We can form matrices of order 12 × 1, 1 × 12, 6 × 2, 2 × 6, r ⎛ cos q sin q ⎞

=
4 × 3, 3 × 4, i.e., matrices of 6 different orders. n ⎝⎜ - sin q cos q ⎠⎟
Again, 12 different real numbers can be arranged in 12! r n ⎛ cos nq sin nq ⎞
ways. \ An =
⎜ ⎟
nn ⎝ - sin nq cos nq ⎠
\ Total number of different matrices = 6 (12)! n/ 2
The correct option is (C) ⎛ n2 + a 2 ⎞ ⎛ cos nq sin nq ⎞

= ⎜ ⎟ ⎜⎝ - sin nq cos nq ⎟⎠
40. Since A is skew-symmetric, therefore A′ = –A ⎝ n2 ⎠
Again, A2 + I = 0  ⇒  A2 = 0 – I = – I
⎛ cos nq sin nq ⎞
n/ 2
\ A.A = –I
An ⎛ a2 ⎞ ⎜ n n ⎟
\
= ⎜1 + 2 ⎟ ⎜ ⎟
⇒ A.A.A′ = –IA′ = I(–A′)
n ⎝ n ⎠ ⎜ - sin nq cos nq ⎟

= –I(–A) = IA = A. ⎝ n n ⎠

Objective_Maths_JEE Main 2017_Ch 5.indd 23 01/01/2008 04:08:41


5.24  Chapter 5

a2 \ B = (I – A)–1
⎡ n ⎤ 2n 2

⎛ a2 ⎞ ⎢⎛
n/ 2
a2 ⎞ a ⎥
2 Hence, (I – A)–1 = I + A + A2 + … + Ak–1
Now,  Lt ⎜1 + 2 ⎟ = Lt ⎢⎜1 + 2 ⎟ ⎥ = e0 = 1 Thus, p = –1.
n→∞ ⎝ n ⎠ n→∞ ⎝ n ⎠ ⎥

⎣ ⎦ The correct option is (A)
1 sin nq 45. Since A satisfies the equation x3 – 5x2 + 4x + kI = 0, ­therefore,
Lt cos nq = Lt =0
n→∞ n n→∞ n A3 – 5A2 + 4A + kI = 0
[Q cos (nq), sin (nq) lie between –1 and 1] A–1 exists if k ≠ 0 since if k = 0, then the above equation gives
A = 0 and in that case A–1 will not exist.
1 n ⎡0 0 ⎤ The correct option is (B)
\
Lt A = ⎢ ⎥ = 0.
n
n→∞ ⎣0 0 ⎦ 46. det M = 1 ⇒ det M′ = 1
The correct option is (B) Now, det (M – I) = det (M – I) det M′
44. Let B = I + A + A2 + … + Ak–1 = det (MM′ – IM′)(Q det A det B = det (AB))
\ B(I – A) = (I + A + A2 + … + Ak–1) (I – A) = det (I – M′) = – det (M′ – I) = –det (M – I)
= I – A + A + A2 – A3 + … + Ak–1 – Ak Thus, det (M – I) = –det (M – I) ⇒ det (M – I) = 0.
= I – Ak = I(Q Ak = 0) The correct option is (A)

More than One Option Correct Type


47. Let B be non-singular, then B–1 exists. 49. The augmented matrix C = [A B]
Now, AB = 0 (given) ⇒ (AB) B–1 = 0B–1 ⎡ 2 -3 6 -5 3 ⎤
 (post-multiplying both sides by B–1) = ⎢⎢ 0 1 -4 1 1 ⎥⎥

⇒ A (BB–1) = 0 (by associativity) ⎢⎣ 4 -5 8 -9 k ⎥⎦
⇒ AIn = 0 (Q BB–1 = In)
⇒ A = 0. ⎡ 2 -3 6 -5 3 ⎤
But A is a non-null matrix. Hence, B is a singular matrix. ~ ⎢⎢ 0 1 -4 1 1 ⎥⎥ [R3 → R3 – 2R1]
Similarly, it can be shown that A is a singular matrix. ⎢⎣ 0 1 -4 1 k - 6 ⎥⎦

The correct option is (A) and (B) ⎡ 2 -3 6 -5 3 ⎤

~ ⎢ 0 1 -4 1 1 ⎥⎥ [R3 → R3 – R2]
⎡ –1 2 5 ⎤ ⎡0 0 a + 6 ⎤
48. A = ⎢⎢ 2 –4 a – 4 ⎥⎥ = ⎢⎢0 0 – a – 6 ⎥⎥ ⎢⎣ 0 0 0 0 k - 7 ⎥⎦
⎢⎣ 1 –2 a + 1 ⎥⎦ ⎢⎣1 –2 a + 1 ⎥⎦  (i)  There is no solution if r (A) ≠ r (C)
(Operating R1 → R1 + R3 and R2 → R2 – 2R3)
Here, r (A) = 2. r (C) = 3 if k – 7 ≠ 0 or k ≠ 7.
(ii)  There are infinite solutions if r (A) = r (C) < 4.
⎡0 0 0 ⎤
Already, r (A) = 2. r (C) = 2 if k – 7 = 0 or k = 7.
= ⎢⎢0 0 – a – 6 ⎥⎥ (Operating R1 → R1 + R2)

The correct option is (A) and (D)
⎢⎣1 –2 a + 1 ⎥⎦
50. Let A be a symmetric matrix.
⎡0 0 0 ⎤ Then, (An)′ = (AAA … n times)′
When a = –6, A = ⎢⎢0 0 0 ⎥⎥ ,  \ r(A) = 1
= (A′ A′ A′ … A′ n times)
⎢⎣1 –2 –5⎥⎦ = (A′ )n = An(Q A′ = A)
n
\ A is also a symmetric matrix.
⎡0 0 0 ⎤
Now, let A be a skew-symmetric matrix. Then A′ = –A.
When a = 6, A = ⎢⎢0 0 –12⎥⎥ ,  \ r(A) = 2


⎢⎣1 –2 –5 ⎥⎦ We have, (An)′ = (AAA … n times)
= (A′ A′ A′ … A′ n times) [Q (AB)′ = B′ A′]
⎡0 0 0 ⎤ = (A′ )n = (– A)n(Q A′ = –A)
When a = 1, A = ⎢⎢0 0 –7⎥⎥ ,  \ r(A) = 2
= (–1)n An
⎢⎣1 –2 –2⎥⎦
⎪⎧ A if n is even ⎪⎫
n

= ⎨ ⎬
⎡0 0 0 ⎤ n
⎩⎪ - A if n is odd ⎭⎪
When a = 2, A = ⎢⎢0 0 –8⎥⎥ ,  \ r(A) = 2.
Hence, An is symmetric if n is even and skew-symmetric if n

⎢⎣1 –2 3 ⎥⎦ is odd.

The correct option is (B) and (D)
The correct option is (A), (B), (C) and (D)

Objective_Maths_JEE Main 2017_Ch 5.indd 24 01/01/2008 04:08:43


Matrices  5.25

1 1
51. Since |A| ≠ 0, therefore A–1 exists. 54. (A) A * I =( AI + IA) = ( A + A) = A
2 2
Now, AA–1 = I = A–1A \ A * I = A,
⇒ (AA–1)′ = I′ = (A–1A)′ \  (A) holds.
⇒ (A–1)′ A′ = I = A′ (A–1)′
1 1
⇒ (A–1)′ A = I = A (A–1)′(Q A′ = A) (B) A * A =
( AA + AA) = ( A2 + A2 ) = A2
2 2
⇒ A–1 = (A–1)′ ⇒ A–1 is symmetric.
\  (B) holds.
Also, since |A| ≠ 0, 1
\ A–1 exists such that (C) A * B = ( AB + BA)
2
AA–1 = I = A–1A  ⇒ |AA–1| = |I| 1 1
B * A = ( BA + AB ) = ( AB + BA)
⇒ |A| |A–1| = 1 (Q |AB| = |A| |B|) 2 2
–1 1 [Q addition is commutative]
⇒ |A | =
| A| \ A * B = B * A,
The correct option is (A) and (D) \  (C) holds
52. (A) For any orthogonal matrix A, we have 1
(D) A * (B + C) = ( A ( B + C ) + ( B + C ) A)
A′A = I 2
Let B be a matrix such that AB = I. 1
= ( AB + AC + BA + CA)
Now we have 2
A′ = A′ I [by property of unit matrix] 1 1
= ( AB + BA) + ( AC + CA)
= A′(AB) = (A′A)B = IB = B 2 2
Therefore, (A′)′(A) = AA′ = AB = I =A*B+A*C
⇒  A′ is orthogonal. \  (D) holds.
(B) For any orthogonal matrix A, we have The correct option is (A), (B), (C) and (D)
A′A = I 55. Given AB = BA
⇒ |A′A| = |I|  ⇒ |A′| |A| = 1 (A) The result AnB = BAn is true for n = 2, since
⇒ |A| ≠ 0, i.e., A is non-singular. A2B = A(AB) = A(BA) = (AB)A = (BA)A = BA2
(C) Let A and B be two orthogonal matrices, therefore, Let the result be true for n = k
(AB)′ (AB) = B′A′AB [by property of transpose] i.e., AkB = BAk

= B′(A′A)B [by asssociative law] For n = k + 1, we have
= B′(IB) = B′B = I Ak+1 B = A(AkB) = A(BAk) = (AB)Ak = (BA)Ak
⇒  AB is orthogonal. = BAAk+1
(D) Let A be orthogonal matrix and B be its inverse matrix. Thus, the result is true for n = k + 1
Then, we have, Hence, by induction the result is true ∀ n ∈ N
A′A = I(1) (B), (C)  Since A and B commute, both these options hold.
and,  AB = I = BA(2) (D) (An – Bn) (An + Bn) = A2n + An Bn – Bn An – B2n
Now, we have, = A2n – B2n.[A and B commute]
(AB)′ (AB) = B′A′AB The correct option is (A), (B), (C) and (D)
= B′(A′A)B = B′(IB)B = B′B(3) 56. We have,
Also, from equation (2), we have 1 0 -2
(AB)′ = I′ = I |A–1| = - 2 1 0 = –2 (–2 + 1) = 2
i.e.,  B′B = I [Using equation (3)] -1 1 0
⇒  B is orthogonal. 1 1
(A) |A| = =
The correct option is (A), (B), (C) and (D) | A-1 | 2
53. We can see that A′ = A. ⎡ 1 0 - 2⎤
1
Therefore, A′A = I (B) adj A = A–1 |A| = ⎢⎢ - 2 1 0 ⎥⎥
2
⇒ A2 = I  ⇒ |A2| = |I| ⎢⎣ - 1 1 0 ⎥⎦
⇒ |A2| = 1  ⇒ |A| = ±1 ⎡ 1/ 2 0 -1⎤
Therefore, a3 + b3 + c3 – 3abc = ± 1
= ⎢⎢ -1 1/ 2 0 ⎥⎥
⇒ a3 + b3 + c3 = 2, 4. ⎢⎣ -1/ 2 1/ 2 0 ⎥⎦
The correct option is (B) and (D)

Objective_Maths_JEE Main 2017_Ch 5.indd 25 01/01/2008 04:08:45


5.26  Chapter 5

⎛ - 1 1⎞ 1
(C) |adj A| = - 1 ⎜
+ ⎟ =
= a2I2 + abIA + baAI + b2A2
⎝ 2 4⎠ 4
1
= a2I + 2abA + b2A2(Q I2 = I, IA = AI)
(D) |A′| = |A| =

= a2I + 2abA – b2I(Q A2 = –I)
2

The correct option is (B) and (D)
= (a2 – b2)I + 2abA

⎡ 1 -1 1⎤ ⎡a2 - b 2 0 ⎤ ⎡ 0 1⎤

= ⎢ ⎥ + 2ab ⎢
57. A = ⎢ 2 -1 0 ⎥⎥

⎢⎣ 0 2
a - b ⎥⎦ 2 ⎥
⎣ - 1 0⎦
⎢⎣ 1 0 0 ⎥⎦
⎡a2 - b 2 2ab ⎤

= ⎢ ⎥
⎡ 1 - 1 1⎤ ⎡ 1 - 1 1⎤ ⎡0 0 1⎤ ⎢⎣ - 2ab a - b 2 ⎥⎦
2

Then,  A2 = ⎢⎢ 2 - 1 0 ⎥⎥ ⎢⎢ 2 - 1 0 ⎥⎥ = ⎢⎢0 - 1 2⎥⎥



On equating corresponding elements in (aI + bA)2 and A2
⎢⎣ 1 0 0 ⎥⎦ ⎢⎣ 1 0 0 ⎥⎦ ⎢⎣ 1 - 1 1⎥⎦
we get
⎡0 0 1⎤ ⎡ 1 - 1 1⎤ ⎡ 1 0 0⎤ a2 – b2 = 0 (1)
and,  A = ⎢0 - 1 2⎥⎥ ⎢⎢ 2 - 1 0 ⎥⎥ =
⎢ 3 ⎢0 1 0 ⎥ = I
⎢ ⎥
2ab = 1 (2)
⎢⎣ 1 - 1 1⎥⎦ ⎢⎣ 1 0 0 ⎥⎦ ⎢⎣0 0 1⎥⎦ From (1),
a = ± b. If a = b, then from (2),
i.e., A(A2) = A2(A) = I

2b2 = 1

⇒ A2 is the inverse of A[|A| ≠ 0 ⇒ A–1 exists]
1 1
Also,  A4 = A3A = IA = A
⇒ b = ±
whence a = ± .
2 2
But  A5 = A4A = A2 ≠ A.
Again, if a = –b, then from (2),


The correct option is (A) and (B) 2b2 = –1

⎡ 0 1⎤ ⎡ 0 1⎤ ⎡ - 1 0⎤ i i
58. A2 = ⎢ = ⎢ ⇒ b = ∓
whence a = ± .
⎥ ⎢ ⎥ ⎥ = –1 2 2
⎣ - 1 0⎦ ⎣ - 1 0⎦ ⎣ 0 - 1⎦

The correct option is (A) and (D)
Also, (aI + bA)2 = (aI + bA) (aI + bA)

Passage Based Questions


59. We have, ⎡ 1 0 0 0⎤
⎛ 1 ⎞
⎡0 1 2 1⎤ ⎡ 1 0 2 1⎤ ~ ⎢⎢0 1 0 0 ⎥⎥ ⎜⎝ R3 → R3 ⎟⎠
⎢ 1 2 3 2⎥ ~ ⎢ 2 1 3 2⎥ 2
⎢ ⎥ ⎢ ⎥ C1 ↔ C2 ⎣⎢0 0 1 1⎥⎦
⎢⎣ 3 1 1 3⎥⎦ ⎢⎣ 1 3 1 3⎥⎦
⎡ 1 0 0 0⎤
~ ⎢⎢0 1 0 0 ⎥⎥ (C4 → C4 - C3 )
⎡1 0 0 0⎤
C3 → C3 - 2C1 ⎢⎣0 0 1 0 ⎥⎦
~ ⎢⎢ 2 1 - 1 0 ⎥⎥
C4 → C4 - C1 Hence, [I3 : 0] is the normal form of A and, therefore, the
⎢⎣ 1 3 - 1 2⎥⎦
rank of the matrix A is 3.
⎡1 0 0 0⎤ The correct option is (C)
⎛ R2 → R2 - 2 R1⎞
~ ⎢0 1 - 1 0 ⎥⎥
⎢ 60. We have,
⎝⎜ R3 → R3 - R1 ⎠⎟
⎢⎣0 3 - 1 2⎥⎦ ⎡ 1 - 1 2 - 3⎤
⎢4 1 0 2⎥⎥
⎡ 1 0 0 0⎤ A= ⎢

⎢0 3 1 4⎥
~ ⎢⎢0 1 0 0 ⎥⎥ (C3 → C3 + C2 ) ⎢ ⎥
⎣ 0 1 0 2⎦
⎢⎣0 3 2 2⎥⎦
⎡1 0 0 0⎤
⎡ 1 0 0 0⎤ ⎢0 ⎛ C2 → C2 + C1 ⎞
5 - 8 14 ⎥⎥
~ ⎢⎢0 1 0 0 ⎥⎥ ( R3 → R3 - 3R2 ) ~⎢ ⎜ C → C - 2C ⎟
⎢0 3 1 4⎥ ⎜ 3 3 1⎟
⎢⎣0 0 2 2⎥⎦ ⎢ ⎥ ⎝ C4 → C4 + 3C1⎠
⎣0 1 0 2⎦

Objective_Maths_JEE Main 2017_Ch 5.indd 26 01/01/2008 04:08:49


Matrices  5.27

⎡1 0 0 0⎤ ⎡ 2 3 4 ⎤ ⎡ - 1 2 2⎤
⎢0 2⎥⎥
1 0 61. A = ⎢⎢ 3 1 2⎥⎥ ~ ⎢⎢ 3 1 2⎥⎥ R1 ↔ R3
~⎢ ( R2 ↔ R4 )
⎢0 4⎥

3 1
⎥ ⎣⎢ - 1 2 2⎥⎦ ⎢⎣ 2 3 4 ⎥⎦
⎣0 5 -8 4⎦
⎡ - 1 2 2⎤
R2 → R2 + 3R1
⎡1 0 0 0⎤ ~ ⎢⎢ 0 7 8⎥⎥
⎢0 R3 → R3 - 2 R1
1 0 0 ⎥⎥ ⎣⎢ 0 7 8⎥⎦
~⎢
⎢0 3 1 - 2⎥
(C4 → C4 - 2C2 )
⎢ ⎥ ⎡ - 1 2 2⎤
5 -8
⎣0 ~ ⎢⎢ 0 7 8⎥⎥
4⎦
R3 → R3 - R2
⎡1 0 0 0⎤ ⎢⎣ 0 0 0 ⎥⎦
⎢0 1 0 0 ⎥⎥
~⎢
⎢0 1 - 2⎥
( R3 → R3 - 3R2 ) ⎡ 1 - 2 - 2⎤
R1 → - R1
0 ⎢ 8 ⎥⎥
⎢ ⎥ ~ ⎢0 1
⎣0 5 -8 4⎦ ⎢
1
7 ⎥ R2 → R2
⎢0 0 0⎦ ⎥ 7
⎡1 0 0 0⎤ ⎣
⎢0 1 0 0 ⎥⎥
The last equivalent matrix is in Echelon form. The number
~⎢
⎢0 0 1 0⎥
(C4 → C4 + 2C3 ) of non-zero rows in this matrix = 2
⎢ ⎥ Rank of A = Number of non-zero rows = 2.

⎣0 5 - 8 - 12⎦

The correct option is (B)
⎡1 0 0 0⎤
⎢0 1 0 0 ⎥⎥ ⎡ R4 → R4 - 5 R1,⎤ ⎡ 1 3 4 3⎤
~⎢ ⎢ ⎥ ⎢ 3
62. A = ⎢ 9 12 9⎥⎥
⎢0 0 1 0⎥ ⎣ R4 → R4 + 8 R3 ⎦
⎢ ⎥ ⎢⎣ - 1 - 3 - 4 - 3⎥⎦
⎣0 0 0 - 12⎦
⎡ 1 3 4 3⎤
⎡1 0⎤ 0 0 R2 → R2 - 3R1
⎢0 ~ ⎢⎢0 0 0 0 ⎥⎥
0 ⎥⎥

1 0 ⎡ -1 ⎤ R3 → R3 + R1
~⎢ = I4 ⎢C4 → C4 ⎥ ⎢⎣0 0 0 0 ⎥⎦
⎢0 0⎥ 0 1 ⎣ 12 ⎦
⎢ ⎥
The equivalent matrix is in Echelon form. The number of
⎣0 1⎦ 0 0
Hence, r(A) = 4.
non-zero rows in this matrix is 1. Therefore, the rank of
A = 1.

The correct option is (D)

Assertion-Reason Type

⎡1 1 .......1⎤ ⎡ n - r n - r .......n - r ⎤
⎢ n - r n - r .......n - r ⎥
63. Here, A = ⎢⎢1 1 .......1⎥⎥
\ B – rI = ⎢

⎣⎢1 1 .......1⎥⎦ ⎢ ....... ....... ............... ⎥
⎢ ⎥
⎡1 1 .......1⎤ ⎡1 1 .......1⎤ ⎣ n - r n - r .......n - r ⎦
\ A = A.A = ⎢⎢1 1 .......1⎥⎥ ⎢⎢1 1 .......1⎥⎥
2
= (n – r) A
⎢⎣1 1 .......1⎥⎦ ⎢⎣1 1 .......1⎥⎦ Hence, (B – rI) [B – (n2 – nr + r) I]


= (B – rI) [(B – rI) – n(n – r) I]
⎡ n n .......n⎤
⎢ n n .......n⎥
= (n – r) A[(n – r) A – n(n – r) I]

= ⎢ ⎥ = nA
= (n – r)2 A (A – nI)
⎢ . . .........⎥
⎢ ⎥
= (n – r)2 A2 – n(n – r)2 AI
⎣ n n .......n⎦

= (n – r)2 ] [A2 – nA]
⎡ n n - r .......n - r ⎤
= (n – r)2 [nA – nA][Q A2 = nA]
⎢n - r .......n - r ⎥⎥
n
= (n – r)2 (0) = 0.
B= ⎢

⎢ ....... ....... .............. ⎥
=BA
⎢ ⎥
⎣n - r n - r .......n ⎦
The correct option is (A)

Objective_Maths_JEE Main 2017_Ch 5.indd 27 01/01/2008 04:08:51


5.28  Chapter 5

Previous Year’s Questions


⎡a b⎤
64. We have A = ⎢ ⎥ so that 71. The product
⎣b a ⎦
⎡ 5 5a a ⎤ ⎡ 5 5a a ⎤
⎡a b⎤ ⎡a A2 = ⎢⎢0 a 5a ⎥⎥ ⎢⎢0 a 5a ⎥⎥
2 b⎤
A =⎢ ⎥⎢
⎣b a ⎦ ⎣b a ⎥⎦ ⎢⎣0 0 5 ⎥⎦ ⎣⎢0 0 5 ⎥⎦

⎡a2 + b2 2ab ⎤
=⎢ ⎥ ⎡ 25 25a + 5a 2 5a + 25a 2 + 5a ⎤
⎢⎣ 2ab 2
a + b ⎥⎦ 2 ⎢ ⎥
⇒ A2 = ⎢ 0
a2 5a 2 + 25a ⎥
⇒ a = a2 + b2, b = 2ab.
⎢ ⎥
⎢⎣ 0 0 25 ⎥⎦

The correct option is (B)
⇒ 625a2 = 25
⎡1 0 0⎤ 1
⇒ |a| =
65. For given A, AA = ⎢⎢0 1 0 ⎥⎥ = 1 . 5
⎢⎣0 0 1 ⎥⎦ The correct option is (C)
The correct option is (B) 72. First row with exactly one zero; total number of cases = 6
66. For AB = I, we have First row 2 zeros we get more cases
A(10 B) = 10 I Total we get more than 7.
Directions: Questions Number 72 to 76 are Assertion-
⎡1 -1 1 ⎤ ⎡ 4 2 2 ⎤ Reason type questions. Each of these questions contains two
⇒ ⎢⎢ 2 1 -3⎥⎥ ⎢⎢ -5 0 a ⎥⎥ statements.
⎢⎣1 1 1 ⎥⎦ ⎢⎣ 1 -2 3 ⎥⎦ Statement 1: (Assertion) and Statement 2: (Reason)
Each of these questions also has four alternative choices,
⎡10 0 5 - a ⎤ ⎡1 0 0 ⎤
only one of which is the correct answer. You have to select
= ⎢⎢ 0 10 a - 5⎥⎥ = 10 ⎢⎢0 1 0 ⎥⎥ if a = 5 . the correct choice.
⎢ 0 0 5 + a ⎥⎦ ⎢⎣0 0 1 ⎥⎦
⎣ The correct option is (C)
The correct option is (B) 73. AT = A, BT = B :
2
67. Given A - A + I = 0 ( A( BA))T = ( BA)T AT = ( AT BT ) A = ( AB ) A = A( BA)
Multiplying A - I on both sides (( AB ) A)T = AT ( AB )T = A( BT AT ) = A( BA) = ( AB ) A
–1 2 –1 –1 –1
A A - A A + A - I = A  ⋅ 0 ∴ Statement 1 is correct

⇒ A - I + A–1 = 0
Statement 2
or A–1 = I - A. ( AB )T = BT AT = BA = AB  (Q AB is commutative)
The correct option is (D)
Statement 2 is also correct but it is not correct explanation of
68. By the principle of mathematical induction (1) is true. Statement 1
The correct option is (A)
The correct option is (A)
69. Given condition A2 - B2 = (A - B) (A + B) ⎛100 ⎞
⇒ A2 - B2 = A2 + AB - BA - B2 74. A = ⎜ 210⎟
⎜ ⎟
⇒ AB = BA. ⎝ 321 ⎠
The correct option is (B) ⎡a⎤ ⎡d ⎤
⎡1 2⎤ ⎡a 0⎤ Let u1 = ⎢b ⎥ ; u2 = ⎢⎢e ⎥⎥
⎢ ⎥
70. Given that A = ⎢ ⎥ B=⎢ ⎥
⎣3 4 ⎦ ⎣0 b⎦ ⎢⎣c ⎥⎦ ⎢⎣ f ⎥⎦

⎡ a 2b ⎤ ⎡1 ⎤ ⎡1 ⎤
⇒ AB = ⎢ ⎥ , and
⎣3a 4b ⎦ Au1 = ⎢0 ⎥ ⇒ u1 = ⎢⎢ -2⎥⎥
⎢ ⎥

⎡ a 0 ⎤ ⎡1 2⎤ ⎡ a 2a ⎤ ⎣⎢0 ⎦⎥ ⎣⎢1 ⎥⎦
BA = ⎢ ⎥⎢ ⎥=⎢ ⎥
⎣ 0 b ⎦ ⎣3 4 ⎦ ⎣3b 4b ⎦ ⎡0 ⎤
⎢ ⎥
Au2 = ⎢1 ⎥
Now, AB = BA only when a = b

⎢⎣0 ⎥⎦

The correct option is (D)

Objective_Maths_JEE Main 2017_Ch 5.indd 28 01/01/2008 04:08:54


Matrices  5.29

⎡0 ⎤ Let AAT = ⎡⎣bij ⎤⎦



⇒ u2 = ⎢⎢1 ⎥⎥
3× 3

Then,  b23 = 0 ⇒ 0 = 2a + 2 - 2b ,

⎣⎢ -2⎥⎦
b13 = 0 ⇒ 0 = a + 4 + 2b
⎡1 ⎤
⇒ 3a + 6 = 0 ⇒ a = -2, b = -1 .

⇒ u1 + u2 = ⎢⎢ -1⎥⎥


The correct option is (C)
⎢⎣ -1⎥⎦
⎡ 5a - b ⎤ T ⎡ 5a 3 ⎤
The correct option is (D) 75. A= ⎢ ⎥ and A = ⎢ - b 2 ⎥
⎣ 3 2 ⎦ ⎣ ⎦
75. For no solution
K +1 8 4k ⎡ 25a 2 + b 2 15a - 2b ⎤
= ≠ (1) AAT = ⎢ ⎥
k k + 3 3k - 1 ⎣ 15a - 2b 13 ⎦
⇒ ( k + 1)( k + 3) - 8k = 0

⎡10 a + 3b 0 ⎤
or, k 2 - 4 k + 3 = 0 ⇒ k = 1, 3 Now, A adj A = |A|I 2 = ⎢
+ 3b ⎥⎦

⎣ 0 10 a
But for k = 1, equation (1) is not satisfied

Given AAT = A. adj A
Hence k = 3.

15a - 2b = 0  (1)

The correct option is (A)
10 a + 3b = 13 (2)
76. B = A-1 A′ ⇒ AB = A′
Solving we get
ABB ′ = A′ B ′ = ( BA)′ = ( A-1 A′ A)′ = ( A-1 AA′ )′ = A .
5a = 2 and b = 3
⇒ BB′ = I .
∴ 5a + b = 5
The correct option is (B)
The correct option is (C)
⎡1 2 2 ⎤ ⎡1 2 a ⎤
77. Given that A = ⎢⎢ 2 1 -2⎥⎥ ⇒ AT = ⎢⎢ 2 1 2⎥⎥
⎣⎢ a 2 b ⎥⎦ ⎢⎣ 2 -2 b ⎥⎦

Objective_Maths_JEE Main 2017_Ch 5.indd 29 01/01/2008 04:08:56


Determinants 6.1

CHaPtER

6 Determinants

Chapter Highlights
Determinants, Minors and cofactors, Expansion of a determinant of order three, Properties of determinants,
Evaluation of determinants using elementary operations, Product of determinants of same order, Solution of
linear equations by determinants

DETERMINANTS a11 a12


|A|= = a11 a22 – a12 a21
A determinant is a pure number associated with a square a21 a22
matrix. Corresponding to each square matrix i.e., the determinant of a 2 × 2 matrix is obtained by taking
the product of the entries on the main diagonal and subtract-
⎡ a11 a12  a1n ⎤ ing from it the product of the entries in the other diagonal.
⎢a a22  a2 n ⎥⎥
A = ⎢ 21 Illustration
⎢    ⎥
⎢ ⎥ 2 3
⎣ an1 an 2  ann ⎦ 1. = 2 × 5 – 4 × 3 = 10 – 12 = – 2.
4 5
there is associated an expression, called the determinant of
A, denoted by det A or | A |, written as x −1 x +1
2. 2
= (x – 1) (x + 1) – (x + 1) (x2 – x + 1)
x − x +1 x +1
a11 a12  a1n
= x2 – 1 – (x3 + 1) = x2 – x3 – 2
a21 a22  a2 n
| A | = det A =
   MINORS AND COFACTORS
an1 an 2  ann
Minor of an Element of a Determinant
A matrix is an arrangement of numbers and it has no fixed
value but a determinant is a number and it has a fixed value. If we take an element of the determinant and delete the row
A determinant having n rows and n columns is called a and the column containing that element, the determinant
determinant of order n. left is called the minor of that element. It is denoted by Mij.
For example, given the 3 × 3 determinant
Determinant of a Square Matrix of Order 1 a11 a12 a13
Let A = [a11] be a 1 × 1 matrix, then the determinant of A is a21 a22 a23
the number a11 itself i.e., | a11| = a11. a31 a32 a33

Determinant of a Square Matrix of Order 2 a22 a23


Then the minor of a11 is M11 = ; the minor of a12
a32 a33
⎡a a12 ⎤ a21 a23
Let A = ⎢ 11 be a 2 × 2 matrix, then is M12 = and so on.
⎣ a21 a22 ⎥⎦ a31 a33

Objective_Maths_JEE Main 2017_Ch 6.indd 1 01/01/2008 04:14:03


6.2  Chapter 6

Cofactor of an Element of a Determinant = 1 (6 – 30) + 3 (0 + 10) + 4 (0 + 4)


The cofactor Cij of an element aij (the element in the ith row = – 24 + 30 + 16 = 22.
and jth column) is defined as Cij = (– 1)i + j Mij. It is denoted We can also expand the determinant along any row or any
by Cij. Thus, column. For example, if we had expanded the above deter-
minant along the first column, then
⎪⎧ M ij , when i + j is even
Cij = ⎨
1 −3 4
⎪⎩− M ij , when i + j is odd 2 5 −3 4
0 2 5 = (1) + 0 + (– 2)
For example, the cofactor of a12 in the 3 × 3 determinant 6 3 2 5

−2 6 3
a11 a12 a13
= 1(6 – 30) – 2 (– 15 – 8)
a21 a22 a23
= – 24 + 46 = 22, as before.
a31 a32 a33

a a23
Method 2: Using Sarrus Rule
1+2 a21 a23
is C12 = (– 1) = – 21 The following diagram called sarrus diagram, enables
a31 a33 a31 a33
us to write the value of the determinant of order 3 very
conveni-entlty.
EXPANSION OF A DETERMINANT OF a1 b1 c1
ORDER THREE Let D = a2 b2 c2 be a determinant of order 3.
a3 b3 c3
Consider the following determinant:
Write the elements as:
a1 b1 c1 a1 b1 c1 a1 b1
a2 b2 c2
a3 b3 c3 b2 c2 a2 b2
a2
We can find the value of this determinant by various
methods.
a3 b3 c3 a3 b3

Method 1 Multiply the elements joined by arrows. Assign the positive


sign to an expression if it is formed by a downward arrow
Step 1: Write the elements of the first row with alter- and negative sign to an expression if it is formed by an
na-tively positive and negative sign, the first element always upward arrow. Note that the first two columns are repeated
has positive sign before it. in the above table to complete the process. The value of the
Step 2: Multiply each signed element by the determinant given determinant is
of second order obtained after deleting the row and the col-
umn in which that element occurs. i.e., a1b2c3 + b1c2a3 + c1a2b3 – a3b2c1 – b3c2a1 – c3a2b1
2 −1 3
a1 b1 c1 For example, to evaluate 5 7 0 , write the elements as
b2 c2 a c2 a b2
a2 b2 c2 = a1 – b1 2 + c1 2 4 1 6
b3 c3 a3 c3 a3 b3
a3 b3 c3 2 –1 3 2 –1
= a1 (b2c3 – b3c2) – b1 (a2c3 – a3c2) + c1 (a2b3
– a3b2)
5 7 0 5 7
For example,
1 −3 4 4 1 6 4 1
2 5 0 5
0 2 5 = (1) + (– 3) (– 1) The value of the given determinant
6 3 −2 3
−2 6 3
= (2) (7) (6) + (– 1) (0) (4) + (3) (5) (1) – (4) (7)
0 2 (3) – (1) (0) (2) – (6) (5) (– 1)
+ (4)
−2 6
= 84 + 15 – 84 + 30 = 45

Objective_Maths_JEE Main 2017_Ch 6.indd 2 01/01/2008 04:14:06


Determinants 6.3

CAUTION SOlvED EXAMPlES

Sarrus rule does not work for determinants of order greater x3 + 4 x x+3 x−2
5 4 3 2
than 3. 1. If x − 2 5x x − 1 = ax + bx + cx + dx
x−3 x + 2 4x
Method 3: Using Cofactors + ex + f, be an identity in x, where a, b, c, d, e, f are
independent of x, then the value of f is
A determinant can also be evaluated by multiplying the
(A) 0 (B) 15
entries of any row (or column) by their cofactors and sum-
(C) 17 (D) None of these
ming the resulting products.
Solution: (C)
a1 b1 c1
Let D = a2 b2 c2 , then 0 3 −2
a3 b3 c3 We have, − 2 0 −1 = f
D = a1C11 + b1C12 + c1C13 = a1M11 – b1M12 + c1M13. −3 2 0

1 2 0 −1 (Putting x = 0 on both sides)


−1 4 1
3 −1 4 1 \ f = 17
For example, = 1 0 −3 3
−2 0 −3 3
3 1 2 a −b 0
4 3 1 2
2. If 0 a b = 0 then
3 4 1 3 −1 4 b 0 a
– 2 −2 −3 3 + 0 + 1 −2 0 −3 (A) a/b is one of the cube roots of unity
4 1 2 4 3 1 (B) a is one of the cube roots of unity
(C) b is one of the cube roots of unity
= 54 – 94 + 13 = 27. (D) None of these
Solution: (A)
NOTE
a −b 0
■ The value of determinant is same when expanded by any We have, 0 a b = 0 ⇒ a3 – b3 = 0
row or any column.
b 0 a
■ The above method of expansion is general and is valid for
determinant of any order. 3
⎛a⎞ a
⇒ ⎜ ⎟ = 1 ⇒ is one of the cube roots of unity.
⎝ b⎠ b

TRICk(S) FOR PROblEM SOlvINg 3. If a, b, c are different, then the value of x satisfying

■ If a row or a column of a determinant consists of all zeros, 0 x2 − a x3 − b


the value of the determinant is zero.
x2 + a 0 x 2 + c = 0 is
■ Always expand a determinant along a row or column with
maximum number of zeros. x4 + b x−c 0
■ If each element above or below the main diagonal of a
determinant is zero, then the value of the determinant is (A) c (B) c
the product of elements along the main diagonal. (C) b (D) 0
Solution: (D)
Since for x = 0, the determinant reduces to the determi-
CAUTION
nant of a skew-symmetric matrix of odd order which is
Sarrus rule does not work for determinants of order greater always zero. Hence, x = 0 is the solution of the given
than 3. equation.

Objective_Maths_JEE Main 2017_Ch 6.indd 3 01/01/2008 04:14:08


6.4  Chapter 6

l 2 + 3l l − 1 l + 3 a11 a12 a13 a11 a21 a31


4. If l + 1 1 − 2l l − 4 = pl4 + ql3 + rl2 + sl + a21 a22 a23 = a12 a22 a32
l−2 l+4 3l a31 a32 a33 a13 a23 a33
t be an identity in l, where p, q, r, s and t are constants, 2. If two adjacent rows (columns) of a determinant are
then the value of t is interchanged, the value of the determinant so obtained
(A) 0 (B) 10 is the negative of the value of the original determinant,
(C)  – 10 (D)  None of these i.e.,
Solution: (B) a11 a12 a13 a21 a22 a23
Putting l = 0 in the given identity, we get a21 a22 a23 = – a11 a12 a13
0 −1 3 a31 a32 a33 a31 a32 a33
1 1 − 4 = t 3. 
If two rows or columns of a determinant are identical
−2 4 0 then its value is zero, i.e.,
a11 a12 a13
⇒ t = 0 + 1 (0 – 8) + 3 (4 + 2) = 10
a11 a12 a13 = 0
5. If f (x) satisfies the equation a31 a32 a33
f ( x − 3) f ( x + 4) f [( x + 1)( x − 2) − ( x − 1) 2 4. 
If each element of a row or column of a determinant
is multiplied by a constant k then the value of the new
5 4 −5 =0
determinant is k times the value of the original deter-
5 6 15 minant, i.e.,
for all real x, then ka11 ka12 ka13 a11 a12 a13
(A) f (x) is periodic with period 7 a21 a22 a23 = k a21 a22 a23
(B) f (x) is periodic with period 1
(C) f (x) is non-periodic a31 a32 a33 a31 a32 a33
(D) f (x) is periodic with no fundamental period 5. 
If any two rows or columns of a determinant are pro-
Solution: (A) portional, then its value is zero, i.e.,
The given determinant = 90f (x – 3) – 10f (x + 4)
a11 a12 a13 a11 a12 a13
+ 10f (x – 3).
So, f (x) satisfies the equation f (x + 4) = f (x – 3). ka11 ka12 ka13 = k a11 a12 a13 = 0
Replacing x by x + 3, we get f (x + 7) = f (x) for all x. a31 a32 a33 a31 a32 a33
Hence, f (x) is periodic with period 7.
6. 
If each element of a row (or column) of a determinant
6. If d is the determinant of a square matrix A of order n,
is the sum of two or more terms, then the determi-
then the determinant of its adjoint is
nant can be expressed as the sum of the two or more
(A) d n (B)  d n–1 determ-inants, i.e.,
m+1
(C) d (D)  d
Solution: (B) a11 a12 a13
We have, | Adj. A | = | A |n – 1 = d n – 1 a21 + c1 a22 + c2 a23 + c3
a31 a32 a33

PROPERTIES OF DETERMINANTS a11 a12 a13
a11 a12 a13
Properties of determinants of order three only are stated = a21 a22 a23 + c1 c2 c3
below. However these properties hold for determinants of a31 a32 a33 a31 a32 a33
any order. These properties help a good deal in the evalua-
tion of determinants. 7. 
If each element of a row (column) of a determinant is
1. 
The value of the determinant remains unchanged multiplied by a constant k and then added to the corre-
if rows are changed into columns and columns are sponding elements of some other row (column), then
changed into rows, i.e., the value of the determinant remains the same, i.e.,

Objective_Maths_JEE Main 2017_Ch 6.indd 4 01/01/2008 04:14:10


Determinants 6.5

a11 a12 a13 a11 a12 a13


I M P O R TA N T P O I N T S
a21 a22 a23 = a21 a22 a23
a31 a32 a33 a31 + ka21 a32 + ka22 a33 + ka23 In general, if r rows (or r columns) become identical when
a is substituted for x, then (x – a)r – 1 is a factor of given
8. If each element of a row (column) of a determinant is determinant.
zero, then its value is zero.
9. If the elements of a determinant that involve x are 10. The sum of the products of the elements of any row
polynomials in x, and if the determinant is equal to (or column) of a determinant with the corresponding
zero when a is substituted for x, the x – a is a factor of co-factors is equal to the value of determinant, i.e., if
given determinant.
a11 a12 a13
Illustration D = a21 a22 a23 , then
1 1 1 a31 a32 a33
let D = a b c . If we put a = b in D, then we
a11C11 + a12C12 + a13C13 = D and so on.
a2 b2 c2
11. The sum of the products of elements of any row (or
have column) of a determinant with the co-factors of the
1 1 1 corresponding elements of any other row (or column)
is zero, i.e., if
b b c =0
a11 a12 a13
b2 b2 c2
D = a21 a22 a23 , then
( first and second column are identical) a31 a32 a33
This implies that (a – b) must be a factor of D. Similarly,
(b – c) and (c – a) are also factors of D. Since the prod- a11C31 + a12C32 + a13C33 = 0 and so on.
uct of the diagonal elements of D is 1.b.c,2 which is a
third degree expression, D is a polynomial of degree
3. But (a – b) (b – c) (c – a) is a factor of D which is I M P O R TA N T P O I N T S
of degree 3 itself. Therefore, the only other factor of D
If D = | aij | is a determinant of order n, then the value of
can be a constant, say k.
the determinant | Aij |, where Aij is the cofactor of aij is
Dn – 1.
1 1 1
D= a b c = k (a – b) (b – c) (c – a) − a2 ab ac o c b
2

For example, ab − b2 bc = c o a .
a2 b2 c2
ac bc −c2 b a o
In order to find the value of k, give values to a, b and c
such that calculations are easy and the two sides do not
vanish. For example, assume a = 0, b = 1, c = 2, we get
EvAlUATION OF DETERMINANTS USINg
1 1 1 ElEMENTARY OPERATIONS
0 1 2 = k (0 – 1) (1 – 2) (2 – 0)
To evaluate determinants of higher order, we should always
0 1 4 try to introduce zeros at the maximum number of places
in a particular row (column) by using the properties of the
or 2 = 2k (on solving the determinant along first
determinant. We denote the rows of the determinant by R1,
column)
R2, R3, ... and columns by C1, C2, C3, ...
Thus k = 1. Hence,
We shall use the following notations to evaluate a
1 1 1 determinant.
a b c = (a – b) (b – c) (c – a). 1. The operation of interchanging the ith row and jth row
a 2
b 2
c 2 will be denoted by Ri ↔ Rj.

Objective_Maths_JEE Main 2017_Ch 6.indd 5 01/01/2008 04:14:12


6.6  Chapter 6

2. The operation of multiplying each element of the ith


row by a number k will be denoted by Ri → kRi. 1 1 1
3. The operation of adding to each element of the ith = a p q r = 0.
row, k times the corresponding elements of the jth row 1 1 1
(j ≠ i) will be denoted by Ri → Ri + kRj.
Similar notations are used for operations on columns cos 2 x cos x ⋅ sin x − sin x
replacing R by C. 9. If f (x) = cos x sin x sin 2 x cos x , then for
sin x − cos x 0
Solved Examples
all x
xp + y x y (A) f (x) = 0 (B)  f (x) = 1
(C) f (x) = 2 (D)  None of these
7. The determinant yp + z y z = 0 if
0 xp + y yp + z Solution: (B)
We have,
(A) x, y, z are in A.P.
(B) x, y, z are in G.P. cos 2 x cos x ⋅ sin x − sin x
(C) x, y, z are in H.P. f  (x) = cos x sin x sin 2 x cos x
(D) xy, yz, zx are in A.P.
sin x − cos x 0
Solution: (B)
The given determinant 1 0 − sin x
= 0 1 cos x
xp + y x y
= sin x − cos x 0
yp + z y z
(Applying C1 → C1 – sin x . C3 and C2 → C2 + cos
− ( xp 2 + 2 yp + z ) 0 0
x . C3)
(Applying R3 → R3– pR1 – R2)
1 0 − sin x
2 2
= – (xp + 2yp + z) (xz – y ) = 0 1 cos x
= 0 if x, y, z are in G. P. 0 − cos x sin 2 x

8. If Tp, Tq, Tr are pth, qth, rth terms of an A.P, then  (Applying R3 → R3 – sin x . R1)
Tp Tq Tr = sin2x + cos2x = 1 for all x. (Expanding along C1)
p q r is equal to
10. The value of the determinant
1 1 1
a2 a 1
(A) p + q + r (B) 0
(C) – 1 (D) 1 cos nx cos ( n + 1) x cos ( n + 2) x is
sin nx sin ( n + 1) x sin ( n + 2) x
Solution: (B)
We have, Tp = a + ( p – 1) d, Tq = a + (q – 1) d, Tr = a + (A)  independent of n
(r – 1) d, where a is the first term and d is the common (B)  independent of a
difference. (C)  independent of x
\ The given determinant (D)  None of these
a + ( p − 1) d a + ( q − 1) d a + ( r − 1) d Solution: (A)
= p q r We have,
1 1 1 a2 a 1
a a a cos nx cos ( n + 1) x cos ( n + 2) x
= p q r [Applying R1 → R1 – (R2 – R3) d] sin nx sin ( n + 1) x sin ( n + 2) x
1 1 1

Objective_Maths_JEE Main 2017_Ch 6.indd 6 01/01/2008 04:14:14


Determinants  6.7

a 2 − 2a cos x + 1 a 1 0 x−a x−b


= 0 cos ( n + 1) x cos ( n + 2) x Let D = x + a 0 x−c .
0 sin ( n + 1) x sin ( n + 2) x x+b x+c 0
(Applying C1 → C1 + C3 – 2 cos x C2) On putting x = a, we get
0 0 a−b
= (a2 – 2a cos x + 1) sin x (Expanding along C1),
D = 2a 0 a − c = (a + c) (a + b) (a – c)
11. If f (x), g (x) and h (x) are three polynomials of degree a+b a+c 0
Clearly D ≠ 0 on expansion along second column, so
f ( x) g ( x) h ( x)
that x = a does not satisfy the equation D = 0. Similarly
2 and D (x) = f ′ ( x ) g ′ ( x ) h ′ ( x ) , then D (x) is a x = b and x = c also do not satisfy. Now, put x = 0, we
f ′′( x ) g ′′ ( x ) h ′′ ( x ) get
0 −a −b
polynomial of degree
(A) 2 (B) 3 D = a 0 −c = 0
(C)  at most 2 (D)  at most 3 b c 0
which is independent of n.
Hence, x = 0 satisfies the equation D = 0.
Solution: (C)
1 k k
Let f (x) = a0x2 + a1x + a2 13. If Un = 2 2
2n k + k +1 k +k
g (x) = b0x2 + b1x + b2 2n − 1 k 2 2
k + k +1
2
h (x) = c0x + c1x + c2 k
Then, and ∑Un = 72 then k =
n =1
f ( x) g ( x) h ( x)
(A) 8 (B) 9
(x) = 2a0 x + a1 , 2b0 x + b1
D 2c0 x + c1 (C)  6 (D)  None of these
2a0 2b0 2c0
Solution: (A)
f ( x) g ( x) h ( x) f ( x) g ( x) h ( x) k

= x 2a0 2b0 2c0 + a1 b1 c1 ∑1 k k


n =1
2a0 2b0 2c0 2a0 2b0 2c0 k
k

f ( x) g ( x) h ( x) ∑Un = 2 ∑n k2 + k + 1 k2 + k
n =1 n =1
= 0 + 2 a1 b1 c1  
k k
a0 b0 c0

2 ∑n− ∑1 k2 k2 + k + 1
n =1 n =1
= 2 [(b1c0 – b0c1) f (x) – (a1c0 – a0c1) g (x) + (a1b0
– a0b1) h (x)] k k k
Hence, degree of D (x) ≤ 2. = k ( k + 1) k 2 + k + 1 k2 + k
12. If a ≠ b ≠ c, one value of x which satisfies the equation k2 k2 k2 + k + 1
0 x−a x−b k 0 k
x+a 0 x − c = 0 is given by, 2 2
= k +k 1 k +k
x+b x+c 0 2 2
k 0 k + k +1
(A) x = a (B) 
x=b
(Applying C2 → C2 – C1)
(C) x = c (D) 
x=0
= k (k2 + k + 1) – k3 = k (k + 1) = 72 (given)
Solution: (D)
⇒ k = 8.

Objective_Maths_JEE Main 2017_Ch 6.indd 7 01/01/2008 04:14:16


6.8  Chapter 6

14. If Scos2a1 = Scos2b1 = Scos2g1 = 1; Scos a1 cos b1 = 16. If a, b and g are the roots of the equation x3 + px + q = 0,
Scos b1 cos g1 = Scos g1 cos a1 = 0, a b g
cos a1 cos a 2 cos a 3 then the value of the determinant b g a is
then the value of cos b1 cos b 2 cos b3 is equal to g a b
cos g 1 cos g 2 cos g 3 (A) q (B) 0
(A) 0 (B) ­– 1 (C) p (D) p2 – 2q
(C)  1 (D)  None of these Solution: (B)
Solution: (C) Since a, b, g are the roots of the equation x3 + px+ q
2 =0
cos a1 cos a 2 cos g 3 \ a + b + g  = 0
cos b1 cos b 2 cos b3
a b g a +b +g b g
cos g 1 cos g 2 cos g 3
So, b g a = a + b + g g a
cos a1 cos a 2 cos g 3 cos a1 cos a 2 cos g 3 g a b a +b +g a b
 = cos b1 cos b 2 cos b3 × cos b1 cos b 2 cos b3 (Applying C1 → C1 + C2 + C3)
cos g 1 cos g 2 cos g 3 cos g 1 cos g 2 cos g 3 0 b g
S cos 2 a1 S cos a1 cos b1 S cos a1 cos g 1 = 0 g a = 0.
= S cos b1 cos a1 0 a b
S cos 2 b1 S cos b1 cos g 1
S cos a1 cos g 1 S cos g 1 cos b1 S cos 2 g 1
1 3 cos q 1
1 0 0 17. If D = sin q 1 3 cos q , then maximum value
= 0 1 0 =1 1 sin q 1
0 0 1 of D is
(A) 10 (B) 14
15. If b2 – ac < 0 and a > 0 then the value of the determinant (C)  1 (D)  None of these
a b ax + by Solution: (A)
b c bx + cy is
1 3 cos q 1
ax + by bx + cy 0
D = sin q 1 3 cos q
(A) positive (B) negative
1 sin q 1
(C) 
zero (D)  b2 + ac
Solution: (B) 1 3 cos q 1
We have, = sin q 1 3 cos q
a b ax + by 0 sin q − 3 cos q 0
b c bx + cy (Applying R3 → R3 – R1)
ax + by bx + cy 0 = –(sin q – 3 cos q)(3 cos q – sin q)

a b 0 = (3 cos q – sin q)2
= b c 0 But – 9 + 1 ≤ 3 cos q – sin q ≤ 9 +1
ax + by bx + cy − ( ax 2 + 2bxy + cy 2 ) Therefore, (3 cos q – sin q)2 ≤ 10
(Applying C3 → C3 – xC1 – y C2) 18. If the determinant
= – (ax2 + 2bxy + cy2) (ac – b2) a b 2aa + 3b
1 b c 2ba + 3c = 0, then
=  (b2 – ac) [(ax + by)2 + y2 (ac – b2)] < 0 
a 2aa + 3b 2ba + 3c 0
(∵ b2 – ac < 0 and a > 0)

Objective_Maths_JEE Main 2017_Ch 6.indd 8 01/01/2008 04:14:18


Determinants  6.9

(A) a, b, c are in H.P. 8 9 10


C3 C5 C5
a is root of 4ax2 + 12bx + 9c = 0 or a, b, c are in
(B) 
9 10 11
G.P. = C4 C6 C8
(C) a, b, c are in G.P. 9
Cn 10
Cn + 2 11
Cn + 4
(D) a, b, c are in A.P.
Solution: (B) (Applying R 2 → R2 + R1 and using nCr + nCr +1 = n + 1Cr + 1 )
Operate R3 → R3 – 2a R1 – 3R2, we get For n = 4, R2 and R3 become identical and, therefore,
the value of the determinant becomes zero.
a b 2aa + 3b
p
b c 2ba + 3c =0 21. The value of q lying between q = 0 and q = and
satisfying the equation 2
2
0 0 − 4 aa − 6ba − 6ba − 9c
1 + sin 2 q cos 2 q 4 sin 4q
2 2
⇒ (4aa + 12ba + 9c) (ac – b ) = 0
sin 2 q 1 + cos 2 q 4 sin 4q = 0 is
⇒ a is a root of 4ax2 + 12bx + 9c = 0 or a, b, c are in
2 2
G. P. sin q cos q 1 + 4 sin 4q
1 1 1 1 1 1 5p 7p 11p
2 2 2 (A)  (B)  or
19. If D1 = x y z and D2 = yz xz xy , then 2 24 24
x y z x y z 3p p
(C)  (D) 
(A) D1 = D2 (B)  D1 = ­– D2 4 24
(C) D1 = – 2D2 (D) 
D2 = 2D1 Solution: (B)
Operate R1 → R1 – R3 and R2 → R2 – R3, we get
Solution: (A)
We have, 1 0 −1
1 1 1 x y z 0 1 −1 =0
1 2 2
D2 = yz xz xy = xyz xyz xyz sin q cos q 1 + 4 sin 4 q
xyz
x y z x 2
y 2
z 2
⇒ 1 + 4 sin 4 q + cos2q + sin2q = 0
x y z 1 1 1 ⇒ 2 + 4 sin 4 q = 0
xyz
= 1 1 1 =­ x y z 1
xyz ⇒ sin 4 q = −
x2 y2 z2 x2 y2 z2 2
7p 11p 7p 11p
1 1 1 ⇒ 4q = or ⇒q= or .
6 6 24 24
= x2 y2 z 2 = D1.
x y z a −1 n 6
n

20. The value of n for which the determinant


22. If Da = ( a − 1) 2
2n 2
4 n − 2 , then ∑ Da =
3 3 2 a =1
8
C3 9
C5 10
C7 ( a − 1) 3n 3n − n
(A) 0 (B)  n
8 C4 9
C6 10
C8 becomes zero is
(C) a (D)  None of these
9 10 11
Cn Cn + 2 Cn + 4
Solution: (A)
(A) n = 2 (B)  n=3 n
(C) n = 4 (D)  None of these ∑ (a − 1) n 6
a =1
Solution: (C)
n n
∑ Da ∑ (a − 1)
We have, 2
We have, = 2n 2 4n − 2
8 9 10 a =1 a =1
C3 C5 C7
8 9 10 n
C4 C6 C8
∑ (a − 1)
3
3n3 3n2 − 3n
9 10 11
Cn Cn + 2 Cn + 4 a =1

Objective_Maths_JEE Main 2017_Ch 6.indd 9 01/01/2008 04:14:22


6.10  Chapter 6

Solution: (A)
n( n − 1)
n 6
2 13 + 3 2 5 5
n( n − 1) ( 2n − 1
= 2n2 4n − 2 We have, 15 + 26 5 10
6
3 + 65 15 5
n2 ( n − 1) 2
3n3 3n2 − 3n
4 13 + 3 2 1
1 1 6 2
= ( 5) 15 + 26 5 2 is
n2 ( n + 1) 2n − 1
= 2n 4n − 2 3 + 65 3 5
2 3
n( n − 1)
3n2 3n2 − 3n (Taking 5 common from C2 and C3)
2
1 1 6 − 3 2 1
n3 ( n − 1)
= 2n − 1 6 n 12n − 6 =5 0 5 2
12
n − 1 6n 6n − 6
0 3 5
1 1 0
n3 ( n − 1) (Applying C1 → C1 – 3 C2 – 13 C3)
= 2n − 1 6 n 0 = 0
12 = – 5 3 (5 – 6 ) (Expanding along C1)
n − 1 6n 0
(Applying C3 → C3 – 6C1)
r −1 n 6
n
23. If the three linear equations x + 4ay + az = 0, x + 3by +
bz = 0 and x + 2cy + cz = 0 have a non-trivial solution,
25. If Dr = ( r − 1) 2
2n 2
4 n − 2 , then ∑ Dr is
3 3 2 r =1
then a, b, c are in ( r − 1) 3n 3n − 3n
equal to
2 1 1
(A)  = + (B)  b2 = ac (A) 1 (B) 2
b a c
(C) 3 (D) 0
(C) 2b = a + c (D)  None of these
Solution: (D)
Solution: (A)
We have,
For a non-trivial solution, we must have,
n
1 4a a 1 4a a ∑ (r − 1) n 6
1 3b b = 0 ⇒ 0 3b − 4 a b − a = 0 r =1
n n
1 2c c 0 2c − 4 a c − a
∑ Dr = ∑ (r − 1)2 2n 2 4n − 2
r =1 r =1
(Applying R2 → R2 – R1, R3 → R3 – R1)
n
⇒ (3b – 4a) (c – a) – (2c – 4a) (b – a) = 0 ∑ (r − 1)3 3n3 3n2 − 3n
2 1 1 r =1
⇒ bc + ab – 2ac = 0 ⇒ = +
b a c [∵ the terms in C1 are dependent on r whereas the
24. The value of the determinant terms in C2 and C3 are constant]
13 + 3 2 5 5 1
( n − 1) n n 6
15 + 26 5 10 is 2
1
3 + 65 15 5 = ( n − 1) n ( 2n − 1) 2n2 4n − 2
6
(A) – 5 3 (5 – 6 ) (B)  – 5 3 (5 + 6) 1
( n − 1) 2 n2 3n3 3n2 − 3n
4
(C) – 5 3 ( 6 – 5) (D)  None of these

Objective_Maths_JEE Main 2017_Ch 6.indd 10 01/01/2008 04:14:25


Determinants  6.11

(A) 0 (B) 1
6 1 6 (C)  – 1 (d) none of these
1 2
= n ( n − 1) 2 ( 2n − 1) 2n 2 ( 2n − 1)
12 Solution: (A)
3n ( n − 1) 3n2 3n ( n − 1)
log a p 1
1
[Taking n (n – 1) common from C1 and n common We have, log b q 1
12
from C2] = 0. [∵ C1 and C3 are identical]. log c r 1

log A + ( p − 1) log R p 1
2 r −1 x 2n − 1
n = log A + ( q − 1) log R q 1
26. If Dr = 2.3r −1 y 3n − 1 , then ∑ Dr is log A + ( r − 1) log R r 1
r −1 n r =1
4.5 z 5 −1 [Let A be the first term and R the common ratio of
(A)  independent of x (B)  independent of y G.P., then
(C)  independent of z (D)  independent of n a = Tp = AR p – 1,
Solution: (A, B, C, D) \ log a = log A + (p – 1) log R.
We have,
b = Tq = AR q – 1,
n
∑ 2 r −1 x 2n − 1 \ log b = log A + (q – 1) log R.
r =1
n n
c = Tr = AR r – 1,
∑ Dr = ∑ 2.3r −1 y 3n − 1
\ log c = log A + (r – 1) log R.]
r =1 r =1
n 1 p 1 p −1 p 1
∑ 4.5r −1 z 5n − 1 = log A 1 q 1 + log R q − 1 q 1
r =1
1 r 1 r −1 r 1
2n − 1 x 2n − 1
0 p 1
= 3n − 1 y 3n − 1
= 0 + log R 0 q 1
5n − 1 z 5n − 1 0 r 1

⎛ n 2n − 1 (Applying C1 → C1 – C2 + C3)
⎜ ∑ r
n −1
∵ D = 1 + 2 + 2 2
+ ... + 2 = , = 2n – 1,
⎝ r=1 2 − 1 = 0.
n
3n − 1 28. If A1B1C1, A2B2C2 and A3B3C3 are three-digit num-
similarly ∑ 2 ⋅ 3r − 1 =2
3 −1
= 3n – 1
bers, each of which is divisible by k, then
r =1
A1 B1 C1
n
5n − 1 ⎞
and ∑ 4 ⋅ 5r − 1 =4·
5 −1
= 5n − 1.⎟ D = A2
B2 C2 is
r =1 ⎠
A3 B3 C3
= 0 (∵ C1 and C3 are identical)
n (A)  divisible by k (B)  divisible by k2
∑ Dr

\ is independent of x, y, z and n. (C)  divisible by 2k (D)  None of these
r =1
Solution: (A)
Since A1B1C1 , A2B2C2 and A3B3C3 are divisible by k,
27. If a, b, c are the pth, qth and rth terms respectively of a
therefore
log a p 1
geometric progression, then log b q 1 is equal to 100A1 + 10B1 + C1 = n1k
log c r 1 100A2 + 10B2 + C2 = n2k
100A3 + 10B3 + C3 = n3k

Objective_Maths_JEE Main 2017_Ch 6.indd 11 01/01/2008 04:14:28


6.12  Chapter 6

where n1, n2, n3 are integers. (A) 0 (B) 1


A1 B1 C1 (c) ­– 1 (D)  2
Now D = A2 B2 C2 Solution: (A)
A3 B3 C3 We have,

A1 B1 100A1 + 10B1 + C1 − 1 + cos B cos C + cos B cos B
= A2 B2 100A2 + 10B2 + C2 cos C + cos A − 1 + cos A cos A
A3 B3 100A3 + 10B3 + C3 − 1 + cos B − 1 + cos A −1

(Applying C3 → C3 + 10C2 + 100C1) − 1 cos C cos B
A1 B1 n1k A1 B1 n1 = cos C − 1 cos A
= A2 B2 n2 k = k A2 B2 n2 = kD1 cos B cos A − 1
A3 B3 n3 k A3 B3 n3
(Applying C1 → C1 – C3 and C2 → C2 – C3)
⇒ D is divisible by k −a cos C cos B
(Since elements of D1 are integers, \ D1 is an integer) 1
= a cos C − 1 cos A
a
a 2 + 2a 2a + 1 1 acos B cos A − 1

29. The determinant 2a + 1 a + 2 1 is 0 cos C cos B
3 3 1 1
= 0 − 1 cos A
(A) > 0 if a > 1 (B)  = 0 if a = 1 a
0 cos A − 1
(C) < 0 if a < 1 (D)  All of these
(Applying C1 → C1 + b C2 + c C3)
Solution: (D)
= 0.
a 2 + 2a 2a + 1 1
Let D = 2a + 1 a + 2 1 31. If the three digit numbers A28, 3B9 and 62C, where A,
3 3 1 B and C are integers between 0 and 9, are divisible by
A 3 6
2
a + 2a − 3 2a − 2 0 a fixed integer k, then the determinant 8 9 C is
= 2a − 2 a −1 0 2 B 2
3 3 1 (A)  divisible by k (B)  divisible by k2

(Applying R1 → R1 – R3, and R2 → R2 – R3) (C)  divisible by 2k (D)  None of these

a 2 + 2a − 3 2a − 2 Solution: (A)
=
2a − 2 a −1 Since A28, 3B9 and 62C are divisible by k

 (Expanding along C3) \ A28 = n1k = 100A + 20 + 8 (1)
( a + 3) ( a − 1) 2 ( a − 1) 3B9 = n2k = 300 + 10B + 9 (2)
=
2 ( a − 1) a −1
62C = n3k = 600 + 20 + C(3)
a+3 2 where n1, n2 and n3 are integers.
= (a – 1)2
2 1
A 3 6
2.
= (a – 1)   (a + 3 – 4) = (a – 1)3.
Now, 8 9 C
Clearly, D > 0 if a > 1; D = 0 if a = 1 and D < 0 if a < 1. 2 B 2

30. If A, B, and C are the angles of a triangle, then the
A 3 6
value of the determinant
= 100 A + 20 + 8 300 + 10B + 9 600 + 20 + C
− 1 + cos B cos C + cos B cos B
2 B 2
cos C + cos A − 1 + cos A cos A is
− 1 + cos B − 1 + cos A −1 (Applying R2 → R2 + 100R1 + 10R3)

Objective_Maths_JEE Main 2017_Ch 6.indd 12 01/01/2008 04:14:31


Determinants  6.13

Solution: (B)
A 3 6 We have,
= n1k n2 k n3 k [Using (1), (2) and (3)]
sec x cos x sec 2 x + cot x cosec x
2 B 2
f (x) = cos 2 x cos 2 x cosec 2 x
A 3 6
1 cos 2 x cos 2 x
= k n1 n2 n3 , which is divisible by k.
2 B 2 cos x
0 0 sec 2 x + − cos x
sin 2 x
 = 0 cos 2 x − cos 4 x cosec 2 x − cos 4 x
x + a a2 a3
1 cos 2 x cos 2 x
32. If x + b b 2 b3 = 0 and a ≠ b ≠ c then x is equal to
x+c c2 c3
(Applying R1 → R1 – R3 sec x and R2 → R2 – R3 cos2 x)
(A) abc/(ab + bc + ca) Expanding along C1, we get
(B) – abc/(ab + bc + ca)
(C) (ab + bc + ca)/(abc) ⎡ 1 cos x ⎤
f (x) = – cos2 x sin2 x ⎢ 2 + 2
− cos x ⎥
(D)  – (ab + bc + ca)/(abc) ⎣ cos x sin x ⎦
Solution: (B) = – (sin2 x + cos3 x – cos3 x sin2 x)
x + a a2 a3 = – (sin2 x + cos3 x – cos3 x + cos5 x)
2 3
We have, x + b b b = 0
2 3 = – sin2 x – cos5 x.
x+c c c
p /2 p /2 p /2
x a2 a3 a a2 a3 \ ∫ f ( x ) dx = ­– ∫ sin 2 x dx − ∫ cos5 x dx
0 0 0
⇒ x b2 b3 + b b 2 b3 = 0
x c2 c3 c c2 c3 1 p 4⋅2
= − ⋅ −
2 2 5 ⋅ 3 ⋅1
1 a2 a3 1 a a2
−p 8
⇒ x 1 b 2 b3 + abc 1 b b 2 = 0 = − = – (15p + 32)/60.
4 15
1 c2 c3 1 c c2
34. The value of the determinant of nth order, being given
⇒ x (a – b) (b – c) (c – a) (ab + bc + ca) x 1 1 ...
 + abc (a – b) (b – c) (c – a) = 0 1 x 1 ...
by , is
1 1 x ...
⇒ x = – abc/(ab + bc + ca)(∵ a ≠ b ≠ c) ... ... ... ...
(A) (x – 1)n – 1 (x + n – 1)
sec x cos x sec 2 x + cot x cosec x (B) (x – 1)n (x + n ­– 1)
33. If f (x) = cos 2 x cos 2 x cosec 2 x , (C)  (1 – x)n – 1 (x + n – 1)
2 2 (D)  None of these
1 cos x cos x
Solution: (A)
p /2
then ∫ f ( x ) dx is equal to x 1 1 ...
0 1 x 1 ...
We have,
1 1 x ...
(A) (15p + 32)/60 (B)  – (15p + 32)/60
(C) (15p + 32)/4 (D)  None of these ... ... ... ...

Objective_Maths_JEE Main 2017_Ch 6.indd 13 01/01/2008 04:14:34


6.14  Chapter 6

x 1 1 ... 1 a2 a3
(1 − x ) ( x − 1) 0 ...
= 3. 1 b 2 b3 = (a – b) (b – c) (c – a) (ab + bc + ca)
(1 − x ) 0 ( x − 1) ...
1 c2 c3
... ... ... ...
[Applying R2 → R2 – R1, R3 → R3 – R1, … , a b c
 Rn → Rn – R1] 4. b c a = – (a3 + b3 + c3 – 3abc)
c a b
= x (x – 1)n – 1 +
( x − 1) n − 1 + ( x − 1) n − 1 + ... + ( x − 1) n − 1

1
= − [a + b + c)[( a − b) 2

( n − 1) times 2
 (Expanding along R1) + ( b − c ) 2 + ( c − a) 2 ]
n −1 n −1
= x ( x − 1) + ( x − 1)
[1 + 1 + ...+ (n – 1) times] Trick(s) for Problem Solving
= ( x − 1) n − 1 (x + n – 1).
If D’ is the determinant obtained by replacing all the elements
of determinant D of order n by their corresponding cofactors,
then D’ = Dn – 1.
PRODUCT OF DETERMINANTS OF SAME
In particular,
ORDER
a1 b1 c1 A1 B1 C1
a1 b1 c1 a1 b1 g 1
If D = a2 b2 c2 , D′ = A B C
Let D1 = a2 b2 c2 and D2 = a 2 b2 g 2 2 2 2
a3 b3 c3 A3 B3 C3
a3 b3 c3 a3 b3 g 3
where A1, B1, C1, ... are cofactors of a1, b1, c1, ... etc., then
Then row by row multiplication of D1 and D2 is given by, D1 = D2.
a1 a1 + b1 b1 + c1 g 1 a1 a 2 + b1 b 2 + c1 g 2
D1 × D2 = a2 a1 + b2 b1 + c2 g 1 a2 a 2 + b2 b 2 + c2 g 2
a3 a1 + b3 b1 + c3 g 1 a3 a 2 + b3 b 2 + c3 g 2 Solved Examples
a1 a 3 + b1b3 + c1g 3 x b b
 a2 a 3 + b2 b3 + c2g 3 x b
35. If D1 = a x b and D2 = are the given
a3 a 3 + b3b3 + c3g 3 a x
a a x
Multiplication can also be performed row by column; determinants, then
column by row or column by column as required in the d
problem. (A) D1 = 3 (D2)2 (B) D1 = 3D2
dx
d
Some Useful Determinants (C)  D1 = 3D22 D1 = 3 (D2)3/2
(D) 
dx
Any of the following determinants can be used directly in
solving problems: Solution: (B)
1 a a2 1 0 0 x b b x b b
d
1. 1 b b 2 = (a – b) (b – c) (c – a) D1 = a x b + 0 1 0 + a x b
dx
1 c c2 a a x a a x 0 0 1

1 a a3 x b x b x b
= + + = 3D2.
2. 1 b b3 = (a – b) (b – c) (c – a) (a + b + c) a x a x a x
1 c c3

Objective_Maths_JEE Main 2017_Ch 6.indd 14 01/01/2008 04:14:37


Determinants  6.15

x 2 x SOLUTION OF LINEAR EQUATIONS BY


36. If x 2 x 6 = Ax4 + Bx3 + Cx2 + Dx + E, then the DETERMINANTS
x x 6 1. Cramer’s Rule: Solution of system of
value of 5A + 4B + 3C + 2D + E is equal to linear equations in two unknowns
The solution of the system of equations
(A) –11 (B) 17
a1x + b1 y = c1   a2x + b2 y = c2
(C) –17 (D) 0
is given by
Solution: (A)
D1 D
x= and y = 2 ,
x 2 x D D
Let D(x) = x 2
x 6 where
x x 6 a1 b1 c b1
D= , D1 = 1
a2 b2 c2 b2

Then, 5A + 4B + 3C + 2D + E = D(1) + D′(1)
a1 c1
Now, D(1) = 0 as R2 and R3 are identical. and D2 = , provided D ≠ 0.
a2 c2
1 0 1 x 2 x x 2 x
2
2. Cramer’s Rule: Solution of system of
D′(x) = x x 6 + 2x 1 0 + x2 x 6 linear equations in three unknowns
x x 6 x x 6 1 1 0
The solution of the system of equations
1 2 1 1 2 1 a1x + b1 y + c1z = d1
D′(1) = 0 + 2 1 0 + 1 1 6 = –17 + 6 = –11 a2x + b2 y + c2z = d2
1 1 6 1 1 0 a3x + b3 y + c3z = d3
\ 5A + 4B + 3C + 2D + E = –11 is given by
D1 D D
x = , y = 2 and z = 3 , where
a1 b1 c1 D D D
37. If D = a2 b2 c2 and a1 b1 c1 d1 b1 c1
a3 b3 c3 D = a2 b2 c2 ; D1 = d2 b2 c2
a1 + pb1 , b1 + qc1 c1 + ra1 a3 b3 c3 d3 b3 c3
D′ = a2 + pb2 b2 + qc2
c2 + ra2 , then a1 d1 c1 a1 b1 d1
a3 + pb3 b3 + qc3 c3 + ra3 D2 = a2 d2 c2 ; D3 = a2 b2 d2 ;
a3 d3 c3 a3 b3 d3
(A)  D′ = D (1 + pqr)
provided D ≠ 0.
(B) D′ = D
(C) D′ = D (1 – pqr)
(D) D′ = D (1 + p + q + r) Conditions for Consistency
The following cases may arise:
Solution: (A)
1. If D ≠ 0, then the system is consistent and has a unique
a1 b1 c1 b1 c1 a1 solution, which is given by Cramer’s rule:
D′ = a2 b2 c2 + pqr b2 c2 a2 D1 D D
b3 c3 a3 x= ,y= 2 ,z= 3
a3 b3 c3 D D D

 (All other determinants will vanish) 2. If D = 0 and atleast one of the determinants D1, D2, D3
is non-zero, the given system is inconsistent, i.e., it has
= (1 + pqr)D no solution.

Objective_Maths_JEE Main 2017_Ch 6.indd 15 01/01/2008 04:14:40


6.16  Chapter 6

3. If D = 0 and D1 = D2 = D3 = 0, then the system is


1 1 −3
consistent and dependent, and has infinitely many
solutions. ⇒ 1+ l 2+l − 8 = 0
1 −1− l 2+l
Homogeneous and Non-homogeneous System
If d1 = d2 = d3 = 0, then the system is said to be, homogene- 1 0 0
ous, otherwise it is called non-homogeneous. ⇒ 1+ l 1 − 5 + 3l = 0
If the system of equations is homogeneous, then
1 −2−l 5+l
D1 = D2 = D3 = 0 (value of the determinant is zero, if one
column has all elements = 0). Thus, (Applying C2 → C2 – C1, C3 → C3 + 3C1)
1. if D ≠ 0, the system has only trivial solution (x = y = z ⇒ (5 + l) + (2 + l) ( – 5 + 3l) = 0
= 0), and
2. if D = 0, the system has a non-trivial solution. In fact it ⇒ 3l2 + 2l – 5 = 0
has infinitely many solutions. ⇒ (l – 1) (3l + 5) = 0
5
⇒ l = 1. −
Solved Examples 3
40. Let l and a be real. The set of all values of l for which
38. If x = cy + bz, y = az + cx, z = bx + ay where x, y, z are the system of linear equations
not all zero, then
l x + (sin a) y + (cos a) z = 0
(A) a2 + b2 + c2 + 2abc = 0
(B) a2 + b2 + c2 – 2abc = 1 x + (cos a) y + (sin a) z = 0
(C) a2 + b2 + c2 + 2abc = 1
– x + (sin a) y – (cos a) z = 0
(D)  None of these
has a non-trivial solution, is
Solution: (C)
(A) [0, 2 ] (B)  [– 2 , 0]
We have, x – cy – bz = 0
(C) [– 2, 2 ] (D)  None of these
cx – y + az = 0
Solution: (C)
bx + ay – z = 0 Since the system has a non-trivial solution,
Since x, y, z are not all zero, so the system will have a l sin a cos a
non-trivial solution. Therefore, therefore 1 cos a sin a = 0
1 −c −b − 1 sin a − cos a
c −1 a = 0
l (– cos2 a – sin2 a) – (– sin a cos a – sin a
⇒ 
b a −1
cos a) – (sin2 a – cos2a) = 0
⇒ 1 (1 – a2) + c (– c – ab) – b (ac + b) = 0
⇒ – l + sin 2a + cos 2a = 0 ⇒ l = sin 2a + cos
⇒ a2 + b2 + c2 + 2abc = 1
2a
39. The value of l for which the equations x + y – 3 = 0,
p
(1 + l) x + (2 + l) y – 8 = 0, x – (1 + l) y + (2 + l) = 0 ⇒ l= 2 cos ⎛⎜ 2 a − ⎞⎟
are consistent is ⎝ 4⎠
(A) 1 (B) 5/3
(C)  – 5/3 (D)  None of these p
Since – 1 ≤ cos ⎛⎜ 2 a − ⎞⎟ ≤ 1 ∀ a ∈ R
Solution: (A, C) ⎝ 4⎠
Here the equations are in two variables x and y. If they
are consistent then the values of x and y obtained from \ – 2 ≤l≤ 2 i.e. l ∈ [– 2 , 2 ].
first two equations should satisfy the third equation
and hence D = 0. i.e.,

Objective_Maths_JEE Main 2017_Ch 6.indd 16 01/01/2008 04:14:42


Determinants  6.17

EXERCISES

Single Option Correct Type

1. Let a1, a2 and b1, b2 be the roots of ax2 + bx + c = 0 a b g


and px2 + qx + r = 0 respectively. If the system of equa-
tions a1 y + a2z = 0 and b1 y + b2z = 0 has a non-trivial 6. Let Dk = 2.3 k
16.9 k
26.27k then the
solution, then (310 − 1) 2 (910 − 1) ( 2710 − 1)
2 2
b ac c ab 10
(A)  = (B)  =
q2 pr r2 pq value of ∑ Dk is
k =1

a2 bc (A)  2 (a + b + g) ab + ag + bg
(B) 
(C)  = (D)  None of these
p 2 qr (C) abg (D)  0

2. a, b, c are in G.P. with common ratio r1 and a, b, g are 7. If M is a 3 × 3 matrix, where M′M = I and det(M) = 1,
in G.P. with common ratio r2. If the equations ax + ay then det(M – I ) =
+ z = 0, bx + by + z = 0, cx + g y + z = 0 have only trivial (A) 1 (B) 0
solution, then (C)  –1 (D)  None of these
(A) a, a = 0 (B)  r1, r2 = 1 8. If [x] denotes the greatest integer less than or equal to
(C) r1, r2 ≠ 1 (D)  r1 = r2 x, then the value of the determinant
3. If the value of a third order determinant is 11, then the [e ] [p ] [p 2 − 6]
value of the determinant formed by its cofactors will [p ] [p 2 − 6] [e ] , then
be
(A) 11 (B) 121 [p 2 − 6] [e ] [p ]
(C) 1331 (D) 14641 (A) –8 (B) 8
1 1 1 (C)  0 (D)  None of these
4. If , and are respectively the pth, qth and rth
a b c 9. If ai, bi, ci ∈ R (i = 1, 2, 3) and x ∈ R and
terms of an A.P., then the value of the determinant a1 + b1 x a1 x + b1 c1
8
C3 9
C5 10
C7 a2 + b2 x a2 x + b2 c2 = 0, then
C4 8 9 10
is a3 + b3 x a3 x + b3 c3
C6 C8
9
Cn 10
Cn + 2 11
Cn + 4 a1 b1 c1
(A)  a2 b2 c2 = 4 (B)  x = ± 1
(A) abc (B)  pqr
a3 b3 c3
(C)  0 (D)  None of these
(C) x = 2 (D)  None of these
5. The value of the determinant 10. The value of the determinant
x+ y 2 2 z
sin q cos q sin 2q
yz + 2 x z 2z ;
⎛ 2p ⎞ ⎛ 2p ⎞ ⎛ 4p ⎞
y+ xz yz z sin ⎜ q + cos ⎜ q + sin ⎜ 2q + is
⎝ 3 ⎟⎠ ⎝ 3 ⎟⎠ ⎝ 3 ⎟⎠
where x, y, z are positive real numbers, is ⎛ 2p ⎞ ⎛ 2p ⎞ ⎛ 4p ⎞
sin ⎜ q − cos ⎜ q − sin ⎜ 2q −
(A) z ( 2 y − z y ) (B) y ( 2z − y z ) ⎝ 3 ⎟⎠ ⎝ 3 ⎟⎠ ⎝ 3 ⎟⎠

(C) x ( 2 y − z y ) (D)  None of these (A) 0 (B) sin q


(C) cos q (D) independent of q

Objective_Maths_JEE Main 2017_Ch 6.indd 17 01/01/2008 04:14:45


6.18  Chapter 6

1 n n ( A)  constant term of f (x) is 4


(B)  coefficieent of x in f (x) is 0
11. If Dk = 2k n2 + n + 2 n2 + n and (C)  constant term in f (x) is a – b
2k − 1 n 2 2
n + n+ 2 (D)  constant term in f (x) is a + b
n
∑ Dk = 48, then n equals ⎡ 3

1 ⎤

k =1 ⎡1 1⎤
2 2 ⎥
(A) 4 (B) 6 17. If P = ⎢ ,A= ⎢ ⎥ and Q = PAP′, then
⎢ 1 3 ⎥ ⎣0 1⎦
(C)  8 (D)  None of these ⎢− ⎥
⎣ 2 2 ⎦
12. If A, B, C are the angles of a triangle and 2005
p′Q P is
1 1 1 ⎡ 1 1⎤ ⎡1 2005⎤
(A)  ⎢ ⎥ (B) 
⎢0
1 + sin A 1 + sin B 1 + sin C = 0,
⎣ 2005 1⎦ ⎣ 1 ⎥⎦
sin A + sin 2 A sin B + sin 2 B sin C + sin 2 C ⎡1 0 ⎤ ⎡ 1 2005⎤
(C)  ⎢ ⎥ (D)  ⎢
then the triangle is a/an
⎣0 1 ⎦ ⎣ 2005 1 ⎥⎦
(A) equilaterral (B) isosceles
(C)  right-angled triangle (D)  any triangle xn xn +2 x n +3
13. If a0, a1 a2, a3, a4 are in A.P with the common 18. If y n yn +2 y n +3
a1a2 a1 a0
zn zn +2 z n +3
difference d, the value of a2 a3 a2 a1 is
a3 a4 a3 a2 ⎛ 1 1 1⎞
= (x – y) (y – z) (z – x), ⎜ + + ⎟ then n =
(A) 2d  (B)  4
2d  3 ⎝ x y z⎠
(C) 2d 2 (D) 
2d (A) –2 (B) –1
14. If a, b, g are different from and are the roots of ax3 + (C) 0 (D) 1
bx2 + cx+ d = 0 and 19. If a, b, g are the roots of the equation ax3 + bx2 + c
25 ab bg ga
(b – g ) (g – a) (a – b) = , then the determinant
2 = 0, then the value of the determinant bg ga ab
a b g ga ab bg
1− a 1− b 1− g (A) a (B)  b
D=
equals (C)  0 (D)  c
a b g
a2 b2 g2 20. If p + q + r = 0 = a + b + c, then the value of the deter-
pa qb rc
25d 25d
(A)  (B)  minant qc ra pb is
2a a
rb pc qa
−25d
(C)  (D)  None of these (A)  0 (B)  pq + qb + rc
a +b +c +d
(C)  1 (D)  None of these
15. Let {D1, D2, D3, ..., Dk} be the set of third order deter-
21. A determinant of second order is made with the ele-
minants that can be made with the distinct nonzero
ments 0 and 1. The number of determinants with
real numbers a1, a2, a3, ..., a9. Then
non-negative values is
k
∑ Di = 0
(A) k = 9! (B)  (A) 3 (B) 10
i =1 (C) 11 (D) 13
(C)  at least one Di = 0 (D)  None of these 2 df j d 2 f j
22. If  fj = ∑ aij x i , j = 1, 2, 3 and if fj′,  fj″ denote ,
(1 + x ) a (1 + 2 x )b 1 i =0 dx dx 2
f1 f2 f3
16. If f (x) = 1 (1 + x ) a
(1 + 2 x )b , a, b being
respectively, then g (x) = f1′ f 2′ f3′ is
(1 + 2 x )b 1 (1 + x ) a
f1′′ f 2′′ f3′′
positive integers, then

Objective_Maths_JEE Main 2017_Ch 6.indd 18 01/01/2008 04:14:48


Determinants  6.19

(A)  a cubic in x (B) a quadratic in x (A) m = 3, n ∈ R (B)  m = 3, n ≠ 10


(C)  linear in x (D)  a constant (C) m = 3, n = 10 (D)  None of these
23. The value of the determinant 1+ x 1 1
29. If x ≠ 0, y ≠ 0, z ≠ 0 and 1 + y 1 + 2 y 1 = 0,
2a1b1 a1b2 + a2 b1 a1b3 + a3b1
D = a1b2 + a2 b1 1 + z 1 + z 1 + 3z
2a2 b2 a2 b3 + a3b2 is
a1b3 + a3b1 a3b2 + a2 b3 2a3b3 then x–1 + y–1 + z–1 is equal to
(A)  –1 (B)  –2
(A) 1 (B) –1 (C)  –3 (D)  None of these
(C) 0 (D)  a1a2a3b1b2b3
x 1 + x2 x3
24. If A + B + C = p, eiq = cos q + i sin q and
30. If D (x) = log(1 + x 2 ) ex sin x then
e 2iA e − iC e − iB 2
cos x tan x sin x
z = e − iC
e 2iB e − iA then
(A) D (x) is divisible by x (B)  D (x) = 0
e − iB e − iA e 2iC (C) D′ (x) = 0 (D)  None of these

(A) Re (z) = 4 (B)  Im (z) = 0 31. The number of values of k for which the linear
(C) Re (z) = – 4 (D)  Im (z) = –1 equations
4x + ky + 2z = 0
25. If xi = ai bici, i = 1,2, 3, are theree-digit positive inte- kx + 4y + z = 0
gers such that each xi is a multiple of 19, then for some 2x + 2y + z = 0
a1 a2 a3 possess a non-zero solution is
integer n, D = b1 b2 b3 is given by (A) 0 (B) 3 (C) 2 (D) 1
c1 c2 c3 32. Let P and Q be 3 × 3 matrices P ≠ Q. If P3 = Q3 and
P2Q = Q2P, then determinant of (P2 + Q2) is equal to:
(A) 19n + 1 (B) 
19n + 2
(C) 19n (D)  19n + 3 (A) –2 (B) 1 (C) 0 (D) –1

26. If the system of equations ax + by + c = 0, bx + cy + a 13 + 3 2 5 5


= 0, cx + ay + b = 0 has a solution then the system of
33. The value of the determinant 15 + 26 5 10
equations
(b + c) x + (c + a) y + (a + b) z = 0 3 + 65 15 5
(c + a) x + (a + b) y + (b + c) z = 0 is equal to:
(a + b) x + (b + c) y + (c + a) z = 0 has (A) 5 3 ( 6 − 5) 5 3( 6 − 5 )
(B) 
(A)  only one solution
(B)  no solution (C) 5( 6 − 5) (D) 
3( 6 − 5 )
(C)  infinite number of solutions 34. Let a, b, c be any real numbers. Suppose that there are
(D)  None of these real numbers x, y, z not all zero such that x = cy + bz, y
27. (b + c) (y + z) – ax = b – c, = az + cx and z = bx + ay. Then a2 + b2 + c2 + 2abc is
(c + a) (z + x) – by = c – a, equal to
(a + b) (x + y) – cz = a – b, (A) 2 (B) –1 (C) 0 (D) 1
where a + b + c ≠ 0, then x =
a a + 1 a −1
c−b a−c
(A)  (B)  35. Let a, b, c be such that b(a + c) ≠ 0. If −b b + 1 b − 1
a+b+c a+b+c
c c −1 c +1
b−a 1
(C)  (D)  a +1 b +1 c −1
a+b+c a+b+c
+ a −1 b −1 c + 1 = 0, then the value
28. The equations x + y + z = 6, x + 2y + 3z = 10, x + 2y +
mz = n give infinite number of values of the triplet (x, ( −1) n + 2 a ( −1) n + 1 b ( −1) n c
y, z) if of ‘n’ is

Objective_Maths_JEE Main 2017_Ch 6.indd 19 01/01/2008 04:14:50


6.20  Chapter 6

(A)  zero (B)  any even integer 41. The value of the determinant of nth order, being given
(C)  any odd integer (D)  any integer x 1 1 ...
36. Let A be a 2 × 2 matrix 1 x 1 ...
by , is
Statement-1: adj (adj A) = A 1 1 x ...
Statement-2: |adj A| = | A | ... ... ... ...
(A)  Statement-1 is true, Statement-2 is true;
(A) (x – 1)n – 1 (x + n – 1)
Statement-2 is a correct explanation for
(B) (x – 1)n (x + n ­– 1)
Statement-1
(C)  (1 – x)n – 1 (x + n – 1)
(B)  Statement-1 is true, Statement-2 is true;
(D)  None of these
Statement-2 is not a correct explanation for
Statement-1 42. The value of the determinant
(C)  Statement-1 is true, Statement-2 is false
(D)  Statement-1 is false, Statement-2 is true x+ y 2 z z

37. If a, b, c, d > 0; x ∈ R and yz + 2 x z 2z ;


(a2 + b2 + c2)x2 – 2(ab + bc + cd) x + b2 + c2 + d2 ≤ 0, y+ xz yz z
33 14 log a where x, y, z are positive real numbers, is
then 65 27 log b = (A) z ( 2 y − z y ) (B) y ( 2z − y z )
97 40 log c
(C) x ( 2 y − z y ) (D)  None of these
(A) 1 (B) –1
(C)  0 (D)  None of these 2 df j d 2 f j
38. The value of the determinant 43. If  fj = ∑ aij x i , j = 1, 2, 3 and if fj′fj″ denote ,
i =0 dx dx 2
2
13 + 3 5 5 f1 f2 f3
15 + 26 5 10 is respectively, then g (x) = f1′ f 2′ f3′ is
3 + 65 15 5
f1′′ f 2′′ f3′′

(A) – 5 3 (5 – 6 ) (B)  – 5 3 (5 + 6) (A)  a cubic in x (B) a quadratic in x


(C)  linear in x (D)  a constant
(C) – 5 3 ( 6 – 5) (D)  None of these
44. If xi = ai bici, i = 1,2, 3, are theree-digit positive inte-
39. If A1B1C1, A2B2C2 and A3B3C3 are three three-digit
gers such that each xi is a multiple of 19, then for some
numbers, each of which is divisible by k, then
a1 a2 a3
A1 B1 C1
integer n, D = b1 b2 b3 is given by
D = A2
B2 C2 is
c1 c2 c3
A3 B3 C3
(A) 19n + 1 (B) 
19n + 2
(A)  divisible by k (B)  divisible by k2 (C) 19n (D)  19n + 3

(C)  divisible by 2k (D)  None of these
45. (b + c) (y + z) – ax = b – c,
40. If the three-digit numbers A28, 3B9 and 62C, where A, (c + a) (z + x) – by = c – a,
B and C are integers between 0 and 9, are divisible by (a + b) (x + y) – cz = a – b,
A 3 6 where a + b + c ≠ 0, then x =
a fixed integer k, then the determinant 8 9 C is c−b a−c
(A)  (B) 
2 B 2 a+b+c a+b+c

(A)  divisible by k (B)  divisible by k2 b−a 1


(C)  (D) 
(C)  divisible by 2k (D)  None of these a+b+c a+b+c

Objective_Maths_JEE Main 2017_Ch 6.indd 20 01/01/2008 04:14:53


Determinants  6.21

1+ x 1 1 (A) 4 (B) – 4
(C) 2 (D) – 2
46. If x ≠ 0, y ≠ 0, z ≠ 0 and 1 + y 1 + 2 y 1 = 0,
1 + z 1 + z 1 + 3z 52. The value of the determinant
then x–1 + y–1 + z–1 is equal to ( a − a1 ) − 2 ( a − a1 ) −1 a1−1
(A)  –1 (B)  –2
( a − a2 ) − 2 ( a − a2 ) −1 a2 −1 is
(C)  –3 (D)  None of these
( a − a3 ) − 2 ( a − a3 ) −1 a3−1
2 2 2
a ( s − a) ( s − a)
47. If 2s = a + b + c and ( s − b) 2
b 2
( s − b) 2 = a 2 Π ( ai − a j ) − a 2 Π ( ai − a j )
(A)  (B) 
( s − c)2 ( s − c)2 c2 p ai Π ( a − ai ) 2 Πai Π ( a − ai ) 2

k (s – a) (s – b) (s – c), then k is equal to Πai Π ( a − ai ) 2 Πai Π ( a − ai ) 2
(C)  −
(D) 
(A) 2 (B) 2s a 2 Π ( ai − a j ) a 2 Π ( ai − a j )
(C) 2s2 (D)  2s3
48. Let a, b be the roots of the equation ax2 + bx + c = 0. 1 1 1
Let sn = an + b n for n ≥ 1. Then, the value of the deter-
a+ x b+x c+x
3 1 + s1 1 + s2 1 1 1 P
53. If = , where Q is the
minant 1 + s1 1 + s2 1 + s3 is a+ y b+ y c+ y Q
1 + s2 1 + s3 1 + s4 1 1 1
( a + b + c) (b 2 − 4 ac) a+z b+z c+z
(A) 
a4 product of denominators, then P is equal to
2
( a + b + c) (b − 4 ac) 2 (A) (a – b) (b – c) (c – a)
(B)  (B) (x – y) ( y – z) (z – x)
a4 (C) (a – b) (b – c) (c – a) (x – y) (y – z) (z – x)
( a + b + c) 2 (b 2 − 4 ac) (D)  None of these
(C) 
a2 54. If a, b, c, d are the roots of the equation ax4 + bx3 + g x2
(D)  None of these + d x + x = 0, then the value of the determinant
a b−c c+b 1+ a 1 1 1
49. The value of the determinant a + c b c − a is 1 1+ b 1 1
2
(A) a + b + c 2 2 is
a−b a+b c 1 1 1+ c 1
(B) abc (a + b + c)
(C) (a2 + b2 + c2) (a + b + c) 1 1 1 1+ d
(D)  None of these
x −d
d − g (B) 
(A) 
cosec a 1 0 a a
1⎛ 3 1 ⎞
50. If 1 2 cosec a 1 = z + 3⎟ ,
2 ⎜⎝ z ⎠ a−b b −a
0 1 2 cosec a (C)  (D) 
a a
then z is equal to
(A) sin a/2 (B)  cos a/2 0 x y z
(C) tan a/2 (D)  None of these −x 0 c b
55. The value of the determinant is
a2 b2 c2 − y −c 0 a
51. If ( a + 1) 2 (b + 1) 2 (c + 1) 2 = k (a – b) (b– c) − z −b −a 0

( a − 1) 2 (b − 1) 2 (c − 1) 2 (A) (ax + by + cz)2 (B) (ax – by + cz)2



(c – a), then k is equal to (C) (ax + by – cz)2 (D)  None of these

Objective_Maths_JEE Main 2017_Ch 6.indd 21 01/01/2008 04:14:56


6.22  Chapter 6

56. The value of the determinant (A) (a2 + b2 + c2)3



(B) (ab + bc + ca)3
b2 + c2 ab ac
(C) (a2 + b2 + c2) (ab + bc + ca)2
2 2
ab c +a bc is (D)  None of these
ca cb a2 + b2 x + a2 ab ac
2 2 2 2 2 2 2
(A) a b c (B)  2a b c 62. If ab x+b bc = 0 and x ( ≠ 0) ∈ R then
(C) 4a2b2c2 (D)  None of these 2
ac bc x+c
x + c1 x+a x+a x is equal to
57. If f (x) = x + b x + c2 x + a and g (x) = (c1 – x) (A) a2 + b2 + c2 (B)  – (a2 + b2 + c2)
x+b x+b x + c3 (C)  2 (a2 + b2 + c2) (D)  None of these
(c2 – x) (c3 – x), then f (0) is equal to 63. The values of m for which the system of equations
bg ( a) − ag (b) bg ( a) + ag (b) 3x + my = m and 2x – 5y = 20 has a solution satisfying
(A)  (B)  the condition x > 0, y > 0, are
( b − a) ( b + a)
− 15 ⎞
bg ( a) − ag (b) bg ( a) + ag (b) (A) m ∈ ⎛⎜ − ∞, ∪ (0, ∞)
(C) 
( b + a)
(D) 
( b − a)
⎝ 2 ⎟⎠
− 15 ⎞
(B) m ∈ ⎛⎜ − ∞, ∪ (30, ∞)
2bc − a 2 c2 b2 ⎝ 2 ⎟⎠
58. If c2 2ca − b 2 a2 − 15 ⎞
(C) m ∈ ⎛⎜ − ∞, ∪ (0, 30)
b 2
a 2
2ab − c 2 ⎝ 2 ⎟⎠
(D)  None of these
= (a3 + b3 + c3 + kabc)2, then k is equal to
(A) 2 (B) – 2 64. If a = cos q + i sin q, b = cos 2q – i sin 2 q , c = cos 3
(C) 3 (D) – 3 a b c
59. The value of the determinant q + i sin 3q and if b c a = 0 then q is equal to
bg bg ′ + b ′g b ′g ′ c a b
is ga ga ′ + g ′a g ′a ′ (A) np (B)  2np
p
ab ab ′ + a ′b a ′b ′ (C) (2n + 1) (D)  None of these
2
(A) (ab ′ – a ′b) (bg  ′ – b ′g) (g a ′ – g  ′a)
65. The value of the determinant
(B) abg (a + b + g ) (a ′ + b ′ + g  ′ )
(C) a ′b ′g  ′ (a + b + g) (a ′ + b ′ + g) 1 1 1
(D)  None of these a a (a + d ) ( a + d ) ( a + 2d )
60. If a ≠ 0, a ≠ 1 and 1 1 1

a+d ( a + d ) ( a + 2d ) ( a + 2d ) ( a + 3d )
x +1 x x
1 1 1
x x+a x = a3 + f (x) . a (a2 + a + 1), then
a + 2d ( a + 2d ) ( a + 3d ) ( a + 3d ) ( a + 4 d )
x x x + a2
where a, d > 0, is
2
(A) f (x) = x (B)  f (x) = x 4d 4
(A)  −
(C) f (x) = x3 (D)  None of these a ( a + d ) 2 ( a + 2d )3 ( a + 3d ) 2 ( a + 4 d )
61. The value of the determinant 4d 4
2 2
(B) 
− bc b + bc c + bc a ( a + d ) 2 ( a + 2d )3 ( a + 3d ) 2 ( a + 4 d )
a 2 + ac − ac c 2 + ac is 4d 4
(C) 
a 2 + ab b 2 + ab − ab a ( a + d ) 2 ( a + 2d )3 ( a + 3d ) 2 ( a + 4 d ) 2
(D)  None of these

Objective_Maths_JEE Main 2017_Ch 6.indd 22 01/01/2008 04:14:59


Determinants  6.23

66. The value of the determinant = ax7 + bx6 + cx5 + dx4 + ex3 + fx2 + gx + h be an iden-
tity in x, where a, b, c, d, e, f, g, h are independent of x,
(b + c) 2 c2 b2
then the value of g is
c 2 ( c + a) 2 a2 is (A) – 213 (B) 213
b2 a2 ( a + b) 2 (C)  0 (D)  None of these

(A)  2 (ab + bc + ca)3 (B) (ab + bc + ca)3 xn yn zn



(C)  4 (ab + bc + ca)3 (D)  None of these 72. If xn + 2 yn + 2 zn + 2
67. If the equations xn + 3 yn + 3 zn + 3
(a + 1)3x + (a + 2)3y = (a + 3)3, (a + 1) x + (a + 2) y
= a + 3, x + y = 1 are consistent then a is equal to
⎛ 1 1 1⎞
(A) 1 (B) – 1 = (x – y) ( y – z) (z – x) ⎜ + + ⎟ then

(C) 2 (D) – 2 ⎝ x y z⎠

68. If the system of equations (A) n = 1 (B) 


n = ­– 1
x sin a + y sin b + z sin g  = 0, x cos a + y cos b + z cos g (C) n = 2 (D) 
n=–2
= 0, x + y + z = 0, where a, b, g are angles of a triangle,
have a non-trivial solution, then the triangle must be 73. The value of the determinant
(A) isosceles (B) equilateral sin a cos b cos a cos b − sin a sin b
(C)  right angled (D)  None of these sin a sin b cos a sin b sin a cos b is
69. If x1 ≠ 0, x2 ≠ 0, x3 ≠ 0, then the determinant cos a − sin a 0

x1 + a1b1 a1b2 a1b3 (A)  is independent of a


a2 b1 x2 + a2 b2 a2 b3 is equal to (B)  independent of b
(C)  independent of a and b
a3b1 a3b2 x3 + a3b3 (D)  None of these
74. If a, b, g are the roots of the equation x3 + px + q = 0,
⎛ a b a b a b ⎞
(A)  x1 x2 x3 ⎜1 + 1 1 + 2 2 + 3 3 ⎟ then the value of the determinant
⎝ x1 x2 x3 ⎠
1+ a 1 1
⎛ a b a b a b ⎞ 1 1+ b 1 is
(B) − x1 x2 x3 ⎜1 + 1 1 + 2 2 + 3 3 ⎟
⎝ x1 x 2 x3 ⎠ 1 1 1+ g
⎛ a b a b a b ⎞
(C)  x1 x2 x3 ⎜1 − 1 1 − 2 2 − 3 3 ⎟ (A) p2 – 2q
⎝ x1 x 2 x3 ⎠ (B) 3pq
(D)  None of these (C) p – q
(D)  None of these
a a+d a + 2d
75. The value of a determinant of third order whose all
70. If a2 ( a + d ) 2 ( a + 2d ) 2 = 0, then elements are 1 or – 1 is
2a + 3d 2 (a + d ) 2a + d (A)  an even number
(A) a + d = 0 (B)  an odd number
(B) d = 0 (C)  a prime number
(C) d = 0 or a + d = 0 (D)  cannot be determined
(D)  None of these 76. If square matrices A and B are such that AAq = AqA,
BBq = BqB and ABq = BqA, then (AB) (AB)q is equal to
x+3 x + 2 ( x + 2) 3
(A) BqAqAB
71. Let x+2 x + 3 ( x + 2) 3 (B) BAqAB
( x + 2) 3 x+2 x+3 (C) BAqABq
(D)  None of these

Objective_Maths_JEE Main 2017_Ch 6.indd 23 01/01/2008 04:15:00


6.24  Chapter 6

x 2 x (A) x3 ± x2 + g  = 0
(B) x3 ± 2x2 + g  = 0
77. Let D (x) = x 2 x 6 = Ax4 + Bx3 + Cx2 + Dx + E. (C) x3 ± x2 – g  = 0
x x 6 (D) x3 ± 2x2 – g  = 0
Then, the value of 5A + 4B + 3C + 2D + E is equal to 80. If a, b, c are the sides of a triangle ABC such that
(A) 9 (B) – 9 (C) 11 (D) – 11 a2 b2 c2
78. If D1 = ( a + 1) 2 (b + 1) 2 (c + 1) 2 = 0, then DABC is
y5 z6 ( z3 − y3 ) x 4 z6 ( x3 − z3 ) x 4 z5 ( y3 − x3 ) ( a − 1) 2 (b − 1) 2 (c − 1) 2
y 2 z3 ( y6 − z6 ) xz 3 ( z 6 − x 6 ) xy 2 ( x 6 − y 6 ) (A)  a right angled triangle
2 3 3 3 3 3 3 2 3 3
y z (z − y ) xz ( x − z ) xy ( y − x ) (B)  an isosceles triangle
(C)  an equilateral triangle
x y2 z3 (D)  None of these
and, D2 = x 4 y5 z 6 , then D1D2 = 81. The set of equations : lx – y + (cos q)z = 0; 3x + y + 2z
x 7
y 8
z 9 = 0; (cos q)x + y + 2z = 0, 0 ≤ q < 2p, has non-trivial
solutions.
2
(A) D 2 (B)  D32 (A)  for no values of l and q
4
(C) D 2 (D)  None of these (B)  for all values of l and q
⎡a b c ⎤ (C)  for all values of l and only two values of q
79. If abc = g, A = ⎢ c a b ⎥ and AA′ = I, then a, b, c are (D)  for only one value of l and all values of q
⎢ ⎥
⎢⎣ b c a ⎥⎦
the roots of the equation.

More than One Option Correct Type

82. The value of l for which the equations x + y – 3 = 0, x2 + x x +1 x−2


(1 + l) x + (2 + l) y – 8 = 0, x – (1 + l) y + (2 + l) = 0
are consistent is 85. If 2 x 2 + 3 x − 1 3x 3 x − 3 = Ax + B, then
2
(A) 1 (B) 5/3 x + 2x + 3 2x − 1 2x − 1
(C)  – 5/3 (D)  None of these
4 0 0 4 0 0
83. Let {D1, D2, D3, ..., Dk} be the set of third order deter- (A) A = 2 3 3 (B) B = 2 3 3
minants that can be made with the distinct non-zero
real numbers a1, a2, a3, ..., a9. Then,
4 0 2 4 0 −1

k 4 0 0 4 0 0
∑ Di = 0
(A) k = 9! (B)  (C) A = 2 3 − 3 (D)  B = 2 3 − 3
i =1
4 0 2 4 0 −1
(C)  at least one Di = 0 (D)  None of these
0 x−a x−b
84. If A + B + C = p, eiq = cos q + i sin q and 86. If x + a 0 x − c = 0, a ≠ b ≠ c, then
e 2iA e − iC e − iB x+b x+c 0
z = e − iC
e 2iB e − iA then (A) x = 0 if b (a + c) ≤ ac
(B) x = ± b ( a + c) − ac if b (a + c) > ac
e − iB e − iA e 2iC
(C) x = 0, ± b ( a + c) − ac if b (a + c) > ac
(A) Re (z) = 4 (B)  Im (z) = 0
(C) Re (z) = – 4 (D)  Im (z) = –1 (D)  None of these

Objective_Maths_JEE Main 2017_Ch 6.indd 24 01/01/2008 04:15:03


Determinants  6.25

92. If a, b, g are non-zero real numbers such that


bc − a 2 ca − b 2 ab − c 2 a2 b2 b2
87. If ca − b 2 ab − c 2 bc − a 2 = b2 a2 b2 , bg ga ab
ga ab bg = 0, then
ab − c 2 bc − a 2 ca − b 2 b2 b2 a2
ab bg ga
then
1 1 1
(A) a 2 = a2 + b2 + c2 (B)  b 2 = ab + bc + ca (A)  + + =0
2
(C)  a  = ab + bc + ca (D)  b 2 = a2 + b2 + c2 g aω bω 2
1 1 1
sin x sin y sin z (B)  + + =0
b aω gω 2
88. The determinant cos x cos y cos z ; 0 < x, y,
1 1 1
cos3 x cos3 y cos3 z (C)  + + =0
p b gω aω 2
z < , is equal to zero if
2 (D) (a b)3 + (b g )3 + (g a)3 = 3a 2b 2g 2
(A) x = y (B)  y=z
p 93. The positive integral solutions of the equation
(C) z = x (D) 
x+y+z=
2 x3 + 1 x2 y x2 z
89. The value of the determinant xy 2 y3 + 1 y 2 z = 30 are
cos (q + a ) − sin (q + a ) cos 2 a xz 2 yz 2 z3 + 1
sin q cos q sin a is (A)  (3, 1, 1) (B)  (1, 3, 1)
− cos q sin q l cos a (C)  (1, 1, 3) (D)  (– 1, 1, 3)
(A)  independent of q for all l ∈ R
(B)  independent of q and a when l = 1 ex sin x 1
(C)  independent of q and a when l = – 1 94. If f  (x) = cos x log(1 + x ) 1 = a + bx + cx2, then
2

(D)  None of these


x x2 1
p
90. The value of q lying between q = 0 and q = and
2 (A) a = 0 (B)  a=1
satisfying the equation
(C) b = – 1 (D)  b = – 2
1 + sin 2 q cos 2 q 4 sin 4q 95. If maximum and minimum values of the determinant
2 2
sin q 1 + cos q 4 sin 4q = 0 is
1 + sin 2 x cos 2 x sin 2 x
2 2
sin q cos q 1 + 4 sin 4q 2
sin x 1 + cos x2
sin 2 x are a and b, then
7p 5p sin 2 x cos 2 x 1 + sin 2 x
(A)  (B) 
24 24
11p (A) a + b 99 = 4
(C) 
p
(D) 
24 (B) a 3 – b 17 = 26
24
(C) (a 2n – b 2n) is always an even integer for n ∈ N
p /2
1 − cos 2nx (D) a triangle can be constructed having its sides as
91. If an = ∫ 1 − cos 2 x
dx , then
a – b, a + b and a + 3b
0
(A) an + 1 is A.M. between an and an + 2
(B) an + 1 is G.M between an and an + 2
(C) an + 1 is H.M. between an and an + 2
a1 a2 a3
(D)  a4 a5 a6 = 0
a7 a8 a9

Objective_Maths_JEE Main 2017_Ch 6.indd 25 01/01/2008 04:15:05


6.26  Chapter 6

Passage Based Questions

Passage I 98. Which of the following statements are true?


Let A = [aij] be an n × n matrix. The matrix A – lI is called If A is any n × n matrix and l is a characteristic root of
the characteristics matrix of A, where l is a scalar and I is A, then
the identity matrix. The determinant | A – lI | is a non-null (A) A and A′ have the same characteristic roots
polynomial of degree n in l and is called the characteristic (B) kl is a characteristic root of kA (k being scalar)
polynomial of A. The equation | A – lI | = 0 is called the (C) ln is a characteristic root of An (n being positive
characteristic equation of A and its roots are called the char- integer)
acteristic roots or latent roots or eigen values of A. The set 1
(D)  is a characteristic root of A–1
of all eigenvalues of the matrix A is called the spectrum of l
A. The product of the eigenvalues of a matrix A is equal to
the determinant A. 99. Which of the following statements are correct?
⎡1 0 2⎤ (A) If A, B are n rowed square matrices and A is
96. The characteristic roots of the matrix A = ⎢⎢0 1 2 ⎥⎥ non-singular, then A–1B and BA–1 has same char-
are acter-istic roots.
⎢⎣1 2 0 ⎥⎦
(B) If A and P are square matrices of same order and
(A) 1 (B) 2
P is non-singular, then A and P–1AP have same
(C) – 2 (D) 3
characteristic roots.
⎡1 − 3 3 ⎤ (C) If A and B be two square matrices of same order,
97. The given values of the matrix A = ⎢⎢ 3 − 5 3 ⎥⎥ are then AB and BA have same characteristic roots.
⎢⎣6 − 6 4 ⎥⎦ (D)  All of these

(A)  4, – 2, – 2, (B)  – 4, 2, – 2
(C)  – 4, 2, 2 (D)  4, – 4, 2

Match the Column Type

1+ x x x2
100. If x 1+ x x2 = px5 + qx4 + rx3 + sx2 + tx + w, then
x2 x 1+ x

Column-I Column-II
  I.  w is equal to (A) 3
 II. t is equal to (B) 1
III.  p + r is equal to (C)  – 1
 IV.  q + s is equal to (D) 0

Assertion-Reason Type

Instructions: In the following questions an Assertion (A) is (B) Assertion(A) is True, Reason(R) is True;
given followed by a Reason. (R). Mark your responses from Reason(R) is not a correct explanation for
the following options: Assertion(A)
(A)  Assertion(A) is True and Reason(R) is (C)  Assertion(A) is True, Reason(R) is False
True; Reason(R) is a correct explanation for (D)  Assertion(A) is False, Reason(R) is True
Assertion(A)

Objective_Maths_JEE Main 2017_Ch 6.indd 26 01/01/2008 04:15:06


Determinants  6.27

101. Assertion: If a, b, c are different, then the value of x Reason: The equations a1x + b1y = 0, a2x + b2 y = 0
0 x 2 − a x3 − b a b
have a non-trivial solution if 1 1 = 0.
satisfying x 2 + a 0 x 2 + c = 0 is 0 a2 b2
x4 + b x − c 0 104. Assertion: a, b, c are in G.P. with common ratio r1
Reason: Determinant of a skew-symmetric matrix of and a, b, g are in G.P. with common ratio r2. If the
odd order is zero. equations ax + a y + z = 0, bx + b y + z = 0, cx + g  y +
z = 0 have only trivial solution, then r1 ≠ r2, r1, r2 ≠ 1.
102. Assertion: Let l and a be real. The set of all values Reason: The equations a1x + b1y + c1z = 0, a2x + b2y
of l for which the system of linear equations + c2z = 0, a3x + b3y + c3z = 0 have only trivial solution
lx + (sina)y + (cosa)z = 0
a1 b1 c1
x + (cosa)y + (sina)z = 0
– x + (sina)y – (cosa)z = 0 if a2 b2 c2 ≠ 0
has a non-trivial solution, is a3 b3 c3
⎣⎡ − 2, 2 ⎤⎦ a 1 1
105. Assertion: If the value of the determinant 1 b 1
Reason: The equations a1x + b1y + c1z = 0, a2x
is positive, then abc > – 8 1 1 c
+ b2y + c2z = 0, a3x + n3y + c3z = 0 have a non-trivial
­solution if Reason: A. M. > G. M.
a1 b1 c1
x + c1 x+a x+a
a2 b2 c2 = 0
1 06. Assertion: If f (x) = x + b x + c2 x + a , then
a3 b3 c3
x+b x+b x + c3
103. Assertion: Let a1, a2 and b1, b2 be the roots of ax2 b g ( a) − a g ( b)
+ bx + c = 0 and px2 + qx + r = 0 respectively. If the f (0) = , where g(x) = (c1 – x) (c2 – x)
b−a
system of equations a1y + a2z = 0 and b1y + b2z = 0 (c3 – x)
b2 ac Reason: f (x) is linear is x.
has a non-trivial solution, then 2 =
q pr

Previous Year’s Questions

107. l, m, n are the pth, qth and rth term of an GP and all 109. If (ω ≠ 1) is a cubic root of unity, then
log l p 1 1 1+ i + ω2 ω2
positive, then log m q 1 equals [2002] 1− i −1 ω 2 − 1 equals [2002]
log n r 1 −i −1 + ω − i −1
(A) 3 (B) 2
(C) 1 (D) Zero (A) Zero
(B) 1
6i −3i 1 (C) i
108. If 4 3i −1 = x + iy , then [2002] (D)  w
20 3 i 110. If the system of linear equations [2003]
(A) x = 3, y = l x + 2ay + az = 0
(B) x = l, y = 3 x + 3by + bz = 0
(C) x = 0, y = 3 x + 4cy + cz = 0
(D) x = 0, y = 0 has a non-zero solution, then a, b, c

Objective_Maths_JEE Main 2017_Ch 6.indd 27 01/01/2008 04:15:08


6.28  Chapter 6

(A)  are in A. P. (A) Zero


(B)  are in G.P. (B)  any even integer
(C)  are in H.P. (C)  any odd integer
(D) satisfy a + 2b + 3c = 0 (D)  any integer
111. If 1, ω, ω2 are the cube roots of unity, then 117. Consider the following system of linear equations:
1 ω n
ω 2n  [2010]
n 2n
x1 + 2x2 + x3 = 3
ω ω 1 (where, n is not a multiple of 3) 2x1 + 3x2 + x3 = 3
ω 2n 1 ωn 3x1 + 5x2 + 2x3 = 1
The system has
is equal to [2003] (A)  exactly 3 solutions
(A) 0   (B)  w   (D) w2
1   (C)  (B)  a unique solution
(C)  no solution
112. If a1, a2, a3, ....,an,.... are in G.P., then the value of the
(D)  infinite number of solutions
determinant [2004]
118. The number of values of k for which the homoge-
log an log an +1 log an + 2
neous system of linear equations
log an + 3 log an + 4 log an + 5 , is
4x + ky + 2z = 0; kx + 4y + z = 0; 2x + 2y + z = 0
log an + 6 log an + 7 log an +8 possess a non-zero solution is [2011]

(A) 2 (B) 1
(A) 0   (B) –2   (C) 
2   (D) 1
(C) Zero (D) 3
1 1 1 119. Let P and Q be 3 by 3 matrices with P ≠ Q . If P3 =
113. If D = 1 1 + x 1 for x ≠ 0, y ≠ 0 then D is Q3 and P2Q = Q2P, then determinant of (P2 + Q2) is
1 1 1+ y equal to [2012]
 [2007]
(A) –2   (B) 1   (C) 0   (D) –1
(A)  divisible by neither x not y
(B)  divisible by both x and y ⎡1 a 3⎤
(C)  divisible by x but not y 120. If P = ⎢⎢1 3 3 ⎥⎥ is the adjoint of a 3 × 3 matrix A
(D)  divisible by y but not x ⎢⎣ 2 4 4 ⎥⎦
114. Let a, b, c be any real numbers. Suppose that there are and |A| = 4, then α is equal to [2013]
real numbers x, y, z not all zero such that x = cy + bz, (A) 11   (B) 5   (C) 0   (D)  4
y = az + cx and z = bx + ay. Then a2 + b2 + c2 + 2abc
121. If a , b ≠ 0 , and f ( n) = a n + b n and
is equal to [2008]
(A)  2   (B) − 1   (C)  0   (D) 1 31 + f (1)1 + f ( 2)
1 + f (1)1 + f ( 2)1 + f (3) = K(1 − a ) (1 − b ) (a − b ) ,
2 2 2
115. Let A be a square matrix all of whose entries are inte-
gers. Then which one of the following is true? 1 + f ( 2)1 + f (3)1 + f ( 4 )
 [2008]
–1
(A) If detA = ± 1, then A exists but all its entries then K isequal to [2014]
1
are not necessarily integers (A) a b    (B)     (C) 1   (D) −1
ab
(B) If detA ≠ ± 1, then A–1 exists and all its entries
are non-integers 122. The set of all values of l for which the system of
(C) If detA = ± 1, then A–1 exists and all its entries linear equations [2015]
are integers 2 x1 − 2 x2 + x3 = l x1

(D)  If detA = ± 1, then A–1 need not exist 2 x1 − 3 x2 + 2 x3 = l x 21

116. Let a, b, c be such that b(a + c) ≠ 0. If − x1 + 2 x2 = l x3

has a non-trivial solution,
a a + 1 a − 1 a +1 b +1 c −1
(A)  is a singleton.
−b b + 1 b − 1 + a − 1 b −1 c + 1 = 0,
(B)  contains two elements.
c c − 1 c + 1 ( −1) a ( −1) b (−
n + 2 n +1
−1) n c (C)  contains more than two elements.
then the value of ‘n’ is [2009] (D)  is an empty set.

Objective_Maths_JEE Main 2017_Ch 6.indd 28 01/01/2008 04:15:11


Determinants  6.29

123. The system of linear equations [2016] (A)  exactly three values of λ.
x+ly− z = 0 (B)  infinitely many values of λ.
(C)  exactly one value of λ.
lx − y − z = 0
(D)  Exactly two values of λ.
x + y − lz = 0

has a non-trivial solution for:

Answer keys

Single Option Correct Type


1. (A) 2.  (C) 3. (B) 4. (C)  5. (A)
6. (D)  7.  (B) 8. (A) 9. (B)  10.  (A, D)
11.  (A) 12. (B) 13.  (A)  14. (D) 15.  (A, B)
16. (B)  17.  (B) 18. (B) 19.  (C) 20. (A)
21.  (D) 22. (D)  23. (C) 24.  (B, C) 25. (C) 
26. (C) 27. (A) 28. (C) 29. (C)  30. (A)
31. (C) 32. (C) 33. (A)  34. (D)  35. (C)
36. (B) 37.  (C)  38. (A) 39.  (A)  40. (A)
41. (A)  42.  (A) 43.  (D)  44. (C) 45. (A)
46. (C) 47.  (D) 48.  (B) 49. (C)  50.  (C)
51. (B)  52.  (B)  53. (C)  54. (B) 55.  (B) 
56. (C) 57.  (A) 58. (D) 59. (A) 60. (A)
61. (B) 62. (B) 63. (B) 64. (B) 65. (B)
66. (A) 67. (D) 68. (A) 69. (A) 70. (C)
71. (A) 72. (B) 73. (B) 74. (C) 75. (A)
76. (A) 77. (D) 78. (B) 79. (A) 80. (B)
81. (A)

More than One Option Correct Type


82.  (A, C) 83.  (A, B) 84.  (B, C) 85.  (A, D) 86.  (A, C)
8 7.  (A, B) 88.  (A, B, C, D) 89.  (A, C) 90.  (A, C) 91.  (A, D)
92.  (A, B, C, D) 93.  (A, B, C) 94.  (A, C) 95.  (A, B, C)

Passage Based Questions


96.  (A, C, D) 97.  (A)  98.  (A, B, C, D) 99. (D)

Match the Column Type


100. I → (B), II → (A), III → (C), IV → (B)

Assertion-Reason Type
101.  (A) 102. 
(A) 103. (A)  104. (A)  105.  (A)
1 06.  (A)

Previous Year’s Questions


107. (D) 108. (D) 109. (A) 110. (C) 111. (A) 112. (A) 113. (B) 114. (D) 115. (C) 116. (C)
117. (C) 118. (A) 119. (C) 120. (A) 121. (C) 122. (B) 123. (A)

Objective_Maths_JEE Main 2017_Ch 6.indd 29 01/01/2008 04:15:11


6.30  Chapter 6

Hints and Solutions

Single Option Correct Type


1
1. Since a1, a2 and b1, b2 are the roots of ax2 + bx + c = 0 and
= A + (q – 1)D
px2 + qx + r = 0 respectively, therefore b
1
−b c = A + (r – 1)D
a1 + a2
= , a1a2 = (1) c
a a where A is the first term and D is the common difference.
−q r 1 1 1
and b1 + b2 =
, b1b2 = (2) bc ca ab
p p a b c
∴ p q r = abc

Since the given system of equations has a non-trivial solution p q r
1 1 1
a1 a 2 1 1 1
∴ = 0 i.e., a1b2 – a2b1 = 0
b1 b 2
A + ( p − 1) D A + ( q − 1) D A + ( r − 1) D
a1 a a + a2 a1a 2

or = 2 = 1 = = abc
p q r
b1 b2 b1 + b 2 b1b 2
1 1 1
2
pb pc ⇒ b = ac
⇒ =
qa ra q2 pr 0 0 0
The correct option is (A) = abc p q r

2. Since a, b, c are in G. P. with common ratio r1 and α, β, γ are 1 1 1
in G. P. with common ratio r2, therefore a ≠ 0, α ≠ 0, b = ar1,
[Applying R1 → R1 – (A – D)R3 – DR2]
c = ar12, β = ar2, γ = α r22
= 0
Also, the system of equations have only trivial solution, so
The correct option is (C)
a a 1 5. We have,

b b 1 ≠0
x+ y 2 z z
c g 1

yz + 2 x z 2z
a a 1 1 1 1 y + xz yz z
⇒ ar1 a r2 1 ≠ 0 ⇒ aα r1 r2 1 ≠0
ar12 a r22 1 r12 r22 1 x+ y 2 1
=z
yz + 2 x z 2
1 0 0
⇒ aα r1 r2 − r1 1 − r1 ≠0 y + xz y z

r12 r22 − r12 1− r12


(Taking z common from C2 and C3)
[Applying C2 → C2 – C1 C3 → C3 – C1]
− y 2 1
1 0 0 =z
0 z 2
⇒ aα (r2 – r1) (1 – r1) r1 1 1 ≠0
0 y z
r12 r2 + r1 1 + r1
⇒ aα (r2 – r1) (1 – r1) (1 – r2) ≠ 0 (Applying C1 → C1 –
y C2 – x C3)
⇒ r1 ≠ r2, r1 ≠ 1, r2 ≠ 1 =–
y ⋅ z (z – 2 y ) = z ( 2 y – z y ).
The correct option is (C)
3. We know that Dc = D3 – 1 = D2 = (11)2 = 121. The correct option is (A)
The correct option is (B) 6. We have,
a b g
1 1 1 10 10 10 10
4. Since , , are pth, qth and rth terms of an A.P.
a b c ∑ Dk = 2 ∑ 3k 16 ⋅ ∑ 9k 26 ⋅ ∑ 27k
k =1 k =1 k =1 k =1
1
⇒ = A + (p – 1)D 310 − 1 2 (910 − 1) ( 2710 − 1)
a

Objective_Maths_JEE Main 2017_Ch 6.indd 30 01/01/2008 04:15:16


Determinants  6.31

a b g n

⎛ 310 − 1⎞ ⎛ 910 − 1⎞ ⎛ 2710 − 1⎞


∑1 n n
k =1

= 2⎜ ⎟ 16 ⋅ ⎜ ⎟ 26 ⋅ ⎜ ⎟
⎝ 3 −1 ⎠ ⎝ 9 −1 ⎠ ⎝ 27 − 1 ⎠ n n

10 10 10
11. ∑ Dk = ∑ 2k n2 + n + 2 n2 + n
3 −1 2 (9 − 1) ( 27 − 1) k =1 k =1
n
a b g ∑ 2k − 1 n2 n2 + n + 2
k =1

= 310 − 1 2 (910 − 1) ( 2710 − 1) = 0
310 − 1 2 (910 − 1) ( 2710 − 1) n n n
(∵R2 and R3 are identical) ⇒ 48 = n + n n + n + 2
2 2
n +n2

The correct option is (D) n 2


n 2 2
n + n+ 2
7. As, M′M = I and | M | = 1
⇒ | M′M| = | I | or | M′M | = | M | (as | I | = 1 = | M |) n 0 n
⇒ | M′ | | M | – | M | = 0 ⇒ | M | (| M′ | – 1) = 0
= n2 + n 2 n2 + n C2 → C2 – C1
⇒ | M | = 0 or | M′ | = 1 n 2
0 n + n+ 2 2

⇒ | M′ | = 1 (∵ | M | = 1)
∴ | M – I | = | M – I | | M′| = | MM′ – M′|
= 2 (n3 + n2 + 2n – n3) = 2 (n2 + 2n)


= | I – M′ | = –| M′ – I | = –| M – I |′ ⇒ 24 = n2 + 2n ⇒ 25 = (n + 1)2
⇒ | M – I | + | M – I | = 0 ⇒ | M – I | = 0 ⇒ n + 1 = 5 (∵ n ∈ N)

The correct option is (B) ∴ n=4
The correct option is (A)
8. Since 2 < e < 3, 3 < π < 4 and 3 < p 2 – 6 < 4, the given deter-
minant reduces to 12. The given determinant

2 3 3 1 1 1
3 3 2 =–8 ∆
= sin A sin B sin C
3 2 3 sin 2 A sin 2 B sin 2 C

The correct option is (A) Operate R3 → R3 + R1–R2, R2 → R2 – R1

9. Clearly x = ± 1 satisfies the given equation.
The correct option is (B) 1 1 1
10. The given determinant = a
b c ∴ ∆ = 0 ⇒ (a – b) (b – c)(c – a) = 0
a2 b2 c2
sin q cos q sin 2q ⇒ a = b or b = c or c = a, i.e., the triangle is isosceles.
2p 2p 4p The correct option is (B)
= 2 sin q cos 2 cos q cos 2 sin 2q cos
3 3 3
a1a2 a1 a0
⎛ 2 p ⎞ ⎛ 2 p ⎞ ⎛ 4 p ⎞
sin ⎜ q − cos ⎜ q − sin ⎜ 2q − 13. The given determinant = a2 2d
3 ⎟⎠ 3 ⎟⎠ 3 ⎟⎠
d d
⎝ ⎝ ⎝
a3 2d d d
[Applying R2 → R2 + R3]

(Applying R2 → R2 – R1, R3 → R3 – R2)

sin q cos q sin 2q a1a2 a1 a0 a1a2 a1 a0


= − sin q − cos q − sin 2q = 0, 2a2 1 1 = d2 2a2 1 1 =d
2

⎛ 2p ⎞ ⎛ 2p ⎞ ⎛ 4p ⎞ 2a3 1 1 2d 0 0
sin ⎜ q − cos ⎜ q − sin ⎜ 2q −
⎝ 3 ⎟⎠ ⎝ 3 ⎟⎠ ⎝ 3 ⎟⎠ (Applying R3 → R3 – R2)

which is independent of θ.
= 2d3 (a1–a0) = 2d 4


The correct option is (A, D)
The correct option is (A)

Objective_Maths_JEE Main 2017_Ch 6.indd 31 01/01/2008 04:15:18


6.32  Chapter 6

14. Taking α, β, γ common from C1, C2,C3 respectively, we get Now differentiating both sides w.r.t. x and putting x = 0, we

get
1 1 1
1− a 1− b 1− g
a 2b 0 1 1 1 1 1 1

∆ = abγ 1 1 1
B = 1 1 1 + 0 a 2b + 1 1 1 = 0
a b g
1 1 1 1 1 1 2b 0 a

Hence, coefficient of x is 0.
1 1 1 1 1
− − The correct option is (B)
1− a 1− b 1− a 1− g 1− a
17. We have,

= abγ 1 0 0
⎡ 3 1 ⎤ ⎡ 3 1⎤
a b −a g −a ⎢ ⎥ ⎢ − ⎥
P′P = ⎢

2 2 ⎥ ⎢ 2 2 ⎥ = ⎡0 0 ⎤
⎢ 1 ⎢0 1 ⎥
3⎥ ⎢ 1 3⎥ ⎣ ⎦
(using C2 → C2 – C1 and C3 → C3 – C1)
⎢− ⎥ ⎢ ⎥
⎣ 2 2 ⎦ ⎣ 2 2 ⎦
abg ( −1)(b − a )(g − a ) 1 − g 1− b ⇒ P′P = I or P′ = P–1
=
(1 − a )(1 − b )(1 − g ) 1 1 As, Q = PAP′
abg (a − b )(b − g )(g − a ) ∴ P′Q2005P = P′[(PAP′) (PAP′) ... 2005 times]P

=
(1 − a )(1 − b )(1 − g )
= ( P ′ P ) A ( P ′ P ) A ( P ′ P ) ... ( P ′ P ) A ( P ′ P )

3 2 2005 times
As α, β, γ are the roots of ax + bx + cx+ d = 0,
2005 2005
ax3 + bx2+ cx2 + d = a(x – α) (x – β) (x – γ)
= IA =A
and abγ = – dla ⎡1 1⎤ 2 ⎡1 1⎤ ⎡1 1⎤ ⎡1 2 ⎤
Now, A = ⎢
⎥,A = ⎢0 1⎥ ⎢0 1⎥ = ⎢0 1 ⎥
( − d / a) ( 25 / 2) 25d ⎣0 1⎦ ⎣ ⎦⎣ ⎦ ⎣ ⎦
Thus, ∆ =
=–
(a + b + c + d ) / a 2( a + b + c + d )
⎡1 2 ⎤ ⎡1 1⎤ ⎡ 1 3⎤ ⎡1 2005⎤

The correct option is (D)
A3 = ⎢ ⎥ ⎢0 1⎥ = ⎢0 1⎥ ... A
2005
= ⎢
⎣ 0 1 ⎦⎣ ⎦ ⎣ ⎦ ⎣0 1 ⎥⎦
⎛ The number of ⎞
⎡1 2005⎤
⎛ The number ⎞ ⎜ arrangements of ⎟ ∴ P′Q2005P = ⎢
⎜ ⎟
⎣0 1 ⎥⎦
15. ⎜ of third-order ⎟ = ⎜ nine different ⎟ = 9!
⎜ ⎟ ⎜ ⎟
⎝ determinants ⎠ ⎜ numbers in ⎟ The correct option is (B)
⎜⎝ nine places ⎟⎠ 18. The degree of lhs (determinant) in x, y, z = 3n + 5
k The degree of the expression in the rhs = 2 ⇒ 3n + 5 = 2 ⇒

Now ∑ Di = D1 + D 2 + D3 + ... + D k n = –1
i =1 The correct option is (B)
Sai Sai Sai ab bg ga
k 1 bg ga
⇒ ∑ Di = Sai Sai Sai = 0 19. bg ga ab = (aβ + br + rα) 1 ga ab
i =1
Sai Sai Sai ga ab bg 1 ab bg

The correct option is (A, B)
(Applying C1 → C1 + C2 + C3)
(1 + x ) a
(1 + 2 x ) b
1 From the given equation, aβ + bγ + gα = 0.
a Thus, the value of the given eterminat is 0.
16. Let 1 (1 + x ) (1 + 2 x )b
The correct option is (C)
(1 + 2 x )b 1 (1 + x ) a 20. Since p + q + r = 0 = a + b + c (given)
= A + Bx + Cx2 +.... ⇒ p3 + q3 + r3 = 3pqr or a3 + b3 + c3 = 3 abc
Putting x = 0, we get pa qb rc
1 1 1 Let ∆ = qc ra
pb
A = 1 1 1 = 0 rb pc qa
1 1 1
⇒ ∆ = pqr (a3 + b3 + c3) – abc (p3 + q3 + r3)

Objective_Maths_JEE Main 2017_Ch 6.indd 32 01/01/2008 04:15:21


Determinants  6.33

∴ ∆ = pqr (3 abc) – abc(3pqr) = 0 since ei(A + B + C) = eiπ = cos π + i sin π) = –1



The correct option is (A)
0 −2eiB 0
21. There are only three determinants of second order with neg-
⇒ z = – −2e iA
0 0
0 1 0 1 1 1 iA iB
ative value, , , −e −e eiC
1 0 1 1 1 0
(Using R1 → R1 + R3, R2 →R2 + R3)


Number of possible determinants with elements 0 and 1are
24 = 16. ⇒ z = –2eiB [– 2 ei (A + C)]

Therefore, number of determinants with non-negative values ∴ z = 4 ei(A + B + C) = 4 eiπ = – 4
is 13.
The correct option is (B, C)

The correct option is (D)
25. ∆ =
22. g′ (x) = g1 (x) + g2 (x) + g3 (x) a1 a2 a3
b1 b2 b3
f1′ f 2′ f3′ (100 a1 + 10b1 + C1 ) (100 a2 + 10b2 + C2 ) (100 a3 + 10b3 + C3 )

where g1(x) = f1′ f 2′ f3′ = 0 (∵ R1 ≡ R2), (Using R3 → R3+ 100R1 + 10R2)

f1′′ f 2′′ f3′′ a1 a2 a3 a1 a2 a3
= b1 b2
b3 = b1 b2 b3
f1 f2 f3 x1 x2 x3 19m1 19m2 19m3
g2(x) = f1′′ f 2′′ f3′′ = 0 (∵ R2≡ R3)
 (where each mi ∈ N)
f1′′ f 2′′ f3′′ a1 a2 a3
= 19 b1 b2
b3 = 19 n
f1 f2 f3 m1 m2 m3
g3(x) = f1′ f 2′ f3′ =0
a1 a2 a3
f1′′′ f 2′′′ f3′′′ where n = b1 b2
b3 is certainly an integer.
(∵ fr′″ = 0 as each fr is a quadratic in x)
m1 m2 m3
Therefore, we have g′(x) = 0 ⇒ g(x) is a constant.
The correct option is (C)

The correct option is (D) 26. For existence of a solution for the first system of equations
2a1b1 a1b2 + a2b1 a1b3 + a3b1 a b c
23. Let ∆ = a1b2 + a2b1 2a2b2 a2b3 + a3b2 b c a =0
a1b3 + a3b1 a3b2 + a2b3 2a3b3 c a b

The second system will have a non-trivial solution if
a1 b1 0 b1 a1 0
b+c c+a a+b
∴ ∆ = a2 b2 0 b2 a2 0 =0

c+a a+b b+c =0
a3 b3 0 b3 a3 0
a+b b+c c+a

The correct option is (C)
b+c c+a a+b a b c
eiA e − i (C + A) e − i ( B + A) Now, c + a a + b b + c = 2 b c a = 0

− i (C + B )
eiB e −i ( A + B ) a+b b+c c+a c a b
24. z = eiA . eiB . eiC e
e −i( B + C ) e−i( A + C ) eiC Remember that the existence of one non-trivial solution
implies existence of infinite number of non-trivial solutions.
eiA − eiB − eic The correct option is (C)
27. Adding all three equations, we get
⇒ z = –1 − e iA
e iB
− eic
(a + b + c) (x + y + z) = 0
− eiA − eiB eic
⇒ x + y + z = 0 since, a + b + c ≠ 0

Objective_Maths_JEE Main 2017_Ch 6.indd 33 01/01/2008 04:15:24


6.34  Chapter 6

From the first equation ⇒ ∆ (x) is divisible by x


(b + c) (– x) – ax = b–c The correct option is (A)
c−b
∴ x= 31. For non-trivial solution of given system of linear equations
a+b+c
4 k 2
The correct option is (A)

k 4 1 =0
28. Each of the first three options contains m = 3. When m = 3,
the last two equations become x + 2y + 3z = 10 and x + 2y + 2 2 1
3z = n. ⇒ 8 + k(2 – k) + 2(2k –8) = 0
Obviously, when n = 10 these equations become the same. ⇒ –k2 + 6k – 8 = 0
So, we are left with only two independent equations to find ⇒ k2 – 6k + 8 = 0
the values of the three unknowns. Consequently, there will
⇒ k = 2, 4
be infinite solutions.
Clearly there exists two values of k.
The correct option is (C)
1 1 1 The correct option is (C)
1+
1+ x 1 1 x x x 32. Subtracting P3 – P2Q = Q3 – Q2P
1 1 1 P2(P – Q) + Q2(P – Q) = 0
29. 1 + y 1 + 2 y 1 = xyz 1 + 2+
y y y (P2 + Q2)(P – Q) = 0
1 + z 1 + z 1 + 3z
1 1 1 If P2 + Q2| ≠ 0 then P2 + Q2 is invertible
1+ 1+ 3+
z z z ⇒ P – Q = 0 contradiction
Hence, |P2 + Q2| = 0
1 1 1 The correct option is (C)
⎛ 1 1 1⎞ 1 1 1
 = xyz ⎜ 3 + + + ⎟ 1 + 2+ 13 + 3 2 5 5
⎝ x y z⎠ y y y
1 1 1 33. 15 + 26 5 10
1+ 1+ 3+
z z z 3 + 65 15 5
[Applying R1 → R1 + R2 + R3 and taking
⎛ 1 1 1⎞ 13 2 1 3 2 1
3 + + + ⎟ common]
⎝⎜ x y 2⎠
= 5 26 5 2 +5 15 5 2

65 3 5 3 3 5
1 0 0 1 2 1 1 1 1
⎛ 1 1 1⎞ 1
= xyz ⎜ 3 + + + ⎟ 1 + 1 −1
= 0+5 3 5 5 2 = 5 3 5 0 2
⎝ x y z⎠ y
3 3 5 3 0 5
1
1+ 0 2
z

= −5 3 (5 − 6 ) = 5 3 ( 6 − 5)
[Applying C2 → C2 – C1 and C3 → C3 – C1]


The correct option is (A)
⎛ 1 1 1⎞
= 2xyz ⎜ 3 + + + ⎟ giving x–1 + y–1 + z–1 = –3
⎝ x y z⎠ 34. The system of equations x – cy – bz = 0, cx – y + az = 0 and

The correct option is (C) 1 −c −b
bx + ay – z = 0 have non-trivial solution if c −1 a = 0
30. Let ∆ (x) = A + Bx + Cx2 + Dx3 +...
b a −1
0 1 0 ⇒ 1(1 – a2) + c(–c – ab) – b(ca + b) = 0
∆(0) = 0 1 0 = 0 ⇒ A = 0
⇒ a2 + b2 + c2 + 2abc = 1
1 0 0 The correct option is (D)
1 0 0 0 1 0 0 1 0
a a +1 a −1 a +1 b +1 c −1
∆′(0) = 0 1 0 + 0 1 1 + 0 1 0 = 1
n
35. − b b + 1 b − 1 + ( −1) a − 1 b − 1 c + 1
1 0 0 1 0 0 0 1 0
c c −1 c +1 a −b c
⇒ ∆ (x) = x + Cx2 + Dx3 +...

Objective_Maths_JEE Main 2017_Ch 6.indd 34 01/01/2008 04:15:27


Determinants  6.35

a a +1 a −1 a +1 a −1 a = – 5 3 (5 – 6) [Expanding along C1].


= − b b + 1 b − 1 + ( −1) n b + 1 b − 1 − b
The correct option is (A)
c c −1 c +1 c −1 c +1 c 39. Since A1B1C1, A2B2C2 and A3B3C3 are divisible by k,
therefore,
a a +1 a −1 a +1 a a −1 100A1 + 10B1 + C1 = n1k
= − b b + 1 b − 1 + ( −1) n + 1 b + 1 −b b − 1
100A2 + 10B2 + C2 = n2k
c c −1 c +1 c −1 c c +1 100A3 + 10B3 + C3 = n3k
where n1, n2, n3 are integers.
a a +1 a −1 a a +1 a −1 A1 B1 C1
= − b b + 1 b − 1 + ( −1) n + 2 − b b + 1 b − 1
Now, ∆ = A2 B2 C2
c c −1 c +1 c c −1 c +1 A3 B3 C3
This is equal to zero only if n + 2 is odd, i.e., n is odd integer. A1 B1 100A1 + 10B1 + C1
The correct option is (C)
= A2 B2 100A2 + 10B2 + C2
36. | adj A | = | A | n–1 = | A |2–1 = | A |
A3 B3 100A3 + 10B3 + C3
adj (adj A) = |A|n–2A = | A |0 A = A
[Applying C3 → C3 + 10C2 + 100C1]

The correct option is (B)
37. We have, A1 B1 n1k A1 B1 n1
(a2 + b2 + c2)x2 – 2(ab + bc + cd)x + b2 + c2 + d2 ≤ 0
= A2 B2 n2 k = k A2 B2 n2 = kD1
⇒ (ax – b)2 + (bx – c)2 + (cx – d)2 ≤ 0 A3 B3 n3k A3 B3 n3
⇒ (ax – b)2 + (bx – c)2 + (cx – d)2 = 0 ⇒ ∆ is divisible by k
b c d  [since elements of D1 are integers, ∴ D1 is an integer].
⇒ = = =x
a b c The correct option is (A)
2
⇒ b = ac or 2logb = loga + logc. 40. Since A28, 3B9 and 62C are divisible by k
33 14 log a 130 54 log a + log c ∴ A28 = n1k = 100A + 20 + 8 (1)
Now, 65 27 log b = 65 27
log b 3B9 = n2k = 300 + 10B + 9 (2)
97 40 log c 97 40 log c 62C = n3k = 600 + 20 + C(3)
[Apply R1 → R1 + R3]
where n1, n2 and n3 are integers.

0 0 0 A 3 6

= 65 27 log b = 0
Now, 8 9 C
97 40 log c 2 B 2

[Apply R1 → R1 – 2R2]
A 3 6

The correct option is (C) = 100 A + 20 + 8 300 + 10B + 9 600 + 20 + C
2 B 2
13 + 3 2 5 5
[Applying R2 → R2 + 100R1 + 10R3]

38. We have, 15 + 26 5 10
3 + 65 15 5 A 3 6
= n1k n2 k n3k  [Using (1), (2) and (3)
13 + 3 2 1 2 B 2
= ( 5 )2
15 + 26 5 2 is A 3 6
3 + 65 3 5 = k n1 n2 n3 , which is divisible by k.
2 B 2

[Taking 5 common from C2 and C3]

The correct option is (A)
− 3 2 1
x 1 1 ...
=5
0 5 2
1 x 1 ...
0 3 5 41. We have,
1 1 x ...
[Applying C1 → C1 – 3 C2 –
3 C3] ... ... ... ...

Objective_Maths_JEE Main 2017_Ch 6.indd 35 01/01/2008 04:15:30


6.36  Chapter 6

x 1 1 ... All these ⇒ g′ (x) = 0 ⇒ g(x) = a constant.


The correct option is (D)
(1 − x ) ( x − 1) 0 ...
= 44. ∆ =
(1 − x ) 0 ( x − 1) ...
... ... ... ... a1 a2 a3
b1 b2 b3
[Applying R2 → R2 – R1, R3 → R3 – R1
(100 a1 + 10b1 + C1 ) (100 a2 + 10b2 + C2 ) (100 a3 + 10b3 + C3 )
    Rn → Rn – R1]
( x − 1) n − 1 + ( x − 1) n − 1 + ... + ( x − 1) n − 1 (Using R3 → R3+ 100R1 + 10R2)

=x(x–1)n–1+
( n − 1) times a1 a2 a3 a1 a2 a3
 [Expanding along R1]
= b1 b2 b3 = b1 b2 b3
= x ( x − 1) n − 1 + ( x − 1) n − 1 [1 + 1 + ...+ (n – 1) times] x1 x2 x3 19m1 19m2 19m3
= ( x − 1) n − 1 (x + n – 1).

 (where each mi ∈ N)
The correct option is (A)
42. We have, a1 a2 a3

= 19 b1 b2 b3 = 19 n
x+ y 2 z z
m1 m2 m3
yz + 2 x z 2z
y + xz yz z a1 a2 a3
where n = b1 b2
b3 is certainly an integer.
x+ y 2 1 m1 m2 m3
=z
yz + 2 x z 2
The correct option is (C)
y + xz y z
45. Adding all three equations, we get
(a + b + c) (x + y + z) = 0

[Taking z common from C2 and C3]
⇒ x + y + z = 0
− y 2 1 since , a + b + c ≠ 0.
From the first equation

=z 0 z 2
(b + c) (– x) – ax = b – c
0 y z
c−b
∴ x =
[Applying C1 → C1 –
y C2 – x C3] a+b+c

The correct option is (A)

=– y ⋅ z (z – 2 y ) = z ( 2 y y – z y ).
1 1 1
1+

The correct option is (A) x x x
1+ x 1 1
1 1 1
43. g′ (x) = g1 (x) + g2 (x) + g3 (x) 46. 1 + y 1 + 2 y 1 = xyz 1 + 2+
y y y
1 + z 1 + z 1 + 3z
f1′ f 2′ f3′ 1 1 1
1+ 1+ 3+
z z z
where, g1(x) = f1′
f 2′ f3′ = 0 (∵R ≡ R2),
1 1 1
f1′′ f 2′′ f3′′ 1 1 1
⎛ 1 1 1⎞ 1 + 2+
= xyz ⎜ 3 + + + ⎟ y y y
f1 f2 f3 ⎝ x y z⎠
1 1 1
1+ 1+ 3+
g2(x) = f1′′ f 2′′ f3′′ = 0 (∵R2 ≡ R3) and z z z
f1′′ f 2′′ f3′′ [Applying R1 → R1 + R2 + R3]
1 0 0
f1 f2 f3 1
⎛ 1 1 1⎞ 1 + 1 −1
g3 (x) = f1′ f 2′ f3′ =0 = xyz ⎜ 3 + + + ⎟ y
⎝ x y z⎠
f1′′′ f 2′′′ f3′′′ 1
1+ 0 2
z
(∵ f r ′′′ = 0 as each fr is a quadratic in x)
[Applying C2 → C2 – C1, C3 → C3 – C1]

Objective_Maths_JEE Main 2017_Ch 6.indd 36 01/01/2008 04:15:34


Determinants  6.37

⎛ 1 1 1⎞ a2 a2
= 2xyz ⎜ 3 + + + ⎟ giving x–1 + y–1 + z–1 = –3 b +g − 0
⎝ x y z⎠ b b

The correct option is (C) = 2 (α + β + γ )2 b2
−g − a + g +a 0
47. Let s – a = α, s – b = β, s – c = γ, a
then β + γ = 2s – (b + c) = 2s – (2s – a) = a. a−b −a ab
Similarly, γ + α = b and α + β = c. a b
[Applying R1 → R1 –
R3 and R2 → R2 – R3]
Also, α + β + γ = 3s – (a + b + c) = 3s – 2s = s. b a

a2 ( s − a) 2 ( s − a) 2 a2 a2
b +g −
Therefore, ( s − b)
2
b 2
( s − b) 2 b b
= 2aβ (α + β + γ )2
( s − c) 2
( s − c) 2
c 2 b2
−g − a + g +a
a
(b + g )2 a2 a2
 [Expanding along C3]
= b2 (g + a ) 2 b2 2
a
b +g
g2 g2 (a + b ) 2 b
= 2aβ (α + β + γ )2
b2
2
(b + g ) − a 2
0 a 2 g +a
a
= b 2 − (g + a ) 2 (g + a ) 2 − b 2 b2 [Applying C1 → C1 + C2]
2 2
0 g − (a + b ) (a + b ) 2 = 2aβ (α + β + γ )2 [(β + γ ) (γ + α) – ab]
[Applying C1 → C1 – C2 and C2 → C2 – C3]
= 2aβ (α + β + γ )2 (g2 + aγ + bγ )
= 2abγ (α + β + γ )3 = 2s3 (s – a) (s – b) (s – c).
(b + g + a ) (b + g − a ) 0 ∴ k = 2s3.
= (b + g + a ) (b − g − a ) (g + a + b ) (g + a − b )  The correct option is (D)
0 (g + a + b ) (g − a − b )
3 1 + s1 1 + s2
2
a 48. Let ∆ = 1 + s1 1 + s2 1 + s3
 b2 1 + s2 1 + s3 1 + s4
(a + b ) 2
3 1+ a + b 1 + a2 + b2
= 1+ a + b 2
1+ a + b 2
1 + a3 + b3
2
b +g −a 0 a
1 + a 2 + b2 1 + a3 + b3 1 + a 4 + b4
= (α + β + γ )2 b − g − a g + a − b b 2
[Sn = a n + b n for n ≥ 1]

0 g − a − b (a + b ) 2
1 1 1 1 1 1
[Taking α + β + γ common from C1 and C2]

= 1 a b × 1 a b = D12 ,
1 a2 b2 1 a2 b2
b +g −a 0 a2
= (α + β + γ )2 b − g − a g + a − b b2 1 1 1
2a − 2b − 2a 2ab where, D1 = 1
a b .

[Applying R3 → R3 – (R1 + R2)


1 a2 b2

1 0 0
b +g −a 0 a2

Now, D1 = 1 a −1 b −1
= 2 (α + β + γ )2 b − g − a g + a − b b 2
1 a2 − 1 b2 − 1
a −b −a ab
[Applying C2 → C2 – C1 and C3 → C3 – C1]


 [Taking 2 common from R3]

Objective_Maths_JEE Main 2017_Ch 6.indd 37 01/01/2008 04:15:37


6.38  Chapter 6

a −1 b −1 1 b−c c+b

= [Expanding along R1] a2 + b2 + c2
2
a −1 b −1 2 =
0 c c−a
a
0 a+c c
1 1 [Applying R2 → R2 – R1, R3 → R3 – R1]


= (α – 1) (β – 1)
a +1 b +1
2 2 2

= (aβ – (α + β) + 1) ⋅ (β – α) = a + b + c (– bc + a2 + ab + ac + bc)

a

= (aβ – (α + β) + 1) ⋅ (a + b ) 2 − 4ab
 [Expanding along C1]
= (a2 + b2 + c2) (a + b + c).

b2
The correct option is (C)

= ⎛ c + b + 1⎞ −
4c
⎜⎝ a a ⎟⎠
a2 a
cosec a 1 0
50. We have, 1 2cosec a 1
⎡∵ a , b are the roots of the equation ax 2 + bx + c = 0, ⎤
⎢ ⎥ 0 1 2cosec a
⎢\ a + b = − b and ab = c ⎥
⎢⎣ a a ⎥⎦
 cosec a 1 0
( a + b + c) b 2 − 4 ac = 1 2cosec a − sin a 1

=
a2 0 1 2cosec a
2 2 [Applying R2 → R2 – sin α R1]

∴ ∆ = D12 = ( a + b + c) × (b − 4 ac)
4
a
2cosec a − sin a 1

The correct option is (B) = cosec α
1 2cosec a
a b−c c+b
 [Expanding along R1]
49. We have, a + c b c−a
= cosec α (4 cosec2 α – 2 – 1)
a−b a+b c

1 ⎡⎛ 2 ⎞ 2 ⎤
a2 b ( b − c ) c ( c + b) = ⎢⎜ ⎟ − 3⎥
1 sin a ⎢⎝ sin a ⎠ ⎥⎦
=
a ( a + c) b2 c ( c − a) ⎣
abc
a ( a − b) b ( a + b) c2
1 ⎡ tan 2 a / 2 + 1 ⎤ ⎡⎛ tan 2 a / 2 + 1⎞ ⎤
2
= ⎢ ⎢
⎥ ⎜ ⎟ − 3⎥

[Multiplying C1, C2 and C3 by a, b and c, respectively] 2 ⎢⎣ tan a / 2 ⎥⎦ ⎢⎝ tan a / 2 ⎠ ⎥
⎣ ⎦

a2 + b2 + c2 b ( b − c ) c ( c + b) 1 ⎛ a a⎞ ⎡⎛ a a⎞
2 ⎤
1 = tan + cot ⎟ ⎢⎜ tan + cot ⎟ − 3⎥
= a2 + b2 + c2 2 ⎜⎝
2
b c ( c − a) 2 2⎠ ⎢⎣ ⎝ 2 2 ⎠ ⎥⎦
abc
a2 + b2 + c2 b ( a + b) c2
1 ⎡⎛ a a⎞
3
a a ⎛ a a⎞⎤
[Applying C1 → C1 + C2 + C3]
 = ⎢⎜ tan + cot ⎟ − 3 tan cot ⎜⎝ tan 2 + cot 2 ⎟⎠ ⎥
2 ⎢⎣⎝ 2 2⎠ 2 2 ⎥⎦
1 b ( b − c ) c ( c + b) 1 ⎛ 3a a⎞
a2 + b2 + c2 = ⎜ tan + cot 3 ⎟
=
1 b2 c ( c − a) 2 ⎝ 2 2⎠
abc
1 b ( a + b) c2 a
∴ z = tan .
2
[Taking a2 + b2 + c2 common from C1]


The correct option is (C)
1 b−c c+b
a2 + b2 + c2 a2 b2 c2
=
⋅ bc 1 b c−a
abc 51. We have, ( a + 1) 2
1 a+b c (b + 1) 2 (c + 1) 2
2 2
( a − 1) (b − 1) (c − 1) 2

[Taking b and c common from C2 and C3, respectively,]

Objective_Maths_JEE Main 2017_Ch 6.indd 38 01/01/2008 04:15:41


Determinants  6.39

a2 b2 c2 a 2 − a1a2
1

=4 a b c ( a1 − a2 ) ( a1 − a3 ) a1a2
=
( a − 1) 2
(b − 1) 2
(c − 1) 2 Π ( a − ai ) 2 a 2 − a1a3
1
a1a3
[Applying R2 → R2 – R3 and then taking


 4 common from R2] ( a1 − a2 ) ( a1 − a3 ) a 2 ( a2 − a3 )
=
a2 b2 c2 a1a2 a3 Π ( a − ai ) 2

=4 a b c
− a 2 Π ( ai − a j )
1 1 1 = .
Πai Π ( a − ai ) 2
[Applying R3 → R3 – (R1 – 2R2)]


The correct option is (B)
a2 b2 − a2 c2 − a2

=4 a b−a c−a 1 1 1
1 0 0 a+x b+x c+x
1 1 1
[Applying C2 → C2 – C1 and C3 → C3 – C1]
53. We have,
a+ y b+ y c+ y
a2 b+a c+a 1 1 1

= 4 (b – a) (c – a) a 1 1 a+z b+z c+z
1 0 0 1 a−b a−c

= 4 (b – a) (c – a) (b – c) = – 4 (a – b) (b – c) (c – a). a+x ( a + x ) (b + x ) ( a + x ) (c + x )
∴ k = – 4.
1 a−b a−c
=

The correct option is (B) a+ y ( a + y ) (b + y ) ( a + y ) (c + y )
1 a−b a−c
( a − a1 ) − 2 ( a − a1 ) −1 a1−1 a+z ( a + z ) (b + z ) ( a + z ) (c + z )
52. We have, ( a − a2 ) − 2 ( a − a2 ) −1 a2 −1 [Applying C2 → C2– C1 and C3 → C3 – C1]

( a − a3 ) − 2 ( a − a3 ) −1 a3−1 1 1 1
a+ x ( a + x ) (b + x ) ( a + x ) (c + x )

= (a – a1) – 2 (a – a2) – 2 (a – a3) – 2 1 1 1
= (a – b) (a – c)
a+ y ( a + y ) (b + y ) ( a + y ) (c + y )
1 ( a − a1 ) a1−1( a − a1 ) 2
1 1 1
1 ( a − a2 ) a2 −1( a − a2 ) 2 a+z (aa + z ) (b + z ) ( a + z ) (c + z )
1 ( a − a3 ) a3−1 ( a − a3 ) 2
(b + x ) (c + x ) (c + x ) b + x
( a − b) ( a − c )
= (b + y ) (c + y ) (c + y ) b + y
1 ( a − a1 ) a1−1 ( a − a1 ) 2 Q
(b + z ) (c + z ) (c + z ) b + z
( a 2 − a1a2 )( a1 − a2 )
1 0 ( a1 − a2 )
= a1a2 (b + x ) (c + x ) c+x b+x
Π ( a − ai ) 2 ( a − b) ( a − c )
( a 2 − a1a3 )( a1 − a3 ) = ( y − x ) ( y + x + b + c) y − x y − x
0 ( a1 − a3 ) Q
a1a3 ( z − x ) ( z + x + b + c) z − x z − x
[Applying R2 → R2 – R1, R3 → R3 – R1]
[Applying R2 → R2 – R1 and R3 → R3 – R1]

( a − b) ( a − c ) ( y − x ) ( z − x )
=
( a 2 − a1a2 ) ( a1 − a2 ) Q
( a1 − a2 )
1 a1a2
= (b + x ) (c + x ) c + x b + x
Π ( a − ai ) 2 ( a 2 − a1a3 ) ( a1 − a3 )
( a1 − a3 ) y+x+b+c 1 1
a1a3

z+ x+b+c 1 1
 [Expanding along C1]

Objective_Maths_JEE Main 2017_Ch 6.indd 39 01/01/2008 04:15:44


6.40  Chapter 6

( a − b) ( a − c ) ( y − x ) ( z − x ) ⎛ 1 1 1 1⎞
=
= abcd ⎜1 + + + + ⎟
Q ⎝ a b c d⎠
(b + x ) (c + x ) c + x b + x
 [Expanding along R1]

  y+x+b+c 1 1
= abcd + (bcd + acd + abd + abc)
z−y 0 0 ⎡∵ a, b, c, d are roots of the equation ⎤
[Applying R3 → R3 – R2]
⎢ 4 3 2 ⎥
⎢a x + b x + g x + d x + x = 0 ⎥
( a − b) ( a − c ) ( y − x ) ( z − x ) ( z − y ) c+x b+x x d ⎢ d ⎥
= = − ⎢\ bcd + acd + abd + abc = − ⎥
Q 1 1 a a ⎢ a ⎥
⎢ x ⎥

 [Expanding along R3] ⎢and abcd = ⎥
( a − b) ( b − c ) ( c − a) ( x − y ) ( y − z ) ( z − x ) ⎣ a ⎦
= x −d
Q = .
∴ P = ( a − b) ( b − c ) ( c − a) ( x − y ) ( y − z ) ( z − x ) . a
The correct option is (B)

The correct option is (C)
0 x y z
1+ a 1 1 1
−x 0 c b
1 1+ b 1 1 55. We have,
54. We have, − y −c 0 a
1 1 1+ c 1
− z −b −a 0
1 1 1 1+ d
0 ax − by + cz y z
1 1 1 1
1+ 1 −x 0 c b
a a a a =
a −y 0 0 a
1 1 1 1
1+ −z 0 −a 0
b b b b
= abcd

1 1 1 1 [Applying C2 → aC2 – bC3 + cC4]

1+
c c c c
1 1 1 1 −x c b
1+ ( ax − by + cz )
d d d d = − −y 0 a
a

[Dividing R1, R2, R3 and R4 by a, b, c and d respectively] − z −a 0

⎛ 1 1 1 1⎞
 [Expanding along C2]
= abcd ⎜1 + + + + ⎟

⎝ a b c d⎠ ax − by + cz 0 0
1 1 1 1 ( ax − by + cz )
= y 0 a
1 1 1 1 a2
1+ z −a 0
b b b b

 × 1 1 1 1
 [Taking (– 1) common from C1 and applying
1+ R1 → aR1 – bR2 + cR3]
c c c c
1 1 1 1 = (ax – by + cz)2.
[Expanding along R1].
1+
The correct option is (B)
d d d d
[Applying R1 → R1 + R2 + R3 + R4 and taking
56. We have,
⎛ 1 1 1 1⎞
 1 + + + + ⎟ common from R1] b2 + c2 ab ac
⎝⎜ a b c d⎠
2 2
ab c +a bc
1 0 0 0
ca cb a + b2
2
1
1 0 0
b
⎛ 1 1 1 1⎞ a (b 2 + c 2 ) ab 2 ac 2

= abcd ⎜1 + + + + ⎟ × 1
⎝ a b c d⎠ 0 1 0 1
c =
a 2b b (c 2 + a 2 ) bc 2
abc
1
0 0 1 a 2c cb 2 c (a2 + b2 )
d
[Applying C2 → C2 – C1, C3 → C3 – C1 and C4 → C4 – C 1]

[Multiplying C1, C2 and C3 by a, b and c, respectively]

Objective_Maths_JEE Main 2017_Ch 6.indd 40 01/01/2008 04:15:48


Determinants  6.41

58. We have,
b2 + c2 b2 c2
abc 2bc − a 2 c2 b2
=
a2 c2 + a2 c2
abc c2 2ca − b 2 a2
a2 b2 a2 + b2
b2 a2 2ab − c 2

[Taking a, b, c common from R1, R2 and R3, respectively]
a b c −a c b
0 b2 c2
= b c a × −b a c

= − 2c 2
c2 + a2 c2 c a b −c b a
− 2b 2 b2 a2 + b2
2
a b c a b c a b c
[Applying C1 → C1 – C2 – C3]
= b c a
b c a = b c a
0 b2 c2 c a b c a b c a b
= – 2 c2
a2 0 = [a (bc – a2) + b (ac – b2) + c (ab – c2)]2
2 2
b 0 a = [a3 + b3 + c3 – 3abc]2
[Taking – 2 common from C1 and then applying C2 → C2 –
∴ k = – 3.
C1 and C3 → C3 – C1] The correct option is (D)
= – 2 [0 – b2(a2c2) + c2 (– a2b2)] = 4a2b2c2.


The correct option is (C) bg bg ′ + b ′g b ′g ′
59. We have, ga ga ′ + g ′a g ′a ′ m
x + c1 x+a x+a
ab ab ′ + a ′b a ′b ′
57. We have, f (x) = x + b x + c2 x + a (1)
x+b x+b x + c3 b g b g
⋅ + 1
b′ g ′ b′ g ′
x + c1 a − c1 0 g a g a
= x + b c2 − b a − c2
= (β ′γ  ′) (γ  ′α ′) (α ′β ′)
⋅ + 1
g ′ a′ g ′ a′
x+b 0 c3 − b a b a b
⋅ + 1
[Applying C2 → C2 – C1 and C3 → C3 – C2]
a′ b′ a′ b′

1 a − c1 0 c1 a − c1 0 [Taking β ′γ  ′, γ  ′α ′ and α ′β ′ common from R1, R2 and R3



= x 1 c2 − b a − c2 + b c2 − b a − c2
respectively]
1 0 c3 − b b 0 c3 − b b g b g
⋅ + 1

So, f (x) is linear. b′ g ′ b′ g ′

Let f (x) = Px + Q, g ⎛a b⎞ ⎛a b⎞
= (α  ′β ′γ  ′ )2
− ⎜⎝ a’ − b ′ ⎟⎠ 0
then, f  (– a) = – aP + Q, f (– b) = – bP + Q
g ′ ⎜⎝ a ′ b ′ ⎟⎠
b f ( − a) − a f ( − b) b ⎛a g ⎞ ⎛a g ⎞
∴ f (0) = 0.P + Q = Q = (2) −
( b − a) b ′ ⎜⎝ a ′ g ′ ⎟⎠ ⎜⎝ a ′ − g ′ ⎟⎠ 0

c1 − a 0 0
[Applying R2 → R2 – R1 and R3 → R3 – R1]


From (1), f (– a) = b − a c2 − a 0
b − a b − a c3 − a ⎛a b⎞ ⎛a g ⎞
= (α ′β ′γ  ′ )2 ⎜ − ⎟ ⎜ − ⎟

= (c1 – a) (c2 – a) (c3 – a). ⎝ a′ b′⎠ ⎝ a′ g ′⎠
Similarly, f (– b) = (c1 – b) (c2 – b) (c3 – b)
b g b g
Also, g (x) = (c1 – x) (c2 – x) (c3 – x) ⋅ + 1
b′ g ′ b′ g ′
∴ g (A)  = f (– a) and g (b) = f (– b)
g
bg ( a) − ag (b) 1 0
So, we get from (2), f (0) =
. g′
( b − a)
b

The correct option is (A) 1 0
b′

Objective_Maths_JEE Main 2017_Ch 6.indd 41 01/01/2008 04:15:51


6.42  Chapter 6

(ab ′ − a ′ b ) (ag ′ − a ′g ) ⎛ g b⎞ x + a2 ab ac
= (α  ′ β  ′γ  ′)2 ⋅
⋅ ⎜⎝ g ′ − b ′ ⎟⎠
a ′b ′ a ′g ′ 62. We have, ab x+b 2
bc =0
2
(ab ′ − a ′ b ) (ag ′ − a ′g ) (g b’ − g ′ b ) ac bc x+c
= (α  ′β  ′γ  ′)2 ⋅

(a ′b ′g ) 2

= (aβ  ′ – α  ′β) (aγ  ′ – α  ′γ) (γ β  ′ – γ  ′β). x + a2 b2 c2
The correct option is (A) ⇒ a2 x + b2 c2 =0
60. We have, 2 2 2
a b x+c
x +1 x x x +1 x x
[Taking a, b, c common from R1, R2, R3, respectively and
x x+a x = x+0 x+a x then multiplying columnwise]
x x x + a2 x+0 x x + a2 1 b2 c2

2 2 2
x x x 1 x x ⇒ (x + a + b + c ) 1 x + b 2
c2 =0
2 2
= x
x+a x + 0 x+a x 1 b x+c
2 2
x x x+a 0 x x+a [Applying C1 → C1 + C2 + C3 and taking (x + a2 + b2 + c2)

common from C1]
x x x 1 x x
= 0 a
0 + 0 x+a x 1 b2 c2
2 2 2
⇒ (x + a + b + c ) 0 x 0 =0
0 0 a2 0 x x + a2
0 0 x
[Applying R2 → R2 – R1 and R3 → R3 – R1 in first determinant]

[Applying R2 → R2 – R1 and R3 → R3 – R1]
x+a x
= x ⋅ a ⋅ a2 + 1
⋅ ⇒ x2 (x + a2 + b2 + c2) = 0
x x + a2 But x ≠ 0, ∴ x = – (a2 + b2 + c2).
 [Expanding both the determinants along C1] The correct option is (B)
= xa + [(x + a) (x + a2) – x2]
3
63. We have,
= xa3 + a2x + ax + a3 = a3 + xa (a2 + a + 1). 3 m
∴ f (x) = x.
D= = – 15 – 2m ,
2 −5
The correct option is (A)
m m

D1 = = – 25m
− bc 2
b + bc c + bc 2 20 − 5
2
61. We have, a + ac − ac c 2 + ac 3 m
2 2

and, D2 = = 60 – 2m.
a + ab b + ab − ab 2 20

So, by Cramer’s rule
− bc ab + ac ac + ab
abc D1 − 25 m 25 m
= ab + bc − ac bc + ab x= = =
abc D − 15 − 2 m 15 + 2 m
ac + bc bc + ca − ab
D2 60 − 2 m 2m − 60

[Multiplying R1, R2, R3 by a, b, c, respectively, and taking a,
and, y= = =
D − 15 − 2m 2m + 15
b, c common from C1, C2 and C3, respectively]
25m
− bc 1 1 Since x > 0 ⇒
> 0 i.e., 25m (2m + 15) > 0
15 + 2m
= (ab + bc + ac)2 ab + bc −1 0
ac + bc 0 −1 ⎛ −15 ⎞

m ∈ ⎜ − ∞, ∪ (0, ∞)(1)
⎝ 2 ⎟⎠
[Applying C3 → C3 – C1 and C2 → C2 – C1, and taking (ab +
ac + bc) common from C2 and C3] 2m − 60
= (ab + bc + ac)2 [1 ⋅ (0 + ac + bc) – 1⋅(bc – ab – bc)] Also, y > 0 ⇒
> 0 i.e., (2m – 60) (2m + 15) > 0
2m + 15
 [Expanding along C3]
⎛ −15 ⎞
= (ab + bc + ac)3. ⇒
m ∈ ⎜ − ∞, ∪ (30, ∞)(2)
⎝ 2 ⎟⎠
The correct option is (B)

Objective_Maths_JEE Main 2017_Ch 6.indd 42 01/01/2008 04:15:55


Determinants  6.43

⎛ −15 ⎞ ( a + d ) ( a + 2d ) a + 2d − 2d
From (1) and (2), we get m ∈ ⎜ − ∞,
⎟ ∪ (30, ∞).
⎝ 2 ⎠ ( a + 2d ) 2d d 0 , C3 → C3 – C2
( a + 3d ) 2d d 0

The correct option is (B)
− 2d
a b c =
a ( a + d ) 2 ( a + 2d )3 ( a + 3d ) 2 ( a + 4 d )
64. We have, b c a = 0
c a b ( a + 2d ) 2d d
( a + 3d ) 2d d
1
⇒ − (a + b + c) [(a – b)2 + (b – c)2 + (c – a)2] = 0
2 − 4d 3 a + 2d 1
⇒ Either a + b + c = 0 or a = b = c =
a ( a + d ) ( a + 2d )3 ( a + 3d ) 2 ( a + 4 d )
2
a + 3d 1
If a + b + c = 0, then we must have
cos θ + cos 3θ + cos 2θ = 0 4d 4
=
and, sin θ + sin 3θ – sin 2θ = 0 a ( a + d ) 2 ( a + 2d )3 ( a + 3d ) 2 ( a + 4 d )
or, cos2θ (2cos θ + 1) = 0
The correct option is (B)
and, sin 2θ (2 cos θ – 1) = 0
The above equations do not hold simultaneously because cos (b + c)2 c2 b2
p
2θ = 0 i.e., θ = then second equation is not satisfied and if 66. We have, c2 ( c + a) 2 a2
4 1 2p 2 2
2cos θ + 1 = 0 or cos θ = − i.e, θ = , then also second b a ( a + b) 2
2 3
equation is not satisfied.
a 2 (b + c)2 a 2c 2 a 2b 2
Therefore, the only possibility is a = b = c. 1
= c 2b 2 b 2 ( c + a) 2 a 2b 2
or, eiθ = e– 2iθ = e3iθ which is satisfied only when a 2b 2c 2
eiθ = 1 i.e., cos θ + isin θ = 1 b 2c 2 a 2c 2 c 2 ( a + b) 2
∴ cos θ = 1 and sin θ = 0. ∴ θ = 2nπ. [Multiplying R1, R2 and R3 by a2, b2 and c2, respectively]

The correct option is (B)
( y + z )2 y2 z2
65. We have, 1
= x2 ( z + x)2 z2
1 1 1 xyz
a a (a + d ) ( a + d ) ( a + 2d ) x2 y2 ( x + y )2
1 1 1 [Putting bc = x, ca = y and ab = z]

a+d ( a + d ) ( a + 2d ) ( a + 2d ) ( a + 3d )
( x + y + z) ( y + z − x) ( x + y + z)
1 1 1 1
= 0 ( x + y + z) ( z + x − y)
a + 2d ( a + 2d ) ( a + 3d ) ( a + 3d ) ( a + 4 d ) xyz
x2 y2
1
=
a ( a + d ) 2 ( a + 2d )3 ( a + 3d ) 2 ( a + 4 d ) 0
( a + d ) ( a + 2d ) a + 2d a ( x + y + z) ( z − x − y)
 × ( a + 2d ) ( a + 3d ) a + 3d a+d ( x + y )2

( a + 3d ) ( a + 4 d ) a + 4 d a + 2d [Applying R1 → R1 – R2, R2 → R2 – R3]
1
= y+z−x y−z−x 0
a ( a + d ) ( a + 2d )3 ( a + 3d ) 2 ( a + 4 d )
2
( x + y + z )2
= 0 z + x − y ( z − x − y)
( a + d ) ( a + 2d ) a + 2d a xyz
x2 y2 ( x + y )2
( a + 2d ) 2d d d
( a + 3d ) 2d d d y+z−x y − z − x 2 ( x − y)
( x + y + z )2
[Applying R3 → R3 – R2, R2 → R2 – R1] = 0 z+x−y − 2x
xyz
1 x2 y2 2 xy
=
a ( a + d ) 2 ( a + 2d )3 ( a + 3d ) 2 ( a + 4 d )
[Applying C3 → C3 – C1 – C2]

Objective_Maths_JEE Main 2017_Ch 6.indd 43 01/01/2008 04:15:59


6.44  Chapter 6

68. Since the given system has a non-trivial solution,


y+z−x 0 −2y
( x + y + z )2 therefore,
= 0 z + x − y − 2x
xyz sin a sin b sin g
x2 y2 2 xy

cos a cos b cos g =0
[Applying R1 → R1 + R2]

1 1 1
x ( y + z − x) 0 − 2 xy
( x + y + z) sin a sin b − sin a sin g − sin a
= 0 y ( z + x − y ) − 2 xy
xy ⋅ xyz ⇒ cos a cos b − cos a cos g − cos a = 0
x2 y2 2 xy
1 0 0

[Multiplying R1, R2 by x and y, respectively] [Applying C2 → C2 – C1, C3 → C3 – C1]

x ( y + z) y2 0 ⎛b + a⎞ ⎛b −a⎞
( x + y + z) sin a 2 cos ⎜ sin ⎜
= x 2
y ( z + x) 0 ⎝ 2 ⎟⎠ ⎝ 2 ⎟⎠
xy ⋅ xyz
x2 y2 2 xy ⇒ ⎛b + a⎞ ⎛a − b⎞
cos a 2 sin ⎜ sin ⎜
⎝ 2 ⎟⎠ ⎝ 2 ⎟⎠
[Applying R1 → R1 + R3, R2 → R2 + R3]

1 0
2
( x + y + z) x ( y + z) y 2
= ⋅ 2xy ⎛g + a⎞ ⎛g −a⎞
xy ⋅ xyz y ( z + x) 2 cos ⎜ ⎟ sin ⎜
x ⎝ 2 ⎠ ⎝ 2 ⎟⎠

 [Expanding along C3]

⎛g + a⎞ ⎛a − g ⎞ = 0
2 sin ⎜ sin ⎜
2 ( x + y + z) 2x ( y + z) y ⎝ 2 ⎟⎠ ⎝ 2 ⎟⎠
= ⋅ xy
xy ⋅ xyz x 2
z+x 0

2 ( x + y + z )2 x + y + z x+ y+z ⎛a − b⎞ ⎛g −a⎞
⇒ 4 sin ⎜ sin ⎜
= ⎝ 2 ⎟⎠ ⎝ 2 ⎟⎠
z x z+x
[Applying R1 → R1 + R2]
⎡ ⎛g + a⎞ ⎛b + a⎞ ⎛b + a⎞ ⎛g + a⎞⎤
⎢sin ⎜⎝ 2 ⎟⎠ cos ⎜⎝ 2 ⎟⎠ − sin ⎜⎝ 2 ⎟⎠ cos ⎜⎝ 2 ⎟⎠ ⎥ = 0
2 ( x + y + z )3 1 1 ⎣ ⎦
= = 2 (x + y + z)3
z x z+x ⎛a − b⎞ ⎛b −g ⎞ ⎛g −a⎞
⇒ – 4 sin ⎜ ⎟ sin ⎜ sin ⎜ =0
⎝ 2 ⎠ ⎝ 2 ⎠ ⎟ ⎝ 2 ⎟⎠
= 2 (ab + bc + ca)3.
The correct option is (A) ⎛a − b⎞
⇒ sin ⎜ =0
67. The three equations in two unknowns will be consistent if ⎝ 2 ⎟⎠

( a + 1)3 ( a + 2)3 − ( a + 3)3 ⎛b −g ⎞ ⎛g −a⎞



or sin ⎜ = 0 or sin ⎜ =0
⎝ 2 ⎟⎠ ⎝ 2 ⎟⎠

( a + 1) ( a + 2) − ( a + 3) = 0
1 1 −1 ⇒ α – β = 0 or β – γ = 0 or γ – α = 0
⇒ α = β or β = γ or γ = α
( a + 1)3 ( a + 2)3 ( a + 3)3 ⇒ triangles must be isosceles.
i.e., ( a + 1) ( a + 2) ( a + 3) = 0
The correct option is (A)
1 1 1 69. We have,
x1 + a1b1 a1b2 a1b3
x3 y3 z3
a2b1 x2 + a2b2 a2b3
⇒ x y z =0 a3b1 a3b2 x3 + a3b3
1 1 1
x1 + a1b1 a1b1b2 a1b1b3
[where x = a + 1, y = a + 2 and z = a + 3] 1
= a2b1 b1 x2 + a2b1b2 a2b1b3
⇒ (x – y) (y – z) (z – x) (x + y + z) = 0 b12
a3b1 a3b1b2 b1x3 + a3b1b3
⇒ (– 1) (– 1) (2) (3a + 6) = 0 ⇒ a + 2 = 0 or a = – 2.
The correct option is (D)
[Multiplying C2 and C3 by b1]

Objective_Maths_JEE Main 2017_Ch 6.indd 44 01/01/2008 04:16:03


Determinants  6.45

71. Differentiating both sides of the given equation w.r.t. x, we


x1 + a1b1 − b2 x1 − b3 x1
1 get
= a2b1 b1x2 0
b12 1 1 3 ( x + 2) 2
a3b1 0 b1x3
x+2 x+3 ( x + 2)3
[Applying C2 → C2 – b2 C1 and C3 → C3 – b3C1]

3
( x + 2) x+2 x+3
x1 + a1b1 − b2 x1 − b3 x1
= a2 x2 0 x+3 x+2 ( x + 2)3
a3 0 x3  + 1 1 3 ( x + 2) 2
( x + 2) 3 x+2 x+3
ab
1+ 1 1 − b2 − b3
x1
x+3 x + 2 ( x + 2) 3
a2
= x1x2x3 1 0  + x+2 x + 3 ( x + 2) 3
x2
a3 3 ( x + 2) 3 1 1
0 1
x3
= (7ax6 + 6bx5 + 5cx4 + 4dx3 + 3ex2 + 2fx + g).

[Taking x1, x2 and x3 common from R1, R2 and R3,
Putting x = 0 on both sides, we get
respectively]
1 1 12 3 2 8 3 2 8
a1b1 a2b2 a3b3
1+ + + 0 0 2 3 8 + 1 1 12 + 2 3 8 = g
x1 x2 x3
8 2 3 8 2 3 12 1 1
a2
= x1x2x3 1 0
x2 ⇒ g = – 189 + 75 – 99. ∴ g = – 213.
a3 The correct option is (A)
0 1
x3 1 1 1
[Applying R1 → R1 + b2R2 + b3R3]
2
y z2 2
72. We have, x y z x
n n n

⎛ ab a b ab ⎞ x 3
y3 z3
= x1x2x3 . ⎜1 + 1 1 + 2 2 + 3 3 ⎟
⎝ x1 x 2 x3 ⎠ 1
= (x – y) (y – z) (z – x) (xy + xz + yz)

The correct option is (A) xyz
1 1 1
70. We have,
⇒ xn + 1 ⋅ yn + 1 ⋅ z n + 1 x 2 y2 z2
a a+d a + 2d
x3 y3 z3

a2 ( a + d ) 2 ( a + 2d ) 2 = 0

= (x – y) (y – z) (z – x) (xy + xz + yz)
2a + 3d 2 (a + d ) 2a + d
1 1 1
a d d
Since the degree of the determinant x 2
y 2
z 2 is 5,
⇒ a 2
d ( d + 2a) d (3d + 2a) = 0 x 3
y 3
z3
2a + 3d −d −d ∴ degree of L.H.S. = 3n + 3 + 5 = 3n + 8.
[Applying C2 → C2 – C1, C3 → C3 – C2]
Also, degree of R.H.S. = 5. ∴ 3n + 8 = 5 ⇒ n = – 1.
The correct option is (B)
a 1 1
2
73. We have,
⇒ d  a 2
d + 2a 3d + 2a = 0
sin a cos b cos a cos b − sin a sin b
2a + 3d −1 −1
sin a sin b cos a sin b sin a cos b
a 1 0 cos a − sin a 0

⇒ d 2 a2 d + 2a 2d = 0 sin a
0 0 −
2a + 3d −1 0 sin b
=
[Applying C3 → C3 – C2] sin a sin b cos a sin b sin a cos b
⇒ d 2 (3a + 3d) = 0 [Expanding along C3] cos a − sin a 0
⇒ d = 0 or a + d = 0. ⎡ cos b ⎤
⎢ Applying R1 → R1 − R2 ⎥
The correct option is (C) ⎣ sin b ⎦


Objective_Maths_JEE Main 2017_Ch 6.indd 45 01/01/2008 04:16:06


6.46  Chapter 6

sin a ⎡⎛ a ⎞⎛ b ⎞
= −
(– sin2α sin β – cos2α sinβ) = c1 ⎢⎜ a2 − 1 b2 ⎟ ⎜ b3 − 1 c3 ⎟

sin b ⎢⎣ ⎝ b1 ⎠ ⎝ c1 ⎠
= sin α, which is independent of β.
⎛ a1 ⎞ ⎛ b1 ⎞ ⎤

The correct option is (B)
–
⎜⎝ a3 − b b3 ⎟⎠ ⎜⎝ b2 − c c2 ⎟⎠ ⎥
1 1 ⎦⎥
1+ a 1 1
Since a1, a2, a3, b1, b2, b3 are 1 or – 1 (1)
74. We have, 1 1+ b 1 a1 b a b
1 1 1+ g ∴ a 2, , b , b , 1 , c , a , 1 b , b , 1 , c are 1 or – 1
b1 2 3 c1 3 3 b1 3 2 c1 2
1 1 1 a1 b a b
+1 ∴ a2, – b2, b3 – 1 c3, a3 – 1 b3, b2 – 1 c2 are 2, – 2 or 0
a b g b1 c1 b1 c1
1 1 1
= abγ
+1 ⎛ a ⎞ ⎛ b ⎞
a b g ∴ ⎜ a2 − 1 b2 ⎟ ⎜ b3 − 1 c3 ⎟ = 4, – 4 or 0 = an even number.
⎝ b1 ⎠ ⎝ c1 ⎠
1 1 1
+1 ∴ From (1), ∆ = an even number ( c1 = 1 or – 1).
a b g
The correct option is (A)
1 1
1
b g 76. Since ABθ = BθA ∴ (ABθ)θ = (BθA)θ
⎛ 1 1 1⎞ 1 1 ⇒ (Bθ)θAθ = Aθ(Bθ)θ ⇒ BAθ = AθB
= abγ ⎜1 + + + ⎟ 1
+1
⎝ a b g⎠ b g Now, AB(AB)θ = AB(BθAθ) = A(BBθ) Aθ

1 1
= A(BθB)Aθ = (ABθ) (BAθ)
1 +1
b g

= BθAAθB = BθAθAB
[Applying C1 → C1 + C2 + C3)

The correct option is (A)
1 1
1 77. ∆′(x) = 4Ax3 + 3Bx2 + 2Cx + D
⎛ 1 1 1⎞ b g
= abγ ⎜1 + + + ⎟
∴ 5A + 4B + 3C + 2D + E = ∆(1) + ∆′(1)
⎝ a b g⎠ 0 1 0
0 0 1 1 2 1
[Applying R2 → R2 – R1, R3 → R3 – R1]
But, ∆(1) = 1 1 6 = 0
[∵ R2, R3 are identical]
⎛ 1 1 1⎞ 1 1 6
= abγ ⎜1 + + + ⎟ [Expanding along C1]

⎝ a b g⎠ 1 0 1 x 2 x x 2 x
= abγ + (bγ + aγ + aβ)
∆′(x) = x 2
x 6 + 2x 1 0 + x2 x 6
[Since α, β, γ are roots of x3 + px + q = 0 x x 6 x x 6 1 1 0
∴ aβ + aγ + bγ = p and abγ = – q]
= – q + p. 1 0 1 1 2 1 1 2 1
The correct option is (C) ∴ ∆′(1) = 1 1 6 + 2 1 0 + 1 1 6
1 1 6 1 1 6 1 1 0
a1 a2 a3
75. Let ∆ = b1 b2 1 2 1 1 2 1
b3 be any determinant of third order.

= 0+ 0 −3 −2 + 1 1 6
c1 c2 c3
0 −1 5 0 0 −6
a1 a1
0 a2 − b2 a3 − b3 = – 17 – 6(1 – 2) = – 17 + 6 = – 11
b1 b1
∴ required value = – 11.
Then, ∆ =
b1 b1
0 b2 − c2 b3 − c3 The correct option is (D)
c1 c1
78. The given determinant D1 is obtained by replacing each
c1 c2 c3 ­element of D2 by its co-factor respectively.
⎡ a1 b1 ⎤ ∴ D1 = D22
⎢ Applying R1 → R1 − R2 , R2 → R2 − R3 ⎥ ∴ D1D2 = D22D2 = D23.
⎣ b c1 ⎦
 1
The correct option is (B)

Objective_Maths_JEE Main 2017_Ch 6.indd 46 01/01/2008 04:16:10


Determinants  6.47

⎡a b c ⎤ ⎡a c b⎤ a2 b2 c2
79. We have, AA′ = ⎢ c a b ⎥ ⎢ b a c ⎥ = I ⇒ a b c = 0 (Apply R3 → R3 – R1)
⎢ ⎥⎢ ⎥
⎢⎣ b c a ⎥⎦ ⎢⎣ c b a ⎥⎦ 1 − 2a 1 − 2b 1 − 2c
⎡ a 2 + b 2 + c 2 ac + ab + ac ab + bc + ca ⎤
⎢ ⎥ a2 b2 c2
⇒ ⎢ ca + ab + bc a 2 + b 2 + c 2 cb + ac + ba ⎥ = I ⇒
⎢ a b c =0
2 2 2 ⎥
⎢⎣ ba + cb + ac bc + ca + ab a + b + c ⎥⎦ 1 1 1
∴ a2 + b2 + c2 = 1 and ab + bc + ca = 0 ⇒ (a – b) (b – c) (c – a) = 0
⇒ a + b + c = ± 1 ⇒ a = b or b = c or c = a
Also, abc = γ The correct option is (B)
∴ a, b, c are the roots of the equation l −1 cos q
x3 ± x2 + γ = 0. 81. Determinant of coefficients = 3 1 2
The correct option is (A) cos q 1 2
2 2 2 2
a b c = cos θ – cos θ + 6 and this is positive for all θ since | cos θ
80. We have, ( a + 1) 2 (b + 1) 2
(c + 1) 2
=0 | ≤ 1, the only solution is therefore the trivial solution.
The correct option is (A)
( a − 1) 2 (b − 1) 2 (c − 1) 2

a2 b2 c2
⇒ 4 a b c =0
2 2 2
( a − 1) (b − 1) (c − 1)
(Apply R2 → R2 – R3)

More than One Option Correct Type


82. Here, the equations are in two variables x and y. If they are k
consistent then the values of x and y obtained from first two
Now, ∑ Di = D1 + D 2 + D3 + ... + D k
equations should satisfy the third equation and hence D = 0. i =1
i.e.,
k
Sai Sai Sai
1 1 −3
⇒ ∑ D i = Sai Sai Sai = 0
⇒ 1+ l 2+l −8 = 0 i =1
Sai Sai Sai
1 −1− l 2+l

The correct option is (A, B)
1 0 0
eiA e − i (C + A) e − i ( B + A)
⇒ 1+ l 1 − 5 + 3l = 0
− i (C + B )
1 −2−l 5+l 84. z = eiA.eiB.eiC e eiB e −i ( A + B )
e −i( B + C ) e −i( A + C )
eiC
[Applying C2 → C2 – C1, C3 → C3 + 3C1]

⇒ (5 + λ) + (2 + λ) ( – 5 + 3λ) = 0 ⇒ 3l2 + 2λ – 5 = 0 eiA − eiB − eic
5 ⇒ z = –1 − eiA eiB − eic
⇒ (λ – 1) (3λ + 5) = 0 ⇒ λ = 1, − .
3
− eiA − eiB eic
The correct option is (A, C)

⎛ The number of ⎞ since ei(A + B + C) = eiπ = cos π + sin π) = –1



⎛ The number ⎞ ⎜ arrangements of ⎟ 0 −2eiB 0
⎜ ⎟
83. ⎜ of third-order ⎟ = ⎜ nine different ⎟ = 9! ⇒ z =– −2e iA
0 0
⎜ ⎟ ⎜ ⎟
⎝ determinants ⎠ ⎜ numbers in ⎟ −e iA
−e iB
eiC
⎜⎝ nine places ⎟⎠
(Using R1 → R1 + R3, R2 → R2 + R3)

Objective_Maths_JEE Main 2017_Ch 6.indd 47 01/01/2008 04:16:12


6.48  Chapter 6

⇒ z = –2eiB {– 2ei (A + C)} a b c


∴ z = 4 ei (A + B + C) = 4 eiπ = – 4 87. Let ∆ = b c a
The correct option is (B, C)
c a b

x2 + x x +1 x−2 Since Dc = D2, where Dc is determinant of co-factors of ∆



85. We have, 2 x 2 + 3 x − 1 3x 3x − 3 bc − a 2 ca − b 2 ab − c 2 a b c
2

x2 + 2x + 3 2x − 1 2x − 1 ⇒ ca − b 2 ab − c 2 bc − a 2 = b c a
ab − c 2 bc − a 2 ca − b 2 c a b
4 0 0
= 2 x 2 + 3x − 1
3x 3x − 3 a b c a b c
2
x + 2x + 3 2x − 1 2x − 1
= b c a × b c a
[Applying R1 → R1 + R3 – R2]
c a b c a b
4 0 0
a2 + b2 + c2 ab + bc + ca ac + ba + bc
= 2 x 2 + 2 3 3x − 3

= ab + bc + ca b 2 + c 2 + a 2
bc + ac + ab
x2 + 4 0 2x − 1
ca + ab + bc bc + ac + ab c 2 + a 2 + b 2
[Applying C1 → C1 – C3 , C2 → C2 – C3]

4 0 0 a2 b2 b2
2 2 2 2
= 2 3 3x − 3
= b2 a2
b 2 where a = a + b + c
4 0 2x − 1 b2 b2 a2

⎡ x2 x2 ⎤ and, b2 = ab + bc + ca.

⎢ Applying R2 → R2 − R1 , R3 → R3 − R1 ⎥
⎢⎣ 2 4 ⎦⎥
The correct option is (A, B)

4 0 0 4 0 0 sin x sin y sin z
= 2 3 3x + 2 3 − 3
88. We have, cos x cos y cos z
4 0 2x 4 0 −1 cos3 x cos3 y cos3 z
4 0 0 4 0 0 tan x tan y tan z
= x 2 3 3 + 2 3 −3
= cos x cos y cos z
1 1 1
4 0 2 4 0 −1
cos 2 x cos 2 y cos 2 z
=xA+B

= cos x cos y cos z

4 0 0 4 0 0
tan x tan y − tan x tan z − tan y
where, A = 2 3 3 and B = 2 3 − 3 .

1 0 0
4 0 2 4 0 −1
cos 2 x cos 2 y − cos 2 x cos 2 z − cos 2 y
The correct option is (A, D)
86. We have, [Applying C2 → C2 – C1 and C3 → C3 – C2]
= – cos x cos y cos z
0 x−a x−b
tan y − tan x tan z − tan y
x + a 0 x−c =0
cos y − cos x cos 2 z − cos 2 y
2 2
x+b x+c 0
 [Expanding along R2]
⇒ (x – a) (x + b) (x – c) + (x – b) (x + a) (x + c) = 0 cos z sin ( x − y ) cos x sin ( y − z )
 [Expanding along R1] =

sin ( x + y ) sin ( x − y ) sin ( y + z ) sin ( y − z )
⇒ 2x (x2 + ac – ab – bc) = 0
⇒ x = 0 or x2 = b (a + c) – ac. = sin (x – y) sin (y – z) sin (z – x) cos (x + y + z)


Therefore, the given determinant is zero only when any two
If b (a + c) > ac, we have three roots 0, ±
b ( a + c) − ac. p
of x, y, z are equal or x + y + z = .
If b (a + c) ≤ ac, we have only one real root x = 0.
2

The correct option is (A, C)
The correct option is (A, B, C, D)

Objective_Maths_JEE Main 2017_Ch 6.indd 48 01/01/2008 04:16:15


Determinants  6.49

for n = 1 and 2.
cos(q + a ) − sin (q + a ) cos 2 a

The correct option is (A, C)
89. We have, sin q cos q sin a
− cos q sin q l cos a 91. We have,
p /2
2 − [cos 2nx + cos 2 ( n + 2) x ]
cos(q + a ) − sin (q + a ) cos 2 a an + an + 2 = ∫ 1 − cos 2 x
dx
1 0
= sin q sin a cos q sin a sin 2 a
sin a cos a p /2
− cos q cos a sin q cos a l cos 2 a 2 − 2 cos 2 ( n + 1) x cos 2 x

= ∫ 1 − cos 2 x
dx
[Multiplying R2 and R3 by sin α and cosα, respectively]
0

p /2
1 1 − cos 2 ( n + 1) x
=
sin a cos a
Also, 2 ⋅ an + 1 = 2
∫ 1 − cos 2 x
dx
0
2 2 ∴ an + an + 2 – 2 ⋅ an + 1
0 0 cos 2 a + sin a + l cos a
p /2
sin q sin a cos q sin a sin 2 a 1 − cos 2 ( n + 1) x ⋅ cos 2 x − 1 + cos 2 ( n + 1) x
− cos q cos a sin q cos a l cos a 2
=2
∫ 1 − cos 2 x
dx
0

p /2
[Applying R1 → R1 + R2 + R3] cos 2 ( n + 1) x ⋅ (1 − cos 2 x )
2
cos 2 a + sin a + l cos a 2
sin q sin a cos q sin a
=2
∫ 1 − cos 2 x
dx
0
=
sin a ⋅ cos a − cos q cos a sin q cos a p /2

sin q cos q
=2
∫ cos 2 ( n + 1) x dx
= (cos2α + λ cos2α) = (1 + λ) cos2α. 0
− cos q sin q p /2
⎡ sin 2 ( n + 1) x ⎤
Therefore, the given determinant is independent of q for all
∴ an + an + 2 – 2an + 1 = 2 ⎢ ⎥
real values of λ. Also, if λ = – 1, then it is independent of q ⎣ 2 ( n + 1) ⎦ 0
and a. 1

The correct option is (A, C)
= (0 – 0) = 0
n +1
90. We have, an + an + 2
∴ an + 1 = (1)
2 2
1 + sin q cos q 4 sin 4q 2
⇒ an + 1 is the A. M. between an and an + 2.
sin 2 q 1 + cos 2 q 4 sin 4q =0
sin 2 q cos 2 q 1 + 4 sin 4q a1 a2 a3 a1 2 a2 a3
1

Now, a4 a5 a6 = a4 2 a5 a6
2
2 cos 2 q 4 sin 4q a7 a8 a9 a7 2 a8 a9
⇒ 2 1 + cos q 2
4 sin 4q =0
2
a1 2 a2 − ( a1 + a3 ) a3
1 cos q 1 + 4 sin 4q 1

= a4 2 a5 − ( a4 + a6 ) a6
2
a7 2 a8 − ( a7 + a9 ) a9
[Applying C1 → C1 + C2]

[Applying C2 → C2 – C1 – C3]

2 cos 2 q 4 sin 4q a1 0 a3
⇒ 0 1 0 =0 1

= a4 0 a6  [using (1)]
−1 0 1 2
a7 0 a9
[Applying R2 → R2 – R1 and R3 → R3 – R1]
= 0.
⇒ 2 + 4 sin 4θ = 0 [Expanding along R2]
The correct option is (A, D)
1 p ⎛ −p ⎞ 92. We have,
⇒ sin4θ = − = – sin = sin ⎜
2 6 ⎝ 6 ⎟⎠
bg ga ab
⎛ −p ⎞ n ga ab bg = 0
⇒ 4θ = nπ + (– 1) ⎜
⎝ 6 ⎟⎠
ab bg ga
p 7p 11p
The values of θ lying between 0 and
are and ⇒ (aβ)3 + (bγ )3 + (gα)3 – 3 (aβ ) (bγ ) (gα) = 0
2 24 24

Objective_Maths_JEE Main 2017_Ch 6.indd 49 01/01/2008 04:16:20


6.50  Chapter 6

⎡ a b c ⎤ ∴ solutions are (3, 1, 1), (1, 3, 1) (1, 1, 3).


⎢ ⎥ The correct option is (A, B, C)
⎢∵ b c a =0 ⎥
⎢ c a b ⎥ 94. Put x = 0 in the given determinant, we get a = 0. Differentiating
⎢ ⎥ f (x) column by column, we get
⎢⎣⇒ a3 + b3 + c3 − 3 abc = 0 ⎥⎦

⇒ (aβ + bγ w 2 + γ aω) (a b ω + bγ w 2 + γ α) ex sin x 1 ex cos x 1
(abw 2 + b g ω + g α) = 0 2x
f ′(x) = − sin x loge (1 + x 2 ) 1 + cos x
1 +0
⇒ aβ + b g w 2 + g a ω = 0 or a b ω + b g w 2 + g α = 0 2
1 + x2
1 x 1 x 2x 1
or, a b w 2 + bgω + g α = 0
1 1 1 1 1 1
= b + 2cx
⇒ + + = 0 or + + =0
g aω bω 2 gω 2 aω b Now, putting x = 0, we have

1 1 1 1 0 1 1 1 1

or, + + = 0.
gω aω 2
b b = 0 0 1 + 1 0 1 = – 1

The correct option is (A, B, C, D) 1 0 1 0 0 1

x3 + 1 x2 y x2z
The correct option is (A, C)
2 3 2
93. We have, xy y +1 y z
1 + sin 2 x cos 2 x sin 2 x
2 2 3
xz yz z +1 2
95. sin x 1 + cos 2 x sin 2 x
2 2
x3 x2 y x2 z 1 x2 y x2z sin x cos x 1 + sin 2 x

= xy 2
y3 + 1 y2z + 0 y3 + 1 y2z
2 cos 2 x sin 2 x
xz 2 yz 2 z3 + 1 0 yz 2 z3 + 1
= 2 1 + cos x 2
sin 2 x [Apply C1 → C1 + C2]
2
x 2
x y2
x z 2 1 cos x 1 + sin 2 x
= x y
2 3
y +1 2 3
y z + ( y + 1) ( z + 1) − y z 3 3 3

2 2 3 2 cos 2 x sin 2 x
z yz z +1
= 0 1 0
[Applying C2 → C2 – yC1 and C3 → C3 – zC1]

−1 0 1
x2 0 0 [Apply R2 → R2 – R1 and R3 → R3 – R1]
= x y
2
1 0 + y3z3 + z3 + y3 + 1 − y3z3 = 2 + sin 2x
z2 0 1 Since the maximum value of sin 2x is 1, and minimum value
of sin 2x is (– 1). Therefore α = 3, β = 1. Now, α – β = 2, α

= x3 + y3 + z3 + 1 + β = 4 and α + 3β = 6. Thus, (α – β ) + (α + β ) = (α + 3β ).
Given: x + y + z + 1 = 30 ⇒ x3 + y3 + z3 = 29
3 3 3 So, α – β, α + β, α + 3β cannot form a triangle. All other
options are correct.
Since 29 = 33 + 1 + 1

The correct option is (A, B, C)

= 13 + 33 + 13

= 13 + 13 + 33

Passage Based Questions


96. The characteristic equation of matrix A is (1 – λ) [(1 – λ) (0 – λ) – 4] + 2[0 × 2 – 1(1 – λ)] = 0

or, (1 – λ) [l2 – λ – 4] – 2 + 2λ = 0

1− l 0 2
or, – l3 + l2 + 4 λ + l2 – λ – 4 – 2 + 2λ = 0

|A – λI | = 0 or
0 1− l 2 =0
or, – l3 + 2l2 + 5λ – 6 = 0

1 2 0−l
or, l3 – 2l2 – 5λ + 6 = 0


Expanding the determinant from first row, we get or, (λ – 1) (λ + 2) (λ – 3) = 0

Objective_Maths_JEE Main 2017_Ch 6.indd 50 01/01/2008 04:16:22


Determinants  6.51

or, λ = 1, – 2, 3 Repeating this process n times, we get


The characteristic roots of the given martix A are 1, – 2 and AnX = lnX
3. Hence, ln is a characteristic root of An.
The correct option is (A, C, D) (D) Let λ ≠ 0 be a characteristic root of a non-singular
97. The characteristic equation of the matrix A is matrix A so that
1− l −3 3 AX = lX(for n × 1 matrix X)
| A – λI | = 0 or
3 −5−l 3 =0 since A is non singular, A–1 exists,
6 −6 4−l ⇒ A–1(AX) = A–1(lX)
⇒ (A–1A)X = λ(A–1X)
1− l −3 0 ⇒ IX = λ(A–1X)

or, 3 −5−l − 2 − l = 0 [C3 → C3 + C2] ⇒ X = λ(A–1X )
6 −6 −2 − l 1
⇒ A–1X = X (∵ λ ≠ 0)
l

Expanding the determinant, we get Hence, λ–1 is a characteristic root of A–1.
(1 – λ) [(– 5 – λ) (– 2 – λ) + 6(– 2 – λ)]
The correct option is (A, B, C, D)
 + 3[3(–2 –λ) – 6(– 2– λ)] = 0 99. (A)  The characteristic equation of the matrix A–1B is
2
or, (– λ + 1) [ l + 7λ + 10 – 6λ – 12]
| A–1B – λI | = 0 (1)
–1 –1
 + 3[– 6 – 3λ + 12 + 6l] = 0 ⇒ |A| |A B – λI| |A | = 0
2
or, (– λ + 1) [ l + λ – 2] + 3[3λ + 6] = 0
⇒ |A (A–1B – λI)A–1| = 0 [∵ | AB | = | A | | B |]
⇒ | A(A–1B)A–1 – AλIA–1| = 0
or, – l3 – l2 + 2λ + l2 + λ – 2 + 9λ + 18 = 0

⇒ |(AA–1) (BA–1) – λ AA–1| = 0 (AA–1 = 1)
or, – l2 + 12λ + 16 = 0 or l3 – 12λ – 16 = 0

⇒ |I(BA–1) – λI| = 0
(λ – 4) (λ + 2)2 = 0 ⇒ λ = 4, – 2, – 2
⇒ |BA–1 – λI| = 0 (2)
∴ The characteristic roots of the given matrix A are 4, – 2,
From (1) and (2), it follows that A–1B and BA–1 have the
and – 2.
same characteristic equation and hence the same char-
The correct option is (A) acter-istic roots.
98. (A) Since the determinant of a matrix is the same as that of (B)  The characteristic equation of the matrix P–1 AP is
its transpose. | P–1AP – λI | = 0 (1)
| A – λI | = | (A – λI)′ | ⇒ | P AP – P–1 λIP | = 0
–1
(P–1P = I)
or, | A – λI | = | (A′ – λI) | [(λI)′ = λI′ = λI] ⇒ | P–1(A – λI)P | = 0
Hence, A and A′ have the same characteristic roots. ⇒ | P–1 | | A – λI | | P | = 0
(B) Let λ be a characteristic root of A so that ⇒ | P–1P | | A – λI | = 0 [∵ | AB | = | A | | B |]
AX = lX(for n × 1 matrix X) ⇒ | A – λI | = 0 (2) [| P–1P | = | I | = 1]
⇒ k(AX) = k(lX) From (1) and (2) it follows that the matrix A and P–1
⇒ (kA)X = (kλ)X AP have the same characteristic equation and hence the
Hence, kλ is a characteristic root of kA. same characteristic roots.
(C)  AX = lX (C)  AB = IAB
⇒ A(AX) = A(lX) (Multiplying by A) = B–1 B(AB)
⇒ A2X = λ(AX) = B–1(BA)B
= λ(lX) (∵ AX = lX) The characteristic roots of AB = characteristic roots of
= l2X B–1 (BA)B = characteristic roots of BA.
Thus, A2X = l2X ⇒ l2 is a characteristic root of A2. The correct option is (D)

Match the Column Type

1 0 0 II. Differentiate both the sides and put x = 0


100. I. Put x = 0 ⇒ 0 1 0 = f ⇒ f = 1 1 1 0 1 0 0 1 0 0
0 0 1 ⇒ 0 1 0 + 1 1 0 + 0 1 0 =e⇒e=3
0 0 1 0 0 1 0 1 1
The correct option is (B)
The correct option is (A)

Objective_Maths_JEE Main 2017_Ch 6.indd 51 01/01/2008 04:16:23


6.52  Chapter 6

2 1 1 0 −1 1
III, IV.  Put x = 1, then 1 2 1 = p + q + r + s + t + w
−1 0 1 = – p + q – r + s – t + w
1 1 2 1 −1 0

⇒ 4 = p + q + r + s + 3 + 1 =–p+q–r+s–3+1
⇒ – p + q – r + s = 2 (2)
⇒ p + q + r + s = 0 (1)
Solving (1) and (2), q + s = 1 and p + r = – 1.

Put x = – 1, then
The correct option is (C, B)

Assertion-Reason Type

101. Since for x = 0, the determinant reduces to the determinant 104. Since a, b, c are in G. P. with common ratio r1 and α, β, γ
of a skew-symmetric matrix of odd order which is always are in G. P. with common ratio r2, therefore a ≠ 0, α ≠ 0, b
zero. Hence, x = 0 is the solution of the given equation. = ar1, c = a r12 , β = ar2, γ = α r22
The correct option is (A) Also, the system of equations have only trivial solution, so
102. Since the system has a non-trivial solution, a a 1
b b 1 ≠0
l sin a cos a c g 1
therefore, 1 cos a sin a = 0
−1 sin a − cos a a a 1 1 1 1
⇒ ar1 a r2 1 ≠ 0 ⇒ aα r1 r2 1 ≠0
⇒ λ (– cos2 α – sin2 α) – (– sin α cos α – sin α cos α) ar12 a r22 1 r12 r22 1
– (sin2 α – cos2α) = 0
⇒ – λ + sin 2α + cos 2α = 0 ⇒ λ = sin 2α + cos 2α 1 0 0
⇒ aα r1 r2 − r1 1 − r1 ≠0
⎛ p⎞
⇒ λ = 2 cos ⎜ 2 a − ⎟
⎝ 4⎠ r12 r22 − r12 1− r12
⎛ p⎞ [Applying C2 → C2 – C1, C3 → C3 – C1]
Since – 1 ≤ cos ⎜ 2 a − ⎟ ≤ 1 ∀ α ∈ R
⎝ 4⎠
1 0 0
∴ – 2 ≤λ≤ 2 i.e., λ ∈ [– 2 , 2 ]. ⇒ aα (r2 – r1) (1 – r1) r1 1 1 ≠0


The correct option is (A) r12 r2 + r1 1 + r1
⇒ aα (r2 – r1) (1 – r1) (1 – r2) ≠ 0
103. Since a1, a2 and b1, b2 are the roots of ax2 + bx + c = 0 and
px2 + qx + r = 0 respectively, therefore, ⇒ r1 ≠ r2, r1 ≠ 1, r2 ≠ 1.
The correct option is (A)
−b c
a1 + a2 = , a1a2 = (1)
a a a 1 1 0 0 1
−q r 105. We have, 1 b 1 = 1 − a b − 1 1
and, b1 + b2 = , b1b2 = (2)
p p 1 1 c 1 − ac 1 − c c
Since the given system of equations has a non-trivial [Apply C1 → C1 – aC3, C2 → C2 – C3]
solution, = (1 – a) (1 – c) – (b – 1) (1 – ac)
a1 a 2 = 1 – a – c + ac – b + abc + 1 – ac
∴ = 0 i.e., a1b2 – a2b1 = 0
b1 b 2 = (2 + abc) – (a + b + c)
a1 a a + a2 a1a 2 Since the value of the given determinant is positive,
or, = 2 = 1 = therefore,
b1 b2 b1 + b 2 b1b 2
abc + 2 > a + b + c
pb pc b2 ac Using A.M. > G.M., we have,
⇒ = ⇒ 2 = .
qa ra q pr a + b + c > 3(abc)1/3

The correct option is (A) i.e., x3 – 3x + 2 > 0 [putting (abc)1/3 = x]

Objective_Maths_JEE Main 2017_Ch 6.indd 52 01/01/2008 04:16:26


Determinants  6.53

⇒ (x – 1)2 (x + 2) > 0 Then, f (– a) = – aα + β(2)


⇒ x > – 2 f (– b) = – bα + β(3)
i.e., x3 = abc > – 8. bf ( − a) − af ( − b)
and, f (0) = β = (4)
The correct option is (A) ( b − a)
106. We have,  [from (2) and (3)]
x + c1 x + a x + a From equation (1), we have,
f (x) = x + b x + c2 x + a (1) c1 − a 0 0
x + b x + b x + c3 (– a) = b − a c2 − a
f  0
x + c1 a − c1 0 b − a b − a c3 − a
= x + b c2 − b a − c2 = (c1 – a) (c2 – a) (c3 – a)
x+b 0 c3 − b Similarly, f (– b) = (c1 – b) (c2 – b) (c3 – b)
Since, g(x) = (c1 – x) (c3 – x) (c2 – x),
[C3 → C3 – C2, C2 → C2 – C1]
therefore, we can see that
1 c − c1 0 c1 a − c1 0
g(A)  = f (– a) and g(b) = f (– b)
= x 1 c2 − b a − c2 + b c2 − b a − c2
Hence, from (4), we have,
1 0 c3 − b c 0 c3 − b
bg ( a) − ag (b)
(0) = .
f 
which proves that f (x) is linear in x. ( b − a)
Let f (x) = ax + β
The correct option is (A)

Previous Year’s Questions


107. ∵l, mand n are the pth, qth and rth term of an GP whose
3i −1
first term is A and common ratio is R. = ( 6i + 4 )
3 i
∴ l = AR P −1
= (6i + 4)(3i 2 + 3)
⇒ log l = log a + ( p − 1)log R
Similarly, log m = log A+ (q− 1) log R, and =0
log n = log A + (r− l)log R 6i −3i 1
log l p 1 Put 4 3i −1 = x + iy
Now log m q 1 20 3 i
log n r 1
⇒ 0 + 0i = x + iy
log A + ( p − 1) p 1 ⇒ x = 0, y = 0
= log A + ( q − 1) q 1 The correct option is (D)
log A + ( r − 1) r 1 1 1+ i + ω2 ω2

0 p 1 109. 1− i −1 ω 2 − 1 ( R1 → R1 + R3 )
= 0 q 1 −i −1 + ω − i −1
0 r 1

[Using the column transformation C1→C2− (C3 log A+ (C2 1− i −1 ω2 −1
– C3)log R)] = 1− i −1 ω2 −1
The correct option is (D) −i −1 + ω − i −1
6i −3i 1
= 0 (∵ Two rows are identical).
108. Determinant 4 3i 1
The correct option is (A)
20 3 i
110. Coefficient determinant is given by
6i + 4 0 0 1 2a a
= 4 3i −1 ( R1 → R1 + R2 ) 1 3b b = 0
20 3 i 1 4c c

Objective_Maths_JEE Main 2017_Ch 6.indd 53 01/01/2008 04:16:30


6.54  Chapter 6

116. LHS of the given equality is


2ac
⇒b= . a a +1 a −1 a +1 b +1 c −1
a +c
n
The correct option is (C) − b b + 1 b − 1 + ( −1) a − 1 b − 1 c + 1
n 2n n 2n c c −1 c +1 a −b c
1+ ω + ω ω ω
n 2n 2n
a a +1 a −1 a +1 b +1 c −1
111. D = 1+ ω + ω ω 1
= − b b + 1 b − 1 + ( −1) n b + 1 b − 1 − b
1 + ω n + ω 2n 1 ωn c c −1 c +1 c −1 c +1 c
=0
a a +1 a −1 a +1 a a −1
(Since 1 + wn +w2n = 0, if n is not a multiple of 3
= − b b + 1 b − 1 + ( −1) n +1 b + 1 − b b − 1
and so the first column is entirely zero)
c c −1 c +1 c −1 c c +1
The correct option is (A)
a a +1 a −1 a a +1 a −1
log an log an +1 log an + 2 = − b b + 1 b − 1 + ( −1) n+ 2
−b b + 1 b − 1
log an + 3 log an + 4 log an + 5 c c −1 c +1 c c −1 C +1
112.
log an + 6 log an + 7 log an + 8
This is equal to zero only if n + 2 is odd i.e. n is odd integer.
C3 → C3 − C2 , C2 → C3 − C1
The correct option is (C)
log an log r log r 1 2 1
= log an + 3 log r log r = 0 ( where r is a common ratio). 117. D= 2 3 1 =0
log an + 6 log r log r 3 5 2

(Since two columns are identical.)
3 2 1
The correct option is (A)
D1 = 3 3 1 ≠ 0
113. The determinant
1 5 2
1 1 1
⇒ Given system, does not have any solution.
D = 1 1+ x 1
⇒No solution.
1 1 1+ y
The correct option is (C)
C2 → C2− C1 and C3→ C3− C2 4k 2
1 0 0 118. k 41 = 0 ⇒ k 2 − 6 k + 8 = 0 ⇒ k = 4, 2
= 1 x 0 = xy 221
1 0 y The correct option is (A)

The correct option is (B) 119. P3 = Q3
114. The system of linear equations P3 – P2Q = Q3 – Q2P
x − cy − bz = 0 P2(P – Q) = Q2 (Q – P)
cx − y + az = 0 P2(P – Q) + Q2 (P – Q) = O
bx + ay − z = 0 (P2 + Q2)(P – Q) = O
have non-trivial solution if ⇒ P2 + Q2 = 0
1 −c −b
The correct option is (C)
c −1 a = 0 ⇒ 1(1 − a 2 ) + c( − c − ab) − b(ca + b) = 0
⎡1 a 3⎤
b a −1
120. P = ⎢1 3 3⎥⎥

⇒ a2 + b2 + c2 + 2abc = 1 ⎢⎣ 2 4 4 ⎥⎦
The correct option is (D)
Since, |Adj A| = |A|2
115. Each entry of A is integer, so the cofactor of every entry is
|Adj A| = 16
an integer and hence each entry in the adjoint of matrix A is
integer. 1 (12 − 12) − α (4 − 6) + 3 (4 − 6) = 16.
Now detA = ± 1 and A−1 =
1
(adj A) implies that all 2 α − 6 = 16.
det( A) 2 α = 22.
1
entries in A are integers.
α = 11.
The correct option is (C)
The correct option is (A)

Objective_Maths_JEE Main 2017_Ch 6.indd 54 01/01/2008 04:16:32


Determinants  6.55

-x1 + 2x2 - lx3 = 0


31 + a + b1 + a 2 + b 2
2−l −2 1
121. 1 + a + b1 + a 2 + b 21 + a 3 + b 3
⇒ 2 −3 − l 2 =0
1 + a 2 + b 21 + a 3 + b 31 + a 4 + b 4
−1 2 −l

111
2
⇒ ( 2 − l )(3l + l 2 − 4) + 2( −2l + 2) + 1( 4 − 3 − l ) = 0
111 100
= 1ab 1aa 2 = 1a − 1b − 1 ⇒ ( 2 − l )( l 2 + 3l − 4) + 4(1 − l + 2) + (1 − l ) = 0
1a 2b 2 1bb 2 1a 2 − 1b 2 − 1 ⇒ ( 2 − l )( l + 4)( l − 1) + 5(1 − l ) = 0

⇒ (1 − l )(( l + 4)( l − 2) + 5) = 0 ⇒ l = 1,1, −3 .
( ( ) ( ))
2
= (a − 1) b 2 − 1 − (b − 1) a 2 − 1 The correct option is (B)

123. We have,
= (a − 1) (b − 1) (a − b ) ⇒ k = 1
2 2 2
1 l −1
The correct option is (C) l −1 −1 = 0 ⇒ l = 0,1, −1
122. Given system 1 1 −l
(2 − l ) x1 − 2x2 + x3 = 0
2x1 − (3 + l ) x2 + 2x3 = 0 The correct option is (A)

Objective_Maths_JEE Main 2017_Ch 6.indd 55 01/01/2008 04:16:34


Permutations and Combinations 7.1

CHaPtER
Permutations and
7 Combinations

Chapter Highlights
Factorial notation, Fundamental principles of counting, Permutation, Key results on permutations, Combination,
Key results on combination, Derangement, Exponent of prime p in n!, Number of divisors

facTorial noTaTion
CAUTION
We often come across products of the form 1 ⋅ 2, 1 ⋅ 2 ⋅ 3,
1 ⋅ 2 ⋅ 3 ⋅ 4, …  The factorial is defined only for whole numbers.
Instead of writing all the factors of such a product We do not define the factorial of proper fractions or
in full, it is convenient to use a special notation. We write negative integers.
1! = 1,  (2n)! ≠ 2(n)!
(m + n)! ≠ m! + n!
2! = 1 ⋅ 2,

 (mn)! ≠ m! n!
3! = 1 ⋅ 2 ⋅ 3,
……………….....
n! = 1 ⋅ 2 ⋅ 3 … n.
fundamenTal PrinciPles of counTing
“n!” denotes the product of the first n natural numbers. We
read ‘n!’ as ‘n factorial’. n! is also written as ‘ |−−
n ’ and read multiplication Principle
as ‘factorial n’. It is easy to see that
If an operation can be performed in ‘m’ different ways;
1! = 1, 2! = 1 ⋅ 2 = 2, 3! = 1 ⋅ 2 ⋅ 3 = 6, 4! = 1 ⋅ 2 ⋅
following which a second operation can be performed in
3 ⋅ 4. = 24, and so on.
‘n’ different ways, then the two operations in succession can
be performed in m × n different ways.
I M P O R TA N T P O I N T S
Illustration
 We know that
Anu wishes to buy a birthday card for the brother Manu and
n! = n (n – 1) (n – 2) (n – 3) … 3 · 2 · 1
send it by post. Five different types of cards are available at
= n (n – 1)! = n (n – 1) (n – 2)! the card-shop, and four different types of postage stamps
= n (n – 1) (n – 2) (n – 3)! and so on. are available at the post-office. In how many ways can she
Thus, if m, n ∈ N and m > n, then m! can be expressed in choose the card and the stamp?
terms of n!
For example Solution
8! = 8 · 7 · 6! She can choose the card in five ways. For each choice of
10! = 10 · 9 · 8 · 7 · 6 · 5! the card she has four choices for the stamp. Therefore, there
Also, m! = n! if and only if m = n are 5 × 4 ways, i.e., 20 ways of choosing the card and the
 Putting n = 1 in n! = n (n – 1)!, we have stamp.
1! = 1 · 0!
\ 0! = 1
 (2n)! = 2n · n! [1 · 3 · 5 … (2n – 1)]

Objective_Maths_JEE Main 2017_Ch 7.indd 1 01/01/2008 04:20:08


7.2 Chapter 7

addition Principle Solution: (A)


Total number of wrong answers
If an operation can be performed in ‘m’ different ways and
another operation, which is independent of the first opera- = 1 · (a1 – a2) + 2 ⋅ (a2 – a3) +, …,
tion, can be performed in ‘n’ different ways. Then either of + (k – 1) (ak – 1 – ak) + kak
the two operations can be performed in (m + n) ways.
= a1 + a2 + a3 +, …, + ak

NOTE 3x 2 + 9x + 17
2. If x is real, the maximum value of is
3x 2 + 9x + 7
The above two principles can be extended for any finite 17 1
(A) (B) (C) 41 (D) 1
number of operations. 7 4
Solution: (C)
Illustration The word BAC cannot be spelt if the m selected
coupons do not contain atleast one of A, B and C.
Suppose there are 5 gates in a stadium, 2 on one side and 3
Number of ways of selecting m coupons which are
on the other. Sohan has to go out of the stadium. He can go
A or B = 2m.
out from any one of the 5 gates. Thus, the number of ways
This also includes the case when all the m coupons
in which he can go out is 5. Hence, the work of going out
are A or all are B.
through the gates on one side will be done in 2 ways and the
Number of ways of selecting m coupons which are
work of going out through the gates on other side will be
B or C = 2m.
done in 3 ways. The work of going out will be done when
This also includes the case when all the m coupons
the Sohan goes out from side I or side II. Thus, the work of
are B or all are C.
going out can be done in (2 + 3) = 5 ways.
Number of ways of selecting m coupons which are
C or A = 2m.
This also includes the case when all the m coupons
are C or all are A.
Number of ways of selecting m coupons when all
E are A = 1m.
D I
Number of ways of selecting m coupons when all
A eI are B = 1m.
Side B
C Sid
I Number of ways of selecting m coupons when all
Fig. 7.1 are C = 1m.
\ Required number = 2m + 2m + 2m – (1m + 1m + 1m)
NOTE = 3 ⋅ 2m – 3 ⋅ 1m = 3 (2m – 1)
Addition theorem of counting is also true for more than two 3. There are 4 candidates for the post of a lecturer in
works. Mathematics and one is to be selected by votes of
5 men. The number of ways in which the votes can be
given is
solved examPles (A) 1048 (B) 1024
(C) 1072 (D) None of these
1. In a certain test, ai students gave wrong answers to at Solution: (B)
least i questions where i = 1, 2, 3, …, k. No student Each man can vote for any one of the 4 candidates and
gave more than k wrong answers. The total number of this can be done in 4 ways.
wrong answers given is Similar is the case with every other man.
(A) a1 + a2 + … + ak (Q Repetition is allowed)
(B) a1 + a2 + … + ak – 1 Hence 5 men can vote in 45 i.e., 1024 ways.
(C) a1 + a2 + … + ak + 1
4. There are 10 lamps in a hall. Each one of them can
(D) None of these
be switched on independently. The number of ways in
which the hall can be illuminated is

Objective_Maths_JEE Main 2017_Ch 7.indd 2 01/01/2008 04:20:09


Permutations and Combinations  7.3

(A) 102 (B) 
18 8 × 7 × 6 × 5 × 4 × 3 × 2 ×1
=
(C) 210 (D) 
1023 2 ×1× 2 ×1× 3 × 2 ×1×1

Solution: (D) = 8 × 7 × 6 × 5 = 1680
Each bulb has two choices, either switched on or off 8. The number of ways in which the letters of the word
\ Required number of ways BALLOON can be arranged so that two L’s do not
= 210 – 1 = 1024 – 1 = 1023 come together, is
(A) 700 (B) 800
(Since in one way when all are switched off, the hall
(C)  900 (D)  None of these
will not be illuminated.)
Solution: (C)
5. A telegraph has 5 arms and each arm is capable of 4
distinct positions, including the position of rest. The There are in all seven letters in the word BALLOON in
total number of signals that can be made is which L occurs 2 times and O occurs 2 times.
\ The number of arrangements of the seven letters
(A) 473 (B) 1023 7!
(C)  1173 (D)  None of these of the word = = 1260.
2! x 2!
Solution: (B) If two L’s always come together, taking them as
Each arm can be set in 4 ways. one letter, we have to arrange 6 letters in which O
\ Five arms can be set in 4 × 4 × 4 × 4 × 4 ways. occurs 2 times.
But this includes the way when all the arms are in \ The no. of arrangements in which the two L’s
the position of rest, when no signal is sent. come together
Hence, required number of signals 6!
= = 6 × 5 × 4 × 3 = 360.
= 45 – 1 = 1024 – 1 = 1023 2!
Hence, the required no. of ways in which the two L’s
6. There are stalls for 10 animals in a ship. The number
do not come together = 1260 – 360 = 900.
of ways the shipload can be made if there are cows,
calves and horses to be transported, animals of each 9. A Letter lock contains 5 rings each marked with four
kind being not less than 10, is different letters. The number of all possible unsuccess-
(A) 59049 (B) 49049 ful attempts to open the lock is
(C)  69049 (D)  None of these (A) 625 (B) 1024 (C) 624 (C) 1023
Solution: (A) Solution: (C)
Each stall can be filled in 3 ways as there are three
types of animals (animals of each category being not 1 1 1 1 1
less than 10).
2 2 2 2 2
Shipload, i.e., filling up of 10 stalls, can be made in
3 3 3 3 3
3 × 3 × … up to 10 times = 310 = 59049
4 4 4 4 4
7. The number of ways in which two 10-paise, two
20-paise, three 25-paise and one 50-paise coins can be Number of options on 1st Ring = 4
distributed among 8 children so that each child gets Number of options on 2nd Ring = 4
only one coin, is Number of options on 3rd Ring = 4
(A) 1720 (B) 1680 Number of options on 4th Ring = 4
(C)  1570 (D)  None of these Number of options on 5th Ring = 4
\ Total number of options/arrangements
Solution: (B)
Total number of coins = 2 + 2 + 3 + 1 = 8. = 4 × 4 × 4 × 4 × 4 = 1024
2 coins are 10 paise, 2 are 20 paise, 3 are 25 paise 10. We have (n + 1) white balls and (n + 1) black balls. In
and 1 is of 50 paise. each set the balls are numbered from 1 to (n + 1). If
\ Required no. of ways these balls are to be arranged in a row so that two con-
8!
= secutive balls are of different colours, then the number
2 ! × 2 ! × 3! × 1!  of these arrangements is

Objective_Maths_JEE Main 2017_Ch 7.indd 3 01/01/2008 04:20:10


7.4 Chapter 7

(A) (2n + 2)! (B) 2 × (2n + 2) Hence, required number of ways


(C) 2 × (n + 1)! (D) 2 [(n + 1)!]2
= 24 × 4 × 120 = 11520
Solution: (A)
Between (n + 1) white balls there are (n + 2) gaps in 13. In a test ai students gave wrong answers to atleast
which (n + 1) black balls are to arranged. Number of i questions, where i = 1, 2, 3, 4, …, k. No one gave
required arrangements wrong answer to greater than k, then the total number
of given wrong answers is
= (n + 1)! (n + 1)! = [(n + 1)!]2
(A) a1 + 2a2 + 3a3 + … + kak
Now between (n + 1) black balls (n + 1) white balls are (B) a1 + a2 + a3 + … + ak
to be filled in number of ways = (n + 1)! (n + 1)! (C) Zero
\ Required number of ways = 2[(n + 1)!]2 (D) a1a2 + a2a3 + … + ak–1ak
Solution: (B)
11. Number of points having position vectoriˆ aiˆ + bjˆ + ckˆ,
Total number of wrong answers
where a, b, c ∈ {1, 2, 3, 4, 5} such that 2a + 3b + 5c is
divisible by 4 is = 1 (a1 – a2) + 2 (a2 – a3) +, …,
(A) 140 (B) 70 + (k – 1) (ak–1 – ak) + kak
(C) 100 (D) None of these
\ Required number of ways = a1 + a2 +, …, + ak
Solution: (A)

4m = 2a + 3b + 5c = 2a + (4 – 1)b + (4 + 1)c PermuTaTion


a b c
4m = 4k + 2 + (–1) + (1) Each of the different arrangements which can be made by
\ a = 1, b = even, c = any number taking some or all of given number of things or objects at a
a ≠ 1, b = odd, c = any number time is called a permutation.

\ Required number = 1 × 2 × 5 × 4 × 3 × 5 = 70
I M P O R TA N T P O I N T S
12. A family consists of grandfather, 5 sons and daughters
and 8 grandchildren. They are to be seated in a row for Permutation of things means arrangement of things. The
dinner. The grandchildren wish to occupy the 4 seats word arrangement is used if order of things is taken into
at each end and the grandfather refuses to have a grand account. Thus, if order of different things changes, then their
child on either side of him. The number of ways in arrangement also changes.
which the family can be made to sit is
(A) 11360 (B) 11520 Notations
(C) 21530 (D) None of these
Let r and n be positive integers such that 1 ≤ r ≤ n. Then,
Solution: (B) the number of permutations of n different things, taken r at
The total number of seats a time, is denoted by the symbol nPr or P (n, r).
= 1 grand father + 5 sons and daughters
key resulTs on PermuTaTions
+ 8 grand children
= 14 1. nPr =
n!
= n(n – 1) (n – 2) … [n – (r – 1)],
(n − r )!
The grand children wish to occupy the 4 seats on either
0 ≤ r ≤ n.
side of the table = 4! ways
2. Number of permutations of n different things taken all at
= 24 ways a time is: nPn = n!.
3. The number of permutations of n things, taken all at a
and grand father can occupy a seat in (5 – 1) ways =
time, out of which p are alike and are of one type, q are
4 ways
alike and are of second type and rest are all different is
(Since 4 gaps between 5 sons and daughters) n!
and the remaining seats can be occupied in 5! ways .
p! q!
= 120 ways (5 seats for sons and daughters)

Objective_Maths_JEE Main 2017_Ch 7.indd 4 01/01/2008 04:20:11


Permutations and Combinations  7.5

4. The number of permutations of n different things taken  All those numbers whose last three digit number is ­divisible
r at a time when each thing may be repeated any number by 8 are divisible by 8.
of times is nr.  All those numbers the sum of whose digits is divisible by
5. Permutations under Restrictions 9 are themselves divisible by 9.
(a) Number of permutations of n different things, taken  All those numbers whose last two digits are divisible by 25
r at a time, when a particular thing is to be always are themselves divisible by 25.
included in each arrangement, is
r ⋅ n – 1Pr – 1
(b) Number of permutations of n different things, taken Solved Examples
r at a time, when s particular things are to be always
included in each arrangement, is 14. The number of positive terms in the sequence
s! [r – (s – 1)] ⋅ n – sPr – s 195 n+3
P3
xn = − , n ∈ N is
(c) Number of permutations of n different things, taken n
4 Pn n +1
Pn + 1
r at a time, when a particular thing is never taken in
each arrangement, is (A) 2 (B) 3
n–1
Pr (C)  4 (D)  None of these
(d) Number of permutations of n different things, taken Solution: (C)
all at a time, when m specified things always come We have,
together, is m! × (n – m + 1)!. 195 n+3
p3
(e) Number of permutations of n different things, taken xn = n
− n +1
4 ⋅ pn pn + 1
all at a time, when m specified things never come 
together, is n! – m!× (n – m + 1)!. 195 ( n + 3) ( n + 2) ( n + 1)
6. Circular Permutations = −
4⋅ n! ( n + 1)! 
(a) Number of circular arrangements (permutations) of
195 ( n + 3) ( n + 2)
n different things is: (n – 1)!. = −
(b) Number of circular arrangements (permutations) of 4 ⋅ n! n! 
2
n different things when clockwise and anti-clockwise 195 − 4 n − 20 n − 24
arrangements are not different, i.e., when observa- =
4 ⋅ n! 
1
tion can be made from both sides is: (n − 1)! . 2
171 − 4 n − 20 n
2 =
(c) Number of circular permutations of n different things, 4 ⋅ n! 
taken r at a time, when clockwise, and anti-clockwise xn is positive.
n
P
orders are taken as different, is = r . 171 − 4 n2 − 20 n
r \ > 0
(d) Number of circular permutations of n different things,
4 ⋅ n!
taken r at a time, when clockwise and anti-clockwise ⇒ 4n2 + 20n – 171 < 0
n
P
orders are not different, is = r . which is true for n = 1, 2, 3, 4.
2r
Hence, the given sequence has 4 positive terms.
15. The number of ways in which the letters of the word
Trick(s) for Problem Solving “STRANGE” can be arranged so that the vowels may
appear in the odd places, is
Numbers Divisible by 2, 3, 4, 5, 8, 9, 25 (A) 1440
 All those numbers having their last digit as an even number
(B) 1470
(i.e., 0, 2, 4, 6 or 8) are divisible by 2. (C) 1370
 All those numbers the sum of whose digits is divisible by
(D)  None of these
3 are themselves divisible by 3.
 All those numbers whose last two digits are divisible by 4
Solution: (A)
are themselves divisible by 4. There are 5 consonants and 2 vowels in the word
 All those numbers whose last digit is either 0 or 5 are STRANGE. Out of 7 places for the 7 letters, 4 places
divisible by 5. are odd and 3 places are even.

Objective_Maths_JEE Main 2017_Ch 7.indd 5 01/01/2008 04:20:12


7.6  Chapter 7

Two vowels can be arranged in 4 odd places in P Each of the digits 3, 4, 5, 6 occurs in 3!
(4, 2) ways = 12 ways and then 5 consonants can be = 3 × 2 = 6 times in unit’s place.
arranged in the remaining 5 places in P(5, 5) ways
\ Sum of the digits in the unit’s place of all the
= 5 × 4 × 5 × 3 × 2 × 1 = 120 ways. numbers
Hence, the required number of ways = (3 + 4 + 5 + 6) × 6 = 18 × 6 = 108
= P (4, 2) × P (5, 5) = 12 × 120 = 1440 18. Three boys and three girls are to be seated around a
−p /2 table, in a circle. Among them, the boy X does not want
16. ∫ [( x + p )3 + cos 2 ( x + 3p )] dx is equal to any girl neighbour and the girl Y does not want any boy
neighbour. The number of such arrangements possible is
−3p / 2
(A) (p/4) – 1 (B)  p4/32 (A) 4 (B) 6
(C) (p4/32) + p/2 (D)  p/2 (C)  8 (D)  None of these

Solution: (A) Solution: (A)
Any number between 1 and 1000000 must be of less As shown in figure, 1, 2 and X are the three boys and
than seven digits. Therefore, it must be of the form 3, 4 and Y are three girls, Boy X will have neighbours
as boys 1 and 2 and the girl Y will have neighbors as
a1 a2 a3 a4 a5 a6 girls 3 and 4.
where a1, a2, a3, a4, a5, a6 ∈ {0, 1, 2, …, 9} 1 and 2 can be arranged in P (2, 2) ways
According to question, sum of the digits = 18 X

Thus, a1 + a2 + a3 + a4 + a5 + a6 = 18
1 2
where 0 ≤ ai ≤ 9, i = 1, 2, 3, …, 9.
Required number
= coefficient of x18 in (1 + x + x2 + … + x9)6
3 4
6
18
⎛ 1 − x 10 ⎞
= coefficient of x in ⎜ ⎟ Y
⎝ 1− x ⎠

= coeff. of x18 in [(1 – x10)6 (1 – x)– 6] = 2! = 2 × 1 = 2 ways.
18 6 10
= coeff. of x in [(1 – C1 x ) (1 – x) ] –6 Also, 3 and 4 can be arranged in P (2, 2) ways

(leaving terms containing powers of x greater than 18) = 2! = 2 × 1 = 2 ways.


Hence, required number of permutations
= coeff. of x18 in (1 – x)– 6 – 6C1.
= 2 × 2 = 4
coeff. of x8 in (1 – x)– 6
19. The number of divisors of 9600 including 1 and 9600 are
= 6 + 18 – 1C18 – 6 ⋅ 6 + 8 – 1C18 = 23C5 – 6 ⋅ 13C8
(A) 60 (B) 58 (C) 48 (D) 46
23 ⋅ 22 ⋅ 21⋅ 20 ⋅19 13 ⋅12 ⋅11⋅10 ⋅ 9
= −6 Solution: (C)
120 120 
= 33649 – 7722 = 25927 Q 9600 = 27 × 3 × 52
17. The sum of the digits in the unit place of all the num- \ No. of divisors = (7 + 1) × (1 + 1) × (2 + 1)
bers formed with the help of 3, 4, 5, 6 taken all at a = 8 × 2 × 3 = 48
time is
(A) 432 (B) 108 (C) 36 (D) 18 Trick(s) for Problem Solving
Solution: (B)
If N = p1a ⋅ p2a … pka , where p1, p2, …, pk are different primes
1 2 k

The total number of numbers that can be formed with


and a1, a2,…, ak are natural numbers, then the total number
the digits 3, 4, 5, 6 taken all at a time of divisors of N including, and N is (a1 + 1) (a2 + 1) …
= P (4, 4) = 4! = 24. (ak + 1).

Objective_Maths_JEE Main 2017_Ch 7.indd 6 01/01/2008 04:20:13


Permutations and Combinations 7.7

20. There are six teachers. Out of them two are primary Solution: (B)
teachers, two are middle teachers and two secondary Four lines intersect each other in 4C2 = 6 points and
teachers. They are to stand in a row, so as the primary 4 circles intersect in 4P2 = 12 points. Each line cuts 4
teachers, middle teachers and secondary teachers are circles into 8.
always in a set. The number of ways in which they can \ 4 lines cut four circles into 32 points.
do so, is \ Required number = 6 + 12 + 32 = 50.
(A) 34 (B) 48
23. There are n seats round a table numbered 1, 2, 3, …, n.
(C) 52 (D) None of these
The number of ways in which m(≤ n) persons can take
Solution: (B) seats is
There are 2 primary teachers. They can stand in a row (A) nPm (B) nCm × (m – 1)!
in P (2, 2) = 2! 1
(C) .n Pm (D) n –1Pm
= 2 × 1 ways = 2 ways 2
There are 2 middle teachers. They can stand in a row in Solution: (A)
P (2, 2) = 2! = 2 × 1 ways = 2 ways. Since the seats are numbered,
There are 2 secondary teachers. They can stand in \ the arrangement is not circular.
a row in P (2, 2) = 2! \ the required number of ways
= 2 × 1 ways = 2 ways = the number of arrangements of n things
These three sets can be arranged in themselves in taken m at a time
= 3! = 3 × 2 × 1 = 6 ways = nPm
Hence, the required number of ways
= 2 × 2 × 2 × 6 = 48 combinaTion
21. A teaparty is arranged for 16 people along two sides of Each of the different groups or selections which can be
a large table with 8 chairs on each side. Four men want made by taking some or all of a number of things (irrespec-
to sit on one particular side and two on the other side. tive of order) is called a combination.
The number of ways in which they can be seated is
6 ! 8! 10 ! 8! 8! 10 ! I M P O R TA N T P O I N T S
(A) (B)
4! 6! 4! 6! Combination of things means selection of things. Obviously,
8! 8! 6 ! in selection of things order of things has no importance.
(C) (D) None of these Thus, with the change of order of things selection of things
6! 4!
does not change.
Solution: (B)
There are 8 chairs on each side of the table. Let the
sides be represented by A and B. Let four persons sit notations
on side A, then number of ways of arranging 4 persons The number of combinations of n different things taken r at
on 8 chairs on side A = 8P4 and then two persons sit on a time is denoted by nCr or C(n, r). Thus,
side B. The number of ways of arranging 2 persons on n! n
Pr
n
8 chairs on side B = 8P2 and the remaining 10 persons Cr = = (0 ≤ r ≤ n)
can be arranged in remaining 10 chairs in 10! ways. r ! ( n − r )! r!
Hence the total number of ways in which the n ( n − 1) ( n − 2) … ( n − r + 1)
=
persons can be arranged r ( r − 1) ( r − 2) … 3 ⋅ 2 ⋅1
8! 8! 10 ! If r > n,
= 8P4 × 8P2 × 10! = n
4! 6! then Cr = 0.
22. The maximum number of points into which 4 circles
and 4 straight lines intersect is NOTE
(A) 26 (B) 50
Selecting things without any order in called combination and
(C) 56 (D) 72
arrangement of things in some order is called permutation.

Objective_Maths_JEE Main 2017_Ch 7.indd 7 01/01/2008 04:20:14


7.8  Chapter 7

Key Results on Combination (e) Number of selections of one or more things out of
n identical things = n.
1. nCr = nCn – r , 0 ≤ r ≤ n (f ) If out of (p + q + r + t) things, p are alike of one
2. nC0 = nCn = 1, nC1 = n kind, q are alike of second kind, r are alike of third
3. If nCx = nCy then either x = y or x + y = n. kind and t are different, then the total number of
selections is
4. nCr + nCr–1 = n + 1Cr , 1 ≤ r ≤ n
(p + 1) (q + 1) (r + 1) 2t – 1
5. r · nCr = n · n–1Cr–1
(g) The number of ways of selecting some or all out of
6. n · n–1Cr–1 = = (n – r + 1) nCr–1 p + q + r items where p are alike of one kind, q are
n
C n − r +1 alike of second kind and rest are alike of third kind
7. n r = ,1≤r≤n
C r −1 r is [(p + 1) (q + 1) (r + 1)] – 1.
8. If n is even then the greatest value of nCr is nCn/2.
Division into Groups
9. If n is odd then the greatest value of nCr is
n 1. (a) Number of ways of dividing m + n different things
C n +1  or  n C n −1
in two groups containing m and n things respec-
2 2
tively (m ≠ n) is
r decreasing numbers starting with n
10. nCr = m+n ( m + n)!
r increasing numbers starrting with 1 Cm =
m ! n!
n ( n − 1) ( n − 2) ....( n − r + 1)
= (b) Number of ways of dividing m + n + p differ-
1⋅ 2 ⋅ 3.....r
ent things in three groups containing m, n and p
11. nPr = r! nCr = n (n – 1) (n – 2) … (n – r + 1). ( m + n + p)!
12. nC0 + nC1 + nC2 + … + nCn = 2n. things respectively (m ≠ n ≠ p) is ,
m ! n! p !
13. nC0 + nC2 + nC4 + … = nC1 + nC3 + nC5 + … = 2n – 1. if the order of the groups is not important and
14. Number of combinations of n different things taken ( m + n + p )!
× 3! , if the order of the groups is
r at a time m ! n! p !
(a) when p particular things are always included = important.
n–p
Cr–p. (c) Number of ways of dividing 2m different things in
(b) when p particular things are never included = two groups, each containing m things and the order
n–p
Cr. ( 2n)!
of the groups is not important, is , of the
(c) when p particular things are not together in any 2 ! ( n !) 2
selection = nCr – n–pCr–p. ( m + n + p)!
groups is not important and × 3!, if
15. (a) Number of selections of r consecutive things out m ! n! p !
of n things in a row = n – r + 1. the order of the groups is important.
(b) Number of selections of r consecutive things out
(d) Number of ways of dividing 2m different things
of n things along a circle
in two groups, each containing m things and the
⎧n, when r < n ( 2n)!
⎨ order of the groups is important, is .
⎩1, when r = n ( n !) 2
16. (a) Number of selections of zero or more things out of (e) Number of ways of dividing 3m different things
n different things in three groups, each containing m things and the
n (3m)!
C0 + nC1 + nC2 + … + nCn = 2n order of the groups is not important, is .
3! ( m !)3
(b) 
Number of combinations of n different things (f ) Number of ways of dividing 3m diferent things
selecting at least one of them is in three groups, each containing m things and the
n
C1 + nC2 + … + nCn = 2n – 1 (3m)!
order of the groups is important, is .
(c) Number of selections of r things (r ≤ n) out of n ( m !)3
identical things is 1. 2. (a) Number of ways of dividing n identical things into
(d) Number of selections of zero or more things out of r groups, if blank groups are allowed is
n identical things = n + 1. n+r–1
Cr – 1

Objective_Maths_JEE Main 2017_Ch 7.indd 8 01/01/2008 04:20:16


Permutations and Combinations  7.9

(b) Number of ways of dividing n identical things Solution: (A)


into r groups, if blank groups are not allowed is We have,
n–1
Cr–1. 5
(c) Number of ways of dividing n identical things into
47
C4 + ∑ 52 − j
C3
r groups such that no group contains less than m j =1

things and more than k (m < k) things is coefficient = 47C4 + 51C3 + 50C3 + 49C3 + 48C3+ 47C3
of xn in the expansion of
= (47C4 + 47C3) + 48C3 + 49C3 + 50C3 + 51C3
(xm + xm + 1 + … + xk)r
= 48C4 + 48C3 + 49C3 + 50C3 + 51C3
3. The number of ways of selecting r things out of n
things of which p are alike and are of one kind, q are = 49C4 + 49C3 + 50C3 + 51C3 = 50C4 + 50C3 + 51C3
alike and are of second, s are alike and are of third kind
= 51C4 + 51C3 = 52C4
and so on, is
= coefficient of xr in
25. 15C8 + 15C9 – 15C6 – 15C7 =
[(1 + x + x2 + … + xp) (1 + x + x2 + … + xq) × (1 + x +
(A) 8 (B) 0
x2 + … + xs) …]
(C)  6 (D)  None of these
4. The number of ways of selecting r things out of n
things of which p are a like and are of one kind, q are Solution: (B)
alike and are of second kind and rest (n – p – q) things We have,
are all different is 15
C8 + 15C9 – 15C6 – 15C7
= coefficient of xr in
[(1 + x + x2 + … + xp) (1 + x + x2 + … + xq) = (15C8 + 15C9) – (15C6 + 15C7)
× (1 + x)n–p–q]
= 16C9 – 16C7(Q nCr + nCr + 1 = n + 1Cr + 1)
5. The number of ways of selecting r things out of n
things of which p are alike and are of one kind, q are = 16C9 – 16C9(Q nCr = nCn – r)
alike and are of second kind, s are alike and are of third = 0
kind when each thing is taken at least once
= coefficient of xr – 3 in ⎛ n⎞ ⎛ n ⎞ ⎛ n ⎞
26. For 2 ≤ r ≤ n, ⎜ ⎟ + 2 ⎜ + =
[(1 + x + x2 + … + xp–1) (1 + x + x2 + … + xq–1) × ⎝ r⎠ ⎝ r − 1⎟⎠ ⎜⎝ r − 2⎟⎠
(1 + x + x2 + … + xs–1) …]
⎛ n + 1⎞ ⎛ n + 1⎞
6. The number of ways in which r identical things can be (A)  ⎜ ⎟ (B) 
2⎜
distributed among n persons when each person can get ⎝ r − 1⎠ ⎝ r + 1⎟⎠
zero or more things ⎛ n + 2⎞ ⎛ n + 2⎞
= coefficient of xr in (1 + x + x2 + … + xr)n (C) 2 ⎜ ⎟ (D) 
⎜⎝ r ⎟⎠
⎝ r ⎠
= coefficient of xr in (1 – x)–n = n + r – 1Cr
Solution: (D)
7. The number of non-negative integral solutions of the
equation x1 + x2 + … + xr = n is n + r – 1Cr.
n
Cr + 2nCr–1 + nCr–2 = (nCr + nCr–1) + (nCr–1 + nCr–2)
8. The number of terms in the expansion of
r n+r–1
= n+1Cr + n+1Cr–1
(a1 + a2 + a3 + … + an) is Cr
⎛ n + 2⎞
= n+2Cr = ⎜
Solved Examples ⎝ r ⎟⎠

27. A gentleman invites 13 guests to a dinner and places
5
24. 47
C4 + ∑ 52− j C3 = 8 of them at one table and remaining 5 at the other,
the tables being round. The number of ways he can
j =1
arrange the guests is
(A) 52C4
11!
(B) 51C4 (A)  (B) 
9!
40
(C) 52C3
12 ! 13!
(D)  None of these (C)  (D) 
40 40

Objective_Maths_JEE Main 2017_Ch 7.indd 9 01/01/2008 04:20:18


7.10  Chapter 7

Solution: (D) p

The number of ways in which 13 guests may be divided


30. ∫ x f (sin x) dx is equal to
0
13! p /2
into groups of 8 and 5 = 13C5 = . p

5! 8!
Now, corresponding to one such group, the 8
(A)  p ∫ p f (cos x ) dx
f (cos x ) dx (B)  ∫
0 0

guests may be seated at one round table in (8 – 1)! i.e., p p /2


p
7! ways and the five guests at the other table in (5 – 1)! (C)  p ∫ f (sin x ) dx (D) 
2 ∫ f (sin x ) dx
i.e., 4! ways. 0 0

But each way of arranging the first group of 8 per- Solution: (C)


sons can be associated with each way of arranging the Let w be a non-negative integer such that
second group of 5, therefore, the two processes can be
performed together in 7! × 4! ways. 3x + y + z + w = 30
Hence required number of arrangements Let a = x – 1, b = y – 1, c = z – 1, d = w,
13! 13! 13! then
= × 7! × 4! = × 7! × 4! =
5! 8! 5 ⋅ 4 ! 8 ⋅ 7! 40 3a + b + c + d = 25, where a, b, c, d ≥ 0 (1)
28. If there are 12 persons in a party, and if each of them Clearly, 0 ≤ a ≤ 8. If a = k, then
shakes hands with each other, then number of hand-
b + c + d = 25 – 3. (2)
shakes happen in the party is
Number of non-negative integral solutions of Eq. (2)
(A) 66 (B) 48
(C)  72 (D)  None of these = n + r – 1Cr
Solution: (A) = 3 + 25 – 3k – 1C25 – 3k
Total number of handshakes
= 27 – 3kC25 – 3k = 27 – 3kC2
= The number of ways of selecting 2 persons
from among 12 persons (27 − 3k ) (26 − 3k )
=
12 x 11 2 
= 12C2 = = 66 3 2
2x 1 = (3k – 53k – 234)
2
29. The number of ways in which a committee of 5 can be
\ Required number
chosen from 10 candidates so as to exclude the young-
8
est if it includes the oldest, is 3
(A) 196 (B) 178
=
2k =0

(3k 2 − 53k + 234)
(C)  202 (D)  None of these 
3 ⎛ 8 × 9 × 17 8×9 ⎞
Solution: (A) = ⎜3 − 53 + 234 × 9⎟ = 1215
2⎝ 6 2 ⎠
There are two different ways of forming the committee
 (i) oldest may be included 31. The number of different ways in which 8 persons can
(ii) oldest may be excluded stand in a raw so that between two particular persons A
 (i)  If oldest is included, then youngest has to be and B there are always two persons, is
excluded and we are to select 4 candidates out (A)  60 (5!)
of 8. This can be done in (B) 4! × 5!
8 8! 8x 7x 6x 5 (C)  15 (4!) × 5!
C4 = = = 70 ways
4! 4! 4x 3x 2 (D)  None of these
(ii) If oldest is excluded, then we are to select 5 candi- Solution: (A)
dates from 9 which can be done in
The number of selections of 4 persons including A, B
9! 9x 8x 7x 6
9
C5 = = = 126 ways considering these four as a group, the number = 6C2 of
5! 4 ! 4x 3x 2x 1 arrangement with the other four = 5!.
 Hence, the total number of ways in which But in each group of four, number of arrange-
­committee can be formed ments  = 2! × 2! therefore, the required number of
= 126 + 70 = 196 ways = 6C2 × 5! × 2! × 2!.

Objective_Maths_JEE Main 2017_Ch 7.indd 10 01/01/2008 04:20:20


Permutations and Combinations  7.11

32. There are 10 points in a plane of which no three points containing 6 questions. He is not permitted to attempt
are coffinear but 4 points are concyclic. The number of more than 5 questions from each group. The number
different circles that can be drawn through atleast 3 of of ways in which he can choose the 7 questions is
these points is (A) 780 (B) 640
(A) 110 (B) 112 (C) 116 (D) 117 (C)  820 (D)  None of these
Solution: (D) Solution: (A)
Since a unique circle can be drawn through three A candidate can attempt 5 questions from group I and
points, therefore a selection of three points results in 2 from group II or 4 from group I and 3 from group II
a circle. So, the maximum number of circles using 10 or 3 from group I and 4 from group III or 2 from group
points is 10C3. Now, out of these 10 points 4 are con- I and 5 from group II. This can be done in
cyclic, hence 4C3 circles are actually single circle. 6
C5 × 6C2 + 6C4 × 6C3 + 6C3 × 6C4 + 6C2 × 6C5
\ Required number of circles = 10C3 – 4C3 + 1 = 117. = 6 × 15 + 15 × 20 + 20 × 15 + 15 × 6
33. The number of ways in which a committee of 3 ladies = 90 + 300 + 300 + 90 = 780
and 4 gentlemen can be appointed from a meeting con-
36. A boy has 3 Library tickets and 8 books of his inter-
sisting of 8 ladies and 7 gentlemen, if Mr X refuses to
est in the library. Out of these 8, he does not want to
serve in a committee if Mr Y is a member is
­borrow Chemistry part II, unless Chemistry part I is
(A) 1960 (B) 1540 also borrowed. The number of ways in which he can
(C)  3240 (D)  None of these choose the three books to be borrowed is
Solution: (B) (A) 41 (B) 32
3 ladies out of 8 can be selected in 8C3 ways and (C)  51 (D)  None of these
4 ­gentlemen out of 7 in 7C4 ways. Solution: (A)
Now each way of selecting 3 ladies is associated
The following are the different possibilities in which
with each way of selecting 4 gentlemen.
three books can be borrowed:
Hence, the required number of ways
  (i) When Chemistry part II is selected, then
= 8C3 × 7C4 = 56 × 35 = 1960 Chemistry part I is also borrowed and the third
book is selected from the remaining 6 books.
We now find the number of committees of 3 ladies and
(ii) When Chemistry part II is not selected, in this case
4 gentlemen in which both Mrs X and Mr Y are mem-
he has to select the three books from the remain-
bers. In this case, we can select 2 other ladies from the
ing 7 books.
remaining 7 in 7C2 ways and 3 other gentlemen from
First choice can be made in 6C1 = 6 ways.
the remaining 6 in 6C3 ways.
Second choice can be made in
\ The number of ways in which both Mrs X and
Mr Y are always included = 7C2 × 8C3 = 21 × 20 = 420. 7 7×6×5
C3 = = 35 ways
Hence, the required number of committes in which 1× 2 × 3
Mrs X and Mr Y do not serve together = 1960 – 420 = Total number of ways in which he can choose the
1540. three books to be borrowed = 6 + 35 = 41.
34. The number of ways in which a team of eleven players 37. The number of words that can be formed from the
can be selected from 22 players including 2 of them ­letters a, b, c, d, e, f, taken 3 at a time, each word
and excluding 4 of them is containing at least one vowel is
(A) 16C11 (B)  16
C5 (C)  16
C9 (D)  20
C9 (A) 96
(B) 84
Solution: (C)
(C) 106
Out of 22 players, 2 are to be included and 4 are to (D)  None of these
be excluded. We have to select a team of 11 players.
So the remaining 9 players are to be selected from the Solution: (A)
remaining 16 players. This can be done in 16C9 ways. The total number of words
35. A candidate is required to answer 7 questions out of = (2C1 × 4C2 + 2C2 × 4C1) 3!
12 questions which are divided into two groups each
= (12 + 4) × 6 = 96.

Objective_Maths_JEE Main 2017_Ch 7.indd 11 01/01/2008 04:20:20


7.12  Chapter 7

38. In an examination a candidate has to pass in each 41. The number of ways in which a mixed doubles game
of the papers to be successful. It the total number of in tennis can be arranged from 5 married couples, if no
ways to fail is 63, how many papers are there in the husband and wife play in the same game, is
examination? (A) 46 (B) 54
(A) 6 (B) 8 (C)  60 (D)  None of these
(C)  14 (D)  None of these
Solution: (C)
Solution: (A) Let the sides of the game be A and B. Given 5 married
Let the number of papers be n. couples, i.e., 5 husbands and 5 wives. Now, 2 husbands
\ Total number of ways to fail or pass for two sides A and B can be selected out of 5 = 5C2 =
n 10 ways.
C0 + nC1 + nC2 + … + nCn = 2n
After choosing the two husbands their wives are
But there is only one way to pass, i.e., when he fails to be excluded (since no husband and wife play in
in none. the same game). So we are to choose 2 wives out of
\ Total number of ways to fail = 2n – 1 remaining 5 – 2 = 3 wives, i.e., 3C2 = 3 ways.
\ From question, 2n – 1 = 63; Again two wives can interchange their sides A and
\ 2n = 64 = 26 B in 2! = 2 ways.
Therefore, the required number of ways = 10 × 3 ×
\ n = 6. 2 = 60.
39. The total number of selections from 4 boys and 3 girls 42. The number of seven letter words that can be formed
if each selection has to contain at least one boy is by using the letters of the word SUCCESS so that the
(A) 106 (B) 120 two C are together but no two S are together, is
(C)  240 (D)  None of these (A) 24 (B) 36
Solution: (B) (C)  54 (D)  None of these
Number of selections of at least one boy from 4 boys Solution: (A)
4 4 4 4 4
= C1 + C2 + C3 + C4 = 2 – 1 Considering CC as single object, U, CC, E can be
Number of selections of any number of girls from arranged in 3! ways
3 girls × U × CC × E ×
3 3 3 3 3
= C0 + C1 + C2 + C3 = 2 Now the three S are to be placed in the four available
places.
\ Required number of selections of at least one boy Hence required number of ways = 3! · 4C3 = 24.
from 4 boys and 3 girls = (24 – 1)23 = 15 × 8 = 120.
43. Four boys picked up 30 mangoes. The number of ways in
40. A boat is to be manned by eight men of whom 2 can which they can divide them if all mangoes be identical, is
only row on bow side and 1 can only row on stroke
(A) 5456 (B) 3456
side; the number of ways in which the crew can be
(C)  5462 (D)  None of these
arranged is
(A) 4360 (B) 5760 Solution: (A)
(C)  5930 (D)  None of these Clearly, 30 mangoes can be distributed among 4 boys
such that each boy can receive any number of mangoes.
Solution: (B)
Hence, total number of ways = 30 + 4 – 1C4 – 1 = 33C3
First we have to select 2 men for bow side and 3 for
stroke side. 33.32.31
= = 5456
\ The number of selections of the crew for two sides 1.2.3
= 5C2 × 3C3
Trick(s) for Problem Solving
For each selection, there are 4 persons each on both
sides who can be arranged in 4! × 4! ways. Number of ways of dividing n idential things into r groups, if
\ Required number of arrangements blank groups are allowed is
5x 4 n + r – 1
Cr – 1
= 5C2 × 3C3 × 4! × 4! = x 1 x 24 x 24 = 5760
2

Objective_Maths_JEE Main 2017_Ch 7.indd 12 01/01/2008 04:20:20


Permutations and Combinations  7.13

44. The number of ways of choosing 10 balls from infinite Solution: (B)


white, red, blue and green balls is Number of ways in which at least one question can be
(A) 70 (B) 84 (C) 286 (D) 86 selected out of 3 are
3
Solution: (C) C1 + 3C2 + 3C3 = 23 – 1 = 7
⎪⎧ Coefficient of x ⎫⎪
10
Number of ways in which at least one question can be
Required ways = ⎨ ⎬
2 4 selected out of 4 are
⎩⎪in (1 + x + x + ...) ⎭⎪
4
4 C1 + 4C2 + 4C3 + 4C4 = 24 – 1 = 15
⎛ 1 ⎞
= Coefficient of x10 in ⎜ \ Total number of ways = 7 × 15 = 105
⎝ 1 − x ⎟⎠

47. Given 5 different green dyes, 4 different blue dyes and
= Coefficient of x10 in (1 – x)–4
3 different red dyes, the number of combinations of
= Coefficient of x10 in dyes that can be chosen by taking atleast one green and
⎛ 5.4 2 4.5.6 3 ⎞ one blue dye is
⎜⎝1 + 4 x + 2 ! x + 3! x + …⎟⎠ (A) 248 (B) 120
(C) 3720 (D) 465
= Coefficient of x10 in (1 + 4C1x + 5C2x2 + 6C3x3
Solution: (A)
+ 7C4x4 + 8C5x5 + 9C6x6 + … 13C10x10)
Number of ways of selecting at least one green dye out
13.12.11 of 5 different green dyes
\ Required number of ways = 13C10 = = 286
3.2.1 5
C1 + 5C2 + 5C3 + 5C4 + 5C5 = 25 – 1
45. In a class tournament where the participants were to
play one game with another, two class players fell ill, Also, at least one blue dye can be selected out of 4
having played 3 games each. If the total number of blue dyes in
games played is 84, the number of participants at the 4
C1 + 4C2 + 4C3 + 4C4 = 24 – 1
beginning was
(A) 22 (B) 15 Again, 3 different red dyes can be selected in
(C)  17 (D)  None of these 3
C0 + 3C1 + 3C2 + 3C3 = 23 ways
Solution: (B) \ Required ways = (25 – 1) (24 – 1) (23) = 3720
Suppose the two players did not play at all so that the
48. Out of 18 points in a plane no three are in the same
remaining (n – 2) players played n – 2C2 matches. Since
straight line except five points which are collinear. The
these two players played 3 matches each, hence the
number of straight lines that can be formed joining
total number of matches is
them is
n–1
C2 + 3 + 3 = 84  (given) (A) 143 (B) 144
( n − 2)( n − 3) (C)  153 (D)  None of these
or = 78 or n2 – 5n + 6 = 156
1.2 Solution: (B)
or n2 – 5n – 150 = 0 or (n – 15) (n + 10) = 0 The number of st. lines
\ n = 15 (n ≠ –10) = 18C2 – (5C2 – 1) = 144
46. An examination paper, which is divided into two
groups consisting of 3 and 4 questions respectively Derangement
carries the note: It is not necessary to answer all the
questions. One question atleast should be answered Rearrangement of objects such that no one goes to its
from each group. The number of ways can an exam- ­original place is called derangement.
inee select the questions is If ‘n’ things are arranged in a row, the number of
(A) 22 ways in which they can be deranged so that none of them
(B) 105 occupies its original place is
(C) 3P3 × 4P4 ⎛ 1 1 1 1⎞ n
1
n ! ⎜1 − + − + … + ( −1) n ⎟ = n ! ∑ ( −1) r
(D) 3C3 × 4C4 ⎝ 1! 2 ! 3! n! ⎠ r =0 r!

Objective_Maths_JEE Main 2017_Ch 7.indd 13 01/01/2008 04:20:21


7.14  Chapter 7

For example, if 4 letters are taken out of 4 different


­envelopes, then the number of ways in which they can be Solved Examples
reinserted in the envelopes so that no letter goes in to its
original envelope 49. The number of diagonals in a polygon of n sides is
⎛ 1 1 1⎞ n ( n − 3) n ( n − 1)
= 4 ! ⎜1 − 1 + − + ⎟ (A)  (B) 
⎝ 2 ! 3! 4 !⎠  2 2
( n − 1) ( n − 2)
⎛ 1 1 1⎞ (C)  (D)  None of these
= 24 ⎜1 − 1 + − + ⎟ = 9 2
⎝ 2 6 24 ⎠
Solution: (A)
Some Useful Results for The number of diagonals + number of sides = number
Geometrical Problems of selections of two vertices from n vertices
1. If n distinct points are given in the plane such that no \ the number of diagonals. = nC2 – n
three of which are collinear, then the number of line n ( n − 1) n 2 − n − 2n n ( n − 3)
segments formed = nC2. = −n = = .
2 2 2
If m of these points are collinear (m ≥ 3), then the
50. If each of 10 points on a straight line be joined to each
­number of line segments is (nC2 – mC2) + 1.
of 10 points on a parallel line then the total number of
2. The number of diagonals in an n-sided closed triangles that can be formed with the given points as
­polygon = nC2 – n. vertices, is
3. If n distinct points are given in the plane such that no (A) 860 (B) 900
three of which are collinear, then the number of trian- (C)  920 (D)  None of these
gles formed = nC3.
If m of these points are collinear (m ≥ 3), then the num- Solution: (B)
ber of triangles formed = nC3 – mC3. A triangle is formed for each selection of 2 points
4. If n distinct points are given on the circumference of a from one line and 1 point from the other line.
circle, then \ The number of triangles
(a) Number of st. lines = nC2 = 10C2 × 10C1 + 10C1 × 10C2
(b) Number of triangles = nC3
10 × 9 10 × 9
(c) Number of quadrilaterals = nC4 and so on = × 10 + 10 × = 900
2 2
5. Number of Rectangles and Squares:
(a) Number of rectangles of any size in a square of Exponent of prime p in n!
n
size n × n is ∑ r 3 and number of squares of any Exponent of a prime p in n! is denoted by Ep (n!) and is
r =1
n given by,
size is ∑ r 2. ⎛ n⎞ ⎛ n ⎞ ⎛ n ⎞ ⎛ n⎞
r =1 Ep (n!) = ⎜ ⎟ + ⎜ 2 ⎟ + ⎜ 3 ⎟ + .... + ⎜ k ⎟ ,
(b) Number of rectangles of any size in a rectangle of ⎝ p⎠ ⎝ p ⎠ ⎝ p ⎠ ⎝p ⎠
np
size n × p (n < p) is (n + 1) (p + 1) and number where pk < n < pk + 1
4 n
of squares of any size is ∑ ( n + 1 − r ) ( p + 1 − r ) . ⎛ n⎞
r =1
and  ⎜⎝ p ⎟⎠ denotes the greatest integer less than or
6. If m parallel lines in a plane are intersected by a family n
equal to .
of other n parallel lines. Then, total number of parallel- p
ograms so formed is For example, exponent of 3 in (100)! is
mC × nC
⎛ 100 ⎞ ⎛ 100 ⎞ ⎛ 100 ⎞ ⎛ 100 ⎞
E3 (100!) = ⎜ + + +
2 2

mn ( m − 1) ( n − 1) ⎝ 3 ⎟⎠ ⎜⎝ 32 ⎟⎠ ⎜⎝ 33 ⎟⎠ ⎜⎝ 34 ⎟⎠

i.e., 
4  = 33 + 11 + 3 + 1 = 48

Objective_Maths_JEE Main 2017_Ch 7.indd 14 01/01/2008 04:20:24


Permutations and Combinations  7.15

Number of Divisors 5. The number of ways in which N can be resolved as a


a a a a
product of two factors is
Let N = p11 . p 22 . p33 ... p k k . where p1, p2, p3, … pk are dif-
⎧1
ferent primes and a1, a2, a3, …, ak are natural numbers then: ⎪⎪ 2 (α1 + 1)(α 2 + 1)… (α k + 1), If N is not a perfect square
1. The total number of divisors of N including 1 and N is ⎨
⎪ 1 [(α + 1)(α + 1)… (α + 1) + 1], If N is a perfect square
= (a1 + 1) (a2 + 1) (a3 + 1) … (ak + 1). ⎪⎩ 2 1 2 k
2. The total number of divisors of N excluding 1 and N is
= (a1 + 1) (a2 + 1) (a3 + 1) … (ak + 1) – 2. 6. The number of ways in which a composite number N
3. The total number of divisors of N excluding 1 or N is can be resolved into two factors which are relatively
= (a1 + 1) (a2 + 1) (a3 + 1) … (ak + 1) – 1. prime (or co-prime) to each other is equal to 2n–1,
4. The susm of these divisors is where n is the number of different factors in N.
= ( p10 + p11 + p12 + ... + p1a )
a2
( p20 + p12 + p22 + ... + p2 )
a
( p k0 + p1k + p k2 + ... + p k k )

EXERCISES

Single Option Correct Type

1. Let y be an element of the set A = {1, 2, 3, 5, 6, 10, 15, 6. The number of ways of selecting 10 balls from the
30} and x1, x2, x3 be integers such that x1x2x3 = y, then unlimited number of red, green, white and yellow
the number of positive integral solutions of x1x2x3 = y is balls, if selection must include 2 red and 3 yellow
(A) 64 (B) 27 balls, is
(C)  81 (D)  None of these (A) 36 (B) 56
p (C)  112 (D)  None of these
2. If m = number of distinct rational numbers ∈
q 7. Let A = {1, 2, 3, 4} and B = {1, 2}. Then, the number
(0, 1) such that p, q ∈ {1, 2, 3, 4, 5} and n = number of onto functions from A to B is:
of mappings from {1, 2, 3} onto {1, 2}, then m – n is (A) 8 (B) 14
(A) 1 (B) –1 (C)  12 (D)  None of these
(C)  0 (D)  None of these
8. Given five line segments of lengths 2, 3, 4, 5, 6 units.
3. The letters of the word RANDOM are written in all Then the number of triangles that can be formed by
possible orders and these words are written out as in a joining these lines is
dictionary then the rank of the word RANDOM is (A) 5C3 – 3 (B)  5C3 – 1
(A) 614 (B) 615 (C) 613 (D) 616 5
(C)  C3 (D)  5
C3 – 2
4. If eight persons are to address a meeting then the num- 9. If a represents the number of permutations of (x + 2)
ber of ways in which a specified speaker is to speak things taken together, b represents the number of per-
before another specified speaker, is mutations of 11 things taken together out of x things,
(A) 40320 (B) 2520 and c represents the number of permutations of (x – 11)
(C)  20160 (D)  None of these things taken together so that a = 182bc, then x =
5. The number of permutations of letters a, b, c, d, e, f, g (A) 15 (B) 12 (C) 10 (D) 18
so that neither the pattern beg nor cad appears is 10. How many different nine digit numbers can be formed
7! 7! from the number 22 33 55 8 88 by rearranging its dig-
(A)  (B)  its so that the odd digits occupy even positions?
3! 3! 2 ! 3! 3!
(C)  4806 (D)  None of these (A) 16 (B) 36 (C) 60 (D) 180

Objective_Maths_JEE Main 2017_Ch 7.indd 15 01/01/2008 04:20:25


7.16  Chapter 7

11. For a game in which two partners play against two (A) 70 (B) 72
other partners, six persons are available. If every pos- (C)  71 (D)  None of these
sible pair must play with every other possible pair,
21. The number of positive integral solutions of 15 < x1 +
then the total number of games played is
x2 + x3 ≤ 20, is equal to
(A) 90 (B) 45 (C) 30 (D) 60
(A) 785 (B) 685
12. A five digit number divisible by 3 is to be formed using (C)  1150 (D)  None of these
the numerals 0, 1, 2, 3, 4 and 5 without repetition. The
22. The number of different 7 digit numbers that can be
total number of ways this can be done is
written using only the three digits 1, 2 and 3 with the
(A) 216 (B) 600 (C) 240 (D) 3125 condition that the digit 2 occurs twice in each number is
13. A box contains two white balls, three black balls and (A) 7P225 (B)  7
C2 25
four red balls. The number of ways in which three balls (C) 7C252 (D)  None of these
can be drawn from the box if atleast one black ball is to
be included in the draw, is 23. The tensdigit of 1! + 2! + 3! + … + 49! is
(A) 32 (B) 64 (A) 1 (B) 2 (C) 3 (D) 4
(C)  128 (D)  None of these 24. Let S be the set of all functions from the set A to the set
14. The sum of all the numbers that can be formed with A. If n (A) = k then n (S ) is
the digits 2, 3, 4, 5 taken all at a time is (A) k! (B)  kk (C) 2k – 1 (d) 2k
(A) 66666 (B) 84844 25. There are three coplanar parallel lines. If any p points
(C)  93324 (D)  None of these are taken on each of the lines, the maximum umber of
15. If the number of ways in which n different things can triangles with vertices at these points is
be distributed among n persons so that at least one per- (A) 3pC3 (B)  p2 (p – 1)
son does not get any thing is 232. Then n is equal to (C) p2 (4p – 1) (D)  p2 (4p – 3)
(A) 3 (B) 4
(C)  5 (D)  None of these 26. The number of ways in which thirty five apples can
be distributed among 3 boys so that each can have any
16. Every body in a room shakes hands with every body number of apples, is
else. The total number of hand shakes is 66. The total (A) 1332 (B) 666
number of persons in the room is (C)  333 (D)  None of these
(A) 11 (B) 12 (C) 13 (D) 14
n 27. The number of non-negative solutions of x1 + x2 + x3
17. m
Cr + 1 + = ∑ k
Cr = +, …, + xn ≤ n (where n is positive integer) is
k =m
n +1
(A) 2nCn – 1 (B)  2n–1Cn – 1
(A) nCr + 1 (B)  Cr + 1 2n+1 2n–1
n
(C)  Cn – 1 (D)  Cn–1 – 1
(C)  Cr (D)  None of these
28. Eleven animals of a circus have to be placed in eleven
18. Two straight lines intersect at a point O. Points A1, cages one in each cage. If 4 of the cages are too small
A2, …, An are taken on one line and points B1, B2, …, for 6 of the animals, then the number of ways of cag-
Bn on the other. If the point O is not to be used, the ing the animals is
number of triangles that can be drawn using these (A) 304800 (B) 504800
points as vertices, is (C)  604800 (D)  None of these
(A) n (n – 1) (B)  n (n – 1)2
2
(C) n (n – 1) (D)  n2 (n – 1)2 29. If n is even and
n
C0 < nC1 < nC2 < … < nCr > nCr+1 > … > nCn
19. If the letters of the word MOTHER are written in all
possible orders and these words are written out as in a then r =
dictionary, then the rank of the word MOTHER is n n −1
(A)  (B) 
(A) 240 (B) 261 (C) 308 (D) 309 2 2
n−2 n+2
20. The number of divisors a number 38808 can have, (C)  (C) 
excluding 1 and the number itself is 2 2

Objective_Maths_JEE Main 2017_Ch 7.indd 16 01/01/2008 04:20:26


Permutations and Combinations  7.17

30. In a network of railways, a small island has 15 stations. 39. The number of two digit numbers which are of the
The number of different types of tickets to be printed form xy with y < x are given by
for each class, if every station must have tickets for (A) 45 (B) 55
other station, is (C)  17 (D)  None of these
(A) 230 (B) 210
40. A crocodile is known to have not more than 68 teeth.
(C)  340 (D)  None of these
The total number of crocodiles with different set of
31. The number of ordered pairs (m, n), m, n ∈ {1, 2,…, teeth is
50} such that 6n + 9m is a multiple of 5 is (A) 68 (B) 68! (C) 1617 (D) 6868
(A) 6250 (B) 1250
41. For x ∈ R, let [x] denotes the greatest integer ≤ x, then
(C)  1875 (D)  None of these
the value of
32. A set contains (2n + 1) elements. The number of sub- ⎛ 1⎞ ⎛ 1 1 ⎞ ⎛ 1 2 ⎞
sets of the set which contain at most n elements is ⎜⎝ 3 ⎟⎠ + ⎜⎝ − 3 − 100 ⎟⎠ + ⎜⎝ − 3 − 100 ⎟⎠

(A) 2n (B)  2n+1
2n–1
(C) 2 (D)  22n ⎛ 1 99 ⎞
+, ..., + ⎜ − − is
⎝ 3 100 ⎟⎠
33. There are n concurrent lines and another line parallel
to one of them. The number of different triangles that (A) –100 (B) – 123 (C) –135 (D) –153
will be formed by the (n + 1) lines, is
42. The total number of ways in which a beggar can be
( n −1) n ( n − 1) ( n − 2) given at least one rupee from four 25 p. coins, three
(A)  (B) 
2 2 50 p. coins and 2 one rupee coins is
n( n +1) ( n + 1) ( n + 2) (A) 54 (B) 53 (C) 51 (D) 48
(C)  (D) 
2 2 43. A student is allowed to select atmost n books from a
34. An n-digit number is a positive number with exactly collection of (2n + 1) books. If the total number of
n digits. Nine hundred distinct n-digit numbers are to ways in which he can select books is 63, then n =
be formed using only the three digits 2, 5 and 7. The (A) 4 (B) 3 (C) 7 (D) 8
smallest value of n for which this is possible is
44. How many different nine digit numbers can be formed
(A) 6 (B) 7 (C) 8 (D) 9 from the number 223355888 by rearranging its digits
35. If all permutations of the letters of the word AGAIN are so that the odd digits occupy even positions?
arranged as in dictionary, the forty ninth word is (A) 16 (B) 36 (C) 60 (D) 180
(A) NAAGI (B)  NAGAI 45. In a certain test there are n questions. In this test 2k
(C)  NAAIG (D) NAIAG ­students gave wrong answers to at least (n – k) ques-
36. The number of ways of choosing n objects out of tions, where k = 0, 1, 2,…, n. If the total number of
(3n + 1) objects of which n are identical and (2n + 1) wrong answers is 4095, then value of n is
are distinct, is (A) 11 (B) 12 (C) 13 (D) 15
(A) 22n (B)  22n+1 46. The number of permutations of the letters a, b, c, d
2n
(C) 2 – 1 (D)  None of these such that b does not follow a, c does not follow b, and
37. In a group of boys, two boys are brothers and in this d does not follow c, is
group 6 more boys are there. In how many ways they (A) 12 (B) 14 (C) 13 (D) 11
can sit if the brothers are not to sit along with each m
other 47. If S = ∑ n + r C k , then
(A) 4820 (B) 1410 r=0
(C)  2830 (D)  None of these (A) S + nCk+1 = n+mCk+1
38. If 20% of three subsets (i.e., subsets containing exactly (B) S + nCk+1 = n+m+1Ck+1
three elements) of the set A = {a1, a2,…, an} contain (C) S + nCk = n+mCk
a1, then the value of n is
(D)  None of these
(A) 15 (B) 16 (C) 17 (C) 18

Objective_Maths_JEE Main 2017_Ch 7.indd 17 01/01/2008 04:20:27


7.18  Chapter 7

48. The number of ways of dividing 15 men and 15 55. From 6 different novels and 3 different d­ ictionaries,
women into 15 couples, each consisting of a man and 4  novels and 1 dictionary are to be selected and
a woman, is arranged in a row on the shelf so that the dictionary
(A) 1240 (B) 1840 is always in the middle. Then the number of such
(C) 1820 (D) 2005 arrangements is
(A)  less than 500
49. Suman writes letters to his five friends. The number of
(B)  at least 500 but less than 750
ways can be letters be placed in the envelopes so that
(C)  at least 750 but less than 1000
atleast two of them are in the wrong envelopes are
(D)  at least 1000
(A) 119 (B) 120
(C)  125 (D)  None of these 56. One ticket is selected at random from 50 tickets num-
bered 00, 01, 02, …, 49. Then the probability that the
50. Statement 1: The number of ways of distributing 10 sum of the digits on the selected ticket is 8, given that
identical balls in 4 distinct boxes such that no box is the product of these digits is zero, equals
empty is 9C3.
1 1 5 1
Statement 2: The number of ways of choosing any 3 (A)  (B)  (C)  (D) 
14 7 14 50
places from 9 different places is 9C3.
(A)  Statement 1 is false, Statement 2 is true 57. In a certain test, ai students gave wrong answers to at
(B) Statement 1 is true, Statement 2, is true; Statement 2 least i questions where i = 1, 2, 3, …, k. No student
is the correct explanation for Statement 1 gave more than k wrong answers. The total number of
(C) Statement 1 is true; Statement 2 is true; Statement 2 wrong answers given is
is not as correct explanation for Statement 1 (A) a1 + a2 + … + ak
(D)  Statement 1 is true, Statement 2 is false (B) a1 + a2 + … + ak – 1
(C) a1 + a2 + … + ak + 1
51. Assuming the balls to be identical except for differ- (D)  None of these

ence in colours, the number of ways in which one or
more balls can be selected from 10 white, 9 green and 58. A gentleman invites 13 guests to a dinner and places
7 black balls is 8 of them at one table and remaining 5 at the other,
(A) 880 (B) 629 (C) 630 (D) 879 the tables being round. The number of ways he can
arrange the guests is
52. The number of 4-digit numbers with distinct digits is
11! 12 ! 13!
(A) 504 (B) 4536 (A)  (B) 9! (C)  (D) 
40 40 40
(C) 4634 (D) 5040
59. There are stalls for 10 animals in a ship. The number
53. In a shop there are five types of ice-creams available. of ways the shipload can be made if there are cows,
A child buys six ice-creams. calves and horses to be transported, animals of each
Statement 1: The number of different ways the child kind being not less than 10, is
can buy the six ice-creams is 10C5. (A) 59049 (B) 49049
Statement 2: The number of different ways the child (C)  69049 (D)  None of these
can buy the six ice-creams is equal to the number of
different ways of arranging 6 A’s and 4 B’s in a row. 60. In an examination a candidate has to pass in each of the
(A) Statement 1 is false, Statement 2 is true papers to be successful. If the total number of ways to
(B) Statement 1 is true, Statement 2 is true, Statement 2 fail is 63, how many papers are there in the examination?
is a correct explanation for Statement 1 (A) 6 (B) 8
(C) Statement 1 is true, Statement 2 is true; Statement 2 (C)  14 (D)  None of these
is not a correct explanation for Statement 1 61. If A denotes the property that two elements of A = {1,
(D)  Statement 1 is true, Statement 2 is false 5, 9, 13 …, 1093} add up to 1094, then the maximum
54. How many different words can be formed by jumbling number of elements in A can be
the letters in the word MISSISSIPPI in which no two S (A) 126 (B) 136 (C) 137 (D) 138
are adjacent?
(A) 8 ⋅ 6C4 ⋅ 7C4 (B) 6 ⋅ 7 ⋅ 8C4
62. ∑ ∑ 10
C j j C i is equal to
0 ≤ i ≤ j ≤ 10
(C) 6 ⋅ 8 ⋅ 7C4 (D) 7 ⋅ 6C4 ⋅ 8C4 (A) 310 (B) 310 – 1 (C)  210 (D) 210 – 1

Objective_Maths_JEE Main 2017_Ch 7.indd 18 01/01/2008 04:20:27


Permutations and Combinations  7.19

63. If eight persons are to address a meeting then the (A) NAAGI (B) NAGAI
­number of ways in which a specified speaker is to (C) NAAIG (D) NAIAG
speak before another specified speaker, is
73. The number of ways of choosing n objects out of
(A) 40320 (B) 2520 (3n + 1) objects of which n are identical and (2n + 1)
(C)  20160 (D)  None of these are distinct, is
64. The number of permutations of letters a, b, c, d, e, f, g (A) 22n (B)  22n+1
so that neither the pattern beg nor cad appears is (C) 22n – 1 (D)  None of these
7! 7!
(A)  (B)  74. If 20% of three subsets (i.e., subsets containing
3! 3! 2 ! 3! 3! exactly three elements) of the set A = {a1, a2, …, an}
(C)  4806 (D)  None of these contain a1, then the value of n is
65. The sum of all the numbers that can be formed with (A) 15 (B) 16 (C) 17 (C) 18
the digits 2, 3, 4, 5 taken all at a time is
75. For x ∈ R, let [x] denotes the greatest integer ≤ x, then
(A) 66666 (B) 84844 the value of
(C)  93324 (D)  None of these
⎡ 1⎤ ⎡ 1 1 ⎤ ⎡ 1 2 ⎤
66. Two straight lines intersect at a point O. Points A1, ⎢ 3 ⎥ + ⎢ − 3 − 100 ⎥ + ⎢ − 3 − 100 ⎥

⎣ ⎦ ⎣ ⎦ ⎣ ⎦
A2, …, An are taken on one line and points B1, B2, …,
Bn on the other. If the point O is not to be used, the ⎡ 1 99 ⎤
+ ... + ⎢ − − ⎥ is
number of triangles that can be drawn using these ⎣ 3 100 ⎦
points as vertices, is
(A) –100 (B) –123 (C) –135 (D) –153
(A) n (n – 1) (B)  n (n – 1)2
(C) n2 (n – 1) (D)  n2 (n – 1)2 76. The total number of ways in which a beggar can be
given at least one rupee from four 25 p. coins, three
67. If the letters of the word MOTHER are written in all 50 p. coins and 2 one rupee coins is
possible orders and these words are written out as in a (A) 54 (B) 53 (C) 51 (D) 48
dictionary, then the rank of the word MOTHER is
(A) 240 (B) 261 (C) 308 (D) 309 77. In a certain test there are n questions. In this test 2k stu-
dents gave wrong answers to at least (n – k) questions,
68. The number of divisors a number 38808 can have, where k = 0, 1, 2, …, n. If the total number of wrong
excluding 1 and the number itself is answers is 4095, then value of n is
(A) 70 (B) 72 (A) 11 (B) 12 (C) 13 (D) 15
(C)  71 (D)  None of these
78. The number of permutations of the letters a, b, c, d
69. The number of different 7 digit numbers that can be such that b does not follow a, c does not follow b, and
written using only the three digits 1, 2 and 3 with the d does not follow c, is
condition that the digit 2 occurs twice in each number is (A) 12 (B) 14 (C) 13 (D) 11
(A) 7P222 (B)  7
C2 25 m
(C) 7C255 (D)  None of these 79. If S = ∑ n + r C k , then
r=0
70. There are 10 points in a plane of which no three points (A) S + nCk+1 = n+mCk+1
are collinear and 4 points are concyclic. The number
of different circles that can be drawn through at least 3 (B) S + nCk+1 = n+m+1Ck+1
of these points is (C) S + nCk = n+mCk
(A) 116 (B) 120 (D)  None of these
(C)  117 (D)  None of these
80. If the number of ways in which n different things can
71. A set contains (2n + 1) elements. The number of sub- be distributed among n persons so that at least one per-
sets of the set which contains at most n elements is son does not get any thing is 232. Then, n is equal to
(A) 2n (B) 2n+1 (C) 22n–1 (D) 22n (A) 3 (B) 4
(C)  5 (D)  None of these
72. If all permutations of the letters of the word AGAIN are
arranged as in dictionary, the forty-ninth word is

Objective_Maths_JEE Main 2017_Ch 7.indd 19 01/01/2008 04:20:28


7.20  Chapter 7

p 90. The sum of five digit numbers which can be formed


81. If m = number of distinct rational numbers∈ (0, 1)
q with the digits 3, 4, 5, 6, 7 using each digit only once
such that p, q ∈ {1, 2, 3, 4, 5} and n = number of in each arrangement, is
mappings from {1, 2, 3} onto {1, 2}, then m – n is (A) 5666600 (B) 6666600
(A) 1 (B) –1 (C)  7666600 (D)  None of these
(C)  0 (D)  None of these
91. The sum of all the numbers that can be formed by
82. Let y be an element of the set A = {1, 2, 3, 5, 6, 10, 15, ­writing all the digits 3, 2, 3, 4 only once is
30} and x1, x2, x3 be integers such that x1x2x3 = y, then (A) 39996 (B) 49996
the number of positive integral solutions of x1x2x3 = (C)  57776 (D)  None of these
y is
92. The sum of all numbers greater than 10000 formed by
(A) 64 (B) 27
using the digits 1, 3, 5, 7, 9, no digit being repeated in
(C)  81 (D)  None of these
any number, is
83. Given 5 different green dyes, 4 different blue dyes and (A) 4666600 (B) 5666600
3 different red dyes, the number of combinations of (C)  6666600 (D)  None of these
dyes that can be chosen by taking at least one green
and one blue dye is 93. The sum of all numbers greater than 1000 formed by
using the digits 0, 1, 2, 3, no digit being repeated in
(A) 248 (B) 120 (C) 3720 (D) 465
any number, is
84. Number of points having position vector aiˆ + bjˆ + ckˆ, (A) 38664 (B) 48664
where a, b, c ∈ {1, 2, 3, 4, 5} such that 2a + 3b + 5c is (C)  58664 (D)  None of these
divisible by 4 is
94. The number of four digit numbers that can be formed
(A) 140 (B) 70 from the digits 0, 1, 2, 3, 4, 5 with at least one digit
(C)  100 (D)  None of these repeated is
2n
85. Cr (0 ≤ r ≤ 2n) is greatest when r is equal to (A) 420 (B) 560
n n +1 (C)  780 (D)  None of these
(A)  (B) 
2 2 95. The number of odd numbers lying between 40000 and
(C) r = n (D)  None of these 70000 that can be made from the digits 0, 1, 2, 4, 5, 7
86. The number of even numbers greater than 100 that if digits can be repeated in the same number is
can be formed by the digits 0, 1, 2, 3 (no digit being (A) 864 (B) 932
repeated) is (C)  766 (D)  None of these
(A) 20 (B) 30 96. A table has provision for 7 seats, 4 being on one side
(C)  40 (D)  None of these facing the window and 3 being on the opposite side.
87. The number of positive numbers less than 1000 and The number of ways in which 7 people can be seated
divisible by 5 (no digit being repeated) is at the table if 2 people, X and Y, must sit on the same
side, is
(A) 150 (B) 154
(C)  166 (D)  None of these (A) 3260 (B) 2160
(C)  3350 (D)  None of these
88. In a certain city, all telephone numbers have six digits,
the first two digits always being 41 or 42 or 46 or 62 97. There are four oranges, five apples and six mangoes in
or 64. The number of telephone numbers having all six a fruit basket. The number of ways in which a person
digits distinct is can make a selection of fruits among the fruits in the
basket, is
(A) 8400 (B) 7200
(C)  9200 (D)  None of these (A) 210 (B) 330
(C)  209 (D)  None of these
89. The total number of ways of selecting five letters from
the letters of the word INDEPENDENT, is 98. The number of zeros at the end of 100! is
(A) 4200 (B) 3320 (A) 36 (B) 18
(C)  3840 (D)  None of these (C)  24 (D)  None of these

Objective_Maths_JEE Main 2017_Ch 7.indd 20 01/01/2008 04:20:29


Permutations and Combinations  7.21

  99. The largest integer n such that 33! is divisible by 2n is (A) 26 (B) 28
(A) 30 (B) 31 (C)  18 (D)  None of these
(C)  32 (D)  None of these 109. The number of ways in which 16 identical things can
100. The number of non-negative integral solutions of x1 + be distributed among 4 persons if each person gets at
x2 + x3 + 4x4 = 20 is least 3 things, is
(A) 436 (B) 536 (A) 33 (B) 35
(C)  602 (D)  None of these (C)  38 (D)  None of these

101. The product of r consecutive positive integers is 110. The number of ways in which 30 marks can be alloted
divisible by to 8 questions if each question carries at least 2 marks, is
(A) r! (B)  (r – 1)! (A) 115280 (B) 117280
(C) (r + 1)! (D)  None of these (C)  116280 (D)  None of these

102. The number of ordered triplets of positive integers 111. In an examination the maximum marks for each of
which are solutions of the equation x + y + z = 100 is the three papers are 50 each. Maximum marks for the
(A) 5081 (B) 6005 fourth paper are 100. The number of ways in which
(C)  4851 (D)  None of these the candidate can score 60% marks in aggregate is
(A) 110256 (B) 110456
103. The number of words that can be formed, with the (C)  110556 (D)  None of these
letters of the work ‘Pataliputra’ without changing the
relative order of the vowels and consonants, is 112. The number of integers between 1 and 1000000 that
(A) 3600 (B) 4200 have the sum of the digits 18, is
(C)  3680 (D)  None of these (A) 25927 (B) 25827
(C)  24927 (D)  None of these
104. On a new year day every student of a class sends a
card to every other student. The postman delivers 600 113. The number of non-negative integral solutions to the
cards. The number of students in the class are system of equations x + y + z + u + t =20 and x + y +
(A) 42 (B) 34 z = 5 is
(C)  25 (D)  None of these (A) 336 (B) 346
(C)  246 (D)  None of these
105. For any positive integers m, n (with n ≥ m), let
114. The number of positive integral solutions of the
and( ) = C , then ( ) + ( ) + ( ) + ... + ( ) =
n
m
n
m
n
m
n −1
m
n− 2
m
m
m inequality 3x + y + z ≤ 30, is

(A) ( ) ( )
n+1 n +1 (A) 1115 (B) 1215
(B)  m +1

m (C)  1315 (D)  None of these
(C) ( ) n
m +1 (D)  None of these 115. In a city no person has identical set of teeth and
there is no person without a tooth. Also, no person
106. The number of 7 digit numbers the sum of whose has more than 32 teeth. If we disregard the shape and
­digits is even, is size of tooth and consider only the positioning of the
(A) 35 × 105 (B) 45 × 105 teeth, then the maximum population of the city is
5
(C) 50 × 10 (D)  None of these (A) 232 (B)  232 – 1
32
107. The number of ways of choosing m coupons out of an (C) 2 + 1 (D)  None of these
unlimited number of coupons bearing the letters A, B 116. Eleven scientists are working on a secret project.
and C so that they cannot be used to spell the word They wish to lock up the documents in a cabinet such
BAC, is that cabinet can be opened if six or more scientists are
(A)  3 (2m – 1) (B)  3 (2m – 1 – 1) present. Then, the smallest number of locks needed is
m
(C)  3 (2 + 1) (D)  None of these (A) 460 (B) 461
108. Six X ’s have to be placed in squares of (C)  462 (D)  None of these
the figure given below, such that each 117. The number of numbers greater than 106 that can be
row contains at least one X. The number formed using the digits of the number 2334203, if all
of different ways in which this can be the digits of the given number must be used, is
done is

Objective_Maths_JEE Main 2017_Ch 7.indd 21 01/01/2008 04:20:29


7.22  Chapter 7

(A) 360 121. A train is going from Delhi to Indore, stops at nine


(B) 420 intermediate stations. Six persons enter the train
(C) 260 during the journey with six different tickets. The num-
(D)  None of these ber of different sets of tickets possessed by them is
118. If ‘n’is an integer between 0 and 21, then the mini- (A) 50C6 (B)  54
C6
45
mum value of n! (21 – n)! is (C)  C6 (D)  None of these
(A)  9! 2! (B)  10! 11! 122. An n-digit number is a positive number with exactly
(C) 20! (D) 21! n digits. Nine hundred distinct n-digit numbers are to
be formed using only the three digits 2, 5 and 7. The
119. In how many ways can 20 oranges be given to four
smallest value of n for which this is possible is
children if each child should get at least one orange?
(A) 5 (B) 6 (C) 7 (D) 8
(A) 869 (B) 969
(C)  973 (D)  None of these 123. If a, b, c are three natural numbers in A.P. such that
a + b + c = 21, then the possible number of values of
120. The total number of 5-digit numbers of different dig-
a, b, c is
its in which the digit in the middle is the largest is
9 9 (A) 13 (B) 14 (C) 15 (D) 16
(A) ∑
n=4
( n
P4 − n −1
) ∑
P3 (B) n P4 124. The number of ways in which a mixed doubles game
n=4 can be arranged from amongst n couples such that no
9
husband and wife play in the same game, is
(C) ∑ n −1
P3 (D)  None of these
n=4 n n 1n 1
(A)  P4 (B)  C4 (C)  P4 (D)  nC4
2 2

More than One Option Correct Type

125. There are 10 points in a plane, no three of which n( n − 2)


(A)  , when n is even
are in the same straight line excepting 4, which are 4
­collinear. Then, number of 1
(B)  ( n − 1) 2 , when n is odd
(A)  straight lines formed by joining them is 40 4
(B)  triangles formed by joining them is 116 n( n − 2)
(C)  , when n is even
(C)  straight lines formed by joining them is 45 2
(D)  triangles formed by joining them is 120 (D)  None of these
126. If N is the number of positive integral solutions of 129. Let N denotes, the greatest number of points in which
x1 x2 x3 x4 = 770. Then, n straight lines and n circles intersect. Then,
(A) N is divisible by 4 distinct primes (A) n/(N – mC2)
(B)  N is a perfect square (B) m/(N – nP2)
(C)  N is a perfect 4th power
(C) N – mC2 is an even integer
(D) N is a perfect 8th power
(D) N – mC2 – nP2 is an even integer
⎡1 1 ⎤ ⎡1 2 ⎤
127. Let E = ⎢ + ⎥ + ⎢ + ⎥ + … + up to 50 terms, 1 30. If n < p < 2n and p is prime and N = 2nCn, then
⎣ 3 50 ⎦ ⎣ 3 50 ⎦ (A) p/N (B)  p does not divide N
then
(C) p2/N (D)  p2 does not divide N
(A)  E is divisible by exactly 2 primes
(B) E is prime 131. If n objects are arranged in a row, the number of ways
(C) E ≥ 30 of selecting three of these objects so that no two adja-
(D) E ≤ 35 cent objects are selected, is
128. The number of ways in which three numbers in A.P. (A) n –2C3 (B)  n –2
Cn–5
n –3 n –3
can be selected from 1, 2, 3, …, n is (C)  C2 (D)  Cn–5

Objective_Maths_JEE Main 2017_Ch 7.indd 22 01/01/2008 04:20:31


Permutations and Combinations  7.23

1 32. If nCr–1 = (k2 – 8) (n + 1Cr), then k belongs to 133. The number of non-negative integral solutions of
x1 + x2 + x3 + x4 ≤ n (where n is a positive integer) is
(A)  [–3, –2 2 ] (B)  [−3, −2 2 )
(A) n + 4 Cn (B)  n+4
C4
(C) [2 2 , 3] (D)  (2 2 , 3] n+3 n+3
(C)  C3 (D)  Cn

Passage Based Questions


Passage 1 141. In how many ways all these coins can be distributed
There are five different boxes and seven different balls. All such that no pot is empty if all coins are different but
the seven balls are to be distributed in the five boxes placed all pots are identical?
in a row so that any box can receive any number of balls. (A) 16 (B) 6 (C) 42 (D) 21
134. In how many ways can these balls be distributed so 142. In how many ways all these coins can be distributed
that no box is empty? such that no pot is empty if all coins are identical but
(A) 71 (B) 16800 all pots are different?
(C)  1775 (D)  None of these (A) 6 (B) 3 (C) 9 (D) 27
135. Suppose, all the balls are identical, then in how many 143. In how many ways all these coins can be distributed if
ways can all these balls be distributed into these all coins are identical and two pots are also identical?
boxes? (A) 1 (B) 10 (C) 9 (D) 11
(A) 110 (B) 220 (C) 330 (D) 1440
144. In how many ways all these coins can be distributed
136. In how many ways can these balls be distributed if out of 4 coins 2 coins are identical and all pots are
so that box 2 and box 4 contain only 1 and 2 balls, different?
respectively? (A) 45 (B) 27
(A) 5522 (B) 8505 (C)  54 (D)  None of these
(C)  2305 (D)  None of these
Passage 3
137. In how many ways can these balls be distributed into
In the World Cup, the tournament is arranged as per the
these boxes if ball 2 can be put into either box 2 or
following rules:
box 4?
In the beginning, 16 teams are taken and divided
(A) 12360 (B) 31250 (C) 13490 (D) 31526 into 2 groups of 8 teams each. Teams of each group play
138. In how many ways can these balls be distributed such a match against each other in the same group. From each
that no box is empty and ball 2 and ball 4 cannot be group, 4 top teams qualify for the next round.
put in the same box? In the next round, two teams play each other in each
group and the losing team goes out of the tournament.
(A) 1200 (B) 15000
Then, four winning teams play for semifinal round and
(C)  3800 (D)  None of these
finally there is one final. The rules of the tournament are
Passage 2 such that every match can result only in a win or a loss and
There are three pots and four coins. All these coins are to not in a tie.
be distributed into these pots where any pot can contain any 145. The total number of matches played in the tourna-
number of coins. ment is
139. In how many ways all these coins can be distributed if (A) 51 (B) 64 (C) 63 (D) 52
all coins are identical but all pots are different? 146. The maximum number of matches that a team going
(A) 15 (B) 16 (C) 17 (D) 81 out of the tournament in the first round can win is
140. In how many ways all these coins can be distributed if (A) 1 (B) 2 (C) 3 (D) 4
all coins are different but all pots are identical? 147. The minimum number of matches that a team must
(A) 14 (B) 21 win in order to qualify for the second round is
(C)  27 (D)  None of these (A) 4 (B) 5 (C) 6 (D) 7

Objective_Maths_JEE Main 2017_Ch 7.indd 23 01/01/2008 04:20:31


7.24  Chapter 7

148. Which of the following statements about a team (A)  it has to win exactly 14 matches
which has already qualified for the second round is (B)  it has to win exactly 5 matches
true? (C)  it has to win exactly 4 matches
To win the world Cup (D)  it has to win exactly 3 matches

Match the Column Type

149. n
(III) If n is even and nC0 < (C) 
n
C1 < nC2 < … < nCr > 2
Column-I Column-II
n
Cr+1 > … > nCn then
  (I) The sum of the digits in the (A) 286
r =…
unit’s place of all the numbers
n ( n − 3)
formed with the help of 3, 4, 5, (IV) There are n concurrent (D) 
6 taken all at a time is… lines and another line 2
  (II) The number of ways in which a (B) 108 parallel to one of them.
committee of 5 can be chosen The number of different
from 10 candidates so as to triangles that will be
exclude the youngest if it formed by the (n + 1)
includes the oldest, is… lines, is…
  (III) The number of divisors of 9600 (C) 196 151.
including 1 and 9600 are…
Column-I Column-II
(IV) The number of ways of (D) 48 n–2 n–2
choosing 10 balls from infinite   (I)  Cr + 2. Cr – 1 + (A)  n + 3
n–2
white, red, blue and green balls Cr – 2 =…
is… n
m
  (II)  Cr + 1 + ∑ k Cr =… (B)  n + 1Cr + 1
150. k =m
n ( nr − 1)
Column-I Column-II If nCn – r + 3 ⋅ nCn – r + 1 + 3 ⋅
  (III)  (C) 
n
Cn – r + 2 + nCn – r + 3 = xCr, n −1
  (I) The number of diagonals (A)  2nCn – 1
in a polygon of n sides then x =…
is… (IV) The total number of (D)  nCr
( n − 1) ( n − 2) permutations of n different
  (II) The number of non- (B) 
2 things taken not more than
negative solutions of
r at a time, when each thing
x1 + x2 + x3 + … + xn ≤
may be repeated any number
n (where n is a positive
of times is…
integer) is…

Assertion-Reason Type
Instructions: In the following questions an Assertion (A) is (C)  Assertion(A) is True, Reason(R) is False
given followed by a Reason (R). Mark your responses from (D)  Assertion(A) is False, Reason(R) is True
the following options:
152. Assertion: In an examination consisting of 9 papers, a
(A)  Assertion(A) is True and Reason(R) is
candidate has to pass in more papers than the ­numbers
True; Reason(R) is a correct explanation for
of papers in which he fails in order to be successful.
Assertion(A)
The number of ways in which he can be unsuccessful
(B)  Assertion(A) is True, Reason(R) is True;
is 256.
Reason(R) is not a correct explanation for
Assertion(A) Reason: nC0 + nC1 + nC2 + … + nCn = 2n

Objective_Maths_JEE Main 2017_Ch 7.indd 24 01/01/2008 04:20:32


Permutations and Combinations  7.25

Previous Year’s Questions

153. A student is to answer 10 out of 13 questions in an (A)  less than 500


examination such that he must choose at least 4 from (B)  at least 500 but less than 750
the first five questions. The number of choices avail- (C)  at least 750 but less than 1000
able to him is [2003] (D)  at least 1000
(A) 140 (B) 196 161. There are two urns. Urn I has 3 distinct red balls and
(C) 280 (D) 346 Urn II has 9 distinct blue balls. From each urn two
154. If nCr denotes the number of combinations of n things balls are taken out at random and then transferred to
taken r at a time, then the expression nCr + 1 + nCr-1+ the other. The number of ways in which this can be
2 × nCr equals [2003] done is [2010]
(A) 36 (B) 66
n+2
(A)  Cr (B)  n + 2Cr + 1 (C) 108 (D) 3
n+1
(C)  Cr (D)  n + 1Cr + 1
162. Statement-1: The number of ways of distributing 10
155. How many ways are there to arrange the letters in identical balls in 4 distinct boxes such that no box is
the word GARDEN with the vowels in alphabetical empty is 9C3 [2011]
order? [2004] Statement-2: The number of ways of choosing any 3
(A) 120 (B) 480 places from 9 different places is 9C3.
(C) 360 (D) 240 (A)  Statement-1 is true, Statement-2 is true;
156. The number of ways of distributing 8 identical balls Statement-2 is not a correct explanation for
in 3 distinct boxes so that none of the boxes is empty Statement-1
is [2004] (B)  Statement-1 is true, Statement-2 is false.
(A)  5 (B)  8
C3 (C)  Statement-1 is false, Statement-2 is true.
(D)  Statement-1 is true, Statement-2 is true;
(C) 38 (D) 21
Statement-2 is a correct explanation for
157. If the letters of word SACHIN are arranged in all Statement-1
possible ways and these words are written out as in 163. Assuming the balls to be identical except for differ-
dictionary, then the word SACHIN appears at serial ence in colors, the number of ways in which one or
number [2005] more balls can be selected from 10 white, 9 green and
(A) 601 (B) 600 7 black balls is [2012]
(C) 603 (D) 602 (A) 880 (B) 629
158. At an election, a voter may vote for any number of (C) 630 (D) 879
candidates, not greater than the number to be elected.
164. Let Tn be the number of all possible triangles formed
There are 10 candidates and 4 are of be elected. If a
by joining vertices of an n-sided regular polygon. If
voter votes for at least one candidate, then the number
Tn+1 − Tn = 10, then the value of n is [2013]
of ways in which he can vote is [2006]
(A) 5 (B) 10
(A) 5040 (B) 6210
(C) 385 (D) 1110 (C) 8 (D) 7
165. The number of integers greater than 6,000 that can
159. How many different words can be formed by jum-
be formed, using the digits 3, 5, 6, 7 and 8, without
bling the letters in the word MISSISSIPPI in which
repetition, is: [2015]
no two S are adjacent? [2008]
(A) 192 (B) 120
(A) 8 ⋅ 6C4 ⋅ 7C4 (B) 6 ⋅ 7 ⋅ 8C4
(C) 72 (D) 216
(C) 6 ⋅ 8 ⋅ 7C4 (D) 7 ⋅ 6C4 ⋅ 8C4
166. If all the words (with or without meaning) having five
160. From 6 different novels and 3 different dictionar- letters, formed using the letters of the word SMALL
ies, 4 novels and 1 dictionary are to be selected and and arranged as in a dictionary; then the position of
arranged in a row on the shelf so that the dictionary the word SMALL is: [2016]
is always in the middle. Then the number of such (A) 58th (B)  46th
arrangements is [2009] (C) 59th (D)  52nd 

Objective_Maths_JEE Main 2017_Ch 7.indd 25 01/01/2008 04:20:32


7.26  Chapter 7

Answer keys

Single Option Correct Type


  1. (A) 2.  (D) 3. (A) 4.  (C) 5. (C) 6.  (B) 7. (B) 8.  (A) 9. (B) 10. (C)
  11. (B) 12.  (A) 13. (B) 14.  (C) 15. (B) 16.  (B) 17. (B) 18. (C) 19. (D) 20.  (A)
  21. (B) 22.  (B) 23. (A) 24.  (B) 25. (D) 26.  (C) 27. (A) 28.  (C) 29. (A) 30.  (B)
  31. (B) 32.  (D) 33. (B) 34. (B) 35. (A) 36.  (A) 37. (D) 38.  (A) 39. (A) 40.  (C)
  41. (C) 42. (A) 43.  (B) 44. (C) 45.  (B) 46. (D) 47.  (B) 48. (A) 49.  (B) 50. (C)
  51. (D) 52.  (B) 53. (A) 54.  (D) 55. (D) 56.  (A) 57.  (A) 58.  (D) 59. (A) 60.  (A)
  61.  (C) 62. (A) 63. (C) 64.  (C) 65. (C) 66.  (C) 67. (D) 68.  (A) 69.  (B) 70. (C)
  71.  (D) 72. (A) 73.  (A) 74. (A) 75. (C) 76.  (A) 77. (B) 78. (D) 79. (B) 80.  (B)
  81. (D) 82.  (A) 83.  (C) 84. (B) 85.  (C) 86. (A) 87.  (B) 88. (A) 89.  (B) 90. (B)
  91. (A) 92.  (C) 93. (A) 94. (C) 95. (A) 96. (B) 97. (C) 98.  (C) 99. (B) 100.  (B)
101. (A) 102. (C) 103. (A) 104. (C) 105. (B) 106. (B) 107.  (A) 108. (A) 109. (B) 110. (C)
111. (C) 112. (A) 113. (A) 114.  (B) 115. (B) 116.  (C) 117. (A) 118.  (B) 119. (B) 120.  (A)
121. (C) 122.  (C) 123. (A) 124.  (C)

More than One Option Correct Type


125.  (A) and (B) 126.  (B), (C) and (D) 127.  (B) and (D) 128.  (A) and (B) 129.  (A), (C) and (D)
130.  (A) and (D) 131.  (A) and (B) 132.  (B) and (D) 133.  (A) and (B)

Passage Based Questions


134.  (B) 135. (C) 136. (B) 137. (B) 138. (B) 139.  (A) 140. (A) 141.  (B) 142. (B) 143.  (C)
1 44. (C) 145. (C) 146. (C) 147.  (A) 148. (D)

Match the Column Type


1 49. (I) → (B); (II) → (C); (III) → (D); (IV) → (A)
150. (I) → (D); (II) → (A); (III) → (C); (IV) → (B)
151. (I) → (D); (II) → (B); (III) → (A); (IV) → (C)

Assertion-Reason Type
152. (A)

Previous Year’s Questions


153. (B) 154. (B) 155. (C) 156. (D) 157. (A) 158. (C) 159. (D) 160. (D) 161. (C) 162. (D)
163. (D) 164. (A) 165. (A) 166. (A)

Objective_Maths_JEE Main 2017_Ch 7.indd 26 01/01/2008 04:20:32


Permutations and Combinations  7.27

Hints and Solutions

Single Option Correct Type


1. Number of solutions of the given equation is the same as the 6. Number of 10 balls selections
number of solutions of the equation = coefficient of x10 in (x2 + x3 + …) (1 + x + x2 +…) (1 + x +
x1x2x3x4 = 30 = 2 × 3 × 5 x2 +…) (x3 + x4 +…)
Here, x4 is there because if x1x2x3 = 15, then x4 = 2 and if = coefficient of x5 in (1 + x + x2 +…)4
x1x2x3 = 5, then x4 = 6 etc.
= coefficient of x5 in (1 – x)–4
x4 is in fact a dummy variable. (4 + 5 – 1)
= C5 = 8C5 = 56 ways.
Each of 2, 3 and 5 will be a factor of exactly one of x1, x2, x3, The correct option is (B)
x4 in 4 ways. 7. Since each element of A can be associated with elements of
∴ Required number = 43 = 64 8 in two ways, therefore the total number of functions from
The correct option is (D) A to B is 2 × 2 × 2 × 2 = 16. Out of these functions, the func-
2. Now, n = 23 – 2 = 6 tions which are not onto are f (x) = 1, ∀ x ∈ A and f (x) = 2 ∀
⎛ 2 1⎞ x ∈ A. Thus, the number of onto functions = 16 – 2 = 14.
Also, m = 4 + 3 + 2 + 1 – 1 = 9 ⎜ as = ⎟
⎝ 4 2⎠ The correct option is (B)
∴ m – n = 3
The correct option is (D) 8. We know that in any trinagle the sum of two sides is always
greater than the third side.
3. A D M N O R (in order)
∴ The triangle will not be formed if we select segments of
Number of words beginning with
lengths (2, 3, 5), (2, 3, 6) or (2, 4, 6).
A _ _ _ _ _ = 5!
Hence number of triangles formed = 5C3 – 3.
D _ _ _ _ _ = 5!
The correct option is (A)
M _ _ _ _ _ = 5!
9. a = x+2Px+2, b = xP11, C = x–11Px–11
N _ _ _ _ _ = 5! x!
O _ _ _ _ _ = 5! ⇒ a = (x + 2)!, b = , c = (x – 11)!
( x − 11)!
R A D _ _ _ = 3! Now, a = 182 bc
x!
R A M _ _ _ = 3! ⇒ (x + 2)! = 182 (x – 11)!
( x − 11)!
R A N D M O = 1
⇒ (x + 2) (x + 1)x! = 182x!
R A N D O M = 1
⇒ (x + 2) (x + 1) = 182 = 14 × 13
∴ Rank of word RANDOM = 614 ⇒ x + 1 = 13
The correct option is (D) ∴ x = 12
4. Let A, B be the corresponding speakers. Without any restric- The correct option is (B)
tion the eight persons can be arranged among themselves in 10. The four digits 3, 3, 5, 5, can be arranged at four even places
8! ways, but the number of ways in which A speaks before 4!
in = 6 ways and the remaining digits viz., 2, 2, 8, 8, 8
B and the number of ways in which B speaks before A make 2! 2! 5!
up 8!. Also number of ways in which A speaks before B can be arranged at the five odd places in = 10 ways.
2! 3!
is exactly same as the number of ways in which B speaks Thus, the number of possible arrangements is (6) (10) = 60.
before A. The correct option is (C)
1
∴ the required number of ways = (8!) = 20160. 11. For one game four persons are required. This can be done
2
The correct option is (C) in 6C4 = 15 ways. Once a set of four persons are selected,
4
5. Total number of permutations = 7! C2
number of games possible will be = 3 games.
Let A be the property that ‘beg’ occurs. 2
B be the property that cad occurs. ∴ Total number of possible games = 3 × 15 = 45.
Number of permutations with A = 5! The correct option is (B)
= that of with B and n (A ∩ B) = 3! 12. We know that a number is divisible by 3 if the sum of its
digits is divisible by 3.
∴ n(A ∪ B) = 5! + 5! – 3! = 234
Now the sum of the digits 1, 2, 3, 4 and 5 is 15, which is
∴ Required number = 7! – 234 = 4806
divisible by 3.
The correct option is (C)

Objective_Maths_JEE Main 2017_Ch 7.indd 27 01/01/2008 04:20:33


7.28  Chapter 7

n
∴ All the five digit numbers formed by the digits 1, 2, 3, 4,
5 are divisible by 3 and their number = 5! = 120.
17. mCr + 1 + ∑ k
Cr
k=m
When we include 0, the four other digits whose sum is divis- = mCr + 1 + Cr + m + 1Cr + … + n – 1Cr + nCr
m
ible by 3 are 1, 2, 4 and 5. m+1
= Cr + 1 + m + 1Cr + … + n – 1Cr + nCr
∴ The number of numbers in this case
m+2
= 4 × 4! = 4 × 24 = 96. =
Cr + 1 + m + 2Cr + … + n – 1Cr + nCr
Hence the required number of numbers    
= 120 + 96 = 216 = nCr + 1 + nCr = n + 1Cr + 1

The correct option is (A)
The correct option is (B)
13. The number of ways of selecting 3 balls out of total 9
18. No. of triangles = 2nC3 – nC3 – nC3
(2 white, 3 black, 4 red balls) is 9C3
9×8×7 2n ( 2n − 1) ( 2n − 2) 2n ( n − 1) ( n − 2)
i.e., = 84 =


6 6 6
1 2
The number of ways of selecting 3 balls out of non-black six = n (n – 1) (3n) = n (n – 1)
3
balls is 6C3
6×8×4 The correct option is (C)
i.e., = 20 19. E H M O R T (alphabetical order)
3 × 2 ×1
Therefore, the number of ways of selecting 3 balls out of Number of words beginning with
9 balls so as to include atleast one black ball = 84 – 20 = 64. E _ _ _ _ _ = 5!
The correct option is (B) H _ _ _ _ _ = 5!
14. The total number of numbers that can be formed with the M E _ _ _ _ = 4!
digits 2, 3, 4, 5 taken all at a time = 4P4 = 4! = 24. Consider M H _ _ _ _ = 4!
the digit in the unit’s place in all these numbers. Each of the M O E _ _ _ = 3!
digits 2, 3, 4, 5 occurs in 3! = 6 times in the unit’s place
M O H _ _ _ = 3!
∴ total for the digits in the unit’s place
M O R _ _ _ = 3!
= (2 + 3 + 4 + 5) 6 = 84
M O T E _ _ = 2!
Since each of the digits 2, 3, 4, 5 occurs 6 times in any one
M O T H E R = 1!
of the remaining places

∴ the required total
∴ Rank of word MOTHER = 309
= 84 (1 + 10 + 102 + 103) = 84 (1111) = 93324.
The correct option is (D)
The correct option is (C)
20. Factorizing the given number, we have
15. Total number of ways = nn
38808 = 23 ⋅ 32 ⋅ 72 ⋅ 11
Number of ways so that each person gets at least one thing
Therefore the total number of divisors
(here exactly one thing) = n!
= (3 +1) (2 + 1) (1 + 1) – 1 = 71.
Given, nn – n! = 232
But this includes the division by the number itself.
11 – 1! = 0, 22 – 2! = 2, 33 – 3! = 21, 44– 4! = 232
Hence, the required number of divisors
∴ n = 4
= 71 – 1 = 70
The correct option is (C)
16. Let total number of persons be n.
Trick(s) for Problem Solving
Since two persons shake hands,
so number of ways of shaking hands = nC2. Let N = p1a ⋅ p2a ⋅ p3a … pka where p1, p2, p3 …pk are dif-
1 2 3 k

According to question, we have nC2 = 66 ferent primes and a1, a2, …, ak are natural numbers then
n! n ( n − 1)( n − 2)! total ­number of divisors of N excluding 1 and N is (a1 + 1)
or = 66  or  = 66 (a2 + 1) … (ak + 1) – 2
( n − 2)! 2! ( n − 2)! 2 × 1
or n2 – n = 132 or n2 – n – 132 = 0

The correct option is (A)
or n2 – 12n + 11n – 132 = 0

21. We have, 15 < x1 + x2 + x3 ≤ 20
or n (n – 12) + 11 (n – 12) = 0  ⇒  n = – 11 or 12

⇒ x1 + x2 + x3 = 16 + r, r = 0, 1, 2, 3, 4.
But, n = –11 is not possible.

Now, number of positive integral solutions of x1 + x2 + x3 =
∴ n = 12.
16 + r is 16 + r – 1C3 – 1 = 15 + r C2

The correct option is (B)

Objective_Maths_JEE Main 2017_Ch 7.indd 28 01/01/2008 04:20:34


Permutations and Combinations  7.29

Thus,
total number of solutions ⎧ No. of Ways of ⎫
4 ⎪ ⎪
+ … + ⎨ distributing ⎬

= ∑ 15+ r C2
15
C2 + 16C2 + 17C2 + 18C2 + 19C2 ⎪ ⎪
r=0 ⎩ n items ⎭
20 15
C3 – C3 = 685 = 1+n–1Cn–1 + 2+n–1Cn–1 + … + n+n–1Cn–1

= n
Cn–1 + n+1Cn–1 + … + 2n–1Cn – 1
Trick(s) for Problem Solving

= (nCn–1
+ nCn) + n+1Cn–1 + … + 2n–1Cn–1 – nCn
The total number of positive integral solutions of the equation
+ n+1Cn–1) + … + 2n–1Cn–1} – nCn
= {(n+1Cn

x1 + x2 + … + xr = n is n – 1Cr – 1
----------------------------------


----------------------------------
The correct option is (B)
22. Other than 2, remaining five places are to be filled by by = (2n–1Cn + 2n–1Cn–1) – nCn

1 and 3 = 2n
Cn – nCn

∴ number of ways for five places

W = 2nCn – 1
= 2 × 2 × 2 × 2 × 2 = 25
The correct option is (A)
For 2, selecting 2 places out of 7 = 7C2
28. 6 large animals can be caged in 7 large cages in 7P6 = 7!
∴ Required no. of ways = 7C2 ⋅ 25. ways. 5 small animals can be caged in remaining 5 cages
The correct option is (B) (4 small + 1 large) in 5! ways. Hence, the number of ways is
23. We know that 1! + 2! + 3! + 4! = 33. 7! × 5! = 5040 × 120 = 604800
Also, 5! = 120, 6! = 720, 7! = 5040, 8! = 40320 and The correct option is (C)
9! = 362880. 29. By the given condition, it is clear that nCr is the greatest
Thus, tens digit of 1! + 2! + … + 9! is 1. among nC0, nC1, …, nCn.
Also, note that n! is divisible by 100 for all n ≥ 10. Therefore, Since n is even,
the tens digit of 1! + 2! + … + 49! is 1. n
∴ nCr is the greatest for r =
The correct option is (A) n 2
∴ r =
24. Each element of the set A can be given the image in the set A 2
in k ways. The correct option is (A)
∴ the required number of functions, i.e., 30. For each pair of stations, two different types of tickets are
n (S) = k × k × … (k times) = kk. required, Now, the number of selections of 2 stations from
The correct option is (B) 15 stations = 15C2.
25. pC × pC × pC + pC × 2 pC × 3C ∴ Required number of types of tickets
1 1 1 2 1 1
15!
= p2(4p – 3) = 2 15C2 = 2 = 15 × 14 = 210
2!13!
The correct option is (D) The correct option is (B)
37! 31. All the numbers of the form 6n will end with 6 and 9m will end
26. Number of ways = 37C2 = = 666 ways
35! × 2! with 9, if m is odd and will end with 1, if m is even. Therefore,
For the distribution equation. 6n + 9m will end with 5 if n is any number and m is odd.
x1 + x2 + x3 + … + xr = n Thus, number of ordered pairs = 50 × 25 = 1250.
The number of ways in which n things can be distributed The correct option is (B)
among r in such a way each can receive none, one or more or 32. Number of subsets
all of n items are n+r–1Cr–1
= 2n+1C0 + 2n+1C1 + … + 2n+1Cn = N (say)

The correct option is (C)
27. In general, we know that ∴ N = 22n+1 – N
For the distribution equation ⇒ 2N = 22n+1

x1 + x2 + x3 + … + xn ≤ n ⇒ N = 22n
Let required ways = W The correct option is (D)
33. The number of triangles = number of selections of 2 lines
⎧ No. of ways of ⎫ ⎧ No. of ways of ⎫ from the (n – 1) lines which are cut by the last line
⎪ ⎪ ⎪ ⎪
⇒ W = ⎨ distributing ⎬ + ⎨ distributing ⎬

( n − 1)! ( n − 1) ( n − 2)
⎪ 1 item ⎪ ⎪ 2 items ⎪ = n – 1C2 =
=
⎩ ⎭ ⎩ ⎭ 2!( n − 3)! 2

The correct option is (B)

Objective_Maths_JEE Main 2017_Ch 7.indd 29 01/01/2008 04:20:36


7.30  Chapter 7

34. Distinct n digit numbers which can be formed using digits 2, 1 2 1 r 1


⇒ − − < − −
≤−
5 and 7 are 3n. We have to find n so that 3 3 3 100 3
3n ≥ 900  ⇒ 3n–2 ≥ 100 ⎛ 1 r ⎞
∴ ⎜ − −
= – 1 for 0 ≤ r ≤ 66
⇒ n – 2 ≥ 5  or  n ≥ 7 ⎝ 3 100 ⎟⎠
The correct option is (B) 67 r
Also, for 67 ≤ r ≤ 100,
≤ ≤1
So, the least value of n is 7. 100 100
35. Starting with the letter A and arranging the other four letters, r 67
⇒ –1 ≤ –
≤−
there are 4! = 24 words. These are the first 24 words. Then, 100 100
starting with G and arranging A, A, I and N in different ways, 1 1 r 1 67
4! ⇒ − − 1 ≤ − −
≤− −
there are = 12 words, the 37th word start with I and 3 3 100 3 100
2! 1! 1!
there are 12 words starting with I. Total goes upto 80 words. ⎛ 1 r ⎞
∴ ⎜ − −
= –2 for 67 ≤ r ≤ 100
Now, 49th word is NAAGI. ⎝ 3 100 ⎟⎠
The correct option is (A) 100
⎛ 1 r ⎞
36. If we choose k (0 ≤ k ≤ n) identical objects, then we must
Hence, ∑ ⎜⎝ − 3 − 100 ⎟⎠ = 67 (– 1) + 2(– 34) = – 135
r=0
choose (n – k) distinct objects. This can be done in 2n+1Cn–k
ways. Thus, the required number of ways The correct option is (C)
n
42. The following four cases are there:
=
∑ 2n+1Cn − k = 2n+1Cn + 2n+1Cn–1 + … + 2n+1C0 (A) At least one one rupee coin and any number of other
k=0 coins = 2 × 4 × 5 = 40 ways.
2n
=2
(B) At least two 50 p. coins and any number of 25 p. coins
The correct option is (A) = 2 × 5 = 10 ways.
37. 6 boys can sit in 6! ways (C) One 50 p. coin and at least two 25 p. coins = 1 × 3 =
× B1 × B2 × B3 × B4 × B5 × B6 × 3 ways.
Now, two brothers can sit in any of the 7 cross marked (×) (D) Four 25 p. coins in one way only ∴ total = 54.
places The correct option is (A)
Therefore, required number of ways = 6! × 7C2 × 2! 43. The number of ways in which at most n books can be selected
= 30240 out of a collection of (2n + 1) books is
2n + 1
The correct option is (D) C1 + 2n + 1C2 + … 2n + 1Cn
38. The number of subsets of A containing exactly three ele- 1 2n + 1

[( C1 + 2n + 1C1) + (2n + 1C2 + 2n + 1C2) + …
ments is nC3 whereas the number of three subsets of A that 2
contain a1 is n–1C2. We are given, … + (2n + 1Cn + 2n + 1Cn)] = 63

n–1
C2 =
20 n
( C3) ⇒ (2n + 1C1 + 2n + 1C2n) + (2n + 1C2 + 2n + 1C2n – 1) + …

100
… + (2n + 1Cn + 2n + 1Cn + 1) = 126

( n − 1) ( n −2) 1 n ( n − 1) ( n −2)
⇒ = ⇒ 2n + 1C1 + 2n + 1C2 + … + 2n + 1C2n = 126
2 5 6
⇒ 22n + 1 – 2 = 126
⇒ n = 15
⇒ 22n + 1 = 128 = 27
The correct option is (A)
⇒ 2n + 1 = 7
39. When zero is in the place of y then there are nine numbers
⇒ n = 3
and when 1 is in the place of y then there are 8 numbers and
9 The correct option is (B)
so on. Hence, required number is ∑ n = 45. 44. The four digits 3, 3, 5, 5 can be arranged at four even places
n =1 4!
The correct option is (A) in = 6 ways and the remaining digits viz., 2, 2, 8, 8, 8
40. The required number is equal to the number of all the subsets 2! 2! 5!
of a 68 elements set i.e., can be arranged at the five odd places in = 10 ways.
2! 3!
68C + 68C + 68C + … + 68C = 268 = 1617 Thus, the number of possible arrangements is (6) (10) = 60.
0 1 2 68


The correct option is (C) The correct option is (C)
r 2 45. The number of students answering at least r questions incor-
41. For 0 ≤ r ≤ 66, 0 ≤ < rectly is 2n–r.
100 3
2 r ∴ The number of students answering exactly r (1 ≤ r ≤ n – 1)
⇒ −
<− ≤0 questions incorrectly is 2n–r – 2n–(r + 1).
3 100

Objective_Maths_JEE Main 2017_Ch 7.indd 30 01/01/2008 04:20:38


Permutations and Combinations  7.31


Also, the number of students answering all questions 5
5! ⎡1 1 ( −1) r ⎤
wrongly is 20 = 1. =
∑ (5 − r )! ⎢⎢ 2! − 3! + … + ⎥
r ! ⎥⎦

Thus, the total number of wrong answers is
r=2 ⎣


1 (2 n–1
–2n–2
) + 2 (2 n–2
–2 n–3
) + 3 (2n–3
–2 n–4
)+… 5! ⎛ 1 ⎞ 5! ⎛ 1 1 ⎞ 5! ⎛ 1 1 1 ⎞
=
⎜ ⎟+ ⎜ − ⎟+ ⎜ − + ⎟
1 0 0 3! ⎝ 2!⎠ 2! ⎝ 2! 3!⎠ 1! ⎝ 2! 3! 4!⎠
+ (n – 1) (2 – 2 ) + n (2 )

= 2n–1 + 2n–2 + … + 20 = 2n –1
5! ⎛ 1 1 1 1 ⎞
+ ⎜ − + − ⎟
Now, 2n – 1 = 4095 ⇒ 2n = 4096 = 212 ⇒ n = 12
0! ⎝ 2! 3! 4! 5!⎠

The correct option is (B) = 10 + 20 + (60 – 20 + 5) + (60 – 20 + 5 – 1)

= 10 + 20 + 45 + 44 = 119.

46. d
c Trick(s) for Problem Solving
a ⎯⎯
→ 2 ways.
If r (0 ≤ r ≤ n) objects occurpy the places assigned to them
d and none of the remaining objects occupies its original places,
a then the number of such ways is
b a d c ⎯⎯
→ 3 ways. ⎡ 1 1 1 1 ⎤
= nCr · (n – r)! ⎢1 − + − + … + ( −1)n − r
d a c ⎣ 1! 2 ! 3 ! (n − r )! ⎥⎦
b c a
The correct option is (B)
a
50. Number of ways to distribute 10 identical balls in four dis-
c ⎯⎯
→ 3 ways. tinct boxes such that no box remains empty = 10 – 1C4 – 1 = 9C3
Number of ways to select 3 different places from 9 places =
9
C3.
d ⎯⎯
→ 3 ways.
Clearly statement-2 is not a correct explanation of
∴ Total number of ways = 11.
statement-1.

The correct option is (D) The correct option is (C)
m 51. (10 + 1)(9 + 1)(7 + 1) – 1 = 11.10.8 – 1 = 879
47. S = ∑ n + r Ck The correct option is (D)
r=0
52. Number of ways to fill thousand’s place = 9
= nCk + n+1Ck + n+2Ck + … + n+mCk
Number of ways to fill hundred’s place = 9
n
= Ck +1 + nCk + n+1Ck + n+2Ck + … + n+mCk – nCk+1 Number of ways to fill ten’s place = 8
=

n+1
Ck+1 + n+1Ck + n+2Ck + … + n+mCk – nCk+1 Number of ways to fill unit place = 7
∴ S = n+m+1Ck+1 – nCk+1 ∴ 9 × 9 × 8 × 7 = 4536

The correct option is (B) The correct option is (B)
48. The number of ways of choosing first couple is (15C1) 53. x1 + x2 + x3 + x4 + x5 = 6
(15C1) = 152. The number of ways of choosing 2nd couple is 5 + 6 – 1
C5 – 1 = 10C4
(14C1) (14C1) = 142, and so on. Thus, the number of ways of The correct option is (A)
choosing the couples is 54. Other than S, seven letters M, I, I, I, P, P, I can be arranged in
152 + 142 + 132 + … + 22 + 12 7!
= 7 ⋅ 5 ⋅ 3.
15 × (15 + 1) [2 (15) + 1] 2! 4 !
= = 1240
6 Now four S can be placed in 8 spaces in 8C4 ways.
Desired number of ways = 7 ⋅ 5 ⋅ 3 ⋅ 8C4 = 7 ⋅ 6C4 ⋅ 8C4.
Trick(s) for Problem Solving The correct option is (D)
55. Four novels can be selected from 6 novels in 6C4 ways. One
Number of ways of dividing n identical things into r groups, if
dictionary can be selected from 3 dictionaries in 3C1 ways.
blank groups are allowed is n + r – 1Cr – 1.
As the dictionary selected is fixed in the middle, the remain-
ing 4 novels can be arranged in 4! ways.
The correct option is (A)
∴ The required number of ways of arrangement
49. Required number of ways
= 6C4 × 3C1 × 4! = 1080
5 ⎡ 1 1 1 ( −1) r ⎤
=
∑ 5 C5 − r r ! ⎢⎢1 − 1! + 2! − 3! + … + ⎥
r ! ⎥⎦
The correct option is (D)
r=2 ⎣

Objective_Maths_JEE Main 2017_Ch 7.indd 31 01/01/2008 04:20:40


7.32  Chapter 7

56. S = {00, 01, 02, … 49} 62. ∑ ∑ 10


C j jCi
Let A be the event that sum of the digits on the selected ticket 0 ≤ i ≤ j ≤ 10
is 8 then = 10C0 × 0C0 + 10C1 (1C0 + 1C1) + 10C2 (2C0 + 2C1 + 2C2)

A = {08, 17, 26, 35, 44}
+ 10C3 (3C0 + 3C1 + 3C2 + 3C3) + …

Let B be the event that the product of the digits is zero
B = {00, 01, 02, 03, … 09, 10, 20, 30, 40} + 10C10(10C0 + 10C1 + … + 10C10)

A ∩ B = {8} 1 = 10C0 · 20 + 10C1 · 21 + 10C2 · 22 + … + 10C10 · 210

P ( A ∩ B ) 50 1
= (1 + 2)10 = 310.
Required probability = P(A/B) = = =
P ( B) 14 14
The correct option is (A)
50
The correct option is (A) 63. Let A, B be the corresponding speakers. Without any restric-
57. Total number of wrong answers tion the eight persons can be arranged among themselves in
8! ways, but the number of ways in which A speaks before
= 1 · (a1 – a2) + 2.(a2 – a3) + … + (k – 1) (ak – 1 – ak) + kak
B and the number of ways in which B speaks before A make
= a1 + a2 + a3 + … + ak up 8!. Also, number of ways in which A speaks before B
The correct option is (A) is exactly same as the number of ways in which B speaks
58. The number of ways in which 13 guests may be divided into before A.
13! 1
groups of 8 and 5 = 13C5 = ∴ the required number of ways = (8!) = 20160.
5! 8! 2
The correct option is (C)
Now, corresponding to one such group, the 8 guests may be
64. Total number of permutations = 7!
seated at one round table in (8 – 1)! i.e., 7! ways and the five
guests at the other table in (5 – 1)! i.e., 4! ways. Let A be the property that ‘beg’ occurs.
But each way of arranging the first group of 8 persons can be B be the property that cad occurs.
associated with each way of arranging the second group of Number of permutations with A = 5!
5, therefore, the two processes can be performed together in = that of with B and n (A ∩ B) = 3!
7! × 4! ways. ∴ n(A ∪ B) = 5! + 5! – 3! = 234
Hence, required number of arrangements ∴ Required number = 7! – 234 = 4806
13! 13! 13! The correct option is (C)
= × 7! × 4! = × 7! × 4! =
5! 8! 5 ⋅ 4 ! 8 ⋅ 7! 40 65. The total number of numbers that can be formed with the
The correct option is (D) digits 2, 3, 4, 5 taken all at a time = 4P4 = 4! = 24. Consider
59. Each stall can be filled in 3 ways as there are three types of the digit in the unit’s place in all these numbers. Each of the
animals (animals of each category being not less than 10). digits 2, 3, 4, 5 occurs in 3! = 6 times in the unit’s place
Shipload, i.e., filling up of 10 stalls, can be made in ∴ total for the digits in the unit’s place
3 × 3 × … up to 10 times = 310 = 59049. = (2 + 3 + 4 + 5) 6 = 84
The correct option is (A) Since each of the digits 2, 3, 4, 5 occurs 6 times in any one
60. Let the number of papers be n. of the remaining places, therefore, the required total
∴ Total number of ways to fail or pass = 84 (1 + 10 + 102 + 103) = 84 (1111) = 93324.
n
C0 + nC1 + nC2 + … + nCn = 2n. The correct option is (C)
But there is only one way to pass, i.e., when he fails in none. 66. Number of triangles = 2nC3 – nC3 – nC3
∴ Total number of ways to fail = 2n – 1. 2n ( 2n − 1) ( 2n − 2) 2n ( n − 1) ( n − 2)
∴ From question, 2n – 1 = 63; =

6 6
∴ 2n = 64 = 26 1
= n (n – 1) (3n) = n2 (n – 1).

∴ n = 6. 3
The correct option is (A) The correct option is (C)
61. Since the elements in A are in A.P., therefore, the number of 67. E H M O R T (alphabetical order)
1093 − 1 Number of words beginning with
elements in A = + 1 = 274.
4 E _ _ _ _ _ = 5!
Since, sum of equidistant terms in A.P. is equal to the sum of H _ _ _ _ _ = 5!
first and last term = 1 + 1093 = 1094.
M E _ _ _ _ = 4!
⇒ maximum number of elements in A, that add up to 1094
M H _ _ _ _ = 4!
274
= = 137. M O E _ _ _ = 3!
2
The correct option is (C) M O H _ _ _ = 3!

Objective_Maths_JEE Main 2017_Ch 7.indd 32 01/01/2008 04:20:41


Permutations and Combinations  7.33

M O R _ _ _ = 3! ( n − 1) ( n −2) 1 n ( n − 1) ( n −2)

=
M O T E _ _ = 2! 2 5 6
M O T H E R = 1! ⇒ n = 15
The correct option is (A)
∴ Rank of word MOTHER = 309 r 2
75. For 0 ≤ r ≤ 66, 0 ≤ <
The correct option is (D) 100 3
68. Factorizing the given number, we have 2 r
⇒ − < − ≤0
38808 = 23 ⋅ 32 ⋅ 72 ⋅ 11 3 100
Therefore, the total number of divisors 1 2 1 r 1
⇒ −
− <− − ≤−
= (3 +1) (2 + 1) (1 + 1) – 1 = 71. 3 3 3 100 3
But this includes the division by the number itself. ⎡ 1 r ⎤

⎢ − 3 − 100 ⎥ = – 1 for 0 ≤ r ≤ 66
Hence, the required number of divisors ⎣ ⎦
= 71 – 1 = 70. 67 r
Also, for 67 ≤ r ≤ 100,
≤ ≤1
The correct option is (A) 100 100
69. Other than 2, remaining five places are to be filled by 1 and 3 r 67
⇒ –1 ≤ –
≤−
∴ number of ways for five places 100 100
= 2 × 2 × 2 × 2 × 2 = 25 1 1 r 1 67
⇒ − − 1 ≤ − −
≤− −
For 2, selecting 2 places out of 7 = 7C2 3 3 100 3 100
∴ Required number of ways = 7C2 ⋅ 25. ⎡ 1 r ⎤
∴ ⎢ − −
⎥ = –2 for 67 ≤ r ≤ 100
The correct option is (B) ⎣ 3 100 ⎦
70. The required number of circles 100
⎡ 1 r ⎤
= (10C3 – 4C3) + 1 = 117.
Hence, ∑ ⎢⎣ − 3 − 100 ⎥⎦ = 67 (–1) + 2(–34) = –135.
The correct option is (C) r=0

71. Number of subsets The correct option is (C)


76. The following four cases are there
= 2n+1C0 + 2n+1C1 + … + 2n+1Cn = N (say)

(A) At least one one rupee coin and any number of other
∴ N = 22n+1 – N
coins is 2 × 4 × 5 = 40 ways.
⇒ 2N = 22n+1

(B) At least two 50 p. coins and any number of 25 p. coins =
⇒ N = 22n. 2 × 5 = 10 ways.
The correct option is (D) (C) One 50 p. coin + at least two 25 p. coins = 1 × 3 =
72. Starting with the letter A and arranging the other four letters, 3 ways.
there are 4! = 24 words. These are the first 24 words. Then, (D) Four 25 p. coins in one way only ∴ total = 54.
starting with G and arranging A, A, I and N in different ways,
4! The correct option is (A)
there are = 12 words, the 37th word starts with I and 77. The number of students answering at least r questions
2! 1! 1!
there are 12 words starting with I. Total goes up to 48 words. ­incorrectly is 2n–r.
Now, 49th word is NAAGI. ∴ The number of students answering exactly
The correct option is (A) r (1 ≤ r ≤ n – 1) questions incorrectly is 2n–r – 2n–(r + 1).
73. If we choose k (0 ≤ k ≤ n) identical objects, then we must Also, the number of students answering all questions
choose (n – k) distinct objects. This can be done in 2n+1Cn–k wrongly is 20 = 1.
ways. Thus, the required number of ways Thus, the total number of wrong answers is
n
1 (2n–1 – 2n–2) + 2 (2n–2 – 2n–3) + 3 (2n–3 – 2n–4) + …
=
∑ 2n+1Cn − k = 2n+1Cn + 2n+1Cn–1 + … + 2n+1C0
+ (n – 1) (21 – 20) + n (20)
k=0

=2 2n
. = 2n–1 + 2n–2 + … + 20 = 2n –1.

The correct option is (A) Now, 2n – 1 = 4095
74. The number of subsets of A containing exactly three e­ lements ⇒ 2n = 4096 = 212
is nC3 whereas the number of three subsets of A that contain ⇒ n = 12.
a1 is n–1C2. We are given, The correct option is (B)

n–1 20 n
C2 = ( C3)
100

Objective_Maths_JEE Main 2017_Ch 7.indd 33 01/01/2008 04:20:43


7.34  Chapter 7

78. d Also, at least one blue dye can be selected out of 4 blue
dyes in
c
C1 + 4C2 + 4C3 + 4C4 = 24 – 1
4
a ⎯⎯
→ 2 ways.
Again, 3 different red dyes can be selected in
d 3
C0 + 3C1 + 3C2 + 3C3 = 23 ways
a ∴ Required ways = (25 – 1) (24 – 1) (23) = 3720
b a d c ⎯⎯
→ 3 ways. The correct option is (C)
d a c 84. 4m = 2a + 3b + 5c = 2a + (4 – 1)b + (4 + 1)c

b c a 4m = 4k + 2a + (–1)b + (1)c

∴ a = 1, b = even, c = any number
a
a ≠ 1, b = odd, c = any number
c ⎯⎯
→ 3 ways.
∴ Required number = 1 × 2 × 5 × 4 × 3 × 5 = 70
The correct option is (B)
d ⎯⎯
→ 3 ways. 2n
2n − r + 1
Cr
85. We have, 2n
= (1)
∴ Total number of ways = 11.
Cr −1 r

The correct option is (D) 2n − r + 1
For 2nCr to be greatest
≥1
m r
79. S = ∑ n + r Ck ⇒ 2n – r + 1 ≥ r ⇒ 2r ≤ 2n + 1

r=0
1
= Ck + n+1Ck + n+2Ck + … + n+mCk
n ⇒ r ≤ n + (2)

2
n
=
Ck +1 + nCk + n+1Ck + n+2Ck + … + n+mCk – nCk+1 2n
Cr +1 2n − ( r + 1) + 1 2n − r
n+1 n+1 n+2 n+m n
From (1), = = .
=
Ck+1 + Ck + Ck + … + Ck – Ck+1 2n
Cr r +1 r +1
∴ S = n+m+1Ck+1 – nCk + 1 2n 2n − r


For Cr to be greatest ≤1
The correct option is (B) r +1
80. Total number of ways = nn ⇒ 2n – r ≤ r + 1 ⇒ 2r ≥ 2n – 1

Number of ways so that each person gets at least one thing 1
⇒ r ≥ n −  (3)
(here, exactly one thing) = n! 2
1 1
Given, nn – n! = 232 From (2) and (3), we get n – ≤r≤n+
2 2
11 – 1! = 0, 22 – 2! = 2, 33 – 3! = 21, 44– 4! = 232
⇒ r = n (since r is a positive integer)
∴ n = 4
Hence, 2nCr is greatest when r = n.
The correct option is (B)
The correct option is (C)
81. Now, n = 23 – 2 = 6
⎛ 2 1⎞ 86. The numbers are of three or four digits.
Also, m = 4 + 3 + 2 + 1 – 1 = 9 ⎜ as = ⎟
⎝ 4 2⎠ To find the number of even numbers of three digits—
∴ m – n = 3
The unit’s place must be filled by 0 or 2.
The correct option is (D)
∴ The number of even numbers of three digits (having or not
82. Number of solutions of the given equation is the same as the having 0 in hundred’s place) = 2P1 × 3P2.
number of solutions of the equation
But the number of even numbers of three digits having 0 in
x1x2x3x4 = 30 = 2 × 3 × 5 hundred’s place = 2P1 (Q unit’s place is naturally filled by 2
Here, x4 is there because if x1x2x3 = 15, then x4 = 2 and if and ten’s place by one of 1 and 3).
x1x2x3 = 5, then x4 = 6, etc. ∴ The number of even numbers of three digits
x4 is in fact a dummy variable. = 2P1 × 3P2 – 2P1.(1)
Each of 2, 3 and 5 will be a factor of exactly one of x1, x2, x3, Similarly, the number of even numbers of four digits
x4 in 4 ways.
= 2P1 × 3P3 – 2P2.(2)
∴ Required number = 43 = 64
Adding (1) and (2), we get the number of even numbers
The correct option is (A) greater than 100
83. Number of ways of selecting at least one green dye out of = (2P1 × 3P2 – 2P1) + (2P1 × 3P3 – 2P2)
5 different green dyes
= 2 × 3 × 2 – 2 + 2 × 3 × 2 × 1 – 2 × 1 = 20.
5
C1 + 5C2 + 5C3 + 5C4 + 5C5 = 25 – 1 The correct option is (A)

Objective_Maths_JEE Main 2017_Ch 7.indd 34 01/01/2008 04:20:45


Permutations and Combinations  7.35

87. Here, the available digits are 0, 1, 2, 3, 4, 5, 6, 7, 8, 9. 5!


(E) 3 alike, 2 alike = 2C1 × 2C1 ×
The numbers can be of one, two or three digits and in each of 3! 2!
= 2 × 2 × 10 = 40
them unit’s place must have 0 or 5 as they must be divisible
by 5. Hence, total number of ways
The number of numbers of one digit = 1 = 720 + 1800 + 400 + 360 + 40 = 3320.
(∵ 5 is the only number). The correct option is (B)
The number of numbers of two digits divisible by 5 = num- 90. The number of numbers of five digits, having 3 in unit’s
ber of all the numbers of two digits divisible by 5 – number place = 4P4 (number of arrangements of the remaining four
of numbers of two digits divisible by 5 and having 0 in ten’s digits)
place = 2P1 × 9P1 – 1, = 4! = 24.
(∵ unit’s place can be filled by either 0 or 5 in first category Similar is the case for other digits, i.e., each digit occurs 24
and only by 5 in the second category) times in the unit’s place.
= 2 × 9 – 1 = 17. Hence, the sum of the digits in unit’s place
The number of numbers of three digits divisible by 5 = num- = 24 × 3 + 24 × 4 + 24 × 5 + 24 × 6 + 24 × 7
ber of all the numbers of three digits divisible by 5 = number = 24 (3 + 4 + 5 + 6 + 7) = 24 × 25 = 600
of numbers of three digits divisible by 5 and having 0 in Similarly, the sum of the digits in each of the other places =
hundred’s place 600.
= 2P1 × 9P2 – 8P1 × 1 = 2 × 9 × 8 – 8 = 136. ∴ The required sum
∴ Required number of numbers = 600 × 10000 + 600 × 1000 + 600 × 100 + 600 × 10 +
= 1 + 17 + 136 = 154. 600 × 1
The correct option is (B) = 600 (10000 + 1000 + 100 + 10 + 1)
88. Suppose, the first two digits are 41, remaining four digits are = 600 × 11111 = 6666600.
to be chosen from 0, 2, 3, 5, 6, 7, 8, 9 so as to make all the The correct option is (B)
digits distinct. 3!
91. The number of numbers having 2 in unit’s place =
∴ Remaining four can be chosen in P (8, 4) ways. 2!
We have 8 letters and they are to be arranged at 4 places. (because the other three places are to be filled by 3, 3, 4 of
which two are identical).
First place can be arranged in 8 different ways.
The number of numbers having 3 in unit’s place = 3!,
Second place can be arranged in 7 different ways.
(because the other three places are to be filled by 2, 3, 4).
Third place can be arranged in 6 different ways.
3!
Fourth place can be arranged in 5 different ways. The number of numbers having 4 in unit’s place = ,
2!
Number of such telephone numbers (because the other three places are to be filled by 3, 2, 3 of
= 8 × 7 × 6 × 5 = 1680. which two are identical)
Now, considering the all given first two digits (41 or 42 or 46 ∴ Sum of the digits in unit’s place
or 64), total number of the telephone numbers 3! 3!
= × 2 + 3! × 3 + × 4 = 6 + 18 + 12 = 36.
= 1680 × 5 = 8400. 2! 2!
The correct option is (A) Similarly, the sum of the digits in other places is 36 each.
89. There are 11 letters in the word INDEPENDENT. ∴ Required sum = 36 × 1000 + 36 × 100 + 36 × 10 + 36 × 1
3N, 3E, 2D, I, P, T (6 types) = 36 (1000 + 100 + 10 +1) = 36 × 1111 = 39996.
Different possibilities of choosing 5 letters are The correct option is (A)
(A) All different 92. All the numbers are of five digits.
Number of ways = 6C5 × 5! = 6 × 120 = 720. The number of numbers having 1 in unit’s place = 4! = 24
(B) 2 alike, 3 different (Q the other four places are to be filled by 3, 5, 7, 9).
5! Similarly, the number of numbers having 3, 5, 7, or 9 in
Number of ways = 3C1 × 5C3 × = 3 × 10 × 60 = 1800
2! unit’s place is 24 in each case.
(C) 3 alike, 2 different
5! ∴ The sum of the digits in unit’s place for all the numbers
Number of ways = 2C1 × 5C2 × = 24 × 1 + 24 × 3 + 24 × 5 + 24 × 7 + 24 × 9
3!
= 2 × 10 × 20 = 400 = 24 (1 + 3 + 5 + 7 + 9) = 24 × 25 = 600.
(D) 2 alike, 2 alike, 1 different Similarly, sum of the digits in other places = 600 in every
5! case.
= 3C2 × 4C1 × = 3 × 4 × 30 = 360
2! 2! ∴ The required sum of all the numbers that can be made

Objective_Maths_JEE Main 2017_Ch 7.indd 35 01/01/2008 04:20:46


7.36  Chapter 7

= 600 × 10000 + 600 × 1000 + 600 × 100 + 600 × 10 + ∴ Number of ways to fill ten thousand’s place = 2P1 = 2
600 × 1 Number of ways to fill unit’s place = 2P1 = 2
= 600 (10000 + 1000 + 100 + 10 +1) = 600 × 11111 Number of ways to fill other three places = 63.
= 6666600. ∴ Required number of numbers = 2 × 2 × 63 = 864.
The correct option is (C) The correct option is (A)
93. All the numbers are of four digits and they do not have 0 in 96. When X and Y sit on the side facing the window—
thousand’s place.
First, we have to select 2 persons for the side facing the
The number of numbers having 0 in unit’s place = 3! ­window and 3 for the opposite side from the remaining 5.
(∵ the other three places are to be filled by 1, 2 or 3). The number of selections for this is equal to 5C2 × 3C3.
The number of numbers having 1 in unit’s place = 2P1 × 2P2 For each of the selections, the number of arrangements
(Q thousand’s place can be filled by one of 2, 3 and the = 4! × 3!.
remaining two places can be filled by the remaining two ∴ The number of ways to sit when X, Y are on the side
digits).
facing the window = 5C2 × 3C3 × 4! × 3!.
Similarly, the number of numbers having 2 or 3 in unit’s
When X and Y sit on the other side
place
As before, the number of ways to sit
= 2P1 × 2P2 in each case.
= 5C1 × 4C4 × 4! × 3!
Thus, the sum of the digits in unit’s place for all the numbers
∴ Required number of ways
= 3! × 0 + 2P1 × 2P2 × 1 + 2P1 × 2P2 × 2 + 2P1 × 2P2 × 3

= 5C2 × 3C3 × 4! × 3! + 5C1 × 4C4 × 4! × 3!
= 4 + 8 + 12 = 24.
5×4
Similarly, the sum of the digits in ten’s and hundred’s places = × 1 × 24 × 6 + 5 × 1 × 24 × 6 = 2160.
= 24 each. 2
The correct option is (B)
Now, the thousand’s place can have only 1 or 2 or 3.
97. In selection of fruits, fruits of the same type are treated as
Number of numbers having 1 in thousand’s place = 3!, (for
identical.
the other three places will be filled by 0, 2, 3).
Number of selections of any number of oranges from
Similarly, the number of numbers having 2 or 3 in ­thousands’s
4 oranges = 4 + 1.
place is 3! in each case.
Number of selections of any number of apples from 5 apples
∴ Sum of the digits in the thousands’s place for all the
= 5 + 1.
numbers = 3! × 1 + 3! × 2 + 3! × 3 = 6 + 12 + 18 = 36.
Number of selections of any number of mangoes from
∴ Required sum of all the numbers
6 mangoes = 6 + 1.
= 36 × 1000 + 24 × 100 + 24 × 10 + 24 × 1
∴ Number of selections of any number of fruits from the
= 36000 + 2400 + 240 + 24 = 38664. basket = (4 + 1) (5 + 1) (6 + 1) = 5 × 6 × 7 = 210.
The correct option is (A) But in one of these selections, there is one which contains
94. The thousand’s place cannot be filled by O. So the number of no fruits. Excluding this selection, the required number of
ways to fill the thousands’s place = 5. selections = 210 – 1 = 209.
The remaining three places can be filled by six digits in 63 The correct option is (C)
ways, as digits can be repeated. 98. In terms of prime factors 100! can be written as 2a ⋅ 3b ⋅ 5c ⋅ 7d
∴ The number of four digit numbers …
= 5 × 63 = 1080. (1) Now,  E2 (100!)
Now, the number of numbers of four digits that do not con-
⎡100 ⎤ ⎡100 ⎤ ⎡100 ⎤ ⎡100 ⎤ ⎡100 ⎤ ⎡100 ⎤
tain any repeated digit = 5P1 × 5P3 {∵ thousand’s place is to = ⎢
⎥+⎢ 2 ⎥+⎢ 3 ⎥+⎢ 4 ⎥+⎢ 5 ⎥+⎢ 6 ⎥
be filled by one of 1, 2, 3, 4, 5 and the remaining three places ⎣ 2 ⎦ ⎣2 ⎦ ⎣2 ⎦ ⎣2 ⎦ ⎣2 ⎦ ⎣2 ⎦
are to be filled by three of the remaining five digits including = 50 + 25 + 12 + 6 + 3 + 1 = 97

0}. But (1) contains numbers which contain no repeated dig- ⎡100 ⎤ ⎡100 ⎤
its as well as those which contain at least one repeated digit. and,  E5 (100!) = ⎢
⎥ + ⎢ 2 ⎥ = 20 + 4 = 24
⎣ 5 ⎦ ⎣5 ⎦
∴ The number of four digit numbers which contain at least
one repeated digit = 1080 – 5P1 × 5P3 100! = 297 ⋅ 3b ⋅ 524 ⋅ 7d …

= 1080 – 5 × 5 × 4 × 3 = 780.
= 273 ⋅ 3b ⋅ (2 × 5)24 ⋅ 7d …

The correct option is (C)
= 273 ⋅ 3b ⋅ (10)24 ⋅ 7d …
95. The numbers are of five digits having 4 or 5 in ten thousand’s
Hence, number of zeros at the end of 100! is 24.
place and 1 or 5 in unit’s place and the remaining digits are
The correct option is (C)
any of the given six digits.

Objective_Maths_JEE Main 2017_Ch 7.indd 36 01/01/2008 04:20:47


Permutations and Combinations  7.37

  99. In terms of prime factors 33! can be written as 2a ⋅ 3b5c ⋅ 7d Since relative order of the vowels and consonants remains
… unchanged, therefore, vowels will occupy only vowel’s
place and consonants will occupy only consonants’s place.
Now, E2 (33!)
Now, 6 consonants can be arranged among themselves in
⎡ 33 ⎤ ⎡ 33 ⎤ ⎡ 33 ⎤ ⎡ 33 ⎤ ⎡ 33 ⎤ 6!
= ⎢ ⎥ + ⎢ 2 ⎥ + ⎢ 3 ⎥ + ⎢ 4 ⎥ + ⎢ 5 ⎥ ways [since there are two p’s and two t’s] and five
⎣ 2 ⎦ ⎣2 ⎦ ⎣2 ⎦ ⎣2 ⎦ ⎣2 ⎦ 2! 2! 5!
= 16 + 8 + 4 + 2 + 1 = 31. vowels can be arranged among themselves in ways,
since a occurs thrice. 3!
Hence, the exponent of 2 in 33! is 31.
6! 5!
∴ Largest value of n is 31. ∴ Required number = . = 3600.
2! 2! 3!
The correct option is (B) The correct option is (A)
100. Number of non-negative integral solutions of the given 104. Let n be the number of students.
equation
Now, number of ways in which two students can be selected
= coefficient of x20 in out of n students is nC2.
(1 – x)–1 (1 – x)–1 (1 – x)–1 (1 – x4)– 1. ∴ number of pairs of students = nC2.
= coefficient of x20 in (1 – x)–3 (1 – x4)–1 But for each pair of students, number of cards sent is 2

= coefficient of x20 in (1 + 3C1x + 4C2 x2 + 5C3 x3 + 6C4 x4+ … (since if there are two students A and B, A will send a card
+ 10C8x8 … + 14C12 x12 + …+ 18C16 x16 + … + 22C20 x20 + …) to B and B will send a card to A).
∴ For nC2 pairs, number of cards sent = 2 ⋅ nC2.
× (1 + x4 + x8 + x12 + x16 + x20 + …)
According to the question, 2 ⋅ nC2 = 600
= 1 + 6C4 + 10C8 + 14C12 + 18C16 + 22C20 n( n − 1)
or, 2 ⋅ = 600
= 1+ 6C2 + 10C2 + 14C2 + 18C2 + 22C2 2!
2
⎛ 6.5 ⎞ ⎛ 10.9 ⎞ ⎛ 14.13 ⎞ ⎛ 18.17 ⎞ ⎛ 22.21⎞ or, n – n – 600 = 0
= 1+ ⎜ ⎟ +⎜ + + +
⎝ 1.2 ⎠ ⎝ 1.2 ⎟⎠ ⎜⎝ 1.2 ⎟⎠ ⎜⎝ 1.2 ⎟⎠ ⎜⎝ 1.2 ⎟⎠ or, (n – 25) (n + 24) = 0
∴ n = 25, –24
= 1 + 15 + 45 + 91 + 153 + 231 = 536.
But n ≠ –24
The correct option is (B)
∴ n = 25.
101. Let r consecutive positive integers be
The correct option is (C)
m, m + 1, m + 2, … (m + r – 1), where m ∈ N
∴ Product = m (m + 1) (m + 2) … (m + r – 1) ⎛ m⎞ ⎛ m + 1⎞ ⎛ m + 2⎞ ⎛ n⎞
105. ⎜⎝ m⎟⎠ + ⎜⎝ m⎟⎠ + ⎜⎝ m⎟⎠
+… + ⎜ ⎟
⎝ m⎠
( m − 1)! m ( m + 1) ( m + 2)…( m + r − 1)
=
( m − 1)! ⎛ m + 1⎞ ⎛ m + 1⎞ ⎛ m + 2⎞ ⎛ n⎞
= ⎜ ⎟ +⎜ ⎟ +⎜ ⎟ +… + ⎜ ⎟
( m + r − 1)! ( m + r − 1)! ⎝ m + 1⎠ ⎝ m⎠ ⎝ m⎠ ⎝ m⎠
= = r !.
( m − 1)! r !( m − 1)! ⎛ m + 2⎞ ⎛ m + 2⎞ ⎛ n⎞
= ⎜ + +…+ ⎜ ⎟
= r!. m + r – 1Cr ⎝ m + 1⎟⎠ ⎜⎝ m⎟⎠ ⎝ m⎠
m+r–1
which is divisible by r!(∵ Cr is a natural number) ⎛ m + 3⎞ ⎛ m + 3⎞ ⎛ n⎞
= ⎜ + + ... + ⎜ ⎟
The correct option is (A) ⎝ m + 1⎟⎠ ⎜⎝ m⎟⎠ ⎝ m⎠
102. The number of triplets of positive integers which are solu-
tions of x + y + z = 100   

= Coefficient of x100 in (x + x2 + x3 + …)3 ⎛ n ⎞ ⎛ n⎞ ⎛ n + 1⎞


= ⎜ + = ⎜

= Coefficient of x 100 3 –3
in x (1 – x) ⎝ m + 1⎟⎠ ⎜⎝ m⎟⎠ ⎝ m + 1⎟⎠
100
= Coefficient of x in ⎛ n ⎞ ⎛ n − 1⎞ ⎛ n − 2⎞ ⎛ m⎞ ⎛ n + 1⎞
Thus, ⎜ ⎟ + ⎜ + +…+ ⎜ ⎟ = ⎜ .
⎡ ( n + 1) ( n + 2) n ⎤ ⎝ m⎠ ⎝ m⎟⎠ ⎜⎝ m⎟⎠ ⎝ m⎠ ⎝ m + 1⎟⎠
x 3 ⎢1 + 3 x + 6 x 2 + … + x + …⎥
⎣ 2 ⎦ The correct option is (B)
(97 + 1) (97 + 2) 106. A number of the seven digits will be of the form.
= = 49 × 99 = 4851.
2 ax1 x2 x3 x4 x5 0, ax1 x2 x3 x4 x5 1, ax1 x2 x3 x4 x5 2,
The correct option is (C) ax1 x2 x3 x4 x5 3, … ax1 x2 x3 x4 x5 9
103. There are eleven letters in the word ‘Pataliputra’ and there where a ∈ {1, 2, 3, … 9}
are two p’s, two r’s, three a’s and four other different letters.
and x1, x2, x3, x4, x5 ∈ {0, 1, 2, 3, …9}.
Number of consonants = 6, number of vowels = 5.

Objective_Maths_JEE Main 2017_Ch 7.indd 37 01/01/2008 04:20:49


7.38  Chapter 7

Since sum of the digits should be even, therefore, ⎛ 1 − x5 ⎞


4
4
if a + x1 + x2 + x3 + x4 + x5 is an even number, then the digit = coefficient of x in ⎜ ⎟
⎝ 1− x ⎠
at units place must be 0, 2, 4, 6, 8 and if a + x1 + x2 + x3 +
x4 + x5 is an odd number, then the digit at units place will be = coefficient of x4 in [(1 – x5)4 (1 – x)– 4]
1, 3, 5, 7, 9. = coefficient of x4 in [(1 – 4x5 + 6x10 – …) (1 – x)– 4]
∴ Required number = 9 × 10 × 10 × 10 × 10 × 10 × 5
= coefficient of x4 in (1 – x)–4
= 9 × 105 × 5 = 45 × 105. 4⋅5⋅6⋅7
The correct option is (B) = 7C4 = = 35
4!
107. The word BAC cannot be spelt if the m selected coupons do The correct option is (B)
not contain atleast one of A, B and C. 110. Required number = coefficient of x30 in (x2 + x3 + … + x16)8
Number of ways of selecting m coupons which are A or
B = 2m. = coefficient of x30 in x16 (1 + x + … + x14)8
8
This also includes the case when all the m coupons are A or ⎛ 1 − x15 ⎞
all are B. = coefficient of x14 in ⎜ ⎟
⎝ 1− x ⎠
Number of ways of selecting m coupons which are B or
C = 2m. = coefficient of x14 in (1– x)– 8 = 21C14 = = 116280.
This also includes the case when all the m coupons are B or The correct option is (C)
all are C. 111. The candidate must score 150 marks.
Number of ways of selecting m coupons which are C or ∴ Required number
A = 2m. = coefficient of x150 in (1 + x + … + x50)3 (1 + x + … + x100)
This also includes the case when all the m coupons are C or 3
⎛ 1 − x 51 ⎞ 1 − x101
all are A. = coefficient of x 150
in ⎜ ⎟
Number of ways of selecting m coupons when all are ⎝ 1− x ⎠ 1− x
A = 1m.
= coefficient of x150 in (1 – x51)3 (1 – x101) (1 – x)–4
Number of ways of selecting m coupons when all are
B = 1m. = coefficient of x150 in (1 – 3x51 + 3x102 – x153)
Number of ways of selecting m coupons when all are  (1 – x101) (1 – x)–4
C = 1m.
[leaving terms containing powers of x greater than 150]
∴ Required number = 2m + 2m + 2m – (1m + 1m + 1m) = coefficient of x150 in (1 – x)–4 – 3. coefficient of x99 in
= 3 ⋅ 2m – 3 ⋅ 1m = 3 (2m – 1). (1 – x)–4 + 3 coefficient of x48 in (1 – x)–4 coefficient of x49
The correct option is (A) in (1 – x)–4
108. First Method: Number of cross marks in 153
= C150 – 3 ⋅ 102C99 + 3 ⋅ 51C48 – 52C49.
1st row 2nd row 3rd row Number of ways
(Number of selections of squares) 153 ⋅ 152 ⋅ 151 102 ⋅ 101 ⋅ 100
= − 3⋅
1 3 2 2C1 × 4C3 × 2C2 = 8 6 6
1 4 1 2C1 × 4C4 × 2C1 = 4 51 ⋅ 50 ⋅ 49 52 ⋅ 51 ⋅ 50
+ 3 · −
2 2 2 2C2 × 4C2 × 2C2 = 6 6 6
2 3 1 2C2 × 4C3 × 2C1 = 8 = 110556
∴ Required number = 8 + 4 + 6 + 8 = 26 The correct option is (C)
Second Method: 112. Any number between 1 and 1000000 must be of less than
Number of ways of putting six cross marks in 8 squares seven digits. Therefore, it must be of the form
when there is no restriction = 8C6 = 28. a1 a2 a3 a4 a5 a6
Number of ways when the six cross marks are put in first where a1, a2, a3, a4, a5, a6 ∈ {0, 1, 2, …, 9}
and second rows = 6C6 = 1. According to question, sum of the digits = 18
Number of ways when the six cross marks are put in second Thus, a1 + a2 + a3 + a4 + a5 + a6 = 18
and third rows = 6C6 = 1. where 0 ≤ ai ≤ 9, i = 1, 2, 3, …, 9.
∴ Required number = 28 – 2 = 26. Required number
The correct option is (A) = coefficient of x18 in (1 + x + x2 + … + x9)6
109. Required number = coefficient of x16 in (x3 + x4 + … + x7)4 6
⎛ 1 − x10 ⎞

= coefficient of x16 in x12 (1 + x + … + x4) = coefficient of x18 in ⎜ ⎟
⎝ 1− x ⎠

Objective_Maths_JEE Main 2017_Ch 7.indd 38 01/01/2008 04:20:51


Permutations and Combinations  7.39


= coefficient of x18 in [(1 – x10)6 (1 – x)–6] But this includes one case where there is no teeth.
18 6 10 –6 ∴ Required number of ways = 232 – 1.
= coefficient of x in [(1 – C1 x ) (1 – x) ]
The correct option is (B)
[leaving terms containing powers of x greater than 18]
116. For any group of five scientists there must be at least one
= coefficient of x18 in (1 – x)– 6 – 6C1. coefficient of x8 in
lock they can not open. Moreover, for any two different
(1 – x)– 6
groups of five scientists, there must be two locks they can-
6 + 18 – 1
= C18 – 6 ⋅ 6 + 8 – 1C18 = 23C5 – 6 ⋅ 13C8 not open, because if both groups cannot open the same
23 ⋅ 22 ⋅ 21 ⋅ 20 ⋅ 19 13 ⋅ 12 ⋅ 11 ⋅ 10 ⋅ 9 lock, there is a group of six scientists among these two
= − 6⋅ groups who will not be able to open the cabinet. Thus, at
120 120
least
= 33649 – 7722 = 25927 11 × 10 × 9 × 8 × 7
11
The correct option is (A) C6 = = 11 × 3 × 2 × 7
1× 2 ×3× 4 × 5
113. Given, x + y + z + u + t = 20 (1) = 462 locks are needed.
x + y + z = 5 (2) The correct option is (C)
Given, system of equations is equivalent to 117. The total number of permutations of the digits of given
x + y + z = 5 (3) number is
and,  u + t = 15 (4) 7!
= 420
Number of non-negative integral solutions of equation (3) 2!3!
But when 0 occupies the first position, the number becomes
= n + r – 1Cr = 3 + 5 – 1C5 = 7C5,
a six-digit number which are not acceptable, since such a
Number of non-negative integral solutions of equation (4) number will become less then 106.
= n + r – 1Cr = 2 + 15 – 1C15 = 16C15 The total number of such six-digit numbers is
6!
∴ Required number = 7C5 × 16C15 = 336. = = 60
2!3!
The correct option is (A)
Hence, the required number is 420 – 60 = 360.
114. Let w be a non-negative integer such that
The correct option is (A)
3x + y + z + w = 30
n!( 21 − n)! 1 21!
Let a = x – 1, b = y – 1, c = z – 1, d = w, then 118. n! (21 – n)! = 21! = 21! 21 = 21
21! Cn Cn
3a + b + c + d = 25, where a, b, c, d ≥ 0 (1)
Clearly, 0 ≤ a ≤ 8. If a = k, then For minimum value, 21Cn is maximum.
b + c + d = 25 – 3k(2) Maximum value of 21 Cn = 21C 21 − 1 = 21C10
Number of non-negative integral solutions of equation (2) 2

n+r–1 3 + 25 – 3k – 1 21! 21!


= Cr = C25 – 3k ∴ Minimum value = 21
= × 10!11! = 10!11!
C10 21!
( 27 − 3k ) ( 26 − 3k )
= 27 – 3k
C25 – 3k = 27 – 3k
C2 = The correct option is (B)
2
119. The maximum number of oranges that a child can get =
3
= (3k2 – 53k – 234) 20 – 3 = 17.
2
3 8 Thus, the problem is equivalent to finding the number of
∴ Required number = ∑ (3k 2 − 53k + 234)
2k=0
integral solutions to the equation
x1 + x2 + x3 + x4 = 20
3 ⎡ 8 × 9 × 17 8×9 ⎤ where 1 ≤ x1, x2, x3, x4 ≤ 17, and x1, x2, x3, x4 denote the
= 3⋅ − 53 + 234 × 9⎥ = 1215.
2 ⎢⎣ 6 2 ⎦ number of oranges given to the four children.
The correct option is (B) Hence, the required number of ways is
115. In the mouth, there are 32 places for the teeth. For each = coefficient of x20 in (x + x2 + x3 + … + x17)4
place, two choices are possible, i.e., either tooth or not.
= coefficient of x16 in (1 + x + x2 + … + x16)4
Choices for 1 teeth = 2
= coefficient of x16 in (1 – x17)4 (1 – x)–4
Similarly choices for 2 teeth = 2 × 2 = 22


Choices for 3 teeth = 2 × 2 × 2 = 23 = coefficient of x16 in (1 – x)–4
16 + 4 –1

…………………………………….. = C16
…………………………………….. 19
= C3 = 969.
Hence, choices for 32 teeth = 232 The correct option is (B)

Objective_Maths_JEE Main 2017_Ch 7.indd 39 01/01/2008 04:20:53


7.40  Chapter 7

120. The smallest number, which can occur in the middle is 4. According to given condition 3n ≥ 900 ⇒ 3n–2 ≥ 100.
The number of numbers with 4 in the middle = 4P4 – 3P3 ∴ n – 2 ≥ 5 ⇒ n ≥ 7.
(∴ The other four places are to be filled by 0, 1, 2 and 3, and The correct option is (C)
a number can not begin with 0) 123. Let b = a + d and c = a + 2d.
Similarly, the number of numbers with 5 in the middle = Given: a + a + d + a + 2d = 21 ⇒ a + d = 7
5
P4 – 4P3, etc.
∴ a + c = 14 and b = 7
∴ The required number of numbers
Now, the number of positive integral solutions of a + c =
= (4P4 – 3P3) + (5P4 – 4P3) + (6P4 –5P3) + … + (9P4 – 8P3) 14 is equal to coefficient of x14 in (x + x2 + x3 +….)2 =
9
Coefficient of x12 in (1 + x + x2…)2 = 13 C12 = 13.
= ∑ ( n P4 − n −1P3 ) The correct option is (A)
n=4

The correct option is (A) 124. A mixed doubles game involves two males and two females.
121. Let D denotes Delhi and I denotes Indore. Two males can be chosen from n males in nC2 ways. Having
chosen two males, now 2 females are to be chosen from
S1 S2 S3 S4 S5 S6 S7 S8 S9 (n –2) females leaving the wives of the two already chosen
D I males. This can be done in n –2C2 ways.
For S1, 9 different tickets are available, one for each of the Hence, the required number of ways
remaining 9 stations. Similarly, for S2, 8 different tickets are = n C2. n –2 C2.2
available and so on. [for every choice of 2 males and 2 females,
Therefore, total number of 9 different tickets  they can be paired in 2 ways].
= 9 + 8 + 7 + 6 + 5 + 4 +3 +2 +1 = 45 n( n − 1) ( n − 2)( n − 3)
Thus, the six different tickets must be any six of these 45 = . .2
2 2
and there are clearly as many different sets of 6 tickets as
n( n − 1)( n − 2)( n − 3) 1 n
they are combinations of 45 things taken 6 at a time. = = . P4
2 2
Hence, the required number = 45C6.
The correct option is (C)
The correct option is (C)
122. Using 2, 5 and 7 with repetition each place of n digit num-
ber can be chosen in 3 ways. Hence, total number of n- digit
numbers = 3 × 3 ×3 … n times = 3n.

More than One Option Correct Type


125. We know that join of any two points gives a line. 126. We have 770 = 2 × 5 × 7 × 11. We can assign 2 to x1 or x2 or
The number of lines obtained from 10 points, no three of x3 or x4. That is 2 can be assigned in 4 ways.
10 × 9 Similarly, each of 5, 7 or 11 can be assigned in 4 ways.
which are collinear = 10C2 = = 45
2 ×1 Thus, the number of positive integral solutions of
4×3 x1 x2 x3 x4 = 770 is 44 = 28 = 256.
Lines obtained from 4 points = 4C2 = =6
2 ×1 The correct option is (B), (C) and (D)
∴ Number of lines lost due to 4 collinear points 1 1 x
127. For 1 ≤ x ≤ 33, < + < 1 therefore,
= 6 – 1 = 5 3 3 50
∴ Required number of lines = 45 – 5 = 40. ⎡1 x ⎤
⎢ + ⎥ = 0 for 1 ≤ x ≤ 33.
Also, We know that any triangle can be obtained by joining ⎣ 3 50 ⎦
any three points not in the same straight line. 1 x 4
For 34 ≤ x ≤ 50, 1 < + <
∴ Number of triangles obtained from 10 points, no three 3 50 3
⎡1 x ⎤
of which are collinear ⇒ ⎢ + ⎥ = 1 for 34 ≤ x ≤ 50
⎣ 3 50 ⎦
10 × 9 × 8
= 10C2 = = 120. Thus, E = 17.
3 × 2 ×1
The correct option is (B) and (D)
Triangle obtained from 4 points = 4C3 = 4
128. Given numbers are 1, 2, 3, … n.
∴ Number of triangles lost due to 4 collinear points = 4.
Let the three selected numbers in A.P. be a, b, c, then
∴ Required number of triangles = 120 – 4 = 116.
a+c
The correct option is (A) and (B) b = or a + c = 2b (1)
2

Objective_Maths_JEE Main 2017_Ch 7.indd 40 01/01/2008 04:20:54


Permutations and Combinations  7.41

From (1) it is clear that a + c should be an even integer. This 2n! ( n + 1)( n + 2)…( 2n)
130. N = =
is possible only when both a and c are odd or both are even. n! n! n!
Case I. When n is even. Let n = 2m ∴ n!N = (n + 1) (n + 2) … (2n)
The number of odd numbers = m Now,  p/(n + 1) (n + 2) … (2n)(Q n < p < 2n)
and number of even numbers = m ∴ p/n! N
∴ number of selections of a and c from m odd numbers = ∴ p/N.(∵ n < p < 2n ⇒ p does not divide n!)
m
C2 If possible, let p2/N.
Number of selections of a and c from m even numbers = ⇒ p2/n! N
m
C2
m ⇒ p2/(n + 1) … (p – 1)p (p + 1) … (2n)
∴ Number of ways in this case = 2 ⋅ C2 = m (m – 1)
⇒ p/(n + 1) … (p –1) (p + 1) … (2n)
n⎛ n ⎞ n ( n − 2) This is impossible.
= ⎜ − 1⎟ = .
2⎝ 2 ⎠ 4 ∴ p2 does not divide N.
Case II. When n is odd. Let n = 2m + 1 The correct option is (A) and (D)
Then, number of odd numbers = m + 1 131. Let A, B,C be the objects selected and x1, x2, x3, x4 be the
and number of even numbers = m number of objects on either side of the selected objects as
∴ Required number in this case = m + 1C2 + mC2 shown below
2 x1 (A) x2 (B) x3 (C) x4
( m + 1) m m ( m − 1) ⎛ n − 1⎞
= + = m2 = ⎜ We have,

2 2 ⎝ 2 ⎟⎠
1 x1 + x2 + x3 + x4 = n– 3
= ( n − 1) 2 . where x1, x4, ≥ 0 and x2, x3 ≥ 1.
4
The correct option is (A) and (B) Hence, the required number of ways
129. Two straight lines can intersect in at most one point. = coefficient of xn–3 in (1 + x + x2 + …)2 (x + x2 + x3 + …)2

Therefore, maximum number of points of intersection of m = coefficient of xn –5 in (1 + x + x2 + …)4
lines = mC2 × 1 = mC2
= coefficient of xn –5 in (1 – x)–4
Again, two circles can intersect in at most two points.
n – 5 + 4 – 1
= Cn–5 = n–2Cn–5 = n–2 C3.
∴ Maximum number of points of intersection of n
The correct option is (A) and (B)
­circles = nC2 × 2 = 2nC2
132. We have, r – 1 ≥ 0, r ≤ n + 1.
A line can intersect a circle in at most 2 points.
∴ Maximum number of points of intersection of m lines ∴ 1 ≤ r ≤ n +1
and n circles = (mn)2 = 2mn 1 r
⇒ ≤ ≤ 1.
∴ N = mC2 + 2 nC2 + 2mn n +1 n +1
= mC2 + nP2 + 2mn n! r !( n + 1 − r )!
Also,  k2 – 8 = ⋅
m n ( r − 1)!( n − r + 1)! ( n + 1)!
(A) N – C2 = P2 + 2mn r
= n(n – 1) + 2mn = n (n –1 + 2m) =
n +1
∴  n / (N – mC2). 1
Thus,  ≥ k2 − 8 ≤ 1
(B) N – nP2 = mC2 + 2mn n +1
m( m − 1) 1
= + 2mn ⇒ 8 < + 8 ≤ k2 ≤ 9
2 n +1
⎡m −1 ⎤ ⇒ 8 < k2 ≤ 9
=m⎢ + 2n⎥
⎣ 2 ⎦ ⇒ –3 ≤ k < – 2 2
If m is even, then (m – 1) is not divisible by 2.
∴  in general m does not divide (N – nP2). or 2 2 <k≤3
(C) N – mC2 = nP2 + 2mn ⇒ k ∈ [–3, –2 2 ) or k ∈ (2 2 , 3]
= 2nC2 + 2mn The correct option is (B) and (D)
= 2(nC2 + mn) 133. Let x5 be such that x1 + x2 + x3 + x4 + x5 = n. We now seek
= N – mC2 is an even integer. the non-negative integral solution of x1 + x2 + x3 + x4 +
(D) N – C2 –nP2 = 2mn is an even integer.
m x5 = n. The number of required solutions = n + 4Cn = n + 4C4.
The correct option is (A), (C) and (D) The correct option is (A) and (B)

Objective_Maths_JEE Main 2017_Ch 7.indd 41 01/01/2008 04:20:57


7.42  Chapter 7

Passage Based Questions


134. There can be two cases Hence, the required number of ways = 16800–1800 = 1500.
(1)  1, 1, 1, 1, 3 (2)  1, 1, 1, 2, 2 The correct option is (B)
Case 1: A box (in which 3 balls can be put) is selected in 139. Required number of ways = 4 + 3 – 1C3 – 1
5
C1 ways. = 6C2 = 15

Now, 3 balls can be selected in 7C3 ways and remaining The correct option is (A)
4 balls can be arranged in 4! ways.
140. Since pots are identical, there will be 4 cases (4, 0, 0),
Hence, total number of ways = 5C1 × 7C3 × 4! (3, 1, 0), (2, 2, 0) and (2, 1, 1) but since all coins are differ-
Case 2: 2 boxes (in which 2–2 balls can be put) are selected ent hence selection of coins matters.
in 5C2 ways. Therefore, for the first case number of selections = 4C4 = 1
Now, any two balls can be selected in 7C2 ways and for the For the second case number of selections = 4C3 × 1C1 = 4
other selected box, 2 balls can be selected from the remain-
For the third case number of selections
ing 5 balls in 5C2 ways and the remaining balls can be
4
­distributed in 3! ways. C2 × 2C2
= =3
Hence, the total number of ways = 5C2 × (7C2 × 5C2) × 3! 2!
∴ The required number of ways For the fourth case number of selections
= (5C1 × 7C3 × 4!) + (5C2 × 7C2 × 5C2 × 3!) = 16800 4
C2 × 2C1 × 1C1
= =6
The correct option is (B) 2!
Hence, the total number of distributions
135. Required number of ways = 7 + 5 – 1C5 – 1
= 1 + 4 + 3 + 6 = 14
= 11C4 = 330 The correct option is (A)
The correct option is (C) 141. Since no box is empty and all pots are identical, so the pos-
136. One ball can be selected from the box 2 in 7C1 ways and two sible case is (1, 1, 2). But since all the coins are different,
balls can be selected out of remaining 6 balls from the box the 2 balls can be selected in 4C2 ways and rest can be put
4 in 6C2 ways and the remaining 4 balls can be distributed 2
C1 × 1C1
in 3 boxes in 34 ways. in ways.
2!
Hence, the required number of ways
Hence, the required number of distribution
= 7C1 × 6C2 × 34 = 8505
4
The correct option is (B) C2 × 2C1 × 1C1
= =6
137. Ball 2 can be distributed between 2 boxes (viz., box 2 and 2!
box 4) in 2 ways. The correct option is (B)
Now, the remaining 6 balls can be distributed in any of the 142. Since no pot is empty and all coins are identical the possible
5 boxes in 56 ways. case is (1, 1, 2). But since all three pots are different hence,
Hence, the required number of ways = 2 × 56 ways a pot (which contains 2 coins together) can be selected in
3
= 31250 ways C1 ways.
The correct option is (B) Hence, the required number of distribution
138. Required number of ways = 3C1 × 1 = 3.
= Total number of ways – number of ways in which ball 2 The correct option is (B)
and ball 4 are together. 143. The possible arrangements are as follows:
Here, total number of ways = 16800. (4, 0, 0) → Can be done in 2 ways, i.e., 4 balls can be put
in either one of the two identical pots or can
Now, consider ball 2 and ball 4 are together and this
be put in different pot.
arrangement is assumed to be a single ball.
(3, 1, 0) → Can be done in 3 ways, either distinct pot can
Thus, we have 6 balls to be distributed into 5 boxes, which
be filled up with 3 balls, or 1 ball or remained
can be done as 1, 1, 1,2 in 5C1 × 6C2 × 4! ways = 1800.
5 empty.
C1 → Number of ways of selecting one box in which two
(2, 2, 0) → Can be done in 2 ways, either distinct pot can
balls are kept together.
be filled by 2 balls or remained empty
6C2 → Number of ways of selecting 2 balls out of 6 balls.
(2, 1, 1) → Can be done in 2 ways either distinct pot can
4! → Number of ways of distribution of remaining 4 balls be filled up with 1 ball or 2 ball.
in remaining 4 boxes.

Objective_Maths_JEE Main 2017_Ch 7.indd 42 01/01/2008 04:20:57


Permutations and Combinations  7.43

Hence, the required number of ways = 2 + 3 + 2 + 2 = 9 Hence, the total number of arrangements = 18 + 3 = 21
The correct option is (C) Thus, the required number of ways
144. The possible arrangements are as follows: = 3 + 18 + 12 + 21 = 54 ways
(4, 0, 0) → Can be done in 3 ways, let the 4 coins be A, The correct option is (C)
A, B and C then all these 4 coins as a single 145. In the first round, total number of matches
packet of coins can be arranged in 3 different
pots in 3 ways 8×7
= 2 × 8C2 = 2 × = 56
(3, 1, 0) → Can be done in 18 ways, out of A, B and C we 1× 2
can select one coin in 3 ways i.e., either A or In the second round, total number of matches
B or C. = 2 (2) = 4 [In each group say I–III, II–IV]
Now, we can arrange this selected coin in any 3 pots in Number of semi-finals = 2
3 ways and the remaining 3 coins as a single packet of coins
Then, final = 1
can be arranged in remaining 2 pots in 2 ways.
Total number of matches = 56 + 4 + 2 + 1 = 63
Hence, the required number of ways = 3 × 3 × 2 = 18
The correct option is (C)
(2, 2, 0) → Can be done in 12 ways
There are two possible cases: 146. Each team will play 7 matches and so any team can win any
(i) (A, A), (B, C) (ii) (A, B), (A, C) number.
In each of two cases we assume that there are two packets If matches between 0 to 7 i.e., (0, 1, 2, 3, 4, 5, 6, 7)
of coins which can be arranged in 3 × 2 = 6 ways. Four teams will be selected (7, 6, 5, 4)
Since there are two cases, hence the total number of Thus, team which wins only 3 matches will be out of the
required ways = 6 × 2 = 12 first round.
(2, 1, 1) → Can be done in 21 ways The correct option is (C)
There are 4 possible cases: 147. Clearly, minimum number of matches that a team must win
  (i) (A, A), (B), (C)   (ii)  (A, B), (A), (C) in order to qualify for second round is 4.
(iii) (A, C), (A), (B) (iv) (B, C), (A), (A) The correct option is (A)
For the first 3 cases in each case all the 3 packets of coins 148. In second round, it has to win one match.
can be arranged in 3! ways. Then, one in semi-final and one final.
Hence, the number of arrangements = 3 × 3! = 18 ∴ total number of matches = 3
Now, in the fourth (iv) case two coins are identical so the The correct option is (D)
third packet of coins can be arranged in 3 pots in 3 ways.

Match the Column Type


149.
  (I) The total number of numbers that can be formed with (ii) 
If oldest is excluded, then we are to select
the digits 3, 4, 5, 6 taken all at a time 5 ­candidates from 9 which can be done in
= P (4, 4) = 4! = 24. 9 9! 9×8×7×6
C5 = = = 126 ways
Each of the digits 3, 4, 5, 6 occurs in 3! 5! 4! 4 × 3 × 2 ×1
= 3 × 2 = 6 times in unit’s place. Hence, the total number of ways in which committee
 ∴  Sum of the digits in the unit’s place of all the can be formed
numbers = 126 + 70 = 196.
= (3 + 4 + 5 + 6) × 6 = 18 × 6 = 108. The correct option is (C)
The correct option is (B) ∵ 9600 = 27 × 3 × 52
(III) 

(II)  There are two different ways of forming the committee ∴  Number of divisors = (7 + 1) × (1 + 1) × (2 + 1)
 (i)  oldest may be included = 8 × 2 × 3 = 48.
(ii)  oldest may be excluded The correct option is (D)
 (i) If oldest is included, then youngest has to be ⎧⎪ Coefficient of x10 ⎫⎪
excluded and we are to select 4 candidates out of   (IV)  Required ways = ⎨ ⎬
2 4
8. This can be done in ⎩⎪in (1 + x + x + …) ⎭⎪
4
8! 8×7×6×5 ⎛ 1 ⎞
8
C4 = = = 70 ways. = Coefficient of x10 in ⎜
4! 4! 4×3× 2 ⎝ 1 − x ⎟⎠

Objective_Maths_JEE Main 2017_Ch 7.indd 43 01/01/2008 04:20:58


7.44  Chapter 7

= Coefficient of x10 in (1 – x)– 4 The number of triangles = number of selections of 2


  (IV) 
lines from the (n – 1) lines which are cut by the last
⎛ 5.4 2 4.5.6 3 ⎞
= Coefficient of x10 in ⎜1 + 4 x + x + x + …⎟ line
⎝ 2! 3! ⎠
( n − 1)! ( n − 1) ( n − 2)
= Coefficient of x10 in (1 + 4C1x + 5C2x2 + 6C3x3 = n – 1C2 = =
2!( n − 3)! 2
+ 7C4x4 + 8C5x5 + 9C6x6 + … 13C10x10) The correct option is (B)
n–2
13.12.11 151.     (I)  Cr + 2 · n – 2Cr – 1 + n – 2Cr – 2
∴  Required ways = 13C10 = = 286
3.2.1 = (n – 2Cr + n – 2Cr – 1) + (n – 2Cr – 1 + n – 2Cr – 2)

The correct option is (A)
= n – 1Cr + n – 1Cr – 1
150.   (I) The number of diagonals + number of sides = number
of selections of two vertices from n vertices = nCr(Q nCr – 1 + nCr = n + 1Cr).
∴  the number of diagonals = nC2 – n The correct option is (D)
n
n ( n − 1) n2 − n − 2n n ( n − 3)
= −n = =   (II)  mCr + 1 + ∑ k
Cr
2 2 2 k=m

The correct option is (D) = mCr + 1 + mCr + m + 1Cr + … + n – 1Cr + nCr


(II) 
In general, we know that for the distribution equation = m + 1Cr + 1 + m + 1Cr + … + n – 1Cr + nCr

x1 + x2 + x3 + … + xr = n
= m + 2Cr + 1 + m + 2Cr + … + n – 1Cr + nCr
 The number of ways in which n things can be distrib-
   
uted among r in such a way each can receive none,
one or more or all of n items are n+r–1Cr–1 = nCr + 1 + nCr = n + 1Cr + 1
∴  For the distribution equation The correct option is (B)

x1 + x2 + x3 + … + xn ≤ n (III)  We have,
n
let required ways = W Cn – r + 3 ⋅ nCn – r + 1 + 3 ⋅ nCn – r + 2 + nCn – r + 3 = xCr

⎧ Number of ways of ⎫ ⇒ 


(nCn – r + nCn – r + 1) + 2 (nCn – r + 1 + nCn – r + 2) +
⎪ ⎪ (nCn – r + 2 + nCn – r + 3) = xCr
⇒  W = ⎨ distributing ⎬
⎪ 1 item ⎪ ⇒  n+1
Cn – r + 1 + 2 ⋅ n + 1Cn – r + 2 + n + 1Cn – r + 3 = xCr
⎩ ⎭
⇒ (n + 1Cn – r + 1 + n + 1Cn – r + 2) + (n + 1Cn – r + 2 +
⎧ Number of ways of ⎫ ⎧ Number of ways of ⎫ n+1
⎪ ⎪ ⎪ ⎪ Cn – r + 3) = xCr
+⎨ distributing ⎬ + … + ⎨ distributing ⎬ n+2
⎪ 2 items ⎪ ⎪ n items ⎪ ⇒  Cn – r + 2 + n + 2Cn – r + 3 = xCr
⎩ ⎭ ⎩ ⎭
n+3
⇒  Cn – r + 3 = xCr
= 1+n–1Cn–1 + 2+n–1Cn–1 + … + n+n–1Cn –1
n+3
= nCn–1 + n+1Cn–1 + … + 2n – 1Cn –1 ⇒  Cr = xCr(Q nCr = nCn – r)

∴  x = n + 3.
= (nCn–1 + nCn) + n+1Cn–1 + … + 2n–1Cn–1
The correct option is (A)
= {(n+1Cn + n+1Cn–1) + … + 2n–1Cn–1} – nCn
  (IV)  Total number of different things = n
----------------------------------
The number of ways taken 1 at a time = nP1 = n
----------------------------------
 Now, if we take 2 at a time (repetition is allowed) then
= (2n–1Cn + 2n–1Cn–1) – nCn first place can be filled in n ways and second place
= 2nCn – nCn can be filled in n ways.

∴  the number of ways taken 2 at a time = nP1 × nP1 = n2
∴  W = 2nCn – 1

Similarly, for 3 at a time = n3
The correct option is (A)
and so on for r at a time = nr
By the given condition, it is clear that nCr is the great-
(III) 
∴  required number of ways = n + n2 + n3 + … + nr
est among nC0, nC1, …, nCn.
= n (1 + n + … + nr–1)
Since n is even,
n n( nr − 1)
∴  nCr is the greatest for r = = .
2 n −1
The correct option is (C) The correct option is (C)

Objective_Maths_JEE Main 2017_Ch 7.indd 44 01/01/2008 04:21:00


Permutations and Combinations  7.45

Assertion-Reason Type
152. The candidate is unsucessful if he fails in 9 or 8 or 7 or 6 or = 9C0 + 9C1 + 9C2 + 9C3 + 9C4
5 papers. 1
= (9C0 + 9C1 + … + 9C9)
∴ The number of ways to be unsucessful 2
= 9C9 + 9C8 + 9C7 + 9C6 + 9C5 1
= × 29 = 28 = 256.
2

Previous Year’s Questions

153. Number of choices is equal to 5C4 × 8C6 + 5C5 × 8C5 160. Since, 4 novels can be selected from 6 novels in 6C4 ways
= 140 + 56. and 1 dictionary can be selected from 3 dictionaries in 3C1
The correct option is (B) ways. As the dictionary selected is fixed in the middle, the
remaining 4 novels can be arranged in 4!ways.
n
154. Cr +1 + nCr −1 + nCr + nCr ∴  The required number of ways of arrangement
= n +1
Cr +1 + n +1
Cr = 6C4 ×3 C1 × 4! = 1080
The correct option is (D)
n+ 2
= Cr +1 . 161. Total number of ways = 3C2 × 9C2

The correct option is (B)
9×8
155. Since in half the arrangement A will be before E and other = 3× = 3 × 36 = 108
2
half E will be before A.
6! The correct option is (C)
Therefore total number of ways = = 360.
2 162. ( n −1) C( r −1) = (10 −1) C( 4 −1) = 9 C3
The correct option is (C)
156. Number of balls = 8 Statement 1 is correct
And, number of boxes = 3 Statement 2 is also correct
Hence number of ways = 7C2 = 21. From 9 we can select 3 in 9C3ways. It is correct explanation.
The correct option is (D) The correct option is (D)
157. Alphabetical order for the name SACHIN is 163. Number of ways of selecting one or more balls from 10
A, C, H, I, N, S white, 9 green, and 7 black balls
No. of words starting with A = 5! = (10 + 1)(9 + 1) (7 + 1) – 1 = 11*10* 8 – 1 = 879.
No. of words starting with C = 5! The correct option is (D)
No. of words starting with H = 5! 1 64. Since n +1C3 − n C3 = 10 ⇒ n C2 = 10 ⇒ n = 5 .
No. of words starting with I =5! The correct option is (A)
No. of words starting with N =5! 165. Four digit numbers, which start from 6, 7, 8 = 3 × 4 × 3 × 2
SACHIN − 1 = 72
So the sum is 601. Five digit numbers = 5! = 120
The correct option is (A) \ Total number of integers = 192.
158. Total number of ways = 10C1 + 10C2 + 10C3 + 10C4 The correct option is (A)
= 10 + 45 + 120 + 210 166. Total number of words which can be formed using all the
= 385 letters of the word ‘SMALL’
The correct option is (C) 5!
= = 60
159. Other than S, seven letters M, I, I, I, P, P, I can be arranged 2!
7! Now, 60th word is → SMLLA
in = 7 . 5 . 3.
2! 4 ! 59th word is → SMLAL
Now four S can be placed in 8 spaces in 8C4 ways. 58th word is → SMALL
Hence, desired number of ways = 7 ⋅ 5 ⋅ 3 ⋅ 8C4 = 7 ⋅ 6C4 ⋅ The correct option is (A)
8
C4.
The correct option is (D)

Objective_Maths_JEE Main 2017_Ch 7.indd 45 01/01/2008 04:21:02


Mathematical Induction 8.1

CHAPTER

8 Mathematical Induction

Chapter Highlights
Mathematical Induction

MATHeMATICAL InDUCTIOn Thus, in order to prove a statement p(n) to be true for all
natural numbers, we have the following working rule:
In drawing mathematical or scientific conclusions, there are
two basic processes of reasoning that are commonly used. wOrkInG rULe
These are deduction and induction. Deduction is the process
of reasoning from general to particular and induction is the ■ Prove that p(1) is true; i.e., p(n) is true for n = 1.
process of reasoning from particular to general. In this unit, ■ Assume p(k) to be true; i.e., p(n) is true for n – k.
we plan to study induction. Induction begins by observa- ■ Prove that p(k + 1) is also true; i.e., p(n) is also true for n =
tions and from observations we arrive at some tentative k + 1.
conclusions, called conjectures. A conjecture may be true
or false. The principle of mathematical induction helps us
in proving some of these conjectures which are true. NOTE
A notation for Statements It is important to note that for the proof by mathematical
induction both the conditions (i) and (ii) as stated above
Consider the statements:
must be fulfilled. The result obtained may be fallacious if
1. n(n + 1) is divisible by 2. only one of these conditions is satisfied. Even if we prove
2. 23n – 1 is divisible by 7. a certain statement for a large number of values of n, say
3. 2n > n, etc. n = 1, 2, …, 100, we cannot say that the statement is true
for all values of n unless we also establish the condition (ii).
All these statements are concerned with n, which takes
values 1, 2, 3, … Such statements are usually denoted by
p(n). By giving particular values to n, we get particular SOLVeD eXAMPLeS
statement.
For example, if the statement 32n – 1 is divisible by
1. If n ∈ N, then 72n + 33n –3 ⋅ 3n – 1 is always divisible by
8 is denoted by p(n), then p(4) is the statement 32.4 – 1 is
divisible by 8. (A) 25 (B) 35
(C) 45 (D) None of these
The Principle of Mathematical Induction Solution: (A)
This principle states: If p(n) is a statement involving natural Putting n = 1 is 72n + 23n–3.3n–1, we get
number n, then 72.1 + 23.1–3 ⋅ 31–1 = 7220.30 = 49 + 1 = 50 (1)
1. if p(1) is true, and Also, for n = 2
2. if p(k + 1) is true whenever p(k) is true, then p(n) is
72.2 + 23.2–3 ⋅ 32–1 = 2401 + 24 = 2425 (2)
true for all natural numbers n.
From Eq. (1) and (2), it is always divisible by 25.

Objective_Maths_JEE Main 2017_Ch 8.indd 1 01/01/2008 04:24:17


8.2  Chapter 8

2. For every natural number n, (n2 – 1) is divisible by p (3) : 23 < 3!, which is not true
(A) 4 (B) 6 p (4) : 24 < 4!, which is true.
(C)  10 (D)  None of these Let p(k) be true if k ≥ 4, i.e., 2k < k !, k ≥ 4
⇒ 2.2k < 2 (k!) ⇒ 2k + 1 < k(k!) (∵ k ≥ 4 >2)
Solution: (B)
⇒ 2k + 1 (k + 1)! ⇒ p(k + 1) is true.
We have, n, (n2 – 1) = (n – 1) n(n + 1). Hence, we conclude that p(n) is not true for n = 2,3 but
It is product of three consecutive natural numbers, so hold true for n ≥ 4.
by Lagrange’s theorem it is divisible by 3! i.e., 6.
8. x(xn–1 – nan – 1) + an(n – 1) is divisible by (x – a)2 for
3. For a positive integer n, (A) n > 1 (B) n > 2
1 1 1 1 (C) all n ∈ N (D)  None of these
Let a(n) = 1 + + + + … + n .Then
2 3 4 2 −1
Solution: (C)
(A) a (100) ≤ 100 (B)  a (100) > 100
Check the options. The condition is satisfied for all
(C) a (200) ≤ 100 (D)  a (200) > 100
n ∈ N.
Solution: (A)
n 9. For natural number n, 2n(n – 1)! < nn, if
It can be proved by induction that > a( n) ≤ n .
2 (A) n < 2 (B) n > 2
200 (C) n ≥ 2 (D)  for no n
∴ < a( 200) ⇒ a(200) > 100 and a(100) ≤ 100.
2
Solution: (B)
4. Let p(n) denote the statement that n2 + n is odd. It is Check the options. The condition is satisfied for n > 3
seen that p(n) ⇒ p(n + 1), Pn is true for all
n
(A) n > 1 (B) n ⎛ n + 1⎞
10. If n is a natural number then ⎜ ≥ n ! is true
(C) n > 2 (D)  None of these ⎝ 2 ⎟⎠
when
Solution: (D) (A) n > 1 (B) n ≥ 1
Since the square of any odd number is always odd an (C) n > 2 (D) n ≥ 2
sum of two odd numbers is always even, so for no ‘n’
Solution: (B)
this statement is true.
Check the options. The condition is true for n ≥ 1.
5. If n ∈ N, then 10n + 3 (4n + 2) + 5 is divisible by
11. Statement 1: For every natural number n ≥ 2,
(A) 7 (B) 5 (C) 9 (D) 17
1 1 1
Solution: (C) + +… + > n
1 2 n
For n = 102 + 3(44) + 5 = 100 + 768 + 5 = 873, which
is divisible by 9. Statement  2: For every natural number n ≥ 2,
n( n +1) < n + 1.
6. The value of the natural numbers n such that the
inequality 2n > 2n + 1 is valid, for (A)  Statement 1 is false, Statement 2 is true
(A) n ≥ 3 (B) n < 3 (B) Statement 1 is true, Statement 2 is true, Statement 2
(C) no n (D)  any n is a correct explanation for Statement 1
(C) Statement 1 is true, Statement 2 is true; Statement 2
Solution: (A) is not a correct explanation for Statement 1
Check through options, the condition 2n > 2n + 1 is (D)  Statement 1 is true, Statement 2 is false
valid for n ≥ 3.
Solution: (C)
7. Let p(n) be statement 2n < n!, where n is a natural num- 1 1 1
ber, then p(n) is true for: P(n) = + +…+
1 2 n
(A) all n (B)  all n > 2
1 1
(C) all n > 3 (D)  None of these P(2) = + > 2
1 2 
Solution: (C)
1 1 1
Let  p(n) : 2n < n! Let us assume that P(k) = + +… + > k
Then, p (1) : 2! < 1!, which is not true 1 2 k
is true.
 p (2) : 22 < 2!, which is not true

Objective_Maths_JEE Main 2017_Ch 8.indd 2 01/01/2008 04:24:19


Mathematical Induction  8.3

1 1 1 1 Statement 1 is correct.
∴ P(k + 1) = + +…+ + > k +1
1 2 k k +1 P(2) = 2 × 3 < 3
has to be true.
If P(k) = k ( k + 1) < (k + 1) is true
1 k ( k + 1) + 1
L.H.S. > k + = Now  P(k + 1) = ( k + 1)( k + 2) < k + 2 has to be
k +1 k +1
true.
Since k ( k + 1) > k (∀ k ≥ 0) Since (k + 1) < k + 2
k ( k + 1) + 1 k +1 ∴ ( k + 1)( k + 2) < (k + 2)
∴ > = k +1 
k +1 k +1 Hence, Statement  2 is not a correct explanation of
Let P(n) = n( n + 1) < n + 1 Statement 1.

EXERCISES

Single Option Correct Type

1. For each natural number n, 3n > n3 for (C)  an odd positive integer
(A) n > 2 (B) n ≥ 3 (D)  None of these
(C) n ≥ 4 (D) n < 4 n7 n5 2n3 n
6. For every positive integer n, + + − is
n5 n3 7 7 5 3 105
2. For n ∈ N , + + n is (A)  an integer
5 3 15
(B)  a rational number
(A)  an integer (B)  a natural number (C)  an odd integer
(C)  a positive fraction (D)  None of these (D)  a negative real number
⎡ cos θ sin θ ⎤ n 7. 10n + 3.4n+2 + k is divisible by 9 for n ∈ N . Then, the
3. If A = ⎢ ⎥ , then for n ∈ N, A is equal to
⎣ − sin θ cos θ ⎦ least positive integral value of k is
(A) 1 (B) 3 (C) 5 (D) 7
⎡ cos n θ sin n θ ⎤ ⎡ cos nθ sin nθ ⎤
(A)  ⎢ ⎥ (B) 
⎢ − sin nθ cos nθ ⎥ 8. The sum of the cubes of three consecutive natural
n n
⎢⎣ − sin θ cos θ ⎥⎦ ⎣ ⎦
numbers is divisible by
⎡cos nθ − sin nθ ⎤ ⎡ n cos θ n sin θ ⎤ (A) 2 (B) 4 (C) 6 (D) 9
(C)  ⎢ ⎥ (D) 
⎢ − n sin θ
⎣ sin nθ cos nθ ⎦ ⎣ n cos θ ⎥⎦ 1 1 1
9. For all n ∈ N, 1 +
+ +… +
1 1 1 2 3 4
4. + + +… to n terms = (A) > n (B)  < n
1.4 4.7 7.10
1 1 (C) ≤ n (D)  ≥ n
(A)  (B) 
5n − 1 3n − 1 10. The statement p(n): 1 × 1! + 2 × 2! + 3 × 3! + …
n n + n × n! = (n + 1)! – 1 is
(C)  (D) 
3n + 1 5n − 1 (A)  true for all n > 1
(B)  not true for any n
5. x2n–1 + y2n–1 is divisible by x + y if n is (C)  true for all n ∈ N
(A)  a positive integer (D)  None of these
(B)  an even positive integer

Answer keys

1. (C) 2.  (B) 3. (B) 4.  (C) 5. (A) 6.  (A) 7. (C) 8.  (D) 9. (D) 10.  (C)

Objective_Maths_JEE Main 2017_Ch 8.indd 3 01/01/2008 04:24:22


CHAPTER

9 Binomial Theorem

Chapter Highlights
Binomial Expression, Binomial Theorem, Pascal's triangle, Middle Term in The Binomial Expansion.

BINOMIAL EXPRESSION 2.
Replacing x by 1 and y by x, we get
(1 + x)n = nC0 + nC1 x + nC2 x2 + ... + nCn xn
An algebraic expression consisting of only two terms is called
3.
Replacing x by 1 and y by – x, we get
a binomial expression. For example, expressions such as
(1 – x)n = nC0 – nC1 x + nC2 x2 – ... + (– 1)n nCn xn.
4
x + a, 4x + 3y, 2x – 4.
Adding (1) and (2), we get
y
are all binomial expressions. (x + y)n + (x – y)n = 2 (xn + nC2 xn – 2 y2 +
n
C4 xn – 4 y4 + ...)
BINOMIAL THEOREM = 2 (sum of terms at odd places).
The last term is nCn yn or nCn – 1 xyn – 1 according as n is
This theorem gives a formula by which any power of a even or odd respectively.
binomial expression can be expanded. It was first given by 5. Subtracting Eq. (2) from (1), we get
Sir Isaac Newton. (x + y)n – (x – y)n = 2 (nC1 xn – 1 y + nC3 xn – 3 y3 + ...)
= 2 (sum of terms at even places)
Binomial Theorem for Positive Integral The last term is nCn – 1 xyn – 1 or nCn yn according as n is
Index even or odd respectively.
If x and y are real numbers, then for all n ∈ N,
(x + y)n = nC0 xn y0 + nC1 xn – 1 y1 + nC2 xn – 2 y2 TRICK(S) FOR PROBLEM SOLVING
+ ...+ nCn – 1 x1 yn – 1 + nCn x0 yn  ...(1)  The coefficient of (r + 1)th term in the expansion of
n (1 + x)n is nCr.
i.e., (x + y)n = ∑ n Cr x n−r y r  The coefficient of x in the expansion of (1 + x) is C .
r n n
r =0 r

Here nC0, nC1, nC2, ..., nCn are called binomial coefficients.
For the sake of convenience, we may denote nCr by Cr ⋅ nCr I M P O R TA N T P O I N T S
 n 
may also be denoted as   .
r  The positive integer n is called the index of the binomial.
 Number of terms in the expansion of (x + y)n is n + 1,
SPECIAL CASES i.e., one more than the index n.
 In the expansion of (x + y)n, the power of x goes on
1.
Replacing y by – y in (1), we get decreasing by 1 and that of y goes on increasing by 1 so
(x – y)n = nC0 xn y0 – nC1 xn – 1 y1 + nC2 xn – 2 y2 that the sum of powers of x and y in any term is n.
... + (– 1)n nCn x0 yn...(2)
9.2  Chapter 9

PASCAL’S TRIANGLE
 The binomial coefficients of the terms equidistant from
the beginning and the end are equal. The coefficients nC0, nC1, nC2, ... , nCn or simply C0, C1, C2
 If n is odd, then number of terms in (x + a)n + ..., Cn are called binomial coefficients and they can be eval-
n +1 uated with the help of Pascal’s triangle as below:
(x – a)n and (x + a)n – (x – a)n are equal to .
2
 If n is even, then the number of terms in the expansion of Exponent of Binomial     Coefficients of successive
     terms in Binomial Theorem
(x + a)n + (x – a)n are  n + 2  and that in the expan-
 2  n = 0
 n n = 1
sion of (x + a)n – (x – a)n are  
 2  n = 2
 xn + yn is divisible by x + y if n is odd as xn + yn = (x + y) n = 3
(xn – 1 – xn – 2y + xn – 3y2 – ... + yn–1)
n = 4
 xn – yn is divisible by x – y if n is even as xn – yn = (x – y)

(xn – 1 + xn–2y + xn–3y2 + ... + yn – 1) n = 5


 The coefficient of xn in the expansion of (x + 1) (x + 2) ...
–1
n = 6
n ( n + 1) I. Each row starts and ends in 1.
(x + n) = .
2 II. The coefficients of an expansion are obtained from
 The coefficient of xn – 1 in the expansion of (x – 1) (x – 2) the coefficients of the previous expansion.
n ( n + 1) III. Each coeffcient is the sum of the two coefficients
... (x – n) = – .
2 which lie on either side of it in the previous row.

General Term in the Expansion of (x + y)n


NOTE
In the binomial expansion of (x + y)n, the (r + 1)th term
 Students are advised to remember the following values: from the beginning is usually called the general term and it
n ( n -1) is denoted by Tr + 1, i.e.,
n
C0 = 1, nC1 = n, nC2 = , Tr + 1 = nCr xn – r yr
2!
n ( n - 1) ( n - 2)
n
C3 = ,
3! SOLVED EXAMPLES
n( n − 1) ( n − 2) ... ( n − r + 1)
n
Cr = 1. The sum of the coefficients in the expansion of
r!
(1 + 5x – 7x3)3165 is
and nCn = 1.
Also, nCr = nCn–r for 1 ≤ r ≤ n.
(A) 1 (B) 23165
(C) 2
3164
(D) –1
i.e., nC0 = nCn, nC1 = nCn – 1, nC2 = nCn – 2, ...
i.e., coefficients of terms equidistant from the beginning Solution: (D)
and end are equal. Putting x = 1 in (1 + 5x – 7x3)3165, the required sum of
n
n 2 coefficients = (1 + 5 – 7)3165 = (– 1)3165 = –1.
 ∑ ( Ci ) = 2nCn
i=0 12
 2 
n 2. The 8th term of 3 x + 2  , when expanded in

n
∑ i ( Ci ) = n ⋅ 2n – 1  3x 
i=0
ascending power of x, is
n
n
 ∑ i (i − 1) ⋅ Ci = n(n – 1)2n – 2 228096 228096
i=2 (A) 3 (B) 
n
n
x x9
 ∑ i(i − 1) ... (i − k + 1) ⋅ Ci = n(n – 1) ... (n – k + 1)⋅
i=k 328179
(C) (D)  none of these
2n – k x9
n
∑ i ( Ci ) = n ⋅ 2n – 1Cn – 1
n 2
 Solution: (A)
i =1
Binomial Theorem  9.3

6
 2 
12
 1  10 ´ 9´8´ 7

When 3 x + 2  is expanded, the power of x goes Hence, t7 = C6 (y )  y  =
10 2 4
y2
 3x    4 ´3´ 2´1
on decreasing as the terms proceed. Hence, it is = 210 y2 n
 2 
12
 1
m 1 + 
expanded in descending powers of x. So  2 + 3 x  , 5. The term independent of x in (1 + x) 


x 
is
 3x 
when expanded, will be in ascending powers of x. (A) Cm
m+n
(B)  Cn
m+n

12 12−7 (C) m+n


C (D)  none of these
 2   2  m–n

Now, t8 in  2 + 3 x  = 12C7 


 3 x 2  ⋅ (3x)7 Solution: (B)
 3x 
We have,
5 n n
12 !  2   1   x + 1
= 7 ! 5! ⋅  2  ⋅ (3x)7 (1 + x) 1 +  = (1 + x)m
m 
 x 
 3x   x
12´11´10 ´ 9´8 25 × 32 (1+ x ) m+ n
= ⋅ == x– n (1 + x)m + n
5´ 4 ´3´ 2 x3 xn
228096 ∴ Required term independent of x = coefficient of
= x0 in
x3
3. If A is the sum of the odd terms and B the sum of even x– n (1 + x)m + n = coefficient of xn in (1 + x)m + n
terms in the expansion of (x + a)n, then A2 – B2 = = m+n
Cn
(A) (x2 + a2)n (B)  (x2 – a2)n 6. The coefficient of x53 in the expansion
(C) 2 (x – a )
2 2 n
(D)  none of these 100
∑ 100
Cm ( x − 3)100−m ⋅ 2m is
Solution: (B) m=0

We have, (A) 100C47 (B) 100


C53
(x + a)n = nC0 xn + nC1 xn – 1 a1 + nC2 xn – 2 a2 (C) – 100C53 (D) –100C100
 + nC3 xn – 3 a3 + ... + nCn xn Solution: (C)
= (nC0 xn + nC2 xn – 2 a2 + ...) 100
+ (nC1 xn – 1 a1 + nC3 xn – 3 a3 + ...) We have, ∑ 100
Cm ( x − 3)100−m ⋅ 2m
= A + B m=0

(x – a)n = nC0 xn – nC1 xn – 1 a1 + nC2 xn – 2 a2 = (x – 3) + 100C1 (x – 3)99 ⋅ 21


100

– nC3 xn – 3 a3 + ... + nCn (– 1)n an + 100C2 (x – 3)98 ⋅ 22 + ... + 100C100 2100


= (nC0 xn + nC2 xn – 2 a2 + ...) = [(x – 3) + 2]100 = (x – 1)100 = (1 – x)100
– (nC1 xn – 1 a1 + nC3 xn – 3 a3 + ...) ∴  coefficient of x54 = 100C53 (– 1)53 = – 100C53
= A – B 7. The coefficient of xm in (1 + x)m + (1 + x)m + 1 +, ..., + (1
∴ A2 – B2 = (A + B) (A – B) = (x + a)n (x – a)n + x)n, m ≤ n is
= (x2 – a2)n
10
(A) nCm (B)  n
Cm + 1
1 2
 (C) Cm + 1
n+1
(D)  none of these

4. The 7th term in  y + y  , when expanded in
  Solution: (C)
descending power of y, is The coefficient of xm in
210 y2 (1 + x)m + (1 + x)m + 1 + (1 + x)m + 2 + ... + (1 + x)n
(a) 2 (b) 
y 210 = mCm + m + 1Cm + m + 2Cm + ... + nCm
= m + 1Cm + 1 + m + 1Cm + m + 2Cm + ... + nCm
(c) 210 y 2 (d)  none of these
(Q  mCm = m + 1Cm + 1 = 1)
Solution: (C) = m + 2Cm + 1 + m + 2Cm + ... + nCm
10
1  (Q  nCr + nCr + 1 = n + 1Cr + 1)
When  + y  is expanded, the powers of y go on
2
= m + 3Cm + 1 + ... + nCm = n + 1Cm + 1.
y 
increasing as the terms proceed. Hence it is expanded 8. The coefficient of x3 in the expansion of (1 – x + x2)6 is
10
 2 1  (A) 50 (B)  – 50

in ascending powers of y. So  y +  , when
 y (C) 68 (D)  none of these
expanded, will be in descending powers of y.
9.4  Chapter 9

Solution: (B)
1
(1 – x + x2)6 = [1 – x (1 – x)]6 = ( 2n + 1 − 1 − n + 1C0 )
n +1
= 6C0 – 6C1 x (1 – x) + 6C2 x2 (1 – x)2
– 6C3 x3 (1 – x)3 + ... to 7 terms 2n − 1
=
= 6C0 – 6C1 x (1 – x) + 6C2 x2 (1 – 2x + x2) n +1
– 6C3 x3 (1 – 3x + 3x2 – x3) + ... to 7 terms
∴ Coefficient of x3 = – 2 ⋅ 6C2 – 6C3, (collecting coef- 11. The coefficient of x5 in the expansion of
ficients of x3 from each term) (1 + x2)5 (1 + x)4 is
6! 6! (A) 40 (B)  50
=–2 - = – 50
2 ! 4 ! 3! 3! (C) – 50 (D)  60
Solution: (D)
9. The value of x in the expression ( x + x log10 x )5 , if the We have, (1 + x2)5 (1 + x)4
third term in the expansion is 10,00,000, is = (1 + 5C1 x2 + 5C2 x4 + ...) (1 + 4C1 x
(A) 10– 1 (B) 101 + 4C2 x2 + 4C3 x3 + 4C4 x4)
(C) 10– 5/2 (D) 105/2 = (1 + 5x2 + 10x4 + ...) (1 + 4x + 6x2 + 4x3 + x4)
Solution: (B, C) The term giving x5 in the above product is
Put log10 x = z (5x2) (4x3) + (10x4) (4x) = (20 + 40) x5 = 60x5
Then, given expression = (x + xz)5. Hence, the coefficient is 60.
Now, T3 = 5C2 x3 (xz)2 = 10x3 + 2z = 106 12. If (1 + x – 2x2)6 = 1 + a1 x + a2 x2 + ... + a12 x12, then
∴  x3 + 2z = 105. a2 + a4 + a6 + ... + a12 =
Taking log, we get
(A) 21 (B)  11
(3 + 2z) log10 x = 5 log10 10
(C) 31 (D)  none of these
⇒ (3 + 2z) z = 5 or 2z2 + 3z – 5 = 0
5 Solution: (C)
⇒ (z – 1) (2z + 5) = 0 ⇒ z = 1, –
2 Given
5
∴ log10 x = 1 or –   ∴  x = 101 or 10–5/2. (1 + x – 2x2)6 = 1 + a1x + a2x2 + ... + a12x12
2 Putting x = 1, we get
C1 n
C n n
C 0 = 1 + a1 + a2 + ... + a12...(1)
10. The value of + 3 + 5 + ... is Putting x = – 1, we get
2 4 6
64 = 1 – a1 + a2 – ... + a12...(2)
2n - 1 2n + 1
(A) (B)  Adding Eq. (1) and (2), we get
n n 64 = 2 (1 + a2 + a4 + ...)
n
n
2 −1 2 +1 ∴ a2 + a4 + a6 + ... + a12 = 31
(C) (D) 
n +1 n +1 13. If 7103 is divided by 25, then the remainder is
(A) 20 (B)  16
Solution: (C)
(C) 18 (D)  15
The rth term of the given expression is
n
C2 r - 1 Solution: (C)
Tr = We have, 7103 = 7 (49)51 = 7 (50 – 1)51
2r
= 7 (5051 – 51C1 5050 + 51C2 5049 – ... – 1)
1 n 1 n +1
Since ⋅ Cr = ⋅ C r +1 = 7 (5051 – 51C1 5050 + 51C2 5049 – ...) – 7 + 18 – 18
r +1 n +1 = 7 (5051 – 51C1 5050 + 51C2 5049 – ...) – 25 + 18
n
C2 r - 1 1 n +1 = k + 18 (say)  Q  k is divisible by 25,
∴ Tr = = ⋅ C2 r
2r n +1 ∴  remainder is 18.
C1n n
C n
C 14. The sum of rational terms in the expansion of
∴  + 3 + 5 + ...
2 4 6 ( 2 + 31/5 )10 is
1 n +1 (A) 31 (B)  41
  = ( C2 + n + 1C4 + ...) (C) 51 (D)  none of these
n +1
Binomial Theorem  9.5

Solution: (B) Solution: (B)


(r + 1)th term in the given expansion is given by The numerator is of the form
tr + 1 = 10Cr 102-r 5r , where r = 0, 1, 2, ..., 10 a3 + b3 + 3ab (a + b) = (a + b)3
2 3 where a = 18 and b = 7
For rational terms
∴  Numerator = (18 + 7)3 = 253.
r = a multiple of 5 = 0, 5, 10 ...(1)
For denominator, 31 = 3, 32 = 9, 33 = 27, 34 = 81, 35 = 243
10 – r = a multiple of 2 = 0, 2, 4, 6, 8, 10 ...(2) 6
C1 = 6, 6C2 = 15, 6C3 = 20
From Eq. (1) and (2) possible values of r are : 0 and 10 6
C4 = 6C2 = 15, 6C5 = 6C1 = 6, 6C6 = 1
∴  sum of rational terms
∴  denominator = 36 + 6C1 35 ⋅ 21 + 6C2 34 ⋅ 22
= t1 + t11 = 10C0 ( 2 )10 (31/5)0 + 10C10 ( 2 )0 (31/5)10 + 6C3 33 ⋅ 23 + 6C4 32 ⋅ 24 + 6C5 3 ⋅ 25 + 6C6 26
= 25 + 32 = 32 + 9 = 41 This is clearly the expansion of
15. In the expansion of (x + a)n if the sum of odd terms be (3 + 2)6 = 56 = (25)3
P and the sum of even terms be Q, then 4PQ = Numerator (25)3
∴  = =1
(A) (x + a)n – (x – a)n (B) (x + a)n + (x – a)n Denominator ( 25)3
(C) (x + a) – (x – a)
2n 2n
(D)  none of these 18. Larger of 9950 + 10050 and 10150 is
Solution: (C) (A) 10150 (B) 9950 + 10050
We have, (C) both are equal (D)  none of these
(x + a)n = xn + nC1 xn – 1 a + nC2 xn – 2 a2 Solution: (A)
+ nC3 xn – 3 a3 + ... We have,
= (xn + nC2 xn – 2 a2 + ...) + (nC1 xn – 1 a 10150 = (100 + 1)50
+ nC3 xn – 3 a3 + ...) 50 × 49
= 10050 + 50 ⋅ 10049 + ⋅ 10048 + ...
=P+Q 1× 2
∴ (x – a)n = P – Q, as the terms are alternatively pos- and 9950 = (100 – 1)50
itive and negative. = 10050 – 50 ⋅ 10049 + 50 × 49 ⋅ 10048 – ...
∴ 4PQ = (P + Q)2 – (P – Q)2 = (x + a)2n – (x – a)2n 1× 2
Subtracting, we get
16. If C0, C1, C2, ..., Cn are the coefficients of the expansion
10150 – 9950 = 2 (50 ⋅ 10049 + × 10047 + ...)
n Ck
of (1 + x)n, then the value of ∑ is
0 k +1 50 × 49 × 48
= 10050 + 2 ⋅ ⋅ 10047 + ... > 10050
2 n -1 1× 2 × 3
(A) 0 (B)  Hence, 10150 > 9950 + 10050.
n
2n+1 −1 19. For all n ∈ N, 24n – 15n – 1 is divisible by
(C) (D)  none of these
n +1 (A) 225 (B)  125
(C) 325 (D)  none of these
Solution: (C) n
Cr 1 Solution: (A)
Here, tr + 1 = = ⋅ nCr
r +1 r +1 We have, 24n = (24)n = (16)n = (1 + 15)n
1 ∴ 24n = 1 + nC1 ⋅ 15 + nC2 152 + nC3 153 + ...
= ⋅ n + 1Cr + 1
n +1 n
Ck ⇒ 24n – 1 – 15n = 152 (nC2 + nC3 ⋅ 15 + ...)
Putting r = 0, 1, 2, ... n and adding, we get ∑ k +1
0
= 225 K, where K is an integer.
Hence, 24n – 15n – 1 is divisible by 225.
1
= (n + 1C1 + n + 1C2 + n + 1C3 + ... + n + 1Cn + 1)
n +1 20. When 599 is divided by 13, the remainder is
1 2n+1 −1 (A) 8 (B)  9
= (2n + 1 – n + 1C0) = (C) 10 (D)  none of these
n +1 n +1
Solution: (A)
17. The value of
We have,
(183 + 73 + 3 ⋅18 ⋅ 7 ⋅ 25)
599 = 53 ⋅ 596 = (125) (625)24
36 + 6 ⋅ 243 ⋅ 2 + 15 ⋅ 81⋅ 4 + 20 ⋅ 27 ⋅ 8 + 15 ⋅ 9 ⋅16 + 6 ⋅ 3 ⋅ 32 + 64is = [13 × 9 + 8] (1 + 48 × 13)24
(A) 0 (B)  1 = (13 × 9 + 8) [1 + 24C1 × (48 × 13)
(C) 2 (D)  none of these + 24C2 (48 × 13)2 +...+ (48 × 13)24]
9.6  Chapter 9
n
= 8 + terms containing powers of 13. ∴  The digit at units place in 22 = 24k = (16)k is 6.
Hence remainder = 8. 100

21. The last digit of the number (32) is 32 Thus, the digit at units place in ∑ r! + 2
r=0
2n
is 0.
(A) 4 (B)  6 (32)
(C) 8 (D)  none of these 24. When 32(32) is divided by 7, the remainder is
(A) 4 (B)  6
Solution: (B)
(C) 8 (D)  none of these
(32)32 = (2 + 3 × 10)32
= 232 + 10k, where k ∈ N Solution: (A)
Therefore, last digits in (32)32 = last digit in (2)32 (32)32 = (25)32 = 2160 = (3 – 1)160
But 21 = 2, 22 = 4, 23 = 8, 24 = 16, 25 = 32 = 160C0 3160 – 160C1 ⋅ 3159
∴ 232 = (25)6 ⋅ 22 = (32)6 ⋅ 4 = (2 + 30)6 ⋅ 4 + ... + 160C159 ⋅ 3 + 160C160 ⋅ 30
= (26 + 10r) 4, r ∈ N = 3k + 1, where k ∈ N
( 32 )
Last digit in 232 = last digit in (2)6 ⋅ 4 = last digit in Now, 32( 32 ) = (32)3k + 1 = (25)3k + 1 = 215k + 5
4×4=6 =2 3(5k + 1)
 ⋅ 2 = (23)5k + 1 ⋅ 4
2

∴  Last digit in (32)32 = 6. = 4(7 + 1)5k + 1


22. If (1 + x)n = C0 + C1 x + C2 x2 + ... + Cn xn, then = 4[5k + 1C0 75k + 1 + 5k + 1C1 75k
+ ... + 5k + 1C5k7 + 5k + 1C5k + 1 ⋅ 70]
C1 C2 Cn
2C0 + 22 ⋅ + 23 ⋅ + ... + 2n + 1 = = 4(7n + 1), where n ∈ N
2 3 n +1 = 28n + 4.
3n+1 −1 3n -1 Therefore, when 32( 32 )
( 32 )
is divided by 7, the remain-
(A) (B) 
n +1 n der is 4.
3n+ 2 −1 25. The number of non zero terms in the expansion of
(C) (D)  none of these
n+2 (1 + 3 2 x )9 + (1 − 3 2 x )9 is
Solution: (A) (A) 9 (B)  0
(C) 5 (D)  10
We have,
n
Cr 1 Solution: (C)
tr + 1 = 2r + 1 = 2r + 1 ⋅ ⋅ n + 1Cr + 1
r +1 n +1 In the expansion of
Putting r = 0, 1, 2, ..., n and adding, we get the required (1 + 3 2 x)9 + (1 – 3 2 x)9
sum 2nd, 4th, 6th, 8th and 10th terms get cancelled.
1 ∴  Number of non-zero terms in
= (2 ⋅ n + 1C1 + 22 ⋅ n + 1C2 + ... + 2n + 1 ⋅ n + 1Cn + 1)
n +1 2 [9C0 + 9C2 (3 2x )2 + ... + 9C8 (3 2x )8] is 5.
1 3n+1 −1 26. The expression [x + (x3 – 1)1/2]5 + [x – (x3 – 1)1/2]5 is a
= [(1 + 2)n + 1 – n + 1C0] = .
n +1 n +1 polynomial of degree
(A) 5 (B)  6
23. For integer n > 1, the digit at units place in the number (C) 7 (D)  8
100 n
2
∑ r ! + 2 is Solution: (C)
r=0
[x + (x3 – 1)1/2]5 + [x – (x3 – 1)1/2]5
(A) 0 (B)  1 = 2 [5C0 x5 + 5C2 x3 (x3 – 1) + 5C4 x (x3 – 1)2]
(C) 2 (D)  3 = 2 [x5 + 10x3 (x3 – 1) + 5x (x3 – 1)2]
Solution: (A) = 5x7 + 10x6 + x5 – 10x4 – 10x3 + 5x
Since the digit at units place in each of 5!, 6!, ..., 100! which is a polynomial of degree 7.
is 0 and 0! + 1! + 2! + 3! + 4! = 34. 27. The value of x, for which the 6th term in the expansion
100 7
 
therefore the digit at units place in ∑ r ! is 4.
r=0
 log2 ( 9x−1 +7 )
of  2 + 1
1 
 is 84, is equal to
 log 2 ( 3x−1 +1) 
 
n

Now, 2 = 2 , k ∈ N (2 is a multiple of 4 form n > 1)


2 4k n 25
Binomial Theorem  9.7

(A) 4 (B)  3 (41/5 + 71/10)45 is 45 + 1,  i.e., 46.


(C) 2 (D)  1 The general term in the expansion is
Solution: (C,D) Tr + 1 = 45Cr ⋅ 455-r ⋅ 10r
4 7
The given expression Tr + 1 is rational if r = 0, 10, 20, 30, 40.
7
 x−1 1  ∴  Number of rational terms = 5.

=  9 + 7 + x−1 
(3 + 1)1/ 5  ∴  Number of irrational terms = 46 – 5 = 41.

Given, T6 = 84 31. In the expansion of (1 + x + x3 + x4)10, the coefficient of
5 x4 is
 1 
⇒ C5 ( 9
x −1
+ 7) 7−5   (A) 40C4 (B)  10
C4
 (3x−1 + 1)1/ 5  = 84
7

(C) 210 (D)  310


1 Solution: (D)
⇒ 7C5 (9x – 1 + 7) ⋅ = 84
x−1
(3 + 1) (1 + x + x3 + x4)10 = [(1 + x) (1 + x3)]10
⇒ 9x – 1 + 7 = 4 (3x – 1 + 1) = (1 + x)10 (1 + x3)10
⇒ 32x – 12 ⋅ 3x + 27 = 0 = (1 + 10C1 x + 10C2 x2 + 10C3 x3 + 10C4 x4 ...)
⇒ (3x – 3) (3x – 9) = 0 × (1 + 10C1 x3 + 10C2 x6 ...)
⇒ 3x = 3, 9 ⇒ x = 1, 2 ∴  Coefficient of x4 = (10C1) (10C1) + 10C4
28. When 337 is divided by 80, the remainder is 10 × 9 × 8 × 7
= 100 + = 100 + 210 = 310
(A) 3 (B)  4 1× 2 × 3 × 4
(C) 6 (D)  none of these 32. If A = 2nC0 · 2nC1 + 2nC1 2n – 1C1 + 2nC2 2n – 2C1 + ..., then A
Solution: (A) is
We have, 337 = 34.9 ⋅ 3 = 3(81)9 = 3(80 + 1)9 (A) 0 (B)  2n
= 3(9C0 ⋅ 809 + 9C1808 + ... + 9C9) (C) n 2 2n
(D) 1
Thus, when 337 is divided by 80, the m remainder is 3. Solution: (C)
n
 a  A = coeff. of x in [2nC0(1 + x)2n

29. If the second term in the expansion  a +  is
13
+ 2nC1 (1 + x)2n – 1 + ...)]
n  a−1 
C = coeff. of x in (1 + (1 + x))2n
14 a5/2, then the value of n 3 is
C2 = coeff. of x in (2 + x)2n
2n
(A) 8 (B)  12  x
= coeff. of x in 22n 1 +  = n · 22n
(C) 4 (D)  none of these  2 
Solution: (C) 33. The greatest integer which divides the number
Given : T2 = 14 a 5/2 101100 – 1 is
1
 a  (A) 100 (B)  1000
⇒ nC1 (a1/13)n – 1 ⋅  −1/ 2  = 14 a5/2 (C) 10000 (D)  100000
 a 
⇒ n ⋅ a(n – 1)/13 ⋅ a3/2 = 14 a5/2 Solution: (C)
⇒ n ⋅ a(n – 1)/13 = 14 a ⇒ n ⋅ a(n – 14)/13 = 14 By Binomial theorem
⇒ n = 14  n ( n −1) 2 
(1 + x)n = 1 + nx + ⋅ x ... + x n 
n
C
14
C3  2 
∴ n 3 = 14 = 4
C2
C2 or (1 + x)n – 1 = nx + n ( n -1) x2 ... + xn
2
30. The number of irrational terms in the expansion of
(41/5 + 71/10)45 is If x = n, (1 + n) – 1 = n + n ( n -1) n2 ... nn
n 2

2
(A) 40 (B)  5
 n ( n − 1) 
(C) 41 (D)  none of these (1 + n)n – 1 = n2 1 + ... + nn−2 
 2 
Solution: (C) Put n = 100,
Total number of terms in the expansion of  100 (100 −1) 
(1 + 100)100 – 1 = (100)2 1 + ... + 100 98 
 2 
9.8  Chapter 9

 100 × 99   1 
n
 1 
log3 8
(101)100 – 1 = (100)2 1 + ... + 100 98  ⇒ 
n
Cn ⋅ −  =  3 
 2    3 ⋅ 9 
2
Clearly (101) – 1 is divisible by
100
n/2 log3 8
(100)2 = 10000 1  1  5
⇒  (−1) n ⋅    =  5/3  = 3− 3 ⋅ 3 log3 2
 2 1 
2n
 2   3 

34. If x  occurs in the expansion of 
p x +  , its coeffi-
 x  1
5
cient is = 2–5 =  
2n 2n  2 
(A) C 4 n- p (B)  C 2 n- p
3 3 ⇒  n = 10
2n 10
(C) C 4 n - p (D)  none of these 3 1 
Therefore, 5th term in  2 −  is
2 
3

Solution: (A) 4
10 − 4  1 
T5 = T4 + 1 = C4 ( 2 ) − 
3 10
Let tr + 1 contains x p.
 2 
r
1
Then, tr + 1 = 2nCr (x2)2n – r   = 2nCr x4n – 3r
 x 
= 10C4 × ( 4) 1 = 10C4 = 10C6
4n - p 4
∴ 4n – 3r = p; or r =
3 37. If (1 – x + x ) = a0 + a1 x + a2 x2 + ... + a2n x2n, then
2 n

2n
Hence, the coefficient of x p = C 4n - p a0 + a2 + a4 + ... + a2n is equal to
3
1 1- 3n
35. Given positive integers r > 1, n > 2 and the coeffi- (A) 3n2 + (B) 
2 2
cients of (3r)th term and (r + 2)th term in the binomial
expansion of (1 + x)2n are equal, then r = 3n -1 3n + 1
(C) (D) 
n n 2 2
(A) , n even (B)  Solution: (D)
2 2
Putting x = – 1, 1 successively in the given equation
(C) n (D) 1
and adding, we shall get the result.
Solution: (A)
We have, t3r = 2nC3r – 1 x3r – 1 Method for Finding the Independent Term
and tr + 2 = 2nCr + 1 xr + 1 or Constant Term
Given, 2n
C3r – 1 = 2nCr + 1
⇒ 3r – 1 = r + 1; or (3r – 1) + (r + 1) = 2n Step I  rite down the general term in the expansion of
W
⇒ 2r = 2 ; or 4r = 2n (x + a)n i.e., (r + 1) th term
⇒ tr+1 = nCrxn–rar
⇒ r = 1 (impossible); or r = n .
2 Step II Separate the constants and variables. Also group
But r is a positive integer greater than 1. So the value them separately.
Setp II Since, we need to find the term independent of x in
of r is n provided n is an even integer (> 2), other- the given binomial expansion, equate to zero the
2
index of x and accordingly we will get the value of r
wise r has no value.
for which there exists a term independent of x in the
36. If the last term in the binomial expansion of expansion.
log 8
 
n
 1  3
3 2 − 1  is   , then the 5th term is Greatest Term (Numerically) in the
 2   3.3 9 
Expansion of (1 + x)n
(A) 2 ⋅ 10C6 (B) 4 ⋅ 10C4
1 10
Method 1
(C) C6 (D) 10
C6
2 1.
Let Tr (the rth term) be the greatest term.
Solution: (D) 2.
Find Tr – 1, Tr , Tr + 1 from the given expansion.
n log3 8
3 1   1  T T
The last term of  2 −  =  3  Put r ≥ 1 and r ≥ 1. This will give an inequal-
3.
 2  3 ⋅ 9  Tr+1 Tr-1
ity from where value or values of r can be obtained.
Binomial Theorem  9.9

4.
Then, find the rth term Tr which is the greatest term. 39. The greatest term (numerically) in the expansion of
1
Method 2 (3 – 5x)11 when x = is
5
( n + 1)| x |
1.
Find the value of k = (A) 55 × 39 (B)  46 × 39
1+| x |
(C) 55 × 3 6
(D)  none of these
2.
If k is an integer, then Tk and Tk + 1 are equal and both
Solution: (A)
are greatest terms.
3.
If k is not an integer, then T(k) + 1 is the greatest term, We have, 11 11
 5x   1  
Q x = 1 
where (k) is the greatest integral part of k. (3 – 5x) = 3 1−  = 311 1− 
11 11
 3  3   5 
| x | ( n + 1)  1 
TRICK(S) FOR PROBLEM SOLVING ∴ m = − < 0
(| x | + 1)  3 
To find the greatest term in the expansion of (x + y)n, write
 y 
n  1 
 −  (11 + 1)
(x + y) = x 1+  and then find the greatest term in
n n
 3 
 x = =3
 
n  1 
1+ y  .  − + 1

 x   3

The greatest terms in the expansion are T3 and T4


∴  Greatest term (when r = 2) = 311 | T2 + 1 |
SOLVED EXAMPLES  1
2
11
C2 −  11⋅10 1
=3 11
 3  = 311 × = 55 × 39
38. The greatest term (numerically) in the expansion of 1⋅ 2 9
3
(2 + 3x)9, when x = , is and greatest term (when r = 3) = 311 | T3 + 1 |
2 3
 1 11⋅10 ⋅ 9
(A)
5´311
(B) 
5´313
=3 11 C3 − 
11
= 311 ×−
1
= 55 × 39
 3 
2 2 1⋅ 2 ⋅ 3 27
7´313 From above, we see that the values of both greatest
(C) (D)  none of these terms are equal.
2
Solution: (C)
We have,
MIDDLE TERM IN THE BINOMIAL
9 9 EXPANSION 
 3x   9  3 
(2 + 3x)9 = 29 1 +  = 29 1 +  Q x = 
 2  4   2  The middle term in the binomial expansion of (x + y)n
depends upon the value of n.
x ( n + 1)
1. If n is even, then there is only one middle term i.e.
( x + 1)  n 
∴ m =  + 1 th term.
 9   2 
  (9 + 1)
 4  2.
If n is odd, then there are two middle terms, i.e.,
9 90 12  n + 1  n + 3 
=   + 1 = =6 ≠ Integer  
4 13 13  2  th and  2  th terms.
The greatest term in the expansion is T[m] + 1 = T6 + 1 = T7
Hence, the greatest term = 29 ⋅ T7 I M P O R TA N T P O I N T S
6
9
= 29 ⋅ T6 + 1 = 29 ⋅ 9C6   When there are two middle terms in the expansion, their
 4  

binomial coefficients are equal.


6
9 9 × 8 × 7 312 13
= 29 ⋅ 9C3   = 29 ⋅ ⋅ = 7´3  Binomial coefficient of the middle term is the greatest

 4  1× 2 × 3 2 12
2 binomial coefficient.
9.10  Chapter 9

SOLVED EXAMPLE 9. C0Cr + C1Cr + 1 + ... Cn – rCn = 2nCn – r or 2nCn + r


10. C1 + 2C2 + 3C3 + ... + n.Cn = n.2n–1
2n
 1  11. C1– 2C2 + 3C3 – ... = 0
40. The greatest coefficient in the expansion of  x + 
 x 12. Cn + 2C1 + 3C2 + ... + (n + 1)Cn = (n + 2)2n–1
is
1⋅ 3 ⋅ 5...( 2n −1) ⋅ 2n 2n !
(A) (B)  Properties of nCr
n! ( n !) 2
If 0 < r < n, n, r ∈ N, then
n!
2
1. r ⋅ nCr = n ⋅ n – 1Cr – 1
 n  
(C)   ! (D)  none of these n
Cr n +1
Cr +1
 2   2. =
r +1 n +1
Solution: (A, B)
n
Since the middle term has greatest coefficient, 3. n
Cr = n – 1Cr – 1
∴ greatest coefficient = coefficient of the middle term r
n
( 2n)! Cr n − r +1
= 2nCn = 4. n =
n! n! Cr -1 r
n
2n ( 2n −1) ( 2n − 2) ( 2n − 3)...4 ⋅ 3 ⋅ 2 ⋅1 Cr r +1
= 5. n C = 6.  nCr – 1 + nCr = n + 1Cr
n! n! r +1 n−r
= 7. n
Cx = nCy ⇒ x = y or x + y = n
[( 2n −1) ( 2n − 3)...3 ⋅1] [2n ( 2n − 2) ( 2n − 4)...4 ⋅ 2]
8. n
Cr = nCn – r.
n! n!
n / 2 if n is even
[1⋅ 3 ⋅ 5...( 2n −1)] 2n [n ( n −1) ( n − 2)...2 ⋅1] 
= 9. Cr is greatest if r =  n − 1 n + 1
n
n! n!  or if n is odd
 2 2
1⋅ 3 ⋅ 5...( 2n −1) 2n n ! 1⋅ 3 ⋅ 5...( 2n −1) 2n Thus, if n is even, the greatest coefficient is nCn/2 and
= = .
n! n! n! n
if n is odd, the greatest coefficient is C n - 1 or C n + 1 ,
n

2 2
pth Term from the End in the Binomial both being equal.
Expansion  of  (x + y)n 10. The greatest term in (1 + x)2n has the greatest coeffi-
pth term from the end in the expansion of (x + y)n is n n +1
cient if n + 1 < x < .
(n – p + 2)th term from the beginning. n

Properties of Binomial Coefficients TRICK(S) FOR PROBLEM SOLVING


In the binomial expansion of (1 + x)n, the coefficients nC0, 1. The number of terms in the expansion of (x + y + z)n,
n
C1, nC2, ... nCn are denoted by C0, C1, C2, ... Cn respectively. ( n + 1) ( n +2)
n where n is a positive integer, is .
1. If n is even, then greatest coefficient = Cn / 2 2
2.
If n is odd, then greatest coefficient is nC(n – 1)/2 or 2. The number of terms in the expansion of (x + y + z + w)n,
n
C(n + 1)/2. ( n + 1) ( n + 2) ( n + 3)
where n is a positive integer, is .
3.
C0 + C1 + C2 + ... + Cn = 2n 6
n n n3
4.
C0 + C2 + C4 + ... = C1 + C3 + C5 + ... = 2n – 1 3. Coefficient of x 1 y 2 z in the expansion of (x + y + z)n is
5.
C0 – C1 + C2 – C3 + C4 – ... + (– 1)n Cn = 0 n!
  
( 2n)! n1 ! n2 ! n2 ! where n = n1 + n2 + n3
6. C02 + C12 + C22 + ... + Cn2 = ( n !) 2 = 2nCn
4. In the expansion of (x1 + x2 + ... + xk)n, the sum of all
7. 2 2
C − C + C − C +...2 2 the coefficients is obtained by putting all the variables xi
0 1 2 3
equal to 1 and it is equal to kn.
0, if n is odd 5. Coefficient of xm in (1 + xr)n (m, r and n ∈ N) is zero, if
= 
( −1) n / 2 ⋅ n cn / 2 , if n is even m is not an integral multiple of r, e.g., coefficient of x1000

in the expansion of (1 + x3)4000 is 0 as 1000 is not an
8.
C0C1 + C1C2 + C2C3 + ... + Cn – 1 Cn = 2nCn – 1 integral multiple of  3.
Binomial Theorem  9.11

43. If (1 + x)n = C0 + C1 x + C2 x2 + ... + Cn xn, then


SOLVED EXAMPLES
ΣΣ (Ci + C j ) 2 =
n 0 ≤i ≤ j ≤ n
C C C C (−1) Cn
41. The value of 0 − 1 + 2 − 3 + ... + (A) (n – 1) ⋅ 2nCn + 22n (B)  n ⋅ 2nCn + 22n
is 1.3 2.3 3.3 4.3 ( n + 1) ⋅ 3
(C) (n + 1) ⋅ Cn + 2
2n 2n
(D)  none of these
1 1
(A) (B)  Solution: (A)
3( n + 1) n + 1
ΣΣ (Ci + C j ) 2   i  = 0, 1, 2, ..., (n – 1)
3 0≤ i ≤ j ≤ n
(C) (D)  none of these j  = 1, 2, 3, ..., n
n +1
and  i < j
Solution: (A) = n ( C02 + C12 + ... + Cn2 ) + 2 Σ Σ Ci Cj 0 ≤ i < j ≤ n
We have, (1 + x)n = C0 + C1x + C2x2 + ... + Cnxn
= n ⋅ 2nCn + [(C0 + C1 + ... + Cn)2
Integrating both sides w.r.t. x form –1 to 0, we get
0 0 – ( C02 + C12 + ... + Cn2 )]
∫ (1 + x) dx = ∫ (C
n 2 n
0 + C1 x + C2 x + ... + Cn x )dx = n ⋅ 2nCn + (2n)2 – 2nCn = (n – 1) ⋅ 2nCn + 22n
−1 −1

44. If (1 + x)n = C0 + C1x + ... + Cnxn, then the value of


C0 + C1 x + C2 x 2 + ... + Cn x n )dx n n
∑ ∑ (Cr + Cs ) is equal to
r=0 s=0
0
n +1 0 
(1 + x )  C1 x 2 C x n + 1  (A) (n + 1)2n + 1 (B) (n + 1)2n
= cos x + + ... + n 
n +1 −1  2 n + 1  −1
(C) n2n + 1 (C)  none of these
C1 C2 C 1 Solution: (A)
⇒  C0 − + +, ..., + (−1) n n =
2 3 n +1 n +1 n n n n n n

∑ ∑ (C r + Cs ) = ∑ ∑C r + ∑ ∑C s
1 C C  r=0 s=0 r=0 s=0 r=0 s=0
∴  The given expression = C0 − 1 + 2 − ...
3 2 3  n  n  n  n 
1 1 1 = ∑ ∑ C  + ∑ ∑ C 
r s

= ⋅ = s=0 r=0 r=0 s=0


3 n + 1 3( n + 1) n n

= ∑2 n
+ ∑2 n

42. If (1 + x)n = C0 + C1 x + C2 x2 + ... + Cn xn , then s=0 r=0

C0C2 + C1C3 + C2C4 + ... + Cn – 2 Cn = = (n + 1)2 + (n + 1)2n n

= (n + 1)2n + 1
( 2n)! ( 2n)!
(A) (B) 
( n !) 2 ( n −1)!( n + 1)! 45. If (1 + x)n = C0 + C1 + ... + Cnxn, then the value of
( 2n)! ∑ ∑ (Cr + Cs ) is equal to
(C) (D)  none of these 0≤r<s≤n
( n − 2)!( n + 2)!
(A) n2n (B) 2n + 1
Solution: (C) (C) (n – 1)2n (D)  none of these
We have,
Solution: (C)
(1 + x)n = C0 + C1 x + C2 x2 + C3 x3 + C4 x4 + ...
n n
+ Cn – 2 xn – 2 + Cn – 1 xn – 1 + Cn xn ...(1)
and (x + 1)n = C0 xn + C1 xn – 1 + C2 xn – 2 + C3 xn – 3
  ∑ ∑ (C
r=0 s=0
r + Cs )

+ C4 xn – 4 + ... + Cn – 2 x2 + Cn – 1 x + Cn ...(2) n

Multiplying Eq. (1) and (2) and equating the coefficients of = ∑ (C


r=0
r + Cr ) + 2 ∑ ∑ (C
0≤r< s≤n
r + Cs )
xn – 2, we get
C0C2 + C1C3 + C2C4 + ... + Cn – 2 Cn  n 

= the coefficient of xn – 2 in (1 + x)2n ⇒ (n + 1)2n + 1 = 2  Cr  + 2
r = 0  0≤r< s≤n

(Cr + Cs ) ∑∑
( 2n)!
= 2nCn – 2 =
( n − 2)! ( n + 2)!
9.12  Chapter 9

1
n
⇒ (n + 1)2n + 1 = 2 ⋅ 2 + 2 ∑ ∑ (C
0≤r< s≤n
r + Cs ) (A) 0 (B) 
2
1
(C) – (D) 1
⇒  ∑∑
0≤r< s≤n
(Cr + Cs ) = (n – 1)2n 2
Solution: (D)

46. The sum of the series ∑


10
20
Cr is We have,
r= 0
1 ( 1 + x 2 + x)
1 20 1 20 ( 1 + x 2 – x)–1 = ×
(A) 219 – ⋅ C10 (B) 219 + ⋅ C10 ( 1 + x 2 − x) ( 1 + x 2 + x)
2 2
(C) 219 (D) 220 (1 + x 2 )1/ 2 + x
 = = (1 + x2)1/2 + x
Solution: (B)
1+ x2 − x2
10 ∴  coefficient of x in the expansion of ( 1 + x 2 – x)–1
We have, ∑
r= 0
20
Cr = 20C0 + 20C1 + ... + 20C10
= coefficient of x in the exp. of [(1 + x2)1/2 + x]
But 20C0 + 20C1 + ... 20C20 = 220 = 1. (Q coefficient of x in the exp. of 1 + x 2 is 0)
and  Q   20C20 = 20C0, 20C19 = 20C1 49. If the expansion of (1 + x)50, the sum of coefficients of
      20
C18 = 20C2, 20C11 = 20C9 add powers of x is
10 (A) 250 (B) 249
∴ ∑r= 0
20
Cr = (20C0 + 20C1 + ... + 20C20) (C) 0 (D)  none of these
– (20C11 + 20C12 + ... + 20C20) Solution: (B)
= 220 + 20C10 – (20C10 + 20C9 + ... + 20C0) The sum of coefficients of odd powers of x
= 50C1 + 50C3 + ... + 50C49
⇒ 2 (20C0 + 20C1 + ... + 20C10) = 220 + 20C10
= 250 – 1 = 249
∴   20C0 + 20C1 + ... + 20C10 = 219 + 1  20C10 50. The value of the sum of the series
2
14
C0 ⋅ 15C1 + 14C1 ⋅ 15C2 + 14C2 ⋅ 15C3 + ... + 14C14 ⋅ 15C15 is
47. n+1
C2 + 2 [2C2 + 3C2 + 4C2 + ... + nC2] =
(A) 29C12 (B) 
29
C10
n ( n + 1) ( 2n + 1) n ( n +1) (C) 29C14 (D) 
29
C16
(A) (B) 
6 2 Solution: (C)
n ( n -1) ( 2n -1) We have,
(C) (D)  none of these
6 (1 + x)14 = 14C0 + 14C1 x + 14C2 x2
Solution: (A) + ... + 14C14 x14 ...(1)
We have, and (x + 1)15 = 15C0 x15 + 15C1 x14 + 15C2 x13
  n + 1
C2 + 2 [2C2 + 3C2 + 4C2 + ... + nC2] + 15C3 x12 + ... + 15C15 ...(2)
= n + 1C2 + 2 [3C3 + 3C2 + 4C2 + ... + nC2] Multiplying Eq. (1) and (2) and equating the coeffi-
= n + 1C2 + 2 [4C3 + 4C2 + ... + nC2] cient of x14, we get
= n + 1C2 + 2 [5C3 + ... + nC2] 14
C0 ⋅ 15C1 + 14C1 ⋅ 15C2 + 14C2 ⋅ 15C3 + ... + 14C14 ⋅ 15C15
= n + 1C2 + 2 ⋅ n + 1C3 = the coefficient of x14 in (1 + x)29 = 29C14
= n + 1C2 + n + 1C3 + n + 1C3
51. If (1 + x)n = C0 + C1x + ... + Cnxn, then the value of
= n + 2C3 + n + 1C3
n n

n ( n + 1) ( n + 2) n ( n + 1) ( n −1) ∑ ∑ Cr Cs is equal to
= + r=0s=0
6 6 (A) 2n (B) 22n
n ( n + 1) ( 2n + 1) (C) 24n (D)  none of these
=
6 Solution: (B)
n n n  n  n
48. The coefficient of x in the expansion of [ 1 + x 2 – x]–1
∑ ∑C C r s = ∑ C ∑ C  = ∑ 2
r s
n
⋅ Cr
in ascending powers of x, when | x | < 1, is r=0 s=0 r=0 s=0 r=0
Binomial Theorem  9.13

⇒ ∑ ∑C r Cs = 1 ( 22 n - 2 nC )
n
n
 n  0≤r< s≤n 2

= 2  Cr  = 2n ⋅ 2n = 22n.
r = 0  53. 79 + 97 is divisible by
(A) 16 (B)  24
52. If (1 + x)n = C0 + C1x + ... + Cnxn, then the value of (C) 64 (D)  72
∑ ∑ Cr Cs is equal to
0≤r<s≤n Solution: (C)
1 1 We have,
(A) ( 22 n - 2 nCn ) (B)  ( 22 n - 2 nCn ) 79 + 97 = (1 + 8)7 – (1 – 8)9
2 4
1 = (1 + 7C1 ⋅ 81 + 7C2 ⋅ 82 + ... + 7C7 ⋅ 87)
(C) ( 2n - 2 nCn ) (D)  none of these – (1 – 9C1 ⋅ 81 + 9C2 ⋅ 82 – ... 9C9 ⋅ 89)
2
Solution: (A) = 16 × 8 + 64 [(7C2 + ... + 7C7 ⋅ 85)
n n n
– (9C2 – ... – 9C9 ⋅ 87)]
∑∑
r=0 s=0
Cr Cs = ∑C
r=0
2
r + ∑ ∑C C
0≤r< s≤n
r s
= 64 (an integer)
Hence, 79 + 97 is divisible by 64.
⇒ 22n =
2n
Cn + 2 ∑ ∑C
0≤r< s≤n
r Cs

EXERCISES

Single Option Correct Type


1. The coefficient of x17 in the expansion of 5. If [x] denotes the greatest integer less than or equal to
(x – 1) (x – 2) (x – 3) ... (x – 18) is x, then [(6 6 + 14) 2 n+1 ]
171 (A) is an even integer (B)  is an odd integer
(A) (B) 342 (C) depends on n (D)  none of these
2
(C) – 171 (D)  684 6. The two consecutive terms in the expansion of
(3x + 2)74, whose coefficients are equal, are
24n
(A) 20th and 21st (B)  30th and 31st
2. The fractional part of 15 is
(C) 40th and 41st (D)  none of these
2 1  x
n
(A) (B)  1  T3
15 15 7. If in the expansion of 2 + x  , T = 7 and the sum
 4  2
4 of the coefficients of 2nd and 3rd terms is 36, then the
(C) (D)  none of these
15 value of x is
 22003  1 1
3. If {x} denotes the fractional part of x, then   is (A) - -
(B) 
 17  3 2
(A) 2/17 (B)  4/17
(C) 8/17 (D)  16/17 1 1
(C) (D) 
3 2
4. The sum of the coefficients of all the integral powers of
8. The interval in which x must lie so that the numeri-
x in the expansion of (1 + 2 x )
80
is cally greatest term in the expansion of (1 – x)21 has the
1 1 80 greatest coefficient is, (x > 0).
(A) (380 + 1) (B)  (3 - 1) 5 6  5 6 
2 2
(A)  ,  (B)  , 
(C) (380 + 1) (D)  (380 – 1) 6 5 6 5
 4 5  4 5
(C)  ,  (D)   , 
5 4 5 4
9.14  Chapter 9

9. If Cr stands for nCr, then the sum of the series 17. If coefficient of xn in (1 + x)101 (1 – x + x2)100 is non-
 n  n zero, then n can not be of the form
2  !  ! (A) 3t + 1 (B)  3t
 2   2  ... + (− 1) n ( n + 1) Cn2 ], (C) 3t + 2
[C02 − 2C12 + 3C22 − (D)  4t + 1
n!
18. The sum of the last ten coefficients in the expansion of
... + (− 1) n ( n + 1) Cn2 ], where n is an even positive integer, is
(1 + x)19 when expanded in ascending powers of x is
(A) 0 (B)  (–1)n/2 (n + 1)
(A) 218 (B) 219
(C) (–1) (n + 2)
n/2
(D)  (– 1)n n
(C) 218 – 19C10 (D)  1  (219 – 1)
2n
10. If (1 + 2x + x2)n = ∑ ar x r , then ar = 2
r =0
(A) (nCr)2 (B) 
n
Cr ⋅ nCr + 1 19. The number of integral terms in the expansion of
(C) 2nCr (D) 
2n
Cr + 1 ( 2 5 + 6 7 )642 is
1 + 4 x + 1 n 1 − 4 x + 1 n  (A) 105 (B)  107
1     
11. If   −    (C) 321 (D)  108
4 x + 1  2   2  
  20. The number of positive terms in the sequence
= a0 + a1x + ... + a5x5, then n equals ( n + 3)
195 P3
(A) 11 (B)  9
xn =  n
− ( n + 1) is
(C) 10 (D)  none of these Pn Pn+1

m 10 20 
 p (A) 14 (B)  11
12. The sum ∑    , (where   = 0 if p < q) is (C) 12 (D)  13
 q 
i = 0  i m − i
maximum when m is 21. The digit at unit’s place in the number 171995 + 111995
–71995 is
(A) 5 (B)  10
(A) 0 (B)  1
(C) 15 (D)  20
(C) 2 (D)  3
13. The number of distinct terms in the expansion of
n 22. The positive integer which is just greater than
 3 
 x + 1 + 1  ; x ∈ R+ and n ∈ N is (1 + 0.0001)1000 is
 x 3  (A) 3 (B)  4
(A) 2n (B) 3n (C) 5 (D)  2
(C) 2n + 1 (C)  3n + 1
23. The coefficient of xn in the polynomial (x + nC0 ) (x + 3
14. The number of terms with integral coefficients in the n
C1) (x + 5 nC2)... (x + (2n + 1) nCn) is
expansion of (171/3 + 351/2x)600 is (A) n.2n (B)  n.2n+1
(A) 100 (B) 50 (C) (n +1).2 n
(D) n.2n–1
(C) 150 (D) 101 24. The interval in which x (> 0) must be so that the great-
 3 i 
5
 3 i 
5
est term in the expansion of (1 + x)2n has the greatest
 
15. If z =  2 + 2  +  2 – 2  , then coefficient is
   
 n − 1 n   n n + 1
(A) Re (z) = 0 (B)  Im (z) = 0 (A)  , 
 (B) 
 , 
 n n − 1  n + 1 n 
(C) Re (z) > 0, Im (z) > 0 (D)  Re (z) > 0, Im (z) < 0
 n n + 2 
16. The greatest value of the term independent of x in the (C)  ,  (D)  none of these
expansion of (x sinα + x–1 cosα)10, α ∈ R, is n + 2 n 
10 ! 25. If n is positive integer and k is a positive integer not
(A) 10 ! (B)  2
25 (5!) exceeding n, then
2
n 
3  Ck 

(C)
1 10 !
(D)  none of these ∑ k  
 C  , where Ck = Ck, is
n

25 (5!) 2 k =1 k −1
Binomial Theorem  9.15

2 (A) 2 (B)  0
(A) n( n + 1) ( n + 2) (B)  n( n + 1) ( n + 2) (C) 1/2 (D)  1
12 12
33. If n is an even integer and a, b, c are distinct, the num-
n( n + 1) 2 ( n + 2) ber of distinct terms in the expansion of (a + b + c)n
(C) (D)  none of these
6 + (a + b – c)n is
6
 1   n
2
 n + 1
2
 1/12 
26. If the fourth term in the expansion of  x log x +1 + x  (A)   
 2 
(B) 
   2 
is equal to 200 and x > 1, then x is equal to  n + 2 
2
 n + 3 
2

(C)  
 2   2 
(D) 
(A) 10 2 (B) 10
(C) 104 (D)  none of these
34. Coefficient of t 24 in (1 + t 2)12 (1 + t 12) (1 + t 24) is
27. The coefficient of λnµn in the expansion of [(1 + λ) (1 + µ)
(A) 12C6 + 3 (B)  12C6 + 1
(λ + µ)]n is
(C) C6
12
(D)  12
C6 + 2
n n

(A) ∑C
r=0
2
r (B)  ∑C
r=0
2
r +2 35. (mC0 + mC1 – mC2 – mC3) + (mC4 + mC5 – mC6 – mC7) + ...
= 0 if and only if for some positive integer k, m =
n n
(A) 4k (B) 4k + 1
(C) ∑C
r=0
2
r +3 (D) Cr3 ∑
r=0
(C) 4k – 1 (D)  4k + 2
36. If the sum of the coefficients in the expansions of
28. If  α = 183 + 73 + 3. 18.7.25, and (1 + 2x)m and (2 + x)n are respectively 6561 and 243,
β  = 36 + 6.243.2 + 15.81.4 + 20.27.8 then the position of the point (m, n) with respect to the
+ 15.9.16 + 6.3.32 + 64 circle x2 + y2 – 4x – 6y – 32 = 0
then the value of αβ–1 is (A) is inside the circle
(A) 1 (B)  5 (B) is outside the circle
(C) 25 (D)  100
n−3
(C) is on the circle
 1  (D) can not be fixed

29. If there is a term containing x in  x + 2  , then
2r
 x  37. Let n(> 1) be a positive integer. Then largest integer m
(A) n –2r is a positive integral multiple of 3 such that (nm + 1) divides 1 + n + n2 + ... + n255 is
(B) n – 2r is even (A) 128 (B)  63
(C) n – 2r is odd (C) 64 (D)  32
(D) none of these
38. The coefficient of xn in the expansion (2x + 3)n –
30. If Pn denotes the product of the binomial
Pn+1 coefficients (2x + 3)n–1 (5 – 2x) + (2x + 3)n–2 (5 – 2x)2 + ... + (– 1)n
in the expansion of (1 + x)n, then equals
Pn (5 – 2x)n is
( n + 1) n nn 1
(A) (B) (A) 2n (B) (n + 1)2n
n! n! 8
(C) (n + 1)2n–3 (D)  – (n + 1)2n–2
( n + 1) n ( n + 1) n+1
(C) (C)
( n +1)! ( n +1)! 39. The value of the sum of the series 3nC0 – 8nC1 +
13n C2 – 18nC3 + ... upto (n + 1) terms is
31. The coefficient of the term independent of x in the
10
(A) 0 (B)  3n
 x +1 x − 1  (B) 5 n
(D)  none of these
expansion of  2 / 3 − 
 is
 x − x1/ 3 + 1 x − x1/ 2  4
40. The value of 2(nC0) + 3  (nC1) +  (nC2) + 5  (nC3)...is
(A) 210 (B)  105 2 3 4
(C) 70 (D)  112 n
2 (1 − n) − 1 n
2 ( n + 3) − 1
1 10 2 n 10 2 (A) (B) 
32. The value of n − n C2 + n 2n
C2 n +1 n +1
81 81 81
10 3
10 2n 2n − 1 2n + 2
− n 2 nC3 + ... + n is (C) (D) 
81 81 n +1 n −1
9.16  Chapter 9

41. Which of the following expansions will have term (A) 0 (B)  2
containing x3 ? (C) 7 (D)  8
 −1 3 25  3 1 24 48. If C0, C1, C2, ..., Cn denote the binomial coefficients in
 x 5 + 2 x 5   x 5 + 2 x− 5  the expansion of (1 + x)n, then
(A)   (B)  
  
23 22
n 1 + r log e 10
− 
 3  3 1 r n
 5
1
5  x 5 + 2 x− 5  ∑ (−1) ⋅ Cr ⋅ =
(C)  x − 2 x  (D)   r=0 (1 + log e 10 n ) r
   (A) 2 (B)  1
(C) 0 (D)  none of these
42. The coefficient of x7 in the expansion of (1 – x – x2 +
x3)6 is 49. If C0, C1, C2, ..., Cn are the coefficients of the expansion
(A) 132 (B)  144 n Ck
(C) –132 (D)  –144 of (1 + x)n, then the value of ∑ is
0 k +1
2 n -1
If n is a positive integer, then ( 3 + 1) − ( 3 − 1)
2n 2n
43. is (A) 0 (B) 
n
(A) an irrational number 2n+1 −1
(B) an odd positive integer (C) (D)  none of these
n +1
(C) an even positive integer
(D) a rational number other than positive integers 50. Larger of 9950 + 10050 and 10150 is
(A) 10150 (B)  9950 + 10050
44. If ai(i = 0, 1, 2, ..., 16) be real constants such that for (C) both are equal (D)  none of these
every real value of x, (1 + x + x2)8 = a0 + a1x2 + a2x2 ...
+ a16x16, then a5 is equal to 51. The greatest coefficient in the expansion of
(A) 502 (B)  504 (x + y + z + w)15 is
(C) 506 (D)  508 15! 15!
n (A) (B) 
45. Statement-1:  ∑ (r + 1)
r=0
n
Cr = ( n + 2)2 n −1
3!( 4 !)3 (3!)3 4 !

n (C) (D)  none of these


Statement-2: ∑ (r + 1)
r=0
n
Cr x r = (1 + x ) n + nx(1 + x ) n − 1
10
52. The sum of the series ∑ 20 Cr is
n r n n −1
r + 1) Cr x = (1 + x ) + nx(1 + x ) r= 0

(A) 219 – 1 ⋅ 20C10 (B) 219 + 1 ⋅ 20C10


(A) Statement-1 is false, Statement-2 is true 2 2
(B) Statement-1 is true, Statement-2 is true, Statement-2 (C) 219 (D)  220
is a correct explanation for Statement-1
(C) Statement-1 is true, Statement-2 is true; Statement-2 53. n+1
C2 + 2 [2C2 + 3C2 + 4C2 + ... + nC2] =
is not a correct explanation for Statement-1 n ( n + 1) ( 2n + 1)
(D) Statement-1 is true, Statement-2 is false (A) (B)  n ( n +1)
6 2
 1
46. In a binomial distribution B n, p =  , if the prob- n ( n -1) ( 2n -1)
 4 (C) (D)  none of these
6
ability of at least one success is greater than equal to
9 54. If A = 2nC0 · 2nC1 + 2nC1 2n – 1C1 + 2nC2 2n – 2C1 + ..., then A
, then n is greater than is
10
1 1 (A) 0 (B)  2n
(A)   3 (B) 
4
log10 - log10 log10 + log10 3
4
(C) n 2 2n
(D) 1
9 4 55. The greatest integer which divides the number
(C) 3 (D) 
log10 4
- log10 log10 - log10 3
4
101100 – 1 is
(A) 100 (B)  1,000
47. The remainder left out when 82n – (62)2n+1 is divided by (C) 10,000 (D)  1,00,000
9 is
Binomial Theorem  9.17

56. Given positive integers r > 1, n > 2 and the coeffi- 64. The coefficient of xn in the polynomial (x + nC0 ) (x + 3
cients of (3r)th term and (r + 2)th term in the binomial n
C1) (x + 5 nC2)... (x + (2n + 1) nCn) is
expansion of (1 + x)2n are equal, then r = (A) n.2n (B)  n.2n+1
(A) n , n even (B)  n (C) (n +1).2 (D) n.2n–1
n

2 2
65. If n is an even integer and a, b, c are distinct, the num-
(C) n (D) 1
ber of distinct terms in the expansion of (a + b + c)n
57. Let n be a positive integer such that + (a + b – c)n is
2 2
(1 + x + x2)n = a0 + a1x + a2x2 + ... + a2nx2n, then ar =  n  n + 1
(A)   (B) 

(A) an–r, 0 ≤ r ≤ 2n (B)  a2n, 0 ≤ r ≤ 2n  2   2 
(C) a2n – r, 0 ≤ r ≤ 2n (D)  none of these 2 2
 n + 2   n + 3 
(C)  
 2   2 
(D) 
 22003 
58. If {x} denotes the fractional part of x, then   is
 17 
2 4 66. (mC0 + mC1 – mC2 – mC3) + (mC4 + mC5 – mC6 – mC7) + ...
(A) (B) 
17 17 = 0 if and only if for some positive integer k, m =
8 16 (A) 4k (B)  4k + 1
(C) (D) 
17 17 (C) 4k – 1 (D)  4k + 2
59. If [x] denotes the greatest integer less than or equal to 67. Let n(> 1) be a positive integer. Then, largest integer m
x, then [(6 6 + 14) 2 n+1 ] such that (nm + 1) divides 1 + n + n2 + ... + n255 is
(A) is an even integer (B)  is an odd integer (A) 128 (B)  63
(C) depends on n (D)  none of these (C) 64 (D)  32
4
60. If Cr stands for nCr, then the sum of the series 68. The value of 2(nC0) + 3  (nC1) +  (nC2) + 5  (nC3)...is
 n  n 2 3 4
2  !  !
 2   2  n
2 (1 − n) − 1 n
2 ( n + 3) − 1
[C02 − 2C12 + 3C22 − (A) (B) 
n! n +1 n +1
... + (− 1) n ( n + 1) Cn2 ], where n is an even positive 2n − 1 2n + 2
integer, is (C) (D) 
n +1 n −1
(A) 0 (B)  (–1)n/2 (n + 1)
(C) (–1) (n + 2)
n/2
(D)  (– 1)n n 69. If A = 2nC0 2nC1 + 2nC1 2n–1C1 + 2nC2 2n–2C1 + ..., then A is
61. The sum of the series (A) 0 (B)  2n
1 1.4.1 1.4.7 1 (C) n 2 (D) 
2n
1
1+ 2 + + + ... is
3 1.2.34 1.2.3 36 70. The coefficient of λnµn in the expansion of
1
3  3  3 [(1 + λ) (1 + µ) (λ + µ)]n is
(A)  
(B) 
 2  Cr2+2
n
2 (A) n C 2 (B) 
r
1
n 3
1  1  3 (C) Cr2+3 (D) 
n Cr
(C)  
(D) 
3  3 
71. The sum to (n + 1) terms of the series
62. If coefficient of xn in (1 + x)101 (1 – x + x2)100 is non-
zero, then n cannot be of the form C C C C
0 − 1 + 2 − 3 +... is
(A) 3t + 1 (B)  3t 2 3 4 5
(C) 3t + 2 (D)  4t + 1 1 1
(A) (B) 
63. The digit at unit’s place in the number 171995 + 111995 n ( n + 1) n+2
–71995 is 1
(C) (D)  none of these
(A) 0 (B)  1 n +1
(C) 2 (D)  3
9.18  Chapter 9

72. Let R = (5 5 + 11)2n + 1 and f = R – [R] where [ ] 1 1


1- mn 1-
denotes the greatest integer function. Then R f = (A) 2 (B) 2mn
(A) 22n + 1 (B)  W24n + 1 2 m -1 2 n -1
(C) 4
2n + 1
(D)  none of these 1
1- m
(C) 2 (D) none of these
73. Let n and k be positive integers such that n ≥ k ( k +1)
2 2 n -1
. The number of solutions (x1, x2, ..., xk), x1 ≥ 1, x2 ≥ 2, 81. If (1 + x)n = C0 + C1 x + C2 x2 + ... + Cn xn, then for n
..., xk ≥ k, all integers, satisfying x1 + x2 + ... + xk = n, is
even, C02 − C12 + C22 − ... + (−1) n Cn2 is equal to
(A) mCk – 1 (B)  m
Ck
(C) Ck + 1
m
(D)  none of these (A) 0 (B)
(-1) n / 2 nCn 2
where m = 1 (2n – k2 + k – 2) (C) n (D) none of these
2 Cn 2
n
74. ∑ n Cr sin rx cos ( n − r ) x = n
n
Ck
r =0 82. ∑ =
k = 0 ( k + 1) ( k + 2)
(A) 2n – 1 sin (n – 1) x (B) 2n sin nx
(C) 2n – 1 sin nx (D)  none of these 2n+1 − n − 3 2n+ 2 − n − 3
(A) (B) 
75. nCn + n + 1Cn + n + 2Cn + ... + n + kCn = ( n + 1) ( n + 2) ( n + 1) ( n + 2)
(A) n + k – 1Cn + 1 (B)  n+k
Cn + 1 2n+ 2 − n + 3
(C) n+k+1
Cn + 1 (D)  none of these (C) (D)  none of these
( n + 1) ( n + 2)
76. If Sn = 1 + q + q2 + q3 + ... + qn and
2 n 83. For all n ∈ N, the integer just above ( 3 + 1)2n is
 q + 1  q + 1  q + 1 divisible by
S’n = 1 +  + + ... +  , q ≠ 1, then
 2   2   2 

(A) 2n + 1 (B)  2n + 1
n+1
C1 + n + 1C2 ⋅ S1 + n + 1C3 ⋅ S2 + ... + n + 1Cn + 1 ⋅ Sn = (C) 2 + 1
n + 1
(D)  none of these
(A) 2n – 1 ⋅ S’n (B)  2n ⋅ S’n 84. If C0, C1, C2, ..., Cn be the coefficients in the expansion
(C) 2 ⋅ S’n
n+1
(D)  none of these of (1 + x)n, then
77. If (1 + x)15 = C0 + C1 x + C2 x2 + ... + C15 x15, then the 22 ⋅ C0 23 ⋅ C1 2 n+ 2 ⋅ Cn
value of C2 + 2 C3 + 3 C4 + ... + 14 C15 is + + ... + is equal to
1⋅ 2 2⋅3 ( n + 1) ( n + 2)
(A) 219923 (B) 16789
(C) 219982 (D)  none of these 3n+1 − 2n − 5 3n+ 2 − 2n − 5
(A) (B) 
78. If a0, a1, a2, ..., a2n be the coefficients in the expansion ( n + 1) ( n + 2) ( n + 1) ( n + 2)
of (1 + x + x2)n in ascending powers of x, then n+ 2

a02 − a12 + a22 − a32 + ... − a22n−1 + a22n = (C) 3 + 2n − 5 (D)  none of these
( n + 1) ( n + 2)
(A) a2n (B)  an
(C) a0 (D)  none of these 85. mCr + mCr – 1 ⋅ nC1 + mCr – 2 ⋅ nC2 + ... + mC1 ⋅ nCr – 1 + nCr =
(A) m + nCr – 1 (B)  m+n
Cr
79. The coefficient of x50 in the expression (C) Cr + 1
m+n
(D)  none of these
(1 + x)1000 + 2x (1 + x)999 + 3x2 (1 + x)998 + ... + 1001 x1000
is 86. If a, b, c and d are any four consecutive coefficients of
(A) 1000C50 (B)  1001
C50 a+b b+c c+d
any binomial expansion, then , , are
(C) C50
1002
(D)  none of these in a b c

80. The sum of the series (A) A.P. (B)  G.P.


 r r r 
∑ (−1) r ⋅ nCr  1 + 3 + 7 + 15 + ... to m terms (C) H.P. (D)  none of these
n

2 r
2 2 r
2 3 r
2 4 r 
r =0
  87. The last two digits of the number 3400 are
is (A) 38 (B)  27
(C) 01 (D)  none of these
Binomial Theorem  9.19

C0 C1 C C (A) 0 (B)  1
88. The sum − + 2 − 3 + ... to (n + 1) terms is (C) –1 (D)  none of these
1.2 2.3 3.4 4.5
1 2n 94. The number of irrational terms in the expansion of
(A) (B) 
( n + 2) ( n + 2)
( )
100
8 5 + 6 2 is
n
2 −1
(C) (D)  none of these (A) 96 (B)  97
( n + 2)
(C) 98 (D)  none of these
89. The sum of the series
(1.2) C2 + (2.3) C3 +......+ (n – 1.n) Cn is 95. Let n be an odd natural number greater than 1. Then,
(A) n (n –1)2n –1 (B)  n (n –1)2n –2 the number of zeros at the end of the sum 99n + 1 is
(C) n (n –1)2 n
(D)  none of these (A) 2 (B)  3
(C) 4 (D)  none of these
90. If n is an even positive integer and k = 3n , then
2
n 1
k 96. ∑ =
∑ (−3) r −1 3nC2 r −1 = r = 0 ( 2r )!( 2n − 2r )!
r =1
(A) 1 (B)  –1 22n 22 n-1
(C) 0 (D)  none of these
(A) (B) 
( 2n)! ( 2n)!
91. The coefficient of x301 in the expansion of
22 n + 1
(1 + x)500 + x (1 + x)499 + x2 (1 + x)498 +.... .+ x500 is
(C) (D)  none of these
( 2n)!
(A) 501C301 (B) 500C301
(C) 501C300 (D) none of these 97. The coefficient of xn in polynomial
(x + 2n + 1C0) (x + 2n +1C1)(x + 2n +1C2)....(x + 2n + 1Cn) is
( 6)
2n
(A) 22n + 1 (B)  22n
92. The fractional part of , n ∈ N is equal to (C) 2 2n – 1
(D)  none of these
5 32 32
1 1 98. If 7 divides 32 , the remainder is
(A) (B) 
3 5

(A)
2 (B) 
4
1
(C) (D)  none of these
6
(C) 8 (D)  none of these
93. The coefficient of xn in the expansion of (x + C0)
(x –3C1) (x + 5C2)..... up to (n + 1) terms, where Cr =
n
Cr is equal to

More than One Option Correct Type

10
is equal to 21 and it is known that the binomial coef-
 3  ficient of the 2nd, 3rd and 4th terms in the expansion
100. If the 4th term in the expansion of 2 + x  has the
 8  represent respectively the first, third and fifth terms of
maximum numerical value, then the range of values of an A.P. (the symbol log stands for logarithm to the base
x is 10), is
64
(A) – 2 ≤ x ≤ 2 (B)  – ≤x≤–2 (A) 1 (B)  0
21 (C) 2 (D)  none of these
(C) 2 ≤ x ≤ 64 (D)  none of these
21 103. nC0 2nCm – nC1 2n–2Cm + nC2 2n – 4 Cm –.....=
101. Three consecutive binomial coefficients can never be in  n  2 n − m
(A) G.P. (B)  H.P. (A)  2 if m ≥ n
m − n
(C) A.P. (D)  A.G.P.
(B) 0 if m < n
102. The value of x, for which the 6th term in the expansion
 n  2 n + m
(C)  2  if m ≥ n
m − n
m
of the binomial  2log( 10 − 3 ) + 5 2( x − 2 )log 3 
x

  (D) 1 if m < n
9.20  Chapter 9

Passage Based Questions


Passage 1 (A) 4th term (B)  5th term
If x, y are real numbers and x,y > 0, then to find the greatest (C) 6th term (D)  none of these
term in the expansion of (x + y)n, we proceed as follows:
20

Let Tr + 1 and Tr be (r + 1)th and rth terms, respectively  1 
3 1 +  is
106. The greatest term in the expansion of
in the expansion of (x + y)n. Then,  3 
Tr + 1 n
C r x n− r y r ( n − r + 1) y 25840 24840
= n = (A) (B) 
n−r + 1 r −1
Tr Cr −1 x y rx 9 9
26840
(C) (D)  none of these
Tr + 1  ( n + 1) y  x + y  9
∴ –1 =  − r  
Tr  x+y  rx  . 10
 3x 
Now, two cases arise: 1 07. If 4th term in the expansion of 2 +  has the
 8
( n + 1) y
Case I : is an integer greatest numerical value, then x belongs to
x+y
( n + 1) y  64 64 

Let = k, 0 < k ≤ n (A) (– ∞, –2] ∪ [2, ∞) (B)  − , 
x+y  21 21
 64   64 
T r +1 (C) − , −2 ∪ 2,  (D)  none of these

Then, −1 > 0 for 1 ≤ r < k  21   21
Tr
i.e., Tr < Tr + 1 for 1 ≤ r < k, Tr > Tr + 1 for k + 1 < r ≤ n Passage 2
and Tk +1 = Tk.
Let a and b be positive integers and b not a perfect square,
Thus, there are two greatest terms, the kth term and the then for every positive integer n, the number is
(a + b )n
(k + 1)th term having equal values. irrational. Also,

( n + 1) y (a + b )n + (a − b )n
Case II : x + y is not an integer. = 2  n C0 a n + nC2 a n−2 ( b ) 2 + nC4 a n−4 ( b ) 4 + ....

 

Let k be its integral part.

Clearly, R.H.S. is an even integer say E.
T r +1
Then, T −1 > 0 for 1 ≤ r < k
n
r Let ( a + b ) = I + F, where I is the integral part and

i.e., Tr < Tr + 1 for 1 ≤ r < k, Tr > Tr + 1 for k + 1 ≤ r ≤ n
n
F is the fractional part of ( a + b ) i.e., 0 < F < 1.
and, Tk + 1 > Tk .
( )
n
Let a − b
= F′, 0 < F ′ < 1
Thus , there is only one greatest term, the (k + 1)th
Then, I + F + F ′ = E.
( n + 1) y
term, where k is the integral part of .
x+y As, I and E are integers, F + F′ , must be an integer.
104. The greatest term in the expansion of ( 1 + x)10, when
But 0 < F + F′ < 2 ⇒ F + F′ = 1 ⇒ I = E – 1.
2
x = is
3 (a + b )n
Thus, integral part of is
4 3
 2  2 (a + b ) n
(a - b ) n
(A) 210   (B) 6300   + .
 3   3 
n
The fractional part of ( a + b ) is 1– ( a - b )
n
 2
5
(C)   (D)  none of these
 3  108. Let R = (5 + 2 6 )n and f = fractional part of R, then
105. The numerically greatest term in the expansion of R (1 – f ) =
1 (A) 1 (B)  –1
(3 –5x)15, when x = is (C) 0 (D)  none of these
5
Binomial Theorem  9.21

109. [(3 + 5 )2n] + 1, where [x] denotes the integral part of 110. If n ∈ N such that (7 + 4 3 )n = I + f, where I ∈ N and
x, is divisible by 0 < f < 1. Then, the value of ( I + f ) (I – f) is
(A) 2n –1 (B) 2n (A) 0 (B)  1
(C) 2 n+1
(D)  none of these (C) 72n (D)  22n

Match the Column Type

111. 112.

Column–I Column–II Column–I Column–II


(A) If 7 is divided by 25, then the 1. 8
103
(A) The number of integral terms in the 1. 210
remainder is expansion of (51/2 + 71/8) 1028 is
(B) The sum of rational terms in the 2. 225 (B) The coefficient of the term 2. 2520
expansion of ( 2 + 31/15)10 is
independent of x in the expansion of
(C) For all n ∈ N, 24n – 15n – 1 is 3. 18 10
divisible by  x +1 x − 1 
  is
 x 2 / 3 − x1/ 3 + 1 − x − x1/ 2 
(D) When 599 is divided by 13, the 4. 41
remainder is (C) The coefficient of x2y3z5 in the 3. 129
expansion (x + y + z)10 is
(D) The least remainder when 1730 is 4. 4
divided by 5 is

Assertion-Reason Type
Instructions: In the following questions an Assertion (A) is n +1 n
n +1
given followed by a Reason (R). Mark your responses from Reason:  Cr + 1 = Cr
r +1
the following options:
(A) Assertion(A) is True and Reason(R) is True; Reason(R) 1 15. Assertion:  The coefficient of xn in the
is a correct explanation for Assertion(A) expansion (2x + 3)n – (2x + 3)n–1 (5 – 2x) +
(2x + 3)n–2 (5 – 2x)2 + ... + (– 1)n
(B) Assertion(A) is True, Reason(R) is True; Reason(R)
(5 – 2x)n is (n + 1)2n
is not a correct explanation for Assertion(A)
(C) Assertion(A) is True, Reason(R) is False Reason: an + an – 1b + an–2b2 + ... + bn
(D) Assertion(A) is False, Reason(R) is True an + 1 − bn + 1
=
113. Assertion: If n is a positive integer and k is a positive a−b
integer not exceeding n, then 116. Assertion:  The interval in which x(x > 0) must lie so
 C 
2 that the numerically greatest term in the expansion of
n
∑ k  k  , where Ck = nCk, is
3

 Ck −1  5 6
k =1 (1 – x)21 has the greatest coefficient is,  ,  .
 6 5 

n ( n + 1) 2 ( n + 2) Reason: If n is odd, then numerically greatest coeffi-
12 n n
Cn+1
Ck n
C n − k +1 cient in the expansion of (1 – x)n is Cn-1 or .
Reason:  =n k = 2 2
Ck −1 Ck −1 k 1 17. Assertion: If n is even positive integer, then the con-
114. Assertion: If Pn denotes the product of the binomial dition that the greatest term in the expansion of (1 + x)
coefficients in the expansion of
n
Pn+1
n
may have the greatest coefficient also is <x<
( n + 1) n n+2

(1 + x)n, then equals
Pn n!
9.22  Chapter 9

n+2
. Reason: nC2 + nC4 + ... = 2n – 1
n
Reason: For even positive integer, the greatest coeffi- 11
C0 11
C1 11 C2
n 120. Assertion: The value of + + +
cient in the expansion of (1 + x)n is Cn 2 . 11
1 2 3
....+ C11 is 1 (212 −1)
118. Assertion:  Sum of the infinite series 12 12
2 1 2 5 1 2 5 8 1 n n–1
1 + ⋅ + ⋅ ⋅ 2 + ⋅ ⋅ ⋅ 3 + ...∞ is Reason: For 0 ≤ k ≤ n, nCk = . Ck–1
3 2 3 6 2 3 6 9 2 k
21/3. 1 21. Assertion: If a1, a2,...an are in A.P. and Sn is the sum of
n ( n -1) 2 first n terms, then
Reason: (1 + x)n = 1 + nx + x +
2! n

∑ Ck Sk =2 (na1 + Sn)
n n–2

n ( n − 1) ( n − 2) 3 k=0
x + ... ∞ , where n is rational..
3! n
Reason:  ∑ k .n Ck = n2n–1
k=0

C C C n
1 19. Assertion:  The value of 1 + 3 + 5 +... is equal
and ∑ k 2 .n Ck = n2 n–1 + n (n –1)2n–2
2 4 6 k=0
2n − 1
to
n +1

Previous Year’s Questions

121. The coefficient of x5 in (1 + 2x + 3x2 + ...)−3/2 is: 126. If the coefficients of rth, (r + 1)th and (r + 2)th terms in
(A) 21 (B)  25 [2002] the binomial expansion of (1 + y)m are in A. P., then m
and r satisfy the equation [2005]
(C) 26 (D)  none of these
(A) m − m(4r − 1) + 4r − 2 = 0
2 2

122. If | x | < 1, then the coefficient of xn in expansion of (1 (B) m2 − m(4r + 1) + 4r2 + 2 = 0


+ x + x2 + x3 + ...)2 is : [2002] (C) m2 − m(4r + 1) + 4r2 − 2 = 0
(A) n (B)  n−1 (D) m2 − m(4r − 1) + 4r2 + 2 = 0
(C) n + 2 (D)  n + 1 6
127. The value of 50C4 + å 56-rC3 is [2005]
r -1
123. The number of integral terms in the expansion of
( 3 + 8 5) 256 is [2003] (A) 55C4 (B) 
55
C3
(A) 32 (B)  33 (C) 56C3 (D) 
56
C4
(C) 34 (D)  35
11
124. The coefficient of the middle term in the binomial é æ öù
128. If the coefficient of x7 in ê ax 2 + çç 1 ÷÷ú equals the
expansion in powers of x of (1 + αx)4 and of (1–αx) is êë èç bx ø÷úû
the same if α equals [2004] 11
é æ öù
(A) -
5
(B) 
3 coefficient of x−7 in ê ax 2 - çç 1 ÷÷ú , then a and b sat-
3 5 êë èç bx ø÷úû
3 10 isfy the relation [2005]
(C) -
(D)  (A) a − b = 1 (B) a + b = 1
10 3
125. The coefficient of xn in expansion of (1 + x) (1 − x)n is a
(B) = 1 (D) ab = 1
(A) (n – 1) (B)  (– 1)n (1 − n) [2004] b
(C) (–1) (n– 1) (D) (–1) n
n–1 2 n–1
Binomial Theorem  9.23

129. If x is so small that x3 and higher powers of x may be 135. The remainder left out when 82n − (62)2n +1 is divided
æ 1 ö
3 by 9 is [2008]
(1 + x )3/ 2 - çç1 + x ÷÷÷ (A) 0 (B) 2
neglected, then çè 2 ø may be approxi-
(C) 7 (D) 8
mated as (1- x )1/ 2 [2005]
3 2 136. The coefficient of x7 in the expansion of the expression
(A) 1 − x (B) 3x + 3 x 2 (1 – x − x2 + x3)6 is [2011]
8 8
(A) –132 (B) –144
3 x 3 (c)132 (D) 144
(C) - x 2 (D) - x 2
8 2 8 137. If n is a natural number, then ( 3 + 1) 2 n - ( 3 -1) 2 n is
130. If the expansion in powers of x of the function (1) an irrational number [2012]
(2) an odd positive integer
1
is a0 + a1x + a2x2 + a3x3 + … , then an is (3) an even positive integer
(1- ax )(1- bx ) (4) a rational number other than positive integers
bn - an an - bn 138. If x = -1 and x = 2 are extreme points of
(A) (B)  [2006]
b-a b-a f ( x ) = a log x + b x 2 + x , then [2013]

(a)
a n+1 - b n+1
(D)
b n+1 - a n+1 (A) a = -6, b = 1 (B) a = -6, b = - 1
b-a b-a 2 2
131. For natural numbers m, n if (1 - y)m (1 + y)n = 1 + a1y (C) a = 2, b = - 1 (D) a = 2, b = 1
+ a2y2 + … , and a1 = a2 = 10, then (m, n) is 2 2
(A) (20, 45) (B)  (35, 20) [2006]
139. If the coefficients of x 3 and x 4 in the expansion of
(C) (45, 35) (D)  (35, 45)
(1 + ax + bx 2 )(1- 2 x )18 , in powers of x, are both zero,
132. In the binomial expansion of (a - b) , n ≥ 5, the sum
n
then (a, b) is equal to [2014]
a æ ö æ ö
of 5th and 6th terms is zero, then equals [2007] (A) çç16, 251÷÷ (B) çç14, 251÷÷
b çè ÷ çè ÷
5 6 3 ø 3 ø
(A) (B)
n- 4 n- 5 æ ö æ 272 ö÷
(C) çç14, 272 ÷÷ (D) çç16, ÷
(C)
n- 5
(D)
n- 4 èç 3 ø÷ çè 3 ÷ø
6 5
140. If X ={4 - 3n -1: n Î N } and Y = {9( n -1) : n Î N } ,
n

133. The sum of the series [2007] where N is the set of natural numbers, then the set
20
C0 − C1 + C2 − C3 + … − … + C10 is
20 20 20 20
X È Y is equal to [2014]
(A) − 20C10 (B)  1 20C10 (A) N (B) Y -X
2 (C) X (D) Y
(C) 0 (D)  2 0
C10 141. The sum of the coefficients of integral powers of x in
æ 1ö ( )
50
the binomial expansion of 1- 2 x is: [2015]
134. In a binomial distribution B çççn, p = ÷÷÷ , if the proba-
è 4ø
bility of at least one success is greater than or equal to (A) 1 (350 ) (B) 1 (350 -1)
9 2 2
, then n is greater than  [2008]
10 (C) 1 (250 + 1) (D) 1 (350 + 1)
2 2
(A) 1 (B) 1
4 3 43
log - log10
10 log + log10
10

(C) 9 (D) 4
4 3 43
log - log10
10 log - log10
10
9.24  Chapter 9

ANSWER KEYS

Single Option Correct Type


1. (C) 2.  (A) 3. (B) 4.  (C) 5. (D) 6.  (A) 7.  (B) 8.  (C) 9. (A) 10. 
(A)
11.  (A) 12.  (B) 13. 
(A) 14.  (C) 15. 
(C) 16.  (B) 17.  (A) 18.  (C) 19. 
(C) 20. 
(A)
21.  (C) 22.  (B) 23. 
(C) 24.  (C) 25. 
(B) 26.  (C) 27.  (B) 28.  (C) 29. 
(C) 30. 
(A)
31.  (B) 32.  (C) 33. 
(D) 34.  (C) 35. 
(B) 36.  (D) 37.  (B) 38.  (C) 39. 
(A) 40. 
(A)
41.  (C) 42.  (C) 43. 
(C) 44.  (B) 45. 
(C) 46.  (C) 47.  (A) 48.  (C) 49. 
(C) 50. 
(A)
51.  (A) 52.  (B) 53. 
(A) 54.  (C) 55. 
(C) 56.  (A) 57.  (C) 58.  (C) 59. 
(A) 60. 
(C)
61.  (B) 62.  (C) 63. 
(B) 64.  (C) 65. 
(C) 66.  (C) 67.  (A) 68.  (B) 69. 
(C) 70. 
(D)
71.  (D) 72.  (C) 73. 
(A) 74.  (C) 75. 
(C) 76.  (B) 77.  (A) 78.  (B) 79. 
(C) 80. 
(B)
81.  (B) 82. (B) 83. 
(A) 84.  (B) 85. 
(B) 86.  (C) 87.  (C) 88.  (A) 89. 
(B) 90. 
(C)
91.  (A) 92. 
(B) 93. 
(A) 94.  (B) 95. 
(A) 96.  (B) 97.  (B) 98.  (B)

More than One Option Correct Type


99.  (A), (B) and (C) 100.  (A), (B), (C) and (D) 101.  (A), (B), (C) and (D) 102.  (A) and (C)

Passage Based Questions


103. (C) 104. (B) 105. (D) 106. (A) 107.  (B) 108. (B) 109. (A)

Match the Column Type


110. (A) → 3; (B) → 4; (C) → 2; (D) → 1 111. (A) → 3; (B) → 1; (C) → 2; (D) → 4

Assertion-Reason Type
112. (A) 113. (A) 114. (A) 115. (A) 116.  (A) 117. (D) 118. (C) 119. (A) 120. (A)

Previous Year’s Questions


121. (D) 122. (D) 123. (B) 124.  (C) 125. (B) 126. (C) 127. (D) 128. (D) 129.  (C) 130. (D)
131. (D) 132. (D) 133. (B) 134.  (A) 135. (B) 136. (B) 137. (A) 138. (C) 139.  (D) 140. (D)
141. (D)

HINTS AND SOLUTIONS

Single Option Correct Type


1
1. Coefficient of x17 = – (1 + 2 + 3 + ... + 18)
= +k
15

= – 18 (1 + 18)
24n 1
2 ∴  Fractional part of is .

= – 9 × 19 = – 171 15 15

The correct option is (C)
The correct option is (B)

24n 16 n (1 + 15) n
2. We have, = = 3. 22003 = (24)500. 23
15 15 15
⇒ 22003 = 8 (16)500
1 + nC115 + nC2 152 + ... + nCn 15n ⇒ 22003 = 8 (17 – 1)500

=
15 ⇒ 22003 = 8[(17)500 – 500C1(17)499 + ...
1 + 15k – 500C499 (17) + 1]

= , where k ∈ N
15
Binomial Theorem  9.25

Substituting n = 8 in Eq. (1), we get


22003 8
⇒ = 8k + ,
17 17 23x = 1 =2–1


where k = (17)499 – 500C1(17)498 + ... + 500C499 2

such that k is an integer 1
⇒ 3x = – 1 ⇒  x =−
 22003  3


∴   = 8 The correct option is (A)
 17 
 17 7. If n is odd, then numerically greatest coefficient in the
The correct option is (C) n
Cn-1 or n Cn+1 .
expansion of (1 – x)n is
4. The coefficients of the integral powers of x are
2 2
80
C0, 80C2 ⋅ 22, 80C4 ⋅ 24, ..., 80C80 ⋅ 280
Therefore in (1 – x)21, the numerically greatest coefficient is
Now, (1 + 2)80 = 80C0 + 80C1 ⋅ 2 + 80C2 ⋅ 22 21
C10 or 21C11. So, the numerically greatest term
+ ... + 80C80 ⋅ 280  ...(1)
=  21
C11x11 or 21C10x10 and
and  (1 – 2)80 = 80C0 – 80C12 + 80C2 ⋅ 22
|  21
C10x10 | > | 21C9.x9 |
– ... + 80C80 ⋅ 280 ...(2) 21! 21!
Adding Eq. (1) and (2), we get ⇒ > x and
10 ! 11! 9 ! 12 !
380 + 1 = 2(80C0 + 80C2 ⋅ 22 + 80C4 ⋅ 24
+ ... + 80C80 ⋅ 280) 21! 21!

x> (Q x > 0)
11! 10 ! 9 ! 12 !
∴ 80
C0 + 80C2 ⋅ 22 + 80C4 ⋅ 24 + ... + 80C80 ⋅ 280 = 1 (380 + 1)
2 6 5 5 6
⇒ x < and x >   ⇒  x ∈  , 

The correct option is (A) 5 6  6 5 
2 n +1
5. Let l + f = (6 6 + 14)
The correct option is (B)
2 n +1
Assuming, f′ = (6 6 − 14)
...(1) 8. C02 − 2C12 + 3C22 − 4C32 + ... + (− 1) n ( n + 1)Cn2
2 n +1 2 n +1
Now, I + f – f′ = (6 6 + 14)
– (6 6 − 14)
2 2 2 2 n 2
= [C0 − C1 + C2 − C3 + ... + (− 1) Cn ]
⇒ I + f – f′ = 2 [ 2 n+1 C1 (6 6 ) 2 n 141
− [C12 − 2C22 + 3C32 − ... + (− 1) n nCn2 ]
2 n +1

+ C3 (6 6 ) 2 n+ 2 (14)3 + ...] n/ 2 n! n
−1 n n
= (− 1)     − (− 1) Cn
2

⇒ I + f – f′ = 2 (Integer) = even ...(2) n n
 !  ! 2 2
   
Now, 0 ≤ f < 1  2  2
Also, 0 ≤ f – f′ < 1 n/ 2 n !  n 
= (− 1) 1+ 
∴ 0 ≤ f – f′ < 0 ⇒ f – f′ = 0 n n  2 
 !  !
Substituting respective values in (2), we get 2 2
I = even integer
 n  n
The correct option is (A)
∴  2  !  !
 2   2  [C02 − 2C12 + 3C22 − ...
T3  n  n
6. Since, = 7 (given) 2  !  ! n! + (− 1) n ( n + 1) Cn2 ] = (− 1) n / 2 ( n + 2)
T2  2   2  [C02 − 2C12 + 3C22 − ...
n
C 2 ( 2 x ) n- 2 ( 4 - x ) 2 n! + (− 1) n ( n
+ 1) Cn2 ] = (− 1) n / 2 ( n + 2)
⇒ =7
n
C1 ( 2 x ) n-1 . ( 4- x )
The correct option is (C)
2n
 n − 1 1
⇒  . = 7 ...(1) 9. We have, (1 + 2x + x2)n = ∑a x r
r

 2  ( 2 x )3 r =0
2n

Also, nC2 + nC1 = 36 ⇒ (1 + x)2n = ∑a x r
r

⇒ n( n − 1) + n = 36
r =0

2 2n 2n r
⇒ n2 + n – 72 = 0 ⇒  ∑ 2n
Cr x r = ∑ a x ⇒ ar = 2nCr.
r
⇒ n = 8, – 9
r =0 r=0
n = – 9 is not possible as in Eq. (1), n – 1 should be positive.
The correct option is (C)
9.26  Chapter 9

10. Given:
Hence, Im (z) = 0
 n n 
The correct option is (B)
1 1 + 4 x + 1  1 − 4 x + 1  
  −    15. Term independent of x = 10C5 (sinα)5 (cosα)5

4 x + 1  2   2  


= 1
C5 sin5 2α
10

= a0 + a1x + a2x2 + ... + a5x5
25
10 !
 Hence, the greatest value = 1
n
1 4 x + 1  
−  −   2 5 (5!) 2

Now,   2 2  
The correct option is (C)

 n−1 16 (1 + x)101 (1 – x + x2)100 = (1 + x) (1 + x3)100
1  n C  1  4x + 1

=  1  2  2
=  (1 + x) (C0 + C1x3 + C2x6 + ... + C100x300)
4 x + 1 
Clearly in this expression xλ will be present if λ = 3t, or
3  λ = 3t + 1
1
n−3  4 x + 1 
+ C3  
n   + ... So, λ can not be of the form 3t + 2.

 3   2  
  The correct option is (A)

17. Sum of last 10 coefficients,


=n
C1 1 + nC 1 ( 4 x + 1) + nC 1 (4x + 1)2
n
2
3 n 5
2 n
2
19
C10 + 19C11 + ... + 19C19 = S (say)
r −1 Also, 19C0 + 19C1 + ... + 19C9 = 19C19 + 19C18 + ... + 19C10
1
+ ... + nCr ( 4 x + 1) 2
+ ... = S  ∴  (nCn = nCn–r)
2n
19

The expansion contains a term x if r -1 = 5  or  r = 11.


5 ∴ 2 S = ∑ 19
Cn = 219 ⇒ S = 218
2 n=0


The correct option is (A)
The correct option is (A)
m 10  20  18. (2.51/2 + 71/6)642 has a general term of the form
11. ∑   
  

i = 0  i  m − i
 642
Cr (2.51/2)642–r (71/6)r
=  642Cr 2642–r. 5321–r/2. 7r/6
= Coefficient of xm in (1 + x)10 (1 + x)20 = 30Cm
and will be rational if only r is a multiple of 2 and 6.
m = 15 (for maximum value)
∴ r must be the LCM of 2 and 6 which is 6.
The correct option is (C)
∴ r takes the values 0, 6, 12, 18,..., 642.
n
 3 1
n
  3 1  There are 108 values.
12.  x + 1 + 3  = 1 +  x + 3 
  x 
x    The correct option is (D)

 1   3 1
n 195 ( n + 3) ( n + 2) ( n + 1)
= n C0 + nC1  x 3 +
 + ... + Cn  x + 3 
n 19. xn = − >0
 3
x   x  n! ( n + 1)!

All the terms are distinct with powers (x3)0, (x3), (x3)2, ... (x3) ⇒  195 > (n + 2) (n + 3).
n
, (x3)–n,... (x3)–1. Hence, (2n+1) terms. Hence, n ≤ 11

The correct option is (C) ∴  n can take the values 1, 2, 3,..., 11.
600-r r
∴  Number of positive terms = 11
13. (d).  tr+1 = 600
Cr 17 3
35 2 x r The correct option is (B)

r 20. We have
As 0 ≤ r ≤ 600 and r and 200 - are integers ⇒  r

2 3   171995 + 111995 – 71995
should be a multiple of 6 =  (7 + 10)1995 + (1 + 10)1995 – 71995
∴  r = 0, 6, 12,... 600 = [71995 + 1995C1. 71994. 101 + 1995C2. 71993. 102 +
The correct option is (D) ... + 1995C1995. 101995] + [1995C0 + 1995C1. 101
+ 1995C2. 102 + ... + 1995C1995. 101995] – 71995
 5 3
 3  i 4 
 3   3  i 2
  + 5
C    = [1995C1. 71994. 101 + ... + 101995]
14. z = 2  
5
 + C 2   4  2  16  + [1995C1. 101 + ... + 1995C1995. 101995] + 1
 2   2  4   
 =  a multiple of 10 + 1.

= Purely real number
Binomial Theorem  9.27


Thus, the units place digit is 1. n n


The correct option is (B) ∴ Series = ∑
p =1
( n − p + 1) p 2 = ∑ (np
p =1
2
− p3 + p 2 )

21. (1 + 0.0001)1000 n n


1000 × 999 −8 1000
= 1 + 1000 × 10–4 + 10 + C3 10−12 + ...

= ∑ (n + 1) p − ∑ p
p =1
2

p =1
3

2
1
= (n + 1) (12 + 22 + 32 + ... + n2)
1 1 1 10
<1 + + + + ... = 1
=
– (13 + 23 + 33 + ... + n3)
10 100 1000 1- 9
10 ( n +1)n ( n + 1) ( 2n + 1) n2 ( n + 1) 2

= −

So, the integer just greater than the given expression must be 2. 6 4

The correct option is (D) n( n + 1) 2  2n + 1 n 

=  − 
22. Given polynomial is 2  3 2
(x + nC0) (x + 3. nC1) (x + 5. nC2) ...(x + (2n + 1). nCn) 2

=  xn+1 + xn [nC0 + 3. nC1 + 5. nC2 + = n( n + 1) ( n + 2)



12
... + (2n + 1). nCn] + xn–1 (...) + ...
The correct option is (B)
∴  Coefficient of xn in the expression is
25. Given, T4 = 200
n n n

∑ (2r +1) C n
= ∑ 2r n
Cr + ∑ n
Cr  1 
3

C3  log x +1  ( x1/12 )3 = 200


r
r=0 r=0 r=0 ⇒ 6

 x 10 
n
n
=  2
∑r r
n−1
Cr −1 + 2n

3
+
2(log10 x+1) 4
1 
 3 1
+ 
r=0
20. x = 200 ⇒ x  2(log x +1) 4
= 10
n

=  2n
∑ n−1
Cr −1 + 2 n


3 1
+ = logx 10 =
1
r=0
2 (log10 x + 1) 4 log10 x

= 2n.2n–1 + 2n = (n + 1) 2n
3 1 1

The correct option is (C) ⇒ + = where y = log10x
2 ( y + 1) 4 y
23. Greatest coefficient in the expansion of (1 + x)2n is 2nCn. We
⇒ y = – 4 or y = 1
are given 2nCn xn is the greatest term.
⇒ log10 x = – 4 or log10 x = 1
∴ 2nCn–1 xn–1 < 2nCn xn
⇒ x = 10–4 or 10
and 2nCn+1 xn+1 < 2nCn xn ∴
⇒ x = 10  ( x > 1)
2n 2n
Cn−1 Cn The correct option is (B)
⇒  2n
<x< 2n
Cn Cn+1
27. General term in (1 + λ)n (1 + µ)n (λ + µ)n is
( 2n)! n! n! ( 2n)! ( n + 1)! ( n − 1)! tp, q, r = (nCpλp) (nCqµq) (nCrλn–rµr)
⇒  <x<
( n − 1)! ( n + 1)! ( 2n)! n! n! ( 2n)! ⇒ tp, q, r = nCp nCq nCr λp+n–r µq+r
The term contains coefficient of λnµn if
n n +1
⇒  <x< p + n − r = n  and q + r = n
n +1 n
⇒ p = r  and q = n – r

The correct option is (B) Now, tr, (n–r), r contains coefficient of λnµn
n
Ck Ck n − k +1 ⇒ Coefficient of λnµn = nCr nCn–r nCr
24. We know that = n =
Ck-1 Ck -1 k ∴ Coefficient of λnµn = (nCr)3
2 The correct option is (D)
n C  n
 n − k + 1
2

∴ ∑ k  k
3
 = ∑k 3   28. The numerator (α) is of the form
 C   k 
k =1 k −1  k =1
a3 + b3 + 3ab (a + b) = (a + b)3
n
∴ α = (18 + 7)3 = 253

= ∑ k (n − k + 1)
k =1
2
Also,  β = 36 + 6C135 ⋅ 21 + 6C234 ⋅ 22 + 6C333 ⋅ 23
+ 6C4 32 ⋅ 24 + 6C533 ⋅ 25 + 6C626
Put n – k + 1 = p ⇒ k = n – p + 1.

which is an expansion of

When k = 1, p = n and when k = n, p = 1.
(3 + 2)6 = 56 = (25)3
9.28  Chapter 9


Hence, required coefficient = 10C4(– 1)4 = 210.
α ( 25)3
∴ = =1
The correct option is (A)
β ( 25)3

The correct option is (A) 1
32. Given expression = ((1 - 10) 2 n ) = 1
 1
n−3 (81) n
29. General Term in  x + 2  is
 x 
The correct option is (D)

1
k 33. Let n = 2m, m ∈ N
tk+1 = n–3Ckxn–3–k  2 
∴ (a + b + c)n + (a + b – c)n = [(a + b) + c]2m
 x 
+ [(a + b) – c]2m
⇒ tk+1 = n–3Ckxn–3(k+1)
= 2[(a + b)2m + 2mC2(a + b)2m–2 c2 +
There is a term containing x2r, if
... + 2mC2mc2m]
n – 3(k+1) = 2r
Therefore, the number of distinct terms in the expansion
⇒ n – 2r = 3(k + 1), k ∈ N
= (2m + 1) + (2m – 1) +
∴ n – 2r is a positive integral multiple of 3.
 
m + 1
The correct option is (A)
... + 3 + 1 =   . (2m + 1 + 1)
 2 
30. Given: Pn = nC0 nC1 nC2... nCn
2 2
 n  
n + 2 
= (m + 1)2 =  + 1 = 
n +1 n +1 n +1
Pn+1 C0 C1 C2 ...n+1 Cn+1
 2 



Now, = n
 2 
Pn C0 nC1 nC2 ...n Cn

The correct option is (C)
Pn+1  n+1 C   n+1 C 
⇒  =
n +1
C0  n 1   n 2  34. Coefficient of t24 in (1 + t2)12 (1 + t12) (1 + t24)
Pn  C   C 
0 1 =  coefficient of t24 in (1 + 12C6 t12 + 12C12 t24)
 C 
n +1  (1 + t12 + t24)

... n n  n+1Cn+1 =  coefficient of t24 in (12C6 + 2)t24 = 12C6 + 2
 Cn 
n +1 n The correct option is (D)
Since,  n + 1Cr + 1 = Cr
r +1 35. Consider
Pn+1  n + 1  n + 1  n + 1 (cosθ – i sinθ)m = mC0 cosmθ – mC1cosm–1θ i sinθ
⇒  = 1  ...
 1   2   n 
1
Pn + ... + mCm (– i sinθ)m  ...(1)
Pn+1 ( n + 1) n (cosθ + i sinθ)m = mC0cosmθ + mC1 cosm–1θ i sinθ
∴  = + ... + mCm (i sin θ)m  ...(2)
Pn n!
Adding (1) and (2), we get

The correct option is (A)
2cos mθ = 2[mC0cosmθ – mC2cosm–2θ sin2θ...] ...(3)
x +1 x −1 Subtracting (1) from (2), we get
31. −
x 2 / 3 − x1/ 3 + 1 x − x1/ 2 2 i sin mθ = 2i [mC1cosm–1θsinθ – mC3cosm–3
θsin3θ...] ...(4)
( x1/ 3 )3 + 13 x −1

= − Adding (3) and (4), we get
x 2 / 3 − x1/ 3 + 1 x1/ 2 ( x1/ 2 −1)
cosmθ + sinmθ = [mC0 cosmθ + mC1 cosm–1θ sinθ
( x1/ 3+ 1) ( x 2 / 3 − x1/ 3 + 1) x1/ 2 + 1 – mC2cosm–2θ sin2θ – mC3cosm–3θ sin3θ...]

= −
x 2 / 3 − x1/ 3 + 1 x1/ 2  π
⇒  2 sin mθ +  = [mC0cosmθ + mC1cosm–1θ sinθ

= x1/3 + 1 – 1 – x–1/2 = x1/3 – x–1/2  4

10 – mC2cosm–2θ sin2θ – mC3cosm–3θ sin3θ ...]

x +1 x − 1 
⇒   2 / 3  Putting θ =  p , we get

 x − x1/ 3 + 1 − x − x1/ 2  = (x – x )
1/3 –1/2 10

4
Tr + 1 for (x1/3 – x–1/2)10 is 10Cr(x1/3)10–r (– 1)r (x –1/2)r  ( m +1)π 
2 sin  1 [(mC + mC – mC
 = m/
For term independent of x,  4  2 2 0 1 2

10 - r r
– mC3) + (mC4 + mC5 – mC6 – mC7) + ...
-  = 0 ⇒  20 – 2r – 3r = 0  ⇒  r  = 4
3 2
+ (mCm–3 + mCm–2 – mCm–1 – mCm)]
Binomial Theorem  9.29

Hence, m + 1 = 4k, for given quantity to be 0.  n 3 n 4 n 5 


⇒  m = 4k – 1, where k ∈ N
=
d  n C0 x 2 + C1 x + C2 x + C3 x ...
dx  2 3 5 
The correct option is (C)  

36. Sum of coefficients in (1 + 2x)m = 3m = 6561 = 38 (1 + x ) n+1 + x( n + 1) (1+ x ) n − 1


⇒ 
⇒ m = 8 Sum of coefficient on (2 + x)n = 3n = 243 = 35 n +1
⇒n=5
3n C1 x 2 4 n C2 x 3
Since S1 <  0, so the point lies inside the circle. = 2 n C0 x +
+ ...
The correct option is (A)
2 3

put x = 1, we get
37. We have, S = 1 + n + n2 + ... + n255 2n+1 + ( n + 1)2n− 1 3 4
= 2n C0 + nC1 + nC2 + ...
1( n256 - 1) 128 n128 − 1 n +1 2 3
⇒  S = = ( n + 1)
n -1 n −1
2n ( n + 3) − 1
∴  S = (n128 + 1) (1 + n + n2 + ... + n127) =
n +1
Thus, the largest value of m for which 1 + n + n2

The correct option is (B)
+ ... + n255 is divisible by nm + 1 is 128.
The correct option is (A) 41. For option (a)
General term
38. The expansion is a G.P. with (n + 1) terms of the form
= 25Cr (x–1/5)25–r (2x3/5)r
a n+1 − b n+1
an + an–1b + an–2b2 + ... + bn = There is a term containing x3 if
a−b
− 25 + r 3r
+ =3
( 2 x + 3) n + 1 − ( 2 x − 5) n+1 5 5

=  ,
8 4r

where a = 2x + 3 and b = 2x – 5 ⇒ − 5 + =3
5
1 ∴ r = 10 i.e. an integer
∴  Coefficient of xn = [(n + 1). 2n(3) – (n + 1). 2n(– 5)]
= (n + 1) ⋅ 2n 8 Hence, T11 will be the term containing x3 and it will be
The correct option is (B) 25
C10210x3.
Similarly, try all the other options, and in none you will have
39. Let S denotes the sum of the series. General term of the
the value of r as an integer, Hence, no other binomial will
series is given by,
have the term of x3.
Tr = (– 1)r (3 + 5r) nCr, where r  =  0, 1, 2,..., n
The correct option is (A)
n

∴  S = ∑ (− 1)
r=0
r
(3 + 5r ) nCr 42. We have (1 – x – x2 + x3)6 = (1 – x)6 (1 – x2)6
Coefficient of x7 in
n n
(1 – x – x2 + x3)6 = 6C1 ⋅ 6C3 – 6C3 ⋅ 6C2 + 6C5 ⋅ 6C1
⇒  S = 3 ∑
r=0
(− 1) r nCr + 5 ∑ (− 1)
r=0
r
r nCr
= 6 × 20 – 20 × 15 + 6 × 6 = –144
The correct option is (D)
⇒  S = 3(C0 – C1 + C2 – C3 + C4 ...)
+ 5(– C1 + 2C2 – 3C3 + 4C4...)
( 3 + 1) − ( 3 − 1)
2n 2n
43.
∴  S  =  0 + 0 = 0
2n − 1 2n − 3 2n − 5
The correct option is (A) = 2[ C1 ( 3 ) + 2 n C3 ( 3 ) + 2 n C5 ( 3 )
2n
+ ....]
40. (1 + x)n = nC0 + nC1x + nC2x2 + nC3x3 2...n nCnxn 2 n − 1 2n − 3 2n − 5
2[ C ( 3 ) + 2n
C3 ( 3 ) + 2 n C5 ( 3 ) + ....]
On integrating between the limits 0 and,1 we get

= which is an irrational number
(1 + x ) n+1 − 1 n n
C1 x 2
n
C x3 n
C x4
= C x+ + 2 + 3 ...
The correct option is (A)
n +1 0
2 3 4
8

Multiplying with x and differentiating, we get 1 − x 3 
44. (1 + x + x ) = 
2 8  = (1 – x3)8(1 – x)–8
d   (1 + x ) n+1 − 1  1 − x 
 x  
 = (1 – 8C1x3 + 8C2x6 – ....)(1 + 8C1x1 + 9C2x2 + 10C3x3 + ...)
dx   n +1 
a5 = coefficient of x5 = 12C5 – 8C19C2 = 792 – 288 = 504
The correct option is (B)
9.30  Chapter 9

n n n n−1
 1   
1 − 1 
45. ∑
r=0
( r + 1) nCr = ∑
r=0
r nCr + nCr = 1 −  −
nx
 1 + nx  1 + nx  1 + nx 
n n
n
n n  nx   nx 

= ∑
r=0
r
r
n −1
Cr − 1 + ∑
r=0
n
Cr =   −   = 0.
1 + nx  1 + nx 

= n 2 + 2 = 2 (n + 2)
n–1 n n–1
The correct option is (C)

Statement-1 is true n
Cr = 1 ⋅ nCr
∑ ( r + 1) nCr x r = ∑r n
Cr x r + ∑ n
Cr x r 49. Here, tr + 1 =
r +1 r +1
n n
= 1 ⋅ n + 1Cr
∑ ∑
n −1 +1
= n
Cr − 1 x r + n
Cr x r n +1
Putting r = 0, 1, 2, ... n and adding we get, ∑ Ck
r=0 r=0 n


= nx(1 + x)n – 1 + (1 + x)n 0 k +1


Substituting x = 1 = 1 {n + 1C1 + n + 1C2 + n + 1C3 + ... + n + 1Cn + 1}

n +1
∑ (r + 1) C n
r = n 2n – 1 + 2n
1 {2n + 1 – n + 1C } = 2n+1 − 1

=

Hence Statement-2 is also true and is a correct explanation n +1
0
n +1
of Statement-1.

The correct option is (C)

The correct option is (B)
9 50. We have,
46. 1 – qn ≥ 10150 = (100 + 1)50
10
n = 10050 + 50 ⋅ 10049 + 50 × 49 ⋅ 10048 + ...

 3 1 log 3 10
⇒   ≤   ⇒  n ≥ 1× 2
 4  10 4
and, 9950 = (100 – 1)50

1 = 10050 – 50 ⋅ 10049 + 50 × 49 ⋅ 10048 – ...



⇒ n ≥ 1× 2
log10 4 - log10 3
Subtracting, we get

The correct option is (A)
10150 – 9950 = 2 [50 ⋅ 10049 + 50 × 49 × 48 × 10047 + ...]

47. 82n – (62)2n+1 = (1 + 63)n – (63 – 1)2n+1 1× 2 × 3
= (1 + 63)n + (1 – 63)2n+1 = 10050 + 2 ⋅ 50 × 49 × 48 ⋅ 10047 + ... > 10050

1× 2 × 3
= (1 + nc163 + nc2 (63)2 + .... + (63)n)
Hence, 10150 > 9950 + 10050.
+ (1 – (2n+1)c163 + (2n+1)c2 (63)2 + .... + (–1)(63)(2n+1))
The correct option is (A)
= 2 + 63(n c1 + nc2 (63) + .... + (63)n–1
– (2n+1)c1 + (2n+1)c2 (63) + .... – (63)(2n)) 51. The greatest coefficient is
∴  Remainder is 2. n!

= r  [Here, n = 15, q = 3, r = 3, k = 4]
The correct option is (B) ( q!) k −r
[( q + 1)!]
48. Let loge10 = x.
The correct option is (A)
10
r n
n 1 + r log e 10 52. We have, ∑ 20
Cr = C0 + C1 + ... + C10
20 20 20

Then, ∑ (−1) ⋅ Cr ⋅

r=0 (1 + log e 10 n ) r r= 0

But 20C0 + 20C1 + ... 20C20 = 220
n 1 + rx and, Q 20C20 = 20C0 , 20C19 = 20C1

= ∑ (−1) r nCr ⋅
r
r=0 (1 + nx)
20
C18 = 20C2 ...  and  20C11 = 20C9
n  1  n n
r
rx ∴ ∑
10
20
Cr = ( C0 + C1 + ... + C20)
20 20 20

= ∑ (−1) r ⋅ nCr   + ∑ (−1) r ⋅ ⋅ n−1Cr −1


r=0 
1 + nx  r=0 r (1 + nx ) r
r= 0
– (20C11 + 20C12 + ... + 20C20)
n  1 
r = 220 + 20C10 – (20C10 + 20C9 + ... + 20C0)
= ∑ (−1) r ⋅ nCr   ⇒ 2 [20C0 + 20C1 + ... + 20C10] = 220 + 20C10
r=0 1 + nx 
nx n  1 
r −1
∴ 20C0 + 20C1 + ... + 20C10 = 219 + 1  20C10
− ⋅ ∑ (−1) r −1 ⋅ n−1
Cr −1   2
1 + nx r = 0 1 + nx  The correct option is (B)
Binomial Theorem  9.31

53. We have, 57. We have,


n + 1C2 + 2 [2C2 + 3C2 + 4C2 + ... + nC2] 2n

(1 + x + x2)n = ∑ ar x r ...(1)
= n + 1C2 + 2 [3C3 + 3C2 + 4C2 + ... + nC2] r=0

= n + 1C2 + 2 [4C3 + 4C2 + ... + nC2] Replacing x by 1 , we get



= n + 1C2 + 2 [5C3 + ... + nC2] x
n
= n + 1C2 + 2 ⋅ n + 1C3  1 1  2n a
1 + +  = ∑ r
 2
= n + 1C2 + n + 1C3 + n + 1C3 x x  r=0 x
r

= n + 2C3 + n + 1C3
Multiplying both sides by x2n, we get
= n ( n + 1)( n + 2) + n ( n + 1)( n − 1)

2n
(1 + x + x2)n = ∑ ar x 2 n−r ...(2)
6 6 r=0

= n ( n + 1)( 2n + 1)

From (1) and (2), we have
6 2n 2n

The correct option is (A) ∑ ar x r = ∑ ar x 2 n−r
r=0 r=0

54. A = coefficient of x in [2nC0(1 + x)2n


On equating the coefficient of x2n – r on both sides, we get
+ 2nC1 (1 + x)2n – 1 + ...] a2n – r = ar for 0 ≤ r ≤ 2n.

= coefficient of x in (1 + (1 + x))2n
The correct option is (C)
= coefficient of x in (2 + x)2n 58. 22003 = (24)500. 23
2n
 x ⇒ 22003 = 8 (16)500
= coefficient of x in 22n 1 +  = n · 22n
 2 ⇒ 22003 = 8 (17 – 1)500
The correct option is (C) ⇒ 22003 = 8[(17)500 – 500C1(17)499 + ...
55. By binomial theorem – 500C499 (17) + 1]
 n ( n − 1) 2 n  2003
(1 + x)n = 1 + nx + ⋅ x ... x  ⇒ 2 = 8k + 8 ,
 2  17 17
or, (1 + x)n – 1 = nx + n ( n -1) x2 ... xn where k = (17)499 – 500C1(17)498 + ... + 500C499
2 such that k is an integer
If x = n, (1 + n)n – 1 = n + 2 n ( n -1 ) n2 ... nn  22003 
2 ∴    = 8
 17  17
 n ( n − 1) n−2 
(1 + n)n – 1 = n2 1 + ...n 
 2 
The correct option is (C)
Put n = 100,
59. Let 1 + f = (6 6 + 14) 2 n+1
 100 (100 − 1) 
(1 + 100)100 – 1 = (100)2 1 + ...100 98  2 n +1
Assuming, f′ = (6 6 − 14)
...(1)
 2 
2 n +1 2 n +1
 100 × 99  Now, I + f – f′ = (6 6 + 14)
– (6 6 − 14)
(101)100 – 1 = (100)2 1 +
...100 98 
 2  ⇒ I + f – f′ = 2 {2 n+1C1 (6 6 ) 2 n 141

Clearly (101)100 – 1 is divisible by
+ 2 n+1C3 (6 6 ) 2 n+ 2 (14)3 + ...}

(100)2 = 10000

The correct option is (C) ⇒ I + f – f′ = 2 {Integer} = even ...(2)
Now, 0 ≤ f < 1
56. We have, t3r = 2nC3r – 1 x3r – 1
Also, 0 ≤ f – f′ < 1
and, tr + 2 = 2nCr + 1 xr + 1.
∴ 0 ≤ f – f′ < 0 {Using (1)}
Given, 2nC3r – 1 = 2nCr + 1
∴ 0 ≤ f – f’ < 0 ⇒ f – f′ = 0
⇒ 3r – 1 = r + 1; or (3r – 1) + (r + 1) = 2n
Substituting respective values in (2), we get
⇒ 2r = 2 ; or 4r = 2n
I = even integer
⇒ r = 1 (impossible); or r = n . The correct option is (A)
2
But r is a positive integer greater than 1. So, the value of 60. C02 − 2C12 + 3C22 − 4C32+ ...

r is n , provided n is an even integer (> 2), otherwise r has + (− 1) n ( n + 1)Cn2


2
2 2 2 2 n 2
no value. = [C0 − C1 + C2 − C3 + ... + (− 1) Cn ]
The correct option is (A)
9.32  Chapter 9

=  xn+1 + xn {nC0 + 3. nC1 + 5. nC2 +


− [C12 − 2C22 + 3C32 − ... + (− 1) n nCn2 ]
n
... + (2n + 1). nCn} + xn–1 (...) + ...
n! −1 n
= (− 1)
n/ 2
− (− 1) 2 n
Cn ∴  Coefficient of xn in the expression is
 n   n  2 2
 !  !
 2   2 
n n n
∑ ( 2r + 1) nCr = ∑ 2r nCr + ∑ n Cr
r=0 r=0 r=0
n!  n
= (− 1) n / 2 1 + 
n n  n
n
 !  ! 2 
=  2 ∑ r n−1
Cr −1 + 2n
2 2 r=0 r
n

=  2n ∑ n−1
Cr −1 + 2n
 n  n
2 !  ! [C02 − 2C12 + 3C22 − ...
r=0
∴   2   2 
= 2n.2n + 2n = (n + 1) 2n
–1

+ (− 1) n ( n + 1) Cn2 ] = (− 1) n / 2 ( n + 2)
The correct option is (C)

The correct option is (C) 65. Let n = 2m, m ∈ N
∴ (a + b + c)n + (a + b – c)n = [(a + b) + c]2m
61. Let S =  1 + 1 + 1.4 1 + 1.4.7 1 + ... + [(a + b) – c]2m
32 1.2 34 1.2.3 36
= 2{(a + b)2m + 2mC2(a + b)2m–2 c2 +
1.4 1.4.7
 1   1  3 3  1 
2
1
3 ... + 2mC2mc2m}
⇒ S = 1 +     +   + 3 3 3   + ...
 3   3  1.2  3  1. 2 . 3  3  Therefore, the number of distinct terms
= (2m + 1) + (2m – 1) +
1  1
1 + 
 1   1  3   m + 1
3
⇒ S = 1 +     + 3   1  ... + 3 + 1 =   . (2m + 1 + 1)
   2 
 3   3  2!  3 
1  1  1 n 
2
 n + 2 
2

1 +  2 +   3 = (m + 1)2 =  + 1 = 



3  3  3  1   2   2 
+   + ...
3!  3

The correct option is (C)
which is an equivalent of

1

1 1 66. Consider,
 1 3  2 3  3 3
S = 1 −  ⇒ S =    =    (cosθ – i sinθ)m = mC0 cosmθ – mC1cosm–1θ i sinθ
 3  3   2 
+ ... + mCm (– i sinθ)m  ...(1)

The correct option is (B) (cosθ + i sinθ)m = mC0cosmθ + mC1 cosm–1θ i sinθ
62. (1 + x)101 (1 – x + x2)100 = (1 + x) (1 + x3)100 + ... + mCm (i sinθ)m  ...(2)
=  (1 + x) (C0 + C1x3 + C2x6 + ... + C100x300) Adding (1) and (2), we get
Clearly, in this expression xλ will be present if λ = 3, or 2cos mθ = 2[mC0cosmθ – mC2cosm–2θ sin2θ...] ...(3)
λ = 3t + 1 Subtracting (1) from (2), we get
So, λ cannot be of the form 3t + 2. 2 i sin mθ = 2i [mC1cosm–1θ sinθ – mC3cosm–3
The correct option is (C) θ sin3θ...] ...(4)
Adding (3) and (4), we get
63. We have,
cosmθ + sinmθ = [mC0 cosmθ + mC1 cosm–1θ sinθ
171995 + 111995 – 71995
– mC2cosm–2θ sin2θ – mC3cosm–3θ sin3θ...]
=  (7 + 10)1995 + (1 + 10)1995 – 71995
= [71995 + 1995C1. 71994. 101 + 1995C2. 71993. 102 + π  
⇒  2 sin mθ +  = [mC0cosmθ + mC1cosm–1θ sinθ
... + 1995C1995. 101995] + [1995C0 + 1995C1. 101  
 4
+ 1995C2. 102 + ... – mC2cosm–2θ sin2θ – mC3cosm–3θ sin3θ ...]

+ 1995C1995. 101995] – 71995 Putting θ =  p , we get

= [1995C1. 71994. 101 + ... + 101995] 4
 
+ [1995C1. 101 + ... + 1995C1995. 101995] + 1 2 sin  ( m +1)π  = 1 [(mC0 + mC1 – mC2
=  a multiple of 10 + 1.  4  2m / 2
Thus, the unit’s place digit is 1.
– mC3) + (mC4 + mC5 – mC6 – mC7) + ...
The correct option is (B)
+ (mCm–3 + mCm–2 – mCm–1 – mCm)]

Hence, m + 1 = 4k, for given quantity to be 0.
64. Given polynomial is
(x + nC0) (x + 3. nC1) (x + 5. nC2) ...(x + (2n + 1). nCn)
Binomial Theorem  9.33

⇒  m = 4k – 1, where k ∈ N ∴ Coefficient of λnµn = (nCr)3


The correct option is (C) The correct option is (D)

67. We have, S = 1 + n + n2 + ... + n255 71. Since (1 – x)n = C0 – C1 x + C2 x2 – C3 x3 + ...


1( n256 - 1) n128 − 1 ∴ x (1 – x)n = C0 x – C1 x2 + C2 x3 – C3 x4 + ...
⇒  S = = ( n128 + 1) 1
n -1 n −1 ⇒ ∫ x (1 − x ) n dx
∴  S = (n128 + 1) (1 + n + n2 + ... + n127) 0
1
Thus, the largest value of m for which 1 + n + n2 = ∫ (C0 x − C1 x 2 + C2 x 3 − C3 x 4 + ...) dx
0
+ ... + n255 is divisible by nm + 1 is 128.
The correct option is (A) = C0 − C1 + C2 −... up to (n + 1) terms
2 3 4
68. (1 + x)n = nC0 + nC1x + nC2x2 + nC3x3 ... nCnxn For L.H.S. put 1 – x = t, ∴ dx = – dt
On integrating w.r.t x between limits 0 and x, we get 1
∴ L.H.S. =
( n + 1)( n + 2)
(1 + x ) n+1 − 1 n 2
= n C0 x + C1 x
The correct option is (D)
n +1 2
n
C x3 n
C x4 72. Here, f = R – [R] is the fraction part of R. Thus if I is the
+ 2 + 3 ... integral part of R, then
3 4

Multiplying with x and differentiating, we get R = I + f = (5 5 + 11)2n + 1, and 0 < f < 1.
Now, Q 5 5 – 11 = 18034 < 1,

d   (1 + x ) n+1 − 1
 x   ∴ if f ‘ = (5 5 – 12)2n + 1, then 0 < f ‘ < 1
dx   n +1 


Now, I + f – f ‘ = (5 5 + 11)2n + 1 – (5 5 – 11)2n + 1
d  n n
C x3 n
C x4 n
C x 5 
 C0 x + 1 + 2 + 3 ...
2

=
= 2 [2n + 1C1 (5 5 )2n × 11
dx  2 3 5 

+ 2n + 1C3 (5 5 )2n – 2 × 113 + ...] ...(1)
(1 + x ) n+1 + x( n + 1) (1+ x ) n − 1
⇒  = An even integer.
n + 1n

n 2
= 2n C0 x + 3 C1 x + 4 C2 x ...
3
⇒ f – f ‘ must also be an integer.
2 3 ⇒ f – f ‘ = 0, Q 0 < f < 1, 0 < f ‘ < 1
put x = 1, we get ⇒ f = f ‘
2n+1 + ( n + 1)2n− 1 ∴ R f = R f ‘ = (5 5 + 11)2n + 1 (5 5 – 11)2n + 1
= 2n C0 + 3 nC1 + 4 nC2 + ...
n +1 2 3 = (125 – 121)2n + 1 = 42n + 1.
2n ( n + 3) - 1 The correct option is (C)

=
n+1 73. The number of solutions of x1 + x2 + ... + xk = n

The correct option is (B) = coefficient of tn in (t + t2 + t3 + ...) (t2 + t3 + ...)
69. A = coefficient of x in [2nC0 (1 + x)2n + 2nC1 ...(tk + tk + 1 + ...)
(1 + x)2n–1 + ...] = coefficient of tn in t1 + 2 + ... + k (1 + t + t2 + ...)k
= coefficient of x in (1 + (1 + x))2n But, 1 + 2 + ... + k = 1 k (k + 1) = r (say)
2
= coefficient of x in (2 + x)2n
2n and, 1 + t + t2 + ... = 1
 x  1- t
= coefficient of x in 22n 1 +  = n.22n
 2  Thus, number of required solutions

The correct option is (C) = coefficient of tn – r in (1 – t)– k
70. General term in (1 + λ)n (1 + µ)n (λ + µ)n is = coefficient of tn – r in (1 + kC1 t + k + 1C2 t2
tp, q, r = (nCpλp) (nCqµq) (nCrλn–rµr) + k + 2C3 t3 + ...)
⇒ tp, q, r = nCp nCq nCr λp+n–r µq+r = k + n – r – 1Cn – r
term contains coefficient of λnµn if = k + n – r – 1Ck – 1 = mCk – 1
p + n − r = n  and q + r = n where, m = k + n – r – 1
⇒ p = r  and q = n – r = k + n – 1 – 1 k (k + 1)

Now, tr, (n–r), r contains coefficient of λnµn 2
⇒ Coefficient of λnµn = nCr nCn–r nCr
9.34  Chapter 9

= 1 [2k + 2n – 2 – k2 – k] = 1 (2n – k2 + k – 2).


1 − q n+1 
2 2 + ... + n + 1Cn + 1 

 1 − q 

The correct option is (A)
1

= [n + 1C1 (1 – q) + n + 1C2 (1 – q2)
n
n 1- q
74. We have, ∑ Cr sin r x cos ( n − r ) x
+ n + 1C3 (1 – q3) + ... + n + 1Cn + 1 (1 – qn + 1)]
r =0

= 1 [(nC0 sin 0x cos nx + nCn sin nx cos 0x)


=
1
[(n + 1C1 + n + 1C2 + ... + n + 1Cn + 1)
2 1- q
+ (nC1 sin x cos (n – 1) x + nCn – 1 sin (n – 1) x ⋅ cos x) – (n + 1C1 ⋅ q + n + 1C2 ⋅ q2 + ... + n + 1Cn + 1 ⋅ qn + 1)]
+ (nC2 sin 2x cos (n – 2) x + nCn – 2 sin (n – 2) x ⋅ cos 2x)
1
+ ... + (nCn sin nx cos 0x + nC0 sin 0x cos nx)]
= [(2n + 1 – n + 1C0) – {(n + 1C0 + n + 1C1 q + n + 1C2 q2
1- q
= 1 [nC0 sin nx + nC1 sin nx + ... + nCn sin nx]
+ ... + n + 1Cn + 1 qn +1 – n + 1C0}]
2 1
n

= [(2n + 1 – 1) – {(1 + q)n + 1 – 1}]
= 1 [nC0 + nC1 + ... + nCn] sin nx = 2 sin nx 1- q
2 2
n
n 1
∴ ∑ Cr sin r x cos ( n − r ) x = 2n – 1 sin nx.
= [2n + 1 – (1 + q)n + 1] = 2n S′n.
r =0 1- q

The correct option is (C)
The correct option is (B)
75. We have, 77. We have, (1 + x)15 = C0 + C1 x + C2 x2 + ... + C15 x15
nCn + n + 1Cn + n + 2Cn + ... + n + kCn 15
= coeff. of xn in (1 + x)n + (1 + x)n + 1 + ... ⇒ (1 + x ) = C0 + C1 + C2 x + C3 x2 + ... + C15 x14
x x
+ (1 + x)n + k. Differentiating both sides w.r.t. x, we get
Now, (1 + x)n + (1 + x)n + 1 + (1 + x)n + 2 + ... 14 15

+ (1 + x)n + k x ⋅ 15 (1 + x ) − 1⋅ (1 + x )
2
x
 (1 + x ) k +1 − 1 C
= (1 + x)n   =– 0 + C2 + 2 C3 x + 3 C4 x2 + ... + 14C15 x13
 x  x2
 
Putting x = 1 on both sides, we get
= 1 (1 + x)n + k + 1 – 1 (1 + x)n 15 ⋅ 214 – 215 = – C0 + C2 + 2 C3 + 3 C4 + ... + 14 C15
x x
Equating the coefficient of xn, we get ⇒ 214 (15 – 2) + 1 = C2 + 2 C3 + 3 C4 + ... + 14 C15
nC0 + n + 1Cn + n + 2Cn + ... + n + kCn ∴ The given series = 214 ⋅ 13 + 1 = 219923.
= n + k + 1Cn + 1 – 0 = n + k + 1Cn + 1 The correct option is (A)
 1  78. (1 + x + x2)n = a0 + a1 x + a2 x2 + a3 x3 + a4 x4 +
There is no term containing x nin (1 + x ) n  .
 x  ... + a2n – 1 x2n – 1 + a2n x2n ...(1)

The correct option is (C)
Replacing x by 1 in (1), we get

76. We have, x

(1 + x + x2)n = a0 x2n + a1 x2n – 1 + a2 x2n – 2 + ...
1 − q n+1  1 − q n+1 
Sn = 1 + q + q2 + ... + qn =   =   ...(1)
+ a2n – 1 x + a2n
 1 − q   1 − q 

Again, replacing x by – x in (1), we get

(1 – x + x2)n = a0 – a1 x + a2 x2 – a3 x3 + ...
2 n
 q + 1  q + 1  q + 1
– a2n – 1 x2n – 1 + a2n x2n ...(2)
and, S′n = 1 + 
+  + ... +  
 2   2   2 

Multiplying (1) and (2), we get
n +1
 q + 1
(1 + x2 + x4)n = (a0 x2n + a1 x2n – 1 + a2 x2n – 2 +
1 − 
 2  2n+1 − ( q + 1) n+1
... + a2n – 1 x + a2n) × (a0 – a1 x + a2 x2 +
= = ...(2)
q +1 (1 − q) ⋅ 2n
... – a2n – 1 x2n – 1 + a2n x2n) ...(3)
1−
2

[Note that (1 – x + x2) (1 + x + x2) = (1 + x2)2 – x2
Now, n + 1C1 + n + 1C2 ⋅ S1 + n + 1C3 ⋅ S2 + ...

= 1 + x2 + x4]
+ n + 1Cn + 1 ⋅ Sn

Finally, replace x by x2 in (1), we get
1 − q  1 − q 2  1 − q3 
= n + 1C1   + n + 1C2   + n + 1C  
(1 + x2 + x4)n = a0 + a1 x2 + ... + an x2n + ... + a2n x4n ...(4)
1 − q   1 − q   1 − q 
3 

Now, equating the coefficients of x2n on the right hand sides
Binomial Theorem  9.35

of (3) and (4), we get ... ... ... ... ... ... ... ... ... ... ... ... ... ... ...
a02 − a12 + a22 − a32 + ... − a22n−1 + a22n = an. ∴ Required sum

The correct option is (B) 1 1 1

= + + +... to m terms
79. Let S = (1 + x) + 2x (1 + x) + 3x (1 + x)
100 999 2 998
+ ... 2 n 2 2 n 23 n
+ 1000 x999 (1 + x) + 1001 ⋅ x1000   1 m 
1 −    1

x 1   2n   1 - mn

This is an A.G.S. of common ratio r =
= = n 2 .
1+ x 2n  1 − 1  2 -1
 x   2 
n

∴   S = x (1 + x)999 + 2x2 (1 + x)998 + ...  


1+ x 
The correct option is (B)
1001
+ 1000 ⋅ x1000 + 1001x
1+ x 81. We have,
On subtracting, we get (1 – x)n = nC0 – nC1 x + nC2 x2 – ... + (– 1)n nCn xn,
  and, (x + 1)n = nC0 xn + nC1 xn – 1 + nC2 xn – 2 + ... + nCn.
1 − x  S = (1 + x)1000 + x (1 + x)999
 1 + x  The given series is the coefficient of xn in the product of
1001 R.H.S. of the above two.
+ x2 (1 + x)998 + ... + x1000 – 1001x
1+ x ∴ Sum of the series = coefficient of xn in (1 – x)n ⋅ (x + 1)n
⇒ S = [(1 + x)1001 + x (1 + x)1000 + x2 (1 + x)999 = coefficient of xn in (1 – x2)n
+ ... + x1000 (1 + x)] – 1001 x1001 = coefficient of xn in
 1001
 [nC0 + nC1 (– x2) + nC2 (– x2)2 + ... + nCn (– x2)n]
1001   x  
(1 + x ) 1 −    Since n is even, let n = 2m. Then,
 1 + x  
=   – 1001 x1001 sum = coefficient of x2m in
x [2mC0 + 2mC1 (– x2) + 2mC2 (– x2)2 + ... + 2mC2m (– x2)2m]
1−
1+ x = 2mCm (– 1)m = nCn/2 (– 1)n/2.
  x 1001  The correct option is (B)
= (1 + x)1002 1 − 
  – 1001 x1001
 
 1 + x   82. We have,

= (1 + x)1002 – x1001 (1 + x) – 1001 x1001 Cr n
 1 n 
tr + 1 = = 1 ⋅  Cr 

= (1 + x)1002 – x1002 – 1002 x1001 ...(1) ( r + 1)( r + 2) r + 2  r + 1 

Now, the coefficient of x50 on the R.H.S. of (1)  1 n+1 

= 1002C50. = 1  ·  
Cr +1 

r + 2  n +1 

The correct option is (C)
1  1 n+1 

= ⋅ ⋅ Cr +1 
80. We have, n + 1  r + 2 
r r
n 1 n 1 1 1 ⋅ n + 2Cr
t1 = 1 ∑ (−1) r ⋅ nCr   = ∑ (−1) r ⋅ nCr  
= ⋅
r =0 
 2 r =0  2  n +1 n + 2
+2

n
Putting r = 0, 1, 2, ..., n and adding, we get the required sum
 1
= 1 −  = 1
1
 2  2n
= [n + 2C2 + n + 2C3 + ... + n + 2Cn + 2]
( n + 1)( n + 2)
 n r n r n
Q ∑ (−1) ⋅ Cr x = (1 − x )  1
 r = 0 
= [2n + 2 – (n + 2C0 + n + 2C1)]
r ( n + 1)( n + 2)
n 3 n  3
t2 = ∑ (−1) r ⋅ nCr  2  = ∑ (−1) r ⋅ nCr   2n + 2 - n - 3
r =0 
2  r =0  4 
= .
n (n + 1)(n + 2)
 3 

= 1 −  = 1 = 1
The correct option is (B)
 4  4n 22n
r r
n 7 n 7 83. Let ( 3 + 1)2n = p + f, where p is the integral part and 0 < f
t3 = ∑ (−1) r ⋅ nCr  3  = ∑ (−1) r ⋅ nCr  
r =0  2  r =0  8  < 1.
 7
n ∴ integer just above ( 3 + 1)2n = p + 1
= 1 −  = 1 = 1

 8 8n 23n
Now, ( 3 + 1)2n = {( 3 + 1)2}n = (4 + 2 3 )n

... ... ... ... ... ... ... ... ... ... ... ... ... ... ...
9.36  Chapter 9

= 2n (2 + 3 )n 85. The given series


Thus, p + f = 2n (2 + 3 )n = mCr ⋅ nC0 + mCr – 1 ⋅ nC1 + ... + mC0 ⋅ nCr,
Also, 0 < 3 – 1 < 1 Now, (1 + x)m = mC0 + mC1 ⋅ x + mC2 x2 + ...
∴ 0 < ( 3 – 1)2n < 1 + mCr xr + ... + mCm xm,
Let f1 = ( 3 – 1)2n = (4 – 2 3 )n = 2n (2 – and, (1 + x)n = nC0 + nC1 x + nC2 x2 + ... + nCr xr + ... + nCn
3 )n,
xn
then 0 < f1 < 1.
The given series is the coefficient of xr in the product of
Now, p + f = 2n (2 + 3 )n
R.H.S. of above two.
= 2n [2n + nC1 2n – 1 3 + nC2 2n – 2 ( 3 )2 ∴ Sum of the series = coefficient of xr in (1 + x)m ⋅
+ ... + nCn ( 3 )n] ...(1) (1 + x)n
f1 = 2n (2 – 3 )n = coefficient of xr in (1 + x)m + n
= 2n [2n – nC1 2n – 1 3 + nC2 2n – 2 ( 3 )2 = m + nCr.
+ ... + (– 1)n ⋅ nCn ( 3 )n] ...(2) The correct option is (B)
(1) + (2) ⇒
86. Let the expansion be that of (1 + x)n.
p + f + f1 = 2n ⋅ 2 [⋅ 2n + nC2 2n – 2 ( 3 )2 + ...]
Let a, b, c, d be the (r + 1)th, (r + 2)th, (r + 3)th and (r + 4)
= an even integer  ...(A) th coefficients.
∴ f + f1 = even number – p = an integer ...(B) ∴ a = nCr , b = nCr + 1, c = nCr + 2, d = nCr + 3.
Also, 0 < f < 1, 0 < f1 < 1 n n
a = Cr Cr
∴ 0 < f + f1 < 2  ...(C)
Now, n
= n +1
a+b C + n
Cr +1 Cr +1
From (B) and (C), f + f1 = 1 ...(D) r

n! ( r + 1)! ( n − r )!
From (A), p + 1 = 2n + 1, an integer.
= × = r +1
Hence, integer just above ( 3 + 1)2n i.e., (p + 1) is divisible r ! ( n − r )! ( n + 1)! n +1
by 2n + 1. b = ( r + 1) + 1 = r + 2 ,

Similarly,
The correct option is (A) b+c n +1 n +1
84. We have, c = ( r + 2) + 1 = r + 3 .
2 r +2 n
Cr r +2 c+d n +1 n +1
tr + 1 = = 2 ⋅
1 nCr
( r + 1)( r + 2) r + 2 r −1 ∴ a c = r + 1 r + 3 = 2r + 4 = 2 ( r + 2)
+ +
r +2 a+b c+d n +1 n +1 n +1 n +1
= 2

1 n + 1Cr
= 2b .
+1
r + 2 n +1
b+c
2 r +2
 1 n+1 

= ⋅  Cr +1  a , b , c are in A.P.


n +1 r + 2  ⇒
a+b b+c c+d
r +2
= 2 ⋅ 1 n+ 2Cr + 2
a + b , b + c , c + d are in H.P.
n +1 n + 2 ∴
a b c
 1 n 1 n+1 
Q C = Cr +1 
The correct option is (C)
 r + 1 r n + 1 

Putting r = 0, 1, 2, ..., n and adding we get, 87. 3400 = (34)100 = (81)100 = (1 + 80)100 = 1 + 100 C1 (80) + C2
100

(80) 2 +....+ 100C100 (80)100



The given expression
= 1 + 8000 + last two digits in each term is 00
1 ∴ Last two digits = 01

= {22 ⋅ n + 2C2 + 23 ⋅ n + 2C3 + ...
( n + 1)( n + 2) The correct option is (C)
+ 2n + 2 ⋅ n + 2Cn + 2}

88. Consider (1 + x)n = C0 + C1x + C2x2 + C3x3 +......
1

= {(1 + 2)n + 2 – n + 2C0 – 2 ⋅ n + 2C1} ....(1)
( n + 1)( n + 2)
Integrating equation (1) w.r. to x, between limits 0 and x, we
3n+ 2 − 2 ( n + 2) − 1 3n + 2 - 2n - 5 get

= = .
( n + 1)( n + 2) (n + 1)(n + 2) x 2 n x
∫0 (C0 + C1 x + C2 x + ......)dx = ∫0 (1 + x ) dx

The correct option is (B)
Binomial Theorem  9.37

x2 x3 (1 + x ) n + 1 − 1  x 
501
⇒ C0x + C1 + C2 + ...... = ...(2) 1 −  
2 3 n +1 1 + x 

Integrating equation (2), taking limits from – 1 to 0, we get = (1 + x) ×
500
= (1 + x)501 – x501
x
1−
 x2 x3  0 (1 + x )
n +1
−1 1+ x
∫−1 C0 x + C1 + C2 + ... dx = ∫−1
0
dx .
Hence, the coefficient of x301 in S = 501C301.
 2 3  n + 1

...(3)
The correct option is (A)
0 0
 C x 2 C x3 C x 4   (1 + x ) n + 2 x  92. We have,
⇒  0 + 1 + 2 + .... =  −
 2 2.3 3.4  −1  ( n + 1)( n + 2 ) n + 1 −1 ( 6 ) 2n = 6n = (1 + 5)n
= C0 + C1.5 + C2.52 + C3.53 +...+ Cn.5n [where Cr = nCr]
C C C 
⇒ –  0 − 1 + 2 − .... 2n
1.2 2.3 3.4  Thus, we have ( 6 )

5
1 1 1
= − =− = + C1 + C2 .5 + C3 .52 + ..... + Cn .5n−1  = 1 + integer
1 
( n + 1)( n + 2) n + 1 n+2
5   5
C0 C1 C 1
∴  − + 2 + ..... = whose fractional part is 1 .

1.2 2.3 3.4 n + 2
5

The correct option is (A)
The correct option is (B)

89. We have, 93. We have,


(1 + x)n = C0 + C1x + C2x2 +...+ Cnxn ...(1) (x + C0) (x –3C1) (x + 5C2)..... up to (n + 1) terms
Differentiating equation (1) w.r.t. x, we get = x n + 1 + [C0 –3C1 + 5C2–.....(n + 1) terms]xn+....
n (1 + x)n –1 = C1 + 2C2x + 3C3x2 + ... + nCnxn–1...(2) Thus, coefficient of xn = C0 – 3C1 + 5C2–....(n + 1) terms
Differentiating equation (2) w. r. t. x, we get
n
= ∑ Cr (−1) r ( 2r + 1)
n(n – 1) (1 + x)n–2 = (1.2)C2 + (2.3) C3x +.... r=0

....+ n(n –1)Cnxn–2...(3)


We have,
n
Putting x = 1 in equation (3), we have
(1 – x2)n = ∑ Cr (−1) r x 2 r
r=0
(1.2) C2 + (2.3)C3 +.......+ (n – 1)n Cn = n(n– 1)2n –2.
n
The correct option is (B)
i.e., x (1 – x2)n = ∑ Cr (−1) r x 2 r + 1 [multiplying by x]
r=0
90. Let n = 2m, then k = 3m n

i.e., (1 – x )n– 2nx2 (1 –x2)n–1 = ∑ Cr (−1) r ( 2r + 1) x 2 r
2
k 3m r=0
r −1
∴ ∑ (−3) C2 r −1 = ∑ (−3) r −1
3n 3m
C2 r −1
r =1 r =1
[differentiating w.r.t.x]

= (–3)0 6mC1 + (–3)1 6m C3 + (–3)2 6mC5+....
Putting x = 1, we have,
= 6mC1– ( 3 ) 2 6 m C3 + ( 3 ) 4 6 m C5−.....
n
∑ Cr (−1) r ( 2r + 1) = 0.
r=0
1  6m 
( 3) ( 3)
3 6m 5 6m

=  3 C1− C3 + C5−.... 
The correct option is (A)
3  

( 5)
100 −r r

=
1
imaginary part of (1 + 3 i )6 m 94. tr +1 = 100
Cr 8
( 6 2 ) . As 2 and 5 are coprime, tr+1
3 will be rational if 100– r is a multiple of 8 and r is a multiple
6m
1 3 
1  of 6. Also, 0 ≤ r ≤ 100

= imaginary part of 26m  + i
3  2 2  ∴ r = 0, 6, 12,....96 ...(1)
1  π π
6m
⇒ 100 – r = 4, 10, 16,.....,100

= imaginary part of 26m cos + i sin 
3  3 3  But 100 – r is to be a multiple of 8, so
100 – r = 0, 8 16, 24,..., 96 ...(2)
1

= × 26 m × 0 = 0 ( Q sin 2m π = 0) The common terms in (1) and (2) are 16, 40, 64 and 88
3 ∴ r = 84, 60, 36, 12 give rational terms.

The correct option is (C) ∴ The number of irrational terms = 101 – 4 = 97
91. The given series The correct option is (B)
 2 500 
1 + x +  x  + .... +  x   95. 1 + 99n =1 + (100 – 1)n = 1+ {nC0100n –nC1.100n–1 +....–
S = (1 + x) 500       nCn} because n is odd
 1 + x 1 + x  1 + x  

9.38  Chapter 9

= 100{nC0.100n–1 –n C1.100n–2+... – n Cn –2.100 + nCn–1} 97. (x + 2n + 1C0) (x + 2n +1C1)(x + 2n +1C2)....(x + 2n + 1Cn)
= 100 × integer whose unit’s place is different from 0. = xn +1 + xn (2n + 1C0 + 2n +1 C1 + 2n +1C2 +....
[ Q nCn–1 = n, has odd digit at unit’s place] +2n + 1Cn) +....
∴ There are two zeros at the end of the sum 99n + 1. ∴ Coeffcient of xn (say)
The correct option is (A) S = 2n + 1C0 + 2n + 1C1 + 2n + 1C2 + ..... + 2n + 1Cn ...(1)
⇒ S = 2n + 1C2n + 1 + 2n + 1C2n + 2n + 1C2n – 1 + ... 2n + 1Cn +1
n 1
96. Let S = ∑ ( Q nCr = nCn – r)...(2)
r=0 ( 2r )!( 2n − 2r )!
On adding (1) and (2), we get
1 n ( 2n)! 1 n 2n 2S = 22n + 1  ∴  S = 22n

= ∑ = ∑ C2 r
( 2n)! r = 0 ( 2r )!( 2n − 2r )! ( 2n)! r = 0 The correct option is (B)
1 2n

=
( 2n)!
( C0 + 2 nC2 + 2nC4 + .... +2n C2n ) 98. 32 = 25 ⇒ (32)32 = (25)32 = 2160 = (3 –1)160
= 3m +1, m ∈ N
Now,
∴ (32)3232 = (32)3m + 1= 25(3m + 1)
 (1+ 1)2n = 2nC0 + 2nC1 + 2nC2 + .....+2nC2n
= 23(5m + 1) 22 = 4.85m + 1
and, (1 – 1)2n = 2nC0 – 2nC1 + 2nC2 – .....+2nC2n
= 4 (7 + 1)5 m + 1 = 4(7n + 1), n ∈ N = 28 n + 4
On adding, we get
∴ When 7 divides (32)3232, remainder = 4
22n = 2(2nC0 + 2nC2 + 2nC4 + .....+2nC2n )
The correct option is (B)
⇒ 22n–1 = 2nC0 + 2nC2 + .....+2nC2n
22 n-1
∴S= .
(2n)!

The correct option is (B)

More than One Option Correct Type


99. SInce T4 is the numerically greatest term in the expansion of or, x ≤ – 2 and x ≥ – 64
10
21
 3x  64
210 1 +  ∴– ≤ x ≤ – 2...(2)
 16  21
T4 T (1) and (2) gives the required range.
∴ ≥ 1 and 4 ≥ 1 The correct option is (B, C)
T3 T5
100. Let Tr, Tr + 1, Tr + 2 be in G.P.
T4 T Tr T

or, ≥ 1 and 5 ≤ 1 ⇒ , 1, r + 2 are in G.P.
T3 T4 Tr +1 Tr +1
Tr +1

Now, = n − r +1 ⋅ x ⇒ r , 1, n − r are in G.P.
Tr r n − r +1 r +1
Taking r = 3 and r = 4 and replacing x by 3 x and n by 10, in
r (n − r)
the above two, we get 16 ⇒ 12 =
( n − r + 1)( r + 1)
11 − 3 3 x 11 − 4 3 x ⇒ n (r + 1) – (r2 – 1) = nr – r2
⋅ ≥ 1 and ⋅ ≤1
3 16 4 16 ⇒n+1=0
or, | x | ≥ 2 and | x | ≤ 64
⇒ n = – 1, which is not possible.
21 Again, if Tr , Tr + 1, Tr + 2 are in H.P.

If x is positive, then | x | = x
1 1 1
⇒ , , are in A.P.
∴ x ≥ 2 and x ≤ 64 Tr Tr +1 Tr + 2
21
∴ 2 ≤ x ≤ 64 ...(1) Tr +1 T
21 ⇒ , 1, r +1 are in A.P.
Tr Tr + 2
If x is negative, then | x | = – x

∴ – x ≥ 2 and – x ≤ 64 ⇒ 2 = n − r +1 + r +1
21 r n−r
Binomial Theorem  9.39

⇒ 2r (n – r) = (n – r)2 + (n – r) + r2 + r ⇒ log [(10 – 3x) ⋅ 3x – 2] = 0 [ Q 20 = 1]


⇒ 2rn – 2r2 = n2 – 2nr + r2 + n – r + r2 + r ⇒ (10 – 3x) ⋅ 3x = 1 ( log 1 = 0)
–2 Q
⇒ n2 + 4r2 – 4nr + n = 0 ⇒ (n – 2r)2 + n = 0 ⇒ 32x – 2 – 10 ⋅ 3x – 2 + 1 = 0
This is not possible as both (n – 2r)2 and n are positive. ⇒ 32x – 10 ⋅ 3x + 9 = 0 ⇒ (3x – 1) (3x – 9) = 0
The correct option is (A, B) ∴ 3x – 1 = 0 which gives x = 0
or, 3x = 9 = 32 which gives x = 2.
101. Given, mC1, mC2 and mC3 are the first, third and fifth terms
of an A.P., which will also be in A.P. of common difference Hence, x = 0 or 2.
2d. The correct option is (B, C)
∴ 2 ⋅ mC2 = mC1 + mC3 7. The given series can be written as
⇒ m (m – 1) = m + m ( m − 1)( m − 2) n
S = ∑ n Cr .2 n−2 rCm (−1) r
1⋅ 2 ⋅ 3 r=0
⇒ 6m – 6 = 6 + m2 – 3m + 2 [ Q m ≠ 0] n

= ∑ n Cr (−1) r × coefficient of xm in (1 + x)2n–2r
⇒ m2 – 9m + 14 = 0 r=0
⇒ (m – 2) (m – 7) = 0 n

= coefficient of xm in ∑ n Cr (−1) r [(1 + x ) 2 ]n−r
Since 6th term is 21, m = 2 is ruled out, r=0

∴ we have m = 7 and
= coefficient of xm in [(1 + x)2 – 1]n
7 −5
= coefficient of xm in (x2 + 2x)n
21 = 7C5  2log(10−3x ) 
5
×  5 2( x −2 ) log 3  (given)
= coefficient of xm in xn (x + 2)n
   

= coefficient of xm–n in (x + 2)n
= 7 ⋅ 6 2log(10−3x ) ⋅ 2( x −2 ) log 3
1⋅ 2  n  2 n − m
= nCm–n2n–(m–n) = 
2 if m ≥ n and 0 if m < n.
m − n
log(10 −3 x
) + log 3x−2
⇒ 21 = 21⋅ 2

The correct option is (A, B)
∴ 2log [(10-3x ) 3x-2 = 1

Passage Based Questions


T r +1 Cr 315-r (-5 x ) r
15
15 − r + 1 −5 x 
101. We have Tr = 10Cr–1xr–1 and Tr + 1 = 10Crxr. ⇒ = =  
Cr -1 316-r (-5 x ) r -1  3 
15
10
Tr r
T r +1 C xr
∴ = 10 r T r +1 16 − r  5 1  1
Tr Cr -1xr-1 ⇒ = ×  – ×  , when x =
Tr r  3 5  5
10
Cr 10! (10 − r + 1)!( r − 1)!

= x = × x T
= 16 − r × 1 , numerically
10 r +1
Cr-1 (10 − r )! r ! 10! ⇒
Tr r 3
T
= 11- r . x
[Neglecting minus sign]
r +1

Tr r T r +1
T 11 − r  2
Now, > 1 (numerically)

r +1
=   × [ Q x = 2/3] Tr
Tr  r  3
Now, ⇒ 16 − r × 1 > 1 ⇒ 16 > 4 r ⇒ r < 4
T r +1 r 3
11 − r  2 2
> 1 ⇒   × >1 ⇒ 22 > 5r ⇒ r < 4 Since 4 is an integer, therefore 4th and 5th terms are numer-
Tr  r  3 5 ically greatest terms.
∴ (4 + 1)th, i.e., 5th term is the greatest term. The correct option is (A, B)
Putting r = 4 in Tr + 1, we get 103. Let (r + 1) th term be the greatest term. Then
4
 2  1 
r
 1 
r −1
T5 = 10C5 x4 ⇒ T5 = 10C5   [ Q x = 2/3] Tr + 1 = 20
Cr   and Tr = 3 20
Cr −1  
 3  3  3   3 
4
 2 
⇒ T5 = 210  
 3  T r +1 20 − r + 1 1 

Now, =  
The correct option is (A) Tr r  3
102. We have, Tr + 1 = 15Cr 315– r (–5x)r ∴ Tr + 1 ≥ Tr ⇒ 20 – r + 1 ≥ 3r
and, Tr = 15Cr – 1 315– r + 1 (–5x)r– 1
9.40  Chapter 9

21 i.e., I + f + g = 2k, k ∈ I+ ...(1)


⇒ 21 ≥ r ( 3 + 1) ⇒ r ≤
3 +1 ∴ f + g = 2k –1 = an integer ...(2)
⇒ r ≤ 7.686 ⇒ r = 7. Since 0 < f, g < 1, therefore we have
 1 
7 0 < f + g < 2 ....(3)
3 C7  
20

Hence, the greatest term is T8 = Thus, using (2) and (3), we have
 3 
f + g = 1

= 25840 .
9 [since the only integral value in (0,2) is 1]
The correct option is (A) i.e., g =1– f  ....(4)
104. Since T4 is the numerically greatest term Therefore, we have,
R(1 –f) = Rg = (5 + 2 6 )n.(5 –2 6 )n
T4 T T T
∴ ³ 1 and 4 ³ 1 ⇒ 4 ³ 1 and 5 £ 1 n
T3 T5 T3 T4 = 52 − 2 6  = 1.
( )
2

 
2
3 
The correct option is (A)

Now, T3 = 10
C2 ( 210−2 )  x ,
 8 
106. Let (3 + 5 )2n = I + f where I and f are the integral and the
 3x 
3 fractional parts of (3 + 5 )2n respectively.
T4 = 10
C3 ( 210−3 )   Let (3 + 5 )2n = g, where g is a fraction.
 8 
4
 3x 
Since 0 < 3 – 5 < 1, therefore

and, T5 = 10
C4 ( 210−4 )  
 8  0 < (3 – 5 ) n < 1 for every positive integer n.
2

 3x 
3
We have,
10
C3 × 27 ×  
T4  8  I + f + g = (3 + 5 )2n + (3 – 5 )2n
∴ = 2
= x
T3  3 x  2
= (14 + 6 5 )n + (14 – 6 5 )n
10
C2 × 28 ×  
 8 
=2n [(7 + 3 5 )n + (7 – 3 5 )n ]
4
 3x 
= 2n. 2[C0. 7n + C2 (7)n–2 (3 5 )2 +....]
C4 × 26 ×  
10

T5  8  21x i.e., I + f + g = (2n + 1) k, k ∈ I+ ...(1)



and, = 3
=
T4  3x  64 i.e.,  f + g = (2n + 1) k– I = an integer  ...(2)
10
C3 × 27 ×  
 8  Since 0 < f, g < 1, therefore we have
0<f+g<2  ....(3)
T4 T
∴ ³ 1 and 5 £ 1 ⇒ x ≥ 1 and 21x £ 1 Thus, using (2) and (3), we have
T3 T4 2 64
f + g = 1 ...(4)
x | ≥ 2 and | x | ≤ 64
⇒ |  [since the only integral value in (0,2) is1]
21 Putting (4) in equation (1), we have
 64 64 
⇒ x ∈ (– ∞. – 2] ∪ [2, ∞) and x ∈ − ,  I + 1 = (2n + 1)k
 21 21
 64   64  ∴  [(3 + 5 ) n ] + 1 is divisible by 2n + 1.
2

⇒ x ∈ − , −2 ∪ 2,  The correct option is (C)


 21   21
The correct option is (C) 107. Let g = (7 – 4 3 )n . Then, 0 < g < 1 as 0 < 7 – 4 3 <1
Now, I + f + g = (7 + 4 3 )n + (7 – 4 3 )n
105. (a).  Let (5 + 2 6 )n = I + f where I and f are the integral
and the fractional parts of (5 + 2 6 )n respectively. = 2 (nC0 7n + nC2 7n –2 (4 3 )2 +...)
Let (5 – 2 6 )n = g where g is a fraction. = an integer
Since 0 < 5 – 2 6 < 1, therefore 0 < (5 – 2 6 )n <1 ⇒ f + g = I  ⇒  g = I – f
for every positive integer n. Thus, (I + f) (I – f ) = (I + f) g = (7 + 4 3 )n (7 – 4 3 )n = 1.
We have, The correct option is (B)
I + f + g = (5 + 2 6 )n + (5–2 6 )n
= 2[C05n +C2(2 6 )2.5n –2+....]
Binomial Theorem  9.41

Match the Column Type


108. I We have, 7103 = 7 (49)51 = 7 (50 – 1)51
where r lies between 0 to 1028. It will be so if r is a
= 7 (5051 – 51C1 5050 + 51C2 5049 – ... – 1)
multiple of 8 between 0 to 1028,
= 7 (5051 – 51C1 5050 + 51C2 5049 – ...) – 7 + 18 – 18
i.e., 0, 8, 16, 32,..., 1024
= 7 (5051 – 51C1 5050 + 51C2 5049 – ...) – 25 + 18
Now, Tn = 1024, a = 0, d = 8
= k + 18 (say)  Q   k is divisible by 25,
∴ 1024 = 0 + (n – 1)8  ⇒  1024 = 8(n – 1)
∴  remainder is 18.
The correct option is (C) ⇒ n – 1 = 1024 = 128 ⇒ n = 129
109. II (r + 1)th term in the given expansion is given by 8
r

The correct option is (C)
10 -r
tr + 1 = 10Cr 5 , where r = 0, 1, 2, ..., 10
2 2 3
x +1 x −1
For rational terms 113. II 2/3

x − x + 1 x − x1 / 2
1/ 3
r = a multiple of 5 = 0, 5, 10 ...(1)
10 – r = a multiple of 2 = 0, 2, 4, 6, 8, 10 ...(2) ( x1/ 3 )3 + 13 x x −1

= −
From (1) and (2), possible values of r are : 0 and 10 x 2 / 3 − x1/ 3 + 1 x1/ 2 ( x1/ 2 −1)
∴  sum of rational terms
= t1 + t11 = 10C0 ( 2 )10 (31/5)0 + 10C10 ( 2 )0 (31/5)10
=
= 25 + 32 = 32 + 9 = 41.

= x1/3 + 1 – 1 – x–1/2 = x1/3 – x–1/2
The correct option is (D) 10
 x +1 x − 1 
110. III We have, 24n = (24)n = (16)n = (1 + 15)n ⇒   2 / 3 −  = (x1/3 – x–1/2)10
∴ 24n = 1 + nC1 ⋅ 15 + nC2 152 + nC3 153 + ...  x − x + 1 x − x1/ 2 
1/ 3

⇒ 24n – 1 – 15n = 152 [nC2 + nC3 ⋅ 15 + ...] Tr + 1 for (x1/3 – x–1/2)10 is 10Cr(x1/3)10–r (– 1)r (x –1/2)r
= 225 k, where k is an integer. For term independent of x,
Hence, 24n – 15n – 1 is divisible by 225. 10 - r r  = 0 ⇒  20 – 2r – 3r = 0  ⇒  r  = 4

-
The correct option is (B) 3 2
111. IV We have, Hence, required coefficient = 10C4(– 1)4 = 210.
599 = 53 ⋅ 596 = (125) (625)24 The correct option is (A)
= [13 × 9 + 8] (1 + 48 × 13)24 114. III Coefficient of x n1 y n2 z n3 in the expansion of
= (13 × 9 + 8) [1 + 24C1 × (48 × 13) (x + y + z)10 is
+ 24C2 (48 × 13)2 +...+ (48 × 13)24] ( n1 + n2 + n3 )! 10!
= = 2520
= 8 + terms containing powers of 13. n1 ! n2 ! n3 ! 2! 3! 5!
The correct option is (B)
Hence, remainder = 8.
115. IV 17 = 2 (mod 5)
The correct option is (A)
(17)5 = (2)5 (mod 5) = 2 (mod 5)
112. I Tr + 1 = 1028Cr (51/2)1028–r. (71/8)r
⇒ (175)6 = (2)6 (mod 5)  ⇒ (17)30 = 4 (mod 5)
= 1028Cr 5514. 5–r/2. 7r/8
The correct option is (D)
Tr + 1 will be integral if both r and r are integers
2 8

Assertion-Reason Type
n
Ck C
116. We know that = n k = n − k +1
When k = 1, p = n and when k = n, p = 1.
Ck -1 Ck -1 k
+n n
C
2
  n − k + 1
2 ∴ Series = ∑ ( n − p + 1) p 2 = ∑ ( np 2 − p3 + p 2 )
∴  ∑ k  k
n n

3
 = ∑ k 3   p =1 p =1
k =1  C k −1 
 k =1  k  n n

= ∑ ( n + 1) p − ∑ p 2 3
n

= ∑ k ( n − k + 1) 2 p =1 p =1

k =1
= (n + 1) [12 + 22 + 32 + ... + n2]
Put n – k + 1 = p ⇒ k = n – p + 1.

– [13 + 23 + 33 + ... + n3]
9.42  Chapter 9

n n
2 2 ∴ n
Cn / 2 xn/2 > C n x2
-1

= ( n +1)n ( n + 1) ( 2n + 1) − n ( n + 1) 2
-1

6 4 n n
and, n C n x 2 > C n
n
+1
2

n( n + 1) 2n + 1 n  n( n + 1) 2 ( n + 2) +1
x2
=  −  = 2 2

2 
 3 
2 12 n 
n n −  + 1 + 1
The correct option is (A) n − +1  2 
⇒ 2 x > 1 and x<1
117. Given: Pn = nC0 nC1 nC2... nCn n n
+1
Pn+1 n +1
C0 n+1C1 n+1C2 ...n−1Cn 2 2

Now, =
Pn n
C0 nC1 nC2 ...n Cn n n
+1
⇒x> 2
and x < 2
Pn+1  n+1C   n+1C 
⇒  =
n +1
C0  n 1   n 2  n
+1
n
Pn  C0   C1  2 2
 n+1C  n and x < n + 2 ∴ n < x < n + 2 .
... n n  n+1Cn+1 ⇒x>
 Cn  n+2 n n+2 n
n +1 n The correct option is (A)

Since,  n + 1Cr + 1 = Cr
r +1 121. Given series is
Pn+1  n + 1  n + 1  n + 1 S = 1 + 2 ⋅ 1 + 2 ⋅ 5 ⋅ 1 + 2 ⋅ 5 ⋅ 8 ⋅ 1 + ...∞
⇒  = 1   ... 1 3 2 3 6 2 2 3 6 9 23
Pn  1   2   n 
and we know that
Pn+1 ( n + 1) n
∴  = (1 + x)n = 1 + nx + n ( n − 1) x 2 + n ( n − 1)( n − 2) x 3 + ...∞

Pn n! 2! 3!
The correct option is (A)
Comparing these two, we get
118. The expansion is a G.P. with (n + 1) terms of the form nx = 2 × 1 ...(1)
a n+1 − b n+1 3 2
an + an–1b + an–2b2 + ... + bn = n ( n − 1) x2 = 2 5 1 ...(2)
a−b and, × ×
n +1 n +1
2 ⋅1 3 6 22
=  ( 2 x + 3) − ( 2 x − 5) ,
8 Now, divide (2) by square of (1), we get
n ( n − 1) 2 2 5 1
x ´ ´

where a = 2x + 3 and b = 2x – 5
⇒ 2 ⋅ 1 = 3 6 4
n2 x 2 2 1 2 1
∴ Coefficient of xn = 1 [(n + 1). 2n(3) – (n + 1). 2n(– 5)] = ´ ´ ´
8 3 2 3 2
(n + 1). 2n -1
⇒ n = 5 ⇒ -1 = 5
n
The correct option is (A) 2n 4 n 2
119. If n is odd, then numerically greatest coefficient in the expan- ⇒ 5n = 2n – 2
n n ⇒ 3n = – 2 ⇒ n = – 2
sion of (1 – x)n is Cn-1 or Cn+1 . 3
2 2 putting value of n in (1), we get

Therefore, in (1 – x)21, the numerically greatest coefficient is
21
C10 or 21C11. So, the numerically greatest term – 2 x= 2 × 1

3 3 2

=  21C11x11 or 21C10x10 and

| 21C10x10 | > | 21C9.x9 | ⇒x=– 2 × 1 × 3 =– 1
3 2 2 2
21! 21! ∴ Sum of given series
⇒ > x and
10! 11! 9! 12! 2/3 −2/3
 1 1
21! 21! = 1 −  =  
= (2)2/3 = (4)1/3.

x> ( Q x > 0)  2   2 
11! 10! 9! 12!
The correct option is (D)
5 6
⇒ x < 6 and x > 5   ⇒  x ∈  ,  122 We have, C1 + C3 + C5 +...
5 6  6 5 
2 4 6
The correct option is (A) n n ( n − 1)( n − 2) n ( n − 1)( n − 2)( n − 3)
120. Since n is even, therefore the greatest coefficient is n Cn / 2 . = + + + ...
2 3!⋅ 4 5!⋅ 6
∴ The greatest term = n Cn / 2 xn/2
Binomial Theorem  9.43

1  ( n + 1)( n) ( n + 1)( n)( n − 1)( n − 2) 1  12 12  1



=  +
=  ∑ Ck −12 C0  = ( 212 − 1)
n + 1  2! 4! 12  k = 0  12
( n + 1)( n)( n − 1)( n − 2)( n − 3)  The correct option is (A)
+ + ...
6!  124. Let d be the common difference of the A.P., then we have,

∑ n Ck Sk = ∑ n Ck . k [2a1 + ( k − 1)d ]
n n

= 1 [n + 1C + n + 1C + ... + ...]
2 4
n +1 k =0 k =0 2
1 [2n – 1] = 2n − 1  d  n n d n

= = a1 −  ∑ k . Ck + ∑ k 2 .n Ck
n +1 n +1  2  k = 0 2k =0
[ Q nC0 + nC2 + nC4 + ... = 2n – 1,  d d
∴ nC2 + nC4 + ... = 2n – 1 – 1]. = a1 −  n2n−1 + [n2n−1 + n( n − 1)2n−2 ]

 2 2
The correct option is (C)
= a1n2n –1+ dn(n–1)2n–3
= a1n2n –1+ n(an – a1)2n–3 [ Q an – a1 = (n –1)d]

123. Using nCk = n . n–1Ck–1, for 0 ≤ k ≤ 11,
k
= n2n –3[4a1 + an – a1] = n2n–3 (2a1 + a1 + an)
Ck 11 12
Ck + 1  n( a1 + an ) 

= = 2n–2  na1 +
 = 2n–2(na1 + Sn).
k +1 12  2 
∴ the given expression = 1 ∑ 12 Ck + 1
11

The correct option is (A)
12 k = 0

Previous Year’s Questions


125. Coefficient of xn in the expansion of (1 + x)(1 − x)
121.
∴ (1 + 2 x + 3 x 2 + ...)-3/ 2 = [(1- x )-2 ]-3/ 2 n
= (1 + x)(nC0 − nC1x + …….. + (−1)n –1 nCn – 1 xn – 1
= (1- x )3 + (−1)n nCn xn) is (−1)n nCn + (−1)n –1 nCn – 1 = (−1)n
Now, coefficient of x5 in (1 + 2x + 3x2 + ...)-3/2 = coef-
(1− n).
ficient of x5 in (1 − x)3 = 0
The correct option is (B)

The correct option is (D)
126. Given that m
Cr -1, mCr , mCr +1 are in A.P.
122.

(1 + x + x 2 + x 3 + ...) 2 = [(1- x )-1 ]2 ⇒
-2
2 mCr = mCr -1 + mCr +1
= (1- x )
m
Cr -1 mCr +1

Now, coefficient of x in (1 + x + x2 + ...)2
n
Þ2= m
+ m
= coefficient of xn in (1 − x)3 = 0 Cr Cr
= n+ 2-1C2-1 = n+1C1 r m-r
= +
= n +1 m - r +1 r +1


The correct option is (D) ⇒ m2 − m (4r + 1) + 4r2 − 2 = 0.
256 256-r 1/8 r
The correct option is (C)
123. General term = Cr ( 3) [(5) ]
For integral terms, r should be 8k And then k 127. The expression 6
assumes values from 0 to 32.Hence, (B) is the cor- C4 + å 56-rC3
50

r -1
rect answer.

The correct option is (B) = C4 + ëêé C3 + C3 + 53C3 + 52C3 + 51C3 + 50C3 ûúù
50 55 54

124. Coefficient of Middle term in (1 + αx)4 = t3 = 4C2⋅ = ( 50C4 + 50C3 ) + 51C3 + 52C3 + 53C3 + 54C3 + 55C3

α Coefficient of Middle term in (1 − αx) = t4 = C3
2 6 6
= ( 51C4 + 51C3 ) + 52C3 + 54C3 + 55C3
(− α)3
Given that 4C2α2 = −6C3.α3
= 55C + 55C = 56C .
⇒ −6 = 20 α 4 3 4

-3
The correct option is (D)
⇒ α =
10

The correct option is (C)
9.44  Chapter 9

128. Tr+1 in the expansion 132 Since the sum of 5th and 6th terms is zero, we have
æ ö
C4 an‑4(−b)4 + nC5 an−5 (− b)5 = 0 Þ çç a ÷÷ = n - 5 + 1
11 r
é 2 1ù 11-r æ 1 ö
n
ê ax + ú = 11Cr (ax 2 ) çç ÷÷÷ çè b ÷ø 5
êë bx úû çè bx ø
The correct option is (D)

= 11Cr (a)11−r (b)−r (x)22−2r−r 133. We have
⇒ 22 − 3r = 7 ⇒ r = 5 (1 + x)20 = 20C0 + 20C1x + … + 20C10x10 + … + 20C20x20
∴ coefficient of x7 = 11C5 (a)6 (b)−5 ……(1)
Put x = 1,

Also, Tr+1 in the expansion 0 = 20C0 − 20C1 + … − 20C9 + 20C10 − 20C11 + … + 20C20
11 r 0 = 2 (20C0 − 20C1 + … − 20C9) + 20C10
é 1 ù æ1ö
ê ax - 2 ú = 11Cr ( ax 2 )11-r çç ÷÷÷ ⇒ 20C0 − 20C1 + … + 20C10 = 1 20C10.
êë bx úû çè bx ÷ø 2
The correct option is (B)
= 11Cr a 11 − r (−1)r × (b) −r (x) −2r (x)11−r
134. 1- q n ³ 9
Now 11 − 3r = −7 ⇒ 3r = 18 ⇒ r = 6 10
∴ coefficient of x-7 = 11C6 a5 × 1 × (b)−6 æ 3ö
n
1
⇒ 11C5 (a)6 (b) −5 = 11C6 a5 (b) −6 Þ çç ÷÷÷ £
çè 4 ø 10
⇒ ab = 1
Þ n ³ - log 3 10
The correct option is (D) 4
é 3 3æ3 ö æ1 ö æ1 ö ù
2
1
1 29. (1- x )1/ 2 ê1 + x + çç -1÷÷ x 2 -1- 3çç x ÷÷ - 3(2) çç x ÷÷ ú Þn³
ê 2 2 èç 2 ø÷ èç 2 ÷ø çè 2 ÷ø ú log10 - log10 3
4
ëê ûú

The correct option is (A)
æ ö ù
2
æ 3 ö÷ 2 æ ö
çç -1÷ x -1- 3çç 1 x ÷÷ - 3(2) çç 1 x ÷÷ ú 135. We can write 82n − (62)2n + 1
èç 2 ø÷ çè 2 ÷ø çè 2 ÷ø ú
úû = (1 + 63)n-(63 − 1)2n + 1
é 3 ù 3 = (1 + 63)n + (1 − 63)2n + 1
= (1- x )1/ 2 ê- x 2 ú = - x 2 (because the higher powers
êë 8 úû 8 = (1 + nc163 + nc2(63)2 + .... + (63)n) + (1 − (2n + 1)c1 63

of x are neglected) + (2n + 1)c2 (63)2 + …+ (−1) (63)(2n + 1))
The correct option is (C) = 2 + 63(nc1 + nc2(63) + .... + (63)n−1 − (2n + 1) c1 + (2n +
130. We have
1)
c2(63) + .... (63)(2n))
(1 − ax)−1 (1 − bx)−1 = (1 + ax + a2x2 + ......) (1 + bx ∴ Reminder is 2
+ b2x2 + ....)
The correct option is (B)
∴ Coefficient of xn = bn + abn−1 + a2bn−2 + .... + an−1b 6
136. éê1- x - x 2 (1- x )ùú = (1- x )6 (1- x 2 )6
b n+1 - a n+1 ë û
+ an = é 6 C0 -6 C1 x +6 C2 ´2 -6 C3 x 3 +6 C4 x 4 ù
b-a = ê ú
ê 6 5 6 6 ú
b n+1 - a n+1 ëê - C 5 x + C 6 x ûú
\ an =
b-a é 6 6 2 6 4 6
´ C - C1 x + C2 x - C3 x + ....ûú 6 ù
ëê 0
The correct option is (D)
Coefficient of
131. We have x 7 =6 C16C3 -6 C36C2 +6 C56C1
(1 − y)m(1 + y)n = [l − m C1y + m C2y2 −....][1 + n C1y
+ n C2y2 + ...] = 120 - 300 + 36 = -144

ïìï m( m -1) n( n -1) ïü
= 1 + ( n - m) + í
+ - mnïý y 2 + ....
The correct option is (B)
ïîï 2 2 ïþï n
137. ( 3 + 1) 2 n - ( 3 -1) 2 n = éê

2 2
∴ a1 = n − m = 10 and a = m + n - m - n - 2mn = 10 ëê
( )
3 +1 ú
ûú
2
2 n
é 2ù
So, n − m = 10 and (m − n)2 - (m + n) = 20 ⇒ m + n = 80 -ê ( )
3 -1 ú = (4 + 2 3) n - (4 - 2 3) n
∴ m = 35, n = 45 ëê ûú
The correct option is (D) é ù
( )
n
= 2n ê 2 + 3 - (2 - 3) n ú
ëê ûú
Binomial Theorem  9.45

ïìïé n C 2n +n C 2n-1 3 +n C 2n-2 3 + ....ù ïüï æ17´8 - 60 ö÷ 2a(-19)


ëê 0 ûú ï 4 çç ÷÷ + =0
= 2 ïí
1 2
n
ý çè 3 ø 6
ïï- é n C 2n-n C 2n-1 3 +n C 2n-2 3 - ....ù ïï
ï
îï ë ê 0 1 2 ú ï
û þï 4 ´ 76 ´6
a=
3´ 2´19
= 2n+1 éê n C1 2n-1 3 +n C3 2n-33 3 + ....ùú = 2n+1 3


ë û Þ a = 16
(some integer)
2´16 ´16 272

which is irrational Þb= - 80 =

The correct option is (A) 3 3
The correct option is (D)
a
138. f ¢( x ) = + 2 b x + 1 2 b x 2 + x + a = 0 has roots –1 140. Set X contains elements of the form
x n n
and 2 4 - 3n -1 = (1 + 3) - 3n -1

The correct option is (C) = 3n +n Cn-1 3n-1.....n C2 32

139. 1(1- 2 x )18 + ax(1- 2 x )18 + bx 2 (1- 2 x )18 = 9(3n-2 +n Cn-1 3n-1... +n C2 )

Coefficient of
Set X has natural numbers which are multiples of 9
(not all)
x 3 : (-2)318 C3 + a(-2) 218 C2 + b(-2)18 C1 = 0
Set Y has all multiples of 9 X È Y = Y

4 ´(17´16) 17
The correct option is (D)
- 2a × + b = 0 ….(i)
(3´ 2) 2
× (-2 x1/ 2 )
50-r r
141. t r +1 =50 Cr × (1)

Coefficient of
x 4 : (-2)418 C4 + a(-2)318 C3 + b(-2) 218 C2 = 0 =50 Cr × 2r × x r / 2 (-1) r
Þ r = an even integer.
16
(4 ´ 20) - 2a × + b = 0 ….(ii) Þ Sumr = of coefficient
3
( )
å 50 C × 22 r = 1 (1 + 2)50 + (1- 2)50 = 1 (350 + 1)
25

From equation (i) and (ii), we get 2r
r =0 2 2
æ17´8 ö æ16 17 ö
4 çç - 20÷÷÷ + 2a çç - ÷÷÷ = 0
The correct option is (D)

èç 3 ø èç 3 2ø
Sequence and Series 10.1

CHaPtER

10 Sequence and Series

Chapter Highlights
Sequence, Series, Progressions, Arithmetic progression (A.P.), Sum of n terms of an A.P., Properties of A.P.,
Arithmetic mean (A.M.), Geometric progression (G.P.), Geometric mean (G.M.), Some special sequences,
Arithmetico-geometric progression (A.G.P.), Method for finding sum of a.G. Series

Sequence Note that sequence (1) is a finite sequence whereas others


are infinite sequences.
A succession of numbers a1, a2, …, an formed according to
some definite rule is called a sequence.
A sequence is a function whose domain is the set N SerieS
of natural numbers and range a subset of real numbers or
complex numbers. A series is obtained by adding or subtracting the terms of
A sequence whose range is a subset of real numbers a sequence.
is called a real sequence. Since we shall be dealing with A series is finite or infinite according as the number
real sequences only, we shall use the term sequence to of terms in the corresponding sequence is finite or infinite.
denote a real sequence.
ProgreSSionS
notation
If the terms of a sequence follow certain pattern, then the
The different terms of a sequence are usually denoted by sequence is called a progression. Following are the three
a1, a2, a3, … or by t1, t2, t3, … The subscript (always a special types of progressions:
natural number) denotes the position of the term in the
sequence. The term at the nth place of a sequence, i.e., tn is 1. Arithmetic Progression (A.P.)
called the general term of the sequence. 2. Geometric Progression (G.P.)
3. Harmonic Progression (H.P.)

notE
arithmetic ProgreSSion (a.P.)
A sequence is said to be finite or infinite according as it
has finite or infinite number of terms. A sequence whose terms increase or decrease by a fixed
number is called an arithmetic progression. The fixed num-
ber is called the common difference of the A.P.
Illustrations
In an A.P., the first term is usually denoted by a, the
1. 1, 4, 7, 10, … 19. In this sequence each term is obtained common difference by d and the nth term by tn. Obviously
by adding 3 to the previous term.
2. 2, – 4, 8, – 16, … In this sequence each term is obtained d = tn – tn – 1
by multiplying the preceding term by – 2. Thus, an A.P. can be written as
3. 2, 3, 5, 7, 11, 13, … This is the sequence of prime
numbers. a, a + d, a + 2d, …, a + (n – 1) d, …

Objective_Maths_JEE Main 2017_Ch 10.indd 1 01/01/2008 04:28:38


10.2 Chapter 10

For example, (A) 21 (B) 28


(C) 14 (D) None of these
1. 1, 3, 5, 7, 9, …
Since, 2nd term – Ist term = 3rd term – 2nd term Solution: (C)
= 4th term – 3rd term By inspection, first common term to both the series is
= … = 2, 23. Second common term = 51.
the sequence 1, 3, 5, 7, … are in A.P. whose first term Third = 79 and so on. These numbers form an A.P.
is 1 and common difference is 2. 23, 51, 79, …
2. 5, 3, 1, – 1, – 3, – 5, – 7, … are in A.P. whose first term
is 5 and common difference is – 2. Since, T15 = 23 + 14 (28) = 23 + 392 = 415 > 407

the nth term of an arithmetic Progression and T14 = 23 + 13 (28) = 387 < 407

If a is the first term and d is the common difference of an \ number of common terms = 14.
A.P., then its nth term tn is given by 2. The number of numbers lying between 100 and 500
tn = a + (n – 1) d that are divisible by 7 but not by 21 is
(A) 57 (B) 19
(C) 38 (D) None of these
trick(S) for Problem Solving
Solution: (C)
To find whether the series is an A.P. The numbers between 100 and 500 that are divisible
Step I: Obtain an (the nth term of the sequence). by 7 are 105, 112, 119, 126, 133, 140, 147, …, 483,
Step II: Replace n by n – 1 in an to get an – 1. 490, 497.
Step III: Calculate an – an – 1.
Let such numbers be n.
Then, 497 = 105 + (n – 1) × 7; or n = 57.
If an – an–1 is independent of n, the given sequence is an A.P.
otherwise it is not an A.P.
The numbers between 100 and 500 that are divisi-
ble by 21 are 105, 126, 147, …, 483.
\ tn = An + B represents the nth term of an A.P. with common
difference A.
Let such numbers be m.
Then 483 = 105 + (m – 1) × 21; or m = 19.
Hence, the required number = n – m = 57 – 19 = 38.

i M P o R ta n t P o i n t S 3. In the series 3, 7, 11, 15, … and 2, 5, 8, … each con-


tinued to 100 terms, the number of terms that are
 If an A.P. has n terms, then the nth term is called the last identical is
term of A.P. and it is denoted by l. That is
(A) 21 (B) 27
l = a + (n – 1) d (C) 25 (D) None of these
 Three numbers a, b, c are in A.P. if and only if
b – a = c – b, i.e., if and only if a + c = 2b. Solution: (C)
 If a is the first term and d the common difference of an Let the nth term of the first series = the mth term of the
A.P. having m terms, then nth term from the end is (m – n second series.
+ 1)th term from the beginning. Thus,
\ 3 + (n – 1) × 4 = 2 + (m – 1) × 3,
nth term from the end = a + (m – n) d.
Any three numbers in an A.P. can be taken as n m

or 4n = 3m or = = k (say)
a – d, a, a + d. Any four numbers in an A.P. can be taken 3 4
as a – 3d, a – d, a + d, a + 3d. Similarly 5 numbers in A.P. \ n = 3k and m = 4k
can be taken as a – 2d, a – d, a, a + d, a + 2d.
As each series is continued to 100 terms,

n = 3k ≤ 100 and m = 4k ≤ 100


Solved examPleS
\ Possible values of k are 1, 2, 3, …, 25 and corre-
1. The number of terms common to two A.P.s 3, 7, 11, …, sponding to each value of k we get one identical term.
407 and 2, 9, 16, …, 709 is Hence there are 25 identical terms.

Objective_Maths_JEE Main 2017_Ch 10.indd 2 01/01/2008 04:28:42


Sequence and Series  10.3

a Solution: (A)
4. If 51 + x + 51 – x, and 25x + 25–x are three consecutive
2 Let d be the common difference of the A.P., then
terms of an A.P., then the values of a are given by
4 = abc = (b – d)b (b + d) = b(b2 – d2)
(A) a ≥ 12 (B)  a > 12
(C) a < 12 (D)  a ≤ 12 ⇒ b3 = 4 + bd2 ≥ 4 (Q b > 0, d2 ≥ 0)

Solution: (A) ⇒ b ≥ 22/3


a
Since 51 + x + 51 – x, , 25x + 25–x are in A.P., we have
2 Thus, the minimum possible value of b is 22/3.
a
2 = 51 + x + 51 – x + 25x + 25–x 7. There are four numbers of which the first three are in
2 1
G.P. whose common ratio is and the last three are
Now put 5x = t so that t > 0, we then have 2
in A.P. If the last number is two less than the first, then
5 1 ⎛ 1⎞ ⎛ 1⎞ the four members are
 a = 5t + + t 2 + = ⎜t2 + 2 ⎟ + 5 ⎜t + ⎟
t t 2 ⎝ t ⎠ ⎝ t⎠ 1 1 1
(A)  3, 1, , – (B)  2, 1, , 0
2 ⎡⎛ 2 ⎤ 3 3 2
1⎞
a = ⎛⎜ t − ⎞⎟ + 2 + 5
1
or ⎢ ⎜ t − ⎟ + 2⎥ 1 1
⎝ t⎠ ⎢⎣⎝ t⎠ ⎥⎦ (C)  4, 1, , – (D)  None of these
4 2
2 2
= ⎛⎜ t − 1⎞⎟ + 5 ⎛ t − 1 ⎞ + 12 ≥ 12. Solution: (B)
⎝ t⎠ ⎜⎝ ⎟ a a
t⎠ The numbers can be taken as a, , , a – 2.
2 4
Thus, values of a are given by the inequality a ≥ 12. a a
By question, , , a – 2 are in A.P.
2 4
5. If 1, logy x, logz y, –15logx z are in A.P., then
(A) x = y–3 (B)  y = z–2 \ 2·
a a
= + a – 2; or a = 2.
3
(C) x = z (D)  None of these 4 2
Solution: (C) 1
Hence, the numbers are 2, 1, , 0.
Let d be the common difference of the A.P. 2
Then, logy x = 1 + d ⇒ x = y1 + d Sum of n terms of an A.P.
logz y = 1 + 2d ⇒ y = z1 + 2d The sum of n terms of an A.P. with first term ‘a’ and com-
mon difference ‘d’ is given by
and –15logx z = 1 + 3d ⇒ z = x–(1 + 3d)/15 n
Sn = [2a + (n – 1) d]
2
\ x = y1 + d = z(1 + 2d) (1 + d)
Trick(s) for Problem Solving
= x–(1 + d)(1 + 2d)(1 + 3d)/15
 If Sn is the sum of n terms of an A.P. whose first term is ‘a’
⇒ (1 + d)(1 + 2d)(1 + 3d) = –15 and last term is l, then
n
⇒ 6d3 + 11d2 + 6d + 16 = 0 Sn = (a + l)
2
 If common difference d, number of terms n and the last
⇒ (d + 2)(6d2 – d + 8) = 0 ⇒ d = –2 term l, are given then
n
\ x = y1 + d = y–1, y = z1 + 2d = z–3 and x = (z–3)–1 = z3. Sn = [2l – (n – 1) d]
2
 tn = Sn – Sn – 1.
6. If three positive real numbers a, b, c are in A.P. such
The sum of an A.P. consisting of odd number of terms = n
that abc = 4, then the minimum possible value of b is 

(middle term), where n is number of terms.


(A) 22/3 (B)  21/3
5/3
(C) 2 (D)  None of these

Objective_Maths_JEE Main 2017_Ch 10.indd 3 01/01/2008 04:28:44


10.4  Chapter 10

Properties of A.P. n

1. If a1, a2, a3, …, an are in A.P., then


\ Sn = ∑ tn
n =1
(a)  a1 + k, a2 + k, …, an + k are also in A.P.
(b)  a1 – k, a2 – k, …, an – k are also in A.P.
n
4 n2 + 1 1 n
⎛ 1 1 ⎞
(c)  ka1, ka2, …, kan are also in A.P.
= ∑ 16
+
32
∑ ⎜⎝ 2n − 1 − 2n + 1⎟⎠
n =1 n =1
a a a
(d)  1 , 2 , …, n , k ≠ 0 are also in A.P. 1 n( n + 1) (2 n + 1) 1 1
k k k = + n+
4 6 16 32
2. If a1, a2, a3, … and b1, b2, b3, … are two A.P.s, then
(a)  a1 + b1, a2 + b2, a3 + b3, … are also in A.P. ⎛ 1 1 1 1 1 ⎞
(b)  a1 – b1, a2 – b2, a3 – b3, … are also in A.P. ⎜⎝1 − 3 + 3 − 5 + ... + 2n − 1 − 2n + 1⎟⎠
(c)  a1b1, a2b2, a3b3, … are also in A.P. 
a a a n 1 ⎛ 1 ⎞
= ( 4 n2 + 6 n + 5) + 1−
(d)  1 , 2 , 3 , … may not be in A.P.
b1 b2 b3 48 32 ⎜⎝ 2n + 1⎟⎠
3. If a1, a2, a3, …, an are in A.P., then n( 4 n2 + 6 n + 5) n
= +
(a)  a1 + an = a2 + an – 1 = a3 + an – 2 = … 48 16( 2n + 1)
ar − k + ar + k \ f (n) = n(4n2 + 6n + 5)
(b)  ar =
, 0 ≤ k ≤ n – r.
2
In other words, in an A.P., the sum of two terms equi- 9. The maximum sum of the series
distant from the beginning and end is a constant and is 1 2
20 + 19 + 18 + 18 + … is
equal to the sum of first term and last term 3 3
4. If nth term of a sequence is a linear expression is n (A) 310 (B) 290
then the sequence is an A.P. (C)  320 (D)  None of these
5. If the sum of first n terms of a sequence is a quadratic
Solution: (A)
expression in n, then the sequence is an A.P.
6. If a1, a2, a3, a4, . . . , an are in A.P., then terms taken The given series is arithmetic whose first term = 20,
at regular intervals from this A.P. are also in A.P. e.g., common difference = – .
a1, a4, a7, a10, . . . also form an A.P. As the common difference is negative, the terms
will become negative after some stage. So the sum is
maximum if only positive terms are added.
Solved Examples Now
14 2 4 34 n4 ⎛ 2⎞
tn = 20 + (n – 1) ⎜ − ⎟ ≥ 0 if 60 – 2 (n – 1) ≥ 0;
8. If + + + ... + = ⎝ 3⎠
13 35 5.7 (2 n − 1) (2 n + 1)
or 62 ≥ 2n or 31 ≥ n
1 n
f ( n) + , then f (n) is equal to \ The first 31 terms are non-negative.
48 16( 2n + 1)
(A) n(4n2 + 3n + 2) (B)  n(4n2 + 6n + 5) 31 ⎡ ⎛ 2⎞ ⎤
\ Maximum sum = S31 = ⎢ 2 × 20 + (31 − 1) ⎜⎝ − 3 ⎟⎠ ⎥
(C) n(4n2 + 5n + 6) (D)  None of these 2 ⎣ ⎦
Solution: (B)
31
= (40 – 20) = 310
n4 2
We have, tn =
(2 n − 1) (2 n + 1) 10. The sum to n terms of the sequence
log a, log ar, log ar2, … is
n2 1 1
= + + n
4 16 16( 4 n2 − 1) (A)  log a2 rn – 1 (B)  n log a2 rn – 1
2
4n 2 + 1 1 ⎛ 1 1 ⎞ 3n
= + − (C)  log a2 rn – 1 (D)  None of these
16 32 ⎜⎝ 2 n − 1 2 n + 1⎟⎠ 2

Objective_Maths_JEE Main 2017_Ch 10.indd 4 01/01/2008 04:28:48


Sequence and Series  10.5

Solution: (A) \ First 24 terms are positive.


The given sequence can be expressed as \ Sum of the positive terms
24 ⎛ −3 ⎞
log a, (log a + log r), (log a + 2 log r) … = S24 = 2 × 10 + 23 × ⎟
2 ⎜⎝ 7⎠
which is clearly an A.P. whose first term is log a and
common difference is log r. ⎛ 69 ⎞ 852
= 12 ⎜ 20 − ⎟ =
The nth term = log a + (n – 1) log r ⎝ 7 ⎠ 7
n 13. The minimum number of terms from the beginning of
Since sum to n terms = (a1 + an) 2 1
2 the series 20 + 22 + 25 + …, so that the sum may
n 3 3
\ Sn = [log a + log a + (n – 1) log r] exceed 1568, is
2 (A) 25 (B) 27
n n (C) 28 (D) 29
 = [2 log a + (n – 1) log r] = log a2 rn – 1
2 2
Solution: (D)
11. Let Sn denotes the sum of n terms of an A.P. whose first 2 8
It is in A.P. for which a = 20, d = 2 =
term is a. If the common difference d = Sn – k Sn – 1 + 3 3
Sn – 2 then k = Now, Sn > 1568
(A) 1 (B) 2 n ⎡ 8⎤
⇒ ⎢ 40 + ( n − 1) ⎥ > 1568
(C)  3 (D)  None of these 2 ⎣ 3⎦
Solution: (B) n 112 + 8n
⇒ × > 1568
2 3
We have, an = Sn – Sn – 1(1)
6
⇒ n2 + 14n > × 1568 = 1176
and an – 1 = Sn – 1 – Sn – 2(2) 8
2
⇒ n + 14n – 1176 > 0,
\ d = an – an – 1
or (n + 42) (n – 28) > 0
= (Sn – Sn – 1) – (Sn – 1 – Sn – 2) As n is positive, n – 28 > 0 i.e., n > 28
\ Minimum value of n = 29.
 [From (1) and (2)]
14. If the first, second and last terms of an A.P. are a, b and
= Sn – 2Sn – 1 + Sn – 2.
2a respectively, then its sum is
12. The sum of positive terms of the series ab ab
(A)  (B) 
4 1 2(b − a) b−a
10 + 9 + 9 + … is
7 7 3ab
(C)  (D)  None of these
352 437 2 ( b − a)
(A)  (B) 
7 7 Solution: (C)
852 Here a1 = a and a2 = b
(C)  (D)  None of these \ Common difference d = a2 – a1 = b – a
7
Let n be the number of terms in the series
Solution: (C)
3 \ an = 2a = a + (n – 1) d
Here, a = 10, d = – .
7 or (n – 1) d = a or (n – 1) (b – a) = a
tn = 10 +(n – 1) ⎛⎜ − ⎞⎟
3 a
Then, \ n–1=
⎝ 7⎠ b−a
a a+b−a b
tn is positive if 10 + (n – 1) ⎛⎜ − ⎞⎟ ≥ 0;
3 or n= +1= =
⎝ 7⎠ b−a b−a b−a
1 n b 3ab
or 70 – 3 (n – 1) ≥ 0 or 73 ≥ 3n; or 24 ≥ n \ Sum = (a1 + an) = (a + 2a) = .
3 2 2(b − a) 2 ( b − a)

Objective_Maths_JEE Main 2017_Ch 10.indd 5 01/01/2008 04:28:51


10.6  Chapter 10

15. If S1 is the sum of an arithmetic series of ‘n’ odd num- 17. If a is the first term, d the common difference and Sk
ber of terms and S2, the sum of the terms of the series S
the sum to k terms of an A.P., then for k x to be inde-
S1 Sx
in odd places, then = pendent of x
S2 (A) a = 2d (B)  a=d
2n n
(A)  (B)  (C) 2a = d (D)  None of these
n +1 n +1
Solution: (C)
n +1 n +1
(C)  (D)  kx
2n n
Skx [ 2a + (kx − 1) d ]
Solution: (A) We have, = 2
Sx x
Let the odd number of terms of an arithmetic series be 2
[ 2a + ( x − 1) d ]
a, a + d, a + 2d, a + 3d, a + 4d, ….., a + (n – 1) d k [( 2a − d ) + kxd ]
=
Then, ( 2a − d ) + xd
n Skx
S1 = {2a + (n – 1) d} For to be independent of x, 2a – d = 0 or 2a = d.
2 Sx
n +1
S2 = a + (a + 2d) + (a + 4d) + … to terms
2 18. The sum of n terms of m A.P.s are S1, S2, S3, …, Sm. If
n +1 ⎡ ⎛ n +1 ⎞ ⎤ the first term and common difference are 1, 2, 3, …, m
= ⎢ 2a + ⎜⎝ 2 − 1⎟⎠ × 2d ⎥ respectively, then S1 + S2 + S3 + … + Sm =
2×2 ⎣ ⎦
1
n +1 (A)  mn (m + 1) (n + 1)
= (2a + (n – 1) d) 4
4
1
(B)  mn (m + 1) (n + 1)
S1 2n 2
\ =
S2 n +1 (C) mn (m + 1) (n + 1)
16. A club consists of members whose ages are in A.P., the (D)  None of these
common difference being 3 months. If the youngest Solution: (A)
member of the club is just 7 years old and the sum
of the ages of all the members is 250 years, then the We have, S1 = (n/2) [2 . 1 + (n – 1)  . 1]
number of members in the club are
(A) 15 (B) 25 S2 = (n/2) [2 . 2 + (n – 1)  . 2]
(C) 20 (D) 30
Sm = (n/2) [2 . m + (n – 1)  . m]
Solution: (B)
n \ S1 + S2 + … + Sm
Sn =  [2a + (n – 1) d]
2 n ( n −1)
1 = n (1 + 2 + 3 … + m) + × (1 + 2 + … + m)
Here a = 1st term = 7 years, d = 3 months = year, 2
4
m ( m + 1) ⎛ n2 − n ⎞
Sn = 250 = ⎜n+ 2 ⎟
2 ⎝ ⎠
n ⎡ 1⎤
\ 250 = ⎢ 2 × 7 + ( n − 1) × 4 ⎥
2 ⎣ ⎦ m ( m + 1) n ( n + 1) 1
= ⋅ = mn (m + 1) (n + 1).
2 2 4
n ⎛ n + 55 ⎞
⇒ 250 = ⎜ ⎟ 19. If the first, second and the last terms of an A.P. are
2⎝ 4 ⎠
a, b, c respectively, then the sum is
⇒ 2000 = n2 + 55n
( a + b) ( a + c − 2b)
⇒ n2 + 55n – 2000 = 0 (A) 
2 ( b − a)
⇒ (n – 25) (n + 80) = 0 ⇒ n = 25. ( b + c ) ( a + b − 2c )
(B) 
\ Number of members in the club = 25. 2 ( b − a)

Objective_Maths_JEE Main 2017_Ch 10.indd 6 01/01/2008 04:28:55


Sequence and Series  10.7

Solution: (C)
( a + c ) ( b + c − 2a)
(C)  The first two digit number which when divided by 4
2 ( b − a)
leaves remainder 1 is 4 . 3 + 1 = 13 and last is 4  .  24
(D)  None of these + 1 = 97.
Solution: (C) Thus, we have to find the sum

We have, first term = a, \ T1 = a 13 + 17 + 21 + … + 97


which is an A.P.
Second term = b, \ T2 = b
\ 97 = 13 + (n – 1) . 4 ⇒ n = 22.
Then common difference d = T2 – T1 = b – a
Also, last term = c. n
c −a+d and Sn = [a + l] = 11 × [13 + 97]
⇒ c = a + (n – 1) d ⇒ n = . 2
d
= 11 × 110 = 1210.
(b + c − 2a)
⇒ n= (Q d = b – a)
(b − a) 22. If there are (2n + 1) terms in A.P., then the ratio of the
sum of odd terms and the sum of even terms is
n (b + c − 2a) (a + c )
\ Sum of n terms Sn = (a + l) = (A) n : (n + 1) (B)  (n + 1) : n
2 2 (b − a) (C) (n – 1) : n (D)  None of these
20. Four different integers form an increasing A.P. If one Solution: (B)
of these numbers is equal to the sum of the squares of Let the A.P. containing (2n + 1) terms be
the other three numbers, then the numbers are
a, a + d, a + 2d, a + 3d, a + 4d, a + 5d, …, a + 2nd.
(A)  – 2, – 1, 0, 1 (B)  0, 1, 2, 3
(C)  – 1, 0, 1, 2 (D)  None of these The sum of odd terms of this A.P.
Solution: (C) = a + (a + 2d) + (a + 4d) + … to (n + 1) terms
Let the numbers be a – d, a, a + d, a + 2d n +1
where a, d ∈ Z and d > 0 = [2a + (n + 1 – 1) × 2d] = (n + 1) (a + nd)
2
Given: (a – d)2 + a2 + (a + d)2 = a + 2d The sum of even terms of this A.P.
⇒ 2d2 – 2d + 3a2 – a = 0 = (a + d) + (a + 3d) + (a + 5d) + … to n terms
1⎡ n
\ d= 1 ± (1 + 2a − 6 a 2 ) ⎤ = [2 (a + d) + (n – 1) × 2d] = n (a + nd)
2 ⎣⎢ ⎦⎥ 2
n +1
Since d is positive integer, Hence, the required ratio = .
n
\ 1 + 2a – 6a2 > 0
Arithmetic Mean (A.M.)
⎛1− 7 ⎞ ⎛1+ 7 ⎞
⇒ ⎜ 6 ⎟ <a< ⎜ 6 ⎟ Single Arithmetic Mean
⎝ ⎠ ⎝ ⎠
Since a is an integer, A number ‘A’ is said to be the single A.M. between two
given numbers a and b provided a, A, b are in A.P.
\ a = 0,
For example, since 2, 4, 6 are in A.P., therefore, 4 is
1 the single A.M. between 2 and 6.
then d = [1 ± 1] = 1 or 0. Since d > 0,
2
\ d = 1. n-Arithmetic Means
Hence, the numbers are – 1, 0, 1, 2. The numbers A1, A2, …, An are said to be the n arithmetic
means between two given numbers a and b provided
21. The sum of all two digit numbers which when divided
by 4, yield unity as remainder, is a, A1, A2, …, An , b are in A.P.
(A) 1100 (B) 1200 For example, since 2, 4, 6, 8, 10, 12 are in A.P., therefore,
(C)  1210 (D)  None of these 4, 6, 8, 10 are the four arithmetic means between 2 and 12.

Objective_Maths_JEE Main 2017_Ch 10.indd 7 01/01/2008 04:28:57


10.8 Chapter 10

inserting Single a.m. between two given Solved examPleS


numbers
1
Let a and b be two given numbers and A be the A.M. 23. Between two numbers whose sum is 2 , an even
6
between them. Then, a, A, b are in A.P. Thus, number of arithmetic means are inserted. If the sum
a+b of these means exceeds their number by unity, then the
A – a = b – A or 2A = a + b, or A = .
2 number of means are
(A) 12 (B) 10
inserting n-arithmetic means between two (C) 8 (D) None of these
given numbers
Solution: (A)
Let A1, A2, …, An be the n arithmetic means between two
Let 2n arithmetic means be A1, A2, A3, …, A2n between
given numbers a and b. Then a, A1, A2, …, An, b are in A.P.
a and b.
Now, b = (n + 2)th term of A.P.
a+b
Then, A1 + A2 + A3 + … + A2n = × 2n
= a + (n + 2 – 1) d = a + (n + 1) d 2
13
b−a
or d= , where d is common difference of A.P. 13n
n +1 = 6 × 2n =
2 6
⎛ b − a⎞
and A1 = a + d = a + ⎜ ,
⎝ n + 1 ⎟⎠ Given: A1 + A2 + A3 + … + A2n = 2n + 1;
13n
⎛ b − a⎞ \ 2n + 1 = ; or 12n + 6 = 13n;
A2 = a + 2d = a + 2 ⎜ , 6
⎝ n + 1 ⎟⎠
\ n = 6.
M M M
\ The number of means = 2n = 2 × 6 = 12.
⎛ b − a⎞
An = a + nd = a + n ⎜
⎝ n + 1 ⎟⎠ 24. If a, b, c are in A.P. and p is the A.M. between a and b
and q is the A.M. between b and c, then
(A) a is the A.M. between p and q
(B) b is the A.M. between p and q
REMEMBER (C) c is the A.M. between p and q
(D) None of these
The sum of n arithmetic means between two given numbers
is n times the single A.M. between them, i.e. if a and b are Solution: (B)
two given numbers and A1, A2, …, An are n arithmetic means
between them, then Q a, b, c are in A.P.,
a + b⎞
A1 + A2 + … + An = n ⎛⎜
⎝ 2 ⎟⎠ \ 2b = a + c (1)
Q p is the A.M. between a and b
a+b
trickS(S) for Problem Solving \ p= (2)
2
 Sum to n terms of the series of the form Q q is the A.M. between b and c
1 1 1 b+c
+ + ... + is \ q= (3)
t1t2 ... t k t2t3 ... t k +1 t n t n +1 ... t n + k −1 2
Adding Eq. (2) and (3)
Sn =
1 ⎛ 1 1 ⎞
− a+b b+c a + c + 2b
(k − 1)(t2 − t1) ⎜⎝ t1t2 ... t k −1 t n +1 t n +2 ... t n + k −1 ⎟⎠ p+q= + =
2 2 2
 Sum to n terms of the series of the form 2b + 2b
(t1t2 … tk) + (t2t3 … tk+1) + … + (tn tn+1 … tn+k–1) is = = 2b [Using (1)]
2
Sn =
1
(tn tn+1 … tn+k – t0 t1 t2 … tk) p+q
\ 2b = p + q or b =
(k + 1)(t2 − t1) 2
Hence, b is the A.M. between p and q.

Objective_Maths_JEE Main 2017_Ch 10.indd 8 01/01/2008 04:29:00


Sequence and Series 10.9

geometric ProgreSSion (g.P.) Solution: (C)


Let A be the first term and R, the common ratio of G.P.
A sequence (finite or infinite) of non-zero numbers in which Then a = ax = ARx – 1, b = ay = ARy – 1
every term, except the first one, bears a constant ratio with
its preceding term, is called a geometric progression. and c = az = ARz – 1
The constant ratio, also called the common ratio of
the G.P., is usually denoted by r. \ (y – z) log a + (z – x) log b + (x – y) log c
For example, in the sequence, 1, 2, 4, 8, …,
2 4 8 = log ay – z + log bz – x + log cx – y
= = = … = 2, which is a constant.
1 2 4
= log (ay – z . bz – x . cx –y)
Thus, the sequence is a G.P. whose first term is 1 and the
common ratio is 2. = log [(ARx – 1) y – z × (ARy – 1)z – x . (ARz – 1)x – y]
nth term of a g.P. = log [Ay – z + z – x + x – y . R(x – 1) (y – z) + (y – 1) (z – x) + (z – 1)
(x – y)
If a is the first term and r is the common ratio of a G.P., then ]
its nth term tn is given by = log (A0 × R0) = log 1 = 0.
tn = arn – 1
26. If p, q, r are in A.P. and x, y, z are in G.P., then
xq – r . yr – p . zp – q =
i M P o R ta n t P o i n t S (A) 1 (B) 2
(C) – 1 (D) None of these
 If a is the first term and r is the common ratio of a G.P.,
then the G.P. can be written as Solution: (A)
a, ar, ar2, …, arn – 1, … (a ≠ 0) Let d be the common difference of A.P. and R (≠ 0), the
 If a is the first term and r is the common ratio of a finite common ratio of G.P., then
G.P. consisting of m terms, then the nth term from the end
is given by arm – n. q = p + d, r = p + 2d
 The nth term from the end of a G.P. with last term l and and y = xR, z = xR2
l
common ratio r is ( n −1) .
r so that q – r = – d, r – p = 2d, p – q = – d
a
 Three numbers in G.P. can be taken as , a, ar;
r
Four numbers in G.P. can be taken as \ xq – r . yr – p . zp – q = x– d . (xR)2d . (xR2)– d
a a = (x– d . x2d . x– d) (R2d × R– 2d )
, , ar, ar3;
r3 r
Five numbers in G.P. can be taken as = (x– d + 2d – d ) . (R2d – 2d )
a a
, , a, ar, ar2 = x0 . R0 = 1 × 1 = 1.
r2 r
 Three numbers a, b, c are in G.P. if and only if 27. If, in a G.P., the (p + q)th term is a and the (p – q)th
term is b, then pth term is
b c
= i.e. if and only if b2 = ac. (A) (ab)1/3 (B) (ab)1/2
a b 1/4
(C) (ab) (D) None of these
Solution: (B)
Solved examPleS Let the G.P. be x, xy, xy2, …
Then tp + q = xy p + q – 1 = a (1)
25. If a, b, c are respectively the xth, yth and zth terms of a p–q–1
G.P., then and tp – q = xy =b (2)
( y – z) log a + (z – x) log b + (x – y) log c = Dividing Eqs (1) by (2),
(A) 1 (B) – 1 a
y2q = ;
(C) 0 (D) None of these b

Objective_Maths_JEE Main 2017_Ch 10.indd 9 01/01/2008 04:29:02


10.10  Chapter 10

1 t
⎛ a ⎞ 2q or tn = 2 .  n+1 ,(Q common ratio = r)
\ y= ⎜ ⎟ 1− r
⎝ b⎠
t n +1 1− r
\ = ; or = r
From Eq. (1), tn 2
p + q −1
⎛ b⎞ 2q 1
x = a ⎜ ⎟ or 1 – r = 2r ; \ r = .
⎝ a⎠ 3

p + q −1
30. The three numbers a, b, c between 2 and 18 are such
p −1
⎛ b⎞ 2q that their sum is 25; the numbers 2, a, b are consec-
\ tp = xy p – 1 = a  . ⎜ ⎟ ⎛ a⎞ 2q
utive terms of an A.P. and the numbers b, c, 18 are
⎝ a⎠ ⎜⎝ b ⎟⎠
consecutive terms of a G.P. The three numbers are
p + q −1 p −1 p + q −1 p −1 (A)  3, 8, 14 (B)  2, 9, 14
= 1− + −
a 2q 2q
×b 2q 2q (C)  5, 8, 12 (D)  None of these

1 1 Solution: (C)
= a2b2 = ab
We have, a + b + c = 25 (1)
28. In a set of four numbers the first three are in G.P. and
the last three are in A.P. with a common difference 6. Q 2, a, b are in A.P., \ 2a = 2 + b (2)
If the first number is same as the fourth, the four num-
bers are Q b, c 18 are in G.P., \ c2 = 18b(3)
(A)  3, 9, 15, 21 (B)  1, 7, 13, 19 From Eq. (1) and (2),
(C)  8, – 4, 2, 8 (D)  None of these
3b + 2c = 48; or 3b = 48 – 2c
Solution: (C)
\ From Eq. (3),
Let the last three numbers in A.P. be b, b + 6, b + 12
and the first number be a. c2 = 6 (48 – 2c) = 288 – 12c
Hence the four numbers are a, b, b + 6, b + 12
or c2 + 12c – 288 = 0;
Given, a = b + 12 (1)
or c2 + 24c – 12c – 288 = 0
and a, b, b + 6 are in G.P. i.e., b2 = a (b + 6)
or (c + 24) (c – 12) = 0;
or b2 = (b + 12) (b + 6) (Q a = b + 12)

or 18b = – 72 \ b = –4, \ c = 12, as c ≠ – 24.


\ From Eq. (3), b = 8 and from (2), a = 5.
From Eq. (1),
31. Let an be the nth term of the G.P. of positive numbers.
a = – 4 + 12 = 8. 100 100
Hence, the four numbers are 8, –4, 2 and 8. Let ∑ a2n = a and ∑ a2n−1 = b, such that a ≠ b, then
n =1 n =1
29. If the first term of an infinite G.P. is 1 and each term is the common ratio is
twice the sum of the succeeding terms, then the com- a b
mon ratio is (A)  (B) 
b a
1 2
(A)  (B)  a b
3 3 (C)  (D) 
3 b a
(C)  (D)  None of these
4 Solution: (A)
Solution: (A) Let a be the first term and r, the common ratio of the
Let the G.P. be 1, r, r2, r3, … given G.P. Then
100
Given: tn = 2 (tn + 1 + tn + 2 + tn + 3 + … to ∞) a= ∑ a2n ⇒ a = a2 + a4 + … + a200
n =1

Objective_Maths_JEE Main 2017_Ch 10.indd 10 01/01/2008 04:29:04


Sequence and Series  10.11

⇒ a = ar + ar3 + … + ar199 Sum of an Infinite G.P. when | r | < 1


2 4 198
⇒ a = ar (1 + r + r + … + r )(1) The sum of an infinite G.P. with first term a and common
100 ratio r is
and b= ∑ a2n−1 ⇒ b = a1 + a3 + … + a199
S∞ =
a
; when | r | < 1, i.e., – 1 < r < 1.
n =1
1− r
⇒ b = a + ar2 + … + ar198
⇒ b = a (1 + r2 + … + r198)(2) caution
From Eqs (1) and (2), we get
If r ≥ 1, then S∞ does not exist.
a
= r.
b

⎛ 1 1 1⎞ Solved Examples
32. If a, b, c are in G.P., then a2 b2 c2 ⎜ 3 + 3 + 3 ⎟ =
⎝a b c ⎠
33. The sum Sn to n terms of the series
(A) a + b + c (B)  ab + ac + bc 1 3 7 15
(C) a3 + b3 + c3 (D)  None of these + + + + ... is equal to
2 4 8 16
Solution: (C) (A) 2n – n – 1 (B)  1 – 2– n
–n
Since a, b, c are in G.P. (C) 2 + n – 1 (D)  2n – 1
b c Solution: (C)
\ = ⇒ b2 = ac (1)
a b We have,
⎛ 1⎞ ⎛ 1⎞ ⎛ 1⎞ ⎛ 1⎞
⎛ 1 1 1⎞ Sn = ⎜1 − ⎟ + ⎜ 1 − ⎟ + ⎜ 1 − 3 ⎟ + ... + ⎜1 − n ⎟
Now, a2 b2 c2 ⎜ + + ⎟ ⎝ 2⎠ ⎝ 4 ⎠ ⎝ 2 ⎠ ⎝ 2 ⎠
⎝ a3 b 3 c 3 ⎠

⎛1 1 1⎞
2 2 2 2 2 2 = n−⎜ + + ... + n ⎟
= b c +a c +a b ⎝2 2 2
2 ⎠
a b c
1 ⎡ ⎛ 1⎞ ⎤
n
⎢1 − ⎜ ⎟ ⎥
ac ⋅ c 2 (b 2 )2 a2 ⋅ ac = 2 ⎢ ⎝ 2 ⎠ ⎥⎦ = n – 1 + 2–n
=
a
+
b
+
c
 [Using (1)] n− ⎣
1
1−
= a3 + b3 + c3. 2
35. The minimum number of terms of the series
1 + 3 + 9 + 27 + …
Sum of n terms of a G.P. so that the sum may exceed 1000, is
The sum of first n terms of a G.P. with first term a and com- (A) 7 (B) 5
mon ratio r(≠ 1) is given by, (C)  3 (D)  None of these

a ( r n − 1) a(1 − r n ) Solution: (A)


Sn = =
r −1 1− r Let, the sum of n terms exceeds 1000.
1(1 − 3n ) 3n − 1
Then > 1000; or > 1000
1− 3 2
Trick(s) for Problem Solving
or 3n > 2001; but 36 = 729 and 37 = 2187;
 When r = 1,
Sn = a + a + a + … upto n terms = na. \ n > 6; but n is a positive integer;
If l is the last term of the G.P., then
\ n = 7, 8, 9, …

lr − a
Sn = r≠1 \ The minimum number of terms = 7.
r −1

Objective_Maths_JEE Main 2017_Ch 10.indd 11 01/01/2008 04:29:07


10.12  Chapter 10

Solution: (A)
x x2 x4
36. The sum of series + + + ... to n n
1− x2 1− x4 1 − x8 n + 2 ⎛ 1⎞ 2( n + 1) − n ⎛ 1 ⎞
We have, tn = ⋅⎜ ⎟ = ⎜ ⎟
infinite terms, if | x | < 1 is n ( n + 1) ⎝ 2 ⎠ n( n + 1) ⎝ 2 ⎠
1 x n −1 n
(A)  (B)  1 ⎛ 1⎞ 1 ⎛ 1⎞
1− x 1− x = ⎜ ⎟ − ⎜ ⎟
n ⎝ 2⎠ n + 1 ⎝ 2⎠
1 x n
(C)  (D) 
1+ x 1+ x \ Sn = ∑ tn
n =1

Solution: (B)
⎡1 ⎛ 1 ⎞ 1 ⎛ 1 ⎞ 1 ⎤ ⎡ 1 ⎛ 1 ⎞ 1 1 ⎛ 1 ⎞ 2 ⎤
2n − 1
1+ x 2n − 1
−1 = ⎢ ⎜ ⎟ − ⎜ ⎟ ⎥+⎢ ⎜ ⎟ − ⎜ ⎟ ⎥
⎢⎣1 ⎝ 2 ⎠ 2 ⎝ 2 ⎠ ⎥⎦ ⎢⎣ 2 ⎝ 2 ⎠ 3 ⎝ 2 ⎠ ⎥⎦
x
We have, tn = =
2n − 1 2n − 1 n −1

1− x (1 + x ) (1 − x 2 )
⎡1 ⎛ 1⎞ n −1 1 ⎛ 1⎞ ⎤
n

1 1  + ... + ⎢ ⎜ ⎟ − ⎜ ⎟ ⎥
= n −1
− n ⎢⎣ n ⎝ 2 ⎠ n + 1 ⎝ 2 ⎠ ⎥⎦
1− x2 1− x2
1
n = 1−
Therefore, Sn = ∑ tn ( n + 1)2 n
n =1
⎡⎛ 1 1 ⎞ ⎛ 1 1 ⎞ 38. If (1 – y) (1 + 2x + 4x2 + 8x3 + 16x4 + 32x5) = 1 – y6,
 = ⎢ − + − (y ≠ 1), then a value of y/x is
⎜⎝ 1 − x 1 − x 2 ⎟⎠ ⎜⎝ 1 − x 2 1 − x 4 ⎟⎠

1
1 ⎞⎤ (A)  (B)  2
⎛ 1 2
+ ... + − n ⎟⎥
⎜ 2n − 1
⎝1 − x 1 − x 2 ⎠ ⎥⎦ 1
 (C)  (D)  4
4
1 1
= − Solution: (B)
1 − x 1 − x2
n

We can rewrite the given expression as


\ The sum to infinite terms 6
(1 – y) [1 − ( 2 x ) ] = 1 – y6,
1 1 − 2x
= lim S n = −1
n→∞ 1− x one of the possible values of y is clearly 2x. Therefore,
⎛ n
⎞ one of the possible values of y/x is 2.
⎜⎝Q nlim x 2 = 0, as | x | < 1⎟
→∞ ⎠
 ⎛1 1 1 ⎞
log 5 ⎜ + + + ...∞⎟
⎝ 4 8 16 ⎠
x 39. The value of (0.2) is
=
1− x (A) 1 (B) 2
1
(C)  (D)  4
37. The sum to n terms of the series 2
2 3
3 ⋅ 1 + 4 ⎛⎜ 1 ⎞⎟ + 5 ⎛⎜ 1 ⎞⎟ + ... is Solution: (D)
1.2 2 2.3 ⎝ 2 ⎠ 3.4 ⎝ 2 ⎠ 1
1 1 1 4 1
1 1 We have, + + + ...∞ = = = 2–1.
(A) 1 − 1−
(B)  4 8 16 1 2
1−
( n + 1)2 n ( n + 1)2n −1 2
1 ⎛1 1 1 ⎞
log 5 ⎜ + + + ...∞⎟ log 5 2−1
(C) 1 − (D)  None of these ⎝ 4 8 16 ⎠ ⎛ 1⎞
( n − 1)2 n − 1 \ (0.2) = ⎜ ⎟
⎝ 5⎠

Objective_Maths_JEE Main 2017_Ch 10.indd 12 01/01/2008 04:29:12


Sequence and Series  10.13

−1 ⎛ A − 1⎞ ⎛ B − 1⎞
log 2 \ a log r = log ⎜ ⎟ and b log r = log ⎜
= −1 1/ 2 5
(5 ) ⎝ A ⎠ ⎝ B ⎟⎠
2 log5 2 log5 ( 2 )2 ⎛ A − 1⎞
= 5 = 5 = 22 = 4 log ⎜
a ⎝ A ⎟⎠
\ =
40. i – 2 – 3i + 4 … to 100 terms = b ⎛ B − 1⎞
log ⎜
(A)  50 (1 – i) (B) 25i ⎝ B ⎟⎠
(C)  25 (1 + i) (D)  100 (1 – i) ⎛ A − 1⎞
= log B − 1 ⎜
Solution: (A) ⎝ A ⎟⎠
B
2 3 100
S = i + 2i + 3i +, …, + 100i 43. Sum of the series : (1 + x) + (1 + x + x2) +
(1 + x + x2 + x3) + … upto n terms is

S . i = i2 + 2i3 +, …, + 99i100 + 100i101
1 ⎡ x 2 (1 − x n ) ⎤
2 3
\ S(1 – i) = (i + i + i +, …, + 100 terms) – 100i 101 (A)  ⎢n − ⎥
1− x ⎣ 1− x ⎦
i (1 − i100 ) i (1 − 1) 1 ⎡ x 3 (1 − x n ) ⎤
= − 100 i = − 100 i = – 100 i
1− i 1− i (B)  ⎢n − ⎥
1− x ⎣ 1− x ⎦

− 100i − 100 i (1 + i ) ⎡
\ S= = = 50 (1 – i) 1 x(1 − x n ) ⎤
1− i 2 (C)  ⎢n − ⎥
1− x⎣ 1− x ⎦
41. The largest value of the positive integer k for which (D)  None of these
nk + 1 divides 1 + n + n2 + … + n127 is divisible by Solution: (A)
(A) 8 (B) 16
We have,
(C) 32 (D) 64
(1 + x) + (1 + x + x2) + (1 + x + x2 + x3)
Solution: (D)  + … upto n terms
We have, 1 − x 2 1 − x3 1 − x 4
= + + + … to n terms
n128 − 1 1− x 1− x 1− x
1 + n + n2 + … + n127 =
n −1 1
= [(1 + 1 + 1 + … n terms)
( n64 − 1) ( n64 + 1) 1− x
=  – (x2 + x3 + x4 + … to n terms)]
n −1
1 ⎡ x (1 − x n ) ⎤
2
= (1 + n + n + … + n63) (n64 + 1)
2 = ⎢ n − ⎥
1 − x ⎢⎣ 1 − x ⎥⎦
\ k = 64 which is divisible by 8, 16, 32 and 64.
Properties of G.P.
42. If A = 1+ ra + r2a + r3a + … ∞ and
a 1. If a1, a2, a3, … are in G.P., then
If B = 1 + rb + r2b + r3b + … ∞, then is equal to
b (a) a1k, a2k, a3k, … are also in G.P.
(A) logB A (B)  log1 – B (1 – A) a a a
(b) 1 , 2 , 3 , … are also in G.P.
k k k
⎛ A − 1⎞
(C) log B − 1 ⎜ (D)  None of these
⎝ A ⎟⎠ (c)
1 1 1
, , , … are also in G.P.
B
a1 a2 a3
Solution:(C) (d) ak1, ak2, a3k … are also in G.P.
1 1 1 A −1 2. If a1, a2, a3, … and b1, b2, b3, … are two G.P.s, then
A= a
⇒ 1 – ra = ⇒ ra = 1 – = (a) a1b1, a2b2, a3b3, … are also in G.P.
1− r A A A
a a a
1 1 1 B −1 (b) 1 , 2 , 3 … are also in G.P.
B = ⇒ 1 – rb = ⇒ rb = 1 – = b1 b2 b3
b B B B
1− r (c) a1 ± b1, a2 ± b2, a3 ± b3, … may not be in G.P.

Objective_Maths_JEE Main 2017_Ch 10.indd 13 01/01/2008 04:29:17


10.14 Chapter 10

3. If a1, a2, a3, …, an are in G.P., then or (r – 4) (r2 + 5r + 21) = 0;


(a) a1 an = a2 an – 1 = a3 an – 2 = . . .
−5 ± 25 − 84
(b) ar = ar − k ar + k , 0 ≤ k ≤ n – r. \ r = 4,
2
4. If a1, a2, a3, …, is a G.P. of positive terms, then log a1,
\ Real value of r is 4. Putting this value in (2),
log a2, log a3, …, is also an A.P. and vice-versa.
5. If a1, a2, a3, …, an–2, an–1, an are in G.P., then a1an = 3
a=
× 4 = 3.
(a2an–1) = (a3an–2) = … = aran–(r–1) 4
In a G.P., the product of two terms equidistant from \ The three numbers are, 3, 3 × 4, 3 × 42,
the beginning and end is a constant and is equal to the i.e., 3, 12, 48.
product of first term and last term.
6. If first term of a G.P. of n terms is a and last term is l,
geometric mean (g.m.)
then the product of all terms of the G.P. is (al)n/2.
7. If there be n quantities in G.P. whose common ratio is Single geometric mean
r and Sm denotes the sum of the first m terms, then the
r A number G is said to be the single geometric mean between
sum of their product taken two by two is Sn Sn–1. two given numbers a and b if a, G, b are in G.P.
r +1
8. If a x , a x , a x , ..., a x are in G.P., then x1, x2, x3, …, xn
1 2 3 n For example, since 2, 4, 8 are in G.P., therefore 4 is
will be in A.P. the G.M. between 2 and 8.
9. Product of a G.P.
n-geometric means
Case I: If number of terms is odd, then Product of
terms = (middle term)No. of terms The numbers G1, G2, …, Gn are said to be the n geometric
Case II: If number of terms is even, then Product of means between two given positive numbers a and b if a, G1,
terms G2, …, Gn , b are in G.P.
= (Geometric mean of middle terms)No. of For example, since 1, 2, 4, 8, 16 are in G.P., therefore
terms
2, 4, 8 are three geometric means between 1 and 16.

inserting Single g.m. between two given


REMEMBER
numbers
Equal non zero numbers are in G.P. Let a and b be two given positive numbers and G be the
G.M. between them. Then a, G, b are in G.P. Thus
G b
Solved examPle = or G2 = ab, or G = ab (Q G > 0)
a G
44. If the sum of three numbers in G.P. is 63 and the prod-
3 inserting n-geometric means between two
uct of the first and the second term is of the third given numbers
term, then the numbers are 4
(A) 3, 12, 48 (B) 4, 12, 36 Let G1, G2, G3, …, Gn be the n geometric means between
(C) 2, 10, 50 (D) None of these two given numbers a and b. Then, a, G1, G2, G3, …, Gn, b
are in G.P.
Solution: (A)
Now, b = (n + 2)th term of G.P.
Let the three numbers be a, ar, ar2
= arn + 1, where r is the common ratio
Given a + ar + ar2 = 63 (1)
1
3 3 b ⎛ b ⎞ n +1
and a . ar = . ar2 or a = r (2) or r n+1
= or r = ⎜ ⎟
4 4 a ⎝ a⎠

3 3 . 3 . 2 1
Putting in (1), r+ r r+ r r = 63 ⎛ b ⎞ n +1
4 4 4 and G1 = ar = a ⎜ ⎟
⎝ a⎠

or r3 + r2 + r – 84 = 0

Objective_Maths_JEE Main 2017_Ch 10.indd 14 01/01/2008 04:29:19


Sequence and Series  10.15

2
i i i
⎛ b ⎞ n +1 234 234 26
2
G2 = ar = a ⎜ ⎟ For example, 0. 234 = 3
= =
⎝ a⎠ 10 1− 999 111

i
M M M  If R = 0.X Y and x denotes the number of digits in X and
2 y denotes the number of digits in Y, then
⎛ b ⎞ n +1
n
Gn = ar = a ⎜ ⎟ R= XY − X
⎝ a⎠
10 x+y
− 10 x
i i
For example, if R = 0.4362, then
Trick(s) for Problem Solving
4362 − 43 4319
R= 4 2
=
 The product of n geometric means between two given 10 − 10 9900
numbers is nth power of the single G.M. between them i.e.
if a and b are two given numbers and G1, G2, …, Gn are n
geometric means between them, then

G1G2G3 … Gn = ( ab)n. Solved Examples


 If A and G are respectively arithmetic and geometric means
45. If two geometric means g1 and g2 and one arithmetic
between two positive numbers a and b then
mean A be inserted between two numbers, then
(A) A > G
g2 g2
1 + 2 =
(B)  the quadratic equation having a, b as its roots is g2 g1
x2 – 2Ax + G2 = 0 (A) 4A (B) 
3A
2 2 (C) 2A (D) 
A
(C)  the two positive numbers are A ± A −G .
Solution: (C)
 If number of terms of any A.P./G.P. is odd, then A.M./G.M.
of first and last terms is middle term of series. Let the two numbers be a and b.
 If number of terms of any A.P./G.P. is even, then A.M./G.M. a+b
of middle two terms is A.M./G.M./H.M. of first and last \ A= or 2A = a + b (1)
2
terms respectively.
th th Again, a, g1, g2, b are in G.P.
 If p , q and rth terms of a G.P. are in G.P., then p, q, r are
in A.P. g1 g b
\ = 2 =
 If a, b, c are in A.P. as well as in G.P. then a = b = c. a g1 g2
a b c

 If a, b, c are in A.P., then x , x , x will be in G.P. (x ≠ ±1) 2
g1 g2 g
 Value of recurring decimal Now = ⇒ 1 = a
a g1 g2
i b
If R = 0.bbb … = 0. b, then R = g2 g2

10′ − 1 b
and = ⇒ 2 = b
5
g1 g2 g1
i 5
For example, 0.5 = =
10′ − 1 9 g12 g2
\ a+b= + 2 (2)
g2 g1
i i ab
If R = 0.ab ab ab … = 0. ab, then R =

102 − 1 \ From Eqs (1) and (2), we get
i i 37 37 g12 g2
For example, 0. 37 = 2
= . 2A = + 2
10 − 1 99 g2 g1

i i i abc
 If R = 0.abc abc abc … , 0. abc then R = and so 46. Let a = 1 . 2 . 3 . 4 . 5. Then
on. 103 − 1
(A) 55 ≥ a (B)  35 ≥ 5!
5
(C) 5 ≥ 6a (D)  None of these

Objective_Maths_JEE Main 2017_Ch 10.indd 15 01/01/2008 04:29:24


10.16  Chapter 10

Solution: (A, B) The required sum


Since A.M. > G.M. = (1 + 2 + 3 + … + 199) – (3 + 6 + 9 + … + 198)
1+ 2 + 3 + 4 + 5 5  – (5 + 10 + 15 + … + 195) + (15 + 30 + 45 +
⇒ ≥ 1⋅ 2 ⋅ 3 ⋅ 4 ⋅ 5
5  … + 195)
⇒ 3 ≥ 5
a ⇒ 35 ≥ a = 5! 199 66 39
= (1 + 199) – (3 + 198) – (5 + 195)
Also, 55 ≥ 1 . 2 . 3 . 4 . 5 = a. 2 2 2
13
47. If a, b, c are positive then the minimum value of + (15 + 195)
2
alog b – log c + blog c – log a + clog a – log b is
(A) 3 (B) 1 = 199 × 100 – 33 × 201 – 39 × 100 + 13 × 105
(C) 9 (D) 16 = 10732
Solution: (A)
49. Sum to 20 terms of the series 1.32 + 2.52 + 3.72 + … is
Since A.M. ≥ G.M.
(A) 178090 (B) 168090
alog b − log c + blog c − log a + c log a − log b (C)  188090 (D)  None of these
\
3 Solution :(C)
3 log b − log c log c − log a log a − log b We have,
≥ a ⋅b ⋅c (1)
tn = [nth term of 1, 2, 3, …] × 
Let x = alog b – log c . blog c – log a . clog a – log b
[nth term of 3, 5, 7, …]2
⇒ log x = (log b – log c) log a + (log c – log a) log b
= n (2n + 1)2 = 4n3 + 4n2 + n.
 + (log a – log b) log c
= 0 ⇒ x = 1. \ Sn = S tn = 4 S n3 + 4 S n2 + S n
2
\ From (1), ⎡ n ( n + 1) ⎤ n ( n + 1) ( 2n + 1) n ( n + 1)
= 4 .  ⎢ ⎥ + 4⋅ +
alog b – log c + blog c – log a + clog a – log b ≥ 3. ⎣ 2 ⎦ 6 2
2
= n2 (n + 1)2 + n (n + 1) (2n + 1)
3 1
Some Special Sequences + n (n + 1);
2
2
1. The sum of first n natural numbers \ S20 = 202 . 212 + × 20 . 21 . 41 
3
n ( n +1) 1
Sn = 1 + 2 + 3 + … + n =
. +  . 20 . 21 = 188090
2 2
2. The sum of squares of first n natural numbers 50. Number of terms in the sequence 1, 3, 6, 10, 15, …,
2 2 2 n ( n + 1) ( 2n + 1)
2 2 5050 is
Sn = 1 + 2 + 3 + … + n = .
6 (A) 50 (B) 75
3. The sum of cubes of the first n natural numbers (C) 100 (D) 125
2
⎡ n ( n + 1) ⎤ Solution: (C)
Sn3 = 13 + 23 + 33 + … + n3 = ⎢
⎥ .
⎣ 2 ⎦ Let S = 1 + 3 + 6 + 10 + 15 +, …, + tn(1)
Solved Examples
then  S = 1 + 3 + 6 + 10 +, …, + tn – 1 + tn (2)
48. The sum of all natural numbers less than 200, that are (1) – (2) ⇒ 0 = (1 + 2 + 3 + 4 + … to n terms) – tn
divisible neither by 3 nor by 5, is n ( n +1)
⇒ tn =
(A) 10730 (B) 10732 2
(C)  15375 (D)  None of these
n ( n +1)
Given, 5050 = ⇒ n2 + n – 10100 = 0
Solution: (B) 2

Objective_Maths_JEE Main 2017_Ch 10.indd 16 01/01/2008 04:29:26


Sequence and Series  10.17

−1 ± 1 + 40400 Trick(s) for Problem Solving


⇒ n=
2
If nth term of a sequence is
−1 ± 40401 Tn = an3 + bn2 + cn + d,
=
2 then the sum of n terms is given by,
−1 ± 201 Sn = STn = aSn3 + bSn2 + cSn + Sd,
= = – 101,100
2 which can be evaluated using the above results.
\ n = 100. (Q n is a positive integer)
n i j
51. ∑∑∑ 1 = Solved Examples
i =1 j =1 k =1

n ( n + 1) ( 2n + 1) n ( n +1) 9 13 17
(A)  (B)  53. The sum of the series 2 + 3 + 4 + ... to
6 2 infinite terms, is 5 .2.1 5 .3.2 5 .4.3
2 2 1
⎛ n ( n + 1) ⎞ n ( n + 1) ( n + 2) (A)  (B) 
(C)  ⎜ (D)  5 5
⎝ 2 ⎟⎠ 6
(C)  1 (D)  None of these
Solution: (D)
Solution: (B)
We have,
The general term of the series is
n i j n i
4r + 1
∑∑∑ 1 = ∑∑ j tr = r , where r ≥ z
i =1 j =1 k =1 i =1 j =1 5 ⋅ r ( r − 1)
n
i (i + 1) 1 ⎡ n n ⎤ 5r − ( r − 1) 1 1
= ∑ 2
= ⎢∑ i 2 + ∑ i ⎥
2 ⎣ i =1
= r
5 ⋅ r ( r − 1)
=
5 r −1
( r − 1)
− r
5 ⋅r
i =1 i =1 ⎦ ∞
\ ∑ tr =
1 ⎡ n ( n + 1) ( 2n + 1) n ( n + 1) ⎤ r=2
= ⎢ +
2 ⎣ 6 2 ⎥⎦
⎛ 1 1 ⎞ ⎛ 1 1 ⎞ ⎛ 1 1 ⎞
 = ⎜ 1 − 2 ⎟ +⎜ 2 − 3 ⎟ +⎜ 3 − 4 ⎟
n ( n + 1) ( n + 2) ⎝ 5 .1 5 .2 ⎠ ⎝ 5 .2 5 .3 ⎠ ⎝ 5 .3 5 .4 ⎠
=
6  + … to infinity
1
52. The sum of the products of the 2n numbers ± 1, ± 2, = (Q terms tend to zero as n → ∞)
5
± 3, …, ± 2n taking two at a time is
n ( n +1) n ( n + 1) ( 2n + 1) 54. For any odd integer n ≥ 1,
(A) – (B) 
2 6 n3 – (n – 1)3 + … + (–1)n–113 =
n ( n + 1) ( 2n + 1) 1
(C) – (D)  None of these (A)  (n – 1)2 (2n – 1)
6 2
1
Solution: (C) (B)  (n – 1)2 (2n – 1)
4
We have, (1 – 1 + 2 – 2 + 3 – 3 + … + n – n)2 1
(C)  (n + 1)2 (2n – 1)
2
= 12 + 12 + 22 + 22 + … + n2 + n2 + 2S,
1
(D)  (n + 1)2 (2n – 1)
where S is the required sum 4
⇒ 0 = 2 (12 + 22 + … + n2) + 2S Solution: (D)
n ( n + 1) ( 2n + 1) Since n is an odd integer (–1)n–1 = 1 and n – 1, n – 3,
⇒ S = – (12 + 22 + … + n2) = – n – 5 etc., are even integers. We have
6

Objective_Maths_JEE Main 2017_Ch 10.indd 17 01/01/2008 04:29:30


10.18  Chapter 10

n3 – (n – 1)3 + (n – 2)3 – (n – 3)3 + … + (–1)n–113 In other words, when | r | < 1 the sum to infinity of an

3 3 3 3 a dr
= n + (n – 1) + (n – 2) + … + 1 arithmetico-geometric series is S∞ = + .
1 − r (1 − r ) 2
 – 2[(n – 1)3 + (n – 3)3 + … + 23]
Method for Finding Sum of A.G.
= n3 + (n – 1)3 + (n – 2)3 + … + 13
Series
⎡⎛ n − 1⎞ 3 ⎛ n − 3 ⎞ 3 3

 ⎟
–2×2 ⎝ 2 ⎠ ⎝ 2 ⎠+ ⎜ ⎟ + +
3 ⎢⎜
... 1 ⎥ Method of Differences
⎢⎣ ⎥⎦
[Q n –1, n – 3 are even integers] Suppose a1, a2, a3, … is a sequence such that the sequence
a2 – a1, a3 – a2, … is either an A.P. or a G.P. The nth term
2 2
⎡ n( n + 1) ⎤ ⎡ 1 ⎛ n − 1⎞ ⎛ n − 1 ⎞ ⎤ ‘a’n of this sequence is obtained as follows:
= ⎢ ⎥ − 16 ⎢ 2 ⎜⎝ 2 ⎟⎠ ⎜⎝ 2 + 1⎟⎠ ⎥
⎣ 2 ⎦ ⎣ ⎦ S = a1 + a2 + a3 + … + an–1 + an
2 2
1 2 ( n − 1) ( n + 1) S = a1 + a2 + … + an–1 + an
= n (n + 1)2 – 16
4 16 × 4 ⇒ an = a1 + [a2 – a1) + (a3 – a2) + … + (an – an–1)]
1 1
= (n + 1)2 [n2 – (n – 1)2] = (n + 1)2(2n – 1). Since the terms within the brackets are either in an A.P. or
4 4 in a G.P., we can find the value of an, the nth term, we can
now find the sum of the n terms of the sequence as
Arithmetico-Geometric progression n

(A.G.P.) S= ∑ ak
k =1
If a1, a2, a3, …, an, … is an A.P. and b1, b2, …, bn, … is G.P.
then the sequence a1b1, a2b2, a3b3, …, anbn, … is said to be Solved Examples
an arithmetico-geometric sequence.
Thus, the general form of an arithmetico geometric
55. Find the sum to n terms of the series:
sequence is a, (a + d) r, (a + 2d) r2, (a + 3d) r3, …
(A) 2 + 5 + 10 + 17 + …
nth term of A.G.P. (B) 3 + 5 + 9 + 17 + …

From the symmetry we obtain that the nth term of this Solution
sequence is [a + (n – 1)d] rn–1. (A) Here, the difference in consecutive terms are 3, 5,
Also, let a, (a + d) r, (a + 2d) r2, (a + 3d) r3, … be an 7, … which are in A.P.
arithmetico-geometric sequence. Let Sn = 2 + 5 + 10 + 17 + … + tn–1 + tn
Then, a + (a + d) r + (a + 2d) r2 + (a + 3d) r3 + … is Shifting every term one place to the right
an arithmetico-geometric series. Sn = 2 + 5 + 10 + … + tn–1 + tn
Subtracting, we get 0 = (2 + 3 + 5 + 7 + … to n
Sum of A.G.P. terms) – tn
⇒ tn = 2 + [3 + 5 + 7 + … to (n – 1) terms]
1. Sum to n terms: The sum of n terms of an arithmeti-
co-geometric sequence a, (a + d) r, (a + 2d) r2, (a + 3d) n −1
=2+ [2 × 3 + (n – 2) × 2]
r3, … is given by, 2
n −1
⎧ a (1 − r n −1 ) [a + ( n − 1)d ]r n =2+ (2n + 2) = 2 + (n – 1) (n + 1)
⎪ + dr − , when r ≠ 1 2
Sn = ⎪⎨1 − r (1 − r )2 1− r = 2 + (n2 – 1) = n2 + 1.
⎪n Putting n = 1, 2, 3, … , n and adding, we get
⎪⎩ 2 [2a + ( n − 1)d ], when r = 1 n
n( )( )
Sn = ∑ k 2 + n = n + 1 2n + 1 + n
2. Sum to infinite terms: Let | r | < 1. Then rn, rn–1 → 0 k =1 6
as n → ∞ and it can also be shown that n . rn → 0 as n n
= (2n + 3n + 1 + 6) = (2n2 + 3n + 7)
2

a dr 6 6
n → ∞. So, we obtain that Sn → + , as (B) Here the differences of consecutive terms are 2, 4,
1 − r (1 − r )2
8, … which are in G.P.
n → ∞.

Objective_Maths_JEE Main 2017_Ch 10.indd 18 01/01/2008 04:29:33


Sequence and Series  10.19

Let Sn = 3 + 5 + 9 + 17 + … + tn–1 + tn 57. The sum of first n terms of the series


Shifting every term one place to the right 1 . 1! + 2 . 2! + 3 . 3! + 4 . 4! + … is
Sn = 3 + 5 + 9 + … + tn–1 + tn (A) (n + 1)! – 1 (B)  n! – 1
Subtracting, we get 0 = (3 + 2 + 4 + 8 + … to n terms) (C) (n – 1)! – 1 (D)  None of these
– tn
⇒ tn = 3 + [2 + 4 + 8 + … to (n – 1) terms] Solution: (A)
n−1 Let Sn = 1 . 1! + 2 . 2! + 3 . 3! + 4 . 4!
= 3 + 2 ( 2 − 1) = 3 + 2n – 2
2 −1  + … + n × n!
= 1 + 2n
Putting n = 1, 2, 3, … , n, we get ⇒ Sn = (2 – 1) 1! + (3 – 1) 2! + (4 – 1) 3!
t1 = 1 + 21
 + (5 – 1) 4! + … + [(n + 1) – 1] n!
t2 = 1 + 22
t3 = 1 + 23 = (2 . 1! – 1!) + (3 . 2! – 2!) + (4 . 3! – 3!)
M
tn = 1 + 2n  + (5 . 4! – 4!) + … + [(n + 1) n! – n!]
Adding column-wise, we get
Sn = n + (2 + 22 + 23 + … + 2n) = (2! – 1!) + (3! – 2!) + (4! – 3!) + (5! – 4!)
n
= n + 2 ( 2 − 1) = 2n+1 + n – 2
 + … + [(n + 1)! – n!]
2 −1
= (n + 1)! – 1! = (n + 1)! – 1.
56. Sum to infinity of the series
2 5 2 11 58. If a, b, c are digits, then the rational number repre-
− + − +… is
3 6 3 24 sented by 0 . cababab … is
4 1 99c + ab 99c + 10 a + b
(A)  (B)  (A)  (B) 
9 3 990 99
2 99c + 10 a + b
(C)  (D)  None of these (C)  (D)  None of these
9 990
Solution: (C) Solution: (C)

2 5 2 11 Let R = 0 . cababab…
Let S=
− + − + ... to ∞(1)
3 6 3 24 ⇒ 102R = ca . bababa …
1
Multiplying both sides by – , the common ratio of and 104R = caba . baba …
G.P. 2
1 ⇒ (104 – 102) R = caba – ca
2 5 8
– S=– + − + ... to ∞(2)
2 6 12 24 caba − ca 1000c + 100a + 10b + a − 10c − a
⇒ R = =
Subtracting Eqs (2) from (1), we have 9900 9900
3 2 3 3 3 99c + 10 a + b
S= − + − + ... to ∞ =
2 3 6 12 24 990
2 ⎛1 1 1 ⎞ 59. The sum of first n terms of the series
= − − + + ... to ∞⎟ 2
3 ⎜⎝ 2 4 8 ⎠ 12 + 2.22 + 32 + 2.42 + 52 + 5.62 + … is n ( n +1)
2
1 when n is even. When n is odd the sum is
2 2 2 1 1
= − = − = n2 ( n + 1) n ( n +1) 2
3 ⎛ ⎞1 3 3 3 (A)  (B) 
1− ⎜ − ⎟ 2 2
⎝ 2⎠ 2
⎡ n ( n + 1) ⎤ n ( n +1)
1 2 2 (C)  ⎢ ⎥ (D) 
\ S= × = ⎣ 2 ⎦ 2
3 3 9

Objective_Maths_JEE Main 2017_Ch 10.indd 19 01/01/2008 04:29:36


10.20  Chapter 10

Solution: (A) Solution: (D)


When n is odd, last term will be n2, \ then the sum is
Let S = 1 + 2 . 2 + 3 . 22 + 4 . 23 + 5 . 24 
2 2 2 2 2 2
1 + 2.2 + 3 + 2.4 + 5 + 2.6 + … + 2 (n – 1) + n 2 2
 + … + 100 . 299
( n − 1) n2 ⎡ n ( n + 1) 2 ⎤ \ 2S = 1 . 2 + 2 . 22 + 3 . 23 + … + 99 . 299 
= + n2 ⎢ Replacing n by n − 1 in ⎥
2 2  + 100 . 2100
⎣ ⎦
Substracting, we get
3 2 2 n3 + n2 n2 ( n + 1)
= n − n + 2n = = – S = 1 + 1 . 2 + 1 . 22 + … + 1 . 299 –100 . 2100
2 2 2
= (1 + 2 + 22 + … + 299) – 100 . 2100
60. The sum of the series
1( 2100 − 1)
1 + 2 . 2 + 3 . 22 + 4 . 23 + 5 . 24 + … + 100 . 299 is = – 100 · 2100 = 2100 – 1 – 100 . 2100
(A) 99 . 2100 + 1 (B)  100 . 2100 2 −1
.  100
(C) 99 2 (D)  . 2100 + 1
99  \ S = 100 . 2100 – 2100 + 1 = 99 . 2100 + 1.

EXERCISES

Single Option Correct Type


1. If a, b, c are positive numbers in A.P. such that their
⎡ 9⎤
product is 64, then the minimum value of b (A)  ⎢1, (– ∞, 0)
(B) 
⎣ 2 ⎥⎦
(A) = 2 (B)  =4
⎡ 9⎤
(C)  ⎡⎢ , 1⎤⎥ (D) 
(C) = 1 (D)  Does not exist 5
⎢1, 5 ⎥
2. If three successive terms of a G.P. with common ratio ⎣9 ⎦ ⎣ ⎦
r(r > 1) form the sides of a DABC and [r] denotes 6. Let Sn (1 ≤ n ≤ 9) denotes the sum of n terms of series
greatest integer function, then [r] + [–r] = 1 + 22 + 333 + … + 999999999, then for 2 ≤ n ≤ 9
(A) 0 (B) 1 1
(A) Sn – Sn–1 = (10n – n2 + n)
(C)  –1 (D)  None of these 9
1
(B) Sn = (10 – n2 + 2n – 2)
n
21
9
3. If ∑ aj = 693, where a1, a2, …, a21, are in A.P., then (C) 9(Sn – Sn–1) = n(10n – 1)
j =1
(D)  None of these
10
∑ a2i +1 is 7. If log 5 x + log5 x + log5 x + … upto 7 terms = 35,
1/3 1/4

i=0 then x is equal to


(A) 361 (B) 396 (A) 5 (B) 25
(C) 363 (D) data insufficient (C)  125 (D)  None of these
∞ ∞
4. Number of increasing geometrical progression(s) with 8. If ∑ x n−1 = a and ∑ y n−1 = b where | x |, | y | < 1,
first term unity, such that any three consecutive terms, n =1 n =1
on doubling the middle become an A.P. is ∞

(A) 0 (B) 1
then ∑ ( xy)n−1 =
n =1
(C) 2 (D) infinity a + b −1
(A) ab (B) 
5. If a1, a2, a3 (with a1 > 0) are in G.P. with common ratio ab
r, then the value of r for which the inequality 9a1 + 5a3 1 ab
(C)  (D) 
> 14a2 holds, cannot be in the interval 1− ab a + b −1

Objective_Maths_JEE Main 2017_Ch 10.indd 20 01/01/2008 04:29:39


Sequence and Series  10.21

9. Let p, q, r ∈ R+ and 27pqr ≥ (p + q + r)3 and 3p + 4q (A) 2 (B) 1


+ 5r = 12 then p3 + q4 + r5 is equal to (C) 4 (D)  2
(A) 3 (B) 6 17. Sum to n terms of the series
(C)  2 (D)  None of these
1 1
10. The sum of the series + is
(1 + x ) (1 + 2 x ) (1 + 2 x ) (1 + 3 x )
1 2 3
2 4 + + + … to n terms
1 + 1 + 1 1 + 2 + 2 1 + 3 + 34
2 4 2
nx n
(A)  (B) 
is (1 + x ) (1 + nx ) (1 + x ) [1 + ( n + 1) x ]
n( n2 + 1) n( n + 1)
(A)  2 (B)  x
n + n +1 2( n2 + n + 1) (C)  (D)  None of these
(1 + x ) (1 + ( n − 1) x )
2
n( n − 1)
(C)  (D)  None of these 18. If a, b, c are distinct positive real numbers and a2 + b2
2( n2 + n + 1)
+ c2 = 1, then ab + bc + ca is
11. a, b, c are three distinct real numbers, which are in G.P. (A)  less than 1 (B)  equal to 1
and a + b + c = xb. Then (C)  greater than 1 (D)  any real number
(A) x < –1 or x > 3 (B)  –1 < x < 3
19. The value of (n – 2)2 + (n – 4)2 + (n – 6)2 + … to n
(C) –1 < x < 2 (D)  0 < x < 1
terms is
12. The sum of the first hundred terms of an A.P. is x and n n
(A)  (n2 + 2) (B)  (n2 + 3)
the sum of the hundred terms starting from the third 3 2
term is y. Then the common difference is n 2 n
(C)  (n – 2) (D)  (n2 – 3)
y−x y−x 3 2
(A)  (B) 
2 50 20. The sum to infinity of the series
y−x y−x 2
(C) 
100
(D)  1 + 2 ⎛⎜1 − 1 ⎞⎟ + 3 ⎛⎜1 − 1 ⎞⎟ + … where n ∈ N, is given
200 ⎝ n⎠ ⎝ n⎠

1 ∞
1 by
13. If λ = ∑ i4 , then ∑ (2i − 1)4 is
(A) n (n – 1)
⎛ 1⎞
(B)  n ⎜1 − ⎟
2

i =1 i =1
⎝ n⎠
14 λ 2
(A)  λ (B)  ⎛ n − 1⎞
15 2 (C) n2 (D) 
⎜⎝ n ⎟⎠
16 15
(C)  λ (D)  λ 21. a1, a2, a3, … are in A.P. with common difference not a
15 16
multiple of 3. Then, maximum number of consecutive
14. The sum of all possible products of the first n natural terms so that all the terms are prime numbers is
numbers taken two at a time is (A) 2 (B) 3
1 1 (C) 5 (D) infinite
(A)  [Sn2 – Sn] (B) [(Sn)2 – Sn]
2 2 22. The coefficient of x49 in the product (x – 1) (x – 3) …
1 1 (x – 99) is
(C)  [Sn2 – Σ(n + 1)] (D)  [(Sn)2 – Sn2]
2 2 (A)  – 992 (B)  1
(C)  – 2500 (D)  None of these
15. The minimum value of 8sin x/8 + 8cos x/8 is
1 3+ 2
23. If x, y, z are three real numbers of the same sign then
3− 2 / 2 x y z
(A) 2 2 2
(B)  the value of + + lies in the interval
y z x
1 3− 2 (A) [2, ∞) (B)  [3, ∞)
3+ 2 / 2
2 2 (C) (3, ∞) (D)  (–∞, 3)
(C) 2 (D) 
24. In a G.P. of alternating positive and negative terms,
16. If log 2 a + log 2 a + log2 a + log2 a + … upto 20
1/2 1/4 1/6 18
/
any term is the A.M. of the next two terms. Then the
terms is 840, then a is equal to common ratio is

Objective_Maths_JEE Main 2017_Ch 10.indd 21 01/01/2008 04:29:44


10.22  Chapter 10

(A) –1 (B) –3
2 1 1⎛ 1 n! ⎞
−1 (A)  − (B)  −

(C) 
–2 (D)  5! ( n + 1)! 4 ⎝ 4 ! ( n + 4)!⎟⎠

2
1⎛ 1 3! ⎞
25. If A = 1 + ra + r2a + r3a + .... as and B = 1+ rb + r2b + (C)  ⎜ − (D)  None of these
a 4 ⎝ 3! ( n + 2)!⎟⎠
r3b + …. as, then is equal to
b 32. If a, b, c, d and p are distinct real numbers such that
(1− A)
(A) log BA (B)  log1− B (a2 + b2 + c2) p2 – 2p (ab + bc + cd) + (b2 + c2 + d2)
≤ 0 then a, b, c, d are in
⎛ A −1 ⎞ (A) A.P. (B) G.P.
(C) log B −1 ⎜ ⎟ (D)  None of these
B ⎝ A ⎠ (C) H.P. (D)  ab = cd

26. If the sum of n terms of an A.P. is cn (n – 1), where 33. If a + b + c = 3 and a > 0, b > 0, c > 0, then the greatest
c ≠ 0, then sum of the squares of these terms is value of a2 b3 c2 is
(A) c2n2(n + 1)2 310 ⋅ 24 39 ⋅ 24
(A)  (B) 
2 77 77
(B)  c2 n (n – 1) (2n – 1)
3 8 4
2 (C)  3 ⋅ 2 (D)  None of these
(C)  2c n (n + 1) (2n + 1) 77
3
(D)  None of these a b aa − b
1
34. If b c ba − c = 0 and α ≠ , then
27. If in an A.P., Sn = p.n2 and Sm = p.m2 where Sr denotes 2
the sum of r terms of the A.P., then Sp is equal to 2 1 0
1 (A) a, b, c are in A.P. (B)  a, b, c are in G.P.
(A)  p3 (B)  mnp (C) a, b, c are in H.P. (D)  None of these
2
(C) p3 (D)  (m + n) p2 35. Suppose a, b, c are in A.P. and a2, b2, c2 are in G.P. If
3
28. If b1, b2 and b3 (b1 > 0) are three successive terms of a a < b < c and a + b + c = , then the value of a is
2
G.P. with common ratio r, the value of r for which the 1 1
inequality b3 > 4b2 – 3b1 holds, is given by (A)  (B) 
2 2 2 3
(A) r > 3 (B)  r < 1
1 1 1 1
(C) r = 2.5 (D)  r = 1.7 (C)  − (D)  −
2 3 2 2
29. If p, q, r are positive and are in A.P., the roots of qua- 36. If a1, a2, …, an are in A.P. with common difference
dratic equation px2 + qx + r = 0 are all real for d ≠ 0, then sum of the series sin d [sec a1 sec a2 + sec
r p a2 sec a3 + … + sec an–1 sec an] is
(A)  − 7 ≥ 4 3 (B)  −7 ≥ 4 3
p r (A) tan an – tan a1 (B) cot an – cot a1
(C) sec an – sec a1 (D) cosec an – cosec a1
(C) all p and r (D) no p and r
37. The first and last term of an A.P. are a and l respec-
30. The sum to n terms of the series tively. If S is the sum of all the terms of the A.P. and the
1 5 19 65 l 2 − a2
+ + + + … is common difference is , then k is equal to
3 9 27 81 k − ( l + a)
(A) S (B)  2S
(3n − 2n ) 2 (3n − 2n )
(A) n – (B)  n – (C) 3S (D)  None of these
2n 3n
38. If a, b, c, d are in G.P., then
(C) 2n – 1 (D) 
3n – 1
(a2 + b2 + c2) (b2 + c2 + d2) =
1 1! 2 ! 3! (A) (ab + ac + bc)2 (B) (ac + cd + ad)2
31. Sum to n terms of the series + + + + … is (C) (ab + bc + cd) 2
(D)  None of these
5! 6 ! 7 ! 8!

Objective_Maths_JEE Main 2017_Ch 10.indd 22 01/01/2008 04:29:47


Sequence and Series  10.23

39. If one geometric mean G and two arithmetic means 46. The first two terms of a geometric progression add up
p and q be inserted between two numbers, then G2 is to 12. The sum of the third and the fourth terms is 48.
equal to If the terms of the geometric progression are alter-
(A) (3p – q) (3q – p) (B) (2p – q) (2q – p) nately positive and negative, then the first term is
(C) (4p – q) (4q – p) (D)  None of these (A) –4 (B) –12
(C) 12 (D) 4
40. The product of n positive integers is 1, then their sum
is a positive integer, that is 47. The
sum to the infinity of the series
(A)  equal to 1 (B)  equal to n + n2 2 6 10 14
1 + + 2 + 2 + 4 + ... is
(C)  divisible by n (D)  never less than n 3 3 3 3
41. A man saves ` 200 in each of the first three months of (A) 2 (B) 3
his service. In each of the subsequent months his sav- (C) 4 (D) 6
ing increases by ` 40 more than the saving of immedi- 48. The sum of positive terms of the series
ately previous months. His total saving from the start
of service will be ` 11040 after 4 1
10 + 9 + 9 + … is
(A)  21 months (B)  18 months 7 7
(C)  19 months (D)  20 months 352 437
(A)  (B) 
42. Statement-1: The sum of the series 1 + (1 + 2 + 4) + 7 7
(4 + 6 + 9) + (9 + 12 + 16) + …+ (361 + 380 + 400) is 852
(C)  (D)  None of these
8000. 7
n
Statement-2:
∑ (k 3 − (k − 1)3 ) = n3 , for any natural 49. The sum of the products of the 2n numbers ±1, ±2, ±3.
number n. k =1 …, ±n taking two at a time is
(A)  Statement-1 is false, Statement-2 is true. n ( n + 1) n ( n + 1)
(B)  Statement-1 is true, statement-2 is true; state- (A)  (B) 

2 2
ment-2 is a correct explanation for Statement-1
(C)  Statement-1 is true, statement-2 is true; n ( n + 1) ( 2n + 1) n ( n + 1) ( 2n + 1)
(C)  (D) –
statement-2 is not a correct explanation for 6 6
Statement-1. 50. If a is the first term, d the common difference and Sk
(D)  Statement-1 is true, statement-2 is false. S
the sum to k terms of an A.P., then for kx to be inde-
43. If 100 times the 100th term of an AP with non-zero pendent of x Sx
common difference equals the 50 times its 50th term,
(A) a = 2d (B)  a=d
then the 150th term of this AP is
(C) 2a = d (D)  None of these
(A) –150
(B)  150 times its 50th term 51. Given that α, γ are roots of the equation Ax2 – 4x + 1 = 0
(C) 150 and β, δ are roots of the equation Bx2 – 6x + 1 = 0. If
(D) zero α, β, γ and δ are in H.P., then
(A) A = 5 (B)  A=–3
44. If the sum of first n terms of two A.P.’s are in the ratio
(C) B = 8 (D)  B=–8
3n + 8 : 7n + 15, then the ratio of their 12th terms is
(A)  8 : 7 (B)  7 : 16 52. The sum of n terms of m A.P.s are S1, S2, S3, …, Sm. If
(C)  74 : 169 (D)  13 : 47 the first term and common difference are 1, 2, 3, …, m
respectively, then S1 + S2 + S3 + … + Sm =
1 3 7 15
45. The sum of n terms of the series + + + + ... 1
2 4 8 16 (A)  mn (m + 1) (n + 1)
is 4
1 1
(A) 2n − n − (B)  1 – 2–n (B)  mn (m + 1) (n + 1)
2 2
1 n (C) mn (m + 1) (n + 1)
(C) n + 2–n – 1 (D)  ( 2 − 1)
2 (D)  None of these

Objective_Maths_JEE Main 2017_Ch 10.indd 23 01/01/2008 04:29:49


10.24  Chapter 10

53. If three positive numbers a, b, c are in H.P., then 61. a, b, c are three distinct real numbers, which are in G.P.
an + cn and a + b + c = xb. Then,
(A) > 2bn (B)  = 2bn (A) x < –1 or x > 3 (B)  –1 < x < 3
n
(C) < 2b (D)  > bn (C) –1 < x < 2 (D)  0 < x < 1
3
54. The sum of first n terms of the series 1
1 ⋅ 1! + 2 ⋅ 2! + 3 ⋅ 3! + 4 ⋅ 4! + … is
62. If a1, a2, a3, a4 are in H.P., then
a
∑ ar ar +1 is a
root of 1a4 r =1
(A) (n + 1)! – 1 (B)  n! – 1
(C) (n – 1)! – 1 (D)  None of these
(A) x2 + 2x + 15 = 0 (B)  x2 + 2x – 15 = 0
55. If a, b, c are digits, then the rational number repre- (C) x2 – 6x – 8 = 0 (D)  x2 – 9x + 20 = 0
sented by 0 ⋅ cababab … is
63. The sum to n terms of the series
99c + ab 99c + 10 a + b 1 5 19 65
(A)  (B)  + + + + … is
990 99 3 9 27 81
99c + 10 a + b (3n − 2n ) 2 (3n − 2n )
(C)  (D)  None of these
990 (A) n – (B)  n –
2n 3n
56. The sum of first n terms of the series n
(C) 2 – 1 (D) n
3 –1
2
12 + 2.22 + 32 + 2.42 + 52 + 5.62 + … is n ( n +1) 64. If a + b + c = 3 and a > 0, b > 0, c > 0, then the greatest
when n is even. When n is odd, the sum is 2 value of a2 b3 c2 is
310 ⋅ 24 39 ⋅ 24
n2 ( n + 1) n ( n +1) 2 (A)  (B) 
(A)  (B)  77 77
2 2 8
4
2 (C)  3 ⋅ 2 (D)  None of these
⎡ n ( n + 1) ⎤ n ( n +1) 77
(C)  ⎢ ⎥ (D) 
⎣ 2 ⎦ 2 65. Let the harmonic mean and the geometric mean of two
57. The sum of the series positive numbers be in the ratio 4 : 5. The two numbers
1 + 2 . 2 + 3 . 22 + 4 . 23 + 5 . 24 + … + 100 . 299 is are in the ratio
(A) 99 . 2100 + 1 (B)  100 . 2100 (A)  1 : 1 (B)  2 : 1 (C)  3 : 1 (D)  4 : 1
(C) 99 . 2 (D) 
100 . 2100 + 1
99 
66. The first and last term of an A.P. are a and l, respec-
58. Four different integers form an increasing A.P. If one tively. If S is the sum of all the terms of the A.P. and the
of these numbers is equal to the sum of the squares of l 2 − a2
the other three numbers, then the numbers are common difference is , then k is equal to
k − ( l + a)
(A)  – 2, – 1, 0, 1 (B)  0, 1, 2, 3 (A) S (B)  2S
(C)  – 1, 0, 1, 2 (D)  None of these (C) 3S (D)  None of these
59. If three successive terms of a G.P. with common ratio 67. If a1, a2, …, an are in A.P. with common difference
r(r > 1) form the sides of a DABC and [r] denotes d ≠ 0, then sum of the series sin d [sec a1 sec a2 + sec a2
greatest integer function, then [r] + [–r] = sec a3 + … + sec an–1 sec an] is
(A) 0 (B) 1 (A) tan an – tan a1
(C)  –1 (D)  None of these (B) cot an – cot a1
60. Let Sn (1 ≤ n ≤ 9) denotes the sum of n terms of series (C) sec an – sec a1
1 + 22 + 333 + … + 999999999, then for 2 ≤ n ≤ 9 (D) cosec an – cosec a1
1 1 1! 2 ! 3!
(A) Sn – Sn–1 = (10n – n2 + n) 68. Sum to n terms of the series + + + + … is
9 5! 6 ! 7 ! 8!
1 2 1 1⎛ 1 n! ⎞
(B) Sn = (10 – n2 + 2n – 2)
n
(A)  − (B)  −
9 5! ( n + 1)! 4 ⎜⎝ 4 ! ( n + 4)!⎟⎠
(C) 9(Sn – Sn–1) = n(10n – 1)
1⎛ 1 3! ⎞
(C)  ⎜ − (D)  None of these
(D)  None of these 4 ⎝ 3! ( n + 2)!⎟⎠

Objective_Maths_JEE Main 2017_Ch 10.indd 24 01/01/2008 04:29:53


Sequence and Series  10.25

n ( n −1)
69. If an and bn be two sequences given by (C)  (1 – 2n) 2 2 ⋅


n n n n

an = ( x )1/2 + ( y )1/2 and bn = ( x )1/2 − ( y )1/2 for all (D)  None of these
n ∈ N. Then, a1a2a3 … an is equal to 75. If 0.272727…, x and 0.727272… are in H.P., then x
x+ y must be
(A) x – y (B)  (A) rational (B) integer
bn
(C)  irrational (D)  None of these
x− y xy
(C)  (D)  76. If a1 = 0 and a1, a2, a3, …, an are real numbers such
bn bn
that | ai | = | ai – 1 + 1 | for all i then the A.M. of the
70. For any odd integer n ≥ 1, numbers a1, a2, …, an has value x where
n3 – (n – 1)3 + … + (–1)n–113 = 1 1
(A) x ≤ – (B)  x≥–
1 2 2
(A)  (n – 1)2 (2n – 1) 1
2 (C) x < – (D)  None of these
1 2
(B)  (n – 1)2 (2n – 1) 77. If a1, a2, a3, …, an are in H.P., then
4
1 a1 a2
(C)  (n + 1)2 (2n – 1) , …,
2 a2 + a3 + ... + an a1 + a3 + ... + an
1 an
(D)  (n + 1)2 (2n – 1) are in
4 a1 + a2 + ... + an −1
71. For a positive integer n, let a (n) = (A)  A.P. (B)  G.P.
(C)  H.P. (D)  None of these
1 1 1 1
1+ + + + ... + n . Then
2 3 4 (2 ) − 1 78. The consecutive numbers of a three digit number form
a G.P. If we subtract 792 from this number, we get a
(A) a (100) ≤ 100 (B)  a (100) > 100 number consisting of the same digits written in the
(C) a (200) ≤ 100 (D)  a (200) > 100 reverse order and if we increase the second digit of the
72. Let α, β, γ be the roots of the equation required number by 2, the resulting number forms an
3x3 – x2 – 3x + 1 = 0. If α, β, γ are in H.P. then A.P. The number is
| α – γ | = (A) 139 (B) 193
1 2 (C)  931 (D)  None of these
(A)  (B) 
3 3 79. The largest term of the sequence
4 1 4 9 16
(C)  (D)  None of these , , , , … is
3 503 524 581 692
73. Suppose a, b > 0 and x1, x2, x3 (x1 > x2 > x3) are roots 16 4
(A)  (B) 
x−a x−b b a 692 524
of + = + and x1 – x2 – x3 = c,
b a x−a x−b 49
(C)  (D)  None of these
then a, b, c are in 1529
(A) A.P. (B)  G.P.
(C)  H.P. (D)  None of these 80. The coefficient of x99 and x98 in the polynomial
(x – 1) (x – 2) (x – 3) … (x – 100) are
74. The coefficient of xn in the product (A)  – 5050 and 12482075
(1 – x) (1 – 2x) (1 – 22 ⋅ x) (1 – 23 ⋅ x) … (1 – 2n ⋅ x) is (B)  – 4050 and 12582075
equal to (C)  – 5050 and 12582075
n ( n −1)
(D)  None of these
(A)  (1 – 2n + 1) 2 2 ⋅
n ( n −1) 81. The three successive terms of a G.P. will form the
(B) (2 n+1
– 1) ⋅2 2 sides of a triangle if the common ratio r satisfies the
inequality

Objective_Maths_JEE Main 2017_Ch 10.indd 25 01/01/2008 04:29:57


10.26  Chapter 10

3 −1 3 +1 π4 π4
(A)  <r< (A)  (B) 
2 2 45 96
π4
(B)  5 − 1 < r < 5 +1 (C)  (D)  None of these
2 2 124

2 −1 89. If the (m + 1) th, (n + 1) th and (r +1)th terms of an A.P.


(C)  < r < 2 +1 are in G.P. and m, n, r are in H.P., then the ratio of the
2 2
(D)  None of these first term of the A.P. to its common difference is
n n
82. If the sides of a right angled triangle are in G.P., then −
(A)  (B) 
3 3
the cosine of the greater acute angle is
n n
1 1 (C)  (D) 

(A)  (B)  2 2
1+ 5 1− 5
90. Let there be n numbers in G.P. whose common ratio
(C)  1 + 5 (D)  None of these is r and Sm denotes the sum of their first m terms. The
2 sum of their products taken two at a time is k Sn Sn–1
83. Sum to n terms of the series 2 + 5 + 14 + 41 + … is where k =
n 1 n 3 r −1 r −1
(A)  + (3n − 1) (B)  + (3n − 1) (A)  (B) 
2 4 2 4 r r +1
n 1 r
(C)  + (3n − 1) (D)  None of these (C)  (D)  None of these
2 2 r +1
84. If the pth, qth and rth terms of both an A.P. and a G.P. 91. If a, b, c, d are distinct integers in A.P. such that d = a2
be respectively a, b and c, then + b2 + c2, then a + b + c + d =
(A) ac ⋅ cb ⋅ ba = ac ⋅ bc ⋅ ab (A) 2 (B) 1
(B) ab – 1 ⋅ bc + 1 ⋅ ca – 1 = ac – 1 ⋅ ba – 1 ⋅ cb + 1 (C)  0 (D)  None of these
(C) ab ⋅ bc ⋅ ca = ac ⋅ ba ⋅ cb 1 1 1
(D)  None of these 92. If Hn = 1 + + + ..... + , then the value of
2 3 n
85. If, in a G.P. of 3n terms, S1 denotes the sum of the first 3 5 2n − 1
n terms, S2 the sum of the second block of n terms and 1 + + + .... + is
2 3 n
S3 the sum of the last n terms, then S1, S2, S3 are in
(A) n – Hn (B)  2n – Hn
(A)  A.P. (B)  G.P. (C) (n – 1) – Hn (D)  n – 2Hn
(C)  H.P. (D)  None of these
93. If am be the mth term of an A.P., then
86. In a geometric series, the first term is a and common
a21 –a22 + a23 – a24 +….+ a22n–1– a22n =
ratio is r. If Sn denotes the sum of n terms and Un
n n −1 2 n
= ∑ Sn, then rSn + (1 – r) un =
(A)  ( a − a22n ) (B)  (a2 − a2 )
2n − 1 1 2n − 1 2 n 1
n =1
n
(A) na (B)  (n – 1) a (C)  ( a 2 − a22n ) (D)  None of these
(C) (n + 1) a (D)  None of these 2n − 1 1

87. In a Dabc, if cot A, cot B, cot C are in A.P. then a2, b2, 1
for n ≥ 1 and a3 = a1, then (a2001)2001=
94. If an + 1 =
c2 are in 1− an
(A)  A.P. (B)  G.P. (A) 1 (B) –1
(C)  H.P. (D)  A.G. P. (C)  0 (D)  None of these
1 1 1 4
88. If 4
+ 4
+ 4
+ .... up to ∞ = π , then the value of 95. If a, b, c are positive numbers in G.P. and log
1 2 3 90 ⎛ 5c ⎞ ⎛ 3b ⎞ ⎛ a⎞
1 1 1 ⎜⎝ a ⎟⎠ , log ⎜⎝ 5c ⎟⎠ and log ⎜⎝ 3b ⎟⎠ are in A.P. then a, b, c
+ + + …. up to ∞ is
14 34 54

Objective_Maths_JEE Main 2017_Ch 10.indd 26 01/01/2008 04:30:01


Sequence and Series  10.27

(A)  form the sides of an equilateral triangle If p, q, r are in A.P. then x, y, z are in
(B)  form the sides of an isosceles triangle (A)  A.P. (B)  G.P.
(C)  form the sides of a right angled triangle (C)  H. P (D)  None of these
(D)  can not form the sides of a triangle
101. If | a | < 1 and | b | < 1, then the sum of the series
96. If a, b, c are in G.P. and log a – log 2b, log 2b – log 3c 1 + (1 + a) b + (1 + a + a2) b2 + (1 + a + a2 + a3) b3 +
and log 3c – log a are in A.P., then a, b, c are the sides … ∞ is equal to
of a triangle which is
1 1
(A)  right angled (A)  (B) 
(1 − b)(1 − ab) (1 − a)(1 − ab)
(B)  acute angled
(C)  obtuse angled 1
(C)  (D)  None of these
(D)  None of these (1 − a)(1 − b)
97. In a sequence of 4n + 1 terms, the first 2n +1 terms 102. If < an > and < bn > be two sequences given by an =
are in A.P. having common difference 2 and the last −n −n −n −n

x 2 + y 2 and bn = x 2 − y 2 ∀ n ∈ N, then the


1
2n + 1 terms are in G.P. having common ratio . If the value of a1 a2 a3 ….an is
2
middle term of the A.P. is equal to the middle term of x+ y x−y
(A)  (B) 
the G.P. then the middle term of the sequence is bn bn
n.2n + 1 n.2n + 1 x2 + y2 x2 − y2
(A)  n
(B)  (C)  (D) 
2 +1 2n − 1 bn bn
n
(C)  n.2 (D)  None of these 103. The sixth term of an A.P. is equal to 2. The value of
2n − 1 the common difference of the A.P. which makes the
product a1 a4 a5 greatest, is
98. If S1, S2 and S3 denote the sums up to n > 1 terms of
8 2
three sequences in A.P. whose first terms are unity and (A)  (B) 
common differences are in H.P. then n = 5 3
3 3
2S3 S1 + S1S2 + S2 S3 (C)  (D) 
(A)  5 4
S1 − 2S2 + S3 104. If the natural numbers are written as
2S3 S1 − S1S2 − S2 S3 1
(B) 
S1 + 2S2 + S3 2 3
2S3 S1 − S1S2 − S2 S3 4 5 6
(C) 
S1 − 2S2 + S3 7 8 9 10
(D)  None of these ......................
......................
99. Sum to n terms of the series 13 + 3.23 + 33 + 3.43 + 53
+….., where n is even, is Then, the sum of the terms of the nth row is
n ( n2 − 1) n ( n2 + 1)
n2 ( n2 − 3n + 1) n2 ( n2 + 3n + 1) (A)  (B) 
(A)  (B)  2 4
2 2
n ( n2 + 1)
2 2 (C)  (D)  None of these
(C)  n ( n + 3n + 1) (D)  None of these 2
4
100. Let a be a fixed real number such that
a− x a− y a− z
= =
px qy rz

Objective_Maths_JEE Main 2017_Ch 10.indd 27 01/01/2008 04:30:05


10.28  Chapter 10

More than One Option Correct Type


105. The H.M. of two numbers is 4. If their A.M. A and 111. The solution of the equations logx + logx 1/2 + log x 1/4
G.M. G satisfy the relation 2A + G2 = 27, then the 1 + 3 + 5 + ..... + ( 2 y − 1)
numbers are + ….= y and
4 + 7 + 10 + .... + (3 y + 1)
(A) 1 (B) 2 20
(C) 3 (D) 6 = is
7 log x
106. If the first and the (2n – 1)th terms of an A.P., G.P. and
(A) x = 105, 10–5/7
H.P. are equal and their nth terms are a, b, c respec-
tively, then 10
(B) y = 10, −
(A) a = b = c (B)  a≥b≥c 7
(C)  a + c = b (D)  ac – b2 = 0 10
(C) x = 10, −
107. The real numbers x1, x2, x3 satisfying the equation 7
x3 – x2 + bx + γ = 0 are in A.P. The intervals in which (D) y = 105, 10–5/7
β and γ lie are
⎛ 1⎤ ⎡ 1 ⎞ 1 12. The sum of of first ten terms of an A.P. is equal to 155
(A) b ∈ ⎜ − ∞, ⎥ (B) b ∈ ⎢ − , ∞⎟
⎝ 3⎦ ⎣ 27 ⎠ and the sum of first two terms of a G.P. is 9. If the
first term of the A.P. is equal to the common ratio of
⎛ 1⎤ ⎡ 1 ⎞ the G.P. and the first term of the G.P. is equal to the
(C) γ ∈ ⎜ − ∞, ⎥ (D) γ ∈ ⎢ − , ∞⎟
⎝ 3⎦ ⎣ 27 ⎠ common difference of the A.P, then
108. If a, b, c are in A.P. and a2, b2, c2 arc in H.P. then (A)  first term of the G.P. is
2
,3
3
(A)  a=b=c
2
a (B)  first term of the A.P. is , 3
(B) – , b, c are in G.P. 3
2 25
c (C)  Common ratio of the G.P. is ,2
(C) – , b, a are in G.P. 2
2 2
(D)  Common difference of the A.P is , 3
a 3
(D) − , b, c are in H.P.
2 n+4

109. If the G.M. between a and b be twice the H.M., then


113. Let (1 + x2)2 (1+ x)n = ∑ ak x k . If a1,a2, a3, are in
k =0
a A.P., then n is equal to
is equal to
b
(A) 1 (B) 2
2+ 3 2− 3 (C) 3 (D) 4
(A)  (B) 
2− 3 2+ 3 114. If a, b, c are non-zero real numbers such that 3
(a2 +b2+ c2 + 1) = 2 (a + b + c + ab + bc + ca), then,
4+ 3 4− 3 a, b, c are in
(C)  (D) 
4− 3 4 + 3 (A)  A.P. (B)  G. P.
110. If a, b, c are in G.P. and x is the A.M. between a and (C) H.P. (D) all equal
b, y the A.M. between b and c, then 115. Let tn = 1
.1...1, then
n times
a c a c
(A)  + = 1 (B)  + = 2 (A) t912 is not prime
x y x y
(B) t951 is not prime
1 1 2 (C) t480 is not prime
(C)  + = (D)  None of these
x y b (D) t91 is not prime

Objective_Maths_JEE Main 2017_Ch 10.indd 28 01/01/2008 04:30:08


Sequence and Series  10.29

Passage Based Questions

Passage 1 119 Sum to infinite terms of the series


In finding the summation of a series, sometimes it is 1 1 1
+ + + .... is
­possible to express the nth term tn as a difference of two 1· 3 3 · 5 5 · 7
terms. Suppose, we want the sum of the series t1 + t2 + … + 1 1
tn. We try to express tn = xn + 1 – xn (A)  (B) 
4 3
hence, t1 + t2 + … + tn = (x2 – x1) + (x3 – x2) + … +
(xn + 1 – xn) = xn + 1 – x1 1
(C)  (D)  None of these
Thus, Sn = xn + 1 – x1. 2
At times, we may have to do partial fractions by trial by
Passage 2
writing the numerator as a difference of extreme
terms. A general arithmetic progression is a, a + d, a + 2d, … and
Suppose, we want to find the sum of the series a general geometric progression is a, ar, ar2, …, then the
sequence a, (a + d)r, (a + 2d)r2, … is called an arithmeti-
1 1 1
+ + + ... up to n terms. co-geometric progression (A. G.P.).
 1 · 2 · 3 2 · 3 · 4 3 · 4·5 Note that each term of the A.G.P. is the product of
1 1 ⎡ ( n + 2) − n ⎤ the corresponding terms of the A.P. a, a + d, a + 2d, …
Then, tn = = and the G.P. 1, r, r2, … . The nth term of the A.G.P. is
n ( n + 1) ( n + 2) 2 ⎢⎣ n ( n + 1) ( n + 2) ⎥⎦
[a + (n – 1)d]rn – 1.
1⎡ 1 1 ⎤ Sum to n terms of A.G.P.
= ⎢ − ⎥ Let Sn = a + (a + d)r + (a + 2d) r2 + …
2 ⎣ n ( n + 1) | n + 1) ( n + 2) ⎦

1⎡ 1 1 ⎤ ( ) (
+ a + n − 2 d r n − 2 + a + n − 1 d r n − 1  (1) )
We easily get Sn = ⎢ − ⎥
2 ⎣1· 2 ( + 1) ( + 2) ⎦
n n ⇒ rSn = ar + (a + d) r2 + … + | a + n −1 d ) r n (2)
Subtracting (2) from (1), we get
116. Sum to n terms of the series

1 +
1
+
1
+ .... is
 (
(1 – r) Sn = a + dr + dr2 + … + drn – 1 – a + n − 1 d r n )
1+ 2 1+ 2 + 3
 = a + ( dr + dr 2 + ... + to n − 1 terms) − ( a + n − 1 d ) r n
n 2n dr (1 − r n −1 )
(A)  (B)  = a+ − | a + n −1 d) rn
n +1 n +1 1− r
n a dr (1 − r n − 1 ) ( a + n − 1 d ) r n
(C)  (D)  None of these ∴ Sn = + −
n −1 1− r (1 − r ) 2 1− r

117. Sum to infinite terms of the series a dr


∴ Sum to infinite terms = S = lim Sn = + ,
where | r | < 1. n→∞ 1 − r (1 − r ) 2
1 1
1 + + + .... is
1+ 2 1+ 2 + 3
(A) 1 (B) 2 120. The sum to infinity of the series
(C)  4 (D)  None of these 3 5 7
1 + + 2 + 3 + ... is
1 1 1 2 2 2
118. Sum to n terms of the series + + + .... is
1· 3 3 · 5 5 · 7 (A) 4 (B) 6
(C)  8 (D)  None of these
n n
(A)  (B)  121. If the sum to infinity of the series
2n + 1 2n − 1
49
n −1 3 + 5r + 7r2 + … is , then r is equal to
(C)  (D)  None of these 9
2n + 1

Objective_Maths_JEE Main 2017_Ch 10.indd 29 01/01/2008 04:30:12


10.30  Chapter 10

124. The sequence {S2n} is


1 1
(A)  (B)  (A) increasing (B) decreasing
4 3
(C) non-monotonic (D) unbounded
1
(C)  (D)  None of these 125. The sequence {S2n + 1} is
2
(A) increasing (B) decreasing
122. If the sum to infinity of the series (C) non-monotonic (D) unbounded
1 1 44
3 + (3 + d ) + (3 + 2d ) 2 + ... is , then d = 126. S2n + 1 – S2n must be equal to
4 4 9
2 n −1 2 n −1
⎛ 1⎞ ⎛ 1⎞
(A) 1 (B) 2 (A) ⎜ ⎟
⎝ 2⎠
(B) 
⎜⎝ 2 ⎟⎠ ( S1 − S2 )
(C)  4 (D)  None of these
2 n −1
123. 31/3 · 91/9 · 271/27 · 811/81… upto ∞ =
3
(C) ⎛⎜ 1 ⎞⎟ ( S2 − S1 ) (D) Zero
(A)  27 (B)  27 ⎝ 2⎠
(C) 4 27 (D)  None of these
127. If S1 > S2, then lim Sn must be equal to
n→∞
Passage 3 (A) S1 – S2 (B) 
S1 + 2S2
We know that arithmetic mean of the positive numbers lie S + 2 S2
1 (C)  1 (D)  None of these
between them. Suppose, S1 >S2 > 0 and Sn + 1 = ( Sn + Sn −1 ) 3
2
We can easily conclude that S3 lies between S2 and S1 and
we can write S2 < S3 < S1.

Match the Column Type

128.
Column-I Column-II
I. Let Sn denotes the sum of n terms of an A.P. whose first term is a. (A) 29
If the common difference d = Sn – k Sn–1 + Sn – 2, then k =
 II. The minimum number of terms from the beginning of the series (B) 4
2 1
20 + 22 + 25 + …, so that the sum may exceed 1568, is
3 3

a
III. If 51 + x + 51 – x, and 25x + 25–x are three consecutive terms of an (C) 2
2
A.P., then a ≥ k, where k =
  IV. If log 2 a + log 2 a + log2 a + log2 a + … upto 20 terms is 840,
1/2 1/4 1/6 18
/ (D) 12
then a is equal to…

129.
Column-I Column-II
2
 I. If the first term of an infinite G.P. is 1 and each term is twice the sum (A) 
of the suceeding terms, then the common ratio is 9
2 5 2 11 3
II. Sum to infinity of the series − + − + ... is (B) 
3 6 3 24 2

Objective_Maths_JEE Main 2017_Ch 10.indd 30 01/01/2008 04:30:15


Sequence and Series  10.31

III.  lim (1 + 3−1 ) (1 + 3−2 ) (1 + 3−4 ) (1 + 3−8 ) ... (1 + 3−2 ) =


n

(C) 1
n→∞

n ⎛ k ⎞ 1
  IV. If ∑ ⎜ ∑ m2 ⎟ = an4 + bn3 + cn2 + dn + e, then a + b + c + d + e = (D) 
k =1 ⎝ m = 1 ⎠ 3

130.
Column-I Column-II
 I. If a, b, c are in A.P., b, c, d are in G.P. and c, d, e are in H.P., then (A) A.P.
a, c, e are in
  II. If 2(y – a) is the H.M. between y – x, y – z then x – a, y – a, z – a (B) G.P.
are in
III. If three numbers are in H.P., then the numbers obtained by subtract- (C) H.P.
ing half of the middle number from each of them are in
  IV. If a, b, c are in G.P., then the equations ax2 + 2bx + c = 0 and dx2 + (D) A.G.P.
d e f
2ex + f = 0 have a common root, if , and are in
a b c

Assertion-Reason Type
Instructions: In the following questions an Assertion (A) is Reason: H.M. > A.M. for unequal numbers
given followed by a Reason (R). Mark your responses from
134. Assertion: The sum of the series
the following options:
(A)  Assertion(A) is True and Reason(R) is 1 2 3
2 4
+ 2 4
+ + ...
True; Reason(R) is a correct explanation for 1 +1 + 1 1+ 2 + 2 1 + 3 + 34
2

Assertion(A) n( n + 1)
(B)  Assertion(A) is True, Reason(R) is True; to n terms is
2( n2 + n + 1)
Reason(R) is not a correct explanation for
Assertion(A) Reason: The nth term of the above series is
(C) Assertion(A) is True, Reason(R) is False 1⎡ 1 1 ⎤
Tn = ⎢ − ⎥
(D) Assertion(A) is False, Reason(R) is True 2 ⎣1 + ( n − 1) n 1 + n ( n + 1) ⎦
1
131. Assertion: Between two numbers whose sum is, 2 135. Assertion: The value of x + y + z is 15 if a, x, y, z,
6
an even number of arithmetic means are inserted. 1 1 1 5
b are in A.P., while the value of + + is if
If the sum of these means exceeds their number by x y z 3
unity, then the number of means are 12 a, x, y, z, b are in H.P. The values of a and b are 9, 1
Reason: If a and b are two given numbers and A1, A2, respectively.
…, An are n arithmetic means between them, then Reason: The sum of n A.M.s between two quantities
a + b⎞ is equal to n times their single mean.
A1 + A2, …, An = n ⎛⎜
⎝ 2 ⎟⎠ 136. Assertion: For every natural number
132. Assertion: If a, b, c are distinct positive real numbers 2n
and a2 + b2 + c2 = 1, then ab + bc + ca is less than 1. n, (n !)3 < nn ⎛⎜ n + 1⎞⎟
⎝ 2 ⎠
Reason: A.M. > G.M. for unequal numbers
Reason: A.M > G.M. for n distinct positive quantities
133. Assertion: If a, b, c, d ∈ R+ and a, b, c, d are in H.P.,
then b + c > a + d

Objective_Maths_JEE Main 2017_Ch 10.indd 31 01/01/2008 04:30:16


10.32  Chapter 10

Previous Year’s Questions

144. Let Tr be the r th term of an A.P. whose first term is


137. If 1, log3 (31− x + 2) , log3 (4 ⋅ 3x− l) are in AP, then
a and common difference is d. If for some positive
x equals: [2002]
1 1
(A) log3 4 (B)  l − log3 4 integers m, n, m ≠ n, Tm = and Tn = , then a − d,
equals n m [2004]
(C) 1 − log, 3 (D)  log4 3
(A) 0 (B) 1
138. The value of 21/4⋅ 41/8⋅ 81/16 . . . ∞ is: [2002] 1 1 1
(C)  (D)  +
(A) 1 (B) 2 mn m n
(C) 3/2 (D) 4
145. The sum of the first n terms of the series 12 + 2 ⋅ 22
139. Fifth term of a GP is 2, then the product of its 9 terms
is : [2002] n( n +1) 2
+ 32 + 2 ⋅ 42 + 52 + 2 ⋅ 62 + ... is when n is
2
(A) 256 (B) 512 even. When n is odd the sum is [2004]
(C)  1024 (D)  None of these
3n( n + 1) n2 ( n + 1)
140. Let Tn denote the number of triangles which can be (A)  (B) 
2 2
formed using the vertices of a regular polygon of n 2
2
sides. If Tn+1 ⋅ = Tn = 21, then n equals : [2002] n( n +1) ⎡ n( n + 1) ⎤
(C)  (D)  ⎢ ⎥
(A) 5 (B) 7 4 ⎣ 2 ⎦
(C) 6 (D) 4 ∞ ∞ ∞
146. If x = ∑ an , y= ∑ bn , z = ∑ cn where a, b, c are
1 1 1 n= 0 n= 0 n= 0
1 41. The sum of the series − + − ……… upto
1.2 2.3 3.4 in A.P. and |a| < 1, |b|< 1, |c|< 1, then x, y, z are in
∞ is equal to [2003]  [2005]
(A)  2 loge 2 (B)  log2 2 − 1 (A) G.P.
(B) A.P.
loge ⎛ 4 ⎞
(C) loge 2 (D)  (C) Arithmetic − Geometric Progression
⎜⎝ ⎟⎠
e (D) H.P.

142. If f : R → R satisfies f (x + y) = f (x) + f (y), for all x, y a1 + a2 + ...a p p 2


1 47. Let a1, a2, a3, … be terms of an A.P. If = , p ≠ q,
n a1 + a2 ... + aq q 2
∈ R and f (1) = 7, then ∑ f (r ) is a1 + a2 + ...a p p
[2003] 2
a
= 2 , p ≠ q, then 6 equals [2006]

r =1 a1 + a2 ... + aq q a21
7n 7( n + 1)
(A)  (B)  41 7
2 2 (A)  (B) 
11 2
7n( n + 1) 2 11
(C) 7n( n + 1) (D)  (C)  (D) 
2 7 41
n
1 n
r tn 148. If a1, a2, … , an are in H.P., then the expression a1 a2 +
143. If Sn = ∑ nC and t n = ∑ nC , then
Sn
is equal to a2 a3 + … + an-1 an is equal to [2006]
r =0 r r =0 r
 [2004] (A) n(a1− an) (B) (n − 1) (a1−an)
1 1 (C) na1an (D) (n − 1)a1an
(A)  n (B)  n −1
2 2 149. In a geometric progression consisting of positive
terms, each term equals the sum of the next two terms.
2n − 1
(C) n − 1 (D)  Then the common ratio of this progression equals
2 [2007]

Objective_Maths_JEE Main 2017_Ch 10.indd 32 01/01/2008 04:30:21


Sequence and Series  10.33

(C) Statement 1 is true, statement 2 is true; statement


1 1
(A)  (1 − 5 ) (B)  5 2 is not a correct explanation for statement 1
2 2 (D)  Statement 1 is true, statement 2 is false
1 156. If 100 times the 100th term of an Arithmetic
(C)  5 (D)  ( 5 − 1)
2 Progression with non zero common difference equals
150. If p and q are positive real numbers such that p2 + q2 the 50 times its 50th term, then the 150th term of this
= 1, then the maximum value of (p + q) is [2007] A.P. is [2012]
(A) 2 (B) 1/2 (A) –150
1 (B)  150 times its 50th term
(C)  (D)  2 (C) 150
2 (D) zero
151. The first two terms of a geometric progression add
157. The sum of first 20 terms of the sequence 0.7, 0.77,
up to 12. The sum of the third and the fourth terms
0.777, . . . is [2013]
is 48. If the terms of the geometric progression are
alternately positive and negative, then the first term is
 [2008]
7
(
(A)  99 − 10 −20
9
) (B) 
7
81
(179 + 10 −20 )
(A) −4 (B)  −12
(C) 12 (D) 4
7
9
(
(C)  99 + 10 −20

) (D) 
7
81
(179 − 10 −20 )
152. The sum to the infinity of the series
158. Let a and b be the roots of equation
2 6 10 14
1 + + 2 + 3 + 4 + ....... is [2009] px 2 + qx + r − 0, p ≠ 0 . If p, q, r are in A.P. and
3 3 3 3
1 1
(A) 2 (B) 3 + = 4 , then the value of a − b is [2014]
(C) 4 (D) 6
a b

153. A person is to count 4500 currency notes. Let an 61 2 17


(A)  (B) 
denote the number of notes he counts in the nth min- 9 9
ute. If a1 = a2 = ...... = a10 = 150 and a10, a11 . . . are in 34 2 13
A.P. with common difference −2, then the time taken (C)  (D) 
9 9
by him to count all notes is [2010]
(A)  34 minutes (B)  125 minutes 159. Three positive numbers form an increasing G.P. If
(C)  135 minutes (D)  24 minutes the middle term in this G.P. is doubled, the new num-
bers are in A.P. Then the common ratio of the G.P. is
154. A man saves Rs. 200 in each of the first three months  [2014]
of his service. In each of the subsequent months his (A)  2 + 3 (B)  3 + 2
saving increases by Rs. 40 more than the saving of
(C) 2 − 3 2+ 3
(D) 
immediate preceding month. His total saving from
the start of service will be Rs. 11040 after [2011] 160. If (10)9 + 2(11)1 (10)8 + 3(11) 2 (10)7 + ..... + 10(11)9,
(A)  19 months (B)  20 months
(C)  21 months (D)  18 months = k(10)9 then k is equal to [2014]
121 441
155. Statement 1: The sum of the series 1 + (1 + 2 + 4) + (A)  (B) 
(4 + 6 + 9) + (9 + 12 + 16) + ...... + (361 + 380 + 400) 10 100
is 8000. (C) 100 (D) 110
n
Statement 2: ∑ ( k 3 = ( k − 1)3 ) = n3 for any natural 161. The sum of first 9 terms of the series
k =1 3 3 3 3 3 3
number n. [2012] 1 1 +2 1 +2 +3
+ + + ...... is: [2015]
1 1+ 3 1+ 3 + 5
(A)  Statement 1 is false, statement 2 is true
(B) Statement 1 is true, statement 2 is true; state- (A) 96 (B) 142
ment 2 is a correct explanation for statement 1 (C) 192 (D) 71

Objective_Maths_JEE Main 2017_Ch 10.indd 33 01/01/2008 04:30:24


10.34  Chapter 10

162. If the 2nd, 5th and 9th terms of a non-constant A.P. are 163. If the sum of the first terms of the series
in G.P., then the common ratio of this G.P. is 2 2 2 2
⎛ 3⎞ ⎛ 2⎞ ⎛ 1⎞ 2 ⎛ 4⎞ 16
 [2016] ⎜⎝1 ⎟⎠ + ⎜⎝ 2 ⎟⎠ + ⎜⎝ 3 ⎟⎠ + 4 + ⎜⎝ 4 ⎟⎠ +…, is m,
5 5 5 5 5
7 8 then m is equal to [2016]
(A)  (B) 
4 5 (A) 99 (B) 102
4 (C) 101 (D) 100
(C)  (D) 
1
3

Answer keys

Single Option Correct Type


  1. (B)  2.  (C) 3. (C) 4.  (B) 5. (D) 6.  (C) 7.  (A) 8.  (D) 9.  (A) 10. (B)
  11.  (A) 12.  (D) 13.  (D) 14.  (D) 15.  (B) 16.  (C) 17.  (B) 18.  (A) 19.  (A)   20.  (C)
  21.  (B) 22.  (C) 23.  (B) 24.  (C) 25.  (C) 26.  (B) 27.  (C) 28.  (A, B) 29.  (B)   30.  (B)
  31.  (B) 32.  (B) 33.  (A) 34.  (B) 35.  (D) 36.  (A) 37.  (B) 38.  (C) 39.  (B) 40.  (D)
  41.  (A) 42.  (B) 43.  (D) 44.  (B) 45.  (C) 46.  (B) 47.  (B) 48.  (C) 49.  (D) 50.  (C)
  51.  (C) 52.  (A) 53.  (A) 54.  (A) 55.  (C) 56.  (A) 57.  (D) 58.  (C) 59.  (C) 60.  (C)
  61.  (A) 62.  (B) 63.  (B) 64.  (A) 65.  (D) 66.  (B) 67.  (A) 68.  (B) 69.  (C) 70.  (D)
  71.  (D) 72.  (C) 73.  (C) 74.  (A) 75.  (A) 76.  (B) 77.  (C) 78.  (C) 79.  (C) 80.  (C)
  81.  (B) 82.  (A) 83.  (B) 84.  (C) 85.  (B) 86.  (A) 87.  (A) 88.  (B) 89.  (D) 90.  (C)
  91.  (A) 92.  (B) 93.  (C) 94.  (B) 95.  (D) 96.  (C) 97.  (B) 98.  (C) 99.  (B) 100.  (C)
101.  (A) 102.  (B) 103.  (A) 104.  (C)

More than One Option Correct Type


105.  (C, D) 106.  (B, D) 107.  (A, D) 108.  (A, B, C) 109.  (A, B)
1 10.  (B, C) 111.  (A, B) 112.  (A, C, D) 113.  (B, C, D) 114. (a, b, c, d)
115.  (a, b, c, d)

Passage Based Questions


116.  (B) 117. (B) 118. (A) 119. (C) 120. (B) 121.  (A) 122. (B) 123.  (C) 124. (A) 125.  (B)
1 26. (B) 127.  (C)

Match the Column Type


128.  I → (C), II → (A), III → (D), IV → (B) 129. I → (D), II → (A), III → (B), IV → (C)
1 30.  I → (B), II → (B), III → (B), IV → (A)

Assertion-Reason Type
131. (A) 132. (A) 133. (A) 134. (A) 135. (A) 136.  (A)

Previous Year’s Questions


137.  (B) 138.  (B) 139.  (B) 140.  (B) 141.  (D) 142.  (D) 143.  (A) 144.  (A) 145.  (D) 146.  (D)
1 47.  (D) 148.  (D) 149.  (D) 150.  (D) 151.  (B) 152.  (B) 153.  (A) 154.  (C) 155.  (B) 156.  (D)
157.  (B) 158.  (D) 159.  (D) 160.  (C) 161.  (A) 162.  (C) 163.  (C)

Objective_Maths_JEE Main 2017_Ch 10.indd 34 01/01/2008 04:30:25


Sequence and Series  10.35

Hints and Solutions

Single Option Correct Type


1. Given a + c = 2b
⎡ 9⎤
a+b+c ⇒
r ∉ ⎢1, ⎥
Also, ≥ 3 abc = 3 64 = 4 ⎣ 5⎦
3

The correct option is (D)
3b
⇒ ≥4 1 2 3
3 6.
Sn = (9) + (99) + (999) + …
9 9 9
⇒ b≥4
1 1
∴ Minimum b=4
= [10 + 2.10 + 3.103 + …] – [1 + 2 + 3 + …]
2
9 9
The correct option is (B)
2. Let sides of triangle be a, ar, ar2. 1 1 n( n + 1)

S− =
Since r > 1, ∴ ar2 is greatest side 9 9 2
∴ a + ar > ar2 ⇒ r2 – r – 1 < 0
S = 10 + 2.102 + 3.103 + … + n 10n
⇒ 10S = 102 + 2.103 + … + (n – 1)10n + n.10n+1

1− 5 1+ 5

<r< ⇒ 1 < r < 1+ 5

–9S = (10 + 102 + 103 + … + 10n) – n.10n+1
2 2 2 n +1
∴ (r) = 1.

⇒ S = n 10 n +1 − 10 − 1
9 81
Also, – 1 + 5 < – r < –1
n +1
2 ∴
Sn = n 10 n +1 − 10 − 1 − 1 n( n + 1)
∴ (– r) = –2
81 9.81 9 2
∴ (r) + (–r) = 1 – 2 = –1
n +1


The correct option is (C) ⇒ 9.Sn = (9n − 1)10 + 1 − n( n + 1)

81 81 2
21
21 10 n
3. ∑ aj = 693 =
2
(a1 + a21) ∴ 9(Sn – Sn–1) =

81
[10(9n – 1) – (9n – 10)]– n
j =1
= n (10n – 1)

a1 + a21 = 66 The correct option is (C)
693

Now, a11 = A.M. = = 33
21

Also, a2 + a20 = a3 + a19 = … = a9 + a13 = a10 + a12 7. log 5
x + log5 x + log5 x + … upto 7 terms
1/3 1/4

10
log x log x log x

∑ a2i +1 = 5 × (a1 + a21) + a11
=
log 5
+
log 5 1/3
+
log 51/4
+ … upto 7 terms
i=0


= 5 × 66 + 33 = 363 log x

= [2 + 3 + 4 + … + 8]
The correct option is (C) log 5
4. Let a, ar, ar2 be any three consecutive terms, then according log x ⎡ 7 ⎤ log x
to the given condition, a, 2ar, ar2 are in A.P.
= ( 2 + 8) ⎥ = (35) = 35 (given)
log 5 ⎢⎣ 2 ⎦ log 5
i.e. 4ar = a + ar2

or r=2± 3 log x

=1
Since, given G.P. is increasing, r = 2 + . 3 Hence, there is log 5
only one such G.P. ⇒ log x = log 5

The correct option is (B) ⇒
x=5
5. Given a1 > 0 and a1, a2, a3 are in G.P.
The correct option is (A)
such that a2 = a1r and a3 = a1r2 ∞
1

∴ 9a1 + 5a3 > 14a2 8. Given a = ∑ x n −1 = 1 + x + x2 + … =
n =1 1− x
⇒ 9a1 + 5a1r2 > 14a1r
and since a1 > 0, we get 9 + 5r2 > 14r a −1
2

x=
⇒ 5r – 14r + 9 > 0 ⇒ (5r – 9) (r – 1) > 0 a

Objective_Maths_JEE Main 2017_Ch 10.indd 35 01/01/2008 04:30:29


10.36  Chapter 10

b −1 ∞ 11. a + b + c = xb
Similarly y =

b
∴ ∑ ( xy)n−1 = 1 + (xy) + (xy)2 + … Divide by b,
n =1
a c
1
+1+ =x
1 b b

= = ⎛ a − 1⎞ ⎛ b − 1⎞ 1
1 − xy 1− ⎜
⎝ a ⎠⎟ ⎝⎜ b ⎠⎟ ⇒

r
+ 1 + r = x, r is the common ratio of the G.P.
2
ab ab ⇒ r + r (1 – x) + 1 = 0. Since r is real, therefore discrimi-


= = nent > 0
ab − ( ab − ( a + b) + 1) a + b −1
⇒ (1 – x)2 – 4 > 0 ⇒ x2 – 2x + 1 – 4 > 0


The correct option is (D) ⇒ x2 – 2x – 3 > 0 ⇒ (x + 1) (x – 3) > 0

⇒ x < –1 or x > 3

9. 27pqr ≥ (p + q + r)3

The correct option is (A)
p +q +r
⇒ (pqr)1/3 ≥ ⇒p=q=r 100
12. Given: [2a + 99d] = x
3 2
Also, 3p + 4q + 5r = 12
100
and
[2(a + 2d) + 99d] = y

p=q=r=1 2

The correct option is (A) On subtraction, 200d = y – x

y−x
10. Let Tn be the nth term of the series ⇒d=

200
1 2 3
The correct option is (D)
2 4 + + +…
1 + 1 + 1 1 + 22 + 24 1 + 32 + 34 1 1 1 1
13. Given: … + + + =λ
14 24 34 44
n n
Then, Tn =
= ∞
1 1 1 1
1+ n + n2 4
(1 + n ) − n2
2 2

∑ ( 2i − 1)4 =
14
+
34
+
54
+ ...
i =1
n 1 1 1 1 ⎡1 1 ⎤

= =
+ + + + ... ∞ − ⎢ 4 + 4 + ... ∞ ⎥
( n2 + n + 1) ( n2 − n + 1) 14 24 34 44 ⎣2 4 ⎦
1⎛ 1 1 ⎞ 1 ⎡1 1 1 ⎤

= − 2 = λ−
+ 4 + 4 + ... ∞ ⎥
2 n − n + 1 n + n + 1⎟⎠

⎝ 2 4 ⎢ 4
2 ⎣1 2 3 ⎦
1 15
1⎡ 1 1 ⎤ =λ–
λ= λ

= ⎢ − ⎥ 16 16
2 ⎣1 + ( n − 1)n 1 + n( n + 1) ⎦

The correct option is (D)

Now
14. Let A = 1.2 + 1.3 + … + 2.3 + 2.4 + … + (n – 1) · n
n
1 ⎡1 1 ⎤ 1⎡ 1 1 ⎤ Now, (1 + 2 + 3 + … + n)2 – (12 + 22 + 32 + … + n2) = 2A

∑ Tr = 2 ⎢⎣1 − 1 + 1.2 ⎥⎦ + 2 ⎢⎣1 + 1.2 − 1 + 2.3 ⎥⎦
r =1
1 ⎡ n2 ( n + 1) 2 n( n + 1) ( 2n + 1) ⎤
⇒ A =
⎢ − ⎥
1⎡ 1 1 ⎤ 2 ⎢⎣ 4 6 ⎥⎦

+ − +…
2 ⎢⎣1 + 2.3 1 + 3.4 ⎥⎦
1

[(Sn)2 – Sn2]
=
1⎡ 1 1 ⎤ 2

+ ⎢ − ⎥
2 ⎣1 + ( n − 1)n 1 + n( n + 1) ⎦
The correct option is (D)

15. A.M. ≥ G.M.


1⎡ 1 ⎤

= ⎢1 − ⎥ ⎛ x π⎞
2 ⎣ 1 + n( n + 1) ⎦ 23 sin x/8 + 23 cos x/8 3 2 cos ⎜ − ⎟
⎝ 8 4⎠

≥ 2
2
n( n + 1)

= . ⎛ x π⎞
2( n2 + n + 1) 3 2 cos ⎜ − ⎟
⎝ 8 4⎠

Now maximum of 2 = 23 2 .1
= 23/ 2

1 3
Trick: Checking for n = 1, 2. S1 = and S2 = which are x x ⎛ 3 ⎞
+1⎟
3 7 sin cos ⎜
given by (b). So, A.M. ≥ 23/
2
⇒ 8 8 +8 8 ≥ 2⎝ 2 ⎠


The correct option is (B)

The correct option is (B)

Objective_Maths_JEE Main 2017_Ch 10.indd 36 01/01/2008 04:30:34


Sequence and Series  10.37

16. log 2 a + log 2 a + log2 a + log 2 a + …


1/2 1/4 1/6 18
/
(1) and (2)
= 2 log2 a + 4 log2 a + 6 log2 a + … + 40 log2 a S 2

= 1 + ⎛⎜1 − 1 ⎞⎟ + ⎛⎜1 − 1 ⎞⎟ + … ∞
= log2 a [2 + 4 + 6 + … + 40] n ⎝ n⎠ ⎝ n⎠
20

=
(2 + 40) log2 a 1
2

= ⎛ 1⎞ = n
= 420 log2 a = 840 (Given) 1 − ⎜1 − ⎟
⎝ n⎠
⇒ log2 a = 2 ⇒ a = 4
The correct option is (C) ⇒ S = n2
The correct option is (C)
17. If tr denotes the rth term of the series, then
21. In this type of A.P. it can be easily shown that exactly one out
x of any 3 consecutive terms will be multiple of 3. So at most
x tr =
(1 + rx ) (1 + ( r + 1) x ) 3 consecutive terms can be prime numbers.
1 1 The correct option is (B)

= − 22. The coefficient of x49 = – [1 + 3 + 5 + … + 99]
1 + rx 1 + ( r + 1) x
= – 2500
n
n
⎡ 1 1 ⎤
⇒ x ∑ t r =
∑ ⎢1 + rx − 1 + ( r + 1) x ⎥ The correct option is (C)
r =1 r =1 ⎣ ⎦ 23. A.M. ≥ G.M.
1/3
1 1 ⎛ x y z⎞ ⎛ ⎞

= − ⇒ ⎜ + + ⎟ 3 ≥ x⋅ y⋅z

1 + x 1 + ( n + 1) x ⎝ y z x⎠ ⎜⎝ y z x ⎟⎠
nx

= ⎛ x y z⎞
(1 + x ) (1 + ( n + 1) x ) ⇒ ⎜ + + ⎟ ≥3

⎝ y z x⎠
n
n

∑ tr =
(1 + x ) [1 + ( n + 1) x ]

The correct option is (B)
r =1 24. Let the G.P. be a – ar + ar2 – ar3 + … with common ratio

The correct option is (B) =–r
18. Since a and b are unequal, By the given condition
2
2 2
a + b > a 2b 2 [A.M. > G.M. for unequal numbers]

a = ar − ar ⇒ 2a = ar2 – ar
2 2

a2 + b2 > 2ab ⇒ 2 = r – r
2

Similarly,
b2 + c2 > 2bc and c2 + a2 > 2ca ⇒
r2 – r – 2 = 0
Hence, 2 (a + b2 + c2) > 2 (ab + bc + ca)
2
⇒ (r – 2) (r + 1) = 0


ab + bc + ca < 1 ⇒
r = 2, –1

The correct option is (A) ∴ Common ratio = –2 or 1

n Hence, common ratio = –2
(∵ common ratio is –ve)
19. The given series = ∑ ( n − 2r ) 2
The correct option is (C)
r =1

n 1 1 1 A −1
25. A = ⇒ 1 − ra = ⇒ ra = 1 − =

= ∑ ( n2 − 4nr + 4r 2 ) 1− r a
A 4 A
1

n ( n + 1) n 1 1 1 B −1

= n · n2 – 4n · + 4 · (n + 1) (2n + 1) B= ⇒ 1 − rb = ⇒ rb = 1 − =
2 6 1 − rb B B B
n 2 ⎛ A −1 ⎞ ⎛ B −1 ⎞

(n + 2) =
\ a log r = log ⎜ and b log r = log ⎜
3 ⎟ ⎟
⎝ A ⎠ ⎝ B ⎠

The correct option is (A)
⎛ A −1 ⎞
2 log ⎜ ⎟
20. Let S = 1 + 2 ⎛⎜1 − 1 ⎞⎟ + 3 ⎛⎜1 − 1 ⎞⎟ +…(1) a
\ =
⎝ A ⎠ ⎛ A −1 ⎞
= log B −1 ⎜ .
⎝ n⎠ ⎝ n⎠ b ⎛ B −1 ⎞ B ⎝
A ⎟⎠
log ⎜ ⎟
⎛ 1⎞ ⎛ 1⎞ ⎛ 1⎞
2
⎝ B ⎠
∴ ⎜1 − ⎟ S =
⎜⎝1 − n ⎟⎠ + 2 ⎜⎝1 − n ⎟⎠ + … (2)
⎝ n⎠
The correct option is (C)

Objective_Maths_JEE Main 2017_Ch 10.indd 37 01/01/2008 04:30:39


10.38  Chapter 10

26. If tr be the rth term of the A.P., then 2



q2 – 4pr ≥ 0 ⇒ ⎛⎜ p + r ⎞⎟ - 4pr ≥ 0
tr = Sr – Sr–1 ⎝ 2 ⎠
= cr (r – 1) – c (r – 1) (r – 2)
= c (r – 1) (r – r + 2) = 2c (r – 1) p2 p

p2 + r2 - 14pr ≥ 0 ⇒ - 14 +1≥0
2 r
r
We have, t12 + t 22 + … + tn2

2
⎛p ⎞ p


= 4c2[02 + 12 + 22 + … + (n – 1)2] ⇒ ⎜
− 7⎟ - 48 ≥ 0 ⇒ −7 ≥ 4 3
⎝ r ⎠ r
( n − 1) n (2 n − 1)
The correct option is (B)

= 4c2
6
1 5 19 65
30. + + + + … to n terms

2 2 3 9 27 81
= c n (n – 1) (2n – 1)
3 ⎛ 2⎞ ⎛ 4⎞ ⎛ 8⎞ ⎛ 16 ⎞

The correct option is (B) = ⎜1 − ⎟ + ⎜1 − ⎟ + ⎜1 − ⎟ + ⎜1 − ⎟ + …

⎝ 3 ⎠ ⎝ 9 ⎠ ⎝ 27 ⎠ ⎝ 81⎠
n
27. Sn = [2a + (n – 1) d] = pn2(1)
2 ⎡ 2 ⎛ 2⎞ ⎤
2 2
=n–
⎢1 + + ⎜ ⎟ + ... to n terms ⎥
Sm =

m
[2a + (m – 1) d] = pm2 3 ⎢⎣ 3 ⎝ 3 ⎠ ⎦⎥
2
n
⎛ 2⎞
2a + ( n − 1) d 1− ⎜ ⎟

⇒ =
n
=n– ·
2 ⎝ 3 ⎠ = n – 2 (3n – 2n)
2a + ( m − 1) d m 3 1− 2 3n
3
⇒ 2am + (n – 1) md = 2an + n (m – 1) d


The correct option is (B)
⇒ 2a (m – n) + d (mn – m – nm + n) = 0

⇒ 2a (m – n) + d (n – m) = 0
( r − 1)! r!
⇒ 2a – d = 0
31. We have, tr = and tr+1 =
( r + 4)! ( r + 5)!
∴ d = 2a

r! r!
∴ (1) gives, 2a + (n – 1) 2a = 2pn

Now, rtr – (r + 5)tr+1 = – =0
( r + 4)! ( r + 4)!
⇒ 2an = 2pn ⇒ a = p ∴ d = 2p

p ⇒
rtr – (r + 1)tr+1 = 4tr+1
Now,
Sp = · [2a + (p – 1) d]
2 n −1 n −1

4 ∑ t r +1 = ∑ [rtr − ( r + 1) tr +1]
p r =1

= · [2p + (p – 1) 2p] r =1
2 ⇒ 4(t2 + t3 + … + tn) = 1t1 - ntn

p

= · [2p + 2p2 – 2p]
⇒ 4 (t1 + t2 + … + tn) = 5t1 - ntn = 5 ⎜
⎛ 0 !⎞ n( n − 1)!

5!⎟⎠ ( n + 4)!
2

p . 2 3

= n 2p = p
2 1 n!

= −

The correct option is (C)
4 ! ( n + 4)!


28. We have, b3 > 4b2 – 3b1 1⎡1 n! ⎤
⇒ t1 + t2 + … + tn =

⇒ b1r2 > 4b1r – 3b1 4 ⎣ 4 ! ( n + 4)! ⎥⎦

⇒ r2 > 4r – 3 (∵ b1 > 0)

The correct option is (B)
⇒ r2 – 4r + 3 > 0 ⇒ (r – 3) (r – 1) > 0
⇒ r > 3 or r < 1 32. We have, (a2 + b2 + c2)p2 – 2p(ab + bc + cd) +
The correct option is (A, B) (b2 + c2 + d2) ≤ 0
29. Since p, q, r are in A.P. so 2q = p + r . The roots of the equa- ⇒ (ap – b)2 + (bp – c)2 + (cp – d)2 ≤ 0
tion px2 + qx + r = 0 are real if and only if Therefore,

Objective_Maths_JEE Main 2017_Ch 10.indd 38 01/01/2008 04:30:43


Sequence and Series  10.39

ap − b = 0 bp − c = 0 cp − d = 0 1 2 1 1 1


= ⎛⎜ 1 − d 2 ⎞⎟ ⇒ − d 2 = ± ⇒ d = ± .
b = ap ⇒ c = bp ⇒ d = cp 16 ⎝ 4 ⎠ 4 4 2
1
b c d ∴
d= (∵ d > 0)

= = =p 2
a b c
1 1
∴ a, b, c, d are in G.P.

a=b–d= −
2 2

The correct option is (B)

33. Taking A.M. and G.M. of 7 numbers The correct option is (D)
36. As a1, a2, a3, … an–1, an are in A.P.,
a a b b b c c d = a2 - a1 = a3 - a2 = … = an - an–1
, , , , , , , we get
2 2 3 3 3 2 2 sin d [sec a1 sec a2 + sec a2 sec a3 + .. + sec an–1 sec an]

1
2⋅
a b
+ 3⋅ + 2 ⋅
c sin (a2 − a1 ) sin (a3 − a2 ) sin (an − an −1 )
⎡⎛ a ⎞ 2 ⎛ b ⎞ 3 ⎛ c ⎞ 2 ⎤ 7 =
+ + .. +
2 3 2 ≥ ⎢

⎜ ⎟ ⎜ ⎟ ⎜ ⎟ ⎥ cos a1 cos a2 cos a2 cos a3 cos an −1 cos an
7 ⎢⎝ 2 ⎠ ⎝ 3 ⎠ ⎝ 2 ⎠ ⎥
⎣ ⎦ = (tan a2 – tan a1) + (tan a3 – tan a2)

1
 + … + (tan an – tan an–1)
3 ⎛ a2b 3c 2 ⎞ 7 37 a2b 3c 2 = tan an – tan a1


≥ ⇒ ≥
7 ⎜⎝ 22 3322 ⎟⎠ 77 22 ⋅ 33 ⋅ 22
The correct option is (A)
n 2S
310 ⋅ 2 4 37. We have, S = (a + l) ⇒ = n (1)
⇒ a2 b3 c2 ≤
2 a+l
77
l−a
310 ⋅ 2 4
Also, l = a + (n – 1) d ⇒ d =
∴ greatest value of a2 b3 c2 =
. n −1
77

The correct option is (A) l −a

= 2S [Using (1)]
34. Expanding along R3, we get −1
a+l
b aa − b a aa − b

2 − +0=0 l 2 − a2
c ba − c b ba − c
=
2S − ( l + a)
⇒ 2 (b2α - bc - acα + bc) - (abα - ac - abα + b2) = 0
∴ k = 2S.

2 2
⇒ 2α (b - ac) - (b - ac) = 0

The correct option is (B)
or (b2 - ac) (2α - 1) = 0
38. As a, b, c, d are in G.P., therefore

or b2 - ac = 0 [Q (2α – 1) ≠ 0] b c d

= = = r (say)

b2 = ac a b c
∴ a, b, c are in G.P.

⇒ b = ar, c = br = ar ⋅ r = ar2,


The correct option is (B)
d = cr = ar2 ⋅ r = ar3. (a2 + b2 + c2) (b2 + c2 + d 2)
35. Let a = b – d and c = b + d,
= (a2 + a2 r2 + a2 r4) (a2 r2 + a2 r4 + a2 r6)
3 1
= a4 r2 (1 + r2 + r4) (1 + r2 + r4)
then a + b + c =
⇒β= .
2 2
= (a2 r + a2 r3 + a2 r5)2
1 1 1

= (a ⋅ ar + ar ⋅ ar2 + ar2 ⋅ ar3)2
−d, , +d
Therefore, the number are
2 2 2

= (ab + bc + cd)2.
(d > 0 as a < b < c)

The correct option is (C)

Now a2, b2, c2 are in G.P. ⇒ (b2)2 = a2c2


39. Let the two numbers be a and b,then

⎛ 1⎞
4 2 2 G = ab or G2 = ab


⇒ ⎜ ⎟ = ⎛⎜ 1 − d ⎞⎟ ⎛⎜ 1 + d ⎞⎟ Also, p and q are two A.M.s between a and b.
⎝ 2⎠ ⎝2 ⎠ ⎝2 ⎠
∴ a, p, q, b are in A.P.

Objective_Maths_JEE Main 2017_Ch 10.indd 39 01/01/2008 04:30:49


10.40  Chapter 10

∴ p – a = q – p and q – p = b – q
1 3 7 15
∴ a = 2p – q and b = 2q – p 45. Sn = + + + + … upto n terms
2 4 8 16
∴ G2 = ab = (2p – q) (2q – p).
The correct option is (B)
⇒ Sn = ⎛⎜1 − ⎞⎟ + ⎛⎜1 − ⎞⎟ + ⎛⎜1 − ⎞⎟ + ...
1 1 1

40. Given x1 · x2 …xn = 1 ⎝ 2⎠ ⎝ 4⎠ ⎝ 8⎠
Since A.M. ≥ G.M. 1⎛ 1⎞
⎛ x + x 2 + ... + x n ⎞ ⎛1 1 1 ⎞ ⎜⎝1 − n ⎟⎠
= n − ⎜ + + + …⎟ = n − 2 2
∴ ⎜ 1
⎟ ≥ (x1 · x2 …xn)1/2 = (1)1/n = 1

⎝2 4 8 ⎠
⎝ n ⎠ 1−
1
⇒ x1 + x2 + … + xn ≥ n.
2
1 –n

Hence x1 + x2 + … + xn can never be less than n.
= n −1+ n = n + 2 – 1
2

The correct option is (D)

The correct option is (C)
41. Total savings = 200 + 200 + 200 + 240 + 280 + … to n
46. Let a, ar, ar2, …
months = 11040
a + ar = 12 (1)
n−2 ar + ar3 = 48
2
⇒ 400 +
( 400 + ( n − 3) ⋅ 40) = 11040 (2)
2
dividing Eq. (2) by (1), we have
⇒ (n – 2)(140 + 20n) = 10640

⇒ 20n2 + 100n – 280 = 10640
ar 2 (1 + r )

=4
⇒ n2 + 5n – 546 = 0
a( r + 1)
⇒ (n – 21)(n + 26) = 0

r2 = 4 if r ≠ –1
⇒ n = 21 as n ≠ –26

r = –2

The correct option is (A)
Also, a = –12 [using (1)].
3 3
(( n − 1) − n )
The correct option is (B)
42. Tn = (n – 1)2 + (n – 1)n + n2 =
( n − 1) − n 2 6 10 14
47. Let S = 1+ + + + + ... (1)

= n3 – (n – 1)3 3 32 33 34

T1 = 13 – 03 1 1 2 6 10

T2 = 23 – 13
S = + + 3 + 4 + ... (2)
3 3 32

3 3

From (1) and (2),
T20 = 203 – 193

S20 = 203 – 03 = 8000 ⎛ 1⎞ 1 4 4 4
S ⎜1 − ⎟ = 1 + +
+ + + ...
The correct option is (B) ⎝ 3⎠ 3 32 33 34
43. 100(a + 99d) = 50(a + 49d)
2a + 198d = a + 49d 2 4 4 ⎛ 1 1 ⎞

S = + ⎜ 1 + + 2 + ...⎟
a + 149d = 0 3 3 3 ⎝
2 3 3 ⎠
T150 = a + 149d = 0 4 4 ⎛ 1 ⎞
2 +
The correct option is (D) ⇒
S = 3 32 ⎜ 1⎟
3 ⎜⎝ 1 − ⎟⎠
3
n
sn 3n + 8 ( 2a + ( n − 1)d ) 3n + 8 4 4 3 4 2 6
44. = ⇒ 2 =
= + = + =
sn′ 7n + 15 n 7n + 15 3 32 2 3 3 2
( 2a′ + ( n − 1)d ′ )
2 2 6

S =
⎛ n − 1⎞ 3 3
a+⎜
⎝ 2 ⎟⎠
d
3n + 8 a + 11d 3( 23) + 8

= = = ⇒
S=3
⎛ n − 1⎞ 7n + 15 a′ + 11d ′ 7( 23) + 15
a′ + ⎜ d′
⎝ 2 ⎟⎠
The correct option is (B)
3
77 7 48. Here, a = 10, d = – .

= = 7
176 16

Then,
⎛ 3⎞
tn = 10 +(n – 1) . ⎜ − ⎟

The correct option is (B) ⎝ ⎠ 7

Objective_Maths_JEE Main 2017_Ch 10.indd 40 01/01/2008 04:30:53


Sequence and Series  10.41

⎛ 3⎞ 1
tn is positive if 10 + (n – 1) ⎜ − ⎟ ≥ 0;
= 1 and d = 1.
Solving (1) and (2), we get
⎝ ⎠ 7 a
1 1 1 1 1
or, 70 – 3 (n – 1) ≥ 0 or 73 ≥ 3n; or 24
≥n ∴
= 1, = 2, = 3 and = 4.
3 a b γ δ
∴ First 24 terms are positive.

∴ Sum of the positive terms
1 1

Since, = A ⇒ A = 3. Also, = B ⇒ B = 8.
aγ bδ
24 ⎡ −3 ⎤

= S24 = 2 × 10 + 23 × ⎥
2 ⎢⎣ 7⎦
The correct option is (C)

52. We have, S1 = (n/2) [2 ⋅ 1 + (n – 1) ⋅ 1]


= 12 ⎡ 20 −


69 ⎤ = 852 .
⎢ 7 ⎥⎦ 7 S2 = (n/2) [2 ⋅ 2 + (n – 1) ⋅ 2]

Sm = (n/2) [2 ⋅ m + (n – 1) ⋅ m]
The correct option is (C) ∴ S1 + S2 + … + Sm
49. We have, n ( n −1)
(1 – 1 + 2 – 2 + 3 – 3 + … + n – n)2 = n (1 + 2 + 3 + … + m) + × (1 + 2 + … + m)
2
2
= 1 + 12 + 22 + 22 + … + n2 + n2 + 25,
m ( m + 1) ⎡ n2 − n ⎤
where S is the required sum. =
⎢n + ⎥
2 ⎢⎣ 2 ⎥⎦
⇒ o = 2(12 + 22 + … + n2) + 2S
m ( m + 1) n ( n + 1) 1
n ( n + 1)( 2n + 1) =
⋅ = mn (m + 1) (n + 1).
⇒ S = – (12 + 22 + … + n2) = –
2 2 4
6

The correct option is (D)
The correct option is (A)

53. For the positive numbers a, c we have harmonic mean H = b


kx
Skx [ 2a + ( kx − 1) d ] {∵ a, b, c are in H.P.} and geometric mean G = ac .
50. We have, = 2 But G > H; ∴ ac > b. (1)
x
Sx
2
[ 2a + ( x − 1) d ] For the positive numbers an, cn, we have

k [( 2a − d ) + kxd ] geometric mean =


an c n

=
( 2a − d ) + xd n n
arithmetic mean = a + c ;

S 2
For kx to be independent of x, 2a – d = 0 or 2a = d.

Sx n n
∵ A.M. > G.M., ∴ a + c
> an c n (2)

The correct option is (C)
2
51. α, β, γ and are in H.P.
From (1) and (2), we get



1 1 1 1
, , , are in A.P. an + c n > ( ac)n > bn, ∴ an + cn > 2bn.
a b γ δ 2

Let d be the common difference of the A.P.
The correct option is (A)
Since, a, γ are roots of Ax2 – 4x + 1 = 0

54. Let Sn = 1 ⋅ 1! + 2 ⋅ 2! + 3 ⋅ 3! + 4 ⋅ 4! + … + n ⋅ n!
a +γ 4/A 1 1 ⇒ Sn = (2 – 1) 1! + (3 – 1) 2! + (4 – 1) 3!

= = 4 or + =4
aγ 1/ A a γ  + (5 – 1) 4! + … + [(n + 1) – 1] n!
= (2 ⋅ 1! – 1!) + (3 ⋅ 2! – 2!) + (4 ⋅ 3! – 3!)
1 1 1  + (5 ⋅ 4! – 4!) + … + [(n + 1) n! – n!]

+ + 2d = 4 or + d = 2 (1)
a a a = (2! – 1!) + (3! – 2!) + (4! – 3!) + (5! – 4!)
Also, β, δ are roots of Bx2 – 6x + 1 = 0
 + … + [(n + 1)! – n!]
b +δ 1 1 6/B 1 1 = (n + 1)! – 1! = (n + 1)! – 1

= + = = 6 or + d + + 3d = 6
bδ b δ 1/ B a a The correct option is (A)
1 55. Let R = 0 ⋅ cababab…

+ 2d = 3 (2) ⇒ 102R = ca ⋅ bababa …
a
and, 104R = caba ⋅ baba …

Objective_Maths_JEE Main 2017_Ch 10.indd 41 01/01/2008 04:30:58


10.42  Chapter 10

⇒ (104 – 102) R = caba – ca



∴ [r] = 1. Also, – 1 + 5 < – r < –1
∴ [– r] = –2
caba − ca 1000c + 100a + 10b + a − 10c − a 2
⇒ R =
=
9900 9900 [r] + [–r] = 1 – 2 = –1


The correct option is (C)
99c + 10a + b

= 1 2 3
990 60. Sn = (9) + (99) + (999) + …
9 9 9
The correct option is (C)
56. When n is odd, last term will be n2, ∴ then the sum is 1 1

= [10 + 2.102 + 3.103 + …] – [1 + 2 + 3 + …]
12 + 2.22 + 32 + 2.42 + 52 + 2.62 + … + 2 (n – 1)2 + n2 9 9
1 1 n( n + 1)

= S−
( n − 1) n2 n ( n + 1)2 ⎤ 9 9 2
= + n2 ⎢ Replacing n by n − 1 in ⎥
2 ⎢ 2 ⎥ S = 10 + 2.102 + 3.103 + … + n 10n
⎣ ⎦
⇒ 10S = 102 + 2.103 + … + (n – 1)10n + n.10n+1

n3 − n2 + 2n2 n3 + n 2 2

–9S = (10 + 102 + 103 + … + 10n) – n.10n+1
=
= = n ( n + 1)
2 2 2 n +1

⇒ S = n 10 n +1 − 10 −1

The correct option is (A)
9 81
57. Let S = 1 + 2 . 2 + 3 . 22 + 4 . 23 + … + 100 . 299 n +1
− 1 1 ( n + 1)

Sn = n 10 n +1 − 10 −

∴ 2S = 1 . 2 + 2 × 22 + 3 × 23 + … + 99 . 299 + 100 . 2100

81 9.81 9 2
Subtracting, we get
n +1
– S = 1 + 1 . 2 + 1 . 22 + … + 1 ⋅ 299 – 100 . 2100 ⇒ 9.Sn = (9n − 1)10
1 n( n + 1)
+ −

= (1 + 2 + 22 + … + 299) – 100 ⋅ 2100 81 81 2
n
100 ∴ 9(Sn – Sn–1) = 10 {10(9n – 1) – (9n – 10)}– n



= 1( 2 − 1) – 100 · 2100 = 2100 – 1 – 100 . 2100
2 −1 81
= n(10n – 1)
∴ S = 100 . 2100 – 2100 + 1 = 99 . 2100 + 1.

The correct option is (C)

The correct option is (D)
61. a + b + c = xb
58. Let the number be a – d, a, a + d, a + 2d Divide by b,
where a, d ∈ Z and d > 0 a c
+ 1 + =x
Given: (a – d)2 + a2 + (a + d)2 = a + 2d b b
⇒ 2d2 – 2d + 3a2 – a = 0 1

+ 1 + r = x, r is the common ratio of the G.P.
1⎡ r
∴d=
1 ± (1 + 2a − 6 a 2 ) ⎤
2 ⎢⎣ ⎥⎦ ⇒ r2 + r (1 – x) + 1 = 0. Since r is real, therefore, discrim-

inent > 0
Since d is positive integer, ∴ 1 + 2a – 6a2 > 0

⇒ (1 – x)2 – 4 > 0 ⇒ x2 – 2x + 1 – 4 > 0

⎛1− 7 ⎞ ⎛1+ 7 ⎞ ⇒ x2 – 2x – 3 > 0 ⇒ (x + 1) (x – 3) > 0

⇒ ⎜
⎟ <a< ⎜ ⎟
⎝ 6 ⎠ ⎝ 6 ⎠ ⇒ x < –1 or x > 3


Since a is an integer,
The correct option is (A)
∴ a = 0,
3

then d =

1
[1 ± 1] = 1 or 0. Since d > 0, ∴ d = 1.
62. ∑ ar ar +1 = a1a2 + a2a3 + a3a4 = 3a1a4
1
2

Hence, the numbers are – 1, 0, 1, 2. 1 1 1 1
Since
a1, a2, a3, a4 are in H.P. , , , are in A.P.

The correct option is (C) a1 a2 a3 a4
1 1
59. Let sides of triangle be a, ar, ar2.
− = d ⇒ a1 – a2 = da1a2(1)
a2 a1
Since r > 1, ∴ ar2 is greatest side
Similarly, a2 – a3 = d a2a3(2)

∴ a + ar > ar2 ⇒ r2 – r – 1 < 0


a3 – a4 = d a3a4 (3)
1− 5 1+ 5
On adding (1), (2) and (3), we get

<r< ⇒ 1 < r < 1+ 5
2 2 a1 – a4 = d[a1a2 + a2a3 + a3a4]

2

Objective_Maths_JEE Main 2017_Ch 10.indd 42 01/01/2008 04:31:03


Sequence and Series  10.43

a − a4 a a 4
∴ a1a2 + a2a3 + a3a4 = 1
= 2 or =
d b b 1
1 1 a −a
The correct option is (D)

= + 3d ⇒ 1 4 = 3a1a4
a4 a1 d
⇒ a1a2 + a2a3 + a3a4 = 3a1a4 n 2S
66. We have, S = (a + l) ⇒ = n (1)
∴ Given expression = 3. It is a root of x2 + 2x – 15 = 0 2 a+l


The correct option is (B) l−a
Also, l = a + (n – 1) d ⇒ d =

n −1
1 5 19 65 l−a
63. + + + + … to n terms
=  [Using (1)]
3 9 27 81 2S
−1
⎛ 2⎞ ⎛ 4⎞ ⎛ 8 ⎞ ⎛ 16 ⎞ a+l
= ⎜1 − ⎟ + ⎜1 − ⎟ + ⎜1 −
+ 1− +…
⎝ ⎠ ⎝ 3⎠ ⎝ 9 27 ⎟⎠ ⎜⎝ 81⎟⎠
=
l 2 − a2
2S − ( l + a)
2 ⎡ 2 ⎛ 2⎞ ⎤
2
=n–
⎢1 + + ⎜ ⎟ + ... to n terms ⎥ ∴ k = 2S

3 ⎢⎣ 3 ⎝ 3 ⎠ ⎥⎦
The correct option is (B)

⎛ 2⎞
n 67. As a1, a2, a3, … an–1, an are in A.P.,
1− ⎜ ⎟ d = a2 – a1 = a3 – a2 = … = an – an–1
2 ⎝ 3⎠
=n– · n = 2 n – (3n – 2n)
3 1− 2 sin d [sec a1 sec a2 + sec a2 sec a3 + .. + sec an–1 sec an]
3n
3 sin (a2 − a1 ) sin (a3 − a2 ) sin (an − an −1 )
The correct option is (B) =
+ + .. +
cos a1 ⋅ cos a2 cos a2 cos a3 cos an −1 cos an
64. Taking A.M. and G.M. of 7 numbers
= (tan a2 – tan a1) + (tan a3 – tan a2)

a a b b b c c
 + … + (tan an – tan an–1)
, , , , , , , we get
2 2 3 3 3 2 2 = tan an – tan a1

1
The correct option is (A)
a b c ⎧⎪⎛ a ⎞ 2 ⎛ b ⎞ 3 ⎛ c ⎞ 2 ⎫⎪ 7
2 ⋅ 2 + 3 ⋅ 3 + 2 ⋅ 2 ≥ ⎨⎜ ⎟ ⎜ ⎟ ⎜ ⎟ ⎬ ( r − 1)! r!
7 ⎪⎩⎝ 2 ⎠ ⎝ 3 ⎠ ⎝ 2 ⎠ ⎪⎭ 68. We have, tr =
( r + 4)!
and tr+1 =
( r + 5)!
1
r! r!
3 ⎛ a 2b3c 2 ⎞ 7 37 2 3 2 rtr – (r + 5)tr+1 =
Now, – =0

≥ ⎜ 2 3 2⎟ ⇒ 7 ≥ a b c ( r + 4)! ( r + 4)!
7 ⎝2 3 2 ⎠ 7 22 ⋅ 33 ⋅ 22

rtr – (r + 1)tr+1 = 4tr+1
10 4
⇒ a b c ≤ 3 ⋅ 2
2 3 2 n −1 n −1
7 7
10 4
⇒ 4
∑ t r +1 = ∑
r =1
[rt r − ( r + 1) t r +1 ]
∴ greatest value of a2 b3 c2 = 3 ⋅ 2
r =1

7 7 ⇒   4(t2 + t3 + … + tn) = 1t1 – ntn


The correct option is (A)
n( n − 1)
(t1 + t2 + … + tn) = 5t1 – ntn = 5 ⎛⎜ ⎞⎟ −
0!
⇒ 4
2ab
⎝ 5!⎠ ( n + 4)
65. Harmonic mean of a, b is H =
a+b
Geometric mean G =
ab
 = 1 − n!
H 4 4
4 ! ( n + 4)!

Given: = , so 2 ab =
G 5 a+b 5 1⎡1 n! ⎤

t1 + t2 + … + tn = −
a+b 5 4 ⎣ 4! ( n + 4)! ⎥⎦


or, =
2 ab 4
The correct option is (B)

By componendo and dividendo n n n n

69. an = x1/2 + y1/2 and bn = x1/2 − y1/2


( a + b )2 9 a+ b 3
= or =
( a − b )2 1 a− b 1
Now, anbn = x1/2 + y1/2
( ) (x
n n
1/2n
− y1/2
n

)
3+1 2 a
= (x ) − ( y )

Again, by componendo and dividendo = 1/2n
2
1/2n
2
2 b 3 −1 ⇒ anbn

Objective_Maths_JEE Main 2017_Ch 10.indd 43 01/01/2008 04:31:11


10.44  Chapter 10

n −1 n −1 n −1
⇒ anbn = x 1/2
− y 1/2 = bn–1(1)
< 1 + 2 + 4 + 8 + ... + 2
2 4 8 2n −1

Now, a1a2a3 … an
1+… +1
⎛ a a a ...a ⎞
=1+ = n.
= ⎜ 1 2 3 n ⎟ bn ( n + 1) times
⎝ bn ⎠ Thus, a (100) < 100

(a1a2 a3 ...an −1 ) (anbn ) 1 ⎛ 1 1⎞ ⎛ 1 1⎞

= Next, a (n) = 1 + + ⎜ + ⎟ + ⎜ + … + ⎟ + …

bn 2 ⎝ 3 4⎠ ⎝ 5 8⎠
1 1
a a a ...a b
+ + ... +

= 1 2 3 n −1 n −1  {using (1)} 2n −1 + 1 2n − 1
bn n −1
( a a a ...a ) ( an −1bn −1 )
> 1+ 1 + 2 + 4 +… + 2 − 1

= 1 2 3 n− 2 2 4 8 2n 2n
bn
1 1 1 1
+ + +… + 1
a a a ...a b
=1+ 2 2 2 2 – n

= 1 2 3 n −1 n − 2    2
n times
bn
⎛ 1⎞ n

………………
= ⎜1 − n ⎟ +
⎝ 2 ⎠ 2

………………
Thus, a (200) > ⎛ 1 −
a1b1 b x −y
1 ⎞ 200 > 100.

= 0 =
= ⎜⎝ 100 ⎟
+
bn bn bn ⎠ 2 2

The correct option is (D)

The correct option is (C)

70. Since n is an odd integer (–1)n–1 = 1 and n – 1, 1 1 1


72. Let α = ,β= and γ  = .
n – 3, n – 5 etc., are even integers. We have, a−d a a+d
n3 – (n – 1)3 + (n – 2)3 – (n – 3)3 + … + (–1)n–113 1 1 1 1

Then, ⋅ ⋅ =–

= n3 + (n – 1)3 + (n – 2)3 + … + 13 a−d a a+d 3

– 2[(n – 1)3 + (n – 3)3 + … + 23] 1 1 1 1 1 1

and, ⋅ + ⋅ + ⋅ =–1
= n + (n – 1) + (n – 2) + … + 13
3 3 3
a−d a a+d a a−d a + d
⎡⎛ n − 1⎞ 3 ⎛ n − 3 ⎞ 3 3
⎤ ⇒ (a – d) a (a + d) = – 3



– 2 × 2 ⎢⎜⎝ 2 ⎟⎠ + ⎜⎝ 2 ⎟⎠ + ... + 1 ⎥
3
⎢⎣ ⎥⎦ (a + d ) + (a − d ) + a


and, =–1
[∵ n –1, n – 3 are even integers] (a − d ) a (a + d )
n( n + 1) ⎤
2
⎡ 1 ⎛ n − 1⎞ ⎛ n − 1 ⎞ ⎤
2 ⇒ 3a = 3 or a = 1 and (1 – d) (1 + d) = – 3

= ⎡
⎥ − 16 ⎢ ⎜ + 1⎟ ⎥
1 – d2 = – 3 or d2 = 4 ⇒ d = ± 2.
⎢ ⎟⎜
or,
⎣ 2 ⎦ ⎣2 ⎝ 2 ⎠ ⎝ 2 ⎠⎦
1

When d = 2, α = – 1, β = 1, γ  = .
2 2
1 2 ( n − 1) ( n + 1) 3
=
n (n + 1)2 – 16
4 16 × 4 1

When d = – 2, α = , β = 1, γ  = – 1.
3
1 1 4
=
(n + 1)2 [n2 – (n – 1)2] = (n + 1)2(2n – 1). Therefore, | α – γ | =

4 4 3

The correct option is (D)
The correct option is (C)

71. We have, x−a x−b b a


73. We have, + = +
1 1 1 1
b a x−a x−b
a (n) = 1 +
+ + + ... + n
x−a
2 3 4 (2 ) − 1 b a x−b

− = −
b x−a x−b a
⎛ 1 1⎞ ⎛ 1 1⎞ ⎛ 1 1⎞

= 1+ ⎜ + ⎟ + ⎜ +… + ⎟ + ⎜ +… + ⎟
⎝ 2 3⎠ ⎝ 4 7⎠ ⎝ 8 15 ⎠ ( x − a) 2 − b 2 a 2 − ( x − b) 2

=
⎛ 1 1 ⎞ b ( x − a) ( x − b) a

 +… + ⎜ n −1 + … + n −1 ⎟
⎝2 2 ⎠ (x − a − b ) (x − a + b ) ( x − b − a) ( x − b + a)

=–
b ( x − a) (x − b ) a

Objective_Maths_JEE Main 2017_Ch 10.indd 44 01/01/2008 04:31:18


Sequence and Series  10.45

⇒ (x – a – b) [a (x – b) (x – a + b) + b (x – b + a)
76. We have, | ai | = | ai – 1 + 1 |
(x – a)] = 0

⇒ ai2 = ai2−1 + 2 ai −1 + 1

⇒ x (x – a – b) [(a + b) x – (a2 + b2)] = 0

2 2 Putting i = 1, 2, 3, …, n + 1, we get

⇒ x = 0, x = a + b or x = a + b .

a12 = 0
a+b
a2 + b 2 2ab
a22 = a12 + 2a1 + 1

Since =a+b– < a + b (Q a, b > 0)
a+b a+b
2 2
a32 = a22 + 2a2 + 1
we take x1 = a + b, x2 = a + b and x3 = 0.

a+b
M M M
2 2
Since x1 – x2 – x3 = c, we get a + b – a + b = c

an2 = an−1
2 + 2an – 1 + 1
a+b
2ab 2

= c ⇒ a, b, c are in H.P.
an+1 = an2 + 2an + 1
a+b
n +1 n n

The correct option is (C)

On adding, we get ∑ ai2
i =1
= ∑ ai2 + 2∑ ai + n
74. The given product i =1 i =1
n
= (– 1)n (x – 1) (2x – 1) (22 x – 1) (23 x – 1) … (2n x – 1)
n 1+2+…+n ⎛ 1⎞
⇒ 2
∑ ai 2
= – n + an+1 ≥–n
= (– 1) ⋅ 2
⋅ (x – 1) ⎜ x − ⎟ i =1
⎝ 2⎠ a + a + ... + an 1 1
⇒ 1 2
≥– ⇒x≥– .
⎛ 1⎞ ⎛ 1⎞ n 2 2

 ⎜⎝ x − 2 ⎟⎠ … ⎜⎝ x − n ⎟⎠
2 2
The correct option is (B)
n ( n +1)
⎛ 1⎞ ⎛ 1⎞ ⎛ 1⎞ 77. Given: a1, a2, a3, …, an are in H.P.
= (– 1)n ⋅ 2
⋅ (x – 1) ⎜ x − ⎟ ⎜ x − 2 ⎟ … ⎜ x − n ⎟
2
⎝ 2⎠ ⎝ 2 ⎠ ⎝ 2 ⎠ 1
1 1 1

, , , …, are in A.P.
∴ coefficient of xn
a1 a2 a3 an
n ( n +1)
⎛ 1 1 1⎞ a1 + a2 + a3 + ... + an a1 + a2 + ... + an
= (– 1)n 2 2 ⋅ ⎜ −1 − − 2 ... n ⎟ ⇒
, , ….
⎝ 2 2 2 ⎠ a1 a2
⎛ 1 ⎞ a1 + a2 + ... + an
n ( n +1) ⎜⎝1 − n+1 ⎟⎠ n ( n − 1)  , are in A.P.
= (–
1)2n + 1 ⋅ 2 2 ⋅ 2 ⋅ = (1 – 2n + 1) ⋅ 2 2 . an
⎛ 1⎞
⎜⎝1 − ⎟⎠ a1 + a2 + a3 + ... + an a + a + ... + an
2 ⇒
– 1, 1 2 – 1, …,

The correct option is (A) a1 a2
a1 + a2 + ... + an
75. Let R = 0.272727…
 – 1, are in A.P.
2 an
⇒ 10 R = 27.2727…
and, 104R = 2727.2727… a2 + a3 + ... + an a + a + ... + an

,, 1 3 …,
⇒ (104 – 102) R = 2727 – 27 a1 a2
2700 3 a1 + a2 + ... + an −1

R= =.
 are in A.P.
9900 11 an
8 a1 a2
Similarly, 0.727272… =
. ⇒
, , …,
11 a2 + a3 + ... + an a1 + a3 + ... + an

Since 0.272727…, x and 0.727272… are in H.P.
an
3 8  are in H.P.

, x, are in H.P. a1 + a2 + ... + an −1
11 11

The correct option is (C)
3 8
2⋅ ⋅
⇒ x =
11 11 = 48 78. Let the three digits be a, ar, ar2.
3 8 121 Then, according to the hypothesis,
+
11 11 100a + 10ar + ar2 + 792 = 100ar2 + 10ar + a
∴ x is rational.

⇒ a (r2 – 1) = 8 (1)

The correct option is (A)

Objective_Maths_JEE Main 2017_Ch 10.indd 45 01/01/2008 04:31:27


10.46  Chapter 10

and, a, ar + 2, ar2 are in A.P.



∴ coeff. of x98 = sum of the product of 1, 2, 3, …, 100 taken

then, 2 (ar + 2) = a + ar2 two at a time
2
⇒ a (r – 2r + 1) = 4 (2) 1
=
[(1 + 2 + 3 + … + 100)2 – (12 + 22 + … + 1002)]

Dividing (1) by (2), 2
a ( r 2 − 1) 8 1 ⎡ 100 × 101 × 201 ⎤

then, = =
(5050) 2 −
2
a ( r − 2 r + 1) 4 2 ⎢⎣ 6 ⎥

( r + 1)( r − 1) 1
=
[(5050)2 – 338350)] = 12582075.

=2 2
( r − 1)2

The correct option is (C)
r +1

=2
r −1 81. Let the sides of the triangle be a, ar, ar2.
∴ r = 3 ∴ from (1), a = 1. If r = 1, then the three terms of G.P. will be a, a, a and hence
Thus, digits are 1, 3, 9 and so the required number is 931. an equilateral triangle will be formed.
The correct option is (C) Thus when r = 1, triangle will be formed (1)
79. The general term of the given sequence is If r > 1, then greatest side will be ar2 and in this case triangle
will be formed if

Tn = n2 a + ar > ar2 ⇒ r2 – r – 1 < 0
500 + 3n3 1− 5

< r < 1+ 5
3 2
d Tn n (1000 − 3n ) 2

then, =
dn (500 + 3n3 )2

r < 1 + 5 [Qr > 1]  (2)

For maximum or minimum of Tn 2
d Tn If r < 1, then greatest side will be a and triangle will be

=0 formed if
dn
1
ar + ar2 > a ⇒ r2 + r – 1 > 0
⎛ 1000 ⎞ 3 −1 − 5
∴ n= ⎜
or r > −1 + 5

⎝ 3 ⎟⎠ ⇒
r<
2 2
1
Now, 6 < ⎛ 1000 ⎞
3
<7 5 −1
⎜⎝ 3 ⎟⎠ ⇒
< r < 1. [Q 0 < r < 1]  (3)
2

Hence, T7 is largest term. So largest term in the given
From (1), (2) and (3), possible values of r are given by

sequence is 49 . 5 −1 5 +1 .
1529 <r<
2 2
The correct option is (C)

The correct option is (B)
80. Consider the equation
(x – 1) (x – 2) (x – 3) … (x – 100) = 0 (1) 82. Let the sides of the right angled triangle be a, ar, ar2 out of
Its root are 1, 2, 3, …, 100. which ar2 is the hypotenuse, then r > 1.
(1) is a polynomial equation in x of degree 100. Coefficient A
of x100 = 1.
Now, sum of the roots of equation (1); taken one at a time
99
i.e., 1 + 2 + 3 + … + 100 = (– 1)1 coeff. of x

coeff. of x100
ar ar 2

= – coeff. of x99
100 × 101
∴ coeff. of x99 = – (1 + 2 + 3 + … + 100) = –
2

= – 5050

Sum of the product of the roots 1, 2, 3, …, 100 taken two at B a C
a time 2 4
Now, a r = a + a r
2 2 2
98

= (– 1) coeff. of x
2
= coeff. of x98
or, r4 – r2 – 1 = 0 ∴ r2 = 1 ± 5
100
coeff. of x 2

Objective_Maths_JEE Main 2017_Ch 10.indd 46 01/01/2008 04:31:31


Sequence and Series  10.47

2 2 1+ 5 . Then, log [ab – c ⋅ bc – a ⋅ ca – b] = 0



Q r > 1 ∴ r > 1 ∴ r =

2 ∴ ab – c ⋅ bc – a ⋅ ca – b = 1;


Angle C is the greater acute angle ∴ ab bc ca = ac ba cb.
a 1 1 The correct option is (C)
Now, cos C =
= = .
ar 2 r2 1+ 5 85. Let the 3n terms of G.P. are a, ar, ar2, …,
The correct option is (A) arn – 1, arn, arn + 1, arn + 2, … ar2n – 1, ar2n,
83. Here, the series is ar2n + 1, ar2n + 2, …, ar2n – 1
n
2 + (2 + 3) + (2 + 3 + 9) + (2 + 3 + 9 + 27) + …

Then, S1 = a + ar + ar2 + … + arn – 1 = a (1 − r )
1− r
the difference of the consecutive terms being 3, 9, 27, …
∴ tn = 2 + 3 + 9 + 27 … to n terms
S2 = arn + arn + 1 + arn + 2 + … + ar3n – 1
= 2 + [3 + 9 + 27 + … to (n – 1) terms] n n

= ar (1 − r )
n −1 1− r
3

= 2 + 3 (1 − 3 ) = 2 – (1 – 3n – 1)
S3 = ar2n + ar2n + 1 + ar2n + 2 + … + ar3n – 1
1− 3 2
n n

= ar (1 − r )
1 1

= + ⋅ 3n; 1− r
2 2
(1 − r n ) 2
1 1 Now, (S2)2 = a2 r2n

∴ Sn = Σ tn =
Σ1+ Σ 3n (1 − r ) 2
2 2
a (1 − r n ) n

=
1
⋅n+
1
(3 + 32 + 33 + … + 3n)
= ⋅ ar2n (1 − r ) = S1S3
2 2 1− r 1− r
Hence,
S , S
1 2 3, S are in G.P.
n 1 3 (1 − 3n ) n 3 n
The correct option is (B)

= + ⋅ = + (3 − 1) .
2 2 1− 3 2 4
n

The correct option is (B) 86. Let r > 1. Then, Sn = a ( r − 1)
r −1
84. Let the A.P. be x, x + y, x + 2y …
un = S1 + S2 + S3 + … + Sn
Then, a = x – (p – 1) y,(1)
2 2 n
b = x + (q – 1) y (2) = a ( r − 1) + a ( r − 1) + a ( r − 1) + ... + a ( r − 1)
c = x + (r – 1) y (3) r −1 r −1 r −1 r −1
∴ b – c = (q – r) y (4) a

= [(r + r2 + r3 + … + rn) – n]
c – a = (r – p) y (5) r −1
a – b = ( p – q) y (6) a ⎡ r ( r n − 1) ⎤

= ⎢ − n⎥
Let the G.P. be u, uv, uv2, … r − 1 ⎢⎣ r − 1 ⎥⎦
Then, a = uvp – 1(7)
∴ r Sn + (1 – r) un


b = uvq – 1,(8)
c = uvr – 1,(9) a ( r n − 1) ar ( r n − 1) an (1 − r )

= r⋅ + (1 − r ) −
r −1 (1 − r ) 2 r −1
Now, log (ab – c ⋅ bc – a ⋅ ca – b)


= (b – c) log a + (c – a) log b + (a – b) log c n n

= – ar ( r − 1) + ar ( r − 1) + an = na
= (q – r) y log (uvp – 1) + (r – p) y log (uvq – 1) 1− r 1− r

 + ( p – q) y log (uvr – 1), using (4), (5), (6),
The correct option is (A)
 (7), (8), (9).
2 2

= y [(q – r) {log u + (p – 1) log v} + (r – p) {log u 87. We have, cos A = b + c − a
 + (q – 1) log v} + (p – q) {log u + (r – 1) log v} 2bc

= y [log u (q – r + r – p + p – q) + log v {(q – r) (p – 1) and, sin A = ka (k is a content)


 + (r – p) (q – 1) + (p – q) (r – 1)}] 2 2 2
∴ cot A = b + c − a and similarly, we have


= y [log u × 0 + log v × 0] = 0. 2abck

Objective_Maths_JEE Main 2017_Ch 10.indd 47 01/01/2008 04:31:35


10.48  Chapter 10

90. Let the n numbers in G.P. be


2 2 2 a2 + b2 − c2
cot B = a + c − b , cot C =
a, ar, ar2,…, arn–1
2abck 2abck
Thus, we have,
⎛1− rn ⎞
b 2 + c 2 − a2 a2 + c 2 − b 2 a2 + b 2 − c 2
Sn = a ⎜ ⎟

Given: , , are ⎝ 1− r ⎠
2abck 2abck 2abck
in A.P. Also,

2 2 2 2 2
⇒ b + c –a , a + c – b , a + b –c are in A.P.
2 2 2 2
S2n = [a + ar + ar2 +….+ arn–1]2

(Multiplying each term by 2abck) ⎛1− rn ⎞

⇒ a2 ⎜ ⎟ = a2 + (ar)2 + (ar2)2 +…..+(arn–1)2 + 2S
⇒ –2a2, – 2b2, – 2c2 are in A.P.
⎝ 1− r ⎠
(Subtracting a2 + b2 + c2 from each term)

where S denotes the sum of the product of the terms of the

⇒ a2, b2, c2 are in A.P. (dividing each term by –2]
G.P. taken two at a time.

The correct option is (A)
2
1 1 1 ⎛1− rn ⎞ ⎛ 1 − r 2n ⎞
88. Let S = + + + ….up to ∞ ⇒ a2 ⎜
⎟ = a2 ⎜ ⎟ + 2S
14 34 54 ⎝ 1− r ⎠ ⎝ 1 − r2 ⎠

π4 1 1 1 1 1 a2 ⎡ (1 − r n )2 1 − r 2 n ⎤

Given: =
4
+ 4 + 4 + 4 + 4 + ….. ⇒
S= ⎢ 2
− ⎥
90 1 2 2 3 4 2 ⎢⎣ (1 − r ) 1 − r 2 ⎥⎦
1 1 1

=S+ + + + ... a2 ⎛ 1 − r n ⎞ ⎡1 − r n 1 + r n ⎤
24 44 64
= ⎜ 1− r ⎟ ⎢ 1− r − 1 + r ⎥
2 ⎝ ⎠ ⎢⎣ ⎥⎦
1 ⎡1 1 1 ⎤

= S+ 4 ⎢ 4
+ 4 + 4 + ....⎥
2 ⎣1 2 3 ⎦ a (1 − r n )(1 + r ) − (1 + r n )(1 − r )

= Sn × ×
2 (1 + r )(1 − r )
= S+
1 π4

×
2 4 90 a 2( r − r n )

= Sn × ×
π 4 2 (1 + r )(1 − r )
π4 ⎛ 1⎞
∴ S =
1− =
90 ⎜⎝ 16 ⎟⎠ 96 r a(1 − r n −1 ) ⎛ r ⎞
= Sn × × =⎜
⎝ 1+ r ⎟⎠ n n–1
S S

The correct option is (B) 1+ r 1− r
89. Let a and d respectively be the first term and common differ- r

k=
ence of the A.P. r +1
2mr
The correct option is (C)
Given: n =
(1)
m+r 91. Let the common difference of the given A.P. be t. Then,
and, (a + nd)2 = (a + md) (a + rd)
d = a2 + b2 + c2 ⇒ a + 3t = a2 + (a + t)2 + (a + 2t)2

2 ⇒ 5t2 + 3 (2a –1) t + 3a2– a = 0 (1)
⎛a ⎞
+ n⎟ = ⎛⎜ + m ⎞⎟ ⎛⎜ + r ⎞⎟
a a
⇒ ⎜
∵ t is real ⇒ D ≥ 0
⎝ d ⎠ ⎝d ⎠⎝d ⎠
⇒ 9 (2a – 1)2–4 (5) (3a2– a) ≥ 0
⇒ (x + n)2 = (x + m) (x + r)
⎡ a ⎤ ⇒ 24 a2 + 16a – 9 ≤ 0

⎢ Putting d = x ⎥
⎣ ⎦ 1
⇒ x2 + 2nx + n2 = x2 + (m + r) x + mr
⇒ −
– 70 < a < −1 + 70
3 12 3 12
n( m + r )
⇒ (m + r – 2n) x = n2 – mr = n 2 −
 ⇒ a = –1, 0
[ ∵ a is integer]
2
3
 [Using (1)] When a = 0, from (1), t = 0, . Rejecting both these values

n since t must be non zero 5

= ( 2n − m − r )
2 4

When, a = – 1, from (1), t = 1, ⇒t=1
n 5
∴ x = − , which is the required ratio.

2 ∴ a + b + c + d = – 1 + 0 + 1 + 2 = 2


The correct option is (D)

The correct option is (A)

Objective_Maths_JEE Main 2017_Ch 10.indd 48 01/01/2008 04:31:40


Sequence and Series  10.49

3 5 ⎛ 2n − 1⎞ ⎛ 5c a ⎞ ⎛ 5c ⎞ ⎛ 3b ⎞
92. 1 + + +… + ⎜
2 3 ⎝ n ⎟⎠ = log ⎜


.
a 3b ⎟⎠
= log ⎜⎝ 3b ⎟⎠ = – log ⎜ ⎟
⎝ 5c ⎠
⎛ 1⎞ ⎛ 1⎞ ⎛ 1⎞
= (2–1) + ⎜ 2 − ⎟ + ⎜ 2 − ⎟ + ..... + ⎜ 2 − ⎟
⎛ 3b ⎞ 5
⎝ 2⎠ ⎝ 3⎠ ⎝ n⎠
i.e., 3 log ⎜ = 0 or b = c (2)
⎝ 5c ⎟⎠ 3
= 2n – ⎜ 1 +
⎛ 1 1 1⎞
+ + ..... + ⎟ = 2n – Hn
From (1) and (2), we have,
⎝ 2 3 n⎠
2

The correct option is (B) a = b = 25c

c 9
93. We have, a21 – a22 + a23 – a24 + ……+ a22n–1 – a22n
Now, we have,
= (a1– a2) [a1 + a2 + a3 +….+ a2n]
5c 8c 25c
[∵ a1 – a2 = a3 – a4 = ……..a2n–1 – a2n] b+c=
+c= < =a
3 3 9
2n


2
[ 2a1 + ( 2n − 1)d ] (1)
=–d×
and hence, a, b, c cannot form the sides of a triangle.

The correct option is (D)

where d is the common difference of the A.P.
Since, a2n = a1 +(2n–1) d
96. Given: b2 = ac  (∵ a, b, c are in G.P.)
and, 2(log 2b – log 3c) = log a – log 2b + log 3c – log a
a2 n − a1

d=  (∵ given terms are in A.P)
2n − 1 2
⎛ 2b ⎞ ⎛ 3c ⎞

Thus, we have from (1), required expression ⇒ log ⎜
⎟ = log ⎜ ⎟ ⇒ b = 3c
a − a2 n ⎝ 3c ⎠ ⎝ 2b ⎠ 2

= 1 [
× n 2a1 + a2 n − a1
2n − 1
]
2
Now, a = b = 3b = 9c

n ( a1 − a2 n )( a1 + a2 n ) n ( a12 − a22n )

= = c 2 4
2n − 1 2n − 1

The correct option is (C) ∴ a is the largest side

1 9c 2 81
94. We have, an + 1 =
b + c −2
a 2+ c2 − c22
1− an Now, cos A =
= 4 16 = negative
1 1 2bc 3
1 = 2× c ×c
∴ a2 =
and a3 = 1 − a2 1 2
1 − a1 1−
1 − a1 ∴ A > 90º, ∴ triangle is obtuse.

1 − a1 1 − a1
The correct option is (C)

= =
1 − a1 − 1 −a1 97. The middle term of the 4n + 1 terms is the (2n + 1)
1 − a1 th term. Let it be m.

Since a3 = a1, ∴ = a1
−a1 The middle term of (2n + 1) terms is the (n + 1) th term.

a21 – a1 + 1 = 0 ⇒ a1 = – w or – w2 Thus, the middle term of the A.P. is
1 1 − a3 = m – (n + 1– 1)2 = m –2n
1 =
Now, a5 = = 1 − a3 and the middle term of the G.P is
1 − a4 1−
1 − a3 n + 1−1
=m ⎛ ⎞
1 − a1 1 m
= = a1 = a3 and so on


⎜⎝ 2 ⎟⎠ =
−a1 2n

a1 = a3 = a5 ………a2001
According to the given condition, we have

Thus, (a 2001) 2001


= (–w)2001 or (–w2)2001 m

m – 2n =


= –1 2n

The correct option is (B) n.2n + 1

m=
95. Given: b2 = ac(1) 2n − 1
⎛ 3b ⎞ ⎛ 5c ⎞ ⎛ a⎞
and, 2 log ⎜ ⎟ = log ⎜ ⎟ + log ⎜ ⎟


The correct option is (B)
⎝ 5c ⎠ ⎝ a⎠ ⎝ 3b ⎠

Objective_Maths_JEE Main 2017_Ch 10.indd 49 01/01/2008 04:31:45


10.50  Chapter 10

98. Let the common difference of the three A.P.s be d1, d2 and d3 a a a a
⇒ − = −  [Using equation (1)]
Then, we have x y y z
n
S1 = [ 2.1 + ( n − 1)d1 ] 1 1 1 1 2 1 1
2 ⇒ − = − ⇒ = +
x y y z y x z
2( S1 − n)

d1 = (1) ∴ x, y, z are in H.P.
n( n − 1)
The correct option is (C)
2(S 2 − n ) 101. We have,
Similarly, d2 =
(2)
n( n − 1)
S = 1 + (1 + a)b + (1 + a + a2) b2 + (1 + a + a2 + a3) b3 +…∞

and, d3 = 2(S 3 − n )(3) = (1 + b + b2 +….∞) + a (b + b2 + b3 +…..∞)
n( n − 1) + a2 (b2 + b3 + ……∞) +….∞

Since d1, d2 and d3 are given to be in H.P, therefore,
1 ab a 2b 2 1
1 1 1 1 = + + + ....∞ =

− = − 1− b 1− b 1− b (1 − b)(1 − ab)
d2 d1 d3 d2

The correct option is (A)
1 1 1 1

− = −
S2 − n S1 − n S3 − n S2 − n 102. We have,

( ) − (y )
− n −1 2 2

 [Using results (1), (2), (3)] 2 − n −1
bn = x 2
S1 − S2 S2 − S3

=
( S2 − n)( S1 − n) ( S2 − n)( S3 − n) = x2( − n −1

− y2
− n −1

) (x 2 − n −1
+ y2
− n −1

)


S1 − S 2 = S 2 − S 3 ⇒ n = 2S3S1 − S1S2 − S2 S3 = bn + 1 an + 1
S1 − n S3 − n S1 − 2S2 + S3

The correct option is (C) an =
bn −1(Putting n in place of n – 1)
i.e.,
bn
99. We have,
Sn = 13 + 3.23 + 33 + 3.43 + 53 +… x−y
b0 b1 b2 bn−1
Let n = 2m. Then, ∴ a1, a2… an = . . .... =
b1 b2 b3 bn bn
S2m = (13 + 33 + 53 …..to m terms)
 + 3 (23 + 43 + 63 + …..to m terms)
The correct option is (B)
= {13 + 23 + 33 + 43 + ……+ (2m–1)3 + (2m)3}
103. Given: a1 + 5d = 2
 – { 23 + 43 +…..+ (2m)3} +3{ 23 + 43 + 63 +….+ (2m)3}
Let y = a1 a4 a5 = a1 (a1 + 3d) (a1 + 4d)
2
⎡ 2m( 2m + 1) ⎤ 3 3 3 3 = (2 – 5d) (2 – 2d) (2 – d)
= ⎢
⎥ + 8 × 2{1 + 2 + 3 + .... + m }
⎣ 2 ⎦ (Putting a1 = 2 – 5d)
2 2 = 2 (4 – 16d + 17d 2 – 5d3)
= m2 (2 m + 1)2 + 16 m ( m + 1)
The value of d at which y attains maxima is given by
4
dy
n2 ( n2 + 3n + 1) n = 0 (by calculus)
=
 [Put m = ] dx
2 2 ⇒ – 16 + 34 d – 15 d2 = 0

The correct option is (B)

1 00. Since p, q, r are in A.P. we have


34 ± 14 2 8
⇒ d= = ,
p – q = q – r = k (say) (1) 30 3 5
a−x a− y a−z d 2y 2
Given: = = Now, = 34 – 30 d > 0 for d =
px qy rz dx 2 3
8
 < 0 for d =
⎛a ⎞ ⎛a ⎞ ⎛a ⎞ ⎛a ⎞ 5
⎜⎝ x − 1⎟⎠ − ⎜⎝ y − 1⎟⎠ ⎜⎝ y − 1⎟⎠ − ⎜⎝ z − 1⎟⎠ 8
Hence, y is maximum for d =
⇒ = 5
p −q q−r

The correct option is (A)
 (by componendo–dividendo)

Objective_Maths_JEE Main 2017_Ch 10.indd 50 01/01/2008 04:31:51


Sequence and Series  10.51

104. Let tn denotes the nth term of the sequence Hence, we have,
1, 2, 4, 7 n
S= ⎡ 2t n + ( n − 1)1⎤⎦
Let S = 1 + 2 + 4 + 7 + ….+ tn 2⎣
Again, S = 1 + 2 + 4 + …..+ tn–1 + tn
n n ( n2 + 1)
On subtracting, we get = ⎡⎣ n2 − n + 2 + n − 1⎤⎦ =
0 = 1 + [1 + 2 + 3 +……(n – 1) terms] – tn 2 2
n ( n − 1) n2 − n + 2
The correct option is (C)
⇒ tn = 1 + =
2 2
which denotes the first term of the nth row which contains
n terms in A.P., having common difference 1.

More than One Option Correct Type


1 05. Let the numbers be a and b. If H is the harmonic mean then 3a2 – d2 = β
H = 4, 2A + G2 = 27  (1)  (Given) ⎛ 1⎞
2
1 1
2 ⇒ β = 3 ⎜ ⎟ − d 2 = – d2 ≤ .
But we know, AH = G . ⎝ 3⎠ 3 3
∴ 4 A = G2; ∴ from (1), 2A + 4A = 27; ⎛ 1⎤
Thus, β ∈ ⎜ −∞, ⎥ .
27 9 ⎝ 3⎦
∴ A= =
6 2 From (3), we get
a+b 9 ⎧ a + b⎫ α (a2 – d2) = – γ
∴ = , ⎨Q A = ⎬
2 2 ⎩ 2 ⎭ 1 ⎛1 2⎞
∴ a + b = 9 (2) ⇒ ⎜ − d ⎟⎠ = – γ
3 ⎝9
2ab 2ab
Again, H=4= ;  ∴  4 =
a+b 9 1 2 1 1
⇒ γ= d – ≥–
∴ ab = 18. (3) 3 27 27
From (2) and (3), we get
⎡ 1 ⎞
Thus, γ ∈ ⎢ − , ∞⎟
a – b = ( a + b) 2 − 4 ab = 9 − 4 × 18 = ± 3
2 (4) ⎣ 27 ⎠
∴  From (2) and (4), 2a = 9 ± 3 and 2b = 9 ∓ 3;
∴  a = 6, 3 and b = 3, 6. ⎛ 1⎤ ⎡ 1 ⎞
Hence, β ∈ ⎜ −∞, ⎥ and γ ∈ ⎢ − 27 , ∞⎟⎠
⎝ 3⎦ ⎣
∴  The numbers are 3 and 6.
The correct option is (C, D) The correct option is (A, D)
106.  Since nth term of the first (2n – 1) terms is the middle term, a+c
108. As a, b, c are in A.P., b = (1)
∴ a is the A.M. (A); b is the G.M. (G); c is the H.M. (H) of 2
the series, whose first and last terms are equal. 2 2
We know that A ≥ G > H and AH = G2 As a2, b2, c2 are in H.P., b2 = 2a c
a + c2
2
∴  a ≥ b ≥ c and ac = b2
⇒ b (a + c ) = 2 a c ⇒ b {(a + c)2 – 2ac} = 2 a2c2
2 2 2 2 2 2
The correct option is (B, D)
⇒ b2 {4b2 – 2ac} = 2 a2c2, using (1)
107. Since x1, x2, x3 are in A.P. we may take x1 = α – d, x2 = α and 2 2 2 2 4 2 2
x3 = α + d. ⇒ b (2b – ac) = a c ; ⇒ 2b – b  ac – (ac) = 0
Since x1, x2, x3 are roots of ⇒ (2b2 + ac) (b2 – ac) = 0.
x3 – x2 + bx + γ = 0 ∴  Either 2b2 + ac = 0 or b2 = ac;
we have, (α – d) + α + (α + d) = 1 (1) ac 2

(α – d) α + α (α + d) + (α – d) (α + d) = β(2) i.e., b2 = –   ∴ ⎛⎜ a + c ⎞⎟ = ac
2 ⎝ 2 ⎠
(α – d) (α) (α + d) = – γ(3) a
1 i.e., – , b, c are in G.P. i.e., (a – c)2 = 0 i.e., a = c;
From (1), we get 3α = 1 or α = 2
3
From (2), we get

Objective_Maths_JEE Main 2017_Ch 10.indd 51 01/01/2008 04:31:55


10.52  Chapter 10

c a+c a+a 2 ⎡ b 2 + bc + ab + b 2 ⎤
or, – , b, a, are in G.P. ∴ b = = =a = ⎢ 2 ⎥
2 2 2 b ⎢⎣ ab + ac + b + bc ⎥⎦
i.e., a = b = c. 2 ⎡ ac + bc + ab + ac ⎤
The correct option is (A, B, C) = ⎢ ab + ac + ac + bc ⎥
b ⎣ ⎦
109. We have, 1 1 2
∴ +  = .
2ab x y b
G= ab , H = and G = 2H.
a+b The correct option is (B, C)
4 ab a b = 4. 111. We have,
∴ ab = or a + b = 4 ab or +
a+b b a log x +log x1/2 + log x1/4 +.....= y
a 1 ⎛ 1 1 ⎞
If = x2, then x + = 4 or x2 + 1 = 4x ⇒ ⎜1 + + + ....⎟ log x....... = y
b x ⎝ 2 4 ⎠
or, x2 – 4x + 1 = 0; ⎛ 1 ⎞
⇒ ⎜ log x = y
4±2 3 ⎝ 1 − 1 / 2 ⎟⎠
∴ x = 4 ± 16 − 4 = =2± 3.
2 2 ⇒ 2 log x = y(1)
Taking the positive sign, 1 + 3 + 5 + ..... + ( 2 y − 1) 20
and, =
2 4 + 7 + 10 + ...... + (3 y + 1) 7 log x
2
x = (2 + 3 ) = (2 + 3)
2

4−3 ⎛ 1 + 2 y − 1⎞
y⎜ ⎟⎠ 20
(2 + 3) 2
(2 + 3) 2 ⎝ 2
= = ⇒ = ⎛ y⎞  [Using (1)]
22 − ( 3 ) 2 ( 2 + 3 )( 2 − 3 ) ⎛ 4 + 3 y + 1⎞ 7⎜ ⎟
y⎜ ⎟⎠ ⎝ 2⎠
⎝ 2
a 2+ 3
∴ = . 2y 40
b 2− 3 ⇒   =  ⇒ 7y2 – 60y – 100 = 0
3y + 5 7 y
a 2− 3
Similarly, taking the negative sign, we get = . 10
b 2+ 3 ⇒ y = 10, – and corresponding x = 105, 10–5/7.
7
The correct option is (A, B)
The correct option is (A, B)
110. Q  a, b, c are in G.P. ∴ b2 = ac (1)
Again, as x is A.M. between a and b, 1 12. Let the G.P. be a, ar, ar2,......
and let the A.P. be a, r + a, r + 2a,...
a+b
∴ x=
(2) Given: a + ar = 9 (1)
2
10
and, y is the A.M. between b and c. and, [ 2r + (10 − 1)a] = 155
2
b+c
∴ y= (3) ⇒ 2r + 9a = 31 (2)
2 9
Putting a = from equation (1) in equation (2), we
a c 2a 2c 1+ r
Consider + = + have,
x y a+b b+c
81
2r + = 31 ⇒ 2r2 –29r + 50 = 0
⎡ a c ⎤ 1+ r
=2 ⎢ + ⎥ [Using (2) and (3)]
⎣a + b b + c⎦ 29 ± 21 25 2
⇒ r = = , 2 and corresponding a = , 3
⎡ ab + ac + ac + bc ⎤ 4 2 3
=2 ⎢ 2 ⎥
⎣ ab + ac + b + bc ⎦ 1 13. We have,

⎡ ab + ac + ac + bc ⎤ ⎡ n( n − 1) 2 n( n − 1)( n − 2) 3 ⎤
=2 ⎢ (1 + 2x2 + x4) ⎢1 + nx + x + x + ....⎥
⎥ = 2 [Using (1)] ⎣ 2 6 ⎦
⎣ ab + ac + ac + bc ⎦
1 1 2 2 = a0 + a1 x + a2 x2 + a3 x3 + ....
Again, + = + Comparing the coefficients, we have
x y a+b b+c
a1 = n
⎡ b+c+a+b ⎤ n( n − 1)
=2 ⎢ 2 ⎥ a2 = 2 +
⎣ ab + ac + b + ac ⎦ 2

Objective_Maths_JEE Main 2017_Ch 10.indd 52 01/01/2008 04:32:03


Sequence and Series  10.53

115. Since t912, t951 and t480 are divisible by 3


n( n − 1)( n − 2)
and, a3 = 2n + ∴ none of them is prime.
6
According to the given condition, we have a1 + a3 = 2a2 1
For t9l, we have t91 = 9.
9...
9
⇒ 3n +
n( n − 1)( n − 2)
= 4 + n (n – 1) 991times
6
⇒ n (n–1) (n–2) = 6 (n2– 4n + 4) =
9
(
1 91 1
9⎣
)
10 − 1 = ⎡(107 )13 − 1⎤

⇒ n3 – 3n2 + 2n = 6n2 – 24n + 24
⎡ (107 )13 − 1⎤ ⎡107 − 1⎤
⇒ n – 9n2 + 26n – 24 = 0
3
= ⎢ ⎥⎢ ⎥
⎢⎣ 10 − 1 ⎥⎦ ⎢⎣ 10 − 1 ⎥⎦
7
∴ n = 2, 3, 4
The correct option is (A, C, D)
= ⎡(107 )12 + (107 )11 + .... + 107 + 1⎤  ⋅
114. Since 3 (a2 + b2 + c2 + 1) – 2 (a + b + c + ab + bc + ca) = 0 ⎣ ⎦
∴ [(a – b)2 + (b –c)2 +(c – a)2 ] + [(a – 1)2 + (b– 1)2  ⎡106 + 105 + .....10 + 1⎤
⎣ ⎦
 + (c –1)2] = 0 = (An integer) (An integer)
⇒ a – b = b– c = c– a = 0 ∴ t91 is not prime.
and, a – 1= b–1 = c – 1= 0 ⇒ a = b = c = 1 The correct option is (A, B, C, D)
⇒ a, b, c are in A. P and G. P. both. Also, they are in H.P.
The correct option is (A, B, C, D)

Passage Based Questions


118. nth term of the given series
1 1
1 16. Here, tn = = 1
1 + 2 + 3 + ... + n n ( n + 1) / 2 =
( n th term of 1,3,5,.......in A.P)( n th term of 3,5,7......in A.P.)
2 (n + 1) − 1
= =2 1
n ( n + 1) n ( n + 1) ∴ Tn =
(1 + ( n − 1) 2) (3 + ( n − 1) 2)
⎛1 1 ⎞
=2 ⎜ − 1
⎝ n n + 1⎟⎠ =
(2 n − 1) (2 n + 1)
Putting n = 1, 2, 3, …, n, we get
1 ⎛ 1 1 ⎞
⎛1 1⎞ ⇒ Tn = ⎜⎝ − ⎟
t1 = 2 ⎜ − 2 2n − 1 2n + 1⎠
⎝ 1 2 ⎟⎠ Putting n = 1, 2, 3, …
⎛ 1 1⎞
t2 = 2 ⎜ − ⎟
1 ⎛ 1 1⎞ 1 1

⎝ 2 3⎠
T1 = ⎜⎝ − ⎟⎠ ∵ –
2 1 3 2n − 1 2n + 1
⎛ 1 1⎞ 1 ⎛ 1 1⎞ 2n + 1 − 2n + 1
t3 = 2 ⎜ − T2 = ⎜⎝ − ⎟⎠ =
⎝ 3 4 ⎟⎠ 2 3 5 (2 n − 1) (2 n + 1)
 1 ⎛ 1 1⎞ 2
T3 = ⎜⎝ − ⎟⎠ = .
⎛1 1 ⎞ 2 5 7 (2 n − 1) (2 n + 1)
tn = 2 ⎜ −
⎝ n n + 1⎟⎠ 
1 ⎛ 1 1 ⎞
⎛ 1 ⎞ 2n Tn = ⎜ − ⎟
Adding, we get Sn = 2 ⎜1 − = 2 ⎝ 2n − 1 2n + 1⎠
⎝ n + 1⎟⎠ n +1
Adding vertically, we get
The correct option is (B)
1 ⎛ 1 ⎞ 1 2n + 1 − 1 n
Sn = ⎜⎝1 − ⎟ = =
2 2 2 2 n + 1⎠ 2 2 n + 1 2n + 1
117. S∞ = lim S n = lim = = 2.
n →∞ n→∞ 1 + 1/ n 1 + 0 n
∴ Sn =
2n + 1
The correct option is (B)
The correct option is (A)

Objective_Maths_JEE Main 2017_Ch 10.indd 53 01/01/2008 04:32:09


10.54  Chapter 10

Multiplying both sides by 9 (1 – r)2, we get


n
119. We have, Sn = ⇒ 44 (1 – r)2 = 27 (1 – r) + 18r
2n + 1
⇒ 44 (1 – 2r + r2) = 27 – 27r + 18r
Dividing numerator and denominator by n, we obtain
⇒ 44 r2 – 79r + 17 = 0
1 1
Sn = . Now when n → ∞, →0 2
2 + 1/ n n ⇒ r = 79 ± ( −79) − 4 × 44 × 17
1 1 2 × 44
∴ S∞ = = .
2+0 2 79 ± 3249 79 ± 57
= =
The correct option is (C) 88 88

120. The series is an arithmetico-geometric series, since each 79 + 57 79 − 57


= ,
term is formed by multiplying corresponding terms of the 88 88
series 136 22 17 1
= , = ,
1, 3, 5, 7, … which is an A.P. 88 88 11 4
1 1 1
and, 1, , , , … which is a G.P. 17
2 Now, > 1 i.e., | r | > 1.
2 2 23 11
Let S denote the sum to infinity of the given series, then 1
3 5 7 But | r | < 1, ∴ r = .
S = 1+ + + + ... (1) 4
2 2 2 23
The correct option is (A)
1
Multiplying both sides by , the common ratio of G.P., 122. The given series is
2
1 1
1 1 3 5 7 3 + (3 + d) + (3 + 2d) +…
we get S= + + 3 + 4 + ... (2) 4 42
2 2 2 2
2 2 1 1
S = 3 + (3 + d ) .
+ (3 + 2d ) . 3 + ....∞ (1)
Subtracting (2) from (1), we get 42 4
Multiplying both sides by 1/4, we get
⎛ 1⎞ ⎛ 3 1⎞ ⎛ 5 3⎞ ⎛ 7 5⎞
⎜ 1 − ⎟ S = 1 + ⎜ − ⎟ + ⎜ 2 − 2 ⎟ + ⎜ 3 − 3 ⎟ + ... 1 1 1 1
⎝ ⎠ 2 ⎝ 2 2⎠ ⎝ 2 2 ⎠ ⎝2 2 ⎠ S = 3 ⋅ + (3 + d ) ⋅ 2 + (3 + 2d ) ⋅ 3 + … ∞
4 4 4 4
⎛ 1 1 ⎞ Subtracting (2) from (1), we get
= 1 + ⎜1 + + 2 + ...⎟
⎝ 2 2 ⎠ ⎛ 1⎞ ⎛1 1 1 ⎞
⎜⎝1 − ⎟⎠ S = 3 + d ⎝⎜ + 2 + 3 + ... ∞⎠⎟
1 1 4 4 4 4
1
S=1+ 1 =1+ 1 =1+2=3 3 1/ 4 d 4
2 1− S=3+d =3+ ∴S=4+ d
2 2 4 1 − 1/ 4 3 9
44
∴ S = 3 × 2 = 6. But S= (Given)
9
The correct option is (B) 44 4
∴ =4+ d
1 21. Let S = 3 + 5r + 7r2 + … ∞(1) 9 9
Multiplying both sides of (1) by r, we get 4 44 44 − 36 8
⇒ d= –4= =
rS = 3r + 5r2 + 7r3 + … ∞(2) 9 9 9 9
Subtracting (2) from (1), we get 8 9
∴ d= × =2
(1 – r) S = 3 + 2r + 2r2 + 2r3 + … to ∞ 9 4
= 3 + 2r (1 + r + r2 + … ∞) Hence, d = 2.
1 2r dr
= 3 + 2r × =3+ a
1− r 1− r Aliter. S∞ = +
1− r (1 − r ) 2
3 2r 44
∴ S= + Here, a = 3. S∞ = (Given), r =
1
1 − r (1 − r )2 9 4
44 44 3 2r 44 3 d Ÿ (1/4) 3 d /4
But S= (Given), ∴ = + . ∴ = + = +
9 9 1 − r (1 − r )2 9 1 − 1/ 4 (1 − r ) 2
3 / 4 ( 3 / 4) 2

Objective_Maths_JEE Main 2017_Ch 10.indd 54 01/01/2008 04:32:17


Sequence and Series  10.55

44 4 1

9
− 4 = d ∴ d = 2.
9
126.
S2n +1 – S2n =
2
( S2n + S2n−1 ) − S2n
The correct option is (B) 1
123. We have, 3 1/3
⋅92/9
⋅ 27 3/27
⋅ 814/81
… to ∞
= − (S − S 2 n −1 )
2 2n
⎛1 2 3 4 ⎞ 2
= = ⎛⎜ − 1 ⎞⎟
+ + + ... to ∞⎟
3⎝⎜ 3 9 27 81 ⎠
⎝ 2⎠
( S2n−1 − S2n− 2 ) and so on.
1 2 3 4
Let S = + + + + … to ∞ Continuing in this manner, we get
3 9 27 81
2 n −1
1
S =
1
+
2
+
8
… to ∞ S2n + 1 –S2n = ⎛ 1 ⎞ (S1 − S2 )
⎜⎝ ⎟⎠
3 9 27 81 2
On subtraction, (A) This recursion relation shows that this choice is not
2 1 1 1 1 1/ 3 1 true.
S = + + + + … to ∞ = = ;
3 3 9 27 81 1 − 1/ 3 2 (C)  is ruled out because S2n + 1 – S2n is alway positive
3 (D)  is ruled out since S2n +1 is strictly greater than S2n.
∴ S = .
4 The correct option is (B)
Hence the given product = 33/4 = 4
27 .
127. From above,
The correct option is (C)
2 n −1
⎛ 1⎞
S + S2 S2n + 1 – S2n = ⎜ ⎟
⎝ 2⎠
(S1 − S2 )
124. and 125. Since S3 = 1 , S2 < S3 < S1
2
which approaches zero as n → ∞
S2 + S3
Again, S4 = ⇒ S2 < S4 < S3< S1
2 ⇒ lim S2 n + 1 = lim S2 n = l (say)
n→∞ n→∞
S + S3
Further S5 = 4 ⇒ S4 < S5 < S3 < S1
2 2Sn + 1 + Sn = Sn + Sn–1 + Sn = 2Sn + Sn –1
Continuing in this manner, we will get
= 2Sn–1 + Sn –2 = 2S2 + S1
S2 < S4 < S6 < S8 <……< S7 < S5 < S3 < S1
On taking limits, we get
⇒ Sequence {S2n} is increasing and {S2n + 1} is decreasing
2l + l = 2S2 + S1
Thus, correct choices of are
1
(a) and (b) respectively. These sequences cannot be ⇒
3
(2S2 + S1 )
l=
unbounded since all terms of the sequence lie between S2
and S1 The correct option is (C)
The correct option is (A)
The correct option is (B)

Match the Column Type


128.
n ⎡ 8⎤
Now, Sn > 1568 ⇒ 40 + ( n − 1) ⎥ > 1568
I .  We have, an = Sn – Sn – 1(1) 2 ⎢⎣ 3⎦
and, an – 1 = Sn – 1 – Sn – 2(2) n 112 + 8n
⇒ ⋅ > 1568
∴ d = an – an – 1 2 3
= (Sn – Sn – 1) – (Sn – 1 – Sn – 2) 6
⇒ n2 + 14n > × 1568 = 1176
[From (1) and (2)] 8
⇒ n2 + 14n – 1176 > 0,
= Sn – 2Sn – 1 + Sn – 2.
or, (n + 42) (n – 28) > 0
The correct option is (C)
As n is positive, n – 28 > 0 i.e., n > 28
2 8 ∴ Minimum value of n = 29.
II.  It is in A.P. for which a = 20, d = 2 =
3 3 The correct option is (A)

Objective_Maths_JEE Main 2017_Ch 10.indd 55 01/01/2008 04:32:22


10.56  Chapter 10

a 1
III.  Since 51 + x + 51 – x, , 25x + 25–x are in A.P., we have 2 2 2 1 1
2 = − = − =
a 3 ⎛ 1⎞ 3 3 3
2 = 51 + x + 51 – x + 25x + 25–x 1− ⎜ − ⎟
2 ⎝ 2⎠

Now, put 5x = t so that t > 0, we then have 1 2 2
∴ S= × =
5 2 1 ⎛ 1⎞ ⎛ 1⎞ 3 3 9
a = 5t + + t + 2 = ⎜t 2 + ⎟ + 5 ⎜t + ⎟ The correct option is (A)
t t ⎝ t ⎠
2 ⎝ t⎠
III. Let S = (1 + 3–1) (1 + 3–2) (1 + 3–4) (1 + 3–8) …
⎛ 1⎞ ⎡⎛ 1⎞
2 ⎤ 2 −2 n

or, a = ⎜ t − ⎟ + 2 + 5 ⎢⎜ t − + 2⎥  (1+ 3 )
⎝ t⎠ ⎟
⎢⎣⎝ t⎠ ⎥⎦ ⇒ (1 – 3– 1) S
= (1 – 3– 1) (1 + 3– 1) (1 + 3– 2) (1 + 3– 4)
2 2
= ⎛⎜ t − 1⎞⎟ + 5 ⎛⎜ t − 1 ⎞⎟ + 12 ≥ 12. (1 + 3– 8) … (1 + 3−2 )
n


⎝ t⎠ ⎝ t⎠ 2
⇒ S = (1 – 3–2) (1 + 3–2) (1 + 3–4) (1 + 3–8) … 
Thus, values of a are given by the inequality a ≥ 12. 3 −2n
The correct option is (D) ( 1 + 3 )
= (1 – 3–4) (1 + 3–4) (1 + 3–8) … (1 + 3−2 )
n


n
IV.  log 2 a + log 2 a + log2 a + log 2 a + …
1/2 1/4 1/6 18
/
= (1 – 3–8) (1 + 3–8) … ( 1 + 3−2 )
= 2 log2 a + 4 log2 a + 6 log2 a + … + 40 log2 a −2n
= (1− 3 ) (1 + 3−2 ) = 1 – ( 3−2n )2
n


= log2 a [2 + 4 + 6 + … + 40] n +1

= 1 – 3−2 .
20
= (2 + 40) log2 a 3 n +1
2 ⇒ S= (1 − 3−2 )
= 420 log2 a = 840 2
⇒ log2 a = 2 ⇒ a = 4 3 n +1
∴ lim S = lim (1 − 3−2 )
The correct option is (B) n→∞ n→∞ 2

1 29. I.  Let the G.P. be 1, r, r2, r3, … 3 3


=
(1 – 0) = .
Given: tn = 2 (tn + 1 + tn + 2 + tn + 3 + … to ∞) 2 2
t The correct option is (B)
or, tn = 2 ⋅ n+1 , (∵ common ratio = r)
1− r
⎛ ⎞
∑ ⎜ ∑ m2 ⎟ = ∑1(1 + 2 + .... + k )
n k n
1− r t 1− r IV.  2 2 2
∴ = n +1 ; or =r
2 tn 2 r =1 ⎝ m = 1 ⎠ k=

1
or, 1 – r = 2r ; ∴ r = . 1 ⎪⎧ n( n + 1) 2 ⎪⎫ 1 ⎧ n( n + 1)( 2n + 1) ⎫
3 = ⎨ ⎬+ ⎨ ⎬
The correct option is (D) 3 ⎩⎪ 2 ⎭⎪ 2 ⎩ 6 ⎭
1 ⎛ n( n + 1) ⎞
2 5 2 11 + ⎜ ⎟
 II. Let S = − + − + ... to ∞(1) 6⎝ 2 ⎠
3 6 3 24

Multiplying both sides by – , the common ratio of


2
1 =
2
{
1 4
n + 4 n3 + 5 n 2 + 2 n }
G.P.
1 2 5 8 ∴ a = 1 ,b = 1 ,c = 5 ,d = 1 ,e = 0
S=– +
– − + ... to ∞(2) 12 3 12 6
2 6 12 24
So, a + b + c + d + e = 1
Subtracting (2) from (1), we have
The correct option is (C)
3
S = 2 − 3 + 3 − 3 + ... to ∞ 130.
I. Since a, b, c are in A.P.
2 3 6 12 24 a+c
∴b= (1)
2 ⎛1 1 1 ⎞ 2
= − ⎜ − + + ... to ∞⎟
3 ⎝2 4 8 ⎠ ∵ b, c, d are in G.P.

Objective_Maths_JEE Main 2017_Ch 10.indd 56 01/01/2008 04:32:28


Sequence and Series  10.57

∴ c2 = bd (2)
a + 2d 1 a
Also, Q c, d, e, are in H.P. i.e. , , ,
2a ( a + d ) 2( a + d ) 2 ( a + 2d ) ( a + d )
2ce
∴ d=
(3) Product of first and third
c+e
Substituting the values of b and d from (1) and (3) a + 2d a
= ⋅
respectively in (2), we get 2a ( a + d ) 2 ( a + 2d ) ( a + d )
a+c e ( a + c) 2
c2 = ⋅ or c = 1 ⎡ 1 ⎤
2 c+e = = ⎢ ⎥
4 (a + d ) 2 ⎣ 2 ( a + d ) ⎦
⇒ c2 + ce = ae + ce
⇒ c2 = ae, which shows that a, c, e are in G.P. ∴ New numbers are in G.P.
The correct option is (B) The correct option is (B)

II. ∵ 2 (y – a) is H.M. between y – x and y – z Solving the equation ax2 + 2bx + c = 0, we get
IV. 
2 1 1
∴ = + −2b ± 4b 2 − 4 ac − b ± b 2 − ac b
2( y − a) y−x y−z x= = =–
2a a a
1 1 [ Q a, b, c are in G.P., ∴ b2 = ac]
= + (1)
( y − a) − ( x − a) ( y − a) − ( z − a) This is also root of
Let x – a = X; y – a = Y; z – a = Z dx2 + 2ex + f = 0
then (1) reduces to 2
∴ ⎛ −b ⎞ ⎛ −b ⎞
1 1 1 d ⎜ ⎟ + 2e ⎜ ⎟ + f = 0
= + ⇒ Y2 = ZX ⎝ a⎠ ⎝ a⎠
Y Y − X Y −Z ⇒ db2 – 2eba + a2 f = 0
∴ X, Y, Z are in G.P. or x – a, y – a, z – a are in G.P.
⇒ dac – 2eba + a2 f = 0 [ Q b2 = ac]
The correct option is (B)
Dividing throughout by a
III. Let the numbers in H.P. be ∴ dc – 2eb + af = 0 ⇒ 2eb = dc + af
1 1 1 Dividing both sides by b
, ,
a a + d a + 2d 2e dc + af 2e dc + af
⇒ = ⇒ = [ b2 = ac]
1 b b2 b ac
Then, the numbers obtained by subtracting
are 2 (a + b ) 2e d f e d f e
⇒ = + ⇒ − = −
b a c b a c b
1 1 1 1 1 1
− , − , −
a 2 (a + d ) a + d 2 (a + d ) a + 2 d 2 ( a + d ) d e f
⇒ , , are in A.P.
a b c
2a + 2d − a 1 2a + 2d − ( a + 2d ) The correct option is (A)
i.e., , ,
2a (a + d ) 2 (a + d ) 2 ( a + 2d ) ( a + d )

Assertion-Reason Type
131. Let 2n arithmetic means be A1, A2, A3, …, A2n between a 132. Since a and b are unequal,
and b.
a+b a2 + b 2
Then, A1 + A2 + A3 + … + A2n = × 2n > a 2b 2 [A.M. > G.M. for unequal numbers]
2 2
13
13n ⇒ a2 + b2 > 2ab
= 6 × 2n =
2 6 Similarly, b2 + c2 > 2bc and c2 + a2 > 2ca
Given, A1 + A2 + A3 + … + A2n = 2n + 1; Hence, 2 (a2 + b2 + c2) > 2 (ab + bc + ca)
13n
∴ 2n + 1 = ; or 12n + 6 = 13n; ∴ n = 6. ⇒ ab + bc + ca < 1
6
∴ The number of means = 2n = 2 × 6 = 12. The correct option is (A)
The correct option is (A)

Objective_Maths_JEE Main 2017_Ch 10.indd 57 01/01/2008 04:32:35


10.58  Chapter 10

135. We know that sum of n A.M.’s between two quantities is


a+c equal to n times their single mean.
133. b = H.M. of a and c > A.M. of a and c =
2 Now, x, y, z are three A.M.’s between a and b
a+c b +d
⇒ b > . Similarly, c > . Adding we get
2 2 x + y + z = 3 ⎜
⎛ a + b ⎞ = 15
b + c > a + d. ⎝ 2 ⎟⎠
The correct option is (A) or, a + b = 10 (1)
134. Let Tn be the nth term of the series a, x, y, z, b are in H.P.

1 2 3 1 1 1 1 1
∴ , , , , are in A.P.
+ + +… a x y z b
1 + 12 + 14 1 + 22 + 2 4 1 + 32 + 34
1 1 1 3 ⎛ 1 1⎞ ⎛ a+b⎞
∴ + + = ⎜ + ⎟ =3 ⎜
Then, Tn =
n
=
n x y z 2 ⎝ a b⎠ ⎝ 2ab ⎟⎠
1 + n2 + n4 (1 + n 2 )2 − n 2
5 3
n or, = ⋅10 , by (1) ∴ ab = 9 (2)
=
3 2ab
( n 2 + n + 1) ( n 2 − n + 1) Hence, a and b are the roots of
t2 – 10 t + 9 = 0 [Using (1) and (2)]
1⎡ 1 1 ⎤
= − ⇒ t = 9, 1
2 ⎢⎣ n2 − n + 1 n2 + n + 1 ⎥⎦
∴ 9, 1 are the required values of a and b.
1⎡ 1 1 ⎤ The correct option is (A)
= ⎢ − ⎥
2 ⎣1 + ( n − 1)n 1 + n( n + 1) ⎦ 136. Since A. M. > G.M. for n distinct positive quantities, we
Now, have,
n
1 ⎡1 1 ⎤ 1⎡ 1 1 ⎤ 13 + 23 + 33 + ..... + n3
∑ Tr ( )
1/ n
= − + − > 13. 23. 33 .....n 3
r =1
2 ⎢⎣1 1 + 1.2 ⎥⎦ 2 ⎢⎣1 + 1.2 1 + 2.3 ⎥⎦ n
2
⎛ n( n + 1) ⎞
> (1.2.3....n) = (n!)3/n
3/ n
1⎡ 1 1 ⎤ ⇒ ⎜
+ − +… ⎝ 4 ⎟⎠
2 ⎣1 + 2.3 1 + 3.4 ⎥⎦

2n

⇒ (n!)3 < n n ⎛
n + 1⎞
1⎡ 1 1 ⎤ ⎜⎝ 2 ⎟⎠
+ −
2 ⎢⎣1 + ( n − 1)n 1 + n( n + 1) ⎥⎦
The correct option is (A)
1⎡ 1 ⎤ n( n + 1)
= ⎢1 − ⎥ = .
2 ⎣ 1 + n( n + 1) ⎦ 2( n 2 + n + 1)

The correct option is (A)

Previous Year’s Questions

1 3
137. Q1,log3 31− x + 2 ,log3 ( 4 ⋅ 3x − 1) are in AP ⇒ t=− ,
3 4
\ 2 log3 (31− x + 2)1/ 2 = log3 3 + log3 ( 4 ⋅ 3x − 1) 3
⇒ 3x = (neglecting the negative value)
1− x 1/ 2 x 4
⇒ log3 (3 + 2) = log3 3( 4 ⋅ 3 − 1)
⎛ 3⎞
Let 3x = t, then ⇒ log3 ⎜ ⎟ = x
⎝ 4⎠
3
+ 2 = 12t − 3 ⇒ x = log3 3 − log3 4
t
⇒ 12t2− 5t − 3 = 0 ⇒ x = 1 − log3 4
⇒ (3t + l)(4t− 3) = 0
The correct option is (B)

Objective_Maths_JEE Main 2017_Ch 10.indd 58 01/01/2008 04:32:42


Sequence and Series  10.59

138. The product 21/4 ⋅ 41/4 ⋅ 81/16 … 142. f (1) = 7


=2 1/ 4
⋅2 2/8 3 /16
⋅2 ... f (1 + 1) = f (1) + f (1)

⇒ f (2) = 2 ×7
1⎡ 2 3 ⎤
1+ ⋅ + ...⎥
4 ⎢⎣ 2 22 ⎦
Also, f (3) = 3 ×7
=2
n


1 ⎤
⎥ \ ∑ f ( r ) = 7(1 + 2 + ......... + n)
1⎢ 1
+ 2 2⎥ r =1
4 ⎢ 1 ⎛ 1⎞ ⎥
⎢1− 1− ⎥
2 ⎜⎝ 2 ⎟⎠ ⎥⎦ n( n + 1)
= 2 ⎢⎣ =7
2
1
[ 2 + 2] The correct option is (D)
= 24 n
r
=2 143. Given t n = ∑ nC
r=0 r
The correct option is (B)

(Q C )
n n
139. Since fifth term of a GP = 2 n−r n−r
Also, t n = ∑ nC = ∑ n
n
r = nCn − r
∴ ar4 = 2 r=0 n−r r=0 Cr
where a and r are the first term and the common ratio Adding above two equalities we write
respectively of a GP. n n
r+n−r n
Now required product 2t n = ∑ n
Cr
= ∑ nC
r=0 r=0
= a × ar × ar2 × ar3 × ar4 × ar5 × ar6 × ar7 × ar8 r
n
= a9r36 = (ar4)9 = 29 = 512 n 1 n t n
The correct option is (B)
⇒ tn = ∑ = sn ⇒ Sn = 2
2 r = 0 nCr 2
n
140. Key Idea : The number of triangles those can be formed The correct option is (A)
using n points = nC3
1
QTn = nC3 144. Given that Tm = = a + ( m − 1)d (1)
n
\ Tn +1 − Tn = 21 1
And Tn = = a + ( n − 1)d (2)
⇒ n +1C3 − nC3 = 21 m
1 1
⇒ nC2 + nC3 − nC3 = 21 From (1) and (2) we get a = ,d=
mn mn
Hence, a − d = 0
(Q nC2 + nC3 = n −1C3 )
The correct option is (A)
⇒ nC2 = 21
145. If n is odd then (n − 1) is even and so the sum of odd terms
n( n − 1) ( n − 1)n2 n2 ( n + 1)
⇒ = 21 ⇒ n2 − n − 42 = 0 = + n2 = .
2 2 2
⇒ ( n − 7)( n + 6) = 0 The correct option is (D)

⇒n=7 (Q n ≠ −6) ∞
1 1
The correct option is (B) 146. x= ∑ an = 1 − a ⇒a = 1−
x
n= 0
1 1 1 ∞
141. − + − .......... 1 1
1.2 2.3 3.4 y= ∑ bn = 1 − b ⇒ b = 1 − y
n= 0
1 1 1 1 1
= 1− − + + − − ......... ∞
1 1
2 2 3 3 4 z= ∑ cn = 1 − c ⇒ c = 1 − z
⎛1 1 1 ⎞ n= 0
= 1 − 2 ⎜ − + − ........⎟ (Rearranging the terms) a, b, c are in A.P.
⎝2 3 4 ⎠
  2b = a + c
⎛ 1 1 1 ⎞
= 2 ⎜1 − + − + ........⎟ − 1 ⎛ 1⎞ 1 1
⎝ 2 3 4 ⎠ 2 ⎜1 − ⎟ = 1 − + 1 −
⎝ y⎠ x y
= 2 log 2 − log e
2 1 1
⎛ 4⎞ = +
= log ⎜ ⎟ . y x z
⎝ e⎠
⇒ x, y, z are in H. P
The correct option is (D) The correct option is (D)

Objective_Maths_JEE Main 2017_Ch 10.indd 59 01/01/2008 04:32:47


10.60  Chapter 10

147. Given condition implies that ⇒ r2 = 4  if r ≠ −1


p ∴ r = −2
[2a + ( p − 1)d ] p 2 2a + ( p − 1)d p
2 1 also, a = −12  (using (1))
= 2 ⇒ 1 =
q
[2a1 + ( q − 1)d ] q 2a1 + ( q − 1)d q The correct option is (B)
2
2 6 10 10
⎛ p − 1⎞ 152. Let S = 1 + + + + + ......  (1)
a1 + ⎜ d 3 32 33 34
⎝ 2 ⎟⎠ p
=
⎛ q − 1⎞ q 1 1 2 6 10
S = + 2 + 3 + 4 + ....  (2)
a1 + ⎜ d
⎝ 2 ⎟⎠ 3 3 3 3 3

a6 Subtracting (2) from (1), we get
For , p = 11, q = 41,
a21 ⎛ 1⎞ 1 4 4 4
S ⎜1 − ⎟ = 1 + + 2 + 3 + 4 + ....
a6 11 ⎝ 3⎠ 3 3 3 3
=
a21 41

2 4 4⎛ 1 1 ⎞
The correct option is (D) S = + 2 ⎜1 + + 2 + .... ⎟
3 3 3 ⎝ 3 3 ⎠
1 1 1 1 1 1 ⎛ ⎞
148. Given that − = − = .... = − = d ( say )
a2 a1 a3 a2 an an −1 2 4 4⎜ 1 ⎟ 4 4 3 4 2 6 2 6
⇒ S= + 2⎜ ⎟= + = + = ⇒ S=
3 3 3 ⎜ 1 − 1 ⎟ 3 32 2 3 3 2 3 3
a1 − a2 a −a a −a ⎜ ⎟
Then a1a2 = , a2 a3 = 2 3 ,...., an−1an = n−1 n ⎝ 3⎠
d d d
⇒S =3
a1 − an The correct option is (B)
\ a1a2 + a2 a3 + .... + an−1an =
d
1 1 153. Till 10th minute number of counted notes = 1500
Also, = + ( n − 1)d
an a1 n
3000 = [2 × 148 + ( n − 1)( −2)] = n[148 − n + 1]
a1 − an 2
⇒ = ( n − 1)a1an
d n2−149n + 3000 = 0
The correct option is (D) n = 125, 24
1 49. Given that arn-1 = arn + arn + 1 n = 125 is not possible.
⇒ 1 = r + r2 Total time = 24 + 10 = 34 minutes.
5 −1 The correct option is (A)
\r =
2 154. 1 2 3 4 5 6 ……
The correct option is (D) 200 200 200 240 280 ……. ……..
150. Using the condition A.M. ≥ G.M., we write Sum = 11040
p2 + q2 120 + 80 + 160 + 40 + 200 + 240 + … = 11040
≥ pq
n
2
2
[2a + ( n − 1)d ] + 80 + 40 = 11040
1
⇒ pq ≤
2 n
(p + q)2 = p2 + q2 + 2pq 2
[240 + ( n − 1)40] = 10920

⇒ p+q≤ 2 n [6 + n − 1] = 546

n (n + 5) = 546
The correct option is (D)
n = 21
1 51. Let a, ar, ar2, ar3 be the first four terms of a G.P., then
a + ar = 12    (1) The correct option is (C)
ar2 + ar3 = 48  (2) 155. Statement 1 has 20 terms whose sum is 8000
dividing (2) by (1), we have And statement 2 is true and supporting statement 1.
ar 2 (1 + r ) th 2 2 2
Q k bracket is (k – 1) + k(k – 1) + k = 3k – 3k + 1.
=4
a( r + 1) The correct option is (B)

Objective_Maths_JEE Main 2017_Ch 10.indd 60 01/01/2008 04:32:51


Sequence and Series  10.61

156. 100(T100 ) = 50(T50 ) 9 1 8 9


160.
S = 10 + 2 ⋅ 11 ⋅ 10 + ... + 10 ⋅ 11

⇒ 2[a + 99d ] = a + 49d 11


⋅ S = 111 ⋅ 108 + ... + 9 ⋅ 119 + 1110
⇒ a + 149d = 0 10

1
⇒ T150 = 0 ⇒ S = 109 + 111 ⋅ 108 + 112 ⋅ 107 + ... + 119 − 1110
10

The correct option is (D)
⎛ ⎛ 11 ⎞10 ⎞
157. tr = .7 + .77 + . . .r times ⎜ ⎜ ⎟ −1 ⎟
1
⇒ S = 109 ⎜ ⎝ ⎠
10 ⎟ − 1110

(
= 7 10 −1 + 10 −2 + 10 −3 + ..... + 10 − r ) 10 ⎜ 11
⎜ −1 ⎟

⎜ 10 ⎟
⎝ ⎠


7
9
(
1 − 10 − r
= ) 1 10 10
⇒ S = 11 − 10 − 11 10

Now, 10
11
7⎛ ⎞   S = 10
20 20
S20 = ∑ t r = ⎜⎜ 20 − ∑10 − r ⎟⎟
9 9
r =1 ⎝ r =1 ⎠   S = 100 ⋅ 10
7⎛ 1
= ⎜ 20 − 1 − 10
9⎝ 9
( −20 ⎞
)7
(
⎟ = 81 179 + 10

−20
)   ⇒ k = 100 .
The correct option is (C)

The correct option is (B)
161. tr =
∑ r 3 = r 2 ( r + 1)2 = 1 (r + 1)2
158.
1 1
+ =4 ∑ (2r − 1) 4r 2 4
a b
1 9
2q = p + r Now, S9 = ∑ ( r + 1)2 , let t = r + 1
4 r =1
⇒ −2(a + b ) = 1 + ab
1 ⎛ 10 2 ⎞
⎛ 1 1⎞
⇒ −2 ⎜ + ⎟ =
1
+1
= ∑ t − 1⎟⎠ = 96 .
4 ⎜⎝ t =1
⎝ a b ⎠ ab  
The correct option is (A)
1
⇒ = −9 162. Let ‘a’ be the first term and d be the common difference
ab
2nd term = a + d, 5th term = a + 4d,
Equation having roots a , b is 9 x 2 + 4 x − 1 = 0 9th term = 4 + 8d
−4 ± 16 + 36 a + 4 d a + 8d 4 d 4
a,b = ∴ Common ratio = = = =
2×9 a+d a + 4 d 3d 3
2 13 The correct option is (C)
a −b = .
9
82 122 16 2
The correct option is (D) 163. Given series is S = + + + …10 terms
52 52 52

159. Let numbers be a, ar , ar 2 42


= ( 22 + 32 + 4 2 + …10 terms)
52
Now, 2( 2ar ) = a + ar 2
[a ≠ 0]
16 ⎛ 11.12.23 ⎞ 16
⇒ 4r = 1 + r 2 = ⎜ − 1⎟ = × 505
25 ⎝ 6 ⎠ 25
⇒ r 2 − 4r + 1 = 0 \ m = 101
⇒r = 2± 3 The correct option is (C)

r = 2 + 3 (Positive value)
The correct option is (D)

Objective_Maths_JEE Main 2017_Ch 10.indd 61 01/01/2008 04:32:57


Limits 11.1

CHaPtER

11 Limits

Chapter Highlights
Limit of a function, Indeterminate forms, Algebra of limits, Evaluation of limits, Algebraic limits, Limit of an
algebraic function when x → ∞, Trigonometric limits, Exponential and logarithmic limits, Evaluation of limits
using L’Hospital’s rule

lImIt oF a FunctIon method for Finding right hand limit


Let a function f be defined at every point in the To evaluate lim f ( x )
x → a+
neighbourhood of a (an open interval about a) except pos-
sibly at a. If as x approaches closer and closer to a, but not 1. Put x = a + h in f (x) to get
equal to a, then the value of the function f (x) approaches a 2. Take the limit as h → 0.
real number l. The number l is referred to as the limit of f (x)
as x tends to a and we write it as left hand limit
lim f ( x ) = l We say that left hand limit of f (x) as x tends to ‘a’ exists
x→a and is equal to l2 if as x approaches ‘a’ through values less
Note that f (x) approaches l means the absolute difference than ‘a’, the values of f (x) approach a definite unique real
between f (x) and l, i.e. |f (x) – l| can be made as small as number l2 and we write
we please.
When the values of f (x) do not approach a single lim f ( x ) = l2
x→ a-
finite value as x approaches a, we say that the limit Does
or f (a – 0) = l2
not exist.

caution method for Finding left hand limit


To evaluate lim f ( x ).
A number is said to be a limiting value only if it is finite and x→ a-
real, otherwise we say that the limit does not exist. 1. Put x = a – h in f (x) to get lim f ( a + h).
h→ 0
2. Take the limit as h → 0.
right hand limit
We say that right hand limit of f (x) as x tends to ‘a’ exists caution
and is equal to l1 if as x approaches ‘a’ through values
greater than ‘a’, the values of f (x) approach a definite For finding lim f( x ), we study the behaviour of the function f
x →a
unique real number l1 and we write in the neighbourhood of ‘a’ and not at ‘a’. Thus, the function
f may or may not be defined at x = a.
lim f ( x ) = l1 or f (a + 0) = l1
x → a+

Objective_Maths_JEE Main 2017_Ch 11.indd 1 01/01/2008 05:00:54


11.2  Chapter 11

and x – ai > 0  for  i = 1, 2, …, m – 1


Solved Examples
x - ai
x 2 - 9 x + 20 \ Ai = = –1, for i = m, m + 1, …, n
1. lim = - ( x - ai )
x→5 x - [ x]
x - ai
(A) 1 and Ai = = 1, for i = 1, 2, …, m – 1
x - ai
(B) 0
(C)  Does not exist Similarly, if x is in the right neighbourhood of ai
(D)  Cannot be determined Then x – ai < 0 for i = m + 1, …., n and x – ai > 0
for i = 1, 2, …, m
Solution: (C) x - ai
\ Ai = = – 1 for i = m + 1, … n
x 2 - 9 x + 20 ( x - 4)( x - 5) - ( x - ai )
lim = lim
x→5 x - [ x] x→5 x - [ x] x - ai
and Ai = = 1 for i = 1, 2, …, m
( x - 4)( x - 5) x - ai
LHL = lim
x → 5- x - [ x]  Now, lim ( A1 A2 ... An ) = (–1)n – m + 1
x → am-
(5 - h - 4)(5 - h - 5)
= lim (h > 0)
h→0 (5 - h) - [5 - h] and lim ( A1 A2 ... An ) = (–1)n – m
x → am+
(1 - h)( - h) (1 - h)( - h) Hence, lim ( A1 A2 ... An ) Does not exist.
= lim = lim
h→0 5-h-4 h→0 (1 - h)  x → am

\ LHL = 0 Indeterminate Forms


( x - 4)( x - 5)
Also, RHL = lim If a unique value cannot be assigned to f (a), then f (x) is
x→5 x - [ x]  said to be inderminate at x = a.
(5 + h - 4)(5 + h - 5) 0
= lim Most general of all indeterminate forms is , others
h→0 (5 + h) - [5 + h]  being 0
(1 + h)( h) (1 + h)h 1 1 0-0 0
= lim = lim 1. ∞ – ∞ = - = = which is indeterminate
h→0 5+h-5 h→0 h 0 0 0 0

and hence is (∞ – ∞)
\ RHL = 1
∞ 1/ 0 0
As LHL ≠ RHL 2. = = which is indeterminate and hence is
∞ 1/ 0 0
\ Limit does not exist. ⎛ ∞⎞
x - ai ⎜⎝ ⎟⎠

2. If Ai = , i = 1, 2, …, n and if a1 < a2 < a3 < … < an.
| x - ai | 1 0
3. 0 × ∞ = 0. = which is indeterminate and hence is
Then lim ( A1 A2 ... An ) , 1 ≤ m ≤ n 0 0
x → am (0 × ∞)
(A)  is equal to (–1)m 4. 1∞

(B)  is equal to (–1)m + 1 Let y = 1∞
(C)  is equal to (–1)m – 1 ⇒ log y = ∞ log 1 = ∞ × 0 which is indeterminate

(D)  Does not exist and hence is 1∞

Solution: (D) 5. 00
x - ai Let y = 00
We have, Ai = , i = 1, 2, …, n ⇒ log y = 0 ⋅ log 0 = 0 × ∞ which is indeterminate
| x - ai |
and hence is 0º
and a1 < a2 < … an – 1 < an. 6. ∞º
Let x be in the left neighbourhood of am. Let y = ∞º
Then, ⇒ log y = 0 ⋅ log ∞ = 0 × ∞ which is indeterminate
x – ai < 0 for i = m, m + 1, … n and hence is ∞º.

Objective_Maths_JEE Main 2017_Ch 11.indd 2 01/01/2008 05:00:58


Limits 11.3

algebra oF lImItS Solution: (B)


Since, 0 ≤ (rx) < 1 for r = 1, 2, 3, …, n
If lim f ( x ) = l and lim g ( x ) = m, then following results n n
x→a x→a
are true: ⇒ 0≤ ∑ (rx) < ∑ (1)
r =1 r =1
1. lim [ f ( x ) + g ( x )] = lim f ( x ) + lim g ( x ) = l + m. n
x→a x→a x→a ⇒ 0≤ ∑ (rx) < n
2. lim [ f ( x ) - g ( x )] = lim f ( x ) – lim g ( x ) = l – m. r =1
x→a x→a x→a
Dividing throughout by n2, we have
3. lim k ⋅ f ( x ) = k ⋅ lim f ( x ) = kl, n
x→a

where k is a constant.
x→a
∑ (rx)
0 r =1 1
≤ <
4. lim [ f ( x ) ⋅ g ( x )] = lim f ( x ) ⋅ lim g ( x ) = lm. n 2
n 2 n
x→a x→a x→a n

⎡ f ( x) ⎤ x → a
lim f ( x )
l ∑ (rx)
5. lim ⎢ ⎥ = = (provided m ≠ 0). r =1 1
x → a ⎣ g ( x) ⎦ lim g ( x ) m ⇒ lim 0 ≤ lim < lim
x→a n→∞ n→∞ n2 n→∞ n
⎛ ⎞ n
6. lim ( fog ) ( x ) = lim f [ g ( x )] = f ⎜ lim g ( x )⎟ =
x→a x→a ⎝ x→a ⎠ ∑ (rx)
f (m). r =1
⇒ 0 ≤ lim <0
n→∞ n2
In particular,
( x ) + (2 x ) + ... + ( nx )
⎛ ⎞ \ 0 ≤ lim <0
(a) lim log g ( x ) = log ⎜ lim g ( x )⎟ = log m. n→∞ n2
x→a ⎝ x→a ⎠
lim g ( x) According to Sandwich Theorem or Squeeze Principle
(b) lim e g ( x ) = e x→a
= em.
x→a ( x ) + ( 2 x ) + ... + ( nx )
n lim =0
⎡ ⎤ n→∞ n2
7. lim[ f ( x )]n = ⎢ lim f ( x ) ⎥ = ln, for all n ∈ N.
x→a ⎣ x → a ⎦
8. Sandwich Theorem (or Squeeze Principle). caution
If f, g and h are functions such that f (x) ≤ g (x) ≤ h (x)
for all x in some neighbourhood of the point a (except The converse of the above result may not be true,
possibly at x = a) and if lim f ( x ) = l = lim h ( x ) , i.e., lim f( x ) = |l| = |l| ⇒ lim f( x ) = l
x→a x→a x →a x →a
then lim g ( x ) = l.
x→a

9. If lim f ( x ) = l, then lim f ( x ) = |l |.


x→a x→a
evaluatIon oF lImItS
i M P o R ta n t P o i n t S The problems on limits can be divided into the following
categories:
Sandwich theorem helps in calculating the limits, when limits
cannot be calculated using the usual method Limits

Solved example Algebraic Trigonometric Exponential and


limits limits logarithmic limits
( x ) + ( 2 x ) + (3 x ) + ... + ( nx )
3. lim , where
n→∞ n2
{x} = x – [x] denotes the fractional part of x, is
algebraIc lImItS
(A) 1 (B) 0
1 The following methods are useful for evaluating limits of
(C) (D) None of these algebraic functions:
2

Objective_Maths_JEE Main 2017_Ch 11.indd 3 01/01/2008 05:01:05


11.4  Chapter 11

Method of Factorization Solution: (A)


⎡ x -3 ⎤
If f (x) and g (x) are polynomials and g (a) ≠ 0, then we have lim ⎢log a ⎥
x →3 ⎣ x + 6 - 3⎦
lim f ( x )
f ( x) x→a f ( a)
lim = = . ⎡ ( x - 3) ( x + 6 + 3) ⎤
x→a g ( x) lim g ( x ) g ( a) = lim ⎢log a ⎥
x→a x →3
⎣ ( x - 3) ⎦
Now, if f (a) = 0 = g (a), then (x – a) is a factor of both f (x)
and g (x). We cancel this common factor (x – a) from both = lim log a ( x + 6 + 3) = loga 6
x →3
the numerator and denominator and again put x = a in the
given expression. If we get a meaningful number then that Standard Formula
number is the limit of the given expression, otherwise we
repeat this process till we get a meaningful number. x n - an
lim = nan – 1,
x→a x-a

Solved Example where n ∈ Q, the set of rational numbers.

3
x2 - 2 3 x + 1 Solved Example
4. lim 2
is equal to
( x - 1)
x →1
x r - 1r
n
1 1 6. lim ∑ =
(A)  (B)  r =1 x - 1
x →1
9 6 n( n + 1)
(A)  0 (B) 
1 2
(C)  (D)  None of these
3 (C)  1 (D)  None of these
Solution: (A) Solution: (B)
3
x2 - 2 3 x + 1 y2 - 2 y + 1
n
x r - 1r
lim = lim We have, lim ∑
r =1 x - 1
x →1

x →1 ( x - 1) 2 y →1 ( y 3 - 1) 2
 
n
n( n + 1)
[Putting 3 x = y; as x → 1, y → 1] = ∑ r ⋅1 r-1
=1+2+3+…+n=
2
.
r =1
( y - 1) 2
= lim
- 1) 2 ( y 2 + y + 1) 2 
y →1 ( y
Limit of an Algebraic Function
1 1 when x → ∞
= lim 2 = .
y →1 ( y + y + 1) 2 9 f ( x)
In order to find the limit of a function of the type as
g ( x)
Method of Rationalization x → ∞, where f (x) and g (x) are algebraic functions of x, it
is convenient to divide all the terms of f (x) and g (x) by the
This method is useful where radical signs (i.e., expressions
highest power of x in numerator and denominator both and
of the form a ± b ) are involved either in the numera-
use the following standard limits:
tor or in the denominator or both. The numerator or (and)
1
the denominator (as required) is (are) rationalised and limit 1. lim =0
x →∞ x
taken after cancelling out the common factors.
1
2. lim p = 0, if p > 0.
x →∞ x
Solved Example
Trick(s) for Problem Solving
⎛ x -3 ⎞
5. The value of lim ⎜ log a ⎟ is
x →3 ⎝ x + 6 - 3⎠ ⎧ ∞, if a > 1
⎪ if a = 1
(A) loga 6 ⎪ 1,
(B) loga 3
 lim a n = ⎨
n→∞ ⎪ 0, if - 1 < a < 1
(C) loga 2 ⎪⎩does not exist, if a ≤ - 1
(D)  None of these

Objective_Maths_JEE Main 2017_Ch 11.indd 4 01/01/2008 05:01:08


Limits  11.5

1 n 3
p
a0 x + a1x p -1
+ … … + ap - 1 x + ap
\ S= ∑ [r - 2nr 2 + (n2 - 2n - 1)r + n2 ] 
2 r =1
 lim
q q -1
x→∞ b0 x + b1x + … … + bq - 1 x + bq 2
1 ⎡ ⎧ n( n + 1) ⎫ ⎧1 ⎫
⎧ a0
⇒ S= ⎢ ⎨ 2 ⎬ - 2n ⎨ 6 n ( n + 1)( 2n + 1)⎬
if p = q 2 ⎣⎩ ⎭ ⎩ ⎭
⎪b ,
= ⎪⎪0,
0
⎧1 ⎫ ⎤
⎨ if p < q + ( n2 - 2n - 1) ⎨ n( n + 1)⎬ + n2 ( n) ⎥
⎪∞, if p > q  ⎩ 2 ⎭ ⎦

⎪⎩ Solving and rearranging, we have
1 4
S = ( n - 11n3 - 19n2 + 6 n) 
24
Some Useful Summations S 1 ⎛ n4 - 11n3 - 19n2 + 6 n ⎞
∴ lim 4 = lim ⎜ ⎟
n ( n + 1) n→∞ n n → ∞ 24 ⎝ n4 ⎠
1. S n = 1 + 2 + 3 + … + n =
2 1 ⎛ 11 19 6 ⎞
2 2 2 2 2 n ( n + 1) ( 2n + 1) = lim ⎜1 - - 2 + 3 ⎟ 
2. S n = 1 + 2 + 3 + … + n = 24 n → ∞ ⎝ n n n ⎠
6
n -1 ⎧ n-r ⎫⎪
⎡ n ( n + 1) ⎤
2 1 ⎪ 1
3. S n3 = 13 + 23 + 33 + … + n3 = ⎢ ⎥
∴ lim
n→∞ n 4 ∑ ⎨⎪(r + 1) ∑ k ⎬⎪ =
24

⎣ 2 ⎦ r =0 ⎩ k =1 ⎭
n
a (1 - r )
4. S arn – 1 = a + ar + ar2 + … + arn – 1 = ; n ⎛ r 3 - 1⎞
­provided r < 1. 1- r 8. lim ∏⎜ 3 ⎟
r = 3 ⎝ r + 1⎠
n→∞

1 6
Solved Examples (A)  (B) 
3 7
2
7. The value of (C) - (D)  None of these
3
1 ⎡ ⎛ n ⎞ ⎛ n -1 ⎞ ⎛n-2 ⎞ ⎤ Solution: (B)
lim ⎢1
n → ∞ n4 ⎢ ⎝
⎜ ∑ k ⎟ + 2 ⎜ ∑ k ⎟ + 3 ⎜ ∑ k ⎟ + ... + n ⋅ 1⎥
⎥⎦ (n – 2)th factor of the series is
⎣ k =1 ⎠ ⎝ k =1 ⎠ ⎝ k =1 ⎠
will be n - 1 n2 + n + 1
tn = ⋅
1 1 1 1 n + 1 n2 - n + 1
(A)  (B)  (C)  (D) 
24 12 6 3 Therefore, required limit = lim t3t 4 t5 ... t n - 2 t n - 1t n
n→∞
Solution: (A) ⎡⎛ 2 3 4 n - 3 n - 2 n - 1⎞
The (r + 1)th term of the series is = lim ⎢⎜ ⋅ ⋅ ... ⋅ ⋅
n→∞ ⎝ 4 5 6
⎣ n -1 n n + 1⎟⎠
n-r 
tr + 1 = ( r + 1) ∑ k
⎛ 13 ⎞ 21 31 n + n +1 ⎤
2
k =1
 ⋅ ⎜ ⎟ ⋅ ⋅ ... 2 ⎥
⎝ 7 ⎠ 13 21 n - n + 1⎦
⇒ tr + 1 = (r + 1)[1 + 2 + 3 + … (n – r) terms] 
1 2 ⋅ 3 n2 + n + 1 6
⇒ tr + 1 = ( r + 1) ( n - r )( n - r + 1) = lim ⋅ = .
n → ∞ n( n + 1) 7 7
2 
1 9. If [x] denotes the integral part of x, then
⇒ tr + 1 = ( r + 1)( n - rn + n - rn + r 2 - r )
2
2 
1 ⎛ n 2 ⎞
⎜ ∑ [k x ]⎟ =
1 lim
⇒ tr + 1 = ( r + 1)( r 2 - (1 + 2n)r + n2 ) n→∞ n3 ⎝
2  k =1 ⎠
1 3 x
⇒ tr + 1 = ( r - 2nr 2 + ( n2 - 2n - 1)r + n2 ) (A) 0 (B) 
2  2
n -1 x x
Now, S= ∑ tr + 1 (C)  (D) 
3 6
r=0


Objective_Maths_JEE Main 2017_Ch 11.indd 5 01/01/2008 05:01:13


11.6  Chapter 11

Solution: (C) Solution: (C)
1 ⎛ n ⎞ 5r + 2 r
n
L = lim 3 ⎜ ∑ [k 2 x ]⎟
n→∞ n ⎝
Required limit = lim
n→∞
∑ 10 r
k =1 ⎠ r =1


2
Since k x – 1 ≤ (k x) < k x2 2 ⎧⎪⎛ 1 ⎞ r ⎛ 1 ⎞ r ⎫⎪
n
= lim ∑ ⎨⎜ ⎟ + ⎜ ⎟ ⎬
n n n n→∞
r =1⎩⎪⎝ 2 ⎠ ⎝ 5⎠ ⎪

⇒ ∑ (k 2 x - 1) ≤ ∑ (k 2 x) < ∑ k 2 x 
k =1 k =1 k =1 ⎧
 ⎛ 1⎞ ⎫
n n
⎛ 1⎞
⎛ n ⎞ n n ⎛ n ⎞ ⎪ 1 - ⎜⎝ ⎟⎠ 1 - ⎜⎝ ⎟⎠ ⎪
⎪1 2 1 5 ⎪
⇒ x ⎜ ∑ k 2 ⎟ - ∑ (1) ≤ ∑ [k 2 x ] < x ⎜ ∑ k 2 ⎟ = lim ⎨ ⋅ + ⎬
⎝ k =1 ⎠ k =1 ⎝ k =1 ⎠ n→∞ 2
⎪ 1 5 1 ⎪
k =1
 1- 1-
⎪ 2 5 ⎪
xn( n + 1)( 2n + 1) ⎩ ⎭
⇒ -n
6  1 5
n
= 1+ =
xn ( n + 1)( 2n + 1) 4 4
≤ ∑ [k x ] <
2
6
k =1 2x - 3 2 x 2 + 5x
3 11. lim f ( x ), where < f (x) < , is
Dividing throughout by n , we have x →∞ x x2
xn ( n + 1)( 2n + 1) 1 (A) 1 (B) 2 (C) –1 (D) –2
3
-
6n n2 
Solution: (B)
n
[k 2 x ] xn ( n + 1)( 2n + 1)
≤ ∑ n3
<
6 n3 lim
2x - 3 ⎛ 3⎞
= lim ⎜ 2 - ⎟ = 2
k =1
x →∞ x x →∞ ⎝ x⎠
x⎛ 1⎞ ⎛ 1⎞ 1 n
[k 2 x ] 2 x 2 + 5x ⎛ 5⎞
⇒ ⎜1 +
6⎝
⎟ ⎜2 +
n⎠ ⎝ n
⎟⎠ - 2 ≤ ∑ 3 and lim = lim ⎜ 2 + ⎟ = 2,
n k =1 n x →∞ x 2 x →∞ ⎝ x⎠
x⎛ 1⎞ ⎛ 1⎞ \ Using Sandwitch theorem, lim f ( x ) = 2.
< ⎜1 + ⎟⎠ ⎜⎝ 2 + ⎟⎠ x →∞
6⎝ n n
Taking limits as n → ∞, we get Trigonometric Limits
⎡x ⎛ 1⎞ ⎛ 1⎞ 1⎤ For finding the limits of trigonometric functions, we use
lim ⎢ ⎜1 + ⎟ ⎜ 2 + ⎟ - 2 ⎥ ≤ L
n→∞ ⎣6 ⎝ n⎠ ⎝ n⎠ n ⎦ trigonometric transformations and simplify. The following
results are quite useful:
x⎛ 1⎞ ⎛ 1⎞
< lim
⎜ 1+ ⎟ ⎜2 + ⎟ sin x
n→∞ 6 ⎝ n⎠ ⎝ n⎠ 1. (a) lim =1
x →0 x
1
Since, as n → ∞, we have → 0 (b) lim cos x = 1
n x →0
x x tan x
⇒ ≤L< (c) lim =1
3 3 x →0 x
According to Squeeze Principle or Sandwich Theorem, sin -1 x
(d) lim =1
we have x→0 x
x tan -1 x
L = . (e) lim =1
3 x→0 x
⎧⎪ 7 sin x 0 p
29 133 5n + 2n ⎫⎪ (f) lim = .
10. lim ⎨ + 2 + 3 + ... + ⎬ is equal to x →0 x 180
n → ∞ ⎪10
⎩ 10 10 10 n ⎭⎪
2. lim f ( x ) = lim f ( a + h) , where a ≠ 0, on taking
3 5 1 x→a h→ 0
(A)  (B) 2 (C)  (D)  x = a + h.
4 4 2

Objective_Maths_JEE Main 2017_Ch 11.indd 6 01/01/2008 05:01:19


Limits  11.7

Some Useful Expansions x


sin 2

1
= lim ⋅ 2 ⋅ 1 + cos x + cos x 
x3 x5 x7 x→0 2 x x
1. sin x = x - + - + ... ∞; x ∈  cos ⋅ cos x
3! 5! 7 ! 2 2
x 2 x 4 x6
1 3
= ⋅1⋅ =
3
2. cos x= 1 - + - + ... ∞; x ∈ 
2! 4 ! 6 ! 2 1 2
x3 2 17 7 62 9 ⎛ x x x x⎞
3. tan x = x + + x3 + x + x + ... ∞ ; 13. lim ⎜ cos cos cos … cos n ⎟ =
3 15 315 2835 n→∞ ⎝ 2 4 8 2 ⎠
p
|x| < x sin x
2 (A)  (B) 
x2 5 4 61 6 p sin x x
4. sec x = 1 + + x + x + ... ∞; 0 < |x| < (C)  0 (D)  None of these
2 24 720 2
1 x 7 3 31 5 Solution: (B)
5. cosec x = + + x + x + ... ∞; 0 < |x| < p
x 6 360 15120 x x x x
Required Limit = lim cos cos 2 cos 3 ... cos n
3 n→∞ 2
1 x x 2 5 2 2 2
6. cot x = - - - x - ... ∞; 0 < |x| < p
x 3 45 945 1 ⎡ x x ⎛ x x ⎞⎤
= lim cos ... cos n - 1 ⎜ 2 sin n cos n ⎟ ⎥
1 x 3 1.3 x 5 1. 3 . 5 x 7 n→∞ x ⎢⎣ 2 2 ⎝ 2 2 ⎠⎦
7. sin– 1 x = x + + + + ... ∞; |x| < 1 2 sin n
2 3 2.4 5 2 . 4 . 6 . 7 2
p ⎡ x ⎛ x ⎞⎤
8. cos– 1 x = - sin -1 x = lim
1 x
2 x ⎢cos 2 ... ⎜⎝ 2 cos n - 1 sin n - 1 ⎟⎠ ⎥
22 sin n ⎣ ⎦
n→∞ 2 2
p ⎧⎪ 1 x 3 1.3 x 5 1.3.5 x 7 ⎪⎫ 2
=
- ⎨x + + ⋅ + ⋅ + ... ∞⎬; |x| < 1
2 ⎪⎩ 2 3 2.4 5 2.4.6 7 ⎭⎪ …………………………………………......................
…………………………………………......................
9. tan–1x =
⎧ x3 x5 x7
1 ⎛ x x⎞ sin x
= lim ⎜⎝ 2 cos sin ⎟⎠ = nlim
⎪ x- + - + ... ∞; | x |<1 n→∞ x 2 2 → ∞ ⎛ x⎞
⎪ 3 5 7 2n sin n 2n sin ⎜ n ⎟
⎨ 2 ⎝2 ⎠
⎪± p - 1 + 1 - 1 + 1 - ... ∞; ⎧ + if x ≥ 1
⎪⎩ 2 x 3x 3 5 x 5 7 x 7 ⎨ ⎧ ⎛ x⎞ ⎫
⎩ - if x ≤ - 1
sin x ⎪⎪ ⎜⎝ n ⎟⎠ ⎪⎪ sin x
2
= lim ⎨ ⎬ = .
x n→∞⎪ ⎛ x ⎞ ⎪ x
Solved Examples sin ⎜ n ⎟
⎪⎩ ⎝ 2 ⎠ ⎪⎭
1 - cos3 x
12. lim = q 1 q q⎞
x → 0 x sin x cos x ⎛ 1 1
14. lim ⎜ tan q + tan + 2 tan 2 + … + n tan n ⎟ =
3 1 n→∞ ⎝ 2 2 2 2 2 2 ⎠
(A) 
2 2 1 1
(A)  (B)  - 2 cot 2q
(C)  1 (D)  None of these q q
(C)  2 cot 2q (D)  None of these
Solution: (A)
1 - cos3 x Solution: (B)
lim
x → 0 x sin x cos x sin 2q
tan 2q =
3
(1 - cos x ) (1 + cos x + cos x ) cos 2q 
= lim
x→0 x sin x cos x  1 2 sin q cos q
=
2 x cot 2q cos 2 q - sin 2 q 
2 sin (1 + cos 2 x + cos x )
= lim 2
x x cos 2 q - sin 2 q
x→0
2 x sin cos cos x ⇒ cot 2q =
2 2 2 sin q cos q 


Objective_Maths_JEE Main 2017_Ch 11.indd 7 01/01/2008 05:01:25


11.8  Chapter 11

cos 2 q sin 2 q 4 4 2 3
⇒ 2cot 2q = -  (A)  (B) – (C)  (D) 
sin q cos q sin q cos q 3 3 3 4
⇒ 2cot 2q = cot q – tan q Solution: (A)
⇒ tan q = cot q – 2 cot 2q(1) ⎡ ⎛p ⎞ ⎛p ⎞⎤
2 ⎢ 3 sin ⎜ + h⎟ - cos ⎜ + h⎟ ⎥
Now, tan q = cot q – 2 cot 2q ⎝ 6 ⎠ ⎝ 6 ⎠⎦
lim ⎣
1 q 1 q h→ 0 3h ( 3 cos h - sin h)
⇒ tan = cot - cot q
2 2 2 2  ⎡ ⎛1 3 ⎞ ⎛ 3 1 ⎞⎤
1 q 1 q 1 2 ⎢ 3 ⎜ cos h + sin h⎟ - ⎜ cos h - sin h⎟ ⎥
⇒ tan 2 = cot - cot q ⎢ ⎝2 2 ⎠ ⎝ 2 2 ⎠ ⎥⎦
2 2
2 22 2 2  = lim ⎣
h→ 0 3h( 3 cos h - sin h)
1 q 1 q 1 q
⇒ n
tan n
= n cot n - n - 1 cot n - 1 2 [ 2 sin h]
2 2 2 2 2 2  = lim
1 q
h→ 0 3 h ( 3 cos h - sin h)
⇒ S = - 2 cot 2q + n cot n 
2 2  sin h
4⋅
Therefore, h
= lim
q 1 q q⎞ h→ 0 3 ( 3 cos h - sin h)
⎛ 1 1 
lim ⎜ tan q + tan + 2 tan 2 + ... + n tan n ⎟
n→∞⎝ 2 2 2 2 2 2 ⎠ 4 4
= =
⎛ 1 q⎞ 3 ( 3 - 0) 3
= lim S = lim ⎜ - 2 cot 2q + n cot n ⎟
n→∞ n→∞⎝ 2 2 ⎠ n
x - sin x n

⎛ q ⎞ 17. If lim is non-zero finite, then n may be


x → 0 x - sin n x
1⎜ n ⎟
= - 2 cot 2q + lim ⎜ 2 ⎟ equal to
n→∞q q
⎜ tan ⎟ (A) 1 (B) 2
⎝ 2n ⎠  (C)  3 (D)  None of these
1
= - 2 cot 2q + Solution: (A)
q
tan ([ - p 2 ]x ) - x 2 tan ([ - p 2 ]) x - sin x
15. lim equals, where [ ] Clearly, for n = 1, lim = 1.
x→0 x - sin x
x→0 sin 2 x
denotes the greatest integer function ⎛ cot 4 x ⎞
18. lim ⎜ cosec3 x ⋅ cot x - 2 cot 3 x ⋅ cosec x + is
(A) 0 (B) 1 x →0 ⎝ sec x ⎟⎠
(C)  tan 10 – 10 (D)  ∞
equal to
Solution: (C) (A) 1 (B) – 1
Since, [–p2] = [–9.87] = –10, therefore (C)  0 (D)  None of these
tan[ - p 2 ]x 2 - x 2 tan[ - p 2 ] Solution: (A)
lim 2
x→0 sin x ⎡ cot 4 x ⎤
2 2 2 lim ⎢cosec3 x ⋅ cot x - 2 cot 3 x ⋅ cosec x + ⎥
tan ( - 10 x ) - x tan ( - 10) x x →0
⎣ sec x ⎦
= lim 2
x→0 x sin 2 x  ⎛ cos x 2 cos3 x cos5 x ⎞
tan (10 x 2 ) x 2 tan 10 = lim ⎜ 4 - + ⎟
= – lim + lim x → 0 ⎝ sin x sin 4 x sin 4 x ⎠

x→0 x2 x→0 x2  2 2
2 cos x (1 - cos x )
tan (10 x ) = lim = lim cos x = 1
= lim ( - 10) + tan 10 x →0 sin 4 x x →0
x→0 10 x 2 
= tan 10 – 10
Exponential and Logarithmic Limits
⎡ ⎛p ⎞ ⎛p ⎞⎤
2 ⎢ 3 sin ⎜ + h⎟ - cos ⎜ + h⎟ ⎥ For finding the limits of exponential and logarithmic func-
⎝ 6 ⎠ ⎝ 6 ⎠⎦
16. lim ⎣ is equal to tions, the following results are useful:
h→ 0 3h ( 3 cos h - sin h)

Objective_Maths_JEE Main 2017_Ch 11.indd 8 01/01/2008 05:01:29


Limits  11.9

ex - 1 ⎛ ⎞
1. lim =1 x 2 x 4 x6 x2
x→0 x ⎜ 1 - + - + ...⎟ - 1 +
⎝ 2! 4 ! 6 ! ⎠ 2 
ax - 1 = lim
2. lim = loge a, a > 0 x→0 x 4
x→0 x
⎛1 ⎞
ax - bx ⎛ a⎞ = lim ⎜ + terms containing x and its powers⎟
3. lim = loge ⎜ ⎟ ; a, b > 0 x → 0 ⎝ 4! ⎠
x→0 x ⎝ b⎠ 
n
(1 + x ) - 1 1 1
4. lim =n = =
x→0 x 4! 24 
n
⎛ 1⎞
5. lim (1 + x )1/ x = lim ⎜1 + ⎟ = e
x→0 n →∞ ⎝ n⎠ caution
6. lim (1 + a h)1/ h = ea If [⋅] denotes the greatest integer function, then
h→ 0
log x lim [ - x ] = [0] = 0
7. lim = 0, (m > 0) x →0
xm
x →∞ Is the above statement true?
log a (1 + x ) No. If fact, lim [ - x ] = lim [0 - x ] = lim - 1 = –1
8. lim = loga e, (a > 0, a ≠ 1) x →0 x →0 x→ 0
x→0 x
Thus, limit must be applied only after removing [⋅] sign.
x
⎛ a⎞
9. lim ⎜1 + ⎟ = ea
x →∞ ⎝ x⎠
⎛ 1 ⎞
f ( x) Solved Examples
10. lim ⎜1 + = e, where f (x) → ∞ as x → ∞.
x →∞ ⎝ f ( x ) ⎟⎠ x sin{x}
19. lim , where {x} denotes the fractional part of x,
x -1
x →1
11. lim (1 + f ( x )1/ f ( x ) = e.
x→a is equal to
(A) –1 (B) 0
Some Useful Expansions (C)  1 (D)  Does not exist
x x 2 x3 Solution: (D)
1. ex = 1 + + + + ... to ∞
1! 2 ! 3! x sin{x}
LHL = lim
x → 1- x - 1
x x 2 x3 
2. e–x = 1 – + - + ... to ∞ Let x = 1 – h, as x → 1, h → 0
1! 2 ! 3!
(1 - h) sin{1 - h}
x 2 x3 ⇒ LHL = lim
3. loge (1 + x) = x – + - ... to ∞, – 1 < x ≤ 1 h→0 h 
2 3
(1 - h) sin (1 - h)
x 2 x3 ⇒ LHL = lim
4. loge (1 ­– x) = – x – - - ... to ∞, – 1 ≤ x < 1 h→0 h 
2 3 (1 - h)
( x log a) 2 \ LHL = lim sin (1) = ∞
5. ax = ex log a = 1 + x log a + + ... to ∞ h→0 h
2! x sin ( x )
n ( n - 1) 2 Now, RHL = lim
6. (1 + x)n = 1 + nx + x + ... to ∞, – 1 < x < 1, x → 1+ x - 1
2! 
n being any negative integer or fraction. Let x = 1 + h, as x → 1, h → 0
(1 + h) sin (1 + h)
The expansion formulae mentioned above can be used with ⇒ RHL = lim
h→0 h 
advantage in simplification and evaluation of limits.
(1 + h) sin h
⇒ RHL = lim = lim (1 + h)
x2 h→0 h h→0

cos x - 1 +
For example, lim 2 \ RHL = (1 + 0) = 1
x→0 x4  Since LHL ≠ RHL,
\ the limit of the function Does not exist at x = 1.

Objective_Maths_JEE Main 2017_Ch 11.indd 9 01/01/2008 05:01:34


11.10  Chapter 11

⎛ b⎞ ⎡ x ⎤ where [x] denotes the greatest integer less than or


lim ⎜ ⎟ ⎢ ⎥ where a > 0, b > 0 and [x] denotes
20. equal to x, then lim f ( x ) is equal to
x →0 ⎝ x⎠ ⎣a⎦
+
x →0
greatest integer less than or equal to x is 1
(A) – (B)  1
1 b 2
(A)  (B)  b (C)  (D) 0 p
a a (C)  (D)  Does not exist
Solution: (D) 4

⎛ b⎞ ⎡ x ⎤ ⎛ b ⎞ ⎡0 + h⎤ Solution: (D)
lim ⎜ ⎟ ⎢ ⎥ = lim ⎜ ⎟ tan -1 ([ - h] - h)
x →0 ⎝ x⎠ ⎣a⎦
+
h → 0 ⎝ 0 + h⎠ ⎢ ⎥
⎣ a ⎦ LHL = lim f (0 - h) = lim
 h→ 0 h→ 0 [ - h] + 2 h 
⎛ b⎞ ⎡ h⎤ -1
tan ( -1 - h)
= lim ⎜ ⎟ ⎢ ⎥ = 0 = lim
h → 0 ⎝ h⎠ ⎣ a ⎦
h→ 0 ( 2h - 1) 
⎧ sin[ x ]
, [ x] ≠ 0 tan -1 (1 + h)
⎪ = lim
21. If f (x) = ⎨ [ x ] , where [x] denotes the h→ 0 (1 - 2h)

⎪0 , [ x] = 0
⎩ p /4 p
greatest integer ≤ x, then lim f ( x ) equals = . =
x →0 1 4
tan -1 ([h] + h)
(A) 0 (B) –1 RHL = lim f (0 + h) = lim
(C)  1 (D)  None of these h→ 0 h→ 0 [h] - 2h 
tan -1 ( h) 1
Solution: (A) = lim =–
h→ 0 -2h 2
sin[ - h] sin( -1)
lim f (0 - h) = lim = lim = sin 1. Since LHL ≠ RHL
h→ 0 h→ 0 [ - h] h→ 0 ( -1)
\ lim f ( x ) Does not exist.
sin[h] x →0
lim f (0 + h) = lim
h→ 0 h→0 [ h] 
Trick(s) for Problem Solving
= 1 [∵ h → 0 ⇒ (h) → 0]
\ lim f ( x ) does not exist.  If lim f( x ) = A > 0 and lim g( x ) = B, then
x →0 x →a x →a

lim [f( x )]g ( x ) = AB


22. Let f (x) = x – [x], where [x] denotes the greatest i­ nteger x →a

≤ x and g (x) = lim


[ f ( x)]2n - 1 , then g (x) =  If lim f( x ) = 1 and lim g( x ) = ∞, then
x →a x →a

[ ]
n→∞ f ( x ) 2 n + 1
lim [f( x )]g( x ) = e
lim
x→a
g( x )[ f ( x )-1]

x →a
(A) 0 (B) 1
(C)  –1 (D)  None of these
Solution: (C)
Solved Examples
As 0 ≤ x – [x] < 1 ∀ x ∈ R, 0 ≤ f (x) < 1.
\ lim [ f ( x ) ]
2n
= 0 729 x - 243x - 81x + 9 x + 3x - 1
n→∞
24. If lim 3
= k(log 3)3,
x→0
x
Thus, for x ∈ R, g (x) = lim
[ f ( x)]2n - 1 then k =
[ ]
n→∞ f ( x ) 2 n + 1

(A) 4 (B) 5
(C)  6 (D)  None of these
0 -1
= = –1 Solution: (C)
0 +1
Required limit
⎧ tan -1 ([ x ] + x ) 243x (3x - 1) - 9 x (32 x - 1) + (3x - 1)
⎪ , [ x] ≠ 0 = lim
23. If f (x) = ⎨ [ x ] - 2 x x→0 x3 
⎪0 , [ x] = 0 (3 - 1) {( 243) - ( 27) x - 9 x + 1}
x x
⎩ = lim
x→0 x3 

Objective_Maths_JEE Main 2017_Ch 11.indd 10 01/01/2008 05:01:40


Limits  11.11

(3x - 1) {( 243) x - ( 27) x - 9 x + 1} ⎛ 1 + tan x ⎞


1/sin x
= lim  27. lim ⎜ is equal to
x→0 x3 ⎟
x → 0 ⎝ 1 + sin x ⎠
(3 - 1) (9 - 1) ( 27 x - 1)
x x
(A) 0 (B) 1
= lim
x→0 x x x  (C)  –1 (D)  None of these
= log 3 ⋅ log 9 ⋅ log 27 Solution: (B)
= log 3 ⋅ 2log 3 ⋅ 3log 3 1 + tan x
Let f (x) =
3
= 6(log 3) = k(log 3) (given) 3 1 + sin x 
\ k = 6 1
and g (x) = .
sin x
25. If a and b be the roots of ax2 + bx + c = 0, then Clearly f (x) → 1 and g (x) → ∞ as x → 0.
lim (1 + ax 2 + bx + c)1 ( x -a ) is 1 /sin x 1 ⎛ 1+ tan x ⎞
x→a ⎛ 1 + tan x ⎞ lim ⎜ - 1⎟
sin x ⎝ 1+ sin x ⎠
\ = e
x →0

(A)  log |a (a – b )| ea (a – b)


(B)  lim ⎜ ⎟
x → 0 ⎝ 1 + sin x ⎠

(C) ea (b – a)
Solution: (B)
(D)  None of these

{Using lim [ f ( x)]
x→a
g( x)
=e
lim g ( x ) [ f ( x ) -1]
x→a
}
1
lim ⎡(1+ ax 2 + bx + c ) -1⎤⎦ 1 ⎛ tan x - sin x ⎞ 1- cos x
( x -a ) ⎣
lim (1 + ax 2 + bx + c)1 ( x -a ) = e lim
x →a
lim ⎜ ⎟
sin x ⎝ 1+ sin x ⎠ cos x (1+ sin x )
= e = e = e0 = 1
x →0 x →0

x →a
lim g ( x )[ f ( x ) -1]
[Using lim [f (x)]g (x) = e x→a
tan
px
x→a ⎛ x⎞ 2a
 provided f (x) → 1 and g (x) → ∞ as x → a] 28. lim ⎜ 2 - ⎟ is equal to
x→a ⎝ a⎠
( ax + bx + c )
2
a ( x -a )( x - b )
lim lim
= e
x →a
( x -a )
= e
x →a
( x -a ) (A) ep/2 (B) 
e2/p (C) 
e–2/p (D) 
e–p/2

[∵ a, b are roots of ax2 + bx + c = 0] Solution: (B)
px ⎛ p x⎞ ⎛ x ⎞
= ea (a – b)
tan ⋅ 2 - -1
⎛ x⎞ 2a lim tan ⎜
⎝ 2 a ⎟⎠ ⎜⎝ a ⎟⎠
lim ⎜ 2 - ⎟ = e
x→a

1 x→a ⎝ a⎠
⎛ sin x ⎞ x - a
26. lim ⎜ ⎟
x → a ⎝ sin a ⎠
, a ≠ np, n is an integer, equals

{Using lim [ f ( x)]x→a
g( x)
=e
lim g ( x ) [ f ( x ) -1]
x→a

(A) ecot a (B) 
etan a (C) 
esin a (D) 
ecos a ⎫
as f ( x ) → 1 and g ( x ) → ∞ as x → a⎬
Solution: (A) ⎭
⎛ x⎞ ⎛ p x⎞ (1- x a )
1 1 lim ⎜1- ⎠⎟ tan ⎝⎜
x→a ⎝
⎟ lim
a 2a ⎠ cot (p x 2 a )
= e = e
x→a

⎛ sin x ⎞ x -a ⎛ sin x - sin a ⎞ x -a 


lim ⎜ = lim ⎜1 +
⎝ sin a ⎟⎠ ⎟ -1 / a
x→a x→a ⎝ sin a ⎠ lim
x→a
⎛ p x⎞ p 2 2 ⎛ p x⎞
- cosec 2 ⎜ ⎟ ⋅ lim sin ⎜
⎝ 2 a ⎟⎠
sin x - sin a
= e ⎝ 2a ⎠ 2a
= e p
= e2/p
x→a

⎡ sin a ⎤ ( x - a )sin a
⎢ ⎧ ⎛ sin x - sin a ⎞ ⎫ sin x - sin a ⎥ a
= lim ⎢ ⎨1 + ⎜ ⎟⎬ ⎥ 29. lim (cos x + a sin bx ) x is equal to
x→a ⎩ ⎝ sin a ⎠ ⎭ x →0
⎢⎣ ⎥⎦
 (A) e - a b (B) 
2

e ab (C)  e-b a
e a b (D) 
2 2 2

sin x - sin a
= lim Solution: (C)
e ( x - a) sin a 
x→a
a a
lim (cos x + a sin bx -1)
2 ⎛ x + a⎞ ⎛ x - a⎞ 1 lim (cos x + a sin bx ) x = e x
x→ 0

= lim cos ⎜ ⎟ sin ⎜ ⋅ x →0


e x-a
x→a
⎝ 2 ⎠ ⎝ 2 ⎟⎠ sin a ⎡ Using lim f ( x ) f ( x ) = e lim f ( x )[ f ( x ) -1]



[ ] x→a

⎛ x + a⎞ ⎡ ⎛ x - a⎞ ⎛ x - a⎞ ⎤ 1  x→a
= lim cos ⎜ sin
e x→a
⎝ 2 ⎟⎠ ⎢⎣ ⎜⎝ 2 ⎟⎠ ⎜⎝ 2 ⎟⎠ ⎥⎦ sin a as f (x) → 1 and f (x) → ∞ as x → a]

cos a a ( - sin x + ab cos bx )
lim 2

= e sin a = ecot a = e x →0
1 = ea b 

Objective_Maths_JEE Main 2017_Ch 11.indd 11 01/01/2008 05:01:46


11.12  Chapter 11

sin x ⎛ ⎛ x ⎞⎞
2

⎛ sin x ⎞ x - sin x ⎜ sin ⎝⎜ 2 n ⎠⎟ ⎟ x 2


30. The value of lim ⎜ ⎟ is -2 lim ⎜ ⎟ ⋅ 2 ⋅n
x →0 ⎝ x ⎠ ⎜ x ⎟ 4n
n→∞

⎜⎝ 2 n ⎟⎠
(A) 1 (B) –1 =e
(C)  0 (D)  None of these x2
-2 × lim
Solution: (B) = e n→∞
4n = e0 = 1
sin x sin x ⎛ sin x ⎞
-1⎟
⎛ sin x ⎞ x - sin x lim ⎜
Evaluation of Limits using
= e x - sin x ⎝ x ⎠
x →0
lim ⎜ ⎟
x →0 ⎝ x ⎠ L’Hospital’s Rule
⎡ Using lim f ( x ) g ( x ) = e lim g ( x )[ f ( x ) -1]

⎢⎣ x→a
[ ] x→a Besides the methods given above to evaluate limits, there
is yet another method for finding limits, usually known as
sin x L’Hospital’s Rule as given below for indeterminate forms:
as f ( x) = → 1 and g ( x )
x
sin x ⎛ 0⎞
1. ⎜ ⎟ form: If lim f ( x ) = 0 and lim g ( x ) = 0, then
sin x x ⎝ 0⎠ x→a x→a
= = → ∞ as x → 0⎤⎦ f ( x) f ′ ( x)
x - sin x sin x lim = lim , provided the limit on the
1- x → a g ( x) x → a g ′ ( x)
x
sin x
lim - R.H.S. exists.
= e x→ 0
x = e– 1 Here, f ′ is derivative of f.
1- cos( x +1)
⎛ ∞⎞
⎛ x 4 + x 2 + x + 1⎞ ( x +1) 2 2. ⎜ ⎟ form: If lim f ( x ) = ∞ and lim g ( x ) = ∞,
31. lim ⎜ ⎟ is equal to ⎝ ∞⎠ x→a x→a
x →-1 ⎝ x2 - x + 1 ⎠ f ( x) f ′ ( x)
then lim = lim , provided the limit on
1/ 2 1/ 2 x → a g ( x) x → a g ′ ( x)
⎛ 2⎞ ⎛ 3⎞
(A) 1 (B)  ⎜ ⎟ (C) ⎜ ⎟ (D) e1/2 the R.H.S. exists.
⎝ 3⎠ ⎝ 2⎠
Solution: (B) Note that sometimes we have to repeat the process if the
1- cos( x +1) 0 ∞
form is or again.
⎛ x 4 + x 2 + x + 1⎞ ( x +1) 2 0 ∞
lim ⎜ ⎟
x →-1 ⎝ x2 - x + 1 ⎠
 Trick(s) for Problem Solving
⎛ x +1⎞
2 sin 2 ⎜
⎝ 2 ⎟⎠
⎛ x4 + x2 + x + 1⎞ ( x + 1) 2
L’Hospital’s Rule is applicable only when
f( x )
becomes of
= lim ⎜ ⎟

g( x )
x →-1 ⎝ x2 - x + 1 ⎠ 0 ∞
 the form or .
⎛ ⎛ x +1⎞ ⎞
2 0 ∞
sin
1 ⎜ ⎜⎝ 2 ⎟⎠ ⎟
0 ∞
 If the form is not or , simplify the given expression till
⎜ ⎟ 1/ 2
⎛ x4 + x2 + x + 1⎞ ⎜⎜ ⎛⎜ x +1⎞⎟ ⎟⎟
2 ⎛ 2⎞ 0 ∞
= lim ⎜ ⎟ ⎝ ⎝ 2 ⎠ ⎠ = ⎜ ⎟ it reduces to the form
0
or

and then use L’Hospital’s
x →-1 ⎝ x2 - x + 1 ⎠ ⎝ 3⎠
 rule. 0 ∞
n For applying L’Hospital’s rule differentiate the numerator
⎛ x⎞ 

32. lim ⎜ cos ⎟ is equal to and denominator separately.


n→∞ ⎝ n⎠
(A) e1 (B)  e–1
(C)  1 (D)  None of these caution
Solution: (C) L’ Hospital’s rule cannot be applied in every problem.
⎛ x ⎞
⎛ x⎞
n lim n ⎜ cos -1⎟ 3 x + sin 2 x ⎛ ∞⎞
lim ⎜ cos ⎟ = e
n→∞ ⎝ n ⎠ Consider the example, lim ⎜ form ⎟⎠ .
n→∞ ⎝
 x →0 3x - sin 2 x ⎝ ∞
n⎠
⎛ x⎞ Here, if we apply L’ Hospital’s rule, we get
lim - n⋅2 sin 2 ⎜ ⎟
⎝ 2n⎠
= e 3 x + sin 2 x = 3 + 2 cos 2 x
n→∞
 lim lim
x →∞ 3 x - sin 2 x x →∞ 3 - 2 cos 2 x

Objective_Maths_JEE Main 2017_Ch 11.indd 12 01/01/2008 05:01:52


Limits  11.13

e x - e - x + 2 sin x ⎛0 ⎞
Now, both the numerator and denominator are undefined = lim  ⎜⎝ form⎟⎠
x →0 24 x 0
because lim cos 2 x Does not exist.
x →∞
We can find the above limit as: e x + e - x + 2 cos x 4 1
= lim = = .
⎛ sin 2 x ⎞ x →0 24 24 6
3 + 2⎜
3 x + sin 2 x = ⎝ 2 x ⎟⎠ = 3 + 2(0)
lim lim 35. If f (2), g (x) be differentiable functions and f (1) =
x →∞ 3 x - sin 2 x x →∞ ⎛ sin 2 x ⎞ 3 - 2(0)
3 - 2⎜ ⎟ f (1) g ( x ) - f ( x ) g (1) - f (1) + g (1)
⎝ 2x ⎠ g (1) = 2 then lim
x →1 g( x) - f ( x)
sin 2 x is equal to
= 1, since lim =0
x →∞ 2 x (A) 0 (B) 1
(C)  2 (D)  None of these
Solution: (C)
Solved Examples
f (1) g ( x ) - f ( x ) g (1) - f (1) + g (1) ⎛0 ⎞
lim  ⎜⎝ form⎟⎠
x →1 g( x) - f ( x) 0
33. If a is a repeated root of ax2 + bx + c = 0, then
f (1) g’( x ) - f’( x ) g (1)
tan ( ax 2 + bx + c) = lim
lim is x →1 g’( x ) - f’( x )
(x - a) x →a 2 

(A) a (B) 
b (C)  c (D) 0 g’( x ) - f’( x )
= 2 lim
x →1 g’( x ) - f’( x )
Solution: (A) 
= 2.
tan ( ax 2 + bx + c)
lim
x →a ( x - a )2 36. Let f (x) be a twice differentiable function and f  ″ (0) = 5,
3 f ( x ) - 4 f (3 x ) + f ( 9 x )
⎛0 2 ⎞ then lim is equal to
 ⎜⎝ form as aa + ba + c = 0⎟⎠ x →0 x2
0
(A) 30 (B) 120
( 2ax + b) sec 2 ( ax 2 + bx + c) (C)  40 (D)  None of these
= lim
x →a 2(x - a)
Solution: (B)
⎛0 ⎞
form as a being a repeated root of ax 2 + bx + c = 0, 3 f ( x ) - 4 f (3 x ) + f ( 9 x ) ⎛0 ⎞
⎜0 ⎟ lim
⎜ ⎟
 ⎜⎝ form⎟⎠
⎝ 2aa + b = 0 ⎠ x →0 x2 0

2a sec 2 ( ax 2 + bx + c) + ( 2ax + b) 2 × 3 f’( x ) - 12 f’(3 x ) + 9 f’(9 x ) ⎛0 ⎞


= lim  ⎜⎝ form⎟⎠
2 2
2 sec ( ax + bx + c) tan( ax + bx + c) 2 x →0 2x 0
= lim 3 f" ( x ) - 36 f" (3 x ) + 81 f" (9 x )
x →a 2  = lim
x →0 2 
2a
= = a. 3 f" (0) - 36 f" (0) + 81 f" (0)
2 = = 24 f  ″(0)
2
e x + e - x + 2 cos x - 4
34. lim is equal to = 24 ⋅ 5 = 120.
x →0 x4
1 1 3- f ( x)
(A) 0 (B) 1 (C)  (D) – 37. If f (9) = 9 and f ′ (9) = 1, then lim is equal
6 6 x →9 3- x
Solution: (C) to
x -x
e +e + 2 cos x - 4 ⎛0 ⎞ (A) 0 (B) 1
lim 4
 ⎜⎝ form⎟⎠ (C)  –1 (D)  None of these
x →0 x 0
e x - e - x - 2 sin x ⎛0 ⎞ Solution: (B)
= lim  ⎜⎝ form⎟⎠
x →0 4 x3 0
3- f ( x) ⎛0 ⎞
lim  ⎜⎝ form⎟⎠
e x + e - x - 2 cos x ⎛0 ⎞ x →9 3- x 0
= lim  ⎜⎝ form⎟⎠
x →0 12 x 2 0

Objective_Maths_JEE Main 2017_Ch 11.indd 13 01/01/2008 05:01:57


11.14  Chapter 11

1 Solution: (B)
0- ⋅ f ′( x )
= lim
2 f ( x)
 ⎛ ln cos x ⎞ ln [1 + (cos x - 1) ]
x →9 1 lim ⎜ ⎟ = xlim
0- x →0 ⎝ 4
1 + x - 1⎠
2 → 0 4
1 + x2 - 1 
2 x
cos x - 1
 [Using L’Hospital’s Rule] = 4 lim
x →0 x2 

( )
x 3 ⎡
= lim ⋅ f’( x ) = × f ′ (9) = 1. x2 ⎤
⎢∵ ln [1 + (cos x - 1) ] ~ (cos x - 1) and
4
x →9 f ( x) 3 1 + x2 - 1 ~ ⎥
⎣ 4⎦
sin 2 x + a sin x ⎡
38. If lim be finite, then the value of a and x2/ 2 x2 ⎤
x →0 x 3 = –4 lim  ⎢∵(1 - cos x ) ~ ⎥
the limit are given by
x →0 x2 ⎣ 2⎦
= –2
(A)  – 2, 1 (B)  – 2, –1
(C)  2, 1 (D)  2, –1 2x - x2
41. lim is equal to
Solution: (B) x x - 22
x→2

sin 2 x + a sin x log 2 - 1 log 2 + 1


Let k = lim (A)  (B) 
x →0 x3 log 2 + 1 log 2 - 1

(C) 1 (D) –1
2 cos 2 x + a cos x
= lim Solution: (A)
x →0
 3x 2
2x - x2 ⎛0 ⎞
 [Using L’Hospital’s Rule] lim x
x → 2 x - 22
⎜⎝ form⎟⎠
 0
We require 2 cos2x + a cos x = 0 for x = 0 as denomi-
x
nator is zero. 2 log 2 - 2 x 4 log 2 - 4 log 2 - 1
= lim x = = .
\ a = –2. x → 2 x (1 + log x ) 4 (1 + log 2) log 2 + 1
2 cos 2 x - 2 cos x ⎛ 0⎞
Hence, k = lim  ⎜⎝ ⎟⎠ sin x - (sin x )sin x
x →0 3x 2 0 42. lim equals
x→
p 1 - sin x + ln sin x
-4 sin 2 x + 2 sin x ⎛ 0⎞ 2
= lim  ⎜⎝ ⎟⎠ (A) 1 (B) 2 (C) 3 (D) 4
x →0 6x 0
-8 cos 2 x + 2 cos x -8 + 2 Solution: (B)
= lim = = –1. Let sin x = h, then as x → p/2, h → 1
x →0 6 6
39. lim x x is equal to \ given limit
x →0
h - hh 1 - hh - hh ln h
(A) 0 (B) 1 = lim = lim
h → 1 1 - h + ln h h →1 -1 + 1 / h
(C)  – 1 (D)  None of these 
 (Using L’ Hospital Rule)
Solution: (B)
Let y = lim x x - h - 2h ln h - hh - 1 - hh (ln h) 2
h h
= lim
x →0  h →1 - 1 / h2
log x ⎛∞ ⎞ 
⇒ log y = lim x log x = lim  ⎜⎝ form⎟⎠ -1 - 1
x →0 x →0 1 x ∞ = = 2
-1
1/ x
= lim = – lim x = 0 p a-x
x → 0 -1/ x 2 x →0 43. The value of lim a 2 - x 2 cot is
x→a 2 a+ x
⇒ y = e0 = 1.
2a 2a
(A)  (B) 

⎛ ln cos x ⎞ p p
40. lim ⎜ ⎟ is equal to
x →0 ⎝ 4
1 + x 2 - 1⎠ 4a 4a
(C)  (D)  –
p p
(A) 2 (B) –2 (C) 1 (D) –1

Objective_Maths_JEE Main 2017_Ch 11.indd 14 01/01/2008 05:02:03


Limits  11.15

Solution: (C) -2 x
p a-x 2 a2 - x 2
lim a 2 - x 2 cot  (0 × ∞ form) = lim 
x→a 2 a+ x x→a p a-x p 2a
- sec 2 × ×
2 a + x 2 2( a + x ) a 2 - x 2
a2 - x 2 ⎛0 ⎞
= lim  ⎜⎝ form⎟⎠ 4a
x→a p a-x 0 =
tan p 
2 a+ x

EXERCISES

Single Option Correct Type

⎛ ⎡100 x ⎤ ⎡ 99 sin x ⎤⎞ 1 - cos 2( x - 1)


1. The value of lim ⎜ ⎢ ⎥ + ⎢ x ⎥⎟ , where [⋅]
x → 0 ⎝ ⎣ sin x ⎦
7. lim
x -1
⎣ ⎦⎠ x →1

represents greatest integer function, is (A)  exists and it equals 2


(A) 199 (B) 198 (B)  exists and it equals – 2
(C)  0 (D)  None of these (C)  Does not exist because (x – 1) → 0
(D) Does not exist because left hand limit is not equal
2. If f (x) = sin x, x ≠ np, to right hand limit
= 2, x = np
x5
where n ∈Z and 8. The value of lim
is
g(x) = x2 + 1, x ≠ 2, 5x x →∞
(A) 1 (B) –1
= 3, x = 2. (C)  0 (D)  None of these
then lim g [ f ( x ) ] is 1
x →0
9. lim (cos x + sin x ) x is equal to
(A) 1 (B) 0 x →0
(C)  3 (D)  Does not exist
(A) e (B) e2 (C) 
e–1 (D) 1
⎛ ⎞
3. The value of lim ⎜ x + x + x - x ⎟ is 2 2 - (cos x + sin x )3
x →∞ ⎝ ⎠ 10. The value of lim is
1 x→
p 1 - sin 2 x
(A)  1 4
2
(C)  0 (D)  None of these 3 2 1
(A)  (B)  (C)  (D) 2
2 3 2
⎡ x +1 p ⎤
4. The value of lim x ⎢ tan -1 - is
x →∞ ⎣ x + 2 4 ⎥⎦ ln(1 + 2h) - 2 ln(1 + h)
11. The value of lim is
1 1 h→ 0 h2
(A)  (B) – (C) 1 (D) – 1 (A) 1 (B) – 1
2 2
(C)  0 (D)  None of these

n→∞
( )
5. lim cos p n2 + n , n ∈ Z is equal to
⎛ 1 e1/ n e 2 / n
12. The value of lim ⎜ + + + ... +
e( n -1)/ n ⎞
is
(A) 0 (B) 1 n→∞ ⎝ n n n n ⎟⎠
(C)  – 1 (D)  None of these
(A) 1 (B) 0 (C) e– 1 (D)  e + 1
nk sin 2 ( n !)
6. lim 0 < k < 1, is equal to x sin ( x - [ x ])
n →∞ n+2 13. lim , where [⋅] denotes the greatest
(A) ∞ (B)  1
x →1 x -1
(C)  0 (D)  None of these ­integer function, is equal to

Objective_Maths_JEE Main 2017_Ch 11.indd 15 01/01/2008 05:02:06


11.16  Chapter 11

(A) 1 (B) –1 (A) (–1)n (B)  (–1)n – 1


(C) ∞ (D)  Does not exist (C)  0 (D)  Does not exist
2 sin x - sin 2 x y3
14. If f (x) = ∫ x3
dx , x ≠ 0, then lim f’( x ) is
x →0
· 23. lim
x →1 x3 - y 2 - 1
as (x, y) → (1, 0) along the line y =
y →0
(A) 0 (B) ∞ (C) –1 (D) 1 x – 1 is given by
(A)  1 (B)  ∞
⎡x⎤ (C)  0 (D)  None of these
⎢⎣ 2 ⎥⎦
15. lim (where [⋅] denotes the greatest integer 1 - 2 + 3 - 4 + 5 - 6 + ... - 2n
x → p / 2 ln(sin x )
24. lim is equal to
n →∞
function) n2 + 1 + 4 n2 - 1
(A)  Does not exist (B)  equals 1 1 1
(C)  equals 0 (D)  equals –1 (A)  (B)  –
3 3
16. lim lim (1 + cos 2 m n !p x ) is equal to 1
m→∞ n→∞ (C) – (D)  None of these
5
(A) 2 (B) 1
(C)  0 (D)  None of these ⎡ x 4 sin (1/ x ) + x 2 ⎤
25. The value of lim ⎢ ⎥ is
x→-∞
⎣ 1 + | x |3 ⎦
⎡ sin([ x - 3]) ⎤
17. lim ⎢ ⎥ , where [⋅] represents greatest (A) 1 (B) –1 (C) 0 (D) ∞
x → 0 ⎣ [ x - 3] ⎦
­
integer function, is 2 x + 23 - x - 6
26. lim is equal to
(A) 0 (B) 1 x→2 2 - x / 2 - 21- x
(C)  Does not exist (D)  sin 1
(A) 8 (B) 4
18. The values of constants a and b so that (C)  2 (D)  None of these
⎛ x2 + 1 ⎞
lim ⎜ - ax - b⎟ = 0 are 8 ⎛ x2 x4 x2 x2 ⎞
x →∞ ⎝ x + 1 ⎠ 27. lim8 ⎜ 1 - cos - cos + cos cos ⎟ is equal
x →0 x ⎝ 2 4 2 4⎠
(A) a= 1, b = –1 (B)  a = –1, b = 1 to
(C) a= 0, b = 0 (D)  a = 2, b = –1 1 1 1 1
(A)  (B) – (C)  (D) –
⎛ 1 1 1 1 ⎞ 16 16 32 32
19. lim ⎜ + + + ... + is equal to
n→∞ ⎝ 1 ⋅ 2 2 ⋅ 3 3⋅ 4 n( n + 1) ⎟⎠ 28. lim [log n -1 ( n) ⋅ log n ( n + 1) ⋅ log n +1 ( n + 2)… log n k
k
-1 ( n )]
n→∞
(A) 1 (B) –1 is equal to
(C)  0 (D)  None of these (A) ∞ (B)  n
(log x ) 2 (C) k (D)  None of these
20. lim , n > 0 is equal to
x →∞ xn ⎡ 1 1 1 1 ⎤
29. lim ⎢ + + + ... + is equal
(A) 1 (B) 0 (C) – 1 (D) ∞ n→∞ ⎣1 ⋅ 3 3 ⋅ 5 5 ⋅ 7 ( 2n + 1) ( 2n + 3) ⎥⎦
21. If the rth term, tr, of a series is given by to
1
r n (A)  1 (B) 
tr = 4
r + r2 + 1
, then lim
n→∞
∑ t r is
1
2
r =1 (C) – (D)  None of these
1 2
nx
(A)  1 (B)  ⎡11/ x + 21/ x + 31/ x + ... + n1/ x ⎤
2 30. The value of lim ⎢ ⎥ is
1 x →∞ n
(C)  (D)  None of these ⎣ ⎦
3
(A) n! (B)  n (C) (n – 1)!   (D)  0
22. lim( -1)[ x ] , where [x] denotes the greatest integer less
x→n 31. lim (1 + x) (1 + x2) (1 + x4) … (1 + x2n), | x | < 1, is
than or equal to x, is equal to n→∞
equal to

Objective_Maths_JEE Main 2017_Ch 11.indd 16 01/01/2008 05:02:10


Limits  11.17

1 1 40. If x1 = 3 and xn + 1= 2 + x n , n≥ 1, then lim xn is


(A)  (B)  (C)  1 – x (D) 
x–1 n→∞
x -1 x -1 equal to
xn (A) –1 (B) 2 (C)  5 (D) 3
32. lim = 0, (n integer), for
x →∞ex 3x +1 - 5 x +1
41. The value of lim is
(A)  no value of n x →∞ 3x - 5 x
(B)  all values of n 1
(A)  5 (B) 
(C)  only negative values of n 5
(D)  only positive values of n (C)  –5 (D)  None of these
x n + x n -1 + x n - 2 + ... + x 2 + x - n 1⎛
n -1 ⎞
33. The value of lim is 42. lim ⎜1 + e1/ n + e 2 / n + ... + e n
x →1 x -1 ⎟ is equal to
n→∞ n
⎝ ⎠
n ( n +1)
(A)  (B) 0 (A) e (B)  –e
2

(C) 
1 (D)  n (C) e– 1 (D)  1 – e

12 + 22 + 32 + ... + r 2 n x + sin x
34. If tr =
13 + 23 + 33 + ... + r 3
and Sn= ∑ ( -1)r ⋅ tr, then 43. lim
x →∞ x - cos x
=
r =1
lim Sn is given by (A) 0 (B) 1
n→∞
(C)  – 1 (D)  None of these
2 2 1 1
(A)  (B) – (C)  (D) – n Sn + 1 - Sn
3 3 3 3 44. If Sn= ∑ ai and lim an = a, then lim is
n→∞ n→∞ n
i =1
35. If lim
(1 + a3 ) + 8e1 x
= 2, then ∑i
x → 0 1 + (1 - b3 ) e1 x equal to i =1

(A) a = 1, b = (–3)1/3 (B)  a = 1, b = 31/3 (A) 0 (B)  a


(C) a = –1, b = –(3)1/3 (D)  None of these (C)  2a (D)  None of these
1 - cos 2q
36. If a = min {x2 + 4x + 5, x∈R} and b = lim 45. The value of lim ⎡ n2 - n3 + n⎤ is
, 3
n q →0 q2 n→∞ ⎢
⎣ ⎥⎦
then the value of ∑ ar ⋅ bn- r is (A) 
1 1
(B) – (C) 
2
(D) –
2
r =0
2 n +1
-1 3 3 3 3
(A)  (B) 2n + 1 – 1
4 ⋅ 2n 4 3
n5 + 2 - n2 + 1
n +1 46. The value of lim is
2 -1 n→∞ 5
n 4 + 2 - 2 n3 + 1
(C)  (D)  None of these
3 ⋅ 2n (A) 1 (B) 0 (C) –1 (D) ∞
1⋅ 2 + 2 ⋅ 3 + 3 ⋅ 4 + ... + n ( n + 1) (cos x - 1)(cos x - e x )
37. lim 3
is equal to 47. The integer n for which lim is
n→∞
n x→0 xn
(A) 1 (B) –1 a finite non-zero number, is
1 (A) 1 (B) 2 (C) 3 (D) 4
(C)  (D)  None of these
3
2 x +3 3 x +5 5 x
log (1 + x + x 2 ) + log (1 - x + x 2 ) 48. The value of lim is
38. lim is equal to x →∞ 3x - 2 + 3 2 x - 3
x →0 sec x - cos x 2
(A)  3
(B) 
(A) 1 (B) –1 (C) 0 (D) ∞ 3
ln x - 1 1
39. lim is equal to (C)  (D)  None of these
| x-e|
x →e 3
1 1 x 3
z 2 - ( z - x)2
(A)  (B)  – 49. lim is equal to
( )
e e x→0 3 4
(C) e (D)  Does not exist 8 xz - 4 x 2 + 3 8 xz

Objective_Maths_JEE Main 2017_Ch 11.indd 17 01/01/2008 05:02:16


11.18  Chapter 11

(A) 
z
(B) 
1 43n - 2 - 9 n + 1
56. lim =
211/ 3 2 23/ 3
⋅z 82 n - 1 - 9 n - 1
n→ 0

(C) 221/3 z (D)  None of these 1


(A)  (B)  81
50. In a circle of radius r, an isosceles triangle ABC is 2
(C)  Does not exist (D)  None of these
inscribed with AB = AC. If the D ABC has perimeter P =
x - ai
57. If Ai = , i = 1, 2, …, n and if a1 < a2 < a3 … < an.
2 ⎡ 2 hr - h2 + 2 hr ⎤ and area A = h 2 hr - h2 , | x - ai |
⎣⎢ ⎦⎥
A Then, lim ( A1 A2 ... An ), 1 ≤ m ≤ n
where h is the altitude from A to BC, then lim 3 is x → am
h→ 0 P +

equal to (A)  is equal to (–1)m


1
(B)  is equal to (–1)m +1
(A) 128 r (B) 
128 r (C)  is equal to (–1)m – 1
1
(C)  (D)  None of these (D)  Does not exist
64 r
⎛ ⎡100 x ⎤ ⎡ 99 sin x ⎤⎞
⎛ 1 - cos{2( x - 2)} ⎞ 58. The value of lim ⎜ ⎢ ⎥ + ⎢ x ⎥⎟ , where [⋅]
x → 0 ⎝ ⎣ sin x ⎦
51. lim ⎜ ⎣ ⎦⎠
x→2⎝

x-2 ⎠ represents greatest integer function, is
1 (A) 199 (B) 198
(A) equals (B)  Does not exist
2 (C)  0 (D)  None of these
1 x5
(C) equals (D) equals - 2 59. The value of lim x is
x →∞ 5
2
(A) 1 (B) –1
⎛ x x x x⎞ (C)  0 (D)  None of these
52. lim ⎜ cos cos cos ... cos n ⎟ =
n→∞ ⎝ 2 4 8 2 ⎠ 60. If the rth term, tr, of a series is given by
x sin x r n
(A) 
sin x
(B) 
x
tr= 4
r + r2 + 1
, then lim
n→∞
∑ t r is
r =1
(C)  0 (D)  None of these 1
(A)  1 (B) 
53. The value of 2
1
(C)  (D)  None of these
1 ⎡ ⎛ n ⎞ ⎛ n -1 ⎞ ⎛n-2 ⎞ ⎤ 3
lim ⎢1
n→∞ n 4 ⎢ ⎝
⎜ ∑ k ⎟ + 2 ⎜ ∑ k ⎟ + 3 ⎜ ∑ k ⎟ + ... + n ⋅ 1⎥
⎥⎦
⎡11/x + 21/x + 31/x + ... + n1/x ⎤
nx

⎣ k =1 ⎠ ⎝ k =1 ⎠ ⎝ k =1 ⎠ 61. The value of lim ⎢ ⎥ is


x →∞ n
will be ⎣ ⎦
1 1 1 1 (A) n! (B)  n (C) (n – 1)!   (D)  0
(A)  (B)  (C)  (D) 
24 12 6 3 62. lim (1 + x )(1 + x 2 )(1 + x 4 )… (1 + x 2 n ), | x | < 1, is equal to
n→∞
54. If [x] denotes the integral part of x, then 1 1
(A)  (B) 
1 ⎛ n ⎞ x -1 1- x
lim 3 ⎜ ∑ [k 2 x ]⎟ =
n→∞ n ⎝ ⎠ (C)  1 – x (D) x–1
k =1
x x x xn
(A) 0 (B)  (C)  (D)  63. lim = 0, (n integer), for
2 3 6 ex
x →∞

(A)  no value of n
⎛ 1 q 1 q 1 q⎞ (B)  all values of n
55. lim ⎜ tan q + tan + 2 tan 2 + … + n tan n ⎟ =
n→∞ ⎝ 2 2 2 2 2 2 ⎠ (C)  only negative values of n
1 1 (D)  only positive values of n
(A)  (B)  - 2 cot 2q 1 - cos 2q
q q 64. If a = min {x2 + 4x + 5, x ∈R} and b = lim
(C)  2 cot 2q (D)  None of these
n q →0 q2
then the value of ∑ a ⋅b
r n-r
is
r=0

Objective_Maths_JEE Main 2017_Ch 11.indd 18 01/01/2008 05:02:21


Limits  11.19

2n + 1 - 1 1 - cos x
(A)  (B) 2n + 1 – 1 72. lim =
4 ⋅ 2n x→ 0 x
2n + 1 - 1 1 1
(C)  (D)  None of these (A)  (B)  –
3 ⋅ 2n 2 2
log (1 + x + x 2 ) + log (1 - x + x 2 ) (C)  Does not exist (D)  None of these
65. lim is equal to
x →0 sec x - cos x x + 7 - 3 2x - 3
73. lim =
(A) 1 (B) –1 (C) 0 (D) ∞ x→ 2 3 x + 6 - 2 3 3x - 5
4
n5 + 2 - 3 n2 + 1 17 34
66. The value of lim is (A)  (B) 
n→∞ 5 23 23
n4 + 2 - 2 n3 + 1
(C)  1 (D)  None of these
(A) 1 (B) 0 (C) –1 (D) ∞
( 2m + x )1/m - ( 2n + x )1/n
2 2 74. lim is equal to
x 3
z - ( z - x) x→ 0 x
67. lim is equal to
( 8xz - 4 x ) 1 1 1 1
x →0 4
3 2 3
+ 8 xz (A)  m - n (B)  m + n
m2 n2 m2 n2
z 1 1 1
(A)  11/3
(B) 23/3 (C)  m -1 - n -1 (D)  None of these
2 2 ⋅z m2 n2
(C) 221/3z (D)  None of these
(cos q ) x - (sin q ) x - cos 2q
75. lim =
68. In a circle of radius r, an isosceles triangle ABC is x→ 4 x-4
inscribed with AB= AC. If the DABC has perimeter P = (A) cos4 q lncos q – sin4 q ln sin q
2 ⎡ 2hr - h2 + 2hr ⎤ and area A = h 2hr - h2 , (B) cos4 q lncos q + sin4 q ln sin q
⎣⎢ ⎦⎥ (C) cos4 q ln sin q – sin4 q lncos q
A
whereh is the altitude from A to BC, then lim 3 is (D)  None of these
h→ 0 P +

equal to 1/ x
1 ⎛ x - 1 + cos x ⎞
(A) 128r (B)  76. lim ⎜ ⎟⎠ =
128r x→ 0 ⎝ x
1
(C)  (D)  None of these (A) e1/2 (B)  e–1/2
64r 1/4
(C) e (D)  None of these
⎛ p⎞ x
cos ⎜ x + ⎟ ⎡ ⎤
⎝ 6⎠ e
69. lim = 77. lim ⎢ ⎥ =
2 cos x ) 2/3
x →p /3 (1 -
⎣ ( )
x →∞ ⎢ 1 + 1/ x x ⎥

(A) 1 (B) –1
e–1 (C) 
(A) e (B)  e1/2 (D) 
e–1/2
(C)  0 (D)  None of these
ln ( 2 - cos 2 x ) ⎡ a sin x ⎤ ⎡ b tan x ⎤
78. lim ⎢ + , where a, b are integers and
70. lim 2
is equal to x→ 0 ⎣ x ⎥⎦ ⎢⎣ x ⎥⎦
ln (sin 3 x + 1)
x →0
[ ] denotes integral part, is equal to
2 2
(A)  (B)  – (A) a+ b (B)  a+b–1
9 9 (C) a– b (D)  a – b – 1
(C)  0 (D)  None of these
[ x ] + [2 x ] + [3 x ] + ... + [nx ]
1 - cos (cx 2 + bx + a) 79. lim =
71. lim , where a is a root of n→∞ 1 + 2 + 3 + ... + n
x → 1/a (1 - xa ) 2
2 (A) x (B)  2x
ax  + bx+ c = 0, is equal to
(C)  0 (D)  None of these
b 2 - 4 ac b 2 - 4 ac
(A) 
2a 2
(B) 2
a ( )
80. lim n2 x1/n - x1/n + 1 , x > 0 is equal to
n→∞
4 ac - b 2 (A) 0 (B)  ex
(C)  (D)  None of these
2a 2 (C) lnx (D)  None of these

Objective_Maths_JEE Main 2017_Ch 11.indd 19 01/01/2008 05:02:27


11.20  Chapter 11

⎡ f ( x) ⎤
1/ x
⎡ f ( x) ⎤
1/ x (A) 0 (B) 1
3
81. If lim ⎢1 + x + = e , then lim ⎢1 + = (C)  –1 (D)  Does not exist
x→ 0 ⎣ x ⎥⎦ x→ 0 ⎣ x ⎥⎦
_ n ≥ 2, then
91. If a1 = 1 and an= n (1 + an – 1), ∨
(A) e (B)  e2
3
(C) e (D)  None of these ⎛ 1⎞ ⎛ 1⎞ ⎛ 1⎞
lim ⎜1 + ⎟ ⎜1 + ⎟ ... ⎜1 + ⎟ =
n→∞ ⎝ a1 ⎠ ⎝ a2 ⎠ ⎝ an ⎠
x x
82. If y = x + , then lim is equal to (A) 0 (B)  e
x x →∞ y
x+ (C) e2 (D)  Does not exist
x
x+ ⎡ 1 ⎞⎤
n
... ∞ - n2 ⎛ 1⎞ ⎛ 1⎞ ⎛
(A) 1 (B) –1
92. lim n ⎢( n + 1) ⎜⎝ n + 2
⎟⎠ ⎜⎝ n + 2 ⎟⎠ ... ⎜⎝ n + n -1 ⎟⎠ ⎥
n→∞ ⎣ 2 2 ⎦
(C)  0 (D)  None of these
(A) e (B)  e2
cos x - (cos x )cos x (C) e 4
(D)  None of these
83. lim =
x → 0 1 - cos x + ln (cos x )
x y - yx 1- k
(A) 0 (B) 1 93. If lim x = , then k =
x→ y x - y y 1+ k
(C)  2 (D)  None of these
(A) log y (B)  ey
(tan x ) tan x - tan x (C) y (D)  None of these
84. The value of lim is
x → p /4 ln (tan x ) - tan x + 1
⎛ 3 1⎞
(A) –2 (B) 1 n r -r +
⎜ r ⎟ is equal to
(C)  0 (D)  None of these 94. lim ∑ cot -1 ⎜ ⎟
n→∞ 2
r =1 ⎜ ⎟
( ) ⎝ ⎠
2 2 2 cos 2 x
85. lim 11/cos x
+ 21/cos x
+ ... + n1/cos x
= (A) 0 (B)  p
x → p /2
n ( n + 1) p
(A) n (B)  (C)  (D)  None of these
2 2
(C) n! (D)  None of these ⎛ ⎡ n sin x ⎤ ⎡ n tan x ⎤⎞
95. The value of lim ⎜ ⎢ + , where [·]
n
⎛ 3⎞ n→∞ ⎝ ⎣ x ⎥⎦ ⎢⎣ x ⎥⎦⎟⎠
86. lim
n→∞
∑ cot -1 ⎜⎝ r 2 + 4 ⎟⎠ =
denotes the greatest integer function, is
r =1
(A) 0 (B) tan–12 (A) n (B)  2n + 1
p (C) 2n – 1 (D)  None of these
(C)  (D)  None of these
4 ⎡ x2 ⎤
87. The value of lim [sin x + cos x], where [·] denotes 96. lim ⎢ ⎥ , where [·] denotes the greatest
x → 0 sin x tan x
x→ 5p / 4 ⎣ ⎦
the greatest integer function, is ­integer function, is
(A) 2 (B) –2 (C) 1 (D) –1 (A) 0 (B) 1
(C)  2 (D)  Does not exist
88. lim lim
m→∞ n→∞
cos 2 (1 - cos 2 (1 - cos 2 (1 ... cos 2 q ))
⎛ 1 + n 1n + 2n + n 2n + 3n + n 3n + 4 n + … + n ( m - 1) n + m n ⎞ 97. lim =
q→ 0 ⎛ p ( q + 4 - 2) ⎞
⎜ ⎟ sin ⎜
⎜⎝ m2 ⎟⎠ q ⎟
⎝ ⎠
1
(A) 0 (B) 1 (C) –1 (D)  (A) 1 (B) 0 (C)  2 (D) – 2
2
⎡ n 1⎤
89. The value of lim ⎢ ∑ r ⎥ , where [·] denotes the tan x - sin{tan -1 (tan x )}
n→∞ ⎢
⎣ r = 1 2 ⎥⎦ 98. Let f (x) = , then lim f ( x ) =
greatest integer, is tan x + cos 2 (tan x ) x→
p
2
1 (A) 1
(A) 0 (B) 1 (C) –1 (D) 
2 (B) –1
90. The value of lim | x | [cos x ] , where [·] denotes the (C) 0
x →∞
greatest integer, is (D)  Does not exist

Objective_Maths_JEE Main 2017_Ch 11.indd 20 01/01/2008 05:02:31


Limits  11.21

⎧ ⎡ 1/2 1/2 -1 -1 103. If a and b are the roots of the quadratic equation

⎪ ⎢⎛ a + x ⎞ 2( ax )1/4
  99. lim ⎨ ⎜ 1/2 1/4 ⎟ - 3/4 1/4 1/2 1/2 1/4 3/4 ⎥
x → a ⎢⎝ a - x ⎠ x -a x +a x -a ⎥ 1 - cos( cx 2 + bx + a)
⎪⎣ ⎦ ax2 + bx+ c = 0, then lim =
⎩ x→
1 2(1 - a x ) 2
a
⎫⎪
4

-2log ⎬ =
a
c ⎛ 1 1⎞ c ⎛ 1 1⎞
⎭⎪ (A)  - (B)  -
2a ⎜⎝ a b ⎟⎠ 2b ⎜⎝ a b ⎟⎠
(A) a3/4 (B)  a
(C) a2 (D)  None of these c ⎛ 1 1⎞
(C)  - (D)  None of these
(log (1 + x ) - log 2) (3.4 x -1
- 3x) ab ⎜⎝ a b ⎟⎠
100. lim =
x →1 {(7 + x )1/3 - (1 + 3 x )1/2 } sin p x 104. Given a real valued function f  such that
9 4 3 4 ⎧ tan 2 {x}
(A)  log (B)  log
p e p e ⎪ 2 2
, x>0
⎪ x - [ x]
9 2 ⎨
(C)  log (D)  None of these f (x) = ⎪ 1 , x=0
p e
⎪ {x} cot {x} , x < 0
(1 - x ) (1 - x 2 ) ... (1 - x 2 n ) ⎩
101. lim = 2
x →1 [(1 - x ) (1 - x 2 ) ... (1 - x n )]2 ⎛ ⎞
( 2n)! The value of cot ⎝ xlim
→0
f ( x )⎟ is
–1 ⎜

(A) n! (B) 
n! (A) 0 (B) 1
( 2n)!
(C)  2 (D)  None of these (C)  –1 (D)  None of these
( n !) x a sin b x
105. If lim , a, b, c∈ R – {0} exists and has
k
kp x →0 sin ( x c )
102. If ∑ cos -1 a r =
2
for any k ≥ 1
­non-zero value, then
r =1
k
(1 + x 2 )1/3 - (1 - 2 x )1/4 (A) a, b, c are in A.P.
and q  = ∑ (a r ) r , then lim is
r =1
x →q x + x2 (B) a, c, b are in A.P.
  (C) a, c, b are in G.P.
1 1
(A) 1 (B) –1 (C)  (D) – (D)  None of these
2 2

More than One Option Correct Type


3x
106. lim lim (1 + cos 2 m n !p x ) is equal to ⎛ a1/x + b1/x + c1/x ⎞
m →∞ n →∞ 108. lim ⎜
x →∞ ⎝
⎟ =
3 ⎠
(A) 2 (B) 1
(C)  0 (D)  None of these (A) (a + b + c) elog (a + b + c)
(B) 
(1 + a3 ) + 8e1/x (C) abc (D)  elog (abc)
107. If lim = 2, then
x→0 1 + (1 - b3 ) e1/x axe x - b log(1 + x ) + cxe - x
109. If lim = 2, then
x→0x 2 sin x
(A) a = –1 (B)  a=1 (A) a = 3 (B)  b = 12
1/3
(C) b = (–3) (D) 31/3 (C) c = 9 (C)  a = –3

Passage Based Questions


Passage 1 However, if lim g ( x ) = 0 = lim f ( x ) , we cannot say
x→a x→a
We know that if lim f ( x ) = l and lim g ( x ) = m(≠ 0), then f ( x)
x→a x→a
  ­anything definite about the existence of lim . Though
x → a g( x)
lim f ( x ) in some cases this limit exists. Any expression of the type
f ( x) x→a
lim = 0 ∞
x → a g( x) lim g ( x ) or is termed as an indeterminate form. Many other
x→a 0 ∞

Objective_Maths_JEE Main 2017_Ch 11.indd 21 01/01/2008 05:02:35


11.22  Chapter 11

expressions like ∞ – ∞, 1∞, ∞0, 00, 0 × ∞ which can be 112. lim | x | sin x equals
x →0
0 ∞
reduced to the form or are also called indeterminate (A) 0 (B) 1
forms. 0 ∞ (C)  –1 (D)  None of these
f ( x) 0 ∞
If is indeterminate at x = a of the type or , 113. If a and b be the roots of ax2 + bx+ c = 0, then
g( x) 0 ∞
then lim (1 + ax 2 + bx + c)1/( x - a ) is
x →a
f ( x) f ′( x )
lim = lim , (A)  log |a(a – b )| ea(a – b)
(B) 
x → a g( x) x → a g ′( x )

wheref ′ is derivative of f. (C) ea(b – a) (D)  None of these


f ′( x ) Passage 3
If , too, is indeterminate at x = a of the type
g ′( x ) Let f, g and h be real valued functions defined on an interval
0 ∞ f ′( x ) f ′′( x )
or , then lim = lim I ⊆ R except possibly for some point c such that
0 ∞ x → a g ′( x ) x → a g ′′( x )
lim f ( x ) = l = lim h( x )
This can be continued till we finally arrive at a deter- x→c x→c
minate result. and,  f (x) ≤ g(x) ≤ h(x), ∀ x ∈ I.
sin 2 x + a sin x
110. If lim be finite, then the value of a Then, lim g ( x ) = l.
x→0 x3 x→c
and the limit are given by 1⋅ 3 ⋅ 5… ( 2n - 1)
(A)  –2, 1 (B)  –2, –1 (C)  2, 1 (D)  2, –1 114. lim =
n→δ +∞ 2 ⋅ 4 ⋅ 6… 2n

p a-x (A) 1 (B) –1


111. The value of lim a 2 - x 2 cot is (C)  0 (D)  None of these
x→0 2 a+ x
2a 2a 4a 4a {x} + {2 x} + {3 x} + ... + {nx}
-
(A)  (B)  -
(C)  (D)  115. lim ,
p p p p n →∞ n2
Passage 2 where {x} = x – [x] denotes the fractional part of x, is
For a function f, let lim f ( x ) ≠ 1 but f (x) is (A) 1 (B) 0
x→a 1
(C)  (D)  None of these
lim
x→a
{ f ( x)} g ( x ) , 2
g(x) ⎛ [12 x x ] + [22 x x ] + ... + [n2 x x ] ⎞
we write { f (x)}g(x) = e log { f ( x )}
e
116.
lim ⎜ lim ⎟ , where [·]
lim g ( x ) log f ( x) x → 0 ⎝ n →∞+
n3 ⎠
⇒ lim { f ( x )}g ( x ) = e
e


x→a

x→a denotes the greatest integer function, is equal to


In case, lim f ( x ) = 1 and lim g ( x ) = ∞, then 1 1
x→a x→a (A) – (B) 
g( x) 3 3
lim { f ( x )} = (1 + f ( x ) - 1) g ( x )
x→a
lim
 (C)  0 (D)  None of these
x→a

lim ( f ( x ) - 1) g ( x )
= e x→a


Match the Column Type


117. n ⎛ r 3 - 1⎞ 1
Column-I Column-II
  (III)  lim
n →∞
∏ ⎜⎝ r 3 + 1⎟⎠ (C) 
3
r=3
1
  (I)  lim ⎡ 3 n2 - n3 + n⎤
n
(A)  ⎛ x⎞
n→∞ ⎢
⎣ ⎥⎦ 9 (IV)  lim ⎜ cos ⎟ (D) 1
n →∞ ⎝ n⎠
3 2 3
x - 2 x +1 6
  (II)  lim 2
(B) 
x →1 ( x - 1) 7

Objective_Maths_JEE Main 2017_Ch 11.indd 22 01/01/2008 05:02:41


Limits  11.23

118.
⎡ x +1 p⎤
  (III)  lim x ⎢ tan -1 - (C) 0
x + 2 4 ⎥⎦
Column-I Column-II
x →∞ ⎣
sin x - (sin x )sin x
  (I)  lim (A) 2 nk sin 2 ( n !) 5
x→
p 1 - sin x + ln sin x (IV)  lim ,0 < k <1 (D) 
2 n→∞ n+2 4
⎧ 7 29 133 1
  (II)  lim ⎨ + 2 + 3 + … (B) –
n→∞ ⎩10 10 10 2
5n + 2n ⎫⎪
+ ⎬
10 n ⎭⎪

Assertion-Reason Type
Instructions: In the following questions an Assertion (A) is e - (1 + x )1/ x e
given followed by a Reason (R). Mark your responses from 122. Assertion: lim =
x →0 x 2
the following options:
(A)  Assertion(A) is True and Reason(R) is ln (1 + x ) - x
Reason: lim =0
True; Reason(R) is a correct explanation for x→0 x
Assertion(A) ln (1 + x ) - x 1
(B)  Assertion(A) is True, Reason(R) is True; and lim 2
=–
x→0 x 2
Reason(R) is not a correct explanation for 1
Assertion(A)
(C)  Assertion(A) is True, Reason(R) is False
( )
123. Assertion: lim [ f ( x )] + x 2 { f ( x )} = e, where f (x) =
x→0
tan x
(D)  Assertion(A) is False, Reason(R) is True and [·], { } denote integral and fractional parts,
x
12 + 22 + 32 + ... + r 2 respectively
119. Assertion: If tr= and
13 + 23 + 33 + ... + r 3 ⎡ tan x ⎤ 2
n ⎢⎣ x ⎥⎦ + x - 1
2 =3
Sn= ∑ ( -1) r . t r , then lim Sn = Reason: lim
x→0 ⎧ tan x ⎫
r =1
n →∞ 3 ⎨ ⎬
r ( r + 1) ( 2r + 1) ⎩ x ⎭
Reason: 12 + 22 + 32 + … + r2 =
6 1 x 1 x 1 x
2 124. Assertion: lim tan + 2 tan 2 + ... + n tan n
⎛ r ( r + 1 ) ⎞ n →∞ 2 2 2 2 2 2
and 13 + 23 + 33 + … + r3 = ⎜ ⎟
⎝ 2 ⎠ 1
= –cot x +
x
120. Assertion: If x1 = 3 and xn + 1 = 2 + xn , n ≥ 1, then
1 x 1 x
lim xn = 2 Reason: cot x + tan = cot
n →∞ 2 2 2 2
Reason: A monotonically decreasing sequence which cot q tan -1 ( m tan q ) - m cos 2 (q/2)
is bounded below is convergent 125. Assertion: lim
q →0 sin 2 (q/2)
1
n →∞ n
(
121. Assertion: lim 1 + e1/n + e 2/n + ... + e( n -1)/n = e – 1 ) = m –
4 3
m
3
Reason: 1 + r + r2 + … + rn – 1 x - tan x 1
Reason: lim =
⎧1 - r n x→0 x 3 3
⎪ if r <1
⎪ 1 - r
= ⎨ .
n
⎪r -1
⎪⎩ r - 1 if r >1

Objective_Maths_JEE Main 2017_Ch 11.indd 23 01/01/2008 05:02:46


11.24  Chapter 11

Previous Year’s Questions

2x
1 - cos 2 x ⎛ a b⎞
126. lim is [2002] 133. If lim ⎜1 + + 2 ⎟ = e 2 , then the values of a and b,
x →∞2x x →∞ ⎝ x x ⎠
(A) l (B) − 1 are  [2004]
(C)  Zero (D)  Does not exist
(A) a ∈ R, b ∈ R (B)  a = 1, b ∈ R
x
⎛ x 2 + 5x + 3⎞ (C) a ∈ R, b = R a = 1 and b = 2
(D) 
127. lim ⎜ 2 ⎟ is equal to [2002]
x →∞ ⎝ x + x + 2 ⎠ 134. Let α and β be the distinct roots of ax2 + bx + c = 0,
(A) e4 e2
(B)  1 - cos( ax 2 + bx + c)
then lim is equal to [2005]
(C) e3 (D) 
e x →a ( x - a )2
2
⎛ x - 3⎞ a2
128. For x ∈ R, lim ⎜ ⎟ is equal to [2002] (A)  (a - b ) 2 (B) 0
x →∞ ⎝ x + 2 ⎠ 2
(A) e (B)  e−1
a2 1
(B) e− 5
(D)  e5 (C) - (a - b ) 2 (D) (a - b ) 2
2 2
xf ( 2) - 2 f ( x ) 135. Let f : R → R be a positive increasing function such
129. Let f (2) = 4 and f  ′ (2) = 4. Then lim is
x→2 x-2 f (3 x ) f (2 x)
given by [2002] that lim = 1 . Then, lim = [2010]
x →∞ f ( x ) x →∞ f ( x )
(A) 2 (B) − 2
(C) − 4 (D) 3 2 3
(A)  (B) 
3 2
⎡ ⎛ x⎞ ⎤ (C) 3 (D) 1
⎢1 - tan ⎜⎝ 2 ⎟⎠ ⎥ [1 - sin x ]
1 30. lim ⎣ ⎦ is [2003] ⎛ 1 - cos{2( x - 2)} ⎞
x →p / 2 ⎡ ⎛ ⎞⎤
x 136. Limit of ⎜
3 ⎟ as x tends to 2 [2011]
⎢1 + tan ⎜⎝ 2 ⎟⎠ ⎥ [p - 2xx ] ⎝ x-2 ⎠
⎣ ⎦
1 (A) equals 2 (B)  equals – 2
(A)  (B)  0
8
1
1 (C) equals (D)  does not exist
(C)  (D)  ∞ 2
32
131. If lim
log(3 + x ) - log(3 - x )
= k , the value of k is 137. The value of lim
(1 - cos 2 x ) (3 + cos x ) is equal to
 x →0 x [2003] x →0 x tan 4 x
 [2013]
1 1
(A)  0 (B)  - (A)  (B)  1
3 2
2 2 1
(C)  (D)  - (C)  2 (D)  -
3 3 4
138. Let f (x) be a forth degree polynomial having extreme
132. Let f (a) = g(a) = k and their nth derivatives f  n(a),
gn(a) exist and are not equal for some n. Further if ⎡ f ( x) ⎤
values at x = 1 and x = 2. If lim ⎢1 + 2 ⎥ = 3 , then
f ( a) g ( x ) - f ( a) - g ( a) f ( x ) + g ( a) x →0 ⎣ x ⎦
lim = 4 , then the f (2) is equal to [2015]
x→a g( x) - f ( x)
value of k is [2003] (A) -4 (B)  0
(C)  4 (D)  -8
(A) 4 (B) 2
(C) 1 (D) 0

Objective_Maths_JEE Main 2017_Ch 11.indd 24 01/01/2008 05:02:50


Limits  11.25

(1 - cos 2 x ) (3 + cos x ) 1
139. The value of lim is equal to 140. Let p = lim (1 + tan 2 x ) 2 x then log p is equal to
x →0 x tan 4 x x →0 +
 [2015]  [2016]
(A) 3 (B) 2 1
(A)  (B) 
2
1 4
(C)  (D)  4 1
2 (C) 1 (D) 
2

Answer keys

Single Option Correct Type


  1. (B) 2. (A) 3.  (A) 4. (B) 5.  (A) 6. (C) 7.  (D) 8. (C) 9.  (A) 10. (A)
  11.  (B) 12. (C) 13.  (D) 14. (D) 15. (C) 16.  (a, b) 17.  (C) 18. (A) 19.  (A) 20. (B)
  21.  (B) 22. (D) 23. (C) 24. (B) 25.  (B) 26. (A) 27.  (C) 28. (C) 29.  (B) 30. (A)
  31.  (B) 32. (B) 33.  (A) 34. (B) 35.  (A) 36. (B) 37.  (C) 38. (A) 39. (D) 40. (B)
  41.  (A) 42. (C) 43.  (B) 44. (A) 45.  (A) 46. (B) 47. (C) 48. (A) 49.  (B) 50. (B)
  51. (B) 52.  (B) 53. (A) 54. (C) 55.  (B) 56. (C) 57.  (D) 58.  (B) 59. (C) 60.  (B)
  61. (A) 62.  (B) 63. (B) 64. (B) 65.  (A) 66. (B) 67.  (B) 68. (B) 69. (C) 70. (A)
  71.  (A) 72. (C) 73. (B) 74.  (C) 75. (A) 76.  (B) 77. (C) 78.  (B) 79. (A) 80.  (C)
  81. (B) 82.  (A) 83. (C) 84. (A) 85.  (A) 86. (B) 87. (B) 88. (D) 89. (A) 90. (B)
  91.  (B) 92. (B) 93.  (A) 94. (C) 95. (C) 96. (A) 97. (C) 98. (A) 99.  (C) 100. (A)
101.  (C) 102. (C) 103. (A) 104. (D) 105. (D)

More than One Option Correct Type


106.  (A) and (B) 107.  (B) and (C) 108.  (C) and (D) 109.  (A), (B) and (C)

Passage Based Questions


110.  (B) 111. (C) 112.  (B) 113. (B) 114.  (C) 115. (B) 116. (B)

Match the Column Type


1 17. (I) → (C); (II) → (A); (III) → (B); (IV) → (D)
118. (I) → (A); (II) → (D); (III) → (B); (IV) → (C)

Assertion-Reason Type
119. (D) 120. (A) 121. (A) 122.  (A) 123. (D) 124.  (A) 125. (C)

Previous Year’s Questions


126. (D) 127. (A) 128. (C) 129. (C) 130. (C) 131. (C) 132. (A) 133. (B) 134. (A) 135. (D)
136. (D) 137. (C) 138. (A) 139.  (B) 140. (D)

Objective_Maths_JEE Main 2017_Ch 11.indd 25 01/01/2008 05:02:51


11.26  Chapter 11

Hints and Solutions

Single Option Correct Type


sin x x ⎡ 1 ⎤
1. We know that lim → 1– and lim → 1+ ⎛ 1⎞ 2
x→0 x → sin x 5. lim cos ⎡p n2 + n ⎤ = lim cos ⎢ np ⎜1 + ⎟ ⎥
x 0
n→∞ ⎢⎣ ⎥⎦ n→∞ ⎢ ⎝ n⎠ ⎥
⎡ x ⎤ ⎡ sin x ⎤ ⎣ ⎦

∴ lim ⎢100 ⎥ + lim 99 = 100 + 98 = 198.
x→0⎣ sin x ⎦ x → 0 ⎢⎣ x ⎥⎦ ⎡ ⎛ 1 1 ⎞⎤

= lim cos ⎢ np ⎜1 + - 2 + ...⎟ ⎥
The correct option is (B) n→∞ ⎣ ⎝ 2 n 8n ⎠⎦
2. g [ f (x)] = [ f (x)]2 + 1, f (x) ≠ 2 ⎛ p p ⎞
= lim cos ⎜ np + -
+ ...⎟
3, f (x) = 2 n→∞ ⎝ 2 8n ⎠
∴ g [ f (x)] = sin2x + 1, x ≠ nπ ⎛ p ⎞
3, x = nπ = – lim sin ⎜ np -
+ ...⎟
n→∞ ⎝ 8n ⎠
RHL = lim g [ f (0 + h)] = lim(sin 2 h + 1) = 1.
p
= – lim ( -1) n -1 sin ⎛⎜ - ...⎞⎟
h→ 0 h→ 0

LHL = lim g [ f (0 - h)] = lim (sin 2 h + 1) = 1.
n→∞ ⎝ 8n ⎠
h→ 0 h→ 0
⎛ p ⎞

lim g [ f ( x )] = 1 = 0
⎜⎝∵ - ... → 0 as n → ∞⎟⎠
x→0 8n

The correct option is (A)
The correct option is (A)
⎡ ⎤ nk sin 2 ( n!) nk sin 2 ( n!)
3. lim ⎢ x + x + x - x ⎥ 6. lim = lim
x→∞ ⎣ ⎦ n→∞ n+2 n→∞ ⎛ 2⎞
n ⎜1 + ⎟
⎝ n⎠
x+ x+ x -x
= lim

x→∞ sin 2 ( n!)
x+ x+ x + x
= lim
n → ∞ 1- k ⎛ 2⎞
n ⎜1 + ⎟
x+ x ⎝ n⎠
= lim

x→∞ a finite quantity
x+ x+ x + x
=

1/ 2
⎛ 1 ⎞ [∵ sin2 (n !) always lies between 0 and 1. Also,

x ⎜1 + ⎟
⎝ x⎠
 since 1 – k > 0, \  n1 – k→ ∞ as n → ∞]
= lim

x→∞ ⎡⎛ 1 1 ⎞
1/ 2 ⎤
= 0.
x ⎢⎜1 + 1+ ⎟ + 1⎥
⎢⎝ x x⎠ ⎥
The correct option is (C)
⎣ ⎦
1 1 1 - cos 2( x - 1) 2 sin 2 ( x - 1)
=
= 7. lim = lim
1+1 2 x →1 x -1 x →1 x -1

The correct option is (A) 2 sin( x - 1)

= lim
⎡ x +1 p ⎤ x →1 ( x - 1)
4. lim x ⎢ tan -1 -
x →∞ ⎣ x + 2 4 ⎥⎦ 2 sin( x - 1) 2 sin(1 - h - 1)
⎛ x +1 ⎞ LHL = lim
= lim -
-1 ( x - 1) (1 - h - 1)
⎡ -1 x + 1 -1 ⎤ -1 ⎜ x + 2 ⎟ x →1- h→ 0
= lim x ⎢ tan
- tan 1⎥ = lim x tan ⎜ ⎟
x →∞ ⎣ x+2 ⎦ x →∞ x + 1
⎜1+ ⎟ 2 - sin h sin h
⎝ x + 2⎠
= lim = – 2 lim
h→ 0 -h h→ 0 h
⎛ 1 ⎞
tan -1 ⎜
-1 ⎞ -1 ⎛ ⎝ 2 x + 3 ⎟⎠ 1
=– 2⋅1=– 2
= lim x tan ⎜
= – lim ⋅
x →∞ ⎝ 2 x + 3 ⎟⎠ x →∞ ⎛ 1 ⎞ ⎛ 3⎞
⎜⎝ ⎟ ⎜⎝ 2 + ⎟⎠ 2 sin( x - 1) 2 sin(1 + h - 1)
2 x + 3⎠ x RHL = lim
= lim
1 1 x →1+ ( x - 1) h→ 0 (1 + h - 1)
= –1 ×
=–
2 2 2 sin h sin h

= lim = 2 lim = 2⋅1= 2

The correct option is (B) h→0 h h→0 h

Objective_Maths_JEE Main 2017_Ch 11.indd 26 01/01/2008 05:02:57


Limits  11.27

Since LHL ≠ RHL,


⎛ h2 ⎞
ln ⎜1 + ⎟
1 - cos 2( x - 1) ⎝ 1 + 2h ⎠
∴ lim
does not exist. = – lim

h→ 0 ⎛ h2 ⎞
x →1 x -1
⎜ 1 + 2h ⎟ (1 + 2h)

The correct option is (D) ⎝ ⎠

x5 x5 x5 ⎛ h2 ⎞
8. lim x
= lim x log 5
= lim , ln ⎜1 + ⎟
x →∞ 5 x →∞ e x →∞ e kx ⎝ 1 + 2h ⎠ 1
= – lim
⋅ = –1
where k = log 5
h→ 0 ⎛ h ⎞ 2 1 + 2h
⎜ 1 + 2h ⎟
x5 ⎝ ⎠
= lim
⎡ log(1 + x ) ⎤
x →∞ ⎛ k 2 x 2 k 3 x 3 k 4 x 4 x 5k 5 k 6 x 6 ⎞
 ⎢ Using xlim = 1⎥
⎜1 + kx + 2! + 3! + 4! + 5! + 6! + ...⎟ ⎣ →0 x ⎦
⎝ ⎠

The correct option is (B)
1
= lim
⎡ 1 e1 n e 2 n e( n -1) n ⎤
x →∞ ⎡⎛ 1 1 k2 1 k3 1 k4 1⎞ 12. lim ⎢ + + + ... + ⎥
⎢⎜ 5 + k ⋅ 4 + ⋅ + ⋅ + ⋅ n→∞ ⎢ n
⎣ n n n ⎥⎦
⎢⎣⎝ x x 2! x 3 3! x 2 4! x ⎟⎠
⎡1 + e1 n + (e1 n ) 2 + ... + (e1 n ) n -1 ⎤
k5 ⎛ k6 ⎞⎤ = lim ⎢

+ + ⎜ x + ...⎟ ⎥ n→∞ ⎢ n ⎥⎦
5! ⎝ 6! ⎠ ⎥⎦ ⎣
1 1 ⋅ ⎡⎣(e1 n ) n - 1⎤⎦ 1
=
=0 = lim
= (e – 1) lim 1 n
∞ n→∞ ⎛ e
n→∞ n ( e1 n - 1) - 1⎞

The correct option is (C) ⎜ 1n ⎟
1 1
⎝ ⎠
lim (cos x + sin x -1) = (e – 1) × 1 = (e – 1).

9. lim (cos x + sin x ) x = e x→0
x
x→0 ( - sin x + cos x )
The correct option is (C)
lim

= e x→0
1
(1 + h)sin (1 + h - [1 + h])

 (Using L’Hospital’s Rule) 13. RHL = lim
h→ 0 1+ h -1

= e1 = e
(1 + h)sin (1 + h - 1)

The correct option is (A)
= lim
h→ 0 h
2 2 - (cos x + sin x )3 ⎛0 ⎞ sin h
10. lim  ⎜⎝ form⎟⎠
= lim(1+ h) =1
x→
p 1 - sin 2 x 0 h→ 0 h
4
-3 (cos x + sin x ) 2 ( - sin x + cos x ) (1 - h)sin (1 - h - [1 - h])
= lim
LHL = lim

p -2 cos 2 x h→ 0 1- h -1
x→
4

 (Using L’Hospital’s Rule) (1 - h)sin (1 - h)

= lim = –∞
h→ 0 -h
-3 (cos x + sin x )(cos 2 x - sin 2 x )
= lim
Since LHL ≠ RHL,

x→
p -2 cos 2 x
4 x sin ( x - [ x ])
∴ lim
does not exist.
-3 (cos x + sin x )cos 2 x x →1 x -1
= lim

x→
p -2 cos 2 x
The correct option is (D)
4
2 sin x - sin 2 x
= lim

3 (cos x + sin x ) 3 ⎛ 1
= ⋅⎜ +
1 ⎞
⎟ =
3 14. f (x) = ∫ x3
dx
x→
p 2 2 ⎝ 2 2⎠ 2
4 d 2 sin x - sin 2 x 2 sin x - sin 2 x
dx ∫

The correct option is (A) ⇒ f ′ (x) =
dx =
x3 x3
ln(1 + 2h) - 2 ln(1 + h)
11. lim 2 sin x - sin 2 x ⎛0 ⎞
h→ 0 h2 ∴
lim f ′( x ) = lim 3
 ⎜⎝ form⎟⎠
x →0 x→0 x 0
ln(1 + h) 2 - ln(1 + 2h)
= – lim
2 cos x - 2 cos 2 x ⎛0 ⎞
h→ 0 h2
= lim  ⎜⎝ form⎟⎠
x→0 3x 2 0
⎛ (1 + h) 2 ⎞
ln ⎜ ⎟ -2 sin x + 4 sin 2 x ⎛0 ⎞
⎝ 1 + 2h ⎠
= lim  ⎜⎝ form⎟⎠
= – lim
x→0 6x 0
h→ 0 h2

Objective_Maths_JEE Main 2017_Ch 11.indd 27 01/01/2008 05:03:03


11.28  Chapter 11

-2 cos x + 8 cos 2 x (log x ) 2 ⎛∞ ⎞



= lim 20. lim  ⎜⎝ ∞ form⎟⎠
x→0 6 x →∞ x n

6 1
= =1 2 log x ⋅
6 x = lim 2 log x  ⎛∞ ⎞
lim ⎜⎝ ∞ form⎟⎠
The correct option is (D) x →∞ n x n -1 x →∞ n x n

p 2
15. ∵ < 1, = lim 2 n = 0
4 x →∞ n x
⎛p⎞ The correct option is (B)
∴ ⎜ ⎟ = 0
⎝ 4⎠
r r
⎛ x⎞ 21. tr = =
⎜⎝ ⎟⎠ r4 + r2 + 1 ( r 2 + 1) 2 - r 2
2
∴ lim =0
x → p / 2 ln(sin x ) 1⎛ 1 1 ⎞
=
⎜ - ⎟
The correct option is (C) 2 ⎝ r 2 - r + 1 r 2 + r + 1⎠
16. We know that |cosθ | ≤ 1 for all θ. 1⎡ 1 1 ⎤
=
⎢ - ⎥
So, if |cos n! px| < 1, 2 ⎣ r ( r - 1) + 1 ( r + 1) r + 1⎦
lim lim (1 + cos 2 m n!p x ) = (1 + 0) = 1 n n
1
m →∞ n→∞

∑ tr = ∑ 2 [ f ( r ) - f ( r + 1)] ,

and if |cosn! px| = 1, r =1 r =1

1 1
= [ f (1) - f ( n + 1)]
2m 2m
lim lim (1 + cos n!p x ) = lim lim (1 + 1 ) where f (r) =

m →∞ n→∞ m →∞ n→∞ r ( r - 1) + 1 2

= lim lim (1 + 1) = 2 1⎡ 1 ⎤ 1
m →∞ n→∞ =

⎢1 - ⎥ → as n → ∞

The correct option is (A) and (B) 2 ⎣ ( n + 1)n + 1⎦ 2
⎡ sin([ x - 3]) ⎤ ⎡ sin( - 4) ⎤
The correct option is (B)
17. LHL = lim ⎢ ⎥ = ⎢ ⎥
x → 0 ⎣ [ x - 3] ⎦ ⎣ -4 ⎦ 22. LHL = lim( -1)[ n - h] = lim( -1) n -1 = (–1)n – 1
-

h→ 0 h→ 0
⎡ sin 4 ⎤ 3p [ n + h]

= ⎢ ⎥ = –1 ∵  p < 4 <
RHL = lim( -1) = lim( -1) n = (– 1)n
⎣ 4 ⎦ 2 h→ 0 h→ 0

⎡ sin[ x - 3] ⎤ ⎡ sin( - 3) ⎤ Since LHL ≠ RHL
RHL = lim ⎢
⎥ = ⎢ ⎥
x → 0 ⎣ [ x - 3] ⎦+
⎣ -3 ⎦ ∴
lim( -1)[ x ] does not exist.
x→n
⎡ sin 3 ⎤ p The correct option is (D)

= ⎢ ⎥ = 0 ∵  < 3 < p.
⎣ 3 ⎦ 2 23. Since y = x – 1,

The correct option is (C) \ x = y + 1.
⎛ x2 + 1 ⎞ As (x, y) → (1, 0) along the line y = x – 1, x = y + 1 holds
18. We have, lim ⎜ - ax - b⎟ = 0
x →∞ ⎝ x + 1 ⎠ throughout.
y3 y3
( x 2 + 1) - ( ax + b)( x + 1) ∴ lim 3 = lim
⇒ lim
=0 x →1 x - y 2 - 1 y → 0 ( y + 1)3 - y 2 - 1
x →∞ x +1 y→0
y3 y2 0
2
x (1 - a) - ( a + b) x - b + 1
lim = lim = =0
⇒ lim
=0 y→0 3
y + 2 y + 3y 2 y→0 2
y + 2y + 3 3
x →∞ x +1
⇒ 1 – a = 0 and a + b = 0
The correct option is (C)
⇒ a = 1 and b = –1.
1 - 2 + 3 - 4 + 5 - 6 + ... - 2n
24. lim
n→∞

The correct option is (A) n2 + 1 + 4 n2 - 1
⎡ 1 1 1 1 ⎤ [1 + 3 + 5 + 7 + ... + ( 2n - 1)] - ( 2 + 4 + 6 + ... + 2n)
19. lim ⎢ + + + ... + ⎥ = lim

n→∞ ⎣1 ⋅ 2 2 ⋅ 3 3⋅ 4 n( n + 1) ⎦ n→∞ 1 1
n 1+ 2
+n 4- 2
⎡⎛ 1 1 ⎞ ⎛ 1 1 ⎞ ⎛ 1 1 ⎞ ⎛1 1 ⎞⎤ n n
= lim ⎢⎜ - ⎟ + ⎜ - ⎟ + ⎜ - ⎟ + ... + ⎜ -

n→∞ ⎣ ⎝ 1 2 ⎠ ⎝ 2 3 ⎠ ⎝ 3 4 ⎠ ⎝ n n + 1⎠ ⎥⎦ n n
[2 ⋅1 + ( n - 1) ⋅ 2] - [2 ⋅ 2 + ( n - 1) ⋅ 2]
= lim
2 2
⎡ 1 ⎤
= lim ⎢1 -
= 1 – 0 = 1. n→∞ ⎛ 1 1⎞
n→∞ ⎣ n + 1⎥⎦ n ⎜ 1+ 2 + 4 - 2 ⎟
⎝ n n ⎠

The correct option is (A)

Objective_Maths_JEE Main 2017_Ch 11.indd 28 01/01/2008 05:03:10


Limits  11.29

n n 8 x2 x2
⋅ 2n - 2 ( n + 1) = lim
⋅ 2 sin 2 ⋅ 2 sin 2
= lim 2 2 8
x→0 x 4 8
n→∞ ⎛ 1 1⎞
n ⎜ 1+ 2 + 4 - 2 ⎟ ⎛
2 2
⎝ n n ⎠ ⎛ x2 ⎞ x2 ⎞
⎜ sin ⎟ ⎛ 2 ⎞ 2 ⎜ sin ⎟ ⎛ x2 ⎞
2
32 x
n2 - n2 - n = lim 8 ⎜ 24 ⎟ ⎜ ⎟ ⎜ 28 ⎟ ⎜ ⎟

= lim
x→0 x
⎜ x ⎟ ⎝ 4⎠ ⎜ x ⎟ ⎝ 8⎠

n→∞ 1⎞ ⎝ 4 ⎠ ⎜⎝ ⎟
1
n ⎜ 1+ 2 + 4 - 2 ⎟ 8 ⎠
⎝ n n ⎠ 1
=

-n 32
= lim

n→∞ ⎛ 1 1⎞
The correct option is (C)
n ⎜ 1+ 2 + 4 - 2 ⎟
⎝ n n ⎠
28. lim ⎡⎣log n -1( n) ⋅ log n ( n + 1) ⋅ log n +1( n + 2)...log n k
-1 ( n
k
)⎤⎦
-1 -1 -1 n→ ∞
= lim
= = ⎡ log n log( n + 1) log( n + 2) log( nk ) ⎤
n→∞ 1 1 1+ 2 3 = lim ⎢
⋅ ⋅ ...
1+ 2 + 4 - 2 ⎥
n→∞ ⎢ log( n - 1) log n log( n + 1) log( nk - 1) ⎥⎦
n n ⎣

The correct option is (B) ⎛ log m ⎞

 ⎜⎝ Using log n m = log n ⎟⎠
⎛ x 4 sin (1 / x ) + x 2 ⎞
25. lim ⎜ ⎟ log nk
x→-∞ ⎝ 1 + | x |3 ⎠ log n ⎛∞ ⎞
= lim
= k lim  ⎜⎝ ∞ form⎟⎠
n→∞ log( n - 1) n→∞ log( n - 1)
1
- y 4 sin + y 2 1/ n
y = k lim
 (Using L’Hospital’s Rule)
= lim n→∞ 1 / n -1
y→∞ 1+ y 3
(Putting x = – y; as x→ – ∞, y→ ∞) ⎛ 1⎞
= k lim ⎜1 - ⎟ = k

n→∞ ⎝ n⎠
⎛ 1⎞
sin
⎜ y⎟ 1
The correct option is (C)
-⎜ ⎟+
⎜ 1 ⎟ y ⎡ 1 1 1 1 ⎤
⎝ y ⎠ -1 + 0 29. lim ⎢ + + + ... + ⎥
= lim = = –1 n→∞ ⎣1 ⋅ 3 3⋅5 5⋅ 7 ( 2n + 1)( 2n + 3) ⎦
y→∞ 1 1+ 0
1+ 3
y ⎡ 1 ⎛ 1 1⎞ 1 ⎛ 1 1⎞ 1⎛ 1 1 ⎞⎤
= lim ⎢ ⎜ - ⎟ + ⎜ - ⎟ ... + ⎜
- ⎟

The correct option is (B) n→∞ ⎣ 2 ⎝ 1 3 ⎠ 2 ⎝ 3 5⎠ 2 ⎝ 2n + 1 2n + 3 ⎠ ⎥⎦

2 x + 23 - x - 6 1⎛ 1 ⎞ 1 1
26. lim = lim
⎜⎝1 - ⎟⎠ = (1 – 0) =
x→2 2 - x / 2 - 21 - x n→∞ 2 2n + 3 2 2
( 22 x + 23 - 6 ⋅ 2 x ) / 2 x
The correct option is (B)
= lim

x→2 1 2
- nx
2x / 2 2x ⎛ 1x 1 1 1 ⎞
1 + 2 x + 3 x + ... + n x
22 x - 6 ⋅ 2 x + 8 ( 2 x - 4 ) ( 2 x - 2) 30. lim ⎜ ⎟
= lim
= lim x →∞ ⎜ n ⎟⎠
x/2 ⎝
x→2 2 -2 x→2 ( 2 x / 2 - 2)
n

( 2 x / 2 + 2) ( 2 x / 2 - 2) ( 2 x - 2) ⎛ 1y + 2 y + 3 y + ... + n y ⎞ y
= lim
= lim ⎜

y→0 ⎝ n ⎠
x→2 ( 2 x / 2 - 2)
= lim ( 2 x / 2 + 2) ( 2 x - 2) = (2 + 2) ⋅ (4 – 2) = 8
lim
n ⎛ 1y + 2 y + 3 y + ... + n y ⎞
-1⎟
x→2 y ⎜⎝ n ⎠
= e
y→0


The correct option is (A) ⎛ 1y + 2 y + 3 y + ... + n y - n ⎞
lim
y→0 ⎜ ⎟⎠

8 ⎡ x 2
x x x 2 2 2⎤
= e
y
27. lim ⎢1 - cos - cos + cos cos ⎥ ⎡ (1y -1) ( 2 y -1) ( 3 y -1) ( n y -1) ⎤
x→0 x 8 ⎢⎣ 2 4 2 4 ⎥⎦ lim
y→0 ⎢
+ + + ...+ ⎥
⎣ y y y y ⎦
=
e
8 ⎡⎛ x2 ⎞ x4 ⎛ x2 ⎞ ⎤
= lim 8 ⎢⎜1 - cos ⎟ - cos ⎜1 - cos ⎟ ⎥
= e (log 1 + log 2 + log 3 + … + log n)

x → 0 x ⎢⎝ 2⎠ 4 ⎝ 2 ⎠ ⎥⎦
⎣ = e log (1 ⋅ 2 ⋅ 3 … n) = n!

8 ⎛ x ⎞⎛
2
x ⎞
2

The correct option is (A)
= lim 8 ⎜1 - cos ⎟ ⎜1 - cos ⎟

x→0 x ⎝ 2 ⎠⎝ 4⎠

Objective_Maths_JEE Main 2017_Ch 11.indd 29 01/01/2008 05:03:16


11.30  Chapter 11

31. lim (1 + x) (1 + x2) (1 + x4) … (1 + x2n) 2


n→∞

lim Sn = –
2 4 2n
n→∞ 3
(1 - x )(1 + x )(1 + x )(1 + x )...(1 + x )
The correct option is (B)
= lim

n→∞ 1- x
(1 + a3 ) + 8e1 x ⎛∞ ⎞
(1 - x 2 )(1 + x 2 )(1 + x 4 )...(1 + x 2 n ) 35. We have 2 = lim ⎜⎝ ∞ form⎟⎠ (1)
= lim
x → 0 1 + (1 - b3 ) e1 x
n→∞ 1- x  

. . . 0 + 8e1 x ( -1 x 2 )

. . . ⇒ 2 = lim

x → 0 0 + (1 - b3 ) e1 x ( -1 x2 )

. . .
 (Using L’Hospital’s Rule)
1 - x 4n + 2 1
= lim
= for |x| < 1 ⇒ 1 – b3 = 4
n→∞ 1- x 1- x
⇒ b3 = – 3
The correct option is (B)
⇒ b = (–3)1/3
32. Case I:  n is a positive integer
(1 + a3 ) + 8e1 x
xn nx n -1 ∴ From Eq. (1), 2 = lim
lim = lim x→0 1 + 4e1 x
x →∞ ex x →∞ ex
⇒ 1 + a3 = 2 i.e., a = 1
n ( n - 1) x n - 2 n!

= lim = … = lim Hence a = 1 and b = (–3)1/3.
x →∞ ex x →∞ e x
The correct option is (A)
 (Using L’Hospital’s Rule repeatedly)
36. x2 + 4x + 5 = (x + 2)2 + 1 ≥ 1. So, a = 1.
=0
1 - cos 2q 2 sin 2 q
Case II:  n is a negative integer. Also, b = lim
= lim = 2.
2
n -m q →0 q q →0 q2
x x n
lim = lim
e x x →∞ e x
x →∞ ∴
∑ ar ⋅ bn - r = bn + abn – 1 + a2 bn – 2 + … + an
r=0
(Putting n = – m, where m is a positive integer)
1 1 ⎡ ⎛ a ⎞ n +1 ⎤ ⎡ ⎛ 1 ⎞ n +1 ⎤
= lim m x = = 0. b n ⎢1 - ⎜ ⎟ ⎥ 2n ⎢1 - ⎜ ⎟ ⎥
x →∞ x e ∞ ⎢⎣ ⎝ b ⎠ ⎥⎦ ⎢⎣ ⎝ 2 ⎠ ⎥⎦
Case III:  n = 0 = =
a 1
1- 1-
xn 1 1 b 2
lim x = lim x = = 0.
x →∞ e x →∞ e ∞ 2n +1 ( 2n +1 - 1)
xn
= = (2n + 1 – 1)
Hence, lim x = 0 for all values of n. 2n +1
x →∞ e

The correct option is (B)
The correct option is (B)
1 ⋅ 2 + 2 ⋅ 3 + 3 ⋅ 4 + ... + n ( n + 1)
x n + x n -1 + x n - 2 + ... + x 2 + x - n 37. lim
⎛0 ⎞ n→∞ n3
33. lim  ⎜⎝ form⎟⎠
x →1 x -1 0
S n ( n + 1) S n 2 + Sn
nx n -1 + ( n - 1) x n - 2 + ... + 2 x + 1 = lim
3
= lim
= lim
n→∞ n n→∞ n3
x →1 1

 (Using L’Hospital’s Rule) 1 ⎡ n ( n + 1)( 2n + 1) n ( n + 1) ⎤
= lim
+
n ( n + 1) n→∞ n3 ⎢⎣ 6 2 ⎥⎦
= n + (n – 1) + … + 2 + 1 =

2 ⎡1 ⎛ 1⎞ ⎛ 1⎞ 1 ⎛ 1 1 ⎞ ⎤

The correct option is (A) = lim ⎢ ⎜1 + ⎟ ⎜ 2 + ⎟ + ⋅ ⎜ + 2 ⎟ ⎥

n→∞ ⎣ 6 ⎝ n⎠ ⎝ n⎠ 2 ⎝ n n ⎠ ⎦
12 + 22 + 32 + ... + r 2
34. tr = 1 1
13 + 23 + 33 + ... + r 3 =
×1×2= .
2 6 3
r ( r + 1)( 2r + 1) ⎛ 2 ⎞
The correct option is (C)
=
⋅⎜
6 ⎝ r ( r + 1) ⎟⎠
log(1 + x + x 2 ) + log(1 - x + x 2 )
2⎛1 1 ⎞ 38. lim
= ⎜ +
⎟ x→0 sec x - cos x
3 r r + 1⎠

log ⎡⎣(1 + x 2 ) 2 - x 2 ⎤⎦
2 ⎡ ⎛ 1 ⎞ ⎛ 1 1⎞ ⎛ 1 1 ⎞ ⎛1 1 ⎞⎤ = lim

∴ Sn = ⎢ - ⎜1+ ⎟ + ⎜ + ⎟ - ⎜ + ⎟ + … ± ⎜ +

3 ⎣ ⎝ 2 ⎠ ⎝ 2 3⎠ ⎝ 3 4 ⎠ ⎝ n n + 1⎟⎠ ⎥⎦ x→0 (1 - cos 2 x ) cos x

2⎛ 1 ⎞ log(1 + x 2 + x 4 ) ⎛0 ⎞

= ⎜ -1 ± ⎟ = lim
⎜⎝ form⎟⎠
3⎝ n + 1⎠ x→0 sin x tan x  0

Objective_Maths_JEE Main 2017_Ch 11.indd 30 01/01/2008 05:03:23


Limits  11.31

log[1 + x 2 (1 + x 2 )] 1 3x +1 - 5 x +1 3 ⋅ 3x - 5 ⋅ 5 x
= lim
⋅ x 2 (1 + x 2 ) ⋅ 41. lim = lim
x→0 x 2 (1 + x 2 ) sin x tan x 2 x →∞ 3x - 5 x x →∞ 3x - 5 x
⋅ ⋅x x
x x ⎛ 3⎞
⎡ log(1 + x ) ⎤ 3⋅ ⎜ ⎟ - 5
= 1. ⎢as lim
= 1⎥ ⎝ 5⎠ -5
⎣ x → 0 x ⎦
= lim =
x →∞ ⎛ 3⎞
x
-1

The correct option is (A) ⎜⎝ ⎟⎠ - 1
⎛ h⎞ 5
loge e ⎜1 - ⎟ - 1
ln(e - h) - 1 ⎝ e⎠ ⎛ ⎞
39. LHL = lim = lim
= 5. ⎜⎝∵ nlim a n = 0, if - 1 < a < 1⎟
h→ 0 | e - h - e | h→ 0 | -h | →∞ ⎠

The correct option is (A)
⎛ h⎞
log e + log ⎜1 - ⎟ - 1
1⎛
n -1 ⎞
⎝ e⎠ 1/ n 2/ n

= lim 42. lim ⎜1 + e + e + ... + e n

h→ 0 h n→∞ n
⎝ ⎠
h h2
( )
n
1/ n
- - 2 - ...
1 1 1- e 1- e

= lim e 2e =– = lim ⋅
= lim
n→∞ ⎛ 1 1 1 ⎞
h→ 0 | e + h - e | e n→∞ n 1 - e1/ n
n ⎜1 - 1 - - ⋅ 2 ...⎟
⎝ n 2! n ⎠
ln (e + h) - 1
RHL = lim
1- e 1- e
h→ 0 |e+h-e| = lim
= =e–1
n→∞ 1 1 -1
-1 - ⋅ ...
⎛ h⎞ 2! n
log e ⎜1 + ⎟ - 1
⎝ e⎠
The correct option is (C)

= lim
h→ 0 |h|
sin x
⎛ h⎞ 1+
x + sin x x 1+ 0
log e + log ⎜1 + ⎟ - 1 43. lim = lim = = 1.
⎝ e⎠ x →∞ x - cos x x →∞ cos x 1- 0

= lim 1-
h→ 0 h x
h h2 ⎡ sin x ⎛1⎞ ⎤
- 2 + ... ⎢∵ lim = lim y sin ⎜ ⎟ = O × (a finite quantity) ⎥
1
= lim e 2e =
 ⎢ x →∞ x y →0 y
⎝ ⎠ ⎥
h→ 0 h e ⎢ cos x ⎥
Since LHL ≠ RHL ⎢ = 0. Similarly lim =0 ⎥
⎣ x →∞ x ⎦
ln x - 1
The correct option is (B)
∴ lim does not exist.
x→e | x - e | n

The correct option is (D) 44. Sn= ∑ ai , lim an = a


n→∞
i =1
40. We have Sn + 1 – Sn = an + 1
x1 = 3, xn + 1 = 2 + xn an + 1 2a
So, lim = lim =0
x2 =
2 + x1 = 2+3 = 5 n → ∞ n( n + 1) n → ∞ n( n + 1)

2
x3 = 2 + x 2 = 2 + 5

The correct option is (A)
∴ x1 > x2 > x3

⎡⎛ 1⎞
13 ⎤
45. lim ⎡ n2 - n3 + n⎤ = lim n ⎢⎜ -1 + ⎟ + 11 3 ⎥
3
can be easily shown by mathematical induction that the
It
sequence x1, x2, …xn, … is a monotonically decreasing ⎢
n→∞ ⎣ ⎥
⎦ n→∞ ⎢ ⎝ n⎠ ⎥⎦

sequence bounded below by 2. So it is convergent.
⎛1 ⎞
Let lim xn = x. Then
⎜⎝ - 1⎟⎠ + 1
n→∞ n
= lim n ⋅ 23 13
n→∞ ⎛1 ⎞ ⎛1 ⎞
xn + 1 =
2 + xn ⎜⎝ - 1⎟ + 1 - ⎜⎝ - 1⎟
n ⎠ n ⎠
⇒ lim xn + 1 = 2 + lim xn
n→∞ n→∞
⎛ a3 + b3 ⎞
⇒ x = 2 + x


 ⎜ Using a + b = 2 ⎟
⎝ a - ab + b 2 ⎠
⇒ x2 – x – 2 = 0
1 1 1
= lim
= =
⇒ (x – 2)(x + 1) = 0
n→∞ ⎛ 1 23 13
1 + 1 + 1 3
⎞ ⎛ 1 ⎞
⇒ x = 2
(∵ xn > 0 ∀n, ∴ x > 0) ⎜⎝ - 1⎟⎠ + 1 - ⎜⎝ - 1⎟⎠
n n

The correct option is (B)

The correct option is (A)

Objective_Maths_JEE Main 2017_Ch 11.indd 31 01/01/2008 05:03:30


11.32  Chapter 11

4 3
n5 + 2 - n 2 + 1 A h 2 hr - h2
46. lim 50. lim = lim
( 2 hr - h )
n→∞ 5 2 3 3
n4 + 2 - n3 + 1 h→0 P h→0
+ +
2
8 + 2 hr
2 1
5
n5 4 4 1 +
- n2 3 3 1 + 2 h ⋅ h 2r - h
= lim
n n
= lim
( )
h→0 3
n→∞ 4 5 2 1 8⋅ h ⋅ h 2r - h + 2r
n 5 1 + 4 - n3 2 2 1 + 3
n n
2r - h
n5 4 4 2 n2 3 1
= lim
( )
3
32
1+ 5 - 3 2 3 1+ 2 h→0
8 2r - h + 2r
n
= lim 4 5
n n n
n→∞ n
5 1+
2 n3 2 2 1 2r 1
32
- 1+ 3
= =
n 4 n3 2
n n
( )
3
8 2 2r 128r

(Dividing the numerator and denominator by the highest

The correct option is (B)
power n3/2)
1 2 1 3 1 ⎛ 1 - cos 2( x - 2) ⎞
4 1+ - 1+ 2 51. lim ⎜ ⎟
n 14
n5 n5 6 n = 0 - 0 = 0. x → 2⎝ x-2 ⎠
= lim

n→∞ 1 5 2 1 0 -1 2 | sin( x - 2) |
7 10
1+ 4 - 2 1+ 3 = lim

n n n x→2 x-2
The correct option is (B) which doesn’t exist as LHL = - 2 whereas

47. Minimum power in numerator on x is 3. So n = 3. RHL =
2
The correct option is (C)
The correct option is (B)
2 x +33 x +55 x x x x x
48. lim 52. Required limit= lim cos cos 2 cos 3 ... cos n
x→∞ 3x - 2 + 2 x - 3 3 2 2 n→∞ 2 2
1 ⎧ x x ⎛ x x ⎞⎫
= lim
cos ... cos n - 1 ⎜ 2 sin n cos n ⎟ ⎬
=
lim
2 x +33 x +55 x n→∞ x ⎨⎩ 2 2 ⎝ 2 2 ⎠⎭
2 sin n
x→∞ 2 3 3 2
x 3- + x 3 2-
x x 1 ⎧ x ⎛ x x ⎞⎫
= lim
cos ... ⎜ 2 cos n - 1 sin n - 1 ⎟ ⎬
3 5 n→∞ 2 x ⎨⎩ 2 ⎝ 2 2 ⎠⎭
1/ 6
+ 3/10 2+ 2 sin n
= lim
x x 2
x→∞ 2 1 3
…………………………………………………………..
3 - + 1/ 6 3 2 -
x x x
…………………………………………………………..

(Dividing the numerator and denominator by the highest ⎛ x x⎞
1
power x1/2) = lim
2 cos sin ⎟
n→∞ x ⎜⎝ 2
n 2 ⎠
2 2 sin n
=
2
3 sin x

The correct option is (A) = lim

n→∞ n ⎛ x⎞
2 sin ⎜ n ⎟
⎝2 ⎠
x 3
z 2 - ( z - x )2
49. lim ⎧ ⎛ x⎞ ⎫
x→0 3
( 8 xz - 4 x 2 + 3 8 xz ) 4 ⎪⎪ ⎜⎝ n ⎟⎠ ⎪⎪
sin x 2 sin x
3 =
lim ⎨ ⎬ =
x 2 xz - x 2 x n → ∞
⎪ sin ⎛⎜ ⎞⎟ ⎪
x x
= lim

x→0 ( 3 x 3
8z - 4 x + 3 8z 3
x )4 ⎪⎩ ⎝ 2n ⎠ ⎪⎭

x4 3 3
2z - x The correct option is (B)
= lim

( )
x→0 4 53. The (r + 1)th term of the series is
x4 3 3
8z - 4 x + 3 8z n-r
3
tr + 1 = ( r + 1) ∑ k
2z
= k =1

( )
4
3
2 8z ⇒ tr + 1 = (r + 1){1 + 2 + 3 + … (n – r) terms}


1 1
= ⇒ tr + 1 = ( r + 1) ( n - r )( n - r + 1)

223 3 ⋅ z 2
1

The correct option is (B) ⇒ tr + 1 = ( r + 1)( n2 - rn + n - rn + r 2 - r )

2

Objective_Maths_JEE Main 2017_Ch 11.indd 32 01/01/2008 05:03:35


Limits  11.33

1
According to Squeeze Principle or Sandwich Theorem, we
⇒ tr + 1 =
( r + 1)( r 2 - (1 + 2n)r + n2 )
2 have
x
1 L= .

⇒ tr + 1 = ( r 3 - 2nr 2 + ( n2 - 2n - 1)r + n2 )
3
2
n -1
The correct option is (C)

Now, S= ∑ tr + 1 55. tan 2θ =
sin 2q
r=0
cos 2q
1 n 3
∴ S =
∑ {r - 2nr 2 + ( n2 - 2n - 1)r + n2}
2 r =1 ⇒

1
=
2 sin q cos q
cot 2q cos 2 q - sin 2 q
2
1 ⎡ ⎧ n( n + 1) ⎫ ⎧1 ⎫
⇒ S =
⎨ ⎬ - 2n ⎨ n ( n + 1)( 2n + 1)⎬ cos 2 q - sin 2 q
2 ⎢⎣ ⎩ 2 ⎭ ⎩ 6 ⎭ ⇒ cot 2θ =

2 sin q cos q
⎧1 ⎫ ⎤ cos 2 q sin 2 q

+ ( n2 - 2n - 1) ⎨ n( n + 1)⎬ + n2 ( n)⎥ ⇒ 2cot 2θ =
-
⎩2 ⎭ ⎦ sin q cos q sin q cos q

Solving and rearranging, we have, ⇒ 2cot 2θ = cot θ – tan θ

1 4 ⇒ tan θ = cot θ – 2 cot 2θ(1)
S=
{n - 11n3 - 19n2 + 6 n}
24 Now, tan θ = cot θ – 2 cot 2θ

S 1 ⎛ n4 - 11n3 - 19n2 + 6 n ⎞
∴ lim 4 = lim
⎜ ⎟ 1 q 1 q
n→∞ n n → ∞ 24 ⎝ n4 ⎠ ⇒
tan = cot - cot q
2 2 2 2
1 ⎛ 11 19 6 ⎞ 1 q 1 q 1

= lim ⎜1 - - 2 + 3 ⎟ ⇒ 2 tan 2 = 2 cot - cot q

24 n → ∞ ⎝ n n n ⎠ 2 2 2 2 2
n -1⎧ n-r ⎫⎪
………
1 ⎪ 1

lim
n→∞ n 4 ∑ ⎨( r + 1) ∑ k ⎬ = 24

………
r = 0⎪
⎩ ⎪
k =1 ⎭
q1 1 q 1 q

The correct option is (A) ⇒
n
n
= n cot n - n - 1 cot n - 1
tan
2 2 2 2 2 2
1 ⎛ n 2 ⎞ 1 q
54. Let L = lim ⎜ ∑ [k x ]⎟ ⇒ S = - 2 cot 2q + n cot n

2 2
n → ∞ n3 ⎝ ⎠
k =1
⎛ 1 q 1 q 1 q⎞

= lim ⎜ tan q + tan + 2 tan 2 +… + n tan n ⎟

Since k2x – 1 ≤ [k2x] < k2x n→∞ ⎝ 2 2 2 2 2 2 ⎠
n n n
⎛ 1 q⎞

∑ ( k 2 x - 1) ≤ ∑ [k 2 x] < ∑ k 2 xa ∴
lim S = lim ⎜ - 2 cot 2q + n cot n ⎟
n→∞ n→ ∞⎝ 2 2 ⎠
k =1 k =1 k =1

⎛ ⎞ n n n ⎛ n ⎞ ⎡ q ⎤
⇒ x ⎜ ∑ k 2 ⎟ - ∑ (1) ≤
∑ [k 2 x ] < x ⎜ ∑ k 2 ⎟ 1 ⎢ 2n ⎥
⎝k =1 ⎠ k =1 ⎝k =1 ⎠
= - 2 cot 2q + lim ⎢ ⎥
k =1 n→∞q ⎢ q
tan n ⎥
n ⎢⎣ 2 ⎥⎦
xn( n + 1)( 2n + 1) xn ( n + 1)( 2n + 1)

- n ≤ ∑ [k 2 x ] <
= - 2 cot 2q +
1
6 k =1 6 q
3

Dividing throughout by n , we have
The correct option is (B)
xn ( n + 1)( 2n + 1) 1 n
[k 2 x ] xn ( n + 1)( 2n + 1) 4 3n - 2 - 9 n + 1

6 n3
- 2 ≤
n
∑ n 3
<
6n3 56. lim
n→∞ 82 n - 1 - 9 n - 1
k =1

x⎛ 1⎞ ⎛ 1⎞ 1 n
[k 2 x ] x ⎛ 1⎞ ⎛ 1⎞ 4 - 2 ⋅ 64 n - 9 ⋅ 9n

⎜1 + ⎟ ⎜2 + ⎟⎠ - 2 ≤ ∑ 3 < ⎜⎝1 + ⎟ ⎜2 + ⎟ = lim

8-1 ⋅ 64 n - 9-1 ⋅ 9n
6⎝ n⎠ ⎝ n n k =1 n 6 n⎠ ⎝ n⎠ n→ + ∞
n
Taking limits as n→ ∞, we get
⎛ 9⎞
4- 2 - 9 ⎜ ⎟
⎝ 64 ⎠
⎧x ⎛ 1⎞ ⎛ 1⎞ 1 ⎫ x⎛ 1⎞ ⎛ 1⎞ = lim

lim ⎨ ⎜1 + ⎟ ⎜ 2 + ⎟ - 2 ⎬ ≤ L < lim ⎜1 + ⎟ ⎜ 2 + ⎟ n→ + ∞ ⎛ 9⎞
n
n → ∞ ⎩6 ⎝ n⎠ ⎝ n⎠ n ⎭ n→∞ 6 ⎝ n⎠ ⎝ n⎠ 8-1 - 9-1 ⎜ ⎟
⎝ 64 ⎠
1
Since, as n→ ∞, we have → 0
4- 2 - 0 1
n = -1
=
x x 8 -0 2

≤L<
3 3

Objective_Maths_JEE Main 2017_Ch 11.indd 33 01/01/2008 05:03:42


11.34  Chapter 11

n
⎛ 64 ⎞ 1
4 -2 ⎜ ⎟ - 9 = lim

⎝ 9⎠ x →∞ ⎡⎛ 1 1 k2 1 k3 1 k4 1⎞
= lim
⎢⎜ 5 + k ⋅ 4 + ⋅ + ⋅ + ⋅
n→- ∞ ⎛ 64 ⎞
n
⎢⎣⎝ x x 2! x 3 3! x 2 4! x ⎟⎠
8-1 ⎜ ⎟ - 9 -1
⎝ 9⎠
k5 ⎛ k6 ⎞⎤
0-9 + + ⎜ x + ...⎟ ⎥
= = 81 5! ⎝ 6! ⎠ ⎥⎦
0 - 9 -1
1
Hence, limit does not exist. =
=0

The correct option is (C)

The correct option is (C)
x - ai
57. We have, Ai = , i = 1, 2, …, n r r
| x - ai | 60. tr = =
4 2
and, a1 < a2 < … an – 1 < an r + r +1 ( r + 1) 2 - r 2
2

Let x be in the left neighbourhood of am 1⎡ 1 1 ⎤


=
-
Then,  x – ai < 0 for i = m, m + 1, … n 2 ⎢⎣ r 2 - r + 1 r 2 + r + 1⎥⎦
and  x – ai > 0 for i = 1, 2, …, m – 1 1⎡ 1 1 ⎤
x - ai =
⎢ - ⎥
∴ Ai = = –1, for i = m, m + 1, …, n 2 ⎣ r ( r - 1) + 1 ( r + 1) r + 1⎦
- ( x - ai )
x - ai n n
1
and,  Ai =
x - ai
= 1, for i = 1, 2, …, m – 1 ∴
∑ tr = ∑ 2 [ f ( r ) - f ( r + 1)] ,
r =1 r =1
Similarly, if x is in the right neighbourhood of ai 1
where,  f (r) =

Then,  x – ai< 0 for i = m + 1, …, n r ( r - 1) + 1
and  x – ai> 0 for i = 1, 2, …, m 1

= [ f (1) - f ( n + 1)]
x - ai 2
∴ Ai = = –1 for i = m + 1, …n
- ( x - ai ) 1⎡ 1 ⎤ 1
x - ai
= ⎢1 - ⎥ → as n→ ∞
and,  Ai = = 1 for i = 1, 2, …, m 2 ⎣ ( n + 1)n + 1⎦ 2
x - ai

The correct option is (B)
Now, 
lim ( A1 A2 ... An ) = (–1)n – m + 1 nx
x → am-
⎡11/x + 21/x + 31/x + ... + n1/x ⎤

and,  lim ( A1 A2 ... An ) = (–1) n–m 61. lim ⎢ ⎥
x → am+
x →∞ ⎢
⎣ n ⎥⎦
n
Hence, lim ( A1 A2 ... An ) does not exist.
x → am ⎡1y + 2 y + 3 y + ... + n y ⎤ y
= lim ⎢


The correct option is (D) y→0 ⎢ n ⎥⎦

sin x n ⎡1y + 2 y + 3 y + ... + n y ⎤
58. We know that lim → 1– lim ⎢ -1⎥
x→0 x =
e y⎣
y→0
n ⎦
x
lim → 1+ lim
⎡1y + 2 y + 3 y + ... + n y - n ⎤
x → 0 sin x y→0 ⎢ ⎥
y
=
e ⎣ ⎦

⎡ x ⎤ ⎡ sin x ⎤ ⎡ (1y -1) ( 2 y -1) ( 3 y -1) ( n y -1) ⎤


∴ lim ⎢100 ⎥ + lim 99 = 100 + 98 = 198. lim
y→0 ⎢
+ + + ...+ ⎥
x→0⎣ sin x ⎦ x → 0 ⎢⎣ x ⎥⎦ =
e ⎣ y y y y ⎦


The correct option is (B) = e (log 1 + log 2 + log 3 + … + log n)
x5

= e log (1 ⋅ 2 ⋅ 3 …. n) = n!
59. lim The correct option is (A)
x →∞ 5 x
62. lim (1 + x) (1 + x2) (1 + x4) … (1 + x2n)
x5 n→∞
= lim
x →∞ e x log 5 (1 - x )(1 + x )(1 + x 2 )(1 + x 4 )...(1 + x 2 n )
= lim

n→∞ 1- x
x5
= lim
, where k = log 5 (1 - x 2 )(1 + x 2 )(1 + x 4 )...(1 + x 2 n )
x →∞ e kx
= lim

x5
n→∞ 1- x
= lim

. . .
x →∞ ⎛ k 2 x 2 k 3 x 3 k 4 x 4 x 5k 5 k 6 x 6 ⎞
⎜1 + kx + 2! + 3! + 4! + 5! + 6! + ...⎟
. . .
⎝ ⎠
. . .

Objective_Maths_JEE Main 2017_Ch 11.indd 34 01/01/2008 05:03:47


Limits  11.35

1 - x 4n + 2 1 4
n5 + 2 - n2 + 1
3
= lim
= for |x| < 1 66. lim
n→∞ 1- x 1- x n→∞ 5 2
n4 + 2 - n3 + 1
The correct option is (B)
63. Case I:  n is a positive integer 2 1
5
n5 4 4 1 +
- n2 3 3 1 + 2
n n
xn nx n -1 = lim

lim = lim n→∞ 4 5 2 3 2 1
x →∞ e x x →∞ ex n 5 1+ 4 - n 2 1+ 3
n n
n ( n - 1) x n - 2 n!

= lim = … = lim n5 4 4 2 n2 3 1
x →∞ ex x →∞ e x
1+ 5 - 3 2 3 1+ 2
32
 [Using L’Hospital’s rule repeatedly] n
= lim 4 5
n n n
=0
n→∞ n
5 1+
2 n3 2 2 1
32
- 1+ 3
Case II:  n is a negative integer. n n 4 n3 2 n

[Dividing the numerator and denominator by the highest
xn x -m
lim x
= lim power n3/2]
x →∞e x →∞ e x

[Putting n = – m, where m is a positive integer] 1 4 2 1 1


1+ 5 - 5 6 3 1+ 2
1 1 n 14
n n n 0-0
= lim m x = =0 = lim
= = 0.
x →∞ x e ∞
n→∞ 1 5 2 2 1 0 -1
1 + - 1 +
Case III:  n = 0 n7 10 n4 n3
xn 1 1
The correct option is (B)
lim x = lim x = =0
x →∞ e x →∞ e ∞
x 3
z 2 - ( z - x )2
xn 67. lim
Hence, lim x = 0 for all values of n. x→0 3
( 8 xz - 4 x 2 + 3 8 xz ) 4
x →∞ e
The correct option is (B) 3
x 2 xz - x 2
64. x2 + 4x + 5 = (x + 2)2 + 1 ≥ 1. = lim

x→0 ( 3 x 3
8z - 4 x + 3 8z 3
x )4
So, a = 1
1 - cos 2q 2 sin 2 q x4 3 3
2z - x 3
2z
Also, b = lim = lim =2 = lim
4
= 4
q →0 q2 q →0 q 2 x→0 4 3 ⎡3 ⎤ 3 ⎡ ⎤ 3
n
x ⎣ 8z - 4 x + 8z ⎦ ⎣2 8z ⎦

∑ ar ⋅ bn - r = bn + abn – 1 + a2 bn – 2 + … + an =

1
r=0
223 3 ⋅ z
⎡ ⎛ a⎞ n +1 ⎤ ⎡ ⎛ 1 ⎞ n +1 ⎤
b n ⎢1 - ⎜ ⎟ ⎥ 2n ⎢1 - ⎜ ⎟ ⎥
The correct option is (B)
⎢⎣ ⎝ b ⎠ ⎥⎦ ⎣⎢
⎝ 2⎠ ⎥

= =
a 1 h 2 hr - h2
1- 1- A
b 2 68. lim 3
= lim 3
h→0 +
P h→0 +

n +1 n +1
- 1) 8 ⎡ 2hr - h2 + 2hr ⎤

=
2 (2
= (2n + 1 – 1) ⎣⎢ ⎦⎥
2n +1
h ⋅ h 2r - h

The correct option is (B)
= lim 3
h→0
2 2 8⋅ h ⋅ h ⎡⎣ 2r - h + 2r ⎤⎦
log(1 + x + x ) + log(1 - x + x )
65. lim
x→0 sec x - cos x 2r - h

= lim
log ⎡⎣(1 + x 2 ) 2 - x 2 ⎤⎦ h→0 3
8 ⎣⎡ 2r - h + 2r ⎤⎦
= lim

x→0 (1 - cos 2 x ) cos x
2r 1
log(1 + x 2 + x 4 ) ⎛0 ⎞
= =
( )
3
= lim
form⎟ 128r
x→0 sin x tan x  ⎝⎜ 0 ⎠ 8 2 2r

The correct option is (B)
log(1 + x 2 (1 + x 2 )) 1
= lim

x→0 2
x (1 + x ) 2
⋅ x 2 (1 + x 2 ) ⋅
sin x tan x 2
⋅ ⋅x 69. lim
(
cos x + p
6 = lim )
cos p + z
2 ( )
x x x →p /3 (1 - 2 cos x ) 2/3
( )
z→ 0 ⎡ 2/3
⎛ log(1 + x ) ⎞ 1 - 2 cos p +z⎤
= 1. ⎜ as lim
= 1⎟ ⎣ 3 ⎦
⎝ x→0 x ⎠

The correct option is (A) [putting x –π/3 = z]

Objective_Maths_JEE Main 2017_Ch 11.indd 35 01/01/2008 05:03:54


11.36  Chapter 11

- sin z 1 -1 1
= lim

=- ⋅ =
(1 - cos z + )
2/3
z→ 0
3 sin z 2 2 2

RHL = lim

1 sin 2
x

1 ( )
⎛ z⎞ ⎛ z⎞ x
-2 sin ⎜ ⎟ cos ⎜ ⎟ x→ 0 2 +

2 1 + cos x
⎝ 2⎠ ⎝ 2⎠
= lim
2/3 2/3 1 1 1
z→ 0 ⎡ ⎛ z⎞⎤ ⎡ ⎛ z⎞ ⎛ z⎞⎤
= ⋅ =
⎢ 2 sin ⎜⎝ 2 ⎟⎠ ⎥ ⎢sin ⎝⎜ 2 ⎟⎠ + 3 cos ⎜⎝ 2 ⎟⎠ ⎥ 2 2 2
⎣ ⎦ ⎣ ⎦

Hence, limit does not exist.
1/3
⎡ ⎛ z⎞⎤ ⎛ z⎞
The correct option is (C)
-21/3 ⎢sin ⎜ ⎟ ⎥ cos ⎜ ⎟
⎣ ⎝ 2 ⎠ ⎦ ⎝ 2⎠ -21/3 ⋅ 0 ⋅1 x + 7 - 3 2x - 3
= lim
= =0 73. lim
( )
z→ 0 ⎡ 2/3 2/3
⎛ z⎞ ⎛ z⎞⎤ 3 x→ 2 3 x + 6 - 2 3 3x - 5
⎢sin ⎜⎝ 2 ⎟⎠ + 3 cos ⎜⎝ 2 ⎟⎠ ⎥
⎣ ⎦ ⎡
( x + 7) - 9( 2 x - 3) a2 - b2 ⎤

The correct option is (C) = lim
× ⎢ using a - b = ⎥
x→ 2 x + 7 + 3 2x - 3 ⎢⎣ a + b ⎥⎦
ln ( 2 - cos 2 x ) ln {1 + (1 - cos 2 x )}
70. lim = lim ( x + 6) 2/3 + 2( x + 6)1/3 (3 x - 5)1/3 + 4(3 x - 5) 2/3
x→ 0 ln2 (sin 3 x + 1) x→ 0 ln2 (1 + sin 3 x )
( x + 6) - 8 (3 x - 5)
1 - cos 2 x 2x2 2

= lim = lim = ⎡
x→ 0 (sin 3 x ) 2 x→ 0 (3 x ) 2 9 a3 - b3 ⎤

 ⎢ using a - b = 2 ⎥

The correct option is (A) ⎢⎣ a + ab + b 2 ⎥⎦
1 - cos (cx 2 + bx + a) -17 ( x - 2)
71. lim = lim

x → 1/a (1 - xa ) 2 x→ z x + 7 + 3 2x - 3
1 - cos (cx 2 + bx + a) (cx 2 + bx + a) 2 ( x + 6) 2/3 + 2( x + 6)1/3 (3 x - 5)1/3 + 4(3 x - 5) 2/3
= lim
⋅ ×

x → 1/a (cx 2 + bx + a) 2 (1 - xa ) 2 -23 ( x - 2)
1 - cos z ( ay 2 + by + c) 2 82/3 + 2.81/3 + 4
-17 17 12 34
= lim
2
⋅ lim 2 2 =
⋅ = ⋅ =
z y →a y ( y - a)
z→ 0 9+3 1 -23 6 23 23
[putting cx2 + bx+ a = z and x = 1/y]

The correct option is (B)
1 a2 ( y - a )2 ( y - b )2 ( 2m + x )1/m - ( 2n + x )1/n
=
lim 74. lim
2 y→ a y 2 ( y - a )2 x→ 0 x
2
[If α, β are roots of ax + bx+ c = 0 then ax + bx+ c = a
2
( 2m + x )1/m - 2 ( 2n + x )1/n - 2
= lim
- lim
(x – α) (x – β)] x→ 0 x x→ 0 x
a 2 (a - b ) 2 a2 ⎡ a-2 b-2
=
= (a + b ) 2 - 4ab ⎤⎦ = lim m
- lim n
2a 2
2a 2 ⎣ a → 2 a - 2m b → 2 b - 2n

⎛ b 2 4c ⎞
2
 [Putting 2m+ x = am and 2n+ x = bn]
a b 2 - 4 ac
=
⎜ - ⎟ = 1 1
2a 2 ⎝ a 2 a⎠ 2a 2 =
-

The correct option is (A) m 2m - 1 n 2n - 1

The correct option is (C)
1 - cos x (cos q ) x - (sin q ) x - cos 2q
72. lim 75. lim
x→ 0 x x→ 4 x-4
1 - cos x 1 (cos q ) y + 4 - (sin q ) y + 4 - (cos 4 q - sin 4 q )
= lim
⋅ = lim

x→ 0 x 1 + cos x y→ 0 y
[Putting x – 4 = y and cos 2θ = cos4 θ – sin4 q]

= lim

2 sin x ( 2) ⋅ 1 ⎡ (cos q ) y - 1⎤ ⎡ (sin q ) y - 1⎤
2(x )
x→ 0 = lim cos 4 q ⎢
4
⎥ - sin q ⎢ ⎥
2 1 + cos x y→ 0 ⎢⎣ y ⎥⎦ ⎢⎣ y ⎥⎦

Now, we have, = cos4 q lncos θ – sin4 q lnsin θ

LHL = lim

-1 sin 2

x

1 ( )
The correct option is (A)
x→ 0 2 x 2 1 + cos x
-

Objective_Maths_JEE Main 2017_Ch 11.indd 36 01/01/2008 05:04:00


Limits  11.37

1/ x 1 ⎛ x - 1 + cos x ⎞ [ x ] + [2 x ] + [3 x ] + ... + [nx ]


⎛ x - 1 + cos x ⎞ ln ⎜
⎝ ⎟⎠ 79. Let f (x) =
76. lim ⎜ ⎟⎠ = lim e x x
1 + 2 + 3 + ... + n
x→ 0 ⎝ x x→ 0

Now, we have,
1 ⎛ 1 - cos x ⎞ x + 2 x + 3 x + ... + nx

Now,  lim ln ⎜1 - ⎟⎠ f (x) ≤
=x
x→ 0 x ⎝ x 1 + 2 + 3 + ... + n
⎛ cos x - 1⎞ ( x - 1) + ( 2 x - 1) + (3 x - 1) + ... + ( nx - 1)
ln ⎜1 + ⎟⎠ and,  f (x) >

cos x - 1 ⎝ x 1 + 2 + 3 + ... + n
= lim

x→ 0 x 2 cos x - 1 x Sn - n 2
=
=x-
x Sn n +1
-2 sin 2 ( x/2) ln (1 + z ) -1
 [∵ x - 1 ≤ [ x ] < x ∀x ∈ R]
= lim
2
⋅ lim =
x→ 0 4( x/2) z→ 0 z 2 Thus, we have,
⎡ cos x - 1 2
x– < f (x) ≤ x

 ⎢ Putting z = , we can see that
n +1
⎣ x
Now, we have,
cos x - 1 ⎛ cos x - 1⎞ -1 ⎤

 lim = lim x ⎜ ⎟⎠ = 0 ⋅ = 0 ⎥ 2
x→ 0 x n → 0 ⎝ x 2
2 ⎦ lim x - = x and lim x = x
n→∞ n +1 n→∞

Hence, the required limit is e–1/2.


Hence, by Sandwich Theorem, we have

The correct option is (B) lim f ( x ) = x
x 1/ y n→∞
⎡ e ⎤ ⎡ e ⎤
The correct option is (A)
77. lim ⎢ ⎥ = lim ⎢ ⎥
⎣( ) ⎦ y → 0 ⎢⎣ (1 + y ) ⎥⎦
( )
x
x →∞ ⎢ 1 + 1/ x ⎥ 1/ y
1 1 ⎛ 1 1 ⎞
-
80. lim n2 x1/n - x n + 1 = lim n2 ⋅ x n + 1 ⎜⎝ x n n + 1 - 1⎟⎠
n→∞ n→∞
1 ⎡ ⎤

( )
e
ln ⎢ ⎥
y ⎢⎣ (1 + y )1/ y ⎥⎦ 1 1

= lim e
y→ 0
= lim x n + 1 x n ( n + 1) - 1 n2
n→∞

Now, we have, 1
1 1
ln e - ln (1 + y ) x n ( n + 1) - 1 n2
1 ⎡ ⎤ n +1
e y
= lim x ⋅ ⋅
lim ln ⎢ ⎥ = lim n→∞ 1
n ( n + 1)
y→ 0 y ⎢⎣ (1 + y ) ⎥⎦
1/ y y→ 0 y n ( n + 1)

= 1 · lnx · 1 = lnx
y - ln (1 + y )

= lim 2 The correct option is (C)
y→ 0 y
81. We have,
⎛ y 2 y3 y 4 ⎞ 1/ x
y-⎜y- + - + ...⎟ ⎡ f ( x) ⎤
⎝ 2 3 4 ⎠ lim ⎢1 + x + = e3

= lim x→ 0 ⎣ x ⎥⎦
y→ 0 y2 ln [1 + x + g ( x )]
⎡ f ( x) ⎤
1 y y 1 2 ⇒ lim e x = e3 ⎢ Putting x = g ( x )⎥

= lim - + - ... = x→ 0 ⎣ ⎦
y→ 0 2 3 4 2 ln [1 + x + g ( x )]
⇒ lim
=3
Hence, the required limit is e1/2.
x→ 0 x

The correct option is (C) Since, the denominator approaches zero, the numerator
should also approach zero for a finite limit to exist.
⎡ a sin x ⎤ ⎡ b tan x ⎤
78. lim ⎢ + Thus, we have,
x→ 0 ⎣ x ⎥⎦ ⎢⎣ x ⎥⎦
lim g ( x ) = 0
x→ 0
⎡ sin x sin x
⎢ x → 0, x → 1 but x < 1
as
Now, using L’Hospital’s rule, the above equation reduces to

1 + g ′( x )
tan x tan x ⎤ lim =3

 while → 1 but > 1⎥ 1 + x + g( x)
x→ 0
x x ⎦ i.e., 1 + g′(0) = 3  ⇒  g′(0) = 2

= (a – 1) + b

Hence, we have,
=a+b–1
1/ x ln [1 + g ( x )]
⎡ f ( x) ⎤

The correct option is (B) lim ⎢1 + = lim e x
x→ 0 ⎣ x ⎥⎦ x→ 0

Objective_Maths_JEE Main 2017_Ch 11.indd 37 01/01/2008 05:04:06


11.38  Chapter 11

g ′( x ) 85. We have,
= lim e1 + g ( x ) = e g ′( 0 )

( )
2 2 2 cos 2 x
x→ 0 lim 11/cos x
+ 21/cos x
+ ... + n1/cos x
x → p /2
= e2

( ) ⎡ 1 ⎤
1/n

The correct option is (B) = lim 1y + 2 y + ... + n y
 ⎢ Putting cos 2 x = y ⎥
y→∞ ⎣ ⎦
x x 1/ y
82. Let y = x + =x+ ⎡⎛ 1 ⎞ y ⎛ 2 ⎞ y ⎛ n⎞ ⎤
y
x y = lim n ⎢⎜ ⎟ + ⎜ ⎟ + ... + ⎜ ⎟ ⎥

x+ y →∞ ⎢ ⎝ n ⎠ ⎝ n⎠ ⎝ n⎠ ⎥
x ⎣ ⎦
x+
... ∞ = n (0 + 0 + … + 1) = n

i.e.,  y2 – xy–
x =0
The correct option is (A)
n
x ± x2 + 4 x ⎛ 3⎞
i.e.,  y =
86. lim
n→∞
∑ cot -1 ⎜⎝ r 2 + 4 ⎟⎠
2 r =1

We can see y is a positive quantity for positive x, therefore n ⎡ ( r - 1/2) ( r + 1/2) + 1⎤
x+ x +4 x 2 = lim

n→∞
∑ cot -1 ⎢⎢ ( r + 1/2) - ( r - 1/2) ⎥⎥
y=
r =1 ⎣ ⎦
2 n
⎛ 1⎞ ⎛ 1⎞

Hence, the required limit is = lim

n→∞
∑ cot -1 ⎜⎝ r - 2 ⎟⎠ - cot -1 ⎜⎝ r + 2 ⎟⎠
x 2x r =1
= lim = lim

x →∞ y x →∞
x + x2 + 4 x ⎡ -1 ⎛ ab + 1⎞ ⎤

 ⎢∵cot ⎜ ⎟ = cot -1 a - cot -1 b ⎥
2 2 ⎣ ⎝ b-a⎠ ⎦
= lim
= =1
x →∞
1 + 1 + 4x -3/2 1 + 1 ⎡ ⎛ 1⎞ ⎛ 3⎞ ⎛ 3⎞ ⎛ 5⎞
= lim ⎢cot -1 ⎜ ⎟ - cot -1 ⎜ ⎟ + cot -1 ⎜ ⎟ - cot -1 ⎜ ⎟ +

The correct option is (A) n→∞ ⎣ ⎝ 2⎠ ⎝ 2⎠ ⎝ 2⎠ ⎝ 2⎠
83. We have, ⎛ 1⎞ ⎛ 1⎞ ⎤
cos x - (cos x )cos x
 ... + cot -1 ⎜ n - ⎟ - cot -1 ⎜ n + ⎟ ⎥
lim ⎝ 2⎠ ⎝ 2⎠ ⎦
x → 0 1 - cos x + ln (cos x )
⎡ ⎛ 1⎞ ⎛ 1⎞ ⎤
⎡ 1 - (cos x )cos x -1 ⎤ = lim ⎢cot -1 ⎜ ⎟ - cot -1 ⎜ n + ⎟ ⎥

n→∞ ⎣ ⎝ 2⎠ ⎝ 2⎠ ⎦
= lim cos x ⎢

x→ 0 ⎢⎣1 - cos x + ln (cos x ) ⎥⎦ ⎛ 1⎞
1 - (1 + t )t = cot–1 ⎜⎝ 2 ⎟⎠ – 0 = tan–1 2

= lim
 [Putting cos x – 1 = t]
t → 0 ln (1 + t ) - t

The correct option is (B)
t 3 (t - 1) t 4 (t - 1) (t - 2) ⎛ p⎞
t2 + + + ... 87. Here, sin x + cos x = 2 sin ⎜ x + ⎟
2! 3! ⎝ 4⎠
= lim 2 3 4
=2
t→ 0 t t t 5p ⎛ p⎞
- + - ... For x →
+ h, 2 sin ⎜ x + ⎟ → - 2 ,
2 3 4 4 ⎝ 4⎠
The correct option is (C)
But greater than – 2
84. We have,
⎡ ⎛ p⎞⎤
(tan x ) tan x - tan x ∴ ⎢ 2 sin ⎜ x + ⎟ ⎥ = –2
(1)
lim ⎣ ⎝ 4⎠⎦
x → p /4 ln (tan x ) - tan x + 1
5p ⎛ p⎞
tt - t ⎛ 0⎞ Also, for x →
- h, 2 sin ⎜ x + ⎟ → - 2 , but greater
= lim
[Putting tan x = t] 4 ⎝ 4⎠
⎜ ⎟
t → 1 ln t - t + 1 ⎝ 0 ⎠ than – 2
⎡ ⎛ p⎞⎤
t t (1 + ln t ) - 1 ⎛ 0 ⎞ ∴ ⎢ 2 sin ⎜ x + ⎟ ⎥ = –2
(2)
= lim
⎜⎝ ⎟⎠ ⎣ ⎝ 4⎠⎦
t→ 1 1 0
-1
From (1) and (2), we get
t
⎛ 1⎞
t t (1 + ln t ) 2 + t t ⎜ ⎟ lim [sin x + cos x ]
⎝t⎠ 1+1 x → 5p /4
= lim
= = –2
t→ 1 - 1 -1 = lim - 2 = –2

x→ 5p /4
t2

The correct option is (B)

The correct option is (A)

Objective_Maths_JEE Main 2017_Ch 11.indd 38 01/01/2008 05:04:12


Limits  11.39

88. lim lim ⎛1 a ⎞


m →∞ n→∞ = lim ⎜ + n ⎟

n→∞ ⎝ n! n! ⎠
⎧⎪1 + n 1n + 2n + n 2n + 3n + ... + n ( m - 1) n + m n ⎫⎪
⎨ ⎬ ⎡1 1 a ⎤
m2 = lim ⎢ +
+ n -1 ⎥  [using (1)]
⎪⎩ ⎪⎭ n→∞ ⎣ n! ( n - 1)! ( n - 1)! ⎦
= lim

m→∞ ⎡1 1 1 1 1 a⎤
= lim ⎢ +
+ + ... + + + 1 ⎥
⎧ n n n ⎫
n→∞ ⎣ n! ( n - 1)! ( n - 2)! 2! 1! 1! ⎦
⎪ ⎛ 1⎞ ⎛ 2⎞ ⎛ ( m - 1) ⎞
1 + 2 n ⎜ ⎟ + 1 + 3n ⎜ ⎟ + 1 + ... + m n ⎜ ⎟ + 1⎪ ⎡1 1 1 1 1 1⎤
⎪ ⎝ 2⎠ ⎝ 3⎠ ⎝ m ⎠ ⎪ = lim ⎢ +
+ + ... + + + ⎥
⎨ lim 2 ⎬ n→∞ ⎣ n! ( n - 1)! ( n - 2)! ( 2)! 1! 1⎦
n→∞ m
⎪ ⎪
⎪ ⎪ [Q  a1 = 1; given]

⎩ ⎭ ⎡ 1 1 1 ⎤
1 + 2 + 3 + ... + m = e ⎢as, e = 1 + + + + ... ∞ ⎥

= lim ⎣ 1 ! 2 ! 3 ! ⎦
m→∞ m2

The correct option is (B)
⎛ ⎛ 1⎞ n
⎛ 2⎞
n

⎜ ⎜ ⎟ → 0 as n → ∞ ; ⎜⎝ ⎟⎠ → 0 as n → ∞; ... ; ⎡ ⎛ 1⎞ ⎛ 1 ⎞⎤
n
⎝ ⎝ 2⎠ 3 92. lim n - n ⎢( n + 1) ⎜ n + ⎟ ... ⎜ n + n -1 ⎟ ⎥
2

n→∞ ⎣ ⎝ 2 ⎠ ⎝ 2 ⎠⎦
⎛ m - 1⎞
n ⎞
⎟⎠ → 0 as n → ∞⎟

 n
⎜⎝ ⎡ ⎛ 1⎞ ⎛ 1 ⎞⎤
m ⎠ ⎢ ( n + 1) ⎜⎝ n + ⎟ ... ⎜ n + n -1 ⎟⎠ ⎥
2⎠ ⎝ 2
m ( m + 1) 1⎛ 1⎞ 1 = lim ⎢

= lim
= lim ⎜1 + ⎟ = n→∞ ⎢ nn ⎥
m →∞ 2m 2 m →∞ 2 ⎝ m⎠ 2 ⎢ ⎥
⎣ ⎦
The correct option is (D) n n
⎛ 1⎞ ⎛ 1 ⎞
89. We know, n+ n n + n -1
⎛ n + 1⎞ ⎜ 2 ⎟ ⎜ 2 ⎟
1⎛ 1⎞ = lim ⎜
⎟ ⋅⎜ ⎟ ... ⎜ ⎟
n→∞ ⎝ n ⎠
n ⎜1 - n ⎟⎠ ⎜
n
⎟ ⎜ n ⎟
1 2⎝ 2 ⎝ ⎠ ⎝ ⎠
∑ r = i.e., sum of n terms of G.P.
r =12
1
1- ⎛ 1⎞ ⎛ 1⎞
n
⎛ 1 ⎞
n n
2 = lim ⎜1 + ⎟ ⋅ ⎜1 + ⎟ ... ⎜1 + n -1 ⎟ (1∞ form)

n→∞ ⎝ n⎠ ⎝ 2n ⎠ ⎝ 2 n⎠
which tends to one as n → ∞ but always remains less than
one. 2n 2n -1. n
n
⎛ 1⎞ ⎛ 1⎞ 2 ⎛ 1 ⎞ 2n - 1
⎡ n 1⎤ = lim ⎜1 + ⎟ ⋅ ⎜1 + ⎟ ... ⎜1 + n -1 ⎟

n→∞ ⎝ n⎠ ⎝ 2n ⎠ ⎝ 2 n⎠
Thus, ⎢ ∑ r ⎥ → 0 as n → ∞

⎢⎣ r = 1 2 ⎥⎦
= e1 · e1/2 · e1/4 … e1/2n–1

⎪⎧ ⎫⎪
an
⎡ n 1⎤ ⎛ 1⎞
∴ lim ⎢ ∑ r ⎥ = 0  … ⎨ using; lim ⎜1 + ⎟ = e a ⎬
n→∞ ⎝ n ⎠
n→∞ ⎢ ⎪⎩ ⎪⎭
⎣ r = 1 2 ⎥⎦ 1
The correct option is (A) 1-
1

90. Here, 0 < cos x < 1; if 0 – h < x < 0 + h = e(1 + 1/2 + 1/4 + …) = e 2 = e2

∴ [cos x] = 0
The correct option is (B)
Hence, lim | x | [cos x ]
x y - yx ⎛0 ⎞
x →0 93. lim ⎜⎝ form⎟⎠
xx - y y 
x→ y 0
= lim | x |0 = lim 1 = 1

x →0 x→0 yx y -1 - y x log y
= lim x
 (applying L-Hospital’s rule)
The correct option is (B) x → y x (1 + x log x ) - 0

a yx y -1 - y x log y
91. We have, an – 1 + 1 = n (1) = lim

n x→ y x x (1 + x log x )
⎛ a + 1⎞ ⎛ a2 + 1⎞ ⎛ an + 1⎞
∴ lim ⎜ 1 ⎟
⎜⎝ a ⎟⎠ ... ⎜⎝ a ⎟⎠ y ⋅ y y -1 - y y log y
n→∞ ⎝ a ⎠
1 2 n
=

y y (log y + 1)
⎛a ⎞ ⎛ a3 ⎞ ⎛ a4 ⎞ ⎛ an +1 ⎞ 1 1 - log y
= lim ⎜ 2 ⎟
⎜⎝ 3 ⎟⎠ ⎜⎝ ⎟⎠ ... ⎜⎝ ⎟ ⋅ =

n→∞ ⎝ 2 ⎠ 4 n + 1⎠ a1 ⋅ a2 ... an 1 + log y
an +1 1 + an
The correct option is (A)
= lim
= lim  [using (1)]
n→∞ ( n + 1)! n→∞ n!

Objective_Maths_JEE Main 2017_Ch 11.indd 39 01/01/2008 05:04:18


11.40  Chapter 11

n⎛ 3 1⎞ cos 2 (1 - cos 2 (1 - cos 2 (1 ... cos 2 q ))


r -r+ 97. lim
94. lim ∑ cot ⎜ -1
r⎟ q→ 0 ⎛ p ( q + 4 - 2⎞
n→∞
r =1 ⎜⎝ ⎟⎠ sin ⎜
2 q ⎟
n ⎝ ⎠
⎛ 2r ⎞
= lim

n→∞
∑ tan -1 ⎜⎝ 1 - r 2 + r 4 ⎟⎠ cos 2 (sin 2 (sin 2 ... (sin 2 q ))
r =1 = lim

q→ 0 ⎛ p ( q + 4 - 2⎞
-1 ⎛ ⎞
n
2r sin ⎜
= lim ∑ tan ⎜
2 2 ⎟ ⎝ q ⎟

n→∞
r =1 ⎝ 1 - (r - r) (r + r) ⎠
n ⎛ (r 2 + r) - (r 2 - r) ⎞ cos 2 (sin 2 (sin 2 ... (sin 2 q )) cos 2 0
= lim
= = 2
= lim

n→∞
∑ tan -1 ⎜ ⎟
⎝ 1 - (r 2 + r) (r 2 - r) ⎠
q→ 0 ⎛
q
⎞ sin
p
r =1
sin ⎜ p lim ⎟
( )
4
n ⎜⎝ q → 0 q q + 4 + 2 ⎟⎠
= lim

n→∞
∑ ⎡⎣tan -1( r 2 + r ) - tan -1( r 2 - r )⎤⎦
r =1
The correct option is (C)
–1 –1 –1 –1 –1
= lim [tan 2 – tan 0) + (tan 6 – tan 2) + (tan 12 –

n→∞ tan x - sin{tan -1(tan x )}
–1 –1 2 –1 2 98. lim f ( x ) = lim
tan 6) + … + {tan (n + n) – tan (n – n)}] x→
p
- x→
p
- tan x + cos 2 (tan x )
2 2
–1 2 –1
= lim {tan (n + n) – tan (0)}
tan x - sin x
n→∞
p
= lim
–1
= tan (∞) – tan (0) = –1 p
x→ - tan x + cos 2 (tan x )
2 2
[∵ tan–1(tan x) = x, where x < π/2]

n ⎛ 3 1⎞ p
r -r+
∴ lim ∑ cot -1 ⎜ r⎟ = sin x
n→∞
r =1 ⎜⎝ ⎟⎠ 2 1-
2 tan x

= lim
The correct option is (C) x→
p cos 2 (tan x )
2 1+
95. We know n ≤ [x] < n + 1  ⇒ [x] = n tan x
n sin x
Here, → n as x → 0 but less than n 1- 0
x
= =1
n tan x 1+ 0
Also, → n as x → 0 but more than n
x ⎡ cos 2 (tan x ) finite quantity ⎤

 ⎢∵ lim = = 0⎥
⎡ n sin x ⎤ p tan x ∞
Thus,  n – 1 ≤ ⎢ ⎥ < n as x → 0 ⎢ x→ - ⎥
⎣ x ⎦ ⎣ 2 ⎦
⎡ n sin x ⎤ sin x
⇒ ⎢ ⎥ =n–1 1+
⎣ x ⎦
Similarly,  lim f ( x ) = lim tan x
p
x→ +
p
x→ + cos 2 (tan x )
⎡ n tan x ⎤ 2 2 1+
Again,  n ≤ ⎢
⎥ < n + 1 as x → 0 tan x
⎣ x ⎦
[∵ tan–1 (tan x) = x – π, if x > π/2]

⎡ n tan x ⎤
⇒ ⎢
⎥ =n
⎣ x ⎦ 1+ 0

= =1
⎛ ⎡ n sin x ⎤ ⎡ n tan x ⎤⎞ 1+ 0
Thus, 
lim ⎜ ⎢ + ∴
lim f ( x ) = 1
x →∞ ⎝ ⎣ x ⎥⎦ ⎢⎣ x ⎥⎦⎟⎠ x→
p
2
= (n – 1) + (n) = (2n – 1) The correct option is (A)
The correct option is (C) 99. Put c = a1/4 and z = x1/4, we get the function whose limit is
96. We know, required, as
x 8
→ 1 , as x → 0, but less than 1 ⎧⎡ 2 ⎤
-1 ⎫
2 -1 1
sin x ⎪ ⎢⎛ c + z ⎞ 2cz ⎥
log c4 ⎪
- 3 -
4

x ⎨ ⎜ ⎟ 2 2

Also, → 1 , as x → 0 but less than 1 ⎢
⎪ ⎣⎝ c - z ⎠
2 2
z - cz + c z - c 3⎥

tan x ⎩ ⎦ ⎭
x2 ⎧⎡ c2 + z 2 -1 ⎫
8
Thus, → 1 as x → 0, but less than 1. ⎪ 2cz ⎤ ⎪
sin x tan x = ⎨⎢
- ⎥ - c ⎬
⎪⎩ ⎢⎣ c - z 2 2
( z - c) ( z + c ) ⎥⎦ ⎪⎭
⎡ x2 ⎤ x2

Hence, lim ⎢ ⎥ = 0 as 0 ≤ < 1 as x → 0 8
= (c – z – c) = z = x
8 2
x → 0 ⎢ sin x tan x ⎥ sin x tan x
⎣ ⎦
Hence, required limit as x → a = a2

The correct option is (A)

The correct option is (C)

Objective_Maths_JEE Main 2017_Ch 11.indd 40 01/01/2008 05:04:23


Limits  11.41

(log (1 + x ) - log 2) (3.4 x -1 - 3 x ) 1 1


100. lim + x + O ( x2 )
x →1 {(7 + x )1/3 - (1 + 3 x )1/2} sin p x 2 3 1
= lim =
x→0 1+ x 2
[log ( 2 + t ) - log 2] [3.4t - 3(t + 1)
= lim [O (x2) means terms containing x2, x3, x4, …]
t →0 {(8 + t )1/3 - ( 4 + 3t )1/2} sin p (t + 1)
The correct option is (C)
 [By putting x = 1 + t]
103. ∵ ax2 + bx+ c = 0 has roots α and β, therefore
⎛ t⎞ a b 1 1
log ⎜1 + ⎟ (3 ( 4t - 1) - 3t ) 2 + + c = 0 i.e., cx2 + bx+ a = 0 has roots and
⎝ 2⎠ x x a b
= - lim
t →0 ⎡
3t ⎞ ⎤ 1⎞ ⎛ 1⎞
1/3 1/2
⎛ t⎞ ⎛ ⎛ 2 b a⎞ ⎛
⎢ 2 ⎜1 + ⎟ - 2 ⎜1 + ⎟ ⎥ sin p t ⇒ c ⎜ x + + ⎟ = c⎜x - ⎟ ⎜x - ⎟
⎢⎣ ⎝ 8⎠ ⎝ 4 ⎠ ⎥⎦ ⎝ cx c ⎠ ⎝ a⎠ ⎝ b⎠

⎛ t⎞ ⎡1 - cos (cx 2 + bx + a) ⎤
log ⎜1 + ⎟ Now, lim ⎢ ⎥
2 ⎠ ⎡ 3 ( 4 - 1) ⎤
t
1 ⎝ 2 (1 - a x ) 2
= - lim ⋅ ⋅⎢ - 3⎥ x→
1 ⎢⎣ ⎥⎦
t →0 p t ⎢⎣ t ⎥⎦ a
1/ 2
2 ⎧ 2 ⎛ cx 2 + bx + a ⎞ ⎫
p

t
= lim ⎨
⎪ sin ⎜ ⎟⎪
⎝ 2 ⎠⎬
sin p t ⎡ t 3t 2 3 ⎤ 1
x→ ⎪ ⎪
⎢1 + 24 - 1 - 8 + terms containing t , t , etc.⎥
2
a ⎩ (1 - a x ) ⎭
⎣ ⎦
3 9 4 ⎛ cx 2 + bx + a ⎞
= ·(3 log 4 - 3) = log sin ⎜ ⎟
p p e = lim ⎝ 2 ⎠
1
x→
The correct option is (A) a 1 - ax
(1 - x ) (1 - x 2 ) ... (1 - x 2 n ) ⎛c ⎛ 1⎞ ⎛ 1⎞⎞
101. lim sin ⎜ ⎜ x - ⎟ ⎜ x - ⎟ ⎟
x →1 [(1 - x ) (1 - x 2 ) ... (1 - x n )]2 ⎝ 2 ⎝ a ⎠ ⎝ b ⎠⎠
= lim
(1 - x )(1 - x 2 )…(1 - x n )(1 - x n +1 )(1 - x n + 2 )…(1 - x 2 n ) 1 ⎛ 1⎞
lim
x→
a -a ⎜ x - ⎟
x →1 [(1 - x )(1 - x 2 )…(1 - x n )]2 ⎝ a⎠

(1 - x n +1 )(1 - x n + 2 )...(1 - x 2 n ) ⎛c ⎛ 1⎞ ⎛ 1⎞⎞ c⎛ 1⎞


= lim sin ⎜ ⎜ x - ⎟ ⎜ x - ⎟ ⎟ x- ⎟
x →1 [(1 - x )(1 - x 2 ) (1 - x 3 ) ... (1 - x n )] ⎝2 ⎝ a⎠ ⎝ b⎠⎠ 2 ⎜⎝ b⎠
= lim · lim
1 c⎛ 1⎞ ⎛ 1⎞ 1 -a
(1 - x n +1 )(1 - x n + 2 )...(1 - x 2 n ) x→
a ⎜⎝ x - ⎟⎠ ⎜ x - ⎟
x→
a
= lim 2 a ⎝ b⎠
x →1 (1 - x )(1 - x ) ... (1 - x )
(1 - x ) (1 - x ) ... (1 - x ) c ⎛ 1 1⎞
 × 1· ⎜ - ⎟
(1 - x ) (1 - x ) ... (1 - x ) 2 n 2 ⎝ a b⎠ c ⎛ 1 1⎞
= = -
1 1 1 ( 2n)! -a 2a ⎜⎝ a b ⎟⎠
= (n + 1) (n + 2) … 2n · · ... =
1 2 n ( n!) 2 The correct option is (A)
The correct option is (C)
104. lim f ( x ) = lim {- h} cot {- h}
p x → 0- h→0
102. Since maximum value of cos–1x =
2
k
kp = lim (1 - h) cot (1 - h) = cot 1
∑ cos a r =-1
is possible if and only if each h→0

r =1 2 tan 2 {h}
p lim f ( x ) = lim
cos–1 a r = ⇒  a r = 0 x→0 +
h→0 h2 - [h]2
2 
tan 2 h2
k = lim =1
∑ (a r )
r
∴ θ = =0 h→0h2
r =1
∴ lim f ( x ) does not exist,
x→0
(1 + x 2 )1/3 - (1 - 2 x )1/4
∴ lim 2 The correct option is (D)
x→q x+x
b
x a sin b x a + b - c ⎛ sin x ⎞
⎛ xc ⎞
(1 + x )2 1/3
- (1 - 2 x )1/4 105. lim = lim x
= lim x→0 sin ( x c ) x→0 ⎝⎜ x ⎠⎟ ⎜⎝ sin ( x c ) ⎟⎠
x→ 0 x + x2
The above limit is non-zero if a + b – c = 0
⎛ 1 2 4 ⎞ ⎛ x 2 ⎞
⎜⎝1 + x + O ( x )⎟⎠ - ⎜⎝1 - + O ( x )⎟⎠ The correct option is (D)
3 2
= lim
x→ 0 x (1 + x )

Objective_Maths_JEE Main 2017_Ch 11.indd 41 01/01/2008 05:04:30


11.42  Chapter 11

More than One Option Correct Type


106. We know that |cosθ | ≤ 1 for all θ. ⎛ a y + b y + c y - 3⎞
ln ⎜ 1 + ⎟
So, if  |cos n! px| < 1, ay + by + cy - 3 ⎝ 3 ⎠
= lim ⋅
lim lim (1 + cos 2 m n!p x ) = (1 + 0) = 1 y→0 y y y
a +b +c -3 y
m →∞ n→∞
and if  |cosn! px| = 1, 3
⎛ a - 1 b - 1 c - 1⎞
y y y
ln (1 + t )
lim lim (1 + cos 2 m n!p x ) = lim lim (1 + 12 m ) = lim ⎜ + + ⋅ lim
m →∞ n→∞ m →∞ n→∞ y→0 ⎝ y y y ⎟⎠ t → 0 t
= lim lim (1 + 1) = 2 ⎡ ay + by + cy - 3 ⎤
m →∞ n→∞
 ⎢ Putting = t⎥
The correct option is (A) and (B) ⎢⎣ 3 ⎥⎦
(1 + a3 ) + 8e1 x ⎛ ∞ ⎞ = ln a + ln b + ln c = ln(abc)
107. We have 2 = lim ⎜ form⎟⎠ (1) Hence, the required limit is eln (abc) = abc
x → 0 1 + (1 - b3 ) e1 x ⎝ ∞
The correct option is (C) and (D)
0 + 8e1 x ( -1 x 2 )
⇒ 2 = lim 109. Using the expansion, we have,
x → 0 0 + (1 - b3 ) e1 x ( -1 x2 )
⎛ x2 ⎞ ⎛ x 2 x3 ⎞
 [Using L’Hospital’s rule] ax ⎜1 + x + + …⎟ - b ⎜ x - + - …⎟
⎝ 2 ! ⎠ ⎝ 2 3 ⎠
⇒ 1 – b3 = 4  ⇒  b3 = –3  ⇒  b = (–3)1/3
⎛ x x 2 x3 ⎞
∴ From (1), + cx ⎜1 - + - + …,⎟
⎝ 1! 2! 3! ⎠
(1 + a3 ) + 8e1 x lim
2 = lim x→0 ⎛ x 3
x ⎞
5
1 + 4e1 x
x→0 x2 ⎜ x - + …
3 ⎝ 3! 5! ⎟⎠
⇒ 1 + a = 2 i.e., a = 1
Hence a = 1 and b = (–3)1/3 ⎛ b ⎞ ⎛ a b c⎞
x ( a - b + c ) - x 2 ⎜ a + - c⎟ + x 3 ⎜ - + ⎟ + …
The correct option is (B) and (C) ⎝ 2 ⎠ ⎝ 2 3 2⎠
⇒ lim
3x x→0 ⎛ x 3
x 5 ⎞
⎛ a1/x + b1/x + c1/x ⎞ x2 ⎜ x - + - …⎟
108. lim ⎜ ⎟ ⎝ 3! 5! ⎠
x→∞ ⎝ 3 ⎠
3/ y
= 2
⎛ ay + by + cy ⎞ ⎡ 1 ⎤ Now, above limit would exist if least power in numerator is
= lim ⎜ ⎟  ⎢ Putting x = y ⎥
y→0 ⎝ 3 ⎠ ⎣ ⎦ greater than or equal to least power in denominator
i.e., coefficient of x and x2 must be zero and coefficient of x3
3 ⎛ ay +by +cy ⎞ should be 2.
ln ⎜
⎝ 3 ⎠⎟
= lim e y b a b c
y→0 i.e., a – b + c = 0, a + – c = 0, - + = 2
2 2 3 2
Now, we have, On solving, we get a = 3, b = 12, c = 9
3 ⎛ ay + by + cy ⎞ The correct option is (A), (B) and (C)
lim ln ⎜ ⎟
y→0 y ⎝ 3 ⎠
3 ⎛ a y + b y + c y - 3⎞
= lim ln ⎜1 + ⎟
y→0 y ⎝ 3 ⎠

Passage Based Questions


2 cos 2 x - 2 cos x ⎛ 0⎞
sin 2 x + a sin x ⎛ 0⎞ Hence,  k = lim 2
 ⎜⎝ ⎟⎠
110. Let k = lim  ⎜⎝ ⎟⎠
x→0 3x 0
x→0 x3 0
-4 sin 2 x + 2 sin x ⎛ 0⎞
2 cos 2 x + a cos x = lim  ⎜⎝ ⎟⎠
= lim x→0 6x 0
x→0 3x 2
-8 cos 2 x + 2 cos x -8 + 2
 [Using L’Hospital’s rule] = lim = = –1
x→0 6 6
We require 2 cos2x + acosx = 0 for x = 0 as denominator is
zero. The correct option is (B)
\ a = –2

Objective_Maths_JEE Main 2017_Ch 11.indd 42 01/01/2008 05:04:34


Limits  11.43

p a-x Hence, by Sandwich Theorem, we have


111. lim a 2 - x 2 cot (0 ⋅ ∞ form)
x→a 2 a+ x 1•3•5 ... ( 2n - 1)
lim =0
n → +∞ 2 •4 •6 ... 2n
a2 - x 2 ⎛0 ⎞ The correct option is (C)
= lim  ⎜⎝ form⎟⎠
x→a p a-x 0 115. Since, 0 ≤ {rx} < 1 for r = 1, 2, 3, …, n
tan
2 a+ x n n n
⇒ 0≤ ∑ {rx} < ∑ (1) ⇒ 0 ≤ ∑ {rx} <n
-2 x r =1 r =1 r =1

2 a2 - x 2 Dividing throughout by n2, we have
= lim
x→a p a-x p 2a n
- sec 2 × ×
2 a + x 2 2( a + x ) a 2 - x 2 0
∑ {rx} 1
r =1
2 ≤ <
n n2 n
4a n
=
p ∑ {rx} 1
r =1
The correct option is (C) ⇒ lim 0 ≤ lim 2
< lim
n→∞ n→∞ n n→∞ n
log e | x | n
lim
lim | x | sin x = lim esin x log | x | = e cosec x
∑ {rx}
x→0
112. e

x→0 x→0 r =1
⇒ 0 ≤ lim <0
lim
1/ x
lim
sin 2 x n→∞ n2
cos ec x cot x x cos x
= e = e
x→0 x→0
{x} + {2 x} + ... + {nx}
∴ 0 ≤ lim <0
2
⎛ sin x ⎞ ⎛ x ⎞
n→∞n2
lim
x→0 ⎜

⎝ x ⎟⎠ ⎜⎝ cos x ⎟⎠ According to Sandwich Theorem or Squeeze Principle
= e = e–(1)2 · (0) = e0 = 1
The correct option is (B) {x} + {2 x} + ... + {nx}
lim =0
2 1 ( x -a )
n→∞ n2
113. lim (1 + ax + bx + c)
x →a The correct option is (B)
1 116. We have,
lim ⎡(1+ ax 2 + bx + c ) -1⎤⎦
( x -a ) ⎣ 12 · xx– 1 ≤ [12xx] < 12xx[Q  x – 1 ≤ [x] < x]
= e
x →a

( ax 2 + bx + c ) a ( x - a )( x - b )
22 · xx– 1 ≤ [22xx] < 22xx
lim lim
= e
x →a
( x -a )
= e
x →a
( x -a ) …

[∵   α, β are roots of ax2 + bx + c = 0]
n2 · xx– 1 ≤ [n2xx] < n2 · xx
= eα (α – β).
Adding the above inequations,
The correct option is (B)
x x Sn 2 - n S [ n2 x x ] x x Sn 2
114. We can see that ≤ ≤
3 3
n n n3
1· 3· 5 ... ( 2n - 1) 1 1 1 1
> · · … to n terms = n n ( n + 1) ( 2n + 1)
2 · 4 · 6 ... 2n 2 2 2 2 1 S [ n2 x x ]
⇒ xx 6 n3 – 2

n n3
1· 3· 5 ... ( 2n - 1) ⎛ 1⎞ ⎛ 1⎞ ⎛ 1⎞
and,  = ⎜1 - ⎟ ⎜1 - ⎟ ... ⎜1 - ⎟ n ( n + 1) ( 2n + 1)
2 · 4 · 6 ... 2n ⎝ 2⎠ ⎝ 4⎠ ⎝ 2n ⎠ ≤ x x
6 n3
⎛ 1⎞⎛ 1⎞ xx S [ n2 x x ] xx
< ⎜1 - ⎟ ⎜1 - ⎟ … to n terms Now, applying lim , we have ≤ ≤
⎝ 2n ⎠ ⎝ 2n ⎠ n→∞ 3 n3 3
n Hence, by Sandwich Theorem, we have
⎛ 1⎞
= ⎜1 - ⎟ S [ n2 x x ]
xx
⎝ 2n ⎠ lim =
n→∞ n3 3
Thus, we have,
n Now, the required unit
1 1· 3· 5 ... ( 2n - 1) ⎛ 1⎞
n < < ⎜1 - ⎟ ⎛
2 2 · 4 · 6 ... 2 n ⎝ 2 n⎠ S [ n2 x x ] ⎞ 1 1
lim ⎜ lim 3 ⎟ = 3 xlim xx =
Now, we have, n→0 ⎝ n→∞+
n ⎠ → 0 + 3
n
1 ⎛ 2n - 1⎞ The correct option is (B)
lim n = 0 and lim ⎜ ⎟ =0
n → +∞ 2 n → +∞ ⎝ 2n ⎠

Objective_Maths_JEE Main 2017_Ch 11.indd 43 01/01/2008 05:04:42


11.44  Chapter 11

Match the Column Type


⎡⎛ 1⎞
13 ⎤ - hh - 2hh ln h - hh - 1 - hh (ln h) 2
117.   (I)  lim ⎡ n2 - n3 + n⎤ = lim n ⎢⎜ -1 + ⎟ + 11 3 ⎥
3
= lim

n→∞ ⎣ ⎥⎦ n→∞ ⎢ ⎝ n ⎠ ⎥⎦ h →1 -1 / h2

⎛1 ⎞ -1 - 1
⎜⎝ - 1⎟⎠ + 1 = =2
n -1
= lim n ⋅
n→∞ ⎛1 ⎞
23
⎛1 ⎞
13 The correct option is (A)
⎜⎝ - 1⎟ + 1 - ⎜⎝ - 1⎟ Required limit =
(II) 
n ⎠ n ⎠
n
5r + 2 r n ⎧⎪⎛ 1 ⎞ r ⎛ 1 ⎞ r ⎫⎪


⎢ Using a + b = 2
a3 + b3 ⎤

lim
n→∞
∑ 10 r
= lim
n→∞
∑ ⎨⎜⎝ 2 ⎟⎠ + ⎜⎝ 5 ⎟⎠ ⎬
a - ab + b 2 ⎥⎦
r =1 ⎪
r = 1⎩ ⎪⎭
⎢⎣
1 1 1 ⎧ ⎛ 1⎞
n
⎛ 1⎞ ⎫
n

= lim = = ⎪⎪ 1 1 - ⎜⎝ ⎟⎠ 1
1- ⎜ ⎟ ⎪
⎝ 5⎠ ⎪
n→∞ ⎛ 1 ⎞
23
⎛1 ⎞
13
1+1+1 3 = lim ⎨ ⋅
2
+ ⎬
⎜⎝ - 1⎟⎠ + 1 - ⎜⎝ - 1⎟⎠ n → ∞⎪2 1 5 1 ⎪
n n 1- 1-
⎪⎩ 2 5 ⎪ ⎭
The correct option is (C) 1 5
3
= 1+ =
x2 - 2 3 x + 1 4 4
 (II)  lim The correct option is (D)
x →1 ( x - 1) 2
⎡ x +1 p ⎤
y2 - 2 y + 1 (III) lim x ⎢ tan -1 -
x + 2 4 ⎥⎦
3
= lim [Putting x = y; as x→ 1, y→ 1] x →∞ ⎣
y →1 ( y 3 - 1) 2
⎡ x +1 ⎤
( y - 1) 2 1 1 = lim x ⎢ tan -1 - tan -1 1⎥
= lim = lim = x →∞ ⎣ x+2 ⎦
- 1) 2 ( y 2 + y + 1) 2
y →1 ( y y →1 ( y 2 + y + 1) 2 9
The correct option is (A) ⎛ x +1 ⎞
-1
-1 ⎜ ⎟
(III)  (n – 2)th factor of the series is = lim x tan ⎜ x + 2 ⎟
x →∞ x +1
n -1 n + n +1 2 ⎜1+ ⎟
tn = ⋅ ⎝ x + 2⎠
n + 1 n2 - n + 1
⎛ -1 ⎞
= lim x tan -1 ⎜
Therefore, required limit = lim t3t 4t5 ... t n - 2t n - 1t n
n→∞
x →∞ ⎝ 2 x + 3 ⎟⎠
⎡⎛ 2 3 4 n - 3 n - 2 n - 1⎞ ⎛ 1 ⎞
tan -1 ⎜
= lim ⎢⎜ ⋅ ⋅ ...
n→∞ ⎝ 4 5 6 n -1

n

n + 1⎟⎠ ⎝ 2 x + 3 ⎟⎠ 1
⎣ = – lim ⋅
x →∞ ⎛ 1 ⎞ ⎛ 3⎞
⎛ 13 ⎞ 21 31 n + n + 1⎤ 2+ ⎟
⎝⎜ 2 x + 3 ⎠⎟ ⎝⎜
2
⋅⎜ ⎟ ⋅ ⋅ … 2 ⎥ x⎠
⎝ 7 ⎠ 13 21 n - n + 1⎥⎦
1 1
= –1 × =–
2 ⋅ 3 n2 + n + 1 6 2 2
= lim ⋅ =
n → ∞ n( n + 1) 7 7 The correct option is (B)
The correct option is (B) nk sin 2 ( n!) nk sin 2 ( n!)
n ⎛ x ⎞ (IV) lim = lim
⎛ x⎞ lim n ⎜ cos -1⎟
⎝ n ⎠
n→∞ n+2 n→∞ ⎛ 2⎞
 (IV)  lim ⎜ cos ⎟ = e
n →∞
n ⎜1 + ⎟
n→∞ ⎝ n⎠ 2
⎝ n⎠
⎛ ⎛ x ⎞⎞
⎜ sin ⎜⎝ 2 n ⎟⎠ ⎟ x 2
-2 lim ⎜ ⎟ ⋅ 2 ⋅n sin 2 ( n!)
⎛ x⎞ n→∞
⎜ x ⎟ 4n = lim
- n⋅2 sin ⎜ ⎟ n → ∞ 1- k ⎛ 2⎞
2
lim ⎜⎝ 2 n ⎟⎠
⎝ 2n⎠ n ⎜1 + ⎟
= e = e
n →∞

x2 ⎝ n⎠
-2 × lim
= e = e0 = 1
n→∞
4n
a finite quantity
The correct option is (D) =

118.   (I)  Let sin x = h, then as x → π/2, h → 1 [∵ sin2 (n !) always lies between 0 and 1. Also,
∴  given limit since 1 – k > 0,
h - hh 1 - hh - hh ln h \  n1 – k → ∞ as n → ∞]
= lim = lim
h → 1 1 - h + ln h h →1 -1 + 1 / h =0
 [Using L’ Hospital rule] The correct option is (C)

Objective_Maths_JEE Main 2017_Ch 11.indd 44 01/01/2008 05:04:48


Limits  11.45

Assertion-Reasoning Type
12 + 22 + 32 + ... + r 2 e y - 1 ln (1 + x ) - x
119. tr = = lim - e ⋅ ⋅
13 + 23 + 33 + ... + r 3 x→0 y x2
2
r ( r + 1)( 2r + 1) ⎛ 2 ⎞ 2⎛1 1 ⎞ ⎡ ln (1 + x ) - x ⎤
= ⋅⎜ = ⎜ + ⎟  ⎢ Putting = y⎥
6 ⎝ r ( r + 1) ⎟⎠ 3 ⎝ r r + 1⎠ ⎣ x ⎦
Now, we have,
2 ⎡ ⎛ 1 ⎞ ⎛ 1 1⎞ ⎛ 1 1 ⎞ ⎛1 1 ⎞⎤
∴  Sn = ⎢ - ⎜1+ ⎟ + ⎜ + ⎟ - ⎜ + ⎟ + … ± ⎜ + ln (1 + x ) - x ⎛ 0 ⎞
3 ⎣ ⎝ 2 ⎠ ⎝ 2 3⎠ ⎝ 3 4 ⎠ ⎝ n n + 1⎟⎠ ⎥⎦ lim ⎜⎝ ⎟⎠
x→0 x 0
2⎛ 1 ⎞ 1
= ⎜ -1 ± ⎟ -1
3⎝ n + 1⎠ 1+ x
= lim =0
2 x→0 1
∴ lim Sn = – ln (1 + x ) - x ⎛ 0 ⎞
n→∞ 3 Also, lim ⎜⎝ ⎟⎠
The correct option is (D) x→0 x2 0
120. We have, 1
-1
1+ x -1 1
x1 = 3, xn + 1 = 2 + xn = lim = lim =–
x→0 2x x → 0 2 (1 + x ) 2
x2 = 2 + x1 = 2+3 = 5
Hence, the required limit is
x3 = 2 + x2 = 2+ 5 ey -1 ln (1 + x ) - x
= –e lim ⋅ lim
∴ x1 > x2 > x3 y→0 y x→0 x2
It can be easily shown by mathematical induction that the
1 e
= –e · 1 · – =
sequence x1, x2, …xn, … is a monotonically decreasing 2 2
sequence bounded below by 2. So, it is convergent. The correct option is (A)
Let lim xn = x. Then, 1
n→∞
123. (
lim [ f ( x )] + x 2
x→0
) { f ( x )}

xn + 1 = 2 + xn [ f ( x )] + x 2 - 1
⎡⎛ 1 ⎞⎤ { f ( x )}
⇒ lim xn + 1 =
n→∞
2 + lim xn
n→∞
x → 0 ⎢⎝
(
= lim ⎢⎜ 1 + [ f ( x )] + x 2 - 1 [ f ( x )] + x ) 2
-1 ⎥

⎠ ⎥⎦
⇒ x = 2 + x ⎣
Now, we have,
⇒ x2 – x – 2 = 0
⇒ (x – 2)(x + 1) = 0 ⎡ tan x ⎤
lim ⎢ + x2 - 1 = 1 + 0 – 1 = 0
x→0 ⎣ x ⎥ ⎦
⇒ x = 2 (∵ xn > 0 ∀ n,  ∴ x > 0)
The correct option is (A) [ f ( x )] + x 2 - 1
and,  lim
x→0 { f ( x )}
1⎛
n -1 ⎞
1/ n 2/ n
121. lim ⎜1 + e + e + ... + e n

n→∞ n ⎡ ⎤
⎝ ⎠ x2 2x4
⎢1 + + + ...⎥ + x 2 - 1
( )
n
1/ n 3 15
1 1- e 1- e = lim ⎣ ⎦
= lim ⋅ = lim ⎧⎪ 2 ⎫⎪
n→∞ n 1 - e1/ n n→∞ ⎡ 1 1 1 ⎤ x→0 x 2x4
n ⎢1 - 1 - - ⋅ 2 ...⎥ ⎨1 + + + ...⎬
⎣ n 2! n ⎦ ⎪⎩ 3 15 ⎪⎭
1- e 1- e ⎡ ⎤
= lim = =e–1 x3 2 x5
n→∞ 1 1 -1  ⎢ tan x = x + + + ...⎥
-1 - ⋅ ... ⎢⎣ 3 15 ⎥⎦
2! n
The correct option is (A) 1 + x2 - 1 1
= lim = lim
ln (1 + x ) x→0 x2
2x 5 x→0 1 2x
+ + ... + + ...
e - (1 + x )1/x e-e x
3 15 3 15
122. lim = lim =3
x→0 x x→0 x
ln (1 + x ) - x ∴ Required limit = e3
e x -1 The correct option is (D)
= lim - e ⋅
x→0 x

Objective_Maths_JEE Main 2017_Ch 11.indd 45 01/01/2008 05:04:54


11.46  Chapter 11

124. lim
1 x 1 x 1 x
tan + 2 tan n + ... + n tan n cot q tan -1( m tan q ) - m
= lim +m
n→∞ 2 2 2 2 2 2 q →0 sin 2 (q/2)
⎛ 1 x⎞ 1 x tan -1( m tan q ) - m tan q
= lim - cot x + ⎜ cot x + tan ⎟ + 2 tan 2 + = lim +m
n→∞ ⎝ 2 2⎠ 2 2 q →0 tan q sin 2 (q/2)
1 x
... + n tan n tan -1( m tan q ) - m tan q
2 2 = lim +m
⎛1 x 1 x⎞ 1 x q →0 q3 / 4
= lim - cot x + ⎜ cot + 2 tan 2 ⎟ + ... + n tan n x - tan x
n→∞ ⎝2 2 2 2 ⎠ 2 2 = lim +m
x→0 x3
⎛ 1 x 1 x⎞ 1 x 4m 3
= lim - cot x + ⎜ 2 cot 2 + 3 tan 3 ⎟ + ... + n tan n
n→∞ ⎝2 2 2 2 ⎠ 2 2 [Putting m tan θ = tan x; as q→ 0, x → 0]
⎡ 1 q 1 q⎤ x - tan x
 ⎢ repeatedly using cot q + 2 tan 2 = 2 cot 2 ⎥ = m + 4m3 xlim
⎣ ⎦ →0 x3

1 x Now, we have,
= lim - cot x + n cot n
n→∞ 2 2 x - tan x ⎛ 0 ⎞ 1 - sec 2 x ⎛ 0 ⎞
lim ⎜ ⎟ = lim ⎜⎝ ⎟⎠
1 ⎛ x/2 ⎞ n
x→0 x 3 ⎝ 0 ⎠ x → 0 3x 2 0
= lim - cot x +
n→∞ x ⎜⎝ tan x/2n ⎟⎠ -2 sec 2 x tan x -1
= lim =
1 q 1 x→0 6x 3
= –cot x + lim = –cot x +
x q → 0 tan q x Hence, the required limit is m – (4/3) m3
The correct option is (A) The correct option is (C)
cot q tan -1( m tan q ) - m cos 2 (q/2)
125. lim
q →0 sin 2 (q/2)
cot q tan -1( m tan q ) - m [1 - sin 2 (q/2)]
= lim
q →0 sin 2 (q/2)

Previous Year’s Questions

126. Key Idea: Limit of a function exists only, if


127. Key Idea : lim (1 + λ x )1/ x = e λ
LHL = RHL. x →∞
x
1 - cos 2 x 1 - 1 + 2 sin x 2 ⎛ x 2 + 5 x + 3⎞

Now, lim = lim
Now, the limit lim ⎜ 2 ⎟
x→0 2x x→0 2x x →∞ ⎝ x + x + 2 ⎠

2 sin x sin x 4x + 1 ⎞
x
 = lim = lim ⎛
x→0 2x x→0 x = lim ⎜1 + 2 ⎟
x →∞ ⎝ x + x + 2⎠
sin x ( 4 x +1) x
Let f ( x ) =

x ⎡ 1
( 4 x +1) ⎤ x 2 + x +2
⎢ ⎛ 4x + 1 ⎞ x + x+2 ⎥
2

sin( 0 - h) = lim ⎜1 + 2 ⎟
LHL = lim x →∞ ⎢⎝ x + x + 2⎠ ⎥
0-h
h→ 0 ⎢⎣ ⎥⎦
Now,
sin h ⎛ 1⎞
4+
= lim = -1 ⎝⎜ x ⎠⎟
h→ 0 - h lim
x →∞ 1 2
1+ - 2
sin( 0 + h) = e x x
RHL = lim
h→ 0 0+h = e 4
and
sin h The correct option is (A)
= lim =1
h→ 0 h x
⎛ x - 3⎞
∵ LHL ≠ RHL 128. The limit lim ⎜ ⎟
x →∞ ⎝ x + 2 ⎠
sin x
\ lim does not exist. ⎡ 5 ⎤
x
xx→0 = lim ⎢1 -

The correct option is (D) x →∞ ⎣ x + 2 ⎥⎦

Objective_Maths_JEE Main 2017_Ch 11.indd 46 01/01/2008 05:05:00


Limits  11.47

⎛ -5 x ⎞
131. By applying L’Hopital Rule, the given limit equals
⎡ ⎛ 5 ⎞ ⎤ ⎜⎝ x + 2 ⎟⎠
1/ ⎜ ⎟ 1 1
⎢⎛ ⎛ -5 ⎞ ⎞ ⎝ x + 2 ⎠ ⎥ +
= lim ⎢⎜1 + ⎜ ⎟ ⎟ ⎥ 3 + x 3 - x =2.
x →∞ ⎝ ⎝ x + 2 ⎠⎠ lim
⎢ ⎥ x→0 1 3
⎣ ⎦
⎛ 5 ⎞ The correct option is (C)
lim ⎜ - 1+ 2 / x ⎠⎟
x →∞ ⎝
= e 132. Applying L. Hospital’s Rule
= e -5 f ( a) g ′( a) - g ( a) f ′( a)
lim =4
Alternative Method: x→ 2a g ′( a) - f ′ ( a)
x
⎛ x - 3⎞ k ( g ′( a) - ff ′( a))
lim ⎜ ⎟ =4
x →∞ ⎝ x + 2 ⎠ ( g ′( a) - f ′( a))
x k = 4.
⎛ 3⎞
⎜⎝1 - x ⎟⎠ The correct option is (A)
= lim x
x →∞ ⎛ 2⎞ 2x
⎛ a b⎞
⎜⎝1 + x ⎟⎠ 133. The limit lim ⎜1 + + 2 ⎟ =
x →∞ ⎝ x x ⎠
e -3 ⎛ 1 ⎞
= 2
= e -5 ⎜ a b ⎟ × 2 x × ⎛⎜ + 2 ⎞⎟
a b
e ⎛ a b ⎞ ⎜⎝ x + x 2 ⎟⎠ ⎝x x ⎠
lim ⎜1 + + 2 ⎟ = e2a

The correct option is (C) x →∞ ⎝ x x ⎠

129. The limit lim


xf ( 2) - 2 f ( x ) ⇒ a = 1, b ∈ R
x→2 x-2
The correct option is (B)
xf ( 2) - 2 f ( x ) + 2 f ( 2) - 2 f ( x )
= lim 134. Let L =
x→2 x-2
⎛ ( x - a )( x - b ) ⎞
f ( 2)( x - 2) - 2{ f ( x ) - f ( 2)} 2 sin 2 ⎜ a ⎟⎠
= lim 1 - cos a( x - a )( x - b ) ⎝ 2
x→2 x-2
lim = lim
x →a ( x - a )2 x →a ( x - a )2
f ( x ) - f ( 2)}
= f ( 2) - 2 lim
x→2 x-2 ⎛ ( x - a )( x - b ) ⎞
sin 2 ⎜ a ⎟⎠
= f ( 2) - 2 f ′( 2) = 4 - 2 × 4 = -4 2 ⎝ 2 a2 ( x - a )2 ( x - b )2
= lim × ×
x →a ( x - a )2 a2 ( x - a )2 ( x - b )2 4
Alternative Solution:
4
⎧ xf ( 2) - 2 f ( x ) ⎫
lim ⎨ ⎬ a 2 (a - b ) 2
x→2 ⎩ x-2 ⎭ Then, the limit L =
.
2
= lim{ f ( 2) - 2 f ′( x )} ( by L′ Hopital’s Rule)
x→2
The correct option is (A)
= f (2) − 2f  ′(2) 135. f (x) is a positive increasing function
= 4 − 2 × 4 = − 4 ⇒ 0 < f (x) < f (2x) < f (3x)
The correct option is (C)
f ( 2 x ) f (3 x )
⇒ 0 < 1 < <
130. The limit, by applying L’Hopital rule, f ( x) f ( x)
⎛ p x⎞ f (2 x) f (3 x )
tan ⎜ - ⎟ (1 - sin x ) ⇒ lim 1 ≤ lim ≤ lim
⎝ 4 2⎠ x →∞ x →∞ f ( x ) x →∞ f ( x )
lim
x →p / 2 ⎛ p x⎞
4 ⎜ - ⎟ (p - 2 x ) 2 By sandwich theorem.
⎝ 4 2⎠
f (2 x)
( - cos x ) ⇒ lim =1
= lim x →∞ f ( x )
x →p / 2 8( -2)(p - 2 x )

The correct option is (D)
- sin x
= lim
x →p / 2 16( -2) 2 sin 2 ( x - 2)
136. lim
1 x→2 x-2
= .
32 2 | sin( x - 2) |

The correct option is (C) lim
x→2 x-2

Objective_Maths_JEE Main 2017_Ch 11.indd 47 01/01/2008 05:05:04


11.48  Chapter 11

1
R.H.L. = 2 , L.H.L. = - 2 ⇒ a = , b = -2
2

Limit does not exist. x4

The correct option is (D) ⇒ f ( x ) = - 2 x3 + 2 x 2
2
⇒ f ( x ) = 8 - 16 + 8 = 0.
(1 - cos 2 x )
137. lim (3 + cos x )
The correct option is (A)
x→0 x(tan 4 x )
2 139. The value of the limit
⎛ sin x ⎞ 1 ⎛ 4 x ⎞
= lim ⎜
x →0 ⎝ x ⎠
⎟ ⋅ 4 ⎜ tan 4 x ⎟ ( 3 + cos x ) 2 sin 2 x × (3 + cos x ) 2 × 4
⎝ ⎠ lim = =2.
1 x→0 ⎛ tan 4 x ⎞ 4
= 2 × 1 × × 1 × ( 3 + 1) = 2. x×⎜ × 4 x
4 ⎝ rx ⎟⎠

The correct option is (C)
The correct option is (B)

⎛ x 2 + f ( x) ⎞
2
1 ⎛ tan x ⎞
⎟ =3.
138. Given that lim ⎜ lim ⎜ ⎟
2⎝
x → 0+ x ⎠
x→0 ⎝ x2 ⎠ 140. We have p = e = e

Since limit exits, the expression x 2 + f ( x ) = ax 4 + bx 3 + 3 x 2 1


\ log p =
⇒ f ( x ) = ax 4 + bx 3 + 2 x 2 2

The correct option is (D)
⇒ f ′( x ) = 4 ax 3 + 3bx 2 + 4 x

Also, f ′( x ) = 0 at x = 1, 2

Objective_Maths_JEE Main 2017_Ch 11.indd 48 01/01/2008 05:05:07


Continuity and Differentiability 12.1

CHaPtER
Continuity and
12 Differentiability

Chapter Highlights
Continuity, Discontinuity of a function, Geometrical meaning of continuity

conTinUiTY 1. f (x) is continuous from right at x = a, i.e.,


lim f (a + h) = f (a)
In this chapter, the concept of a continuous function is h→0

defined in terms of limits. Most of the results in calculus 2. f (x) is continuous from left at x = b, i.e.,
are not true unless we are dealing with functions that are lim f (b – h) = f (b)
h→0
continuous. We may intuitively think of continuous func- 3. f (x) is continuous at each point of the open interval
tions as those functions whose graphs we can draw without (a, b).
lifting the pencil. A formal definition of continuity follows:
continuity at end points of an interval
continuity of a function at a Point
For continuity of f (x) at the end points of an interval [a, b],
A function f (x) is said to be continuous at an interior point we must have
x = a of its domain if lim f (x) = f (a). In other words a
x→a
• lim f ( a + h) = f (a) at x = a
function f (x) is said to be continuous at a point x = a pro- h→0
vided lefthand limit righthand limit and value of function • lim f (b - h) = f (b) at x = b
are equal: h→0

I M P O R TA N T P O I N T S Trick(s) for Problem solving


A function f (x) is continuous at a point x = a if For continuity of f (x) at the end points of an interval [a, b],
lim f (a – h) = lim f (a + h) = f (a) we must have
h→0 h→0
(i) lim f (a + h) = f (a) at x = a.
h→0

(ii) lim f (b – h) = f (b) at x = b.


continuity of a function on An interval h→0

Continuity on an Open Interval


A function f (x) is said to be continuous on an open interval
(a, b) if it is continuous at each point of (a, b). DisconTinUiTY of A fUncTion
A function f (x), which is not continuous at a point x = a, is
Continuity on a Closed Interval
said to be discontinuous at that point.
A function f (x) is said to be continuous on a closed interval
[a, b] if

Objective_Maths_JEE Main 2017_Ch 12.indd 1 01/01/2008 05:24:38


12.2  Chapter 12

10. Logarithmic functions, Exponential functions,


Trick(s) for Problem Solving Trigonometric functions, Inverse circular functions
The discontinuity may arise due to any of the following
and Absolute value functions are continuous in their
situations: domain.
(i)  lim f (a – h) ≠ lim f (a + h), i.e., LHL and RHL exist,
h→0 h→0 Types of Discontinuity
but are not equal.
(ii) lim f (a – h) = lim f (a + h) ≠ f (a), i.e., LHL and Removable Discontinuity
h→0 h→0
If lim f (a – h) and lim f (a + h) exist and are equal, but
RHL exist and are equal, but are different from f (a). h→0 h→0
(iii) f (a) is not defined. are not equal to f (a), then the function f (x) is said to have
(iv) Atleast one of the limits lim f (a – h) or lim f (a + a removable discontinuity at x = a. However, by suitably
h→0 h→0 defining the function at x = a, f (x) can be made continuous
h) does not exist or atleast one of these limits is ∞ at x = a.
or – ∞.
Discontinuity of the First Kind
If lim f (a – h) and lim f (a + h) exist but are not equal,
h→0 h→0
Some Useful Results on Continuous then the function f (x) is said to have a discontinuity of the
Functions first kind at x = a.
We also say that f (x) has jump discountinuity at x = a.
1. If f and g are continuous at x = a, then We define lim f ( x ) - lim f ( x ) = jump of the func-
(A) f + g is continuous at x = a. x → a- x → a+
(B) f – g is continuous at x = a. tion at x = a.
(C) fg is continuous at x = a. If lim f (a – h) exists but is not equal to f (a), then
h→0
(D) f /g is continuous at x = a, provided g (a) ≠ 0. the function f (x) is said to have a discontinuity of the first
(e)  kf is continuous at x = a, where k is any real kind from the left at x = a.
constant. Similarly, if lim f (a + h) exists but is not equal
(f) [ f (x)]m/n is continuous at x = a, provided [ f (x)]m/n h→0
is defined on an interval containing a, and m and n to f (a), then the function f (x) is said to have a discontinuity
are integers. of the first kind from the right at x = a.
2. If f is continuous at a and g is continuous at f (a), then Discontinuity of the Second Kind
gof is continuous at a.
3. If f is continuous at x = a and g is discontinuous at If atleast one of the limits lim f (a – h) or lim f (a + h)
h→0 h→0
x = a, then f + g and f – g are discontinuous at x = a, does not exist or atleast one of these limits is ∞ or – ∞,
whereas fg may be continuous at x = a. then the function f (x) is said to have a discontinuity of the
4. If f is continuous at x = a and f (a) ≠ 0, then there exits second kind at x = a. We also say that f (x) has an infinite
an open interval (a – d, a + d  ) such that ∀ x ∈ (a – d, discontinuity at x = a.
a + d  ), f (x) has the same sign as f (a). If lim f (a – h) does not exist or is equal to ∞ or –
h→0
5. If f is a continuous function defined on [a, b] such that ∞, then the function f (x) is said to have a discontinuity of
f (a) .  f (b) < 0, then there exists atleast one solution of the second kind from the left at x = a. Discontinuity of the
the equation f (x) = 0 in the open interval (a, b). second kind from the right is similarly defined.
6. If f is a continuous function defined on [a, b] and k
is any real number between f (a) and f (b), then there Oscillating Discontinuity
exists atleast one solution of the equation f (x) = k in f (x) has oscillating discontinuity at x = a if lim and lim
the open interval (a, b). h→0 h→0

7. If a function f is continuous on a closed interval [a, b], lie in a certain range but do not approach to a definite value
then it is bounded on [a, b] i.e., there exists real num- at x = a.
bers k and K such that
GEOMETRICAL MEANING OF CONTINUITY
k ≤ f (x) ≤ K for all x ∈ [a, b]
8. Every polynomial is continuous at every point of the 1. A function f (x) will be continuous at a point x = a,
real line. if there is no break or cut or gap in the graph of the
9. Every rational function is continuous at every point function y = f (x) at the point [a, f (a)]. Otherwise, it is
where its denominator is different from zero. discontinuous at that point.

Objective_Maths_JEE Main 2017_Ch 12.indd 2 01/01/2008 05:24:41


Continuity and Differentiability  12.3

2. A function f (x) will be continuous on the closed


­interval [a, b] if the graph of the function y = f (x) is and lim f (x) = lim f (0 – h)
an unbroken line (curved or straight) from the point x → 0- h→0

[a, f (a)] to the point [b, f (b)]. e( - h) - h - 1


= lim
Y h→0 ( - h) - h

)
f(x
e -1 - h - 1 e -1 - 1

y=
= lim =
h→0 ( - 1 - h) -1
[a, f(a)]
1
=1–
f(a + h) e
f(a)
f(a – h)

Since lim f (x) ≠ lim f (x), therefore, f (x) is not


x → 0+ x → 0-
X continuous at x = 0.
O x=a–h x=a x=a+h
1
2. If f (x) = x sin , x ≠ 0, then the value of the function
Fig. 12.1  f (x) has a continuous graph at x = a x
at x = 0, so that the function is continuous at x = 0, is
Y
(A) 0 (B) – 1
y = f(x)
(C) 1 (D) indeterminate
[a, f(a)]
Solution: (A)
For f (x) to be continuous at x = 0, we must have
f(a + h)

f (0) = lim f (x)


f(a)
f(a – h)

x→0

1
X ⇒ f (0) = lim x sin = 0 × a finite quantity = 0
O x=a–h x=a x=a+h x→0 x
(h > 0)
Hence f (0) = 0.
Fig. 12.2  f (x) has a discontinuous graph at x = a ⎧ ⎛ 1 1⎞
2-⎜ + ⎟
⎪ ⎝ | x| x ⎠
3. The function f (x) = ⎨( x + 1) , x ≠ 0 is
Solved Examples ⎪0 ,x=0

(A)  discontinuous at only one point
⎧ e[ x ] + x - 1 (B)  discontinuous exactly at two points
⎪ ,x≠0 (C)  continuous everywhere
1. If f (x) = ⎨ [ x ] + x then
⎪ (D)  None of these
⎩ 1 ,x=0
Solution: (A)
(A)  lim f (x) = –1 The only doubtful point is x = 0.
x→0 +

1 ⎛ 1 1⎞
(B)  lim f (x) = –1 2-⎜ - ⎟
⎝ h h⎠
x → 0- e L.H.L. = lim f (0 – h) = lim ( - h + 1)
h→0 h→0
(C) f (x) is continuous at x = 0
2
(D) f (x) is discontinuous at x = 0. = (1 – h) = 1
⎛ 1 1⎞
2-⎜ + ⎟
Solution: (D) R.H.L. = lim f (0 + h) = lim ( h + 1) ⎝ h h⎠
h→0 h→0
lim f (x) = lim f (0 + h) 2
x→0 +
x→0 +
2-
= lim (1 + h) h
e( h) + h - 1 h→0
= lim
h → 0 ( h) + h = lim (1 + h)2 [(1 + h)1/h ]– 2
h→0

h
e -1 = 1 × e– 2 = e– 2
= lim = 1 Since LHL ≠ RHL,
h→0 h
\ f (x) is not continuous at x = 0.

Objective_Maths_JEE Main 2017_Ch 12.indd 3 01/01/2008 05:24:45


12.4  Chapter 12

(A)  continuous for all real x


1
4. If f (x) = , then the points of discontinuity of the (B)  continuous only at some values of x
1- x (C)  discontinuous for all real x
function f [ f { f (x)}] are (D)  discontinuous only at some values of x
(A)  {0, – 1} (B)  {0, 1}
Solution: (C)
(C)  {1, – 1} (D)  None of these
Let x0 be any arbitrary real number.
Solution: (B) Case I: x0 is rational
1
We have, f (x) = . Then f (x0) = 1
1- x
As at x = 1, f (x) is not defined, x = 1 is a point of dis- In any vicinity of a rational point there are irrational
continuity of f (x). points, where f (x) = 0. Hence, in any vicinity of x0
there are points x for which
⎛ 1 ⎞ 1 x -1
If x ≠ 1, f [ f (x)] = f ⎜ ⎟ = =
⎝1 - x⎠ 1 - 1 / (1 - x ) x | D y | = | f (x0) – f (x) | = 1

\ x = 0, 1 are points of discontinuity of f [ f (x)]. Case II: x0 is irrational


Then f (x0) = 0
If x ≠ 0, x ≠ 1,
In any vicinity of an irrational point there are rational
⎛ x - 1⎞ 1 points at which f (x) = 1. Hence, it is possible to find
f [ f { f (x)}] = f ⎜ = = x
⎝ x ⎟⎠ ( x - 1) the values of x for which
1-
x
| D y | = | f (x0) – f (x) | = 1
⎧ e[ x ] + | x | - 2 Thus, in both cases, the difference D y does not tend to
⎪ , x≠0
5. If f (x) = ⎨ [ x ] + | x | , ([ . ] denotes the great- zero as D x → 0. Therefore, x0 is a point of discontinu-
⎪ -1 x=0 ity. Since x0 is an arbitrary point, the Dirichlet function
⎩ f (x) is discontinuous at each point.
est integer function) then
(A) f (x) is continuous at x = 0 ⎧ x, when x is rational
(B)  lim f ( x ) = – 1 7. If f (x) = ⎨ , then
x → 0+ ⎩1 - x, when x is irrational
(C)  lim f ( x ) = 1 (A) f (x) is continuous for all real x
x → 0- (B) f (x) is discontinuous for all real x
(D)  None of these (C) f (x) is continuous only at x = 1/2
Solution: (D) (D) f (x) is discontinuous only at x = 1/2
⎧ e[ x ] + | x | - 2 Solution: (C)
⎪ , x≠0
(x) = ⎨ [ x ] + | x |
f  Let a be any real number.
⎪- 1 , x=0
⎩ lim f (x) = lim x = a
x→a x→a
e[ x ] + | x | - 2 e -1 - 2 (when x → a through rational values)
lim f ( x ) = lim =
x→0 -
x → 0 [ x] + | x |
-
-1 lim f (x) = lim (1 – x) = 1 – a
x→a x→a
e[ x ] + | x | - 2
lim f ( x ) = lim (when x → a through irrational values)
x→0 +
x → 0 [ x] + | x |
+


lim f (x) will exist only when a = 1 – a
x→a
ex - 2
= lim → -∞ 1
x→0 x i.e., when a = .
+


2
6. The Dirichlet function, defined as 1
Thus if x ≠ , then lim f (x) will not exist and hence
⎧1 if x is rational 2 x→a
f (x) = ⎨ , is 1
⎩0 if x is irrational f (x) will be discontinuous at x = a where a ≠ .
2

Objective_Maths_JEE Main 2017_Ch 12.indd 4 01/01/2008 05:24:49


Continuity and Differentiability  12.5

2
1 ⎛ 1⎞ 1 ⇒ log L = lim tan 2 x log sin 2 x
Also, lim f (x) = and f ⎜ ⎟ = x → p /4
x→
1 2 ⎝ 2⎠ 2
2
1 log sin 2 x ⎛ ∞ ⎞
Hence, f (x) is continuous at x = . = lim ⎜ ⎟
2
x → p /4 cot 2 2 x ⎝ ∞ ⎠
2 cot 2 x 1
⎧ a /|sin x | -p = lim =–
⎪(1 + |sin x |) ,
6
<x<0 x → p /4 2
-2 cot 2 x cosec 2 x ⋅ 2 2

⎪ p
8. Let f (x) = . ⎨e tan 2 x /tan 3 x ,0< x< The values or L = e–1/2 \ f (p/4) = e–1/2 = 1/ e
⎪ 6
⎪ b ,x=0 10. Let a function f : R → R satisfy the equation f (x + y) =
⎪ f (x) + f (y) for all x, y. If the function f (x) is continuous

of a and b so that f (x) may be continuous at x = 0 are at x = 0, then
-2 2 (A) f (x) = 0 for all x
(A) a = , b = e2/3 (B)  a = , b = e– 2/3 (B) f (x) is continuous for all positive real x
3 3
2 (C) f (x) is continuous for all x
(C) a = , b = e2/3 (D)  None of these (D)  None of these
3
Solution: (C) Solution: (C)
We have, Since f (x) is continuous at x = 0,
lim f (0 – h) = lim [1 + | sin (– h) |]a/| sin (– h ) | \ lim f (x) = f (0)
h→0 h→0 x→0

Take any point x = a, then at x = a
= lim (1 + sin h)a/sin h
h→0
lim f (x) = lim f (a + h) = lim [ f (a) + f (h)]
x→a h→0 h→0
= lim [(1 + sin h)1/sin h]a = ea,
h→0
[Q f (x + y) = f (x) + f ( y)]
tan 2h/tan 3h
lim f (0 + h) = lim e
= f (a) + lim f (h) = f (a) + f (0)
h → 0 h→0
h→0
⎛ tan 2 h 2 3h ⎞ 2 2
× ×
lim
h→0 ⎜
⎝ 2h 3 tan 3h ⎠⎟ 1× ×1 = f (a + 0) = f (a)
= e = e 3 = e3
and f (0) = b. \ f (x) is continuous at x = a. Since x = a is any arbi-
For f to be continuous at x = 0, we must have trary point, therefore f (x) is continuous for all x.

lim f (0 – h) = lim f (0 + h) = f (0) ⇒ ea = e2/3 = b ⎛ 1⎞


h→0 h→0 11. If f (x) is continuous in [0, 1] and f ⎜ ⎟ = 2, then
2 ⎝ 2⎠
⇒ a= and b = e2/3 ⎛ n ⎞
3 lim f ⎜ ⎟ is equal to
n→∞ ⎝ 2 n + 1⎠
2

9. The function f (x) = (sin 2 x ) tan 2 x is not defined at (A)  0 (B)  ∞


x = p/4. The value of f (p/4) so that f is continuous at (C)  2 (D)  None of these
x = p/4 is
(A)  e (B)  1/ e Solution: (C)
(C)  2 (D)  None of these Since f (x) is continuous in [0, 1], therefore,

Solution: (B) ⎛ n ⎞ ⎛ n ⎞
lim f ⎜ ⎟ = f ⎜ lim ⎟
f is continuous at x = p/4, if n→∞ ⎝ 2 n + 1⎠ ⎝ n → ∞ 2 n + 1⎠
lim f ( x ) = f (p/4).
x → p /4
⎛ 1⎞
= f ⎜ ⎟ = 2
Now, L = lim (sin 2 x ) tan 2 2 x ⎝ 2⎠
x → p /4

Objective_Maths_JEE Main 2017_Ch 12.indd 5 01/01/2008 05:24:54


12.6  Chapter 12

12. The function f (x) = (x), where (x) denotes the smallest Differentiability of a Fucntion
integer ≥ x, is
The function, f (x) is differentiable at a point P, iff there
(A)  continuous everywhere
exists a unique tangent at the point P. In other words, f (x)
(B)  continuous at integral points only
is differentiable at a point P iff the curve does not have P
(C)  continuous at non-integral points only
as a corner point, i.e., “the function is not differentiable at
(D)  None of these
those points on which function has jumps (or holes) and
Solution: (C) sharp edges.”
Let x = n, n ∈ Z Consider the function f (x) = |x|. Let us draw the graph
of this function
Then, L.H.L. = lim (x) = n; R.H.L. = lim (x) = n + 1
x→n x→n
x<n x>n Y

Since, L.H.L. ≠ R.H.L., therefore f (x) is discontinuous


at all integers n. f(x) = –x f(x) = x
f′(x) = –1 f′(x) = 1
Now, let x = p, n < p < n + 1, where n is an integer.
X
Then, L.H.L. = lim (x) = n + 1, R.H.L. O
x→ p
x< p
Fig. 12.3
= lim (x) = n + 1
x→ p
x> p which shows that f (x) is not differentiable at x = 0 as f (x)
f ( p) = ( p) = n + 1 has sharp edge at x = 0.
Since L.H.L. = R.H.L. = f ( p), therefore, f (x) is con-
tinuous at all non-integral points p.
Differentiability of a Function at a Point
Let f be a function defined on an interval I ⊆ R. We say that
13. Let f be a function defined and continuous on [2, 5]. If f is differentiable at an interior point c ∈ I provided
f (x) takes rational values for all x and f (4) = 8 then the
value of f (3.7) is f ( x ) - f (c)
lim exists and is finite.
(A) 0 (B) 8 x→c x-c
(C)  – 1 (D)  None of these We denote this limit by f ′(c), called the derivative of f at c.
Solution : (B)
Since f is continuous on [2, 5], therefore f assumes Trick(s) for Problem Solving
atleast once, every values between f (2) and f (5). But
it is given that f (x) takes only rational values for all x In view of the definition of limit of a function f, one may
and there are irrational values also between f (2) and observe that f  ′(c) exists provided
f (5), this is possible only if f (x) has a constant ratio- f ( x ) - f (c ) f ( x ) - f (c )
lim = f  ′(c) = lim
nal value at all points between x = 2 and x = 5. Since x → c- x-c x → c+ x-c
f (4) = 8, \ f (3 . 7) = 8.
The limit on the left, denoted by L f  ′(c), is called the left hand
3
14. Let f (x) = [x – 3], where [.] denotes the greatest inte- derivative of f at c and the limit on the right, denoted by R
f  ′(c), is called the right hand derivative of f at c.
ger function. Then the number of points in the interval
(1, 2) where the function is discontinuous, is Thus, f (x) is differentiable at x = c if L f  ′(c) = R f  ′(c).
(A) 4 (B) 2
(C)  6 (D)  None of these Differentiability of a Function on an
Solution: (C) Interval
Let g (x) = x3 – 3, then g (x) is an increasing function A function f (x) is said to be differentiable on an open inter-
on the interval (1, 2). Since g (1) = – 2 and g (2) = 5, val (a, b) if f (x) is differentiable at every point of this inter-
therefore between – 2 and 5 there are 6 points where val (a, b).
f (x) is discontinuous (as [x3 – 3] is discontinuous at the It is differentiable on a closed interval [a, b] if it is
points where x3 – 3 is an integer). differentiable on the open interval (a, b) and the limits.

Objective_Maths_JEE Main 2017_Ch 12.indd 6 01/01/2008 05:24:56


Continuity and Differentiability  12.7

f ( a + h) - f ( a) f (1 - h) - f (1)
R f ′(a) = lim We have, L f ′(1) = lim
h→0 ( a + h) - a h→0 -h
f ( b - h) - f ( b ) (1 - h) [1 - h] - 1 [1]
and L f ′ (a) = lim exist. = lim
h→0 ( b - h) - b h→0 -h
0 -1
= lim →∞
Some Important Results on Differentiability h→0 -h
1. Every polynomial function, exponential function and \ f ′(1) does not exist.
constant function is differentiable at each point of the f ( 2 - h) - f ( 2)
Also, L f ′ (2) = lim
real line. h→0 -h
2. Logarithmic functions, trigonometric functions and ( 2 - h) [2 - h] - ( 2 - 1) [2]
inverse trigonometric functions are differentiable in = lim
h→0 -h
their domain.
3. The sum, difference, product and quotient of two dif- 2-h-2
= lim = 1
ferentiable functions is differentiable. h→0 -h
4. The composition of differentiable functions is a differ- f ( 2 + h) - f ( 2)
entiable function. and R f ′(2) = lim
h→0 h
5. If a function is not differentiable but is continuous at a
point, it geometrically implies there is a sharp corner (1 + h) [2 + h] - ( 2 - 1) [2]
= lim
or a kink at that point. h→0 h
6. If f (x) and g (x) both are not differentiable at a point, 2 + 2h - 2
then the sum function f (x) + g (x) and the product func- = lim = 2.
h→0 h
tion f (x) × g (x) can still be differentiable at that point. \ f ′ (2) also does not exist.

⎧ p 1
Trick(s) for Problem Solving ⎪ x cos , x ≠ 0
16. If f (x) = ⎨ x , then at x = 0, f (x) is
 If a function f (x) is differentiable at a point x = a then it is ⎪⎩ 0 , x= 0
continuous at x = a.
(A)  continuous if p > 0 and differentiable if p > 1
 If f (x) is only continuous at a point x = a, there is no guar-
(B)  continuous if p > 1 and differentiable if p > 0
antee that f (x) is differentiable there.
(C)  continuous and differentiable if p > 0
 If f (x) is not differentiable at x = a then it may or may not
(D)  None of these
be continuous at x = a.
 If f (x) is not continuous at x = a, then it is not differentiable Solution: (A)
at x = a. Continuity at x = 0:
 If left hand derivative and right hand derivative of f (x) at x 1
= a are finite (they may or may not be equal) then f (x) is L.H.L. = lim f (0 – h) = lim (– h) p cos
h→0 h→0 h
continuous at x = a.
= 0 if p > 0
1
R.H.L. = lim f (0 + h) = lim h p cos
h→0 h→0 h
Solved Examples = 0 if p > 0
⎧ x [ x ], 0 ≤ x < 2 and f (0) = 0.
15. If f (x) = ⎨ , where [.] denotes the
⎩( x - 1)[ x ], 2 ≤ x ≤ 3 \ f (x) is continuous at x = 0 if p > 0
greatest integer function, then Differentiability at x = 0 :
(A) both f ′ (1) and f ′ (2) do not exist f (0 - h) - f (0)
L f ′(0) = lim
(B) f ′ (1) exists but f ′ (2) does not exist h→0 -h
(C) f ′ (2) exists but f ′ (1) does not exist 1
(D) both f ′ (1) and f ′ (2) exist ( - h) p cos -0
= lim h
Solution: (A) h→0 -h

Objective_Maths_JEE Main 2017_Ch 12.indd 7 01/01/2008 05:25:00


12.8  Chapter 12

(A)  continuous as well as differentiable at x = 0


1
= lim (– h)p – 1 cos = 0 if p – 1 > 0, (B)  continuous but not differentiable at x = 0
h→0 h (C)  differentiable but not continuous at x = 0
i.e., p > 1; (D)  None of these
f (0 + h) - f (0) Solution: (D)
R f ′(0) = lim
h→0 h 1 1
lim f (0 – h) = lim = = 1
1
h p cos - 0
h→0 h→0 1 + e -1/ h 1+ 0
= lim h
h→0 h 1 e -1/ h
and lim f (0 + h) = lim = lim
1 h→0 h→0 1 + e1/ h h→0 e -1/ h + 1
= lim hp – 1 cos = 0 if p > 1
h→0 h 0
\ f (x) is differentiable at x = 0 if p > 1. = = 0
1
⎧ 1 Since lim f (0 – h) ≠ lim f (0 + h), therefore, f (x) is
⎪ x sin , x ≠ 0
17. Let g (x) = x f (x), where f (x) = ⎨ x . h→0 h→0
⎪⎩0, not continuous and hence not differentiable at x = 0.
x=0
At x = 0,
(A) g is differentiable but g′ is not continuous ⎧ e1/ x - 1
⎪ ,x≠0
(B)  g is differentiable while f is not 19. If f (x) = ⎨ e1/ x + 1 , then f (x) is
(C) both f and g are differentiable ⎪
⎩ 0 ,x=0
(D) g is differentiable and g′ is continuous.
(A)  continuous as well as differentiable at x = 0
Solution: (A, B) (B)  continuous but not differentiable at x = 0
⎧ 2 1 (C)  differentiable but not continuous at x = 0
⎪ x sin , x≠0 (D)  None of these
We have, g (x) = ⎨ x
⎪⎩0, x=0 Solution: (D)
For x ≠ 0, e -1/ h - 1 0 -1
lim f (0 – h) = lim -1/ h
=
1 ⎛ 1⎞ 1 h→0 h→0 e +1 0 +1
g′ (x) = x2 cos ⎜⎝ - 2 ⎟⎠ + 2x sin
x x x = – 1
e1/ h - 1 1 - e -1/ h
1 1 and lim f (0 + h) = lim = lim
= – cos + 2x sin h→ 0 h→ 0 e1/ h + 1 h→ 0 1 + e -1/ h
x x
For x = 0 1- 0
1 = = 1
x 2 sin - 0 1+ 0
g ( x ) - g ( 0) x
g′ (0) = lim = lim
x→0 x-0 x→0 x Since lim f (0 – h) ≠ lim f (0 + h), therefore, f (x) is
h→0 h→0
1
= lim x sin = 0. not continuous and hence not differentiable at x = 0.
x →0 x
⎧ 1 1 ⎧ x(e1/ x - e -1/ x )
⎪2 x sin - cos , x≠0 ⎪ , x≠0
\ g′ (x) = ⎨ x x 20. The function f (x) = ⎨ e1/ x + e -1/ x is
⎪⎩0, x=0 ⎪
⎩0 ,x=0
1
g′ is not continuous at x = 0 as cos is not continu- (A) continuous everywhere but not differentiable at
x x=0
ous at x = 0. Also, f is not differentiable at x = 0.
(B)  continuous and differentiable everywhere
⎧ 1 (C)  not continuous at x = 0
⎪ ,x≠0 (D)  None of these
18. If f (x) = ⎨1 + e1/ x , then f (x) is
⎪ 0 , x= 0 Solution: (A)

Objective_Maths_JEE Main 2017_Ch 12.indd 8 01/01/2008 05:25:05


Continuity and Differentiability  12.9

Differentiability at x = 0 :
( 5 - h - 2) - ( 5 - 2)
f (0 - h) - f (0) = lim
Lf ′ (0) = lim h→0 -h
h→0 -h 2-3
= lim → ∞
- h (e -1/ h - e1/ h ) h→0 -h
= lim
h→0 - h (e -1/ h + e1/ h ) Hence, f ′ (2 . 5) = 0 while f ′ (5) does not exist.
e - 2/ h - 1
= lim = – 1 ⎛ p⎞ ⎛p⎞
h→0 e - 2/ h + 1 22. If f (x) = [tan x], x ∈ ⎜ 0, ⎟ , then f ′ ⎜ ⎟ is equal to
⎝ 3 ⎠ ⎝ 4⎠
f (0 + h) - f (0) (A) 1 (B) 0
Rf ′(0) = lim
h→0 h (C)  does not exist (D)  None of these

h (e1/ h - e -1/ h ) Solution: (C)
= lim ⎛p ⎞ ⎛p⎞
h→0 h (e1/ h + e -1/ h ) f ⎜ - h⎟ - f ⎜ ⎟
⎛p⎞ ⎝4 ⎠ ⎝ 4⎠
L f ′ ⎜ ⎟ = lim
1 - e - 2/ h ⎝ 4⎠ h→0 -h
= lim = 1
h → 0 1 + e - 2/ h
⎡ ⎛ p ⎞ ⎤ ⎡ p⎤
Since L f ′(0) ≠ R f ′ (0), \ f (x) is not differentiable at ⎢ tan ⎜⎝ 4 - h⎟⎠ ⎥ - ⎢ tan 4 ⎥
= lim ⎣ ⎦ ⎣ ⎦
x = 0. h→0 -h
But since L f  ′ (0) and R f ′ (0) are finite, therefore f (x)
is continuous at x = 0. 0 -1
= lim → ∞
Hence, f (x) is continuous every where but not dif- h→0 -h
ferentiable at x = 0. ⎛p⎞
\ f ′ ⎜ ⎟ does not exist.
21. If f (x) = [x – 2], then ⎝ 4⎠
1
(A) f ′ (2.5) = and f ′ (5) = 3
2 ⎧| x |
(B) f ′ (2.5) = 0 and f ′ (5) = 3 ⎪ ,x≠0
23. If sgn (x) = ⎨ x , then the function
(C) f ′ (2.5) = 0 and f ′ (5) does not exist ⎪⎩0 , x = 0
(D) both f ′ (2.5) and f ′ (5) do not exist
f (x) = sgn [sgn (x)] is
Solution: (C) (A)  continuous as well as differentiable at x = 0
We have (B)  continuous but not differentiable at x = 0
f ( 2 ⋅ 5 - h) - f ( 2 ⋅ 5) (C)  differentiable but not continuous at x = 0
L f ′ (2 . 5) = lim (D)  neither differentiable nor continuous at x = 0
h→0 -h
( 2 ⋅ 5 - h - 2) - ( 2 ⋅ 5 - 2) Solution: (D)
= lim We have,
h→0 -h ⎧ ⎛ | x |⎞
⎪sgn ⎜ ⎟ , x ≠ 0
0 sgn [sgn (x)] = ⎨ ⎝ x⎠
= lim = 0 ⎪sgn (0)
-h h→0 ⎩ ,x=0
f ( 2 ⋅ 5 + h) - f ( 2 ⋅ 5)
and R f ′ (2 . 5) = lim ⎧ | x|
h→0 h ⎪ x ,x≠0
= ⎪⎨ | x |
( 2 ⋅ 5 + h - 2) - ( 2 ⋅ 5 - 2) ⎪
x
= lim
h→0 h ⎪⎩ 0 ,x=0
0
= lim = 0 ⎧x
h→0 h ,x≠0
\ f (x) = ⎪⎨ | x |
\  f ′ (2 . 5) = 0 ⎪0 , x = 0

f (5 - h) - f (5)
Also, L f ′ (5) = lim Continuity at x = 0 :
h→0 -h

Objective_Maths_JEE Main 2017_Ch 12.indd 9 01/01/2008 05:25:11


12.10  Chapter 12

Solution: (B)
-h
L.H.L. = lim f (0 – h) = lim =–1 f ( x + h) - f ( x )
h→0 h→0 | - h| f ′ (x) = lim
h→0 h
and f (0) = 0
f ( x + h) - f ( x + 0 )
\f (x) is not continuous and hence not differentia- = lim
h→0 h
ble at x = 0.
⎧| x | ⎫ f ( x ) ⋅ f ( h) - f ( x ) ⋅ f (0)
⎪ , x ≠ 0⎪ = lim
24. If f (x) = x5 sgn x, where sgn x = ⎨ x ⎬ , then f (x) h→0 h
is ⎪⎩ 0 , x = 0⎪⎭
f ( h) - f ( 0 )
(A)  differentiable as well as continuous at x = 0 = lim ⋅ f ( x)
(B)  continuous but not differentiable at x = 0 h→0 h
(C)  differentiable but not continuous at x = 0 = f ′ (0) . f (x) = 2f (x).(Q f ′ (0) = 2)
(D)  neither differentiable nor continuous at x = 0
df df
Now, = 2f or = 2 dx ⇒ d (log f – 2x) = 0
Solution: (A) dx f
Since, \ log f – 2x = c, \ f = e2x + c = ec × e2x = Ae2x,
⎧| x | ⎧1, x > 0
⎪ , x≠0 ⎪ where A = ec = constant.
sgn x = ⎨ x or sgn x = ⎨0, x = 0
⎪⎩0 , x = 0 ⎪ - 1, x < 0 27. Let f (x + y) = f (x) .  f ( y) for all x, y where f (0) ≠ 0. If

f (5) = 2 and f ′ (0) = 3, then f ′ (5) is equal to
⎧ x5 , x > 0 (A) 6 (B) 0
5 ⎪
Therefore, f (x) = x sgn x = ⎨0 ,x=0 (C)  1 (D)  None of these
⎪ 5
⎩- x , x < 0 Solution: (A)
f (5 + h) - f (5)
Clearly, f (x) is continuous as well as differentiable f ′ (5) = lim
at x = 0. h→0 h

25. Let f (x) = | x | and g (x) = [x], where [.] denotes the f (5 + h) - f (5 + 0)
= lim
greatest integer function. Then (fog) ‘ (– 2) is h→0 h
(A)  0 (B)  does not exist f (5) ⋅ f ( h) - f (5) ⋅ f (0)
(C) – 1 (D) 1 = lim
h→0
h
Solution: (B) [Q f (x + y) = f (x) .  f ( y) for all x, y]
( fog) (x) = f (g (x)) = f ([x]) = | [x] | ⎛ f ( h) - f (0) ⎞
= ⎜ lim ⎟⎠ ⋅ f (5)
⎝h→0 h
Now, L ( fog) ′ (– 2) = lim
h→0 = f ′ (0) .  f (5) = 3 . 2 = 6
( fog ) ( - 2 - h) - ( fog ) ( - 2)
-h 28. Let f : R → R be a function such that

⎛ x + y⎞ f ( x) + f ( y)
|[ - 2 - h]| - |[ - 2]| f ⎜ ⎟ = , f (0) = 0 and f’ (0) = 3.
= lim ⎝ 3 ⎠ 3
h→0 -h
Then
| - 3 | - | - 2| -1 (A) f (x) is a quadratic function
= lim
 = lim →–∞
h→0 -h h→0 h (B) f (x) is continuous but not differentiable
(C) f (x) is differentiable in R
\ ( fog) ′(– 2) does not exist. (D) f (x) is bounded in R
26. Let f (x + y) = f (x) f ( y) for all x, y, where f (0) ≠ 0. If Solution: (C)
f ′ (0) = 2, then f (x) is equal to We have,
(A) Aex (B)  Ae2x ⎛ x + y⎞ f ( x) + f ( y)
f⎜ = , f (0) = 0 and f ′ (0) = 3
(C) 2x (D)  None of these ⎝ 3 ⎟⎠ 3

Objective_Maths_JEE Main 2017_Ch 12.indd 10 01/01/2008 05:25:15


Continuity and Differentiability  12.11

f ( x + h) - f ( x ) 1 + h f ( h) - 1
f ′ (x) = lim = f (x) lim
h→0 h h→0 h
⎛ 3 x + 3h ⎞ = f (x) lim f (h)
f⎜ - f ( x) h→0
= ⎝ 3 ⎟⎠
lim = f (x) . 1 = f (x)
h→0 h
f ( x - h) - f ( x )
f (3 x ) + f (3h) f (3 x ) + f (0) and L f ′ (x) = lim
- h→0 -h
= 3 3
lim
h→0 h f ( x ) f ( - h) - f ( x )
= lim
f (3h) - f (0) h→0 -h
= lim =3
h→0 3h f ( - h) - 1
= f (x) lim
\ f 
(x) = 3x + c, Q f (0) = 0 ⇒ c = 0 h→0 -h

\ f (x) = 3x 1 - h f ( - h) - 1
= f (x) lim
h→0 -h
29. If f (x + y) = 2 f (x) . f ( y) for all x, y, where f ′ (0) = 3 and = f (x) . 1 = f (x).
f (4) = 2, then f ′ (4) is equal to
Hence f ′ (x) exists and is equal to f (x).
(A) 6 (B) 12
(C)  4 (D)  None of these 31. Let f (x + y) = f (x) .  f ( y) for all x, y ∈ R and f (x) = 1 +
x f (x) log 2 where lim f (x) = 1. Then f ′ (x) is equal
Solution: (B) x→0
to
f ( 4 + h) - f ( 4)
f ′ (4) = lim (A)  log 2 f (x) (C)  log [ f (x)]2
h→0 h (C)  log 2 (D)  None of these
f ( 4 + h) - f ( 4 + 0) Solution: (A)
= lim
h→0 h f ( x + h) - f ( x )
f ′ (x) = lim
h→0 h
2 f ( 4) ⋅ f ( h) - 2 f ( 4) f (0)
= lim f ( x ) ⋅ f ( h) - f ( x )
h→0h = lim
h→0 h
[Using f (x + y) = 2f (x) .  f ( y) for all x, y]
[Q f (x + y) = f (x) .  f ( y)]
f ( h) - f (0) ⎛ f ( h) - 1⎞
= 2f (4) lim
h→0 h = f (x) lim ⎜ ⎟⎠
h→0 ⎝ h
= 2f (4) × f ′ (0) = 2 × 2 × 3 = 12 1 + h f ( h) log 2 - 1
= f (x) lim
h→0 h
30. If a function f : R → R be such that f (x + y) = f (x) . f ( y) [Q f (x) = 1 + x f (x) log 2]
for all x, y ∈ R where f (x) = 1 + x f (x) and lim f (x)
= 1, then x→0 = f (x) log 2 lim f (h)
h→0
(A) f ′ (x) does not exist ⎡ ⎤
(B) f ′ (x) = 2f (x) for all x = f (x) × log 2 × 1 ⎢⎣Q hlim f ( h) = 1⎥
→0 ⎦
(C) f ′ (x) = f (x) for all x
= log 2 f (x)
(D)  None of these
Solution: (C) 32. Let f (x + y) = f (x) .  f ( y) and f (x) = 1 + x g (x)
f ( x + h) - f ( x ) G (x) where lim g (x) = a and lim G (x) = b. Then
R f ′ (x) = lim x→0 x→0
h→0 h f ′(x) = k f (x), where k is equal to
f ( x ) f ( h) - f ( x ) a
= lim (A)  (B)  1 + ab
h→0 h b
(C) ab (D)  None of these

Objective_Maths_JEE Main 2017_Ch 12.indd 11 01/01/2008 05:25:21


12.12  Chapter 12

Solution: (C)
⎧2 x, x ≥ 0
f ( x + h) - f ( x ) ⎪
f ′ (x) = lim and 4x – 3y = 5y if y < 0 ⇒ y = ⎨ 1
h→0 h ⎪⎩ 2 x, x < 0
f ( x ) ⋅ f ( h) - f ( x ) Clearly, y is continuous at x = 0 but not differentiable
= lim
h h→0 at x = 0.
[Q f (x + y) = f (x) .  f (y)]
⎧2, x ≥ 0
⎛ f ( h) - 1⎞ dy ⎪
= f (x) .  lim ⎜ Also, = ⎨1
h→0 ⎝
⎟ dx ⎪⎩ 2 , x < 0
h ⎠

1 + h g ( h) G (h) - 1
= f (x) lim 35. The function f (x) = max. {(1 – x), (1 + x), 2}, x ∈ (– ∞,
h→0 h ∞), is
= f (x) lim g (h) G (h) (A)  continuous at all points
h→0
(B)  differentiable at all points
= f (x) lim g (h) lim G (h) (C) differentiable at all points except at x = 1 and x
h→0 h→0
=–1
= ab f (x). \ k = ab (D) continuous at all points except at x = 1 and x = – 1,
where it is discontinuous
33. Let f and g be differentiable functions satisfying g ′ (a) Solution: (A, C)
= 2, g (a) = b and fog = I (identity function). Then f ′
(b) is equal to ⎧1 - x, x ≤ -1
2 ⎪
(A)  2 (B)  f (x) = ⎨2, -1 < x ≤ 1
3 ⎪1 + x, x >1
1 ⎩
(C)  (D)  None of these
2 lim f ( x ) = lim (1 - x ) = 2 = lim f ( x )
x → -1- x → -1- x → -1+
Solution: (C)
We have, fog = I and lim f ( x ) = 2, so f is continuous at all points.
x →1
⇒ ( fog) (x) = x for all x f ( -1 + h) - f ( -1)
f ′ (– 1–) = lim
⇒ f [ g (x)] = x ⇒ f ′ [ g (x)] × g ′ (x) = 1 h → 0- h
1+1- h - 2
⇒ f ′ [ g (a)] × g ′ (a) = 1 = lim =–1
h→ 0 -
h
1 1 +
⇒ f ′ [ g (a)] = = f ′ (– 1 ) = 0
g ′ ( a) 2
[Q g ′ (a) = 2] Similarly, f ′ (1–) = 0 and f ′ (1+) = 1, so f is differentia-
ble everywhere except at x = – 1, 1.
1
⇒ f ′ (b) = [Q g (a) = b] Y
2

34. If 4x + 3| y | = 5y, then y as a function of x is


x
y

+
=

1
1

(A)  not continuous at x = 0


=

x

(B)  not defined for all real x 2


dy 1 X
(C)  = for x < 0 O
dx 2
(D)  derivable at x = 0 36. Let f : R → R be a function defined by f (x) = max
Solution: (C) {x, x3}. The set of all points where f (x) is not differ-
entiable is
We have, 4x + 3 | y | = 5y (A)  {– 1, 1} (B)  {– 1, 0}
⇒ 4x + 3y = 5y if y ≥ 0 (C)  {0, 1} (D)  {– 1, 0, 1}

Objective_Maths_JEE Main 2017_Ch 12.indd 12 01/01/2008 05:25:25


Continuity and Differentiability  12.13

Solution: (D) We have,

If x < – 1, then x > x3. So, f (x) = x. f (x) = [x] + | 1 – x |, – 1 ≤ x ≤ 3


⎧- x , -1≤ x < 0
If x = – 1, then x = x3. So, f (x) = x. ⎪1 - x , 0 ≤ x <1
⎪⎪
If – 1 < x < 0, then x < x3. So, f (x) = x3. = ⎨x ,1≤ x < 2
⎪1 + x ,2≤ x<3

If x = 0, then x = x3. So, f (x) = x3. ⎪⎩5 ,x=3

If 0 < x < 1, then x > x3. So, f (x) = x. The only doubtful points are x = – 1, 0, 1, 2 and 3. It
can be easily seen that f (x) is differentiable at x = – 1
If x = 1, then x = x3. So, f (x) = x. but not differentiable at x = 0, 1, 2 and 3.
Hence, the required points are 0, 1, 2 and 3.
If x > 1, then x < x3. So, f (x) = x3.
39. Let f (x) = a + b | x | + c | x |4, where a, b and c are real
constants. Then f (x) is differentiable at x = 0 if
Thus, f (x) = x, x ≤–1
(A) a = 0 (B)  b=0
x3, – 1 < x ≤ 0 (C) c = 0 (D)  None of these
Solution: (B)
x, 0 <x≤1 Since f (x) is differentiable at x = 0, therefore,
x3, x > 1. L f ′ (0) = R f ′ (0)
Clearly, f (x) is not differentiable at x = – 1, 0, 1.
f (0 - h) - f (0) f (0 + h) - f (0)
⇒ lim = lim
2 f ( x) - 5 h→0 -h h→0 h
37. If g (x) = (x + 2x + 3) f (x), f (0) = 5 and lim
= 4, then g ′(0) is equal to x→0 x
( a + b | - h | + c | - h |4 ) - a
(A) 22 (B) 20 ⇒ lim
(C)  18 (D)  None of these
h→0 -h
4
Solution: (A)  = lim ( a + b | h | + c | h | ) - a
h→0 h
f ( x) - 5
We have, lim = 4
x→0 x
bh + ch4 bh + ch4
f ( x ) - f ( 0) ⇒ lim = lim
⇒ lim = 4 [Q f (0) = 5] h→0 -h h→0 h
x→0 x
⇒ f ′ (0) = 4. ⇒ lim (– b – ch3) = lim (b + ch3)
h→0 h→0
Since, g (x) = (x2 + 2x + 3) f (x)
⇒ – b = b i.e. b = 0
2
⇒ g ′ (x) = (x + 2x + 3) f ′ (x) + (2x + 2) f (x)
1
40. If f (x) = , where [.] denotes the greatest integer
\ g ′ (0) = 3 f ′ (0) + 2 f (0) = 3 (4) + 2 (5) = 22. [sin x ]
function, then
38. The points where the function f (x) = [x] + | 1 – x |, (A) Domain of f (x) is (2np + p, 2np + 2p) ∪
–1 ≤ x ≤ 3, where [.] denotes the greatest integer func-
⎧ p⎫
tion, is not differentiable, are ⎨2np + ⎬ where n ∈ I
(A) x = – 1, 0, 1, 2, 3 ⎩ 2⎭
(B) x = – 1, 0, 2 (B) f (x) is continuous when x ∈ (2np + p, 2np + 2p)
(C) x = 0, 1, 2, 3 (C) f (x) is not differentiable at x = p/2
(D) x = – 1, 0, 1, 2 (D)  None of these

Solution: (C) Solution: (A, C)

Objective_Maths_JEE Main 2017_Ch 12.indd 13 01/01/2008 05:25:28


12.14  Chapter 12

The function f (x) is defined when – 1 ≤ sin x < 0 or 1


sin x = 1. -1
p⎫ ⎡ ⎛p ⎞⎤
⎧ sin
⎢ ⎜⎝ 2 - h⎟⎠ ⎥
⇒ x ∈ ((2n + 1) p, (2n + 2) p) ∪ ⎨2np + ⎬ , n ∈ I.
⎩ 2⎭ = lim ⎣ ⎦
h→0 -h
When x ∈ (2np + p, 2np + 2p), f (x) = – 1
\ f (x) is a constant function. 1
-1
Hence f (x) is continuous when x ∈ (2np + p, 2np + [cos h]
= lim
2p). h→0 -h
⎛p ⎞ ⎛p⎞
f ⎜ - h⎟ - f ⎜ ⎟ = does not exist.
⎝2 ⎠ ⎝ 2⎠
Now, L f ′ (p/2) = lim Hence, f (x) is not differentiable at x = p/2.
h→0 -h

EXERCISES

Single Option Correct Type

⎧3, x < 0 ⎛ 2 x - 1⎞
1. If f (x) = ⎨ , then 5. The function f (x) = [x] cos ⎜ p , where [.]
⎩2 x + 1, x ≥ 0 ⎝ 2 ⎟⎠
(A) both f (x) and f ( | x | ) are differentiable at x = 0 denotes the greatest integer function, is discontinuous
(B) f (x) is differentiable but f ( | x | ) is not differentia- at
ble at x = 0 (A) all x (B)  all integer points
(C) f ( | x | ) is differentiable but f (x) is not differentia- (C) no x (D)  x which is not an integer
ble at x = 0 6. The left-hand derivative of f (x) = [x] sin (π x) at x = k,
(D) both f (x) and f ( | x | ) are not differentiable at x = 0 k an integer and [x] = greatest integer ≤x, is
2. Let f (x) = cos x and g (x) = [x + 2], where [.] denotes (A)  (– 1)k (k – 1) π
⎛p⎞ (B)  (– 1)k – 1 ⋅ (k – 1) π
the greatest integer function. Then, (gof )′ ⎜ ⎟ is (C)  (– 1)k ⋅ kπ
⎝ 2⎠
(A) 1 (B) 0 (D)  (– 1)k – 1 ⋅ kπ.
(C)  – 1 (D)  does not exist
7. If lim f (x) = l = lim g (x) and lim f (x) = m =
⎧ 1 x → a+ x → a- x → a-
⎪ | x |≥1
3. Let f (x) = ⎨ | x | . If f (x) is continuous and g (x), then the function f (x) ⋅ g (x)
⎪ax 2 + b | x | < 1 (A)  is not continuous at x = a

differentiable at any point, then (B)  has a limit when x → a and it is equal to lm
(C)  is continuous at x = a
1 3 1 3 (D)  has a limit when x → a but it is not equal to lm
(A) a = , b = – (B) a = - ,b=
2 2 2 2
(C) a = 1, b = – 1 (D)  None of these 8. Let [x] denotes the greatest integer less than or equal
to x. If f (x) = [x sin p x], then f (x) is
4. Let f (x) be a function satisfying f (x + y) = f (x) f (y) for
(A)  continuous at x = 0
all x, y∈R. If f (x) = 1 + xf (x) + x2f (x) ψ (x), where
(B)  continuous in (– 1, 0)
lim f (x) = a and lim ψ (x) = b, then f ′(x) is equal to
x→0 x→0 (C)  differentiable at x = 1
(A) (a + b) f (x) (B)  af (x) (D)  differentiable in (– 1, 1)
(B) bf (x) (D)  None of these

Objective_Maths_JEE Main 2017_Ch 12.indd 14 01/01/2008 05:25:31


Continuity and Differentiability  12.15

9. The function f (x) = [x]2 – [x2] (where [ x] is the ­greatest


p 3p
integer less than or equal to x), is discontinuous at (C)  ⎧⎨ x : + 2np ≤ x ≤ ⎫
+ 2np ⎬
(A)  all integers ⎩ 4 4 ⎭
5p 7p
(B)  all integers except 0 and 1  ∪ ⎧⎨ x : + 2np ≤ x ≤

+ 2np ⎬
(C)  all integers except 0 ⎩ 4 4 ⎭
(D)  all integers except 1 (D)  None of these
10. Let f : R→R be any function. Define g:R → R by g (x) 1
16. If f (x) = , then the points of discontinuity of the
= | f (x) | for all x. Then g is 1- x
(A)  onto if f is onto function f 3n(x), where f n = fof ... of (n times), are
(B)  one-one if f is one-one (A) x = 2 (B)  x = 0
(C)  continuous if f is continuous (C) x = 1 (D)  continuous everywhere
(D)  differentiable if f is differentiable
1
11. Let f (x) be a function satisfying the condition 17. The function f (x) = are tan has
x-5
f (– x) = f (x), for all real x. If f ′ (0) exists, then its value (A)  discontinuity of the first kind at x = 5
is (B)  discontinuity of the second kind at x = 5
(A) 0 (B) 1 (C)  removable discontinuity at x = 5
(C)  – 1 (D)  None of these (D)  continuous at x = 5
⎧ x (3e1/ x + 4) n
,x≠0

12. If f (x) = ⎨ 2 - e1/ x , then f (x) is 18. If f (x) = ∑ ak | x - 1|k , where ai∈R then
k =0

⎩ 0 ,x=0 (A) f (x) is continuous at x = 1 for all ak∈R
(A)  continuous as well as differentiable at x = 0 (B) f (x) is differentiable at x = 1 for all ak∈ R
(B)  continuous but not differentiable at x = 0 (C) f (x) is differentiable at x = 1, provided a2k + 1 = 0
(C)  differentiable but not continuous at x = 0 (D) f (x) is continuous at x = 1, provided a2k = 0
(D)  None of these ⎛ 2x ⎞
1 1 19. If f (x) = cos– 1 ⎜⎝ 1 + x 2 ⎟⎠ , then f (x) is differentiable on
13. The function f (x) = 2 , where u = , is
u +u-2 x -1
discontinuous at the points (A) (– ∞, ∞) (B)  (– ∞, ∞) \ {0}
(C) (– ∞, ∞) \ {– 1, 1} (D)  None of these
1 1
(A) x = – 2, 1, (B)  x = , 1, 2 20. The set of points of discontinuities of the function
2 2
(C) x = 1, 0 (D)  None of these f (x) = x - ⎡⎣ x ⎤⎦ , where [x] denotes the greatest inte-

⎛ -p p ⎞ ger less than or equal to x, contains the set


14. Let f (x) = [3 + 2cos x], x ∈ ⎜ , , where [.]
⎝ 2 2 ⎟⎠ (A) (–∞, 0) (B)  {n2 : n ∈N}
denotes the greatest integer function. The number of (C) N (D)  {2n : n ∈N}
points of discontinuity of f (x) is 21. If f (x) = | 3 – x | + (3 + x), where (x) denotes the least
(A) 3 (B) 2 integer greater than or equal to x, then
(C)  5 (D)  None of these (A) f (x) is continuous as well as differentiable at x = 3
15. The set of points of continuity of the function (B) f (x) is continuous but not differentiable at x = 3
(C) f (x) is differentiable but not continuous at x = 3
f (x) =
1 (D) f (x) is neither differentiable nor continuous at x = 3
- cos 2 x is
2
22. Let
p 3p
(A)  ⎧⎨ x : + 2np ≤ x ≤ ⎫
+ 2np , n ∈ I ⎬ ⎧1 + cos x sin 2 x
⎩ 4 4 ⎭ ⎪ ⋅ , x≠p
f (x) = ⎨ (p - x ) log (1 + p 2 - 2p x + x 2 )
2 .
5p 7p ⎪
(B)  ⎧⎨ x : + 2np ≤ x ≤

+ 2np , n ∈ I ⎬ ⎩ k , x=p
⎩ 4 4 ⎭ If f (x) is continuous at x = π, then k is equal to

Objective_Maths_JEE Main 2017_Ch 12.indd 15 01/01/2008 05:25:33


12.16  Chapter 12

30. The value of f (0) so that the function


1 1
(A)  (B)  31+ x - 41+ x
4 2 f (x) = becomes continuous at x = 0, is
-1 -1 x
(C)  (D)  1 7
2 4
(A)  (B) 
12 12
23. Let f (x) be a continuous function defined for 1 ≤ x ≤ 3.
(C)  0 (D)  None of these
If f (x) takes rational values for all x and f (2) = 10, then
f (1⋅5) is equal to f ( h) - f (0)
31. If f is an even function such that lim has
(A) 0 (B) 10 h→0 h
(C)  not defined (D)  any constant some finite non-zero value, then
x (A) f is continuous and derivable at x = 0
1
24. If f (x) = ∫ t cos t dt, then the number of points of (B) f is continuous but not derivable at x = 0
0 (C) f may be discontinuous at x = 0
d­ iscontinuity of f (x) in the interval (0, π) is (D)  None of these
(A) 1 (B) 2 32. If f is differentiable function satisfying f (0) = 0, and if
(C)  0 (D)  None of these
f ( x)
3 g (x) = , then the value, that should be assigned to
25. If f (x) = ( - 1)[ x ] , where [.] denotes the greatest integer x
function, then g (0), so that g is continuous at ‘0’ is
(A) f (x) is discontinuous for x = n1/3, where n∈ I (A) 1 (B) 0
(B) f (3/2) = 1 (C) f (0) (D)  f ′ (0)
(C) f ′ (x) = 0 for – 1 < x < 1 1
(D)  None of these 33. Let f (x) = , [ . ] being the greatest integer func-
[sin x ]
⎡ 1 ⎤ tion, then
26. If f (x) = ⎢ (cos x + sin x ) ⎥ , 0 < x < 2π, where [.] (A) f (x) is not continuous,
⎣ 2 ⎦
denotes the greatest integer function, then the number where x ∈ (2nπ, 2nπ + π), n ∈ I
of points of discontinuity of f (x) is p
(B) f (x) is differentiable at x =
(A) 5 (B) 4 4
(C)  3 (D)  None of these p
(C) f (x) is differentiable at x =
2
27. Let f (x) = a [x] + b e| x | + c | x |2, where a, b and c are (D)  None of these
real constants. If f (x) is differentiable at x = 0, then
34. The function
(A) b = 0, c = 0, a ∈ R
(B) a = 0, c = 0, b ∈ R ⎪⎧1 - 2 x + 3 x 2 - 4 x 3 + ... to ∞, x ≠ - 1
f (x) = ⎨ is
(C) a = 0, b = 0, c ∈ R ⎩⎪ 1 , x = -1
(D)  None of these (A)  continuous and derivable at x = – 1
(B)  neither continuous nor derivable at x = – 1
⎛ p ⎞
28. If f (x) = [x] sin ⎜ , where [.] denotes the great- (C)  continuous but not derivable at x = – 1
⎝ [ x + 1] ⎟⎠ (D)  None of these
est integer function, then the points of discontinuity of
f in the domain are x x
35. If f (x) = x + + + ... to ∞, then at x = 0,
(A) Z (B)  Z \{0} 1 + x (1 + x ) 2
f (x)
(C) R \[– 1, 0) (D)  None of these
(A)  has no limit
29. Let f be a function satisfying f (x + y) = f (x) + f ( y) and (B)  is discontinuous
f (x) = x3f (x) for all x and y, where f (x) is a continuous (C)  is continuous but not differentiable
function then f ′ (x) is equal to (D)  is differentiable
(A) g  (0) (B)  g ′ (x) 36. The function f (x) = [x2] + [– x]2, where [.] denotes the
(C)  0 (D)  None of these greatest integer function, is

Objective_Maths_JEE Main 2017_Ch 12.indd 16 01/01/2008 05:25:35


Continuity and Differentiability  12.17

(A)  continuous and derivable at x = 2


1 3 1 3
(B)  neither continuous nor derivable at x = 2 (A) p = , q = p=
(B)  ,q=–
(C)  continuous but not derivable at x = 2 2 2 2 2
(D)  None of these 5 1 3 1
(C) p = , q = p=– ,q=
(D) 
37. If the function 2 2 2 2
⎧ a
-p
⎪(1 - |tan x |) |tan x | , <x<0 42. If f : R→R is a fucntion defined by f (x) = [x] cos
⎪ 4 ⎛ 2 x - 1⎞
⎪ ⎜⎝ ⎟ p , where [x] denotes the greatest integer
f (x) = ⎨ b ,x=0 2 ⎠
⎪ sin 3 x fucntion, then f is
⎪ sin 2 x p
⎪⎩e ,0< x< (A)  continuous for every real x
4
(B)  discontinuous only at x = 0
is continuous at x = 0, then (C)  discontinuous only at non-zero integral values
-3 3 3 of x
(A) a = b = (B)  a = b = e3/2
2 2 2 (D)  continuous only at x = 0
-3
(C) a = , b = e3/2 (D)  None of these 43. Consider the fucntion, f (x) = | x – 2 | + | x – 5 |, x ∈R.
2
Statement-1: f ′(4) = 0
∞ Statement-2: f is continuous in [2, 5] differentiable in
xn
38. If f (x) = ∑ n! (log a)n, then at x = 0, f (x) (2, 5) and f (2) = f (5)
n=0
(A)  Statement-1 is false, Statement-2 is true
(A)  has no limit
(B)  Statement-1 is true, statement-2 is true; state-
(B)  is discontinuous
ment-2 is a correct explanation for Statement-1
(C)  is continuous but not differentiable
(C)  Statement-1 is true, statement-2 is true; state-
(D)  is differentiable
ment-2 is not a correct explanation for statement-1
39. The values of constants a and b so as to make the (D)  Statement-1 is true, statement-2 is false
⎧1 44. If f (x) = | x | + [x – 1], where [.] is greatest integer
⎪ , | x| ≥ 1
function f (x) = ⎨ | x | continuous as well function, then f (x) is:
⎪ax 2 + b, | x | < 1 (A)  continuous at x = 0 as well as at x = 1

(B)  continous at x = 0 but not at x = 1
as differentiable for all x, are
(C)  continuous at x = 1 but not at x = 0
-1 3 1 3
(A) a = , b = (B) a = , b = (D)  neither continuous at x = 0 nor at x = 1
2 2 2 2
45. Amongst the following functions, a function that is
-1 -3
(C) a = ,b= (D)  None of these differentiable at x = 0 is
2 2
(A)  cos (|x|) – |x|
40. If f (x) = [tan2x] (where [ . ] denotes the greatest integer (B)  cos (|x|) + |x|
function), then (C)  sin (|x|) + |x|
(D)  sin (|x|) – |x|
(A)  lim f ( x ) does not exist
x→0
46. Let f (x) = x2 – 8x + 12, x∈ [2, 6].
(B) f (x) is continuous at x = 0
(C) f (x) is non-differentiable at x = 0 Statement-1: f  ′(c) = 0 for some c ∈ (2, 6)
(D) f (0) = 1. Statement-2: f is continuous on [2, 6] and differentia-
ble on (2, 6) with f (2) = f (6)
41. The values of p and q for which the function (A)  Statement-1 is true, Statement-2 is true,
⎧ sin( p + 1) x + sin x Statement-2 is a correct explanantion for
⎪ , x<0 Statement-1
x
⎪⎪ (B)  Statement-1 is true, Statement-2 is true,
f (x) = ⎨q, x=0
Statement-2 is not a correct expalantion for
⎪ 2
⎪ x+x - x, x=0
Statement-1
⎪⎩ x 3/ 2 (C)  Statement-1 is true, Statement-2 is false
(D)  Statement-1 is false, Statement-2 is true
is continuous for all x in R, are

Objective_Maths_JEE Main 2017_Ch 12.indd 17 01/01/2008 05:25:38


12.18  Chapter 12

(A) f (x) is a quadratic function


⎧ ⎛ 1 ⎞
⎪( x - 1) sin ⎜ if x ≠ 1 (B) f (x) is continuous but not differentiable
47. Let f (x) = ⎨ ⎝ x - 1⎟⎠ . Then which (C) f (x) is differentiable in R
⎪ 0, if x = 1 (D) f (x) is bounded in R

of the following is true?
⎧ x, when x is rational
(A) f is neither differentiable at x = 0 nor at x = 1 53. If f (x) = ⎨ , then
(B) f is differentiable at x = 0 and at x = 1 ⎩1 - x, when x is irrational
(C) f is differentiable at x = 0 but not at x = 1 (A) f (x) is continuous for all real x
(D) f is differentiable at x = 1 but not at x = 0 (B) f (x) is discontinuous for all real x
(C) f (x) is continuous only at x = 1/2
48. Let f (x) = x | x | and g(x) = sin x. (D) f (x) is discontinuous only at x = 1/2.
Statement-1: gof is differentiable at x = 0 and its
derivative is continuous at that point. 54. The points where the function f (x) = [x] + | 1 – x |, – 1
Statement-2: gof is twice differentiable at x = 0. ≤ x ≤ 3, where [.] denotes the greatest integer function,
(A)  Statement-1 is true, Statement-2 is true; is not differentiable, are
Statement-2 is a correct explanation for (A) x = – 1, 0, 1, 2, 3 (B)  x = – 1, 0, 2
Statement-1 (C) x = 0, 1, 2, 3 (D)  x = – 1, 0, 1, 2
(B)  Statement-1 is true, Statement-2 is true; 55. Let a function f  : R → R satisfy the equation f (x + y) =
Statement-2 is not a correct explanation for f (x) + f (y) for all x, y. If the function f (x) is continuous
Statement-1 at x = 0, then
(C)  Statement-1 is true, Statement-2 is false
(A) f (x) = 0 continuous for all x
(D)  Statement-1 is false, Statement-2 is true
(B) f (x) is continuous for all positive real x
⎧ ⎛ 1 1⎞
2-⎜ + ⎟ (C) f (x) is continuous for all x
⎪ ⎝ | x| x ⎠
49. The function f (x) = ⎨( x + 1) , x ≠ 0 is (D)  None of these
⎪0 ,x=0 ⎛ 2 x - 1⎞
⎩ 56. The function f (x) = [x] cos ⎜ p , where [.]
(A)  discontinuous at only one point ⎝ 2 ⎟⎠
(B)  discontinuous exactly at two points denotes the greatest integer function, is discontinuous
(C)  continuous everywhere at
(D)  None of these (A) all x (B)  all integer points
⎧ e[ x ] + | x | - 2 (C) no x (D)  x which is not an integer.
⎪ , x≠0
50. f (x) = ⎨ [ x ] + | x | , ([ . ] denotes the greatest 57. The function f (x) = [x]2 – [x2] (where [ x] is the great-
⎪ -1 , x = 0 est integer less than or equal to x), is discontinuous at

integer function), then (A)  all integers
(A) f (x) is continuous at x = 0 (B)  all integers except 0 and 1
(B)  lim f ( x ) = – 1 (C)  all integers except 0
x → 0+ (D)  all integers except 1
(C)  lim f ( x ) = 1
x → 0- 1 1
58. The function f (x) = , where u = , is
(D)  None of these u +u-2 2 x -1
51. The Dirichlet function, defined as discontinuous at the points
⎧1 if x is rational 1 1
f (x) = ⎨ , is (A) x = – 2, 1, (B)  x = , 1, 2
⎩0 if x is irrational 2 2
(A)  continuous for all real x (C) x = 1, 0 (D)  None of these
(B)  continuous only at some values of x
(C)  discontinuous for all real x n

(D)  discontinuous only at some values of x 59. If f (x) = ∑ ak | x - 1|k , where ai∈R, then
k =0
52. Let f : R → R be a function such that (A) f (x) is continuous at x = 1 for all ak ∈R
⎛ x + y⎞ f ( x) + f ( y) (B) f (x) is differentiable at x = 1 for all ak ∈ R
f ⎜ = , f (0) = 0 and f  ′ (0) = 3.
⎝ 3 ⎟⎠ 3 (C) f (x) is differentiable at x = 1, provided a2k + 1 = 0
(D) f (x) is continuous at x = 1, provided a2k = 0
Then,

Objective_Maths_JEE Main 2017_Ch 12.indd 18 01/01/2008 05:25:41


Continuity and Differentiability  12.19

66. Let f (x) = [n + p sin x], x ∈ (0, π), n ∈ Z and p is a


⎛ p ⎞
60. If f (x) = [x] sin ⎜ , where [.] denotes the great- prime number, where [ · ] denotes the greatest integer
⎝ [ x + 1] ⎟⎠ function. Then, the number of points where f (x) is not
est integer function, then the points of discontinuity of differentiable, are
f in the domain are
(A) 0 (B) 2(p – 1)
(A) Z (B)  Z \{0} (C) 2p – 1 (D)  None of these
(C) R \[– 1, 0) (D)  None of these
67. The function y = f  (x), defined parametrically as
61. If f is differentiable function satisfying f (0) = 0 and if
x = 2t – | t – 1 | and y = 2t2 + t | t |, is
f ( x) (A)  continuous and differentiable for x ∈ R
g (x) = , then value, that should be assigned to
x (B) continuous for x ∈ R and differentiable for x ∈
g (0), so that g is continuous at ‘0’ is R – {2}.
(A) 1 (B) 0 (C) continuous for x ∈ R and differentiable for x ∈
(C) f (0) (D)  f ′(0) R – {–1, 2}
(D)  None of these
62. The value of f (0) so that the function
cos -1 (1 - {x}2 ) sin -1 (1 - {x}) 68. If f  (x) is a continuous function for all real values of x
f (x) =
, x ≠ 0 ( {x} satisfying x2 + (f (x) – 2)x + 2 3 - 3 - 3 f (x) = 0, ∀
{x} - {x}3
x ∈ R, then the value of f ( 3 ) is
denotes fractional part of x) becomes continuous at
x = 0 is (A)  3 (B)  1– 3
p p (C)  2(1 – 3 ) (D) 2( 3 – 1)
(A)  (B) 
2 4
69. The jump of the function at the point of discontinuity
p i.e., x = 1 of the function
(C)  (D)  None of these
2 log ( 2 + x ) - x 2 n sin x
f (x) = lim is
63. If the function f (x) defined as n →∞ 1 + x 2n
⎧ -p (A)  sin 1 – log 3 (B)  sin 1 + log 3
⎪(sin x + cos x )cosec x , <x<0 (C)  –sin 1 + log 3 (D)  None of these
⎪ 2
⎪ ⎧ x -1
f (x) = ⎨
a , x=0 , x ≠1
⎪ 1/x ⎪ 1
⎪ e +e +e
2/ x 3/ x
p 70. The function f (x) = ⎨ x -1
, 0< x< e +1
⎪⎩ ae -2 +1/x + be -1+ 3/x 2 ⎪
⎩ 0 , x =1
is continuous at x = 0, then (A)  is continuous
(A) a = e, b = 1 (B)  a = 1, b = e (B)  has removable discontinuity
1 (C)  has jump discontinuity
(C)  a = , b = 1 (D)  None of these (D)  has infinite discontinuity
e
(128a + ax )18/
-2 71. Let f: R → R be a real valued function such that
64. If the function f (x) = 1/5
is continuous | f (x) – f (y) | ≤ | x – y |2 ∀x, y ∈R. Then, the function
(32 + bx ) - 2
at x = 0, then the value of
a
is
h(x) = ∫ f ( x) dx is
b (A) continuous ∀x ∈R
3 (B)  discontinuous at x = 0 only
(A)  f (0) (B) 28/5f (0)
5 (C)  discontinuous at all integral points
(D) h(0) = 0
64
(C)  f (0) (D)  None of these
5 72. If f is a continuous function from R to R and f (f (a)) = a
for some a ∈R, then the equation f (x) = x has
65. Let f : R+→ R satisfies the equation f (xy) = exy – x – y (A)  no solution
{eyf (x) + exf (y)}, ∀x, y ∈ R+. If f  ′(1) = e, then f (x) = (B)  exactly one solution
(A) exlog | x | (B)  e–x log | x | (C)  at most one solution
(C) e2x log | x | (D)  None of these (D)  at least one solution

Objective_Maths_JEE Main 2017_Ch 12.indd 19 01/01/2008 05:25:43


12.20  Chapter 12

73. Let f be a continuous function on R such that (A) f (x) = 0 (B) 


f (x) < 0
n2 (C) f (x) > 0 (D) 
f (x) = x
( )
f (1/4n) = sin e n e - n + 2
2

. Then, the value of f (0) is


n +1 76. A function f  : R →R, where R is the set of real numbers
1 satisfies the equation
(A)  1 (B) 
2
⎛ x + y ⎞ f ( x ) + f ( y ) + f ( 0)
(C)  0 (D)  None of these f ⎜ =
⎝ 3 ⎟⎠ 3
74. Let f be a continuous and differentiable function in
for all x, y in R. If the function f is differentiable at x =
(a, b), lim f ( x ) → ∞ and lim f ( x ) → - ∞ . If f  ′(x) 0, then f is
x → a+ x → b-
2
(A) linear (B) quadratic
+ f   (x) ≥ –1 for a < x <b, then (C) cubic (D) biquadratic
(A) b – a ≤π (B)  b – a ³π
(C) b – a = π (D)  None of these
75. Let f be a differentiable function such that f (x + y) =
f (x) + f (y) + 2xy – 1 for all real x and y. If f  ′(0) = cos
α, then ∀x ∈R

More than One Option Correct type

⎧ p 1 ⎧ 1
⎪ x cos , x ≠ 0 ⎪ | x |≥1
77. If f (x) = ⎨ x , then at x = 0, f (x) is 81. Let f (x) = ⎨ | x | . If f is continuous and
⎪⎩ 0, x = 0 ⎪ax 2 + b | x | < 1

(A)  continuous if p > 0 differentiable at every point, then
(B)  differentiable if p> 1
(C)  continuous if p> 1 1 1
(A) a = a= -
(B) 
(D)  differentiable if p> 0 2 2
⎧ 1
⎪ x sin , x ≠ 0 3 -3
78. Let g (x) = x f (x), where f (x) = ⎨ x . (C) b = (D) b=
⎪⎩0, x=0 2 2
At x = 0, 82. Let [x] denotes the greatest integer less than or equal
(A) g is differentiable but g′ is not continuous to x. If f (x) = [x sin px], then f (x) is
(B)  g is differentiable while f is not (A)  continuous at x = 0
(C) both f and g are differentiable (B)  continuous in (– 1, 0)
(D) g is differentiable and g′ is continuous (C)  differentiable at x = 1
79. The function f (x) = max. {(1 – x), (1 + x), 2}, (D)  differentiable in (– 1, 1)
x ∈ (– ∞, ∞), is ⎧ x (3e1/ x + 4)
(A)  continuous at all points ⎪ ,x≠0
83. If f (x) = ⎨ 2 - e1/ x , then f (x) is
(B)  differentiable at all points ⎪
(C) differentiable at all points except at x = 1 and x ⎩ 0, x=0
= – 1. (A)  continuous at x = 0
(D) continuous at all points except at x = 1 and x = – 1, (B)  not continuous at x = 0
where it is discontinuous. (C)  differentiable at x = 0
(D)  not differentiable at x = 0
80. The function f (x) = (x), where (x) denotes the smallest
integer ≥ x, is 1
84. If f (x) = , then the points of discontinuity of the
(A)  continuous at integral points 1- x
(B)  continuous at non-integral points function f 3n(x), where f n = fof ... of (n times), are
(C)  discontinuous at integral points (A) x = 2 (B)  x = 0
(D)  discontinuous at non-integral points (C) x = 1 (D)  continuous everywhere

Objective_Maths_JEE Main 2017_Ch 12.indd 20 01/01/2008 05:25:45


Continuity and Differentiability  12.21

85. The function 91. Let f (x) = x3 – x2 + x + 1 and

⎪⎧1 - 2 x + 3 x 2 - 4 x 3 + ... to ∞ , x ≠ -1 ⎧ max . f (t ) 0 ≤ t ≤ x for 0 ≤ x ≤ 1


f (x) = ⎨ is g(x) = ⎨ . Then, in
⎩⎪ 1 , x = -1 ⎩ 3- x 1< x ≤ 2
the interval [0, 2], g(x) is
(A)  continuous at x = – 1
(B)  neither continuous nor derivable at x = – 1 (A)  continuous for all x
(C)  derivable at x = – 1 (B)  differentiable for all x
(D)  not derivable at x = – 1 (C)  discontinuous at x = 1
(D)  not differentiable at x = 1
86. The function F(x), defined as
92. Let f (x) = x4 – 8x3 + 22x2 – 24x and
2n
f ( x) + x g( x)
F(x) = lim
⎧ min . f (t ) x ≤ t ≤ x + 1, - 1 ≤ x ≤ 1
n →∞ 1 + x 2n g(x) = ⎨
⎩ x - 10 x >1
shall be continuous everywhere, if Then, in the interval [–1, ∞), g(x) is
(A) f (1) = g(1) (B)  f (–1) = g(–1) (A)  continuous for all x
(C) f (1) = –g(1) (D)  f (–1) = –g(1) (B)  discontinuous at x = 1
87. If the function f (x) defined as (C)  differentiable for all x
(D)  not differentiable at x = 1
⎧ 3 , x=0 p
⎪ 1/ x 93. If f (x) = [tan x] + tan x - [tan x ] , 0 ≤ x < , where
f (x) = ⎨⎛
ax + bx 3 ⎞ 2
⎪⎜ 1 + ⎟ ,x>0 [ · ] denotes the greatest integer function, then
⎩⎝ x2 ⎠
⎡ p⎞
(A) f (x) is continuous in ⎢0, ⎟
is continuous at x = 0, then ⎣ 2⎠
(A) a = 0 (B)  b = e3 (B) f (x) is not continuous at x = 0
(C) a = 1 (D)  b = ln 3 p
(C) f (x) is continuous at x = 0,
4
sin 3 x + a sin 2 x + b sin x (D) f (x) has infinite points of discontinuity
88. If the function f (x) = ,x≠0
x5 e1/x - e -1/x
is continuous at x = 0, then 94. Let f (x) = g′(x) , where g′ is the derivative
e1/x + e -1/x
(A) a = –4 (B)  b=5 of g and is a continuous function, then lim f ( x ) exists
(C) a = 4 (D)  f (0) = 1 if x →0

89. Let f  ′′(x) be continuous at x = 0. (A) g(x) is a polynomial


(B) g(x) = x
2 f ( x ) - 3a f ( 2 x ) + bf (8 x )
If lim exists and f (0) ≠ 0, (C) g(x) = x2
x→0 sin 2 x (D) g(x) = x3h(x), where h(x) is a polynomial
f  ′(0) ≠ 0, then
-7 1 95. If the function f (x), defined as
(A) a = (B) b= ⎧
9 3
⎪ a (1 - x sin x ) + b cos x + 5
7 1 ⎪ ,x<0
(C) a = (D) b = – x 2
9 3 ⎪⎪
f (x) = ⎨
3 ,x=0
⎧⎪ x-3 , x<0 ⎪ 1/ x
90. If f (x) = ⎨ 2 ⎪⎧⎪ ⎛ cx + dx 3 ⎞ ⎫⎪
⎩⎪ x - 3 x + 2, x ≥ 0 ⎪⎨1 + ⎜ 2 ⎟⎬ ,x>0
⎪⎩⎪⎩ ⎝ x ⎠ ⎪⎭
and g(x) = f (| x |) + | f (x) |, then g(x) is
is continuous at x = 0, then
(A)  continuous in R – {0}
(A) a = –1 (B)  b = –4
(B)  continuous in R
(C) c = 0 (D)  loge3
(C)  differentiable in R – {0, 1, 2}
(D)  differentiable in R – {1, 2}

Objective_Maths_JEE Main 2017_Ch 12.indd 21 01/01/2008 05:25:48


12.22  Chapter 12

Passage Based Questions


Passage 1 ⎧ x(e1/ x - e -1/ x )
Let f be a real valued function defined on an open internal ⎪ , x≠0
99. The function f (x) = ⎨ e1/ x + e -1/ x is
I ⊂ R. If x0 ∈I, then we define g with domain I – {x0} by set- ⎪
f ( x ) - f ( x0 ) ⎩0 ,x=0
ting g(x) = , for all x ∈ I – {x0}. If lim g ( x )
x - x0 x→ x
0
(A) continuous everywhere but not differentiable at
exists and is finite, we denote it by f  ′(x0) and say that f is x=0
derivable at x0. f  ′(x0) is called the derivative of f at x0. If (B)  continuous and differentiable everywhere
lim g ( x ) exists and is finite, we denote it by Rf  ′(x0) and (C)  not continuous at x = 0
x → x0+ (D)  None of these
say that f is derivable from right at x0. ∞
xn
If lim g ( x ) exists and is finite, we denote it by
x → x0-
100. If f (x) = ∑ n !
(log a)n, then at x = 0, f (x)
n=0
Lf  ′(x0) and say that f is derivable from left at x0.
(A)  has no limit
It is obvious that f is derivable at x0 iff Lf  ′(x0) and
(B)  is continuous
Rf  ′(x0) both exist and are equal. Also, if this condition be
(C)  is continuous but not differentiable
satisfied, then the common value is nothing else but f  ′(x0).
(D)  is differentiable
⎧3, x < 0 ⎧x
96. If f (x) = ⎨ , then
101. Let f (x) = ⎨∫
⎩2 x + 1, x ≥ 0 ⎪ (5 + | 1 - t |) dt , x > 2
0
, then at x = 2
(A) both f (x) and f (| x |) are differentiable at x = 0 ⎪
(B) f (x) is differentiable but f (| x |) is not differentiable ⎩ 5x + 1 , x≤2
at x = 0 (A) f (x) is continuous
(C) f (| x |) is differentiable but f (x) is not differentiable (B) f (x) is not continuous
at x = 0 (C) f (x) is differentiable
(D) both f (x) and f (| x |) are not differentiable at (D) f (x) is not differentiable
x = 0. Passage 3
97. Let f (x) = cos x and g (x) = [x + 2], where [.] denotes Let f be a function defined on an interval I. If f  be discon-
⎛p⎞ tinuous at a point p ∈ I, then we say that
the greatest integer function. Then, (gof )  ′ ⎜ ⎟ is
⎝ 2⎠ (i) f has a removable discontinuity at p if lim f ( x )
x→ p
(A) 1 (B) 0 exists but is not equal to f (p).
(C)  – 1 (D)  does not exist (ii) f   has a discontinuity of the first kind at p if lim f ( x )
and lim f ( x ) exist but are unequal. x → p - 0
98. The left-hand derivative of f (x) = [x] sin (π x) at x = k, x→ p + 0
k an integer and [x] = greatest integer ≤ x, is (iii) f  has a discontinuity of the second kind at p if neither
(A)  (– 1)k (k – 1) π (B)  (– 1)k – 1 ⋅ (k – 1) π of lim f ( x ) and lim f ( x ) exists.
x→ p - 0 x→ p + 0
k
(C)  (– 1) ⋅ kπ (D)  (– 1)k – 1 ⋅ kπ.
1
102. The function f (x) = are tan has
x-5
Passage 2
(A)  discontinuity of the first kind at x = 5
Let f be a real-valued function defined on an interval I. If (B)  discontinuity of the second kind at x = 5
f be derivable at a point x0 ∈ I, then it is continuous at x0. (C)  removable discontinuity at x = 5
The converse of the above statement does not hold. That (D)  continuous at x = 5.
is, a function may be continuous at a point but may fail
to be derivable at that point. Thus, derivability is a more 1 - u2
103. The function f (x) = , where u = tan x, has
restrictive property than continuity. In fact, there are func- 2 + u2
tions which are continuous everywhere but differentiable p
(A) discontinuity of the first kind at x = nπ ± ,
nowhere. n∈I 2
If Rf  ′(x0) and Lf  ′(x0) are finite (they may or may not p
(B) discontinuity of the second kind at x = nπ ± ,
be equal), then f (x) is continuous at x = x0. n∈I 2

Objective_Maths_JEE Main 2017_Ch 12.indd 22 01/01/2008 05:25:51


Continuity and Differentiability  12.23

(A)  discontinuity of first kind at x = 0


p
(C)  removable discontinuity at x = nπ ± ,n∈I (B)  removable discontinuity at x = 1
2 (C)  discontinuity of first kind at x = 1
p
(D)  continuous at x = nπ ± ,n∈I (D)  removable discontinuity at x = 0
2
1
104. The function f (x) = t3, where 105. Let f (x) = , where [ · ] denotes the greatest inte-
[cos x ]
⎧ x - 1, x≤0 p
⎪ x + 1, ger function. Then, the function f (x) has at x =
⎪ 0 < x <1 2
t = ⎨ (A)  removable discontinuity
⎪ 1, x =1 (B)  discontinuity of first kind from left
⎪⎩3 - x, 1< x (C)  discontinuity of second kind from left
has (D)  None of these

Match the Column Type


106.
Column-I Column-II
2
    I.  Let f and g be differentiable functions satisfying g′(a) = 2, (A) 
g(a) = b and fog = I (identity function). Then, f  ′(b) = 3

⎧ a/|sin x | p
⎪(1 + |sin x |) , - <x<0
6

⎪ tan 2 x/tan 3 x p 1
   II. Let f (x) = ⎨ e , 0< x< The value of (B) 
⎪ 6 12
⎪ e 2/3 , x=0


a so that f (x) may be continuous at x = 0 is
1
III.  The value of f (π/4) so that the function f (x) = (sin 2x)tan22x (C) 
is continuous at x = π/4 is 2

31+ x - 41+ x
 IV.  The value of f (0) so that the function f (x) = (D)  - 1
becomes continuous at x = 0, is x 2

107.
Column-I Column-II

⎛ 1⎞ ⎛ n ⎞
  I. If f (x) is continuous in [0, 1] and f ⎜ ⎟ = 2, then lim f⎜ ⎟ = (A) 6
⎝ 2⎠ n →∞ ⎝ 2 n + 1⎠
 II. If a function f, defined and continuous on [2, 5], takes rational values (B) 2
for all x and f (4) = 8, then f (3·7) =
III. The number of points in the interval (1, 2), where the function (C) 3
f (x) = [x3 – 3] ([ · ] denotes the greatest integer function) is discon-
tinuous, is
 IV. The number of points of discontinuity of the function f (x) = [3 + 2 cos (D) 8
⎛ p p⎞
x], x ∈ ⎜ - , ⎟ , where [ · ] denotes the greatest integer function, is
⎝ 2 2⎠

Objective_Maths_JEE Main 2017_Ch 12.indd 23 01/01/2008 05:25:52


12.24  Chapter 12

108.
Function Character of discontinuity
  I.  f (x) = |2 sin 2x | + 2 at x = 0 (A)  Oscillating discontinuity
⎧ px
⎪tan , x <1
 II.  f (x) = ⎨ 2 at x = 1 (B)  Infinite discontinuity
⎪⎩ x - 1, 1 ≤ x < 2

⎧ 1
⎪sin , x ≠ 0
III.  f (x) = ⎨ x at x = 0 (C)  Removable discontinuity
⎪⎩ 0 , x = 0

| x + 2|
 IV.  f (x) = at x = –2 (D)  Jump discontinuity
tan -1 ( x + 2)

109. The equationshave at least one root on the interval

Equations Interval
  I.  sin x – x + 1 = 0 (A)  (–2, 1/2)
2
 II.  x2/4 – sin px + =0 (B)  (0, 1)
3
2
III.  x3/4 – sin px + =0 (C)  (0, 3π/2)
3
 IV.  2x– 3x = 0 (D)  (–2, 2)

Assertion-Reason Type
Instructions: In the following questions an Assertion (A) is 112. Assertion: Let f: R → R be any function. Define
given followed by a Reason (R). Mark your responses from g: R → R by g(x) = | f (x) | for all x. Then, g is contin-
the following options: uous if f is continuous.
(A)  Assertion(A) is True and Reason(R) is True; Reason(R) Reason: Composition of two continuous functions is
is a correct explanation for Assertion(A) continuous
(B)  Assertion(A) is True, Reason(R) is True; Reason(R) is
not a correct explanation for Assertion(A) ⎛ 2x ⎞
113. Assertion: If f (x) = cos–1 then f (x) is dif-
⎜ ⎟,
(C)  Assertion(A) is True, Reason(R) is False ⎝ 1 + x2 ⎠
(D)  Assertion(A) is False, Reason(R) is True ferentiable everywhere
110. Assertion: Let f (x + y) = f (x) f (y) for all x, y, where ⎧ -2
f (0) ≠ 0. If f  ′(0) = 2, then f (x) = Ae2x, where A is a ⎪1 + x 2 , if | x | < 1

constant. Reason: f  ′(x) = ⎨
⎪ 2 , if | x | > 1
Reason: f  ′(x) = f (x) ⎪⎩1 + x 2
111. Assertion: Let f: R → R be a function defined by
f (x) = max. {x, x3}. Then, f (x) is not differentiable at n
[2rx ]
x = –1, 0, 1
114. Assertion: The function f (x) = lim
n →∞
∑ n2
,
⎧x , x ≤ -1 r =1
⎪ 3 where [ · ] denotes the greatest integer function, is
x , -1 < x ≤ 0
Reason: f (x) = ⎪⎨ continuous everywhere.
⎪x , 0 < x ≤ 1 Reason: f (x) = x, ∀ x
⎪ x3 , x > 1

Objective_Maths_JEE Main 2017_Ch 12.indd 24 01/01/2008 05:25:54


Continuity and Differentiability  12.25

115. Assertion: The function f (x) = 117. Assertion: Let f be a function such that f (xy) = f (x)·
cos p x - x 2 n sin ( x - 1) f (y), ∀ y ∈ R and f (1 + x) = 1 + x (1 + g(x)), where
lim is discontinuous at
n→∞ 1 + x 2 n +1 - x 2 n lim g ( x ) = 0, then
x →0
x=±1
⎧ cos p x 2
f ( x) 1 1 ⎛ 5⎞
, | x| < 1
⎪ 1+ x
∫ f ′( x) ⋅1 + x 2 dx =
2
log ⎜ ⎟
⎝ 2⎠
⎪ 1
⎪ - 1 + sin 2 , x = - 1 f ( x)
Reason: f (x) = ⎨
-1 , x =1 Reason: f  ′(x) =
⎪ x
⎪ - sin ( x - 1) 118. Assertion: The function y = f (x), defined parametri-
⎪ , | x |>1 cally as y = t2 + t | t |, x = 2t – | t |, t ∈ R, is continuous
⎩ x -1
for all real x.
116. Assertion: If f (x) = sgn (x) and g(x) = x (1 – x2), then ⎧⎪2 x 2 , x ≥ 0
Reason: f (x) = ⎨
fog (x) and gof (x) are continuous everywhere ⎪⎩ 0 , x < 0

⎧ -1, x ∈ ( -1, 0) ∪ (1, ∞)



Reason: fog (x) = ⎨ 0, x ∈{-1, 0, 1}
⎪ 1, x ∈ ( -∞, - 1) ∪ (0, 1)

and, gof (x) = 0, ∀ x ∈ R

Previous Year’s Questions

1 - tan x p ⎡ p⎤ ⎧ ⎛ 1 ⎞
119. Let f ( x ) = , x ≠ , x ∈ ⎢0, ⎥ . If f (x) is ⎪( x - 1) sin ⎜ if x ≠ 1
4x - p 4 ⎣ 2⎦ 1 22. Let f (x) = ⎨ ⎝ x - 1⎟⎠ Then which
⎡ p⎤ ⎡p ⎤ ⎪ 0, if x = 1
continuous in ⎢0, ⎥ , then f ⎢ ⎥ is [2004] ⎩
⎣ 2⎦ ⎣4⎦ one of the following is true? [2008]
1
(A)  1 (B)  (A) f is neither differentiable at x = 0 nor at x = 1
2 f is differentiable at x = 0 and at x = 1
(B) 
1
(C) - (D)  -1 f is differentiable at x = 0 but not at x = 1
(C) 
2 (D) f is differentiable at x = 1 but not at x = 0
120. Let f : R → R be a function defined by f (x) = min 123. Consider the following relations:
{x + 1, |x| + 1}. Then which of the following is true?
R = {(x, y)| x, y are real numbers and x = wy for some
 [2007]
rational number w};.
(A) f (x) ≥ 1 for all x ∈R
(B) f (x) is not differentiable at x = 1 ⎧⎛ m p ⎞ ⎫
⎪⎜ , ⎟ m, n, p and q∈ Ζ ⎪
(C) f (x) is differentiable everywhere S = ⎨⎝ p q ⎠ ⎬ Then [2010]
(D) f (x) is not differentiable at x = 0 ⎪ ⎪
⎩such that n, q ≠ 0 and qm = pn ⎭
121. The function f : R ~ {0} → R given by
(A) neither R nor S is an equivalence relation
1 2 (B) S is an equivalence relation but R is not an equiv-
f ( x) =
- 2x
x e -1 alence relation
can be made continuous at x = 0 by defining f (0) as (C) R and S both are equivalence relations
 [2007] (D) R is an equivalence relation but S is not an equiv-
(A) 2 (B)  -1 alence relation
(C) 0 (D) 1

Objective_Maths_JEE Main 2017_Ch 12.indd 25 01/01/2008 05:25:56


12.26  Chapter 12

124. The real values of p and q for which the function Statement 1: f ′( 4) = 0
⎧ sin( p + 1) x + sin x Statement 2: f is continuous in [2, 5], differentiable
⎪ ,x < 0 in (2, 5) and f (2) = f (5).
⎪ x
⎪ (A)  Statement 1 is false, statement 2 is true
f ( x ) = ⎨q, x = 0 is continuous for (B) Statement 1 is true, statement 2 is true; statement
⎪ 2
⎪ x+ x - x ,x > 0 2 is a correct explanation for statement 1
⎪⎩ x 3/ 2 (C) Statement 1 is true, statement 2 is true; statement
2 is not a correct explanation for statement 1
all x in R, is [2011] (D)  Statement 1 is true, statement 2 is false
5 1 3 1 ⎪⎧k x + 1, 0 ≤ x ≤ 3
(A) p = ,q = (B)  p = - , q = 127. If the function g ( x ) = ⎨ is differ-
2 2 2 2
⎩⎪ mx + 2, 3 < x ≤ 5
1 3 1 3 entiable, then the value of k + m is: [2015]
(C) p = , q = (D)  p = ,q = -
2 2 2 2
16 10
(A)  (B) 
125. If f : R → R is a function defined by 5 3
⎛ 2 x - 1⎞ (C) 4 (D) 2
f ( x ) = [ x ]cos ⎜ p , where [x] denotes the
⎝ 2 ⎟⎠
128. For x ∈ R, f (x) = |log2 – sinx| and g (x) = f (f (x)), then:
greatest integer function, then f is [2012]  [2016]
(A)  continuous for every real x (A) g is differentiable at x = 0 and g′(0) = – sin(log2)
(B)  discontinuous only at x = 0 (B) g is not differentiable at x = 0
(C)  discontinuous only at non-zero integral values of x (C) g′ (0) = cos (log2)
(D)  continuous only at x = 0 (D) g′ (0) = – cos (log2)
126. Consider the function f ( x ) = x - 2 + x - 5 , x ∈ R.
 [2012]

Answer keys
Single Option Correct Type
1. (C) 2.  (D) 3. (B) 4. (B) 5. (C)
6.  (A) 7. (B) 8.  (a, b, d) 9. (D) 10.  (C)
11. (A) 12.  (B) 13. (B) 14.  (A) 15. (C)
16.  (B, C) 17. (A) 18.  (A, C) 19.  (A, B) 20.  (A, B)
21. (D) 22.  (B) 23. (B) 24.  (C) 25. (A)
26.  (B) 27. (C) 28. (B) 29. (C) 30.  (A)
31. (B) 32.  (D) 33. (A) 34.  (B) 35. (B)
36. (B) 37. (C) 38.  (D) 39. (A) 40. (B)
41. (D) 42. (A) 43.  (C) 44. (D) 45. (D)
46. (A) 47.  (A) 48. (C) 49. (A) 50.  (D)
51.  (C) 52. (C) 53. (C) 54. (C) 55. (C)
56. (A) 57. (D) 58.  (B) 59. (C) 60.  (B)
61. (D) 62.  (D) 63. (A) 64.  (C) 65. (A)
66.  (C) 67. (B) 68.  (C) 69. (B) 70.  (A)
71. (A) 72.  (D) 73. (A) 74.  (B) 75. (C)
76.  (A)

Objective_Maths_JEE Main 2017_Ch 12.indd 26 01/01/2008 05:25:58


Continuity and Differentiability  12.27

More than One Option Correct Type


77.  (a, b) 78.  (a, b) 79.  (a, c) 80.  (b, c) 81.  (b, c)
82.  (a, b, d) 83.  (a, d) 84.  (b, c) 85.  (b, d) 86.  (a, b)
87.  (a, d) 88.  (a, b, d) 89.  (b, c) 90.  (a, c) 91.  (a, d)
92.  (a, d) 93.  (a, c) 94.  (c, d) 95.  (a, b, c, d)

Passage Based Questions


 96. (C) 97. (D) 98.  (A) 99. (A) 100.  (b, d) 101.  (a, d) 102.  (A)
1 03. (c) 104.  (a, b) 105. (c)

Match the Column Type


106.  I → (c), II → (a), III → (d), IV → (b), 107. I → (b), II → (d), III → (a), IV → (c)
1 08.  I → (c), II → (b), III → (a), IV → (d) 109. I → (c), II → (a), III → (d), IV → (b)

Assertion-Reason Type
110. (C) 111. (A) 112.  (A) 113. (D) 114.  (A)
115. (A) 116. (D) 117. (a) 118. (a)

Previous Year’s Questions


119. (C) 120. (C) 121. (D) 122. (A) 123. (A) 124. (B) 125. (A) 126. (B) 127. (D) 128. (C)

Objective_Maths_JEE Main 2017_Ch 12.indd 27 01/01/2008 05:25:58


12.28  Chapter 12

Hints and Solutions

Single Option Correct Type


1. We have, 3. The given function is clearly continuous at all points except
f ( 0 - h) - f ( 0 ) 3 -1 possibly at x = ± 1.
L f  ′(0) = lim = lim →–∞ For f (x) to be continuous at x = 1, we must have
h→0 -h h→0 - h

∴ f (x) is not differentiable at x = 0


lim f ( x ) = lim f ( x ) = f (1)
x → 1- x → 1+
Also, if x < 0 or x ≥ 0 then | x | ≥ 0
∴ f ( | x | ) = 2 | x | + 1 for all x. 1
⇒ lim ax 2 + b = lim
x →1 x →1| x|
f ( 0 + h) - f ( 0 )
∴ R f  ′(0) = lim
h→0 h ⇒ a + b = 1 (1)
2h + 1 - 1 Now, for f (x) to be differentiable at x = 1, we must have


= lim =2
h→0 h
f ( x ) - f (1) f ( x ) - f (1)
f ( 0 - h) - f ( 0 )
lim = lim
and L f  ′(0) = lim
x → 1- x -1 x → 1+ x -1
h→0 -h
1
2 ( - h) + 1 - 1 -1

= lim =2 ax 2 + b - 1 |x|
h→0 -h ⇒ lim = lim
x →1 x -1 x →1 x - 1

∴ f (| x |) is differentiable at x = 0. 1
The correct option is (C) -1
ax 2 - a |x|
⇒ lim = lim
⎛p⎞ x →1 x - 1 x → 1- x - 1
2. L (gof ) ′ ⎜ ⎟
⎝ 2⎠
⎛p ⎞ ⎛p⎞  (∵ a + b = 1 ∴ b – 1 = – a)
( gof ) ⎜ - h⎟ - ( gof ) ⎜ ⎟
⎝ 2 ⎠ ⎝ 2⎠ -1

= hlim
→0 ⇒ lim a ( x + 1) = lim ⇒ 2a = – 1
-h x →1 x →1 x
⎡ ⎛p ⎞ ⎤ ⎡ p ⎤ 1
⎢cos ⎜⎝ 2 - h⎟⎠ + 2⎥ - ⎢cos 2 + 2⎥ ⇒
a= -
⎣ ⎦ ⎣ ⎦ 2

= lim
h→0
-h 1 3
Putting a = -
in (1), we get b = .
2 2
(sin h + 2) - ( 2) 2-2
The correct option is (B)

= lim = lim =0
h→0 -h h→0 -h
4. We have,
⎛p ⎞ ⎛p⎞
( gof ) ⎜ + h⎟ - ( gof ) ⎜ ⎟ f ( x + h) - f ( x )
⎛p⎞ ⎝2 ⎠ ⎝ 2⎠ f  ′(x) = lim
R( gof ) ′ ⎜ ⎟ = lim h→0 h
⎝ 2⎠ h→0 h
f ( x ) ⋅ f ( h) - f ( x )

= lim
⎡ ⎛p ⎞ ⎤ ⎡ p ⎤ h→0h
⎢cos ⎜⎝ 2 + h⎟⎠ + 2⎥ - ⎢cos 2 + 2⎥
⎣ ⎦ [Using f (x + y) = f (x) ⋅ f ( y)]

= lim ⎣ ⎦
h→0 h f ( h) - 1

= f (x) lim
h→0 h
( - sin h + 2) - ( 2)

= lim 1 + h f ( h) + h2 f ( h) ψ ( h) - 1
h→0 h
= f (x) lim
h→0 h
1- 2
= f (x) lim [f (h) + h f (h) ψ (h)]

= lim →–∞ h→0
h→0 h

= f (x) (a + 0 ⋅ a⋅ b) = a ⋅ f (x).
∴ (gof ) is not differentiable at x = π/2.

The correct option is (B)
The correct option is (D)

Objective_Maths_JEE Main 2017_Ch 12.indd 28 01/01/2008 05:26:06


Continuity and Differentiability  12.29

5. For n ∈ I,
It follows that f (x) = 0 if 0 ≤ x <
2.
2x - 1

lim f ( x ) = lim [ x ] cos p This shows that f (x) must be continuous at x = 1.

x→n +
x→n +
2

However, at points x other than integers and not lying
2n - 1 between 0 and 2 , f (x) ≠ 0.

= n cos p =0
2
Thus, f is discontinuous at all integers except 1.
2x - 1
The correct option is (D)

and lim f ( x ) = lim [ x ] cos p
x→n -
x→n -
2
10. Let h (x) = | x | for all x. Clearly, h (x) is continuous for all x.
2n - 1

= ( n - 1)cos p = 0. Then g (x) = | f (x) | = h[ f (x) ] = (hof ) (x) for all x.
2
Since composition of two continuous functions is continu-
Hence, f is continuous for x = n ∈ I. Since the functions
ous, therefore, g is continuous if f is continuous.
2x - 1
g (x) = [x] and h (x) = cos p are continuous on The correct option is (C)
2
x ∈ R – I, so f is continuous everywhere. 11. Since f  ′ (0) exists,

The correct option is (C) ∴ R f  ′ (0) = L f ′(0)

[k - h]sin p ( k - h) - [k ]sin p k f ( 0 + h) - f ( 0 ) f ( 0 - h) - f ( 0 )
6. f ′ (k – 0) = lim ⇒ lim = lim
h→0 -h h→0 h h→0 -h

k -1 f ( - h) - f (0)

= lim ( -1) ( k - 1)sin p h - k × 0
= – lim
h→0 -h h→0 h
k -1 f ( h) - f ( 0 )

= lim ( -1) ⋅ ( k - 1)sin p h
= – lim
h→0 -h h→0 h

= (– 1)k ⋅ (k – 1) π. [Qf (– h) = f (h)]


The correct option is (A) f ( h) - f ( 0 )
⇒ 2 lim = 0 ⇒ 2 f  ′(0) = 0
h→0 h
7. lim f (x) ⋅ g (x) = lim f (x) ⋅ lim g (x) = ml [Qf  ′(0) exists]
x→a- x→a- x→a-
⇒ f  ′(0) = 0
and lim f (x) ⋅ g (x) = lim f (x) ⋅ lim
g (x) = lm The correct option is (A)
x→a+ x→a+ x→a+

12. We have,
∴ lim f (x) ⋅ g (x) = lm
x→a f ( 0 - h) - f ( 0 )
L f ′(0) = lim

The correct option is (B)
h→0 -h

⎡ - h (3e -1/ h + 4) ⎤ ⎛ -1⎞


8. We have lim f (0 – h) = lim [– h sin (– p h)]
= lim ⎢ - 0⎥ ⋅ ⎜ ⎟
⎦⎥ ⎝ h ⎠
- 1/ h
⎣ 2-e
h→0 h→0 h→0 ⎢


= lim [h sin π h] = 0,
h→0 0+4

= =2
lim f (0 + h) = lim [h sin p h] = 0. and f (0) = 0 2-0
h→0 h→0
f ( 0 + h) - f ( 0 )
∴ f (x) is continuous at x = 0. R f  ′(0) = lim
h→0 h
It can be easily seen that f (x) is continuous in (– 1, 0). f (x)
is not differentiable at x = 1 but it is differentiable in (– 1, 1). ⎡ h (3e1/ h + 4) ⎤1

= lim ⎢ 1/ h
- 0⎥
The correct option is (A), (B), and (D.) ⎣ 2-e
h→0 ⎢ ⎥⎦ h
9. Note that f (x) = 0 for each integral value of x. ⎛ 3 + 4e -1/ h ⎞ 3+0
2
= lim ⎜ -1/ h ⎟ = 0 -1 = – 3
Also, if 0 ≤ x < 1, then 0 ≤ x < 1
h → 0 ⎝ 2e - 1⎠
∴ [x] = 0 and [x2] = 0 ⇒ f (x) = 0 for 0 ≤ x < 1
Since L f ′(0) ≠ R f ′(0), \f ′(x) is not differentiable at

Next, if 1 ≤ x <
2 , then x = 0. But f (x) is continuous at x = 0 (as L f ′(0) and R f ′(0)
1 ≤ x2< 2 ⇒ [x] = 1 and [x2] = 1
are finite).

The correct option is (B)
Thus, f (x) = [x]2 – [x2] = 0 if 1 ≤ x < 2 .

Objective_Maths_JEE Main 2017_Ch 12.indd 29 01/01/2008 05:26:11


12.30  Chapter 12

17. We have,
1 1
13. The function u = suffers a discontinuity at the point
x = 1. x -1
lim f (5 – h) = lim tan– 1 (5 - h) - 5
h→0 h→0

The function f (x) =



1
suffers a discontinuity at the ⎛ -1⎞
u2 + u - 2
= lim tan– 1 ⎜⎝ h ⎟⎠
h→0
points where u2 + u – 2 = 0 i.e., u = – 2 and u = 1. Using these -p
values of u, the corresponding values of x are obtained by
= tan– 1 (– ∞) =
2
solving the equations
1
1 1 and lim f (5 + h) = lim tan– 1 (5 + h) - 5


–2= and 1 = i.e. x = 1/2 and x = 2. h→0 h→0
x -1 x -1
⎛ 1⎞

Hence, the composite function is discontinuous at three
= lim tan– 1 ⎜⎝ h ⎟⎠
points x = 1/2, x = 1 and x = 2. h→0
p

The correct option is (B)
= tan– 1 (∞) =
2
⎛ -p p ⎞
14. 3 ≤ 3 + 2 cos x ≤ 5 for x ∈ ⎜ , Since lim f (5 – h) ≠ lim f (5 + h), therefore, f (x) has

⎝ 2 2 ⎟⎠ h→0 h→0
discontinuity of the first kind at x = 5.
f (x) = [3 + 2 cos x] is discontinuous at those points where
3 + 2 cos x is an integer.
The correct option is (A)
-p p
Now, 3 + 2 cos x = 3 if cos x = 0. So, x = , 18. Since | x – 1 |, | x – 1 |2, etc, are continuous at x = 1 \ f (x) is
2 2
continuous at x =1 for all ak ∈ R.
 (not possible)
1 Also, | x – 1 |2, | x – 1 |4, etc, are all differentiable at x = 1,
3 + 2 cos x = 4 if cos x =
2 whereas | x – 1 |, | x – 1 |3, etc, are not differentiable at x = 1.
p -p Therefore, f (x) is differentiable at x = 1 for all a2k + 1 = 0.
So x has two values and .
3 3 The correct option is (A) and (C)
3 + 2 cos x = 5 if cos x = 1. So, x = 0.
19. f  ′(x) = -1 d ⎛ 2x ⎞
\The number of values of x = 2 + 1 = 3. ×

⎛ 2x ⎞
2 dx ⎜⎝ 1 + x 2 ⎟⎠
The correct option is (A) 1- ⎜
⎝ 1 + x 2 ⎟⎠
15. The function f (x) is continuous at all points where
1 1 - (1 + x 2 ) 2 (1 - x 2 )

– cos2x ≥ 0 ⇒ | cos x | ≤
= ×
2 2 (1 + x 2 ) 2 - 4 x 2 (1 + x 2 ) 2

p 3p ⎧ -2

+ 2nπ ≤ x ≤ + 2nπ 2⎪1 + x 2 , if | x | < 1
4 4 - 2 1- x ⎪

= ⋅ = ⎨
1 + x 2 |1 - x 2 | ⎪ 2 , if | x | > 1
5p 7p

or + 2nπ ≤ x ≤ + 2nπ, n ∈ I. ⎪⎩1 + x 2
4 4
The correct option is (C)
Clearly, f (x) is differentiable everywhere except at the points
where | x | = 1 i.e. x = ± 1.
16. Clearly, x = 1 is a point of discontinuity of the function
1 Hence, f (x) is differentiable on (– ∞, ∞) \ { – 1, 1}.

f (x) = .
The correct option is (C)
1- x
⎛ 1 ⎞ x -1
If x ≠ 1, then ( fof ) (x) = f [ f (x)] = f ⎜ ⎟⎠ = x , which 20. The function ⎡⎣ x ⎤⎦ has discontinuity at every x = n2 and x
is discontinuous at x = 0. ⎝ 1 - x is not defined on (–∞, 0). Hence the set of discontinuities of
If x ≠ 0 and x ≠ 1, then
f (x) is (–∞, 0) ∪ {n2 : n ∈N}. This clearly contains the sets
in (a) and (b).
⎛ x - 1⎞

(  fo fof ) (x) = f [( fof ) (x)] = f ⎜ = x, ∴ (a) and (b) are the correct answers.
⎝ x ⎟⎠
The correct option is (A, B)

which is continuous everywhere.
Hence, f  3n (x) = ( fofof )n(x) = x, which is continuous
21. We have
everywhere. lim f (3 – h) = lim | 3 – (3 – h) | + (3 + 3 – h)

h→0 h→0
So, the only points of discontinuity are x = 0 and x = 1.


The correct option is (B) and (C)
= lim (h + 6) = 6,
h→0

Objective_Maths_JEE Main 2017_Ch 12.indd 30 01/01/2008 05:26:16


Continuity and Differentiability  12.31

27. Since [x] and e| x | are not differentiable at x = 0, therefore, for



lim f (3 + h) = lim | 3 – (3 + h) | + (3 + 3 + h) f (x) to be differentiable at x = 0, we must have a = 0, b = 0
h→0 h→0
and c can be any real number.

= lim (h + 7) = 7 The correct option is (C)
h→0
28. [x + 1] = 0 if 0 ≤ x + 1 < 1 i.e. – 1 ≤ x < 0.
Since lim f (3 – h) ≠ lim f (3 + h), therefore, f (x) is not

h→0 h→0 Thus domain of f = R\[– 1, 0).
continuous and hence not differentiable at x = 3.
⎛ p ⎞
The correct option is (D)
We have, sin ⎜ continuous at all points of R
⎝ [ x + 1] ⎟⎠
22. Since f (x) is continuous at x = π
\[– 1, 0) and [x] continuous on R \ Z, where Z denotes the
∴ lim f (x) = f (π) ⇒ lim f (π – h) = k set of integers. Thus the points where f can possibly be dis-
x→p h→0
continuous are ..., – 3, – 2, – 1, 0, 1, 2, ...
1 + cos (p - h)
⇒ k = lim ⎛ p ⎞
h→0 (p - p + h) 2 For 0 ≤ x < 1, [x] = 0 and sin ⎜
is defined.
⎝ [ x + 1] ⎟⎠
sin 2 (p - h) ∴ f (x) = 0 for 0 ≤ x < 1.

 ·
log [1 + p - 2p (p - h) + (p - h) 2 ]
2
Also, f is not defined on [– 1, 0), so the continuity of f at 0
means continuity of f from right at 0. Since f is continuous
1 - cos h sin 2 h

= lim 2
⋅ from right at 0, so f is continuous at 0. Hence the set of points
h→0 h log (1 + h2 ) of discontinuity of f is Z \ {0}.
1 ⎛ sin h / 2 ⎞ h2
2
⎛ sin h ⎞
2 The correct option is (B)

= lim ⋅ ⎜ ⎟ ⋅ ⋅⎜ ⎟
h→0 2 ⎝ h / 2 ⎠ log (1 + h2 ) ⎝ h ⎠ f ( x + h) - f ( x )
29. f  ′(x) = lim
h→0 h
1

= . \k = 1/2
2 f ( x ) + f ( h) - f ( x )

= lim
The correct option is (B) h
h→0

23. A continuous function f (x) defined on 1 ≤ x ≤ 3 having only [Qf (x + y) = f (x) + f ( y)]

rational values must be a constant function. 3

= lim h f ( h) [Qf (x) = x3f (x)]
∴ f (2) = 10 ⇒ f (x) = 10, for all x h→0 h
∴ f (1.5) = 10
= h2f (h) = 0 × f (0)
The correct option is (B)
[f is continuous at x = 0, ∴ lim f (h) = f(0)]
x h→0
1 1 =0
24. We have, f (x) = ∫ t cos dt ⇒ f  ′(x) = x cos .
0
t x The correct option is (C)
Clearly f  ′(x) exists and is finite in the interval (0, π). 30. For f (x) to be continuous at x = 0, we must have
Therefore, f (x) is differentiable in the interval (0, π).
31+ x - 41+ x
Hence, f (x) is continuous in the interval (0, π). f (0) = lim
x→0 x
The correct option is (C)
25. Let x3 = n, n∈ I ⇒ x = n1/3 ⎡ 1 (1 /3)( -2 /3) 2 ⎤
∴ f (x) = (– 1)n = ± 1 =
⎢1 + 3 x + x + ...⎥
lim ⎣ 2! ⎦
Hence, f (x) is discontinuous for x = n1/3, n ∈ I. x→0 x
The correct option is (A) ⎡ 1 (1 /4) ( -3 /4) 2 ⎤
- ⎢1 + x + x + ...⎥
⎡ 1 ⎤ ⎣ 4 2! ⎦
26. We have, f (x) = ⎢ (cos x + sin x )⎥
⎣ 2 ⎦  x

⎡ ⎛ p⎞⎤ ⎡ 1 ⎛ -1 3 ⎞ ⎤

= ⎢cos ⎜ x - ⎟ ⎥ ⎢ +⎜ + ⎟ x + terms containing x 2 and ⎥
⎣ ⎝ 4⎠⎦ x ⎢12 ⎝ 9 32 ⎠ ⎥

Clearly, f (x) is discontinuous at all those points where cos ⎢⎣ higher powers ⎥⎦
 = lim
⎛ p⎞ p p 3p x→0 x
⎜⎝ x - ⎟⎠ is an integer i.e. x – = 0, , p ,
4 4 2 2
1 1
p 3p 5p 7p = \ f (0) =

i.e., x= , , and . 12 12
4 4 4 4

The correct option is (B)
The correct option is (A)

Objective_Maths_JEE Main 2017_Ch 12.indd 31 01/01/2008 05:26:21


12.32  Chapter 12

36. Continuity at x = 2 :
f ( h) - f ( 0 )
31. Let f  ′(0 + ) = lim = k (say) L.H.L. = lim f (2 – h) = lim [(2 – h)2] + [– 2 + h]2
h→0 h h→0 h→0

f ( 0 ) - f ( 0 - h)
= lim {3 + (– 2)2} = 7.
∴ f   ′(0 – ) = lim h→0
h→0 h
f ( 0 ) - f ( h) R.H.L. = lim f (2 + h) = lim [(2 + h)2] + [– 2 – h]2

h→0 h→0

= lim =–k
h→0 h 2

= lim {4 + (– 3) } = 13.
∴ f  ′(0+) ¹f  ′(0–), but both are finite
h→0

so f  ′ (x) is continuous at x = 0 but not differentiable at


Since L.H.L. ≠ R.H.L., \f (x) is not continuous at x = 2.

x = 0. Differentiability at x = 2 :


The correct option is (B)
f ( 2 - h) - f ( 2) 7-8
f ( x) LHD = lim
= lim → ∞.
32. We have, f (0) = 0, g (x) = , h→0 ( 2 - h) - 2 h → 0 -h
x So, f (x) is not differentiable at x = 2.
f ( x) Hence, the function f (x) is neither continuous nor deriv-
lim g ( x ) = lim = xlim
→0
f’( x ) = f  ′(0)
x→0 x→0 x able at x = 2.

Thus the required value is f  ′(0). The correct option is (B)

The correct option is (D) 37. Since the function f (x) is continuous at x = 0, therefore,
1
33. Since f  (x) = , ∴ sin x∉ [0, 1) lim f (0 – h) = f (0) = lim f (0 + h)
[sin x ] h→0 h→0
sin 3h
⇒ x∉ [2nπ, (2n + 1) p] – (4n + 1) π/2, n∈I. a
⇒ lim = b = lim e sin 2 h
∴ f (x) is not continuous if x∈ (2nπ, 2nπ + π), n∈I. h→0 (1 - |tan h |) |tan h |
h→0
The correct option is (A) sin 3h / 3h 3
34. For x ≠ – 1, we have ⇒ lim [(1 – | tan h | )– 1/|tan h |]– a = b = lim ⋅
h→0 h→0 e sin 2 h / 2 h2

f (x) = 1 – 2x + 3x2 – 4x3 + ...∞ -3


⇒ e– a = b = e3/2 ⇒ a = and b = e3/2.
1 2
= (1 + x)– 1 = .
1+ x
The correct option is (C)
1 38. We have
lim f (– 1 – h) = lim
→–∞
h→0 1-1- h h→0 ∞ ∞
xn ( x log a) n
So, f (x) is not continuous at x = – 1.
f (x) = ∑ n! (log a)n = ∑ n!
-1 n=0 n=0
-1
f ( - 1 - h) - f ( -1)

Also, lim = lim h
= ex log a= e log a = ax.
x

h→0 ( -1 - h) - ( -1) h→0 -h


f ( 0 - h) - f ( 0 ) a- h - 1
1+ h L f  ′(0) = lim = lim

= lim → ∞. h→0 -h h→0 -h
h→0 h2

= logea
So, f (x) is not derivable at x = – 1.
f ( 0 + h) - f ( 0 ) h
Hence, f (x) is neither continuous nor derivable at x = – 1. R f  ′(0) = lim
= lim a - 1
The correct option is (B)
h→0 h h→0 h
35. For x≠ 0, we have = logea
x /1+ x x /1+ x Since L f  ′(0) = R f  ′(0),
f (x) = x + =x+ = x + 1.
1 x /1+ x ∴ f (x) is differentiable at x = 0.
1-
1+ x Since every differentiable function is continuous, therefore,

For x = 0, f (x) = 0 f (x) is continuous at x = 0.
⎧ x + 1, x ≠ 0 The correct option is (D)

Thus, f (x) = ⎨ 39. Since f (x) is continuous for all x, therefore, it is continuous
⎩ 0 ,x=0
at x = 1 also.

Clearly, lim f (x) = lim f (x) = 1 ≠ f (0).
x → 0- x → 0+ ∴ f (1) = lim f (1 – h) ⇒ 1 = lim [a (1 – h)2 + b]
h→0 h→0

So, f (x) is discontinuous and hence not differentiable at
x = 0. ⇒ a + b = 1
(1)

The correct option is (B) Also, f (x) is differentiable at x = 1

Objective_Maths_JEE Main 2017_Ch 12.indd 32 01/01/2008 05:26:28


Continuity and Differentiability  12.33

f (1 - h) - f (1) f (1 + h) - f (1)
⇒ lim = lim
h→0 -h h→0 h

1
a (1 - h) 2 + b - 1 -1
⇒ lim = lim |1 + h |
h→0 -h h→0
h
2 5
( a + b - 1) + ( h - 2h) a2 1-1- h
The correct option is (C)
⇒ lim = lim
h→0 -h h → 0 h (1 + h)
44. At x = 0 and 1, f  (x) = |x| + [x – 1]
= continuous + discontinuous
⇒ 2a = – 1 (Using a + b = 1)
= discontinuous
-1 The correct option is (D)
∴ a=
2
45. RHD of sin(| x |) – | x | at x = 0 is 1 – 1 = 0
3
Hence, a + b = 1 ⇒ b = 1 – a =
. LHD of sin(| x |) – | x | at x = 0 is (–1) – (–1) = 0,
2

The correct option is (A) so differentiable at x = 0
The correct option is (D)
⎛ -p p ⎞
40. If x ∈⎜ , then f (x) = [tan2x] = 0 which is continuous
⎝ 4 4 ⎟⎠ 46. y
and differentiable at x = 0.


The correct option is (B) x
0 2 6
41. The given function f is continuous at x = 0 if lim f (0 - h)
h→0
= f (0) = hlim f ( 0 + h)
Statement-2 is true, all conditions of Rolle’s theorem are
→0
­satisfied so f ′(c) = 0 for some c ∈ (2, 6).
1
The correct option is (A)
⇒ p + 2 = q =
2
f (1 + h) - f (1)
47. f ′(1) = lim
3 1 h→0 h
⇒ p=– ,q=
2 2 ⎛ 1 ⎞
(1 + h - 1)sin ⎜ -0

The correct option is (D) ⇒ f ′(1) = ⎝ 1 + h - 1⎟⎠
lim
42. Doubtful points are x = n, n ∈I h→0 h

2 x - 1⎞ h ⎛ 1⎞
L.H.L = lim [ x ] cos ⎛⎜
p

= lim sin ⎜ ⎟
x→n ⎝ 2 ⎟⎠
- h→0 h ⎝ h⎠

⇒ f ′(1) = lim sin ⎛⎜ ⎞⎟


1
2n - 1⎞

= (n – 1)cos ⎛⎜ p=0 ⎝ h⎠
⎝ 2 ⎟⎠
h→0

∴ f is not differentiable at x = 1.
2n - 1⎞
R.H.L = lim [ x ] cos ⎛⎜
p f ( h) - f ( 0 )
x→n ⎝ 2 ⎟⎠
+ Similarly, f ′(0) = lim

h→0 h
2n - 1⎞ ⎛ 1 ⎞

= n cos ⎛⎜ p =0 ( h - 1)sin ⎜ - sin (1)
⎝ 2 ⎟⎠ ⇒ f ′(0) = ⎝ h - 1⎟⎠
lim
f (n) = 0
h→0 h

Hence continuous for all real x. ⇒ f is also not differentiable at x = 0.

The correct option is (A) The correct option is (A)

43. f (x) = 3 2 ≤ x ≤ 5 48 . f (x) = x | x | and g(x) = sin x


⎪⎧ - sin x , x < 0
2
f ′(x) = 0 2 < x < 5
gof (x) = sin(x | x |) = ⎨

f ′(4) = 0 2
⎩⎪ sin x , x ≥ 0

Objective_Maths_JEE Main 2017_Ch 12.indd 33 01/01/2008 05:26:32


12.34  Chapter 12

Thus, in both cases, the difference ∆ y does not tend to zero



⎧⎪ -2 x cos x 2 , x < 0
∴ (gof )′ (x) = ⎨ as ∆ x→ 0. Therefore, x0 is a point of discontinuity. Since x0
2
⎩⎪ 2 x cos x , x ≥ 0 is an arbitrary point, the Dirichlet function f (x) is discontin-
uous at each point.
Clearly, L(gof)′(0) = 0 = R(gof)′(0)


The correct option is (C)
∴ gof is differentiable at x = 0 and also its derivative is con-
tinuous at x = 0 52. We have,
⎧⎪ -2 cos x 2 + 4 x 2 sin x 2 , x < 0 ⎛ x + y⎞ f ( x) + f ( y)
Now, (gof)″ (x) = ⎨
f⎜ = , f (0) = 0 and f  ′(0) = 3
2 2 2 ⎝ 3 ⎟⎠ 3
⎩⎪ 2 cos x - 4 x sin x , x ≥ 0
∴ L(gof)″ (0) = –2 and R(gof)″(0) = 2 f ( x + h) - f ( x )
f  ′(x) = lim
∴ L(gof)″ (0) ≠ R(gof)″(0) h→0 h
∴ gof(x) is not twice differentiable at x = 0. ⎛ 3 x + 3h ⎞
f⎜ - f ( x)
The correct option is (C)
= ⎝ 3 ⎟⎠
lim
49. The only doubtful point is x = 0. h→0 h
⎛1 1⎞ f (3 x ) + f (3h) f (3 x ) + f (0)
L.H.L. = lim f (0 – h) = lim
2-⎜ -
⎝h ⎟
= -
h⎠ 3 3
h→0 h→0 ( - h + 1) lim
h→0 h

= lim (1 – h)2 = 1
h→0 f (3h) - f (0)
⎛ 1 1⎞
= lim =3
R.H.L. = lim f (0 + h) = lim
2-⎜ + ⎟ h→0 3h
⎝ h h⎠
h→0 h→0 ( h + 1) ∴ f  (x) = 3x + c, ∴ f (0) = 0 ⇒ c = 0
2-
2
∴ f (x) = 3x

= lim (1 + h) h = lim (1 + h)2 [(1 + h)1/h ]– 2
h→0 h→0 The correct option is (C)

=1×e =e –2 –2 53. Let a be any real number
Since L.H.L ≠ R.H.L, \ f (x) is not continuous at x = 0.
lim f (x) = lim x = a
x → a x→a

The correct option is (A) (when x → a through rational values)
⎧ e[ x ] + | x | - 2 lim f (x) = lim (1 – x) = 1 – a
⎪ , x≠0 x → a x→a
50. f (x) = ⎨ [ x ] + | x | (when x → a through irrational values)
⎪- 1 , x=0 1
⎩ lim f (x) will exist only when a = 1 – a or a =
x→a 2
e[ x ] + | x | - 2 e -1 - 2 1
lim f ( x ) = lim = Thus, if x ≠ , then lim f (x) will not exist and hence f (x)
x → 0- x → 0- [ x] + | x | -1 2 x→a
1
will be discontinuous at x = a where a ≠
e[ x ] + | x | - 2 2
lim f ( x ) = lim 1 ⎛ 1⎞ 1
x→0 [ x] + | x | Also, lim f (x) = and f ⎜ ⎟ =
+
x→0 +

x→
1 2 ⎝ 2⎠ 2
x
= lim e - 2 → - ∞
2

1
x→0 +
x Hence, f (x) is continuous at x =
.
2
The correct option is (D)
The correct option is (C)
51. Let x0 be any arbitrary real number.
54. We have,
Case I: x0 is rational
f (x) = [x] + | 1 – x |, – 1 ≤ x ≤ 3
Then, f (x0) = 1
⎧- x , - 1 ≤ x < 0
In any vicinity of a rational point there are irrational points, ⎪1 - x , 0 ≤ x < 1
where f (x) = 0. Hence, in any vicinity of x0 there are points x ⎪⎪
for which
= ⎨x ,1≤ x < 2
| ∆ y | = | f (x0) – f (x) | = 1 ⎪1 + x , 2 ≤ x < 3

Case II: x0 is irrational ⎪⎩5 ,x=3
Then, f (x0) = 0 The only doubtful points are x = – 1, 0, 1, 2 and 3. It can be

In any vicinity of an irrational point there are rational points easily seen that f (x) is differentiable at x = – 1 but not differ-
at which f (x) = 1. Hence, it is possible to find the values of x entiable at x = 0, 1, 2 and 3.
for which
Hence, the required points are 0, 1, 2 and 3.
| ∆ y | = | f (x0) – f (x) | = 1
The correct option is (C)

Objective_Maths_JEE Main 2017_Ch 12.indd 34 01/01/2008 05:26:38


Continuity and Differentiability  12.35

55. Since f (x) is continuous at x = 0,


Hence, the composite function is discontinuous at three
points x = 1/2, x = 1 and x = 2.
∴ lim f (x) = f (0)
x→0
The correct option is (B)
Take any point x = a, then at x = a

59. Since | x – 1 |, | x – 1 |2, etc, are continuous at x = 1 \ f (x) is
lim f (x) = lim f (a + h) = lim [ f (a) + f (h)] continuous at x =1 for all ak ∈R.
x→a h→0 h→0
Also, | x – 1 |2, | x – 1 |4, etc., are all differentiable at x = 1,
= f (a) + lim f (h) = f (a) + f (0) whereas | x – 1 |, | x – 1 |3, etc., are not differentiable at x = 1.
h→0
Therefore, f (x) is differentiable at x = 1, provided a2k + 1 = 0.
= f (a + 0) = f (a)
The correct option is (C)
∴ f (x) is continuous at x = a. Since x = a is any arbitrary
point, therefore f (x) is continuous for all x. 60. [x + 1] = 0 if 0 ≤ x + 1 < 1 i.e., – 1 ≤ x < 0
The correct option is (C) Thus, domain of f = R\[– 1, 0)
56. For n ∈ I, ⎛ p ⎞

We have, sin ⎜ continuous at all points of R\[– 1, 0)

lim f ( x ) = lim [ x ] cos
2x - 1
p ⎝ [ x + 1] ⎟⎠
x→n +
x→n +
2 and [x] continuous on R \ Z, where Z denotes the set of inte-
2n - 1 gers. Thus, the points where f can possibly be discontinuous

= n cos p =0
2 are ..., – 3, – 2, – 1, 0, 1, 2, ...
2x - 1 ⎛ p ⎞

and, lim f ( x ) = lim [ x ] cos p For 0 ≤ x < 1, [x] = 0 and sin ⎜
is defined.
x → n- x → n- 2 ⎝ [ x + 1] ⎟⎠
2n - 1 ∴ f (x) = 0 for 0 ≤ x < 1

p =0
= ( n - 1)cos Also, f is not defined on [– 1, 0), so the continuity of f at 0
2
means continuity of f from right at 0. Since f is continuous
Thus, f is continuous for x = n ∈ I. Since the functions

from right at 0, so f is continuous at 0. Hence, the set of
2x - 1
g (x) = [x] and h (x) = cos p are continuous on x ∈ points of discontinuity of f is Z \ {0}.
2
The correct option is (B)
R – I, so f is continuous everywhere.

The correct option is (A) f ( x)
61. We have, f (0) = 0, g (x) = ,
x
57. Note that f (x) = 0 for each integral value of x.
f ( x)
Also, if 0 ≤ x < 1, then 0 ≤ x2 < 1 lim g ( x ) = lim = xlim
→0
f’( x ) = f  ′(0)
x→0 x→0 x
∴ [x] = 0 and [x2] = 0 ⇒ f (x) = 0 for 0 ≤ x < 1
Next, if 1 ≤ x < 2 , then Thus, the required value is f  ′(0).
1 ≤x2< 2 ⇒ [x] = 1 and [x2] = 1 The correct option is (D)
62. We have,
Thus, f (x) = [x]2 – [x2] = 0 if 1 ≤ x < 2
It follows that f (x) = 0 if 0 ≤ x < 2
cos -1(1 - {h}2 ) sin -1 (1 - {h})
lim f (0 + h) = lim
This shows that f (x) must be continuous at x = 1.

h→0 h→0 {h} - {h}3

However, at points x other than integers and not lying cos -1(1 - h2 ) sin -1(1 - h)

=
between 0 and 2 , f (x) ≠ 0 h (1 - h2 )

Thus, f is discontinuous at all integers except 1.

The correct option is (D) sin -1(1 - h) cos -1(1 - h2 )

= lim ⋅ lim
1
h→0 1- h 2 h→0 -h
58. The function u = suffers a discontinuity at the point x
= 1. x -1
cos -1(1 - 2 sin 2 θ )

= sin–1 1 · lim
1 θ →0 2 sin θ
The function f (x) =
suffers a discontinuity at the
u2 + u - 2
 [Putting 1 – h2 = cos 2q]
points where u2 + u – 2 = 0 i.e., u = – 2 and u = 1. Using these p 2θ p
values of u, the corresponding values of x are obtained by
= ⋅ lim =
2 2 θ →0 sin θ 2
solving the equations
cos -1(1 - {- h}2 ) sin -1(1 - {- h})
1 1 lim f (0 - h) = lim

–2=
and 1 = i.e., x = 1/2 and x = 2 h→0 h→0 {- h} - {- h}3
x -1 x -1

Objective_Maths_JEE Main 2017_Ch 12.indd 35 01/01/2008 05:26:42


12.36  Chapter 12


gives a=2
cos -1(1 - (1 - h) 2 ) sin -1(1 - (1 - h))

= lim 3
Now, we have,
h→0 (1 - h) - (1 - h)
(128a + ax )18/
- 2 ⎛ 0⎞
cos -1( h ( 2 - h)) sin -1 h
f (0) = lim ⎜ ⎟

= lim x→0 (32 + bx ) - 2 ⎝ 0 ⎠
1/5
h→0 (1 - h) ( 2 - h) h
2
cos -1( h ( 2 - h)) sin -1 h ( 256 + 2 x ) -7/8 5 2 -7 5

= lim ⋅ lim
= lim 8 = · =
h → 0 (1 - h) ( 2 - h) h → 0 h x→0 b -4/5 4b 2 -4 32b
(32 + bx )
5
cos -1 0

= ⋅1 = π/4 5
2
gives b=
32 f (0)
Since R.H.L. ≠ L.H.L., therefore no value of f (0) can make

Hence, we have,
f continuous at x = 0

a 64
The correct option is (D)
= f ( 0)
63. We have,
b 5
cosec ( - h )
The correct option is (C)
lim f ( x ) = lim [sin ( - h) + cos( - h)]

x → 0- h→0

- cosec h 65. Given that;



= lim (cos h - sin h)]
h→0 f  (x y) = exy– x – y {ey f (x) + ex f (y)} ∀ x, y ∈ R+
1

cos h - sin h - 1 Putting x = y = 1, we get

= lim (1 + (cos h - sin h - 1)] cos h - sin h - 1 - sin h
f (1) = e–1 {e1f (1) + e1f (1)}
h→0
⇒ f (1) = 0
cos h - sin h - 1 f ( x + h) - f ( x )

= ⎡ lim (1 + y )1/ y ⎤
lim
h→0
- sin h Now, f (x) = lim

⎢⎣ y → 0 ⎥⎦ h→0 h
⎧ ⎛ h⎞ ⎫
f ⎨ x ⎜1 + ⎟ ⎬ - f ( x )
cos h - sin h - 1 ⎛ 0 ⎞ ⎩ ⎝ x⎠⎭

Now, lim ⎜⎝ ⎟⎠ = lim

h→ 0 -sin h 0 h→0 h
- sin h - cos h 0 - 1 ⎛ h⎞ ⎛ h⎞
⎪⎧ 1 + x h ⎞ ⎪⎫
h

= lim = =1 x ⎜1 + ⎟ - x - ⎜1 + ⎟
⎝ x⎠ ⎝ x⎠ ⎛
h→0 - cos h -1 lim e ⎨e f ( x ) + e x f ⎜1 + ⎟ ⎬ - f ( x )
h→0
⎪⎩ ⎝ x⎠⎪
=

Thus, xlim
→0
f ( x) = e - h
h
Now, we have,
h -1 - + x ⎛ h⎞
eh f ( x) + e x f ⎜1 + ⎟ - f ( x )
e 1/h
+e 2/h
+e 3/h ⎝ x⎠

lim f ( x ) = lim = lim

x → 0+ h→0 ae -2 +1/- h
+ be -1+ 3/h
h→0 h
h
e -2/h + e -1/h + 1 h -1- +x ⎧ ⎛ h⎞ ⎫

= lim f ( x ) (e h - 1) + e x
⎨ f ⎜1 + ⎟⎠ - f (1)⎬
h→0 -2
( ae ) e -2/h
+ (be ) -1 ⎩ ⎝ x ⎭
= lim

h→0 h
0 + 0 +1 e (Q f (1) = 0)


= =
( ae -2 ) 0 + (be -1 ) b
h
f ( x ) (e h - 1) h -1- + x ⎧ ⎛ h⎞ ⎫
e + lim e x
⎨f ⎜⎝1 + ⎟⎠ - f (1)⎬
If f is continuous at x = 0, then e = a = gives a = e and h h→0 ⎩ x ⎭
b=1 b = lim

h→0 h
⋅x
The correct option is (A) x
64. If f is continuous at x = 0, then
e x -1 ⋅ f ′(1)
/
18 = f ( x) +

(128a + ax ) -2 x
f (0) = lim
x→0 (32 + bx )1/5 - 2 ⎧ ⎛ h⎞
⎪⎪ f ⎜1 + ⎟ - f (1)
As x → 0, the denominator → 0. Thus, for limit to exist the
⎝ x⎠ e h - 1 ⎪⎫
⎨Q lim = f ′(1) and lim = 1⎬
numerator must also → 0. Thus, we have ⎪ h→0 h h
h→0
⎭⎪
(128a)1/8 = 2
⎪⎩ x


Objective_Maths_JEE Main 2017_Ch 12.indd 36 01/01/2008 05:26:48


Continuity and Differentiability  12.37


Thus,
ex
= f (x) +
· f ′(1) [Qf  ′(1) = e]
ex  ⎧1 2
⎪ 9 ( x + 1) , x < -1
x

f  ′(x) = f (x) + e
x ⎪
⎪1
ex y = f (x) = ⎨ ( x + 1) 2
, -1 ≤ x < 2
⇒ = f  ′(x) – f (x) ⎪3
x ⎪ 3( x - 1) 2 , x ≥1

1 x x ⎩
⇒ = e f ′( x ) - f ( x ) e
x e 2 x
We have to check continuity and differentiability at x = –1

and 2.
1 d ⎧ f ( x) ⎫

= ⎨ ⎬ Differentiability at x = –1;

x dx ⎩ e x ⎭
f ( -1 - h) - f ( -1)

Integrating both sides w.r.t. ‘x’ we get L.H.D. = Lf  ′(–1) = lim

h→0 -h
f ( x)
log | x | + c =

ex 1
x
= ( -1 - h + 1) 2 - 0 = 0

or, f (x) = e {log | x | + c} lim 9

Since f (1) = 0 ⇒ c = 0 h→0 -h

Thus, f (x) = exlog | x | f ( -1 + h) - f ( -1)

The correct option is (A) R.H.D. = Rf  ′(–1) = lim

h→0 h

66. Here, 1
f  (x) = [n + p sin x] is not differentiable at those points where
= ( -1 + h + 1) 2 - 0 = 0
n + p sin x is integer. lim 3
h→0 h
As p is a prime number.
⇒ n + p sin x is an integer if sin x = 1, –1, r/p
Therefore, f (x) is differentiable and hence continuous at
r x = –1
p -p r Differentiability at x = 2;
i.e., x= , , sin -1 , π – sin–1 p , where 0 ≤ r ≤ p – 1
2 2 p
f ( 2 - h ) - f ( 2)
L.H.D. Lf  ′(2) = lim

-p -h
h→0
But x ≠
, 0.
2 1
p r ( 2 - h + 1) 2 - 3
∴ Function is not differentiable at x = , sin -1 ,
= =2
lim 3
2 p h→0 -h
r
–1
π – sin p , where 0 < r ≤ p – 1 f ( 2 + f ) - f ( 2)
R.H.D. R f  ′(2) = lim

h→0 h

So, the required number of points are,
2

= 1 + 2(p – 1) = 2p – 1
= lim 3( 2 + h - 1) - 3 = 6

The correct option is (C) h→0 h
Hence, f (x) is not differentiable at x = 2. Since Lf  ′(2) and
67. Here, x = 2t – | t – 1 | and y = 2t2 + t | t | Rf  ′(2) are finite, therefore f (x) is continuous at x = 2. Hence,
Now, when t < 0; f (x) is continuous for all x and differentiable for all x except
x = 2.
x = 2t – {– (t – 1)} = 3t – 1
The correct option is (B)
1
and y = 2t2 – t2 = t2⇒ y = ( x + 1) 2 68. As  f (x) is continuous for all x ∈ R
9
when 0 ≤ t < 1 ; Thus,

x = 2t – (–(t – 1)) = 3t – 1 lim f ( x ) = f ( 3 )
x→ 3
1 x2 - 2x + 2 3 - 3
and y = 2t2 + t2 = 3t2⇒ y = ( x - 1) 2

where f (x) = ,x≠ 3
3 3-x
when t > 1;


x = 2t – (t – 1) = t + 1 x2 - 2x + 2 3 - 3
∴ lim f ( x ) = lim
and y = 2t2 + t2 = 3t2⇒ y = 3(x – 1)2
x→ 3 x→ 3 3-x

Objective_Maths_JEE Main 2017_Ch 12.indd 37 01/01/2008 05:26:52


12.38  Chapter 12

∴ h(x) of a linear function of x which is continuous for all


(2 - 3 - x) ( 3 - x)

= lim x ∈R.
x→ 3 ( 3 - x)
The correct option is (A)

= 2 (1 – 3) 72. If f (a) = 0, then obviously, x = a is the solution
Let f (a) > a. Since g(x) = f (x) – x,therefore g(a) > 0 and
Thus, f ( 3 ) = 2 (1 – 3 )

g(f (a)) = f (f (a)) – f (a) = a – f (a) < 0

The correct option is (C)
Since g is continuous from R to R so at least for one
c∈ (a, f (a)), g(c) = 0
⎪⎧ 0 if x < 1
2
69. We know that lim x 2 n = ⎨ Similarly, we can argue for f (a) <a
2
⎩⎪∞ if x > 1
n→∞
The correct option is (D)
2
∴ for x < 1, we have 73. As f is continuous on R, so f (0) = lim f ( x )
x→0
log( 2 + x ) -x 2 n sin x

f (x) = lim
n→∞ 1 + x 2n = lim f ( xn ) for any sequence xnsuch that lim xn = 0.
n→∞ n→∞

= log (2 + x)
and for x2 > 1,
Thus, f (0) = lim f (1/4 n)

n→∞
2n
log( 2 + x ) -x sin x
f (x) = lim
⎛ 1 ⎞
1 + x 2n = lim ⎜ (sin e n ) e - n +
2
n→∞

n→∞ ⎝ 1 + 1 / n2 ⎠
log( 2 + x )
- sin x
=0+1=1

= lim x 2n = –sin x
n→∞ 1 ⎛
(sin e n ) e - n = 0 as (sin e n )e - n ≤ e - n
2 2 2

+ 1 ⎜⎝ nlim
x 2n 
→∞

∴ lim f ( x ) = log (2 + 1) = log 3 and lim f ( x ) = –sin 1


x →1� x →1+
and e - n → 0 as n → ∞

2

)
\ jump of discontinuity at x = 1
The correct option is (A)
74. We have f  ′(x) + f  2(x) ≥ –1
= lim f ( x ) - lim f ( x ) = sin 1 + log 3
x →1+ x →1- f ′( x )
⇒ + 1 ≥ 0, forx ∈ (a, b)(1)

The correct option is (B) 1 + f 2 ( x)
h
70. lim f ( x ) = lim = 0 ⎛⎜Q lim e1/h = ∞⎞⎟ ∴
d
(tan -1 f ( x ) + x ) =
f ′( x )
+ 1 ≥ 0 (from (1))
x →1+ h→0 e1/h
+1 ⎝ h→0 ⎠ dx 1 + ( f ( x )) 2
-h h(x) = tan–1f (x) + x, is a non-decreasing function in the
⇒ 
lim f ( x ) = lim = 0 ⎡⎢Q lim e -1/h = 0 ⎤⎥
-1/h
x →1- e
h→0 +1 ⎣ h→0 ⎦ interval (a, b)
Therefore, f (x) is continuous at x = 1 ⇒ xlim h( x ) ≤ lim h( x )
→a x→b
The correct option is (A)
71. Since | f (x) – f (y) | ≤ | x – y |2 x ≠ y ⇒ lim (tan -1 f ( x ) + x ) ≤ lim (tan -1 f ( x ) + x )
x → a+ x → b-
f ( x) - f ( y)
∴ ≤|x–y| p p
x-y ⇒ +a≤- +b
2 2
Taking limit as y →x, we get
Hence, b – a ³π

f ( x) - f ( y)
The correct option is (B)
lim ≤ lim | x - y |
y→ x x-y y→ x
75. We have,
f ( x) - f ( y) f ( x + h) - f ( x )
⇒ lim ≤ lim ( x - y ) f  ′(x) = lim

y→ x x-y y→ x h→0 h
⇒ | f ′(x) | ≤ 0 ⇒ | f ′(x) | = 0 [∵ | f ′(x)| ≥ 0] f ( x ) + f ( h) + 2hx - 1 - 1 f ( x )

= lim
∴ f  ′(x) = 0 ⇒ f (x) = 6 (constant) h→0 h

⎧ f ( h) - 1 ⎫
∴ h(x) = ∫ f ( x) dx = ∫ c dx = c x + d, where d is constant
= lim ⎨2 x + ⎬
h→0 ⎩ h ⎭
of integration

Objective_Maths_JEE Main 2017_Ch 12.indd 38 01/01/2008 05:26:57


Continuity and Differentiability  12.39

Now, substituting x = y = 0 in the given functional relation, 76. Since f (x) is differentiable at x = 0
we get, f (0 + h) - f (0)
(0) = f (0) + f (0) + 0 – 1 ⇒ f (0) = 1
f  ⇒ lim = a (say)(1)
h→0 h
f ( h) - f ( 0 ) f ( x + h) - f ( x )
∴ f  ′(x) = 2x + lim = 2x + f  ′(0) Now, f ′(x) = lim

h→0 h h→0 h
⎛ 3 x + 3h ⎞ ⎛ 3 x + 3•0 ⎞
⇒ f  ′(x) = 2x + cos α f⎜ - f⎜
⎝ 3 ⎟⎠ ⎝ 3 ⎠

Integrating, f (x) = x2 + x cos α + C
⇒ f  ′(x) = lim
h→0 h

Here, x = 0 and f (0) = 1
∴ 1 = C f (3 x ) + f (3h) + f (0) - f (3 x ) - f (0) - f (0)
⇒ f  ′(x) = lim
⇒ 2
f (x) = x + x cos α + 1
h→0 3h

f (3h) - f (0)
It is a quadratic in x with discriminant ⇒ f  ′(x) = lim
h→0 3h
D = cos2 α – 4 < 0
⇒ f  ′(x) = f  ′(0)
and coefficient of x2 = 1 > 0
⇒ f  ′(x) = a [from (1)] (say)
∴ f (x) > 0 ∀x ∈R ∴ f (x) = ax + b, which is linear

The correct option is (C) The correct option is (A)

More than One Option Correct Type


77. Continuity at x = 0
1 ⎧ 2 1
L.H.L. = lim f (0 - h) = lim ( - h) p cos ⎪ x sin , x≠0
h→ 0 h→ 0 h 78 We have, g (x) = ⎨ x
⎪⎩0, x=0

= 0 if p> 0
1 For x ≠ 0,

R.H.L. = lim f (0 + h) = lim h p cos
1 ⎛ 1⎞ 1
h→ 0 h→0 h g′(x) = x2 cos ⎜ - ⎟ + 2x sin

= 0 if p> 0 x ⎝ x2 ⎠ x
and, f (0) = 0.
1 1

= – cos + 2x sin
∴ f (x) is continuous at x = 0 if p > 0 x x

For x=0
f (0 - h) - f (0) 1

L f  ′0) = lim x 2 sin -0
h→ 0 -h g ( x ) - g ( 0) x
g′(0) = lim = lim
1
x→0 x-0 x→0 x
p
( - h) cos - 0

= h 1
lim
= lim x sin =0
h→ 0 -h x→0 x
1 ⎧ 1 1

= lim ( - h) p -1 cos = 0 if p > 0, ⎪2 x sin - cos , x≠0
h→ 0 h ∴ g′(x) = ⎨ x x
⎪⎩0, x=0
f ( 0 + h) - f ( 0 )
and, R f  ′(0) = lim

h→ 0 h 1
g′ is not continuous at x = 0 as cos is not continuous at
1 x
h p cos -0 x = 0. Also, f is not differentiable at x = 0.

= h
lim
The correct option is (A, B)
h→ 0 h
1 ⎧1 - x, x ≤ -1

= lim h p -1 cos = 0 if p> 1 ⎪
h→ 0 h 79. f (x) = ⎨2, -1 < x ≤ 1
⎪1 + x, x >1
∴ f (x) is differentiable at x = 0 if p> 1. ⎩
lim f ( x )
The correct option is (A, B) x → - 1+

lim f ( x ) = lim (1 - x ) = 2 =
x → -1-
x → -1
-

Objective_Maths_JEE Main 2017_Ch 12.indd 39 01/01/2008 05:27:02


12.40  Chapter 12

and, lim f ( x ) = 2, so f is continuous at all points.



x →1 -1 1
⇒ lim a ( x + 1) = lim ⇒a= -
– f ( -1 - h) - f ( -1) x →1 x →1 x 2
f ′ (– 1 ) = lim
h→o -
-h 1 3
Putting a = -
in (1), we get b =
1+1+ h - 2 2 2

= lim =–1
The correct option is (B, C)
h→o -
-h
f ′ (– 1 ) = 0. Similarly, f ′ (1–) = 0 and f ′ (1+) = 1, so f is
+
82. We have, lim f (0 – h) = lim [– h sin (– ph)]
differentiable everywhere except at x = – 1, 1. h→0 h→0

Y
= lim [h sin π h] = 0,
h→0

lim f (0 + h) = lim [h sinph] = 0. andf (0) = 0

x
y

+ h → 0 h→0
=

1
1

∴ f (x) is continuous at x = 0.
=

x

2 It can be easily seen that f (x) is continuous in (– 1, 0).  f (x)


X is not differentiable at x = 1 but it is differentiable in (– 1, 1).
O
The correct option is (A, B, D)

The correct option is (A, C)
83. We have,
80. Let x = n, n ∈Z
f ( 0 - h) - f ( 0 )
Then, L.H.L. = lim (x) = n; R.H.L. = lim (x) = n + 1 L f  ′(0) = lim
x→n x→n h→0 -h
x<n x<n
Since, L.H.L. ≠ R.H.L., therefore f (x) is discontinuous at all
⎡ - h (3e -1/ h + 4) ⎤ ⎛ -1⎞

= lim ⎢ - 0⎥ ⋅ ⎜ ⎟
⎦⎥ ⎝ h ⎠
integers n. - 1/ h
⎣ 2-e
h→0 ⎢
Now, let x = p, n < p ≤ n + 1, where n is an integer.

0+4
Then, L.H.L. = lim (x) = n + 1,

= =2
x→ p 2-0
x< p
f ( 0 + h) - f ( 0 )
R.H.L. = lim (x) = n + 1
R f  ′(0) = lim
x→ p h→0 h
x> p
f ( p) = ( p) = n + 1 ⎡ h (3e1/ h + 4) ⎤1

= lim ⎢ - 0⎥
Since L.H.L. = R.H.L. = f ( p), therefore, f (x) is continuous ⎣ 2-e
h→0 ⎢ 1/ h
⎥⎦ h
at all non-integral points p.
The correct option is (B, C) ⎛ 3 + 4e -1/ h ⎞ 3+0

= lim ⎜ -1/ h ⎟ = 0 -1 = – 3
h → 0 ⎝ 2e - 1⎠
81. The given function is clearly continuous at all points except
possibly at x = ± 1.
Since L f  ′(0) ≠ R f  ′(0), \ f (x) is not differentiable at

For f (x) to be continuous at x = 1, we must have x = 0. But f (x) is continuous at x = 0 (as L f  ′(0) and R f  ′(0)

lim f ( x ) = xlim
→1
f ( x ) = f (1)
+
are finite).
x → 1-

The correct option is (A, D)
1
⇒ lim ax 2 + b = lim 84. Clearly, x = 1 is a point of discontinuity of the function
x →1 x →1| x|
1
⇒ a + b = 1 (1) f (x) = .
1- x
Now, for f(x) to be differentiable at x = 1, we must have ⎛ 1 ⎞ x -1
If x ≠ 1, then ( fof ) (x) = f [ f (x)] = f  ⎜ ⎟ = , which
f ( x ) - f (1) f ( x ) - f (1) ⎝1 - x⎠ x

lim = lim is discontinuous at x = 0.
x → 1- x -1 x → 1 + x -1
If x ≠ 0 and x ≠ 1, then

1
2 -1 ⎛ x - 1⎞
ax + b - 1 |x| ( fofof ) (x) = f [( fof ) (x)] = f ⎜
= x,
⇒ lim = lim ⎝ x ⎟⎠
x →1 x -1 x →1 x - 1

which is continuous everywhere.
1 Hence, f  3n (x) = ( fofof )n(x) = x, which is continuous

ax 2 - a -1
⇒ lim = x everywhere.
x →1 x - 1 lim
x →1 x - 1
So, the only points of discontinuity are x = 0 and x = 1.

 {∵ a + b = 1 ∴ b – 1 = – a}
The correct option is (B, C)

Objective_Maths_JEE Main 2017_Ch 12.indd 40 01/01/2008 05:27:07


Continuity and Differentiability  12.41

85. For x ≠ – 1, we have For f (x) to be continuous at x = 0, we have



f (x) = 1 – 2x + 3x2 – 4x3 + ...∞
1 xlim
→0
f ( x ) = f (0) ⇒ eb= 3
+

= (1 + x)– 1 =
1+ x ∴ b = ln 3
1
lim f (– 1 – h) = lim →–∞ The correct option is (A, D)
h→0 h→0 1-1- h
88. We have
So, f (x) is not continuous at x = – 1

sin 3 x + a sin 2 x + b sin x ⎛ 0 ⎞
-1
lim f ( x ) = lim ⎜⎝ ⎟⎠
-1 x→0 x→0 x5 0
f ( - 1 - h) - f ( -1)

Also, lim = lim h
h→0 ( -1 - h) - ( -1) h→0 -h 3 cos 3 x + 2a cos 2 x + b cos x

= lim
x→0 5x 4
1+ h For a finite limit to exist, the numerator must be 0 at x = 0,


= lim →∞
h→0 h2 since the denominator is 0 at x = 0
So, f (x) is not derivable at x = – 1.
i.e., 3 + 2a + b = 0 (1)
Hence, f (x) is neither continuous nor derivable at x = – 1.
Now, we have,
The correct option is (B, D) 3 cos 3 x + 2a cos 2 x + b cos x ⎛ 0 ⎞
lim f ( x ) = lim
⎜⎝ ⎟⎠
86. We have, x→0 x→0 5x4 0
F(x) = f (x), | x | < 1 ⎡ ⎤ -9 sin 3 x - 4 a sin 2 x - b sin x ⎛ 0⎞
⎢⎣ nlim x 2 n = 0 for | x | < 1⎥
= lim ⎜⎝ ⎟⎠
→∞ ⎦ x→0 20 x 3 0
f ( x) + g( x) -27 cos 3 x - 8a cos 2 x - b cos x

= , |x|=1
2
= lim
x→0 60 x 2
-2 n

= lim x f ( x ) + g ( x ) For a finite limit to exist, the numerator must be 0 at x = 0,

n→∞ x -2 n + 1 since the denominator is 0 at x = 0
⎡ ⎤ i.e. 27 + 8a + b = 0
(2)

= g(x), | x | > 1 ⎢⎣ nlim x -2 n = 0 for | x | > 1⎥
→∞ ⎦
Now, we have,
81 sin 3 x + 16 a sin 2 x + b sin x

Thus, the function F(x) shall be continuous everywhere if lim f ( x ) = lim

f (x) and g(x) are continuous everywhere and if F(x) is con- x→0 x→0 120 x
tinuous at x = ± 1, we have 81 ⎛ sin 3 x ⎞ 16 a ⎛ sin 2 x ⎞ b ⎛ sin x ⎞

= lim ⎜ ⎟+ ⎜ ⎟+ ⎜ ⎟
f (1) + g (1) x→0 40 ⎝ 3 x ⎠ 60 ⎝ 2 x ⎠ 120 ⎝ x ⎠

f (1) = = g(1) ⇒ f (1) = g(1)
2
81 16 a b
f ( -1) + g ( -1)
+ = +

and, f (–1) = = g(–1) ⇒ f (–1) = g(–1) 40 60 120
2
Solving equations (1) and (2), we have
The correct option is (A, B)
a = –4, b = 5
87. We have, For f to be continuous at x = 0, we have

1/h
⎛ 3⎞
lim f (0 + h) = lim ⎜1 + ah + bh ⎟

f  (0) = lim f ( x ) =
81 64
- +
5
=1
h→0 h→0 ⎝ 2
h ⎠ x→0 40 60 120
1 ⎛
ln ⎜1 +
ah + bh3 ⎞
f  (0) = 1

= lim e h ⎝ h2 ⎟⎠

The correct option is (A, B, D)
h→0

For limit to exist, we have 89. We have,
ah + bh3 2
lim 2
= 0 i.e., lim a + bh = 0, lim
2 f ( x ) - 3af ( 2 x ) + bf (8 x )
h→0 h h→0 h x→0 sin 2 x

which is possible only if a = 0.
2 f ( x ) - 3a f ( 2 x ) + bf (8 x )
Now, we have = lim
x→0 x2
1/h
lim f (0 + h) = lim (1 + bh)

For the limit to exist, we have
h→0 h→0
2f (0) – 3af (0) + b f (0) = 0

b/bh

= lim (1 + bh) = eb i.e., 3a – b = 2
[Q f (0) ≠ 0, given]  (1)
h→0

Objective_Maths_JEE Main 2017_Ch 12.indd 41 01/01/2008 05:27:13


12.42  Chapter 12

2 f ′( x ) - 6 a f ′( 2 x ) + 8b f ′(8 x ) ⎪⎧3 x 2 - 2 x + 1 ; 0 ≤ x ≤ 1

= lim
Also, g′(x) = ⎨
x→0 2x ⎩⎪ -1 ; 1< x ≤ 2

For the limit to exist, we have which clearly shows g(x) is continuous for all x ∈ [0, 2] but

2f  ′(0) – 6af  ′(0) + 8bf  ′(0) = 0
g(x) is not differentiable at x = 1
i.e., 3a – 4b = 1
[Qf ′(0) ≠ 0, given]  (2)
The correct option is (A, D)

Solving equations (1) and (2), we have
92. Here, f (x) = x4 – 8x3 + 22x2 – 24x

a = 7/9 and b = 1/3
⇒ f  ′(x) = 4x3 – 24x2 + 44x – 24

The correct option is (B, C)
or, f  ′(x) = 4(x – 1) (x – 2) (x – 3)
⎪⎧ | x | - 3 , | x | < 0 which shows f (x) is increasing in [1, 2] ∪ [3, ∞) and decreas-
90. We have, f | x | = ⎨ 2
⎩⎪| x | -3 | x | + 2, | x | ≥ 0 ing in (–∞, 1] ∪ [2, 3].
Since | x | < 0 is not possible, so we get,
Thus, minimum f (x) ;x ≤ t ≤ x + 1, – 1 ≤ x ≤ 1

f ( | x | ) = | x | 2 – 3 | x | + 2, | x | ≥ 0 ⎧ f ( x + 1), - 1 ≤ x ≤ 0
⇒ minimum f (x) = ⎨

⎧⎪ x 2 + 3 x + 2, x < 0 ⎩ f (1) , 0 < x ≤ 1

= ⎨ (1)
2
⎩⎪ x - 3 x + 2, x ≥ 0 ⎧ f ( x + 1), - 1 ≤ x ≤ 0

Thus, g(x) = ⎨
f (1), 0 < x ≤1

Again, ⎪ x - 10,
⎩ x >1
⎧⎪ | x - 3 | x<0
| f (x) | = ⎨ 2

⎩⎪| x - 3 x + 2 | x ≥ 0 ⎧( x + 1) 4 - 8( x + 1)3 + 22 ( x + 1) 2 - 24, - 1 ≤ x ≤ 0


= ⎨ 1 - 8 + 22 - 24, 0 < x ≤1
⎧ ( x 2 - 3 x + 2), 0 ≤ x < 1
⎪⎪ ⎪ x - 10 , x >1

= ⎨ -( x 2 - 3 x + 2), 1 ≤ x < 2 (2) ⎩
⎪ 2 ⎧ x 4 - 4 x 3 + 4 x 2 - 9, - 1 ≤ x ≤ 0
⎪⎩ ( x - 3 x + 2), 2≤ x

⇒ g(x) = ⎨ - 9, 0 < x ≤1

From (1) and (2), we get ⎪ x - 10, x >1


g(x) = f (| x |) + | f (x) |
⎧4 x 3 - 12 x 2 + 8 x, - 1 ≤ x ≤ 0
⎧ ⎪
⎪ x 2 + 2 x + 5, Also, g′(x) = ⎨
0, 0 < x ≤1
x<0
⎪ ⎪ x >1
⎪ 2 ⎩ 1,

= ⎨ 2 x - 6 x + 4, 0 ≤ x < 1
⎪ 0 , 1≤ x < 2
which clearly shows g(x) is continuous in [–1,∞) but not

⎪⎩2 x - 6 x + 4, x ≥ 2
2 differentiable at x = 1

The correct option is (A, D)
⎧ 2 x + 2, x<0
⎪ 4 x - 6, 0 < x < 1 93. f (x) = [tan x] + tan x - [tan x ] = [t] + t - [t ] , where


and, g′(x) = ⎨ t = tan x. Clearly, 0 ≤ t < ∞ at 0 ≤ x <π /2. Possible points of
⎪ 0 , 1< x < 2
⎪⎩4 x - 6, discontinuity may be, at which t ∈ N.
x>2
Let t = k ∈ N

Clearly, g(x) is continuous in R – {0} and differentiable in
R – {0, 1, 2} L.H.L. at t = k = lim [t ] + t - [t ]

t →k-
The correct option is (A, C)
91. Here, f (x) = x3 – x2 + x + 1
= lim [k - h] + ( k - h) - [k - h]
h→0
⇒ f  ′(x) = 3x2 – 2x + 1, which is strictly increasing in (0, 2)
⎧ f ( x) ; 0 ≤ x ≤ 1
∴ g(x) = ⎨


h→0
{
= lim k - 1 + k - h - k + 1] = k }
⎩3 - x ; 1 < x ≤ 2
[asf (x) is increasing so f (x) is maximum when 0 ≤ t ≤ x]

R.H.L. at t = k = lim [t ] + t - [t ]
t→k+

⎧⎪ x 3 - x 2 + x + 1 ; 0 ≤ x ≤ 1
So, g(x) = ⎨

= lim [k + h] + k + h - [k + h]
⎩⎪ 3- x ; 1< x ≤ 2 h→0

Objective_Maths_JEE Main 2017_Ch 12.indd 42 01/01/2008 05:27:17


Continuity and Differentiability  12.43

⎛ b⎞
= lim k + k + h - k = k
( a + b + 5) + ⎜ - a - ⎟ x 2 + ...
h→0 ⎝ 2⎠
=
=3
∴ The function is continuous at t = k ∈ N x2
Thus, function f (x) is continuous for all x∈ [o, π /2)
 [By expansion of sin x and cos x]
The correct option is (A, C) If lim f ( x ) = 3 exists, then a + b + 5 = 0

x → 0-
1/ x -1/ x
e -e -2/x
94. lim = lim 1 - e =1 b
x → 0+ e1/x + e -1/x x → 0 1 + e -2/x
+
and, – a – = 3 ⇒ a = –1 and b = –4
2
e1/x - e -1/x
2/x

and, lim = lim e - 1 = –1 ⎛ ⎛ 3⎞⎞
1/x
Since, lim ⎜1 + ⎜ cx + dx ⎟ ⎟
1/ x -1/ x
x→0 e +e -
x → 0 e 2/x + 1 -
exists.
2
Hence, lim f ( x ) exists if g′(0) = 0 x→0 ⎝ ⎝ x ⎠⎠
+

x→0 -

If g(x) = ax + b, a ≠ 0 then lim f ( x ) = a and


x → 0+ cx + dx 3
⇒ lim =0⇒c=0
lim f ( x ) = –a. Hence xlim f ( x ) exists if g(x) = x2 x→0 +
x2
→0
x → 0
-

d
Now, lim (1 + dx ) = lim ⎡(1 + dx )1/dx ⎤ = ed
1/x
or, g(x) = x3h(x), where h(x) is a polynomial.
x→0 x→0 ⎣ ⎦
+ +

The correct option is (C, D)


95. Since, f (x) is continuous at x = 0 so at x = 0 both left and So, ed= 3 ⇒ d = loge3

right limits must exist and both must be equal to 3. Hence, a = –1, b = –4, c = 0 and d = loge3

Now,
The correct option is (A, B, C, D)
a (1 - x sin x ) + b cos x + 5

x2

Passage Based Questions


96. We have,
⎡ ⎛p ⎞ ⎤ ⎡ p ⎤
f ( 0 - h) - f ( 0 ) 3 -1 ⎢cos ⎜⎝ 2 - h⎟⎠ + 2⎥ - ⎢cos 2 + 2⎥
L f  ′(0) = lim = lim →–∞
= lim ⎣ ⎦ ⎣ ⎦
h→0 -h h→0 - h
h→0 -h
∴ f (x) is not differentiable at x = 0 [sin h + 2] - [2] 2-2
Also, if x < 0 or x ≥ 0 then | x | ≥ 0
= lim = lim =0
h→0 -h h→0 -h
∴ f ( | x | ) = 2 | x | + 1 for all x.
⎛p ⎞ ⎛p⎞
f ( 0 + h) - f ( 0 ) ( gof ) ⎜ + h⎟ - ( gof ) ⎜ ⎟
∴ R f ′(0) = lim ⎛p⎞ ⎝2 ⎠ ⎝ 2⎠
h→0 h R( fog)′ ⎜ ⎟ = lim
⎝ 2⎠ h→0 h
2h + 1 - 1

= lim =2 ⎡ ⎛p ⎞ ⎤ ⎡ p ⎤
h→0 h cos ⎜ + h⎟ + 2⎥ - ⎢cos + 2⎥

= lim ⎢⎣ ⎝2 ⎠ ⎦ ⎣ 2 ⎦
f ( 0 - h) - f ( 0 ) h→0
and, L f ′(0) = lim
h
h→0 -h
[ - sin h + 2] - [2]
2 ( - h) + 1 - 1
= lim

= lim =2 hh→0
h→0 -h
1- 2
∴ f ( | x | ) is differentiable at x = 0. = lim →–∞
h→0 h
The correct option is (C)
∴ (gof ) is not differentiable at x = π/2.
⎛p⎞ The correct option is (D)
97. L (gof )′ ⎜ ⎟
⎝ 2⎠
[k - h]sin p ( k - h) - [k ]sin p k
⎛p ⎞ ⎛p⎞ 98. f ′(k – 0) = lim
( gof ) ⎜ - h⎟ - ( gof ) ⎜ ⎟ h→0 -h
⎝2 ⎠ ⎝ 2⎠

= lim k -1
-h = lim ( -1) ( k - 1)sin p h - k × 0
h→0


h→0 -h

Objective_Maths_JEE Main 2017_Ch 12.indd 43 01/01/2008 05:27:23


12.44  Chapter 12

k -1
= lim ( -1) ⋅ ( k - 1)sin p h
2
= 6 – 1 + 4x + x - 4 - 1
h→0 -h 2 2 2
= (– 1)k ⋅ (k – 1) π.
2
= 1 + 4x + x

The correct option is (A) 2
99. Differentiability at x = 0:
⎪⎧1 + 4 x + x 2 , if x > 2
f ( 0 - h) - f ( 0 ) f (x) = ⎨
Lf  ′(0) = lim
⎪⎩ 5 x + 1 , if x ≤ 2
h→0 -h
f ( 2 + h) - h( 2)
- h (e -1/ h - e1/ h ) e- 2/ h - 1 We have, Rf  ′(2) = lim

= lim = lim = – 1. h→0 h
h→0 - h (e -1/ h + e1/ h ) h→0 e- 2/ h + 1
( 2 + h) 2
f ( 0 + h) - f ( 0 ) h (e1/ h - e -1/ h ) = 1 + 4 ( 2 + h) + - 11
Rf ′(0) = lim
= lim lim 2
h→0 h h → 0 h ( e1/ h + e - 1/ h ) h→0 h
- 2/ h
1- e 2
h

= lim =1 11 + 6 h + - 11
h→0 1 + e- 2/ h = lim 2 =6
h→0 h
Since L f ′(0) ≠ Rf ′(0), \ f (x) is not differentiable at x = 0.
But since L f ′(0) and R f ′(0) are finite, therefore f (x) is con- f ( 2 - h) - f ( 2)
and, Lf  ′(2) = lim
tinuous at x = 0. h→0 -h
Hence, f (x) is continuous everywhere but not differentia- 5 ( 2 - h) - 1 - 11
ble at x = 0. = lim
h→0 h
The correct option is (A)
11 - 5h - 11
100. We have, = lim = –5

h→0 h

xn ( x log a) n
f (x) = ∑ n !
(log a)n = ∑ n!
∴ f (x) is not differentiable at x = 2
n=0 n=0 Since Rf  ′(2) and Lf  ′(2) are finite, therefore f (x) is continu-
ous at x = 2
= ex log a= e log a = ax
x


The correct option is (A, D)
f ( 0 - h) - f ( 0 ) a- h - 1
L f ′(0) = lim = lim 102. We have,
h→0 -h h→0 -h 1
= logea lim f (5 – h) = lim tan– 1 (5 - h) - 5
h→0 h→0
f ( 0 + h) - f ( 0 ) ah - 1
R f ′(0) = lim = lim ⎛ -1⎞
h→0 h h→0 h = lim tan– 1 ⎜⎝ h ⎟⎠
h→0
= logea
-p
Since L f ′(0) = R f ′(0), = tan– 1 (– ∞) =
2
∴ f (x) is differentiable at x = 0.
1
Since every differentiable function is continuous, there- and lim f (5 + h) = lim tan– 1 (5 + h) - 5
fore, f (x) is continuous at x = 0. h→0 h→0

The correct option is (B, D) ⎛ 1⎞


= lim tan– 1 ⎜⎝ h ⎟⎠
101. We have, for x > 2 h→0
5 p
= tan– 1 (∞) =
.
f (x) = ∫ {5 + | 1 - t |} dt 2
0
Since lim f (5 – h) ≠ hlim
→0
f (5 + h), therefore, f (x) has
1 x h→0

= ∫ {5 + 1 - t} dt + ∫ {5 + t - 1} dt ­ iscontinuity of the first kind at x = 5.


d
0 1
The correct option is (A)
[Since x > 2] 103. The function u = tan x is discontinuous at nπ ± π/2, n ∈ I
1 x
⎛ t2 ⎞ ⎛ t2 ⎞ 1 - u2
= ⎜ 6t - ⎟ + ⎜ 4t + ⎟ The function f (x) =
⎝ 2⎠
0
⎝ 2⎠
1
2 + u2

Objective_Maths_JEE Main 2017_Ch 12.indd 44 01/01/2008 05:27:30


Continuity and Differentiability  12.45

is continuous at every u ∈ R. Hence, f (x) is continuous on


x∈ R ∼ n ∈ I At x = 1, we have lim f ( x ) = (1 + 1)3 = 8 and f (1) = 1.
x →1-
Now, we have,
1 Since lim f ( x ) = 8 ≠ f (1), therefore, f has a removable
-1
1 - u2 2 x →1
lim f ( x ) = lim = lim u = –1 discontinuity at x = 1.
p u→∞ 2 + u 2 u→∞ 2
x → np ± + 1 The correct option is (A, B)
2
u2
p 105. We have,
Hence, the points x = nπ ± n ∈ I have removable
2 [cos x] = 1, x = 0
discontinuity.
= 0, 0 <x ≤ π/2

The correct option is (C)
= –1, π/2 <x < 3π/2
104. The function = 0, 3π/2 ≤ x < 2π
f (x) = t3 = (x – 1)3, x ≤ 0 = 1, x = 2π
= (x + 1)3, 0 < x < 1 1
Therefore, f (x) = = 1, x = 0
= 1, x = 1 [cos x ]
= (3 – x)3, 1 < x ∞, 0 <x ≤ π/2
may have discontinuities at x = 0, 1. –1, π/2 <x < 3π/2
At x = 0, we have ∞, 3π/2 ≤x < 2π
1, x = 2π
xlim
→0
f ( x ) = f (0) = (0 – 1)3 = –1
-
Clearly, f (x) has discontinuity of second kind from left
at x = π/2
xlim
→0
f ( x ) = (0 + 1)3 = 1
+
The correct option is (C)
Since xlim
→0
f ( x ) and lim f ( x ) both exist but are not
-
x → 0+
equal, therefore f has discontinuity of first kind at x = 0.

Match the Column Type


1 06. I. We have, fog = I For f to be continuous at x = 0, we must have
⇒ ( fog) (x) = x for all x
lim f (0 – h) = hlim
→0
f (0 + h) = f (0) ⇒ea = e2/3 = b
⇒ f [ g (x)] = x ⇒ f ′[g (x)] ⋅ g′ (x) = 1 h→0

1 1 2
⇒ f ′ [ g (a)] ⋅ g ′ (a) = 1 ⇒f ′ [ g (a)] = = ⇒ a= and b = e2/3.
g’ ( a) 2 3
[ g ′ (a) = 2]
The correct option is (A)
1
⇒ f ′ (b) = [Qg (a) = b].
2 p
For f to be continuous at x =
III. , we must have
The correct option is (C) 4
II. We have, p
f = lim (sin 2 x ) tan 2 2 x
lim f (0 – h) = lim [1 + | sin (– h) |] a/|sin (– h ) | 4 n→p
h→0 4
h→0
sin 2 x
= lim (1 + sin h)a/sin h = lim
h→0 n→
p cot 2
4
2 cos 2 x
= hlim
→0
[(1 + sin h)1/sin h]a = ea, = lim
n→
p -2 coot 2 x cosec 2 2 x ⋅ 2
4
lim f (0 + h) = lim etan 2h/tan 3h 1 1
h→0 h→0 = lim - ⋅ sin m3 2 x = - .
p 2 2
n→
⎡ tan 2 h 2 3h ⎤ 2 2 4
lim
h→0 ⎢
× × ⎥ 1× ×1
= e ⎣ 2h 3 tan 3h ⎦
= e 3 = e3
The correct option is (D)
and f (0) = b

Objective_Maths_JEE Main 2017_Ch 12.indd 45 01/01/2008 05:27:34


12.46  Chapter 12

IV. For f to be continuous at x = 0 we must have 108. I. f (x) = | 2 sgn 2x | + 2


3
1+ x - 4 1+ x lim f ( x ) = xlim f ( 0 + h)
f (0) = lim →0 +
x → 0+
x →0 x
=|2×1|+2=4
1 1
-
3(1 + x ) 2 / 3 4(1 + x )3/ 4 lim f ( x ) = hlim f ( 0 - h)
= lim x → 0- →0
x →0 1
1 1 1 = hlim | 2 sgn ( -2h)| + 2
= - = →0
3 4 12
= | 2(–1) | + 2 = 4
The correct option is (B)
∴  lim f ( x ) = 4 ≠ f (0)
107. I. Since f (x) is continuous in [0, 1], therefore, x→ 0

[∵ f (0) = | 2 sgn (2.0) | + 2 = 0 + 2 = 2]


⎛ n ⎞ ⎛ n ⎞
lim f ⎜ ⎟ = f ⎜ nlim ⎟ ∴ f (x) has a removable discontinuity at x = 0
n→∞ ⎝ 2 n + 1⎠ ⎝ → ∞ 2 n + 1⎠
The correct option is (C)
⎛ 1⎞ px p
= f ⎜ ⎟ = 2. II. lim f ( x ) = lim tan = tan = ∞
⎝ 2⎠ x →1- x →1 2 2
The correct option is (B) ∴ lim f ( x ) does not exist
II. Since f is continuous on [2, 5], therefore f assumes at x →1-

least once, every values between f (2) and f (5). But it is ∴ f (x) has infinite discontinuity at x = 1.
given that f (x) takes only rational values for all x and The correct option is (B)
there are irrational values also between f (2) and f (5),
1 1
this is possible only if f (x) has a constant rational value For any x ≠ 0, –1 ≤ sin
III.  ≤ 1, but as x → 0, sin
at all points between x = 2 and x = 5. Since f (4) = 8, x x
does not approach to any particular value but oscillates
\ f (3⋅7) = 8.
between –1 and 1.
The correct option is (D)

The correct option is (A)
Let g (x) = x3 – 3, then g (x) is an increasing function
III. 
on the interval (1, 2). Since g (1) = – 2 and g (2) = 5, | x + 2|
IV. lim f ( x ) = lim
therefore between – 2 and 5 there are 6 points where x → - 2+ x → - 2+ tan -1( x + 2)
f (x) is discontinuous (as [x3 – 3] is discontinuous at the x+2
points where x3 – 3 is an integer). = lim =1
x → -2 tan -1( x + 2)
The correct option is (A)
| x + 2|
lim f ( x ) = lim
⎛ -p p ⎞ x → - 2- x → - 2- tan -1( x + 2)
IV. 3 ≤ 3 + 2 cos x ≤ 5 for x ∈ ⎜ ,
⎝ 2 2 ⎟⎠ - ( x + 2)
= lim =1
f (x) = [3 + 2 cos x] is discontinuous at those points where x → -2 tan -1( x + 2)
3 + 2 cos x is an integer.
∴ Both the limits exist but are unequal
-p p ∴ f (x) has jump discontinuity at x = –2.
Now, 3 + 2 cos x = 3 if cos x = 0. So, x = ,
2 2 The correct option is (D)
 (not possible) 109. I. Let f (x) = sin x – x + 1.
1 3p 3p
3 + 2 cos x = 4 if cos x = ⎛ 3p ⎞
2 f  (0) = 1 > 0 and f ⎜ ⎟ = –1 – +1=– < 0.
⎝ 2⎠ 2 2
p -p Thus, by intermediate value theorem, there is ax ∈
So, x has two values and
3 3 ⎛ 3p ⎞
3 + 2 cos x = 5 if cos x = 1. So, x = 0 ⎜⎝ 0, ⎟⎠ such that f (x) = 0.
2
\The number of values of x = 2 + 1 = 3 Similarly, argue for II, III and IV.
The correct option is (C) The correct option is (C)

Objective_Maths_JEE Main 2017_Ch 12.indd 46 01/01/2008 05:27:39


Continuity and Differentiability  12.47

Assertion-Reason Type

Clearly, f (x) is differentiable everywhere except at the


f ( x + h) - f ( x )
110. f  ′(x) = lim points where | x | = 1 i.e., x = ± 1.
h→0 h
Hence, f (x) is differentiable on (– ∞, ∞) \ { – 1, 1}.
f ( x + h) - f ( x + 0 ) The correct option is (D)
= lim
h→0 h
f ( x ) ⋅ f ( h) - f ( x ) ⋅ f ( 0 ) 114. We have,
= lim n n n
h→0 h [2rx ] 2rx [2rx ]
(x) = lim
f 
n→∞
∑ n 2
= lim
n→∞
∑ n2 - lim
n→∞
∑ n2
f ( h) - f ( 0 ) r =1 r =1 r =1
= lim ⋅ f ( x)
h→0 h Now, we have 0 ≤ {2rx} < 1
= f ′(0) ⋅ f (x) = 2f (x).(Qf  ′(0) = 2) n n n

df df
i.e., ∑ 0 ≤ ∑ {2rx} < ∑ 1
Now, = 2f or = 2 dx ⇒ d (log f – 2x) = 0 r =1 r =1 r =1
dx f
n
∴ log f – 2x = c, \f = e2x + c = ec ⋅ e2x = Ae2x, 0 {2rx} n
i.e., 2
≤∑ 2
<
whereA = ec = constant. n r =1 n n2
The correct option is (C)
n
111. (a). If x < – 1, then x > x3. So, f (x) = x {2rx} 1
i.e., 0 ≤ lim
n→∞
∑ n 2
< lim
n→∞ n
=0
If x = – 1, then x = x3. So, f (x) = x r =1

If – 1 < x < 0, then x < x3. So, f (x) = x3 n


{2rx}
If x = 0, then x = x3. So, f (x) = x3 ⇒ lim
n→∞
∑ n2
=0
r =1
If 0 < x < 1, then x > x3. So, f (x) = x  {By Sandwich Theorem}
If x = 1, then x = x3. So, f (x) = x
n
If x > 1, then x < x3. So, f (x) = x3 1
Thus, we have f (x) = 2x · lim
n→∞ n 2 ∑r
Thus, f (x) = x, x ≤ – 1 r =1

f (x) = x3, – 1 < x ≤ 0 n ( n + 1) 1


= 2x · lim = 2x · = x
f (x) = x, 0 < x ≤ 1 n→∞ 2n2 2
f (x) = x3, x > 1 Thus, f (x) is continuous everywhere.
Clearly, f (x) is not differentiable at x = – 1, 0, 1. The correct option is (A)
The correct option is (A) 115. We have,
112. Let h (x) = | x | for all x. Clearly, h (x) is continuous for cos (p x ) - x 2 n sin ( x - 1)
f (x) = lim
all x. n→∞ 1 + x 2 n +1 - x 2 n
Then, g (x) = | f (x) | = h[ f (x) ] = (hof ) (x) for all x.
cos(p x ) - 0
Since composition of two continuous functions is continu- i.e., f (x) = ,|x|<1
1+ x - 0
ous, therefore, g is continuous if f is continuous.
The correct option is (A) cos(p x ) - sin ( x - 1)
= , |x|=1
1 + 1-1
-1 d ⎛ 2x ⎞
113. f  ′(x) = × cos (p x )
⎛ 2x ⎞
2 dx ⎜⎝ 1 + x 2 ⎟⎠ - sin ( x - 1)
= lim x 2n
1- ⎜
⎝ 1 + x 2 ⎟⎠ n→∞ 1
+ x -1
x 2n
- (1 + x 2 ) 2 (1 - x 2 )
= × - sin ( x - 1)
2 2
(1 + x ) - 4 x 2 (1 + x 2 ) 2 = , |x|>1
x -1
⎧ -2 cos(p x )
⎪1 + x 2 , if | x | < 1 i.e., f (x) = , |x|<1
- 2 1- x 2
⎪ 1+ x
= ⋅ = ⎨
1 + x 2 |1 - x 2 | = –1 + sin 2, x = –1
⎪ 2 , if | x | > 1
⎪⎩1 + x 2 = –1 , x=1

Objective_Maths_JEE Main 2017_Ch 12.indd 47 01/01/2008 05:27:43


12.48  Chapter 12

sin ( x - 1) ⎛ ⎛ h⎞ ⎞
=– , |x|>1 f ⎜ x ⎜1 + ⎟ ⎟ - f ( x )
x -1 ⎝ ⎝ x⎠⎠
= lim
At x = –1, we have, h→0 h
cos p ( -1 + h)
lim f ( -1 + h) = lim ⎛ h⎞
h→0 h→0 1 + ( -1 + h) f ( x ) ⋅ f ⎜1 + ⎟ - f ( x )
⎝ x⎠
- cos( p h) ⇒ f  ′(1) = lim
= lim = –∞ h→0 h
h→0 h
[given f (x y) = f (x) · f (y)]
f (–1) = –1 + sin 2
implies discontinuity at x = –1
At x = 1, we have, ⇒
lim
⎧ h
⎩ f  ′(x) =
x

f ( x ) ⋅ ⎨1 + 1 + g h ⎬ - f ( x )
x ⎭ ( ( ))
h→0 h
- sin (1 + h - 1)
lim f (1 + h) = lim [given f (1 + x) = 1 + x (1 + g(x))]
h→0 h→0 1 + h -1
- sin h -1
= lim =
h → 0 1 + (1 - h) 2
⇒ f  ′(x) =
lim
⎧ h
f ( x ) ⎨1 + 1 + g h
⎩ x x ( ( ))

- 1⎬

cos p (1 - h) -1 h
lim f (1 - h) = lim = h→0
h→0 h → 0 1 + (1 - h) 2
f ( x)
f (1) = – 1 ⇒ (1) f  ′(x) =
x
implies discontinuity at x = 1
 ⎡⎢as lim g ( x ) = 0 ⎤⎥
The correct option is (A) ⎣ h→0 ⎦
116. We have,
f (x) = sgn (x) = –1, x < 0 2
f ( x) 1
2
x
∴ ∫ ⋅ dx = ∫ dx  [using (1)]
= 0, x = 0 f ′( x ) 1 + x 2 1 + x2
1 1
= 1, x > 0
1
( )
2
and, g(x) = x (1 – x2) = log | 1 + x 2 |
2 1
Now, fog(x) = –1, x(1 – x2) < 0


= 0, x(1 – x2) = 0
= 1, x(1 – x2) > 0
=
1⎡
2⎣
( )
log 5 ⎤
2⎦

Solving the inequality The correct option is (A)


x(1 – x2) < 0 118. We have, y = t2 + t | t | and x = 2t – | t |
i.e., x(x – 1) (x + 1) > 0 i.e., x ∈ (–1, 0) ∪ (1, ∞) When t ≥ 0
Thus, we have, x = 2t – t = t, y = t2 + t2 = 2t2
⎧ -1, x ∈ ( -1, 0) ∪ (1, ∞) ∴ x = t and y = 2t2
⎪ ⇒ y = 2x2 ∀x ≥ 0
fog(x) = ⎨ 0, x ∈{-1, 0, 1}
⎪ 1, x ∈ ( -∞, - 1) ∪ (1, 0) Also, when t < 0

x = 2t + t = 3t and y = t2 – t2 = 0
which is continuous everywhere except at x ∈ {–1, 0, 1}
⇒ y = 0 for all x < 0
Also,
gof (x) = f (1 – f 2) = –1 [1 – (–1)2], x < 0 ⎪⎧2 x 2 , x ≥ 0
Hence, f (x) = ⎨ which is clearly
= 0 (1 – 02), x = 0 ⎩⎪ 0 , x < 0
= 1 (1 – 12), x > 0 continuous for all x.
i.e., gof (x) = 0, ∀x ∈ R The correct option is (A)
which is continuous everywhere.
The correct option is (D)
117. We know,
f ( x + h) - f ( x )
f  ′(x) = lim
h→0 h

Objective_Maths_JEE Main 2017_Ch 12.indd 48 01/01/2008 05:27:46


Continuity and Differentiability  12.49

Previous Year’s Questions


123. xRy need not implies yRx
1 - tan x p 1 - tan x 1
119. f ( x) = ⇒ f ( ) = lim =- m p
4x - p 4 x→
p 4x - p 2 S: s ⇔ qm = pn
4 n q
The correct option is (C) m m
s reflexive
120. Given that f (x) = min{x + 1, |x| + 1} n n
Now, f (x) = x + 1 ∀ x ∈ R. m p p m
s ⇒ s symmetric
n q q n
y = –x + 1 y=x+1
m p p r
s , s
n q q s
(0, 1) ⇒ qm = pn, ps = rq
⇒ ms = rn transitive
S is an equivalence relation.
The correct option is (B)

The correct option is (C) sin( p + 1) + sin x x + x2 - x
124. lim = q = lim
x→0 x x→0 x 3/ 2
1 2
121. f (0) = lim - 1
x→0 x e2 x - 1 lim( p + 1)cos( p + 1) × + cos x = q =
x→0 2
e2 x - 1 - 2 x 1 3 1
= lim ⇒ p +1+1 =
⇒ p - ;q =
x→0 x(e 2 x - 1) 2 2 2

2e 2 x - 2 The correct option is(B)
= lim
x → 0 (e 2 x - 1) + 2 xe 2 x ⎛ 2 x - 1⎞ ⎛ 1⎞
125. f ( x ) = [ x ]cos ⎜ p = [ x ]cos ⎜ x - ⎟ p
⎝ 2 ⎟⎠ ⎝ 2⎠
4e 2 x
= lim =1 = [x] sin p x is continuous for every real x.
4e + 4 xe 2 x
x→0 2x
The correct option is (A)
The correct option is (D)
126. f (x) = 7 – 2x; x < 2
122. Using differentiation formula, we write
= 3; 2 ≤ × ≤ 5
f (1 + h) - f (1) = 2x – 7; x > 5
f ′(1) = lim
h→ 0 h f  (x) is constant function in [2, 5]
⎛ 1 ⎞ f is continuous in [2, 5] and differentiable in (2, 5) and f (2)
(1 + h - 1)sin ⎜ -0
⎝ 1 + h - 1⎟⎠ h ⎛ 1⎞ = f (5)
⇒ f ′(1) = lim = lim sin ⎜ ⎟
h→ 0 h h → 0 h ⎝ h⎠ by Rolle’s theorem f ′( 4) = 0

⎛ 1⎞ \ Statement 2 and statement 1 both are true and statement
⇒ f ′(1) = lim sin ⎜ ⎟ 2 is correct explanation for statement 1.
h→ 0 ⎝ h⎠
The correct option is(B)
∴ f is not differentiable at x = 1.
127. For f (x) to be continuous
f ( h) - f ( 0 ) 2k = 3m + 2
Similarly, f ′(0) = lim
h→ 0 h
⇒ 2k - 3m = 2  (1)
⎛ 1 ⎞ Also, for f (x) to be differentiable
( h - 1)sin ⎜ - sin(1)
⎝ h - 1⎟⎠ k
⇒ f ′(0) = lim =m
h→ 0 h
4
⇒ f is also not differentiable at x = 0. ⇒ k = 4m.
The correct option is (A) from (1), 8m - 3m = 2

Objective_Maths_JEE Main 2017_Ch 12.indd 49 01/01/2008 05:27:50


12.50  Chapter 12

⇒ 5m = 2 128. In the neighbourhood of x = 0, f (x) = log 2 – sin x



2 \ g ( x ) = f ( f ( x )) = log 2 - sin( f ( x ))
⇒ m= log 2 - sin(log 2 - sin x )
5
Since g (x) is differentiable at x = 0,
2 8 2 8 10
\ k = 4 × = \k+m= + = =2. \ g ′( x ) = - cos(log 2 - sin x ) ( - cos x )
5 5      5 5 5
The correct option is (D) ⇒ g ′(0) = cos(log 2)
The correct option is (C)

Objective_Maths_JEE Main 2017_Ch 12.indd 50 01/01/2008 05:27:51


Differentiation 13.1

CHaPtER

13 Differentiation

Chapter Highlights
Derivative of a function, Derivative at a point, Standard derivatives, Rules for differentiation, Derivative of para-
metric functions, Derivative of implicit functions, Differentiation of a function with respect to another function,
Logarithmic differentiation, Successive differentiation

dERIvaTIvE oF a FunCTIon dERIvaTIvE aT a poInT


Let y = f (x) be a function defined on the interval [a, b]. Let The value of f ′ (x) obtained by putting x = a, is called
for a small increment δ x in x, the corresponding increment the derivative of f (x) at x = a and it is denoted by f ′(a) or
in the value of y be δ y. Then ⎧ dy ⎫
⎨ ⎬ .
y = f (x) and y + δ y = f (x + δ x) ⎩ dx ⎭ x = a
On subtraction, we get
STandaRd dERIvaTIvES
δy = f (x + δx) – f (x)
δy f ( x + δ x) - f ( x)
algebraic Functions
or =
δx δx d n
1. ( x ) = nxn – 1
Taking limit on both sides when δx → 0 we have, dx
d d
δy f ( x + δ x) - f ( x) [ f ( x )]n = n[ f (x)]n – 1 dx f ( x )
lim = lim dx
δ x→0 δx δ x→0 δx
d ⎛ 1⎞ –n–1
if this limit exists, is called the derivative or differential 2. ⎜ ⎟ = –nx
dx ⎝ x n ⎠
dy
coefficient of y with respect to x and is written as or n
dx d ⎛ 1 ⎞ -n d
f ′(x). ⎜ ⎟ = f ( x)
dx ⎝ f ( x ) ⎠ n - 1 dx
[ f ( x )]
dy δy f ( x + δ x) - f ( x)
\ = lim = lim d 1
dx δ x→0 δ x δ x→0 δx 3. x =
dx 2 x
d 1 d
f ( x) = f ( x)
I M P O R TA N T P O I N T S dx 2 f ( x ) dx
The instantaneous rate of change of f (x) at x, which is
written as f ′(x) or
df
is called the derivative of f at x. SolvEd ExamplES
dx
Geometrically, it represents the slope of the tangent at the
1. If Sn denotes the sum of n terms of a G.P. whose
point (x, y) on the curve y = f (x). dSn
common ratio is r, then (r – 1) is equal to
dr

Objective_Maths_JEE Main 2017_Ch 13.indd 1 01/01/2008 05:32:18


13.2  Chapter 13

(A) (n – 1) Sn + n Sn – 1 (B) (n – 1) Sn – n Sn – 1 = P′ (x) P″ (x) + y2 P″′ (x) – P′ (x) P″ (x)


(C) (n – 1) Sn (D)  None of these
⎡ dy ⎤
Solution: (B) ⎢∵ 2 y dx = P ′(x ) ⎥
 ⎣ ⎦
We have,
n
a ( r - 1) = y2 P′″ (x) = P (x) P′″ (x).[∵ y2 = P (x)]
Sn =
r -1
3. Let f   be a twice differentiable function such that
⇒ (r – 1) Sn = arn – a
f ″ (x) = – f (x) and f ′(x) = g (x). If
Differentiating both sides with respect to r, we get h (x) = [ f (x)]2 + [g (x)]2 and h (5) = 11 then h (10) =
dSn (A) 11 (B) 0
(r – 1) + Sn = narn – 1 – 0
dr (C)  – 1 (D)  None of these
dSn
⇒ (r – 1) = narn – 1 – Sn Solution: (A)
dr
We have,
= n (nth term of G.P.) – Sn
h (x) = [ f (x)]2 + [g (x)]2
= n (Sn – Sn – 1) – Sn Differentiating both sides with respect to x, we get

= (n – 1) Sn – n Sn – 1. h′ (x) = 2 f (x) × f ′(x) + 2g (x) g′ (x)

2. If y2 = P (x), a polynomial of degree n ≥ 3, then = 2 f (x) g (x) + 2g (x) g′ (x)


[∵ f ′(x) = g (x)]
d ⎛ 3 d2 y⎞
2 y =
dx ⎜⎝ dx 2 ⎟⎠ = 2 g (x) [ f (x) + g′ (x)](1)
(A) – P (x) . P″′ (x) (B)  P (x) × P′″ (x)
But g (x) = f ′(x) ⇒ g′ (x) = f ″ (x) = – f (x)
(C) P (x) . P″ (x) (D)  None of these
[∵ f ″ (x) = – f (x)]
Solution: (B)
We have, \ From (1),
y2 = P (x)(1) h′ (x) = 2 g (x) [ f (x) – f (x)] = 0
dy
⇒ 2y = P′ (x)(2) ⇒ h (x) = constant for all x.
dx
dy . dy d2 y Given h (5) = 11,
⇒ 2    + 2y  .  2 = P″ (x)
dx dx dx \ h (10) = 11
2
⎛ dy ⎞ d2 y 4. A function f (x) is so defined that for all x, [ f (x)]n
⇒ 2y2 ⎜ ⎟ + 2y3  .  2 = y2 P″ (x)
⎝ dx ⎠ dx = f (nx). If f ′(x) denotes derivative of f (x) with respect
to x, then f ′(x) × f (nx) =
d2 y 2
⎛ dy ⎞ (A) f (x) (B)  0
⇒ 2y dx 2 = y2 P″ (x) – 2y2 ⎜ ⎟ 3
⎝ dx ⎠ (C) f (x) .  f ′(nx) (D)  None of these
1 Solution: (C)
= y2 P″ (x) – [P′ (x)]2
2 We have,
 [from (2)]
[ f (x)]n = f (nx)
d ⎛ 3 d2 y⎞ Differentiating with respect to x, we get
⇒ 2 y
dx ⎜⎝ dx 2 ⎟⎠ n [ f (x)]n – 1 × f ′(x) = f ′(nx) . n

dy 1
= 2y P″ (x) + y2 P″′ (x) – 2P′ (x) × P″ (x) ⇒ [ f (x)]n – 1 .  f ′(x) = f ′(nx)
dx 2

Objective_Maths_JEE Main 2017_Ch 13.indd 2 01/01/2008 05:32:21


Differentiation  13.3

⇒ [ f (x)]n .  f ′(x) = f ′(nx) .  f (x) = [ f (3)]2 – [ f ′(3)]2


[Multiplying both sides by f (x)]
= 25 – 16 = 9
⇒ f (nx) .  f ′(x) = f ′ (nx) . f (x)
⎛ 1⎞ dy
[∵ [ f (x)]n = f (nx)] 7. If 8 f (x) + 6 f ⎜ ⎟ = x + 5 and y = x2 f (x), then at
⎝ x⎠ dx
5. If f (x) = (1 – x)n, then the value of x = – 1 is equal to
1
(A)  0 (B) 
f"(0) f n ( 0) 14
(0) + f ′(0) +
f  + ... + is equal to 1
2! n! (C) – (D)  None of these
14
(A) 2n (B)  0
(C) 2n – 1 (D)  None of these Solution: (C)
We have,
Solution: (B)
⎛ 1⎞
We have, 8 f (x) + 6 f ⎜ ⎟ = x + 5 for all x (1)
⎝ x⎠
(x) = (1 – x)n, f ′(x) = – n (1 – x)n – 1,
f  Therefore,
⎛ 1⎞ 1
f ″ (x) = n (n – 1) (1 – x)n – 2, 8 f ⎜ ⎟ + 6 f (x) = + 5 (2)
⎝ x⎠ x
f ″′ (x) = – n (n – 1) (n – 2) (1 – x)n – 3... ⎛ 1⎞
⎜⎝ Putting x = ⎟
 x⎠
f n (x) = (– 1)n n (n – 1) (n – 2) ... 1 From (1) and (2), we have
1 ⎛ 6 ⎞
f (x) = ⎜ 8 x - + 10⎟⎠
⇒ f (0) = 1, f ′(0) = – n, f ″ (0) = n (n – 1), 28 ⎝ x
1
f ″′ (0) = – n (n – 1) (n – 2) ... f n (0) = (– 1)n n! \ y = x2 f (x) = (8x3 – 6x + 10x2)
28
Therefore, dy 1
f"(0) f n ( 0) ⇒ = (24x2 + 20x – 6)
(0) + f ′(0) +
f  + ... + dx 28
2! n!
dy ⎤ 1 1
\ = (24 – 20 – 6) = –
=1–n+
n ( n - 1) n( n - 1)( n - 2)
- + ... +
( -1) n n ! dx ⎥⎦ x = -1 28 14
2! 3! n!
8. Let f (x) be a polynomial function satisfying f (x) . 
= nC0 – nC1 + nC2 – nC3 + ... + (– 1)n nCn
⎛ 1⎞ ⎛ 1⎞
f  ⎜ ⎟ = f (x) + f ⎜ ⎟ . If f (4) = 65 and l1, l2, l3 are in
= (1 – 1)n = 0. ⎝ x⎠ ⎝ x⎠
G.P. then f ′(l1), f ′(l2), f ′(l3) are in
6. If f ′(x) = f (x) and f  ′ (x) = f (x) for all x. Also, f (3) = 5 (A)  A.P. (B)  G.P.
and f ′(3) = 4. Then the value of [ f (10)]2 – [ f (10)]2 is (C)  H.P. (D)  None of these
(A) 0 (B) 9
(C)  41 (D)  None of these Solution: (B)
Since f (x) is a polynomial function satisfying
Solution: (B)
⎛ 1⎞ ⎛ 1⎞
d f (x) .  f ⎜ ⎟ = f (x) + f ⎜ ⎟ ,
{[ f (x)]2 – [ f (x)]2} = 2 [ f (x) .  f ′(x) – f (x) .  f  ′ (x)] ⎝ x⎠ ⎝ x⎠
dx
= 2 [ f (x) . f (x) – f (x) . f (x)] \ f (x) = xn + 1 or f (x) = – xn + 1

[∵ f ′(x) = f (x) and f  ′ (x) = f (x)] If f (x) = – xn + 1, then f (4) = – 4n + 1 ≠ 65
= 0 So, f (x) = xn + 1.
2 2
⇒ [ f (x)] – [f (x)] = constant
Since f (4) = 65 \ 4n + 1 = 65 ⇒ n = 3
2 2 2 2
\ [ f (10)] – [f (10] = [ f (3)] – [f (3)]
\ f (x) = x3 + 1 ⇒ f ′(x) = 3x2

Objective_Maths_JEE Main 2017_Ch 13.indd 3 01/01/2008 05:32:24


13.4  Chapter 13

2 2 2 (A) 12 (B) 23
\ f ′(l1) = 3l1 , f ′(l2) = 3l2 , f ′(l3) = 3l3
(C)  – 13 (D)  None of these
Since l1, l2, l3 are in G.P.,
Solution: (B)
\ f ′(l1), f ′(l2), f ′(l3) are also in G.P. Let f (x) = a (x – 3)3 + b (x – 3)2 + c (x – 3) + d
Then,
9. Let f (x) = (x3 + 2)30. If f n (x) is a polynomial of degree
20, where f n (x) denotes the nth derivative of f (x) with f (3) = 1 = d ⇒ d = 1
respect to x, then the value of n is
f ′(3) = – 1 = c ⇒ c = – 1
(A) 60 (B) 40
(C)  70 (D)  None of these
f ″ (3) = 0 = 2b ⇒ b = 0
Solution: (C)
f (x) is a polynomial of degree 90. f ′(x) reduces the f ″′ (3) = 12 = 6a ⇒ a = 2.
degree of f (x) by one. Thus, in order to get a polyno-
mial of degree 20, we must reduce the degree of f (x) \ f (x) = 2 (x – 3)3 – x + 4 ⇒ f ′(x) = 6 (x – 3)2 – 1
by 70. Hence, f (x) should be differentiated 70 times to
get a polynomial of degree 20. \ f ′(1) = 6 (4) – 1 = 23.

\ n = 70. 12. A triangle has two of its vertices at P (a, 0),


Q (0, b) and the third vertex R (x, y) is moving along
10. If f (x) + f (y) + f (z) + f (x) .  f (y) .  f (z) = 14 for all x, y, z the straight line y = x. If A be the area of the triangle,
∈ R, then
dA
(A) f (0) = 2 then =
dx
(B) f ′(x) = 0, for all x ∈ R
(C) f ′(x) > 0, for all x ∈ R a-b a-b
(A)  (B) 
(D)  None of these 2 4
Solution: (A, B) a+b a+b
(C)  (D) 
We have, 2 4
(x) + f (y) + f (z) + f (x) .  f (y) .  f (z) = 14
f  (1) Solution: (C)
for all x, y, z ∈ R Area of D PQR = A
Putting x = y = z = 0, we get 1
= [x (b – 0) + 0 (0 – y) + a (y – b)]
3 f (0) + [ f (0)]3 = 14 2
1
= (bx + ax – ab) (As y = x)
⇒ [ f (0)]3 + 3 f (0) – 14 = 0 2
dA 1
⇒ f (0) = 2. \ = (a + b)
dx 2
Now, putting y = z = x in (1), we get
3 f (x) + [ f (x)]3 = 14 Y
Differentiating with respect to x, we get
Q (0, b)
3 f ′(x) + 3 [ f (x)]2 .  f ′(x) = 0 R (x, y)
2
⇒ 3 f ′(x) {1 + [ f (x)] } = 0
x
=
y

⇒ f ′ (x) = 0, for all x.

11. Let f (x) be a polynomial of degree 3 such that f (3) = 1, X


O P (a, 0)
f ′(3) = – 1, f ″ (3) = 0 and f ″′ (3) = 12. Then the value
of f ′(1) is

Objective_Maths_JEE Main 2017_Ch 13.indd 4 01/01/2008 05:32:25


Differentiation  13.5

Exponential Functions Trigonometric Functions


d x d
1. (e ) = ex 1. (sin x ) = cos x
dx dx
d d d d
e[ f ( x )] = {e[ f ( x )]} f ( x) sin[ f ( x )] = cos[ f ( x )] f ( x)
dx dx dx dx
d x d
2. ( a ) = ax loge a 2. (cos x ) = – sin x
dx dx
d d d d
a[ f ( x )] = {a[ f ( x )] loge a} f ( x) cos[ f ( x )] = - sin[ f ( x )] f ( x)
dx dx dx dx
d
3. (tan x ) = sec2x
dx
Solved Example
d d
tan[ f ( x )] = sec 2 [ f ( x )] f ( x)
13. Let f (x) = 22x – 1 and f (x) = – 2x + 2x log 2. If f ′(x) > dx dx
f ′(x), then
d
(A) 0 < x < 1 (B)  0≤x<1 4. (cot x ) = – cosec2 (x)
dx
(C) x > 0 (D)  x≥0
d d
Solution: (C) cot [ f ( x )] = - cosec 2 [ f ( x )] f ( x)
dx dx
Since f ′(x) > f  ′ (x)
d
2x – 1 x
5. (sec x ) = sec x tan x
⇒ 2 2 log 2 > – 2 log 2 + 2 log 2 dx
d d
⇒ 22x > – 2x + 2 sec[ f ( x )] = sec[ f ( x )]tan[ f ( x )] f ( x)
dx dx
⇒ 22x + 2x – 2 > 0 d
6. (cosec x ) = – cosec x cot x
x
⇒ (2 – 1) (2 + 2) > 0 x dx
d d
⇒ 2x – 1 > 0 (∵ 2x + 2 > 0 for all x) cosec[ f ( x )] = - cosec[ f ( x )] cot [ f ( x )] f ( x)
dx dx
⇒ 2x > 1

\ x > 0. Solved Examples


⎛p⎞
14. If f (x) = | cos x – sin x |, then f ′ ⎜ ⎟ is equal to
Logarithmic Functions ⎝ 2⎠
(A) 1 (B) – 1
d 1
1. (log e x ) = (C)  0 (D)  None of these
dx x
Solution: (A)
d 1 d p
[log e f ( x )] = f ( x) When 0 < x < , cos x > sin x
dx f ( x ) dx 4
\ cos x – sin x > 0
d 1
2. (log a x ) = p p
dx x loge a When < x < , cos x < sin x
4 2
d 1 d \ cos x – sin x < 0
[log a f ( x )] = f ( x)
dx f ( x ) loge a dx p
When < x < p, cos x – sin x < 0
2

Objective_Maths_JEE Main 2017_Ch 13.indd 5 01/01/2008 05:32:30


13.6  Chapter 13

p dy
\ When < x < p, cos x – sin x < 0 17. If y = sin -1 ⎣⎡ x - ax - a - ax ⎤⎦ , then =
4 dx
p 1
\ | cos x – sin x | = –(cos x – sin x), when <x<p (A)  (B)  sin x ⋅ sin a
4 sin a - ax
p
\ f (x) = –cos x + sin x, when <x<p
4 1
(C)  (D) Zero
⇒ f ′(x) = sin x + cos x 2 x 1- x

⎛p⎞ p p Solution: (C)


⇒ f ′ ⎜ ⎟ = sin + cos =1+0=1
⎝ 2⎠ 2 2 Put x = sin2q and a = sin2f, then
15. If f (x) = (ax + b) sin x + (cx + d) cos x, then the values
of a, b, c and d such that f ′(x) = x cos x for all x are y = sin–1 (sin q cos f – cos q sin f]
(A) b = c = 0, a = d = 1 (B)  b = d = 0, a = c = 1
(C) c = d = 0, a = b = 1 (D)  None of these = sin–1 [sin (q – f)] = q – f

Solution: (A) = sin -1 x - sin -1 a


We have,
dy 1 1
f ′(x) = a sin x + (ax + b) cos x + c cos x \ = ×
dx 1- x 2 x
 – (cx + d) sin x
d2
But f ′(x) = x cos x for all x (given). 18. If y = sin x, then 2
(cos7x) is equal to
dy
\ x cos x = (a – d) sin x + (b + c) cos x + ax (A)  35 cos x – 42 cos5x
3

cos x – cx sin x (B)  35 cos3x + 42 cos5x


(C)  42 cos3x – 35 cos5x
Equating the coefficients of sin x, cos x, x cos x and x (D)  None of these
sin x, we get
Solution: (A)
a – d = 0, b + c = 0, a = 1, c = 0
dy
We have, y = sin x ⇒ = cos x.
\ b = c = 0 and a = d = 1 dx
d2 d ⎛ d 7 ⎞
Now, (cos7x) = ⎜⎝ dy cos x ⎟⎠
16. Let f (x) = sin x, g (x) = 2x and h (x) = cos x. If f (x) dy 2 dy
⎛p⎞
= [go ( f h)] (x), then f  ″ ⎜ ⎟ is equal to d ⎛ dx ⎞
⎝ 4⎠ = ⎜ 7 cos6 x ⋅ ( - sin x ) ⎟
dy ⎝ dy ⎠
(A) 4 (B) 0
(C)  – 4 (D)  None of these d
= (– 7sin x cos5x)
dy
Solution: (C)
= – 7 [cos x × cos5x – 5 cos4x
We have, dx
sin2x]
dy
( f h) (x) = f (x) . h (x) = sin x cos x = – 42 cos5x + 35 cos3x
\ [ go ( f h)] (x) = g [( f h) (x)] = g [ f (x) . h (x)]

= g (sin x cos x) = 2sin x cos x = sin 2x


Inverse Circular Functions

f (x) = sin 2x d 1
i.e., 1. (sin -1 x ) = ;|x|<1
dx 1 - x2
⇒ f  ′ (x) = 2 cos 2x and f  ″ (x) = – 4 sin 2x
d 1 d
⎛p⎞ p [sin -1 f ( x )] = - f ( x ) ; | f  (x)| < 1
\ f  ″ ⎜ ⎟ = – 4 sin = – 4. dx 1 - [ f ( x )]2 dx
⎝ 4⎠ 2

Objective_Maths_JEE Main 2017_Ch 13.indd 6 01/01/2008 05:32:33


Differentiation  13.7

= [tan–12 – tan–11 + tan–13 – tan–12 + ... + tan–1x


d 1
2. (cos -1 x ) = - ; |x| < 1
dx 1 - x2  – tan–1 (x – 1) + tan–1 (x + 1) – tan–1x]

d 1 d = [tan–1 (x + 1) – tan–11]
[cos -1 f ( x )] = - f ( x ) ; |f  (x)| < 1
dx 1 - [ f ( x )]2 dx dy 1
\ =
dx 1 + ( x + 1) 2
d ⎛ 1 ⎞
3. tan -1 x = ⎜ ⎟ ;x=
dx ⎝ 1 + x2 ⎠ Rules for Differentiation
d 1 d 1. The derivative of a constant function is zero, i.e.,
[tan -1 f ( x )] = 2
f ( x ) ; f (x) ∈ 
dx 1 + [ f ( x )] dx d
(c) = 0
d ⎛ 1 ⎞ dx
4. cot -1 x = - ⎜ ⎟ ;x=
dx ⎝ 1 + x2 ⎠ 2. The derivative of constant times a function is constant
times the derivative of the function, i.e.,
d 1 d
[cot -1 f ( x )] = - f ( x ) ; f  (x) ∈  d d
dx 2
1 + [ f ( x )] dx [c .  f (x)] = c [ f (x)]
dx dx
d 1 3. The derivative of the sum or difference of two function
5. (sec -1 x ) = ; |x| > 1 is the sum or difference of their derivatives, i.e.,
dx | x | x2 - 1
d d d
d 1 d [ f (x) ± g (x)] = { f (x)} ± [g (x)]
[sec -1 f ( x )] = f ( x ) ; dx dx dx
dx | f ( x ) | [ f ( x )]2 - 1 dx
| f (x)| >1 Product Rule of Differentiation
⎛ ⎞ The derivative of the product of two functions
d 1
6. (cosec -1 x ) = - ⎜ ⎟ ; |x| > 1 d d .
dx ⎜⎝ | x | x 2 - 1 ⎟⎠ [ f (x) .  g (x)] = f (x)    [g (x)] 
dx dx
d
d 1 d  + g(x) .  [ f (x)]
[cosec -1 f ( x )] = – f ( x ) ; dx
dx | f ( x ) | [ f ( x )] - 1 dx
2
= (first function) × (derivative of second function)
| f  (x)| > 1  + (second function) × (derivative of first function)

Quotient Rule of Differentiation


Solved Example
The derivative of the quotient of two functions
x
1 dy d d
19. If y = ∑ tan -1 1 + r + r 2 then
dx
is equal to d ⎡ f ( x) ⎤
=
g ( x ) ⋅ [ f ( x )] - f ( x ) ⋅ [ g ( x )]
dx dx
r =1
dx ⎢⎣ g ( x ) ⎥⎦
1 1 [ g ( x )]2
(A)  (B)  2
1+ x 2
1 + (1 + x ) (denom. × derivative of num.)
(C)  0 (D)  None of these - ( num. × derivative of denom.)
=
Solution: (B) (denominator) 2
We have,
x x
Solved Examples
1 -1 ⎛
( r + 1) - r ⎞
y = ∑ tan -1
1+ r + r2
= ∑ tan ⎜
⎝ 1 + ( r + 1) r ⎟⎠ d ⎛ 1 + x 4 + x8 ⎞
r =1 r =1 20. If ⎜ 2 4 ⎟ = ax3 + bx then
dx ⎝ 1 + x + x ⎠
x
(A) a = 4, b = 2 (B)  a = 4, b = – 2
= ∑ ⎡⎣tan -1 (r + 1) - tan -1 r ⎤⎦ (C) a = – 2, b = 4 (D)  None of these
r =1

Objective_Maths_JEE Main 2017_Ch 13.indd 7 01/01/2008 05:32:39


13.8  Chapter 13

Solution: (B) 2
⎛ 2 x - 1⎞ ⎛ 2 + 2 x - 2 x ⎞
2
We have, (B) sin ⎜ 2 ⎟ ⎜
⎝ x + 1⎠ ⎝ ( x + 1) ⎟⎠
2 2
d ⎡ (1 + x 2 + x 4 ) (1 - x 2 + x 4 ) ⎤ 3
⎢ 2 4 ⎥ = ax + bx 2
⎛ 2 x - 1⎞ ⎛ 2 + 2 x - x ⎞
2
dx ⎣ (1 + x + x ) ⎦ (C) sin ⎜ 2 ⎟ ⋅ ⎜
⎝ x + 1⎠ ⎝ ( x 2 + 1) 2 ⎟⎠
d
⇒ (1 – x2 + x4) = ax3 + bx
dx (D)  None of these
⇒ – 2x + 4x3 = ax3 + bx
⇒ a = 4 and b = – 2. Solution: (B)

dy ⎛ 2 x - 1⎞
21. If y = (1 + x) (1 + x2) (1 + x4) ... (1 + x2n), then at We have, y = f ⎜ 2 ⎟
⎝ x + 1⎠
x = 0 is dx
⎛ 2 x - 1⎞ ⎡ ( x + 1) 2 - ( 2 x - 1) ⋅ 2 x ⎤
2
(A) – 1 (B) 1 dy
⇒ = f ′ ⎜ 2 ⎟ ⋅⎢ ⎥
(C)  0 (D)  None of these dx ⎝ x + 1⎠ ⎣ ( x 2 + 1) 2 ⎦
Solution: (B) 2
⎛ 2 x - 1⎞ ⎡ 2 + 2 x - 2 x ⎤
2
We have, = sin ⎜ 2 ⎟ ⋅ ⎢ ⎥
⎝ x + 1⎠ ⎣ ( x 2 + 1) 2 ⎦
y = (1 + x) (1 + x2) (1 + x4) ... (1 + x 2 )
n

⎡ ⎛ 2 x - 1⎞ ⎛ 2 x - 1⎞ ⎤
2
2 4
(1 - x ) (1 + x )(1 + x )(1 + x )...(1 + x ) 2n ⎢∵ f’( x ) = sin x 2 , \ f’ ⎜ 2 ⎟ = sin ⎜ 2 ⎟ ⎥
= ⎢⎣ ⎝ x + 1⎠ ⎝ x + 1⎠ ⎥
1- x  ⎦
n +1
2
1- x 1
= 23. If g is the inverse of f and f ′(x) = , then g′ (x) is
1- x 1 + x3
equal to
(1 - x ) ⋅ -2n +1 ⋅ x 2 -1 + (1 - x 2 )
n +1 n +1

dy
⇒ = 1
dx (1 - x ) 2 (A) 1 + [g (x)]3 (b)
1 + [ g ( x )]3
dy ⎤ (C) [g (x)]3 (D)  None of these
\ = 1.
dx ⎥⎦ x =0
Solution: (A)
Derivative of a Function of a Function We have, g = inverse of f = f –1
(Chain Rule)
If y is a differentiable function of t and t is a differentiable ⇒ g (x) = f –1 (x) ⇒ f [g (x)] = x
function of x i.e., y = f (t) and t = g (x), then Differentiating with respect to x, we get
dy dy dt
= . f ′ [g (x)] × g′ (x) = 1
dx dt dx
Similarly, if y = f (u), where u = g (v) and v = h (x), then, 1
\ g′ (x) = = 1 + [ g (x)]3
f’[ g ( x ) ]
dy dy du dv
= . .
dx du dv dx ⎡ 1 1 ⎤
⎢∵ f ′ ( x ) = 3
, \ f ′ [ g ( x )] = 3⎥
⎣ 1+ x 1 + [ g ( x )] ⎦

24. If u = f (x3), v = g(x2), f ′(x) = cos x and g′(x) = sin x,
Solved Examples
du
then =
⎛ 2 x - 1⎞ dy dv
22. If y = f ⎜ 2 ⎟ and f ′(x) = sin x2, then is equal to
⎝ x + 1⎠ dx 1
2
(A)  x cos x 3 cosec x 2
⎛ 2 x - 1⎞ ⎛ 2 + 2 x + x ⎞ 2
2
(A) sin ⎜ 2 ⎟ ⋅ ⎜
⎝ x + 1⎠ ⎝ ( x 2 + 1) 2 ⎟⎠ 3
(B)  x cos x 3 cosec x 2
2

Objective_Maths_JEE Main 2017_Ch 13.indd 8 01/01/2008 05:32:42


Differentiation  13.9

4/5 4/5
3 ⎛ p⎞ ⎛p⎞ p ⎛p⎞
(C)  x sec x 3cosec x 2 ⇒ f ′ ⎜ 5 ⎟ = – 5 ⎜ ⎟ ⋅ sin = – 5 ⎜ ⎟ .
2 ⎝ 2⎠ ⎝ 2⎠ 2 ⎝ 2⎠
(D)  None of these
Solution: (B) Trick(s) for Problem Solving
We have, u = f (x3)
du If y = un, where u is a function of x, then
⇒ = f ′( x 3 ) ⋅ 3 x 2 = 3x2 cos x3
dx dy dy du du ⎡ dy ⎤
= × = nu n - 1 ×  ∵ = nu n -1⎥
Also, 2
v = g(x ) dx du dx dx ⎢⎣ du ⎦

dv
⇒ = g′(x2) . 2x = 2x sin x2
dx
Derivative of Parametric Functions
du du/dx 3 x 2 cos x 3
⇒ = = Sometimes x and y are separately given as functions of
dv dv/dx 2 x sin x 2
a single variable t (called a parameter) i.e., x = f (t) and
3 y = g (t). In this case,
= x cos x 3 cosec x 2
2 dy dy / dt f ′ (t )
= =
25. Let f be a function defined for all x ∈ R. If f is differen- dx dx / dt g ′ (t )
tiable and f (x3) = x5 for all x ∈ R (x ≠ 0), then the value
d2 y d ⎛ dy ⎞
of f ′(27) is and = ⎜ ⎟
(A) 15 (B) 45 dx 2 dx ⎝ dx ⎠
(C)  0 (D)  None of these d ⎛ dy ⎞ dt d ⎛ dy ⎞ dx
= ⎜ ⎟ × = ⎜ ⎟
Solution: (A) dt ⎝ dx ⎠ dx dt ⎝ dx ⎠ dt
We have, for all x (x ≠ 0)
f (x3) = x5
Solved Examples
Differentiating with respect to x, we get dy
27. Let y = x3 – 8x + 7 and x = f (t). If = 2 and x = 3 at
5 dt
f ′(x3) × 3x2 = 5x4 ⇒ f ′(x3) = x2 dx
3 t = 0, then at t = 0 is given by
dt
3 5 2
\ f ′(27) = f ′(3 ) = (3) = 15 19
3 (A)  1 (B) 
2
⎛p ⎞ 2
26. If f (x) = sin ⎜ [ x ] - x 5 ⎟ , 1 < x < 2 and [x] denotes the (C)  (D)  None of these
⎝2 ⎠ 19
⎛ p⎞
greatest integer less than or equal to x, then f ′ ⎜ 5 ⎟ Solution: (C)
is equal to ⎝ 2⎠
We have,
4/5 4/5
⎛p⎞ ⎛p⎞ dy
(A) 5 ⎜ ⎟ (B)  – 5 ⎜ ⎟ y = x3 – 8x + 7 ⇒ = 3x2 – 8
⎝ 2⎠ ⎝ 2⎠ dx
(C)  0 (D)  None of these It is given that when t = 0, x = 3.
dy
Solution: (B) \ When t = 0, = 3 × 32 – 8 = 19.
dx
Since, 1 < x < 2. dy dy dt
Also, = (1)
\ [x] = 1 dx dx dt
dy dy
⎛p ⎞ Since, when t = 0,
= 19 and = 2,
\ f  (x) = sin ⎜ - x 5 ⎟ = cos x5 dx dt
⎝2 ⎠
2 dx 2
⇒ f  ′(x) = – sin x5 . 5x4 \ from (1), 19 = ⇒ =
dx dt dt 19

Objective_Maths_JEE Main 2017_Ch 13.indd 9 01/01/2008 05:32:47


13.10  Chapter 13

28. If x = f (t) cos t – f ′(t) sin t, y = f (t) sin t + f ′(t) cos t Step 1: Differentiate each term of the equation f (x, y) = 0
2 2 d
⎛ dx ⎞ ⎛ dy ⎞ with respect to x, keeping in mind that (y2) =
then ⎜ ⎟ + ⎜ ⎟ is equal to dx
⎝ dt ⎠ ⎝ dt ⎠ dy d 3 dy
2y ; (y ) = 3y2 and so on.
(A) f (t) – f ″ (t) (B) [ f (t) – f ″ (t)]2 dx dx dx
2 dy
(C) [ f (t) + f ″ (t)] (D)  None of these Step 2: Collect the terms containing on one side and
dy dx
Solution: (C) the terms not involving on the other side.
dx
dx
= – f (t) sin t + f ′(t) cos t – f ′(t) cos t – f ″ (t) sin t dy dy
dt Step 3: Divide by coefficient of to get as a func-
dx dx
= – [ f (t) + f ″ (t)] sin t tion of x or y or both.
dy
and = f (t) cos t + f ′(t) sin t – f ′(t) sin t  Trick(s) for Problem Solving
 dt + f ″ (t) cos t
Shorter Method for Finding the Derivative of an Implicit
= [ f (t) + f ″ (t)] cos t
Function
2 2 Step 1: Take all the terms of the function to be differentiated
⎛ dx ⎞ ⎛ dy ⎞ 2
\ ⎜⎝ ⎟⎠ + ⎜⎝ ⎟⎠ = [ f (t) + f ″ (t)] to the left hand side and put left hand side equal to
dt dt f (x, y).
29. If x = sec q – cos q, y = secnq – cosnq, then Step 2.
dy derivatie of f (x,y ) w.r.t. x treating y as constant
⎛ dy ⎞
2 =– .
(x2 + 4) ⎜ ⎟ is equal to  dx derivatie of f (x,y ) w.r.t. y treating x as constant
⎝ dx ⎠
(A) n2 (y2 – 4) (B)  n2 (4 – y2)
(C) n2 (y2 + 4) (D)  None of these
Solution: (C) Differentiation of a function with
dx respect to another function
= sec q tan q + sin q = tan q (sec q + cos q)
dq If y = f (x) and z = g (x), then in order to find the derivative
dy of f (x) with respect to g (x), we use the formula
= n [secnq tan q + cosn – 1q sinq]
dq
dy dy / dx f ′( x )
= n tan q (secnq + cosnq) = =
dz dz / dx g ′( x )
dy dy dq n tan q (sec n q + cos n q )
\ = =
dx dx dq tan q (sec q + cos q ) Solved Examples
2 2 n n 2
⎛ dy ⎞ n (sec q + cos q )
⇒ ⎜ ⎟ = 30. The derivative of f (tan x) with respect to g (sec x) at x
⎝ dx ⎠ (sec q + cos q ) 2 p
= , where f ′(1) = 2 and g′ ( 2 ) = 4, is
4
(sec n q - cos n q ) 2 + 4 1
= n2 . (A)  (B)  2
(sec q - cos q ) 2 + 4 2
n 2 ( y 2 + 4) (C)  1 (D)  None of these
=
( x 2 + 4) Solution: (A)
2 Let y = f (tan x) and u = g (sec x)
⇒ (x2 + 4) ⎛⎜ dy ⎞⎟ = n2 ( y2 + 4)
⎝ dx ⎠ dy
⇒ = f ′(tan x) sec2x
dx
du
Derivative of Implicit Functions and = g′ (sec x) . sec x tan x
dx
The derivative of an implicit function, given by the relation dy dy du f ′(tan x ) sec 2 x
f (x, y) = 0 in which y is not expressible explicitly in terms \ = =
du dx dx g ′(sec x ) sec x tan x
of x, can be found by the following steps:

Objective_Maths_JEE Main 2017_Ch 13.indd 10 01/01/2008 05:32:51


Differentiation  13.11

⎛ p⎞ Properties of Logarithms
f ′ ⎜ tan ⎟
⎛ dy ⎞ ⎝ 4⎠ 1. loge (mn) = logem + logen
\ ⎜⎝ ⎟⎠ p =
du x = ⎛ p⎞ p ⎛ m⎞
4 g ′ ⎜ sec ⎟ sin 2. loge ⎜ ⎟ = logem – logen
⎝ 4⎠ 4 ⎝ n⎠
f ′(1) 2×2 1 3. loge (m)n = n logem
= = =
1 4 2 4. logee = 1
g ′( 2 ) ⋅
2 loge m
5. logn m =
31. If y = x – x2, then the derivative of y2 with respect to loge n
x2 is 6. logn m × logm n = 1.
(A) 2x2 + 3x – 1 (B)  2x2 – 3x + 1
2
(C) 2x + 3x + 1 (D)  None of these Trick(s) for Problem Solving
Solution: (B)
Shorter Methods of Finding the Derivative of a Function
Let u = y2 and v = x2 of the from [f  (x)]g(x)
dy
du dy ⎛ dy ⎞ If y = [f  (x)]g(x), then to find , in addition to the method
Then, = 2y = 2y (1 – 2x) ⎜∵ = 1 - 2 x⎟ dx
dx dx ⎝ dx ⎠ discussed above, we can also apply any of the following two
dv methods:
and = 2x Method 1
dx
Step 1: Express y = [ f  (x)]g (x) = eg (x) log f  (x)
du du dx 2 y (1 - 2 x ) ( x - x 2 ) (1 - 2 x )
\ = = = (∵ ax = ex log a)
dv dv dx 2x x dy
(∵ y = x – x2) Step 2:  Differentiate with respect to x to obtain .
dx
= 2x2 – 3x + 1 Method 2
Step 1: Evaluate
Logarithmic Differentiation A = Differential coefficient of y treating f  (x) as constant.
Step 2: Evaluate
If differentiation of an expression or an equation is done
after taking log on both sides, then it is called logarithmic B = Differential coefficient of y treating g (x) as constant.
differentiation. This method is useful for the function hav- dy
Step 3: = A + B.
ing following forms: dx

1. y = [ f  (x)]g(x)

2. y = 1
f ( x ). f 2 ( x )...
where gi(x) ≠ 0
Solved Examples
g1 ( x ). g2 ( x )...
(where i = 1, 2, 3, ...), fi(x) and gi(x) both are 32. If f (x) = log5 log3 x, then f  ′ (e) is equal to
differentiable. (A) e log 5 (B)  – e log 5
1
(i) Case I: y = [ f  (x)]g(x), where f  (x) and g(x) are (C)  (D)  None of these
function of x. To find the derivative of this type of e log 5
functions we proceed as follows: Let y = [ f  (x)] Solution: (C)
g(x)
. Taking logarithm of both the sides, we have We have,
log y = g(x). log  f  (x) and then we differentiate ⎛ log x ⎞
f (x) = log5 log3 x = log5 ⎜
with respect to x. ⎝ log 3 ⎟⎠
f ( x ). f 2 ( x ) = log5 (log x) – log5 (log 3)
(ii) Case II: y = 1
g1 ( x ). g2 ( x ) log (log x )
= – log5 (log 3)
Taking logarithm of both the sides, we have log 5
log y = log [f1(x)] + log [f2(x)] – log [g1(x)] 1 1 1
– log [g2(x)] f  ′ (x) = ⋅ ⋅ –0
log 5 log x x
and differentiating with respect to x, we get
1 1 1 1
1 dy f ′( x ) f 2′( x ) g1′( x ) g2′ ( x ) \ f  ′ (e) = ⋅ ⋅ = .
= 1 + - - log 5 log e e e log 5
y dx f1 ( x ) f 2 ( x ) g1 ( x ) g2 ( x )

Objective_Maths_JEE Main 2017_Ch 13.indd 11 01/01/2008 05:32:55


13.12 Chapter 13

a + b +2 x
⎛ a + x⎞ dy y 2 f ′( x )
33. If f (x) = ⎜ , then f ′(0) is equal to \ =
⎝ b + x ⎟⎠ dx f ( x )[1 - y log f ( x )]

⎛ a+b 1
a a2 - b2 ⎞ ⎛ a ⎞ 3. If y = f ( x ) + ,
(A) ⎜ 2 log + ⎜ ⎟ 1
⎝ b ab ⎟⎠ ⎝ b ⎠ f ( x) +
f ( x) +
a+b ∞
⎛ a b2 - a2 ⎞ ⎛ a ⎞
(B) ⎜ 2 log + ⎜ ⎟
⎝ b ab ⎟⎠ ⎝ b ⎠ then y = f (x) +
1

dy
=
yf ′( x )
y dx 2 y - f ( x)
a+b
⎛ a a2 + b2 ⎞ ⎛ a ⎞
(C) ⎜ 2 log + ⎜ ⎟
⎝ b ab ⎟⎠ ⎝ b ⎠ differentiation of Inverse Trigonometric
(D) None of these Functions
For problems involving inverse trigonometric functions,
Solution: (B) first try for a suitable substitution to simplify it and then
We have, differentiate. If no such substitution is found, then differen-
tiate directly.
log f (x) = (a + b + 2x) [log (a + x) – log (b + x)]
Differentiating both sides with respect to x, we get I M P O R TA N T P O I N T S
f ′( x )
= 2 [log (a + x) – log (b + x)] Substitutions to Reduce the Function to a Simpler
f ( x) Form
⎡ 1 1 ⎤ Expressions Substitutions
+ (a + b + 2x) ⎢ - ⎥
⎣a + x b + x⎦
a2 - x 2 Put x = a sin q or x = a cos q
⎡ ⎛ 1 1⎞ ⎤
⇒ f ′(0) = f (0) ⎢ 2 (log a - log b) + ( a + b) ⎜ - ⎟ ⎥
⎣ ⎝ a b⎠ ⎦ x 2 - a2 Put x = a sec q or x = a cosec q
a+b
⎡ a b2 - a2 ⎤ a2 + x 2 Put x = a tan q or x = a cot q
= ⎛⎜ ⎞⎟
a
⎢ 2 log + ⎥
⎝ b⎠ ⎣ b ab ⎦ a-x a+x
or Put x = a tan q
a+x a-x
differentiation of Infinite Series
a-x a+x
If y is given in the form of an infinite series of x and we or Put x = a cos q
a+x a-x
dy
have to find out , then we remove one or more terms, it
dx
does not affect the series.
Some useful Trigonometric and Inverse
1. If y = f ( x) + f ( x) + f ( x ) + ... ∞ ,
Trigonometric Transformations
y= f ( x) + y mx
then 1. 1 + cos mx = 2 cos2
2
dy dy
⇒ y2 = f (x) + y ⇒ 2 y = f ′(x) + ; mx
dx dx 2. 1 – cos mx = 2 sin2
2
dy f ′( x ) mx
\ = 2 tan
dx 2y - 1 2
3. sin mx =
2 mx
2. If y = f ( x ) f ( x )
f (x)f (x)

then y = f (x)y 1 + tan


2
\ log y = y log f (x) mx mx
1 - tan 2 cot 2 -1
1 dy y ⋅ f ′( x ) dy 4. cos mx = 2 = 2
⇒ = + log f ( x ). mx mx
y dx f ( x) dx 1 + tan 2 cot 2 +1
2 2

Objective_Maths_JEE Main 2017_Ch 13.indd 12 01/01/2008 05:33:00


Differentiation  13.13

⎛p ⎞ 1 + tan x Differentiation of a function given in the


5. tan ⎜ + x ⎟ = form of a determinant
⎝4 ⎠ 1 - tan x
⎛p ⎞ 1 - tan x f ( x ) g ( x ) h( x )
6. tan ⎜ - x ⎟ =
⎝4 ⎠ 1 + tan x If y = p( x ) q( x ) r ( x ) , then
⎛ x+ y⎞ u( x ) v ( x ) w ( x )
7. tan–1x + tan–1y = tan–1 ⎜ ,
⎝ 1 - xy ⎟⎠
provided x, y > 0 and xy < 1 f ′( x ) g ′( x ) h ′( x )
dy
⎛ x+ y⎞ = p( x ) q( x ) r ( x )
8. tan–1x + tan–1y = p + tan–1, ⎜ dx
⎝ 1 - xy ⎟⎠ u( x ) v ( x ) w ( x )
provided x, y > 0 and xy > 1 f ( x ) g ( x ) h( x ) f ( x ) g ( x ) h( x )
⎛ x- y⎞  + p ′( x ) q ′( x ) r ′( x ) + p( x ) q( x ) r ( x )
9. tan–1x – tan–1y = tan–1 ⎜ ; if x, y > 0
⎝ 1 + xy ⎟⎠ u( x ) v ( x ) w ( x ) u ′( x ) v ′( x ) w ′( x )
p
10. sin–1x + cos–1x = = tan–1x + cot–1x Note that differentiation of a determinant can be done in
2 columns also.
–1 –1
= sec x + cosec x
11. sin–1x ± sin–1y = sin–1 ( x 1 - y 2 ± y 1 - x 2 ) , Trick(s) for Problem Solving
2 2
provided x, y ≥ 0 and x + y ≤ 1

If a is a repeated root of the equation f  (x) = 0, repeated n
12. sin–1x ± sin–1y = p – sin–1 ( x 1 - y 2 ± y 1 - x 2 ) , times, then f  (x) can be written as
if x, y ≥ 0 and x2 + y2 > 1
f  (x) = (x – a)n g(x)(1)
where g(a) ≠ 0.
13. cos–1x ± cos–1y = cos–1 ( xy ∓ 1 - x 2 1 - y 2 ) ,
Clearly, form (1)
if x, y > 0 and x2 + y2 ≤ 1

f  (a) = 0, f ′(x) = 0, f ″(a) = 0, ... f  (n – 1)(a) = 0
14. cos–1x ± cos–1y = p – cos–1 ( xy ∓ 1 - x 2 1 - y 2 ) ,
if x, y > 0 and x2 + y2 > 1

⎛ 2x ⎞
15. sin–1 ⎜ = 2 tan–1x Solved Examples
⎝ 1 + x 2 ⎟⎠
⎛ 1 - x2 ⎞ 34. If fr (x), gr (x), hr (x), r = 1, 2, 3 are polynomials in x
16. cos–1 ⎜ 2⎟
= 2 tan–1x such that fr (a) = gr (a) = hr (a), r = 1, 2, 3 and
⎝1+ x ⎠
1- x 1 f1 ( x ) f 2 ( x ) f3 ( x )
17. tan–1 = cos–1x F (x) = g1 ( x ) g2 ( x ) g3 ( x ) , then F  ′ (a) is equal
1+ x 2
to h1 ( x ) h2 ( x ) h3 ( x )

Successive Differentiation
(A) a (B)  –a
dy (C)  0 (D)  None of these
Let y = f (x) be a function of x, then is again a function
dx
of x and is called the first derivative of y with respect to x. Solution: (C)
If the first derivative is differentiable, its derivative is called We have,
second derivative of the original function and is denoted F  ′ (x) =
d2 y f1′( x ) f 2′( x ) f3′( x ) f1 ( x ) f 2 ( x ) f3 ( x )
by or y2. If the second derivative is differentiable, its
dx 2 g1 ( x ) g2 ( x ) g3 ( x ) + g1′( x ) g2′ ( x ) g3′ ( x )
derivative is called the third derivative of the original func-
h1 ( x ) h2 ( x ) h3 ( x ) h1 ( x ) h2 ( x ) h3 ( x )
d3 y 
tion and is denoted by or y3 and so on. This process of
dx 3 f1 ( x ) f 2 ( x ) f3 ( x )
differentiating a function more than once is called succes-  + g1 ( x ) g2 ( x ) g3 ( x )
sive differentiation. h1′( x ) h2′ ( x ) h3′ ( x )

Objective_Maths_JEE Main 2017_Ch 13.indd 13 01/01/2008 05:33:03


13.14  Chapter 13

\ F  ′ (a) = Solution: (C)


f1′( a) f 2′( a) f3′( a) f1 ( a) f 2 ( a) f3 ( a) We have,
g1 ( a) g2 ( a) g3 ( a) + g1′( a) g2′ ( a) g3′ ( a) D=
h1 ( a) h2 ( a) h3 ( a) h1 ( a) h2 ( a) h3 ( a) f g h

f1 ( a) f 2 ( a) f3 ( a) xf ′ + f xg ′ + g xh ′ + h
 + g1 ( a) g2 ( a) g3 ( a) x 2 f ′′ + 4 xf ′ + 2 f x 2 g ′′ + 4 xg ′ + 2 g x 2 h ′′ + 4 xh ′ + 2h
h1′( a) h2′ ( a) h3′ ( a)
f g h
= 0 + 0 + 0 [∵ fr (a) = gr (a) = hr (a), r = 1, 2, 3] = 0 = xf ′ xg ′ xh ′
35. If f (x), g (x), h (x) are polynomials in x of degree 2 and x 2 f ′′ + 4 xf ′ x 2 g ′′ + 4 xg ′ x 2 h ′′ + 4 xh ′
f g h
(Applying R2 → R2 – R1 and R3 → R3 – 2R1)
F (x) = f ′ g ′ h ′ , then F  ′ (x) is equal to
f ′′ g ′′ h ′′ f g h
(A) 1 (B) 0 = xf ′ xg ′ xh ′ (Applying R3 → R3 – 4R2)
(C)  – 1 (D)  None of these 2 2 2
x f ′′ x g ′′ x h ′′
Solution: (B)
F  ′ (x) = f g h f g h
3
f′ g ′ h′ f g h f g h = x f′ g ′ h′ = f′ g′ h′
 f′ g ′ h ′ + f ′′ g ′′ h ′′ + f ′ g ′ h′ f ′′ g ′′ h ′′ 3
x f ′′ x 3 g ′′ x 3 h ′′
f ′′ g ′′ h ′′ f ′′ g ′′ h ′′ f ′′′ g ′ ′′ h ′′′
f g h
= 0 + 0 + 0 = 0.
[Since f (x), g (x), h (x) are polynomials of degree 2, \ D′ = f′ g′ h′
\ f ″′ (x) = g″′ (x) = h″′ (x) = 0.] ( x 3 f ′′ ) ′ ( x 3 g ′′ ) ′ ( x 3 h ′′ ) ′

36. If f, g, h are differentiable functions of x and


f g h n! xn 2
D=
( xf ) ′ ( xg ) ′ ( xh) ′ np
37. If f (x) = cos x cos 4 then the value of
2
( x 2 f ) ′′ ( x 2 g ) ′′ ( x 2 h) ′′
np
then D′ (the derivative of D with respect to x) is given sin x sin 8
2
by
f′ g′ h′ dn
(A)  f g h n [f (x)]x = 0 is
dx
( x 3 f ′′ ) ′ ( x 3 g ′′ ) ′ ( x 3 h ′′ ) ′ (A) 0 (B) 1
(C)  – 1 (D)  None of these
f g h
(B)  f′ g′ h′ Solution: (A)

( x 2 f ′′ ) ′ ( x 2 g ′′ ) ′ ( x 2 h ′′ ) ′
n! n! 2
n
f g h d ⎛ np ⎞ np
[ f (x)] = cos ⎜ x + ⎟ cos 4
(C)  f′ g′ h′ dx n ⎝ 2⎠ 2
( x 3 f ′′ ) ′ ( x 3 g ′′ ) ′ ( x 3 h ′′ ) ′ ⎛ np ⎞ np
sin ⎜ x + ⎟ sin 8
(D)  None of these ⎝ 2⎠ 2

Objective_Maths_JEE Main 2017_Ch 13.indd 14 01/01/2008 05:33:06


Differentiation  13.15

Solution: (D)
n! n! 2 A( x ) B( x ) C ( x )
n
d np np Let D (x) = A(a ) B(a ) C (a ) , then
\ [ f (x)]x = 0 = cos cos 4 = 0
dx n 2 2 A′(a ) B ′(a ) C ′(a )
np np
sin sin 8 A′ ( x ) B ′ ( x ) C ′ ( x )
2 2
D′ (x) = A(a ) B(a ) C (a )
(C1 and C2 are identical)
A′(a ) B ′(a ) C ′(a )
38. If a is a repeated root of a quadratic equation f (x) = 0
So D (a) = 0 = D′ (a), therefore a is a repeated root
and A (x), B (x), C (x) be polynomials of degree > 2,
of D (x) = 0 and a is a repeated root of the quadratic
then the determinant
equation f (x) = 0, so D is divisible by f  (x).
A( x ) B( x ) C ( x )
A(a ) B(a ) C (a ) is divisible by
A′(a ) B ′(a ) C ′(a )
(A) A (x) (B) 
B (x)
(C) C (x) (D) 
f (x)

EXERCISES

Single Correct Options Type


⎛p⎞
1. If f (x) = x 2 - 10 x + 25 , then the derivative of f (x) on 4. If f (x) = cos x cos 2x cos 4x cos 8x, then f ′ ⎜ ⎟ is
⎝ 4⎠
the interval [0, 7] is (A) – 1 (B) 2
(A) 1 (B) – 1 (C)  2 (D)  None of these
(C)  0 (D)  Does not exist ⎛ d 2 ⎞⎛ 2 ⎞
y d x
5. If y = enx, then ⎜ 2 ⎟ ⎜ 2 ⎟ is equal to
2. If the capital letters denote the cofactors of the corre- ⎝ dx ⎠ ⎝ dy ⎠
sponding small letters in the determinant (A) nenx (B)  n2enx
nx
(C) – ne (D) – ne– nx
a1 b1 c1
D = , a2
b2 c2 then the value of 6. If the parametric equation of a curve is given by
q
a3 b3 c3 x = cos q + log tan and y = sin q, then the points for
2 2
d y
A1 B1 C1 which = 0 are given by
D′ = A2
B2 C2 is dx 2
(A) q = np, n∈Z
A3 B3 C3 (B) q = (2n + 1)p/2, n∈Z
(C) q = (2n + 1) p, n∈ Z
(A) 0 D2 (D) 
(B) 2D (C)  D
(D) q = 2np, n∈Z.
⎛ p⎞ ⎛ p⎞ 7. Let f (x) = x - 1 + x + 24 - 10 x - 1 ; 1 < x < 26
3. If f (x) = cos2x + cos2 ⎜ x + ⎟ + sin x sin ⎜ x + ⎟ and
⎝ 3 ⎠ ⎝ 3⎠ be real valued function. Then f  ′(x), for 1 < x < 26 is
⎛ 5⎞
g ⎜ ⎟ = 3 then (gof ) (x) is equal to 1
⎝ 4⎠ (A)  0 (B) 
(A) 1 (B) 2 x -1
(C)  3 (D)  None of these (C) 2 x - 1 - 5 (d)  None of these

Objective_Maths_JEE Main 2017_Ch 13.indd 15 01/01/2008 05:33:08


13.16  Chapter 13

(A)  – 1 if 2 ≤ x < 3 (B)  1 if 2 ≤ x < 3


⎛ log e / x 3
–1 ⎜
( )⎞⎟ + tan -1


log (e 4 x 3 )

⎟ (C)  1 for all x > 2 (D)  None of these
8. If y = tan ⎜ log (ex 3 )
d2 y
⎝ ⎟⎠ (
⎜⎝ log e / x12 ) ⎟⎠ , then 16. If f (x) = | (x – 4) (x – 5) |, then f ′ (x) is equal to
is equal to (A)  – 2x + 9, for all x∈R
dx 2 (B) 2x – 9 if 4 < x < 5
(A) 1 (B) 0 (C)  – 2x + 9 if 4 < x < 5
(C)  – 1 (D)  None of these (D)  None of these
d
x3 sin x cos x 17. If 2f (sin x) + f (cos x) = x, then f ( x ) is
dx
9. Let f (x) = 6 -1 0 , where p is a constant. (A) sin x + cos x (B) 2
p p2 p3 1
(C)  (D)  None of these
d3 1 - x2
Then [ f (x)] at x = 0 is
dx 3 18. Let f (x) = |x – a|; (a > 0) and g(x) = f  { f  [ f (x)]}. Then
(A) p (B)  p + p2 g′(a); (a > 3a)
3
(C) p + p (D)  independent of p (A)  does not exist (B)  equal to 3
10. The function y defined by the equation xy – log y = 1 (C)  equal to 1 (D)  None of these
satisfies x ( yy″ + y′ 2) – y″ + kyy′ = 0. The value k is 19. Let f (x) be the inverse of the function f (x) and
(A) – 3 (B) 3 1 d
(C)  1 (D)  None of these f  ′(x)= 5
then f ( x ) is
1+ x dx
11. If the function y(x) represented by x = sin t, 1 1
(A)  (B) 
⎛ p p⎞ 1 + [f ( x )]5
1 + [ f ( x )]5
y = aet 2 + bet 2 , t∈ ⎜ - , ⎟ satisfies the equation
⎝ 2 2⎠
(C) 1 + [f (x)]5 (D) 1 + [f (x)]5
(1 – x2) y″ – xy′ = ky, then k is equal to
(A) 1 (B) – 2 1 dy dx
20. If y = then + =
(C)  2 (D)  None of these 1+ y x 4
1 + x4
df -1 ( x )
x2 - x (A) 0 (B) 1
12. If f (x) = , then
is equal to
x2 + 2x dx x y
(C)  (D) 
3 3 y x
(A) – 2
(B)  2
(1 - x ) (1 - x )
f ( x) f ′( x )
21. If y = and z = , then
1 f( x) f ′( x )
(C)  (D)  None of these
(1 - x ) 2 f ′′ f ′′ 2( y - z )
- + (f ′ ) 2 =
13. Let F (x) = f (x) g (x) h (x) for all real x, where f (x), g f f ff
(x) and h (x) are differentiable functions. At some point d2 y 1 d2 y
(A)  (B) 
x0, if F  ′ (x0) = 21 F (x0), f ′ (x0) = 4 f (x0), g′ (x0) = – 7 dx 2 y dx 2
g (x0) and h′ (x0) = kh (x0) then k is equal to
d2 y
(A) 24 (B) 12 (C)  y (D)  None of these
(C)  – 12 (D)  – 24 dx 2
22. Let g(x) be the inverse of an invertible function f (x),
14. If f (x) is a polynomial of degree n (> 2) and f (x) = which is differentiable for all real x, then g′′(f (x))
f (k – x), (where k is a fixed real number), then degree equals
of f ′ (x) is
f ′′( x )
(A) n (B)  n–1 (A) -
(C) n – 2 (D)  None of these [ f ′( x )]3

15. If f (x) = | x – 1 | and g (x) = f { f [ f (x)]}, then for x > 2, f ′( x ) f ′′( x ) - [ f ′( x )]2
(B)
g′ (x) is equal to f ′( x )

Objective_Maths_JEE Main 2017_Ch 13.indd 16 01/01/2008 05:33:12


Differentiation  13.17

f ′( x ) f ′′( x ) - [ f ′( x )]2 d2x


(C)  31. equals
[ f ′( x )]2 dy 2
-1
(D)  None of these ⎛ d 2 y ⎞ ⎛ dy ⎞ -3 ⎛ d2 y⎞
(A) - ⎜ 2 ⎟ ⎜ ⎟ (B)  ⎜ 2 ⎟
⎝ dx ⎠ ⎝ dx ⎠ ⎝ dx ⎠
23. Let f (x) = xn, n being a non-negative integer. The value
of n for which equality ⎛ d2 y⎞
-1 -3
⎛ d 2 y ⎞ ⎛ dy ⎞ -2
⎛ dy ⎞
f ′(a + b) = f ′(a) + f ′(b) is valid for all a, b > 0 is (C) - ⎜ 2 ⎟ ⎜⎝ ⎟⎠ (D) 
⎜⎝ 2 ⎟⎠ ⎜⎝ ⎟⎠
⎝ dx ⎠ dx dx dx
(A) 5 (B) 1
(C) 2 (D) 4 32. Let y be an implicit function of x defined by,
24. The solution set of f  ′(x) >  g′(x) where f  (x) = (1/2)52x + 1 x2x – 2xx cot y – 1 = 0
and g(x) = 5x + 4x log 5 is Then y′(1) equals
(A) (1, ∞) (B)  (0, 1) (A) – 1 (B) 1
(C) (0, ∞) (D)  [0, ∞) (C)  log 2 (D)  – log 2

dn 33. If y and z are the functions of x and if y2 + z2 = l2, then


n
25. If In =( x log x ), then In – nIn – 1 =
dx n d ⎛ y ⎞ d ⎛ z2 ⎞
y ⎜ ⎟+ is equal to
(A) n (B)  n–1 dx ⎝ l ⎠ dx ⎜⎝ l ⎟⎠
(C) n! (D)  (n – 1)!
z dz z dx
(A)  (B) 
26. If u = f (x3), v = g(x2), f  ′(x) = cos x and g′(x) = sin x l dx l dz
du l dz
then is (C)  (D)  None of these
dv z dx
3
(A)  x ⋅ cos x 3 ⋅ cosec x 2 34. If Sn denotes the sum of n terms of a G.P. whose com-
2 dSn
2 mon ratio is r, then (r – 1) is equal to
(B)  sin x 3 ⋅cosec x 2 dr
3 (A) (n – 1) Sn + nSn – 1 (B) (n – 1) Sn – nSn – 1
(C) tan x (C) (n – 1) Sn (D)  None of these
(D)  None of these
⎛ x + x2 + ... + xn ⎞
27. If f (x) be a differentiable function such that f (xy) = f (x) 35. Let f ⎜ 1 ⎟⎠ =
+ f (y) for all x and y, then f (e) + f (1/e) = ⎝ n
(A) 1 (B) 0 f ( x1 ) + f ( x2 ) + ... + f ( xn )
,
(C)  –1 (D)  None of these n
where all xi∈R are independent to each other and n∈N.
28. If for all x, y the function f is defined by f (x) + f (y) + If f (x) is differentiable and f  ′(0) = a, f (0) = b then f  ′(x)
f (x) .  f (y) = 1 and f (x) > 0 then is equal to
(A) f ′(x) does not exist (A) a (B)  0
(B) f ′(x) = 0 for all x (C) b (D)  None of these
(C) f ′(0) < f  ′(1)
(D)  None of these 36. If y2 = P (x), a polynomial of degree n≥ 3, then

29. Let 3f (x) – 2f (1/x) = x, then f ′(2) is equal to d ⎛ 3 d2 y⎞


2 y =
dx ⎜⎝ dx 2 ⎟⎠
2 1 7
(A)  (B)  (C) 2 (D)  (A) – P (x) × P″′ (x) (B) P (x) × P′″ (x)
7 2 2
(C) P (x) × P″ (x) (D)  None of these
d2 y 37. A function f (x) is so defined that for all x, [ f (x)]n =
30. If x+ y+ y - x = c then 2
equals
dx f (nx). If f ′ (x) denotes derivative of f (x) with respect to
x, then f ′ (x) × f (nx) =
2 -2 2 -2
(A)  2
(B)  2
(C)  (D)  (A) f (x) (B)  0
c c c c
(C) f (x) × f ′ (nx) (D)  None of these

Objective_Maths_JEE Main 2017_Ch 13.indd 17 01/01/2008 05:33:16


13.18  Chapter 13

38. If f, g, h are differentiable functions of x and 42. If f (x) is a polynomial of degree n (> 2) and
f g h f (x) = f (k – x), (where k is a fixed real number), then
degree of f ′ (x) is
D=
( xf ) ′ ( xg ) ′ ( xh) ′
(A) n (B)  n–1
( x 2 f ) ′′ ( x 2 g ) ′′ ( x 2 h) ′′ (C) n – 2 (D)  None of these
f ( x) f ′( x )
then D′ (the derivative of D with respect to x) is given 43. If y = and z = , then
by f( x) f ′( x )
f ′′ f ′′ 2( y - z )
f′ g′ h′ - + (f ′ ) 2 =
f f ff
(A)  f g h
( x 3 f ′′ ) ′ ( x 3 g ′′ ) ′ ( x 3 h ′′ ) ′ d2 y 1 d2 y
(A)  (B) 
dx 2 y dx 2
f g h
d2 y
(B)  f′ g′ h′ (C)  y (D)  None of these
dx 2
( x 2 f ′′ ) ′ ( x 2 g ′′ ) ′ ( x 2 h ′′ ) ′
44. The solution set of f ′(x) > g′(x) where f (x) = (1/2)52x + 1
f g h and g(x) = 5x + 4x log 5 is
(C)  f′ g′ h′ (A) (1, ∞) (B)  (0, 1)
(C) (0, ∞) (D)  [0, ∞)
( x 3 f ′′ ) ′ ( x 3 g ′′ ) ′ ( x 3 h ′′ ) ′
45. If for all x, y the function f is defined by f (x) + f (y) +
(D)  None of these f (x) .  f (y) = 1 and f (x) > 0, then
39. If a is a repeated root of a quadratic equation f (x) = 0 (A) f ′(x) does not exist
and A (x), B (x), C (x) be polynomials of degree (B) f ′(x) = 0 for all x
A( x ) B( x ) C ( x ) (C) f ′(0) < f ′(1)
> 2, then the determinant A(a ) B(a ) C (a ) is (D)  None of these
A′(a ) B ′(a ) C ′(a ) d2 y
46. If x+ y+ y - x = c then equals
divisible by dx 2
(A) A (x) (B)  B (x)
(C) C (x) (D)  f (x) 2 -2
(A)  (B) 
2
c c2
40. If the capital letters denote the cofactors of the corre-
sponding small letters in the determinant 2 -2
(C)  (D) 
a1 b1 c1 c c
n
D = a2 b2 c2 , then the value of
a3 b3 c3
47. Let f (x) = ∏ (cos (2k - 1) x + i sin (2k - 1) x) , then
k =1

A1 B1 C1 (Re f (x))′′ + i(Im f (x))′′ is equal to


D′ = A2
(A) n2f (x) (B)  – n4f (x)
B2 C2 is 2
(C) – n f (x) (D)  n4f (x)
A3 B3 C3
d
(A) 0 (B) 2D 48. If 2f (sin x) + f (cos x) = x, then f ( x ) is
(C) D2 (D) D dx
(A) sin x + cos x (B) 2
x2 - x df -1 ( x )
41. If f (x) = 2 , then is equal to 1
x + 2x dx (C)  (D)  None of these
1 - x2
3 3
(A) – 2
(B)  2
(1 - x ) (1 - x )
49. Let f (x) = x -1 + x + 24 - 10 x - 1 ; 1 < x < 26
1
(C)  (D)  None of these be a real valued function. Then, f  ′(x), for 1 < x < 26 is
(1 - x ) 2

Objective_Maths_JEE Main 2017_Ch 13.indd 18 01/01/2008 05:33:20


Differentiation  13.19

(A) n (B) 0
1

(A) 
0 (B)  (C) 2n (D) 
2n – 1
x -1
(C) 2 x - 1 - 5 (D)  None of these ⎛ x x - x-x ⎞
dy
–1 ⎜
58. If y = cot ⎝ ⎟ , then at x = 1, equals
2 ⎠ dx
50. If f (x) = | x – 2 | and g(x) = f {f (x)}, then g′(x) for x > 2
is (A) 0 (B) 1
(A) – 1 (B) 1 (C)  –1 (D)  None of these
(C)  0 (D)  does not exist
1 + cos 2q
59. If y = , then
51. The derivative of the function represented parame-­ 1 - cos 2q
trically as x = 2t – | t |, y = t3 + t2 | t | at t = 0 is
⎛p⎞ ⎛ 3p ⎞
(A) 0 (B) 1 (A) y′ ⎜ ⎟ = y′ ⎜ ⎟
⎝ 4⎠ ⎝ 4⎠
(C)  –1 (D)  does not exist
52. A polynomial f (x) leaves remainder 15 when divided ⎛p⎞ ⎛ 3p ⎞
(B) y′ ⎜ ⎟ · y ′ ⎜ ⎟ = – 4
by (x – 3) and (2x + 1) when divided by (x – 1)2. When ⎝ 4⎠ ⎝ 4⎠

f is divided by (x – 3) (x – 1)2, the remainder is ⎛p⎞ ⎛ 3p ⎞
(A) 2x2 + 2x + 3 (B)  2x2 – 2x – 3 (C)  y ′ ⎜ ⎟ and y ′ ⎜ ⎟ do not exist
2
⎝ 4⎠ ⎝ 4⎠
(C) 2x – 2x + 3 (D)  None of these
(D)  None of these
53. If for a non-zero x, the function f (x) satisfies the
60. Let f (x) be a polynomial function of second degree.
equation
If f (1) = f (–1) and a1, a2, a3 are in A. P., then f  ′(a1),
⎛ 1⎞ 1 f  ′(a2), f  ′(a3) are in
af (x) + bf ⎜ ⎟ = – 5 (a ≠ b),
⎝ x⎠ x
(A)  A.P. (B)  G.P.
then f ′(x) is equal to (C)  H.P. (D)  None of these
1 ⎛ a ⎞ 1 ⎛ a ⎞
(A)  2 2 ⎜
+ b⎟ (B)  ⎜ + b⎟⎠ ⎛ 1⎞ dy
b -a ⎝ x 2 ⎠ a - b ⎝ x2
2 2 61. If 5f (x) + 3f ⎜ ⎟ = x + 2 and y = x f (x) then at
⎝ x⎠ dx
⎛ a
1 ⎞ x = 1 is equal to
(C)  2 ⎜⎝ 2 - b⎟⎠
2
(D)  None of these
a -b x (A) 1 (B) –1
d3x 7 7
54. If x = cos7q and y = sinq, then = (C)  (D)  –
dy 3 8 8
105 105 62. Let f (x) be a polynomial function of degree 2 and f (x)
(A)  sin 4q (B) sin 2q > 0 for all x ∈R. If g(x) = f (x) + f  ′(x) + f ″(x), then for
4 2
any x
105
(C)  cos 4q (D)  None of these (A) g(x) > 0 (B)  g(x) < 0
4 (C) g(x) = 0 (D)  g(x) ≤ 0
⎛ x + 1⎞ -1 ⎛ x - 1⎞ 63. Let f (x + y) = f (x) + f (y) + 2xy – 1 ∀x, y ∈R. If f (x) is
dy
55. If y = sec–1 ⎜⎝ x - 1 ⎟⎠ + sin ⎜⎝ x + 1⎟⎠ , then = differentiable and f  ′(0) = sinq, then
dx
(A) f (x) > 0 ∀x ∈R (B) f (x) < 0 ∀x ∈R
(A) 0 ∀x ∈R (B) 0 ∀x ∈ (0, ∞) (C) f (x) = sin q ∀x ∈R (D)  None of these
(A) 0 ∀x ∈R – {0} (D)  None of these
64. If f (x) = x3 + x2f  ′(1) + x f  ″(2) + f  ″′(3) for all x ∈R,
⎛ x -1 - x ⎞ then which of the following is false?
56. If f (x) = cos ⎝ x -1 + x ⎟⎠ , then f  ′(x) is
–1 ⎜
(A) f (0) + f (2) = f (1) (B)  f (0) + f (3) = 0
(A) odd (B) even (C) f (1) + f (3) = f (2) (D)  f (1) + f (3) = f (0)
(C)  periodic (D)  None of these
⎛ x + y⎞ 1
65. Let f ⎜ = [ f ( x ) + f ( y )] for real x and y. If
57. If f (x) = (1 – x)n, then the value of ⎝ 2 ⎟⎠ 2
f ′′(0) f n ( 0) f  ′(0) exists and equals –1 and f (0) = 1 then the value
f (0) + f  ′(0) +
+ ... + is of f (2) is
2! n!

Objective_Maths_JEE Main 2017_Ch 13.indd 19 01/01/2008 05:33:23


13.20  Chapter 13

(A) 1 (B) –1
(C)  0 (D)  None of these a-x

(A) 
1 (B) 
x-b
66. Let the function f satisfy the equation
f (x + y) = f (x) f (y) for all x and y and f (x) = 1 + xg(x) 1
(C)  ( a - x ) ( x - b) (D) 
where lim g ( x ) = log a. If f  n(x) = k f (x), then k = ( a - x ) ( x - b)
x →0

(A) log a (B)  n log a


⎛ 1 ⎞ d2 y dy
(C) (log a)n (D)  n(log a)n 70. If y3 – y = 2x, then ⎜ x 2 - ⎟ +x =
⎝ 27 ⎠ dx 2 dx
67. Let f be a differentiable function satisfying f (x + y) =
y
1 (A) y (B) 
f (x) + f (y) + xy. If lim f ( h) = 3 , then 3
h→ 0 h
y y
(A) f (x) = 3x (B)  f (x) = 3x + x2 (C)  (D) 
9 27
x2
(C) f (x) = 3x + (D)  None of these
2 1 - 2x 2 x - 4 x3 4 x3 - 8x7
71. If x < 1, then + + + ...∞ =
68. If f (x) = x + tan x and f is the inverse of g, then g′(x) 3 is 3 7 1 - x + x 2 1 - x 2 + x 4 1 - x 4 + x8
equal to 1 - 2 x 2 x - 4 x 4 x - 8 x
+ + + ...∞ =
1 1 - x + x 2 1 - x 12 + x 4 1 - x 4 + x8
(A)  (B)  1 1 + 2x
1 + [ g ( x ) - x ]2 2 - [ g ( x ) - x ]2 (A)  (B)  2
2
1 1+ x + x 1+ x + x
(C)  (D)  None of these 2
2 + [ g ( x ) - x ]2 1- x + x
(C)  (D) 1
a-x 1 + x + x2
69. If y = ( a - x ) ( x - b) - ( a - b) tan–1 x - b , then
dy
=
dx

More than One Option Correct Type

72. If f (x) = xm, m being a non-negative integer, then the 76. If f (x – y), f (x) . f (y) and f (x + y) are in A. P. for all x, y
value of m for which f ′ (a + b) = f ′ (a) + f ′ (b ), for all and f (0) ≠ 0, then
a, b > 0, is (A) f ′(3) + f ′(–3) = 0 (B)  f (3) + f (– 3) = 0
(A) 1 (B) 2 (C) f  ′(2) + f  ′(– 2) = 0 (D)  f  ′(3) = f  ′(– 3)
(C)  0 (D)  None of these
77. A function f : (0, ∞) →R satisfies the equation
73. If f (x) = x3+ x2f  ′(1) + xf  ′′(2) + f  ′′′(3) for all x∈ ⎛ x⎞
where f (x) is a polynomial of degree 3, then f (xy) = 2f (x) – f ⎜ ⎟ .
⎝ y⎠
(A) f (0) + f (2) = f (1) (B)  f (0) + f (3) = 0 If f is differentiable on R and f (1) = 0, f ′(1) = 1, then
(C) f (1) + f (3) = f (2) (D)  None of these
⎛ 1⎞ 1
74. If f (x – y), f (x) . f (y) and f (x + y) are in A.P. for all x, y (A) f (y) = –f ⎜ ⎟ (B)  f  ′(x) =
⎝ y⎠ x
and f (0) ≠ 0, then x
(C) f (x) = ln x (D)  f (x) = e
(A) f (2) = f (– 2) (B)  f (3) + f (– 3) = 0
(C) f  ′(2) + f  ′(– 2) = 0 (D)  f  ′(3) = f  ′(– 3) 78. If f : R →R be a function such that f (x) = x3 + x2f ′(1) +
x f ″(2) + f ′″(3) is differentiable for every x ∈R, then
75. If f (x) + f (y) + f (z) + f (x) . f (y) . f (z) = 14 for all
(A) f ′(1) = –5 (B)  f ″(2) = 2
x, y, z ∈R, then
(C) f ′″(3) = 6 (D)  None of these
(A) f (0) = 2
(B) f ′ (x) = 0, for all x ∈ R 79. If f (x – y) + f (x + y) = 2 f (x) f (y) ∀x, y ∈R,then
(C) f ′ (x) > 0, for all x ∈ R (A) f is even (B)  f is odd
(D)  None of these (C) f ′ is even (D)  f ′ is odd

Objective_Maths_JEE Main 2017_Ch 13.indd 20 01/01/2008 05:33:27


Differentiation  13.21

80. If f (x) = x2 + xg′(1) + g″(2) and g(x) = f (1) · x2 + xf  ′(x) (A) 1 (B) –1
+ f ″(x), then (C) 0 (D) independent of a
(A) f (x) = x2 – 3x (B)  f (x) = x2 + 3x 84. Let f (x) = x3 + 3x2 – 33x – 33 for x > 0 and g be its
(C) g(x) = 3x + 2 (D)  g(x) = –3x + 2 inverse, then the value of k such that kg′(2) = 1 is equal
to
n
1
81. If ∑ rx r -1 =
(1 - x ) 2
⋅{1 + ax n + bx n +1}, then (A) –36
(C) 72
(B) 51
(D) 36
r =1

(A) a = (n + 1) (B)  b = n 85. If F (x) = f (x) g(x) and f  ′(x) g′(x) = c, then
(C) a = – (n + 1) (D)  b = – n
⎛ f g⎞
(A) F  ′ = c ⎜ +
82. If y = f (x) = min f (t); –3 ≤ t ≤ x where f (x) = || x – 1 ⎝ f ′ g ′ ⎟⎠
| – | x + 1||, then
(A) f (x) is non-differentiable at x = 0, –1 F ′′ f ′′ g ′′ 2c
(B)  = + +
(B) f (x) is non-differentiable at x = 1, –1 F f g fg

(C) f ″(100) = 0 F ′′′ f ′′′ g ′′′
10 (C)  = +
F f g
(D)  ∫ f ( x ) dx = 5
-3 F ′′′ f ′′′ g ′′′
(D)  = +
x n
sin x - cos x F ′′ f ′′ g ′′
83. If f (x) = n ! sin ( np 2) cos ( np 2) , then f n(0) for
a a2 a3
n = 2m + 1 is

Passage Based Questions


Passage 1 Passage 2
⎛ p p⎞
Let f be a function such that f : (– 1, 1) → ⎜ - , ⎟ . Let f Let f : R  →  R be a function satisfying the condition
⎝ 2 2⎠
⎛ x + y⎞ f ( x) + f ( y)
f ⎜ = , where k ≠ 0, 2. The function
satisfy the equation f (x) + f (y) = f ( x 1 - y2 + y 1 - x2 ) ⎝ k ⎟⎠ k
f (x) is differentiable on R and f ′(0) = m.
86. The function f (x) is
89. f ′(x) is equal to
(A) even (B) odd
(A) m (B)  2m
(C)  constant (D)  None of these
(C) m + 1 (D)  0
87. If f (x) is differentiable on (– 1, 1) and f ′(0) = 1, then 90. The function f (x) is equal is
f ′(x) is equal to (A) mx (B)  mx + 1
1 1 (C)  – 2m x (D)  None of these
(A)  -
(B) 
2
1- x 1 - x2 Passage 3
1 1 A function f : R→ [1, ∞) satisfies the equation f (xy) = f (x)
(C)  -
(D)  f (y) – f (x) – f (y) + 2. The function f is differentiable on R
2
1+ x 1 + x2 and f (2) = 5.

88. The function f (x) is equal to 91. f ′(x) is equal to


(A) cos–1 x f ( x) - 1 f ( x) + 1
(A)  f ′(1) (B)  f ′(1)
(B) sin–1 x x x
(C) tan–1x 1 - f ( x)
(C)  f ′(1) (D)  None of these
(D) sec–1 x x

Objective_Maths_JEE Main 2017_Ch 13.indd 21 01/01/2008 05:33:29


13.22  Chapter 13

92. f (x) is equal to


(A) x2 – 1 (B)  1 – x2 ∂ ⎛ ∂z ⎞ ∂2 z
(C) x2 + 1 (D)  None of these or fxy, , denoted by or fyy are called second
∂y ⎜⎝ ∂y ⎟⎠ ∂y 2
Passage 4 order partial derivatives of z = f (x, y).
⎛ p p⎞
A function f: R → ⎜ - , ⎟ satisfies the equation
⎝ 2 2⎠ x y ∂u ∂u
96. If u = sin -1 + tan -1 , then x +y =
⎛ x + y⎞ y x ∂x ∂y
f (x) + f (y) = f ⎜ . The function f is differentiable on
⎝ 1 - xy ⎟⎠ (A) 0 (B) 1
(C)  – 1 (D)  None of these
R and f ′(0) = 2.
Passage 6
93. The function f is
If U and V are two functions of x having derivatives of the
(A)  an even function (B)  an odd function nth order, then (UV)n = UnV + nC1Un–1V1 + nC2Un–2V2 + ...
(C)  a constant function (D)  None of these + nCrUn–rVr + ... + nCn UVn.
94. f ′(x) is equal to dn y
97. If y = x2 sin x, then = ( x 2 - n2 + n) sin
1 1 dx n
(A)  (B) 
2 ⎛ np ⎞ ⎛ np ⎞
1+ x 1 - x2 ⎜⎝ x + ⎟⎠ + k cos ⎜⎝ x + ⎟ , where k =
2 2⎠
2 2
(C)  (D) 
2 2
(A) nx (B)  2nx
1+ x x -1 (C) – nx (D) 
– 2nx
95. f (x) is equal to n
⎛ y⎞ ⎛ x⎞
(A) tan–1x (B)  2 tan–1 x 98. If cos -1 ⎜ ⎟ = log ⎜ ⎟ , then x2yn+2 + (2n + 1)x yn+1
⎝ b⎠ ⎝ n⎠
(C)  4 tan–1 x (D)  None of these
+ kyn = 0 where k =
Passage 5 (A) n2 (B)  2n2
2
Let z = f (x, y) be a function of two variables x and y. (C) – n (D)  – 2n2
The partial derivative with respect to x of the function z = 99. If f (x) = tan x, then
f ( x + Dx, y ) - f ( x, y ) f n (0) – nC2  f n–2 (0) + nC4  f n–4 (0) – ... =
f (x, y) at (x, y) is defined as lim ,
Dx → 0 Dx
∂z np np
provided the limit exists and is finite. It is denoted by (A) sin (B)  cos
∂x 2 2
∂f ∂z np
or or fx. Clearly, is the derivative of z = f (x, y) with (C) tan (D)  None of these
∂x ∂x 2
respect to x, regarding y as a constant. Similarly, we can
∂z ∂ ⎛ ∂z ⎞ ∂2 z ∂ ⎛ ∂z ⎞ dn
define . , denoted by or fxx, 100. If In = ( x n log x ) , then In = n In–1 + k, where k =
⎜⎝ ⎟⎠ ⎜ ⎟, dx n
∂y ∂x ∂x ∂x 2 ∂y ⎝ ∂x ⎠
∂2 z ∂ ⎛ ∂z ⎞ ∂2 z (A) n ! (B)  (n – 1) !
denoted by or fyx, ⎜ ⎟ , denoted by (C) (n – 2) ! (D)  None of these
∂y ∂x ∂x ⎝ ∂y ⎠ ∂x ∂y

Objective_Maths_JEE Main 2017_Ch 13.indd 22 01/01/2008 05:33:33


Differentiation  13.23

Match the Column Type


101. 102.
Column-I Column-II Column-I Column-II
I. The derivative of f (tan x) with (A) 3  I. The function y defined by the equa- (A) 24
p tion xy – log y = 1 satisfies x(yy″ +
respect to g (sec x) at x = ,
4 y′2) – y″ + k yy′ = 0. The value of k is
( )
where f ′(1) = 2 and g′ 2 = 4,  II. If the function y(x)
­represented by x = sin t, y =
(B) 2

is ⎛ p p⎞
aet 2 + bet 2 , t ∈ ⎜ - , ⎟
II. Let y = x3 – 8x + 7 and x = f (t). (B)  – 4 ⎝ 2 2⎠
dy s­ atisfies the equation (1 – x2)y″ – xy′
If = 2 and x = 3 at t = 0, then
dt = ky, then k is equal to
dx
at t = 0 is III. Let F (x) = f (x) g(x) h(x) for all real (C) 4
dt x, where f (x), g(x) and h(x) are dif-
2 ferentiable functions. At some point
III. Let f (x) = sin x, g(x) = 2x and h(x) (C) 
19 x0, if F  ′(x0) = 21 F (x0), f ′(x0) = 4
= cosx. If f (x) = [go (f h)] (x), f (x0), g′(x0) = – 7g(x0) and h′(x0) =
⎛p⎞ kh(x0) then k is equal to
then f ′′ ⎜ ⎟ is equal to
⎝ 4⎠   IV. Let f (x) = xn, n being a non-negative (D) 3
2 2⎛ p⎞ 1⎛ p⎞ integer. The number of values of
  IV. If f (x) = cos x + cos ⎜ x + ⎟ + sin x sin ⎜ x + ⎟
(D) 
⎝ 3⎠ ⎝2 3⎠ n for which the equality f ′(a + b)
2 2⎛ p⎞ ⎛ p⎞ ⎛ 5⎞ = f ′(a) + f ′(b) is valid for all a, b
cos x + cos ⎜ x + ⎟ + sin x sin ⎜ x + ⎟ and g ⎜ ⎟
⎝ 3⎠ ⎝ 3⎠ ⎝ 4⎠ > 0, is
= 3, then (g o f  ) (x) is equal to

Assertion-Reason Type
Instructions: In the following questions an Assertion (A) is Reason: f (x) = ± xn + 1
given followed by a Reason (R). Mark your responses from
104. Assertion: If y = (1 + x) (1 + x2) (1 + x4)... (1 + x2n),
the following options:
dy
(A)  Assertion(A) is True and Reason(R) is then at x = 0 is 1.
True; Reason(R) is a correct explanation for dx n +1

Assertion(A) 1 - x2
Reason: y =
(B)  Assertion(A) is True, Reason(R) is True; 1- x
Reason(R) is not a correct explanation for 105. Assertion: If f (x) = (cos x + i sin x) (cos 2x + i sin 2x)
Assertion(A) (cos 3x + i sin 3x) ... (cos nx + i sin nx) and f (1) = 1
(C) Assertion(A) is True, Reason(R) is False 2
⎛ n ( n + 1) ⎞
(D)  Assertion(A) is False, Reason(R) is True then f ″(1) is equal to - ⎜ ⎟ .
⎝ 2 ⎠
103. Assertion: Let f (x) be a polynomial function
satisfying n ( n - 1) n ( n - 1)
Reason: f (x) = cos x + i sin x
2 2
⎛ 1⎞ ⎛ 1⎞
f ( x ) . f ⎜ ⎟ = f ( x ) + f ⎜ ⎟ . If f (4) = 65 and l1, l2,
⎝ x⎠ ⎝ x⎠
l3 are in G.P., then f ′(l1), f ′(l2), f ′(l3), are also in G.P.

Objective_Maths_JEE Main 2017_Ch 13.indd 23 01/01/2008 05:33:35


13.24  Chapter 13

Previous Year’s Questions


d2 y dy
106. If y = (x + 1 + x 2 )n , then(1 + x2) 2
+x is : 113. If f is a real-valued differentiable function satisfying
dx dx | f (x) -f (y)| ≤ (x-y)2, x, y ∈R and f (0) = 0, then f (1)
 [2002]
equals [2005]
(A) n2y (B) 
–n2y
(A) -1 (B)  0
2x2y
(C) –y (D) 
(C) 2 (D) 1
dy
107. If sin y = x sin (a + y), then is : [2002] x
dx 114. The set of points where f ( x ) = is differentia-
1+ | x |
sin a sin 2 (a + y ) ble is [2006]
(A)  2 (B) 
sin (a + y ) sin a (A) (-∞, 0) ∪ (0, ∞)
(B) (-∞, -1) ∪ (-1, ∞)
sin 2 (a - y ) (C) (-∞,∞)
(C) sin a sin2 (a + y) (D) 
sin a (D) (0, ∞)
dy
108. If xy= ex-y, then
dy
is: [2002] 115. If xm . ym = (x + y)m + n, then is [2006]
dx dx
1+ x 1 - log x
(A)  (B)  y x+ y
1 + log x 1 + log x (A)  (B) 
x xy
log x
(C)  not defined (D)  x
(1 + log x ) 2 (C) xy (D) 
y
109. Let f (x) be a polynomial function of second degree.
116. Let y be an implicit function of x defined by x2x - 2xx
If f (1) = f (-1) and a, b, c are in A. P., then f  ′(a),
coty - 1 = 0. Then y  ′(1) equals [2009]
f  ′(B) and f  ′(C)  are in [2003]
(A) - 1 (B)  1
(A) A.P.
(C)  log 2 (D)  - log 2
(B) G.P.
(C)  H. P. 117. Let f : (-1, 1) → R be a differentiable function such
(D)  arithmetic-geometric progression that  f (0) = -1 and f  ′(0) = 1. Let g(x) = [f (2f (x) + 2)]2.
Then g  ′(0) = [2010]
110. If f (x) = x, then the value of
(A) -4 (B)  0
f ′(1) f ′′(1) f ′′′(1) ( -1) n f n (1)
f (1) -
+ - + ... + is (C) -2 (D)  4
1! 2! 3! n!
 [2003] d2x
n n-1 118. is equal to [2011]
(A) 2 (B)  2 dy 2
(C) 0 (D) 1 -1 -3
⎛ d2 y⎞ ⎛ dy ⎞
dy (A) - ⎜ 2 ⎟ ⎜⎝ ⎟⎠
111. If x = e y + e
y + ... to ∞

, x > 0, then is [2004] ⎝ dx ⎠ dx


dx
x 1
(A) (B)  ⎛ d 2 y ⎞ ⎛ dy ⎞ -2
1+ x x (B) ⎜ 2 ⎟ ⎜ ⎟
1- x 1+ x ⎝ dx ⎠ ⎝ dx ⎠
(C) (D) 
x x ⎛ d 2 y ⎞ ⎛ dy ⎞ -3
(C) - ⎜ 2 ⎟ ⎜ ⎟
112. Suppose f (x) is differentiable x = 1 and ⎝ dx ⎠ ⎝ dx ⎠
1
lim f (1 + h) = 5, then f  ′(1) equals [2005] -1
h→ 0 h ⎛ d2 y⎞
(D) ⎜ 2 ⎟
(A) 3 (B) 4 ⎝ dx ⎠
(C) 5 (D) 6

Objective_Maths_JEE Main 2017_Ch 13.indd 24 01/01/2008 05:33:39


Differentiation  13.25

dy 1
119. If y = sec(tan -1 x ) , then at x = 1 is equal to 120. If g is the inverse of a function f and f ′( x ) = ,
 dx [2013] 1 + x5
1 then g′(x) is equal to [2014]
(A)  (B)  1
2 (A) 1 + x5 (B) 
5x4
1
(C)  2 (D) 
1
2 1 + {g ( x )}
5
(C) (D) 
1 + {g ( x )}
5

Answer keys

Single Option Correct Type


1. (D) 2. (C)  3.  (C)  4. (C)  5. (D)  6. (A) 7.  (A)  8. (B)  9.  (D)  10. (B)
11.  (C)  12. (B)  13. (A)  14.  (B) 15. (A)  16.  (C)  17. (C)  18.  (C)  19. (C) 20. (A) 
21. (B)  22.  (A)  23. (C) 24.  (C)  25. (D)  26.  (A)  27. (B)  28.  (B)  29. (B)  30.  (A)
31. (A)  32.  (A) 33. (A)  34.  (B)  35. (A)  36.  (B)  37. (C)  38.  (C) 39.  (D)  40. (C) 
41. (B) 42. (B)  43.  (B)  44. (C)  45.  (B)  46. (A)  47.  (B)  48. (C)  49.  (A) 50. (D) 
51.  (A)  52. (C)  53.  (A)  54. (A)  55.  (B)  56. (A)  57.  (B) 58. (C)  59.  (B)  60. (A) 
61.  (C)  62. (A)  63.  (A)  64. (D)  65.  (B) 66. (C)  67.  (C)  68. (C)  69. (B)  70. (C) 
71.  (B)

More than One Option Correct Type


72.  (B) and (C) 73.  (A), (B) and (C) 74.  (A) and (C) 75.  (A) and (B) 76.  (A) and (C)
77.  (A), (B) and (C) 78.  (A), (B) and (C) 79.  (A) and (D) 80.  (A) and (D) 81.  (B) and (C)
82.  (A), (C) and (D) 83.  (C) and (D) 84.  (A), (B) and (C) 85.  (A), (B) and (C)

Passage Based Questions


86.  (B)  87. (A)  88. (B)  89. (A)  90.  (A) 
91. (A)  92. (C)  93. (B) 94.  (C)  95. (B) 
96.  (A)  97. (D)  98. (B)  99. (A) 100. (B)

Match the Column Type


101.  I → (D), II → (C), III → (B), IV → (A) 102. I → (D), II → (B), III → (A), IV → (B)

Assertion-Reason Type
103. (A)  104. 
(A)  105. (C)

Previous Year’s Questions


106. (A) 107. (B) 108. (D) 109.  (A) 110.  (C)
111.  (C) 112.  (C) 113. (B) 114. (C) 115. (A)
116. (A) 117. (A) 118. (C) 119. (D) 120. (D)

Objective_Maths_JEE Main 2017_Ch 13.indd 25 01/01/2008 05:33:41


13.26  Chapter 13

Hints and Solutions

Single Option Correct Type


dy d2y
1. We have, 5. y = enx ⇒ = n ⋅ e nx , = n2 × enx
dx dx 2
f (x) =
x 2 - 10 x + 25 = ( x - 5) 2 = | x – 5 |
dx 1
⎧ x - 5, x ≥ 5⎫
Now, nx = log y ⇒ n ⋅ =

= ⎨ ⎬ dy y
⎩5 - x, x < 5⎭

Clearly, f (x) is differentiable at all points on the interval d2x 1 d2x
⇒ n = - ⇒n = – e– 2nx
[0, 7] except at x = 5. dy 2
y 2
dy 2
∴ The derivative of f (x) on the interval [0, 7] does not exist.

d2 y d2x ⎛ - e - 2 nx ⎞ – nx

The correct option is (D)
Now, 2
⋅ 2 = n2 ⋅ e nx ⎜ ⎟ = – ne
dx dy ⎝ n ⎠
D 0 0 The correct option is (D)
2. We have, ∆∆′ = 0 D 0 = ∆3 6. We have,
0 0 D
dx sec 2 q / 2 1
∴ ∆′ = ∆2 = – sin θ + = – sin θ +
dq 2 tan q / 2 q q
2 sin cos
The correct option is (C) 2 2
⎛ p⎞ ⎛ p⎞ 1 1 - sin 2 q cos 2 q
3. f ′ (x) = – 2 cos x sin x – 2 cos ⎜ x + ⎟ sin ⎜ x + ⎟
= – sin θ + = =
⎝ 3⎠ ⎝ 3⎠ sin q sin q sin q
⎛ p⎞ ⎛ p⎞ dy
 + cos x sin ⎜ x + ⎟ + sin x cos ⎜ x + ⎟
and = cos θ
⎝ 3⎠ ⎝ 3⎠ dq
⎛ 2p ⎞ ⎛ p⎞

= – sin 2x – sin ⎜ 2 x + ⎟ + sin ⎜⎝ x + x + ⎟⎠ dy dy dq cos q sin q
⎝ 3⎠ 3 ∴ = = = tan θ
dx dx dq cos 2 q
⎛ p⎞ p ⎛ p⎞

= – 2 sin ⎜ 2 x + ⎟ cos + sin ⎜⎝ 2 x + ⎟⎠ d2y dq sin q
⎝ 3⎠ 3 3
Also, 2
= sec2θ ⋅ =
dx dx cos 4 q
⎛ p⎞ ⎛ p⎞ d2y

= – sin ⎜ 2 x + ⎟ + sin ⎜⎝ 2 x + ⎟⎠ = 0. ∴ = 0 ⇒ sin θ = 0 ⇒ θ = nπ, n ∈ Z
⎝ 3⎠ 3 dx 2
⇒ f  (x) = constant for all x.
The correct option is (A)
p p 5
But, f (0) = cos2 0 + cos2 3 + sin 0 ⋅ sin = 7. f (x) = x - 1 + 25 + ( x - 1) -10 x - 1
3 4
5
∴ f (x) = for all x.
= x - 1 + ( 5 - x - 1) 2
4
⎛ 5⎞
Thus, (g o f ) (x) = g [ f (x)] = g ⎜ ⎟ = 3
= x -1 + |5 - x -1| = 5
⎝ 4⎠
The correct option is (C) (∵ x - 1 < 5 for 1 < x < 26)
∴ f ′(x) = 0
2 sin x cos x cos 2 x cos 4 x cos 8 x sin16 x
4. f (x) = = 4 The correct option is (A)
2 sin x 2 sin x
8. We have,
1 ⎡ sin x ⋅ cos16 x ⋅ 16 - sin16 x ⋅ cos x ⎤ ⎛ 1 - 3 log x ⎞ ⎛ 4 + 3 log x ⎞
⇒ f ′ (x) = y = tan -1 ⎜ + tan -1 ⎜
16 ⎢⎣ sin 2 x ⎥
⎦ ⎟
⎝ 1 + 3 log x ⎠ ⎝ 1 - 12 log x ⎟⎠
⎡ 1 1 ⎤ = tan–11 – tan–1 (3 log x) + tan–14 + tan–1 (3 log x)
⎢ ⋅ 1 ⋅ 16 - ⋅0⎥
⎛p⎞ 1 ⎢ 2 2 ⎥= ⎡ -1 -1 -1 ⎛ A - B ⎞ ⎤
∴ f ′ ⎜ ⎟ = 2 ⎢ Using tan A - tan B = tan ⎜⎝ 1+AB ⎟⎠ ⎥
⎝ 4⎠ 16 ⎢ ⎛ 1 ⎞
2 ⎥
⎢ ⎥ ⎢ ⎥
⎢⎣ ⎜
⎝ 2⎠ ⎟ ⎥⎦ ⎢ ⎥
-1 -1 -1 ⎛ A + B ⎞
⎢ and tan A + tan B = tan ⎜ ⎟ ⎥
The correct option is (C) ⎣ ⎝ 1 - AB ⎠ ⎦


Objective_Maths_JEE Main 2017_Ch 13.indd 26 01/01/2008 05:33:48


Differentiation  13.27

= tan–11 + tan–1 (4)



So domain of f= R\{0, – 2}.
dy 2
Then, in this domain, we have
∴ = 0 and hence d y = 0.
dx dx 2 x -1
y = f (x) = ⇒ yx + 2y = x – 1
The correct option is (B) x+2
9. We have, 2y +1 2x + 1
or x =
, i.e., f –1 (x) =
3 x 2 cos x - sin x 1- y 1- x
 f ′(x) = 6 -1 0 df -1( x ) 2 (1 - x ) + 2 x + 1 3
2 3 ∴ = =
p p p dx (1 - x ) 2 (1 - x ) 2
The correct option is (B)
6 x - sin x - cos x
13. We have,
f ″ (x) = 6 -1 0
F (x) = f (x) g (x) h (x)
p p2 p3 ⇒ logF (x) = log f (x) + log g (x) + log h (x)
6 - cos x sin x Differentiating both the sides with respect to x, we get
and f ″′ (x) = 6
-1 0 F’( x ) f’( x ) g’( x ) h’( x )

= + +
F (x ) f (x ) g (x ) h(x )
p p2 p3
F’( x0 ) f’( x0 ) g’( x0 ) h’( x0 )
6 -1 0 ⇒ = + +
F (x0 ) f (x0 ) g (x0 ) h(x0 )
∴ f ″′ (0) = 6
-1 0 =0
⇒ 21 = 4 – 7 + k ⇒ k = 24
2 3
p p p The correct option is (A)
(QR1 and R2 are identical)
The correct option is (D) 14. Clearly, f (x) must be of the form
10. Differentiating the given equation with respect to x, we get f (x) = a0 [xn+ (k – x)n] + a1 [xn – 1 + (k – x)n – 1]
1  + ... + an – 1 [x + (k – x)] + an
xy′ + y ⋅ 1 – y′ = 0
y It may be noted that n must be even for otherwise f (x) will
become a polynomial of degree n – 1.
⇒ xyy′ – y′ + y2 = 0 i.e.(xy– 1) y′ + y2 = 0
Clearly,  f ′ (x) is a polynomial of degree n – 1.
Differentiating again with respect to x, we get
The correct option is (B)
(xy– 1) y″ + y′ (xy′ + y ⋅ 1) + 2yy′ = 0
15. We have,
⇒ x (yy″ + y′ 2) – y″ + 3yy′ = 0
(x) = x – 1
f  ( ∵x > 2)
∴ k=3
f [ f (x)] = f (x – 1) = | (x – 1) – 1 | = | x – 2 |
The correct option is (B)
= (x – 2) (∵x > 2)
11. We have,
∴ g (x) = f { f [ f (x)]} = f (x – 2)
dx
= cost = | (x – 2) – 1 | = | x – 3 |
dt
dy ⎧ x - 3, if x ≥ 3 ⎫

and = 2 (a + b) et 2
= ⎨ ⎬
dt ⎩ - x + 3, if 2 ≤ x < 3⎭
dy dy dt 2 ( a + b) e t 2 2y ⎧1, if x ≥ 3 ⎫
∴ = = = ∴ g′ (x) = ⎨ ⎬
dx dx dt cos t 1 - x2 ⎩ -1, if 2 ≤ x < 3⎭
2 The correct option is (A)
⇒ (1 – x2) ⎛⎜ dy ⎞⎟ = 2y2 16. f (x) = | (x – 4) (x – 5) |
⎝ dx ⎠
Differentiating with respect to x, we get ⎧( x - 4)( x - 5) if ( x - 4)( x - 5) ≥ 0⎫

= ⎨ ⎬
(1 – x2) 2y′ y″ – 2x (y′ )2 = 4yy′ ⎩ -( x - 4)( x - 5) if ( x - 4)( x - 5) < 0 ⎭
⇒ (1 – x2) y″ – xy′ = 2y (dividing by 2y′ ) ⎧⎪ x 2 - 9 x + 20 if x ≤ 4 or x ≥ 5⎫⎪
∴ k = 2
= ⎨ ⎬
2
The correct option is (C) ⎩⎪ -( x - 9 x + 20) if 4 < x < 5 ⎭⎪
2 ⎧2 x - 9 if x ≤ 4 or x ≥ 5⎫
12. We have,  f (x) = x - x ∴
f ′ (x) = ⎨ ⎬
2
x + 2x ⎩-2 x + 9 if 4 < x < 5 ⎭
Clearly, f (0) and f (– 2) are not defined.
The correct option is (C)

Objective_Maths_JEE Main 2017_Ch 13.indd 27 01/01/2008 05:33:52


13.28  Chapter 13

17. 2 f (sin x) + f (cos x) = x(1) 22. Given that g–1 (x) = f (x)
p ⇒ x = g[ f (x)] or g′[ f (x)] f  ′(x) = 1

Replace x by -x
2 1 f ′′( x )
⇒ g′[ f (x)] = ⇒ g′′[ f (x)] × f ′(x) = –
p f ′( x ) [ f ′( x )]2
2 f (cos x) + f (sin x) = - x (2)

2 f ′′( x )
p ⇒ g′′[ f (x)] = –


Solving we get, 3f (sin x) = + 3 x [ f ′( x )]3
2
p d 1 The correct option is (A)
∴ f (x) = + sin -1 x ∴ f ( x) =
6 dx 1- x 2 23. f (x) = xn ⇒ f ′(x) = nxn – 1
The correct option is (C) f ′(a + b) = f ′(a) + f ′(b)
18. f (x) = | x – a | = x – a (if x > a) ⇒ n(a + b)n – 1 = nan – 1 + nbn – 1
f  [ f (x)] = f (x – a) = | x – 2a | = x – 2a (if x > 2a) ⇒ (a + b)n – 1 = an – 1 + bn – 1
f { f [ f (x)]} = f (| x – 2a |) = | x – 3a | Which is true for n = 2 and false for n = 1 and n = 4.
= x – 3a (if x > 3a) Also, for n = 0, f (x) = 1,
⇒ g′(α) = 1, ∀ α > 3a. So, f ′(x) = 0; f ′(a + b) = 0
The correct option is (C) So, f ′(a + b) = f ′(a) + f ′(b)
19. f (x) = f –1 (x) ⇒ x = f  [f (x)] Hence, there are two values of n.
⇒ 1 = {f  ′[f (x)]} ⋅ f  ′(x) The correct option is (C)
1 24. f ′(x) = (1/2)52x + 1 (log 5)(2) = log 5 ⋅ (52x + 1)
⇒ f  ′(x) =
Also, g′(x) = 5x log 5 + 4 log 5
f ′[f ( x )]
So {x : f ′(x) > g′(x)}
1 1
Now f  ′(x) =
⇒ f  ′[f (x)] = = {x : log 5 × 52x + 1 > log 5 5x + 4 log 5}
1+ x 5
1 + [f (x )]5
= {x : 52x + 1 > 5x + 4}
1 = {t = 5x : 5t2 – t – 4 > 0}
∴ f  ′(x) = = 1 + [f (x)]5
f ′[f ( x )] = {t = 5x : (5t + 4)(t – 1) > 0}

The correct option is (C) = {t = 5x : t > 1 or t < – 4/5}
= {t = 5x : t > 1} = (0, ∞).
1 dy 1
20. y = ⇒ = - 2 The correct option is (C)
x dx x
n -1
25. In = d ( x n - 1 + nx n - 1 log x )
1 x4 + 1 dx n - 1
1+ 4
1 + y4 x = x2 1 dy

Now, = = 2= - ⇒ In = (n – 1)! + nIn – 1⇒In – nIn – 1 = (n – 1)!.
1 + x4 1 + x4 4
x +1 x dx
The correct option is (D)
du

Writing in the form of differentials, we have du f ′( x 3 ) ⋅ 3 x 2 cos x 3 ⋅ 3 x 2
26. = dx = =
dx dy dx dy dv dv g ′( x 2 ) ⋅ 2 x sin x 2 ⋅ 2 x
= - ⇒ + = 0.
1+ x 4
1 + y4 dx
1 + x4 1 + y4
du 3
The correct option is (A) ∴ = x cos x 3 ⋅cosec x 2
dv 2
dy f ′f - f f ′ The correct option is (A)
21. =
dx f2 27. Put x = y = 1, we get f (1) = 0
1 ⎛ 1⎞
d2y ( f ′′f - f f ′′ )f 2 - ( f ′f - f f ′ )2ff ′ Put y = , we get f (x) + f ⎜ ⎟ = f (1) = 0
2 = x ⎝ x⎠
dx f4
⎛ 1⎞
∴ f (e) + f ⎜ ⎟ = 0
1 d2y f ′′f - f f ′′ 2( f ′f - f f ′ ) ⎝ e⎠
∴ = - (f ′ ) 2
y dx 2 ff f f 2 ⋅f ′ The correct option is (B)
f ′′ f ′′ 2( y - z ) 28. Putting x = 0, y = 0, we get

= - + (f ′ ) 2 2f (0) + [ f (0)]2 = 1 ⇒ f (0) = 2 - 1 [∵ f (x) > 0]
f f ff
Putting y = x, 2f (x) + [f (x)]2 = 1

The correct option is (B)

Objective_Maths_JEE Main 2017_Ch 13.indd 28 01/01/2008 05:33:58


Differentiation  13.29

Differentiating with respect to x, we get


⎡ dy ⎤
2f ′(x) + 2f (x) ⋅ f ′(x) = 0 or f ′(x) [1 + f (x)] = 0 2 x 2 x (1 + log x ) - 2 ⎢ x x (cosec 2 y ) + cot yx x (1 + log x )⎥ = 0
 ⎣ dx ⎦
⇒ f ′(x) = 0, because f (x) > 0
p
The correct option is (B) Now at ⎛⎜1, ⎞⎟

⎝ 2⎠
⎛ 1⎞
29. 3 f ( x ) - 2 f ⎜ ⎟ = x(1)
⎝ x⎠ ⎛ ⎛ dy ⎞ ⎞
2(1 + log 1) - 2 ⎜1( -1) ⎜ ⎟ + 0⎟ = 0
⎝ dx ⎠ ⎛⎜1, p ⎞⎟
1 ⎛ 1⎞ 1 ⎝ ⎝ 2⎠ ⎠
Put x =
, then 3 f ⎜ ⎟ – 2f (x) = (2)
x ⎝ x⎠ x
⎛ dy ⎞ ⎛ dy ⎞

Solving (1) and (2), we get ⇒ 2 + 2⎜ ⎟ = 0 ⇒ ⎜⎝ ⎟⎠ ⎛ p ⎞ = –1
⎝ dx ⎠ ⎛⎜1, p ⎞⎟ dx ⎜1, ⎟
⎝ ⎝ 2⎠
2⎠
2 3 2
5f (x) = 3 x +
⇒f  ′(x) = - 2
x 5 5x
The correct option is (A)
3 2 1
∴ f ′(2) = - = 33. We have, λ = y2 + z2
5 20 2
The correct option is (B) d ⎛ y ⎞ d ⎛ z2 ⎞

Now, y ⎜ ⎟+
30. Given: x+ y+ y - x = c(1) dx ⎝ l ⎠ dx ⎜⎝ l ⎟⎠

( y + x) - ( y - x) d ⎛ y ⎞ d ⎛ z2 ⎞

=c = y ⎜ ⎟+ ⎜ ⎟
x+ y - y-x dx ⎜⎝ y 2 + z 2 ⎟⎠ dx ⎜⎝ y 2 + z 2 ⎟⎠
2x ⎡ dz ⎞ ⎤
⇒ y+x - (2) y-x = ⎛ dy
c ⎢ y⎜ y +z ⎟⎥
2 dy ⎝ dx dx ⎠
= ⎢ y +z ⎥
2

By adding Eq. (1) and (2), we get -
⎢ dx 2
y +z 2 ⎥
2x 4x 2 y⎢ 2 2 ⎥
2 y+x = c+ ⇒ 4(y + x) = c 2 + 2 + 4 x ⎢⎣ y +z ⎥⎦
c c
dy 8x ⎡ ⎛ dy dz ⎞ ⎤
∴ 4 = 2 ⎢ z2 ⎜ y +z ⎟ ⎥
dx dz ⎝ dx dx ⎠
c ⎢ ( y + z ) 2z
2 2
- ⎥
2
⎢ dx 2
y +z 2 ⎥
2 +⎢ ⎥
∴ d y = 2 ⎢⎣ y2 + z2 ⎥⎦
dx 2 c 
The correct option is (A) 1 ⎡ dy 2 ⎛ dy dz ⎞
= 2
⎢y (y + z ) - y
2
⎜⎝ y +z ⎟
31. We have ( y 2 + z 2 )3/ 2 ⎣ dx dx dx ⎠
d2x d ⎛ dx ⎞ d ⎛ 1 ⎞ dz 2 ⎛ dy dz ⎞ ⎤
2 = ⎜ ⎟ = ( y + z2 ) - z2 ⎜ y
dy dy ⎝ dy ⎠ dy ⎜⎝ dy /dx ⎟⎠ + 2z
dx ⎝ dx
+ z ⎟⎥
dx ⎠ ⎦

d ⎛ 1 ⎞ dx 1 ⎡ dz ⎤

= ⋅ = z ( y2 + z2 ) ⎥
dx ⎜⎝ dy /dx ⎟⎠ dy ( y 2 + z 2 )3/ 2 ⎢⎣ dx ⎦
1 d2y 1 z
dz z dz
- ⋅ ⋅ = = .

= ⎛ dy ⎞
2
dx 2 ⎛ dy ⎞ 2 dx l dx2
⎜⎝ ⎟⎠ y +z
⎜⎝ ⎟⎠ dx
dx

The correct option is (A)
1 d2y -3
- ⋅ 2 ⎛ dy ⎞ ⎛ d y ⎞
2
a ( r n - 1)

= ⎛ dy ⎞ dx = - ⎜⎝ ⎟⎠ ⎜⎝ 2 ⎟⎠
3
34. We have, Sn =
⎜⎝ ⎟⎠ dx dx r -1
dx
⇒ (r – 1) Sn = arn – a
The correct option is (A)
Differentiating both sides with respect to r, we get
32. x2x– 2xx cot y – 1 = 0 (1)
dSn
Now x = 1, (r – 1) + Sn = narn – 1 – 0
dr
1 – 2 cot y – 1 = 0
p dSn
⇒ cot y = 0 ⇒ y = ⇒ (r – 1) = narn – 1 – Sn
2 dr
Now differentiating eq., (1) with respect to ‘x’ = n [nth term of G.P.] – Sn

Objective_Maths_JEE Main 2017_Ch 13.indd 29 01/01/2008 05:34:04


13.30  Chapter 13

= n (Sn – Sn – 1) – Sn ⇒ [ f (x)]n – 1 ⋅ f ′ (x) = f ′ (nx) ⇒ [ f (x)]n ⋅ f ′ (x) = f ′ (nx) ⋅


= (n – 1) Sn – nSn – 1. f (x)
The correct option is (B)  [Multiplying both sides by f (x)]
35. Differentiating the given equation with respect to x1, we get ⇒ f (nx) ⋅ f ′ (x) = f ′ (nx) ⋅ f (x)[∵ [ f (x)]n = f (nx)]
1 ⎛ x + x2 + .. + xn ⎞ f ′( x1 ) The correct option is (C)
f′⎜ 1 ⎟⎠ = 38. We have,
n ⎝ n n

dx f g h
[Since all xi′s are independent to each other, ∴ i = 0
dx j  ∆= xf ′ + f xg ′ + g xh′ + h
dxi
if i ≠ j and = 1 if (i = j)] 2 2 2
dx j x f ′′ + 4 xf ′ + 2 f x g ′′ + 4 xg ′ + 2 g x h′′ + 4 xh′ + 2h

On putting x1 = x2 = ... = xn–1 = 0


f g h

= xf ′ xg ′ xh′
⎛ x⎞
and xn = x, we get f  ′ ⎜ ⎟ = f  ′(0) = a.

⎝ n⎠ x 2 f ′′ + 4 xf ′ x 2 g ′′ + 4 xg ′ x 2 h′′ + 4 xh′
⎛ x⎞ [Applying R2 → R2 – R1 and R3 → R3 – 2R1]

On integrating, we get nf ⎜ ⎟ = ax + c

⎝ n⎠
f g h
Since f (0) = b, we have c = nb


= xf ′ xg ′ xh′ [Applying R3 → R3 – 4R2]
⎛ x⎞
∴ nf ⎜ ⎟ = ax + nb ⇒nf (x) = nax + nb x 2 f ′′ x 2 g ′′ x 2 h′′
⎝ n⎠
⇒ f (x) = ax + b. f g h f g h
∴ f  ′(x) = a, ∀x ∈R
=x 3 f′ g ′ h′ = f′ g′ h′
The correct option is (A) f ′′ g ′′ h′′
x 3 f ′′ x 3 g ′′ x 3h′′
36. We have,
y2 = P (x)(1) f g h
dy ∴ ∆′ = f′ g′ h′
⇒ 2y = P′ (x)(2)
dx ( x 3 f ′′ )′ ( x 3 g ′′ )′ ( x 3h′′ )′
dy dy 2
⇒ 2 ⋅ + 2y⋅ d y = P″ (x)
The correct option is (C)
dx dx dx 2
2 A( x ) B( x ) C ( x )
⎛ dy ⎞ 2
⇒ 2y ⎝ dx ⎟⎠
2⎜
+ 2y ⋅ d y = y2P″ (x)
3 39. Let ∆ (x) = A(a ) B(a ) C (a ) , then
dx 2 A′(a ) B ′(a ) C ′(a )
2
d2y ⎛ dy ⎞
⇒ 2y3 2 = y2P″ (x) – 2y2 ⎜⎝ dx ⎟⎠ A′( x ) B ′( x ) C ′( x )
dx
∆′ (x) =
A(a ) B(a ) C (a )
1

= y2P″ (x) – [P′ (x)]2 [from (2)] A’(a ) B’(a ) C’(a )
2
d ⎛ 3 d2y⎞ So, ∆ (α) = 0 = ∆′ (α), therefore α is a repeated root of ∆ (x)

⇒2 y
dx ⎜⎝ dx 2 ⎟⎠ and α is a repeated root of the quadratic equation f (x) = 0, so
∆ is divisible by f (x).
dy 1
The correct option is (D)

= 2y P″ (x) + y2P″′ (x) – 2P′ (x) ⋅ P″ (x)
dx 2

= P′ (x) P″ (x) + y2P″′ (x) – P′ (x) P″ (x) D 0 0
40. We have, ∆∆′ = 0 D 0 = ∆3
⎡ dy ⎤
⎢∵ 2 y dx = P’(x )⎥ 0 0 D
 ⎣ ⎦ ∴ ∆′ = ∆2.
= y2P′″ (x)
The correct option is (C)
= P (x) P″′ (x)[∵y2 = P (x)]
2
The correct option is (B) 41. We have, f (x) = x - x
37. We have, [ f (x)]n = f (nx) x2 + 2x
Clearly, f (0) and f (– 2) are not defined.
Differentiating with respect to x, we get
So, domain of f = R\{0, – 2}
n [ f (x)]n – 1 .  f ′ (x) = f ′ (nx) . n

Objective_Maths_JEE Main 2017_Ch 13.indd 30 01/01/2008 05:34:08


Differentiation  13.31


Then, in this domain, we have
2x
x -1 ⇒ y+x - y-x = (2)
y = f (x) = ⇒yx + 2y = x – 1 c
x+2

By adding (1) and (2) we get
2y +1 2x + 1
or, x =
, i.e., f –1 (x) = 2x 2
1- y 1- x 2 y+x = c+ ⇒ 4(y + x) = c 2 + 4 x + 4 x
c c2
df -1( x ) 2 (1 - x ) + 2 x + 1 3
∴ = = dy 8x
dx (1 - x ) 2 (1 - x ) 2 ∴ 4 = 2
dx c
The correct option is (B) d2y 2
∴ 2
= 2
42. Clearly, f (x) must be of the form dx c
f (x) = a0 [xn+ (k – x)n] + a1 [xn – 1 + (k – x)n – 1] The correct option is (A)
 + ... + an – 1 [x + (k – x)] + an. n

It may be noted that n must be even for otherwise f (x) will


47. f (x) = ∏ (cos( 2k - 1) x + i sin ( 2k - 1) x)
k =1
become a polynomial of degree n – 1.
Clearly, f ′ (x) is a polynomial of degree n – 1. n n

The correct option is (B)


= cos ∑ ( 2k - 1) x + i sin ∑ ( 2k - 1) x
k =1 k =1
dy f ′f - f f ′
43. =
dx f2 = cos n2x + i sin n2x
(Using De Moivre’s Theorem)
d2y ( f ′′f - f f ′′ )f 2 - ( f ′f - f f ′ )2ff ′

= ∴ (Re f (x))′′ = – n4cos n2x
dx 2 f4
and, (Imf (x))′′ = – n4 sin n2x. Thus,
1 d2y f ′′f - f f ′′ 2( f ′f - f f ′ ) (Re f (x))′′ + i(Imf (x))′′ = – n4 [cos n2x + i sin n2x]
∴ = - (f ′ ) 2
y dx 2 ff f f 2 ⋅f ′ = – n4f (x).
f ′′ f ′′ 2( y - z ) The correct option is (B)

- = + ⋅ (f ′ ) 2
f f ff 48. 2 f (sin x) + f (cos x) = x(1)

The correct option is (B) p

Replace x by -x
44. f ′(x) = (1/2)5 2x + 1
(log 5)(2) = log 5 ⋅ (5 )2x + 1 2
x
Also, g′(x) = 5 log 5 + 4 log 5 p
2f (cos x) + f (sin x) =
- x (2)
So, {x: f ′(x) > g′(x)} 2
= {x: log 5 × 52x + 1> log 5 5x + 4 log 5} p
Solving we get, 3f (sin x) =
+ 3x
= {x: 52x + 1> 5x + 4} 2
= {t = 5x : 5t2 – t – 4 > 0} p d 1
= {t = 5x : (5t + 4)(t – 1) > 0} ∴ f (x) = + sin -1 x ∴ f ( x) =
6 dx 1 - x2
= {t = 5x : t > 1 or t< – 4/5}
= {t = 5x : t > 1} = (0, ∞).
The correct option is (C)
The correct option is (C) 49. f (x) = x - 1 + 25 + ( x - 1) -10 x - 1
45. Putting x = 0, y = 0, we get
2f (0) + { f (0)}2 = 1 ⇒ f (0) = 2 - 1 (∵ f (x) > 0)
= x - 1 + ( 5 - x - 1) 2
Putting y = x, 2f (x) + {f (x)}2 = 1
Differentiating with respect to x, we get
= x -1 + |5 - x -1| = 5
2f ′(x) + 2f (x) × f ′(x) = 0 or f ′(x) {1 + f (x)} = 0
[∵ x - 1 < 5 for 1 < x < 26]

⇒ f ′(x) = 0, because f (x) > 0
The correct option is (B) ∴ f ′(x) = 0
The correct option is (A)
46. Given: x+ y+ y - x = c(1)
50. We have,
( y + x) - ( y - x) 2x
⇒ = f (x) = 2 – x, x < 2
x+ y - y-x c
= x – 2, x ≥ 2

Objective_Maths_JEE Main 2017_Ch 13.indd 31 01/01/2008 05:34:13


13.32  Chapter 13


Thus, we have, From equations (1) and (3), we have

g(x) = f {f (x)} = 2 – f, f < 2 f (3) = 15 = R(3)

= f – 2, f ≥ 2 i.e., 9a + 3b + c = 15 (4)
From equations (2) and (3), we have
g ( x ) = 2 - ( 2 - x ), 2 - x < 2⎫ f (1) = 3 = R(1)

i.e., ⎬x<2
= ( 2 - x ) - 2, 2 - x ≥ 2⎭ i.e., a + b + c = 3 (5)
= 2 - ( x - 2), x - 2 < 2⎫ From equations (2) and (3), we have
⎬x≥2 f  ′(1) = 2 = R′(1)
= ( x - 2) - 2, x - 2 ≥ 2⎭
i.e., 2a + b = 2 (6)
i.e., g(x) = x, 0 < x < 2
Solving equations (4), (5) and (6), we get
= –x, x ≤ 0
a = 2, b = –2, c = 3
= 4 – x, 2 ≤ x < 4
The correct option is (C)
= x – 4, 4 ≤ x
53. We have,
i.e., g(x) = – x, x ≤ 0
= x, 0 < x < 2 ⎛ 1⎞ 1
af (x) + bf ⎜ ⎟ =
– 5 (1)
⎝ x⎠ x
= 4 – x, 2 ≤ x < 4
= x – 4, 4 ≤ x 1

Substituting in place of x, we have
Hence, we have for x > 2 x
g′(x) = –1, 2 < x < 4 ⎛ 1⎞
a f ⎜ ⎟ + bf (x) = x – 5
(2)
= 1, 4 ≤ x ⎝ x⎠
The derivative of g(x) does not exist at x = 4. ⎛ 1⎞

Eliminating f ⎜ ⎟ from equations (1) and (2), we have
The correct option is (D) ⎝ x⎠
51. We have, ⎛1 ⎞
x = 2t – | t |, y = t3 + t2 | t | (a2 – b2) f (x) = a ⎜ - 5⎟ – b(x – 5)

⎝x ⎠
⇒ x = 3t, y = 0 when t < 0
1 ⎡a ⎤
x = t, y = 2t3 when t ≥ 0 ⇒ f (x) = - bx + 5 (b - a)⎥
a 2 - b 2 ⎢⎣ x ⎦
Eliminating the parameter t, we get
1 ⎡ a ⎤
⎪⎧ 0 , x < 0 ∴ f  ′(x) = 2
+ b⎥
y = ⎨ 3 b - a 2 ⎢⎣ x 2 ⎦
⎩⎪2 x , x ≥ 0

The correct option is (A)
Differentiating with respect to x, we get
54. We have, x = cos7θ and y = sinθ
dy ⎪⎧ 0 , x < 0
= ⎨ 2 Differentiating with respect to θ, we get
dx ⎩⎪6 x , x ≥ 0
dx

Hence, the function is differentiable everywhere and its = –7 cos6θ sinθ
derivative at x = 0 (t = 0) is 0 dq

The correct option is (A) dy

and, = cos θ
52. Since function f (x) leaves remainder 15 when divided by dq
x – 3, therefore f (x)can be written as
Thus, we have,

f (x) = (x – 3) l(x) + 15 (1) dx
= –7 cos5θ sinθ
dy
Also, f (x) leaves remainder 2x + 1 when divided by

(x – 1)2. Thus, f (x) can also be written as
d2x d ⎛ dx ⎞ d ⎛ dx ⎞ dq

and, = = ⎜⎝ dy ⎟⎠ · dy
dy 2 dy ⎜⎝ dy ⎟⎠ dq
(x) = (x – 1)2m (x) + 2x + 1
f  (2)

It R(x) be the remainder when f (x) is divided by 1

= (35 cos4θ sin2θ – 7cos6θ)
2
(x – 3) (x – 1) , then we may write cos q

= 35 cos3θ sin2θ – 7cos5θ
(x) = (x – 3) (x – 1)2n(x) + R(x)(3)
f 

= 35 cos3θ (1 – cos2θ) – 7 cos5θ
Since (x – 3) (x – 1)2 is a polynomial of degree three, the


= 35 cos3θ – 42 cos5θ
remainder has to be a polynomial of degree less than or
equal to two. Thus, let d 3x d ⎛ d2x ⎞ d ⎛ d 2 x ⎞ dq

and, = ⎜ ⎟ = ·
R(x) = ax2 + bx+ c
dy 3 dy ⎝ dy 2 ⎠ dq ⎜⎝ dy 2 ⎟⎠ dy

Objective_Maths_JEE Main 2017_Ch 13.indd 32 01/01/2008 05:34:16


Differentiation  13.33

f n(x) = (–1)nn!

1

= (–105 cos2θ sinθ + 210 cos4qsn θ) Thus, f (0) = 1, f  ′(0) = –n, f  ″(0) = n (n – 1), ...,

cos q
f n(0) = (–1)nn!


= 105 sinqcosθ (2 cos2θ – 1)
( n)
=
105
sin 2qcos 2θ =
105
sin 4θ Hence, f (0) + f  ′(0) + f ′′(0) + ... + f (0)

2 4 2! n!
The correct option is (A) n ( n - 1) n!

=1–n+ + ... + ( -1) n
55. We have, 2! n!
n n n nn
⎛ x + 1⎞ -1 ⎛ x - 1⎞ = C0 – C1 + C2 + ... + (–1) Cn
y = sec–1 ⎜⎝ x - 1⎟⎠ + sin ⎜⎝ x + 1⎟⎠ = 0 [Putting x = 1 in the expansion of (1 – x)n]
⎛ x - 1⎞ p The correct option is (B)
-1 ⎛ x - 1⎞
= cos–1 ⎜⎝ x + 1⎟⎠ + sin ⎜⎝ x + 1⎟⎠ = 2 58. We have,
⎛ xx - x-x ⎞ ⎛ x 2 x - 1⎞
–1 ⎜ ⎟
y = cot ⎝ –1
⎜ ⎟

However, the above function is defined only for values of x, 2 ⎠ = cot ⎝ 2 x x ⎠
given by
⎛ 2x x ⎞
x -1
–1 ≤
≤1
= – tan–1 ⎜⎝ 1 - x 2 x ⎟⎠
x +1
x -1 x -1
= –2 tan–1 (xx)

i.e., + 1 ≥ 0 and –1≤0
x +1 x +1 -2

and, y′ = · x x (1 + ln x )
2x 2 1 + x2x

i.e., ≥ 0 and ≥0
x +1 x +1 -2
Hence, y′(1) =
·1 = –1
i.e., x < –1 or ≥ 0 and x > –1
1+1
i.e., x ≥ 0
The correct option is (C)

Hence, we have, 59. We have,
p
y = , x≥ 0
1 + cos 2qcos 2 q = | cot θ |
2 y=
=
1 - cos 2qsin 2 q
dy

and, = 0, x > 0 In the neighbourhood of θ = π/4, we have

dx

y = cot θ
The correct option is (B) ⎛p⎞ ⎛p⎞
56. We have, and, y′ = –cosec2θ ⇒ y′ ⎜ ⎟ = –cosec2 ⎜⎝ 4 ⎟⎠ = –2

⎝ 4⎠
⎛ x -1 - x ⎞ ⎛ 1 - x2 ⎞
f (x) = cos ⎜⎝ x -1 + x ⎟⎠ = cos ⎝ 1 + x 2 ⎟⎠
–1 –1 ⎜ 3p
In the neighbourhood of θ =
, we have
4

and,
y = –cot θ
-1 (1 + x 2 ) ( -2 x ) - (1 - x 2 )( 2 x )
·
and, y′ = cosec2θ
f  ′(x) =
⎛ 1 - x2 ⎞
2 (1 + x 2 ) 2
1- ⎜ ⎛ 3p ⎞ ⎛ 3p ⎞
⎟ ⇒ y′ ⎜ ⎟ = cosec2 ⎜ ⎟ = 2
⎝ 1 + x2 ⎠ ⎝ 4⎠ ⎝ 4⎠
-1 -4 x 2x ⎛ p ⎞ ⎛ 3p ⎞

= · = Hence, y′ ⎜ ⎟ y′ ⎜ ⎟ = – 4

4x 2 (1 + x 2 ) 2 | x | (1 + x 2 ) 2 ⎝ 4⎠ ⎝ 4 ⎠
which is an odd function, since The correct option is (B)
f  ′(–x) = –f  ′(x) 60. Let f (x) = ax2 + bx+ c
The correct option is (A) Then, f ′(x) = 2ax + b
57. We have, Also, f (1) = f (–1)
f (x) = (1 – x)n ⇒ a+ b + c = a – b + c ⇒ 2b = 0 ⇒b = 0
f  ′(x) = –n (1 – x)n – 1 ∴ f ′(x) = 2ax
f  ″(x) = (–1)2n(n – 1) (1 – x)n – 2 ∴ f ′(a1) = 2aa1, f  ′(a2) = 2aa2 and f  ′(a3) = 3aa3
. . . As a1, a2, a3 are in A.P., we have
. . . f  ′(a1), f  ′(a2), f  ′(a3) are in A. P.
. . . The correct option is (A)

Objective_Maths_JEE Main 2017_Ch 13.indd 33 01/01/2008 05:34:20


13.34  Chapter 13

∴ f (x) = x2 + x sin θ + 1 > 0 ∀ x ∈R


⎛ 1⎞
61. We have, 5f (x) + 3f ⎜ ⎟ = x + 2 (1) [∵D = sin2θ – 4 < 0]
⎝ x⎠
1 The correct option is (A)
Put x = , we get
x 64. f  (x) = x3 + x2f  ′(1) + x f  ″(2) + f  ″′(3)
⎛ 1⎞ 1 ∴ f  ′(x) = 3x2 + 2xf  ′(1) + f  ″(2)
5f ⎜ ⎟ + 3f (x) = + 2 (2)
⎝ x⎠ x ⇒ f  ″(x) = 6x + 2f  ′(1)
Solving (1) and (2), we get ⇒ f  ″′(x) = 6
3 Now, f  ′(1) = 3 + 2f  ′(1) + f  ″(2)
16f (x) = 5x –
+ 4 (3)
x ⇒ f  ″(2) + f  ′(1) + 3 = 0 (1)
∴ y = x  f  (x) Again, f  ″(2) = 12 + 2f  ′(1)
1 ⎧ 3 ⎫ 1 ⎡ 2 ⇒ f  ″(2) – 2f  ′(1) – 12 = 0 (2)
⇒y=x· ⎨5 x - + 4⎬ = 5 x - 3 + 4 x ⎤⎦ f  ″′(3) = 6
16 ⎩ x ⎭ 16 ⎣ Again, (3)
From (1) and (2)
dy 1

or, = [10 x + 4] 2f  ″(2) + f  ″(2) – 6 = 0 ⇒ f  ″(2) = 2 (4)
dx 16
∴ (1) gives f  ′(1) + 2 + 3 = 0
⎛ dy ⎞ 10 + 4 7 ⇒ f  ′(1) = –5 (5)

Therefore, ⎜⎝ ⎟⎠ = =
dx at x = 1 16 8 ∴ f (x) = x3 – 5x2 + 2x + 6
∴ f (0) = 6,
The correct option is (C)
f (1) = 1 – 5 + 2 + 6 = 4
62. Let, f (x) = ax2 + bx+ c
f (2) = 8 – 20 + 4 + 6 = –2
Then, g(x) = f (x) + f  ′(x) + f  ″(x)
f (3) = 27 – 45 + 6 + 6 = –6
= (ax2 + bx+ c) + (2ax + b) + 2a
f (0) + f (2) = 6 – 2 = f (1)
= ax2 + (b + 2a)x+ (c + b + 2a)(1)
∴ (a) is true
As f (x) > 0 ∀ x, we have
f (0) + f (3) = 6 – 6 = 0
a > 0 and D < 0
f (1) + f (3) = (1 – 5 + 2 + 6) – 6
i.e., a > 0 and b2 – 4ac < 0 (2)
= –2 = f (2)
Now, D = (b + 2a)2 – 4a (c + b + 2a)
f (1) + f (3) = –2 ≠ f (0) [Q f (0) = 6]
= b2 + 4a2 + 4ab – 4ac – 4ab – 8a2
∴ (d) is false
= b2 – 4a2 – 4ac
The correct option is (D)
= b2 – 4ac – 4a2
65. Putting y = 0 in the given functional equation,
= (b2 – 4ac) – (4a2),
2
where b – 4ac < 0 from (2) f ( x ) + f ( 0) 1
we get, f (x/2) =
= [1 + f ( x )] 
∴ D = –ve as b2 – 4ac –(4a2) < 0 (Q 2 2 f (0) = 1)
Hence, g(x) is a quadratic in which a > 0 and D < 0
⇒ f (x) = 2f (x/2) – 1 (1)
⇒ g(x) > 0 for all x
f ( 0 + h) - f ( 0 )
The correct option is (A) Since f  ′(0) = –1, we get lim
= –1
h→0 h
63. We have,
f (x + y) = f (x) + f (y) + 2xy – 1 f ( h) - 1
⇒ lim = –1
Put x = y = 0 ⇒ f (0) = 2f (0) – 1 ⇒ f (0) = 1
h→0 h

Now,
f ( x + h) - f ( x )

Also, f  ′(x) = lim
h→0 h f ( x + h) - f ( x )
f  ′(x) = lim
f ( x ) + f ( h) + 2 xh - 1 - f ( x )
h→0 h

= lim
h→0 h f ( 2 x ) + f ( 2 h)

= - f ( x)
f ( h) - 1 lim 2

= 2x + lim h→0 h
h→0 h

 [using (1)]

= 2x + f  ′(0) = 2x + sinθ
Integrating, we get f (x) = x2 + x sinθ + c
1 ⎡1 ⎤

= lim {2 f ( x ) - 1 + 2 f ( h) - 1} - f ( x )⎥

f (0) = 1 ⇒ 1 = c h→0 h ⎢⎣ 2 ⎦

Objective_Maths_JEE Main 2017_Ch 13.indd 34 01/01/2008 05:34:23


Differentiation  13.35

1 f ( h) - 1 a-b
lim
h
[ f ( x) - 1 + f ( h) - f ( x)] = hlim h
=–1 =
sin 2θ – (a – b)θ
h→0 →0 2
dy
Thus, f  ′(x) = –1, so we get f (x) = –x + c dy dq = ( a - b) cos 2q - ( a - b)

=
But f (0) = 1, therefore, 1 = f (0) = –0 + c dx dx (b - a) sin 2q
dq
⇒ c=1
1 - cos 2q
Thus, f (x) = 1 – x
= = tan θ = a - x
∴ f (2) = 1 – 2 = –1 sin 2q x-b
The correct option is (B) The correct option is (B)
70. We have,
f ( x + h) - f ( x )
66. f  ′(x = lim y3 – y = 2x
h→0 h
Differentiating both sides with respect to x, we get
f ( x ) { f ( h) - 1}

= lim dy dy 2
h→0 h (3y2 – 1)
=2⇒ = (1)
dx dx 3y2 - 1
f ( x ) {1 + hg ( h) - 1}

= lim
Again, differentiating both sides with respect to x, we get
h→0 h
dy
d2y -2 · 6 y

= hlim
→0
f ( x ) g ( h) = log a f (x)

= dx
dx 2
(3 y - 1)
2
 n n n 2
Therefore, f (x) = (log a) f (x), so k = (log a)


The correct option is (C)
using (1), we get
f ( x + h) - f ( x ) d2y -24 y
67. We have, f  ′(x) = lim =
( )

3 (2)
h→0 h dx 2
3y2 - 1
f ( x ) + f ( h) + xh - f ( x )
Now,

= lim
h→0 h 2
⎛ 2 1 ⎞ d y xdy
1
⎜⎝ x - ⎟⎠ +

= lim f ( h) + x 27 dx 2 dx
h→0 h
⎛ 1 ⎞ ⎛ -24 y ⎞ 2x

=3+x
= ⎜ x2 - ⎟ ⎜ 3⎟
+
2 ⎝ ⎠
27 ⎝ (3 y - 1) ⎠ (3 y 2 - 1)
2
Integrating, we get f (x) = 3x + x + c

2
[From (1) and (2)]
Putting x = y = 0 in the given equation,
⎛ y 2 ( y 2 - 1) 2 1 ⎞ ⎛ -24 y ⎞ y ( y 2 - 1)
we get, f (0) = 0 ⇒ c = 0
= ⎜ - ⎟⎜ +
27 ⎠ ⎝ (3 y 2 - 1)3 ⎟⎠

⎝ 4 3y2 - 1
2
∴ f (x) = 3x + x
3
(Qy – y = 2x)

2 {27 y 2 ( y 2 - 1) 2 - 4} ( -24 y ) y( y 2 - 1)
The correct option is (C)
= +
( )
3
108 3y2 - 1 3y2 - 1
68. We have,
f (x) = x + tan x y ⎪⎧ -54 y ( y - 1) + 8 9( y 2 - 1) ⎪⎫
2 2 2

= ⎨ + ⎬
⇒ f (f –1 (y)) = f –1(y) + tan f –1(y) 9 ⎩⎪ (3 y 2 - 1)3 3 y 2 - 1 ⎭⎪
⇒ y = g(y) + tan g(y) or x = g(x) + tan g(x)
On differentiating, we get 1 = g′(x) + sec2 g(x) g′(x) y ⎪⎧ -2(1 + a ) (a - 2) 2 + 8 ⎪⎫ 3(a - 2)

= ⎨ ⎬+
9 ⎩⎪ a3 ⎭⎪ a
1
⇒ g′(x) =
1 + sec 2 g ( x ) where α = 3y2 – 1
1 y

= =
2 + [ g ( x ) - x ]2 9
The correct option is (C)

The correct option is (C)
71. We have,
69. Let x = a cos2θ + b sin2θ
(1 + x + x2) (1 – x+ x2) = (1 – x2)2 – x2 = 1 + x2 + x4
∴ a – x = a – a cos2θ – b sin2θ = (a – b) sin2θ
Now, (1 + x + x2) (1 – x + x2) (1 – x2 + x4)
and, x – b = a cos2θ + b sin2θ – b = (a – b) cos2θ
= (1 + x2 + x4) (1 – x2 + x4)
∴ y = (a – b) sin qcos θ – (a – b) tan–1 tan θ
= 1 + x4 + x8

Objective_Maths_JEE Main 2017_Ch 13.indd 35 01/01/2008 05:34:28


13.36  Chapter 13


Continuing in this way, we have
Differentiating both sides with respect to x, we get
(1 + x + x2) (1 – x+ x2) (1 – x2 + x4) (1 – x4 + x8)
1 + 2x -1 + 2 x -2 x + 4 x 3
⇒ + +
(
... 1 - x 2

n -1

+ x2
n

) 1+ x + x 2
1- x + x 2
1 - x2 + x4


 -4 x 3 + 8 x 7
+ + ... = 0
= (1 + x
2n
+x 2n + 1
) 1 - x 4 + x8

Now, for x < 1, x∞ = 0. 1 - 2x 2 x - 4 x3 4 x3 - 8x7


Taking limits as n → ∞ in (1), we get
Hence, + +
2 2 4
1- x + x 1- x + x 1 - x 4 + x8
(1 + x + x2) (1 – x+ x2) (1 – x2 + x4) (1 – x4 + x8) ... = 1
1 + 2x
Taking logarithm of both sides, we get
 + ... =
1 + x + x2
⇒ ln (1 + x + x2) + ln (1 – x + x2) + ln (1 – x2 + x4)
The correct option is (B)
 + ln (1 – x4 + x8) + ... = 0

More than One Option Correct Type


72. We have, f ′ (α + β) = f ′ (α) + f ′ (β) 75. We have,
⇒ (α + β)m – 1 = am – 1 + bm – 1 f (x) + f (y) + f (z) + f (x) ×f (y) ×f (z) = 14 (1)
Since, for m > 2, the above equality is not valid for all x, y, z ∈R
∴ we must have m = 2. Putting x = y = z = 0, we get
Also, for m = 0, f ′ (x) = 0 for all x. So the equality is trivially 3 f (0) + [ f (0)]3 = 14 ⇒ [ f (0)]3 + 3 f (0) – 14 = 0
true. ⇒ f (0) = 2
The correct option is (B) and (C) Now, putting y = z = x in (1), we get
73. Since f (x) is a polynomial of degree 3, 3 f (x) + [ f (x)]3 = 14
∴ f (x) = x3 + ax2 + bx + c Differentiating with respect to x, we get
Comparing with the given equation, we have 3 f ′ (x) + 3 [ f (x)]2 × f ′ (x) = 0
a = f ′(1), b = f ′′(2), c = f ′′′(3) ⇒ 3 f ′ (x) [1 + { f (x)}2] = 0
Now, f ′(x) = 3x2 + 2ax + b ⇒ f ′(x) = 0, for all x.
f ″(x) = 6x + 2a The correct option is (A) and (B)
f ″′(x) = 6 76. Since f (x – y), f (x) ⋅ f (y) and f (x + y) are in A.P.
∴ a = f ′(1) = 3 + 2a + b ⇒ f (x + y) + f (x – y) = 2f (x) .  f ( y)(1)
b = f ′′(2) = 12 + 2a Putting x = 0, y = 0 in (1), we get
c = f ′′′(3) = 6 f  (0) + f (0) = 2f (0) .  f (0) ⇒f (0) = 1 {∵ f (0) ≠ 0}
Solving the above system of equations Putting x = 0, y = x in (1), we get
3 + a + b = 0 and b = 12 + 2a f  (x) + f (–x) = 2f (0) .  f (x) ⇒ f (2) = f (–2), f (3) = f (–3)
∴ a = – 5, b = 2 Differentiating f (x) = f (–x) with respect to x, we have
Now, f (x) = x3 – 5x2 + 2x +6 f  ′(x) + f  ′(– x) = 0
Thus, f (0) = 6, f (1) = 4, f (2) = – 2, f (3) = – 6. ∴ f  ′(2) + f  ′(– 2) = 0, f  ′(3) + f  ′(– 3) = 0
The correct option is (A), (B) and (C) The correct option is (A) and (C)
74. Since f (x – y), f (x) ⋅ f (y) and f (x + y) are in A.P. 77. We have,
⇒ f (x + y) + f (x – y) = 2f (x) .  f ( y)(1) ⎛ x⎞
f (x y) = 2f (x) – f ⎜ ⎟ (1)
Putting x = 0, y = 0 in (1), we get ⎝ y⎠
f  (0) + f (0) = 2f (0) .  f (0) ⇒ f (0) = 1 [∵ f (0) ≠ 0] Putting x = 1 in equation (1), we get

Putting x = 0, y = x in (1), we get 1
2f (1) = f (y) –

f  (x) + f (–x) = 2 f (0)  .  f (x) ⇒ f (2) = f (–2), f (3) = f (–3) y
Differentiating f (x) = f (–x) with respect to x, ⎛ 1⎞

f (y) = –f ⎜ ⎟ [f (1) = 0; given] (2)
f  ′(x) + f  ′(– x) = 0 ⎝ y⎠
∴ f  ′(2) + f  ′(– 2) = 0, f  ′(3) + f  ′(– 3) = 0 f ( x + h) - f ( x )
The correct option is (A) and (C) Now, f  ′(x) = lim

h→0 h

Objective_Maths_JEE Main 2017_Ch 13.indd 36 01/01/2008 05:34:29


Differentiation  13.37

Putting x = 0 in equation (1), we have


⎧ ⎛ h⎞ ⎫
f ⎨ x ⎜1 + ⎟ ⎬ - f ( x ) f (–y) + f (y) = 2f (0) + f (y) = 2f (y)

= ⎝ x⎠⎭
lim ⎩ ⇒ f (–y) = f (y)(2)
h→0 h
⇒ f is even
⎛ h⎞ ⎛1 h ⎞ Differentiating equation (2) with respect to y, we get
f ⎜1 + ⎟ - f (1) f ⎜ + 2 ⎟ + f ( x)
2 ⎝ x⎠ 1 ⎝x x ⎠ –f  ′(–y) = f  ′(y)

= lim - 2⋅
h→0 x h x h
⇒ f  ′ is odd.
x x2
The correct option is (A) and (D)

 [using (2)] 80. Let g′(1) = a and g′(2) = b(1)
Then, f (x) = x2 + ax + b, f (1) = 1 + a + b
2 1 ⎛ 1⎞ f  ′(x) = 2x + a, f  ″(x) = 2

= - f ′⎜ ⎟
x x2 ⎝ x ⎠ ∴ g(x) = (1 + a + b)x2 + (2x + a) · x + 2
= x2(3 + a + b) + ax + 2

Differentiating (2) with respect to y, we get
⇒ g′(x) = 2x(3 + a + b) + a
⎛ 1⎞ 1 Hence, g′(1) = 2(3 + a + b) + a(2)

f  ′(y) = f  ′ ⎜ ⎟ 2
⎝ y⎠ y g′(2) = 4(3 + a + b) + a(3)
2 1 From (1), (2) and (3), we have,

f  ′(x) = - f ′( x ) i.e. f  ′(x) =
x x a = 2(3 + a + b) + a and b = 2(3 + a + b)
1 i.e., 3 + a + b = 0 and b + 2a + 6 = 0
⇒ df = dx
x Hence, b = 0 and a = –3
On integrating, we get So f (x) = x2 – 3x and g(x) = –3x + 2
f  (x) = lnx + c, where c is a constant The correct option is (A) and (D)
Since, f (1) = 0 ⇒c = 0. Thus, we have 81. Let S = 1 + x + x2 + x3 + x4 + ... + xn
f (x) = lnx which is a geometric progression
1 (1 - x n +1 )
The correct option is (A) , (B) and (C) ∴ S = 1 + x + x2 + x3 + ... + xn=
1- x
78. We have, On differentiating both sides, we get,
f (x) = x3 + x2f  ′(1) + xf  ″(2) + f ′″(3)(1) 0 + 1 + 2x + 3x2 + 4x3 + ... + nxn – 1


Differentiating with respect to x, we have
f  ′(x) = 3x2 + 2xf  ′(1) + f  ″(2)(2)
= ( )
(1 - x ) ⋅ [ -( n + 1) x n ] - 1 - x n +1 ⋅ ( -1)
(1 - x ) 2
Differentiating with respect to x, we get
n
f  ″(x) = 6x + 2f  ′(1)(3) 1

∑ r xr - 1 =
(1 - x ) 2
{1 - ( n + 1) x n + n •x n + 1}
Differentiating with respect to x, we get r =1
f  ″′(x) = 6 (4)
Thus, a = –(n + 1) and b = n

Putting x = 3 is equation (4), we get

The correct option is (B) and (C)
f  ″′(3) = 6
Putting x = 1 in equation (2) and in equation (3), we get 82. From the graph it is clear that f (x) is non-differentiable at
f  ′(1) = 3 + 2f  ′(1) + f  ″(2) x = 0, –1
i.e., f  ′(1) + f  ″(2) = –3 (5) Y
and, f  ″(2) = 12 + 2f  ′(1)(6)
(–1, 2) 2 y = f (x)
Solving equation (5) and equation (6), we have
f  ′(1) = –5 and f  ″(2) = 2 f (x) = –2x
The correct option is (A), (B) and (C) 1
79. We have, 1 2
f (x – y) + f (x + y) = 2 f (x) f (y)(1) X
–3 –2 –1 0(0, 0)
Putting y = 0 in equation (1), we have
f (x) {f (0) – 1} = 0
⇒ f (0) = 1

Objective_Maths_JEE Main 2017_Ch 13.indd 37 01/01/2008 05:34:31


13.38  Chapter 13

Also, f  ″(100) = 0
⇒ (x – 5) (x + 1) (x + 7) = 0
⎡ dy ⎤ ∴ x = –7, –1, 5


 ⎢∵on x -axis, y = 0 ⇒ dx = 0 ⇒ f ′( x ) = 0 ⎥
⎣ ⎦
Thus, we have,
10 -1 0 10 k = f  ′(–1) = 3(–1)2 + 6(–1) –33 = 3 – 6 – 33 = –36

∫ f ( x ) dx = ∫ f ( x ) dx + ∫ f ( x ) dx + ∫ f ( x ) dx
k = f  ′(–7) = 3(–7)2 + 6(–7) –33 = 147 – 63 – 33 = 51
-3 -3 -1 0
k = f  ′(5) = 3(5)2 + 6(5) –33 = 75 + 30 – 33 = 72
-1 0 10

The correct option is (A), (B) and (C)

= ∫ 2 dx + ∫ ( -2 x) dx + ∫ 0 dx
-3 -1 0 85. Given F (x) = f (x) · g(x)(1)
Differentiating both sides with respect to x,we get
-1 0

= 2x + - x2 F  ′(x) = f  ′(x) g(x) + g′(x) f (x)
-3 -1


= (–2 – (–6)) + (–0 + (–1)2) = 4 + 1 = 5 ⎡ f ( x) g( x) ⎤
⇒ F  ′(x) = f  ′(x) g′(x) ⎢ + ⎥

The correct option is (A), (C) and (D) ⎣ f ′ ( x ) g ′( x ) ⎦
⎡ f g⎤
⇒ F  ′ = c ⎢ + ⎥ ⇒ (a) is correct
⎛ np ⎞
n! sin ⎜ x + ⎟
⎝ 2⎠
- cos x + np( 2 )
⎣ f ′ g′ ⎦
Again, differentiating both sides w.r.t, x we get
83. f n(x) = n!
⎛ np ⎞
sin ⎜ ⎟
⎝ 2⎠
cos np ( 2) F  ″(x) = f  ″(x) · g(x) + g″(x) · f (x) + 2f  ′(x) · g′(x)
⇒ F  ″(x) = f  ″(x) · g(x) + g″(x) · f (x) + 2c(2)
a a2 a3 Dividing both sides by F (x) = f (x) · g(x)
{Qf  ′(x) g′(x) = c}

n! sin np ( 2) ( 2)
- cos np
Then,
F ′′( x )
F ( x)
=
f ′′( x ) g ′′( x )
f ( x)
+
g( x)
+
2c
f ( x) g( x)
∴ f n(0) = n!
sin ( np ) cos ( np ) F ′′ f ′′ g ′′ 2c
2 2
or, = + +
2 3 F f g fg
a a a
⇒ (b) is correct

n! sin np ( 2) 0


Again, given f  ′(x) g′(x) = c
Differentiating both sides with respect to x, we get

= n! sin ( np ) 0 =0 f  ′(x) g″(x) + g′(x) f  ″(x) = 0 (3)
2
From (2), F  ″(x) = f  ″(x) + g(x) + g″′(x) f (x) + 2c
a a2 a3
Differentiating both sides with respect to x, we get
F ″′(x) = f  ″(x) g′(x) + f  ″′(x) g(x) + g″(x) f  ′(x) + g″(x) f (x) + 0
 (∵ n = ( 2m + 1)) = f ″′(x) g(x) + g″′(x) f (x) + 0 [from (3)]
The correct option is (C) and (D)
Now, dividing both sides by F (x) = f (x) g(x), we get
84. We have,
1 F ′′′( x ) f ′′′( x ) g ′′′( x )
g(f (x)) = x ⇒ g′(f (x)) f  ′(x) = 1 ⇒ g′(f (x) =
= +
( f ′ ( x )) F ( x) f ( x) g( x)
Now, f (x) = 2 ⇒ x3 + 3x2 – 33x – 33 = 2
F ′′′ f ′′′ g ′′′
⇒ x3 + 3x2 – 33x – 35 = 0
or, = + ⇒ (c) is correct
F f g
⇒ x3 – 5x2 + 8x2 – 40x + 7x – 35 = 0

The correct option is (A), (B) and (C)
⇒ (x – 5) (x2 + 8x + 7) = 0

Passage Based Questions


86. We have, Putting x = 0 and y = 0 in equation (1), we get
f (x) + f (y) = f ( x 1 - y + y 1 - x ) (1) 2 2 f (0) + f (0) = f (0)
i.e., f (0) = 0
Putting y = – x in equation (1), we have

So, by (1)
(x) + f (– x) = f ( x
f  1 - x2 - x 1 - x 2 )cos -1 = f (0)
f (x) + f (– x) = 0 (2)

Objective_Maths_JEE Main 2017_Ch 13.indd 38 01/01/2008 05:34:35


Differentiation  13.39

Therefore, the function f (x) is odd.


f ( kx ) + f ( h) - k f ( x )
The correct option is (B) = lim
(2)
h→ 0 h
87. We have,

[using (1)]
f ( x + h) - f ( x )
Putting kx in place x and 0 in place of y in equation (1), we
f ′(x) = lim
h→ 0 h get
f ( x + h) + f ( - x ) ⎛ kx + 0 ⎞ f ( kx ) + f (0)

= lim  [using (2)]
f ⎜ =
h→ 0 h ⎝ k ⎟⎠ k


=
lim
{
f ( x + h) 1 - x 2 - x 1 - ( x + h) 2 } i.e., f (kx) – kf (x) = – f (0)


Putting the above result in equation (2), we get
h→ 0 h
f ( h) - f ( 0 )

[using (1)] f ′(x) = lim = f ′(0)
h→ 0 h
2

= lim f ( y ) ⋅ lim ( x + h) 1 - x - x 1 - ( x + h) 2 i.e., f ′(x) = m[ f  ′(0) = m (given)]
y→0 y h→ 0 h The correct option is (A)
⎡ Putting ( x + h) 1 - x 2 - x 1 - ( x + h) 2 = y ⎤ 90. We have, f ′(x) = m
 ⎣⎢ ⎦⎥
Now, we have, df
⇒ =m
dx
f ( y) f ( y ) - f ( 0)
lim = lim = f ′(0) = 1 [Q f (0) = 0]
y →0 y y→0 y i.e., f (x) = mx + c, where c is a constant
(3)
Putting x = 0 and y = 0 in equation (1), we get

( x + h) 1 - x 2 - x 1 - ( x + h) 2

and, lim 2 f ( 0)
h→ 0 h f (0) =
k

=
1 - x 2 + lim
x ( 1 - x 2 - 1 - ( x + h) 2 ) ⇒
⎛ 2⎞
f (0) ⎜1 - ⎟ = 0 or f (0) = 0
h→ 0 h ⎝ k⎠
Therefore, from (3), we get c = 0
x (1 - x - 1 + ( x + h) 2 )
2
1 - x 2 + lim
( )

= Hence, we have,
h→ 0
h 1 - x2 + 1 - ( x + h) 2 f  (x) = mx
The correct option is (A)
x2 1

= 1 - x2 + = 91. Given:
1- x 2 1 - x2
f  (xy) = f (x) f (y) – f (x) – f (y) + 2 (1)
1 Therefore,
Hence, f ′(x) =
(3) f ( x + h) - f ( x )
1 - x2 f ′(x) = lim
h→ 0 h
The correct option is (A)
⎧ ⎛ h⎞ ⎫
88. Integrating (3), we get f ⎨ x ⎜1 + ⎟ ⎬ - f ( x )

= ⎩ ⎝ x⎠⎭
f  (x) = sin–1x + C, where C is a contant. lim
h→ 0 h
Since f (0) = 0, we get c = 0.
∴ f (x) = sin–1x ⎛ h⎞ ⎛ h⎞
f ( x ) f ⎜1 + ⎟ - f ( x ) - f ⎜1 + ⎟ + 2 - f ( x )
= ⎝ x⎠ ⎝ x⎠
The correct option is (B) lim
h→ 0 h
⎛ x + y⎞ f ( x) + f ( y)
89. Given: f ⎜ = (1)
[using (1)]
⎝ k ⎟⎠ k
⎛ h⎞

We have, f ⎜1 + ⎟ - 2
⎝ x⎠ f ( x ) - 1 (2)
⎛ h⎞ = lim ⋅
f ⎜ x + ⎟ - f ( x) h→ 0 h x
f ′(x) = ⎝ k⎠
lim x
h→ 0 h/k Putting x = 1 and y = 2 in equation (1), we have
⎛ kx + h ⎞ f  (2) = f (1) + (2) – f (1) – f (2) + 2
f ⎜ - f ( x)

= ⎝ k ⎟⎠ ⇒ 5 = 5 f (1) – f (1) – 5 + 2
lim
h→ 0 h/k [Qf (2) = 5; given]

Objective_Maths_JEE Main 2017_Ch 13.indd 39 01/01/2008 05:34:39


13.40  Chapter 13

i.e., f (1) = 2
1 f (δ ) - f ( 0) f ′(0)
Equation (2), thus reduces to
= lim =
1 + x2 δ →0 δ 1 + x2
⎛ h⎞
f ⎜1 + ⎟ - f (1)
f ( x ) - 1 ⎝ x⎠ 2
f ′(x) = ⋅ lim
= [ f ′(0) = 2; given]
x h→ 0 h 1 + x2
x
The correct option is (C)
f ( x) - 1

= ⋅ f ′ (1) (3) 2
x 95. We have, f ′(x) =
1 + x2
The correct option is (A)
92. From (3), we have, ⎛ 2 ⎞
⇒ df = ⎜ ⎟ dx
df dx ⎝ 1 + x2 ⎠
= f ′(1)
f -1 x On integrating, we get
On integrating, we get f  (x) = 2 tan–1x + C, where C is a constant.
ln ( f – 1) = f ′(1) lnx + C, where C is a constant Now, using the condition f (0) = 0, we get C = 0.
On using the condition f (1) = 2, we get Hence, we have,
ln (2 – 1) = f ′(1) ln 1 + C f  (x) = 2 tan–1 x
⇒ C = 0 The correct option is (B)
Using the condition f (2) = 5, we get 96. We have
ln (5 – 1) = f ′(1) ln 2 ∂u 1 1 1 ⎛ - y⎞
= ⋅ +
⇒ f ′(1) =
ln 4
ln 2
=2

∂x 1 - ( x / y) 2 y 2 ⎜
1 + ( y / x) ⎝ x ⎠
2 ⎟
{ }
Hence, we have, ∂u x xy
∴ x = - (1)
In[ f (x) – 1] = 2 ln x = ln x2 ∂x y -x2 2 x + y2
2

∴ f (x) = 1 + x2
∂u ⎛ - x⎞
1 1 1
The correct option is (C) ⎜⎝ y ⎟⎠ + ⋅
{ ( )}

Now, =
∂y 1- x / y 2
1 +
2
y 2
/ x2 x
⎛ x + y⎞
93. Given f (x) + f (y) = f ⎜ (1)
⎝ 1 - xy ⎟⎠ ∂u -x xy
∴ y = + (2)
Putting y = – x in equation (1), we get ∂y y -x 2 2 x + y2
2

f  (x) + f (– x) = f (0)


Putting x = 0 and y = 0 in equation (1), we get
Adding (1) and (2), we get
f  (0) + f (0) = f (0) ∂u ∂u
x +y = 0.
⇒ f (0) = 0 ∂x ∂y
Thus, we have, The correct option is (A)
f  (x) + f (– x) = 0 97. Let U = sin x and V = x2 so that y = UV
which proves that f (x) is an odd function. ⎛ np ⎞
We have, Un = sin ⎜ x +

The correct option is (B) ⎝ 2⎠
94. We have,
dny ⎛ np ⎞ 2 n ⎡ p⎤
f ( x + h) - f ( x ) ∴ = sin ⎜ x + ⎟ ⋅ x + C1 sin ⎢ x + ( n - 1) ⎥ ⋅ 2 x
f ′(x) = lim dx n ⎝ 2⎠ ⎣ 2⎦
h→ 0 h
⎡ p⎤
f ( x + h) + f ( - x )  + n C2 sin ⎢ x + ( n - 1) ⎥ ⋅ 2
= lim [Qf is odd] ⎣ 2⎦
h→ 0 h
⎛ h ⎞ ⎛ np ⎞ ⎡p ⎛ np ⎞ ⎤
f ⎜ = x 2 sin ⎜ x + ⎟ - 2nx sin ⎢ - ⎜⎝ x +
= ⎝ 1 + x 2 + xh ⎟⎠  [using equation (1)] ⎝ 2⎠ ⎣2

2 ⎠ ⎥⎦
lim
h→ 0 h
⎡ ⎛ np ⎞ ⎤
⎛ h ⎞ - n( n - 1) sin ⎢ n - ⎜ x + ⎟
f ⎜
⎝ 1 + x 2 + xh ⎟⎠
- f ( 0) ⎣ ⎝ 2 ⎠ ⎥⎦
= lim
1 

h→ 0 h 1 + x 2 + xh np ⎞ np ⎞
⎛ ⎛
1 + x 2 + xh = ( x 2 - n2 + n) sin ⎜ x + ⎟ - 2nx cos ⎜⎝ x + ⎟
⎝ 2⎠ 2⎠
[Qf (0) = 0]

Objective_Maths_JEE Main 2017_Ch 13.indd 40 01/01/2008 05:34:43


Differentiation  13.41

∴ k = – 2nx + [xyn+1 + nC1yn. 1] + n2yn = 0


The correct option is (D) or, x2 yn+2 + 2n x yn+1 + n(n – 1) yn + x yn+1 + n yn + n2yn = 0
⎛ y⎞ or, x2 yn+2 + (2n +1) x yn+1 + 2n2 yn = 0 ∴k = 2n2
98. Step 1: We have cos -1 ⎜ ⎟ = n (log x – log n) The correct option is (B)
⎝ b⎠
⇒ y = b cos [n (log x – log n)](1) 99. Apply the theorem to f (x) cosx = sin x.
Differentiating, we get The correct option is (A)
100. We have,
n
y1 = - b sin [ n (log x - log n)] ⋅
d n -1 ⎧ d n
Squaring both sides, we get
x In =
dx n -1 ⎨
⎩ dx
( ⎫
x log x ⎬

)
x2y12 = n2b2sin2[n (log x – log n)] n -1

= n2b2 [1 – cos2 {n(log x – log n)}]


In = d
dx n -1
{
n x n -1 log x + x n -1 }
Using (1), we obtain n -1
d n -1 n -1
x2y12 = n2b2 – n2y2 = n d
dx n -1
x {
n -1
log x + }
dx n -1
(x )
Differentiating again, we get
∴ In = n In–1 + (n – 1) !
2xy12 + 2x2y1y2 = – 2n2y y1
∴ k = (n – 1) !
Cancelling out 2y1, we get x2y2 + xy1 + n2y = 0 (2)
The correct option is (B)
Step 2 : Differentiating (2) n times by the theorem, we get
[x2yn+2 + nC1yn+1 . 2x + nC2yn. 2]

Match the Column Type


101. I.  Let y = f (tan x) and u = g (sec x) We have, ( f h) (x) = f (x) ⋅ h (x) = sin x cos x
III. 
dy ∴ [ go ( f h)] (x) = g [( f h) (x)] = g [ f (x) ⋅ h (x)]
⇒ = f ′ (tan x) sec2x
dx = g (sin x cos x) = 2sin x cos x = sin 2x
du i.e., f (x) = sin 2x
and = g′ (sec x) ⋅ sec x tan x
dx ⇒ f  ′ (x) = 2 cos 2x and f  ″ (x) = – 4 sin 2x
dy dy du f ′(tan x ) sec 2 x p
⎛ ⎞ p
∴ = = ∴ f  ″ ⎜ ⎟ = – 4 sin = – 4
du dx dx g ′(sec x ) sec x tan x ⎝ 4⎠ 2
⎛ p⎞ The correct option is (B)
f ′ ⎜ tan ⎟
dy ⎤ ⎝ 4⎠ ⎛ p⎞ ⎛ p⎞
∴ = f ′ (x) = – 2 cos x sin x – 2 cos ⎜ x + ⎟ sin ⎜ x + ⎟
IV. 
du ⎥⎦ x = p ⎛ p ⎞ p ⎝ 3⎠ ⎝ 3⎠
4 g ′ ⎜ sec ⎟ sin
⎝ 4⎠ 4 ⎛ p⎞ p⎞

 + cos x sin ⎜ x + ⎟ + sin x cos ⎜ x + ⎟
f ′(1) 1 ⎝ 3⎠ ⎝ 3⎠
= = 2×2 =
1 4 2
g ′( 2 ) ⋅ ⎛ 2p ⎞ ⎛ p⎞
2 = – sin 2x – sin ⎜ 2 x + ⎟⎠ + sin ⎝⎜ x + x + ⎠⎟
⎝ 3 3
The correct option is (D)
⎛ p⎞ p ⎛ p⎞
dy = – 2 sin ⎜ 2 x + ⎟ cos + sin ⎜⎝ 2 x + ⎟⎠
 II.  We have, y = x3 – 8x + 7 ⇒ = 3x2 – 8 ⎝ 3 ⎠ 3 3
dx
It is given that when t = 0, x = 3 ⎛ p⎞ ⎛ p⎞
= – sin ⎜ 2 x + ⎟ + sin ⎜⎝ 2 x + ⎟⎠ = 0
dy ⎝ 3⎠ 3
∴ When t = 0, = 3 ⋅ 32 – 8 = 19
dx
⇒ f (x) = constant for all x.
dy dy dt p p 5
Also, = (1) But, f (0) = cos2 0 + cos2 3 + sin 0 ⋅ sin =
dx dx dt 3 4
dy dy 5
Since, when t = 0, = 19 and = 2, ∴ f (x) = for all x.
dx dt 4
2 dx 2 ⎛ 5⎞
∴ from (1), 19 = ⇒ = Thus, (gof ) (x) = g [ f (x)] = g ⎜ ⎟ = 3
dx dt dt 19 ⎝ 4⎠
The correct option is (C) The correct option is (A)

Objective_Maths_JEE Main 2017_Ch 13.indd 41 01/01/2008 05:34:48


13.42  Chapter 13

102. I. Differentiating the given equation with respect to x, we III. 


We have,
get F (x) = f (x) g (x) h (x)
1 ⇒ log F (x) = log f (x) + log g (x) + log h (x)
xy′ + y⋅ 1 – y′ = 0
y Differentiating both the sides with respect to x, we get
⇒ xyy′ – y′ + y2 = 0 i.e., (xy – 1) y′ + y2 = 0
F ′( x ) f ′ ( x ) g ′ ( x ) h′ ( x )
Differentiating again with respect to x, we get   = + +
F ( x) f ( x ) g ( x ) h( x )
(xy – 1) y″ + y′ (xy′ + y ⋅ 1) + 2yy′ = 0
F ′( x0 ) f ′( x0 ) g ′( x0 ) h′( x0 )
⇒ x (yy″ + y′ 2) – y″ + 3yy′ = 0 ⇒ = + +
F ( x0 ) f ( x0 ) g ( x0 ) h( x0 )
∴k=3
The correct option is (D) ⇒ 21 = 4 – 7 + k ⇒ k = 24
II.  We have, The correct option is (A)
dx IV. f (x) = xn ⇒ f ′(x) = nxn – 1
= cos t
dt f ′(a + b) = f ′(a) + f ′(b)
dy ⇒ n(a + b)n – 1 = nan – 1 + nbn – 1
and, = 2 (a + b) et 2
dt
⇒ (a + b)n – 1 = an – 1 + bn – 1
2
dy dy dt 2 ( a + b) e t 2y
∴ = = = Which is true for n = 2 and false for n = 1 and n = 4.
dx dx dt cos t 1 - x2 Also, for n = 0, f (x) = 1,
2
⇒ (1 – x ) ⎛⎜ dy ⎞⎟
2
= 2y2 So, f ′(x) = 0; f ′(a + b) = 0
⎝ dx ⎠
So, f ′(a + b) = f ′(a) + f ′(b)
Differentiating with respect to x, we get
Hence, there are two values of n.
(1 – x2) 2y′ y″ – 2x (y′ )2 = 4yy′
The correct option is (B)
⇒ (1 – x2) y″ – xy′ = 2y [dividing by 2y′ ]
∴k=2
The correct option is (B)

Assertion-Reason Type
103. Since f (x) is a polynomial function satisfying
dy ⎤
∴ =1
⎛ 1⎞ ⎛ 1⎞
f (x) ⋅ f ⎜ ⎟ = f (x) + f ⎜ ⎟ , dx ⎥⎦ x =0
⎝ x⎠ ⎝ x⎠
The correct option is (A)
∴ f (x) = xn+ 1 or f (x) = – xn + 1 105. f (x) = (cosx + i sin x) (cos 2x + i sin 2x)
If f (x) = – xn + 1, then f (4) = – 4n + 1 ≠ 65 (cos 3x + i sin 3x) ... (cosnx+ i sin nx)
So, f (x) = xn + 1 = cos (x + 2x + 3x + ... + nx) + i sin (x + 2x + 3x + ... + nx)
Since f (4) = 65 ∴ 4n + 1 = 65 ⇒ n = 3
n ( n + 1) n ( n + 1)
∴ f (x) = x3 + 1 ⇒f ′ (x) = 3x2 = cos x + i sin x
2 2
∴ f ′ (l1) = 3l12 , f ′ (l2) = 3l22 , f ′ (l3) = 3l32
n ( n + 1) ⎡ n ( n + 1) n ( n + 1) ⎤
Since l1, l2, l3 are in G.P., ⇒ f ′ (x) = - sin x + i cos x⎥
2 ⎢⎣ 2 2 ⎦
∴ f ′ (l1), f ′ (l2), f ′ (l3) are also in G.P.
The correct option is (A) 2
⇒ f ″ (x) = – ⎛⎜ n ( n + 1) ⎞⎟ ⎛⎜ cos n ( n + 1) x + i sin n ( n + 1) x⎞⎟
104. We have, ⎝ 2 ⎠ ⎝ 2 2 ⎠
y = (1 + x) (1 + x2) (1 + x4) ... (1 + x 2 )
n

2
2 4 2 n
= – ⎛⎜ n ( n + 1) ⎞⎟ f ( x )
= (1 - x )(1 + x )(1 + x )(1 + x )...(1 + x ) ⎝ 2 ⎠
1- x
2 2
⎛ n ( n + 1) ⎞
f (1) = – ⎛⎜ n ( n + 1) ⎞⎟ .
n +1
2
= 1- x ∴ f ″ (1) = – ⎜
⎝ ⎟
1- x 2 ⎠ ⎝ 2 ⎠
The correct option is (C)
(1 - x ) ⋅ -2n +1 ⋅ x 2 -1 + (1 - x 2 )
n +1 n +1
dy
⇒ =
dx (1 - x ) 2

Objective_Maths_JEE Main 2017_Ch 13.indd 42 01/01/2008 05:34:53


Differentiation  13.43

Previous Year’s Questions

106. Given y = ( x + 1 + x 2 ) n ⇒ a + b + c = a – b + c also 2b = a + c


f  ′ (x) = 2ax + b = 2ax
∴ On differentiating with respect to x, we get
f  ′(a) = 2a2
dy ⎛ 2x ⎞
= n( x + 1 + x 2 ) n -1 ⎜1 + ⎟ f  ′ (b) = 2ab
dx ⎝ 2 1 + x2 ⎠ f  ′ (c) = 2ac
n( x + 1 + x 2 ) n So they are in AP.
=
1 + x2 The correct option is (A)

2
110. We have
⎛ dy ⎞
⇒ (1 + x 2 ) ⎜ ⎟ = n2 y 2 f ′( x ) = 1
⎝ dx ⎠

On differentiating again with respect to x, we get f ′′( x ) = 0
2 And all other derivatives are 0.
dy d 2 y ⎛ dy ⎞ dy
(1 + x 2 ) ⋅ 2 ⋅ + 2 x ⎜ ⎟ = n2 2 y So the expression =1 – 1 + 0 + 0 + 0 + …
dx dx 2 ⎝ dx ⎠ dx
=0
d2y
dy The correct option is (C)
⇒ (1 + x 2 ) 2
= n2 y
+x
dx dx y + e y + .... ∞
111. Given that x = e y + e ⇒ x = e y+ x
The correct option is (A)
107. Since, sin y = x sin (a + y) we have that dy 1 1- x
sin y ⇒ Inx - x = y ⇒= -1 = .
x= dx x x
sin( a + y) The correct option is (C)
On differentiating with respect to y, we get
f (1 + h) - f (1)
dx sin( a + y )cos y - sin y cos( a + y ) 112. The value of f ′(1) = lim .
= h h→ 0
dy sin 2 ( a + y ) As function is differentiable so it is continuous as it is given
sin( a + y - y ) f (1 + h)
= that lim = 5 and hence f (1) = 0
sin 2 ( a + y ) h→ 0 h

dx sin a f (1 + h)
⇒ = Hence f ′(1) = lim =5
dy sin 2 ( a + y ) h→ 0 h

The correct option is (B) The correct option is (C)
108. ∵ x y = e x - y
f ( x + h) - f ( x )
Taking log on both sides, we obtain 113. f ′( x ) = lim implies
h→ 0 h
y log x = (x− y) logee
f ( x + h) - f ( x ) ( h) 2
x f ′( x ) = lim ≤ lim
⇒y= h→ 0 h h→ 0 h
1 + log x
⇒ | f  ′(x)| ≤ 0 ⇒ f  ′(x) = 0 ⇒ f (x) = constant
On differentiating with respect to x, we get
As f (0) = 0 we have f (1) = 0.
1
(1 + log x ) ⋅ 1 - x ⋅ This correct option is (B)
dy x
= 114. Representing the given function as
dx (1 + log x )2
log x ⎧ x ⎧ x
= ⎪⎪1 - x , x < 0 ⎪ (1 - x )2 , x < 0
(1 + log x )2 ⎪
f ( x) = ⎨ ⇒ f ′( x ) = ⎨
dy log x ⎪ x , x≥0 ⎪ x , x≥0
⇒ = ⎪⎩1 + x ⎪⎩ (1 + x )2
dx (1 + log x ) 2

The correct option is (D) ∴ f  ′(x) exist at everywhere.
The correct option is (C)
1 09. f (x) = ax2 + bx + c
115. Given equation xm⋅yn = (x + y)m + n
So that, f (1) = a + b + c
⇒ mlnx + nlny = (m + n) ln (x + y)
And, f (–1) = a-b + c

Objective_Maths_JEE Main 2017_Ch 13.indd 43 01/01/2008 05:34:56


13.44  Chapter 13

m n dy m + n ⎛ dy ⎞ ⇒ g ′(0) = 2f (2f (0) + 2) . f  ′(2f (0) + 2) . 2(f  ′(0)


\ + = ⎜1 + ⎟ = 4 f (0) f   ′(0)
x y dx x + y ⎝ dx ⎠
= 4(−1) (1) = −4
⎛ m m + n ⎞ ⎛ m + n n ⎞ dy
⇒⎜ - = - The correct option is (A)
⎝ x x + y ⎟⎠ ⎜⎝ x + y y ⎟⎠ dx

⎛ 1 ⎞
my - mx ⎛ my - mx ⎞ dy dy y d ⎛ dx ⎞ d ⎜ 1 d ⎛ dy ⎞
⇒ = ⇒ = ⎜ ⎟ = ⎛ dy ⎞ ⎟ = - ⎜ ⎟
x( x + y ) ⎜⎝ y( x + y ) ⎟⎠ dx
118. dy ⎝ dy ⎠ dy ⎜ ⎜ ⎟ ⎟ ⎛ dy ⎞ dy ⎝ dx ⎠
2
dx x ⎝ ⎝ dx ⎠ ⎠
⎜⎝ ⎟⎠
The correct option is (A) dx
116. Given equation x2x − 2xxcot y − 1 = 0 (1) -2
⎛ dy ⎞ 1 d ⎛ dy ⎞ ⎛ d 2 y ⎞ ⎛ dy ⎞ -3
Now at x = 1, = -⎜ ⎟ ⎜ ⎟ = - ⎜ 2 ⎟ ⎝⎜ dx ⎟⎠
p ⎝ dx ⎠ ⎛ dy ⎞ dx ⎝ dx ⎠ ⎝ dx ⎠
1 − 2 cot y − 1 = 0 ⇒ cot y = 0 ⇒ y = ⎜⎝ ⎟⎠
2 dx
Now, differentiating eq. (1) with respect to ‘x’, we get The correct option is (C)
2x2x( 1 + logx ) − 2
119. Given, y = sec(tan -1 x )
⎡ x 2 dy x ⎤
 ⎢ x ( - cos ec y ) dx + cot yx (1 + log x )⎥ = 0
⎣ ⎦ \
dy
dx
( ) (
= sec tan -1 x tan tan -1 x ⋅
1
)
1 + x2
⎛ p⎞
Now at ⎜1, ⎟ ,
⎝ 2⎠ dy 1 1
x=1 = 2 ×1× = .
dx 2 2
⎛ ⎞
⎛ dy ⎞
2(1 + log1) - 2 ⎜1( -1) ⎜ ⎟ + 0⎟ = 0 The correct option is (D)
⎜ ⎝ dx ⎠ ⎛1, p ⎞ ⎟
⎝ ⎜⎝
2⎠
⎟ ⎠ 120. f (g (x)) = x

f  ′ (g (x)) g′ (x) = 1
⎛ dy ⎞ ⎛ dy ⎞
⇒ 2 + 2⎜ ⎟ =0⇒⎜ ⎟ = -1 g′ (x) = 1 + (g(x))5
⎝ dx ⎠ ⎛1, x ⎞ ⎝ dx ⎠ ⎛1, x ⎞
⎝ 2⎠ ⎜ ⎟ ⎜⎝ ⎟⎠
2 The correct option is (D)
The correct option is (A)
⎛ d ⎞
117. g ′( x ) = 2( f ( 2 f ( x ) + 2)) ⎜ ( f ( 2 f ( x ) + 2))⎟
⎝ dx ⎠
= 2 f ( 2 f ( x ) + 2) f ′( 2 f ( x ) + 2) ⋅ ( 2 f ′( x ))

Objective_Maths_JEE Main 2017_Ch 13.indd 44 01/01/2008 05:34:58


Applications of Derivatives 14.1

CHaPtER

14 Applications of Derivatives

Chapter Highlights
Tangents and normals, Slope of tangent, Equation of tangent, Slope of normal, Equation of normal, Angle of
intersection of two curves, Length of tangent, length of normal, sub-tangent and subnormal, Length of intercept
made on axes by the tangent, Length of perpendicular from origin to the tangent, Increasing and decreasing
functions (monotonicity), Test for monotonicity of functions, Maxima and minima of functions, Method to deter-
mine the points of local maxima and local minima, Greatest and least values of a function in a closed interval
(absolute maximum and absolute minimum), Concavity and convexity of a function, Point of inflexion, Rolle’s
dy
and lagrange’s mean value theorem, Rolle’s theorem, Lagrange’s mean value theorem, Application of as a
dx
rate measure.

tangEntS and normalS Equation of tangEnt


geometrical meaning of derivative The equation of a tangent to a curve y = f (x) at a given point
at a point P (x1, y1) is given by

The derivative of a function f (x) at a point x = a is the slope


⎛ dy ⎞
of the tangent to the curve y = f (x) at the point [a, f (a)]. y – y1 = ⎜ ⎟ (x – x1)
⎝ dx ⎠ ( x , y )
1 1

SlopE of tangEnt [Using point slope form of equation of the straight line]
Consider a curve y = f (x) and a point P(x, y) on this curve.
If tangent to the curve at P(x, y) makes an angle θ with
dy SlopE of normal
the positive direction of x-axis, then, at the point P(x, y):
dx
= tan θ = m = gradient or slope of tangent to the curve at The normal to a curve at a point P(x1, y1) is a line perpen-
P(x, y). dicular to the tangent at P and passing through P. Slope of
the normal
Y
−1 −1 ⎛ dx ⎞
= = = −⎜ ⎟
Slope of tangent ⎛ dy ⎞ ⎝ dy ⎠ P ( x , y )
y = f(x) P = (x, y) ⎜⎝ ⎟⎠ 1 1

dx P ( x ⋅ y ) 1 1

θ
X
O
Fig. 14.1

Objective_Maths_JEE Main 2017_Ch 14.indd 1 01/01/2008 05:42:52


14.2  Chapter 14

Equation of Normal Thus, if m be the slope of a line, then the slope of a line
1
The equation of normal to a curve y = f (x) at a given point perpendicular to it is − and parallel to it is m.
m
P(x1, y1) is given by
−1
Solved Examples
y – y1 = (x – x1)
⎛ dy ⎞
⎜⎝ ⎟⎠ 1. If the tangent at each point of the curve
dx ( x , y )
1 1
2 3
x – 2ax2 + 2x + 5
y=
Y 3
en
t makes an acute angle with the positive direction of
No x-axis, then
ng

rm
Ta

al P (x1, y1) (A) a ≥ 1 (B)  – 1 ≤ a ≤ 1


(C) a ≤ – 1 (D)  None of these
y = f (x)
Solution: (B)
X 2 3
O
We have, y = x – 2ax2 + 2x + 5
Fig. 14.2 3
dy
Y dy ⇒  = 2x2 – 4ax + 2
=0 dx
dx
dx
=0
Since, the tangent makes an acute angle with the posi-
dy tive direction of x-axis, therefore,
dy dy
<0 >0
dx dx dy
≥ 0 ⇒ 2x2 – 4ax + 2 ≥ 0 for all x
dx
⇒ 16a2 – 16 ≤ 0
X (∵ Disc. = (4a)2 – 4 (2) (2) ≤ 0)
O 2
⇒ a – 1 ≤ 0 i.e. (a – 1) (a + 1) ≤ 0
Fig. 14.3 ⇒–1≤a≤1

2. If m be the slope of a tangent to the curve e2y = 1 + 4x2,


Trick(s) for Problem Solving then
(A) m < 1 (B)  |m|≤1
dy
■ If > 0, the tangent makes an acute angle with the x-axis. (C) | m | > 1 (D)  None of these
dx
dy Solution: (B)
If < 0, the tangent makes an obtuse angle with the x-axis. dy

dx We have, e2y = 1 + 4x2  ⇒ e2y · 2 = 8x
dx
dy
■ If = 0, the tangent is parallel to x-axis. dy 4x
dx ⇒ = 2y
■ If the tangent is perpendicular to x-axis, then dx e
dy dy
= ∞, i.e., = 0. 4x
dx dx =
■ If the tangent is equally inclined to the axes, then
1 + 4x2
dy 4x
= tan 45º or tan 135º = ± 1. ∴ Slope of tangent = m =
dx 1 + 4x2
■ The slope of a line having equation ax + by + c = 0 is given by
4| x |
a coefficient of x ⇒|m|= ≤1
m = − = − 1 + 4 | x |2
b coefficient of y
■ The two lines having slopes m1 and m2 are ⎡∵ (1 − 2 | x |) 2 ≥ 0 ⇒ 1 + 4 | x |2 −4 | x | ≥ 0 ⎤
(i) perpendicular if m1m2 = –1 and ⎢ ⎥
⎢ 4| x| ⎥
(ii) parallel if m1 = m2. ⇒ ≤1
⎢ 1 + 4 | x |2 ⎥
⎣ ⎦

Objective_Maths_JEE Main 2017_Ch 14.indd 2 01/01/2008 05:42:54


Applications of Derivatives  14.3

Solution: (B)
3. The tangent to the curve x + y = 4 is equally 1
At the point x = , y = e–1
inclined to the axes at the point 2
(A)  (1, – 2) (B)  (4, 4) 1
Since, x = > 0, ∴ y = e–2x
(C)  (4, – 4) (D)  (– 4, 4) 2
Solution: (B) Differentiating with respect to x, we get

We have, x + y = 4 (1) dy dy ⎤ −2
= – 2e–2x ⇒ = – 2e–1 =
dx dx ⎥⎦ ⎛⎜ 1 , 1 ⎞⎟ e
1 1 dy dy y ⎝ 2 e⎠
⇒ + = 0, or =– .
2 x 2 y dx dx x Thus, the equation of normal is
Since the tangent is equally inclined to the axes, ⎛ 1⎞ e ⎛ 1⎞ 2
⎜⎝ y − ⎟⎠ = ⎜⎝ x − ⎟⎠ i.e., 2e (ex – 2y) = e – 4
dy e 2 2
= tan 45º or tan 135º i.e., 1 or – 1. Thus
dx
6. If the line ax + by + c = 0 is a tangent to the curve
y xy = 4, then
– = ± 1. This gives y = x
x (A) a > 0, b > 0 (B)  a > 0, b < 0
(C) a < 0, b > 0 (D)  a < 0, b < 0
From (1), x + x = 4 ⇒ x = 4. Also, y = x = 4.
The point is (4, 4). Solution: (A, D)
x2 y2 dy
4. The angle between the tangents to the curve 2 + 2  = 1 We have, xy = 4 ⇒ x · +y·1=0
a b dx
at the points (a, 0) and (0, b) is
dy y 4
π π i.e., =– = – 2 (∵ xy = 4)
(A)  (B)  dx x x
4 2
4
π ∴ Slope of tangent = –
(C)  (D)  None of these x2
3
a
Solution: (B) Slope of the line ax + by + c = 0 is = – .
b
x2 y2 Since the given line is a tangent to the curve
We have, + =1
a2 b2 4 a a
∴ – 2
=– ⇒ >0
x b b
2x 2 y dy dy − b2 x
⇒ + = 0 ⇒ = It is possible only when a > 0, b > 0 or a < 0, b < 0.
a2 b 2 dx dx a2 y
7. If the line ax + by + c = 0 is a normal to the curve
dy ⎤ π xy = 1, then
∴ tan θ1 = ⎥ = ∞ ⇒ θ1 =
dx ⎦ ( a, 0 ) 2 (A) a > 0, b > 0 (B)  a > 0, b < 0
(C) a < 0, b > 0 (D)  a < 0, b < 0
dy ⎤
and tan θ2 = = 0 ⇒ θ2 = 0
dx ⎥⎦ ( 0, b ) Solution: (B, C)
1
We have xy = 1 ⇒ y =
Hence, the angle between the two tangents is x
π π dy 1
θ = θ1 – θ2 = –0= ∴  =– 2
2 2 dx x
5. The equation of the normal to the curve y = e–2 | x | at ∴  The slope of the normal = x2
1 If ax + by + c = 0 is normal to the curve xy = 1
the point where the curve cuts the line x = is a a
2 then x2 = – ∴– >0
(A) 2e (ex + 2y) = e2 – 4   (B)  2e (ex – 2y) = e2 – 4 b b
(C) 2e (ey – 2x) = e2 – 4  (D)  None of these ⇒  a > 0, b < 0 or a < 0, b > 0

Objective_Maths_JEE Main 2017_Ch 14.indd 3 01/01/2008 05:42:58


14.4  Chapter 14

x y 1 1
8. The line + = 1 touches the curve y = be–x/a at the (A) a = , b = 1, c =
a b 2 2
point 1 1
⎛ a⎞ (B) a = 1, b = , c =
(A) (– a, ba) (B)  ⎜⎝ a, ⎟⎠ 2 2
b
⎛ b⎞ 1 1
(C)  ⎜ a, ⎟ (D)  None of these (C) a = , c = , b = 1
⎝ a⎠ 2 2
Solution: (D) (D)  None of these
We have, y = be–x/a(1) Solution: (A)
dy b –x/a The given curve is y = ax2 + bx + c (1)
⇒    =– e Since the point (– 1, 0) lie on it
dx a
x y ∴ a – b + c = 0 (2)
Since the line + = 1 touches (1)
a b Also, y = 2x is a tangent to (1) at x = 1, so that y = 2.
Since the point (1, 2) lies on (1),
−1 / a b –x/a b b –x/a
∴ =– e ⇒– =– e ∴ a + b + c = 2 (3)
1/ b a a a
x dy ⎤
⇒  1 = e–x/a ⇒ – = 0 i.e., x = 0. Also = ( 2ax + b) ](1, 2) = 2,
a dx ⎥⎦ (1, 2)
∴  y = be0 = b. ∴ 2a + b = 2 (4)
Hence, the required point is (0, b). 1 1
Solving (2), (3) and (4): a = , b = 1, c =
9. The curve y = ax3 + bx2 + cx is inclined at 45º to x-axis 2 2
at (0, 0) but it touches x-axis at (1, 0), then the values
of a, b, c are given by
(A) a = 1, b = – 2, c = 1 Angle of Intersection of Two Curves
(B) a = 1, b = 1, c = – 2
(C) a = – 2, b = 1, c = 1 Let y = f (x) and y = g (x) be two curves intersecting at a
(D) a = – 1, b = 2, c = 1 point P(x1, y1). Then, the angle of intersection of these two
curves is defined as the angle between the tangents to the
Solution: (A) two curves at their point of intersection.
We have, y = ax3 + bx2 + cx
dy Y
⇒   = 3ax2 + 2bx + c y = f (x)
dx y = g (x)
θ
dy ⎤
∴   ⎥ = c = tan 45º = 1 (Given) P (x1, y1)
dx ⎦ ( 0, 0 )
θ1 – θ2
⇒  c = 1
θ2 θ1
dy ⎤ O X
Also, = 3a + 2b + c = 0
dx ⎥⎦ (1, 0 ) Fig. 14.4
(∵ x-axis is tangent at (1, 0))
⇒  3a + 2b + 1 = 0 If θ is the required angle of intersection, then,
which is true if a = 1, b = – 2. θ = ± (θ1 – θ2),
Hence, a = 1, b = – 2, c = 1. where θ1 and θ2 are the inclinations of tangents to the
2
10. If the line y = 2x touches the curve y = ax + bx + c at curves y = f (x) and y = g (x) respectively at the point P.
the point where x = 1 and the curve passes through the
point (– 1, 0), then the values of a, b and c are

Objective_Maths_JEE Main 2017_Ch 14.indd 4 01/01/2008 05:43:01


Applications of Derivatives 14.5

Since m1 = m2, therefore the two curves touch each


trick(S) for problEm Solving
other at (1, 2).
Short-cut method to find the angle of intersection 12. The angle of intersection of the curves y = 2 sin2x and
Find f ′ (x) and g′ (x). π

y = cos 2x at x = is
■ If f ′ (x) × g′ (x) = – 1, then the two curves are said to cut 6
each other orthogonally, wherever they cut. π π
(A) (B)
■ If the product is not – 1, solve the equation of the two 4 3
curves to get their point of intersection. If (θ, θ ) be their π
point of intersection, then find f ′ (θ ) and g′ (θ ). Let (C) (D) None of these
m1 = f ′ (θ ) and m2 = g′ (θ ). 2
■ If θ is the angle between the tangents, then Solution: (B)
m1 − m2 f ′(α ) − g′(α ) We have,
tan θ = ± =±
1 + m1m2 1 + f ′(α )g′(α ) y = 2sin2x (1)
and y = cos 2x (2)
Repeat this process for other points of intersection.
Differentiating (1) with respect to x, we get
dy
I M P O R TA N T P O I N T S = 4 sin x cos x
dx
The two curves are said to touch each other at their point of dy ⎤ ⎛ 1⎞ 3
intersection (α, β ), if the slope of their tangents at (α, β ) ⇒ ⎥ = 4 ·⎜ ⎟ ·= 3 = m1 (say)
dx ⎦ x = π ⎝ 2⎠ 2
are equal. 6

Differentiating (2) with respect to x, we get


SolvEd ExamplES dy dy ⎤ π
= – 2 sin 2x ⇒ = – 2 sin
dx dx ⎥⎦ x = π 3
11. The two curves y2 = 4x and x2 + y2 – 6x + 1 = 0 at the 6

point (1, 2)
= – 3 = m2 (say).
(A) Intersect orthogonally
π Hence, angle between the two curves is
(B) Intersect at an angle
3
(C) Touch each other ⎛ m − m2 ⎞ π 2π
θ = ± tan–1 ⎜ 1 = ± tan–1 3 = or
(D) None of these ⎝ 1 + m1m2 ⎟⎠ 3 3
Solution: (C)
We have, y2 = 4x (1) n n
x y ⎛ x⎞ ⎛ y⎞
and 2 2
x + y – 6x + 1 = 0 (2) 13. The line + = 2 touches the curve ⎜ ⎟ + ⎜ ⎟  = 2
a b ⎝ a ⎠ ⎝ b⎠
Differentiating (1) with respect to x, we get at the point (a, b) for
(A) n = 2 only (B) n = – 3 only
dy dy 2
2y =4⇒ = (C) n is any real number (D) None of these
dx dx y
Solution: (C)
dy ⎤ 2
∴ ⎥ = = 1 = m1 (say) n n
dx ⎦ (1, 2) 2 ⎛ x⎞ ⎛ y⎞
We have, ⎜ ⎟ + ⎜ ⎟ = 2
⎝ a⎠ ⎝ b⎠
Differentiating (2) with respect to x, we get
nx n −1 ny n −1 dy
dy dy 3− x ⇒ + =0
2x + 2y –6=0⇒ = an b n dx
dx dx y
dy ⎤ 3 −1 dy ⎤ b n ⋅ a n −1 b
∴ = = 1 = m2 (say) ⇒ = – =–

dx ⎦ (1, 2) 2 dx ⎥⎦ ( a, b ) a n b n −1 a

Objective_Maths_JEE Main 2017_Ch 14.indd 5 01/01/2008 05:43:05


14.6 Chapter 14

∴ The equation of tangent at (a, b) is Since the two curves intersect at right angles,
b x y ⎛ 1⎞ ⎛ y⎞
y–b=– (x – a) ⇒ + = 2. ∴ m1m2 = – 1 ⇒ ⎜ ⎟ ⎜⎝ − x ⎟⎠ = – 1 ⇒ x1 = 1
a a b ⎝ y1 ⎠ 1
x y
∴ The line + = 2 touches the curve at (a, b), for and hence from y12 = 2x1, we get y12 = 2.
all n. a b
Since (x1, y1) also lies on 2xy = k
4 1 ∴ k2 = 4 x12 y12 = 4 × 1 × 2 = 8
14. The least value of a for which + =a
sin x 1 − sin x
⎛ π⎞ 16. If the curve y = x2 + bx + c touches the line y = x at the
has at least one solution in the interval ⎜ 0, ⎟ is point (1, 1), then the values of x for which the curve
⎝ 2⎠
has a negative gradient are
(A) 9 (B) 8
1 1
(C) 6 (D) 4 (A) x < (B) x >
2 2
Solution: (A) 1 1
(C) x < – (D) x > –
4 1 2 2
f (x) = + =a
sin x 1 − sin x Solution: (A)
4 cos x cos x We have, y = x2 + bx + c
⇒ f ′ (x) = − +
2
sin x (1 − sin x ) 2 dy
⇒ = 2x + b
dx
⎛ 1 4 ⎞
= cos x ⎜ − Since the curve touches the line y = x at the point (1, 1)
⎝ (1 − sin x ) 2
sin 2 x ⎟⎠
∴ ( 2 x + b) ](1, 1) = 1 i.e., 2 + b = 1 ⇒ b = – 1.
∴ f ′(x) = 0
Also, the curve passes through the point (1, 1)
1 4 ⎛ π⎞
⇒ − = 0 as cos x ≠ 0 in ⎜ 0, ⎟ ∴ 1 = 1 + b + c i.e., c = – b = 1
(1 − sin x ) 2 2
sin x ⎝ 2⎠
dy
The given x = 2/3. Substituting in f (x) = 0, we get a = 9. ∴ y = x2 – x + 1 ⇒ = 2x – 1
dx
15. The curves y2 = 2x and 2xy = k cut at right angles if dy 1
Now, < 0 ⇒ 2x – 1 < 0 ⇒ x <
(A) k2 = 8 (B) k2 = 4 dx 2
2
(C) k = 2 (D) None of these
Solution: (A) REMEMBER
Let P (x1, y1) be the point of intersection of the two
curves. If one angle between the tangents (acute/obtuse) is θ, then
We have, the other angle between the tangents (obtuse/acute) is
(180º – θ ).
dy ⎛ dy ⎞ ⎤ Generally, we take the acute angle to be the angle of intersec-
y2 = 2x ⇒ 2y = 2 ⇒ m1 = ⎜ ⎟ ⎥
dx ⎝ dx ⎠ ⎦ ( x , y ) tion of the given curves.
1 1

1
=
y1 lEngth of tangEnt, lEngth
dy of normal, Sub-tangEnt and
and 2xy = k ⇒ x +y=0
dx Subnormal
Let the tangent and normal at the point P(x, y) on the curve
⎛ dy ⎞ ⎤ y
⇒ m2 = ⎜ ⎟ ⎥ =– 1 meet the axis of x at the points T and N respectively. Let M
⎝ dx ⎠ ⎦ ( x , y ) x1
1 1 be the foot of the ordinate at P. Then,

Objective_Maths_JEE Main 2017_Ch 14.indd 6 01/01/2008 05:43:09


Applications of Derivatives  14.7

Y
y = f (x) Solved Examples

t
en
No

ng
rm 17. The sub-normal at any point of the curve

Ta
al P (x, y)
x2y2 = a2 (x2 – a2) varies as
θ
(A) (abscissa)– 3 (B) (abscissa)3
–3
θ (C) (ordinate) (D)  None of these
O X
T M N
Solution: (A)
Fig. 14.5 We have, x2y2 = a2 (x2 – a2)(1)

1. Length of the tangent = PT = | y cosec θ | dy


⇒  x2 × 2y + y2 · 2x = a2 · 2x
dx
2
⎛ dy ⎞ dy a2 − y 2
y 1+ ⎜ ⎟ ⇒  =
⎝ dx ⎠ dx xy
= y 1 + cot 2 θ =
dy
dy a 2 − y 2 x 2 (a2 − y 2 )
dx ∴ Sub-normal = y = =
dx x x3
a4
2. Length of the normal = PN = | y sec θ | = [∵ from (1) x2 (a2 – y2) = a4]
x3
2
⎛ dy ⎞ ⇒ The sub-normal varies inversely as the cube of its
= y 1 + tan 2 θ = y 1 + ⎜ ⎟
⎝ dx ⎠ abscissa.
18. The sub-tangent at any point of the curve xmyn = am + n
varies as
y (A) (abscissa)2 (B) (ordinate)2
3. Subtangent = TM = | y cot θ | = (C) abscissa (D) ordinate
⎛ dy ⎞
⎜⎝ ⎟⎠
dx Solution: (C)
We have, xmyn = am + n
⎛ dy ⎞ ⇒  m log x + n log y = (m + n) log a
4. Subnormal = MN = | y tan θ | = y ⎜ ⎟ .
⎝ dx ⎠
m n dy dx nx
∴ + =0⇒ =–
x y dx dy my
Length of Intercept made on Axes dx nx
by the Tangent ∴ Sub-tangent = y = ∝x
dy m
Equation of tangent at any point (x1, y1) to the curve y = f (x)
19. For the parabola y2 = 4ax, the ratio of the sub-tangent
⎛ dy ⎞
is y – y1 = ⎜ ⎟ ( x − x1 ) . to the abscissa is
⎝ dx ⎠ ( x , y )
1 1 (A)  1 : 1 (B) 2 : 1
⎛ ⎞ (C) x : y (D) x2 : y
⎜ y1 ⎟ Solution: (B)
 Then, x-intercept = x1 – ⎜ ⎟
⎜ ⎛ dy ⎞ ⎟ We have, y2 = 4ax
⎜ ⎜⎝ dx ⎟⎠ ⎟
⎝ (x , y ) ⎠ dy dy 2a
1 1
⇒ 2y = 4a i.e., =
dx dx y
⎛ ⎛ dy ⎞ ⎞
 and y-intercept = y1 – ⎜ x1 ⎜ ⎟ ⎟ y y y2 4 ax
⎝ ⎝ dx ⎠ ( x , y ) ⎠ 1 1
Sub-tangent =
dy / dx
=
2a / y
=
2a
=
2a
= 2x.

∴ Sub-tangent : Abscissa = 2x : x = 2 : 1.

Objective_Maths_JEE Main 2017_Ch 14.indd 7 01/01/2008 05:43:12


14.8  Chapter 14

Length of perpendicular from Strictly Decreasing Function


origin to the Tangent A function f (x) is said to be a strictly decreasing function
Length of perpendicular from origin (0, 0) to the tangent on an interval I, if
drawn at point P(x1, y1) to the curve y = f (x) is x1 < x2 ⇒ f (x1) > f (x2) ∀ x1, x2 ∈ I

⎛ dy ⎞ Y
y1 − x1 ⎜ ⎟
⎝ dx ⎠ ( x , y )
p= 1 1

2
⎛ dy ⎞
1+ ⎜ ⎟
⎝ dx ⎠ f (x1)
f (x2)
X
Increasing and Decreasing O x1 x2
Functions (Monotonicity) Fig. 14.8 Strictly Decreasing Function

Increasing Function Monotonic Function


A function f(x) is said to be an increasing function on an A function f (x) is said to be monotonic on an interval I if it
interval I, if is either increasing or decreasing on I.
x1 < x2 ⇒ f (x1) ≤ f (x2), ∀ x1, x2 ∈ I

Y
Test for Monotonicity of Functions
1. f (x) is increasing in [a, b] if f ′ (x) ≥ 0, ∀ x ∈ [a, b].
2. f (x) is strictly increasing in [a, b] if f ′ (x) > 0, ∀ x ∈
f (x2)
[a, b].
f (x1) 3. f (x) is decreasing in [a, b] if f ′ (x) ≤ 0, ∀ x ∈ [a, b].
4. f (x) is strictly decreasing in [a, b] if f ′ (x) < 0, ∀ x ∈
x1 x2 X
O [a, b].
Fig. 14.6 Increasing Function
Trick(s) for Problem Solving
Strictly Increasing Function
■ If a function f  (x) is strictly increasing (strictly decreasing)
A function f (x) is said to be a strictly increasing function on an interval I, then f –1 exists and is also strictly increasing
on an interval I, if (strictly decreasing).
■ If f  (x) is monotonic on an interval I, then f  (x) has at the
x1 < x2 ⇒ f (x1) < f (x2), ∀ x1, x2 ∈ I
most one zero in the interval I.
Y ■ If the functions f  (x) and g (x), both are increasing or

decreasing on an interval I, then the composite function


(gof ) (x) is an increasing function on I.
■ If the function f  (x) is increasing (decreasing) and g (x)
f (x2) decreasing (increasing) on an interval I, then the composite
f (x1) function (gof ) (x) is decreasing on the interval I.
X ■ A function may be increasing in some interval I1 and
O x1 x2
decreasing in some other interval I2.
Fig. 14.7 Strictly Increasing Function

Decreasing Function Solved Examples


A function f (x) is said to be a decreasing function on an
interval I, if 20. If f (x) is a polynomial function such that, f (x) > f (x), ∀
x ≥ 1 and f (1) = 0, then
x1 < x2 ⇒ f (x1) ≥ f (x2), ∀ x1, x2 ∈ I
(A) f (x) < 0 ∀ x ≥ 1 (B)  f (x) ≥ 0 ∀ x ≥ 1
Decreasing Function (C) f (x) = 0, ∀ x ≥ 1 (D)  None of these.

Objective_Maths_JEE Main 2017_Ch 14.indd 8 01/01/2008 05:43:12


Applications of Derivatives  14.9

Solution: (B) But f ′ (n) ≠ 0 for any n ∈ N. Hence f (n) has no


We have, f ′(x) > f ′(x) > f (x), ∀ x ≥ 1 critical point.
But the function f (n) is increasing for n < 8 and it
⇒  f ′(x) – f (x) > 0, ∀ x ≥ 1 starts decreasing for n ≥ 8.
⇒  e–x[ f ′(x) – f (x)] > 0, ∀ x ≥ 1 49 8
Here, f (7) = and f (8) = .
d 543 89
⇒  [e–xf (x)] > 0, ∀ x ≥ 1 Clearly, f (7) > f (8).
dx
49
∴  e–xf (x) is an increasing function, ∀ x ≥ 1 Hence, the largest value is .
543
⇒  e–xf (x) ≥ e–1f (1)(∴  f (x) is a polynomial)
⇒  e–xf (x) ≥ 0 [∴  f (1) = 0] x

⇒  f (x) ≥ 0, ∀ x ≥ 1
23. Let f (x) = ∫ log 2 {log3 [log 4 (cos t + a)]} dt be increas-
0
ing for all real values of x, then
21. The subtangent, ordinate and subnormal to the parab-
ola y2 = 4ax at a point (different from the origin) are in (A) a ≥ 5 (B)  0≤a<4
(C) a < 0 (D)  None of these
(A)  GP (b) AP
(C)  HP (d) None of these Solution: (A)
Solution: (A) We have, f ′ (x) = log 2 log3 log 4 (cos x + a)
We have, y2 = 4ax
Clearly, f ′(x) ≥ 0 for all x
dy dy 2a ⇒  x = ± 1
⇒ 2y = 4a i.e., = .
dx dx y
∴ f (x) is increasing for all real x, provided f (x) is defined.
y y y2
Sub-tangent = = = Now, f (x) is defined, if
dy / dx 2a / y 2a
log3 log4(cos x + a) > 0, ∀ x ∈ R
dy 2a
Sub-normal = y =y× = 2a
dx y ⇒ log4(cos x + a) > 4° = 1, ∀ x ∈ R
2
y ⇒ cos x + a > 4, ∀ x ∈ R
Clearly, , y, 2a are in G.P.
2a ⇒  a > 4 – cos x, ∀ x ∈ R
22. The largest term in the sequence ⇒  a > 5 (∴ 1 ≤ cos x ≤ 1)
2
n
xn = , n ∈ N, is ⎛ π⎞
3
n + 200 24. Let f ′(sin x) < 0 and f ″(sin x) > 0 ∀ x ∈ ⎜ 0, ⎟ and
⎝ 2⎠
49 8 g(x) = f (sin x) + f (cos x), then g (x) is decreasing in
(A)  (B) 
543 89 ⎛ π⎞ ⎛π π⎞
1 ⎜⎝ , ⎟⎠
(A)  ⎜ 0, ⎟ (B) 
(C)  (D)  None of these ⎝ 4⎠ 4 2
52
⎛ π⎞
Solution: (A) (C)  ⎜ 0, ⎟ (D)  None of these
⎝ 2⎠
n2
Let  f (n) = xn = ,n∈N Solution: (A)
n3 + 200
We have, g′(x) = f ′(sin x) · cos x – f ′(cos x) · sin x
( n3 + 200) ⋅ 2n − n2 ⋅ 3n2 ⇒  g″(x) = –f ′(sin x) · sin x + f ″(sin x) · cos2x
⇒ f ′ (n) = 3 2
( n + 200)
+ f ″(cos x) · sin2x – f ′(cos x) · cos x
n ( 400 − n3 )
= ⎛ π⎞
( n3 + 200) 2 > 0, ∀ x ∈ ⎜ 0, ⎟
⎝ 2⎠

Objective_Maths_JEE Main 2017_Ch 14.indd 9 01/01/2008 05:43:15


14.10  Chapter 14

27. The function f (x) = 2x2 – log | x |, x ≠ 0 is increasing in


⎛ π⎞ ⎛π⎞
⇒ g′(x) is increasing in . ⎜ 0, ⎟ Also, g′ ⎜ ⎟ = 0 the interval
⎝ 2⎠ ⎝ 4⎠
⎛ 1 ⎞ ⎛1 ⎞
(A)  ⎜ − , 0⎟ ∪ ⎜ , ∞⎟
⎛π π⎞ ⎛ π⎞ ⎝ 2 ⎠ ⎝2 ⎠
⇒ g′(x) > 0 ∀ x ∈ ⎜ , ⎟ and g′(x) < 0 ∀ x ∈ ⎜ 0, ⎟
⎝ 4 2⎠ ⎝ 4⎠
⎛ 1⎞ ⎛ 1⎞
(B)  ⎜ −∞, − ⎟ ∪ ⎜ 0, ⎟
⎛ π⎞ ⎝ 2 ⎠ ⎝ 2⎠
Thus, g(x) is decreasing in ⎜ 0, ⎟
⎝ 4⎠ ⎛ 1 1⎞
(C)  ⎜ − , ⎟
⎝ 2 2⎠
25. Let f (x) = (1 + b2) x2 + 2bx + 1 and m (b) the minimum (D)  None of these
value of f (x) for a given b. As b varies, the range of m
(b) is Solution: (A)
(A)  [0, 1] (B)  (0, 1/2] We have, f (x) = 2x2 – log | x |, x ≠ 0
⎡1 ⎤ Case I. When x < 0, f (x) = 2x2 – log | x | = 2x2 – log (– x)
(C)  ⎢ , 1⎥ (D) 
(0, 1]
⎣2 ⎦ 1 1 4x2 − 1
∴  f ′ (x) = 4x – (– 1) = 4x – =
Solution: (D) ( −x ) x x
We have, f (x) = (1 + b2) Case II. When x > 0, f (x) = 2x2 – log | x | = 2x2 – log x
1
⎡ 2 2b b2 ⎤ b2 ∴  f ′ (x) = 4x –
⎢x + x + ⎥ − +1 x
⎣ 1 + b2 (1 + b 2 ) 2 ⎦ 1 + b 2 1 4x2 − 1
Thus, when x < 0 or x > 0, f ′ (x) = 4x – =
2 x x
⎛ 2 b ⎞ 1 1
= (1 + b ) ⎜ x + 2⎟
+ 2
≥ 4x2 − 1
⎝ 1+ b ⎠ 1+ b 1 + b2 Sign scheme for :
x
1
∴ m (b) = . So, range of m (b) = (0, 1]. –∞ –ve +ve 0 –ve +ve +∞
1 + b2 – 1 1
2 2

Thus, f (x) is an increasing function in the interval


26. If f (x) = xex (1 – x) then f (x) is
⎡ 1 ⎤ ⎛ 1 ⎞ ⎛1 ⎞
(A)  increasing on ⎢ − , 1⎥ ⎜ − 2 , 0⎟ ∪ ⎜ 2 , ∞⎟ .
⎣ 2 ⎦ ⎝ ⎠ ⎝ ⎠
(B)  decreasing on R
(C)  increasing on R
Maxima and Minima of Functions
⎡ 1 ⎤
(D)  decreasing on ⎢ − , 1⎥
⎣ 2 ⎦ Local Maximum
Solution: (A) A function y = f (x) is said to have a local maximum value
We have, f ′ (x) = ex (1 – x) + x · ex (1 – x) · (1 – 2x) at a point x = a, if f (x) ≤ f (a), ∀ x ∈ (a – h, a + h), for small
h > 0, i.e. f (a) is the greatest of all the values of f (x) in the
= ex (1 – x) [1 + x (1 – 2x)] = ex (1 – x) · (– 2x2 + x + 1). interval (a – h, a + h).
Now, the sign-scheme for – 2x2 + x + 1 is
Y Local maximum

–ve –1/2 +ve 1 –ve

⎡ −1 ⎤
f (a + h)
f (a – h)

∴ f ′ (x) ≥ 0 if x ∈ ⎢ , 1⎥ because ex (1 – x) is always


f (a)

⎣2 ⎦
positive.
⎡ 1 ⎤ O X
So, f (x) is increasing on ⎢ − , 1⎥ . a–h a a+h
⎣ 2 ⎦ Fig. 14.9

Objective_Maths_JEE Main 2017_Ch 14.indd 10 01/01/2008 05:43:18


Applications of Derivatives 14.11

The point x = a is called a point of local maximum of the Local maximum


Y
function f (x).

f ′ (x
f′ (x) = 0

)>

)<0
f ′ (x
local minimum
A function y = f (x) is said to have a local minimum value at
a point x = a, if f (x) ≥ f (a), ∀ x ∈ (a – h, a + h), for small
h > 0, i.e., f (a) is the smallest of all the values of f (x) in the
X
interval (a – h, a + h). O a–h a a+h
Fig. 14.11
Y
(ii) If f ′ (x) changes sign from negative to positive
as x increases through a, then x = a is a point of
Local minimum minimum.
f (a + h)
f (a – h)

f (a)

0
f ′ (x

)>
f′ (x) = 0
)<

f ′ (x
O X
a–h a a+h

0
Fig. 14.10 Local minimum

The point x = a is called a point of local minimum of the


function f (x).
X
O a–h a a+h
Fig. 14.12
REMEMBER
(iii) f ′ (x) does not change sign as x increases through
(i) The points at which a function attains either the local a, then x = a is neither a point of maximum nor a
maximum value or local minimum value are called point of minimum. Such a point is called a point
the extreme points and both local maximum and local of inflexion.
minimum values are called the extreme values of the
function f (x). We repeat this process for other values of x and
(ii) The local maximum and local minimum values are also examine them for maxima or minima.
known as relative maximum and relative minimum values
respectively.

CAUTION
mEthod to dEtErminE thE pointS ■ A function may have maxima or minima at a point without
of local maxima and local being derivable at the point.
If f (a) does not exist, the there is no question of extrema
minima ■

at x = a.
method i (first derivative test)
1. For the function y = f (x), find f ′ (x).
2. Put f ′ (x) = 0 and solve this equation for x. Let its roots
be a, b, c etc. These points are called stationary points
method ii (Second derivative test)
or critical points. 1. For the function y = f (x), find f ′ (x) and f ″ (x).
3. At x = a, determine the sign of f ′ (x) for values of 2. Put f ′ (x) = 0 and solve this equation for x. Let its roots
x slightly less than a and that for values of x slightly be a, b, c etc.
greater than a. 3. At x = a
(i) If f ′ (x) changes sign from positive to negative (i) if f ″ (a) < 0, then x = a is a point of local maxima;
as x increases through a, then x = a is a point of (ii) if f ″ (a) > 0, then x = a is a point of local minima;
maximum. (iii) if f ″ (a) = 0, we cannot say any thing.

Objective_Maths_JEE Main 2017_Ch 14.indd 11 01/01/2008 05:43:19


14.12 Chapter 14

grEatESt and lEaSt valuES of Solution: (A)


a function in a cloSEd intErval Let y = x – xp, where x is the fraction
(abSolutE maximum and dy
⇒ = 1 – pxp – 1
abSolutE minimum) dx
dy
If f (x) is continuous in an interval [a, b], then greatest or For maximum or minimum, =0
absolute maximum value of f (x) = max. [ f (a), f (b), values dx
of f (x) at all critical points in (a, b)]. 1/( p − 1)
p–1 ⎛ 1⎞
Also, least or absolute minimum value of f (x) = min. ⇒ 1 – px =0⇒x= ⎜ ⎟ .
⎝ p⎠
[f (a), f (b), values of f (x) at all critical points in (a, b)].
d2 y
Now, = – p (p – 1) xp – 2
dx 2
I M P O R TA N T P O I N T S p − 2 /( p − 1)
d2 y ⎤ ⎛ 1⎞
∴ ⎥ = – p (p – 1) ⎜ ⎟ <0
If a function is defined and continuous on an interval which dx 2 ⎦ x = ⎛ 1 ⎞
1/( p − 1 )
⎝ p⎠
is not a closed interval, then it cannot have any greatest ⎜ ⎟
⎝ p⎠
or least value other than local maximum or local minimum
1
values. ⎛1⎞ ( p −1)
∴ y is maximum at x = ⎜ ⎟
⎝ p⎠
30. The shortest distance of the point (0, 0) from the curve
SolvEd ExamplES 1
y = (ex + e–x) is
2
28. If P (x) = a0 + a1x2 + a2x4 + ... + anx2n be a polynomial (A) 2 (B) 1
in x ∈ R with 0 < a1 < a2 ... < an , then P (x) has (C) 3 (D) None of these
(A) no point of minimum
(B) only one point of minimum Solution: (B)
(C) only two points of minimum Let P(x, y) be the point on the curve which is nearest
(D) None of these to O (0, 0).
1 2x
Solution: (B) Let z = OP2 = x2 + y2 = x2 + (e + e–2x + 2).
We have, 4
dz 1
P (x) = a0 + a1x2 + a2 x4 + ... + an x2n ⇒ = 2x + (e2x – e–2x)
dx 2
⇒ P′ (x) = 2a1x + 4a2 x3 + ... + 2nan x2n – 1 dz
For maximum or minimum, =0
For maximum or minimum, P′ (x) = 0 dx
⇒ x (2a1 + 4a2 x2 + ... + 2nan x2n – 2) = 0 1 2x –2x
⇒ 2x + (e – e ) = 0
2
⇒ x = 0 ( each ai > 0 and powers of x are even)
e −2 x − e 2 x
Now, P˝ (x) = 2a1 + 12a2 x2 + ... + 2n (2n – 1) an x2n –2 ⇒ = 2x
2
∴ P˝ (x)]x = 0 = 2a1 > 0 i.e. P(x) has a minimum at 1 0
⇒ x = 0 is a solution and then y = (e + e0) = 1.
x = 0 only. 2
d2 y
29. The fraction exceeding its pth power by the greatest Also, 2
= 2 + e2x + e–2x > 0,
number possible, where p ≥ 2, is dx
1 p −1
hence, z is minimum.
⎛ 1⎞ p −1 ⎛ 1⎞
(A) ⎜ ⎟ (B) ⎜ ⎟ ∴ The shortest distance OP = 0 2 + 12 = 1.
⎝ p⎠ ⎝ p⎠

(C) p1/p – 1 (D) None of these

Objective_Maths_JEE Main 2017_Ch 14.indd 12 01/01/2008 05:43:22


Applications of Derivatives  14.13

31. Let the function f (x) be defined as nπ cos nπ − sin nπ cos nπ


At x = nπ, f ″ (x) = =
⎪⎧tan α − 3 x , 0 < x < 1
−1 2
( nπ ) 2 nπ
f (x) = ⎨
⎩⎪ − 6 x, x ≥1 n
( −1)
=
f (x) can have a maximum at x = 1 if the value of α is nπ
(A) 0 (B) 2 ∴ Extreme points are x = nπ, n = 1, 2, 3, ..., where
(C)  1 (D)  None of these the maximum occurs at x = π, 3π, 5π, ... and the mini-
mum occurs at x = 2π, 4π, 6π, ...
Solution: (D)
We have, 34. The difference between the greatest and the least value
of the function
⎧ − 6 x, 0 < x < 1
f ′ (x) = ⎨ x
⎩ − 6, x ≥ 1 f (x) = ∫ (6t 2 − 24) dt on [1, 3] is
0
∴  f ′ (1 – h) = – 6 (1 – h) < 0
(A) 14 (B) 10
and f ′ (1 + h) = – 6 < 0 (C)  4 (D)  None of these
Since f ′ (x) does not change sign as x passes Solution: (A)
through 1, therefore, f (x) does not have a maximum or x
minimum at x = 1, whatever be the value of α. We have,  f (x) = ∫ (6t 2 − 24) dt
0
32. If the roots of the equation x3 – ax2 + 4x – 8 = 0 are real
and positive, then the minimum value of a is ⇒ f ′ (x) = (6x2 – 24) × 1
(A) 2 (B) 6
3
f ′ (x) = 0 ⇒ 6x2 – 24 = 0 ⇒ x = ± 2.
(C) 3 4 (D)  None of these
But x = – 2 ∉ [1, 3]. So x = 2 is the only critical point.
Solution: (B)
1
Let α, β, γ be the roots of the given equation. 1
⎛ 6t 3 ⎞
Now,   f (1) = ∫ (6t 2 − 24) dt = ⎜ − 24t ⎟ = – 22,
Then, α + β + γ  = a, αβ + αγ + βγ  = 4, αβγ  = 8. 0 ⎝ 3 ⎠ 0
α +β +γ 2
Since AM ≥ GM ⇒ ≥ 3 αβγ 2
⎛ 6t 3 ⎞
3 (2) = ∫ (6t 2 − 24) dt = ⎜
f  − 24t ⎟ = – 32
a 0 ⎝ 3 ⎠ 0
⇒  ≥ 38 ⇒a≥6
3 3
3
⎛ 6t 3 ⎞
∴ The minimum value of a = 6 and f (3) = ∫ (6t − 24) dt = ⎜ 2
− 24t ⎟ = – 18.
x 0 ⎝ 3 ⎠ 0
sin t
33. For the function f (x) = ∫ t
dt , where x > 0,
Hence, the greatest value of f (x) is – 18 which it
0
attains at x = 3 and the least value is – 32 which is
(A)  maximum occurs at x = nπ, n even
attained at x = 2.
(B)  minimum occurs at x = nπ, n odd
Thus, the difference between the greatest and the
(C)  maximum occurs at x = nπ, n odd
least value of the function
(D)  minimum occurs at x = nπ, n even
= f (3) – f (2) = – 18 + 32 = 14
Solution: (C, D)
We have, 4
− x3 + x 2 )
35. The minimum value of e( x is
sin x x cos x − sin x
f ′ (x) = and f ″ (x) = (A) e (B)  e2
x x2
(C)  1 (D)  None of these
For maximum or minimum, f ′ (x) = 0
Solution: (C)
sin x
⇒   = 0 ⇒ sin x = 0; x ≠ 0.
x − x3 + x 2 )
is minimum when (x4 – x3 + x2) is minimum.
4

e( x
∴ x = nπ ; n = 1, 2, 3, ... (∵ x > 0)

Objective_Maths_JEE Main 2017_Ch 14.indd 13 01/01/2008 05:43:24


14.14  Chapter 14

Since  x4 – x3 + x2 = x2 (x2 – x + 1) Solution: (B)


⎡⎛ Let α and β be the roots of the given equation. Then
3⎤
2
1⎞
= x2 ⎢⎜ x − ⎟ + ⎥ α + β = k and αβ = (2k – 3)
⎢⎣⎝ 2⎠ 4 ⎥⎦
Let   z = α3 + β3 = (α + β )3 – 3αβ (α + β )
≥ 0, ∀ x ∈ R = k3 – 3k (2k – 3) = k3 – 6k2 + 9k
∴ The minimum value of (x4 – x3 + x2) is 0 for x = 0. dz
( x 4 − x3 + x ) 0
∴  = 3k2 – 12k + 9 = 3 (k2 – 4k + 3)
Hence, the minimum value of e is e = 1. dk
36. The minimum value of the function = 3 (k – 1) (k –3)
f (x) = 2 | x – 2 | + 5 | x – 3 |, ∀ x ∈ R is dz
Now, = 0 ⇒ 3 (k – 1) (k – 3) = 0 ⇒ k = 1, 3
(A) 3 (B) 2 dk
(C) 5 (D) 7
d2z ⎤
Solution: (B) Also, ⎥ = (6 k − 12) k =3
= 6 (3) – 12 > 0
dk 2 ⎦ k = 3
We have,
 f (x) = 2 | x – 2 | + 5 | x – 3 | Hence, z is minimum when k = 3.
= 2 (2 – x) + 5 (3 – x) = 19 – 7x, if x < 2 39. If f (x) = | x | + | x – 1 | + | x – 2 |, then
= 5, if x = 2 (A) f (x) has minima at x = 1
= 2 (x – 2) + 5 (3 – x) = 11 – 3x, if 2 < x < 3 (B) f (x) has maxima at x = 0
= 2 (3 – 2) = 2, if x = 3 (C) f (x) has neither maxima nor minima at x = 0
(D) f (x) has neither maxima nor minima at x = 2.
= 2 (x – 2) + 5 (x – 3) = 7x – 19, if x > 3
Solution:  (A, C, D)
Thus, we find that f (x) has a minimum value 2 at x = 3.
We have,
37. If (x – a)2n (x – b)2m + 1, where m and n are positive
integers and a > b, is the derivative of a function f, then (x) = | x | + | x – 1 | + | x – 2 |
f 
(A) x = a gives neither a maximum nor a minimum ⎧ − 3 x + 3, x<0
(B) x = a gives a maximum ⎪ − x + 3, 0 ≤ x <1

(C) x = b gives a minimum = ⎨
(D) x = b gives neither a maximum nor a minimum ⎪ x + 1, 1≤ x < 2
⎪⎩3 x − 3, x≥2
Solution: (A, C)
⎧ −3 x<0
Let f ′ (x) = (x – a)2n (x – b)2m + 1 ⎪does not exist x=0

The extreme values of f are given by f ′ (x) = 0 ⎪ −1 0 < x <1

⇒ (x – a)2n · (x – b)2m+1 = 0 ⇒ x = a, b. ⇒  f ′ (x) = ⎨does not exist x =1
2m+1 ⎪1 1< x < 2
For x < b, (x – b) is negative and for x > b, ⎪
(x – b)2m + 1 is positive ( 2m + 1 is odd). ⎪does not exist x=2
Thus, f ′ changes sign from negative to positive as ⎪3 x>2

x passes through b and so, f has a minimum at x = b.
Since 2n is an even integer, (x – a)2n does not Clearly, f (x) has minima at x = 1 and neither maxima
change sign as x passes through a i.e., f ′ (x) does not nor minima at x = 0 and x = 2.
change sign as x passes through a. Hence, f  has neither
40. The function
a maximum nor a minimum at x = a.
x

∫ t (e − 1) (t − 1) (t − 2)3 (t − 3)5 dt
t
38. The value of k so that the sum of the cubes of the roots f (x) =
of the equation x2 – kx + (2k – 3) = 0 assumes the min- −1
imum value, is has a local minimum at x =
(A) k = 1 (B)  k=3 (A) 0 (B) 1
(C) k = 0 (D)  None of these (C) 2 (D) 3

Objective_Maths_JEE Main 2017_Ch 14.indd 14 01/01/2008 05:43:26


Applications of Derivatives  14.15

Solution: (B, D)
This requires cos x = 1 and cos 2 x = 1
We have,
⇒  x = 2nπ and 2 x = 2mπ, n, m ∈ I.
f ′ (x) = x (ex – 1) (x – 1) (x – 2)3 (x – 3)5
∴ f ′ (x) = 0 ⇒ x = 0, 1, 2, 3 2mπ m
⇒ 2nπ = ⇒n=
2 2
Sign scheme for f ′ (x) :
This is possible only when n = m = 0.
−∞ –ve 0 –ve 1 +ve 2 –ve 3 +ve ∞ ∴ There is only one value (x = 0) at which f (x) attains
its maximum value.
Clearly, f (x) has local minimum at x = 1 and x = 3.
43. The minimum value of 27cos 2x ⋅ 81sin 2x is
⎧| x |, 0 < | x |≤ 2 1
41. Let f (x) = ⎨ . Then at x = 0, f has (A)  (B)  –5
⎩1, x = 0 243
1
(A)  a local maximum (B)  no local maximum (C)  (D)  None of these
(C)  a local minimum (D)  no extremum 5
Solution: (A)
Solution: (A)
Let y = 27cos2x ⋅ 81sin2x = 33cos2x + 4sin2x
We have,
⎧ − x, − 2 ≤ x < 0 y will be minimum when 3cos 2x + 4sin 2x is minimum.

(x) = ⎨1,
f  x=0 Let Z = 3cos 2x + 4sin 2x
⎪ x, 0< x≤2 Put 3 = r cos θ, 4 = r sin θ

The graph of f (x) is 4
Then,  r = 32 + 4 2 = 5 and tan θ =
3
⎛ 4⎞
i.e., θ = tan–1 ⎜⎝ 3 ⎟⎠
y = –x y=x
1 ∴ Z = 5 cos (2x – θ ) ⇒ – 5 ≤ Z ≤ 5
1
∴ Min. Z = – 5 ⇒ Min. y = 3–5 =
O 243
–2 2

44. If h (x) = f (x) + f (– x), then h (x) has got an extreme
Clearly, from the graph, value at a point where f ′ (x) is
(A)  even function (B)  odd function
f (0) = 1, f (0 – ε) < 1, (C)  zero (D)  None of these
f (0 + ε) < 1
Solution: (A)
where ε is small and positive. We have, h′ (x) = f ′ (x) – f ′ (– x)
∴ f (x) has a local maximum at x = 0.
For extreme values of h (x), h′ (x) = 0
42. The number of values of x where the function
⇒  f ′ (x) = f ′ (– x) ⇒ f ′ (x) is an even function.
f (x) = cos x + cos ( 2 x) attains its maximum is
x
(A) 0 (B) 1 cos t π
(C) 2 (D) infinite 45. Let f (x) = ∫ t
dt (x > 0); then for x = (2n + 1),
2
0
Solution: (B) f (x) has
We have, f (x) = cos x + cos ( 2 x) (A)  minima when n = 0, 2, 4, ...
(B)  maxima when n = 0, 2, 4, 6, ...
⇒ | f (x) | = cos x + cos 2 x ≤ | cos x | + cos 2x (C)  neither max. nor min. when n = – 1, – 3, – 5, ...
(D)  None of these
= 1 + 1 = 2, ∀ x ∈ R.
∴ Maximum value of f (x) = 2.

Objective_Maths_JEE Main 2017_Ch 14.indd 15 01/01/2008 05:43:28


14.16  Chapter 14

Solution: (B)
1 − x + x2
cos x 48. For all real x, the minimum value of is
We have, f ′ (x) = . 1 + x + x2
x
π 1
(A) 0 (B) 
∴  f ′ (x) = 0 ⇒ cos x = 0 ⇒ x = (2n + 1) , n ∈ I. 3
2
(C) 1 (D) 3
− x sin x − cos x
Also, f ″ (x) = .
x2 Solution: (B)
π π 1 − x + x2
− ( 2n + 1) sin ( 2n + 1) − 0 Let Z =
∴  f" ( x ) ]x = ( 2 n +1) π = 2
2
2 1 + x + x2
2 ⎡ π⎤
⎢⎣( 2n + 1) 2 ⎥⎦ ⇒  Z + Zx + Zx2 = 1 – x + x2

− 2 ( −1) n ⇒  Zx2 – x2 + Zx + x + Z – 1 = 0
= .
( 2n + 1) π ⇒  x2 (Z – 1) + x (Z + 1) + (Z – 1) = 0
 < 0, for n = 0, 2, 4, 6, ... For x to the real, B2 – 4AC ≥ 0
∴ f (x) has maxima when n = 0, 2, 4, 6, … ⇒ (Z + 1)2 – 4 (Z – 1) (Z – 1) ≥ 0
− 3)3 + 27
⇒  Z2 + 2Z + 1 – 4Z2 + 8Z – 4 ≥ 0
2

46. The minimum value of 2( x is


(A) 1 (B) 2 ⇒  – 3Z2 + 10Z – 3 ≥ 0 i.e. 3Z2 – 10Z + 3 ≤ 0
(C) 227 (D)  None of these
1
Solution: (A) ⇒ (Z – 3) (3Z – 1) ≤ 0 ⇒ ≤Z≤3
3
2( x − 3) + 27 is minimum when (x2 – 3)3 + 27 is
2 3

1
minimum. Therefore, minimum value of Z = .
3
Since (x2 – 3)3 + 27 = x6 – 9x4 + 27x2 49. If log10(x3 + y3) – log10(x2 + y2– xy) ≤ 2 then the maxi-
= x2 (x4 – 9x2 + 27) mum value of xy is
⎡⎛ 9⎞
2
27 ⎤ (A) 2500 (B) 3000
= x2 ⎢⎜ x 2 − ⎟ + ⎥ (C) 1200 (D) 3500
⎢⎣⎝ 2⎠ 4 ⎥⎦
Solution: (A)
≥ 0, for all x,
∴ minimum value of (x2 – 3)3 + 27 = 0. ( x3 + y3 )
log10 ≤ 2 and x + y > 0
− 3)3 + 27 x 2 + y 2 − xy
is 20 = 1
2

Thus, minimum value of 2( x


π ⇒ 0 < x + y ≤ 100
47. f (x) = 1 + [cos x] x, in 0 < x ≤
2 ⇒  maximum value of xy = 2,500.
(A)  has a minimum value 0
(B)  has a maximum value 2
⎡ π⎤ Concavity and Convexity
(C)  is continuous in ⎢0, ⎥
⎣ 2⎦ of a Function
π
(D)  is not differentiable at x =
2 Concavity of Function
Solution: (C) If f ′′(x) > 0 in the interval (a, b), then shape of f (x) in inter-
π val (a, b) is concave when observed from upwards (i.e.,
Since f (x) = 1 in 0 < x < (as [cos x] = 0)
2 concave upwards) or convex down.
Geometrically, a curve is concave upward in the
⎡ π⎤ interval [a, b] if all points on the curve lie above the tangent
∴ f (x) is continuous in ⎢0, ⎥
⎣ 2⎦ to the curve at any point in the interval [a, b].

Objective_Maths_JEE Main 2017_Ch 14.indd 16 01/01/2008 05:43:30


Applications of Derivatives 14.17

higher order derivative test to determine


DO NOT FORGET local maxima, local minima and point of
If the curve is concave upward, then the slope of the tangent inflexion
increases as x increases i.e., f ′(x) is strictly increasing in
Let f (x) be a differentiable function in an interval I and let
[a, b]
x = a be a point lying in the interior of I such that
1. f ′(a) = f ′′(a) = f ′′′(a) = ... = f n – 1(a) = 0 and
convexity of function 2. f n(a) exists and is non-zero, then:
(a) If n is even and f n(a) < 0, then x = a is a point of
If f ′′(x) < 0 in the interval (a, b), then shape of f (x) in interval
local maximum.
(a, b) is convex when observed from upwards (i.e., convex
(b) If n is even and f n(a) > 0, then x = a is a point of
upwards) or concave down.
local minimum.
Y (c) If n is odd and f n(a) > 0, then x = a is a point of
Concave up inflexion where shape of curve changes from con-
vex upwards to concave upwards.
Convex up (d) If n is odd and f n(a) < 0, then x = a is a point of
inflexion, where shape of curve changes from con-
cave upwards to convex upwards.
X

I M P O R TA N T P O I N T S
Convex down
This test is used only when second derivative fails to decide
between local maximum and local minimum.
i.e., when at x = a, f′(a) = 0 and fn(a) = 0.
Concave down

Fig. 14.13
Geometrically, a curve is concave downward in the interval trick(S) for problEm Solving
[a, b] if all points on the curve lie above the tangent to the
■ Maxima and minima occur alternately,that is between two
curve at any point in the interval [a, b].
maxima there is one minimum and vice-versa.
■ If f (x) → ∞ as x → a or b and f ′(x) = 0 only for one value
of x (say c) between a and b, then f (c) is necessarily the
DO NOT FORGET minimum and the least value.
If the curve is concave downward, then the slope of the If f (x) → – ∞ as x → a or b, then f (c) is necessarily the
tangent decreases as x increases i.e., f ′(x) is strictly decreas- maximum and the greatest value.
ing in [a, b]. ■ If a function is strictly increasing in [a, b], then f (a) is local
⇒ f ″(x) < 0 ∀ x ∈ [a, b] minimum and f (b) is local maximum.
■ If a function is strictly decreasing in [a, b], then f (a) is local
maximum and f (b) is local minimum.
point of inflExion
If at x = a, the shape of the curve changes from concave to rollE’S and lagrangE’S mEan valuE
convex or from convex to concave, then x = a is known as thEorEm
the point of inflexion.
rolle’s theorem
method to Evaluate point of inflexion If a function f defined on the closed interval [a, b], is
Points of inflexion can be obtained by equating f ′′(x) = 0. It 1. continuous on [a, b],
is not necessary that all values of x which are obtained by 2. derivable on (a, b), and
equating f ′′(x) = 0 are points of inflexion. Only those values 3. f (a) = f (b), then there exists atleast one real number c
of x for whcih f ′′(x) changes sign are points of inflexion. between a and b (a < c < b) such that f ′ (c) = 0.

Objective_Maths_JEE Main 2017_Ch 14.indd 17 01/01/2008 05:43:31


14.18  Chapter 14

Geometrical Interpretation Solved Examples


Y
50. If the polynomial equation
anxn + an – 1xn – 1 + ... + a2x2 + a1x + a0 = 0,
n positive integer, has two different real roots α and β,
then between α and β, the equation

a b
X nanxn – 1 + (n – 1) an – 1xn – 2 + ... + a1 = 0 has
O
(A)  exactly one root (B)  atmost one root
Fig. 14.14 (C)  atleast one root (D)  no root
Y Solution: (C)
Let f (x) = anxn + an – 1xn–1 + ... + a2x2 + a1x + a0, which
is a polynomial function in x of degree n. Hence f (x) is
continuous and differentiable for all x.
Let α < β. We are given, f (α) = 0 = f (β ).
a b
X By Rolle’s theorem, f ′ (c) = 0 for some value c,
O
α<c<β
Fig. 14.15 Hence the equation
Y f ′ (x) = nanxn–1 + (n – 1) an – 1xn–2 + ... + a1 = 0
has atleast one root between α and β.
51. If a + b + c = 0, then the equation 3ax2 + 2bx + c = 0
has, in the interval (0, 1)
(A)  atleast one root (B)  atmost one root
(C)  no root (D)  None of these
X
O a b
Solution: (B)

Fig. 14.16
Let   f (x) = ax3 + bx2 + cx, x ∈ [0, 1].
∴  f ′ (x) = 3ax2 + 2bx + c.
The conclusion is that there is a point c between a and b
such that the tangent to the graph at [c, f (c)] is parallel to Since f (x) is a polynomial function of x, it is con-
the x-axis. tinuous and differentiable for all x ∈ [0, 1].
Also, f (0) = 0; f (1) = a + b + c = 0.
Algebraic Interpretation ∴ f (0) = f (1)
Applying Rolle’s theorem, f ′ (k) = 0 for atleast one
Between two zeros a and b of f (x) (i.e., between two roots value k, 0 < k < 1. Hence k is a root of the equation
a and b of f (x) = 0) there exists atleast one zero of f ′(x).
3ax2 + 2bx + c = 0, where 0 < k < 1

52. If the equation anxn + an – 1xn – 1 + ... + a1x = 0 has a


Trick(s) for Problem Solving positive root x = α, then the equation nanxn – 1 + (n – 1)
Suppose a and b are two real numbers such that an – 1xn – 2 + ... + a1 = 0 has a positive root, which is
■ f(x) and its derivative f  ′(x) are continuous for a ≤ x ≤ b. (A)  smaller than α
(B)  greater than α
f(a) and f  (b) have opposite signs.
(C)  equal to α

■ f  ′(x) is different from zero for all values of x between a (D)  greater than or equal to α
and b.
Then, there is one and only real root of the equation f(x) = 0 Solution: (A)
between a and b.
Let f (x) = anxn + an –1xn–1 + ... + a1x.

Objective_Maths_JEE Main 2017_Ch 14.indd 18 01/01/2008 05:43:31


Applications of Derivatives  14.19

Then f (α) = 0 (Given). Also f (0) = 0. Solution: (A)


Moreover, f (x) is continuous and differentiable in Let α, β (α < β) be any two real roots of
[0,  α] as it is a polynomial function of x. Hence, by f  (x) = e–x – sin x.
Rolle’s theorem, there exists a c in (0, α) such that Then, f (α) = 0 = f (β )
f ′(x) = 0 for x = c i.e.
Moreover, f (x) is continuous and differentiable for
nanxn – 1 + (n – 1) an – 1xn – 2 + ... + 2a2x + a1 = 0. x ∈ [α, β].
Hence, from Rolle’s theorem, there exists atleast
a0 a a a
53. If + 1 + 2 + ... + n −1 + an = 0, then the equa- one x in (α, β ) such that
n +1 n n −1 2
tion a0xn + a1xn – 1 + ... + an – 1x + an = 0 has, in the f ′ (x) = 0 ⇒ – e– x – cos x = 0
interval (0, 1), ⇒ – e–x (1 + ex cos x) = 0 ⇒ ex cos x = – 1.
(A)  exactly one root (B)  atleast one root
(C)  atmost one root (D)  no root 56. If a, b, c be non-zero real numbers such that
1
Solution: (B)
∫ (1 + cos
8
x ) ( ax 2 + bx + c) dx
Let 0
x n +1 xn x n −1 x2 2
f (x) = a0 + a1 + a2 + ... + an −1 + an x = ∫ (1 + cos8 x ) ( ax 2 + bx + c) dx = 0,
n +1 n n −1 2
0
Then f (x) is continuous and differentiable in [0, 1],
as it is a polynomial function of x. then the equation ax2 + bx + c = 0 will have
(A) one root between 0 and 1 and other root between
Also, f (0) = 0 1 and 2
a0 a a a (B)  both the roots between 0 and 1
and + 1 + 2 + ... + n −1 + an = 0.
f (1) = (C)  both the roots between 1 and 2
n +1 n n −1 2
(Given) (D)  None of these

Hence, by Rolle’s theorem, there exists atleast one real Solution: (A)
number c ∈ (0, 1) such that f ′ (c) = 0 i.e., c is a root of y

the equation a0xn + a1xn – 1 + ... + an – 1x + an = 0. Let f ( y) = ∫ (1 + cos8 x ) ( ax 2 + bx + c) dx


0
54. The equation x log x = 3 – x has, in the interval (1, 3),
⇒ f ′ (y) = (1 + cos8y) (ay2 + by + c)(1)
(A)  exactly one root (B)  atmost one root
1
(C)  atleast one root (D)  no root
Now, f (1) = ∫ (1 + cos8 x ) ( ax 2 + bx + c) dx = 0
Solution: (C) 0
Let f (x) = (x – 3) log x 2

Then, f (1) = – 2 log 1 = 0 and f (3) = (3 – 3) log 3 = 0. and f (2) = ∫ (1 + cos8 x ) ( ax 2 + bx + c) dx = 0


0
As, (x – 3) and log x are continuous and differentiable
in [1, 3], therefore (x – 3) log x = f (x) is also continu- Also, f (0) = 0
ous and differentiable in [1, 3]. Hence, by Rolle’s theo- ∴  f (0) = f (1) = f (2)
rem, there exists a value of x in (1, 3) such that Now by Rolle’s theorem for f (x) in [0, 1].
f ′ (x) = 0 f ′ (α) = 0, for atleast one α, 0 < α < 1
1 and by Rolle’s theorem for f (x) in [1, 2],
⇒ log x + (x – 3) = 0
x f ′ (β ) = 0, for atleast one β, 1 < β < 2.
⇒ x log x = 3 – x. From (1),
55. Between any two real roots of the equation ex sin x = 1, f ′ (α) = 0 ⇒ (1 + cos8α) (aα2 + bα + 2) = 0.
the equation ex cos x = – 1 has But 1 + cos8α ≠ 0,
(A)  atleast one root (B)  exactly one root ∴ aα2 + bα + c = 0,
(C)  atmost one root (D)  no root

Objective_Maths_JEE Main 2017_Ch 14.indd 19 01/01/2008 05:43:32


14.20  Chapter 14

i.e.,  α is a root of the equation ax2 + bx + c = 0. Solved Examples


Similarly f ′ (β) = 0 ⇒ aβ2 + bβ + c = 0,
i.e.,  β is a root of the equation ax2 + bx + c = 0. 58. If f (x) is differentiable in the interval [2, 5], where
But the equation ax2 + bx + c = 0, being a quadratic f (2) =
1 1
and f (5) = , then there exists a number c,
equation, cannot have more than two roots. 5 2
∴ The equation ax2 + bx + c = 0 has one root α 2 < c < 5 for which f ′(c) =
between 0 and 1 and other root β between 1 and 2. 1 1
(A)  (B) 
57. If 27a + 9b + 3c + d = 0, then the equation 4ax3 + 3bx2 2 5
+ 2cx + d = 0 has atleast one real root lying between 1
(C)  (D)  None of these
(A)  0 and 1 (B)  1 and 3 10
(C)  0 and 3 (D)  None of these
Solution: (C)
Solution: (C) As f (x) is differentiable in [2, 5], therefore, it is also
Let f (x) = ax4 + bx3 + cx2 + dx continuous in [2, 5]. Hence, by mean value theorem,
Then, there exists a real number c in (2, 5) such that
f (0) = 0 and f (3) = 81a + 27b + 9c + 3d 1 1

= 3 (27a + 9b + 3c + d) f (5) − f ( 2) 1
f ′(c) = ⇒ f ′ (c) = 2 5 =
= 0 (∵ 27a + 9b + 3c + d = 0) 5−2 3 10

Therefore 0 and 3 are roots of the polynomial f (x). 59. The value of c in Lagrange’s theorem for the function
So, by Rolle’s theorem, there exists atleast one root of
the polynomial f ′ (x) = 4ax3 + 3bx2 + 2cx + d lying ⎧ ⎛ 1⎞
⎪ x cos ⎜ ⎟ , x ≠ 0
between 0 and 3. f (x) = ⎨ ⎝ x⎠ in the interval [– 1, 1] is
⎪0, x=0

Lagrange’s Mean Value Theorem 1
(A) 0 (B) 
2
If a function f defined on the closed interval [a, b], is 1
(C) – (D)  non-existent in the interval
1. continuous on [a, b] 2
2. derivable on (a, b), then there exists atleast one real Solution: (D)
number c between a and b (a < c < b) such that f (x) is continuous in the interval [– 1, 1], but f (x) is not
differentiable at x = 0. Hence mean value theorem is
f ( b) − f ( a)
f ′(c) = not applicable. So, no c can be found.
b−a
60. Let f be a function which is continuous and differen-
tiable for all real x. If f (2) = – 4 and f ′ (x) ≥ 6 for all
Geometrical Interpretation x ∈ [2, 4], then
The theorem states that between two points A and B on the (A) f (4) < 8 (B)  f (4) ≥ 8
graph of f there exists atleast one point where the tangent is (C) f (4) ≥ 12 (D)  None of these
parallel to the chord AB. Solution: (B)
Y
By mean value theorem, there exists a real number c
∈ (2, 4) such that
B
f ( 4) − f ( 2) f ( 4) + 4
f ′(c) = ⇒ f ′ (c) =
A 4−2 2
Since f ′ (x) ≥ 6 ∀ x ∈ [2, 4]
O X
x=a x=c x=b f ( 4) + 4
∴  f ′(c) ≥ 6 ⇒ ≥ 6 ⇒ f (4) + 4 ≥ 12
Fig. 14.17 2
⇒ f (4) ≥ 8

Objective_Maths_JEE Main 2017_Ch 14.indd 20 01/01/2008 05:43:34


Applications of Derivatives  14.21

dy And this is known as the rate of change of y with respect to


Application of as a rate measure x for the same value x = x1.
dx dy
The importance of the derivable functions in various Hence represents the actual rate of increases in y
­practical problems of day to day life rests on the fact that dx dy
the derivatives give us a measure of the rate of change of a per unit increase in x for the particular value of x or is
dx
function with respect to tis independent variable. the rate at which y is changing with respect to x.
Let y = f (x) be the given function. Take a fixed value
x1 for x and the corresponding value of y for x = x1 is say dy dy / dt dy dy dx
Again, = ⇒ = ×
y1, i.e., y1 = f (x1). Take another value x2 of x and the corre- dx dx / dt dt dx dt
sponding value of y be y2.
Let δx and δy denote the increments in x and y respec- In order words, if the rate of change of variables x and y are
δy y − y1 f ( x2 ) − f ( x1 ) taken relative to time t, we have
tively. The increment ratio = 2 =
δx x2 − x1 x2 − x1 dy
Rate of change of y = × rate of change of x.
is called the average rate of change with respect to x in the dx
interval [x1, x2]. If we continue choosing the values of x in
such a way that the interval [x1, x2] shrinks to zero, i.e., dx
→ 0 then according to the definition of derivative.

δy f ( x2 ) − f ( x1 ) ⎛ dy ⎞
lim = lim = f ′(x1) = ⎜ ⎟
δx → 0 δ x x2 → x1 x2 − x1 ⎝ dx ⎠ x = x
1

EXERCISES

Single Option Correct Type

1. The set of values of x for which log (1 + x) < x, is π


5. If 0 < α < β <
then
(A) x < 0 (B)  x>0 2
(C) 0 < x < 1 (D)  None of these tan β α tan β α
(A)  < (B)  >
2. Let f (x) = cos 2πx + x – [x], where [⋅] denotes the tan α β tan α β
greatest integer function. Then the number of points tan α α tan α α
(C)  < (D)  >
in [0, 10] at which f (x) assumes its local maximum tan β β tan β β
value, is
6. If a < 0, the function (eax + e–ax) is a monotonic
(A) 10 (B) 9
decreasing function for all values of x, where
(C) 0 (D) infinite
(A) x > 0 (B)  x<0
sin x (C) x > 1 (D)  x<1
3. The function f (x) =
is decreasing in the interval
x 7. The range of values of a for which the function
⎛ π ⎞ ⎛ π⎞ f (x) = x3 + (a + 2) x2 + 3ax + 5
(A)  ⎜ − , 0⎟ (B)  ⎜⎝ 0, ⎟⎠
⎝ 2 ⎠ 2 may be monotonic in R, is
(C) (0, π) (D)  None of these (A) a < 1 (B)  1 <a < 4
b (C) a > 4 (D)  None of these
4. If ax +≥ c for all positive x, where a, b> 0, then
x 8. The values of k for which the function
c2 c2 f (x) = kx3 – 9x2 + 9x + 3 may be increasing on R are
(A) ab < (B)  ab ≥
4 4 (A) k > 3 (B)  k<3
c (C) k ≤ 3 (D)  None of these
(C) ab ≥ (D)  None of these
4

Objective_Maths_JEE Main 2017_Ch 14.indd 21 01/01/2008 05:43:37


14.22  Chapter 14

9. The least possible value of k for which the function 18. The curve y – exy+ x = 0 has a vertical tangent at the
f (x) = x2 + kx+ 1 may be increasing on [1, 2] is point
(A) 2 (B) – 2 (A)  (1, 1) (B)  at no point
(C)  0 (D)  None of these (C)  (0, 1) (D)  (1, 0)
10. If f (x) = 2x3 + 9x2 + lx + 20 is a decreasing function 19. The set of all values of a for which the function f (x) =
of x in the largest possible interval (– 2, – 1) then l is (a2 – 3a + 2)(cos2 x/4 – sin2 x/4) + (a – 1)x + sin 1 does
equal to not possess critical points is
(A) 12 (B) – 12 (A) [1, ∞) (B)  (0, 1) ∪ (1, 4)
(C)  6 (D)  None of these (C)  (– 2, 4) (D)  (1, 3) ∪ (3, 5)
11. Let f ′(x) > 0 and g′(x) < 0 for all x ∈ R. Then, ⎧⎪ − x 3 + log 2 b 0 < x < 1
(A) f [g(x)] > f [g(x – 1)] 20. Let f (x) = ⎨ . Then set of val-
⎩⎪ 3x x ≥1
(B) f [g(x)] > f [g(x + 1)]
(C) g[ f (x)] > g [ f (x – 1)] ues of b for which f (x) has least value at x = 1 is:
(D) g[ f (x)] < g[ f (x + 1)] (A) R+ (B)  (0, 16]
(C) [16, ∞) (D)  None of these
12. If the function f (x) = 3 cos | x | – 6ax + b increases for
all x∈ R, then the range of values of a is given by 21. If at any point on a curve the sub-tangent and s­ ub-normal
1 1 are equal, then the length of the normal is equal to
(A) a > – (B)  a<–
2 2 (A)  2 ordinate (B) ordinate
(C) a ≤ b (D)  a≥b (C)  2 ordinate (D)  None of these
13. The equation x + ex = 0 has x2
(A)  only one real root 22. Tangent is drawn to the ellipse + y 2 = 1 at
27
(B)  only two real roots (3 3 cos θ , sin θ ) , where θ ∈ (0, θ/2). Then, the value
(C)  no real root
of θ such that sum of intercepts on axes made by this
(D)  None of these
tangent is minimum, is
14. The value of a in order that f (x) = sin x – cos x – ax + b π π
decreases for all real values is given by (A)  (B) 
3 6
(A) a ≥ 2 (B) 
a< 2 π π
(C)  (D) 
(C) a ≥ 1 (D) a<1 8 4
15. Let f and g be increasing and decreasing functions 23. The minimum value of a tan2x + b cot2x equals the
respectively from [0, ∞) to [0, ∞). Let h (x) = f [g (x)]. maximum value of a sin2 θ + b cos2 θ where a > b > 0,
If h (0) = 0, then h (x) is when
(A)  always zero (B)  always negative (A) a = b (B)  a = 2b
(C)  always positive (D)  strictly increasing (C) a = 3b (D)  a = 4b
16. If f ″ (x) < 0 ∀ x ∈ (a, b), then f ′ (x) = 0 24. A function f is such that f ′(a) = f ″(a) = f ″′(a) = ... =
(A)  exactly once in (a, b) f (2n) (a) = 0 and f  has a local maximum value b at x =
(B)  atmost once in (a, b) a, if f (x) is
(C)  atleast once in (a, b) (A) (x – a)2n+2
(D)  None of these (B) b – 1 – (x + 1 – a)2n–1
17. The two tangents to the curve ax2 + 2hxy + by2 = 1, (C) b– (x – a)2n+2
a > 0 at the points where it crosses x-axis, are (D) (x – a)2n+2 – b
(A) parallel
1 1
(B) perpendicular 25. If P = x3 – 3
and Q = x – , x ∈ (0, x) then m
­ inimum
π x x
(C)  inclined at an angle value of P/Q2
4
(D)  None of these (A)  is 2 3 (B)  is – 2 3
(C)  does not exist (D)  None of these

Objective_Maths_JEE Main 2017_Ch 14.indd 22 01/01/2008 05:43:39


Applications of Derivatives  14.23

26. If the area of the triangle included between the axes (C) the perimeter of ΔABC is minimum when it is
and any tangent to the curve xny = an is constant, then isosceles
n is equal to (D) the perimeter of ΔABC is maximum when it is
(A) 1 (B) 2 isosceles
3 1 33. Let f (x) = 1 + 3x2 + 32x4 + ... + 330 ⋅ x60. Then f (x) has
(C)  (D) 
2 2 (A)  atleast one maximum
27. If f (x) and g (x) are differentiable functions for 0 ≤ x ≤ (B)  exactly one maximum
1 such that f (0) = 2, g (0) = 0, f (1) = 6, g (1) = 2, then (C)  atleast one minimum
in the interval (0, 1), (D)  exactly one minimum
(A) f ′ (x) = 0 for all x 34. A function f is such that f ′ (4) = f ″ (4) = 0 and f has
(B) f ′ (x) = 2g′ (x) for atleast one x minimum value 10 at x = 4. Then f (x) =
(C) f ′ (x) = 2g′ (x) for atmost one x (A)  4 + (x – 4)4 (B)  10 + (x – 4)4
(D)  None of these (C) (x – 4) 4
(D)  None of these
sin ( x + a) 35. The range of values of k for which the function
28. If y = ; a ≠ b, then y has
sin ( x + b) f (x) = (k2 – 7k + 12) cosx + 2 (k – 4) x + log 2
(A)  maximum at x = 0 does not possess critical points, is
(B)  minimum at x = 0 (A)  (1, 5) (B)  (1, 5) – {4}
(C)  neither maximum nor minimum (C)  (1, 4) (D)  None of these
(D)  None of these
36. The minimum value of the function
29. For a differentiable curve y = f (x) having atleast two x p x−q 1 1
f (x) = + , where + = 1, p > 1 is
extremum in the interval [a, b], p q p q
(A)  two of its maximum values occur successively (A) 1 (B) 0
(B)  two of its minimum values occur successively (C)  2 (D)  None of these
(C)  maximum and minimum values occur alternatively x2 − 1
(D)  None of the above 37. If f (x) = 2 , for every real number x, then the
x +1
30. The points on the curve xy2 = 1 which are nearest to ­minimum value of f
the origin are (A)  does not exist because f is unboundecd
⎡⎛ 1 ⎞ 1/ 3 −1/ 6 ⎤ ⎡⎛ 1 ⎞ 1/ 3 −1/ 6 ⎤ (B)  is not attained even though f is bounded
⎛ 1⎞ (C)  is equal to 1
(A)  ⎢⎜ ⎟ , ± ⎜ ⎟ ⎥ (B) 
⎢⎜ ⎟ , 2 ⎥
⎢⎣⎝ 2 ⎠ ⎝ 2⎠ ⎥⎦ ⎢⎣⎝ 2 ⎠ ⎥⎦ (D)  is equal to – 1
⎛ ⎛ 1⎞
−1/ 6 ⎞ 38. If a differentiable function f (x) has a relative minimum
(C)  ⎜ 21/ 3 , ± ⎜ ⎟ ⎟ (D)  None of these at x = 0, then the function y = f (x) + ax + b has a rela-
⎝ ⎝ 2⎠ ⎠ tive minimum at x = 0 for
31. N characters of information are held on magnetic tape, (A) all a > 0 (B)  all b > 0
in batches of x characters each; the batch process- (C) all a and b (D) all b if a = 0
ing time is α + βx2 seconds; α, β are constants. The
39. On the curve x3 = 12y, the abscissa changes at a faster
­optimum value of x for fast processing is
rate than the ordinate. Then, x belongs to the interval
α β
(A)  (B)  (A)  (– 4, 4) (B)  (–3, 3)
β α
(C)  (–2, 2) (D)  None of these
α β 40. The maximum value of radius vector where
(C)  (D) 
β α c4 a2 b2
2
= + ; (a, b > 0) is
32. AB is a diameter of a circle and C is any point on the r sin 2 t cos 2 t
circumference of the circle, then c4
(A) (a + b)2 (B) 2
(A)  area of ΔABC is maximum when it is an isosceles ( a + b)
(B)  area of ΔABC is minimum when it is an isosceles c2
(C)  (D)  c2(a + b)
a+b

Objective_Maths_JEE Main 2017_Ch 14.indd 23 01/01/2008 05:43:41


14.24  Chapter 14

41. Let f (x) and g(x) be defined and differentiable for (C)  decreases in [0, 1) and increases in (1, 2]
x ≥ x0 and f (x0) = g(x0), f ′(x) > g′(x) for x > x0, then (D)  increases in [0, 1) and decreases in (1, 2]
(A) f (x) < g(x), x > x0 (b) f (x) = g(x), x > x0 50. A spherical balloon is filled with 4500π cubic meters
(C) f (x) > g(x), x > x0 (d) None of these of helium gas. If a leak in the balloon causes the gas
42. If α and β (α < β ) be two different real roots of the to escape at the rate of 72π cubic meters per minute,
equation ax2 + bx + c = 0, then then the rate (in meters per minute) at which the radius
b b of the balloon decreases 40 minutes after the leakage
(A) α > – (B)  β<– began is
2a 2a
(A) 9/7 (B) 7/9
b b
(C) α < – < β β<–
(D)  <α (C) 2/9 (D) 9/2
2a 2a
51. Let a, b ∈ R be such that the fucntion  f given by f (x) =
1 ln|x| + bx2 + ax, x ≠ 0 has extreme values at x = –1 and
43. If f ′ (x) = for all x and f (0) = 0, then
1 + x2 x = 2.
(A) f (2) < 0.4 (B)  f (2) > 2 Statement 1: f has local maximum at x = –1 and at
(C) 0.4 < f (2) < 2 (D)  f (2) = 2 x = 2.
44. The interval in which λ should be if f (x) = sin3 x + λ 1 −1
Statement 2: a = and b = .
sin2 x(–π/2 < x < π/2) has exactly one maximum and 2 4
one minimum is (A)  Statement-1 is false, Statement-2 is true.
⎛ 1 1⎞ (B) Statement-1 is true, statement-2 is true, statement-2
(A)  (– 1, 1) (B)  ⎜ − , ⎟ is a correct explanation for Statement-1.
⎝ 2 2⎠
(C) Statement-1 is true, statement-2 is true; statement-2
⎛ − 3 3 ⎞ ⎛ − 3 ⎞ ⎛ 3⎞
(C)  ⎜ , ⎟ (D)  ⎜⎝ , 0 ⎟ ∪ ⎜ 0, ⎟ is not a correct explanation for Statement-1.
⎝ 2 2⎠ 2 ⎠ ⎝ 2⎠ (D)  Statement-1 is true, statement-2 is false.
45. Twenty metre of wire is available to fence off a flower 52. Each side of a square is increasing at the uniform rate
bed in the form of a sector. If the flower bed has the of 1 m/sec. If after some time the area of the square
maximum surface then radius is is increasing at the rate of 8 m2/sec, then the area of
(A) 10 (B) 5/2 square at that time in sq. meters is:
(C) 5 (D) 15/2 (A) 4 (B) 9
46. If f ″(x) > 0, ∀ x ∈ R, f ′(3) = 0 and g(x) = f (tan2x – 2 (C) 16 (D) 25
tan x + 4), 0 < x < π/2, then g(x) is increasing in 53. Let a, b, c ∈ R, a > 0 and function f : R → R be defined
⎛ π⎞ ⎛ π⎞ ⎛π π⎞ by f (x) = ax2 + bx + c
(A)  ⎜ 0, ⎟ (B)  ⎜ 0, ⎟ ∪ ⎜ , ⎟
⎝ 4⎠ ⎝ 4⎠ ⎝ 4 2⎠ Statement 1: b2 < 4ac ⇒ f (x) > 0, for every value of x.
⎛π π⎞ ⎛ π⎞ Statement  2: f is strictly decreasing in the interval
(B)  ⎜ , ⎟ (D)  ⎜⎝ 0, ⎟⎠
⎝ 4 2⎠ 2 ⎛ −b ⎞
⎜⎝ − ∞, ⎟⎠ and strictly increasing in the interval
2a
47. The normal to the curve x = a(1 + cos θ ), y = a sin θ
⎛ −b ⎞
at θ always passes through the fixed point ⎜⎝ , ∞⎟⎠ .
2a
(A) (a, a) (B)  (a, 0)
(A) Statement-1 is true, Statement-2 is true, Statement-2
(C) (0, a) (D)  None of these
is a correct explanation for Statement-1.
48. If the tangent to the curve 2y3 = ax2 + x3 at the point (B) Statement-1 is true, Statement-2 is true, Statement-2
(a, a) cuts off intercepts α and β on the coordinate axes is not a correct explanation for Statement-1.
such that α2 + β 2 = 61, then a = (C)  Statement-1 is true, Statement-2 is false.
(A) ± 30 (B)  ±5 (D)  Statement-1 is false, Statement-2 is true.
(C) ± 6 (D)  ± 61 54. How many real solutions does the equation x7 + 14x5 +
49. If x ∈ [0, 2] and g(x) = f (x) + f(2 – x). Also, f ′′(x) < 0, 16x3 + 30x – 560 = 0 have?
then g(x) (A) 7 (B) 1
(A)  increases in [0, 2] (B)  decreases in [0, 2] (C) 3 (D) 5

Objective_Maths_JEE Main 2017_Ch 14.indd 24 01/01/2008 05:43:42


Applications of Derivatives  14.25

55. Given P(x) = x4 + ax3 + bx2 + cx + d such that x = 0 is (C)  equal to α


the only real root of P′(x) = 0 . If P(–1) < P(1) , then in (D)  greater than or equal to α
the interval [–1, 1]
62. If a, b, c be non-zero real numbers such that
(A) P(–1) is the minimum and P(1) is the maximum
1
of P
∫ (1 + cos
8
x ) ( ax 2 + bx + c) dx
(B) P(–1) is not minimum but P(1) is the maximum of
0
P
2
(C) P(–1) is the minimum and P(1) is not the maxi-
= ∫ (1 + cos8 x ) ( ax 2 + bx + c) dx = 0,
mum of P
0
(D) neitherP(–1) is the minimum nor P(1) is the
­maximum of P then the equation ax2 + bx+ c = 0 will have
(A) one root between 0 and 1 and other root between
56. Let f: [2, 7] → [0, ∞) be a continuous and differentiable 1 and 2
function. Then, (B)  both the roots between 0 and 1
( f (7) 2 + ( f ( 2)) 2 + f ( 2) f (7) (C)  both the roots between 1 and 2
(f (7) – f (2)) is equal to
3 (D)  None of these
(A) 5f 2(c) f ′(C)  (B) 5f ′(c) 63. Let f be a function which is continuous and differen-
(C) f (c) f ′(C) (D)  None of these tiable for all real x. If f (2) = – 4 and f ′ (x) ≥ 6 for all
where c ∈ (2, 7). x ∈ [2, 4], then
57. The fraction exceeding its pth power by the greatest (A) f (4) < 8 (B)  f (4) ≥ 8
number possible, where p ≥ 2, is (C) f (4) ≥ 12 (D)  None of these
1 p −1
⎛ 1⎞ p −1 ⎛ 1⎞ b
(A)  ⎜ ⎟ (B)  ≥ c for all positive x, where a, b > 0, then
64. If ax +
⎝ p⎠ ⎜⎝ p ⎟⎠ x
c2 c2
(C) p1/p – 1 (D)  None of these (A) ab < (B)  ab ≥
4 4
58. Let the function f (x) be defined as c
(C) ab ≥ (D)  None of these
⎪⎧tan −1 α − 3 x 2 , 0 < x < 1 4
f (x) = ⎨
⎩⎪ − 6 x, x ≥1 65. Let f  be a continuous, diferentiable and bijective func-
f (x) can have a maximum at x = 1 if the value of α is tion. If the tangent to y = f (x) at x = a is also the nor-
(A) 0 (B) 2 mal to y = f (x) at x = b, then there exists at least one
(C)  1 (D)  None of these c ∈ (a, b) such that
⎧| x |, 0 < | x |≤ 2 (A) f ′(c) = 0 (B)  f ′(c) > 0
59. Let f (x) = ⎨ . Then, at x = 0, f  has
⎩1, x = 0 (C) f ′(c) < 0 (D)  None of these
(A)  a local maximum (B)  no local maximum 66. The values of k for which the function
(C)  a local minimum (D)  no extremum f(x) = kx3 – 9x2 + 9x + 3 may be increasing on R are
x
cos t π (A) k > 3 (B)  k<3
60. Let f (x) = ∫ t
dt (x > 0); then for x = (2n + 1) ,
2
(C) k ≤ 3 (D)  None of these
0
f (x) has 67. The least possible value of k for which the function
(A)  minima when n = 0, 2, 4, ... f (x) = x2 + kx+ 1 may be increasing on [1, 2] is
(B)  maxima when n = 0, 2, 4, 6, ... (A) 2 (B) – 2
(C)  neither max. nor min. when n = – 1, – 3, – 5, ... (C)  0 (D)  None of these
(D)  None of these
68. Let a + b = 4, a < 2 and g(x) be a monotonically
61. If the equation anxn + an – 1xn – 1 + ... + a1x = 0 has a increasing function of x.
positive root x = α, then the equation a b

nanxn – 1 + (n – 1) an – 1xn – 2 + ... + a1 = 0 has a positive Then, f (a) = ∫ g ( x) dx + ∫ g ( x) dx


root, which is 0 0

(A)  smaller than α (A)  increases with increase in (b – a)


(B)  greater than α (B)  decreases with increase in (b – a)

Objective_Maths_JEE Main 2017_Ch 14.indd 25 01/01/2008 05:43:44


14.26  Chapter 14

(C)  increases with decrease in (b – a) 77. A function f is such that f ′ (4) = f ″ (4) = 0 and f has
(D)  None of these minimum value 10 at x = 4. Then f (x) =
69. The equation x + ex = 0 has (A)  4 + (x – 4)4 (B)  10 + (x – 4)4
4
(C) (x – 4) (D)  None of these
(A)  only one real root
(B)  only two real roots 78. The range of values of k for which the function
(C)  no real root f (x) = (k2 – 7k + 12) cosx + 2 (k – 4) x + log 2
(D)  None of these does not possess critical points, is
70. The value of a in order that (A)  (1, 5) (B)  (1, 5) – {4}
(C)  (1, 4) (D)  None of these
f (x) = sin x – cos x – ax + b
decreases for all real values is given by 79. If a differentiable function f (x) has a relative minimum
(A) a ≥ 2 (B)  a< 2 at x = 0, then the function y = f (x) + ax + b has a rela-
tive minimum at x = 0 for
(C) a ≥ 1 (D)  a<1
(A) all a > 0 (B)  all b > 0
71. If f ″(x) < 0 ∀ x ∈ (a, b), then f ′(x) = 0 (C) all a and b (D) all b if a = 0
(A)  exactly once in (a, b)
80. Let f (x) and g(x) be defined and differentiable for
(B)  at most once in (a, b)
x ≥ x0 and f (x0) = g(x0), f ′(x) > g′(x) for x > x0, then
(C)  at least once in (a, b)
(D)  None of these (A) f (x) < g(x), x > x0 (b) f (x) = g(x), x > x0
(C) f (x) > g(x), x > x0 (d) None of these
72. The minimum value of a tan2x + b cot2x equals the max-
imum value of a sin2 θ + b cos2 θ where a > b > 0, when 81. If α and β (α < β) be two different real roots of the
(A) a = b (B)  a = 2b equation ax2 + bx + c = 0, then
(C) a = 3b (D)  a = 4b b b
(A) α > – (B)  β<–
2a 2a
x2 − 1 b b
73. If f ( x ) = , for every real number, then mini-
x2 + 1 (C) α < – < β β<–
(D)  <α
2a 2a
mum value of f
(A)  Does not exist 82. If p(x) = a0 + a1x+a2x2 + ... + anxn and |p(x)| ≤ |ex–1 – 1|
(B)  Is note attained even through f is bounded for all x≥ 0, then |a1 + 2a2 + 3a3 + ... + nan|
(C)  Is equal to 1 (A) ≤ 1 (B)  ≥1
(D)  Is equal to –1 (C) ≥ 0 (D)  ≤0

74. If y = a log|x| + bx2 + x has its extremum values at 83. The maximum value of radius vector where
x = –1 and x = 2, then c4 a2 b2
2 = + ; (a, b> 0) is
(A) a = 2, b = –1 r sin t cos 2 t
2

(B) a = 2, b = –1/2 c4
(C) a = –2, b = 1/2 (A) (a + b)2 (B) 2
( a + b)
(D)  None of these
c2
75. If f (x) and g(x) are differentiable functions for 0 ≤ x ≤ 1 (C)  (D)  c2(a + b)
a+b
such that f (0) = 2, g (0) = 0, f (1) = 6, g(1) = 2, then in
the interval (0, 1), ⎧⎪ − x 3 + log 2 b 0 < x < 1
84. Let f (x) = ⎨ . Then, the set of
(A) f ′ (x) = 0 for all x ⎩⎪ 3x x ≥1
(B) f ′ (x) = 2g′(x) for at least one x values of b for which f (x) has least value at x = 1 is
(C) f ′ (x) = 2g′(x) for at most one x (A) R+ (B)  (0, 16]
(D)  None of these (C) [16, ∞) (D)  None of these
76. The difference between the greatest and least values of 85. The second drivative f ″(x) of the function f (x) exists
1 1 for all x in [0, 1] and satisfies | f ″(x)| ≤ 1. If f (0) = f (1),
the function f ( x ) = cos x + cos 2 x − cos 3 x is
2 3 then for all x in [0, 1]
(A) 2/3 (B) 8/7 (A) | f ′(x) | < 1 (B)  | f ′(x) | > 1
(C) 9/4 (D) 3/8 (C) | f ′(x) | = 1 (D)  f (x) is constant

Objective_Maths_JEE Main 2017_Ch 14.indd 26 01/01/2008 05:43:46


Applications of Derivatives  14.27

86. Let the function f be defined as 93. If the function


⎧ P( x) ⎧ 3 b3 − b 2 + b − 1
⎪ , x≠2 ⎪− x + 2 , 0 ≤ x <1
f ( x) = ⎨ x − 2 f ( x) = ⎨ b + 3b + 2
⎪ ⎪ 2 x − 3, 1≤ x ≤ 3
⎩ 7, x = 2 ⎩

where P(x) is a polynomial such that P ′″(x) is identi- has the least value at x = 1, then all possible real values
cally equal to 0 and P(3) = 9. If f (x) is continuous at of b are
x = 2, then (A)  (–1, 1) (B)  (–2, –1) ∪ [1, ∞)
(A) P(x) = 2x2 – x – 6 (B)  P(x) = 2x2 + x – 6 (C)  (–2, 1) (D)  None of these
2
(C) P(x) = 2x – x + 6 (D)  None of these
| x + 1|
87. The equation x5 – 3x – 1 = 0 has, in the interval [1, 2] 94. The function f (x) = is strictly decreasing in the
interval x2
(A)  at least one root (B)  at most one root
(C)  no root (D)  a unique root (A) (– ∞, – 2) ∪ (0, 1) (B)  (– 2, 0) ∪ (1, ∞)
(C)  (– 2, – 1) ∪ (0, ∞) (D)  None of these
88. If the equation x – sin x = k has a unique root in
95. If the equation ax2 + bx + c = 0, a, b, c, ∈R has at least
⎡ π π⎤
⎢ − 2 , 2 ⎥ , then the range of values of k are one root in (0, 1), then
⎣ ⎦
(A) 2a + 3b + 6c = 0 (B)  a + 3b + 6c = 0
⎛ π π ⎞ ⎡ π π ⎤ (C) 2a + b + 6c = 0 (D)  2a + 3b + c = 0
(A)  ⎜1 − , − 1⎟ (B) ⎢1 − , − 1⎥
⎝ 2 2 ⎠ ⎣ 2 2 ⎦
96. The range of values of a so that the equation x3 – 3x +
⎡ π ⎤ a = 0 has three real and distinct roots is
(C)  ⎢0, + 1⎥ (D)  None of these
⎣ 2 ⎦ (A) (– ∞, – 2) ∪ (2, ∞) (B)  (– 2, 0)
89. The largest term in the sequence (C)  (– 2, 0) (D)  (– 2, 2)
n x2 y2 x2 y2
an = 2 , n ∈ N is 97. The curves + = 1 and + = 1 will cut
n + 10

You might also like